You are on page 1of 992

SA

NK
A
LP
JA
UH
A
RI
FUNDAMENTAL’S OF MATHEMATICS

SETS
A set is a collection of well defined objects which are distinct from each other.

METHODS TO WRITE A SET :


(i) Roster Method or Tabular Method : In this method a set is described by listing elements,
separated by commas and enclose then by curly brackets.
(ii) Set builder form (Property Method) : In this we write down a property or rule which gives us
all the element of the set.

RI
TYPES OF SETS
Null set or empty set : A set having no element in it is called an empty set or a null set or void set, it is
denoted by φ or { }.

A
Singleton set : A set consisting of a single element is called a singleton set.
Finite set : A set which has only finite number of elements is called a finite set.
Order of a finite set : The number of distinct elements in a finite set A is called the order of this set

UH
and denoted by O(A) or n(A). It is also called cardinal number of the set.
e.g. A = {a, b, c, d} ⇒ n(A) = 4
Infinite set : A set which has an infinite number of elements is called an infinite set.
Equal sets : Two sets A and B are said to be equal if every element of A is member of B, and every
element of B is a member of A. If sets A and B are equal, we write A = B and if A and B are not equal
then A ≠ B
JA
Equivalent sets : Two finite sets A and B are equivalent if their cardinal number is same i.e. n(A) =
n(B)
e.g. A = {1, 3, 5, 7}, B = {a, b, c, d} ⇒ n(A) = 4 and n(B) = 4
⇒ A and B are equivalent sets
Note - Equal sets are always equivalent but equivalent sets may not be equal
LP

SUBSET AND SUPERSET :


Let A and B be two sets. If every element of A is an element of B then A is called a subset of B and B is
called superset of A. We write it as A ⊂ B.
e.g. A = {1, 2, 3, 4} and B = {1, 2, 3, 4, 5, 6, 7} ⇒ A⊂ B
If A is not a subset of B then we write A ⊄ B
A

PROPER SUBSET :
If A is a subset of B but A ≠ B then A is a proper subset of B. Set A is not proper subset of A so this is
NK

improper subset of A
Note : (i) The total number of subsets of a finite set containing n elements is 2 n.
(ii) Number of proper subsets of a set having n elements is 2n – 1.

POWER SET :
Let A be any set. The set of all subsets of A is called power set of A and is denoted by P(A)
SA

UNIVERSAL SET :
A set consisting of all possible elements which occur in the discussion is called a universal set and is
denoted by U.
e.g. if A = {1, 2, 3}, B = {2, 4, 5, 6}, C = {1, 3, 5, 7} then U = {1, 2, 3, 4, 5, 6, 7} can be taken as the
universal set.
SOME OPERATION ON SETS :
(i) Union of two sets : A ∪ B = {x : x ∈ A or x ∈ B}
e.g. A = {1, 2, 3}, B = {2, 3, 4} then A ∪ B = {1, 2, 3, 4}
(ii) Intersection of two sets : A ∩ B = {x : x ∈ A and x ∈ B}
e.g. A = {1, 2, 3}, B = {2, 3, 4} then A ∩ B = {2, 3}
(iii) Difference of two sets : A – B = {x : x ∈ A and x ∉ B}. It is also written as A ∩ B'.
Similarly B – A = B ∩ A' e.g. A = {1, 2, 3}, B = {2, 3, 4} ; A – B = {1}
(iv) Symmetric difference of sets : It is denoted by A ∆ B and A ∆ B = (A – B) ∪ (B – A)
(v) Complement of a set : A' = {x : x ∉ A but x ∈ U} = U – A
e.g. U = {1, 2,........, 10}, A = {1, 2, 3, 4, 5} then A' = {6, 7, 8, 9, 10}

LAWS OF ALGEBRA OF SETS (PROPERTIES OF SETS):


(i) Commutative law : (A ∪ B) = B ∪ A ; A ∩ B = B ∩ A
(ii) Associative law : (A ∪ B) ∪ C = A ∪ (B ∪ C) ; (A ∩ B) ∩ C = A ∩ (B ∩ C)
(iii) Distributive law : A ∪ (B ∩ C) = (A ∪ B) ∩ (A ∪ C) ; A ∩ (B ∪ C) = (A ∩ B) ∪ (A ∩ C)
(iv) De-morgan law : (A ∪ B)' = A' ∩ B' ; (A ∩ B)' = A' ∪ B'
(v) Identity law : A ∩ U = A ; A ∪ φ = A
(vi) Complement law : A ∪ A' = U, A ∩ A' = φ, (A')' = A
(vii) Idempotent law : A ∩ A = A, A ∪ A = A

SOME IMPORTANT RESULTS ON NUMBER OF ELEMENTS IN SETS :


If A, B, C are finite sets and U be the finite universal set then
n(A ∪ B) = n(A) + n(B) – n(A ∩ B)

RI
(i)
(ii) n(A – B) = n(A) – n(A ∩ B)
(iii) n(A ∪ B ∪ C) = n(A) + n(B) + n(C) – n(A ∩ B) – n(B ∩ C) – n(A ∩ C) + n(A ∩ B ∩ C)
(iv) Number of elements in exactly two of the sets A, B, C
= n(A ∩ B) + n(B ∩ C) + n(C ∩ A) – 3n(A ∩ B ∩ C)

A
(v) Number of elements in exactly one of the sets A, B, C
= n(A) + n(B) + n(C) – 2n(A ∩ B) – 2n(B ∩ C) – 2n(A ∩ C) + 3n(A ∩ B ∩ C)

UH
Intervals :
Intervals are basically subsets of R and are commonly used in solving inequalities or in finding
domains. If there are two numbers a, b ∈ R such that a < b, we can define four types of intervals as
follows :
JA
Name Representation Discription
Open Interval (a, b) {x : a < x < b} i.e. end points are not included.
{x : a ≤ x ≤ b} i.e. end points are also included. This is possible only when
Close Interval [a, b] both a and b are finite.
Open - Closed Interval (a, b] {x : a < x ≤ b} i.e. a is excluded and b is included.
Close - Open Interval [a, b) {x : a ≤ x < b} i.e. a is included and b is excluded.
LP

Note : (i) (a, ∞) = {x : x > a} (ii) [a, ∞) = {x : x ≥ a} (iii) (– ∞, b) = {x : x < b}


(iv) (–∞, b] = {x : x ≤ b} (v) (– ∞, ∞) = {x : x ∈ R}

Graph of polynomial
A

To plot a graph of polynomial, several sets of Points (x, y) are required.

dy
The key points are (i) stationary points ( where =0)
NK

dx

(ii) y-intercept ( where x is zero)

(iii) x-intercept ( where y is zero)

(iv) behaviour of polynomial at x tends to ± ∞


SA

and

Logarithm of A Number :
The logarithm of the number N to the base ' a ' is the exponent indicating the power to which the base ' a
' must be raised to obtain the number N. This number is designated as loga N. Hence:
logaN = x ⇔ ax = N , a > 0, a ≠ 1 & N > 0

Domain of Definition :
The existence and uniqueness of the number loga N can be determined with the help of set of
conditions, a > 0 & a ≠ 1 & N > 0.
The Principal Properties of Logarithm:
Let M & N are arbitrary positive numbers, a > 0, a ≠ 1, b > 0, b ≠ 1 and α, β are any real numbers,
then :
(i) loga (M.N) = loga M + loga N ; in general loga (x1 x2 ......xn) = logax1 + loga x2 + ........+ loga xn
(ii) loga (M/N) = loga M − loga N (iii) loga Mα = α. loga M
1 loga M
(iv) logaβ M = logaM (v) logb M = (base changing theorem)
β loga b
1
(vi) logba = (vii) ax = e x n a
loga b
log N
(viii) alogc b = blogc a (ix) a a = N, a > 0, a ≠ 1 & N > 0
Note : (i) If the number and the base are on the same side of the unity, then the
logarithm is positive.

RI
(ii) If the number and the base are on the opposite sides of unity, then the logarithm is
negative.

Logarithmic Equation :

A
The equality loga x = loga y is possible if and only if x = y i.e.
loga x = loga y ⇔ x = y
Always check validity of given equation, (x > 0, y > 0, a > 0, a ≠ 1)

UH
Logarithmic Inequality :
Let 'a' is a real number such that
(i) If a > 1, then logax > loga y ⇒ x>y
(ii) If a > 1, then logax < α ⇒ 0 < x < aα
JA
(iii) If a > 1, then logax > α ⇒ x > aα
(iv) If 0 < a < 1, then logax > logay ⇒ 0<x<y
(v) If 0 < a < 1, then loga x < α ⇒ x > aα

Form - I : f(x) > 0, g(x) > 0, g(x) ≠ 1


Form Collection of system
LP

 f(x) ≥ 1 , g(x) > 1


(a) logg(x) f(x) ≥ 0 ⇔ 
0 < f(x) ≤ 1 , 0 < g(x) < 1
 f(x) ≥ 1 , 0 < g(x) < 1
(b) logg(x) f(x) ≤ 0 ⇔ 
0 < f(x) ≤ 1 , g(x) > 1
A

 f(x) ≥ (g(x))a , g(x) > 1


(c) logg(x) f(x) ≥ a ⇔ 
0 < f(x) ≤ (g(x)) , 0 < g(x) < 1
a
NK

0 < f(x) ≤ (g(x))a , g(x) > 1


(d) logg(x) f(x) ≤ a ⇔ 
 f(x) ≥ (g(x))
a
, 0 < g(x) < 1

Form - II : When the inequality of the form


Form Collection of system
SA

 f(x) ≥ g(x), φ(x) > 1,


(a) logφ(x) f(x) ≥ logφ(x) g(x) ⇔ 
0 < f(x) ≤ g(x);0 < φ(x) < 1
 0 < f(x) ≤ g(x), φ(x) > 1,
(b) logφ(x) f(x) ≤ logφ(x) g(x) ⇔ 
f(x) ≥ g(x) > 0, 0 < φ(x) < 1

Absolute value function / modulus function :


 x if x≥0
The symbol of modulus function is f (x) = x and is defined as: y = x=  .
− x if x<0
Properties of modulus : For any a, b ∈ R
(i) |a| ≥ 0 (ii) |a| = |–a|
(iii) |a| ≥ a, |a| ≥ –a (iv) |ab| = |a| |b|
a |a|
(v) = (vi) |a + b| ≤ |a| + |b| ; Equality holds when ab ≥ 0
b |b|
(vii) |a – b| ≥ ||a| – |b|| ; Equality holds when ab ≥ 0

RI
Irrational function :
An irrational function is a function y = f(x) in which the operations of addition, substraction,
multiplication, division and raising to a fractional power are used.

A
x 3 + x1/ 3
For example y = is an irrational function
2x + x
(a) The equation f(x) = g(x), is equivalent to the following system

UH
f(x) = g2(x) & g(x) ≥ 0
(b) The inequation f(x) < g(x), is equivalent to the following system
f(x) < g2(x) & f(x) ≥ 0 & g(x) ≥ 0
(c) The inequation f(x) > g(x), is equivalent to the following system
JA
g(x) ≤ 0 & f(x) ≥ 0 or g(x) ≥ 0 & f(x) > g 2(x)

Greatest integer function or step up function :


The function y = f (x) = [x] is called the greatest integer function where [x] equals to the
greatest integer less than or equal to x. Graph of greatest integer function is
LP
A
NK

Properties of greatest integer function :


(a) x − 1 < [x] ≤ x (b) [x ± m] = [x] ± m iff m is an integer.
 0; if x is an int eger
SA

(c) [x] + [y] ≤ [x + y] ≤ [x] + [y] + 1 (d) [x] + [− x] = 


 −1 otherwise
Fractional part function:
It is defined as y = {x} = x − [x]. It is always non-negative and varies from [0, 1). The period of this
function is 1 and graph of this function is as shown.
Properties of fractional part function :

0 , if x is an int eger
(a) {x ± m} = {x} iff m is an integer (b) {x} + {–x} = 
1 , otherwise
 1 for x>0
Signum function : f (x) = sgn (x) =  0 for x=0
−1 for x<0

Trigonometric functions of sum or difference of two angles:


(a) sin (A ± B) = sinA cosB ± cosA sinB
(b) cos (A ± B) = cosA cosB  sinA sinB
(c) sin²A − sin²B = cos²B − cos²A = sin (A+B). sin (A− B)

RI
(d) cos²A − sin²B = cos²B − sin²A = cos (A+B). cos (A − B)
tan A ± tan B cot A cot B  1
(e) tan (A ± B) = (f) cot (A ± B) =
1  tan A tan B cot B ± cot A

A
(g) sin (A + B + C) = sin A cos B cos C + sin B cos A cos C + sin C cos A cos B – sin A sin B sin C
(h) cos (A + B + C) = cos A cos B cos C – cos A sin B sin C – sin A cos B sin C – sin A sin B cos C
tan A + tan B + tanC − tan A tan B tan C
(i) tan (A + B + C) = .
1 − tan A tan B − tan B tan C − tan C tan A

UH
S1 − S3 + S5 − ......
(j) tan (θ1 + θ2 + θ3 + ....... + θn) =
1 − S2 + S4 − .......
where Si denotes sum of product of tangent of angles taken i at a time
JA
Transformation formulae :
C+D C−D
(i) sin(A+B) + sin(A − B) = 2 sinA cosB (a) sinC + sinD = 2 sin cos
2 2
C+D C−D
(ii) sin(A+B) − sin(A − B) = 2 cosA sinB (b) sinC − sinD = 2 cos sin
2 2
C+D C−D
(iii) cos(A+B) + cos(A − B) = 2 cosA cosB (c) cosC + cosD= 2 cos cos
LP

2 2
C+D D −C
(iv) cos(A − B) − cos(A+B) = 2 sinA sinB (d) cosC − cosD = 2 sin sin
2 2

Multiple and sub-multiple angles :


A

θ θ
(a) sin 2A = 2 sinA cosA Note : sin θ = 2 sin cos etc.
2 2
(b) cos 2A = cos²A − sin²A = 2cos²A − 1 = 1 − 2 sin²A
NK

θ θ
Note : 2 cos² = 1 + cos θ, 2 sin² = 1 − cos θ.
2 2
2 tan A 2 tan 2θ
(c) tan 2A = Note : tan θ =
1 − tan2 A 1 − tan 2 2θ
SA

2 tan A 1 − tan2 A
(d) sin 2A = , cos 2A =
1 + tan2 A 1 + tan2 A
(e) sin 3A = 3 sinA − 4 sin3A
(f) cos 3A = 4 cos3A − 3 cosA
3 tan A − tan3 A
(g) tan 3A =
1 − 3 tan2 A
Important trigonometric ratios of standard angles :
(a) sin n π = 0 ; cos n π = (−1)n ; tan n π = 0, where n ∈ Ι
π 3 −1 5π
(b) sin 15° or sin = = cos 75° or cos ;
12 2 2 12
π 3 +1 5π
cos 15° or cos = = sin 75° or sin ;
12 2 2 12
3 −1 3 +1
tan 15° = = 2 − 3 = cot 75° ; tan 75° = = 2 + 3 = cot 15°
3 +1 3 −1
π 5 −1 π 5 +1
(c) sin or sin 18° = = cos 72° ; cos 36° or cos = = sin 54°
10 4 5 4

Conditional identities:
If A + B + C = π then :
(i) sin2A + sin2B + sin2C = 4 sinA sinB sinC
A B C
(ii) sinA + sinB + sinC = 4 cos cos cos
2 2 2
(iii) cos 2 A + cos 2 B + cos 2 C = − 1 − 4 cos A cos B cos C
A B C
(iv) cos A + cos B + cos C = 1 + 4 sin sin sin

RI
2 2 2
(v) tanA + tanB + tanC = tanA tanB tanC
A B B C C A
(vi) tan tan + tan tan + tan tan =1
2 2 2 2 2 2

A
A B C A B C
(vii) cot + cot + cot = cot . cot . cot
2 2 2 2 2 2
(viii) cot A cot B + cot B cot C + cot C cot A = 1

UH
Sine and Cosine series:

sin 2  n − 1 
(i) sin α + sin (α + β) + sin (α + 2β ) +...... + sin {α + (n − 1) β} = sin  α + β
β  2 
2
sin

JA
sin 2  n − 1 
(ii) cos α + cos (α + β) + cos (α + 2β ) +.... + cos {α + (n − 1) β} = cos  α + β
β  2 
sin 2
where : β ≠ 2mπ, m ∈ Ι
Product series of cosine angles
sin2n θ
cos θ . cos 2θ . cos22θ . cos23θ ...... cos2n–1θ =
LP

2n sin θ
Range of trigonometric expression:

Range of E = a sin θ + b cos θ is − a 2 + b 2 , a 2 + b 2 


 
A

Trigonometric Equation :
An equation involving one or more trigonometric ratios of an unknown angle is called a trigonometric
NK

equation.

Solution of Trigonometric Equation :


A solution of trigonometric equation is the value of the unknown angle that satisfies the equation.
Thus, the trigonometric equation may have infinite number of solutions (because of their periodic
SA

nature) and can be classified as :


(i) Principal solution (ii) General solution.

Principal solutions :
The solutions of a trigonometric equation which lie in the interval [0, 2π) are called Principal solutions.

General Solution :
The expression involving an integer 'n' which gives all solutions of a trigonometric equation is called
General solution. General solution of some standard trigonometric equations are given below.

General Solution of Some Standard Trigonometric Equations :


 π π
(i) If sin θ = sin α ⇒ θ = n π + (−1)n α where α ∈  − ,  , n ∈ Ι.
 2 2
(ii) If cos θ = cos α ⇒ θ = 2nπ ± α where α ∈ [0, π], n ∈ Ι.
 π π
(iii) If tan θ = tan α ⇒ θ = nπ + α where α ∈  − ,  , n ∈ Ι.
 2 2
(iv) If sin² θ = sin² α ⇒ θ = n π ± α, n ∈ Ι.
(v) If cos² θ = cos² α ⇒ θ = n π ± α, n ∈ Ι.
(vi) If tan² θ = tan² α ⇒ θ = n π ± α, n ∈ Ι. [Note: α is called the principal angle ]

Some Important deductions :


(i) sinθ = 0 ⇒ θ = nπ, n ∈ Ι
π
(ii) sinθ = 1 ⇒ θ = (4n + 1) ,n ∈ Ι
2
π
⇒ θ = (4n – 1) , n ∈ Ι

RI
(iii) sinθ = – 1
2
π
(iv) cosθ = 0 ⇒ θ = (2n + 1) , n ∈ Ι
2
(v) cosθ = 1 ⇒ θ = 2nπ, n ∈ Ι

A
(vi) cosθ = – 1 ⇒ θ = (2n + 1)π, n∈Ι
(vii) tanθ = 0 ⇒ θ = nπ, n ∈ Ι

UH
JA
LP
A
NK
SA
 Marked questions are recommended for Revision.

SUBJECTIVE QUESTIONS

Section (A) : Representation of sets, Types of sets, subset and power set

A-1. Which of the following collections is not a set ?


(i) The collection of natural numbers between 2 and 20
(ii) The collectihon of numbers which satisfy the equation x 2 –5x + 6 = 0

RI
(iii) The collection of prime numbers between 1 and 100.
(iv) The collection of all intelligent women in Jalandhar.

A-2. Write the set A = {x : x is a positive prime < 10} in the tabular form

A
A-3. Which of the following is the empty / non-empty set
(i) {x : x is a real number and x2 – 1 = 0} (ii) {x : x is a real number and x2 + 1 = 0}

UH
(iii) {x : x is a real number and x – 9 = 0}
2
(iv) {x : x is a real number and x2 = x + 2}

A-4. Which of the folowing sets is an finite / infinite set ?


(i) Set of divisors of 24
(ii) Set of all real number which lie between 1 and 2
JA
(iii) Set of all humman beings living in India.
(iv) Set of all three digit natural numbers

A-5. If A = {x : –3 < x < 3, x ∈ Z} then find the number of subsets of A.

A-6. Find Power set of the set A = {φ, {φ}}.


LP

Section (B) : Operations on sets, Law of Algebra of sets

B-1. Sets A and B have 3 and 6 elements respectively. What can be the minimum number of elements in
A∪B?
A

B-2. Given the sets A = {1, 2, 3}, B = {3, 4}, C = {4, 5, 6}, then find A ∪ (B ∩ C).
NK

B-3. Let A = {x : x ∈ R, –1 < x < 1} , B = {x : x ∈ R, x ≤ 0 or x ≥ 2} and A ∪ B = R – D, then find set D

B-4. Find the smallest set A such that A ∪ {1, 2} = {1, 2, 3, 5, 9}

B-5. If A = {2, 3, 4, 8, 10}, B = {3, 4, 5, 10, 12}, C = {4, 5, 6, 12, 14} then find (A ∩ B) ∪ (A ∩ C).
SA

B-6. Let U = {1, 2, 3, 4, 5, 6, 7, 8, 9, 10}, A = {1, 2, 5}, B = {6, 7}, then show that A ∩ B′ is same as set A.

B-7. If A = {x : x = 4n + 1, n ≤ 5, n ∈ N} and B {3n : n ≤ 8, n ∈ N}, then find (A – (A – B)).

B-8. If aN = {ax : x ∈ N} and bN ∩ cN = dN, where b, c ∈ N, b ≥ 2, c ≥ 2 are relatively prime, then find
relation between d,b and c.

Section (C) : Cardinal number Problems

C-1. Let n(U) = 700, n(A) = 200, n(B) = 300 and n(A ∩ B) = 100, then find n(A' ∩ B')
C-2. In a group of 1000 people, there are 750 people, who can speak Hindi and 400 people, who can speak
Bengali.
(i) Find number of people who can speak Hindi only.
(ii) Number of people who can speak Bengali only is
(iii) Number of people who can speak both Hindi and Bengali is

C-3. A class has 175 students. The following data shows the number of students obtaining one or more
subjects : Mathematics 100, Physics 70, Chemistry 40, Mathematics and Physics 30, Mathematics and
Chemistry 28, Physics and Chemistry 23, Mathematics & Physics & Chemistry 18. How many students
have offered Mathematics alone ?

C-4. 31 candidates appeared for an examination, 15 candidates passed in English, 15 candidates passed in
Hindi, 20 candidates passed in Sanskrit. 3 candidates passed only in English. 4. candidates passed
only in Hindi, 7 candidates passed only in Sanskrit. 2 candidates passed in all the three subjects How
many candidates passed only in two subjects ?

RI
C-5. In a survery, it was found that 21 persons liked product A, 26 liked product B and 29 liked product C. If
14 persons liked products A and B, 12 liked products C and A, 13 persons liked products B and C and
8 liked all the three products then

A
(i) Find the number of persons who liked the product C only
(ii) The number of persons who like the products A and B but not C

Section (D) : Graphs of polynomial

UH
D-1. Draw the graph of following function
(i) y = 4x3 – 30x2 + 72x – 55 (ii) y = x 3 + x2 + x – 3
(iii) y = x4 – 8x3 + 22x2 – 24x + 8.5 (iv) y = x4 – 6x2 – 8x + 13
(v) y = x4 – 4x3 + 8x2 – 8x – 21 (vi) y = x4 + 2x2 + 4x + 1
JA
D-2. Find the number of solution of the following equation x 4 – 6x2 – 8x – 3 = 0

D-3. Find the range of 'λ' for which equation x3 + x2 – x – 1 – λ = 0 has 3 real solution.

Section (E) : Rational inequaties, Modulus & Graphical transformations


LP

E-1. Solve the following rational in equalities


(x – 1)(x + 2) (1– x)3 (x + 2)4
(i) <0 (ii) ≥0
(x – 3)(x + 3) (x + 9)2 (x – 8)
A

(x 2 – 3x + 1)3 x(2x – 3 x )
(iii) ≤0 (iv) >0
(x − 1)(x + 2) (x 2 + x + 1)(x – 1)
(x – 1)(x – 2)(x – 3) 3x 2 – 7x + 8
NK

(v) ≤1 (vi) 1< ≤2


(x + 1)(x + 2)(x + 3) x2 + 1

(3 x – 5 x )(x – 2)
E-2. Find the number of positive integral value of x satisfying the inequality ≥0
(x 2 + 5x + 2)
SA

x –1
E-3. If 1 < < 7 then find the range of
x+2
1
(i) x (ii) x2 (iii)
x

E-4. Define and plot


(i) y = |x – 2| + 3 |x – 3| (ii) y = ||x – 2| – 3| + |x|
(iii) y = |x – 1| + |x – 4| – 2 |x + 1|

E-5. Solve for x


(i) |x + 1| = 4x + 3
(ii) |x + 1| = |x + 3|
(iii) 7|x – 2| – |x – 7| = 5
(iv) ||x – 1| – 2| = 6x + 8
(v) |2x2 – 3x + 1| = |x2 + x – 3|

E-6. Solve for x


(i) 2|x+1| + 2|x| = 6 and x∈Ι
(ii) x + x + 1 + |x – 3| ≤ |x + 2x – 2|
2 2

(iii) |2x – 4| – 2|x2 + x – 3| + 2|x – 1||x + 1| = 0

E-7. Solve the following in equalities


(i) |x + 7| > 5 (ii) |x + 3| < 10
2x – 1
(iii) (x + 2) < |x2 + 3x + 5| (iv) >2
x –1
x 2 – | x | –12

RI
(v) |x – 6| ≤ x2 – 5x + 9 (vi) ≥ 2x
x–3
(vii) (|x – 1| – 3) (|x + 2| – 5) < 0 (viii) |x – 1| + |x – 2| + |x – 3| ≤ 6

A
E-8. Find the number of solution of the following equation
(i) |||x – 1| – 2| – 1| = 1 (ii) 2|(x – 1) (x – 5)| = (x – 3)2

E-9. If graph of y = (x – 1)(x – 2) is

(0,2)

UH
1
JA
2

–1/4

then draw the graph of the following


LP

(i) y = |(x – 1) (x – 2)| (ii) |y| = (x – 1) (x – 2)


(iii) y = (|x| – 1) (|x| – 2) (iv) y = |(|x| – 1) (|x| – 2)|
(v) |y| = |(|x| – 1) (|x| – 2)|

E-10. Let graph of y = f(x) is


A

(–6,2) (6,2)
(0,2)
(1,1) (2,1)
NK

(–1,1)
(7,0)

(–4,0) (–2,0) (3,0) (5,0)


(–3,–1) (4,–1) (8,–1)

Now draw the graph of following


SA

(i) y = 2f (–x)
(ii) y =f (|x + 1|)
(iii) y = – f(|x| + 1)
(iv) |y + 1| = f(2x – 1) ∀ x ∈ [–1,3]
Section (F) : Irrational inequality, logarithmic equation & logarithimic inequality
F-1. Solve the following inequlities
(i) x –1 < x – 3 (ii) x–3 > 7–x
(iii) x 2 + 4x + 9 > x + 2 (iv) 4 – x < 2x – x 2
(v) 3
(x – 4)(x – 6) > 2 (vi) x 2 + 3x + 5 < x2 + x + 1
1 3 1 1 2x 2 + 7x – 4 1
(vii) 2
– < – (viii) ≤
x x x 2 x+4 2
| x+2| – | x|
(ix) ≥0
8 – x3

F-2. Find the value of

RI
1
log7 5 1
(i) (log105)(log1020) + (log102)2 (ii) 3 5 +
( − log10 0.1)

A
1 log 2
(iii) log0.75 log2 (iv) 5 5
+ 9log3 7 − 8log2 5
0.125
1+ log7 2
 1 

UH
log3 5 log5 7 log3 7 log5 3
(v)  49  + 5 – log1/ 5 7 (vi) 7 +3 −5 −7
 

F-3. Let log102 = a and log103 = b determine the following in term of a and b
(i) log4100 + 2log27100 (ii) log144 45
JA
F-4. Prove that
1 1 1
(i) + + =1 (ii) (log210) (log280) – (log25) (log2160) = 4
log bc (abc) log ca abc log ab (abc)
log 2 2
+1)3
loga b logb a a 21/ 4
– 3log27 (a – 2a
(iii) a = b (iv) = a2 + a + 1
( 7 4log49 a
) – a –1
LP

F-5 Solve the following equations :


(i) logx(4x – 3) = 2 (ii) log2(x – 1) + log2(x – 3) = 3
(iii) log2 (log8 (x2 – 1)) = 0 (iv) 4log2 x − 2x − 3 = 0
A

F-6. Solve the following equations


log x +7
log2 (9 – 2x ) 10

(i) =1 (ii) x 4 = 10(log10 x +1)


3–x
NK

(iii) (log10(100x))2 + (log10(10x))2 = 14 + log10(1/x)


(iv) log105 + log10(x + 10) – 1 = log10(21x – 20) – log10(2x – 1)
(v) 52x = 32x + 2.5x + 2.3x
F-7 Solve the following inequalities
(i) log5 (3x – 1) < 1 (ii) (log.5x)2 + log.5x – 2 ≤ 0
SA

(iii) log3(x + 1) + log3(x + 7) ≥ 3 (iv) log1/2 log3(x2 + 5) + 1 ≤ 0


F-8. Solve the following inequalities
x –1
(i) |log3x| – log3x – 3 < 0 (ii) ≤1
log3 (9 – 3 x ) – 3
(iii) log x −1 (x–2) > 0 (iv) logx (x3 – x2 – 2x) < 3
x −5
F-9 Solve the following inequalities
 3x – 2 
x
2
(i) 15x – 25.3x – 9.5x + 225 ≥ 0 (ii) 8.  x x 
>1+  
3 –2  3
Section (G) : Greatest integer function, fractional part & signum function

G-1. Solve for x (where [•] denotes greatest integer function and {•} represent fractional part function)
(i) [2x] = 1 (ii) {x}2 + [x] = 2 (iii) 6{x}2 – 5{x} + 1 = 0 (iv) 6[x]2 – 5[x] – 1 = 0

G-2 Solve the following equations


(where [•] denotes greatest integer function and {•} represent fractional part function)
(i) 2[x] + 3{x} = 4x – 1 (ii) 4[x] = x + {x} (iii) [x] + 2{–x} = 3x

G-3. Solve the following equations (where [•] denotes greatest integer function and {•} represent fractional
part function and sgn represents signum function)
(i) [x] + |x – 2| ≤ 0 and x ∈ [–1,3] (ii) [2x] – 2x = [x + 1]
(iii) [x2] + 2 [x] = 3x, 0 ≤ x ≤ 2

G-4. Solve the following inequalities (where [•] denotes greatest integer function and {•} represent fractional
part function)

RI
1
(i) [x + [x]] < 0 (ii) [2x2 – x] < 1 (iii) {x} <
2

G-5 Solve the following equations

A
 x 2 − 5x + 4 
(i) sgn ({[x]}) = 0 (ii) sgn(x2 – 2x – 8) = –1 (iii) sgn  = – 1
 {x} 

UH
G-6 Find the number of solution of equation (where sgn represent signum function)
(i) sgn (x) = |x| (ii) sgn (x2 – 1) = (x + 1)2

Section (H) : Trigonometric Equations


JA
H-1. Solve the following equation
θ π π
(i) 5 cos2θ + 2cos2 + 1 = 0, – < θ <
2 2 2
(ii) sin7θ + sin4θ + sinθ = 0 , 0 ≤ θ ≤ π
(iii) tanθ + sec θ = 3 , 0 ≤ θ ≤ 2π
LP

H-2. Find the most general solution of the following


(i) sin6x = sin4x – sin2x
(ii) sec4x – sec2x = 2
π  π 
A

(iii) tan  + x  + tan  – x  = 2


4  4 

H-3. Solve the following inequations


NK

(i) (sinx – 2) (2sinx–1) < 0 (ii) (2cosx – 1) (cosx) ≤ 0


(iii) sinx + 3 cosx ≥ 1 (iv) cos2x + sinx ≤ 2
(v) tan2x > 3

H-4 Find the number of solution of the following equation


SA

x
(i) |sinx| = (ii) n|x| = sinπx
10

1
1
≤1
2
H-5. Solve the inequation 2 cos x
y2 – y +
2
Marked questions are recommended for Revision.
* Marked Questions may have more than one correct option.

OBJECTIVE QUESTIONS
1. Which of the following are true ?
(A) [3, 7] ⊆ (2, 10) (B) (0, ∞) ⊆ (4, ∞) (C) (5, 7] ⊆ [5, 7) (D) [2, 7] ⊆ (2.9, 8)

2. The shaded region in the given figure is

RI
(A) A ∩ (B ∪ C) (B) A ∪ (B ∩ C) (C) A ∩ (B – C) (D) A – (B ∪ C)

A
3. Consider the following statements :
1. N ∪ (B ∩ Z) = (N ∪ B) ∩ Z for any subset B of R, where N is the set of positive integers, Z is

UH
the set of integers, R is the set of real numbers.
2. Let A = {n ∈ N : 1 ≤ n ≤ 24, n is a multiple of 3}. There exists no subset B of N such that the
number of elemets in A is equal to the number of elements in B.
Which of the above statements is/are correct ?
(A) 1 only (B) 2 only (C) Both 1 and 2 (D) Neither 1 nor 2
JA
4. Which of the following venn-diagrams best represents the sets of females, mothers and doctors ?
LP

5. In a college of 300 students, every student reads 5 newspapers and every newspaper is read by 60
students. The number of newspaper is-
(A) at least 30 (B) at most 20 (C) exactly 25 (D) exactly 30

6. In a town of 10,000 families it was found that 40% families buy newspaper A, 20% families buy
A

newspaper B and 10% families buy newspaper C, 5% families buy A and B, 3 % buy B and C and 4%
buy A and C. If 2% families buy all the three news papers, then number of families which buy
newspaper A only is
NK

(A) 3100 (B) 3300 (C) 2900 (D) 1400

7. Let A1, A2 and A3 be subsets of a set X. Which one of the following is correct ?
(A) A1 ∪ A2 ∪ A3 is the smallest subset of X containing elements of each of A1, A2 and A3
(B) A1 ∪ A2 ∪ A3 is the smallest subset of X containing either A1 or A2 ∪ A3 but not both
SA

(C) The smallest subset of X containing A1 ∪ A2 and A3 equals the smallest subset of X containing
both A1 and A2 ∪ A3 only if A2 = A3
(D) None of these
8. Let A, B, C be distinct subsets of a universal set U. For a subset X of U, let X ' denote the complement
of X in U.
Consider the following sets :
1. ((A ∩ B) ∪ C)′ ∩ B′)′ = B ∩ C
2. (A ′ ∩ B ′) ∩ (A ∪ B ∪ C′) = (A ∪ (B ∪ C))′
Which of the above statements is/are correct ?
(A) 1 only (B) 2 only (C) Both 1 and 2 (D) Neither 1 nor 2

9. Let U be set with number of elements in U is 2009.


Consider the following statements :
Ι : If A, B are subsets of U with n (A ∪ B) = 280, then n(A′ ∩ B′) = x13 + x 32 = y13 + y 32
for some positive integers x1, x2 y1, y2
ΙΙ : If A is a subset of U with n (A) = 1681 and out of these 1681 elements, exactly 1075 elements
belong to a subset B of U , then n (A – B) = m2 + p1 p2 p3 for some positive integer m and distinct primes
p1, p2, p3
Which of the statements given above is / are correct ?
(A) I only (B) II only (C) Both I and II (D) Neither I nor II.

10. Consider the following statements :


1. If A = {(x, y) ∈ [R × R : x3 + y3 = 1] and B = {(x, y) ∈ [R : x – y = 1]}, then A ∩ B contains exactly one
elements.
2. If A = {(x, y) ∈ [R × R : x 3 + y3 = 1] and B = {(x, y) ∈ [R : x + y = 1]}, then A ∩ B contains exactly two

RI
elements.
Which of the above statements is/are correct ?
(A) 1 only (B) 2 only (C) Both 1 and 2 (D) Neither 1 and 2

11. In a class of 42 students, the number of students studying different subjects are 23 in Mathematics, 24

A
in Physics, 19 in Chemistry, 12 in Mathematics and Physics 9 in Mathematics and Chemistry, 7 in
Physics and Chemistry and 4 in all the three subjects. The number of students who have taken exactly
one subject is

UH
(A) 15 (B) 30 (C) 22 (D) 27

12. In an examination of a certain class, at least 70% of the students failed in Physics, at least 72% failed in
Chemistry, at least 80% failed in Mathematics and at least 85% failed in English. How many at least
must have failed in all the four subjects ?
(A) 5%
JA (B) 7%
(C) 15% (D) Cannot be determined due to insufficient data

13.* A and B are two sets such that n(A) = 3 and n(B) = 6, then
(A) minimum value of n(A ∪ B) = 6 (B) minimum value of n(A ∪ B) = 9
(C) maximum value of n(A ∪ B) = 6 (D) maximum value of n(A ∪ B) = 9
LP

14. The number of solution of equation x3 – 21x –20 = ex is/are


(A) 0 (B) 1 (C) 2 (D) 3

x 2 – 2x + 2|a|
15. If a ≠ 0 then complete set of solution of > 0 is
A

x 2 – a2
(A) (–∞, –|a|) ∪ (|a|, ∞) (B) (–∞,–a) ∪ (a,∞)
(C) (–∞,–a) ∪ [a,∞) (D) (–|a|, |a|)
NK

|x| 1
16. The complete set of solution of equation 1– = is {a,b} then
1+ | x | 2
(A) a + b = 8 (B) a + b = 3 (C) a + b = 0 (D) a + b = – 3
SA

17. The smallest integral value of a such that |x + a – 3| + |x –2a| = |2x – a –3| is true ∀ x ∈ R is
(A) 0 (B) 1 (C) 2 (D) 3

18. Number of positive integral solution of the equation |x 2 – 3x – 3| > |x2 + 7x – 13| is/are
(A) 0 (B) 1 (C) 2 (D) 3

 2 | x | –2 
19. If (x2 – 2|x|) (|2x| – 2) – 9  2  ≤ 0 then
 x – 2 | x |
(A) x ∈ (–∞, – 1] ∪ (0,1] ∪ (2,3] (B) x ∈ (–∞, – 1] ∪ (2,3] ∪ {1}
(C) x ∈ (–∞,3] (D) x ∈ [–3 – 2) ∪ [–1,0) ∪ (0,1] ∪ (2,3]
x+2
20. Number of solution of pair of equations y = ||x| – 2| – 2| and y = equals to
2
(A) 1 (B) 2 (C) 3 (D) 4
21*. For making graph of equations |y| = |f(|x|)| through y = f(x) which order of step is right among the
following order of
Step Ι : y = f(|x|) (replace x by |x|)
Step ΙΙ : y = |f(x)| (take modulus of R.H.S)
Step ΙΙΙ : |y| = f(x) (replace y by |y|)
(A) Ι, ΙΙ, ΙΙΙ (B) ΙΙ, Ι, ΙΙΙ (C) ΙΙΙ, ΙΙ, Ι (D) ΙΙΙ, Ι, ΙΙ

22. The sum of all the integral values of a {where a ∈ [–10, 10)} such that the graph of the function
f(x) = ||x – 2| – a| – 3 has exactly three x-intercepts is

(A) 10 (B) 5 (C) 3 (D) 0

23. Let graph of y = f(x) is

RI
(1, 1)

A
(–1, –1)

UH
and graph of y = g(x) is
(1, 1)
JA
(–1, –1)
The graph of y = |f(x) + g(x)| is same as
(A) y = |f(x)| + |g(x)| (B) y = 2|x| (C) y = |x| (D) y = 2

3
LP

24. Complete set of solution of inequation – 2 – x < 2 is


2– x
(A) (–∞,1) (B) (–∞,1] (C) (1,∞) (D) [1,∞)

25. Complete set of solution of inequation 3x 2 + 5x + 7 – 3x 2 + 5x + 2 > 1 is (–a, – b] ∪ [–c,d) (where a,


A

b, c ∈ R+) then a + b + c + d equation


(A) 4 (B) 3 (C) 2 (D) 1
NK

3
26. Complete set of solution of a inequation x + 2 x –1 + x – 2 x –1 >
2
(A) (–∞,1) (B) (–∞,1] (C) [1, ∞) (D) (1, ∞)

27. If a, b, c are distinct positive number but no one among them is equal to one and
SA

logba logca + logab logcb + logac logbc = 3, then value of abc is


(A) 2 (B) 3 (C) 0 (D) 1

1
28. Let α, β, are two real solution of equation (log 10x)2 + log10x2 = (log102)2 –1, then equal to
αβ
(A) 20 (B) 3 (C) 10 (D) 1

29. Let a, b, c, d are positive integer such that logab = 3/2 and logcd = 5/4. If a – c = 9, then value of (b – d)
is equal to
(A) 20 (B) 93 (C) 10 (D) 1

30. The values of a for which the equation 2(log3x)2 – |log3x| + a = 0 posses four real solution
1 1
(A) –2 < a < 0 (B) 0 < a < (C) 0 < a < 5 (D) – < a < 0
8 8
x 2 + 6x + 9
31. If log 1 < – log2(x + 1) then complete set of values of x is
2 2(x + 1)
(A) (–1, 1 + 2 2 ) (
(B) 1– 2 2,2 ) (C) (–1, ∞) (D) ( 1– 2 2, 1 + 2 2 )

32. The least positive integer x, which satisfies the inequality log x
(x2 – 10x + 22) > 0 is equal to
log2  
2

(A) 3 (B) 4 (C) 7 (D) 8

log0.3 (x – 2)
33. Complete set of solution of equation ≥0
|x|
(A) [1, 2) ∪ (2, 3]

RI
(B) [1, 3] (C) (2, 3] (D) {1}

34. The solution set of the inequality |9x – 3x+1 + 15| < 2.9x – 3x is
(A) (–∞,1) (B) (1,∞) (C) (–∞,1] (D) [2, ∞)

A
35. The complete set of values of x satisfying the equation x 2 . 2x+1 + 2|x–3| + 2 = x2.2|x–3|+ 4 + 2x–1 is
 1 1  1  1 1
(A) [3,∞) (B)  – ,  ∪ (3,∞) (C)  – ∞ –  (D)  – ,  ∪ [3,∞)

UH
 2 2  2  2 2

  x     x     x    x 
36. If f(x) = {x} +  x +  2 
+ x +  2 
+ x +  2 
........+  x +  2 
, then values of [f(
 1 + x    1 + 2x    1 + 3x    1 + 99x  
JA
3 )] is (where [•] denotes greatest integer function and {•} represent fractional part function)
(A) 5050 (B) 4950 (C) 17 (D) 73

37. The number of solution of the equation sgn ({x}) = |1– x| is/are (where {•} represent fractional part
function and sgn respresent signum function)
(A) 4 (B) 3 (C) 2 (D) 1
LP

x 2 – 5x + 6 sgn(x)
38. The complete set of solution of Inequality ≥ 0 is (where sgn respresent signum
xsgn(x – 1) + 1
function)
(A) (–∞, –1] ∪ [0,2] ∪ [3,∞) (B) (–∞, 0] ∪ [2,∞)
A

(C) (–∞,2] ∪ [3,∞) (D) (–∞, –1] ∪ [0,∞)

 3 3  2π  3  4π 
NK

1
39. sin α + sin  3 + α  + sin  3 + α   is equal to
sin3α     
4 3 –3 –4
(A) (B) (C) (D)
3 4 4 3
SA

40.* If (m + 2) sinθ + (2m – 1) cosθ = 2m + 1 then


3 2m 2m 4
(A) tanθ = (B) tanθ = 2 (C) tanθ = (D) tanθ =
4 m +1 m2 – 1 3

π
41. * Let 0 ≤ θ ≤ and x = X cosθ + Ysinθ , y = Xsinθ – Ycosθ such that x 2 + 4xy + y2 = aX2 + bY2,
2
where a, b are constants then
π
(A) a = –1, b = 3 (B) θ = π/4 (C) a = 3, b = – 1 (D) θ =
3

42. Let (1 + tan 1º) (1 + tan2º) ............(1 + tan45º) = 2 k then k equals to


(A) 21 (B) 22 (C) 23 (D) 24
43. The number of solution of 2 cosx = |sinx| where x ∈ [0,4π] is/are
(A) 2 (B) 3 (C) 4 (D) 1

44. * If the equation sin (πx2) – sin(πx2 + 2πx) = 0 is solved for positive roots, then in the increasing sequence
of positive root
−1 + 7 −1 + 3
(A) first term is (B) first term is
2 2
−1 + 11
(C) third term is 1 (D) third term is
2

45. In (0, 6π), the number of solutions of the equation tanθ + tan 2θ + tan 3θ = tan θ.tan2θ.tan3θ is /are
(A) 15 (B) 17 (C) 20 (D) 12

RI
 nπ 
46. If 2tan2x – 5 secx – 1 = 0 has 7 different roots in 0, , n ∈ N, then the greatest value of n is
 2 

A
(A) 15 (B) 13 (C) 14 (D) 16

UH
47. The number of integral values of a for which the equation cos 2x + a sin x = 2a − 7 possesses a
solution.
(A) 0 (B) 1 (C) 3 (D) 5

48. If the arithmetic mean of the roots of the equation 4cos 3x – 4cos2x – cos(π + x) – 1 = 0 in the interval
JA
[0, 315] is equal to kπ , then the value of k is
(A) 10 (B) 20 (C) 50 (D) 80

49. Number of solution of sinx cosx – 3 cosx + 4 sinx – 13 > 0 in [0,2π] is equal to
(A) 0 (B) 1 (C) 2 (D) 4
LP

50. The solution of 5 − 2 sin x ≥ 6 sin x – 1 is


(A) [π (12n – 7)/6, π (12n + 7)/6] (n ∈ Z) (B) [π (12n – 7)/6, π (12n + 1)/6] (n ∈ Z)
(C) [π (2n – 7)/6, π (2n + 1)/6] (n ∈ Z) (D) [π (12n – 7)/3, π (12n + 1)/3] (n ∈ Z)
A
NK
SA
* Marked Questions may have more than one correct option.
Marked questions are recommended for Revision.
PART - I : JEE (ADVANCED) / IIT-JEE PROBLEMS (PREVIOUS YEARS)

sin4 x cos4 x 1
1.* If + = , then [IIT-JEE - 2009 ,Paper-1, (4, –1), 80]
2 3 5
8
2 sin8 x cos x 1
(A) tan2x = (B) + =
3 8 27 125
1 sin8 x cos8 x 2
(C) tan2 x = (D) + =

RI
3 8 27 125

π 6
 (m − 1) π   mπ 
2*. For 0 < θ <
2
, the solution(s) of ∑ cos ec  θ +
 4 

cos ec  θ +
 4 
= 4 2 is(are)

A
m =1

[IIT-JEE - 2009, Paper-2, (4, –1), 80]


π π π 5π
(A) (B) (C) (D)

UH
4 6 12 12

1
3. The maximum value of the expression is
sin2 θ + 3 sin θ cos θ + 5cos2 θ
[IIT-JEE-2010, Paper-1, (3, 0)/84]
JA
1 1 1
4. The positive integer value of n > 3 satisfying the equation = + is
π  2π   3π 
sin   sin   sin  
n  n   n 
[IIT-JEE-2010, Paper-1, (3, 0)/84]
LP

 π π nπ
5. The number of values of θ in the interval  – ,  such that θ ≠ 5 for n = 0, ±1, ± 2 and
 2 2
tanθ = cot 5θ as well as sin 2θ = cos 4θ is [IIT-JEE-2010, Paper-1, (3, 0)/84]
A

6. Let (x0, y0) be the solution of the following equations


(2x)n2 = (3y)n3
3nx = 2ny .
NK

Then x0 is [IIT-JEE 2011, Paper-1, (3, –1), 80]


1 1 1
(A) (B) (C) (D) 6
6 3 2

7. Let P = {θ : sin θ – cos θ = 2 cos θ} and Q = {θ : sin θ + cos θ = 2 sin θ} be two sets. Then
SA

(A) P ⊂ Q and Q – P ≠ ∅ (B) Q ⊂ / P


(C) P ⊂/ Q (D) P = Q [IIT-JEE 2011, Paper-1, (3, –1), 80]

 θ θ
8.* Let θ, φ ∈ [0, 2π] be such that 2cosθ(1 – sinφ) =  tan + cot  sin2θ cosφ – 1, tan(2π – θ) > 0 and
 2 2
3
–1 < sinθ < – . Then φ cannot satisfy [IIT-JEE 2012, Paper-1, (4, 0), 70]
2
π π 4π 4π 3π 3π
(A) 0 < φ < (B) < φ< (C) < φ< (D) < φ < 2π
2 2 3 3 2 2
 
1 1 1 1
9. The value of 6 + log3  4– 4– 4– ....  is [IIT-JEE 2012, Paper-1, (4, 0), 70]

2 3 2 3 2 3 2 3 2 
 

10.* If 3x = 4x – 1 , then x = [JEE (Advanced) 2013, Paper-2, (3, –1)/60]


2log3 2 2 1 2log2 3
(A) (B) (C) (D)
2log3 2 – 1 2 – log2 3 1– log4 3 2log2 3 – 1

11. For x ∈ (0, π), the equation sinx + 2 sin 2x – sin 3x = 3 has
[JEE (Advanced) 2014, Paper-2, (3, –1)/60]
(A) infinitely many solutions (B) three solutions
(C) one solution (D) no solution

RI
12. The number of distinct solutions of the equation [JEE (Advanced) 2015, P-1 (4, 0) /88]
5
cos2 2x + cos4 x + sin4 x + cos6 x + sin6x = 2 in the interval [0, 2π] is
4

A
π π
13. Let – < θ<– . Suppose α1 and β1 are the roots of the equation x2 – 2x sec θ + 1 = 0 and α2 and
6 12

UH
β2 are the roots of the equation x2 + 2x tan θ – 1 = 0. If α1 > β1 and α2 > β2 , then α1 + β2 equals
(A) 2(secθ – tan θ) (B) 2sec θ (C) – 2tan θ (D) 0
[JEE (Advanced) 2016, Paper-1, (3, –1)/62]

 π
JA
14. Let S = x ∈ (– π, π) : x ≠ 0,±  . The sum of all distinct solutions of the equation
 2
3 sec x + cosec x + 2(tan x – cot x) = 0 in the set S is equal to
[JEE (Advanced) 2016, Paper-1, (3, –1)/62]

7π 2π 5π
LP

(A) – (B) – (C) 0 (D)


9 9 9

13


1
15. The value of is equal to [JEE (Advanced) 2016, Paper-2 (3, –1)/62]
 π (k − 1)π   π kπ 
A

k = 1 sin +  sin + 
4 6  4 6 
(A) 3 − 3 (
(B) 2 3 − 3 ) (
(C) 2 3 − 1 ) (
(D) 2 2 + 3 )
NK

16. Let α and β be nonzero real numbers such that 2(cos β – cos α) + cos α cos β = 1. Then which of the
following is/are true? [JEE(Advanced) 2017, Paper-2,(4, –2)/61]
α
  β
  α β
(A) 3 tan   – tan   = 0 (B) tan   – 3 tan   = 0
2 2 2 2
SA

α β α β


(C) tan   + 3 tan   = 0 (D) 3 tan   + tan   = 0
 
2  
2  
2 2
1 1

17. The value of ((log 2 9)2 ) log2 (log2 9) × ( 7 ) log4 7 is ___________.[JEE(Advanced) 2018, Paper-1,(3, 0)/60]
PART - II : JEE (MAIN) / AIEEE PROBLEMS (PREVIOUS YEARS)
1. If A, B and C are three sets such that A ∩ B = A ∩ C and A ∪ B = A ∪ C, then
[AIEEE-2009, (4, – 1), 144]
(1) A = C (2) B = C (3) A ∩ B = φ (4) A = B

2. Let A and B denote the statements [AIEEE 2009 (4, –1), 144]
A : cos α + cos β + cos γ = 0
B : sin α + sin β + sin γ = 0
3
If cos (β – γ) + cos (γ – α) + cos (α – β) = – , then :
2
(1) A is false and B is true (2) both A and B are true

RI
(3) both A and B are false (4) A is true and B is false

4 5 π
3. Let cos(α + β) = and let sin(α – β) = , where 0 ≤ α, β ≤ . Then tan 2α =

A
5 13 4
[AIEEE 2010 (4, –1), 144]
56 19 20 25
(1) (2) (3) (4)

UH
33 12 7 16

4. If A = sin2 x + cos4 x, then for all real x : [AIEEE 2011 (4, –1), 120]
3 13 3 13
(1) ≤ A ≤ 1 (2) ≤ A ≤1 (3) 1 ≤ A ≤ 2 (4) ≤ A ≤
4 16
JA 4 16

5. Let X = {1, 2, 3, 4, 5}. The number of different ordered pairs (Y, Z) that can formed such that
Y ⊆ X, Z ⊆ X and Y ∩ Z is empty, is : [AIEEE-2012, (4, – 1), 120]
(1) 52 (2) 35 (3) 25 (4) 53
LP

6. In a ∆PQR, if 3 sin P + 4 cos Q = 6 and 4 sin Q + 3 cos P = 1, then the angle R is equal to :
[AIEEE-2012, (4, –1)/120]
5π π π 3π
(1) (2) (3) (4)
6 6 4 4
A

7. Let A and B two sets containing 2 elements and 4 elements respectively. The number of subsets of
A × B having 3 or more elements is [AIEEE - 2013, (4, –1), 120]
NK

(1) 256 (2) 220 (3) 219 (4) 211

tan A cot A
8. The expression + can be written as : [AIEEE - 2013, (4, –1),120]
1 − cot A 1 − tan A
(1) sinA cosA + 1 (2) secA cosecA + 1 (3) tanA + cotA (4) secA + cosecA
SA

9. If X = {4n – 3n – 1 : n ∈ N} and Y = {9(n – 1) : n ∈ N}, where N is the set of natural numbers, then X ∪ Y
is equal to [JEE(Main)2014,(4, – 1), 120]
(1) X (2) Y (3) N (4) Y – X
1
10. Let fk (x) = (sinkx + coskx) where x ∈ R and k ≥ 1. Then f4(x) – f6(x) equals
k
[JEE(Main)2014,(4, – 1), 120]
1 1 1 1
(1) (2) (3) (4)
4 12 6 3

11. Let A and B be two sets containing four and two elements respectively. Then the number of subsets of
the set A × B, each having at least three elements is: [JEE(Main)2015,(4, –1), 120]
(1) 219 (2) 256 (3) 275 (4) 510

12. If the angles of elevation of the top of a tower from three collinear points A, B and C, on a line leading to
the foot of the tower, are 30º, 45º and 60º respectively, then the ratio, AB : BC , is

RI
[JEE(Main)2015,(4, – 1), 120]
(1) 3 :1 (2) 3: 2 (3) 1 : 3 (4) 2 : 3

If 0 ≤ x < 2π, then the number of real values of x, which satisfy the equation

A
13.
cosx + cos2x + cos3x + cos4x = 0, is [JEE Main 2016, (4, –1),120]
(1) 5 (2) 7 (3) 9 (4) 3

UH
14. If 5(tan2 x – cos2x) = 2cos2x + 9, then the value of cos4x is : [JEE Main 2017, (4, –1),120]
−3 1 2 7
(1) (2) (3) (4) −
5 3 9 9
JA
15. Let a vertical tower AB have its end A on the level ground. Let C be the mid-point of AB and P be a
point on the ground such that AP = 2AB. If ∠BPC = β, then tanβ is equal to
[JEE Main 2017, (4, –1),120]
6 1 2 4
(1) (2) (3) (4)
7 4 9 9
LP

16. Let S = {x ∈ R : x ≥ 0 and 2| x – 3| + x ( x – 6) + 6 = 0}. Then S : [JEE Main 2017, (4, –1),120]
(1) contains exactly two elements. (2) contains exactly four elements.
(3) is an empty set. (4) contains exactly one element
A

 π  π  1
17. If sum of all the solutions of the equation 8 cosx.  cos  + x  .cos  − x  −  = 1 in [0, π] is kπ, then k
 6  6  2
NK

is equal to : [JEE(Main)2018,(4, – 1), 120]


8 20 2 13
(1) (2) (3) (4)
9 9 3 9

18. PQR is a triangular park with PQ = PR = 200 m. A T.V. tower stands at the mid-point of QR. If the
SA

angles of elevation of the top of the tower at P, Q and R are respectively 45º, 30º and 30º, then the
height of the tower (in m) is : [JEE(Main)2018,(4, – 1), 120]
(1) 100 3 (2) 50 2 (3) 100 (4) 50

19. Let S = {x ∈ R : x ≥ 0 and 2| x – 3| + x ( x – 6) + 6 = 0}. Then S : [JEE(Main)2018,(4, – 1), 120]


(1) contains exactly two elements. (2) contains exactly four elements.
(3) is an empty set. (4) contains exactly one element

20. In a class 140 students numbered 1 to 140, all even numbered students opted Mathematics course,
those whose number is divisible by 3 opted Physics course and those whose number is divisible 5
opted Chemistry course. Then the number of student who did not opt for any of the three courses is :
[JEE(Main) 2019, Online (10-01-19),P-1 (4, – 1), 120]
(1) 38 (2) 42 (3) 102 (4) 1
 π 3
21. The sum of all values of θ∈  0,  satisfying sin2 2θ + cos4 2θ = is :
 2 4
[JEE(Main) 2019, Online (10-01-19),P-1 (4, – 1), 120]
π 3π 5π
(1) π (2) (3) (4)
2 8 4

π π π π
22. The value of cos 2
. cos 3
. .... . cos 10
. sin is :
2 2 2 210
[JEE(Main) 2019, Online (10-01-19),P-2 (4, – 1), 120]
1 1 1 1
(1) (2) (3) (4)
1024 2 512 256

RI
23. If sin 4 θ + 4 cos 4 β + 2 = 4 2 sin α cos β ; α, β ∈ [0, π] , then cos(α + β) – cos(α – β) is equal to
[JEE(Main) 2019, Online (12-01-19),P-2 (4, – 1), 120]

A
(1) – 2 (2) 0 (3) 2 (4) –1

UH
JA
LP
A
NK
SA
EXERCISE # 1
Section (A) :
A-1. (iv) A-2. {2,3,5,7}

A-3. (i) non-empty (ii) empty (iii) non-empty (iv) non-empty


A-4. (i) Finite (ii) Infinite (iii) Finite (iv) Finite

A-5. 32 A-6. P(A) = {φ, {φ}, {{φ}}, {φ, {φ}}} = {φ, {φ}, {{φ}}, A}

Section (B) :

RI
B-1. 6 B-2. {1, 2, 3, 4}

B-3. {x : 1 ≤ x < 2}

A
(3) {x : 1 ≤ x ≤ 2}
(4) {x : 1 < x < 2}
B-4. {3, 5, 9} B-5. {3, 4, 10} B-7. {9, 21} B-8. d = bc

UH
Section (C) :
C-1. 300 C-2. (i) 600 (ii) 250 (iii) 150 C-3. 60 C-4. 15
C-5. (i) 12 (ii) 6
JA
Section (D) :

(2,1)

(1,0)
D-1. (i) (ii)
LP

• •(0,–3)
• (0,–55) (3,–1)

y
A

(0, 8.5)
(2,1/2)
• (0,13)
• (–1,16)
(iii) (iv) x
NK

• •
(2,-11)
• •
(1,–1/2) (3,–1/2)

y
SA

(0,1)

(–1,0)
(v) (0,–21) (vi)
(1,–24)

 –32 
D-2. 2 D-3.  27 ,0 
 
Section (E) :
 3– 5  3+ 5
E-1. (i) (–3,–2) ∪ (1,3) (ii) {–2} ∪ [1, 8) (iii)  –2,  ∪  1, 
 2   2 
(iv) (–∞,1) – {0} (v) (–3,–2) ∪ (–1,∞) (vi) [1,6]

E-2. 2

 5  25   2 
E-3. (i)  – ∞,– 2  (ii)  4 ,∞  (iii)  – 5 ,0 
     

 –4x + 11 x ∈ (– ∞,2) (2,3)


 (3,1)
E-4. (i) y =  –2x + 7 x ∈ [2,3)
 4x – 11 x ∈ [3, ∞ )

RI
(2,5)

A
(5,5)
 –2x – 1 x ∈ (– ∞,–1)
 1 x ∈ [–1,0) (–1,1) (0,1)

UH

(ii)  2x + 1 x ∈ [0,2)
 5 x ∈ [2,5)

 2x – 5 x ∈ [5, ∞ )
JA
(–1,7)
LP

–2 –1 0 1 2 3 4 5

(1,–1)
A

(4,–7)
NK

(iii)
2 1 13 9
E-5. (i) – (ii) –2 (iii) , (iv) –
3 3 4 7
1± 7
SA

(v) 2,
3

E-6. (i) –2, 1 (ii) [3,∞) (iii) [–1,1] ∪ [2,∞)

E-7. (i) (–∞,–12)∪ (–2,∞) (ii) (–13,7)


3 
(iii) R (iv)  4 ,1 ∪ (1, ∞)
 
(v) (–∞,1]∪ [3,∞) (vi) (–∞,3)
(vii) (–7,–2) ∪ (3,4) (viii) [0,4]

E-8. (i) 5 (ii) 4


(0,2)
(0,2)


1 (1,0) (2,0)
E-9. (i) 4 (ii)
(1,0) (2,0)

(0,–2)

(0,2)

RI
(0,2)
(–1,0) (–1,0)
1

(iii) (–2,0) –
1
(2,0) (iv) 4

A
4 (–2,0) (–1,0) (1,0) (2,0)

UH
(0,2)

(v) (–1,0) (1,0)


(–2,0) (2,0)
JA
(0,–2)

(–6,4) (0,4)
(6,4)
LP

(–2, 2)
(–1, 2) (2, 0)
(–7,0)
E-10. (i) (–5, 0) (–3, 0) (4, 0)

(–8,–2) (–4,–2) (3,–2)


A

(–1, 2) (5, 2)
(–7, 2)
(–3, 1) (0, 1) (1, 1)
NK

(–2, 1)
(ii) (6, 0)
(–8, 0) (–6, 0) (–4, 0) (2, 0) (4, 0)

(–9, –1) (–5, –1) (3, –1) (7, –1)


SA

(iii)
(–7, 1) (–3, 1) (3, 1) (7, 1)

(–6, 0) (–4, 0) (–2, 0) (2, 0) (4, 0)


(6, 0)
(–1, –1) (1, –1)
(–5, –2) (5, –2)
1 
 2 ,1
 
3 
 2 ,0 
(0,0) (1,0)  
(iv)
(2,–1)
(0,–2) (1,–2)

1 
 2 ,–3 
 

Section (F) :

F-1. (i) (5, ∞) (ii) (5,7] (iii) R x∈φ


(iv) (v) (–∞,2) ∪ (8, ∞)
 1 1 8
(vi) (–∞,–2) (vii) x ∈  0,  (viii) x ∈ (–∞,–4) ∪  ,  (ix) x ∈ [–1,2)
 3 2 7

RI
F-2. (i) 1 (ii) 2 (iii) 1
1
(iv) –72 (v) 7+ (vi) 0
196

A
1 4 1  2b + 1– a 
F-3. (i) + (ii)
a 3b 2  2b + 4a 

UH
F-5 (i) 3 (ii) 5 (iii) ±3 (iv) 3

F-6. (i) 0 (ii) 10, 10–4 (iii) 10, 10–9/2 (iv) 10, 3/2 (v) 1
JA
1  1 
F-7 (i)  ,2  (ii)  ,4  (iii) [2,∞) (iv) (–∞, –2] ∪ [2, ∞)
3  2 

 1 
F-8. (i)  3 / 2 , ∞  (ii) [log3 9 /10,2) (iii) (5, ∞) (iv) (2,∞)
3 
LP

F-9 (i) R (ii) (0, log 3 3 )


2

Section (G) :
1   1 1
A

G-1. (i) x ∈  ,1


2 
(ii) 2 (iii) x ∈  n + 3 ,n + 2 
n∈Ι
(iv) x ∈ [1,2)

G-2 (i) x ∈φ (ii) {0} (iii) {0,2/5, –1/5}


NK

1
G-3. (i) no solution (ii) –1, – (iii) 0,1
2
 −1   1
G-4. (i) x ∈ (–∞,0) (ii) x ∈  ,1 (iii) x ∈  n,n + 2 
 2  n∈Ι
SA

G-5 (i) x ∈ R (ii)x ∈ (–2, 4) (iii) (1, 2) ∪ (2, 3) ∪ (3, 4) G-6 (i) 2 (ii) 2

Section (H) :
π −π π π 3 π 2π 4 π 8 π π
H-1. (i) , (ii) 0, , , , π, , , (iii)
3 3 4 2 4 9 9 9 6
nπ π
H-2. (i) x = or x = mπ± where m,n∈I
4 6
π
(ii) x = (2n+1) where n ∈ Ι
10
(iii) x = mπ, m∈Ι
π 5π  −π −π π π
H-3. (i) x ∈   6 + 2nπ, 6 + 2nπ  (ii) x∈ ∪ [ +2nπ, + 2nπ] ∪ [ +2nπ, + 2nπ]
n∈Ι  n∈I 2 3 3 2
π π  π π  π π 
(iii) x∈ ∪ [– +2nπ, 2nπ+ ] (iv) x∈ R (v) x ∈   – 2 + 2nπ,– 3 + 2nπ  ∪  + 2nπ, + 2nπ 
n∈I 6 2 n∈Ι 3 2 
1
H-4 (i) 11 (ii) 6 H-5. y= and x = nπ,where n ∈ Ι
2

EXERCISE # 2
1. (A) 2. (D) 3. (A) 4. (D) 5. (C) 6. (B) 7. (A)

8. (B) 9. (C) 10. (C) 11. (C) 12. (B) 13.* (AD) 14. (C)

RI
15. (A) 16. (C) 17. (B) 18. (A) 19. (D) 20. (C) 21*. (B)

22. (C) 23. (A) 24. (A) 25. (A) 26. (C) 27. (D) 28. (C)

A
29. (B) 30. (B) 31. (A) 32. (D) 33. (C) 34. (B) 35. (D)

36. (D) 37. (D) 38. (A) 39. (C) 40.* (CD) 41. * (BC) 42. (C)

UH
43. (C) 44. * (BC) 45. (B) 46. (A) 47. (D) 48. (C) 49. (A)

50. (B)
JA
EXERCISE # 3
PART-I

1.* (AB) 2*. (CD) 3. 2 4. (n = 7) 5. 3 6. (C) 7. (D)


LP

8.* (ACD) 9. (4) 10.* (ABC) 11. (D) 12. 8 13. (C) 14. (C)

15. (C) 16. (BC)/ BONUS 17. (8)

PART-II
A

1. (2) 2. (2) 3. (1) 4. (1) 5. (2) 6. (2) 7. (3)


NK

8. (2) 9. (2) 10. (2) 11. (1) 12. (1) 13. (2) 14. (4)

15. (3) 16. (1) 17. (4) 18. (3) 19. (1) 20. (1) 21. (2)

22. (3) 23. (1)


SA
FUNDAMENTAL OF MATHEMATICS-II

He is unworthy of the name of man who is ignorant of the fact that the diagonal of square is incommensurable with its side .......Plato

Absolute value function / modulus function :


 x if x≥0
The symbol of modulus function is f (x) = x and is defined as: y = x=  .
− x if x<0

RI
Properties of modulus : For any a, b ∈ R
(i) |a| ≥ 0 (ii) |a| = |–a|

A
(iii) |a| ≥ a, |a| ≥ –a (iv) |ab| = |a| |b|
a |a|
(v) = (vi) |a + b| ≤ |a| + |b| ; Equality holds when ab ≥ 0
b |b|

UH
(vii) |a – b| ≥ ||a| – |b|| ; Equality holds when ab ≥ 0

Example # 1 : Solve the following linear equations


(i) x |x| = 4 (ii) |x – 3| + 2|x + 1| = 4
Solution : (i) x|x| = 4
JA
If x > 0
∴ x2 = 4 ⇒ x=±2
∴ x = 2 ( x ≥ 0)
If x<0 ⇒ – x2 = 4 ⇒ x2 = – 4 which is not possible
(ii) |x – 3| + 2|x + 1| = 4

case Ι : If x ≤ –1
LP

∴ –(x – 3) – 2(x + 1) = 4
⇒ – x + 3 – 2x – 2 = 4 ⇒ – 3x + 1 = 4
⇒ – 3x = 3 ⇒ x=–1

case ΙΙ : If – 1 < x ≤ 3
A

∴ –(x – 3) + 2(x + 1) = 4
⇒ – x + 3 + 2x + 2 = 4 ⇒ x = – 1 which is not possible
case ΙΙΙ : If x > 3
x – 3 + 2(x + 1) = 4
NK

3x – 1 = 4 ⇒ x = 5/3 which is not possible ∴ x = – 1 Ans.

Rational function :
g(x)
A rational function is a function of the form, y = f (x) = , where g (x) & h (x) are polynomial
h(x)
SA

functions.

Irrational function :
An irrational function is a function y = f(x) in which the operations of addition, substraction,
multiplication, division and raising to a fractional power are used.
x 3 + x1/ 3
For example y = is an irrational function
2x + x
(a) The equation f(x) = g(x), is equivalent to the following system
f(x) = g2(x) & g(x) ≥ 0
(b) The inequation f(x) < g(x), is equivalent to the following system
f(x) < g2(x) & f(x) ≥ 0 & g(x) ≥ 0
(c) The inequation f(x) > g(x), is equivalent to the following system
g(x) ≤ 0 & f(x) ≥ 0 or g(x) ≥ 0 & f(x) > g 2(x)

Example # 2 : Solve : x + 2 > 2 1 − x 2


Solution : 4(1 – x2) < (x + 2)2 and x+2≥0 & 1 – x2 ≥ 0
 –4 
x∈  – ∞, ∪ (0, ∞) ...(1)
 5 
x∈[–2, ∞) ...(2)
x∈[–1, 1] ...(3)
(1) ∩ (2) ∩ (3)
 4
 −1, − 5  ∪ (0, 1]
 

Self Practice Problem :

RI
(1) 2x 2 + x − 6 < x (2) 5−x > x + 1
(3) x+3+ x + 4x − 5 > 0
2
(4) x – 4–x ≥1
3  4 + 7 

A
Ans. (1)  , 2  (2) (–∞, 1) (3) (–∞, –1] ∪ [5, ∞) (4)  , 4
 2   2 

UH
Graphs Related to modulus :
If graph of y = f(x) is
JA
then draw graph of
LP

(a) y = – f(x) (b) y = f(–x) (c) y = f(|x|) (d) y = f(–|x|)


(e) y = |f(x)| (f) |y| = f(x) (g) |y| = – f(x)
y

1
A

–4 –3 3
x
–2 –1 1 2
NK

–1

y
y

1
1
SA

–3 3
–4 –3 3 4 x
x –2 –1 1 2
–2 –1 1 2

–1 –1

x
–3 –2 –1 1 2 3 4
Graphical Trasformation :

If graph of y = f(x) is

RI
y

(0, 1)

A
(1, 0)
x

UH
(–2, 0) –1

(0, –2)
JA
then graph of (a) y – k = f(x – h) (b) y = kf(x), (k > 0) (c) y = f(kx), (k > 0)

y
(h, 1 + k)
LP

(–2 + h, k) (1 + h, k)
(h, k)
A

x
NK

(h, –2+k)
SA
RI
Example # 3 : y = |x2 + 4x + 3| Solution : –3 –1

A
UH
Example # 4 : |y–1| = sin x Solution :
JA
–3
Example # 5 : y = –x2 + 4|x| – 3 Solution :
LP

–3 3
Example # 6 : y = ||x |–3| Solution :
A
NK

x
Example # 7 : y = sin   Solution : period is 6π
3
Example # 8 : y = sin x − 3 Solution : Graphical Transformation
SA

Example # 9 : y = |–n |–x|| Solution :

Example # 10 : |y| = x2 – 3x + 2 Solution :


 Marked questions are recommended for Revision.

PART - I : SUBJECTIVE QUESTIONS


Section (A) : Modulus Function & Equation

A-1. Write the following expression in appropriate intervals so that they are bereft of modulus sign
(i) |x2 – 7x + 10| (ii) |x3 – x| (iii) |2x – 2|
(iv) |x2 – 6x + 10| (v) |x – 1| + |x2 – 3x + 2| (vi) x 2 − 6x + 9
(vii) 2(x–1) + |x + 2| – 3|x+1|

RI
A-2. Draw the labled graph of following
(i) y = |7 – 2x| (ii) y = |x – 1| – |3x – 2|

A
(iii) y = |x – 1| + |x – 4| + |x – 7| (iv) y = |4x + 5| (v) y = |2x – 3|

A-3. Solve the following equations

UH
(i) |x| + 2 |x – 6| = 12 (ii) ||x + 3| – 5| = 2
(iii) |||x – 2| – 2 | – 2| = 2 (iv) |4x + 3| + |3x – 4| = 12

A-4. Solve the following equations :


(i) x2 – 7|x| – 8 = 0 (ii) |x2 – x + 1| = |x2 – x – 1|
JA
(iii) |x3 – 6x2 + 11x – 6| = 6 (iv) |x2 – 2x| + x = 6
(v) |x2 – x – 6| = x + 2.

A-5. Find the number of real roots of the equation


2
(i) x −3 x +2=0 (ii) ||x – 1|– 5| = 2 (iii) |2x 2 + x – 1| = |x2 + 4x + 1|
LP

A-6. Find the sum of solutions of the following equations :


(i) x2 – 5|x| – 4 = 0
(ii) (x – 3)2 + |x – 3| – 11 = 0 (iii) |x|3 – 15x2 – 8|x| – 11 = 0
(iv) ||x – 3| – 4| = 1 (v) 2|x| + 3|x| + 4|x| = 9
A

A-7. Find number of solutions of the following equations


(i) |x – 1| + |x – 2| + |x – 3| = 9 (ii) |x – 1| + |x – 2| + |x – 3| + |x – 4| = 4
NK

(iii) |x| + |x + 2| + |x – 2| = p, p ∈ R

A-8. Find the minimum value of f(x) = |x – 1| + |x – 2| + |x – 3|

A-9 If x2 – |x – 3| – 3 = 0, then |x| can be


SA

A-10. If |x3 – 6x2 + 11x – 6| is a prime number then find the number of possible integral values of x.

Section (B) : Modulus Inequalities


B-1. Solve the following inequalities :
(i) |x – 3| ≥ 2 (ii) | |x – 2| – 3| ≤ 0 (iii) ||3x – 9| + 2 | > 2
(iv) |2x – 3| – |x| ≤ 3 (v) |x – 1| + |x + 2| ≥ 3 (vi) ||x – 1| – 1| ≤ 1

B-2. Solve the following inequalities :


3 3x | x + 3 | +x
(i) 1+ >2 (ii) ≤1 (iii) >1
x x −4 2 x+2
(iv) |x2 + 3x| + x2 – 2 ≥ 0 (v) |x + 3| > |2x – 1|
B-3. Solve the following inequalities
| x + 2 | −x
(i) |x3 – 1| ≥ 1 – x (ii) x 2 – 4x + 4 ≥1 (iii) <2
x
| x−2|
(iv) >0 (v) |x – 2| > |2x – 3| (vi) |x + 2| + |x – 3| < |2x + 1|
x−2

B-4. Solve the following equations


(i) |x3 + x2 + x + 1| = |x3 + 1| + |x2 + x|
(ii) |x2 – 4x + 3| + |x2 – 6x + 8| = |2x – 5|
(iii) |x2 + x + 2| – |x2 – x + 1| = |2x + 1|
(iv) |x2 – 2x – 8| + |x2 + x – 2| = 3 | x + 2|
(v) | 2x – 3 | + | x + 5 | ≤ | x – 8 |

Find the solution set of the inequalities |x2 + x – 2| ≤ 0 and |x2 – x + 2| ≥ 0

RI
B-5.

Section (C) : Miscellaneous Modulus Equations & Inequations

A
C-1. Write the following expression in appropriate intervals so that they are bereft of modulus sign
2
− 4x + 5
(i) |log10x| + |2x–1 – 1| (ii) |(log2x)2 – 3(log2x) + 2| (iii) | 5 x − 25 |

UH
C-2. Solve the equations log100 l x + y l = 1/2 , log10y – log10|x| = log1004 for x and y.

C-3. Solve the inequality


(i) (log2x)2 – |(log2x) – 2| ≥ 0
(ii) 2 | log3x | + log3x ≥ 3
JA x
(iii). Find the complete solution set of 2x + 2 ≥ 2 2

3 x 2 − 10 x + 3
C-4. Find the number of real solution(s) of the equation x − 3 =1

C-5. If x, y are integral solutions of 2x2 – 3xy – 2y2 = 7, then find the value of |x + y|
LP

C-6. If x, |x + 1|, |x – 1| are three terms of an A.P., then find the number of possible values of x

Section (D) : Irrational Inequations


D-1. Solve the following inequalities :
A

2x − 1
(i) <1 (ii) x– 1 −|x| < 0 (iii) x 2 − x − 6 < 2x – 3
x−2
x
NK

(iv) x 2 − 6x + 8 ≤ x +1 (v) x 2 − 7x + 10 + 9 log4   ≥ 2x + 14x − 20 − 2x 2 – 13


8
1
(vi) x–3< x 2 + 4x − 5 (vii) x 2 − 5x − 24 > x + 2 (viii) 4 − x2 ≥
x
SA

x+7
(ix) > 3−x
x +1
D-2. Solve the equation a(2x – 2) + 1 = 1 – 2x for every value of the parameter a.

Section (E) : Transformation of curves

E-1. Draw the graph of followings —


(i) y = – |x + 2| (ii) y = | | x – 1 | – 2|
(iii) y = |x + 2| + |x – 3| (iv) |y| + x = – 1

E-2. Draw the graphs of the following curves :


1 y
(i) y= – (ii) =–1 (iii) |y – 3| = |x – 1|
| 2x + 1| | x | −1
| x 2 – 1|
(iv) y= nx
(x 2 – 1)

E-3. Draw the graph of y = log1/2 (1 – x).

E-4. Find the set of values of λ for which the equation |x 2 – 4|x| – 12| = λ has 6 distinct real roots.

E-5. If y = f(x) has following graph

RI
Then draw the graph of
(i) y = |f(x)| (ii) y = f(|x|) (iii) y = f(– |x|) (iv) y = | f ( |x| ) |

E-6. If y = f(x) is shown in figure given below, then plots the graph for
(A) y = f(|x + 2|) (B) |y – 2| = f(–3x).

A
UH
JA
E-7. Find the number of roots of equation
(i) 3|x|– |2 – |x|| = 1 (ii) x + 1 = x · 2x

E-8 Find values of k for which the equation |x2 – 1| + x = k has


(i) 4 solution (ii) 3 solutions (iii) 1 solution (iv) 2 solutions
LP
A
NK
SA
 Marked questions are recommended for Revision.
* Marked Questions may have more than one correct option.
1. Number of integral values of 'x' satisfying the equation 3|x + 1| – 2.3x = 2.|3x – 1| + 1 are
(A) 1 (B) 2 (C) 3 (D) 4

2. |x2 + 6x + p| = x2 + 6x + p ∀ x ∈ R where p is a prime number then least possible value p is


(A) 7 (B) 11 (C) 5 (D) 13

3. If (log10x)2 – 4|log10x| + 3 = 0, the product of roots of the equation is :


(A) 3 (B) 104 (C) 108 (D) 1

RI
4. The equaiton ||x – 1| + a| = 4 can have real solutions for x if a belongs to the interval
(A) (–∞, 4] (B) (4, ∞) (C) (–4, ∞) (D) (–∞, –4) U(4, ∞)

A
5. The number of values of x satisfying the equation | 2x + 3 | + | 2x – 3 | = 4x + 6, is
(A) 1 (B) 2 (C) 3 (D) 4

| x2 − 4 x |+3

UH
log3 ≥0
x2 +| x − 5 |
(A) 1 (B) 2 (C) 3 (D) 4

7. If |x + 2| + y = 5 and x – |y| = 1 then the value of (x + y) is


JA
(A) 1 (B) 2 (C) 3 (D) 4

8. The number of value of x satisfying the equation x − 1A = (x − 1)7 , where A = log3 x² − 2 logx9
(A) 1 (B) 2 (C) 0 (D) 3

9. The number of integral value of x satisfying the equation log 3


x−2 − log3 x − 2 = 2
LP

(A) 1 (B) 2 (C) 3 (D) 4

10. The sum of all possible integral solutions of equation


||x2 – 6x + 5| – |2x2 – 3x + 1|| = 3|x2 – 3x + 2| is
(A) 10 (B) 12 (C) 13 (D) 15
A

11. The complete solution set of the inequality (|x – 1| – 3) (|x + 2| – 5) < 0 is (a, b) ∪ (c, d) then the value
of |a| + |b| + |c| + |d| is
(A) 14 (B) 15 (C) 16 (D) 17
NK

3 |x| − 2
12. The product of all the integers which do not belong to the solution set of the inequality
| x | −1
≥ 2 is
(A) –1 (B) –4 (C) 4 (D) 0
SA

13. Let f(x) = |x – 2| and g(x) = |3 – x| and


A be the number of real solutions of the equation f(x) = g(x)
B be the minimum value of h(x) = f(x) + g(x)
C be the area of triangle formed by f(x) = |x – 2|, g(x) = |3 – x| and x-axis and α < γ < β < δ where α < β
are the roots of f(x) = 4 and γ < δ are the roots of g(x) = 4, then the value of sum of digits of
α 2 + β2 + γ 2 + δ 2
.
ABC
(A) 7 (B) 8 (C) 11 (D) 9

14*. If f(x) = |x + 1| – 2 | x – 1| then


(A) maximum value of f(x) is 2. (B) there are two solutions of f(x) = 1.
(C) there is one solution of f(x) = 2. (D) there are two solutions of f(x) = 3.
a
15*. The solution set of inequality |x| < , a ∈ R, is
x
(
(A) – –a,0 ) if a < 0 ( )
(B) 0, a if a > 0 (C) φ if a = 0 (D) (0, a) if a > 0

 1 
16*. If a and b are the solutions of equation : log5  log64 | x | − + 25 x  = 2x, then
 2 
(A) a + b = 0 (B) a2 + b2 = 128 (C) ab = 64 (D) a – b = 8

17. The number of solution of the equation log3|x – 1| . log4|x – 1| . log5|x – 1|


= log5|x – 1| + log3|x – 1| . log4|x – 1| are
(A) 3 (B) 4 (C) 5 (D) 6

RI
(x 2 + 2)( x 2 − 16 )
18. Find the number of all the integral solutions of the inequality ≤0
(x 4 + 2)(x 2 − 9)
(A) 1 (B) 2 (C) 3 (D) 4

A
1 − 21 − 4x − x 2
19. Find the complete solution set of the inequality ≥0

UH
x +1
(A) 2 6 − 2, 3  (B)  −2 −2 6, −1 )
  
(C)  −2 − 2 6, − 1 ∪ 2 6 − 2, 3  (D)  −2 −2 6, −1) ∪ 2 6 − 2, 3 
      
JA
| x+2| – | x|
20. The solution set of the inequality ≥ 0 is
4 – x3
3 3 3 3
(A) [–1, 4) (B) [1, 4) (C) [–1, 2) (D) [0, 4)
LP

2 4
21. The number of integers satisfying the inequality log1/ 2 x + 4log2 x < 2 (4 – log16x ) are

(A) 2 (B) 3 (C) 4 (D) 5

22. If f1(x) = | | x | – 2| and fn(x) = |fn – 1(x) –2| for all n ≥ 2, n ∈ N, then number of solution of the equation
A

f2015(x) = 2 is
(A) 2015 (B) 2016 (C) 2017 (D) 2018
NK

23. If graph of y = f(x) in (–3,1), is as shown in the following figure


SA

and g(x) = n(f(x)), then the graph of y = g(– |x|) is

(A) (B)
(C) (D)

24*. Solution set of inequality ||x| – 2| ≤ 3 – |x| consists of :


(A) exactly four integers (B) exactly five integers
(C) Two prime natural number (D) One prime natural number

25*. If a ≠ 0, then the inequation |x – a| + |x + a| < b


 −b b 

RI
(A) has no solutions if b ≤ 2 |a| (B) has a solution set  ,  if b > 2 |a|
 2 2
 −b b 
(C) has a solution set  ,  if b < 2 |a| (D) All above
 2 2

A
26. The equation ||x – a| – b| = c has four distinct real roots, then

UH
(A) a > b – c > 0 (B) c > b > 0
(C) a > c + b > 0 (D) b > c > 0

27*. If graph of y = f(x) is as shown in figure


y
2
JA
x
–2 –1 0 2
–1

then which of the following options is/are correct ?


y
LP

(A) Graph of y = f(–|x|) is


x
–2 –1 0 1 2
A

(B) Graph of y = f(|x|) is


NK

y
1
(C) Graph of y = |f(|x|)| is
x
–2 –1 O 1 2
SA

(D) Graph of |y| = f(x) is –1


x
–2 O 2

28*. Consider the equation x 2 – 4 | x | + 3 =p

(A) for p = 2 the equation has four solutions


(B) for p = 2 the equation has eight solutions
(C) there exists only one real value of p for which the equation has odd number of solutions
(D) sum of roots of the equation is zero irrespective of value of p
29*. Consider the equation |nx| + x = 2, then
(A) The equation has two solutions (B) Both solutions are positive
(C) One root exceeds one and other in less than one (D) Both roots exceed one

30*. Consider the equation ||x – 1 | – |x + 2|| = p. Let p 1 be the value of p for which the equation has exactly

one solution. Also p2 is the value of p for which the equation has infinite solution. Let α be the sum of all
the integral values of p for which this equation has solution then
(A) p1 = 0 (B) p2 = 3 (C) α = 6 (D) p1 + p2 = 4

RI
31. Number of the solution of the equation 2x = |x – 1| + |x + 1| is
(A) 0 (B) 1 (C) 2 (D) ∞

A
32. Number of the solution of the equation x2 = |x – 2| + |x + 2| – 1 is
(A) 0 (B) 3 (C) 2 (D) 4

UH
33. f(x) is polynomial of degree 5 with leading coefficient = 1, f(4) = 0. If the curve y = |f(x)| and y = f(|x|) are
same, then the value of f(5) is
(A) 405 (B) –405 (C) 45 (D) –45
JA
34. The area bounded by the curve y ≥ |x – 2| and y ≤ 4 – |x – 3| is
13 15
(A) (B) 7 (C) (D) 8
2 2
LP
A
NK
SA
PART - I : JEE (ADVANCED) / IIT-JEE PROBLEMS (PREVIOUS YEARS)
 Marked questions are recommended for Revision.

* Marked Questions may have more than one correct option.

1. Draw the graph of y = |x|1/2 for –1 ≤ x ≤ 1.

2. The number of real solutions of the equation |x| 2 – 3|x| + 2 = 0 is :


(A) 4 (B) 1 (C) 3 (D) 2

3. If p, q, r are any real numbers, then

RI
1
(A) max (p, q) < max (p, q, r) (B) min (p, q) = (p + q – |p – q|)
2
(C) max (p, q) < min(p, q, r) (D) None of these

A
4. Let f(x) = |x – 1|. Then
(A) f(x2) = (f(x))2 (B) f(x + y) = f(x) + f(y) (C) f(|x|) = |f(x)| (D) None of these

UH
5. If x satisfies |x – 1| + |x – 2| + |x – 3| ≥ 6, then
(A) 0 ≤ x ≤ 4 (B) x ≤ –2 or x ≥ 4 (C) x ≤ 0 or x ≥ 4 (D) None of these

6. Solve |x2 + 4x + 3| + 2x + 5 = 0.

7. If p, q, r are positive and are in A.P., then roots of the quadratic equation px 2 + qx + r = 0 are real for
JA
r r
(A) −7 ≥ 4 3 (B) −7 < 4 3
p p
(C) all p and r (D) no p and r

8. The function f(x) = |ax – b| + c |x| ∀ x ∈ (–∞, ∞), where a > 0, b > 0, c > 0, assumes its minimum value
only at one point if
LP

(A) a ≠ b (B) a ≠ c (C) b ≠ c (D) a = b = c

9. Find the set of all solutions of the equation 2|y| – | 2y–1 – 1| = 2y–1 + 1

2
The sum of all the real roots of the equation x − 2 + x − 2 − 2 = 0 is ______.
A

10.

11. If α & β (α < β) are the roots of the equation x 2 + bx + c = 0, where c < 0 < b, then
(A) 0 < α < β (B) α < 0 < β < α (C) α < β < 0 (D) α < 0 < α < β
NK

12. If f(x) = x2 + 2bx + 2c2 and g(x) = – x2 – 2cx + b2 are such that min f(x) > max g(x), then the relation
between b and c, is
(A) no relation (B) 0 < c < b/2 (C) |c| < 2 |b| (D) |c| > 2 |b|
SA

PART - II : JEE (MAIN) / AIEEE PROBLEMS (PREVIOUS YEARS)


1. Product of real roots of the equation t2x2 + |x| + 9 = 0
(1) is always positive (2) is always negative (3) does not exist (4) none of these

2. The number of real solutions of the equation x2 – 3|x| + 2 = 0 is


(1) 3 (2) 2 (3) 4 (4) 1

3. The sum of the roots of the equation, x 2 + |2x – 3| –4 = 0, is :

(1) – 2 (2) 2 (3) –2 (4) 2


4. The equation 3x 2 + x + 5 = x – 3, where x is real , has :
(1) exactly four solutions (2) exactly one solutions
(3) exactly two solutions (4) no solution

1
5. The domain of the function f(x) = is :
| x | −x

(1) (–∞ , ∞) (2) (0, ∞) (3) (–∞, 0) (4) (–∞, ∞) – {0}

 1
6. If x is a solution of the equation, 2x + 1 – 2x – 1 = 1,  x ≥  , then 4x 2 – 1 is equal to

RI
 2

3 1
(1) 2 (2) (3) 2 2 (4)
4 2

A
1 1
7. Let α and β be the roots of equation px2 + qx + r = 0, p ≠ 0. If p, q ,r are in the A.P. and + = 4, then
α β

UH
the value of |α – β| is :

34 2 13 61 2 17
(1) (2) (3) (4)
9 9 9 9
JA
8. Let S = {x ∈ R : x ≥ 0 and 2| x – 3| + x ( x – 6) + 6 = 0}. Then S :
(1) contains exactly two elements. (2) contains exactly four elements.
(3) is an empty set. (4) contains exactly one element
LP
A
NK
SA
EXERCISE # 1

PART-I
Section (A) :

A-1. (i) x2 – 7x + 10, x > 5 or x ≤ 2 ; –(x2 – 7x + 10), 2 < x ≤ 5


(ii) x3 – x, x ∈ [–1, 0] ∪ [1, ∞) ; x – x3 , x ∈ (–∞ , –1) ∪ (0, 1)

(iii) 2x – 2, x ≥ 1 ; 2 – 2x , x < 1 (iv) x2 – 6x + 10, x ∈ R

RI
(v) x2 – 2x + 1, x ≥ 2 ; 4x – x2 – 3, 1 ≤ x < 2 ; x2 – 4x + 3, x < 1
(vi) x – 3, x ≥ 3 ; 3 – x, x < 3
(vii) 2x–1 + x + 2 – 3x+1 x ≥ –1 ; 2x–1 + x + 2 – 3–(x+1) –2 ≤ x < –1
x–1
2 –x–2–3 –(x+1) x < –2

A
A-2. (i) (ii)

UH
(iii)
JA
LP

(iv) (v)
A

11 13
A-3. (i) x = 0, 8 (ii) x = –10, –6, 0, 4 (iii) x = 0, ± 4, 8 (iv) x=– ,
7 7
NK

A-4. (i) ±8 (ii) 0, 1 (iii) 0, 4 (iv) –2, 3 (v) x ∈ {– 2, 2, 4}

A-5. (i) 4 (ii) 4 (iii) 4

A-6. (i) 0 (ii) 6 (iii) 0 (iv) 12 (v) 0


SA

A-7. (i) 2 (ii) Infinite


(iii) p<4 no solution p=4 one solution p>4 Two solution

A-8. 2 A-9 2, 3 A-10. 0

Section (B) :
B-1. (i) x ∈ (–∞, 1] ∪ [5, ∞) (ii) x = 5 or x = – 1 (iii) x ∈ R – {3} (iv) x ∈ [0, 6]
(v) R (vi) [− 1, 3]

B-2. (i) x ∈ (–1, 0) ∪ (0, 3) (ii) x ∈ (–∞, –4] ∪ [–1, 1] ∪ [4, ∞)


 2 1   2 
(iii) x ∈ (–5, –2) ∪ (–1, ∞) (iv) x ∈  −∞, −  ∪  , ∞  (v) x ∈ − , 4
 3 2   3 
B-3. (i) x ∈ (–∞, –1] ∪ [0, ∞) (ii) x ∈ (–∞, 1] ∪ [3, ∞) (iii) x ∈ (–∞, 0) ∪ (1, ∞)
(iv) x ∈ (2, ∞) (v) (1, 5/3) (vi) (2, ∞)
 1 
B-4. (i) {–1} ∪ [0, ∞) (ii) [1, 2] ∪ [3, 4] (iii) x ∈ − , ∞ 
 2 
 3
(iv) [1, 4] ∪ {–2} (v)  –5, 2 
 
B-5. {–2, 1}

Section (C) :

C-1. (i) log10x + 2x – 1 – 1 x≥1


–(log10x + 2x–1 – 1) 0<x<1
(ii) (log2x)2 – 3(log2x) + 2 x ∈ (0, 2] ∪ [4, ∞)

RI
–((log2x)2 – 3(log2x) + 2 ) x ∈ (2, 4)
2
− 4x + 5
(iii) 5x − 25 x ∈ (–∞, 1] ∪ [3, ∞)
x 2 − 4x + 5
25 − 5 x ∈ (1, 3)

A
C-2. x = 10/3, y = 20/3 & x = – 10, y = 20

UH
 1  1 1
C-3. (i) x ∈  0,  ∪ 2, ∞ ) (ii)  0, 27  ∪ 3, ∞ ) (iii) ( −∞,log2 ( 2 − 1)] ∪  , ∞ )
 4   2
C-4. 3 C-5. 4 C-6. 2

Section (D) :
JA
1 
D-1. (i)  2 , 2  ∪ (5, ∞) (ii) [– 1, ( 5 – 1)/2) (iii) x ∈ [3, ∞)
 
 7 − 21   7 + 21 
(iv) x∈  , 2 ∪  4,  (v) x=2
 2   2 
LP

(vi) (– ∞, – 5] U [1, ∞) (vii) (–∞, –3] (viii) [–2, 0) ∪ [ 2 − 3 , 2 + 3 ]


(ix) (– 1, 1) ∪ (2, 3]

D-2. x = log2a where, a ∈ (0, 1]


A

Section (E) :
NK

E-1. (i) (ii)


SA

(iii) (iv)
E-2. (i) (ii)

RI
(iii) (iv)

A
UH
E-3.

E-4. λ ∈ (12, 16)


JA
E-5. (i) (ii)
LP

(iii) (iv)
A
NK

E-6. (A)
SA

(B)

E-7. (i) 2 (ii) 2

 5 5 5 
E-8 (i) k ∈  1,  (ii) k = 1, (iii) k=–1 (iv) k ∈  ∞  ∪ (–1, 1)
 4 4 4 
EXERCISE # 2
1. (B) 2. (B) 3. (D) 4. (A) 5. (A) 6. (A*) 7. (C)

8. (B) 9. (A) 10. (D) 11. (C) 12. (A) 13. (D) 14. (ABC)

15. (ABC) 16. (AB) 17. (D) 18. (B) 19. (D) 20. (A) 21. (B)

22. (C) 23. (D) 24. (BD) 25. (AB) 26. (D) 27. (ACD)

28. (ACD) 29. (ABC) 30. (ABC) 31. (C) 32. (C) 33. (A) 34. (C)

EXERCISE # 3

RI
PART-I

2. (A) 3. (B) 4. (D) 5. (C) 6. x = –1 – 3 or –4

A
7. (A) 8. (B) 9. {–1} ∪ [1, ∞) 10. 4 11. (B) 12. (D)

UH
PART-I
1. (3) 2. (3) 3. (2) 4. (4) 5. (3) 6. (2) 7. (2)

8. (1)
JA
LP
A
NK
SA
COMPLEX NUMBER

The shortest path between two truths in the real domain passes through the complex domain. ......Hadamard, Jacques

The complex number system


A complex number (z) is a number that can be expressed in the form z = a + ib where a and b are real
numbers and i2 = –1. Here ‘a’ is called as real part of z which is denoted by (Re z) and ‘b’ is called as
imaginary part of z, which is denoted by (Im z).
Any complex number is :
(i) Purely real, if b = 0 ;

RI
(ii) Imaginary, if b ≠ 0.
(iii) Purely imaginary, if a = 0

Note :

A
(a) The set R of real numbers is a proper subset of the Complex Numbers. Hence the complete
number system is N ⊂ W ⊂ I ⊂ Q ⊂ R ⊂ C.

UH
(b) Zero is purely real as well as purely imaginary but not imaginary.

(c) i= − 1 is called the imaginary unit.

Also i² = − 1; i3 = − i ; i4 = 1 etc.
(d) a b = ab only if atleast one of a or b is non - negative.
JA
(e) If z = a + ib, then a – ib is called complex conjugate of z and written as z = a – ib

(f) Real numbers satisfy order relations where as imaginary numbers do not satisfy order relations
i.e. i > 0, 3 + i < 2 are meaningless.
LP

Self Practice Problems


(1) Write the following as complex number
(i) −16 (ii) x , (x > 0) (iii) –b + −4ac , (a, c> 0)
(2) Write the following as complex number
A

(i) x (x < 0) (ii) roots of x2 – (2 cosθ) x + 1 = 0


Answers : (1) (i) 0 + 4i (ii) x + 0i (iii) –b + i 4ac
NK

(2) (i) 0+i x (ii) cos θ + i sin θ , cos θ – i sin θ

Algebraic Operations:
SA

Fundamental operations with complex numbers


In performing operations with complex numbers we can proceed as in the algebra of real numbers,
replacing i2 by – 1 when it occurs.
1. Addition (a + bi) + (c + di) = a + bi + c + di = (a + c) + (b + d) i
2. Subtraction (a + bi) – (c + di) = a + bi – c – di = (a – c) + (b – d) i
3. Multiplication (a + bi) (c + di) = ac + adi + bci + bdi2 = (ac – bd) + (ad+ bc)i
a + bi a + bi c − di ac − adi + bci − bdi2 ac + bd + (bc − ad)i ac + bd bc − ad
4. Division = . = = = 2 + 2 i
c + di c + di c − di c −d i
2 2 2 2
c +d 2
c +d 2
c + d2
Inequalities in imaginary numbers are not defined. There is no validity if we say that imaginary number
is positive or negative.
e.g. z > 0, 4 + 2i < 2 + 4 i are meaningless.
In real numbers if a2 + b2 = 0 then a = 0 = b however in complex numbers,
z12 + z22 = 0 does not imply z1 = z2 = 0.
Example # 1 : Find the multiplicative inverse of 4 + 3i .
Solution : Let z be the multiplicative inverse of 4 + 3i then
z (4 + 3i) = 1
1 4 − 3i 4 − 3i 4 − 3i 4 − 3i
z= × = = . Ans.
4 + 3i 4 − 3i 16 + 9 25 25

Self Practice Problem :

(3) Simplify in + in+1 + in+2 + in+3 , n ∈ Ι . Ans. 0

Equality In Complex Number :


Two complex numbers z1 = a1 + ib1 & z2 = a2 + ib2 are equal if and only if their real and
imaginary parts are equal respectively

RI
i.e. z1 = z2 ⇔ Re(z1) = Re(z2) and Ιm (z1) = Ιm (z2).

Example # 2 : Find the value of x and y for which

A
(x4 + 2xi) – (3x2 + yi) = (3 – 5i) + (1 + 2iy) , where x, yR
Solution : (x4 + 2xi) – (3x2 + yi) = (3 – 5i) + (1 + 2iy)
⇒ x4 – 3x2 – 4 = 0 ⇒ x2 = 4 ⇒ x = ± 2
and 2x – y = – 5 + 2y

UH
2x + 5 = 3y
when x = 2 ⇒ y=3
and x=–2 ⇒ y = 1/3 Ans. (2,3) or (–2,1/3)

Example # 3 : Find the value of expression x4 + 4x3 + 5x2 + 2x + 3, when x = – 1 + i.


JA
Solution : x=–1+i
(x + 1)2 = i2
x2 + 2x + 2 = 0
now, x4 + 4x3 + 5x2 + 2x + 3 = (x2 + 2x + 2) (x2 + 2x – 1) + 5 = 5

Example # 4 : Find the square root of – 21 – 20i


LP

Solution : Let x + iy = –21– 20i


(x + iy)2 = – 21 – 20 i
x2 – y2 = – 21 ----- (i)
xy = – 10 ----- (ii)
From (i) & (ii)
A

x2 = 4 ⇒ x=±2
when x = 2, y = – 5 and x = –2, y = 5
x + iy = (2 – i5) or (–2 + i5)
NK

Self Practice Problem

(4) Solve for z : z = i z2

(5) Given that x, y ∈ R, solve : 4x² + 3xy + (2xy − 3x²)i = 4y² − (x 2/2) + (3xy − 2y²)i
SA

3 1 3K
Answers : (4) ± – i, 0, i (5) x = K, y = K∈R
2 2 2

Representation of a complex number :


To each complex number there corresponds one and only one point in plane, and conversely to each
point in the plane there corresponds one and only one complex number. Because of this we often refer
to the complex number z as the point z.
(a) Cartesian Form (Geometric Representation) :
Every complex number z = x + i y can be represented by a point on the Cartesian plane known as
complex plane (Argand diagram) by the ordered pair (x, y).
Length OP is called modulus of the complex number which is denoted by z & θ is called argument
or amplitude.
y
z = x 2 + y 2 and tan θ =   (angle made by OP with positive x−axis)
x
Note :
(i) Argument of a complex number is a many valued function. If θ is the argument of
a complex number then 2nπ + θ; n ∈ I will also be the argument of that complex number. Any two
arguments of a complex number differ by 2nπ.

RI
(ii) The unique value of θ such that − π < θ ≤ π is called the principal value of the argument.
Unless otherwise stated, amp z implies principal value of the argument.
(iii) By specifying the modulus & argument a complex number is defined completely. For the complex

A
number 0 + 0 i the argument is not defined and this is the only complex number which is only given by
its modulus.
(b) Trignometric/Polar Representation :

UH
z = r (cos θ + i sin θ) where z = r; arg z = θ ; z = r (cos θ − i sin θ)
Note : cos θ + i sin θ is also written as CiS θ
(c) Euler's Formula :
z = reiθ, |z| = r, arg z = θ
JA
z = re−iθ
eiθ = cos θ + i sin θ.
eiθ + e−iθ eiθ − e−iθ
Note : If θ is real then cos θ = ; sin θ =
2 2i
(d) Vectorial Representation :
LP

Every complex number can be considered as the position vector of a point. If the point P represents
→ →
the complex number z then, OP = z &  OP  = z.

Agrument of a Complex Number :


A

Argument of a non-zero complex number P(z) is denoted and defined by arg(z) = angle which OP makes with
the positive direction of real axis.

If OP = |z| = r and arg(z) = θ, then obviously z = r(cosθ + isinθ), called the polar form of z.
NK

'Argument of z' would mean principal argument of z(i.e. argument lying in (– π, π] unless the context requires
otherwise. Thus argument of a complex number z = a + ib = r(cosθ + isinθ) is the value of θ satisfying rcosθ = a
b
and rsinθ = b. Let θ = tan–1
a
SA

π
(ii) a = 0, b > 0 p.v. arg z =
2
RI
π
(vi) a = 0, b < 0 p.v. arg z = –

A
2

UH
JA
Example # 5 : Solve for z if (z)2 + 2|z| = 0.
Solution : Let z = x + iy
LP

⇒ x2 – y2 – 2ixy + 2 x 2 + y 2 = 0 ⇒ x2 – y2 + 2 x 2 + y 2 = 0 and 2xy = 0


when x = 0, ⇒ – y2 + 2|y| = 0
⇒ y = 0, 2, – 2
⇒ z = 0, 2i, – 2i
y=0 ⇒
A

when x2 + 2|x| = 0
⇒ x=0⇒z=0
Ans. z = 0, 2i, –2i.
NK


Example # 6 : Find the modulus and principal argument of complex number z = 1 + i tan α, π < α <
2

Solution : |z| = 1 + tan2 α = |secα| = – secα , where π < α <
2
tan α
SA

Arg (z) = tan–1 = tan– 1 (tanα) = α–π


1
Ans. – secα , α–π

Self Practice Problems


(2 + i) (3 – 4i)
(6) Find the principal argument and |z|. If z =
3+i
(7) Find the |z| and principal argument of the complex number
z =–8(cos 310º – i sin 310°)
5 2
Answers : (6) – π/4 , (7) 8, –130°
2
Demoivre’s Theorem :
(i) (cos θ + i sin θ )n = cos nθ + i sin nθ where n ∈ Ι
(ii) (cos θ1 + i sin θ1) (cos θ2 + i sin θ2) (cosθ3 + i sin θ2) (cos θ3 + i sin θ3) .....(cos θn + i sin θn)
= cos (θ1 + θ2 + θ3 + ......... θn) + i sin (θ1 + θ2 + θ3 + ....... + θn) where n ∈ Ν
(iii) If p, q ∈ Z and q ≠ 0, then (cos θ + i sin θ) p/q can take 'q' distinct values which are equal to
 2kπ + pθ   2kπ + pθ 
cos   + i sin   where k = 0, 1, 2, 3, ......, q – 1
 q   q 
Note : Continued product of the roots of a complex quantity should be determined using theory of equations.

Self practice problems :


(8) Prove the identity: cos 5θ = 16 cos5θ – 20 cos3θ + 5 cos θ;

1 1 3
(9) Prove that identity: cos4θ = cos 4θ + cos 2θ +
8 2 8

RI
Geometrical Representation of Fundamental Operations :

(i) Geometrical representation of addition.

A
UH
If two points P and Q represent complex numbers z1 and z2 respectively in the Argand plane, then the
sum z1 + z2 is represented by the extremity R of the diagonal OR of parallelogram OPRQ having OP
and OQ as two adjacent sides.
JA
(ii) Geometric representation of substraction.
LP

(iii) Modulus and argument of multiplication of two complex numbers.

Theorem : For any two complex numbers z1, z2 we have |z1 z2| = |z1| |z2| and
A

arg (z1z2) = arg (z1) + arg (z2).


Proof : z1 = r1 eiθ1 , z2 = r2 eiθ2
NK

z1z2 = r1r2 ei( θ1 +θ2 ) ⇒ |z1z2| = |z1| |z2|


arg (z1z2) = arg (z1) + arg (z2)
i.e. to multiply two complex numbers, we multiply their absolute values and add their arguments.
Note : (i) P.V. arg (z1z2) ≠ P.V. arg (z1) + P.V. arg (z2)
(ii) |z1 z2 .... zn| = |z1| |z2| ..... |zn|
SA

(iii) arg (z1z2 .... zn) = arg z1 + arg z2 + ..... + arg zn

(iv) Geometrical representation of multiplication of complex numbers.


Let P, Q be represented by z1 = r1 eiθ1 , z2 = r2 eiθ2 repectively. To find point R representing
complex number z1z2 , we take a point L on real axis such that OL = 1 and draw triangle OQR similar to
triangle OLP. Therefore

OR OP ˆ =θ
= ⇒ OR = OP.OQ i.e. OR = r1r2 and QOR 1
OQ OL
ˆ = LOP
LOR ˆ + POQ
ˆ + QOR
ˆ =θ +θ –θ +θ =θ +θ
1 2 1 1 1 2

Hence, R is represented by z1z2 = r1r2 ei( θ1 +θ2 )

(v) Modulus and argument of division of two complex numbers.


z1 |z | z 
Theorem : If z1 and z2 (≠0) are two complex numbers, then = 1 and arg  1  = arg(z1)–arg (z2)
z2 | z2 |  z2 
z 
Note : P.V. arg  1  ≠ P.V. arg (z1) – P.V. arg (z2)
 z2 
(vi) Geometrical representation of the division of complex numbers.
Let P, Q be represented by z1 = r1eiθ1 , z2 = r2 eiθ2 respectively. To find point R representing complex
z1

RI
number , we take a point L on real axis such that OL = 1 and draw a triangle OPR similar to OQL.
z2
OP OR r1 ˆ = LOP
ˆ – ROP
ˆ =θ –θ
Therefore = ⇒ OR = and LOR 1 2
OQ OL r2

A
z1 r
= 1 ei( θ1 −θ2 )

UH
JA
Hence, R is represented by
z2 r2

Conjugate of a complex Number :


Conjugate of a complex number z = a + ib is denoted and defined by z = a – ib.

In a complex number if we replace i by – i, we get conjugate of the complex number. z is the mirror
LP

image of z about real axis on Argand's Plane.

Geometrical representation of conjugate of complex number.


A
NK

|z| = | z |
arg (z) = – arg (z)
General value of arg (z) = 2nπ – P.V. arg (z)
SA

Properties
z+z z−z
(i) If z = x + iy, then x = ,y=
2 2i
(ii) z= z ⇔ z is purely real
(iii) z+ z =0 ⇔ z is purely imaginary
(iv) Relation between modulus and conjugate. |z|2 = z z
(v) z=z
(vi) (z1 ± z2 ) = z1 ± z2
(vii) (z1 z2 ) = z1 z2 , In general (zn ) = (z)n
 z1  (z1 )
(viii)  = (z2 ≠ 0)
 z2  (z2 )
Theorem : Imaginary roots of polynomial equations with real coefficients occur in conjugate pairs
Note : If w = f(z), then w = f( z )
Theorem : |z1 ± z2|2 = |z1|2 + |z2|2 ± (z1 z2 + z1 z2) = |z1|2 + |z2|2 ± 2 Re(z1 z2 )
= |z1|2 + |z2|2 ± 2 |z1| |z2| cos (θ1 – θ2)

z −1
Example # 7 : If is purely imaginary, then prove that | z | = 1
z +1
 z − 1 z − 1  z − 1
Solution : Re   =0 ⇒ +   =0
 z + 1 z + 1  z + 1 
z −1 z −1
⇒ + =0 ⇒ zz –z + z –1+ zz –z + z –1 = 0
z +1 z +1
⇒ zz =1 ⇒ | z |2 = 1 ⇒ | z | = 1 Hence proved

RI
Example # 8 : If z1 and z2 are two complex numbers and c > 0, then prove that
|z1 + z2|2 ≤ (1 + c) |z1|2 + (1 + c–1) |z2|2
Solution : We have to prove : |z1 + z2|2 ≤ (1 + c) |z1|2 + (1 + c–1) |z2|2

A
i.e. |z1|2 + |z2|2 + z1 z 2 + z 1z2 ≤ (1 + c) |z1|2 + (1 +c–1) |z2|2
1
or z1 z 2 + z 1z2 ≤ c|z1|2 + c–1|z2|2 or c|z1|2 + |z |2 – z1 z 2 – z 1 z2 ≥ 0
c 2

UH
2
 1 
(using Re (z1 z 2) ≤ |z1 z 2|) or  c z1 − | z2 |  ≥ 0 which is always true.
 c 
Example # 9 : Let z1 and z2 be complex numbers such that z1 ≠ z2 and |z1| = |z2|. If z1 has positive real part

z1 + z2
JA
and z2 has negative imaginary part, then show that is purely imaginary.
z1 − z2

π π
Solution : z1 = r (cosθ + i sin θ), – <θ<
2 2
z2 = r (cosφ + i sin φ), –π<φ<0
LP

z1 + z2 θ−φ π θ−φ 3π
⇒ = – i cot  , – < <
z1 − z2  2  4 2 4
Hence purely imaginary.
A

Self Practice Problem


(10) If |z + α| > | αz + 1| and | α |> 1, then show that |z| < 1.
(11) If z = x + iy and f(z) = x2 – y2 – 2y + i(2x – 2xy), then show that f(z) = z 2 +2iz
NK

Distance, Triangular Inequality


If z1 = x1 + iy1, z2 = x2 + iy2 , then distance between points z1, z2 in argand plane is

|z1 – z2| = (x1 − x 2 )2 + (y1 − y 2 )2


SA

In triangle OAC
OC ≤ OA + AC
OA ≤ AC + OC
AC ≤ OA + OC
using these in equalities we have ||z1| – |z2|| ≤ |z1 + z2| ≤ |z1| + |z2|
Similarly from triangle OAB
we have ||z1| – |z2|| ≤ |z1 – z2| ≤ |z1| + |z2|
Note :
(a) ||z1| – |z2|| = |z1 + z2|, |z1 – z2| = |z1| + |z2| iff origin, z1 and z2 are collinear and origin lies between z1
and z2.

(b) |z1 + z2| = |z1| + |z2|, ||z1| – |z2|| = |z1 – z2| iff origin, z1 and z2 are collinear and z1 and z2 lies on the
same side of origin.

Example # 10 : If |z – 5 – 7i| = 9, then find the greatest and least values of |z – 2 – 3i|.
Solution : We have 9 = |z – (5 + 7i)| = distance between z and 5 + 7i.
Thus locus of z is the circle of radius 9 and centre at 5 + 7i. For such a z (on the circle), we
have to find its greatest and least distance as from 2 + 3i, which obviously 14 and 4.
Example # 11 : Find the minimum value of |z| + |z – 2|

RI
Solution : |z| + |z – 2| ≥ |z + 2 – z|
|z| + |z – 2| ≥ 2

Example # 12 : If θi ∈ [π/6, π/3], i = 1, 2, 3, 4, 5, and z 4 cos θ1 + z3 cos θ2 + z2 cos θ3. + z cos θ4 + cosθ5 = 2 3 ,

A
3
then show that |z| >
4

UH
Solution : Given that cosθ1 . z4 + cosθ2 . z3 + cosθ3 . z2 + cosθ4 . z + cosθ5 = 2√3

or |cosθ1 . z4 + cosθ2 . z3 + cosθ3 . z2 + cosθ4 . z + cosθ5| = 2√3

2√3 ≤ |cosθ1 . z4 | + |cosθ2 . z3 | + |cosθ3 . z2 | + cosθ4 . z| + |cosθ5 |


JA
 θi ∈ [π/6, π/3]

1 3
∴ ≤ cosθi ≤
2 2

3 4 3 3 3 3
2 3 ≤ |z| + |z|3 + |z|2 + |z| +
LP

2 2 2 2 2
3 ≤ |z|4 + |z|3 + |z|2 + |z|
Case I : If |z| > 1, then above result is automatically true
Case II : If |z| < 1, then
A

3 < |z| + |z|2 + |z|3 + |z|4 +|z|5 + .........∞


|z| 3 3
NK

3< ⇒ 3 – 3 |z| < |z| ⇒ |z| > Hence by both cases, |z|>
1− | z | 4 4

3
Example # 13 : z− = 2, then find maximum and minimum value of | z |.
z
SA

3 3
Solution : z− ≤ z−
z z

Let |z| = r
3 3
r− ≤2 ⇒–2≤r– ≤2
r r

r2 + 2r – 3 ≥ 0 ........(i) and r2 – 2r – 3 ≤ 0 ........(ii)


⇒ r ∈ [1, 3]
from (i) and (ii)
|z|max = 3 and |z|min = 1.
Self Practice Problem
(12) |z – 3| < 1 and |z – 4i| > M then find the positive real value of M for which there exist at least
one complex number z satisfying both the equation.
1 1
(13) If z lies on circle |z| = 2, then show that ≤
z − 4z + 3
4 2
3

Answers : (12) M ∈ (0, 6)


Important results :

RI
eminating from origin making an angle θ with positive direction of real axis

A
UH
θ with positive direction of real axis

π 2π
Example # 14 : Solve for z, which satisfy Arg (z – 3 – 2i) = and Arg (z – 3 – 4i) = .
6 3
Solution : From the figure, it is clear that there is no z, which satisfy both ray
JA
LP

Example # 15 : Sketch the region given by


(i) π/2 ≥ Arg (z – 1 – i) ≥ π/3 (ii) |z| ≤ 4 & Arg (z – i – 1) > π/4
Solution :
π/3
Ιm
A

π/3
NK

Self Practice Problems


(14) Sketch the region given by
SA

(i) |Arg (z – i – 2)| < π/4 (ii) Arg (z + 1 – i) ≤ π/6

(15) Consider the region |z – 4 – 3i| ≤ 3. Find the point in the region which has
(i) max |z| (ii) min |z|
(iii) max arg (z) (iv) min arg (z)

32 24 8 6 28 96
(15) (i) +i (ii) +i (iii) +i (iv) 4 + 0i
5 5 5 5 25 25
Rotation theorem :
(i) If P(z1) and Q(zz) are two complex numbers such that |z1| = |z2|, then z2 = z1 eiθ where θ = ∠POQ

 z − z2  z3 − z2 iθ
(ii) If P(z1), Q(z2) and R(z3) are three complex numbers and ∠PQR = θ, then  3  = e
 z1 − z2  z1 − z2

z3 − z 4 z3 − z 4
(iii) If P(z1), Q(z2), R(z3) and S(z4) are four complex numbers and ∠STQ= θ, then = eiθ
z1 − z2 z1 − z2

RI
A
 z − 1 π
Example # 16 : If arg   = then interpret the locus.

UH
 z + 1 4
 z − 1 π  1− z  π
Solution : arg   = ⇒ arg   =
 z + 1 4  −1 − z  4

 1− z 
Here arg   represents the angle between lines joining –1 and z, and 1 and z. As this
JA
 −1 − z 
angle is constant, the locus of z will be a larger segment of circle. (angle in a segment is
constant).

Example # 17 : If A(2 + 3i) and B(3 + 4i) are two vertices of a square ABCD (take in anticlock wise order) then
find C and D.
LP

Solution : Let affix of C and D are z3 and z4 respectively.


Considering ∠DAB = 90º and AD = AB

z 4 − (2 + 3 i) AD 2
we get = e
A

(3 + 4i) – (2 + 3i) AB
NK

⇒ z4 – (2 + 3i) = (1 + i) i ⇒ z4 = 2 + 3i+ i – 1 = 1 + 4i
SA


z3 − (3 + 4 i) CB –
and = e 2 ⇒ z3 = 3 + 4i – (1 + i) (–i)
(2 + 3i) – (3 + 4i) AB
⇒ z3 = 3 + 4i + i – 1 = 2 + 5i

Self Practice Problems

(16) Let ABC be an isosceles triangle inscribed in the circle |z|= r with AB = AC. If z 1, z2, z3
represent the points A, B, C respectively, show that z2 z3 = z12

(17) Check that z1z2 and z3z4 are parallel or, not
where, z1 = 1 + i z3 = 4 + 2i
z2 = 2 – i z4 = 1 – i
(18) P is a point on the argand diagram on the circle with OP as diameter, two point Q and R are
taken such that ∠POQ = ∠QOR = θ. If O is the origin and P, Q, R are represented by complex
z1, z2, z3 respectively then show that z22 cos 2θ = z1z3cos2θ

(19) If a, b, c ; u, v, w are complex numbers representing the vertices of two triangles such that
c = (1 – r) a + rb, w = (1 – r) u + rv where r is a complex number show that the two triangles are
similiar.
Answers : (17) z1z2 and z3z4 are not parallel.

Cube Root of Unity :


−1 + i 3 −1 − i 3
(i) The cube roots of unity are 1, , .
2 2

RI
(ii) If ω is one of the imaginary cube roots of unity then 1 + ω + ω² = 0. In general 1 + ω r + ω2r = 0;
where r ∈ Ι but is not the multiple of 3.

A
(iii) In polar form the cube roots of unity are :
2π 2π 4π 4π
cos 0 + i sin 0; cos + i sin, cos + i sin
3 3 3 3

UH
(iv) The three cube roots of unity when plotted on the argand plane constitute the verties of an
equilateral triangle.

(v) The following factorisation should be remembered :


(a, b, c ∈ R & ω is the cube root of unity)
JA
a3 − b3 = (a − b) (a − ωb) (a − ω²b) ; x2 + x + 1 = (x − ω) (x − ω2) ;
a + b = (a + b) (a + ωb) (a + ω b) ; a + ab + b = (a – bω) (a – bω )
3 3 2 2 2 2

a3 + b3 + c3 − 3abc = (a + b + c) (a + ωb + ω²c) (a + ω²b + ωc)

Example # 18 : Find the value of ω200 + ω198 + ω193.


ω200 + ω198 + ω193
LP

Solution :
ω2 + 1 + ω = 0.

1 1 1
Example # 19 If W is an imaginary cube root of unity then find the value of + –
1 + 2w 2 + w 1 + w
A

1 1 1 1 1 1
Solution : + – = + –
1 + w + w 1 + (1 + w) 1+ w –w 2 + w 1– w 2 –w 2
1 1 1 w(1 + w) + w 2 + 1– w 2 1+ w + w2
NK

= + + = = =0
w(1– w) (1– w 2 ) w 2 w 2 (1– w 2 ) w 2 (1– w 2 )
Ans. 0

Self Practice Problem


100

∑ (1 + ω
SA

(20) Find r
+ ω2r )
r =0

(21) It is given that n is an odd integer greater than three, but n is not a multiple of 3. Prove that
x3 + x2 + x is a factor of (x + 1)n – xn – 1
(22) If x = a + b, y = aα + bβ , z = aβ + bα where α, β are imaginary cube roots of unity show that
xyz = a3 + b3
5 2
 n 1 
(23) 2
If x – x + 1 = 0, then find the value of ∑
n =1
x + n 
 x 
Answers : (20) 102 (23) 8

nth Roots of Unity :


If 1, α1, α2, α3..... αn − 1 are the n, nth root of unity then :
(i) They are in G.P. with common ratio ei(2π/n)
(ii) 1p + α 1p + α p2 +.... +α pn − 1 = 0 if p is not an integral multiple of n
= n if p is an integral multiple of n
(iii) (1 − α1) (1 − α2)...... (1 − αn − 1) = n &
(1 + α1) (1 + α2)....... (1 + αn − 1) = 0 if n is even and 1 if n is odd.

RI
(iv) 1. α1. α2. α3......... αn − 1 = 1 or −1 according as n is odd or even.

Example # 20 : Find the roots of the equation z5 = – 32i, whose real part is negative.

A
Solution : z5 = – 32i
π
i(4n −1)
z5 = 25 e 2 , n = 0, 1, 2, 3, 4.
π

UH
i(4n −1)
z= 2e 10

π 3π 7π 11π 15 π 7π 11π
−i i i i i i i
z = 2e 10
, 2e 10
, 2e 10 , 2e 10 , 2e 10 roots with negative real part are 2e 10 , 2e 10 .

6
 2πk 2πk 
∑  sin − cos
JA
Example # 21 : Find the value
k =1
7 7 
6 6 6 6
 2πk   2πk  2πk 2πk
Solution : ∑  sin
k =1
 – 
7  k =1 
cos ∑
7 
= ∑ sin
k =0
7
– ∑ cos
k =0
7
+1

6
= ∑ (Sum of imaginary part of seven seventh roots of unity)
LP

k =0
6
– ∑
k =0
(Sum of real part of seven seventh roots of unity) + 1 = 0 – 0 + 1 = 1
A

Self Practice Problems


4

∑2–α
1
(24) If 1, α1, α2 , α3 , α4 are the fifth roots of unity then find
i=1 i
NK

(25) If α, β, γ are the roots of x3 – 3x2 + 3x + 7 = 0 and ω is a complex cube root of unity then prove
α –1 β –1 γ –1
that + + = 3ω2
β –1 γ –1 α –1
SA

(26) Find all values of (− 256)1/4. Interpret the result geometrically.


49
Answers : (24)
31
  2r + 1   2r + 1  
(26) 4 cos   π + isin   π  , r = 0, 1, 2, 3; vertices of a square in a
  4   4  
circle of radius 4 & centre (0, 0)

The Sum Of The Following Series Should Be Remembered :


sin ( nθ / 2 )  n + 1
(i) cos θ + cos 2 θ + cos 3 θ +..... + cos n θ = cos   θ.
sin ( θ / 2 )  2 
sin ( nθ / 2 )  n + 1
(ii) sin θ + sin 2 θ + sin 3 θ +..... + sin n θ = sin   θ.
sin ( θ / 2 )  2 

Note : If θ = (2π/n) then the sum of the above series vanishes.

Geometrical Properties :
Section formula
If z1 and z2 are affixes of the two points P and Q respectively and point C divides the line segment
joining P and Q internally in the ratio m : n then affix z of C is given by
mz2 + nz1
z= where m, n > 0

RI
m+n
mz2 − nz1
If C divides PQ in the ratio m : n externally then z =
m−n

A
Note : If a, b, c are three real numbers such that az1 + bz2 + cz3 = 0 ; where a + b + c = 0 and a,b,c are not all
simultaneously zero, then the complex numbers z1, z2 & z3 are collinear.

UH
(1) If the vertices A, B, C of a ∆ are represented by complex numbers z 1, z2, z3 respectively and a, b, c are
the length of sides then,
z1 + z2 + z3
(i) Centroid of the ∆ ABC = :
3
JA
(ii) Orthocentre of the ∆ ABC =
( a sec A ) z1 + ( b sec B ) z2 + ( c sec C ) z3
or
z1 tan A + z2 tanB + z3 tan C
a sec A + b sec B + c sec C tan A + tan B + tanC
(iii) Incentre of the ∆ ABC = (az1 + bz2 + cz3) ÷ (a + b + c).
(iv) Circumcentre of the ∆ ABC = :
LP

(Z1 sin 2A + Z2 sin 2B + Z3 sin 2C) ÷ (sin 2A + sin 2B + sin 2C).

(2) amp(z) = θ is a ray emanating from the origin inclined at an angle θ to the positive x− axis.

(3) z − a = z − b is the perpendicular bisector of the line joining a to b.


A

(4) The equation of a line joining z1 & z2 is given by, z = z1 + t (z1 − z2) where t is a real parameter.
NK

(5) z = z1 (1 + it) where t is a real parameter is a line through the point z 1 & perpendicular to the line joining
z1 to the origin.

(6) The equation of a line passing through z1 & z2 can be expressed in the determinant form as
SA

z z 1
z1 z1 1 = 0. This is also the condition for three complex numbers z, z 1, z2 to be collinear. The above
z2 z2 1
equation on manipulating, takes the form α z + α z + r = 0 where r is real and α is a non zero complex
constant.

z − z2 π
(7) The equation of the circle described on the line segment joining z 1 & z2 as diameter is arg =±
z − z1 2
or (z − z1) ( z − z 2) + (z − z2) ( z − z 1) = 0.
z3 − z1 z 4 − z2
(8) Condition for four given points z1, z2, z3 & z4 to be concyclic is the number . should be
z3 − z2 z 4 − z1
real. Hence the equation of a circle through 3 non collinear points z1, z2 & z3 can be taken as
( z − z2 ) ( z3 − z1 ) is real ⇒ ( z − z2 ) ( z3 − z1 ) = ( z − z2 ) ( z3 − z1 ) .
( z − z1 ) ( z3 − z2 ) ( z − z1 ) ( z3 − z2 ) ( z − z1 ) ( z3 − z2 )

 z − z1 
(9) Arg   = θ represent (i) a line segment if θ = π
 z − z2 
(ii) Pair of ray if θ = 0 (iii) a part of circle, if 0 < θ < π.

(10) If |z – z1| + |z – z2| = K > |z1 – z2| then locus of z is an ellipse whose focii are z1 & z2

RI
z − z1
(11) If =k where k ∈ (0, 1) ∪ (1, ∞), then locus of z is circle.
z − z2

A
(12) If z – z1  – z – z2 = K < z1 – z2 then locus of z is a hyperbola, whose focii are z1 & z2.

UH
Match the following columns :

Column - Ι Column - ΙΙ
(i) If | z – 3+2i | – | z + i | = 0, (i) circle
then locus of z represents ..........
JA
 z − 1 π
(ii) If arg  = , (ii) Straight line
 z + 1 4
then locus of z represents...
(iii) if | z – 8 – 2i | + | z – 5 – 6i | = 5 (iii) Ellipse
then locus of z represents .......
LP

 z − 3 + 4i  5π
(iv) If arg  = , (iv) Hyperbola
 z + 2 − 5i  6
then locus of z represents .......

(v) If | z – 1 | + | z + i | = 10 (v) Major Arc


A

then locus of z represents ........


(vi) |z–3+i|–|z+2–i|=1 (vi) Minor arc
NK

then locus of z represents .....

(vii) | z – 3i | = 25 (vii) Perpendicular bisector of a line segment


 z − 3 + 5i 
(viii) arg  =π (viii) Line segment
 z+i 
SA

Ans. Ι (i) (ii) (iii) (iv) (v) (vi) (vii) (viii)


ΙΙ (ii),(vii)(v) (viii) (vi) (iii) (iv) (i) (viii)

Example # 22 : If z1, z2 & z3 are the affixes of three points A, B & C respectively and satisfy the condition

|z1 – z2| = |z1| + |z2| and |(2 - i) z1 + iz3 | = |z1| + |(1 – i) z1 + iz3| then prove that ∆ ABC in a right

angled.
Solution : |z1 – z2| = |z1| + |z2|

⇒ z1, z2 and origin will be collinear and z1, z2 will be opposite side of origin

Similarly |(2 - i) z1 + iz3 | = |z1| + |(1 – i) z1 + iz3|

⇒ z1 and (1 – i) z1 + iz3 = z4 say, are collinear with origin and lies on same
side of origin. Let z4 = λz1 , λ real

then (1 – i) z1 + iz3 = λz1

(z3 − z1 ) z − z1
⇒ i (z3 – z1) = (λ – 1) z1 ⇒ = (λ – 1) I ⇒ 3 =meiπ/2 , m = λ – 1
−z1 0 − z1

⇒ z3 – z1 is perpendicular to the vector 0 – z1 .

i.e. also z2 is on line joining origin and z1

so we can say the triangle formed by z1 , z2 and z3 is right angled.

RI
A
UH
JA
LP
A
NK
SA
Marked questions are recommended for Revision.

PART - I : SUBJECTIVE QUESTIONS


Section (A) : Algebra of Complex Numbers and Its Representation and Demoivre’s
Theorem

A-1. Find the real values of x and y for which the following equation is satisfied :
(1 + i) x − 2i (2 − 3i) y + i
(i) + =i
3+i 3−i
x y 5 + 6i
(ii) + =

RI
1 + 2i 3 + 2i 8i − 1

(iii) (2 + 3i) x2 – (3 – 2i) y = 2x – 3y + 5i

A
(iv) 4x2 + 3xy + (2xy – 3x2) i = 4y2 – (x2/2) + (3xy – 2y2) i

1 + 2(sin θ)i
A-2_. Let z =
1 − (sin θ)i

UH
(i) Find the number of values of θ∈[0, 4π] such that z is purely imaginary.
(ii) Find the sum of all values of θ∈[0, 4π] such that z is purely real.

A-3. (i) Find the real values of x and y for which z1 = 9y2 – 4 – 10ix and z2 = 8y2 – 20i are conjugate
complex of each other.
JA
(ii) Find the value of x4 – x3 + x2 + 3x – 5 if x = 2 + 3i

A-4. Find
(i) the square root of 7 + 24 i (ii) i + −i

A-5. Solve the following for z :


LP

z2 – (3 – 2i) z = (5i – 5)

A-6. Simplify and express the result in the form of a + bi :


2
 4i3 − i 
(i) − i (9 + 6 i) (2 − i)−1 (ii)  
A

 2i + 1 
π
1 −i
(iii)
(1 − cos θ) + 2 i sin θ
(iv) ( )
3 +i e 6
NK

A-7. Convert the following complex numbers in Eulers form


 π π
(i) z = –π (ii) z = 5i (iii) z = – 3–i (iv) z = –2  cos + isin 
 5 5 
SA

A-8. Find the modulus, argument and the principal argument of the complex numbers.
18 π 18 π
(i) z = 1 + cos + i sin (ii) z = −2 (cos 30° + i sin 30°)
25 25
i −1
(iii) (tan 1 – i)2 (iv)
 2π  2π
i  1 − cos  + sin
 5  5

A-9. Dividing polynomial f(z) by z – i, we get the remainder i and dividing it by z + i, we get the remainder
1 + i. Find the remainder upon the division of f(z) by z 2 + 1.

A-10. If ( 3 + i)100 = 299 (a + ib), then find


(i) a 2 + b2 (ii) b

A-11. If n is a positive integer, prove the following


θ nθ
(i) (1 + cos θ + i sin θ)n + (1 + cos θ – i sin θ)n = 2n + 1 cosn cos .
2 2
n
+1 nπ
(ii) (1 + i)n + (1 – i)n = 2 2 . cos
4

m
 icot θ + 1 
A-12. Show that ei2mθ   = 1.
 icot θ − 1 

 π  π 
A-13. If xr = cos  r  + i sin  r  , prove that x1 x2 x3....... upto infinity = i.
3  3 

Section (B) : Argument / Modulus / Conjugate Properties and Triangle Inequality

RI
B-1. If z = x + iy is a complex number such that z = (a + ib) 2 then
(i) find z
(ii) show that x2 + y2 = (a2 + b2)2

A
B-2. If z1 and z2 are conjugate to each other, then find arg (–z1z2).

UH
z −1
B-3. If z (≠ – 1) is a complex number such that is purely imaginary, then find |z|
z +1

4
B-4. If |z – 2| = 2 |z – 1|, where z is a complex number, prove |z|2 = Re (z) using
3
JA
(i) polar form of z, (ii) z = x + iy, (iii) modulus, conjugate properties

B-5. For any two complex numbers z1, z2 and any two real numbers a, b show that
|az1 – bz2|2 + |bz1 + az2|2 = (a2 + b2) (|z1|2 + |z2|2)

1 − z1z2
B-6. If z1 and z2 are two complex numbers such that |z1| < 1 < |z2| then prove that < 1.
LP

z1 − z2

z−w
B-7. If k > 0, |z| = |w| = k and α = , then find Re(α).
k 2 + zw
A

z+i
B-8. (i) If w = is purely real then find arg z.
z−i
NK

z + 4i
(ii) If w = is purely imaginary then find |z + 3i|.
z + 2i

B-9. If a = eiα , b = eiβ, c = eiγ and cos α + cos β + cos γ = 0 = sin α + sin β + sin γ, then prove the following
(i) a+b+c=0 (ii) ab + bc + ca = 0
(iii) a2 + b 2 + c 2 = 0 (iv) Σ cos 2α = 0 = Σ sin 2α
SA

B-10. If |z – 1 + i| + |z + i| = 1 then find range of principle argument of z.

Section (C) : Geometry of Complex Number and Rotation Theorem

C-1. If z − 2 + i = 2, then find the greatest and least value of z.

C-2. If |z + 3| ≤ 3 then find minimum and maximum values of


(i) |z| (ii) |z – 1| (iii) |z + 1|

C-3. Interpret the following locus in z ∈ C.


(i) 1 < z − 2i < 3 (ii) Im (z) ≥ 1
 z + 2i 
(iii) Arg (z − 3 – 4i) = π/3 (iv) Re   ≤ 4 (z ≠ 2i)
 iz + 2 
C-4. If O is origin and affixes of P, Q, R are respectively z, iz, z + iz. Locate the points on complex plane. If
∆PQR = 200 then find
(i) |z| (ii) sides of quadrilateral OPRQ

C-5. The three vertices of a triangle are represented by the complex numbers, 0, z 1 and z2. If the triangle is
equilateral, then show that z12 + z22 = z1z2 . Further if z0 is circumcentre then prove that z12 + z22 = 3z02.

C-6. Let z1 and z2 be two roots of the equation z2 + az + b = 0, z being complex. Further, assume that the
origin, z1 and z2 form an equilateral triangle. Then show that a2 = 3b.

C-7. Let z1 = 10 + 6i and z2 = 4 + 6i. If z is any complex number such that the argument of (z – z 1) / (z – z2) is
π/4, then find the length of arc of the locus.

RI
 z − 8i  π
C-8. Let Ι: Arg   =±
 z+6  2
 z − 8i 

A
ΙΙ : Re   =0
 z+6 
Show that locus of z in Ι or ΙΙ lies on x2 + y2 + 6x – 8y = 0. Hence show that locus of z can also be
z − 8i z + 8i

UH
represented by + = 0. Further if locus of z is expressed as |z + 3 – 4i| = R, then find R.
z+6 z+6

C-9. Show that z z + (4 – 3i)z + (4 + 3i) z + 5 = 0 represents circle. Hence find centre and radius.

z1 + z2 π
JA
C-10. If z1 & z2 are two complex numbers & if arg = but z1 + z2 ≠ z1 − z2 then identify the figure
z1 − z2 2
formed by the points represented by 0, z1, z2 & z1 + z2.

Section (D) : Cube root and nth Root of Unity.

If ω (≠ 1) be a cube root of unity and (1 + ω4)n = (1 + ω2)n then find the least positive integral value of n
LP

D-1.

D-2. When the polynomial 5x3 + Mx + N is divided by x2 + x + 1, the remainder is 0. Then find M + N.

Show that (1 – ω + ω2) (1 – ω2 + ω4) (1 – ω4 + ω8) .......... to 2n factors = 22n


A

D-3.

D-4. Let ω is non-real root of x3 = 1


NK

(i) If P = ωn , (n ∈ N) and
P
Q = (2nC0 + 2nC3 + .........) + (2nC1 + 2nC4 + .........)ω + (2nC2 + 2nC5 + .........)ω2 then find .
Q
ω ω2 ω3 1 − ω2
(ii) If P = 1 – + − ...... upto ∞ terms and Q = then find value of PQ.
2 4 8 2
SA

D-5. If x = 1 + i 3 ; y = 1 – i 3 and z = 2, then prove that xp + yp = zp for every prime p > 3.

D-6. Solve (z – 1)4 – 16 = 0. Find sum of roots. Locate roots, sum of roots and centroid of polygon formed by
roots in complex plane.

D-7. Find the value(s) of the following


3 3/4
1 −3  1 −3 
(i)  +  (ii)  + 
2 2  2 2 
 
Hence find continued product if two or more distinct values exists.

D-8. If 1, α1, α2, α3, α4 be the roots of x5 − 1 = 0, then find the value
ω − α1 ω − α2 ω − α3 ω − α4
of . . . (where ω is imaginary cube root of unity.)
ω − α1 ω − α 2 ω − α3 ω2 − α 4
2 2 2

2π 2π
D-9. a = cos + i sin then find the quadratic equation whose roots are α = a + a2 + a4 and
7 7
β = a 3 + a5 + a6

PART - II : ONLY ONE OPTION CORRECT TYPE


Section (A) : Algebra of Complex Numbers and Its Representation and
Demoivre’s Theorem

RI
A-1. If z is a complex number such that |z| = 4 and arg(z) = , then z is equal to
6
(A) – 2 3 + 2i (B) 2 3 +i (C) 2 3 – 2i (D) – 3 +i

A
A-2. The complex numbers sin x + i cos 2x and cos x – i sin 2x are conjugate to each other, for

(A) x = nπ (B) x = 0 (C) x = (D) no value of x
2

UH
n
 1+ i 
A.3. The least value of n (n ∈ N), for which   is real, is
 1− i 
(A) 1 (B) 2 (C) 3 (D) 4
JA
1
A-4. In G.P. the first term & common ratio are both
2
( )
3 + i , then the modulus of nth term is :
(A) 1 (B) 2n (C) 4n (D) 3n

A-5. If z = (3 + 7i) (p + iq), where p, q ∈ Ι – {0}, is purely imaginary, then minimum value of |z| 2 is
3364
(A) 0 (B) 58 (C) (D) 3364
LP

3
x y
A-6. If z = x + iy and z1/3 = a − ib then
a b
(
− = k a2 − b2 where k = )
(A) 1 (B) 2 (C) 3 (D) 4
A

π  1− π i π −i   |z| 
A-7. If z = (1 + i)4  +  , then   equals

4  π + i 1+ π i   amp(z) 
(A) 1 (B) π (C) 3π (D) 4
NK

A-8. The set of values of a ∈ R for which x2 + i(a – 1) x + 5 = 0 will have a pair of conjugate imaginary roots
is
(A) R (B) {1}
(C) {a : a2 – 2a + 21 > 0} (D) {0}
SA

A-9. Let z is a complex number satisfying the equation, z 3 – (3 + i) z + m + 2i = 0, where m ∈ R. Suppose


the equation has a real root α, then find the value of α4 + m4
(A) 32 (B) 16 (C) 8 (D) 64

n
 1 + i tan α  1 + i tannα
A-10. The expression   − when simplified reduces to :
 1 − i tan α  1 − i tannα
(A) zero (B) 2 sin n α (C) 2 cos n α (D) none

A-11. If (cosθ + i sinθ) (cos 2θ + i sin 2θ) ... (cos nθ + i sin nθ) = 1, then the value of θ is
3mπ 2mπ 4mπ mπ
(A) ,m∈Z (B) ,m∈Z (C) ,m∈Z (D) ,m∈Z
n(n + 1) n (n + 1) n (n + 1) n (n + 1)
A-12. Let principle argument of complex number be re-defined between (π, 3π], then sum of principle
arguments of roots of equation z6 + z3 + 1 = 0 is
(A) 0 (B) 3π (C) 6π (D) 12π

Section (B) : Argument / Modulus / Conjugate Properties and Triangle Inequality


z −1
B-1. If |z| = 1 and ω = (where z ≠ –1), the Re(ω) is
z +1
1 z 1 2
(A) 0 (B) − (C) . (D)
| z + 1|2 z + 1 | z + 1|2 | z + 1|2

(1 + b + ia)
B-2. If a2 + b2 = 1, then =
(1 + b − ia)

RI
(A) 1 (B) 2 (C) b + ia (D) a + ib

B-3. If (2 + i)(2 + 2i) (2 + 3i) ...... (2 + ni) = x + iy, then the value of 5.8.13. .......(4 + n 2)

(x 2
+ y2 )

A
(A) (x2 + y2) (B) (C) 2(x2 + y2) (D) (x + y)

B-4. If z = x + iy satisfies amp (z – 1) = amp (z + 3) then the value of (x – 1) : y is equal to

UH
(A) 2 : 1 (B) 1 : 3 (C) – 1 : 3 (D) does not exist

B-5. If z1 = − 3 + 5 i; z2 = − 5 − 3 i and z is a complex number lying on the line segment joining


z1 & z2, then arg(z) can be :
3π π π 5π
(A) − (B) −
JA
(C) (D)
4 4 6 6

B-6. If (1 + i)z = (1 – i) z then z is


t t
(A) t(1 – i), t ∈ R (B) t (1 + i), t ∈ R (C) , t ∈ R+ (D) , t ∈ R+
1+ i 1− i
LP

B-7. Let z and ω be two non-zero complex numbers such that |z| = |ω| and arg z = π – arg ω, then z equals
(A) ω (B) –ω (C) ω (D) – ω

z1 3π z1
A

B-8. If z1 and z2 are two non-zero complex numbers such that = 2 and arg(z1z2) = , then is
z2 2 z2
equal to
NK

(A) 2 (B) –2 (C) –2i (D) 2i

B-9. Number of complex numbers z such that |z| = 1 and | z / z + z / z |= 1 is (arg(z) ∈ [0, 2π])
(A) 4 (B) 6 (C) 8 (D) more than 8
SA

B-10. If |z1| = |z2| and arg (z1/z2) = π, then z1 + z2 is equal to


(A) 1 (B) 3 (C) 0 (D) 2

B-11. The number of solutions of the system of equations Re (z 2) = 0, |z| = 2 is


(A) 4 (B) 3 (C) 2 (D) 1

B-12. If |z2 – 1| = |z2| + 1, then z lies on :


(A) the real axis (B) the imaginary axis (C) a circle (D) an ellipse

B-13. If |z – 2i| + |z – 2| ≥ ||z| – |z – 2 – 2i||, then locus of z is


(A) circle (B) line segment (C) point (D) complete x-y plane

Section (C) : Geometry of Complex Number and Rotation Theorem


z − 5i
C-1. The complex number z = x + iy which satisfy the equation = 1 lie on :
z + 5i
(A) the x-axis (B) the straight line y = 5
(C) a circle passing through the origin (D) the y-axis

C-2. The inequality |z – 4| < | z – 2| represents :


(A) Re(z) > 0 (B) Re(z) < 0 (C) Re (z) > 2 (D) Re(z) > 3

C-3. Let A, B, C represent the complex numbers z 1, z2, z3 respectively on the complex plane. If the
circumcentre of the triangle ABC lies at the origin, then the orthocentre is represented by the complex
number :
(A) z1 + z2 − z3 (B) z2 + z3 − z1 (C) z3 + z1 − z2 (D) z1 + z2 + z3

RI
π
C-4. If Arg (z – 2 – 3i) = , then the locus of z is
4

A
π/4
(A) (2, 3) (B)

UH
π/4
(2, 3)
JA
| z + 1 − i | = 2
C-5. The system of equations  , where z is a complex number has :
Re z ≥ 1
(A) no solution (B) exactly one solution
LP

(C) two distinct solutions (D) infinite solution

C-6. The locus of z which lies in shaded region is best represented by


i
A

1
NK

–i

−π π −π
≤ arg z ≤ (B) |z| = 1, ≤ arg z ≤ 0
2 2 2
π −π
(C) |z| ≥ 0, 0 ≤ arg z ≤ (D) |z| ≤ 1, ≤ arg z ≤ π
2 2
SA

C-7. The equation |z – 1|2 + |z + 1|2 = 2 represents


(A) a circle of radius '1' (B) a straight line
(C) the ordered pair (0, 0) (D) None of these

z−2 π
C-8. If arg  = then locus of z is :
z−4 3
(A) equileteral triangle (B) arc of circle
π
(C) arc of ellipse (D) two rays making angle between them
3

C-9. The region of Argand diagram defined by |z – 1| + |z + 1| ≤ 4 is :


(A) interior of an ellipse (B) exterior of a circle
(C) interior and boundary of an ellipse (D) exterior of ellipse
C-10. The vector z = − 4 + 5i is turned counter clockwise through an angle of 180º & stretched 1.5 times. The
complex number corresponding to the newly obtained vector is :
15 15 15 15
(A) 6 − i (B) − 6 + i (C) 6 + i (D) −6 − i
2 2 2 2

C-11. The points z1, z2, z3, z4 in the complex plane are the vertices of a parallelogram taken in order if and
only if :
(A) z1 + z4 = z2 + z3 (B) z1 + z3 = z2 + z4 (C) z1 + z2 = z3 + z4 (D) z1 z3 = z2 z4

C-12.Complex numbers z1 , z2 , z3 are the vertices A, B, C respectively of an isosceles right angled triangle
with right angle at C and (z1 – z2)2 = k(z1 – z3) (z3 – z2), then find k
(A) 1 (B) 2 (C) 3 (D) –2

RI
C-13. If z1, z2, z3 are vertices of an equilateral triangle inscribed in the circle |z| = 2 and if z 1 = 1 + i 3 , then
(A) z2 = – 2, z3 = 1 + i 3 (B) z2 = 2, z3 = 1 – i 3
(C) z2 = – 2, z3 = 1 – i 3 (D) z2 = 1 – i 3 , z3 = – 1 – i 3

A
Section (D) : Cube root of unity and nth Root of Unity.

Let z1 and z2 be two non real complex cube roots of unity and |z –z1|2 + |z – z2|2 = λ be the equation of a

UH
D-1.
circle with z1, z2 as ends of a diameter then the value of λ is
(A) 4 (B) 3 (C) 2 (D) 2

D-2. If x = a + b + c, y = aα + bβ + c and z = aβ + bα + c, where α and β are imaginary cube roots of unity,


JA
then xyz =
(A) 2(a3 + b3 + c3) (B) 2(a3 – b3 – c3) (C) a3 + b3 + c3 – 3abc (D) a3 – b3 – c3

1 ωn ω2n
D-3. If 1, ω, ω2 are the cube roots of unity, then ∆ = ωn ω2n 1 is equal to-
ω
2n
1 ω n
LP

(A) 0 (B) 1 (C) ω (D) ω2

D-4. If x2 + x + 1 = 0, then the numerical value of


2 2 2 2 2
 1  2 1   3 1   4 1   27 1 
A

 x + x  +  x + 2  +  x + 3  +  x + 4  + ...... +  x + 27  is equal to
   x   x   x   x 
(A) 54 (B) 36 (C) 27 (D) 18
NK

x3 x6 x 4 x7 x2 x5 x8
D-5. If a = 1 + + + ...... , b=x+ + + ...... , c = + + + ......
3! 6! 4! 7! 2! 5! 8!
then find a3 + b3 + c3 – 3abc.

(A) 1 (B) 2 (C) 3 (D) 4


SA

D-6. If equation (z – 1)n = zn = 1(n ∈ N) has solutions, then n can be :


(A) 2 (B) 3 (C) 6 (D) 9

−2
− α −1|
If α is non real and α = 5 1 then the value of 2|1 + α + α + α
2
D-7. is equal to
(A) 4 (B) 2 (C) 1 (D) 8

D-8. If α = ei8π/11 then Real (α + α2 + α3 + α4 + α5) equals to :


1
(A) (B) 1 (C) – (D) –1
2
PART - III : MATCH THE COLUMN
1. Match the column
Column – Ι Column – ΙΙ
(Complex number Z) (Principal argument of Z)
2
(1 + i)5 (1 + )
3i
(A) Z= . (p) π
−2i ( − 3 + i )

6π  6π  7π
(B) Z = sin + i  1 + cos is (q) −
5  5  18
 11π   11π  9π
(C) Z = 1 + cos   + i sin   (r)
 9   9  10

RI
(D) Z = sinx sin(x – 60) sin(x + 60)
 π 5π
where x∈  0,  and x ∈ R (s) –
 3 12
(t) 0

A
2. Column I Column II
(one of the values of z)

UH
π π
(A) z4 – 1 = 0 p. z = cos + i sin
8 8
π π
(B) z4 + 1 = 0 q. z = cos – i sin
8 8
π π
JA
(C) iz4 + 1 = 0 r. z = cos + i sin
4 4
(D) iz4 – 1 = 0 s. z = cos 0 + i sin 0

3. Which of the condition/ conditions in column II are satisfied by the quadrilateral formed by z 1, z2, z3, z4 in
order given in column I ?
Column - I Column-II
LP

(A) Parallelogram (p) z1 – z4 = z2 – z3


(B) Rectangle (q) |z1 – z3| = |z2 – z4|
z1 − z2
(C) Rhombus (r) is real
z3 − z 4
z1 − z3
A

(D) Square (s) is purely imaginary


z2 − z 4
z1 − z2
(t) is purely imaginary
NK

z3 − z 2

4. Let z1 lies on | z | = 1 and z2 lies on | z | = 2.


Column – Ι Column – ΙΙ
(A) Maximum value of | z1 + z2 | (p) 3
SA

(B) Minimum value of | z1 – z2 | (q) 1


(C) Minimum value of | 2z1 + 3z2 | (r) 4
(D) Maximum value of | z1 – 2z2| (s) 5
 Marked questions are recommended for Revision.
PART - I : ONLY ONE OPTION CORRECT TYPE

1 
1. sin–1  (z − 1)  , where z is nonreal, can be the angle of a triangle if
 i 
(A) Re(z) = 1, Ιm(z) = 2 (B) Re(z) = 1, 0 < Ιm (z) ≤ 1
(C) Re(z) + Ιm(z) = 0 (D) Re(z) = 2, 0 < Ιm (z) ≤ 1

2. If |z|2 – 2iz + 2c(1 + i) = 0, then the value of z is, where c is real.

RI
(A) z = c + 1 i(–1 ± 1 − 2c − c 2 ), where c∈[–1 – 2 , –1 + 2 ]
(B) z = c – 1 i(–1 ± 1 − 2c − c 2 ), where c∈[–1 – 2 , –1 + 2 ]
(C) z = 2c + 1 i(–1 ± 1 − 2c − c 2 ), where c∈[–1 – 2 , –1 + 2 ]

A
(D) z = c + 1 i(–1 ±) 1 − 2c − c , where c∈[–1 –
2
2,1+ 2 ]

3. If (a + ib)5 = α + iβ , then (b + ia)5 is equal to

UH
(A) β + iα (B) α – iβ (C) β – iα (D) – α – iβ

1 + z + z2
4. Let z be non real number such that ∈ R, then value of 7|z| is
1 − z + z2
(A) 1 (B) 3
JA (C) 5 (D) 7

5. If |z1| = 2, |z2| = 3, |z3| = 4 and |z1 + z2 + z3| = 2, then the value of |4z2z3 + 9z3z1 + 16z1z2|
(A) 24 (B) 48 (C) 96 (D) 120

6. The minimum value of |3z–3| + |2z–4| equal to


(A) 1 (B) 2 (C) 3 (D) 4
LP

7. If |z1 – 1| < 1, |z2 – 2| < 2, |z3 – 3| < 3, then |z1 + z2 + z3|


(A) is less than 6 (B) is more than 3
(C) is less than 12 (D) lies between 6 and 12
A

8. Let O = (0, 0) ; A=(3, 0) ; B = (0, –1) and C=(3, 2),then minimum value of |z|+|z–3|+|z+i|+|z–3–2i| occur
at
(A) intersection point of AB and CO (B) intersection point of AC and BO
(C) intersection point of CB and AO (D) mean of O, A, B, C
NK

9. Given z is a complex number with modulus 1. Then the equation [(1 + ia)/(1 – ia)] 4 = z in 'a' has
(A) all roots real and distinct (B) two real and two imaginary
(C) three roots real and one imaginary (D) one root real and three imaginary

10. The real values of the parameter ‘a’ for which at least one complex number z = x + iy satisfies both the
SA

equality z − ai = a + 4 and the inequality z − 2 < 1.


 21 5   7 5 5 7  21 7 
(A)  − , −  (B)  − , −  (C)  ,  (D)  − , 
 10 6   2 6 6 2  10 2 

11. The points of intersection of the two curves z − 3 = 2 and z = 2 in an argand plane are:
1 1 3 7 7 3
(A)
2
(
7±i 3 ) (B)
2
3±i 7( ) (C)
2
±i
2
(D)
2
±i
2

12. The equation of the radical axis of the two circles represented by the equations,
z − 2 = 3 and z − 2 − 3 i = 4 on the complex plane is :
(A) 3iz – 3i z – 2 = 0 (B) 3iz – 3i z + 2 = 0 (C) iz – i z + 1 = 0 (D) 2iz – 2i z + 3 = 0
 z −1 + 4 
13. If log1/2   > 1, then the locus of z is
3 z − 1 − 2
 
(A) Exterior to circle with center 1 + i0 and radius 10
(B) Interior to circle with center 1 + i0 and radius 10
(C) Circle with center 1 + i0 and radius 10
(D) Circle with center 2 + i0 and radius 10

14. Points z1 & z2 are adjacent vertices of a regular octagon. The vertex z3 adjacent to z2 (z3 ≠ z1) is
represented by :
1 1
(A) z2 + (1 ± i) (z1 + z2) (B) z2 + (1 ± i) (z1 − z2)
2 2
1
(C) z2 + (1 ± i) (z2 − z1) (D) none of these

RI
2

15. If p = a + bω + cω2; q = b + cω + aω2 and r = c + aω + bω2 where a, b, c ≠ 0 and ω is the non-real


complex cube root of unity, then :

A
(A) p + q + r = a + b + c (B) p2 + q2 + r2 = a2 + b2 + c2
(C) p2 + q2 + r2 = 2(pq + qr + rp) (D) None of these

16. The points z1 = 3 + 3 i and z2 = 2 3 + 6 i are given on a complex plane. The complex number lying

UH
on the bisector of the angle formed by the vectors z1 and z2 is :
(3 + 2 3 ) 3 +2
(A) z = + i (B) z = 5 + 5 i
2 2
(C) z = − 1 − i (D) none
JA
17. Let ω be the non real cube root of unity which satisfy the equation h(x) = 0 where h(x) = x f(x 3) + x2g(x3).
If h(x) is polynomial with real coefficient then which statement is incorrect.
(A) f(1) = 0 (B) g(1) = 0 (C) h(1) = 0 (D) g(1) ≠ f(1)

18. If 1, α1, α2, α3,......., αn − 1 be the nth roots of unity, then the value of
π 2π 3π (n − 1) π
LP

sin . sin . sin ........ sin equals


n n n n
n n n +1 n
(A) n (B) n − 1 (C) n − 1 (D)
2 2 2 2n + 1
A

PART - II : SINGLE AND DOUBLE VALUE INTEGER TYPE


N
1. If a and b are positive integer such that N = (a + ib) 3 – 107i is a positive integer then find the value of
2
NK

2. Let z,w be complex numbers such that z + iw = 0 and arg zw = π. If Re(z) < 0 and principal arg

z= then find the value of a + b. (where a & b are co-prime natural numbers)
b

∑ (1 + i)
−r
If x = 91/3 91/9 91/27 ...∞, y = 41/3 4–1/9 41/27 ... ∞, and z =
SA

3. and principal argument of P = (x + yz) is


r =1
 a
–tan–1 
 b 
then determine a2 + b2. (where a & b are co-prime natural numbers)
 

4. z1, z2 ∈ c and z12 + z22 ∈ R,


z1(z12 – 3z22) = 2, z2 (3z12 – z22) = 11
If z12 + z22 = λ then determine λ2

z +1
5. Let |z| = 2 and w = where z, w∈C (where C is the set of complex numbers), then find product of
z −1
maximum and minimum value of |w|.
6. A function 'f' is defined by f(z) = (4 + i)z2 + αz + γ for all complex number z, where α and γ are complex
numbers if f(1) and f(i) are both real and the smallest possible values of |α| + |γ| is p then determine p 2.

π
7. If z and ω are two non-zero complex numbers such that |zω| = 1, and arg (z) – arg(ω) = , then find
2
the value of 5 i z ω.

8. Number of complex number satisfying |z| = max {|z – 1|, |z + 1|}.

π
9. If z1 & z2 both satisfy the relation, z + z = 2 z − 1 and arg (z1 − z2) = , then find the imaginary part
4
of (z1 + z2).

RI
10. If a1, a2, a3.... an, A1, A2, A3..... An, k are all real numbers and number of imaginary roots of the equation
A12 A 22 A n2
+ + ...... + = k is α. Then find the value of α + 15.
x − a1 x − a2 x − an

A
n

∑a z
1 r
11. How many complex number z such that | z | < and r = 1 where |ar| < 2.
3

UH
r =1

12. If a variable circle S touches S1 : |z – z1| = 7 internally and S2 : |z – z2| = 4 externally while the curves S1
& S2 touch internally to each other, (z1 ≠ z2). If the eccentricity of the locus of the centre of the curve S is
‘e’ find the value of 11e.
JA
z−2
13. Given that, z − 1 = 1, where ' z ' is a point on the argand plane. = α i tan (arg z). Then
2z
1
determine .
α4

π π
LP

14. Area of the region formed by z ≤ 4 & – ≤ arg z ≤ on the Argand diagram is expressed in the
2 3

form of . Then find the value of ab (where a & b are co-prime natural number)
b
A

15. The points A, B, C represent the complex numbers z1, z2, z3 respectively on a complex plane & the
1
angle B & C of the triangle ABC are each equal to ( π − α ) . If
2
NK

α
(z2 − z3)² = λ (z3 − z1) (z1 − z2) sin2 then determine λ.
2

1 1 1
16. If ω and ω2 are the non-real cube roots of unity and a, b, c ∈ R such that + + = 2ω2
a+ω b+ω c+ω
SA

1 1 1 1 1 1
and + + = 2ω. If + + = λ then determine λ4
a+ω 2
b+ω 2
c+ω 2 a +1 b +1 c +1

 n n n 
17. If L = lim  + + ...... + 
n →∞ (1 − nω)(1 − nω )
2
(2 − nω)(2 − nω )
2
(n − nω)(n − nω ) 
2

π
then find the value of {where ω is non real cube root of unity}.
3L

6
 2 πk 2 πk 
18. The value of ∑  sin
k =1  7
− icos
7 
= α then find α4
r
 i 2π   1+ Z1 + Z 2 + Z3 + ......... + Z 7  aπ
19. Let Zr =  e 15  . If arg  = , then b – a equals. (where a & b are co-
 
   1+ Z8 + Z9 + Z10 + ....... + Z14  b
prime natural number)

1 1 1
20. If A1, A2, ......., An be the vertices of an n-sided regular polygon such that = + , then
A1A 2 A1A 3 A1A 4
find the value of n

PART - III : ONE OR MORE THAN ONE OPTIONS CORRECT TYPE


1. If the biquadratic x4 + ax3 + bx2 + cx + d = 0 (a, b, c, d ∈ R) has 4 non real roots, two with sum 3 + 4i
and the other two with product 13 + i.

RI
(A) b = 51 (B) a = –6 (C) c = –70 (D) d = 170

2. The quadratic equation z2 + (p + ip′) z + q + iq′ = 0; where p, p′, q, q′ are all real.
(A) if the equation has one real root then q ′2 − pp ′ q ′ + qp ′2 = 0.

A
(B) if the equation has two equal roots then pp ′ = 2q ′.
(C) if the equation has two equal roots then p 2 − p′2 = 4q
(D) if the equation has one real root then p ′2 − pp ′ q ′ + q ′2 = 0.

UH
3. The value of in + i−n, for i = − 1 and n ∈ Ι is :
2n (1+ i)2n (1+ i)2n (1− i)2n (1+ i)2n 2n 2n 2n
(A) + (B) + (C) + (D) +
(1− i)2n 2n 2n 2n 2n (1− i)2n (1+ i)2n (1− i)2n
JA
4.  If amp (z1z2) = 0 and |z1| = |z2| = 1, then
(A) z1 + z2 = 0 (B) z1z2 = 1 (C) z1 = z2 (D) z1 = z2

π
5. Let z1 and z2 are two complex numbers such that (1 – i)z1 = 2z2 and arg(z1z2) = , then arg(z2) is equal
2
LP

to
(A) 3π/8 (B) π/8 (C) 5π/8 (D) – 7π/8

6. If |z1 + z2|2 = |z1|2 + |zz|2 (where z1 and z2 are non-zero complex numbers), then
A

z1 z1
(A) is purely real (B) is purely imaginary
z2 z2
z1 π
(C) z1z2 + z2 z1 = 0 (D) amp may be equal to
NK

z2 2

7. a, b, c are real numbers in the polynomial, P(z) = 2z4 + az3 + bz2 + cz + 3. If two roots of the equation
P(z) = 0 are 2 and i. Then which of the following are true.
SA

11 11
(A) a = – (B) b = 5 (C) c = – (D) a = –11
2 2

8. If Z =
(1 + i)(1 + 2i)(1 + 3i) ...... (1 + ni) , n ∈ N then principal argument of Z can be
(1 − i)( 2 − i)( 3 − i) ...... (n − i)
π π
(A) 0 (B) (C) – (D) π
2 2

9.  For complex numbers z and w, if |z|2w – |w|2 z = z – w. Which of the following can be true :
(A) z = w (B) z w = 1 (C) z = w + 2 (D) z w = 1

10. If z satisfies the inequality |z – 1 – 2i| ≤ 1, then which of the following are true.
(A) maximum value of |z| = 5 +1 (B) minimum value of |z| = 5 − 1
3
(C) maximum value of arg(z) = π/2 (D) minimum value of arg(z) = tan–1  
4

3
11. The curve represented by z = , θ ∈ [0, 2π)
2 + cos θ + i sin θ
(A) never meets the imaginary axis (B) meets the real axis in exactly two points
(C) has maximum value of |z| as 3 (D) has minimum value of |z| as 1

12. POQ is a straight line through the origin O . P and Q represent the complex number a + i b and c + i d
respectively and OP = OQ. Then which of the follwoing are true :

RI
(A) |a + i b| = | c + i d| (B) a + c = b + d
(C) arg (a + i b) = arg (c + i d) (D) none of these

13. Let i = −1 . Define a sequence of complex number by z1 = 0, zn+1 = zn2 + i for n ≥ 1. Then which of the

A
following are true.
(A) |z2050| = 3 (B) |z2017| = 2 (C) |z2016| = 1 (D) |z2111| = 2

UH
14. If |z1| = |z2| = ...... = |zn| = 1 then which of the following are true.
1
(A) z1 =
z1
JA
1 1 1
(B) |z1 + z2 + ..... + zn| = + + ...... + .
z1 z2 zn
1 1 1
(C) Centroid of polygon with 2n vertices z1, z2, ....... zn , , ..... (need not be in order) lies on
z1 z2 zn
real axis.
1 1 1
LP

(D) Centroid of polygon with 2n vertices z1, z2, ...... zn , ,.... (need not be in order) lies on
z1 z2 zn
imaginary axis.

1 1
15. If 2 cosθ = x + and 2 cosφ = y + , then
A

x y
1 x y
(A) xn + n
= 2 cos (nθ), n ∈ z (B) + = 2 cos (θ – φ)
x y x
NK

1 1
(C) xy + = 2 cos (θ + φ) (D) xmyn + m n
= 2cos(mθ + n φ), m, n ∈ z
xy x y

z−α
16. If = k, k > 0 where, z = x + iy and α = α1 + iα2, β = β1 + iβ2 are fixed complex numbers. Then
z−β
SA

which of the following are true


 k 2β − α 
(A) if k ≠ 1 then locus is a circle whose centre is  2
 k − 1 
 
k(α − β)
(B) if k ≠ 1 then locus is a circle whose radius is
1− k2
(C) if k = 1 then locus is perpendicular bisector of line joining α = α1 + iα2 and β = β1 + iβ2
 k 2α − β 
(D) if k ≠ 1 then locus is a circle whose centre is  2
 k − 1 
 
 z − 1 − 2i  π
17. The locus of equation Arg  = represents part of circle in which
 z+3+i  3
 3 1 2  5
(A) centre is  −1 − + i +  (B) radius is
 2 2 3   3
 3 1 2  5
(C) centre is  −1 − − i +  (D) radius is
 2 2 3   3

18. The equation ||z + i| – |z – i|| = k represents


(A) a hyperbola if 0 < k < 2 (B) a pair of ray if k > 2
(C) a straight line if k = 0 (D) a pair of ray if k = 2

19. The equation |z – i| + |z + i| = k, k > 0, can represent

RI
(A) an ellipse if k > 2 (B) line segment if k = 2
(C) an ellipse if k = 5 (D) no locus if k = 1

20. If |z1| = |z2| = |z3| = 1 and z1, z2, z3 are represented by the vertices of an equilateral triangle then

A
(A) z1 + z2 + z3 = 0 (B) z1z2z3 = 1
(C) z1z2 + z2z3 + z3z1 = 0 (D) z23 + z33 = 2z13

UH
21. Let z1, z2, z3 be three distinct complex numbers satisfying, z1−1 = z2−1 = z3−1= 1. Let A, B & C
be the points representing vertices of equilateral triangle in the Argand plane corresponding to z 1, z2
and z3 respectively. Which of the following are true
(A) z1 + z2 + z3 = 3 (B) z12 + z22 + z32 = 3
3 3
JA
(C) area of triangle = (D) z1z2 + z2z3 + z3z1 = 1
4

22. If 1, α1, α2, α3,......., αn − 1 be the nth roots of unity, then which of the following are true
1 1 1 n −1
(A) + + ...... + =
1 − α1 1 − α 2 1 − αn−1 2
LP

(B) (1 − α1) (1 − α2) (1 − α3)........ (1 − αn − 1) = n.


(C) (2 – α1) (2 − α2) (2 − α3)........ (2 − αn − 1) = 2n – 1
1 1 1 n
(D) + + ...... + =
1 − α1 1 − α 2 1 − αn−1 2
A

23. Which of the following are true.


x n + 2
(A) cos x + nC1 cos 2x + nC2 cos 3x +..... + nCn cos (n + 1) x = 2n. cosn . cos  x
NK

2  2 
x n + 2
(B) sin x + nC1 sin 2x + nC2 sin 3x +..... + nCn sin (n + 1) x = 2n. cosn . sin   x
2  2 
x  nx 
(C) 1 + nC1 cosx + nC2 cos 2x +..... + nCn cos nx = 2n. cosn . cos  
2  2 
SA

x  nx 
(D) nC1 sinx + nC2 sin 2x +..... + nCn sin n x = 2n. cosn . sin  
2  2 

24. If α, β, γ are distinct roots of x3 – 3x2 + 3x + 7 = 0 and ω is non-real cube root of unity, then the value of
α −1 β −1 γ −1
+ + can be equal to :
β −1 γ −1 α −1
(A) ω2 (B) 2ω2 (C) 3ω2 (D) 3ω

25. If z is a complex number then the equation z2 + z z + z2 = 0 is satisfied by


(ω and ω2 are imaginary cube roots of unity)
(A) z = k ω where k ∈ R (B) z = k ω2 where k is non negative real
(C) z = k ω where k is positive real (D) z = k ω2 where k ∈ R.
26. If α is imaginary nth (n ≥ 3) root of unity. Which of the following are true.
n −1 n −1
nα 2rπ n π
(A) ∑ (n − r) α
r =1
r
=
1− α
(B) ∑ (n − r) sin
r =1
n
= cot .
2 n
n −1 n −1
2rπ
∑ (n − r) cos ∑ (n − r) α
n r n
(C) =– (D) =
r =1
n 2 r =1
1− α

27. Which of the following is true


(A) roots of the equation z10 − z5 − 992 = 0 with real part positive = 5
(B) roots of the equation z10 − z5 − 992 = 0 with real part negative = 5
(C) roots of the equation z10 − z5 − 992 = 0 with imaginary part non-negative = 6
(D) roots of the equation z10 − z5 − 992 = 0 with imaginary part negative = 4

RI
PART - IV : COMPREHENSION
Comprehension # 1 (Q. No. 1 - 2)

A
Let (1 + x)n = C0 + C1x + C2x2 + .......+ Cnxn. For sum of series C0 + C1 + C2 + ........., put x = 1. For sum of
series C0 + C2 + C4 + C6 + ........, or C1 + C3 + C5 + ........ add or substract equations obtained by putting
x = 1 and x = – 1.

UH
For sum of series C0 + C3 + C6 + ........ or C1 + C4 + C7 + ....... or C2 + C5 + C8 + ....... we subsitute x = 1,
x = ω, x = ω2 and add or manipulate results.

Similarly, if suffixes differe by ‘p’ then we substitute pth roots of unity and add.
JA
1. C0 + C3 + C6 + C9 +........ =
1  n nπ 1  n nπ 1  n nπ 1  n nπ
(A) 2 − 2cos 3  (B) 2 + 2 cos 3  (C) 3 2 − 2 sin 3  (D) 2 + 2sin 3 
3   3     3  

2. C1 + C5 + C9 +.... =
LP

1  n nπ 1  n nπ
(A) 2 − 2n / 2 2cos (B) 2 + 2n / 2 2cos
4  4  4  4 
1  n nπ 1  n nπ
(C)  2 − 2n / 2 2sin (D) 2 + 2n / 2 2sin
4  4  
4  4 
A

Comprehension # 2 (Q. No. 3 to 6)

As we know eiθ = cosθ + isinθ and (cosθ1 + isinθ1).(cosθ2 + isinθ2) = cos(θ1 + θ2) + i sin(θ1 + θ2)
NK

3
Let α, β, γ ∈ R such that cos (α – β) + cos (β – γ) + cos (γ – α) = –
2

3. Σ sin (α + β) = Σ cos (α + β) =
(A) 0 (B) 3cosα cosβ cosγ (C) 3 cos (α + β + γ) (D) 3
SA

4. Σcos(2α – β – γ )
(A) 0 (B) 3cosα cosβ cosγ (C) 3 cos (α + β + γ) (D) 3

5. Σ cos 3 α =
(A) 0 (B) 3cosα cosβ cosγ (C) 3 cos (α + β + γ) (D) 3

Σ cos3 (θ + α )
6. If θ ∈ R then =
Π cos(θ + α )
(A) 0 (B) 3cosα cosβ cosγ (C) 3 cos (α + β + γ) (D) 3

Comprehension # 3(Q. No. 7 to 8)


ABCD is a rhombus. Its diagonals AC and BD intersect at the point M and satisfy BD = 2AC. Let the
points D and M represent complex numbers 1 + i and 2 – i respecetively.
If θ is arbitary real, then z = reiθ, R1 ≤ r ≤ R2 lies in annular region formed by concentric circles
| z | = R1, | z | = R 2.

7. A possible representation of point A is


i i 3 3
(A) 3 – (B) 3 + (C) 1 + i (D) 3 – i
2 2 2 2

8. If z is any point on segment DM then w = eiz lies in annular region formed by concentric circles.
1
(A) |w |min = 1, |w|max = 2 (B) | w |min = , | w |max = e
e
1 1
(C) |w |min = 2 , | w |max = e2 (D) | w |min = , | w |max = 1
e 2

RI
Comprehension # 4 (Q. No. 9 to 10)
Logarithm of a complex number is given by
loge(x + iy) = loge(|z|eiθ)

A
= loge|z| + logeeiθ
= loge |z| + iθ

(x 2
+ y2 )

UH
= loge + i arg(z)
∴ loge(z) = loge|z| + i arg(z)
1  −1 y 
In general loge (x + iy) = loge (x² + y²) + i  2n π + tan x  where n ∈ Ι.
2  
JA
9. Write loge (1 + 3 i) in (a + ib) form
π π
(A) loge2 + i(2nπ + ) (B) loge3 + i(nπ + )
3 3
π π
(C) loge2 + i(2nπ + ) (D) loge2 + i(2nπ – )
6 3
LP

10. Find the real part of (1 – i)–i.


1  1 
(A) eπ/4 + 2nπ cos  loge 2  (B) e–π/4 + 2nπ cos  loge 2 
 2   2 
 1 
(loge 2 )
A

(C) e–π/4 + 2nπ cos (D) e–π/2 + 2nπ cos  loge 2 


 2 
NK
SA
PART - I : JEE (ADVANCED) / IIT-JEE PROBLEMS (PREVIOUS YEARS)
 Marked questions are recommended for Revision.

15
1. Let z = cos θ + i sin θ . Then the value of
m =1
∑ Ιm (z 2m −1 ) at θ = 2º is

[IIT-JEE-2009, Paper-I, (3, – 1), 80]


1 1 1 1
(A) (B) (C) (D)
sin2º 3 sin2º 2sin2º 4 sin2º

RI
2. Let z = x + iy be a complex number where x and y are integers. Then the area of the rectangle whose
vertices are the roots of the equation zz 3 + zz3 = 350 is [IIT-JEE-2009, Paper-I, (3, – 1), 80]
(A) 48 (B) 32 (C) 40 (D) 80

A
3*. Let z1 and z2 be two distinct complex numbers and let z = (1 – t) z1 + tz2 for some real number t with
0 < t < 1. If Arg(w) denotes the principal argument of a nonzero complex number w, then
(A) |z – z1| + |z – z2| = |z1 – z2| (B) Arg (z – z1) = Arg (z – z2)

UH
z − z1 z − z1
(C) =0 (D) Arg (z – z1) = Arg (z2 – z1)
z 2 − z1 z2 − z1
[IIT-JEE-2010, Paper-1, (3, 0)/84]

2π 2π
JA
4. Let ω be the complex number cos + i sin . Then the number of distinct complex numbers z
3 3
z +1 ω ω2
satisfying ω z + ω2 1 = 0 is equal to [IIT-JEE-2010, Paper-1, (3, 0)/84]
ω2 1 z+ω
LP

5. Match the statements in Column-I with those in Column-II. [IIT-JEE-2010, Paper-2, (8, 0)/79]
[Note : Here z takes values in the complex plane and Im z and Re z denote, respectively, the
imaginary part and the real part of z.]

Column-I Column-II
A

4
(A) The set of points z satisfying (p) an ellipse with eccentricity
5
|z – i| z|| = |z + i|z|| is contained in
NK

or equal to

(B) The set of points z satisfying (q) the set of points z satisfying Im z = 0
|z + 4| + |z – 4| = 10 is contained in
or equal to
1
SA

(C) If |w| = 2, then the set of points z = w – (r) the set of point z satisfying |Im z| ≤ 1
w
is contained in or equal to
1
(D) If |w| = 1, then the set of points z = w + (s) the set of points z satisfying |Re z| ≤ 2
w
is contained in or equal to
(t) the set of points z satisfying |z| ≤ 3

6. If z is any complex number satisfying |z – 3 – 2i| ≤ 2, then the minimum value of |2z – 6 + 5i| is
[IIT-JEE 2011, Paper-1, (4, 0), 80]
πi
7. Let ω = e 3 , and a, b, c, x, y, z be non-zero complex numbers such that
a+b+c=x
a + bω + cω2 = y
a + bω2 + cω = z.
| x |2 + | y |2 + | z |2
Then the value of is [IIT-JEE 2011, Paper-2, (4, 0), 80]
| a |2 + | b |2 + | c |2

8. Let z be a complex number such that the imaginary part of z is non zero and a = z 2 + z + 1 is real. Then
a cannot take the value [IIT-JEE 2012, Paper-1, (3, –1), 70]
1 1 3
(A) –1 (B) (C) (D)
3 2 4
1
9. Let complex numbers α and lies on circles (x – x0)2 + (y – y0)2 = r2 and (x – x0)2 + (y – y0)2 = 4r2,
α
respectively. If z0 = x0 + iy0 satisfies the equation 2|z0|2 = r2 + 2, then |α| =

RI
[JEE (Advanced) 2013, Paper-1, (2, 0)/60]
1 1 1 1
(A) (B) (C) (D)
2 2 7 3

A
3 +i  1
10.* Let w = and P = {wn : n = 1,2,3,....}. Further H1 = z ∈ C : Re z>  and H2 =
2  2

UH
 1
z ∈ C : Re z < –  , where C is the set of all complex numbers. If z1 ∈ P ∩ H1, z2 ∈ P ∩ H2 and O
 2

represents the origin, then ∠z1O z2 = [JEE (Advanced) 2013, Paper-2, (3, –1)/60]

π π 2π 5π
(A) (B)
JA (C) (D)
2 6 3 6

11.* Let ω be a complex cube root of unity with ω ≠ 1 and P = [pij] be a n × n matrix with pij = ω i +j
. Then

P2 ≠ 0, when n = [JEE (Advanced) 2013, Paper-2, (3, –1)/60]


(A) 57 (B) 55 (C) 58 (D) 56
LP

Paragraph for Question Nos. 12 to 13


Let S = S 1 ∩ S 2 ∩ S 3 , where
A

  z – 1+ 3 i  
S 1 = {z ∈ C : |z| < 4}, S 2 = z ∈ C : Ιm   > 0  and
  1– 3 i  
NK

S 3 : {z ∈ C : Re z > 0}.

12. Area of S = [JEE (Advanced) 2013, Paper-2, (3, –1)/60]


10π 20π 16π 32π
SA

(A) (B) (C) (D)


3 3 3 3

13. min | 1– 3i – z | = [JEE (Advanced) 2013, Paper-2, (3, –1)/60]


z∈S

2– 3 2+ 3 3– 3 3+ 3
(A) (B) (C) (D)
2 2 2 2

 2kπ   2kπ 
14. Let z k = cos   + isin   ; k = 1, 2,...9 . [JEE (Advanced) 2014, Paper-2, (3, –1)/60]
 10   10 
List I List II
P. For each z k there exists a z j such that z k . z j = 1 1. True

Q. There exists a k ∈ {1,2,....,9} such that z 1 . . z = z k has 2. False


no solution z in the set of complex numbers.

| 1– z1 || 1– z 2 | ..... | 1– z 9 |
R. equals 3. 1
10

9
 2kπ 
S. 1 – ∑ cos  10
k =1
 equals

4. 2

P Q R S
(A) 1 2 4 3

RI
(B) 2 1 3 4
(C) 1 2 3 4
(D) 2 1 4 3

A
 kπ   kπ 
15. For any integer k, let α k = cos   + i sin   , where i = –1 . The value of the
 
7  7 
12

∑α

UH
k +1 – αk
k =1
expression 3
is [JEE (Advanced) 2015, P-2 (4, 0) / 80]
∑α
k =1
4k –1 – α 4k – 2

−1 + 3 i ( −z)r z 2s 
JA
16. Let z = , where i =−1 and r, s ∈ {1,2,3}. Let P =  2s  and Ι be the identity matrix of
2  z zr 
order 2. Then the total number of ordered pairs (r, s) for which P2 = – Ι is
[JEE (Advanced) 2016, Paper-1, (3, 0)/62]

 1 
17. Let a, b ∈Rand a2 + b2 ≠ 0. Suppose S = z ∈ R : z = , t ∈ R, t ≠ 0  , where i = −1 .
LP

 a + ibt 
If z = x + iy and z ∈ S, then (x, y) lies on [JEE (Advanced) 2016, Paper-2, (4, –2)/62]
1  1 
(A) the circle with radius and centre  ,0  for a > 0 , b ≠ 0
2a  2a 
A

1  1 
(B) the circle with radius – and centre  ,0  for a < 0, b ≠ 0
2a  2a 
(C) the x-axis for a ≠ 0, b = 0
NK

(D) the y-axis for a = 0, b ≠ 0

18. Let a, b, x and y be real numbers such that a – b = 1 and y ≠ 0. If the complex number z = x + iy
 az + b 
satisfies Im   = y, then which of the following is(are) possible value(s) of x ?
 z +1 
SA

[JEE(Advanced) 2017, Paper-1,(4, –2)/61]


(A) 1 – 1 + y2 (B) – 1– 1 − y2 (C) 1 + 1 + y2 (D) – 1 + 1 − y2
19. For a non-zero complex number z, let arg(z) denote the principal argument with –π < arg(z) ≤ π. Then,
which of the following statement(s) is (are) FALSE ?
π
(A) Arg(–1 –i) = , where i = –1 [JEE(Advanced) 2018, Paper-1,(4, –2)/60]
4

(B) The function f : R → (–π, π], defined by f(t) = arg(–1 + it) for all t ∈ R, is continuous at all points
of R, where i = –1

z 
(C) For any two non-zero complex numbers z1 and z2 , arg  1  − arg(z1 ) + arg(z2 ) is an integer
 z2 
multiple of 2π
(D) For any three given distinct complex numbers z1, z2 and z3, the locus of the point z satisfying
 (z − z1 )(z 2 − z3 ) 
the condition arg   = π, lies on a straight line.
 (z − z3 )(z2 − z1 ) 

20. Let s, t, r be non-zero complex numbers and L be the set of solutions z = x + iy (x, y ∈ R, i = –1 ) of
the equation sz + t z + r = 0, where z = x – iy. Then,which of the following statement(s) is (are) TRUE ?

(A) If L has exactly one element, then |s| ≠ |t| [JEE(Advanced) 2018, Paper-2,(4, –2)/60]

(B) If |s| = |t|, then L has infinitely many elements

(C) The number of elements in L ∩ {z : |z – 1 + i| = 5} is at most 2

RI
(D) If L has more than one element, then L has infinitely many elements

PART - II : JEE (MAIN) / AIEEE PROBLEMS (PREVIOUS YEARS)

A
4
1. If z − = 2, then the maximum value of |z| is equal to : [AIEEE 2009, (4, –1), 144]

UH
z
(1) 5 +1 (2) 2 (3) 2 + 2 (4) 3 +1

2. If α and β are the roots of the equation x 2 – x + 1 = 0, then α2009 + β2009 = [AIEEE 2010, (4, –1), 144]
(1) – 1 (2) 1 (3) 2 (4) –2
JA
3. The number of complex numbers z such that | z – 1| = | z + 1| = |z – i| equals
[AIEEE 2010, (4, –1), 120]
(1) 1 (2) 2 (3) ∞ (4) 0

4. If ω(≠1) is a cube root of unity, and (1 + ω) 7 = A + Bω . Then (A, B) equals[AIEEE 2011, I, (4, –1), 120]
LP

(1) (0, 1) (2) (1, 1) (3) (1, 0) (4) (–1, 1)

5. Let α, β be real and z be a complex number. If z 2 + αz + β = 0 has two distinct roots on the line Re z =
1, then it is necessary that : [AIEEE- 2011, I, (4, –1), 120]
(1) β ∈ (0, 1) (2) β ∈ (–1, 0) (3) |β| = 1 (4) β ∈ (1, ∞)
A

 1+ z 
6. If z is a complex number of unit modulus and argument θ, then arg   equals :
 1+ z 
π
NK

(1) –θ (2) −θ (3) θ (4) π – θ


2
[AIEEE - 2013, (4, –1), 120]

1
7. If z a complex number such that |z| ≥ 2, then the minimum value of z + :
2
SA

(1) is strictly greater than 5/2 [JEE(Main) 2014, (4, – 1), 120]
(2) is strictly greater than 3/2 but less than 5/2
(3) is equal to 5/2
(4) lie in the interval (1, 2)

8. A complex number z is said to be unimodular if |z| = 1. Suppose z 1 and z2 are complex numbers such
z – 2z2
that 1 is unimodular and z2 is not unimodular. Then the point z1 lies on a :
2 – z1z2
[JEE(Main) 2015, (4, – 1), 120]
(1) straight line parallel to x-axis (2) straight line parallel to y-axis
(3) circle of radius 2 (4) circle of radius 2
2 + 3i sin θ
9. A value of θ for which is purely imaginary, is : [JEE(Main) 2016, (4, – 1), 120]
1– 2i sin θ
π  3  1  π
(2) sin–1 
 4 
(1) (3) sin–1   (4)
6    3 3

1 1 1
10. Let ω be a complex number such that 2ω + 1 = z where z = −3 . If 1 −ω − 1 ω2 = 3k, then k is
2

1 ω2 ω7
equal to : [JEE(Main) 2017, (4, – 1), 120]
(1) –z (2) z (3) –1 (4) 1

RI
11. If α, β ∈ C are the distinct roots, of the equation x 2 – x + 1 = 0, then α101 + β107 is equal to :
[JEE(Main) 2018, (4, – 1), 120]
(1) 1 (2) 2 (3) –1 (4) 0

A
12. Let α and β be two roots of the equation x 2 + 2x + 2 = 0, then α15 + β15 is equal to :
[JEE(Main) 2019, Online (09-01-19),P-1 (4, – 1), 120]
(1) 512 (2) –256 (3) 256 (4) –512

UH
13. Let z be a complex number such that |z| + z = 3 + i, (where i = − 1 ) then |z| is equal to :
[JEE(Main) 2019, Online (11-01-19),P-2 (4, – 1), 120]
34 41
(1) (2) 5/4 (3) 5/3 (4)
3 4
JA
14. Let z1 and z2 be two complex numbers satisfying |z1| = 9 and |z2 – 3 – 4i| = 4. Then the minimum value
of |z1 – z2| is : [JEE(Main) 2019, Online (12-01-19),P-2 (4, – 1), 120]
(1) 0 (2) 1 (3) 2 (4) 2
LP
A
NK
SA
EXERCISE - 1

PART – I

 5 3K
A-1. (i) 3, –1 (ii) x = 1 and y = 2 (iii) (1, 1)  0,  (iv) x = K, y = ,K∈R
 2 2

A-2_. (i) 8 (ii) 10 π A-3. (i) [(–2, 2) ; (–2, –2)] (ii) – (77 + 108 i)

A-4. (i) ± (4 + 3i) (ii) ± 2 + 0i or 0 ± 2 I A-5. z = (2 + i) or (1 – 3i)

RI
θ
− cot
21 12 1 2
A-6. (i) − i (ii) 3 + 4 i (iii) + i (iv) 2
5 5  θ θ
2  1 + 3cos2  1 + 3 cos2

A
 2 2
π 5π 4π
i −i −i
A-7. (i) πeiπ (ii) 5e 2 (iii) 2e 6 c (iv) 2e 5

UH
9π 9π 9π
A-8. (i) z = 2 cos Principal Arg z = , arg z = + 2kπ, k ∈ Ι
25 25 25
5π 5π
(ii) Modulus = 2, Arg = 2 k π – , k ∈ Ι, Principal Arg = −
6 6
JA
(iii) Modulus = sec21, arg = 2kπ + (2 – π), Principal Arg = (2 – π)
1 π 11π 11π
(iv) Modulus = cosec , arg z = 2kπ + , Principal Arg =
2 5 20 20

iz 1
A-9. + +i A-10. (i) 4 (ii) 3
2 2
LP

Section (B) :

π
B-1. (i) (a – ib)2 B-2. π B-3. 1 B-7. 0 B-8. (i) ± (ii) 1
A

 π π
B-10. arg z∈  − , − 
NK

 2 4

Section (C) :
C-1. 5+2& 5 −2 C-2. (i) 0, 6 (ii) 1, 7 (iii) 0, 5
SA

C-3. (i) The region between the concentric circles with centre at (0, 2) & radii 1 and 3
units
(ii) The part of the complex plane on or above the line y = 1
(iii) a ray emanating from the point (3 + 4i) directed away from the origin & having equation,
3 x − y + 4 − 3 3 = 0, x > 3
1 1
(iv) Region outside or on the circle with centre + 2i and radius
2 2
π
C-4. (i) |z| = 20 (ii) OP = OQ = PR = QR = 20 C-7. 9 C-8. 5 C-9. –4 – 3 i, 2 5
2

C-10. a rhombous but not a square


Section (D) :

D-1. 3 D-2. –5 D-3. 4n D-4. (i) 1 (ii) 1

D-6. z = – 1, 3, 1 – 2 i, 1 + 2 i
1 + 2i

–1 1 3 4

1 – 2i

Sum = 4
centroid = 1

RI
(6n +1) i
D-7. (i) – 1 (ii) e 4 , n = 0, 1, 2, 3. Continued product = 1

D-8. ω D-9. x2 + x + 2 = 0

A
PART - II

UH
Section (A) :
A-1. (A) A-2. (D) A.3. (B) A-4. (A) A-5. (D) A-6. (D) A-7. (D)
A-8. (B) A-9. (A) A-10. (A) A-11. (C) A-12. (D)
Section (B) :
JA
B-1. (A) B-2. (C) B-3. (A) B-4. (D) B-5. (D) B-6. (A) B-7. (D)
B-8. (D) B-9. (C) B-10. (C) B-11. (A) B-12. (B) B-13. (D)
Section (C) :
C-1. (A) C-2. (D) C-3. (D) C-4. (A) C-5. (B) C-6. (A) C-7. (C)
LP

C-8. (B) C-9. (C) C-10. (A) C-11. (B) C-12. (B) C-13. (C)
Section (D) :
D-1. (B) D-2. (C) D-3. (A) D-4. (A) D-5. (A) D-6. (C) D-7. (A)
A

D-8. (C)
PART - III
NK

1. (A) → (s), (B) → (r), (C) → (q), (D) → (p) 2. A → s; B → r; C → p; D → q.


3. a → p, r; b → p,q,r, t; c → p,r,s; d → p,q,r,s, t. 4. (A) → (p), (B) → (q), (C) → (r), (D) → (s)

EXERCISE - 2
SA

PART - I
1. (B) 2. (A) 3. (A) 4. (D) 5. (B) 6. (B) 7. (C) 8. (C)
9. (A) 10. (A) 11. (B) 12. (B) 13. (A) 14. (C) 15. (C) 16. (B)
17. (D) 18. (B)
PART - II
1. 99 2. 7 3. 13 4. 25 5. 1 6. 2 7. 5
8. 0 9. 2 10. 15 11. 0 12. 3 13. 16 14. 60
15. 4 16. 16 17. 3 18. 11 9. 1 20. 7
PART - III
1. (ABCD) 2. (ABC) 3. (BD) 4. (BC) 5. (BD) 6. (BCD)
7. (ABC) 8. (ABCD) 9. (ABD) 10. (ABCD) 11. (ABCD) 12. (AB)
13. (BCD) 14. (ABC) 15. (ABCD) 16. (ABC) 17. (AB) 18. (ACD)
19. (ABCD) 20. (ACD) 21. (ABC) 22. (ABC) 23. (ABCD) 24. (CD)
25. (BC) 26. (ABC) 27. (ABCD)
PART - IV
1. (B) 2. (D) 3. (A) 4. (D) 5. (C) 6. (D) 7. (A)
8. (B) 9. (A) 10. (B) 9. (A) 10. (B)

EXERCISE - 3

RI
PART - I
1. (D) 2. (A) 3*. (ACD) 4. 1

A
5. (A) - (q,r), (B)-(p), (C) - (p,s,t), (D) - (q,r,s,t) 6. 5

7. Bonus (w = eiπ / 3 is a typographical error, because of this the answer cannot be an integer.)

UH

i
3
(if w = e then answer comes out to be 3)

8. (D) 9. (C) 10.* (CD) 11.* (BCD) 11. (B) 13. (C) 14. (C)
JA
15. 4 16. 1 17. (ACD) 18. (BD) 19. (ABD) 20. (ACD)

PART - II
LP

1. (1) 2. (2) 3. (1) 4. (2) 5. (4) 6. (3)


7. (4) 8. (3) 9. (3) 10. (1) 11. (1) 12. (2) 13. (3)
14. (1)
A
NK
SA
1. If the equation z4 + a1z3 + a2z2 + a3z + a4 = 0 where a1, a2, a3, a4 are real coefficient different from zero,
a3 aa
has a purely imaginary root, then find the value of + 1 4 .
a1a2 a2 a3

2. If |z1| = 2, |z2| = 3, |z3| = 4 and |2z1 + 3z2 + 4z3| = 4, then find the value of |8z2z3 + 27z3z1 + 64z1z2|

3. If | z |2 + A z 2 + Az 2 + Bz + Bz + c = 0 represents a pair of intersecting lines with angle of


intersection ‘θ’ then find the value of |A|.

RI
4. If z 2 + αz + β = 0 (α, β are complex numbers) has a real root then prove that
2
( α − α ) ( αβ − αβ ) = (β − β ) .

A
5. If z1, z2, z3 be three complex number such that
z12 z2 z2

UH
|z1| = |z2| = |z3| = 1 and + 2 + 3 +1=0
z 2 z3 z1z3 z1z 2
then sum of all the possible values of |z1 + z2 + z3|.

1
6. Number of complex number (z) satisfying |z|2 = |z|n –2z2 + |z|n –2z + 1 such that Re(z) ≠ – and
JA
2
n = 2λ + 1, λ ∈ N.

7. Let z1 & z2 be any two arbitrary complex numbers then prove that
z1 z 1 z1 z
(i) |z1 + z2| = | z 2 | + 2 | z1 | (ii) z1 + z2 ≥ (z1 + z2 ) + 2 .
| z1 | | z2 | 2 z1 z2
LP

8. Prove that
z
(i) − 1 ≤ | arg z|. (ii) |z – 1| ≤ ||z| – 1| + |z| |arg z|.
|z|
A

9. Prove that
|Img(zn)| ≤ n |Img (z)||z|n –1 , n∈I⊕
NK

10. If z − 1 + z + 3 ≤ 8 then find the range of values of z − 4 .

11. Show that all the roots of the equation a1z3 + a2z2 + a3z + a4 = 3, where |ai| ≤ 1, i = 1, 2, 3, 4 lie outside
the circle with centre origin and radius 2/3.
SA

12. Consider the locus of the complex number z in the Argand plane is given by Re(z) –2 = |z – 7 + 2i|. Let
P(z1) and Q (z2) be two complex number satisfying the given locus and also satisfying
 z - (2 + αi)  π
arg  1  = (α ∈ R) then find the minimum value of PQ
 z 2 - (2 + αi)  2

z − z1
13. Find the mirror image of the curve = a, a ∈ R+ a ≠ 1 about the line |z – z1| = |z – z2|.
z − z2
z 
14. Let z1 and z2 are the two compelx numbers satisfying |z – 3 – 4i| = 3. Such that Arg  1  is maximum
 z2 
then find the value of |z1 – z2|.
z  π
15. If z1 and z2 are the two complex numbers satisfying |z – 3 – 4i| = 8 and Arg  1 = then find the
 z2  2
range of the values of |z1 – z2|.

16. If |z – z1| = |z1| and |z – z2| = |z2| be the two circles and the two circles touch each other then prove that
z 
Img  1  = 0
 z2 

p q r
17. If q r p = 0; where p, q, r are the modulus of non-zero complex numbers u, v, w respectively,
r p q

RI
2
w  w −u
prove that, arg = arg   .
v  v −u 

 z − iz1 

A
18. If |z2 + iz1| = |z1| + |z2| and |z1| = 3 & |z2| = 4, if affix of A, B, C are z1, z2,  2  respectively. Then
 1− i 
find the area of ∆ABC

UH
19. Find the locus of mid-point of line segment intercepted between real and imaginary axes, by the line
az + az + b = 0 , where ‘b’ is real parameter and ‘a’ is a fixed complex number such that Re(a) ≠ 0,
Ιm(a) ≠ 0.

20. Given z1 + z2 + z3 = A, z1 + z2 ω + z3 ω2 = B, z1 + z2 ω2 + z3 ω = C, where ω is cube root of unity,


JA
(a) express z1, z2, z3 in terms of A, B, C.
(b) prove that, |A|2 + |B|2 + |C|2 = 3 (z 1
2
+ z2
2
+ z3
2
).
(c) prove that A3 + B3 + C3 – 3ABC = 27z1 z2 z3

n −1
If w ≠ 1 is nth root of unity, then find the value of ∑ | z1 + w k z 2 |2
LP

21.
k = 0

a b c
22. Let a, b, c be distinct complex numbers such that = = = k, (a, b, c ≠ 1). Find the value
1− b 1− c 1− a
A

of k.

2 πi 20
23. If α = e 7
and f(x) = A0 + ∑A xk , then find the value of,
NK

k
k =1

f(x) + f(αx) + ..... + f(α6x) independent of α.

2π 2π
24. Given, z = cos + i sin , ‘n’ a positive integer, find the equation whose roots are,
SA

2n + 1 2n + 1
α = z + z3 + ...... + z2n – 1 and β = z2 + z4 + ..... + z2n.

 2π   4π   6π   2n π  1
25. Prove that cos   + cos   + cos   +..... + cos   = − When tc n ∈ N.
 2n + 1   2n + 1   2n + 1   2n + 1  2

π 2π kπ 2k + 1
26. Proof that (i) sin sin ........sin =
2k + 1 2k + 1 2k + 1 2k
π 2π kπ 1
(ii) cos cos ........cos = k
2k + 1 2k + 1 2k + 1 2

27. If Zr , r = 1, 2, 3, ......, 2m, m ∈ N are the roots of the equation


2m
1
Z2m + Z2m–1 + Z2m–2 + ....... + Z + 1 = 0, then prove that ∑
r =1 Zr − 1
=–m

28. The points represented by the complex numbers a, b, c lie on a circle with centre O and radius r. The
a −1 + b −1 − 2c −1
tangent at c cuts the chord joining the points a, b at z. Show that z =
a −1b −1 − c −2

29. Show that for the given complex numbers z1 and z2 and for a real constant c the equation
(z1 + λz 2 )z + (z1 + λ z2 )z + c = 0
represents a family of concurrent lines and and also find the fixed point of the family.
(where λ is a real parameter)

30. Let z1 , z2 , z3 are three pair wise distinct complex numbers and t1, t2, t3 are non-negative real numbers

RI
such that t1 + t2 + t3 = 1. Prove that the complex number z = t1z1 + t2z2 + t3z3 lies inside a triangle with
vertices z1, z2, z3 or on its boundary.

A
UH
sec θ
1. 1 2. 96 3. 5. 3 6. 2 10. [1, 9] 12. 10
2
JA
z − z2 24  103 – 5, 103 + 5  25
13. =a 14. 15.
 
18. 19. az + az = 0
z − z1 5 4

A +B+C A + B ω2 + C ω A + B ω + C ω2
20. (a) z1 = , z2 = , z3 =
3 3 3
LP

sin2 nθ 2π cz 2
24. z2 + z + = 0, where θ = 29. z=
sin θ
2 2n + 1 z1z2 − z2 z1
A
NK
SA
QUADRATIC EQUATION
JEE (ADVANCED) SYLLABUS

Quadratic equations with real coefficients, relations between roots and coefficients, formation of quadratic
equations with given roots, symmetric functions of roots.

JEE (MAIN) SYLLABUS

RI
Quadratic equations in real and complex number system and their solutions. Relation between roots and
co-efficients, nature of roots, formation of quadratic equations with given roots.

A
A man is like a fraction whose numerator is what he is and whose denominator is what he thinks of himself. The larger the denominator the
smaller the fraction......... Tolstoy, Count Lev Nikolgevich

UH
1. Polynomial :
A function f defined by f(x) = anxn + an – 1xn – 1 + .......+ a1x + a0
where a0, a1, a2, ......, an ∈ R is called a polynomial of degree n with real coefficients (an ≠ 0, n ∈ W).
If a0, a1, a2, ....., an ∈ C, it is called a polynomial with complex coefficients.
JA
2. Quadratic polynomial & Quadratic equation :
A polynomial of degree 2 is known as quadratic polynomial. Any equation f(x) = 0, where f is a
quadratic polynomial, is called a quadratic equation. The general form of a quadratic equation is
ax2 + bx + c = 0 .......(i)
Where a, b, c are real numbers, a ≠ 0.
If a = 0, then equation (i) becomes linear equation.
LP

3. Difference between equation & identity :


If a statement is true for all the values of the variable, such statements are called as identities. If the
statement is true for some or no values of the variable, such statements are called as equations.
Example : (i) (x + 3)2 = x2 + 6x + 9 is an identity
A

(ii) (x + 3)2 = x2 + 6x + 8, is an equation having no root.


(iii) (x + 3)2 = x2 + 5x + 8, is an equation having – 1 as its root.
A quadratic equation has exactly two roots which may be real (equal or unequal) or imaginary.
NK

a x2 + b x + c = 0 is:
 a quadratic equation if a ≠ 0 Two Roots
 a linear equation if a = 0, b ≠ 0 One Root
 a contradiction if a = b = 0, c ≠ 0 No Root
 an identity if a=b=c=0 Infinite Roots
SA

If ax2 + bx + c = 0 is satisfied by three distinct values of ' x ', then it is an identity.


Example # 1 : (i) 3x2 + 2x – 1 = 0 is a quadratic equation here a = 3.
(ii) (x + 1)2 = x2 + 2x + 1 is an identity in x.
Solution : Here highest power of x in the given relation is 2 and this relation is satisfied by three different
values x = 0, x = 1 and x = – 1 and hence it is an identity because a polynomial equation of n th
degree cannot have more than n distinct roots.
4. Relation Between Roots & Co-efficients:
(i) The solutions of quadratic equation, a x2 + b x + c = 0, (a ≠ 0) is given by
− b ± b − 4 ac
2
x=
2a
The expression, b − 4 a c ≡ D is called discriminant of quadratic equation.
2

(ii) If α, β are the roots of quadratic equation,


a x2 + b x + c = 0 .......(i)
then equation (i) can be written as
a(x – α) (x – β) = 0 or ax2 – a(α + β)x + a αβ = 0 ......(ii)
equations (i) and (ii) are identical,
b coefficient of x
∴ by comparing the coefficients sum of the roots, α + β = – = –
a coefficient of x 2
c constant term
and product of the roots, αβ = =
a coefficient of x 2
b c
(iii) Dividing the equation (i) by a, x2 + x+ =0
a a
 −b  c
⇒ x2 –  x+ a =0 ⇒ x2 – (α + β)x + αβ = 0
 a 
⇒ x2 – (sum of the roots) x + (product of the roots) = 0

RI
Hence we conclude that the quadratic equation whose roots are α & β is x 2 – (α + β)x + αβ = 0

Example # 2 : If α and β are the roots of ax2 + bx + c = 0, find the equation whose roots are α+2 and β+2.
Solution : Replacing x by x – 2 in the given equation, the required equation is

A
a(x – 2)2 + b(x – 2) + c = 0 i.e., ax2 – (4a – b)x + (4a – 2b + c) = 0.

Example # 3 : The coefficient of x in the quadratic equation x 2 + px + q = 0 was taken as 17 in place of 13, its
roots were found to be – 2 and – 15. Find the roots of the original equation.

UH
Solution : Here q = (– 2) × (– 15) = 30, correct value of p = 13. Hence original equation is
x2 + 13x + 30 = 0 as (x + 10) (x + 3) = 0
∴ roots are – 10, – 3

Self practice problems :


JA
(1) If α, β are the roots of the quadratic equation cx 2 – 2bx + 4a = 0 then find the quadratic equation
whose roots are
α β
(i) , (ii) α2 , β 2 (iii) α + 1, β + 1
2 2
1+ α 1 + β α β
(iv) , (v) ,
1− α 1 − β β α
LP

(r + 1)2 b2
(2) If r be the ratio of the roots of the equation ax 2 + bx + c = 0, show that = .
r ac
Answers : (1) (i) cx2 – bx + a = 0
(ii) c2x2 + 4(b2 – 2ac)x + 16a2 = 0
A

(iii) cx2 – 2x(b + c) + (4a + 2b + c) = 0


(iv) (c – 2b + 4a)x2 + 2(4a – c) x + (c + 2b + 4a) = 0
(v) 4acx2 + 4(b2 – 2ac) x + 4ac = 0
NK

5. Theory Of Equations :
If α1, α2, α3,......αn are the roots of the equation;
f(x) = a0xn + a1xn-1 + a2xn-2 +.... + an-1x + an = 0 where a0, a1,....,an are all real & a0 ≠ 0 then,
a a a a
∑ α1 = − 1 , ∑ α1 α2 = + 2 , ∑ α1 α2 α3 = − 3 ,....., α1 α2 α3........αn = (−1)n n
SA

a0 a0 a0 a0

Note : (i) If α is a root of the equation f(x) = 0, then the polynomial f(x) is exactly divisible by (x − α) or
(x − α) is a factor of f(x) and conversely.
(ii) Every equation of nth degree (n ≥ 1) has exactly n roots & if the equation has more than n roots,
it is an identity.
(iii) If the coefficients of the equation f(x) = 0 are all real and α + iβ is its root, then α − iβ is also a
root. i.e. imaginary roots occur in conjugate pairs.
(iv) An equation of odd degree will have odd number of real roots and an equation of even degree
will have even numbers of real roots.
(v) If the coefficients in the equation are all rational & α + β is one of its roots, then
α− β is also a root where α, β ∈ Q & β is not square of a rational number.
(vi) If there be any two real numbers 'a' & 'b' such that f(a) & f(b) are of opposite signs, then
f(x) = 0 must have odd number of real roots (also atleast one real root) between ' a ' and ' b '.
(vii) Every equation f(x) = 0 of degree odd has atleast one real root of a sign opposite to that of its
last term. (If coefficient of highest degree term is positive).

Example # 4 : If 2x3 + 3x2 + 5x + 6 = 0 has roots α, β, γ then find α + β + γ, αβ + βγ + γα and αβγ.


Solution : Using relation between roots and coefficients, we get
3 5 6
α+β+γ==– , αβ + βγ + γα = , αβγ = – = – 3.
2 2 2
Self practice problems :
(3) If 2p3 – 9pq + 27r = 0 then prove that the roots of the equations rx 3 – qx2 + px – 1 = 0 are in
H.P.
(4) If α, β, γ are the roots of the equation x 3 + qx + r = 0 then find the equation whose roots are

RI
(a) 2α + 2β + γ, α + 2β + 2γ, 2α + β + 2γ
r r r
(b) − , − , −
α β γ
(c) (α + β)2, (β + γ)2, (γ + α)2

A
(d) −α3, −β3, −γ3
Answers : (4) (a) x3 + qx – r = 0 (b) x3 – qx2 – r2 = 0
(c) x3 + 2qx2 + q2 x – r2 = 0 (d) x3 – 3x2r + (3r2 + q3) x – r3 = 0

UH
6. Nature of Roots:
Consider the quadratic equation, a x2 + b x + c = 0 having α, β as its roots;
D ≡ b2 − 4 a c
JA
D=0 D≠0
Roots are equal i.e. α = β = − b/2a Roots are unequal
& the quadratic expression can be expressed
as a perfect square of a linear polynomial
LP

a, b, c ∈ R & D > 0 a, b, c ∈ R & D < 0


Roots are real Roots are imaginary α = p + i q, β = p − i q
A

a, b, c ∈ Q & a, b, c ∈ Q &
NK

D is square of a rational number D is not square of a rational number


⇒ Roots are rational ⇒ Roots are irrational
↓ i.e. α = p + q , β = p − q
a = 1, b, c ∈ Ι & D is square of an integer
⇒ Roots are integral.
SA

Example # 5 : For what values of m the equation (1 + m) x2 – 2(1 + 3m)x + (1 + 8m) = 0 has equal roots.
Solution : Given equation is (1 + m) x2 – 2(1 + 3m)x + (1 + 8m) = 0 ........(i)
Let D be the discriminant of equation (i).
Roots of equation (i) will be equal if D = 0.
or 4(1 + 3m)2 – 4(1 + m) (1 + 8m) = 0
or 4(1 + 9m2 + 6m – 1 – 9m – 8m2) = 0
or m2 – 3m = 0 or, m(m – 3) = 0
∴ m = 0, 3.

Example # 6 : Find all the integral values of a for which the quadratic equation (x – a) (x – 10) + 1 = 0 has
integral roots.
Solution : Here the equation is x2 – (a + 10)x + 10a + 1 = 0. Since integral roots will always be rational it
means D should be a perfect square.
From (i) D = a2 – 20a + 96.
⇒ D = (a – 10)2 – 4 ⇒ 4 = (a – 10)2 – D
If D is a perfect square it means we want difference of two perfect square as 4 which is
possible
only when (a – 10)2 = 4 and D = 0.
⇒ (a – 10) = ± 2 ⇒ a = 12, 8

Example # 7 : If the roots of the equation (x – a) (x – b) – k = 0 be c and d, then prove that the roots of the
equation (x – c) (x – d) + k = 0, are a and b.
Solution : By given condition (x – a) (x – b) – k ≡ (x – c) (x – d)
or (x – c) (x – d) + k ≡ (x – a) (x – b)
Above shows that the roots of (x – c) (x – d) + k = 0 are a and b.

Example # 8 : Determine 'a' such that x 2 – 11x + a and x2 – 14x + 2a may have a common factor.
Solution : Let x – α be a common factor of x2 – 11x + a and x2 – 14x + 2a.

RI
Then x = α will satisfy the equations x2 – 11x + a = 0 and x2 – 14x + 2a = 0.
∴ α2 – 11α + a = 0and α2 – 14α + 2a = 0
Solving (i) and (ii) by cross multiplication method, we get a = 0, 24.

A
Example # 9 : Show that the expression x2 + 2(a + b + c)x + 3(bc + ca + ab) will be a perfect square
if a = b = c.
Solution : Given quadratic expression will be a perfect square if the discriminant of its corresponding
equation is zero.

UH
i.e. 4(a + b + c)2 – 4.3 (bc + ca + ab) = 0
or (a + b + c)2 – 3(bc + ca + ab) = 0
1
or ((a – b)2 + (b – c)2 + (c – a)2) = 0
2
which is possible only when a = b = c.
JA
Self practice problems :
(5) For what values of 'k' the expression (4 – k)x2 + 2(k + 2)x + 8k + 1 will be a perfect square ?
(6) If (x – α) be a factor common to a1x2 + b1x + c and a2x2 + b2x + c, then prove that
α(a1 – a2) = b2 – b1.
LP

(7) If 3x2 + 2αxy + 2y2 + 2ax – 4y + 1 can be resolved into two linear factors, Prove that 'α' is a root
of the equation x2 + 4ax + 2a2 + 6 = 0.
(8) Let 4x2 – 4(α – 2)x + α – 2 = 0 (α ∈ R) be a quadratic equation. Find the values of 'α' for which
(i) Both roots are real and distinct.
A

(ii) Both roots are equal.


(iii) Both roots are imaginary
(iv) Both roots are opposite in sign.
(v) Both roots are equal in magnitude but opposite in sign.
NK

(9) If P(x) = ax2 + bx + c, and Q(x) = – ax2 + dx + c, ac ≠ 0 then prove that P(x) . Q(x) = 0 has
atleast two real roots.

Answers. (5) 0, 3
(8) (i) (– ∞, 2) ∪ (3, ∞) (ii) α ∈ {2, 3} (iii) (2, 3) (iv) (– ∞, 2) (v) φ
SA

7. Graph of Quadratic Expression :


 the graph between x, y is always a parabola.

 b D 
 the co−ordinate of vertex are  − ,− 
 2a 4a 

 If a > 0 then the shape of the parabola is concave upwards & if a < 0 then the shape of the
parabola is concave downwards.
 the parabola intersect the y−axis at point (0, c).

 the x−co−ordinate of point of intersection of parabola with x−axis are the real roots of the
quadratic equation f (x) = 0. Hence the parabola may or may not intersect the x−axis.

8. Range of Quadratic Expression f(x) = ax2 + bx + c.

RI
(i) Range :
 D 
If a > 0 ⇒ f (x)∈  − , ∞
 4 a 

A
 D 
If a < 0 ⇒ f (x)∈  − ∞ , − 
 4a 

UH
D
Hence maximum and minimum values of the expression f (x) is − in respective cases and it
4a
b
occurs at x = − (at vertex).
2a
(ii) Range in restricted domain:
JA
Given x ∈ [x1, x2]
b
(a) If − ∉ [x1, x2] then,
2a
f (x)∈ [min { f(x1 ),f(x 2 )} , max { f(x1 ),f(x 2 )}]
b
(b) If − ∈ [x1, x2] then,
LP

2a
  D   D 
f (x)∈  min  f ( x1 ) , f ( x 2 ) , −  , max  f ( x1 ) , f ( x 2 ) , − 
  4a   4 a  
A

9. Sign of Quadratic Expressions :


The value of expression f (x) = a x2 + b x + c at x = x0 is equal to y−co−ordinate of the point on parabola
y = a x2 + b x + c whose x−co−ordinate is x0. Hence if the point lies above the x−axis for some x = x0,
NK

then f (x0) > 0 and vice−versa.

We get six different positions of the graph with respect to x−axis as shown.
SA

(a) a>0
(b) D>0
(c) Roots are real & distinct.
(d) f(x) > 0 in x ∈ (– ∞, α) ∪ (β, ∞)
(e) f(x) < 0 in x ∈ (α, β)

(b) D=0
(c) Roots are real & equal.
(d) f(x) > 0 in x ∈ R – {α}
(b) D<0
(c) Roots are imaginary.
(d) f(x) > 0 ∀ x ∈ R.

(b) D>0
(c) Roots are real & distinct.
(d) f(x) < 0 in x ∈ (– ∞, α) ∪ (β, ∞)
(e) f(x) > 0 in x ∈ (α, β)

RI
(b) D=0
(c) Roots are real & equal.

A
(d) f(x) < 0 in x ∈ R – {α}

UH
(b) D<0
(c) Roots are imaginary.
(d) f(x) < 0 ∀ x ∈ R.
JA
Example # 10 : If c < 0 and ax2 + bx + c = 0 does not have any real roots then prove that
(i) a–b+c<0 (ii) 9a + 3b + c < 0.
Solution : c < 0 and D < 0 ⇒ f(x) = ax2 + bx + c < 0 for all x ∈ R
⇒ f(– 1) = a – b + c < 0
and f(3) = 9a + 3b + c < 0
LP

Example # 11 : Find the range of f(x) = x2 – 5x + 6.


D b  25 − 24  5 1
Solution : minimum of f(x) = – at x = – =–   at x = =–
4a 2a  4  2 4
maximum of f(x) → ∞
A

 1 
Hence range is  − , ∞ 
 4 
x2 − x + 4
NK

Example # 12 : Find the range of rational expression y = if x is real.


x2 + x + 4
x2 − x + 4
Solution : y = ⇒ (y – 1)x 2 + (y + 1) x + 4(y – 1) = 0 ........(i)
x2 + x + 4
SA

case-Ι : if y ≠ 1, then equation (i) is quadratic in x


and  x is real
∴ D ≥ 0 ⇒ (y + 1) 2 – 16(y – 1) 2 ≥ 0 ⇒ (5y – 3) (3y – 5) ≤ 0
3 5
∴ y ∈  ,  – {1}
5 3
case-ΙΙ : if y = 1, then equation becomes
2x = 0⇒ x = 0 which is possible as x is real.
3 5
∴ Rangec  , 
5 3

x+3
Example # 13 : Find the range of y = 2
, if x is real.
2x + 3x + 9
x+3
Solution : y= 2
2x + 3x + 9
⇒ 2yx2 + (3y – 1)x + 3(3y – 1) = 0 .......(i)
case-Ι : if y ≠ 0, then equation (i) is quadratic in x
 x is real
∴ D≥0
⇒ (3y – 1)2 – 24y (3y – 1) ≥ 0
⇒ (3y – 1) (21y + 1) ≤ 0
 1 1
y∈  − ,  – {0}
 21 3 
case-ΙΙ : if y = 0, then equation becomes
x = –3 which is possible as x is real
 1 1

RI
∴ Range y∈  − , 
 21 3 

Self practice problems :

A
(10) If c > 0 and ax2 + 2bx + 3c = 0 does not have any real roots then prove that
(i) 4a – 4b + 3c > 0 (ii) a + 6b + 27c > 0 (iii) a + 2b + 6c > 0
(a − b)2
If f(x) = (x – a) (x – b), then show that f(x) ≥ –

UH
(11) .
4
(12) Find the least integral value of 'k' for which the quadratic polynomial
(k – 1) x2 + 8x + k + 5 > 0 ∀ x ∈ R.
x 2 + 34x − 71
(13) Find the range of the expression , if x is a real.
JA
x 2 + 2x − 7
mx 2 + 3x − 4
(14) Find the interval in which 'm' lies so that the expression can take all real
−4x 2 + 3x + m
values, x ∈ R.
Answers : (12) k = 4 (13) (– ∞, 5] ∪ [9, ∞) (14) m ∈ (1, 7)
LP

10. Location of Roots :


Let f (x) = ax² + bx + c, where a > 0 & a, b, c ∈ R.
A

(i) Conditions for both the roots of f (x) = 0 to be greater than a specified number‘x0’ are
NK

b² − 4ac ≥ 0 & f (x0) > 0 & (− b/2a) > x0.


(ii) Conditions for both the roots of f (x) = 0 to be smaller than a specified number ‘x0’ are
b² − 4ac ≥ 0 & f (x0) > 0 & (− b/2a) < x0.
(iii) Conditions for a number ‘x0’ to lie between the roots of f (x) = 0 is f (x0) < 0.
SA

(iv) Conditions that both roots of f (x) = 0 to be confined between the numbers x1 and
x2, (x1 < x2) are b² − 4ac ≥ 0 & f (x1) > 0 & f (x2) > 0 & x1 < (− b/2a) < x2.
(v) Conditions for exactly one root of f (x) = 0 to lie in the interval (x1, x2) i.e.
x1 < x < x2 is f (x1). f (x2) < 0.

Example # 14 : Let x2 – (m – 3) x + m = 0 (m ∈ R) be a quadratic equation, then find the values of 'm' for which
(a) both the roots are greater than 2.
(b) both roots are positive.
(c) one root is positive and other is negative.
(d) One root is greater than 2 and other smaller than 1
(e) Roots are equal in magnitude and opposite in sign.
(f) both roots lie in the interval (1, 2)

Condition - Ι : D≥0 ⇒ (m – 3)2 – 4m ≥ 0 ⇒ m2 – 10m + 9 ≥ 0


⇒ (m – 1) (m – 9) ≥ 0
⇒ m ∈ (– ∞, 1] ∪ [9, ∞) ......(i)

Condition - ΙΙ : f(2) > 0 ⇒ 4 – (m – 3)2 + m > 0 ⇒ m < 10 ......(ii)


b m−3
Condition - ΙΙΙ : – >2 ⇒ >2 ⇒ m>7 ......(iii)
2a 2
Intersection of (i), (ii) and (iii) gives m ∈ [9, 10)

RI
Condition - Ι D≥0 ⇒ m ∈ (– ∞, 1] ∪ [9, ∞)

A
Condition - ΙΙ f(0) > 0 ⇒ m>0
b m−3
Condition - ΙΙΙ − >0 ⇒ >0 ⇒ m>3
2a 2

UH
intersection gives m ∈ [9, ∞) Ans.

Condition - Ι f(0) < 0 ⇒ m<0 Ans.


JA
Condition - Ι f(1) < 0 ⇒ 4<0 ⇒ m∈φ
Condition - ΙΙ f(2) < 0 ⇒ m > 10
Intersection gives m ∈ φ Ans.
LP

(e) sum of roots = 0 ⇒ m=3


and f(0) < 0 ⇒ m<0 ∴ m∈φ Ans.
A

Condition - Ι D ≥ 0 ⇒ m ∈ (– ∞, 1] ∪ [9, ∞)
NK

Condition - ΙΙ f(1) > 0 ⇒ 1 – (m – 3) + m > 0 ⇒ 4 > 0 which is true ∀ m ∈ R


Condition - ΙΙΙ f(2) > 0 ⇒ m < 10
b m−3
Condition - ΙV 1 < – <2 ⇒ 1< <2 ⇒ 5<m<7
2a 2
intersection gives m ∈ φ Ans.
SA

Example # 15 : Find all the values of 'a' for which both the roots of the equation (a – 2)x 2 – 2ax + a = 0 lies
in the interval (– 2, 1).
Solution : Case-I : f(–2) > 0 ⇒ 4(a – 2) + 4a + a > 0
8
9a – 8 > 0 ⇒ a>
9
f(1) > 0 ⇒ a – 2 – 2a + a > 0
– 2 > 0 not possible ∴ a∈φ
Case-II : a–2<0 ⇒ a<2
8
f(–2) < 0 ⇒ a<
9
f(1) < 0 ⇒ a∈R
b 4
–2< –<1 ⇒ a<
2a 3
D≥0 ⇒ a≥0
 8
intersection gives a ∈ 0, 
 9
 8
complete solution a ∈ 0,  ∪ {2}
 9
Self practice problems :
(15) Let x2 – 2(a – 1)x + a – 1 = 0 (a ∈ R) be a quadratic equation, then find the value of 'a' for which
(a) Both the roots are positive (b) Both the roots are negative
(c) Both the roots are opposite in sign. (d) Both the roots are greater than 1.
(e) Both the roots are smaller than 1.
(f) One root is small than 1 and the other root is greater than 1.
(16) Find the values of p for which both the roots of the equation 4x 2 – 20px + (25p2 + 15p – 66) = 0

RI
are less than 2.
(17) Find the values of 'α' for which 6 lies between the roots of the equation x 2 + 2(α – 3)x + 9 = 0.
(18) Let x2 – 2(a – 1)x + a – 1 = 0 (a ∈ R) be a quadratic equation, then find the values of 'a' for

A
which
(i) Exactly one root lies in (0, 1). (ii) Both roots lies in (0, 1).
(iii) Atleast one root lies in (0, 1).
(iv) One root is greater than 1 and other root is smaller than 0.

UH
(19) Find the values of a, for which the quadratic expression ax 2 + (a – 2) x – 2 is negative for
exactly two integral values of x.
Answers : (15) (a) [2, ∞) (b) φ (c) (– ∞, 1) (d) φ (e) (– ∞, 1] (f) (2, ∞)
 3
(– ∞, –1) (17)  −∞, − 
(16)
JA
 4
(18) (i) (– ∞, 1) ∪ (2, ∞) (ii) φ (iii) ( – ∞, 1) ∪ (2, ∞) (iv) φ
(19) [1, 2)

11. Common Roots:


LP

Consider two quadratic equations, a1 x2 + b1 x + c1 = 0 & a2 x2 + b2 x + c2 = 0.


(i) If two quadratic equations have both roots common, then the equations are identical and their
co-efficient are in proportion.
a1 b1 c
i.e. = = 1
A

a2 b2 c2
(ii) If only one root is common, then the common root ' α ' will be :
c a − c 2 a1 b1 c 2 − b2 c1
α= 1 2 =
NK

a1 b2 − a2 b1 c1 a2 − c 2 a1
Hence the condition for one common root is :
2
⇒ ( c1 a 2 − c 2 a1 ) = ( a1 b2 − a2 b1 ) ( b1 c 2 − b2 c1 )

Note : If f(x) = 0 & g(x) = 0 are two polynomial equation having some common root(s) then those common
SA

root(s) is/are also the root(s) of h(x) ≡ a f(x) + bg (x) = 0.

Example # 16 : If x2 – ax + b = 0 and x2 – px + q = 0 have a root in common and the second equation has
ap
equal roots, show that b + q = .
2
Solution : Given equations are : x2 – ax + b= 0 ........ (i)
and x2 – px + q = 0. ........ (ii)
Let α be the common root. Then roots of equation (ii) will be α and α. Let β be the other root of
equation (i). Thus roots of equation (i) are α, β and those of equation (ii) are α, α.
Now α+β=a ........ (iii)
αβ = b ........ (iv)
2α = p ........ (v)
α2 = q ........ (vi)
L.H.S. = b + q = αβ + α2 = α(α + β) ........ (vii)
ap (α + β) 2α
and R.H.S. = = = α (α + β) ........ (viii)
2 2
from (vii) and (viii), L.H.S. = R.H.S.

Example # 17 : If a, b, c ∈ R and equations ax2 + bx + c = 0 and x2 + 2x + 9 = 0 have a common root, show that
a : b : c = 1 : 2 : 9.
Solution : Given equations are : x2 + 2x + 9 = 0 ........(i)
and ax2 + bx + c = 0 ........(ii)
Clearly roots of equation (i) are imaginary since equation (i) and (ii) have a common root,
therefore common root must be imaginary and hence both roots will be common.
Therefore equations (i) and (ii) are identical
a b c
∴ = =
1 2 9
∴ a:b:c=1:2:9

RI
Self practice problems :

A
(20) If the equations ax2 + bx + c = 0 and x3 + x – 2 = 0 have two common roots then show that
2a = 2b = c.
a b c

UH
(21) If ax2 + 2bx + c = 0 and a1x2 + 2b1x + c1 = 0 have a common root and , , are in A.P.
a1 b1 c1

show that a1, b1, c1 are in G.P. JA


12. Graphs of Polynomials
y = anxn + ............ + a1x + a0. The points where y' = 0 are called turning points which are critical in
plotting the graph.
LP

Example # 18 : Draw the graph of y = 2x3 – 15x2 + 36x + 1

Solution. y′ = 6x2 – 30x + 36 = 6(x – 3) (x – 2)


A

x 2 3 ∞ –∞

y 29 28 ∞ –∞
NK

Example # 19 : Draw the graph of y = –3x4 + 4x3 + 3,


SA

Solution. y′ = –12x3 + 12x


y′ = –12x2 (x – 1)

x 0 1 ∞ –∞
y 3 4 –∞ –∞
 Marked questions are recommended for Revision.

PART - I : SUBJECTIVE QUESTIONS

Section (A) : Relation between the roots and coefficients ; Quadratic Equation
A-1. For what value of 'a', the equation (a2 – a – 2)x2 + (a2 – 4)x + (a2 – 3a + 2) = 0, will have more than two
solutions ? Does there exist a real value of 'x' for which the above equation will be an identity in 'a' ?

If α and β are the roots of the equation 2x 2 + 3x + 4 = 0, then find the values of

RI
A-2.
α β
(i) α2 + β2 (ii) +
β α

A
A-3. If α and β are the roots of the equation ax 2 + bx + c = 0, then find the equation whose roots are given by
1 1
(i) α+ ,β+ (ii) α2 + 2, β2 + 2
β α

UH
α β
A-4. If α ≠ β but α2 = 5α – 3, β2 = 5β – 3, then find the equation whose roots are and .
β α

A-5. In copying a quadratic equation of the form x2 + px + q = 0, the coefficient of x was wrongly written
as – 10 in place of – 11 and the roots were found to be 4 and 6. Find the roots of the correct equation.
JA
3 + 5 −1
A-6. (i) Find the value of the expression 2x3 + 2x2 – 7x + 72 when x = .
2
−1 + 15
(ii) Find the value of the expression 2x3 + 2x2 – 7x + 72 when x =
2
(iii) Solve the following equation 22x + 2x+2 – 32 = 0
LP

A-7. Let a, b, c be real numbers with a ≠ 0 and let α, β be the roots of the equation ax 2 + bx + c = 0. Express
the roots of a3x2 + abcx + c3 = 0 in terms of α, β

A-8. If α, β are roots of x2 – px + q = 0 and α – 2, β + 2 are roots of x 2 – px + r = 0, then prove that


A

16q + (r + 4 – q)2 = 4p2.

A-9. If one root of the equation ax2 + bx + c = 0 is equal to nth power of the other root, then show that
(acn)1/(n + 1) + (anc)1/(n + 1) + b = 0.
NK

A-10. If the sum of the roots of quadratic equation (a + 1)x 2 + (2a + 3)x + (3a + 4) = 0 is –1, then find the
product of the roots.

A-11. Find the least prime integral value of '2a' such that the roots α, β of the equation 2 x 2 + 6 x + a = 0
α β
SA

satisfy the inequality + < 2.


β α
Section (B) : Relation between roots and coefficients ; Higher Degree Equations
B-1. If α and β be two real roots of the equation x 3 + px2 + qx + r = 0 (r ≠ 0) satisfying the relation αβ + 1 = 0,
then prove that r2 + pr + q + 1 = 0.

B-2. If α, β, γ are the roots of the equation x 3 + px2 + qx + r = 0, then find the value of
 1 1  1 
 α −  β −  γ −  .
 βγ   γα   αβ 
B-3. (i) Solve the equation 24x3 – 14x2 – 63x + λ = 0, one root being double of another. Hence find the
value(s) of λ.
(ii) Solve the equation 18x 3 + 81x2 + λx + 60 = 0, one root being half the sum of the other two.
Hence find the value of λ.
B-4. If α, β, γ are roots of equation x3 – 6x2 + 10x – 3 = 0, then find cubic equation with roots 2α + 1, 2β + 1,
2γ + 1.

α β
B-5. If α, β and γ are roots of 2x 3 + x2 – 7 = 0, then find the value of ∑  +  .
α, β, γ 
β α

B-6. Find the roots of 4x3 + 20x2 – 23x + 6 = 0 if two of its roots are equal.

Section (C) : Nature of Roots


C-1. If 2 + i 3 is a root of the equation x2 + px + q = 0 (where p, q ∈ R and i2 = –1), then find the ordered
pair (p, q).

C-2. If the roots of the equation x2 – 2cx + ab = 0 are real and unequal, then prove that the roots of

RI
x2 – 2(a + b) x + a2 + b2 + 2c2 = 0 will be imaginary.

C-3. For what values of k the expression kx2 + (k + 1)x + 2 will be a perfect square of a linear polynomial.

A
C-4. Show that if roots of equation (a2 – bc) x2 + 2(b2 – ac) x + c2 – ab = 0 are equal, then either
b = 0 or a3 + b3 + c3 = 3abc

UH
1 1 1
C-5. If a, b, c ∈ R, then prove that the roots of the equation + + = 0 are always real and
x −a x −b x−c
cannot have roots if a = b = c.

1 1 1
C-6. If the roots of the equation + = are equal in magnitude but opposite in sign, then
(x + p)
JA
(x + q) r
show that p + q = 2 r and that the product of the roots is equal to (−1/2) (p² + q²).

C-7. (i) If – 2 + iβ is a root of x3 + 63x + λ = 0 (where β ∈ R – {0}, λ ∈ R and i2 = –1), then find roots of
equation.
−1
(ii) If + iβ, is a root of 2x3 + bx2 + 3x + 1 = 0 (where b,β ∈ R – {0} and i2 = –1), then find the
2
LP

value(s) of b.

C-8. Solve the equation x4 + 4x3 + 5x2 + 2x – 2 = 0, one root being – 1 + −1 .

C-9. Draw graph of y = 12x3 – 4x2 – 3x + 1. Hence find number of positive zeroes.
A

Section (D) : Range of quadratic expression and sign of quadratic expression


NK

D-1. Draw the graph of the following expressions :


(i) y = x2 + 4x + 3 (ii) y = 9x2 + 6x + 1 (iii) y = – 2x2 + x – 1
D-2. Find the range of following quadratic expressions :
(i) f(x) = –x2 + 2x + 3 ∀x∈R
(ii) f(x) = x2 – 2x + 3 ∀ x ∈ [0, 3]
∀ x ∈ (0, 1]
SA

(iii) f(x) = x2 – 4x + 6
D-3. If x be real, then find the range of the following rational expressions :
x2 + x + 1 x 2 − 2x + 9
(i) y= (ii)  y =
x2 + 1 x 2 – 2x − 9
kx 2 + 2(k + 1)x + (9k + 4)
D-4. Find the range of values of k, such that f(x) = is always negative.
x 2 − 8x + 17

D-5. x2 + (a − b) x + (1 − a − b) = 0, a, b ∈ R. Find the condition on ' a ' for which


(i) Both roots of the equation are real and unequal ∀ b ∈ R .
(ii) Roots are imaginary ∀ b ∈ R

Section (E) : Location of Roots


E-1. If both roots of the equation x2 – 6ax + 2 – 2a + 9a2 = 0 exceed 3, then show that a > 11/9.
E-2. Find all the values of 'K' for which one root of the equation x² − (K + 1) x + K² + K − 8 = 0, exceeds 2 &
the other root is smaller than 2.
E-3. Find all the real values of 'a', so that the roots of the equation
(a2 – a + 2) x2 + 2(a – 3) x + 9 (a4 – 16) = 0 are of opposite sign.

E-4. Find all the values of 'a', so that exactly one root of the equation x 2 – 2ax + a2 – 1 = 0, lies between the
numbers 2 and 4, and no root of the equation is either equal to 2 or equal to 4.

E-5. If α & β are the two distinct roots of x² + 2 (K − 3) x + 9 = 0, then find the values of K such that
α, β ∈ (− 6, 1).

Section (F) : Common Roots & Graphs of Polynomials

RI
F-1. If one of the roots of the equation ax2 + b x + c = 0 be reciprocal of one of the roots of
a1 x2 + b1 x + c1 = 0, then prove that (a a1 − c c1)2 = (b c1 − a b1) (b1c − a1b).

A
F-2. Find the value of 'a' so that x2 – 11 x + a = 0 and x2 – 14x + 2a = 0 have a common root.

F-3. If ax2 + bx + c = 0 and bx2 + cx + a = 0 have a common root and a, b, c are non-zero real numbers, then
a3 + b3 + c 3

UH
find the value of .
abc

F-4. If x2 + px + q = 0 and x2 + qx + p = 0, (p ≠ q) have a common root, show that 1 + p + q = 0 ; show that


their other roots are the roots of the equation x2 + x + pq = 0.
JA
F-5. Draw the graphs of following :
(i) y = 2x3 + 9x2 – 24x + 15 (ii) y = – 3x4 + 4x3 + 12x2 – 2

F-6. Find values of ‘k’ if equation x3 – 3x2 + 2 = k has


(i) 3 real roots (ii) 1 real root

PART - II : ONLY ONE OPTION CORRECT TYPE


LP

Section (A) : Relation between the roots and coefficients quadratic equation

A-1. The roots of the equation (b – c) x2 + (c – a) x + (a – b) = 0 are


A

c −a a−b b−c c −a
(A) ,1 (B) ,1 (C) ,1 (D) ,1
b−c b−c a−b a−b
NK

A-2. If α, β are the roots of quadratic equation x 2 + p x + q = 0 and γ, δ are the roots of x2 + p x – r = 0,
then (α − γ) . (α − δ) is equal to :
(A) q + r (B) q – r (C) – (q + r) (D) – (p + q + r)

A-3. Two real numbers α & β are such that α + β = 3, α − β = 4, then α & β are the roots of the quadratic
SA

equation:
(A) 4x2 − 12x − 7 = 0 (B) 4x2 − 12x + 7 = 0 (C) 4x2 − 12x + 25 = 0 (D) none of these

A-4. For the equation 3x2 + px + 3 = 0, p > 0 if one of the roots is square of the other, then p is equal to:
(A) 1/3 (B) 1 (C) 3 (D) 2/3

A-5. Consider the following statements :


S1 : If the roots of x2 – bx + c = 0 are two consecutive integers, then value of b2 – 4c is equal to 1.
S2 : If α, β are roots of x2 – x + 3 = 0 then value of α4 + β4 is equal 7.
S3 : If α , β, γ are the roots of x3 – 7x2 + 16 x – 12 = 0 then value of α2 + β2 + γ2 is equal to 17.
State, in order, whether S1, S2, S3 are true or false
(A) TTT (B) FTF (C) TFT (D) FTT
Section (B) : Relation between roots and coefficients ; Higher Degree Equations

B-1. If two roots of the equation x3 − px2 + qx − r = 0, (r ≠ 0) are equal in magnitude but opposite in sign,
then:
(A) pr = q (B) qr = p (C) pq = r (D) None of these
1+ α 1+ β 1+ γ
B-2. If α, β & γ are the roots of the equation x 3 − x − 1 = 0 then, + + has the value equal to:
1− α 1− β 1− γ
(A) zero (B) − 1 (C) − 7 (D) 1

B-3. Let α, β, γ be the roots of (x – a) (x – b) (x – c) = d, d ≠ 0, then the roots of the equation


(x – α) (x – β) (x – γ) + d = 0 are :
a b c
(A) a + 1, b + 1, c + 1 (B) a, b, c (C) a – 1, b – 1, c – 1 (D) , ,
b c a

RI
α 3 + β3 + γ 3
B-4_. If α, β, γ are the roots of the equation x 3 + ax + b = 0 then value of 2 is equal to :
α + β2 + γ 2
3b −3b

A
(A) (B) (C) 3b (D) 2b
2a 2a
B-5_. If two of the roots of equation x4 – 2x3 + ax2 + 8x + b = 0 are equal in magnitude but opposite in sign,
then value of 4a + b is equal to :

UH
(A) 16 (B) 8 (C) –16 (D) –8

Section (C) : Nature of Roots


C-1. If one roots of equation x2 – 3 x + λ = 0 , λ ∈ R is 3 + 2 then other root is
(A) 3–2 (B) – 2 (C) 2 – 3 (D) 2
JA
C-2. If roots of equation 2x2 + bx + c = 0 ; b, c ∈ R, are real & distinct then the roots of equation
2cx2 + (b – 4c) x + 2c – b + 1 = 0 are
(A) imaginary (B) equal (C) real and distinct (D) can't say

C-3. Let one root of the equation x2 + x + m = 0 is square of other root. If m∈R then
 1  1 1 
LP

(A)  ∈  – ∞,  ∪ {1} (B) ∈(–∞,0] (C)  ∈  – ∞,  (D)  ∈  ,1


 4   9  4 
C-4. If a, b, c are integers and b2 = 4(ac + 5d2), d ∈ N, then roots of the quadratic equation ax2 + bx + c = 0
are
(A) Irrational (B) Rational & different (C) Complex conjugate (D) Rational & equal
A

C-5. Let a, b and c be real numbers such that 4a + 2b + c = 0 and ab > 0. Then the equation
ax2 + bx + c = 0 has
NK

(A) real roots (B) imaginary roots (C) exactly one root (D) none of these
C-6. Consider the equation x2 + 2x – n = 0, where n ∈ N and n ∈ [5, 100]. Total number of different values of
'n' so that the given equation has integral roots, is
(A) 4 (B) 6 (C) 8 (D) 3

Section (D) : Range of quadratic expression and sign of quadratic expression


SA

D-1. If α & β (α < β) are the roots of the equation x 2 + bx + c = 0, where c < 0 < b, then
(A) 0 < α < β (B) α < 0 < β2 < α2
(C) α < β < 0 (D) α < 0 < α2 < β2

D-2. Which of the following graph represents the expression f(x) = a x2 + b x + c (a ≠ 0) when
a > 0, b < 0 & c < 0 ?

(A) (B) (C) (D)

D-3. The expression y = ax2 + bx + c has always the same sign as of 'a' if :
(A) 4ac < b2 (B) 4ac > b2 (C) 4ac = b2 (D) ac < b2
D-4. The entire graph of the expression y = x2 + kx – x + 9 is strictly above the x-axis if and only if
(A) k < 7 (B) –5 < k < 7 (C) k > – 5 (D) none of these

D-5. If a, b ∈ R, a ≠ 0 and the quadratic equation ax 2 − bx + 1 = 0 has imaginary roots then a + b + 1 is:
(A) positive (B) negative (C) zero (D) depends on the sign of b

D-6. If a and b are the non-zero distinct roots of x2 + ax + b = 0, then the least value of x2 + ax + b is
3 9 9
(A) (B) (C) – (D) 1
2 4 4

D-7. If y = – 2x2 – 6x + 9, then


(A) maximum value of y is –11 and it occurs at x = 2
(B) minimum value of y is –11 and it occurs at x = 2
(C) maximum value of y is 13.5 and it occurs at x = –1.5

RI
(D) minimum value of y is 13.5 and it occurs at x = –1.5

D-8. If f(x) = x2 + 2bx + 2c2 and g(x) = – x2 – 2cx + b2 are such that min f(x) > max g(x), then the relation
between b and c, is

A
(A) no relation (B) 0 < c < b/2 (C) c2 < 2b (D) c2 > 2b2

Section (E) : Location of Roots

UH
E-1. If b > a, then the equation (x − a) (x − b) − 1 = 0, has:
(A) both roots in [a, b] (B) both roots in (− ∞, a)
(C) both roots in [b, ∞) (D) one root in (− ∞, a) & other in (b, ∞)

E-2. If α, β are the roots of the quadratic equation x 2 − 2p (x − 4) − 15 = 0, then the set of values of 'p' for
JA
which one root is less than 1 & the other root is greater than 2 is:
(A) (7/3, ∞) (B) (− ∞, 7/3) (C) x ∈ R (D) none of these

E-3. If α, β be the roots of 4x2 – 16x + λ = 0, where λ ∈ R, such that 1 < α < 2 and 2 < β < 3, then the
number of integral solutions of λ is
(A) 5 (B) 6 (C) 2 (D) 3
LP

E-4. Set of real values of k if the equation x2 – (k–1)x + k2 = 0 has atleast one root in (1,2) is
(A) (2, 4) (B) [–1, 1/3] (C) {3} (D) φ

Section (F) : Common Roots & Graphs of Polynomials


A

F-1. If the equations k (6x2 + 3) + rx + 2x2 – 1 = 0 and 6k (2x2 + 1) + px + 4x2 – 2 = 0 have both roots
common, then the value of (2r – p) is
(A) 0 (B) 1/2 (C) 1 (D) none of these
NK

F-2. If 3x2 – 17x + 10 = 0 and x2 – 5x + λ = 0 has a common root, then sum of all possible real values of λ is
29 26 29
(A) 0 (B) − (C) (D)
9 9 3
F-3. If a, b, p, q are non−zero real numbers, then two equations 2a2 x2 − 2 ab x + b2 = 0 and
p2 x2 + 2 pq x + q2 = 0 have :
SA

(A) no common root (B) one common root if 2 a2 + b2 = p2 + q2


(C) two common roots if 3 pq = 2 ab (D) two common roots if 3 qb = 2 ap

x 3 – 4x
F-4. The graphs of y = is
4
y

30
y

127
3
(A) 2 (B)
15 O
–1 2
2 x
x –2
–4 O 1

RI
y
(C) 2 (D)
3

A
–2 −2 2 x
3

UH
F-5. The graphs of y = x4 – 2x2 + 5 is
y
y
30

127
JA
2
(A) 15 (B) 3
O
2 –1 2 x
x
–4 O 1 –2
LP

y
2
3
(C) –2 −2 2 x
(D)
A

3
NK

PART - III : MATCH THE COLUMN


1. Column – Ι Column – ΙΙ
(A) If α, α + 4 are two roots of x2 – 8 x + k = 0, (p) 4
SA

then possible value of k is

1 1
(B) If α, β are roots of x2 + 2x – 4 = 0 and , are (q) 0
α β
−3
roots of x2 + qx + r = 0 then value of is
q+r

(C) If α, β are roots of ax2 + c = 0, ac ≠ 0, then (r) 12


α3 + β3 is equal to

(D) If roots of x2 – kx + 36 = 0 (s) 10


are Integers then number of values of k =
2. If graph of the expression f(x) = ax2 + bx + c (a ≠ 0) are given in column-II, then Match the items in
column-I with in column-II (where D = b2 – 4ac)
Column-I Column-II

abc
(A) >0 (p)
D

abc

RI
(B) <0 (q)
D

A
(C) abc > 0 (r)

UH
JA
(D) abc < 0 (s)
LP

3. Let y = Q(x) = ax2 + bx + c be a quadratic expression. Match the inequalities in Column-I with possible
graphs in Column-II.
Column-I Column-II

Q(x) > 0, ∀ x ∈ (2, 7)


A

(A) (p)
NK

(B) Q(x) > 0, ∀ x ∈ (– ∞, 1) (q)


SA

(C) Q(x) < 0, ∀ x ∈ (1, 6) (r)

(D) Q(x) < 0, ∀ x ∈ (– ∞, – 1) (s)

(t)
 Marked questions are recommended for Revision.

PART - I : ONLY ONE OPTION CORRECT TYPE

1. Let a > 0, b > 0 & c > 0. Then both the roots of the equation ax 2 + bx + c = 0
(A) are real & negative (B) have negative real parts
(C) are rational numbers (D) have positive real parts

2. If the roots of the equation x2 + 2ax + b = 0 are real and distinct and they differ by atmost 2m, then b
lies in the interval

RI
(A) (a2 – m2, a2) (B) [a2 – m2, a2) (C) (a2, a2 + m2) (D) none of these

3. The set of possible values of λ for which x2 – (λ2 – 5λ + 5)x + (2λ2 – 3λ – 4) = 0 has roots, whose sum

A
and product are both less than 1, is
 5  5  5
(A)  −1 , (B) (1, 4) (C) 1 ,  (D)  1 ,
 2   2  2 

UH
4. If p, q, r, s ∈ R, then equaton (x2 + px + 3q) (–x2 + rx + q) (–x2 + sx – 2q) = 0 has
(A) 6 real roots (B) atleast two real roots
(C) 2 real and 4 imaginary roots (D) 4 real and 2 imaginary roots
JA
5. If coefficients of biquadratic equation are all distinct and belong to the set {–9, – 5, 3, 4, 7}, then
equation has
(A) atleast two real roots
(B) four real roots, two are conjugate surds and other two are also conjugate surds
(C) four imaginary roots
(D) None of these
LP

6. Find the set of all real values of λ such that the root of the equation
x2 + 2(a + b + c)x + 3λ (ab + bc + ca) = 0 are always real for any choice of a, b, c (where a, b, c
represents sides of scalene triangle).
 4  4  1 5 4 5
A

(A)  −∞,  (B)  , ∞  (C)  ,  (D)  , 


 3  3  3 3 3 3

Let p, q, r, s ∈ R, x2 + px + q = 0, x2 + rx + s = 0 such that 2 (q + s) = pr then


NK

7.
(A) atleast one of the equation have real roots.
(B) either both equations have imaginary roots or both equations have real roots.
(C) one of equations have real roots and other equation have imaginary roots
(D) atleast one of the equations have imaginary roots.
SA

8. The equation, πx = − 2x2 + 6x − 9 has:


(A) no solution (B) one solution (C) two solutions (D) infinite solutions

9. If (λ2 + λ – 2)x2 + (λ + 2) x < 1 for all x ∈ R, then λ belongs to the interval
 2 2 
(A) (–2, 1) (B)  −2,  (C)  , 1 (D) none of these
 5  5 

10. Let conditions C1 and C2 be defined as follows : C1 : b2 – 4ac ≥ 0, C2 : a, –b, c are of same sign. The
roots of ax2 + bx + c = 0 are real and positive, if
(A) both C1 and C2 are satisfied (B) only C2 is satisfied
(C) only C1 is satisfied (D) none of these
x2 − x + c
11. If 'x' is real, then can take all real values if :
x 2 + x + 2c
(A) c ∈ [0, 6] (B) c ∈ [− 6, 0] (C) c ∈ (− ∞, − 6) ∪ (0, ∞) (D) c ∈ (− 6, 0)

12. If both roots of the quadratic equation (2 − x) (x + 1) = p are distinct & positive, then p must lie in the
interval:
(A) (2, ∞) (B) (2, 9/4) (C) (– ∞, – 2) (D) (– ∞, ∞)

13. If two roots of the equation (a – 1) (x2 + x + 1)2 – (a + 1) (x4 + x2 + 1) = 0 are real and distinct, then 'a'
lies in the interval
(A) (–2, 2) (B) (– ∞, –2) ∪ (2, ∞) (C) (2, ∞) (D) (–∞, –2)

RI
14. The equations x3 + 5x2 + px + q = 0 and x3 + 7x2 + px + r = 0 have two roots in common. If the third root
of each equation is represented by x1 and x2 respectively, then the ordered pair (x1, x2) is:
(A) (− 5, − 7) (B) (1, − 1) (C) (− 1, 1) (D) (5, 7)

A
15. If a, b, c are real and a2 + b2 + c2 = 1, then ab + bc + ca lies in the interval:
1   1   1
(A)  , 2  (B) [0, 2] (C)  − , 1 (D)  −1, 
2   2   2

UH
PART - II : SINGLE AND DOUBLE VALUE INTEGER TYPE

1. Find number of integer roots of equation x (x + 1) (x + 2) (x + 3) = 120.


JA
2
−3 2
−3
2. Find product of all real values of x satisfying (5 + 2 6 )x + (5 − 2 6 )x = 10

3. If a, b are the roots of x2 + px + 1 = 0 and c, d are the roots of x2 + qx + 1 = 0. Then find the value of
(a − c) (b − c) (a + d) (b + d)/(q2 − p2).
LP

4. α, β are roots of the equation λ (x2 – x) + x + 5 = 0. If λ1 and λ2 are the two values of λ for which the
 λ1 λ 2 
α β λ + λ 
roots α, β are connected by the relation + = 4, then the value of  2 1  is
A

β α  14 
 
 
NK

5. Let one root of equation ( – m) x2 + x + 1 = 0 be double of the other. If  be real and 8m ≤ k then find
the least value of k.

6. Let α, β be the roots of the equation x2 + ax + b = 0 and γ, δ be the roots of x2 – ax + b – 2 = 0. If αβγδ =


1 1 1 1 5
SA

24 and + + + = , then find the value of a.


α β γ δ 6

7. If a > b > 0 and a3 + b3 + 27ab = 729 then the quadratic equation ax2 + bx – 9 = 0 has roots
α, β (α < β). Find the value of 4β – aα.

8. Let α and β be roots of x2 – 6(t2 – 2t + 2)x – 2 = 0 with α > β. If an = αn – βn for n ≥ 1, then find the
a100 − 2a98
minimum value of (where t ∈ R)
a99

9. If α, β, γ, δ are the roots of the equation x 4 − Kx3 + Kx2 + Lx + M = 0, where K, L & M are real numbers,
then the minimum value of α2 + β2 + γ2 + δ2 is – n. Find the value of n.
2x
10. Consider y = , where x is real , then the range of expression y2 + y − 2 is [a, b]. Find the value of
1 + x2
(b – 4a).

11. If the roots of the equation x3 + Px2 + Qx − 19 = 0 are each one more than the roots of the equaton
x3 − Ax2 + Bx − C = 0, where A, B, C, P & Q are constants, then the value of A + B + C is equal to :

12. If one root of the equation t2 – (12x)t – (f(x) + 64x) = 0 is twice of other, then find the maximum value of
the function f(x), where x ∈ R.

13. The values of k, for which the equation x2 + 2 (k − 1) x + k + 5 = 0 possess atleast one positive root, are
(– ∞, – b]. Find value of b.

RI
14. Find the least value of 7a for which atleast one of the roots of the equation x 2 – (a – 3) x + a = 0 is
greater than 2.

A
15. If the quadratic equations 3x2 + ax + 1 = 0 & 2x2 + bx + 1 = 0 have a common root, then the value of the
expression 5ab − 2a2 − 3b2 is

The equations x2 − ax + b = 0, x3 − px2 + qx = 0, where a, b, p, q ∈ R – {0} have one common root & the

UH
16.
ap
second equation has two equal roots. Find value of .
q+b
16λ
17. If x – y and y – 2x are two factors of the expression x 3 – 3x2y + λxy2 + µy3, then + 4µ is
11
JA
PART - III : ONE OR MORE THAN ONE OPTIONS CORRECT TYPE
1. Possible values of 'p' for which the equation (p2 – 3p + 2)x2 – (p2 – 5p + 4)x + p – p2 = 0 does not
possess more than two roots is/are
(A) 0 (B) 1 (C) 2 (D) 4
LP

2. If a, b are non-zero real numbers and α, β the roots of x 2 + ax + b = 0, then


(A) α2, β2 are the roots of x2 – (2b – a2) x + a2 = 0
1 1
(B) , are the roots of bx2 + ax + 1 = 0
α β
A

α β
(C) , are the roots of bx2 + (2b – a2) x + b = 0
β α
NK

(D) (α – 1), (β – 1) are the roots of the equation x 2 + x (a + 2) + 1 + a + b = 0

3. If α, β are the roots of ax2 + bx + c = 0 (a ≠ 0) and α + δ, β + δ are the roots of,


Ax2 + Bx + C = 0 (A ≠ 0) for some constant δ, then
1 B b 1 b B 
(A) δ = − (B) δ = a − A 
SA

2  A a  2  
b2 − 4ac B2 − 4 A C b2 + 4ac B2 + 4 A C
(C) = (D) =
a2 A2 a2 A2

4. If one root of the equation 4x2 + 2x – 1 = 0 is ‘α’, then


−1 + 5 1+ 5
(A) α can be equal to (B) α can be equal to
4 4
(C) other root is 4α3 – 3α. (D) other root is 4α3 + 3α

5. If α, β are roots of x2 + 3x + 1 = 0, then


(A) (7 – α) (7 – β) = 0 (B) (2 – α) (2 – β) = 11
2 2
α2 β2  α   β 
(C) + =–2 (D)   +  = 18
3α + 1 3β + 1  1+ β   α + 1
6. If both roots of x2 – 32x + c = 0 are prime numbers then possible values of c are
(A) 60 (B) 87 (C) 247 (D) 231

7. Let f(x) = x2 – a(x + 1) – b = 0, a, b ∈ R – {0}, a + b ≠ 0. If α and β are roots of equation f(x) = 0, then
1 1 2
the value of 2 + 2 – is equal to
α − aα β − aβ a+b
 a  a2
(A) 0 (B) f(a) + a + b (C) f(b) + a + b (D) f   + +a+b
2 4
8. If f(x) is a polynomial of degree three with leading coefficient 1 such that f(1) = 1, f(2) = 4, f(3) = 9, then

RI
3
6 6
(A) f(4) = 22 (B) f   =  
5 5
(C) f(x) = x3 holds for exactly two values of x. (D) f(x) = 0 has a root in interval (0, 1).

A
9. Let P(x) = x32 – x25 + x18 – x11 + x4 – x3 + 1. Which of the following are CORRECT ?
(A) Number of real roots of P(x) = 0 are zero.

UH
(B) Number of imaginary roots of P(x) = 0 are 32.
(C) Number of negative roots of P(x) = 0 are zero.
(D) Number of imaginary roots of P(x) + P(–x) = 0 are 32.

If α, β are the real and distinct roots of x 2 + px + q = 0 and α4, β4 are the roots of x2 – rx + s = 0, then the
10.
JA
equation x2 – 4qx + 2q2 – r = 0 has always (given α ≠ –β)
(A) two real roots (B) two negative roots
(C) two positive roots (D) one positive root and one negative root

11. x2 + x + 1 is a factor of a x3 + b x2 + c x + d = 0, then the real root of above equation is


LP

(a, b, c, d ∈ R)
(A) − d/a (B) d/a (C) (b – a)/a (D) (a – b)/a

12. If – 5 + iβ, – 5 + iγ, (where β2 ≠ γ2 ; β, γ ∈ R and i2 = –1) are roots of x3 + 15x2 + cx + 860 = 0, c ∈ R,
then
A

(A) c = 222
(B) all the three roots are imaginary
(C) two roots are imaginary but not complex conjugate of each other.
NK

(D) – 5 + 7i 3 , – 5 – 7i 3 are imaginary roots.


13. Let f(x) = ax2 + bx + c > 0, ∀ x ∈ R or f(x) < 0, ∀ x ∈ R. Which of the following is/are CORRECT ?
(A) If a + b + c > 0 then f(x) > 0, ∀ x ∈ R (B) If a + c < b then f(x) < 0, ∀ x ∈ R
(C) If a + 4c > 2b then f(x) < 0, ∀ x ∈ R (D) ac > 0.
SA

14. Let x1 < α < β < γ < x4, x1 < x2 < x3. If f(x) is a cubic polynomial with real coefficients such that
(f(α))2 + (f(β))2 + (f(γ))2 = 0, f(x1) f(x2) < 0, f(x2) f(x3) < 0 and f(x1) f(x3) > 0 then which of the following are
CORRECT ?
(A) α ∈ (x1, x2), β ∈ (x2, x3) and γ ∈ (x3, x4) (B) α ∈ (x1, x3), β, γ ∈ (x3, x4)
(C) α, β ∈ (x1, x2) and γ ∈ (x4, ∞) (D) α ∈ (x1, x3), β ∈ (x2, x3) and γ ∈ (x2, x4)

15. If f(x) is cubic polynomial with real coefficients, α < β < γ and x 1 < x2 be such that f(α) = f(β) = f(γ) =
f′ (x1) = f ′ (x2) = 0 then possible graph of y = f(x) is (assuming y-axis vertical)

(A) (B)
(C) (D)

3 4 5
16. Let f(x) = + + , then f(x) = 0 has
x−2 x−3 x−4
(A) exactly one real root in (2, 3) (B) exactly one real root in (3, 4)
(C) 3 different roots (D) atleast one negative root
17. If the quadratic equations ax + bx + c = 0 (a, b, c ∈ R, a ≠ 0) and x2 + 4x + 5 = 0 have a common root,
2

then a, b, c must satisfy the relations:


(A) a > b > c (B) a < b < c
(C) a = k; b = 4k; c = 5k (k ∈ R, k ≠ 0) (D) b2 − 4ac is negative.

18. If the quadratic equations x2 + abx + c = 0 and x2 + acx + b = 0 have a common root, then the equation

RI
containing their other roots is/are :
(A) x2 + a (b + c) x − a2bc = 0 (B) x2 − a (b + c) x + a2bc = 0
(C) a (b + c) x2 − (b + c) x + abc = 0 (D) a (b + c) x2 + (b + c) x − abc = 0

A
19. Consider the following statements.
S1 : The equation 2x2 + 3x + 1 = 0 has irrational roots.

UH
S2 : If a < b < c < d, then the roots of the equation (x – a) (x – c) + 2 (x – b) (x – d) = 0 are real and
distinct.
S3 : If x2 + 3x + 5 = 0 and ax2 + bx + c = 0 have a common root and a, b, c ∈ N, then the minimum
value of (a + b + c) is 10.
The value of the biquadratic expression x4 − 8 x3 + 18 x2 − 8 x + 2, when x = 2 + 3 , is 1
S4 :
JA
Which of the following are CORRECT ?
(A) S2 and S4 are true. (B) S1 and S3 are false.
(C) S1 and S2 are true. (D) S3 and S4 are false.

20. If the equations x2 + a x + 12 = 0, x2 + b x + 15 = 0 & x2 + (a + b) x + 36 = 0 have a common positive


LP

root, then which of the following are true ?


(A) ab = 56 (B) common positive root is 3
(C) sum of uncommon roots is 21. (D) a + b = 15.

If x2 + λx + 1 = 0, λ ∈ (–2 , 2) and 4x3 + 3x + 2c = 0 have common root then c + λ can be


A

21.
1 1 3
(A) (B) – (C) 0 (D)
2 2 2
NK

PART - IV : COMPREHENSION
Comprehension # 1 (Q. No. 1 & 2)
If x, y ∈ R then some problems can be solved by direct observing extreme cases
SA

e.g. (i) (x – 3)2 + (y – 2)2 = 0 is possible only for x = 3 and y = 2


(ii) if x ≥ 3, y ≥ 2 and xy ≤ 6 then x = 3 & y = 2

1. The least value of expression x2 + 2 xy + 2 y2 + 4 y + 7 is :


(A) 1 (B) 2 (C) 3 (D) 4

2. Let P(x) = 4x2 + 6x + 4 and Q(y) = 4y2 – 12y + 25. If x, y satisfy equation P(x).Q(y) = 28, then the value
of 11y – 26x is -
(A) 6 (B) 36 (C) 8 (D) 42
Comprehension # 2 (Q. No. 3 & 4)

In the given figure ∆OBC is an isosceles right triangle in which AC is a median, then answer the
following questions :
Y

y = x2 + bx + c
C

O A B X

3. Roots of y = 0 are
(A) {2, 1} (B) {4, 2} (C) {1, 1/2} (D) {8, 4}

RI
4. The equation whose roots are (α + β) & (α – β), where α, β (α > β) are roots obtained in previous
question, is
(A) x2 – 4x + 3 = 0 (B) x2 – 8x + 12 = 0 (C) 4x2 – 8x + 3 = 0 (D) x2 – 16x + 48 = 0

A
Comprehension # 3 (Q. No. 5 to 7)
Consider the equation x4 – λx2 + 9 = 0. This can be solved by substituting x2 = t such equations are

UH
called as pseudo quadratic equations.

5. If the equation has four real and distinct roots, then λ lies in the interval
(A) (–∞, –6) ∪ (6, ∞) (B) (0, ∞) (C) (6, ∞) (D) (–∞, –6)

6. If the equation has no real root, then λ lies in the interval


JA
(A) (–∞, 0) (B) (–∞, 6) (C) (6, ∞) (D) (0, ∞)

7. If the equation has only two real roots, then set of values of λ is
(A) (–∞, –6) (B) (–6, 6) (C) {6} (D) φ

Comprehension # 4
LP

To solve equation of type,


ax2m + bx2m – 1 + cx2m – 2 + ......... + kxm + ......... + cx2 + bx + a = 0, (a ≠ 0) → (Ι)
divide by xm and rearrange terms to obtain
 1   1   1 
a  xm + m  + b  xm−1 + m−1  + c  xm−2 + m−2  + ......... + k = 0
A

 x   x   x 
Substitutions like
1 1
t=x+ or t=x– helps transforming equation into a reduced degree equation.
NK

x x

8. Roots of equation x4 – 10x3 + 26x2 – 10x + 1 = 0 are


(A) 2 ± 3 , 3 ± 2 (B) 2 ± 3 , 3 ± 2 2 (C) 3 ± 2,3±2 2 (D) 8 ± 3 , 3 ± 2
SA

9. Roots of equation x5 – 5x4 + 9x3 – 9x2 + 5x – 1 = 0 are


3 ± 5 1± i 3 5± 3 3±i 3± 5 3±i 5 ± 3 1± i 3
(A) 1, , (B) 1, , (C) 1, , (D) 1, ,
2 2 2 2 2 2 2 2

10. Roots of equation x6 – 4x4 + 4x2 – 1 = 0 are


1± i 5 −1 ± 5 1± 5 −1 ± i 5
(A) ± 1, , (B) ± 1, ,
2 2 2 2
1 ± 5 −1 ± 5 −1 ± 5 −1 ± i 5
(C) ± 1, , (D) ± 1, , .
2 2 2 2
Marked questions are recommended for Revision.
* Marked Questions may have more than one correct option.
PART - I : JEE (ADVANCED) / IIT-JEE PROBLEMS (PREVIOUS YEARS)
1. The smallest value of k, for which both the roots of the equation x 2 – 8kx + 16(k2 – k + 1) = 0 are real,
distinct and have values atleast 4, is [IIT-JEE 2009, Paper-2, (4, –1)/ 80]

2. Let p and q be real numbers such that p ≠ 0, p3 ≠ q and p3 ≠ – q. If α and β are nonzero complex
α β
numbers satisfying α + β = – p and α3 + β3 = q, then a quadratic equation having and as its roots
β α
is [IIT-JEE 2010, Paper-1, (3, –1)/ 84]

RI
(A) (p3 + q) x2 – (p3 + 2q)x + (p3 + q) = 0 (B) (p3 + q) x2 – (p3 – 2q)x + (p3 + q) = 0
(C) (p3 – q) x2 – (5p3 – 2q)x + (p3 – q) = 0 (D) (p3 – q) x2 – (5p3 + 2q)x + (p3 – q) = 0

3. Let α and β be the roots of x2 – 6x – 2 = 0, with α > β . If an = αn – βn for n ≥ 1, then the value of

A
a10 − 2a8
is [IIT-JEE 2011, Paper-1, (3, –1), 80]
2a9
(A) 1 (B) 2 (C) 3 (D) 4

UH
4. A value of b for which the equations [IIT-JEE 2011, Paper-2, (3, –1), 80]
x2 + bx – 1 = 0
x2 + x + b = 0
have one root in common is
JA
(A) – 2 (B) – i 3 (C) i 5 (D) 2

5. The quadratic equation p(x) = 0 with real coefficients has purely imaginary roots. Then the equation
p(p(x)) = 0 has [JEE (Advanced) 2014, Paper-2, (3, –1)/60]
(A) only purely imaginary roots (B) all real roots
(C) two real and two purely imaginary roots (D) neither real nor purely imaginary roots
LP

6*. Let S be the set of all non-zero real numbers α such that the quadratic equation αx 2 – x + α = 0 has two
distinct real roots x1 and x2 satisfying the inequality |x1 – x2| < 1. Which of the following intervals is(are) a
subset(s) of S ? [JEE (Advanced) 2015, P-2 (4, –2)/ 80]
 1 1   1   1   1 1
(A)  – , –  (B)  – , 0 (C)  0,  (D)  , 
A

 2 5  5   5  5 2

π π
7. Let – < θ<– . Suppose α1 and β1 are the roots of the equation x2 – 2x sec θ + 1 = 0 and α2 and β2
NK

6 12
are the roots of the equation x2 + 2x tan θ – 1 = 0. If α1 > β1 and α2 > β2 , then α1 + β2 equals
[JEE (Advanced) 2016, Paper-1, (3, –1)/62]
(A) 2(secθ – tan θ) (B) 2sec θ (C) – 2tan θ (D) 0
SA

Comprehension (Q-8 & 9)


Let p, q be integers and let α, β be the roots of the equation, x 2 – x – 1 = 0 where α ≠ β.
For n = 0,1,2,...., let an = pαn + qβn.

FACT : If a and b are rational numbers and a + b 5 = 0, then a = 0 = b.

8. a12 = [JEE(Advanced) 2017, Paper-2,(3, 0)/61]


(A) a11 + 2a10 (B) 2a11 + a10 (C) a11 – a10 (D) a11 + a10

9. If a4 = 28, then p + 2q = [JEE(Advanced) 2017, Paper-2,(3, 0)/61]


(A) 14 (B) 7 (C) 21 (D) 12
PART - II : JEE (MAIN) / AIEEE PROBLEMS (PREVIOUS YEARS)
1. Sachin and Rahul attempted to solve a quadratic equaiton. Sachin made a mistake in writing down the
constant term and ended up in roots (4, 3). Rahul made a mistake in writing down coefficient of x to get
roots (3, 2). The correct roots of equation are : [AIEEE- 2011, II, (4, –1), 120]
(1) 6, 1 (2) 4, 3 (3) –6 , –1 (4) –4 , –3

2. Let for a ≠ a1 ≠ 0, f(x) = ax2 + bx + c, g(x) = a1x2 + b1x + c1 and p(x) = f(x) – g(x). If p(x) = 0 only for
x = –1 and p(–2) = 2, then the value of p(2) is : [AIEEE- 2011, II, (4, –1), 120]
(1) 3 (2) 9 (3) 6 (4) 18

3. The equation esinx – e–sinx – 4 = 0 has : [AIEEE- 2012 (4, –1), 120]

RI
(1) infinite number of real roots (2) no real roots
(3) exactly one real root (4) exactly four real roots

4. If the equations x2 + 2x + 3 = 0 and ax2 + bx + c = 0, a,b,c ∈ R, have a common root, then a : b : c is

A
[AIEEE - 2013, (4, –1), 120]
(1) 1 : 2 : 3 (2) 3 : 2 : 1 (3) 1 : 3 : 2 (4) 3 : 1 : 2

5. If a ∈ R and the equation – 3(x – [x])2 + 2 (x – [x]) + a2 = 0 (where [x] denotes the greatest integer ≤ x)

UH
has no intgeral solution, then all possible values of a lie in the interval :
[JEE(Main)2014,(4, – 1), 120]
(1) (–2, –1) (2) (–∞, –2) ∪ (2, ∞) (3) (–1, 0) ∪ (0, 1) (4) (1, 2)

1 1
JA
6. Let α and β be the roots of equation px2 + qx + r = 0, p ≠ 0. If p, q ,r are in the A.P. and + = 4, then
α β
the value of |α – β| is : [JEE(Main)2014,(4, – 1), 120]
34 2 13 61 2 17
(1) (2) (3) (4)
9 9 9 9
LP

a10 – 2a8
7. Let α and β be the roots of equation x2 – 6x – 2 = 0. If an = αn – βn, for n ≥ 1, then the value of
2a9
is equal to : [JEE(Main)2015,(4, – 1), 120]
(1) 6 (2) – 6 (3) 3 (4) –3
A

8. The number of all possible positive integral values of α for which the roots of the quadratic equation,
6x2 – 11x + α = 0 are rational numbers is :
NK

[JEE(Main) 2019, Online (09-01-19),P-2 (4, – 1), 120]


(1) 3 (2) 4 (3) 5 (4) 2

9. If λ be the ratio of the roots of the quadratic equation in x, 3m 2x2 + m(m – 4)x + 2 = 0, then the least
1
value of m for which λ + = 1, is : [JEE(Main) 2019, Online (12-01-19),P-1 (4, – 1), 120]
SA

λ
(1) –2 + 2 (2) 4 – 3 2 (3) 2 – 3 (4) 4 – 2 3
PART - I
Section (A) :

7 7
A-1. a = 2; No real value of x A-2. (i) – (ii) –
4 8

A-3. (i) ac x2 + b(a + c) x + (a + c)2 = 0 (ii) a2 x2 + (2ac – 4a2 – b2) x + 2b2 + (c – 2a)2 = 0

RI
A-4. 3x2 – 19x + 3 = 0 A-5. 8, 3 A-6. (i) 4 (ii) 72 (iii) 2

A-7. α2β and αβ2 A-10. 2 A-11. 11

A
Section (B) :
(r + 1)3 3 3 −5 −1 25
B-2. – B-3. (i) roots are , , , λ = 45 or , − 1, , λ = –25 .
r2 4 2 3 2 12

UH
−4 3 −5
(ii) roots are ,− , , λ = 121 B-4. x3 – 15x2 + 67x – 77 = 0.
3 2 3

1 1
B-5. –3 B-6. , ,–6
2 2
JA
Section (C) :

C-1. (– 4, 7) C-3. 3±2 2 C-7. (i) 4, – 2 ± i 5 3 (ii) 3 or 4


LP

C-8. –1± 2,–1± −1 C-9. . Two positive roots


A

Section (D) :
NK

D-1. (i) (ii) (iii)


SA

1 3  −4 
D-2. (i) (–∞, 4] (ii) [2, 6] (iii) [3, 6) D-3. (i)  ,  (ii)  −∞ , 5  ∪ (1, ∞)
2 2  

 1
D-4.  −∞, − 2  D-5. (i) a>1 (ii) a ∈ φ.
 

Section (E) :
E-2. K ∈ (− 2, 3) E-3. a ∈ (–2, 2) E-4. a ∈ (1, 5) – {3} E-5. 6 < K < 6.75
Section (F) :
F-2. a = 0, 24 F-3. 3
F-5. (i) (ii)

F-6. (i) k∈[–2,2] (ii) k∈(–∞,–2) ∪ (2, ∞)

RI
PART - II

A
Section (A) :

A-1. (B) A-2. (C) A-3. (A) A-4. (C) A-5. (A)

UH
Section (B) :

B-1. (C) B-2. (C) B-3. (B) B-4. (A) B-5. (C)
JA
Section (C) :

C-1. (B) C-2. (C) C-3. (A) C-4. (A) C-5. (A) C-6. (C)

Section (D) :
LP

D-1. (B) D-2. (B) D-3. (B) D-4. (B) D-5. (A) D-6. (C)

D-7. (C) D-8. (D)

Section (E) :
A

E-1. (D) E-2. (B) E-3. (D) E-4 (D)


NK

Section (F) :
F-1. (A) F-2. (C) F-3. (A) F-4. (C) F-5. (D)

PART - III
SA

1. (A) → (r), (B) → (p), (C) → (q), (D) → (s) 2. (A → r); (B → p,q,s); (C → s); (D → p,q,r)

3. (A) q, s, t (B) p, t (C) r (D) q, s.

EXERCISE - 2
PART – I
1. (B) 2. (B) 3. (D) 4. (B) 5. (A) 6. (A) 7. (A)

8. (A) 9. (B) 10. (A) 11. (D) 12. (B) 13. (B) 14. (A)

15. (C)
PART - II
1. 2 2. 8 3. 1 4. 73 5. 9 6. 10 7. 13

8. 6 9. 1 10. 9 11. 18 12. 32 13. 1 14. 63

15. 1 16. 2 17. 1

PART - III
1. (ACD) 2. (BCD) 3. (BC) 4. (AC) 5. (BCD) 6. (BC) 7. (ABD)

8. (ABCD) 9. (ABCD) 10. (AD) 11. (AD) 12. (AD) 13. (ABD) 14. (AD)

RI
15. (AC) 16. (AB) 17. (CD) 18. (BD) 19. (AB) 20. (ABC) 21. (AB)

PART - IV

A
1. (C) 2. (B) 3. (A) 4. (A) 5. (C) 6. (B) 7. (D)

UH
8. (B) 9. (A) 10. (C)

EXERCISE - 3
PART – I
1. 2 2. (B) 3. (C) 4. (B) 5. (D) 6. (AD) 7. (C)
JA
8. (D) 9. (D)

PART - II
1. (1) 2. (4) 3. (2) 4. (1) 5. (3) 6. (2) 7. (3)
LP

8. (1) 9. (2)
A
NK
SA
1. Find the number of values of x satisfying the relation
 n    j−1 n    n −1 



(x − αi )  n−1  
   i=1 ∏
(x − αi ) ∏ (x − αi )  
 3


∏ (x − α )  i
α13  in=2 +   j−1
j= 2  
∑ i= j+1
n
 αj  +
 

i=1
n −1  αn – x = 0 (where n ≥ 5).
3 3



 i= 2

(α1 − αi ) 


 
  i=1
( α j −∏α i ) ∏ ( α j − α i )  




∏ ( αn − αi ) 


 i= j+1   i=1

2. Prove that roots of a2x2 + (b2 + a2 − c2) x + b2 = 0 are not real, if a + b > c and a − b < c.
(where a, b, c are positive real numbers)

RI
1 4 4 1 1
3. Solve the inequality, – + – < .
x − 1 x − 2 x − 3 x − 4 30

A
4. If three real and distinct numbers a, b, c are in G.P. (i.e., b 2 = ac) and a + b + c = x b, then prove that
x < − 1 or x > 3.

5. If Vn= αn + βn, where α, β are roots of equation x2 + x – 1 = 0. Then prove that Vn + Vn–3 = 2 Vn–2 and

UH
hence evaluate V7 (n is a whole number)

6. Find all ‘m’ for which f (x) ≡ x2 − (m − 3) x + m > 0 for all values of ‘x’ in [1, 2].
JA
7. Find the values of a, for which the quadratic expression ax 2 + (a – 2) x – 2 is negative for exactly two
integral values of x.

8. (i) Solve for real values of 'x' :


x2 – 2a |x – a| – 3a2 = 0, a ≤ 0
3
 1  1
(ii) Find the number of real roots of  x +  +  x +  = 0
LP

 x   x
9. If α, β are roots of the equation x2 – 34x + 1 = 0, evaluate 4
α – 4β , where 4 . denotes the principal
value.

10. Find the values of 'a' for which the equation


A

2 2
(x 2
+x+2 ) ( )( ) (
− ( a − 3 ) x2 + x + 2 x2 + x + 1 + ( a − 4 ) x2 + x + 1 ) = 0 has atleast one real root.
NK

11. Show that the quadratic equation x2 + 7x – 14(q2 + 1) = 0 where q is an integer, has no integral roots.

12. (
Find the integral values of 'a' for which the equation x 4 − a2 − 5a + 6 x 2 − a2 − 3a + 2 ) ( ) = 0 has only
real roots.
SA

13. If α, β; β, γ and γ, α are the roots of ai x 2 + bi x + c i = 0 ; i = 1, 2, 3 then show that


1
 3  a − b + c  
2
(α + β + γ ) + ( αβ+ βγ + αγ ) + αβγ = ± ∏

 i = 1


i
a
i

i
i

 
−1
14. Suppose that a1 > a2 > a3 > a4 > a5 > a6 and

p = a 1 + a2 + a3 + a4 + a5 + a6

q = a1a3 + a3a5 + a5a1 + a2a4 + a4a6 + a6a2

r = a1a3a5 + a2a4a6 ,

then show that roots of the equation 2x3 – px2 + qx – r = 0 are real.

15. If β + cos2α, β + sin2α are the roots of x2 + 2bx + c = 0 and γ + cos4α, γ + sin4α are the roots of
X2 + 2BX + C = 0, then prove that b2 – B2 = c – C.

16. Find the set of values of 'a' if (x2 + x)2 + a (x2 + x) + 4 = 0 has

RI
(i) all four real & distinct roots.
(ii) four roots in which only two roots are real and distinct.
(iii) all four imaginary roots.

A
(iv) four real roots in which only two are equal.

17. f(x) = x2 + bx + c , where b,c ∈ R, if f(x) is a factor of both x 4 + 6x2 + 25 and 3x4 + 4x2 + 28x + 5 then find

UH
f(x).

18. Let ax4 + bx3 + x2 +(3–a) x + 3 = 0 and x2 + (2–a) x + 3 = 0 have common roots. If a∈(–1,5) then find
|a+12b|
JA
19. How many quadratic equations are there which are unchanged by squaring their roots ?

20. Let P(x) = x5 + x2 + 1 have zeros α1, α2, α3, α4, α5 and Q(x) = x2 – 2, then find
5 5 5
(i) ∏
i =1
Q(αi ) (ii) ∑
i =1
Q(αi ) (iii) ∑ ∑
1 ≤ i < j ≤5
Q(αi ) Q(α j ) (iv) ∑ Q (α )
i =1
2
i
LP

21. If a, b, c are non–zero, unequal rational numbers then prove that the roots of the equation
(abc2)x2 + 3a2 cx + b2 cx – 6a2 – ab + 2b2 = 0 are rational.

22. If a, b, c represents sides of a ∆ then prove that equation x 2 – (a2 + b2 + c2)x + a2b2 + b2c2 + c2a2 = 0 has
A

imaginary roots.

If x1 is a root of ax2 + bx + c = 0, x2 is a root of − ax2 + bx + c = 0 where 0 < x1 < x2, show that the
NK

23.
equation ax2 + 2 bx + 2c = 0 has a root x3 satisfying 0 < x1 < x3 < x2.

24. Find the number of positive real roots of x4 – 4x – 1 = 0.


SA

25. If (1 + k) tan2x – 4 tan x – 1 + k = 0 has real roots tan x 1 and tan x2, where tan x1 ≠ tan x2 , then find k.

26. Let ∆2 be the discriminant and α, β be the roots of the equation ax2 + bx + c = 0. Then
find equation whose roots are 2aα + ∆ and 2a β – ∆.

πe eπ ππ + ee
27. Prove that + + = 0 has one real root in (e, π) and other in (π, π + e).
x−e x−π x−π−e

28. If α, β2 are integers, β2 is non-zero multiple of 3 and α + iβ, – 2α are roots of x 3 + ax2 + bx – 316 = 0,
a, b, β ∈ R, then find a, b.
29_. Let polynomial f(x) = ax4 + bx3 + cx2 + dx + e have integral coefficient (where a > 0) If there exist four
distinct integer α1, α2, α3, α4 (α1 < α2 < α3 < α4) such that f(α1) = f(α2) = f(α3) = f(α4) = 5 and equation
f(x) = 9 has integeral roots then find
 α + α2 + α3 + α 4   α + α2 + α3 + α 4 
(i) f 1  (ii) f ' 1 
 4   4 
(iii) Range of f(x) in [α2, α3]

(iv) Difference of largest and smallest root of equation f(x) = 9

30. If x and y both are non-negative integral values for which (xy – 7)2 = x2 + y2, then find the sum of all
possible values of x.

RI
1. Infinite 3. (– ∞, – 2) ∪ (– 1, 1) ∪ (2, 3) ∪ (4, 6) ∪ (7, ∞) 5. –29

A
6. (− ∞, 10) 7. [1, 2)

UH
8. (i) x = a (1 – 2 ), x = a ( 6 – 1) (ii) 0

19
9. ±2 10. 5<a< 12. a ∈ {1, 2}
3
JA
 65   65 
16. (i) a ∈ (– ∞, – 4) (ii) a ∈  , ∞ (iii) a ∈  −4, (iv) a ∈ φ
 4   4 

17. x2 – 2x + 5 18. 3 19. 4 20. (i) – 23 (ii) – 10 (iii) 40 (iv) 20

24. 1 25. (− ) (
5, − 1 ∪ −1, 5 ) 26. x2 + 2b x + b2 = 0 or x2 + 2bx – 3b2 + 16 ac = 0
LP

28. a = 0, b = 63 29. (i) 9 (ii) 0 (iii) [5, 9] (iv) 2 5

30. 14
A
NK
SA
BINOMIAL THEOREM

JEE (ADVANCED) SYLLABUS

Binomial theorem for a positive integral index, properties of binomial coefficients.

JEE (MAIN) SYLLABUS

RI
Binomial theorem for a positive integral index, general term and middle term, properties of Binomial coefficients
and simple applications.

A
bvious” is the most dangerous word in mathematics......... Bell, Eric Temple

Binomial expression :

UH
Any algebraic expression which contains two dissimilar terms is called binomial expression.
1 1
For example : x + y, x2y + 2 , 3 – x, x 2 + 1 + 3 etc.
xy (x + 1)1/ 3
Terminology used in binomial theorem :
JA
n(n − 1)(n − 2)........3 . 2 . 1 ; if n ∈N
n! = 
 1 ; if n=0
Note : n! = n . (n – 1)! ; n∈N
LP

Mathematical meaning of nCr : The term nCr denotes number of combinations of r things choosen from n
n !
distinct things mathematically, nCr = , n, r ∈ W, 0 ≤ r ≤ n
(n − r)! r!
n
Note : Other symbols of of nCr are   and C(n, r).
A

r 
Properties related to nCr :

(i) nCr = nCn – r


NK

Note : If n
Cx = n Cy ⇒ Either x = y or x + y = n
(ii) n
Cr + n Cr – 1 = n + 1 Cr
n
Cr n −r +1
(iii) n
=
Cr −1 r
SA

n n–1 n(n − 1) n–2 n(n − 1)(n − 2).........(n − (r − 1))


(iv) n
Cr = Cr–1 = Cr–2 = ............. =
r r(r − 1) r (r − 1)(r − 2).......2 .1
(v) If n and r are relatively prime, then Cr is divisible by n. But converse is not necessarily true.
n

Statement of binomial theorem :


(a + b)n = nC0 anb0 + nC1 an–1 b1 + nC2 an–2 b2 +...+ nCr an–r br +...... + nCn a0 bn
where n ∈ N
n
or (a + b)n = ∑
r =0
n
Cr an−r br

Note : If we put a = 1 and b = x in the above binomial expansion, then


or (1 + x)n = nC0 + nC1 x + nC2 x2 +... + nCr xr +...+ nCn xn
n
or (1 + x)n = ∑
r =0
n
Cr xr

Example # 1 : Expand the following binomials :


4
 3x 2 
 1 −
5
(i) (x + 2 ) (ii) 
 2 
2 3 4 5
Solution : (i) (x + 2 )5 = 5C0x5 + 5C1x4 ( 2) + 5
C2 x 3 ( 2) + 5C3 x2 ( 2) + 5C4x ( 2) + 5 C5 ( 2)
= x5 + 5 2 x4 + 20x3 + 20 2 x2 + 20x + 4 2

RI
4 2 3 4
 3x 2   3x 2  4  3x 2   3x 2   3x 2 
(ii)  1 −  = 4C0 + 4C1  −  + C2  −  + 4C3  −  + 4C4  − 
 2   2   2   2   2 
27 27 6 81 8

A
= 1 – 6x2 + x4 – x + x
2 2 16
10
2 
Example # 2 : Expand the binomial  + x  up to four terms

UH
x 
10 10 9 8 7
2  2 2 2 2
Solution :  x + x = 10C0   + 10C1   x + 10
C2   x2 + 10C3   x3 + ....
  x x x x
Self practice problems :
JA
6
 y
(1) Write the first three terms in the expansion of  2 −  .
 3
5
 x2 3 
Expand the binomial 
 3 x 
(2) + .
 
x10
LP

80 2 5 7 10 4 135 243
Ans. (1) 64 – 64y + y (2) + x + x + 30x + 2 + 5 .
3 243 27 3 x x

Observations :
(i) The number of terms in the binomial expansion (a + b) n is n + 1.
A

(ii) The sum of the indices of a and b in each term is n.


(iii) The binomial coefficients (nC0, nC1 ..........nCn) of the terms equidistant from the beginning and
the end are equal, i.e. nC0 = nCn, nC1 = nCn–1 etc. { nCr = nCn–r}
NK

(iv) The binomial coefficient can be remembered with the help of the following pascal’s Triangle
(also known as Meru Prastra provided by Pingla)
SA

Regarding Pascal’s Triangle, we note the following :

(a) Each row of the triangle begins with 1 and ends with 1.
(b) Any entry in a row is the sum of two entries in the preceding row, one on the immediate left and
the other on the immediate right.
Example # 3 : The number of dissimilar terms in the expansion of (1 + x 4 – 2x2 )15 is
(A) 21 (B) 31 (C) 41 (D) 61
Solution : (1 – x2)30
Therefore number of dissimilar terms = 31.
General term :
(x + y)n = nC0 xn y0 + nC1 xn–1 y1 + ...........+ nCr xn–r yr + ..........+ nCn x0 yn
(r + 1)th term is called general term and denoted by Tr+1.
Tr+1 = nCr xn–r yr
Note : The rth term from the end is equal to the (n – r + 2)th term from the begining, i.e. n
Cn – r + 1 xr – 1 yn – r + 1
7
 3x 
Example # 4 : Find (i) 15th term of (2x – 3y)20 (ii) 4th term of  − y
 5 
Solution : (i) T14 + 1 = 20C14 (2x)6 (–3y)14 = 20C14 26 314 x6.y14

RI
4 4
 3x  3
(ii) T3 + 1 = 7C3   (–y)3 = –7C3   x4y3
 5  5
600
 1 1

A
Example # 5 : Find the number of rational terms in the expansion of  2 3 + 3 5 
 
 
600
 1 1

UH
Solution : The general term in the expansion of  2 3 + 3 5  is
 
 
600 − r r 600 − r
 1  51  600 r
Tr+1 = 600Cr  2 3   
3 = C r
2 3
3 5

   
JA
The above term will be rational if exponent of 3 and 2 are integers
600 − r r
It means and must be integers.
3 5
The possible set of values of r is {0, 15, 30,45............,600}
Hence, number of rational terms is 41
Middle term(s) :
LP

th
n+2
(a) If n is even, there is only one middle term, which is   term.
 2 
th th
 n + 1  n +1 
(b) If n is odd, there are two middle terms, which are   and  + 1 terms.
 2   2 
A

Example # 6 : Find the middle term(s) in the expansion of


11
 y2 
(i) (1 + 2x)12 (ii)  2y − 
NK

 2 
Solution : (i) (1 + 2x)12
 12 + 2  th
Here, n is even, therefore middle term is   term.
 2 
It means T7 is middle term T7 = 12C6 (2x)6
SA

11
 y2 
(ii)  2y − 
 2 
 11 + 1  th  11 + 1  th
Here, n is odd therefore, middle terms are   &  2 + 1 .
 2   
It means T6 & T7 are middle terms
5 6
 y2   y2  11
C6 17
T6 = 11C5 (2y)6  −  = –2 11C5 y16 ⇒ T7 = 11C6 (2y)5  −  = y
 2   2  2
16
 1 
Example # 7 : Find term which is independent of x in  x 2 – 6 
 x 
r
 1 
Solution : Tr + 1 = 16Cr (x2)16–r  – 6 
 x 
For term to be independent of x, exponent of x should be 0
32 – 2r = 6r ⇒ r=4 ∴ T5 is independent of x.

Numerically greatest term in the expansion of (a + b)n, n ∈ N


Binomial expansion of (a + b)n is as follows : –
(a + b)n = nC0 anb0 + nC1 an–1 b1 + nC2 an–2 b2 +...+ nCr an–r br +...... + nCn a0 bn
If we put certain values of a and b in RHS, then each term of binomial expansion will have certain
value. The term having numerically greatest value is said to be numerically greatest term.
Let Tr and Tr+1 be the rth and (r + 1)th terms respectively
Tr = nCr–1 an–(r–1) br–1

RI
Tr+1 = nCr an–r br
Tr +1 n
Cr an−r br n −r +1 b
Now, = n n −r +1 r −1
= .
Tr Cr −1 a b r a

A
Tr +1
Consider ≥1
Tr

UH
 n − r + 1 b n +1 a n +1
  ≥1 ⇒ –1≥ ⇒r≤
 r  a r b
1+
a
b
n +1
Case - Ι When is an integer (say m), then
a
JA
1+
b
(i) Tr+1 > Tr when r < m (r = 1, 2, 3 ...., m – 1)
i.e. T2 > T1, T3 > T2, ......., Tm > Tm–1
(ii) Tr+1 = Tr when r = m
i.e. Tm+1 = Tm
(iii) Tr+1 < Tr when r > m (r = m + 1, m + 2, ..........n )
LP

i.e. Tm+2 < Tm+1 , Tm+3 < Tm+2 , ..........Tn+1 < Tn

Conclusion :
n +1
When is an integer, say m, then Tm and Tm+1 will be numerically greatest terms (both terms are
A

a
1+
b
equal in magnitude)
NK

Case - ΙΙ
When is not an integer (Let its integral part be m), then
(i) Tr+1 > Tr when r < (r = 1, 2, 3,........, m–1, m)
i.e. T2 > T1 , T3 > T2, .............., Tm+1 > Tm
(ii) Tr+1 < Tr when r > (r = m + 1, m + 2, ..............n)
SA

i.e. Tm+2 < Tm+1 , Tm+3 < Tm+2 , .............., Tn+1 < Tn

Conclusion :
When is not an integer and its integral part is m, then T m+1 will be the numerically greatest term.

Note : (i) In any binomial expansion, the middle term(s) has greatest binomial coefficient.
In the expansion of (a + b)n

If n No. of greatest binomial coefficient Greatest binomial coefficient


Even 1 n
Cn/2
Odd 2 n
C(n – 1)/2 and nC(n + 1)/2
(Values of both these coefficients are equal )
(ii) In order to obtain the term having numerically greatest coefficient, put a = b = 1, and proceed
as discussed above.

1
Example # 8 : Find the numerically greatest term in the expansion of (7 – 3x) 25 when x = .
3
n +1 25 + 1 26
Solution : m= = =
a 7 8
1+ 1+
b –1
[m] = 3 ([m] denotes GIF)
∴ T4 is numerically greatest term

RI
Self practice problems :
9
 3
(3) Find the term independent of x in  x 2 − 
 x
(4) The sum of all rational terms in the expansion of (31/7 + 51/2)14 is

A
(A) 32 (B) 32 + 57 (C) 37 + 52 (D) 57
18
 1
(5) Find the coefficient of x–2 in (1 + x2 + x4)  1– 2 
 x 

UH
(6) Find the middle term(s) in the expansion of (1 + 3x + 3x 2 + x3)2n
2
(7) Find the numerically greatest term in the expansion of (2 + 5x) 21 when x = .
5
Ans. (3) 28.37 (4) B (5) –681
(6) 6n
C3n . x3n (7) T11 = T12 = 21C10 221
JA
Example # 9 : Show that 7n + 5 is divisible by 6, where n is a positive integer.
Solution : 7n + 5 = (1 + 6)n + 5 = nC0 + nC1 . 6 + nC2 . 62 + ....... + nCn 6n + 5.
= 6. C1 + 62. C2 + ....... + Cn . 6n + 6.
= 6λ, where λ is a positive integer
Hence, 7n + 5 is divisible by 6.
LP

Example # 10 : What is the remainder when 781 is divided by 5.


Solution : 781 = 7.780 = 7. (49)40 = 7 (50 – 1)40
= 7 [40C0 (50)40 – 40C1 (50)39 + .......... – 40C39 (50)1 + 40C40 (50)0]
= 5(k) + 7(where k is a positive integer) = 5 (k + 1) + 2
A

Hence, remainder is 2.
Example # 11 : Find the last digit of the number (13)12.
Solution : (13)12 = (169)6 = (170 – 1)6
NK

= 6C0 (170)6 – 6C1 (170)5 + ........ – 6C5 (170)1 + 6C6 (170)0


Hence, last digit is 1
Note : We can also conclude that last three digits are 481.
Example-12 : Which number is larger (1.1)100000 or 10,000 ?
Solution : By Binomial Theorem
SA

(1.1)100000 = (1 + 0.1)100000 = 1 + 100000C1 (0.1) + other positive terms


= 1 + 100000 × 0.1 + other positive terms
= 1 + 10000 + other positive terms
Hence (1.1)100000 > 10,000
Self practice problems :
(8) If n is a positive integer, then show that 6n – 5n – 1 is divisible by 25.
(9) What is the remainder when 3257 is divided by 80 .
(10) Find the last digit, last two digits and last three digits of the number (81) 25.
(11) Which number is larger (1.3)2000 or 600
Ans. (9) 3 (10) 1, 01, 001 (11) (1.3)2000.
Some standard expansions :
(i) Consider the expansion
n
(x + y)n = ∑
r =0
n
Cr xn–r yr = nC0 xn y0 + nC1 xn–1 y1 + ...........+ nCr xn–r yr + ..........+ nCn x0 yn ....(i)

(ii) Now replace y → – y we get


n
(x – y)n = ∑
r =0
n
Cr (– 1) r xn–r yr = nC0 xn y0 – nC1 xn–1 y1 + ...+ nCr (–1)r xn–r yr + ...+ nCn (– 1)n x0 yn ....(ii)

(iii) Adding (i) & (ii), we get


(x + y)n + (x – y)n = 2[nC0 xn y0 + nC2 xn – 2 y2 +.........]

RI
(iv) Subtracting (ii) from (i), we get
(x + y)n – (x – y)n = 2[nC1 xn – 1 y1 + nC3 xn – 3 y3 +.........]

Properties of binomial coefficients :

A
(1 + x)n = C0 + C1x + C2x2 + ......... + Cr xr + .......... + Cnxn ......(1)
where Cr denotes nCr

(1) The sum of the binomial coefficients in the expansion of (1 + x) n is 2n

UH
Putting x = 1 in (1)
n
C0 + nC1 + nC2 + ........+ nCn = 2n ......(2)
n
or ∑
r =0
n
Cr = 2n
JA
(2) Again putting x = –1 in (1), we get
n
C0 – nC1 + nC2 – nC3 + ............. + (–1)n nCn = 0 ......(3)
n
or ∑ (−1)
r =0
r n
Cr = 0

(3) The sum of the binomial coefficients at odd position is equal to the sum of the binomial
LP

coefficients at even position and each is equal to 2n–1.


from (2) and (3)
n
C0 + nC2 + nC4 + ................ = nC1 + nC3 + nC5 + ................ = 2n–1
A

(4) Sum of two consecutive binomial coefficients


n
Cr + nCr–1 = n+1Cr
n! n!
L.H.S. = nCr + nCr–1 = +
(n − r)! r! (n − r + 1)! (r − 1)!
NK

n! 1 1 
= +
(n − r)! (r − 1)!  r n − r + 1

n! (n + 1)
=
(n − r)! (r − 1)! r(n − r + 1)
SA

(n + 1)!
=
= n+1Cr = R.H.S.
(n − r + 1)! r!
(5) Ratio of two consecutive binomial coefficients
n
Cr n −r +1
n
=
Cr −1 r
n n(n − 1) n–2 n(n − 1)(n − 2).........(n − (r − 1))
(6) n
Cr = n–1
Cr–1 = Cr–2 = ............. =
r r(r − 1) r (r − 1)(r − 2).......2 .1

Example # 13 : If (1 + x)n = C0 + C1x + C2x2 + ............. + cnxn, then show that


(i) C0 + 4C1 + 42C2 + .......... + 4n Cn = 5n. (ii) 3C0 + 5C1 + 7. C2 + ........ + (2n + 3) Cn = 2n (n + 3).
C1 C C Cn 2n +1 – 1
(iii) C0 + + 2 + 3 + ......... + =
2 3 4 n +1 n +1
Solution : (i) (1 + x)n = C0 + C1 x + C2x2 + ........... + Cnxn
put x = 4
C0 + 4C1 + 42C2 + .......... + 4n Cn = 5n.
(ii) L.H.S. = 3C0 + 5C1 + 7. C2 + ........ + (2n + 3) Cn
n n n
= ∑ (2r + 3) . nCr = 2∑ r. n
Cr + 3∑ n Cr
r =0 r =0 r =0

n n
= 2n ∑
r =1
n −1
Cr −1 + 3 ∑
r =0
n
Cr = 2n . 2n – 1 + 3. 2n = 2n (n + 3) RHS

(iii) Ι Method : By Summation

RI
C C C Cn
L.H.S. = C0 + 1 + 2 + 3 + ......... +
2 3 4 n +1
n
n
Cr 1 n
n + 1  2n +1 – 1
=∑ .= ∑ . n + 1 Cr + 1  . n
Cr = n+1 Cr +1  = R.H.S.

A
r=0 r +1 n +1 r =0 r +1  n +1
ΙΙ Method : By Integration

UH
(1 + x)n = C0 + C1x + C2x2 + ...... + Cn xn. Integrating both sides, within the limits 0 to 1.
1 1
 (1 + x)n + 1   x2 x3 x n +1 
  = C0 x + C1 + C2 + ..... + Cn 
 n + 1 0  2 3 n + 1 0
2n +1 1  C C C 
– =  C0 + 1 + 2 + ..... + n  – 0
n +1 n +1  2 3 n + 1
JA
C C C Cn 2n +1 – 1
C0 + 1 + 2 + 3 + ......... + = Proved
2 3 4 n +1 n +1

Example # 14 : If (1 + x)n = C0 + C1x + C2x2 + ........+ Cnxn, then prove that


(i) C0C1 + C1C2 + C2C3 + .......... + Cn – 1 Cn = 2nCn – 1 or 2nCn + 1
LP

(ii) 12. C12 + 22 . C22 + 32. C32 + ......... + n2 Cn2 . = n2. 2n – 2Cn–1
Solution : (i) (1 + x)n = C0 + C1x + C2x2 + ......... + Cn xn. ........(i)
(x + 1)n = C0xn + C1xn – 1+ C2xn – 2 + ....... + Cn x0 ........(ii)
Multiplying (i) and (ii)
(C0 + C1x + C2x2 + ......... + Cnxn) (C0xn + C1xn – 1 + ......... + Cnx0) = (1 + x)2n
A

Comparing coefficient of xn–1,


C0C1 + C1C2 + C2C3 + .......... + Cn – 1 Cn = 2nCn – 1 or 2nCn + 1
(ii) (1 + x)n = C0 + C1x + C2x2 + ......... + Cn xn. ........(i)
NK

differentiating w.r.t x......


n(1 + x)n–1 = C1 + 2C2x + 3C3x2 + ......... + nCn xn–1.
multiplying by x.......
n x(1 + x)n–1 = C1x + 2C2x2 + 3C3x3 + ......... + nCn xn
Now differentiate w.r.t. x.....
SA

n(1 + x)n–1 + n (n–1)x.(1+x)n–2 = 12C1 + 22C2x + 32C3x2 + ....... + n2Cnxn–1 ........(ii)


(x + 1)n = C0xn + C1xn – 1+ C2xn – 2 + ....... + Cn x0 ........(iii)
multiplying (ii) & (iii) and comparing the cofficient of x n–1
12. C12 + 22 . C22 + 32. C32 + ......... + n2 Cn2. = n ( 2n–1
)
Cn–1 – 2n–2 Cn–2 + n2 2n–2Cn–2
=n 2 2n–2
Cn–1 = R.H.S.
Example # 15 : Find the summation of the following series –
(i) mC0 + m+1C1 + m+2C2 + .............. + nCm (ii) nC3 + 2 . n+1C3 + 3. n+2C3 + ......... + n . 2n–1C3
Solution : (i) Ι Method : Using property, nCr +nCr–1 = n+1Cr
m
C0 + m+1C1 + m+2C2 + .............. + nCm
m
Cm + m+1Cm + m+2Cm + .............. + nCm
m +1
= Cm+1 +m+1 Cm + m+2Cm + .............. + nCm { mCm = m+1Cm+1}
  
m+ 2
= Cm+1 +m+ 2 Cm + .................. + nCm = m+3Cm+1 + ............. + nCm = nCm+1 + nCm = n+1Cm+1
  

ΙΙ Method
m
Cm + m+1Cm + m+2Cm + .......... + nCm

RI
The above series can be obtained by writing the coefficient of x m in
(1 + x)m + (1 + x)m+1 + ......... + (1 + x)n
Let S = (1 + x)m + (1 + x)m+1 +.............. + (1 + x)n
n −m +1
(1 + x)m (1 + x ) − 1 n +1 m
  (1 + x ) − (1 + x )

A
= =
x x
n +1
= coefficient of x in m
(1 + x )

(1 + x )m
= n + 1Cm +1 + 0 = n + 1Cm +1

UH
x x
(ii) n
C3 + 2 . n+1C3 + 3 . n+2C3 + .......... + n . 2n–1C3
The above series can be obatined by writing the coefficient of x 3 in
(1 + x)n + 2 . (1 + x)n+1 + 3 . (1 + x)n+2 + ........... + n . (1 + x)2n–1
Let S = (1 + x)n + 2 . (1 + x)n+1 + 3. (1 + x)n+2 + ........... + n (1 + x)2n–1 .....(i)
JA
(1 + x)S = (1 + x)n+1 + 2 (1 + x)n+2 + ............. + (n – 1) (1 + x)2n–1 + n(1 + x)2n ....(ii)
Subtracting (ii) from (i)
– xS = (1 + x)n + (1 + x)n+1 + (1 + x)n+2 + .............. + (1 + x)2n–1 – n(1 + x)2n
(1 + x)n (1 + x)n − 1
 
= – n (1 + x)2n
x
n(1 + x)2n
−(1 + x)2n + (1 + x)n
S= +
LP

2 x
x
x3 : S (coefficient of x3 in S)
−(1 + x)2n + (1 + x)n n(1 + x)2n
x3 : +
x2 x
A

Hence, required summation of the series is – 2nC5 + nC5 + n . 2nC4



Example # 16 : Prove that C1 – C3 + C5 – ........ = 2n/2 sin .
4
NK

Solution : Consider the expansion (1 + x)n = C0 + C1 x + C2 x2 + ...... + Cn xn ....(i)


putting x = – i in (i) we get
(1 – i)n = C0 – C1i – C2 + C3i + C4 + ....... (– 1)n Cn in
  nπ   nπ  
or 2n 2 cos  −  + i sin  −   = (C0 – C2 + C4 – ....) – i (C1 – C3 + C5 – .....) ....(ii)
  4   4 
SA


Equating the imaginary part in (ii) we get C1 – C3 + C5 – ........ = 2n/2 sin .
4
Self practice problems :
(12) Prove the following
(i) 5C0 + 7C1 + 9C2 + ............. + (2n + 5) Cn = 2n (n + 5)
42 43 4n+1 5n+1 − 1
(ii) 4C0 + . C1 + C2 + .............. + Cn =
2 3 n +1 n +1
(iii) n
C0 . n+1Cn + nC1 . nCn–1 + nC2 . n–1Cn–2 + ........... + nCn . 1C0 = 2n–1 (n + 2)
(iv) 2
C2 + 3C2 + ......... + nC2 = n+1C3
Binomial theorem for negative and fractional indices :
n(n − 1) 2 n(n − 1)(n − 2) 3
If n ∈ R, then (1 + x)n = 1 + nx + x + x + ................
2 ! 3 !
n(n − 1)(n − 2).......(n − r + 1) r
.................. + x + .................... ∞.
r !
Remarks
(i) The above expansion is valid for any rational number other than a whole number if | x | < 1.
(ii) When the index is a negative integer or a fraction then number of terms in the expansion of
(1 + x)n is infinite, and the symbol nCr cannot be used to denote the coefficient of the general
term.
(iii) The first term must be unity in the expansion, when index ‘n’ is a negative integer or fraction

RI
  n  y n (n − 1)  y 
2 
 xn  1 +  = xn 1 + n .
y y
+   + .....  if <1
  x 

x 2 ! x 

x

(x + y)n = 

A

 n x
n  x n (n − 1)  x 
2 
n x
 
y 1 +  = y  1 + n . +   + .....  if <1
  y   y 2 !  
y  y

UH
n(n − 1)(n − 2).........(n − r + 1) r
(iv) The general term in the expansion of (1 + x)n is Tr+1 = x
r !
(v) When ‘n’ is any rational number other than whole number then approximate value of (1 + x) n is
1 + nx (x2 and higher powers of x can be neglected)
(vi) Expansions to be remembered (|x| < 1)
JA
(a) (1 + x)–1 = 1 – x + x2 – x3 + .......... + (–1)r xr + .........∞
(b) (1 – x)–1 = 1 + x + x2 + x3 + .......... + xr + .........∞
(c) (1 + x)–2 = 1 – 2x + 3x2 – 4x3 + .......... + (–1)r (r + 1) xr + ...........∞
(d) (1 – x)–2 = 1 + 2x + 3x2 + 4x3 + ............. + (r + 1)xr + ........... ∞

Example # 17 : Prove that the coefficient of xr in (1 – x)–n is n+r–1Cr


LP

Solution: (r + 1)th term in the expansion of (1 – x)–n can be written as


−n( −n − 1)( −n − 2)......( −n − r + 1)
Tr +1 = (–x)r
r !
n(n + 1)(n + 2)......(n + r − 1) n(n + 1)(n + 2)......(n + r − 1) r
= (–1)r (–x)r = x
A

r ! r !
(n − 1)! n(n + 1)......(n + r − 1) r (n + r − 1)! n+r–1
= x Hence, coefficient of xr is = Cr Proved
(n − 1) ! r ! (n − 1)! r!
NK

Example-18 : If x is so small such that its square and higher powers may be neglected, then find the value of
(1 − 2x)1/ 3 + (1 + 5x)–3 / 2
(9 + x)1/ 2
2 15x
x + 1− 1−
SA

−1/ 2
(1 − 2x)1/ 3 + (1 + 5x)–3 / 2
3 2 = 1  2 − 49 x   1 + x 
Solution : =  6   9 
(9 + x)1/ 2  x
1/ 2
3 
3 1 + 
 9
1  49   x  1  x 49  x 49 149
= 2 − x  1 −  =  2− − x = 1 – – x=1– x
3  6   18  2  9 6  18 12 36

Self practice problems :


(13) Find the possible set of values of x for which expansion of (3 – 2x) 1/2 is valid in ascending
powers of x.
2 3
2 1.3  2  1.3.5  2 
(14) If y = +   + 3 !  5  + ............., then find the value of y + 2y
2
5 2 ! 5  
2 − 3x
(15) The coefficient of x50 in is
(1 − x)3
(A) 500 (B) 1000 (C) –1173 (D) 1173
 3 3
Ans. (13) x ∈− ,  (14) 4 (15) C
 2 2
n n
Multinomial theorem : As we know the Binomial Theorem (x + y)n = ∑ n Cr xn–r yr = ∑ (n − r)!
n!
xn–r yr
r =0 r =0
r!


n!
putting n – r = r1 , r = r2 therefore, (x + y)n = xr1 . yr2
r1 +r2
r
=n 1
! r 2 !
Total number of terms in the expansion of (x + y)n is equal to number of non-negative integral solution

RI
of r1 + r2 = n i.e. n+2–1C2–1 = n+1C1 = n + 1
In the same fashion we can write the multinomial theorem


n!
(x1 + x2 + x3 + ........... xk)n = x1r1 . xr22 ...xkrk

A
r
r +r +...+r =n 1
! r2 !... rk !
1 2 k

Here total number of terms in the expansion of (x1 + x2 + .......... + xk)n is equal to number of non-
negative integral solution of r1 + r2 + ........ + rk = n i.e. n+k–1Ck–1

UH
Example # 19 : Find the coefficient of a2 b3 c4 d in the expansion of (a – b – c + d)10


(10)!
Solution : (a – b – c + d)10 = (a)r1 ( −b)r2 ( −c)r3 (d)r4
r
r +r +r +r =10 1
! r !r
2 3 ! r4 !
1 2 3 4

we want to get a2 b3 c4 d this implies that r1 = 2, r2 = 3, r3 = 4, r4 = 1


JA
(10)!
∴ coeff. of a2 b3 c4 d is (–1)3 (–1)4 = – 12600
2! 3! 4! 1!
11
 7
Example # 20 : In the expansion of  1 + x +  , find the term independent of x.
 x
11 r
 7 r2  7 

3
(11)!
LP

r1
Solution :  1 x = (1) (x)
x  x
+ +
 r ! r2 ! r3 !
r1 +r2 +r3 =11 1  
7
The exponent 11 is to be divided among the base variables 1, x and in such a way so that we
x
get x0. Therefore, possible set of values of (r1, r2, r3) are (11, 0, 0), (9, 1, 1), (7, 2, 2), (5, 3, 3), (3, 4, 4),
A

(1, 5, 5)

Hence the required term is


NK

(11)! 0 (11)! (11)! (11)! (11)! (11)!


(7 ) + 71 + 72 + 73 + 74 + 75
(11)! 9! 1 !1 ! 7! 2 ! 2 ! 5! 3 ! 3 ! 3! 4 ! 4 ! 1 ! 5 ! 5 !
(11)! 2 ! (11)! 4 ! (11) ! 6 ! (11) ! 8 !
=1+ . 71 + . 72 + . 73 + . 74
9 ! 2 ! 1 ! 1 ! 7 ! 4 ! 2 ! 2 ! 5 ! 6 ! 3 ! 3 ! 3 ! 8 ! 4 ! 4 !
(11) ! (10) ! 5
SA

+ . 7
1 ! 10 ! 5 ! 5 !
5
= 1+11C2 . 2C1 . 71 + 11C4 . 4C2 . 72 + 11C6 . 6C3 . 73 + 11C8 . 8C4 . 74 + 11C10 . 10C5 . 75 = 1 + ∑
r =1
11
C2r . 2rCr . 7r

Self practice problems :


(16) The number of terms in the expansion of (a + b + c + d + e) n is
(A) n+4C4 (B) n+3Cn (C) n+5Cn (D) n + 1
(17) Find the coefficient of x2 y3 z1 in the expansion of (x – 2y – 3z)7
(18) Find the coefficient of x17 in (2x2 – x – 3)9
7!
Ans. (16) A (17) 24 (18) 2304
2! 3! 1!
 Marked questions are recommended for Revision.

PART - I : SUBJECTIVE QUESTIONS

Section (A) : General Term & Coefficient of xk in (ax +b)n


A-1. Expand the following:
5 4
2 x  2 2
(i)  x − 2  , (x ≠ 0) (ii)  y +  , (y ≠ 0)
 

RI
 y

n
 1 
A-2. In the binomial expansion of  3 2 +  , the ratio of the 7th term from the begining to the 7th term
3
 3

A
from the end is 1 : 6 ; find n.

5 5
(
Find the degree of the polynomial x + (x 3 − 1) 2
1
) + ( x − (x − 1) 2
1
).

UH
3
A-3.

A-4. Find the coefficient of


(i) x6y3 in (x + y)9 (ii) a5 b7 in (a – 2b)12
JA11 11
 1   1 
A-5. Find the co-efficient of x7in  ax 2 +  and of x–7 in  ax −  and find the relation between
 b x2
 b x  
'a' & 'b' so that these co-efficients are equal. (where a, b ≠ 0).

A-6. Find the term independent of 'x' in the expansion of the expression,
9
3 1 
LP

(1 + x + 2 x3)  x 2 −  .
 2 3 x

A-7. (i) Find the coefficient of x5 in (1 + 2x)6(1 – x)7.


(ii) Find the coefficient of x4 in (1 + 2x)4 (2 – x)5
A

n
 1 
A-8. In the expansion of  x 3 − 2  , n ∈ N, if the sum of the coefficients of x5 and x10 is 0, then n is :
 x 
NK

Section (B) : Middle term, Remainder & Numerically/Algebraically Greatest terms


B-1. Find the middle term(s) in the expansion of
7
x y
 − 
SA

(i) (ii) (1 – 2x + x2)n


y x

B-2. Prove that the co-efficient of the middle term in the expansion of (1 + x) 2n is equal to the sum of the
co-efficients of middle terms in the expansion of (1 + x)2n − 1.

B-3. (i) Find the remainder when 798 is divided by 5


(ii) Using binomial theorem prove that 6n – 5n always leaves the remainder 1 when divided by 25.
(iii) Find the last digit, last two digits and last three digits of the number (27) 27.

B-4. Which is larger : (9950 + 10050) or (101)50.


1
B-5. (i) Find numerically greatest term(s) in the expansion of (3 – 5x)15 when x =
5
(ii) Which term is the numerically greatest term in the expansion of (2x + 5y)34, when x = 3 & y = 2 ?

B-6. Find the term in the expansion of (2x – 5)6 which have
(i) Greatest binomial coefficient (ii) Greatest numerical coefficient
(iii) Algebraically greatest coefficient (iv) Algebraically least coefficient

Section (C) : Summation of series, Variable upper index & Product of binomial
coefficients
C-1. If C0, C1, C2, ... Cn are the binomial coefficients in the expansion of (1 + x)n then prove that :

RI
( 3.2 − 1) 32.22 − 1 33.23 − 1 3n.2n − 1 23n − 3n
(i)_ C1 + C2 + C3 + ........ + Cn =
2 22 23 2n 2n
C1 C C Cn n(n + 1)
(ii) + 2 2 + 3 3 +........ + n =
C0 C1 C2 Cn − 1 2

A
C0 C1C2 ........Cn − 1(n + 1)n
(iii) (C0 + C1) (C1 + C2) (C2 + C3) (C3 + C4)........ (Cn − 1 + Cn) = .
n!

UH
(iv) Co − 2C1 + 3C2 − 4C3 +.... + (−1)n (n+1) Cn = 0
42 43 4n+1 5n+1 − 1
(v) 4C0 + . C1 + C2 + .............. + Cn =
2 3 n +1 n +1
22 .C0 23 .C1 24 .C2 2n + 2 .Cn 3n + 2 − 2n − 5
(vi) + + + ...... + =
1. 2 2 . 3 3 . 4
JA
(n + 1 ) (n + 2) (n + 1) (n + 2)

C-2. Prove that


22 .C1 23 .C2 24 .C3 2n + 1 .Cn 3n + 1 − 1
2.Co + + + + ...... =
2 3 4 n + 1 n + 1
LP

C-3. Prove that nCr + n–1Cr + n–2Cr + .............. + rCr = n+1Cr+1

C-4. If (1 + x)n = C0 + C1 x + C2 x2 + ......... + Cn xn, prove that


(2n)!
(i) C0 C3 + C1 C4 + ......... + Cn – 3 Cn =
(n + 3) ! (n − 3)!
A

(2n)!
(ii) C0 Cr + C1 Cr + 1 + .......... + Cn – r Cn =
(n + r) ! (n − r)!
NK

(iii) C0 – C1 + C2 – C3 + ........ + (–1) Cn2 = 0 or (–1)n/2 Cn/2 according as n is odd or even.


2 2 2 2 n

Section (D) : Negative & fractional index, Multinomial theorem

D-1. Find the co-efficient of x6 in the expansion of (1 − 2 x)−5/2.


SA

4 + 2x − x 2
D-2. (i) Find the coefficient of x12 in
(1 + x)3
3 − 5x
(ii) Find the coefficient of x100 in
(1 − x)2

D-3. Assuming ' x ' to be so small that x2 and higher powers of ' x ' can be neglected, show that,
−4
(1+ 34 x ) (16 − 3x )1/ 2
is approximately equal to, 1 −
305
x.
(8 + x)2 / 3 96

D-4. (i) Find the coefficient of a5 b4 c7 in the expansion of (bc + ca + ab)8.


(ii) Sum of coefficients of odd powers of x in expansion of (9x 2 + x – 8)6

D-5. Find the coefficient of x7 in (1 – 2x + x3)5 .

PART - II : ONLY ONE OPTION CORRECT TYPE


Section (A) : General Term & Coefficient of xk in (ax +b)n
2m +1
x y
A-1. The (m + 1)th term of  +  is:
y x

RI
(A) independent of x (B) a constant
(C) depends on the ratio x/y and m (D) none of these

A-2. The total number of distinct terms in the expansion of, (x + a) 100 + (x − a)100 after simplification is :

A
(A) 50 (B) 202 (C) 51 (D) none of these

183 + 73 + 3 . 18 . 7. 25

UH
A-3. The value of, 6
is :
3 + 6. 243. 2 + 15 . 81. 4 + 20. 27. 8 + 15. 9 . 16 + 6.3. 32 + 64
(A) 1 (B) 2 (C) 3 (D) none

15
 17 
A-4. In the expansion of ,  3 − +3 2 the 11th term is a :
JA
 4 
 
(A) positive integer (B) positive irrational number
(C) negative integer (D) negative irrational number.
n
 a  n
C
A-5. If the second term of the expansion a1/ 13 +  is 14a5/2, then the value of n 3 is:
 a−1  C2
LP

(A) 4 (B) 3 (C) 12 (D) 6

A-6. In the expansion of (71/3 + 111/9)6561, the number of terms free from radicals is:
(A) 730 (B) 729 (C) 725 (D) 750
A

A-7. The value of m, for which the coefficients of the (2m + 1) th and (4m + 5)th terms in the expansion of
(1 + x)10 are equal, is
(A) 3 (B) 1 (C) 5 (D) 8
8
NK

 1
A-8. The co-efficient of x in the expansion of (1 − 2 x3 + 3 x5)  1 +  is :
 x
(A) 56 (B) 65 (C) 154 (D) 62

A-9. Given that the term of the expansion (x1/3 − x−1/2)15 which does not contain x is 5 m, where m∈ N,then m=
SA

(A) 1100 (B) 1010 (C) 1001 (D) 1002


4 3
 1  1
A-10. The term independent of x in the expansion of  x −   x + is:
 x  x 
(A) − 3 (B) 0 (C) 1 (D) 3

Section (B) : Middle term, Remainder & Numerically/Algebrically Greatest terms


8
k 
B-1. If k ∈ R+ and the middle term of  + 2  is 1120, then value of k is:
 2 
(A) 3 (B) 2 (C) 1 (D) 4
B-2. The remainder when 22003 is divided by 17 is :
(A) 1 (B) 2 (C) 8 (D) 7

B-3. The last two digits of the number 3400 are:


(A) 81 (B) 43 (C) 29 (D) 01

B-4. The last three digits in 10 ! are :


(A) 800 (B) 700 (C) 500 (D) 600

10 n

∑r
Cr
B-5. The value of . n
is equal to
r =1 Cr −1
(A) 5 (2n – 9) (B) 10 n (C) 9 (n – 4) (D) n –2

RI
n −1 n


Cr
B-6. n
=
r =0 Cr + nCr + 1

A
n n +1 n n (n − 1)
(A) (B) (C) (n + 1) (D)
2 2 2 2 (n + 1)

UH
B-7. Find numerically greatest term in the expansion of (2 + 3 x)9, when x = 3/2.
(A) 9C6. 29. (3/2)12 (B) 9C3. 29. (3/2)6 (C) 9C5. 29. (3/2)10 (D) 9C4. 29. (3/2)8

B-8. The greatest integer less than or equal to ( 2 + 1)6 is


(A) 196 (B) 197 (C) 198 (D) 199
JA
Section (C) : Summation of series, Variable upper index & Product of binomial
coefficients
11 11 11 11
C0 C1 C2 C10
C-1. + + + ...... + =
1 2 3 11
LP

211 − 1 211 − 1 311 − 1 311 − 1


(A) (B) (C) (D)
11 6 11 6

C0 C C C Cn
– 1 + 2 – 3 +.......... + (–1)n
A

C-2. The value of is :


1.3 2.3 3.3 4.3 (n + 1) . 3
3 n +1 1
(A) (B) (C) (D) none of these
n +1 3 3(n + 1)
NK

5
C-3. The value of the expression 47C4 + ∑
j=1
52 − j
C3 is equal to :

(A) 47C5 (B) 52C5 (C) 52C4 (D) 49C4


SA

 50   50   50   50   50   50  n
C-4. The value of     +    +...........+     is, where Cr =  
n

0  1  1  2  49   50  r 
2
 100   100   50   50 
(A)   (B)   (C)   (D)  
 50   51   25   25 

Section (D) : Negative & fractional index, Multinomial theorem

D-1. If x < 1, then the co-efficient of xn in the expansion of (1 + x + x2 + x3 +.......)2 is


(A) n (B) n − 1 (C) n + 2 (D) n + 1
D-2. The co−efficient of x4 in the expansion of (1 − x + 2x2)12 is:
(A) 12C3 (B) 13C3 (C) 14C4 (D) 12C3+ 3 13C3 + 14C4

D-3. If (1 + x)10 = a0 + a1x + a2x2 +......+ a10x10, then value of


(a0 – a2 + a4 – a6 + a8 – a10)2 + (a1 – a3 + a5 – a7 + a9)2 is
(A) 210 (B) 2 (C) 220 (D) None of these

PART - III : MATCH THE COLUMN

1. Column – I Column – II

RI
(A) If (r + 1)th term is the first negative term in the expansion (p) divisible by 2
of (1 + x)7/2 , then the value of r (where 0 < x < 1) is

(B) If the sum of the co-efficients in the expansion of (q) divisible by 5

A
(1 + 2x)n is 6561, and Tr is the greatest term
in the expansion for x = 1/2 then r is

UH
(C) n
Cr is divisible by n, (1 < r < n) if n is (r) divisible by 10

(D) The coefficient of x4 in the expression (s) a prime number


(1 + 2x + 3x2 + 4x3 + ......up to ∞)1/2 is c, (c ∈ N),
then c + 1 (where | x | < 1) is
JA
LP
A
NK
SA
Marked questions are recommended for Revision.

PART - I : ONLY ONE OPTION CORRECT TYPE


1. In the expansion of
21
 a b 
 3 +3  , the term containing same powers of a & b is
 b a 
(A) 11 term
th
(B) 13th term (C) 12th term (D) 6th term

RI
2. Consider the following statements :
S1 : Number of dissimilar terms in the expansion of (1 + x + x 2 + x3)n is 3n + 1
S2 : (1 + x) (1 + x + x2) (1 + x + x2 + x3)...... (1 + x + x2 +...... + x100) when written in the ascending
power of x then the highest exponent of x is 5000.

A
n−r
S3 : ∑
k =1
n − kC
r = nCr+1

UH
3n − 1
S4 : If (1 + x + x2)n = a0 + a1x + a2 x2 + ...... + a2n x2n, then a0 + a2 + a4 + ....... + a2n =
2
State, in order, whether S1, S2, S3, S4 are true or false
(A) TFTF (B) TTTT (C) FFFF (D) FTFT
JA
n
Cr + 4 nCr +1 + 6 nCr + 2 + 4 nCr + 3 + nCr + 4 n+k
3. If n n n n
= then the value of k is :
Cr + 3 Cr +1 + 3 Cr + 2 + Cr + 3 r +k
(A) 1 (B) 2 (C) 4 (D) 5

4. The co-efficient of x5 in the expansion of (1 + x)21 + (1 + x)22 +....... + (1 + x)30 is :


(A) 51C5 (B) 9C5 (C) 31C6 − 21C6 (D) 30C5 + 20C5
LP

100
5. The coefficient of x52 in the expansion ∑
m =0
100
Cm (x – 3)100–m. 2m is :

(A) 100C47 (B) 100C48 (C) –100C52 (D) –100C100


A

40
6. The sum of the coefficients of all the integral powers of x in the expansion of 1 + 2 x ( ) is :
1 40 1 40
NK

(A) 340 + 1 (B) 340 – 1 (C) (3 – 1) (D) (3 + 1)


2 2
n

∑ (−1) r n (1 + rn10)
7. Cr . =
r =0 (1 + n10n )r
(A) 0 (B) 1/2 (C) 1 (D) None of these
SA

10
 
x −1
The coefficient of the term independent of x in the expansion of  2 
x +1
8. 1
− 1  is :
 3 
 x − x3 + 1 x − x2 
(A) 70 (B) 112 (C) 105 (D) 210

9. Coefficient of xn − 1 in the expansion of, (x + 3)n + (x + 3)n − 1 (x + 2) + (x + 3)n − 2 (x + 2)2 +..... + (x + 2)n
is :
(A) n+1C2(3) (B) n−1C2(5) (C) n+1C2(5) (D) nC2(5)

10. Let f(n) = 10n + 3.4n +2 + 5, n ∈ N. The greatest value of the integer which divides f(n) for all n is :
(A) 27 (B) 9 (C) 3 (D) None of these
n n
(101)100
∑ ∏
ar
11. If (1 + x)n = ar xr and br = 1 + and br = , then n equals to :
r =0
ar −1 r =1
100 !
(A) 99 (B) 100 (C) 101 (D) 102

6
12. Number of rational terms in the expansion of 1 + 2 + 5 ( ) is :
(A) 7 (B) 10 (C) 6 (D) 8

13. If S = 404 C4 − 4 C1.303 C4 + 4 C2 .202 C4 − 4 C3 .101C4 = (101)k then k equals to :

RI
(A) 1 (B) 2 (C) 4 (D) 6

2 2
14. 10 2 2
C0 −10 C1 +10 C2 − .... −
2
( 10
C9) +( 10
C10 ) =

A
2
(A) 0 (B) ( 10
C )
5 (C) –10C5 (D) 2 9C5

UH
n
15. The sum ∑
r =0
(r + 1) Cr2 is equal to :

(A)
(n + 2 ) ( 2n − 1) ! (B)
(n + 2 ) ( 2n + 1) !
(C)
(n + 2 ) ( 2n + 1) !
(D)
(n + 2 ) ( 2n − 1) !
n ! ( n − 1) ! n ! (n − 1) ! n ! (n + 1) ! n ! (n + 1) !
JA
16. If (1 + x + x2 + x3)5 = a0 + a1x + a2x2 +....................... + a 15x15, then a10 equals to :
(A) 99 (B) 101 (C) 100 (D) 110
n n
n − 2r
∑ ∑
1
17. If an = n
, the value of n
is :
r =0 Cr r =0 Cr
LP

n 1
(A) a (B) a (C) nan (D) 0
2 n 4 n

18. The sum of: 3.nC0 − 8.nC1 + 13.nC2 − 18.nC3 +.... upto (n+1) terms is (n ≥ 2):
(A) zero (B) 1 (C) 2 (D) none of these
A

n −1  3
 n


4Cr
19. If  n n
 = then n =
r =0  Cr + Cr +1 
5
NK

(A) 4 (B) 6 (C) 8 (D) None of these

n
 1 
20. The number of terms in the expansion of  x 2 + 1 + 2  , n ∈ N, is :
 x 
SA

(A) 2n (B) 3n (C) 2n + 1 (D) 3n + 1

PART - II : SINGLE AND DOUBLE VALUE INTEGER TYPE


1 1 1 1 2
1. If + + + ...... + = (2k –1 – 1) then find the value of k.
1!10! 2!9! 3!8! 10!1! k!

8
 1 
2. If the 6 term in the expansion of  8 / 3 + x 2 log10 x  is 5600, then x =
th

x 

3. The number of values of ' x ' for which the fourth term in the expansion,
8
 2 log 4x + 44 1 
 55 5 +  is 336, is :
 3 x −1 
 5log5 2 + 7 

4. If second, third and fourth terms in the expansion of (x + a) n are 240, 720 and 1080 respectively,
then n is equal to

5
P + Q
5. Let the co-efficients of xn in (1 + x)2n & (1 + x)2n − 1 be P & Q respectively, then   =
 Q 

n
 −x 5x 

RI
6. In the expansion of  3 4 + 3 4  , the sum of the binomial coefficients is 256 and four times the term
 
 
with greatest binomial coefficient exceeds the square of the third term by 21n, then find 4x.

A
19
( −2)k −λ
7. If ∑ k!(19 − k)! = 19!
k =1
then find λ.

UH
8. The value of p, for which coefficient of x50 in the expression
(1 + x)1000 + 2x (1 + x)999 + 3x² (1 + x)998 +..... + 1001 x1000 is equal to 1002Cp , is :

 31001 
9. If { x } denotes the fractional part of ' x ', then 82  =
JA
 82 

n
x 2
10. The index ' n ' of the binomial  +  if the only 9th term of the expansion has numerically the
5 5
greatest coefficient (n ∈ N), is :
LP

11. The number of values of ' r ' satisfying the equation, 39 C3r − 1 − 39Cr 2 = 39 Cr 2 − 1 − 39C3r is :

12. Find the value of


A

6
C0. 12C6.– 6C1 11C6+ 6C2 10C6 – 6C3 . 9C6 + 6C4 . 8C6 – 6C5 . 7C6 + 6C6 . 6C6

−1
 n  Ck  
2
k3
If n is a positive integer & Ck = nCk, find the value of  ∑ 
NK

13.   is :
  
n(n + 1) .(n + 2)  k − 1  
2 C
 k =1 

 10 10   10 10 
∑ ∑
C
14. The value of the expression  Cr   ( −1)K K K  is :
   2 
 r =0   K =0 
SA

100
15. The value of λ if ∑
m = 97
100
Cm . mC97 = 2λ . 100C97 , is :

16. If (1 + x + x² +... + xp)6 = a0 + a1x + a2x²+...+a6p x6p, then the value of :


1
[a1 + 2a2 + 3a3 +.... + 6p a6p] is :
p(p + 1)6

17. If (2nC1)²+ 2. (2nC2)² + 3. (2nC3)² +... + 2n. (2nC2n)² = 18 . 4n–1C2n–1, then n is :


n
(n + k ) .2n+1 − 1

2r + 3 n
18. If . Cr = then 'k' is
r =0
r +1 n +1
n
( −1)r .Cr
∑ (r + 1)(r + 2)(r + 3) =
1
19. If , then a + b is
r =0
a (n + b )

3n
20 ∑
k=1
6 nC
2k−1 (− 3)k is equal to :

x x y2
21. If x is very large as compare to y, then the value of k in = 1+
x+y x−y kx 2

RI
PART - III : ONE OR MORE THAN ONE OPTIONS CORRECT TYPE

A
20
 1 
1. In the expansion of  3 4 + 
4
 6

UH
(A) the number of irrational terms is 19 (B) middle term is irrational
(C) the number of rational terms is 2 (D) 9th term is rational

2
 1+ x 
2. The coefficient of x4 in   , | x | < 1, is
 1− x 
JA
(A) 4 (B) –4 (C) 10 + 4C2 (D) 16

3. 79 + 97 is divisible by :
(A) 16 (B) 24 (C) 64 (D) 72

n
4. The sum of the series ∑ (−1) r −1
. n
Cr (a − r) is equal to :
LP

r =1
(A) 5 if a = 5 (B) –5 if a = 5 (C) –5 if a = –5 (D) 5 if a = –5

1000n
5. Let an = for n ∈ N, then an is greatest, when
n!
A

(A) n = 997 (B) n = 998 (C) n = 999 (D) n = 1000

6. n
C0 – 2.3 nC1 + 3.32 nC2 – 4.33 nC3 +..........+ (–1)n (n +1) nCn 3n is equal to
NK

 3n   3
(A) 2n  + 1 if n is even (B) 2n  n +  if n is even
 2   2
 3n   3
(C) – 2n  + 1 if n is odd (D) 2n  n +  if n is odd
 2   2
SA

7. Element in set of values of r for which, 18Cr − 2 + 2. 18Cr − 1 + 18Cr ≥ 20C13 is :
(A) 9 (B) 5 (C) 7 (D) 10

8. The expansion of (3x + 2)–1/2 is valid in ascending powers of x, if x lies in the interval.
(A) (0, 2/3) (B) (–3/2, 3/2) (C) (–2/3, 2/3) (D) (–∞, –3/2) (3/2, ∞)

9. If (1 + 2x + 3x2)10 = a0 + a1x + a2x2 +.... + a20x20, then :


(A) a1 = 20 (B) a2 = 210 (C) a4 = 8085 (D) a20 = 22. 37. 7
10. In the expansion of (x + y + z)25
(A) every term is of the form 25Cr. rCk. x25 – r. yr – k. zk (B) the coefficient of x8 y9 z9 is 0
(C) the number of terms is 325 (D) none of these
11. If (1 + x + 2x2)20 = a0 + a1x + a2x2 +......... + a40x40, then a0 + a2 + a4.......+ a38 is equal to :
(A) 219 (230 + 1) (B) 219(220 – 1) (C) 239 – 219 (D) 239 + 219
2n
 n + 1
12. nn   is (n ∈ N)
 2 
3 3
 n + 1  n + 1
(A) Less than   (B) Greater than or equal to  
 2   2 
(C) Less than (n!)3 (D) Greater than or equal to (n!)3.

RI
13. If recursion polynomials Pk(x) are defined as P1(x) = (x – 2)2, P2 (x) = ((x – 2)2 – 2)2
P3 (x) = ((x – 2)2 – 2)2 – 2)2 .......... (In general Pk (x) = (Pk – 1 (x) – 2)2, then the constant term in
Pk (x) is

A
(A) 4 (B) 2 (C) 16 (D) a perfect square

PART - IV : COMPREHENSION

UH
Comprehension # 1 (Q. No. 1 to 3)
Consider, sum of the series ∑∑
0≤i< j≤n
f(i) f(j)

In the given summation, i and j are not independent.


JA
n n n   n 
In the sum of series f(i) f( j) =
 ∑∑
 f(i) 
 j=1  ∑ ∑
f( j)   i and j are independent. In this summation,
i=1 j=1 i=1   
three types of terms occur, those when i < j, i > j and i = j.
Also, sum of terms when i < j is equal to the sum of the terms when i > j if f(i) and f(j) are symmetrical.
So, in that case
LP

n n

∑∑ f(i)f( j) = ∑∑ f(i)f( j)
i=1 j=1 0 ≤i< j≤n

+ ∑∑ f(i)f( j) + ∑∑ f(i)f( j)
0 ≤i< j≤n i= j

∑∑ f(i)f( j) + ∑∑ f(i)f( j)
A

=2
0 ≤i< j≤n i= j
n n

∑∑ f(i)f( j) – ∑∑ f(i)f( j)
NK

∑∑ f(i)f( j) =
i=0 j=0 i= j

0 ≤i< j≤n
2
When f(i) and f(j) are not symmetrical, we find the sum by listing all the terms.

∑∑ n n
SA

1. Ci C j is equal to
0 ≤i< j≤n

22n – 2n
Cn 22n + 2n
Cn 22n – n Cn 22n + n Cn
(A) (B) (C) (D)
2 2 2 2

n m
2. Let 0C0 = 1, then ∑ ∑
m=0 p=0
n
Cm . m
Cp is equal to

(A) 2n–1 (B) 3n (C) 3n–1 (D) 2n

3. ∑∑ ( n
Ci + n
Cj )
0 ≤i≤ j≤n
(A) (n + 2)2n (B) (n + 1)2n (C) (n – 1)2n (D) (n + 1)2n–1
Comprehension # 2 (Q. No. 4 to 6)

Let P be a product given by P = (x + a1) (x + a2) ......... (x + an)


n
and Let S1 = a1 + a2 + ....... + an = ∑a
i=1
i , S2 = ∑∑ a .a , S = ∑ ∑ ∑ a .a .a
i< j
i j 3
i< j<k
i j k and so on,

then it can be shown that


P = xn + S1 xn – 1 + S2 xn – 2 + ......... + Sn.

4. The coefficient of x8 in the expression (2 + x)2 (3 + x)3 (4 + x)4 must be


(A) 26 (B) 27 (C) 28 (D) 29

RI
5. The coefficient of x203 in the expression (x – 1) (x2 – 2) (x3 – 3) .......... (x20 – 20) must be
(A) 11 (B) 12 (C) 13 (D) 15

6. The coefficient of x98 in the expression of (x – 1) (x – 2) ......... (x – 100) must be

A
(A) 12 + 22 + 32 + ....... + 1002
(B) (1 + 2 + 3 + ....... + 100) 2 – (12 + 22 + 32 + ....... + 1002)
1
(C) [(1 + 2 + 3 + ....... + 100)2 – (12 + 22 + 32 + ....... + 1002)]

UH
2
(D) None of these

Comprehension # 3 (Q.No. 7 to 9)
Let (7 + 4 3 )n = Ι + f = nC0.7n + nC1.7n – 1 .(4 3 )1 + ....... .............(i)
where Ι & f are its integral and fractional parts respectively.
JA
It means 0 < f < 1
Now, 0 < 7 – 4 3 < 1 ⇒ 0 < (7 – 4 3 )n < 1
Let (7 – 4 3 )n = f′ = nC0.7n – nC1.7n – 1 .(4 3 )1 + ....... .............(ii)
⇒ 0 < f′ < 1
Adding (i) and (ii) (so that irrational terms cancelled out)
Ι + f + f′ = (7 + 4 3 )n + (7 – 4 3 )n
LP

= 2 [nC0 7n + nC2 7n – 2 (4 3 )2 + ..........]


Ι + f + f′ = even integer ⇒ (f + f′ must be an integer)
0 < f + f′ < 2 ⇒ f + f′ = 1
with help of above analysis answer the following questions
A

n
7. ( )
If 3 3 + 5 = p + f, where p is an integer and f is a proper fraction, then find the value of
n
NK

(3 )
3 − 5 , n ∈ N, is
(A) 1 – f, if n is even (B) f, if n is even (C) 1 – f, if n is odd (D) f, if n is odd

n
8. (
If 9 + 80 ) = Ι + f, where Ι, n are integers and 0 < f < 1, then :
SA

n
(A) Ι is an odd integer (B) Ι is an even integer (C) (Ι + f) (1 – f) = 1 (D) 1 − f = 9 − 80 ( )
9. The integer just above ( 3 + 1)2n is, for all n ∈ N.
(A) divisible by 2n (B) divisible by 2n + 1 (C) divisible by 8 (D) divisible by 16
 Marked questions are recommended for Revision.

PART - I : JEE (ADVANCED) / IIT-JEE PROBLEMS (PREVIOUS YEARS)


* Marked Questions may have more than one correct option.
1. Coefficient of t24 in (1 + t2)12 (1 + t12) (1 + t24) is: [IIT-JEE-2003, Scr, (3, – 1), 84]
(A) 12C6 + 3 (B) 12C6 + 1 (C) 12C6 (D) 12C6 + 2

n n  n   n − 1 k −2  n   n − 2  n n − k  n

RI
2. Prove that 2k     – 2k −1     + 2   
k
 –...... + (– 1)     =  .
0 k   1   k − 1  2 k − 2 k   0  k 
[IIT-JEE-2003, Main, (2, 0), 60]

A
3. If (n – 1)Cr = (k2 – 3) nCr+1, then an interval in which k lies is [IIT-JEE-2004, Scr, (3, – 1), 84]
(A) (2, ∞) (B) (– ∞, – 2) (C)  − 3, 3  (D) ( 3,2
  

UH
4. The value of [IIT-JEE-2005, Scr, (3, – 1), 84]
 30   30   30   30   30   30   30   30 
    –     +     – .......... +     is :
 0   10   1   11   2   12   20   30 
 
30  30 
( )
(A) 60 (B)   (C)   (D) None of these
20
JA
 10   15 

5. For r = 0, 1, ...., 10, let Ar, Br and Cr denote, respectively, the coefficient of xr in the expansions of
10
(1 + x)10 , (1 + x)20 and (1 + x)30 . Then ∑ A (B
r =1
r 10Br − C10 A r ) is equal to
LP

[IIT-JEE 2010, Paper-2, (5, –2)/79]


(A) B10 – C10 (B) A10 (B210 – C10 A10) (C) 0 (D) C10 – B10

6. The coefficients of three consecutive terms of (1 + x)n+5 are in the ratio 5 : 10 : 14. Then n =
A

[JEE (Advanced) 2013, Paper-1, (4, – 1)/60]

7. Coefficient of x11 in the expansion of (1 + x2)4 (1 + x3)7 (1 + x4)12 is


[JEE (Advanced) 2014, Paper-2, (3, –1)/60]
NK

(A) 1051 (B) 1106 (C) 1113 (D) 1120

8. The coefficient of x9 in the expansion of (1 + x) (1 + x2) (1 + x3) ......(1 + x100) is


[JEE (Advanced) 2015, P-2 (4, 0) / 80]

9. Let m be the smallest positive integer such that the coefficient of x 2 in the expansion of
SA

(1 + x)2 + (1 + x)3 +........+ (1 + x)49 + (1 + mx)50 is (3n + 1) 51C3 for some positive integer n. Then the
value of n is [JEE (Advanced) 2016, Paper-1, (3, 0)/62]

10. Let X = (10C1)2 + 2(10C2)2 + 3(10C3)2 + ....... + 10(10C10)2 where 10Cr, r ∈ {1, 2, ......., 10} denote binomial
1
coefficients. Then the value of X is _______ . [JEE (Advanced) 2018, Paper-1, (3, 0)/60]
1430
PART - II : JEE (MAIN) / AIEEE PROBLEMS (PREVIOUS YEARS)
10 10 10
1. Let S1 = ∑ j ( j – 1) 10
Cj , S2 = ∑j 10
Cj and S3 = ∑j 2 10
Cj. [AIEEE 2009, (4, –1), 144]
j =1 j =1 j =1

Statement -1 : S3 = 55 × 29 .
Statement -2 : S1 = 90 × 28 and S2 = 10 × 28.
(1) Statement-1 is true, Statement-2 is true ; Statement -2 is not a correct explanation for Statement -1.
(2) Statement-1 is true, Statement-2 is false.
(3) Statement -1 is false, Statement -2 is true.
(4) Statement -1 is true, Statement -2 is true; Statement-2 is a correct explanation for Statement-1.

RI
2. The coefficient of x7 in the expansion of (1 – x – x2 + x3)6 is : [AIEEE 2011, (4, –1), 120]
(1) 144 (2) – 132 (3) – 144 (4) 132

A
2n 2n
3. If n is a positive integer, then ( 3 +1 ) – ( )
3 −1 is : [AIEEE 2012, (4, –1), 120]

(1) an irrational number (2) an odd positive integer

UH
(3) an even positive integer (4) a rational number other than positive integers

10
 x +1 x −1 
4. The term independent of x in expansion of  2 / 3 −  is :[AIEEE - 2013, (4, –1),120]
x − x + 1 x − x1/ 2 
1/ 3
JA
(1) 4 (2) 120 (3) 210 (4) 310

5. If the coefficients of x3 and x4 in the expansion of (1 + ax + bx2) (1 – 2x)18 in powers of x are both zero,
then (a, b) is equal to [JEE(Main) 2014, (4, – 1), 120]
 272   272   251   251 
(1)  14, (2)  16, (3)  16, (4)  14,
 3   3   3   3 
LP

6. The sum of coefficients of integral powers of x in the binomial expansion of (1 – 2 x )50 is
[JEE(Main) 2015, (4, – 1), 120]
1 50 1 50 1 50 1 50
A

(1) (3 + 1) (2) (3 ) (3) (3 – 1) (4) (2 + 1)


2 2 2 2

n
If the number of terms in the expansion of  1– 2 + 4  , x ≠ 0, is 28, then the sum of the coefficients of
NK

7.
 x x2 
all the terms in this expansion, is [JEE(Main) 2016, (4, – 1), 120]
(1) 2187 (2) 243 (3) 729 (4) 64
SA

8. The value of (21C1 – 10C1) + (21C2 – 10C2) + (21C3 – 10C3) + (21C4 – 10C4) +........+ (21C10 – 10C10) is
[JEE(Main) 2017, (4, – 1), 120]
21
(1) 2 – 2 11 21
(2) 2 – 2 10 20
(3) 2 – 2 9 (4) 220 – 210

5 5
9. The sum of the co-efficients of all odd degree terms in the expansion of  x + x 3 − 1  +  x − x 3 − 1  ,
   
(x > 1) is : [JEE(Main) 2018, (4, – 1), 120]
(1) 1 (2) 2 (3) –1 (4) 0
2 403 k
10. If the fractional part of the number is , then k is equal to :
15 15
[JEE(Main) 2019, Online (09-01-19),P-1 (4, – 1), 120]
(1) 14 (2) 8 (3) 6 (4) 4

3

20 20
Ci−1  k
11. If ∑  20 20  = , then k equals : [JEE(Main) 2019, Online (10-01-19),P-1 (4, – 1), 120]
i=1  Ci + Ci–1  21
(1) 50 (2) 400 (3) 200 (4) 100

25
12. If ∑{ 50
Cr .50 −r C25 −r = K
} ( 50
)
C25 , then K is equal to :

RI
r =0

[JEE(Main) 2019, Online (10-01-19),P-2 (4, – 1), 120]


(1) 225 (2) 225 –1 (3) (25)2 (4) 224

A
2 n
 q + 1  q + 1  q + 1
13. Let Sn = 1 + q + q2 + …. + qn and Tn = 1 +   +   + …. +   .
 2   2   2 

UH
where q is a real number and q ≠ 1 . If 101C1 + 101C2 . S1 + …..+ 101C101 . S100 = αT100 then α is equal to
[JEE(Main) 2019, Online (11-01-19),P-2 (4, – 1), 120]
(1) 200 (2) 299 JA (3) 2100 (4) 202
LP
A
NK
SA
EXERCISE - 1
PART - I

Section (A) :

5 3 3 5
2 2 2 x x x 32 16
A-1. (i)   – 5   + 10   – 10   + 5   –   (ii) y8 + 8y5 + 24y2 + + 4
 
x  
x  
x  
2  
2 2 y y

RI
A-2. n=9 A-3. 7 A-4. (i) 9C3 (ii) – 27 . 12C7

11C a6 a5 17
A-5. 5 5
, 11C6 6
, ab = 1 A-6. A-7. (i) 171 (ii) –438
b b 54

A
A-8 15

UH
Section (B) :

35x 35y (2n)! n


B-1. (i) – , (ii) (–1)n x B-3. (i) 4 (iii) 3, 03, 803
y x n! n!
JA
B-4. 10150 B-5. (i) T4 = – 455 × 312 and T5 = 455 × 312 (ii) 22

B-6. (i) T4 (ii) T5, T6 (iii) T5 (iv) T6

Section (D) :
15015
D-1. D-2. (i) 142 (ii) – 197 D-4. (i) 280 (ii) 25 D-5. 20
16
LP

PART - II
Section (A) :
A-1. (C) A-2. (C) A-3. (A) A-4. (B) A-5. (A) A-6. (A)
A

A-7. (B) A-8. (C) A-9. (C) A-10. (B)

Section (B) :
NK

B-1. (B) B-2. (C) B-3. (D) B-4. (A) B-5. (A) B-6. (A)
B-7. (A) B-8. (B)
Section (C) :
SA

C-1. (B) C-2. (C) C-3. (C) C-4. (B)


Section (D) :
D-1. (D) D-2. (D) D-3. (A)
PART - III
1. (A) → (q, s), (B) → (q,s), (C) → (s), (D) → (p, s)
EXERCISE - 2
PART - I
1. (B) 2. (A) 3. (C) 4. (C) 5. (B) 6. (D) 7. (A)
8. (D) 9. (C) 10. (B) 11. (B) 12. (B) 13. (C) 14. (C)
15. (A) 16. (B) 17. (D) 18. (A) 19. (A) 20. (C)

PART - II
1. k = 11 2. 10 3. 2 4. 5 5. 35 6. 2 7. 2
8. 50 9. 3 10. n = 12 11. 2 12. 1 13. 12 14. 1

RI
15. 3 16. 3 17. 9 18. 2 19. 5 20. 0 21. 2

PART - III

A
1. (ABCD) 2. (CD) 3. (AC) 4. (AC) 5. (CD) 6. (AC)
7. (ACD) 8. (AC) 9. (ABC) 10. (AB) 11. (BC) 12. (BD)

UH
13. (AD)
PART - IV

1. (A) 2. (B) 3. (A) 4. (D) 5. (C) 6. (C) 7. (AD)


JA
8. (ACD) 9. (ABC)

EXERCISE - 3

PART - I
LP

1. (D) 3. (D) 4. (B) 5. (D) 6. 6 7. (C) 8. 8


9. 5 10. 646
A

PART - II
NK

1. (2) 2. (3) 3. (1) 4. (3) 5. (2) 6. 1


7. (3) or Bonus 8. (4) 9. (2) 10. (2) 11. (4) 2. (1)
13. (3)
SA
1. Find the coefficient of x49 in
 C1   2 C2   2 C3   2 C50 
x +  x + 2  x +3  ........  x + 50 
 C 0   C 1   C 2   C49 

6
6  2 
2. The expression, ( 2x 2 + 1 + 2x 2 − 1 ) +

 2x 2
+ 1 + 2x 2
− 1
 is a polynomial of degree

3. Find the co-efficient of x5 in the expansion of (1 + x2)5 (1 + x)4.

RI
5
4. Prove that the co-efficient of x15 in (1 + x +x3 + x4)n is ∑
r =0
n n
C15 −3r Cr .

A
5. If n is even natural and coefficient of xr in the expansion of
(1 + x )n is 2n, (|x| < 1), then prove that r ≥ n
1− x

UH
6. Find the coefficient of xn in polynomial (x + 2n+1C0) (x + 2n+1C1)........(x + 2n+1Cn).

n  r −1 
7. Find the value of ∑  ∑ n
Cr Cp 2p  .
r

 p =0 
r =1
JA
Comprehension (Q-8 to Q.10)
For k, n ∈ N, we define
B(k, n) = 1.2.3......... k + 2.3.4........(k+1) + .........+ n(n + 1).......(n + k – 1), S 0(n) = n and Sk(n)
= 1k + 2k + ......... + nk.
To obtain value B(k, n), we rewrite B(k, n) as follows
LP

B(k,n) = k!  k Ck + k +1Ck + k + 2Ck + ......... + n+k −1Ck  = k! n+k Ck +1 ( )


 
n ( n + 1) ........ (n + k )
=
k +1
A

n n!
where Ck =
k! (n − k)!
NK

8. Prove that S2(n) + S1(n) = B(2, n)

9. Prove that S3(n) + 3S2(n) = B(3, n) – 2B(1, n)

10. If (1 + x)p = 1 + pC1 x + pC2x2 +..........+ pCp xp, p ∈ N , then show that k+1
C1 Sk(n) + k+1
C2 Sk–1(n) +.........+
SA

k+1
Ck S1(n) + k+1Ck+1 S0(n) = (n + 1)k+1 – 1

11. Show that 25n – 20n – 8n + 3n, n ∈ I+ is divisible by 85.


n +1
C2
 2n 
Prove that C1 ( C2) ( C3) ........( Cn) ≤ 
 n + 1 
12. n n 2 n 3 n n
.
 
1/ n
(n + 1) p + (n − 1)q  p 
13. If p is nearly equal to q and n > 1, show that =  . Hence find the approximate
(n − 1)p + (n + 1)q  q 
1/ 6
 99 
value of   .
 101 
14. If (18x2 + 12x + 4)n = a0 + a1x + a2x2 + .... + a2n x2n , then prove that
2n − 2 2n − 4
ar = 2n 3r ( 2n n
Cr + C1 Cr + n
C2 Cr + ... )
15. Prove that 1². C0 + 2². C1 + 3². C2 + 4². C3 +.... + (n+1)² Cn = 2n−2 (n+1) (n+4).

16. If (1 − x)−n = a0 + a1 x + a2 x2 + a3 x3 +........, find the value of, a0 + a1 + a2 +....... + an.

32
17. Find the remainder when 3232 is divided by 7.

18. If n is an integer greater than 1, show that : a − nC1(a−1) + nC2(a−2) −..... + (−1)n (a − n) = 0.

RI
19. If (1 + x)n = p0 + p1 x + p2 x2 + p3 x3 +......., then prove that :
nπ nπ
(a) p0 − p2 + p4 −....... = 2n/2 cos (b) p1 − p3 + p5 −....... = 2n/2 sin

A
4 4

20. Show that if the greatest term in the expansion of (1 + x) 2n has also the greatest co-efficient, then ' x '
n n +1

UH
lies between, & .
n +1 n

21. Prove that if ' p ' is a prime number greater than 2, then (2 + 5 )p  − 2p+1 is divisible by p, where [. ]
 
denotes greatest integer function.
JA
n
1 3r 7r   1 
22. If ∑
r =0
(− 1)r. nCr  r + 2 r + 3 r + ........
 2 2 2
to m terms  = k  1 − m n  , then find the value of k.
  2 

2 n
q + 1  q + 1  q + 1
qn  , q ≠ 1,
LP

23. Given sn= 1 + q + q² +..... + & Sn = 1 + +   +.... + 


2  2   2 
prove that n+1C1 + n+1C2.s1 + n+1C3.s2 +....+ n+1Cn+1.sn = 2n. Sn.

24. If (1+x)15 = C0 + C1. x + C2. x2 +.... + C15. x15, then find the value of : C2 + 2C3 + 3C4 +.... + 14C15
A

1 n 2 3 4 ( − 1)n + 1 n n 1
25. Prove that, C1− nC2+ nC3− nC4 +..... + . Cn=
2 3 4 5 n + 1 n + 1
NK

n
26. Prove that ∑r
r =0
2 n
C r pr qn – r = npq + n2p2, if p + q = 1.
SA

27. Prove that : (n−1)². C1 + (n−3)². C3 + (n−5)². C5 +..... = n (n + 1)2n−3

28. Prove that nCr + 2 n +1C


r +3 n+2C
r +....... + (n + 1) 2nC
r = nCr+2 + (n + 1) 2n + 1C
r+1 – 2n+1C
r+2

1 1 3 1 3 5 1 3 5 7
29. Show that, 3=1+ + . + . . + . . . + ......
3 3 6 3 6 9 3 6 9 12

30. If (1 + x)n = C0 + C1x + C2x2 +...... + Cnxn, show that for m ≥ 2


C0 – C1 + C2 –......... + (– 1)m – 1Cm – 1 = (–1)m – 1 n – 1Cm – 1.
31. If (1+x)n = C0 + C1x + C2x² +.... + Cn xn, then show that the sum of the products of the Ci’s taken two at
∑ ∑ Ci C j 2n !
a time, represented by is equal to 22n−1 − .
0≤i< j≤n 2 (n !)2

32. If a0, a1, a2,..... be the coefficients in the expansion of (1 + x + x²) n in ascending powers of x, then prove
that :
(i) a0 a1 − a1 a2 + a2 a3 −.... = 0
(ii) a0a2 − a1a3 + a2a4 −..... + a2n − 2 a2n = an + 1
(iii) E1 = E2 = E3 = 3n−1; where E1= a0 + a3 + a6 +...; E2 = a1 + a4 + a7 +...& E3 = a2 + a5 + a8 +...

RI
1198

A
1. 22100 2. 6 3. 60 6. 22n 7. 4n – 3n 13.
1202

(2n)! 1

UH
16. 17. 4 22. 24. 212993
(n!)2 2n − 1

JA
LP
A
NK
SA
CIRCLE

Four circles to the kissing come, The smaller are the benter. The bend is just the inverse of The distance from the centre. Through their intrigue left
Euclid dumb There’s now no need for rule of thumb. Since zero bend’s a dead straight line And concave bends have minus sign, The sum of squares of
all four bends Is half the square of their sum. ....... Soddy, Frederick

A circle is a locus of a point in a plane whose distance from a fixed point (called centre) is always constant
(called radius).
Equation of a circle in various forms :
(a) The circle with centre as origin & radius ‘r’ has the equation; x 2 + y2 = r2.

RI
A
(b) The circle with centre (h, k) & radius ‘r’ has the equation; (x − h) 2 + (y − k)2 = r2.

(c) The general equation of a circle is x2 + y2 + 2gx + 2fy + c = 0

UH
with centre as (−g, −f) & radius = g2 + f 2 − c .
This can be obtained from the equation (x – h)2 + (y – k)2 = r2
⇒ x2 + y2 – 2hx – 2ky + h2 + k2 – r2 = 0
Take – h = g, – k = f and h2 + k2 – r2 = c
Condition to define circle :-
JA
g² + f² − c > 0 ⇒ real circle.
g² + f² − c = 0 ⇒ point circle.
g² + f² − c < 0 ⇒ imaginary circle, with real centre, that is (– g, – f)

Note : That every second degree equation in x & y, in which coefficient of x 2 is equal to coefficient of y2
& the coefficient of xy is zero, always represents a circle.
LP

(d) The equation of circle with (x1, y1) & (x2, y2) as extremeties of its diameter is:
(x − x1) (x − x2) + (y − y1) (y − y2) = 0.
A

This is obtained by the fact that angle in a semicircle is a right angle.


NK

∴ (Slope of PA) (Slope of PB) = – 1


y − y1 y − y 2
⇒ . = – 1 ⇒ (x – x1) (x – x2) + (y – y1) (y – y2) = 0
x − x1 x − x 2
Note that this will be the circle of least radius passing through (x 1, y1) & (x2, y2).
SA

Example # 1 Find the equation of the circle whose centre is (0, 3) and radius is 3.
Solution. The equation of the circle is (x – 0)2 + (y – 3)2 = 32
⇒ x2 + y2 – 6y = 0

Example # 2 Find the equation of the circle which passes through(1, –1) and two of its dimeter are
x + 2y – 5 = 0 and x – y + 1 = 0
Solution. Let P be the point of intersection of the lines
x + 2y – 5 = 0 ..........(i)
and x–y+1=0 ..........(ii)
Solving (i) and (ii), we get x = 1, y = 2. So, coordinates of centre are (1, 2). since circle passes
through (1,–1) so
radius = (1 − 1)2 + (2 + 1)2 ⇒ radius = 3
Hence the equation of the required circle is (x – 1)2 + (y + 2)2 = 9
Example # 3 If the equation ax2 + (b – 3)xy + 3y2 + 6ax + 2by – 3 = 0 represents the equation of a circle then
find a, b
Solution. ax2 + (b – 3)xy + 3y2 + 6ax + 2by – 3 = 0
above equation will represent a circle if
coefficient of x2 = coefficient of y2
a=3
coefficient of xy = 0
b=3

Example # 4 Find the equation of a circle whose diametric end points are (x 1, y1) and (x2, y2) where x1, x2 are
the roots of x2 – ax + b = 0 and y1, y2 are the roots of y2 – by + a = 0.

RI
Solution. We know that the equation of the circle described on the line segment joining (x 1, y1) and
(x2, y2) as a diameter is (x – x1) (x – x2) + (y – y1) (y – y2) = 0.
x2 + y2 – (x1+ x2)x – (y1 + y2)y + x1x2 + y1 y2 = 0
Here, x1 + x2 = a , x1x2 = b

A
y1 + y2 = b , y1y2 = a
So, the equation of the required circle is

UH
x2 + y2 – ax – by + a + b = 0
Self practice problems :
(1) Find the equation of the circle passing through the point of intersection of the lines
x + 3y = 0and 2x – 7y = 0 and whose centre is the point of intersection of the lines x + y + 1 = 0
and x – 2y + 4 = 0.
JA
(2) Find the equation of the circle whose centre is (1, 2) and which passes through the point (4, 6)
(3) Find the equation of a circle whose radius is 6 and the centre is at the origin.

Answers :
(1) x2 + y2 + 4x – 2y = 0 (2) x2 + y2 – 2x – 4y – 20 = 0 (3) x2 + y2 = 36.
LP

Intercepts made by a circle on the axes:


The intercepts made by the circle x2 + y2 + 2gx + 2fy + c = 0 on the co−ordinate axes
are 2 g2 − c (on x-axis) & 2 f 2 − c (on y-axis) respectively.
A

If g2 > c ⇒ circle cuts the x axis at two distinct points.

g2 = c ⇒ circle touches the x−axis.


NK

g2 < c ⇒ circle lies completely above or below the x−axis.


SA

AB = 2AD = 2 r 2 − CD2 = 2 r 2 − f 2 = 2 g2 + f 2 − c − f 2 = 2 g2 − c

Example # 5 Find the locus of the centre of the circle whose x and y intercepts are a and b respectively.
Solution. Equation of circle is x2 + y2 + 2gx + 2fy + c = 0
x intercept = a
a2
2 g2 – c = a g2 – c = ...... (i)
4
y intercept = b
b2
2 f2 – c = b f2 – c = ...... (ii)
4
subtracting equation (i) and (ii)
a2 – b2
g 2 – f2 =
4
a2 – b2
hence locus of centre is x2 – y2 =
4
Self practice problems :
(4) Find the equation of a circle which touches the positive axis of y at a distance 3 from the origin
and intercepts a distance 6 on the axis of x.
(5) Find the equation of a circle which touches positive y-axis at a distance of 2 units from the
origin and cuts an intercept of 3 units with the positive direction of x-axis.
Answers : (4) x2 + y2 ± 6 2 x – 6y + 9 = 0 (5) x2 + y2 – 5x – 4y + 4 = 0

Parametric equations of a circle:

RI
The parametric equations of (x − h)2 + (y − k)2 = r2 are: x = h + r cos θ ; y = k + r sin θ ; − π < θ ≤ π
where (h, k) is the centre, r is the radius & θ is a parameter.

A
UH
Example # 6 Find the parametric equations of the circle x 2 + y2 + 4x + 6y + 9 = 0
Solution. We have : x2 + y2 + 4x + 6y + 9 = 0
JA
⇒ (x + 2)2 + (y + 3)2 = 22
So, the parametric equations of this circle are
x = –2 + 2 cos θ , y = –3 + 2 sin θ.

Example # 7 Find the equation of the following curve in cartesian form


x+ y = cosθ , x – y = sinθ where θ is the parameter.
LP

Solution. We have : x+ y = cosθ ....... (i)


x – y = sinθ ....... (ii)
(i)2 + (ii)2
⇒ (x + y)2 + (x – y)2 = 1
1
x2 + y2 =
A

2
1
Clearly, it is a circle with centre at (0, 0) and radius .
2
NK

Self practice problems :


(6) Find the parametric equations of circle x2 + y2 – 6x + 4y – 12 = 0
(7) Find the cartesian equations of the curve x = 1 + 2 cos θ, y = 2 – 2 sin θ
Answers : (6) x = 3 + 5 cos θ, y = –2 + 5 sin θ (7) (x – 1) 2 + (y – 2)2 = 2
SA

Position of a point with respect to a circle:


The point (x1, y1) is inside, on or outside the circle S ≡ x2 + y2 + 2gx + 2fy + c = 0.
according as S1 ≡ x1² + y1² + 2gx1 + 2fy1 + c <, = or > 0.

Note : The greatest & the least distance of a point A (lies outside the circle) from a circle with centre
C & radius r is AC + r & AC − r respectively.

Example # 8 Check whether the point (1, 2) lies in smaller or larger region made by circle
x2 + y2 – 4x + 2y – 11 = 0 and the line x + y = 0
Solution : We have x2 + y2 – 4x + 2y – 11 = 0 or S = 0,
x+y =0


(2,–1)

where S = x2 + y2 – 4x + 2y – 11.
For the point (1, 2), we have S1 = 12 + 22 – 4 × 1 +2 × 2 – 11 < 0
Hence, the point (1, 2) lies inside the circle
Points (1,2) and (2, –1) lie on same side of the line x + y = 0
Hence the point (1,2) lies in the larger region.

RI
Self practice problem :

(8) How are the points (0, 1) (3, 1) and (1, 3) situated with respect to the circle
x2 + y2 – 2x – 4y + 3 = 0?

A
Answer : (8) (0, 1) lies on the circle ; (3, 1) lies outside the circle ; (1, 3) lies inside the circle.

Line and a circle:

UH
Let L = 0 be a line & S = 0 be a circle. If r is the radius of the circle & p is the length of the perpendicular
from the centre on the line, then:
(i) p>r ⇔ the line does not meet the circle i. e. passes out side the circle.
(ii) p=r ⇔ the line touches the circle. (It is tangent to the circle)

(iii) p<r
JA
the line is a secant of the circle.
(iv) p=0 ⇒ the line is a diameter of the circle.

Also, if y = mx + c is line and x2 + y2 = a2 is circle then


(i) c2 < a2 (1 + m2) ⇔ the line is a secant of the circle.
(ii) c = a (1 + m ) ⇔
2 2 2
the line touches the circle. (It is tangent to the circle)
(iii) c2 > a2 (1 + m2) ⇔ the line does not meet the circle i. e. passes out side the
LP

circle.
A

These conditions can also be obtained by solving y = mx + c with x 2 + y2 = a2 and making the
discriminant of the quadratic greater than zero for secant, equal to zero for tangent and less the zero for
NK

the last case.

Example # 9 For what value of λ, does the line x + y = λ touch the circle x 2 + y2 – 2x – 2y = 0
Solution. We have : x + y = λ ......(i) and x 2 + y2 – 2x – 2y = 0 ....... (ii)
If the line (i) touches the circle (ii), then
SA

length of the ⊥ from the centre (1, 1) = radius of circle (ii)


1+ 1− λ
⇒ = 2 ⇒ 2–λ = 2 ⇒ λ = 0 or 4
12 + 12
Hence, the line (i) touches the circle (ii) for λ = 0 or 4

Self practice problem :


(9) Find the range of values of m for which the line y = mx + 2 cuts the circle x 2 + y2 = 1 at distinct
points
Answers : (9) m∈(–∞, – 3 ) ∪ ( 3 ,∞)
Slope form of tangent :
y = mx + c is always a tangent to the circle x2 + y2 = a2 if c2 = a2 (1 + m2). Hence, of tangent is
 a 2m a 2 
y = mx ± a 1 + m2 and the point of contact is  − ,  .
 c c
 
Point form of tangent :
(i) The equation of the tangent to the circle x2 + y2 = a2 at its point (x1, y1) is, x x1 + y y1 = a².
(ii) The equation of the tangent to the circle x2 + y2 + 2gx + 2fy + c = 0 at its point
(x1, y1) is : xx1 + yy1 + g (x+x1) + f (y+y1) + c = 0.

Note : In general the equation of tangent to any second degree curve at point (x 1, y1) on it can be obtained by
x + x1 y + y1
replacing x2 by x x1, y2 by yy1, x by , y by ,

RI
2 2
x y + x y1
xy by 1 and c remains as c.
2

A
Parametric form of tangent :
The equation of a tangent to circle x2 + y2 = a2 at (a cos α, a sinα)is x cosα + ysinα=a.
 a cos α + β a sin α + β 

UH
NOTE : The point of intersection of the tangents at the points P(α) & Q(β) is  2 , 2 
 cos α − β α−β
cos 2 
 2 

Example # 10 Find the equation of the tangent to the circle x 2 + y2 – 2x – 2y – 11 = 0 at (3, 4).
Solution. Equation of tangent is
JA
x+3 y+4
3x + 4y – 2   – 2  – 11 = 0
 2   2 
or 2x + 3y – 18 = 0
Hence, the required equation of the tangent is 2x + 3y – 18 = 0

Example # 11 Find the equation of tangents to the circle x2 + y2 – 4x + 2y = 0 which are perpendicular to the
line x + 2y + 4 = 0
LP

Solution. Given circle is x2 + y2 – 4x + 2y = 0 .......(i)


and given line is x + 2y + 4 = 0 .......(ii)
Centre of circle (i) is (2,–1)) and its radius 5 is Equation of any line
2x – y + k = 0 perpendicular to the line (ii) .......(iii)
A

If line (iii) is tangent to circle (i) then


| 4 + 1+ k |
= 5 or |k + 5| = 5 or k = 0, – 10
5
NK

Hence equation of required tangents are 2x – y = 0 and 2x – y – 10 = 0

Self practice problem :

(10) Find the equation of the tangents to the circle x 2 + y2 – 2x – 4y – 4 = 0 which are
(i) parallel,
SA

(ii) perpendicular to the line 3x – 4y – 1 = 0


Answer.
(10) (i) 3x – 4y + 20 = 0 and 3x – 4y – 10 = 0 (ii) 4x + 3y + 5 = 0 and 4x + 3y – 25 = 0

Normal :
If a line is normal / orthogonal to a circle, then it must pass through the centre of the circle. Using this
y +f
fact normal to the circle x2 + y2 + 2gx + 2fy + c = 0 at (x1, y1) is; y − y1 = 1 (x − x1).
x1 + g
Example # 12 Two normals of a circle are 2x + 3y = 5 and 3x – 4y + 1 = 0. Find its equation having radius 2
Solution. Since point of intersection of normals is the centre of the circle
point of intersection of lines 2x + 3y = 5 and 3x – 4y + 1 = 0 is (1,1)
equation of circle having centre (1,1) and radius 2 is
(x – 1)2 + (y – 1)2 = 4

Self practice problem :

(11) Find the equation of the normal to the circle x2 + y2 – 2x – 4y + 3 = 0 at the point (2, 3).

Answer : (11) x – y + 1 = 0

Pair of tangents from a point :


The equation of a pair of tangents drawn from the point A (x 1, y1) to the circle
x2 + y2 + 2gx + 2fy + c = 0 is : SS1 = T².
Where S ≡ x2 + y2 + 2gx + 2fy + c ; S1 ≡ x1² + y1² + 2gx1 + 2fy1 + c
T ≡ xx1 + yy1 + g(x + x1) + f(y + y1) + c.

RI
A
Example # 13 Find the equation of the pair of tangents drawn to the circle x 2 + y2 + 4x – 6y + 9 = 0 from
thepoint (2, 1)

UH
Solution. Given circle is S = x2 + y2 + 4x – 6y + 9 = 0
Let P ≡ (2, 1)
For point P, S1 = 16
Clearly P lies outside the circle
JA
and T ≡ 2x + y + 2 (x + 2) – 3(y + 1) + 9 = 0
i.e T ≡ 2(2x – y + 5)
Now equation of pair of tangents from P(2, 1) to circle (1) is SS 1 = T2
or 16 (x2 + y2 + 4x – 6y + 9) = 4(2x – y + 5)2 or 12y2 – 16x – 56y + 16xy + 44 = 0
or 3y2 – 4x – 14y + 4xy + 11 = 0
LP

Self practice problems :

(12) Find the joint equation of the tangents through (7, 1) to the circle x 2 + y2 = 25.

Answer : (12) 12x2 – 12y2 + 7xy – 175x – 25y + 625 = 0


A

Length of a tangent and power of a point :


The length of a tangent from an external point (x 1, y1) to the circle
NK

S ≡ x2 + y2 + 2gx + 2fy + c = 0 is given by L = x12 + y12 + 2 g x1 + 2 f1 y + c = S1 .


SA

AP = length of tangent
AP2 = AD . AE
Square of length of the tangent from the point A is also called the power of point w.r.t. a circle.

Power of a point w.r.t. a circle remains constant.

Power of a point P is positive, negative or zero according as the point ‘A’ is outside, inside or on the circle
respectively.

Example # 14 Find the angle between the tangents drawn from the point (2, 0) to the circle x 2 + y2 = 1
Solution. Given circle is x2 + y2 = 1 .........(i)
Given point is (2, 0).
Now length of the tangent from (2, 0) to circle (i) = 22 + 0 2 − 1 = 3
1
tanα =
3
3
1
α (2, 0)

π
α=
6
π
so angle between tangents = 2α =
3
Self practice problems :

(13) The lenght of tangents from P(1, –1) & Q(3, 3) to a circle are 2 and 6 respectively. Then

RI
find the lenght of tangent from R (– 1, – 5) to the same circle.
(14) Find the lenght of tangent drawn from any point on circle x 2 + y2 + 4x + 6y – 3 = 0 to the circle
x2 + y2 + 4x + 6y + 4 = 0.

A
Answer. (13) 38 (14) 7

Director circle :
The locus of the point of intersection of two perpendicular tangents is called the director circle of the

UH
given circle. The director circle of a circle is the concentric circle having radius equal to times the
original circle.
Proof : JA
AC = r cosec 45º = r 2
Example # 15 Find the equation of director circle of the circle x 2 + y2 + 6x + 8y – 2 = 0
Solution : Centre & radius of given circle are (–3, –4) & 27 respectively.
LP

Centre and radius of the director circle will be (–3, –4) &
27. 2 = 54 respectively.
∴ equation of director circle is (x + 3)2 + (y + 4)2 = 54
⇒ x2 + y2 + 6x + 8y –29 = 0
A

Self practice problems :


(15) Find the angle between the tangents drawn from (5, 7 ) to the circle x2+y2 =16
π
NK

Answer (15)
2

Chord of contact :
If two tangents PT1 & PT2 are drawn from the point P(x1, y 1) to the circle
S ≡ x2 + y2 + 2gx + 2fy + c = 0, then the equation of the chord of contact T 1T2 is:
SA

xx1 + yy1 + g (x + x1) + f (y + y1) + c = 0.

Note : Here R = radius; L = length of tangent.


(a) Chord of contact exists only if the point ‘P’ is not inside.

2LR
(b) Length of chord of contact T1 T2 = .
R2 + L2
(c) Area of the triangle formed by the pair of the tangents & its chord
RL3
of contact =
R2 + L2
 2RL 
(d) Tangent of the angle between the pair of tangents from (x 1, y1) =  2 
 L − R2
 
(e) Equation of the circle circumscribing the triangle PT1 T2 is:
(x − x1) (x + g) + (y − y1) (y + f) = 0.

Example # 16 Find the equation of the chord of contact of the tangents drawn from (0, 1) to the
circle x2 + y2 – 2x + 4y = 0
Solution. Given circle is x2 + y2 – 2x + 4y = 0 .......(i)
Let P = (0, 1)
For point P (0, 1), T = x . 0 + y . 1 – (x + 0) + 2(y + 1)

RI
i.e. T = x – 3y – 2
Now equation of the chord of contact of point P(0, 1) w.r.t. circle (i) will be
x – 3y – 2 = 0

A
Example # 17 If the chord of contact of the tangents drawn from (α, β) to the circle x 2 + y2 = a2
subtends right angle at the centre then prove that α2 + β2 = 2a2.
π
Solution. ∠QOR = ∠QPR =

UH
2
so OQPR is a square
Q

O P
JA
R
OQ2 = PQ2
a2 = α2 + β2 – a2
α2 + β2 = 2a2

Self practice problems :


LP

(16) Find the co-ordinates of the point of intersection of tangents at the points where the line
x – 2y + 1 = 0 meets the circle x2 + y2 = 25
(17) If the chord of contact of the tangetns drawn from a point on circle x 2 + y2 = a2 to another circle
x2 + y2 = b2 touches the circle x2 + y2 = c2 then prove that a,b,c are in G.P.
405 √ 3
A

Answers : (16) (–25, 50) (17) sq. unit ; 4x + 6y – 25 = 0


52
NK

Equation of the chord with a given middle point:


The equation of the chord of the circle S ≡ x2 + y2 + 2gx + 2fy + c = 0 in terms of its mid point M (x1, y1) is
xx1 + yy1 + g (x + x1) + f (y + y1) + c = x12 + y12 + 2gx1 + 2fy1 + c which is designated by T = S1.
SA

Notes : (i) The shortest chord of a circle passing through a point ‘M’ inside the circle , is one chord whose
middle point is M.
(ii) The chord passing through a point ' M ' inside the circle and which is at a maximum distance
from the centre is a chord with middle point M.
Example # 18 Find the equation of the chord of the circle x 2 + y2 + 2x – 2y – 4 = 0, whose middle point is
(0, 0)
Solution. Equation of given circle is S ≡ x2 + y2 + 2x – 2y – 4 = 0
Let L ≡ (0, 0)
For point L(0, 0), S1 = –4 and
T ≡ x.0 + y (0) + (x + 0) – (y + 0) – 4 i.e. T≡x–y–4
Now equation of the chord of circle (i) whose middle point is L(0, 0) is
T = S1 or x – y = 0
Second Method : Let C be the centre of the given circle, then C ≡ (–1, 1). L ≡ (0, 0) slope of
CL = –1
∴ Equation of chord of circle whose middle point is L, is y – 0 = 1(x – 0)
( chord is perpendicular to CL) or x–y=0
Self practice problems :
(18) Find the equation of that chord of the circle x 2 + y2 = 15, which is bisected at
(3, 2)
(19) A variable chord is drawn through the origin to the circle x 2 + y2 – 2ax = 0. Find the locus of the
centre of the circle drawn on this chord as diameter.
Answers : (18) 3x + 2y – 13 = 0 (19) x2 + y2 – ax = 0

Equation of the chord joining two points of circle :

RI
The equation of chord PQ to the circle x2 + y2 = a2 joining two points P(α) and Q(β) on it is given by the
equation of a straight line joining two point α & β on the circle x 2 + y2 = a2 is
α+ β α+β α−β
x cos + y sin = a cos .

A
2 2 2

Common tangents to two circles:


Case Number of Tangents Condition

UH
(2 direct and 2 transverse) r1 + r2 < c1 c2.
JA
LP
A

(Here C1C2 is distance between centres of two circles.)

Notes : (i) The direct common tangents meet at a point which divides the line joining centre of circles
NK

externally in the ratio of their radii.


Transverse common tangents meet at a point which divides the line joining centre of circles
internally in the ratio of their radii.
(ii) Length of an external (or direct) common tangent & internal (or transverse) common tangent to
the two circles are given by:
Lext= d2 − (r1 − r2 )2 & Lint = d2 − (r1 + r2 )2 , where d = distance between the centres of the
SA

two circles and r1, r2 are the radii of the two circles. Note that length of internal common tangent
is always less than the length of the external or direct common tangent.

Example # 19 Examine if the two circles x2 + y2 – 4x – 6y + 9 = 0 and x2 + y2 – 10x – 6y + 18 = 0 intersect


or not
Solution. Given circles are x2 + y2 – 4x – 6y + 9 = 0 ...........(i)
and x2 + y2 – 10x – 6y + 18 = 0 ...........(ii)
Let A and B be the centres and r1 and r2 the radii of circles (i) and (ii) respectively, then
A ≡ (2, 3), B ≡ (5, 3), r1 = 2, r2 = 4
Now AB = 3 and r1 + r2 = 6, |r1 – r2| = 2
Thus |r1 – r2| < AB < r1 + r2, hence the two circles intersect.
Self practice problems :
(20) Find the position of the circles x2 + y2 – 10x + 4y – 20 = 0 and x2 + y2 + 14x – 6y + 22 = 0 with
respect to each other.
Answer : (20) touch externally

Orthogonality of two circles:


Two circles S1= 0 & S2= 0 are said to be orthogonal or said to intersect orthogonally if the tangents at
their point of intersection include a right angle. The condition for two circles to be orthogonal is:
2 g1 g2 + 2 f1 f2 = c1 + c2.
Proof :
(C1C2)2 = (C1P)2 + (C2P)2
⇒ (g1 – g2)2 + (f1 – f2)2 = g12 + f12 – c1 + g22 + f22 – c2

RI
⇒ 2g1g2 + 2f1f2 = c1 + c2

A
Notes :

UH
(a) The centre of a variable circle orthogonal to two fixed circles lies on the radical axis of two circles.
(b) If two circles are orthogonal, then the polar of a point 'P' on first circle w.r.t. the second circle passes
through the point Q which is the other end of the diameter through P. Hence locus of a point which
moves such that its polars w.r.t. the circles S1 = 0, S2 = 0 & S3 = 0 are concurrent in a circle which is
orthogonal to all the three circles.
JA
(c) The centre of a circle which is orthogonal to three given circles is the radical centre provided the radical
centre lies outside all the three circles.

Example # 20 If the circles x2 + y2 + 2g1x + 2f1y + c1 = 0 and 2x2 + 2y2 + 2g2x + 2f2y + c2 = 0 are orthogonal to
c
each other then prove that g1g2 + f1f2 = c1 + 2
2
Solution. Given circles are x2 + y2 + 2g1x + 2f1y + c1 = 0 ...........(i)
LP

and 2x2 + 2y2 + 2g2x + 2f2y + c2 = 0


c
or x2 + y2 + g2x + f2y + 2 = 0 ..........(ii)
2
Since circles (i) and (ii) cut orthogonally
A

g  f  c
∴ 2g1  2  + 2f1  2  = c1 + 2
 2  2 2
c2
NK

g1g2 + f1f2 = c1 +
2

Self practice problems :


(21) For what value of λ the circles x2 + y2 + 8x + 3y + 9 = 0 and x2 + y2 + 2x – y – λ = 0 cut
orthogonally.
SA

(22) Find the equation to the circle which passes through the origin and has its centre on the line
x – y = 0 and cuts the circle x2 + y2 – 4x – 6y + 10 = 0 orthogonally.
5
Answer : (21) (22) x2 + y2 – 2x – 2y = 0
2

Radical axis and radical centre:


The radical axis of two circles is the locus of points whose powers w.r.t. the two circles are equal. The
equation of radical axis of the two circles S1 = 0 & S2 = 0 is given by
S1 − S2 = 0 i.e. 2 (g1 − g2) x + 2 (f1 − f2) y + (c1 − c2) = 0.
The common point of intersection of the radical axes of three circles taken two at a time is called the
radical centre of three circles. Note that the length of tangents from radical centre to the three circles
are equal.

Notes :
(a) If two circles intersect, then the radical axis is the common chord of the two circles.
(b) If two circles touch each other, then the radical axis is the common tangent of the two circlesat the

RI
common point of contact.
(c) Radical axis is always perpendicular to the line joining the centres of the two circles.
(d) Radical axis will pass through the mid point of the line joining the centres of the two circles only if

A
the two circles have equal radii.
(e) Radical axis bisects a common tangent between the two circles.
(f) A system of circles, every two which have the same radical axis, is called a coaxial system.

UH
(g) Pairs of circles which do not have radical axis are concentric.

Example # 21 Find the co-ordinates of the point from which the lengths of the tangents to the
following three circles be equal.
x2 + y2 = 1
JA
x2 + y2 – 8x + 15 = 0
x2 + y2 + 10y + 24 = 0
Solution : Here we have to find the radical centre of the three circles. First reduce them to standard form
in which coefficients of x2 and y2 be each unity. Subtracting in pairs the three radical axes are
x=2 ; 8x + 10y + 9 = 0
10y + 25 = 0
 5
LP

solving any two, we get the point  2, −  which satisfies the third also. This point is called the
 2
radical centre and by definition the length of the tangents from it to the three circles are equal.

Self practice problem :


A

(23) Find the point from which the tangents to the three circles x 2 + y2 – 4x + 7 = 0,
2x2 + 2y2 – 3x + 5y + 9 = 0 and x2 + y2 + y = 0 are equal in length. Find also
this length.
NK

Answer : (23) (2, – 1) ; 2.

Family of Circles:
This article is aimed at obtaining the equation of a group of circles having a specific characteristic.
For example, the equation x2 + y2 + 4x + 2y + λ = 0 where λ is arbitrary, represents a family of circles
with fixed centre (–2, –1) but variable radius. We have the following results for some other families of
SA

circles.
(a) The equation of the family of circles passing through the points of intersection of two circles
S1 = 0 & S2 = 0 is : S1 + K S2 = 0
(K ≠ −1, provided the co−efficient of x2 & y2 in S1 & S2 are same)
(b) The equation of the family of circles passing through the point of intersection of a circle S = 0 &
a line L = 0 is given by S + KL = 0.
(c) The equation of a family of circles passing through two given points
(x1, y1) & (x2, y2) can be written in the form:
x y 1
(x − x1) (x − x2) + (y − y1) (y − y2) + K x1 y1 1 = 0, where K is a parameter.
x2 y2 1
(d) The equation of a family of circles touching a fixed line y − y1 = m (x − x1) at the fixed point (x1,
y1) is (x − x1)2 + (y − y1)2 + K (y − y1 − m (x − x1)) = 0, where K is a parameter.
(e) Family of circles circumscribing a triangle whose sides are given by L1 = 0, L2 = 0 and L3 = 0 is
given by; L1L2 + λ L2L3 + µ L3L1 = 0 provided co−efficient of xy = 0 and co−efficient of x 2 =
co−efficient of y2.
(f) Equation of circle circumscribing a quadrilateral whose side in order are represented by the
lines L1 = 0, L2 = 0, L3 = 0 & L4 = 0 are u L1L3 + λ L2L4 = 0 where values of u & λ can be found
out by using condition that co−efficient of x2 = co−efficient of y2 and co−efficient of xy = 0.

Example # 22 Find the equation of the circle passing through the point (1, 1) and points of
intersection of the circles x2 + y2 + 13x – 3y = 0 and 2x2 + 2y2 + 4x – 7y – 25 = 0.
Solution. Any circle through the intersection of given circles is S1 + λS2 = 0

RI
or x2 + y2 + 13x – 3y + λ(x2 + y2 + 2x – 7y/2 – 25/2) = 0
This circle passes through (1, 1)
1 + 1 + 13 – 3 + λ(1 + 1 + 2 – 7/2 – 25/2) = 0
λ=1
Putting the value of λ in (i) the required circle is 4x 2 + 4y2 + 30x – 13y – 25 = 0

A
Example # 23 Find the equations of smallest circle which passes through the points of intersection of the line x
+ y = 1 and the circle x2 + y2 = 9.

UH
Solution. The required circle by S + λL = 0 is
x2 + y2 – 9 + λ (x + y – 1) = 0 ....(i)
 λ λ
centre (– g, – f) =  − , − 
 2 2
centre lies on the line x + y = 1
JA
λ λ
– – =1
2 2
λ = –1
Putting the value of λ in (i) the required circle is
x2 + y2 –x – y – 8 = 0
LP

Example # 24 Find the equation of circle passing through the points A(1, 1) & B(0, 3) and
5
whose radius is .
2
Solution. Equation of AB is 2x + y – 3 = 0
∴ equation of circle is
A

(x – 1) (x) + (y – 1) (y – 3) + λ(2x + y – 3)
= 0 or x2 + y2 + (2λ – 1)x + (λ – 4)y + 3 – 3λ = 0
2
 2λ − 1   λ − 4 
NK

5
 2  +  2  + 3λ − 3 = 2
   
λ=1
∴ equation of circle is x2 + y2 + x – 3y = 0
SA

Example # 25 A variable circle always touhces x + y = 2 at (1, 1), cuts the circle x 2 + y2 + 4x + 5y – 6 = 0.
Prove that all common chords pass through a fixed point. Also find the point.
Solution : Equation of circle is (x – 1)2 + (y – 1)2 + λ(x + y – 2) = 0
x2 + y2 + x (λ – 2) + y (λ – 2) + 2 – 2λ = 0
common chord of this circle with x2 + y2 + 4x + 5y – 6 = 0 is
(λ – 6)x + (λ – 7)y + 8 – 2λ = 0
λ(x + y – 2) + (–6x – 7y + 8) = 0
this chord passes through the point of intersection of the lines x + y – 2 = 0 and –6x – 7y + 8
= 0 which is (6, –4)
Example # 26Find the equation of circle circumcscribing the triangle whose sides are 3x – y –
12 = 0,
5x – 3y – 28 = 0 & x + y – 4 = 0.
Solution : L1L2 + λL2L3 + µL1L3 = 0
(3x – y – 12) (5x – 3y – 28) + λ(5x – 3y – 28) (x + y – 4) + µ (3x – y – 12) (x + y – 4) = 0
coefficient of x2 = coefficient of y2
⇒ 5λ + 3µ + 15 = 3 – 3λ – µ
2λ + µ + 3 = 0 ...........(ii)
coefficient of xy = 0
⇒ λ+µ–7=0 ..........(iii)
Solving (ii) and (iii), we have
λ = – 10, µ = 17
Puting these values of λ & µ in equation (i), we get 2x 2 + 2y2 – 9x + 11y + 4 = 0

Self practice problems :


(24) Find the equation of the circle passing through the points of intersection of the circles
x2 + y2 – 6x + 2y + 4 = 0 and x2 + y2 + 2x – 4y – 6 = 0 and with its centre on the line y = x.
(25) Find the equation of circle circumscribing the quadrilateral whose sides are x + y = 10,

RI
x – 7y + 50 = 0, 22x – 4y + 125 = 0 and 2x – 4y – 5

Answers : (24) 7x2 + 7y2 – 10x – 10y – 12 = 0


125
(25) x2 + y2 =

A
2

UH
JA
LP
A
NK
SA
Marked questions are recommended for Revision.

PART - I : SUBJECTIVE QUESTIONS


Section (A) : Equation of circle, parametric equation, position of a point
A-1. Find the equation of the circle that passes through the points (1, 0), (– 1, 0) and (0, 1).

A-2. ABCD is a square in first quadrant whose side is a, taking AB and AD as axes, prove that the equation
to the circle circumscribing the square is x2 + y2 = a(x + y).

RI
A-3. Find the equation to the circle which passes through the origin and cuts off intercepts equal to 3 and 4
from the positive axes.

A-4. Find equation of circle which touches x & y axis & perpendicular distance of centre of circle from

A
3x + 4y + 11 = 0 is 5. Given that circle lies in Ist quadrant.

A-5. Find the equation to the circle which touches the axis of x at a distance 3 from the origin and intercepts
a distance 6 on the axis of y.

UH
A-6. Find equation of circle whose cartesian equation are x = –3 + 2 sin θ, y = 4 + 2 cos θ

A-7. Find the values of p for which the power of a point (2, 5) is negative with respect to a circle
x2 + y2 − 8x − 12y + p = 0 which neither touches the axes nor cuts them.
JA
Section (B) : Line and circle, tangent, pair of tangent
B-1. If radii of the largest and smallest circle passing through the point (1, –1) and touching the circle
x2 + y2 + 2 2 y – 2 = 0 are r1 and r2 respectively, then find the sum of r1 and r2 .

B-2. Find the points of intersection of the line x – y + 2 = 0 and the circle 3x 2 + 3y2 – 29x – 19y + 56 = 0. Also
LP

determine the length of the chord intercepted.

B-3. Show that the line 7y – x = 5 touches the circle x 2 + y2 – 5x + 5y = 0 and find the equation of the other
parallel tangent.

B-4. Find the equation of the tangents to the circle x 2 + y2 = 4 which make an angle of 60º with the positive x-
A

axis in anticlockwise direction.

B-5. Show that two tangents can be drawn from the point (9, 0) to the circle x 2 + y2 = 16; also find the
NK

equation of the pair of tangents and the angle between them.

B-6. If the length of the tangent from (f, g) to the circle x 2 + y2 = 6 be twice the length of the tangent from
(f, g) to the circle x2 + y2 + 3x + 3y = 0, then will f2 + g2 + 4f + 4g + 2 = 0 ?

Section (C) : Normal, Director circle, chord of contact, chord with mid point
SA

C-1. Find the equation of the normal to the circle x 2 + y2 = 5 at the point (1, 2)

C-2. Find the equation of the normal to the circle x 2 + y2 = 2x, which is parallel to the line x + 2y = 3.
C-3. Find the equation of director circle of the circle (x + 4) 2 + y2 = 8
C-4. Tangents are drawn from the point (h, k) to the circle x2 + y2 = a2; prove that the area of the triangle
a(h2 + k 2 − a2 )3 / 2
formed by them and the straight line joining their points of contact is c.
h2 + k 2
C-5. Find the equation of the chord of the circle x 2 + y2 + 6x + 8y + 9 = 0 whose middle point is (– 2, – 3).

C-6. Tangents are drawn to the circle x2 + y2 = 12 at the points where it is met by the circle
x2 + y2 – 5x + 3y – 2 = 0; find the point of intersection of these tangents.
Section (D) : Position of two circle, Orthogonality, Radical axis and radical centre
D-1. Find the equations to the common tangents of the circles x 2 + y2 – 2x – 6y + 9 = 0 and
x2 + y2 + 6x – 2y + 1 = 0

D-2. Show that the circles x2 + y2 –2x – 6y – 12 = 0 and x2 + y2 + 6x + 4y – 6 = 0 cut each other orthogonally.

D-3. Find the equation of the circle passing through the origin and cutting the circles
x2 + y2 – 4x + 6y + 10 = 0 and x2 + y2 + 12y + 6 = 0 at right angles.

D-4. Given the three circles x2 + y2 – 16x + 60 = 0, 3x2 + 3y2 – 36x + 81 = 0 and x2 + y2 – 16x – 12y + 84 = 0,
find (1) the point from which the tangents to them are equal in length and (2) this length.

Section (E) : Family of circles , Locus, Miscellaneous

RI
E-1. If y = 2x is a chord of the circle x2 + y2 – 10x = 0, find the equation of a circle with this chord as diameter.

E-2. Find the equation of a circle which touches the line 2x – y = 4 at the point (1, –2) and
(i) Passes through (3, 4) (ii) Radius = 5

A
E-3. Show that the equation x2 + y2 – 2x – 2λy – 8 = 0 represents for different values of λ a system of circles
passing through two fixed points A and B on the x-axis, and also find the equation of that circle of the

UH
system the tangent to which at A and B meet on the line x + 2y + 5 = 0.

E-4. Consider a family of circles passing through two fixed points A (3, 7) and B (6, 5). Show that the chords
in which the circles x2 + y2 – 4x – 3 = 0 cuts the members of the family are concurrent at a point. Also
find the co-ordinates of this point.
JA
E-5. Find the equation of the circle circumscribing the triangle formed by the lines x + y = 6, 2x + y = 4 and
x + 2y = 5.

1 1 1
E-6. Prove that the circle x2 + y2 + 2ax + c2 = 0 and x2 + y2 + 2by + c2 = 0 touches each other if + =
a2 b2 c 2
LP

PART - II : ONLY ONE OPTION CORRECT TYPE


Section (A) : Equation of circle, parametric equation, position of a point
A-1. The radius of the circle passing through the points (1, 2), (5, 2) & (5, − 2) is:
A

(A) 5 2 (B) 2 5 (C) 3 2 (D) 2 2


NK

A-2. The centres of the circles x2 + y2 – 6x – 8y – 7 = 0 and x2 + y2 – 4x – 10y – 3 = 0 are the ends of the
diameter of the circle
(A) x2 + y2 – 5x – 9y + 26 = 0 (B) x2 + y2 + 5x – 9y + 14 = 0
2 2
(C) x + y + 5x – y – 14 = 0 (D) x2 + y2 + 5x + y + 14 = 0

A-3. The circle described on the line joining the points (0, 1), (a, b) as diameter cuts the x−axis in points
SA

whose abscissa are roots of the equation:


(A) x² + ax + b = 0 (B) x² − ax + b = 0 (C) x² + ax − b = 0 (D) x² − ax − b = 0

A-4. The intercepts made by the circle x2 + y2 – 5x – 13y – 14 = 0 on the x-axis and y-axis are respectively
(A) 9, 13 (B) 5, 13 (C) 9, 15 (D) none

A-5. Equation of line passing through mid point of intercepts made by circle x 2 + y2 – 4x – 6y = 0 on
co-ordinate axes is
(A) 3x + 2y – 12 = 0 (B) 3x + y – 6 = 0 (C) 3x + 4y – 12 = 0 (D) 3x + 2y – 6 = 0

A-6. Two thin rods AB & CD of lengths 2a & 2b move along OX & OY respectively, when ‘O’ is the origin.
The equation of the locus of the centre of the circle passing through the extremities of the two rods is:
(A) x² + y² = a² + b² (B) x² − y² = a² − b² (C) x² + y² = a² − b² (D) x² − y² = a² + b²
A-7. Let A and B be two fixed points then the locus of a point C which moves so that (tan∠BAC)
π π
(tan ∠ABC)=1, 0 < ∠BAC < , 0 < ∠ABC < is
2 2
(A) Circle (B) pair of straight line (C) A point (D) Straight line

A-8. STATEMENT-1 : The length of intercept made by the circle x 2 + y2 – 2x – 2y = 0 on the x-axis is 2.
α β
STATEMENT-2 : x2 + y2 – αx – βy = 0 is a circle which passes through origin with centre  ,  and
 2 2
α 2 + β2
radius
2
(A) STATEMENT-1 is true, STATEMENT-2 is true and STATEMENT-2 is correct explanation for
STATEMENT-1

RI
(B) STATEMENT-1 is true, STATEMENT-2 is true and STATEMENT-2 is not correct explanation
for STATEMENT-1
(C) STATEMENT-1 is true, STATEMENT-2 is false
(D) STATEMENT-1 is false, STATEMENT-2 is true

A
Section (B) : Line and circle, tangent, pair of tangent
B-1. Find the co-ordinates of a point p on line x + y = – 13, nearest to the circle x 2 + y2 + 4x + 6y – 5 = 0

UH
(A) (– 6, – 7) (B) (– 15, 2) (C) (– 5, – 6) (D) (– 7, – 6)
B-2. The number of tangents that can be drawn from the point (8, 6) to the circle x 2 + y2 – 100 = 0 is
(A) 0 (B) 1 (C) 2 (D) none
B-3. Two lines through (2, 3) from which the circle x2 + y2 = 25 intercepts chords of length 8 units have
equations
JA
(A) 2x + 3y = 13, x + 5y = 17 (B) y = 3, 12x + 5y = 39
(C) x = 2, 9x – 11y = 51 (D) y = 0, 12x + 5y = 39
B-4. The line 3x + 5y + 9 = 0 w.r.t. the circle x 2 + y2 – 4x + 6y + 5 = 0 is
(A) chord dividing circumference in 1 : 3 ratio (B) diameter
(C) tangent (D) outside line
LP

B-5. If one of the diameters of the circle x2 + y2 – 2x – 6y + 6 = 0 is a chord to the circle with centre (2, 1),
then the radius of the circle is
(A) 3 (B) 2 (C) 3/2 (D) 1
B-6. The tangent lines to the circle x² + y² − 6x + 4y = 12 which are parallel to the line 4x + 3y + 5 = 0 are
given by:
A

(A) 4x + 3y − 7 = 0, 4x + 3y + 15 = 0 (B) 4x + 3y − 31 = 0, 4x + 3y + 19 = 0
(C) 4x + 3y − 17 = 0, 4x + 3y + 13 = 0 (D) 4x + 3y − 31 = 0, 4x + 3y – 19 = 0
B-7. The condition so that the line (x + g) cosθ + (y + f) sin θ = k is a tangent to x 2 + y2 + 2gx + 2fy + c = 0 is
(A) g2 + f2 = c + k2 (B) g2 + f2 = c2 + k (C) g2 + f2 = c2 + k2 (D) g2 + f2 = c + k
NK

B-8. The tangent to the circle x2 + y2 = 5 at the point (1, –2) also touches the circle
x2 + y2 – 8x + 6y + 20 = 0 at
(A) (–2, 1) (B) (–3, 0) (C) (–1, –1) (D) (3, –1)

B-9. The angle between the two tangents from the origin to the circle (x − 7)² + (y + 1)² = 25 equals
SA

π π π π
(A) (B) (C) (D)
4 3 2 6
B-10. A point A (2, 1) is outside the circle x² + y² + 2gx + 2fy + c = 0 & AP, AQ are tangents to the circle.
The equation of the circle circumscribing the triangle APQ is:
(A) (x + g) (x − 2) + (y + f) (y − 1) = 0 (B) (x + g) (x − 2) − (y + f) (y − 1) = 0
(C) (x − g) (x + 2) + (y − f) (y + 1) = 0 (D) (x – g) (x − 2) + (y – f) (y − 1) = 0

B-11. A line segment through a point P cuts a given circle in 2 points A & B, such that PA = 16 & PB = 9, find
the length of tangent from points to the circle
(A) 7 (B) 25 (C) 12 (D) 8

B-12.The length of the tangent drawn from any point on the circle x² + y² + 2gx + 2fy + p = 0 to the circle
x² + y² + 2gx + 2fy + q = 0 is:
(A) q − p (B) p − q (C) q + p (D) 2q + p
B-13. The equation of the diameter of the circle (x – 2)2 + (y + 1)2 = 16 which bisects the chord cut off by the
circle on the line x – 2y – 3 = 0 is
(A) x + 2y = 0 (B) 2x + y – 3 = 0 (C) 3x + 2y – 4 = 0 (D) 3x – 2y – 4 = 0

B-14. The locus of the point of intersection of the tangents to the circle x 2 + y2 = a2 at points whose parametric
π
angles differ by is
3
4a2 2a2 a2 a2
(A) x2 + y2 = (B) x2 + y2 = (C) x2 + y2 = (D) x2 + y2 =
3 3 3 9

Section (C) : Normal, Director circle, chord of contact, chord with mid point
C-1. The equation of normal to the circle x 2 + y2 – 4x + 4y – 17 = 0 which passes through (1, 1) is

RI
(A) 3x + y – 4 = 0 (B) x – y = 0 (C) x + y = 0 (D) 3x – y – 4 = 0

C-2. The normal at the point (3, 4) on a circle cuts the circle at the point (–1, –2). Then the equation of the
circle is
(A) x2 + y2 + 2x – 2y – 13 = 0 (B) x2 + y2 – 2x – 2y – 11 = 0

A
2 2
(C) x + y – 2x + 2y + 12 = 0 (D) x2 + y2 – 2x – 2y + 14 = 0

C-3. The co-ordinates of the middle point of the chord cut off on 2x – 5y + 18 = 0 by the circle

UH
x2 + y2 – 6x + 2y – 54 = 0 are
(A) (1, 4) (B) (2, 4) (C) (4, 1) (D) (1, 1)

C-4. The locus of the mid point of a chord of the circle x² + y² = 4 which subtends a right angle at the origin is:
(A) x + y = 2 (B) x² + y² = 1 (C) x² + y² = 2 (D) x + y = 1
JA
C-5. The chords of contact of the pair of tangents drawn from each point on the line 2x + y = 4 to the circle
x2 + y2 = 1 pass through the point
 1 1
(A) (1, 2) (B)  ,  (C) (2, 4) (D) (4, 4)
2 4
C-6. The locus of the centers of the circles such that the point (2, 3) is the mid point of the chord 5x + 2y = 16 is:
LP

(A) 2x − 5y + 11 = 0 (B) 2x + 5y − 11 = 0 (C) 2x + 5y + 11 = 0 (D) 2x − 5y – 11 = 0

C-7. Find the locus of the mid point of the chord of a circle x² + y² = 4 such that the segment intercepted by
the chord on the curve x² − 2x − 2y = 0 subtends a right angle at the origin.
(A) x² + y² − 2x − 2y = 0 (B) x² + y² + 2x − 2y = 0 (C) x² + y² + 2x + 2y = 0 (D) x² + y² – 2x + 2y = 0
A

Section (D) : Position of two circle, Orthogonality, Radical axis and radical centre
Number of common tangents of the circles (x + 2)² + (y−2)² = 49 and (x − 2)² + (y + 1)² = 4 is:
NK

D-1.
(A) 0 (B) 1 (C) 2 (D) 3

D-2. The equation of the common tangent to the circle x 2 + y2 – 4x – 6y–12=0 and x2 + y2 + 6x+18y + 26 = 0
at their point of contact is
SA

(A) 12x + 5y + 19 = 0 (B) 5x + 12y + 19 = 0 (C) 5x – 12y + 19 = 0 (D) 12x –5y + 19 = 0

D-3. Equation of the circle cutting orthogonally the three circles x 2 + y2 – 2x + 3y – 7 = 0,


x2 + y2 + 5x – 5y + 9 = 0 and x2 + y2 + 7x – 9y + 29 = 0 is
(A) x2 + y2 – 16x – 18y – 4 = 0 (B) x2 + y2 – 7x + 11y + 6 = 0
(C) x2 + y2 + 2x – 8y + 9 = 0 (D) x2 + y2 + 16x – 18y – 4 = 0

D-4. If the length of a common internal tangent to two circles is 7, and that of a common external tangent is
11, then the product of the radii of the two circles is:
(A) 18 (B) 20 (C) 16 (D) 12
Section (E) : Family of circles , Locus, Miscellaneous
E-1. The locus of the centre of the circle which bisects the circumferences of the circles x² + y² = 4
& x² + y² − 2x + 6y + 1 = 0 is :
(A) a straight line (B) a circle (C) a parabola (D) pair of straight line

E-2. Equation of a circle drawn on the chord x cos α + y sin α = p of the circle x 2 + y2 = a2 as its diameter, is
(A) (x2 + y2 – a2) –2p (xsinα + ycosα – p) = 0 (B) (x2 + y2 – a2) –2p (xcosα + ysinα – p) = 0
(C) (x2 + y2 – a2) + 2p (xcosα + ysinα – p) = 0 (D) (x2 + y2 – a2) –p (xcosα + ysinα – p) = 0

E-3. Find the equation of the circle which passes through the point (1, 1) & which touches the circle

RI
x² + y² + 4x − 6y − 3 = 0 at the point (2, 3) on it.
(A) x² + y² + x − 6y + 3 = 0 (B) x² + y² + x − 6y – 3 = 0
(C) x² + y² + x + 6y + 3 = 0 (D) x² + y² + x − 3y + 3 = 0

A
E-4. Find the equation of circle touching the line 2x + 3y + 1 = 0 at (1, – 1) and cutting orthogonally the circle
having line segment joining (0, 3) and (– 2, – 1) as diameter.

UH
(A) 2x2 + 2y2 – 10x– 5y + 1 = 0 (B) 2x2 + 2y2 – 10x+ 5y + 1 = 0
(C) 2x2 + 2y2 – 10x– 5y – 1 = 0 (D) 2x2 + 2y2 + 10x– 5y + 1 = 0
E-5. Equation of the circle which passes through the point (–1, 2) & touches the circle x 2 + y2 – 8x + 6y = 0
at origin, is -
JA
3
(A) x2 + y2 – 2x – y = 0 (B) x2 + y2 + x – 2y = 0
2
3 3
(C) x2 + y2 + 2x + y = 0 (D) x2 + y2 + 2x – y = 0
2 2
E-6. Two circles are drawn through the point (a, 5a) and (4a, a) to touch the axis of ‘y’. They
LP

intersect at an angle of θ then tanθ is -


40 9 1 1
(A) (B) (C) (D)
9 40 9 3
A

PART - III : MATCH THE COLUMN


1. Column – I Column – II
NK

(A) Number of values of a for which the common chord of the circles x 2 + y2 = 8 (p) 0
and (x – a)2 + y2 = 8 subtends a right angle at the origin is

(B) The number of circles touching all the three lines 3x + 7y = 2, 21x + 49y = 5 (q) 2
and 9x + 21y = 0 are
SA

(C) The length of common chord of circles x2 + y2 – x – 11y + 18 = 0 and (r) 5

x2 + y2 – 9x – 5y + 14 = 0 is

(D) Number of common tangents of the circles x2 + y2 – 2x = 0 and (s) 3


x2 + y2 + 6x – 6y + 2 = 0 is

2. Column – I Column – II
(A) If director circle of two given circles C1 and C2 of equal radii touches each other, (p) 13
then ratio of length of internal common tangent of C1 and C2 to their radii equals to

(B) Let two circles having radii r1 and r2 are orthogonal to each other. If length of their (q) 7
common chord is k times the square root of hormonic mean between squares of
their radii, then k4 equals to

(C) The axes are translated so that the new equation of the circle (r) 4

λ2
x² + y² − 5x + 2y − 5 = 0 has no first degree terms and the new equation x 2 + y2= ,
4
then the value of λ. is

(D) The number of integral points which lie on or inside the circle x 2 + y2 = 4 is (s) 2

RI
A
UH
JA
LP
A
NK
SA
PART - I : ONLY ONE OPTION CORRECT TYPE

 1  1  1  1
1. If  a ,  ,  b ,  ,  c ,  &  d ,  are four distinct points on a circle of radius 4 units, then abcd is
 a  b  c  d
equal to:
(A) 4 (B) 16 (C) 1 (D) 2

2. From the point A (0, 3) on the circle x² + 4x + (y − 3)² = 0 a chord AB is drawn & extended to a point M
such that AM = 2 AB. The equation of the locus of M is :

RI
(A) x² + 8x + y² = 0 (B) x² + 8x + (y − 3)² = 0
(C) (x − 3)² + 8x + y² = 0 (D) x² + 8x + 8y² = 0

A
3. If tangent at (1, 2) to the circle c1: x2 + y2 = 5 intersects the circle c2: x2 + y2 = 9 at A & B and tangents

at A & B to the second circle meet at point C, then the co−ordinates of C is

UH
 9 18   9 18 
(A) (4, 5) (B)  ,  (C) (4, − 5) (D)  , 
 15 5  5 5 

 7
4. A circle passes through point  3,  touches the line pair x2 – y2 – 2x + 1 = 0. Centre of circle lies
 2 
JA

inside the circle x2 + y2 – 8x + 10y + 15 = 0. Co-ordinate of centre of circle is
(A) (4, 0) (B) (5, 0) (C) (6, 0) (D) (0, 4)

5. The length of the tangents from any point on the circle 15x 2 + 15y2 – 48x + 64y = 0 to the two circles
LP

5x2 + 5y2 – 24x + 32y + 75 = 0 and 5x2 + 5y2 – 48x + 64y + 300 = 0 are in the ratio
(A) 1 : 2 (B) 2 : 3 (C) 3 : 4 (D) 2 : 1

6. The distance between the chords of contact of tangents to the circle; x² + y² + 2gx + 2fy + c = 0 from the
origin & the point (g, f) is:
A

g2 + f 2 − c g2 + f 2 − c g2 + f 2 + c
(A) g2 + f 2 (B) (C) (D)
2 2 g2 + f 2 2 g2 + f 2
NK

7. If from any point P on the circle x² + y² + 2gx + 2fy + c = 0, tangents are drawn to the circle
x² + y² + 2gx + 2fy + c sin²α + (g² + f²) cos²α = 0, then the angle between the tangents is:
α α
(A) α (B) 2 α (C) (D)
SA

2 3
8. The locus of the mid points of the chords of the circle x² + y² + 4x − 6y − 12 = 0 which subtend an angle
π
of radians at its circumference is:
3
(A) (x − 2)² + (y + 3)² = 6.25 (B) (x + 2)² + (y − 3)² = 6.25
(C) (x + 2)² + (y − 3)² = 18.75 (D) (x + 2)² + (y + 3)² = 18.75

9. If the two circles, x2 + y2 + 2 g1x + 2 f1y = 0 & x2 + y2 + 2 g2x + 2 f2y = 0 touch each other then:
f1 f
(A) f1 g1 = f2 g2 (B) = 2 (C) f1 f2 = g1 g2 (D) f1 + f2 = g1 + g2
g1 g2

10. A circle touches a straight line x + my + n = 0 & cuts the circle x² + y² = 9 orthogonally. The locus of
centres of such circles is:
(A) (x + my + n)² = (² + m²) (x² + y² − 9) (B) (x + my − n)² = (² + m²) (x² + y² − 9)
(C) (x + my + n)² = (² + m²) (x² + y² + 9) (D) (x + my – n)² = (² + m²) (x² + y² − 9)

11. The locus of the point at which two given unequal circles subtend equal angles is:
(A) a straight line (B) a circle (C) a parabola (D) an ellipse

12. A circle is given by x2 + (y – 1)2 = 1. Another circle C touches it externally and also the x-axis, then the
locus of its centre is
(A) {(x, y) : x2 = 4y} U {(x, y) : y ≤ 0} (B) {(x, y) : x 2 + (y – 1)2 = 4} U {(x, y) : y ≤ 0}
(C) {(x, y) : x2 = y} U {(0, y) : y ≤ 0} (D) {(x, y) : x2 = 4y} U {(0, y) : y ≤ 0}

RI
13. The locus of the centre of a circle touching the circle x2 + y2 – 4y – 2x = 4 internally and tangent on
which from (1, 2) is making a 60° angle with each other.
(A) (x – 1)2 + (y – 2)2 = 2 (B) (x – 1)2 + (y – 2)2 = 4

A
(C) (x + 1)2 + (y – 2)2 = 4 (D) (x + 1)2 + (y + 2)2 = 4

14. STATEMENT-1 : If three circles which are such that their centres are non-collinear, then exactly one

UH
circle exists which cuts the three circles orthogonally.
STATEMENT-2 : Radical axis for two intersecting circles is the common chord.
(A) STATEMENT-1 is true, STATEMENT-2 is true and STATEMENT-2 is correct explanation for
STATEMENT-1
JA
(B) STATEMENT-1 is true, STATEMENT-2 is true and STATEMENT-2 is not correct explanation
for STATEMENT-1
(C) STATEMENT-1 is true, STATEMENT-2 is false
(D) STATEMENT-1 is false, STATEMENT-2 is true

 3  4
LP

15. The centre of family of circles cutting the family of circles x2 + y2 + 4x  λ –  + 3y  λ –  – 6


 2   3 
(λ + 2) = 0 orthogonally, lies on
(A) x – y – 1 = 0 ( B) 4x + 3y – 6 = 0 (C) 4x + 3y + 7 = 0 (D) 3x – 4y – 1 = 0
A

16. The circle x² + y² = 4 cuts the circle x² + y² + 2x + 3y − 5 = 0 in A & B. Then the equation of the circle on
AB as a diameter is:
NK

(A) 13(x² + y²) − 4x − 6y − 50 = 0 (B) 9(x² + y²) + 8x − 4y + 25 = 0


(C) x² + y² − 5x + 2y + 72 = 0 (D) 13(x² + y²) − 4x − 6y + 50 = 0

PART-II: NUMERICAL VALUE QUESTIONS


SA

INSTRUCTION :
 The answer to each question is NUMERICAL VALUE with two digit integer and decimal upto two digit.
 If the numerical value has more than two decimal places truncate/round-off the value to TWO decimal placed.
1. Find maximum number of points having integer coordinates (both x, y integer) which can lie on a circle
with centre at ( )
2, 3 is (are)

2. If equation of smallest circle touching the circles x² + y² − 2y − 3 = 0 and x² + y² − 8x − 18y + 93 = 0 is


x2 + y2 – 4x – fy + c = 0 then value of f + c is
3. A line meets the co−ordinate axes in A and B. A circle is circumscribed about the triangle OAB. If
d1 and d2 are the distances of the tangent to the circle at the origin O from the points A and B
respectively and diameter of the circle is λ1d1 + λ2d2, then find the value of λ 1 + λ2.
4. A circle is inscribed (i.e. touches all four sides) into a rhombous ABCD with one angle 60º. The distance
from the centre of the circle to the nearest vertex is equal to 1. If P is any point of the circle, then
2 2 2 2
PA + PB + PC + PD is equal to :

5. Let x & y be the real numbers satisfying the equation x 2 − 4x + y2 + 3 = 0. If the maximum and minimum
values of x2 + y2 are M & m respectively, then find the numerical value of (M + m).
6. Find absolute value of 'c' for which the set,
{(x, y)x2 + y2 + 2x ≤ 1} ∩ {(x, y)5x − 12y + c ≥ 0} contains only one point is common.

RI
7. A rhombus is inscribed in the region common to the two circles x² + y² − 4x − 12 = 0 and
x² + y² + 4x − 12 = 0 with two of its vertices on the line joining the centres of the circles then area of the
rhombus is
If (α, β) is a point on the circle whose centre is on the x-axis and which touches the line x + y = 0 at

A
8.
(2, –2), then find the greatest value of ‘α’ is
9. Two circles whose radii are equal to 4 and 8 intersect at right angles, then length of their common

UH
chord is

10. A variable circle passes through the point A (a, b) & touches the x−axis and the locus of the other end
of the diameter through A is (x − a)² = λby , then find the value of λ
11. Let A be the centre of the circle x² + y² − 2x − 4y − 20 = 0. Suppose that the tangents at the points
JA
B (1, 7) & D (4, − 2) on the circle meet at the point C. Find the area of the quadrilateral ABCD.
12. If the complete set of values of a for which the point (2a, a + 1) is an interior point of the larger segment
of the circle x2 + y2 − 2x − 2y − 8 = 0 made by the chord whose equation is 3x − 4y + 5 = 0 is (p,q) then
value of p + q is
13. The circles x2 + y2 + 2ax + cy + a = 0 and x2 + y2 – 3ax + dy – 1 = 0 intersect in two distinct points P and
LP

Q, then find the number of values of ‘a’ for which the line 5x + by – a = 0 passes through P and Q.

14. The circumference of the circle x2 + y2 − 2x + 8y − q = 0 is bisected by the circle x2 + y2 + 4x + 12y +


p = 0, then find p + q
A

15. A circle touches the line y = x at a point P such that OP = 4 2 where O is the origin. The circle
contains the point (−10, 2) in its interior and the length of its chord on the line x + y = 0 is 6 2 . If the
equation of the circle x2 + y2 + 2g x + 2fy + 3c = 0, then value of g + f + c is
NK

PART - III : ONE OR MORE THAN ONE OPTIONS CORRECT TYPE


1. The equation of circles passing through (3, –6) touching both the axes is
(A) x2 + y2 – 6x + 6y + 9 = 0 (B) x2 + y2 + 6x – 6y + 9 = 0
SA

(C) x2 + y2 + 30x – 30y + 225 = 0 (D) x2 + y2 – 30x + 30y + 225 = 0

2. Equations of circles which pass through the points (1, –2) and (3, – 4) and touch the x-axis is
(A) x2 + y2 + 6x + 2y + 9 = 0 (B) x2 + y2 + 10x + 20y + 25 = 0
(C) x2 + y2 – 6x + 4y + 9 = 0 (D) x2 + y2 + 10x + 20y – 25 = 0

3. The centre of a circle passing through the points (0, 0), (1, 0) & touching the circle x 2 + y2 = 9 is :

3 1 1   1 1 1 
(A)  ,  (B)  , 2  (C)  ,  (D)  , − 2 
2 2 2  2 2 2 
x y
4. The equation of the circle which touches both the axes and the line + = 1 and lies in the first
3 4
quadrant is (x – c)2 + (y – c)2 = c2 where c is
(A) 1 (B) 2 (C) 4 (D) 6

5. Find the equations of straight lines which pass through the intersection of the lines x − 2y − 5 = 0,
7x + y = 50 & divide the circumference of the circle x² + y² = 100 into two arcs whose lengths are in the
ratio 2 : 1.
(A) 3x – 4y − 25 = 0 (B) 4x + 3y − 25 = 0 (C) 4x − 3y − 25 = 0 (D) 3x + 4y − 25 = 0

RI
6. Tangents are drawn to the circle x2 + y2 = 50 from a point ‘P’ lying on the x-axis. These tangents meet the
y-axis at points ‘P1’ and ‘P2’. Possible coordinates of ‘P’ so that area of triangle PP1P2 is minimum, is/are

(A) (10, 0) (B) (10 2 , 0) (C) (–10, 0) (D) (– 10 2 , 0)

A
7. If (a, 0) is a point on a diameter segment of the circle x 2 + y2 = 4, then x2 – 4x – a2 = 0 has

UH
(A) exactly one real root in (– 1, 0] (B) Exactly one real root in [2, 5]
(C) distinct roots greater than-1 (D) Distinct roots less than 5

8. The tangents drawn from the origin to the circle x2 + y2 – 2rx – 2hy + h2 = 0 are perpendicular if
JA
(A) h = r (B) h = – r (C) r2 + h2 = 1 (D) r2 = h2

9. The equation (s) of the tangent at the point (0, 0) to the circle where circle makes intercepts of length
2a and 2b units on the coordinate axes, is (are) -
(A) ax + by = 0 (B) ax – by = 0 (C) x = y (D) bx + ay = ab
LP

10. Consider two circles C1 : x2 + y2 – 1 = 0 and C2 : x2 + y2 – 2 = 0. Let A(1,0) be a fixed point on the circle
C1 and B be any variable point on the circle C2. The line BA meets the curve C2 again at C.
Which of the following alternative(s) is/are correct?
A

(A) OA2 + OB2 + BC2 ∈ [7, 11], where O is the origin.


(B) OA2 + OB2 + BC2 ∈ [4, 7], where O is the origin.
NK

1
(C) Locus of midpoint of AB is a circle of radius .
2
π
(D) Locus of midpoint of AB is a circle of area .
2
SA

11. One of the diameter of the circle circumscribing the rectangle ABCD is x – 3y + 1 = 0.
If two verticles of rectangle are the points (– 2, 5) and (6, 5) respectively, then which of the following
hold(s) good?
(A) Area of rectangle ABCD is 64 square units.
(B) Centre of circle is (2, 1)
(C) The other two vertices of the rectangle are (– 2, – 3) and (6, – 3)
(D) Equation of sides are x = – 2, y = – 3, x = 5 and y = 6.

12. Three concentric circles of which the biggest is x2 + y2 = 1, have their radii in A.P. If the line y = x + 1
cuts all the circles in real and distinct points.The permissible values of common difference of A.P. is/are
(A) 0.4 (B) 0.6 (C) 0.01 (D) 0.1
13. If 4² − 5m² + 6 + 1 = 0. Prove that x + my + 1 = 0 touches a definite circle, then which of the following
is/are true.

(A) Centre (0, 3) (B) centre (3, 0) (C) Radius 5 (D) Radius 5

14. If the circle C1: x² + y² = 16 intersects another circle C2 of radius 5 in such a manner that the common

chord is of maximum length and has a slope equal to 3/4, then the co−ordinates of the centre of C 2 are:

 9 12   9 −12   −9 −12   −9 +12 


(A)  ,  (B)  , (C)  , (D)  ,
5 5  5 5   5 5   5 5 

RI
15. For the circles x2 + y2 – 10x + 16y + 89 – r2 = 0 and x2 + y2 + 6x – 14y + 42 = 0 which of the following
is/are true.
(A) Number of integral values of r are 14 for which circles are intersecting.

A
(B) Number of integral values of r are 9 for which circles are intersecting.
(C) For r equal to 13 number of common tangents are 3.

UH
(D) For r equal to 21 number of common tangents are 2.

16. Which of the following statement(s) is/are correct with respect to the circles S 1 ≡ x2 + y2 – 4 = 0

and S2 ≡ x2 + y2 – 2x – 4y + 4 = 0?
JA
(A) S1 and S2 intersect at an angle of 90°.

6 8
(B) The point of intersection of the two circle are (2, 0) and  ,  .
5 5
4
(C) Length of the common of chord of S1 and S2 is .
5
LP

(D) The point (2, 3) lies outside the circles S1 and S2.

17. Two circles, each of radius 5 units, touch each other at (1, 2). If the equation of their common tangent is
4x + 3y = 10. The equations of the circles are
A

(A) x² + y² + 6x + 2y − 15 = 0 (B) x² + y² − 10x − 10y + 25 = 0


(C) x² + y² – 6x + 2y − 15 = 0 (D) x² + y² − 10x + 10y + 25 = 0
NK

18. x2 + y2 = a2 and (x – 2a)2 + y2 = a2 are two equal circles touching each other. Find the equation of circle
(or circles) of the same radius touching both the circles.

(A) x2 + y2 + 2ax + 2 3 ay + 3a2 = 0 (B) x2 + y2 – 2ax + 2 3 ay + 3a2 = 0


SA

(C) x2 + y2 + 2ax – 2 3 ay + 3a2 = 0 (D) x2 + y2 – 2ax – 2 3 ay + 3a2 = 0

19. The circle x2 + y2 − 2 x − 3 k y − 2 = 0 passes through two fixed points, (k is the parameter)

(
(A) 1 + 3, 0 ) (
(B) − 1 + 3, 0 ) (
(C) − 3 − 1, 0 ) (
(D) 1 − 3, 0 )
20. Curves ax2 + 2hxy + by2 – 2gx – 2fy + c = 0 and a′x2 – 2hxy + (a′ + a – b)y2 – 2g′x – 2f ′y + c = 0 intersect at

 g′ + g f ′ + f 
four concyclic point A, B, C and D. If P is the point  ,  , then which of the following is/are true
 a′ + a a′ + a 

(A) P is also concyclic with points A, B, C, D (B) PA, PB, PC in G.P.


(C) PA2 + PB2 + PC2 = 3PD2 (D) PA, PB, PC in A.P.
PART - IV : COMPREHENSION
Comprehension # 1 (Q. No. 1 to 3) 
Let S1, S2, S3 be the circles x2 + y2 + 3x + 2y + 1 = 0, x2 + y2 – x + 6y + 5 = 0 and x2 + y2 + 5x – 8y + 15 = 0, then
1. Point from which length of tangents to these three circles is same is
(A) (1, 0) (B) (3, 2) (C) (10, 5) (D) (– 2, 1)

2. Equation of circle S4 which cut orthogonally to all given circle is


(A) x2 + y2 – 6x + 4y – 14 = 0 (B) x2 + y2 + 6x + 4y – 14 = 0
(C) x2 + y2 – 6x – 4y + 14 = 0 (D) x2 + y2 – 6x – 4y – 14 = 0

RI
3. Radical centre of circles S1, S2, & S4 is

 3 8  4 3
(A)  – ,–  (B) (3, 2) (C) (1, 0) (D)  – , – 

A
 5 5  5 2

Comprehension # 2 (Q. No. 4 to 6)

UH
Two circles are S1 ≡ (x + 3)2 + y2 = 9

S2 ≡ (x – 5)2 + y2 = 16
with centres C1 & C2
4. A direct common tangent is drawn from a point P (on x-axis) which touches S1 & S2 at Q & R,
JA
respectively. Find the ratio of area of ∆PQC1 & ∆PRC2.
(A) 3 : 4 (B) 9 : 16 (C) 16 : 9 (D) 4 : 3

5. From point 'A' on S2 which is nearest to C1, a variable chord is drawn to S1. The locus of mid point of the
LP

chord.
(A) circle (B) Diameter of s1
(C) Arc of a circle (D) chord of s1 but not diameter
A

6. Locus obtained in question 5 cuts the circle S1 at B & C, then line segment BC subtends an angle on
the major arc of circle S1 is
NK

3 π 4 π 1 3 π 4
(A) cos–1 (B) – tan–1 (C) – tan–1 (D) cot–1  
4 2 3 2 2 4 2 3
SA
 Marked questions are recommended for Revision.

PART - I : JEE (ADVANCED) / IIT-JEE PROBLEMS (PREVIOUS YEARS)


1. Two parallel chords of a circle of radius 2 are at a distance 3 + 1 apart. If the chords subtend at the
π 2π
center, angles of and , where k > 0, then the value of [k] is
k k
[Note : [k] denotes the largest integer less than or equal to k] [IIT-JEE - 2010, Paper-2, (3, 0), 79]

2. The circle passing through the point (–1, 0) and touching the y-axis at (0, 2) also passes through the

RI
point [IIT-JEE 2011, Paper-2, (3, –1), 80]
 3   5   3 5
(A)  − , 0  (B)  − , 2  (C)  − ,  (D) (–4, 0)
 2   2   2 2

A
3. The straight line 2x – 3y = 1 divides the circular region x2 + y2 ≤ 6 into two parts.
 3   5 3   1 1   1 1 
If S =  2,  ,  ,  ,  , −  ,  ,   , [IIT-JEE 2011, Paper-2, (4, 0), 80]
 4   2 4   4 4   8 4 

UH
then the number of point(s) in S lying inside the smaller part is

4. The locus of the mid-point of the chord of contact of tangents drawn from points lying on the straight
line 4x – 5y = 20 to the circle x2 + y2 = 9 is [IIT-JEE 2012, Paper-1, (3, –1), 70]
JA
(A) 20(x2 + y2) – 36x + 45y = 0 (B) 20(x2 + y2) + 36x – 45y = 0
(C) 36(x2 + y2) – 20x + 45y = 0 (D) 36(x2 + y2) + 20x – 45y = 0

Paragraph for Question Nos. 5 to 6


A tangent PT is drawn to the circle x2 + y2 = 4 at the point P( 3 , 1). A straight line L, perpendicular to
LP

PT is a tangent to the circle (x – 3)2 + y2 = 1. [IIT-JEE 2012, Paper-2, (3, –1), 66]

5. A common tangent of the two circles is


(A) x = 4 (B) y = 2 (C) x + 3y=4 (D) x + 2 3y=6
A

6. A possible equation of L is
(A) x – 3y=1 (B) x + 3y=1 (C) x – 3 y = –1 (D) x + 3 y=5
NK

7*. Circle(s) touching x-axis at a distance 3 from the origin and having an intercept of length 2 7 on y-axis
is (are) [JEE (Advanced) 2013, Paper-2, (3, –1)/60]
(A) x + y – 6x + 8y + 9 = 0
2 2
(B) x + y2 – 6x + 7y + 9 = 0
2

(C) x2 + y2 – 6x – 8y + 9 = 0 (D) x2 + y2 – 6x – 7y + 9 = 0
SA

8*. A circle S passes through the point (0, 1) and is orthogonal to the circles (x – 1) 2 + y2 = 16
and x2 + y2 = 1. Then [JEE (Advanced) 2014, Paper-1, (3, 0)/60]
(A) radius of S is 8 (B) radius of S is 7
(C) centre of S is (–7, 1) (D) centre of S is (–8, 1)

9*. The circle C1 : x2 + y2 = 3, with centre at O, intersects the parabola x2 = 2y at the point P in the first
quadrant. Let the tangent to the circle C1 at P touches other two circles C2 and C3 at R2 and R3,

respectively. Suppose C2 and C3 have equal radii 2 3 and centres Q2 and Q3, respectively. If Q2 and
Q3 lie on the y-axis, then [JEE (Advanced) 2016, Paper-1, (4, –2)/62]

(A) Q2Q3 = 12 (B) R2R3 = 4 6


(C) area of the triangle OR2R3 is 6 2 (D) area of the triangle PQ2Q3 is 4 2

10*. Let RS be the diameter of the circle x2 + y2 = 1, where S is the point (1, 0). Let P be a variable point
(other than R and S) on the circle and tangents to the circle at S and P meet at the point Q. The normal
to the circle at P intersects a line drawn through Q parallel to RS at point E. Then the locus of E passes
through the point(s) [JEE (Advanced) 2016, Paper-1, (4, –2)/62]
1 1   1 1 1 1  1 1
(A)  3 ,  (B)  4 , 2  (C)  3 , – 
 (D)  4 , – 2 
 3    3  

11. For how many values of p, the circle x2 + y2 + 2x + 4y – p = 0 and the coordinate axes have exactly

RI
three common points? [JEE(Advanced) 2017, Paper-1,(3, 0)/61]

A
PARAGRAPH “X” [JEE(Advanced) 2018, Paper-1, (3, –1)/60]
Let S be the circle in the xy-plane defined by the equation x 2 + y2 = 4.
(There are two questions based on PARAGRAPH “X”, the question given below is one of them)

UH
12. Let E1E2 and F1F2 be the chords of S passing through the point P0(1, 1) and parallel to the x-axis and
the y-axis, respectively. Let G1G2 be the chord of S passing through P0 and having slope –1. Let the
tangents to S at E1 and E2 meet at E3, the tangents to S at F1 and F2 meet at F3, and the tangents to S
at G1 and G2 meet at G3. Then, then, the points E3, F3, and G3 lie on the curve
JA
(A) x +y = 4 (B) (x – 4)2 + (y – 4)2 = 16 (C) (x – 4)(y – 4) = 4 (D) xy = 4

13. Let P be a point on the circle S with both coordinates being positive. Let the tangent to S at P intersect
the coordinate axes at the points M and N. Then, the mid-point of the line segment MN must lie on the
LP

curve
(A) (x + y)2 = 3xy (B) x2/3 + y2/3 = 24/3 (C) x2 + y2 = 2xy (D) x2 + y2 = x2y2

14*. Let T be the line passing through the points P(–2, 7) and Q(2, –5). Let F 1 be the set of all pairs of circles
A

(S1, S2) such that T is tangent to S1 at P and tangent to S2 at Q, and also such that S1 and S2 touch
each other at a point, say, M. Let E1 be the set representing the locus of M as the pair (S1, S2) varies in
F1. Let the set of all straight line segments joining a pair of distinct points of E 1 and passing through the
NK

point R(1, 1) be F2. Let E2 be the set of the mid-points of the line segments in the set F 2. Then, which of
the following statement(s) is (are) TRUE [JEE(Advanced) 2018, Paper-2, (4, –2)/60]

4 7
(A) The point (–2, 7) lies in E1 (B) The point  ,  does NOT lie in E2
5 5
SA

1   3
(C) The point  ,1 lies in E2 (D) The point  0,  does NOT lie in E1
2   2

PART - II : JEE (MAIN) / AIEEE PROBLEMS (PREVIOUS YEARS)


1. The circle x2 + y2 = 4x + 8y + 5 intersects the line 3x – 4y = m at two distinct points if
[AIEEE 2010, (4, –1), 144]
(1) – 35 < m < 15 (2) 15 < m < 65 (3) 35 < m < 85 (4) – 85 < m < – 35

2. The two circles x2 + y2 = ax and x2 + y2 = c2(c > 0) touch each other if:
[AIEEE-2011, I, (4, –1), 120]
(1) 2|a| = c (2) |a| = c (3) a = 2c (4) |a| = 2c
3. The equation of the circle passing through the point (1, 0) and (0, 1) and having the smallest radius is -
(1) x2 + y2 – 2x – 2y + 1 = 0 (2) x2 + y2 – x – y = 0 [AIEEE-2011, II, (4, –1), 120]
(3) x2 + y2 + 2x + 2y – 7 = 0 (4) x2 + y2 + x + y – 2 = 0

4. The length of the diameter of the circle which touches the x-axis at the point (1, 0) and passes through
the point (2, 3) is : [AIEEE- 2012, (4, –1), 120]
10 3 6 5
(1) (2) (3) (4)
3 5 5 3
5. The circle passing through (1, –2) and touching the axis of x at (3, 0) also passes through the point
[AIEEE - 2013, (4, –1),120]
(1) (–5, 2) (2) (2, – 5) (3) (5, – 2) (4) (–2, 5)

RI
6. Let C be the circle with centre at (1, 1) and radius = 1. If T is the circle centred at (0, y), passing through
origin and touching the circle C externally, then the radius of T is equal to :
[JEE(Main) 2014, (4, – 1), 120]

A
1 1 3 3
(1) (2) (3) (4)
2 4 2 2

7. Locus of the image of the point (2, 3) in the line (2x – 3y + 4) + k (x – 2y + 3) = 0, k ∈ R, is a

UH
[JEE(Main) 2015, (4, – 1), 120]
(1) straight line parallel to x-axis (2) straight line parallel to y-axis
(3) circle of radius 2 (4) circle of radius 3

8. The number of common tangents to the circles x2 + y2 – 4x –6y – 12 = 0 and x2 + y2 + 6x + 18y + 26 = 0,


JA
is [JEE(Main) 2015, (4, – 1), 120]
(1) 1 (2) 2 (3) 3 (4) 4
9. The centres of those circles which touch the circle, x2 + y2 – 8x – 8y – 4 = 0, externally and also touch
the x-axis, lie on : [JEE(Main) 2016, (4, – 1), 120]
(1) an ellipse which is not a circle (2) a hyperbola
(3) a parabola (4) a circle
LP

10. If one of the diameters of the circle, given by the equation, x 2 + y2 – 4x + 6y – 12 = 0, is a chord of a
circle S, whose centre is at (– 3, 2), then the radius of S is : [JEE(Main) 2016, (4, – 1), 120]
(1) 5 3 (2) 5 (3) 10 (4) 5 2
A

11. Let the orthocenter and centroid of a triangle be A (–3, 5) and B(3,3) respectively. If C is the
circumcentre of this triangle, then the radius of the circle having line segement AC as diameter, is :
NK

[JEE(Main) 2018, (4, – 1), 120]


5 3 5
(1) 3 (2) (3) 10 (4) 2 10
2 2

12. Three circles of radii, a, b, c (a < b < c) touch each other externally, If they have x-axis as a common
SA

tangent, then : [JEE(Main) 2019, Online (09-01-19),P-1 (4, – 1), 120]


1 1 1 1 1 1
(1) a, b, c are in A.P. (2) = + (3) a, b, c are in A.P. (4) = +
a b c b a c

13. If a circle C passing through the point (4, 0) touches the circle x 2 + y2 + 4x – 6y = 12 externally at the
point (1, –1), then the radius of C is: [JEE(Main) 2019, Online (10-01-19),P-1 (4, – 1), 120]

(1) 2 5 (2) 57 (3) 4 (4) 5

14. If a variable line, 3x + 4y – λ = 0 is such that the two circles x 2 + y2 – 2x –2y + 1 = 0 and
x2 + y2 –18x –2y +78 = 0 are on its opposite sides, then the set of all values of λ is the interval:
[JEE(Main) 2019, Online (12-01-19),P-1 (4, – 1), 120]
(1) (2, 17) (2) [12, 21] (3) [13, 23] (4) (23,31)

15. If a circle of radius R passes through the origin O and intersects the coordinate axes at A and B, then
the locus of the foot of perpendicular from O on AB is -
[JEE(Main) 2019, Online (12-01-19),P-2 (4, – 1), 120]
(1) (x2+y2) (x + y) = R2xy (2) (x2 + y2)3 = 4R2x2y2
(3) (x2 + y2)2 = 4Rx2y2 (4) (x2 + y2)2 = 4R2x2y2
16. The sum of the squares of the lengths of the chords intercepted on the circle, x 2 + y2 = 16, by the lines,
x + y =n, n ∈ N, where N is the set of all natural numbers, is:
[JEE(Main) 2019, Online (08-04-19),P-1 (4, – 1), 120]

RI
(1) 105 (2) 210 (3) 320 (4) 160

17. If a tangent to the circle x2 + y2 = 1 intersects the coordinate axes at distinct points P and Q, then the
locus of the mid-point of PQ is : [JEE(Main) 2019, Online (09-04-19),P-1 (4, – 1), 120]

A
(1) x2 + y2 – 4x2y2 = 0 (2) x2 + y2 – 16x2y2 = 0 (3) x2 + y2 – 2x2y2 = 0 (4) x2 + y2 – 2xy = 0

18. The locus of the centres of the circles, which touch the circle, x 2 + y2 = 1 externally, also touch the

UH
y-axis and lie in the first quadrant, is : [JEE(Main) 2019, Online (10-04-19),P-2 (4, – 1), 120]

(1) x = 1 + 4 y , y ≥ 0 (2) y = 1 + 4 x , x ≥ 0 (3) x = 1 + 2y , y ≥ 0 (4) y = 1 + 2x , x ≥ 0


JA
19. If a line, y = mx + c is a tangent to the circle, (x – 3)2 + y2 = 1 and it is perpendicular to a line L1, where
 1 1 
L1 is the tangent to the circle, x2 + y2 = 1 at the point  ,  ; then:
 2 2

[JEE (Main) 2020, Online (08-01-20),P-2 (4, –1), 120]


LP

(1) c2 + 7c + 6 = 0 (2) c2 + 6c + 7 = 0 (3) c2 – 6c + 7 = 0 (4) c2 – 7c + 6 = 0

20. If the curves, x2 – 6x + y2 + 8 = 0 and x2 – 8y + y2 + 16 – k = 0, (k > 0) touch each other at a point, then
the largest value of k is ________ [JEE(Main) 2020, Online (09-01-20),P-2 (4, 0), 120]
A
NK
SA
 Marked Questions may have for Revision Questions.

1. Find the equation of the circle passing through the points A(4, 3), B(2, 5) and touching the axis of
y. Also find the point P on the y−axis such that the angle APB has largest magnitude.

2. Let a circle be given by 2x (x − a) + y (2y − b) = 0, (a ≠ 0, b ≠ 0). Find the condition on a & b if two

 b
chords, each bisected by the x−axis, can be drawn to the circle from  a , 

RI
 2

3. A circle is described to pass through the origin and to touch the lines x = 1, x + y = 2. Prove that the

(
radius of the circle is a root of the equation 3 − 2 2 ) t2 − 2

A
2 t + 2 = 0.

4. If (a, α) lies inside the circle x2 + y2 = 9 : x2 – 4x – a2 = 0 has exactly one root in (– 1, 0), then find the

UH
area of the region in which (a, α) lies.

5. Let S ≡ x2 + y2 + 2gx + 2fy + c = 0 be a given circle. Find the locus of the foot of the perpendicular
drawn from the origin upon any chord of S which subtends right angle at the origin.
JA
6. A ball moving around the circle x² + y² − 2x − 4y − 20 = 0 in anti−clockwise direction leaves it
tangentially at the point P(−2, −2). After getting reflected from a straight line it passes through the centre
5
of the circle. Find the equation of this straight line if its perpendicular distance from P is . You can
2
LP

assume that the angle of incidence is equal to the angle of reflection.

7. The lines 5x + 12 y − 10 = 0 and 5x − 12y − 40 = 0 touch a circle C 1 of diameter 6 unit. If the centre of

C1 lies in the first quadrant, find the equation of the circle C2 which is concentric with C1 and cuts of
A

intercepts of length 8 on these lines.

8. The chord of contact of tangents drawn from a point on the circle x 2 + y2 = a2 to the circle
NK

x2 + y2 = b2 touches the circle x2 + y2 = c2. Show that a, b, c are in G.P.

9. Find the locus of the middle points of chords of a given circle x 2 + y2 = a2 which subtend a right angle at
the fixed point (p, q).
SA

10. Let a 2 − bm2 + 2 d + 1 = 0, where a, b, d are fixed real numbers such that a + b = d 2. If the line
x + my + 1 = 0 touches a fixed circle then find the equation of circle

11. The centre of the circle S = 0 lies on the line 2x − 2y + 9 = 0 and S = 0 cuts orthogonally the circle
x² + y² = 4. Show that circle S = 0 passes through two fixed points and also find their co−ordinates.

12. Prove that the two circles which pass through the points (0, a), (0, − a) and touch the straight line y = m
x + c will cut orthogonaly if c2 = a2 (2 + m2).

13. Consider points A ( 13, 0) and B (2 13, 0) lying on x-axis. These points are rotated in an-
2
anticlockwise direction about the origin through an angle of tan–1   . Let the new position of A and B
3
2 13
be A′ and B′ respectively. With A′ as centre and radius a circle C1 is drawn and with B′ as a
3
13
centre and radius circle C2 is drawn. Find radical axis of C1 and C2.
3
14. P(a, b) is a point in the first quadrant. If the two circles which pass through P and touch both the
co−ordinate axes cut at right angles, then find condition in a and b.

15. Prove that the square of the tangent that can be drawn from any point on one circle to another circle is
equal to twice the product of perpendicular distance of the point from the radical axis of two circles and
distance between their centres.

RI
16. Find the equation of the circle which cuts each of the circles, x² + y² = 4, x² + y² − 6x − 8y + 10 = 0
& x² + y² + 2x − 4y − 2 = 0 at the extremities of a diameter.

A
17. Show that if one of the circle x2 + y2 + 2gx + c = 0 and x2 + y2 + 2g1x + c = 0 lies within the other,
then gg1 and c are both positive.

UH
18. Let ABCD is a rectangle. Incircle of ∆ABD touches BD at E. Incircle of ∆CBD toches BD at F.

If AB = 8 units, and BC = 6 units, then find length of EF.


JA
19. Let circles S1 and S2 of radii r1 and r2 respectively (r1 > r2) touches each other externally. Circle S radii r
touches S1 and S2 externally and also their direct common tangent. Prove that the triangle formed by
joining centre of S1, S2 and S is obtuse angled triangle.

20. Circles are drawn passing through the origin O to intersect the coordinate axes at point P and Q such
LP

that m. OP + n. OQ is a constant. Show that the circles pass through a fixed point.

21. A triangle has two of its sides along the axes, its third side touches the circle x2 + y2 − 2 ax − 2 ay + a2 = 0.
A

Find the equation of the locus of the circumcentre of the triangle.

22. Let S1 be a circle passing through A(0, 1), B(–2, 2) and S2 is a circle of radius 10 units such that AB is
NK

common chord of S1 and S2. Find the equation of S2.

23. The curves whose equations are


S = ax2 + 2hxy + by2 + 2gx + 2fy + c = 0
SA

S′ = a′x2 + 2h′xy + b′y2 + 2g′x + 2f′y + c′ = 0


intersect in four concyclic points then find relation in a, b, h, a′, b′, h′.

24. A circle of constant radius ‘r’ passes through origin O and cuts the axes of coordinates in points P and
Q, then find the equation of the locus of the foot of perpendicular from O to PQ.

25. The ends A , B of a fixed straight line of length ‘a’ and ends A′ and B′ of another fixed straight line of
length ‘b’ slide upon the axis of X & the axis of Y (one end on axis of X & the other on axis of Y). Find
the locus of the centre of the circle passing through A, B, A′ and B′.
EXERCISE - 1
PART - I
Section (A):
A-1. x2 + y2 = 1 A-3. x2 + y2 – 3x – 4y = 0 A-4. x2 + y2 – 4x – 4y + 4 = 0
A-5. x2 + y2 ± 6 √2y ± 6x + 9 = 0 A-6. (x + 3)2 + (y – 4)2 = 4 A-7. (36, 47)
Section (B):
B-1. 2 B-2. (1, 3), (5, 7), 4 2 B-3. x – 7y – 45 = 0
B-4. 3x – y ± 4 = 0

RI
 8 65 
B-5. 16x2 – 65y2 – 288x + 1296 = 0, tan–1   B-6. Yes
 49
 
Section (C):

A
C-1. 2x – y = 0 C-2. x + 2y – 1 = 0 C-3. (x + 4) 2 + y2 = 16
 18 
C-5. x+y+5=0 C-6.  6, − 5 
 

UH
Section (D):
D-1. x = 0, 3x + 4y = 10, y = 4 and 3y = 4x.

 33  1
D-3. 2(x2 + y2) – 7x + 2y = 0 D-4.  4 , 2 ; 4
JA
 
Section (E):
E-1. x2 + y2 – 2x – 4y = 0.
E-2. (i) (x – 1)2 + (y + 2)2 + 20 (2x – y – 4) = 0 (ii) (x – 1)2 + (y + 2)2 ± 20 (2x – y – 4) = 0
 52 23 
LP

E-4.  3 ,– 9  E-5. x2 + y2 – 17x – 19y + 50 = 0


 
PART - II
Section (A):
A-1. (D) A-2. (A) A-3. (B) A-4. (C) A-5. (D) A-6. (B) A-7. (A)
A

A-8. (C)

Section (B):
NK

B-1. (A) B-2. (B) B-3. (B) B-4. (B) B-5. (A) B-6. (B) B-7. (A)
B-8. (D) B-9. (C) B-10. (A) B-11. (C) B-12. (A) B-13. (B) B-14. (A)

Section (C):
C-1. (A) C-2. (B) C-3. (A) C-4. (C) C-5. (B) C-6. (A) C-7. (A)
SA

Section (D):
D-1. (B) D-2. (B) D-3. (A) D-4. (A)

Section (E):
E-1. (A) E-2. (B) E-3. (A) E-4. (A) E-5. (D) E-6. (A)
PART - III
1. (A) → (q),( B) → (p), (C) → (r), (D) → (s) 2. (A) → (s), (B) → (r), (C) → (q), (D) → (p)

EXERCISE - 2
PART - I
1. (C) 2. (B) 3. (D) 4. (A) 5. (A) 6. (C) 7. (B)
8. (B) 9. (B) 10. (A) 11. (B) 12. (D) 13. (B) 14. (D)
15. (B) 16. (A)

PART - II
1. 01.00 2. 32.88 or 32.89 3. 02.00 4. 11.00 5. 10.00 6. 13.38

7. 13.85 or 13.86 8. 06.82 or 06.83 9. 07.15 10. 04.00

11. 75.00 12. 01.30 13. 00.00 14. 10.00 15. 18.66 or 18.67

PART - III
1. (AD) 2. (BC) 3. (BD) 4. (AD) 5. (CD) 6. (AC) 7. (ABCD)

RI
8. (ABD) 9. (AB) 10. (ACD) 11. (ABC) 12. (CD) 13. (BC) 14. (BD)
15. (AC) 16. (ACD) 17. (AB) 18. (BD) 19. (AD) 20. (BCD)

A
PART - IV
1. (B) 2. (D) 3. (A) 4. (B) 5. (C) 6. (A)

UH
EXERCISE - 3
PART - I
1. 3 2. (D) 3. 2 4. (A) 5. (D) 6. (A) 7. (AC)
JA
8. (BC) 9. (ABC) 10. (A, C) 11. (2) 12. (A) 13. (D) 14. (BD)

PART - II
1. (1) 2. (2) 3. (2) 4. (1) 5. (3) 6. (2) 7. (3)

8. (3) 9. (3) 10. (1) 11. (1) 12. (2) 13. (4) 14. (2)
LP

15. (2) 16. (2) 17. (1) 18. (4) 19. (2) 20. 36
A
NK
SA
1. x² + y² − 4x − 6y + 9 = 0 OR x² + y² − 20x − 22y + 121 = 0, P(0, 3), θ = 45°

 9  2  
2. (a² > 2b²) 4. 4  5 + cot −1  
 2  5  

c
5. x2 + y2 + gx + fy + =0 6. (4 3 − 3) x − (4 + 3 3 ) y−(39 − 2 3 ) = 0
2

RI
7. x2 + y2 − 10 x − 4 y + 4 = 0 9. 2 x2 + 2y2 − 2 p x − 2 q y + p2 + q2 − a2 = 0

 1 1

A
10. x2 + y2 – 2dx + d2 – b = 0 11. (− 4, 4);  − , 
 2 2

UH
13. 9x + 6y = 65 14. a2 − 4ab + b2 = 0

16. x² + y² − 4x − 6y − 4 = 0 18. 2
JA
2x y
21. 2 (x + y) − a = 22. x2 + y2 + 2x – 3y + 2 ± 7 (x + 2y – 2) = 0
a

a – b a′ – b′
23. = 24. (x2 + y2)2 (x–2 + y–2) = 4r2
h h′
LP

25. (2ax − 2by)² + (2bx − 2ay)² = (a² − b²)²


A
NK
SA
CONIC SECTION

Everything should be made as simple as possible, but not simpler...... Einstein, Albert

This chapter focusses on parabolic curves, which constitutes one category of various curves obtained
by slicing a cone by a plane, called conic sections. A cone (not necessarily right circular) can be out in
various ways by a plane, and thus different types of conic sections are obtained.

Let us start with the definition of a conic section and then we will see how are they obtained by slicing a
right circular cone.

1. Definition of Conic Sections:

RI
A conic section or conic is the locus of a point which moves in a plane so that its distance from a fixed
point is in a constant ratio to its perpendicular distance from a fixed straight line.
• The fixed point is called the Focus.

A
• The fixed straight line is called the Directrix.
• The constant ratio is called the Eccentricity denoted by e.

UH
PS
=e
PM
• The line passing through the focus & perpendicular to the directrix is called the Axis.
• A point of intersection of a conic with its axis is called a Vertex.
JA
If S is (p, q) & directrix is x + my + n = 0
| x + my + n |
Then PS = (x – α )2 + (y – β)2 & PM =
 2 + m2
PS
= e ⇒ (2 + m2) [(x – p)2 + (y – q)2] = e2 (x + my + n)2
PM
LP

Which is of the form ax2 + 2hxy + by2 + 2gx + 2fy + c = 0


1.1 Section of right circular cone by different planes
A right circular cone is as shown in the figure – 1
A

A(vertex)
Generator α
NK

β Plane
Axis

B
SA

Circular base
α is angle between generator and axis.
β is angle between plane and axis

Section of a right circular cone by a plane passing through its vertex is a pair of straight lines.
Section of a right circular cone by a plane not passing through vertex is either circle or parabola
or ellipse or hyperbola which is shown in table below :
Type of 3-D view of section of right circular cone Condition of condition of conic in
conic section with plane conic in ax2 + by2 + 2hxy + 2gx +
definition of 2fy + c = 0
conic
Two distinct e > 1, focus lies a h g
real lines on directrix = 0, h2 > ab
Plane A(vertex) h b f
g f c

Generator

Q Circular base

RI
Plane passes through vertex A and
0≤β<α
Two real e = 1, focus lies a h g
same lines Plane on directrix h b f = 0, h2 = ab,

A
A (vertex) g f c
(either g2 = ac or f2 = bc)

UH
Generator

Circular base
JA
Plane passes through vertex A and β = α

Two 0 < e < 1, focus a h g


imaginary Plane A(vertex) lies on directrix = 0, h2 < ab
h b f
lines/point
g f c
LP

Generator
A

Circular base
Plane passes through vertex A and β > α
Parabola e = 1, focus a h g
does not lies
NK

A(vertex) h b f ≠ 0, h2 = ab
on directrix
g f c
Plane

Generator
SA

Circular base

Plane does not passes through vertex A


and β = α
Ellipse 0 < e < 1, focus a h g
A(vertex)
does not lies h b f ≠ 0, h2 < ab,
on directrix
Generator α g f c
(either a ≠ b or h ≠ 0)
β Plane
Axis

Circular base
Plane does not passes through
vertex A and α < β < 90
Circle e = 0, focus a h g
does not lies h b f ≠ 0,

RI
A(vertex)
on directrix
Generator α g f c
a = b, h = 0
Plane

A
Axis

UH
Circular base
Plane does not passes through
vertex A and β = 90
Hyperbola Axis e > 1, focus a h g
does not lies h b f ≠ 0, h2 > ab
Generator on directrix
JA g f c
A(vertex)

Plane does not passes through vertex A and


0≤β<α
LP

Note : (i) Pair of real parallel lines is not the part of conic but it is part of general two degree equation.
a h g
For it h b f = 0, h2 = ab, (either g2 > ac or f2 > bc)
A

g f c

⇒ General two degree equation can represent real curve other than conic section.
NK

a h g
(ii) For rectangular hyperbola h b f ≠ 0, h2 > ab, a + b = 0
g f c

2. Elementary Concepts of Parabola


SA

2.1 Definition and terminology of parabola


Y

M P

N
A X
Z S

L'

P'

2
y = 4ax
A parabola is the locus of a point, whose distance from a fixed point (focus) is equal to
perpendicular distance from a fixed straight line (directrix). Four standard forms of the parabola
are y² = 4ax; y² = − 4ax; x² = 4ay; x² = − 4ay
For parabola y² = 4ax.
(i) Vertex is (0, 0) (ii) focus is (a, 0)
(iii) Axis is y = 0 (iv) Directrix is x + a = 0
Focal Distance: The distance of a point on the parabola from the focus.
Focal Chord : A chord of the parabola, which passes through the focus.
Double Ordinate: A chord of the parabola perpendicular to the axis of the symmetry.
Latus Rectum: A double ordinate passing through the focus or a focal chord perpendicular to
the axis of parabola is called the Latus Rectum (L.R.).

RI
For y² = 4ax. ⇒ Length of the latus rectum = 4a.
⇒ ends of the latus rectum are L(a, 2a) & L’ (a, − 2a).

A
NOTE :
(i) Perpendicular distance from focus on directrix = half the latus rectum.
(ii) Vertex is middle point of the focus & the point of intersection of directrix & axis.

UH
(iii) Two parabolas are said to be equal if they have the same latus rectum.

Example # 1: Find the equation of the parabola whose focus is at (– 1, – 2) and the directrix is
x – 2y + 3 = 0.
Solution : Let P(x, y) be any point on the parabola whose focus is S(– 1, – 2) and the directrix
JA
x – 2y + 3 = 0. Draw PM perpendicular to directrix x – 2y + 3 = 0. Then by definition,
SP = PM

⇒ SP2 = PM2
2
 x − 2y + 3 
LP

⇒ (x + 1) + (y + 2) = 
2 2

 1+ 4 
⇒ 5 [(x + 1)2 + (y + 2)2] = (x – 2y + 3)2
⇒ 5(x2 + y2 + 2x + 4y + 5) = (x2 + 4y2 + 9 – 4xy + 6x – 12y)
A

⇒ 4x2 + y2 + 4xy + 4x + 32y + 16 = 0


This is the equation of the required parabola.
Example # 2 : Find the vertex, axis, focus, directrix, latusrectum of the parabola, also draw their rough
NK

sketches. x2 – 2x + 4y + 9 = 0.
Solution : The given equation is x2 – 2x + 4y + 9 = 0
⇒ (x – 1)2 = – 4(y + 2)
which of the form X2 = –4bY
Vertex -
SA

(X, Y) ≡ (0, 0) y
x=1
(x, y) ≡ (1, –2)
x
Axis (1, –2)
X=0⇒x=1
Focus-
(X, Y) = (0, –b)
(x, y) ≡ (1, –1 –2) = (1, –3)
Directrix -
Y=b⇒y+2=1
y = –1
Latusrectum -
The length of the latusrectum of the given parabola is 4b = 4.
Self Practice Problems :

(1) Find the equation of the parabola whose focus is the point (0, 0)and whose directrix is the
straight line 4x – 3y – 2 = 0.
(2) Find the extremities of latus rectum of the parabola y = x 2 – 2x + 3.
(3) Find the latus rectum & equation of parabola whose vertex is origin & directrix is x + y = 2.

(4) Find the equation of the parabola whose focus is (–1, 1) and whose vertex is (1, 2). Also find
its axis and latusrectum.
1 9 3 9
Ans. (1) 9x2 + 16y2 + 24xy + 16x – 12y – 4 = 0 (2)  ,   , 
2 4 2 4

RI
(3) 4 2 , x2 + y2 – 2xy + 8x + 8y = 0
(4) (2y – x – 3)2 = – 20 (y + 2x – 4), Axis 2y – x – 3 = 0. LL′ = 4 5 .

2.2 Parametric representation of parabola

A
The simplest & the best form of representing the co−ordinates of a point on the parabola is
(at², 2at) i.e. the equations x = at² & y = 2at together represents the parabola y² = 4ax, t being
the parameter.

UH
Parametric form for : y2 = – 4ax (–at2, 2at)
x = 4ay
2
(2at , at2)
x = – 4ay
2
(2at , – at2)
Example # 3: Find the parametric equation of the parabola (x + 1)2 = –6 (y + 2)
−3
 ⇒
Solution: 4a = – 6 a=
JA , y + 2 = at2
2
3 2
x + 1 = 2 at ⇒ x = –1 – 3t, y = –2 – t
2
Self Practice Problems:

(5) Find the parametric equation of the parabola x2 = 4a(y – 1)


LP

Ans. x = 2at, y = 1 + at2


.
2.3 Position of a point relative to a parabola:
The point (x1 , y1) lies outside, on or inside the parabola y² = 4ax according as the expression
A

y12 − 4ax1 is positive, zero or negative.


Outside

Inside
NK

P(x1, y1)

S1 : y12 – 4ax1
S1 < 0 → Inside
S1 > 0 → Outside
SA

Example # 4 : Check whether the point (4, 5) lies inside or outside the parabola y 2 = 4x.
Solution : y2 – 4x = 0
 S1 ≡ y12 – 4x1 = 25 – 16 = 9 > 0
∴ (3, 4) lies outside the parabola.
Self Practice Problems :

(6) Find the set of value's of α for which (α, – 2 – α) lies inside the parabola y 2 + 4x = 0.
Ans. α ∈ (– 4 – 2 3 , – 4 + 2 3 )
3. Elementary Concepts of Ellipse

3.1 Definition of Ellipse


It is locus of a point which moves in such a way that the ratio of its distance from a fixed point
called focus and a fixed line called directrix (not passes through fixed point and all points and
line lies in same plane) is constant (e = eccentricity), which is less than one.

Example # 5 : Find the equation to the ellipse whose focus is the point (– 1, 1), whose directrix is the straight
1
line x – y + 3 = 0 and eccentricity is .
2
Solution : Let P ≡ (h, k) be moving point,

RI
A
2
PS 1 1 h−k +3
e= = ⇒ (h + 1) + (k – 1) = 
2 2

PM 2 4  2 

UH
locus of P(h, k) is
8 {x2 + y2 + 2x – 2y + 2} = (x2 + y2 – 2xy + 6x – 6y + 9)
7x2 + 7y2 + 2xy + 10x – 10 y + 7 = 0.
Self Practice Problems :
1
(7) Find the equation to the ellipse whose focus is (0, 0) directrix is x + y – 1 = 0 and e = .
JA
2
Ans. 3x2 + 3y2 – 2xy + 2x + 2y – 1 = 0.

3.2 Standard Equation of Ellipse


Standard equation of an ellipse referred
LP
A

to its principal axes along the co−ordinate


x2 y2
NK

axes is 2 + 2 = 1, where a > b & b² = a² (1 − e²).


a b
b2
Eccentricity: e = 1 − , (0 < e < 1)
a2
Focii : S ≡ (a e, 0) & S′ ≡ (− a e, 0).
SA

a a
Equations of Directrices: x = & x = − .
e e
Major Axis: The line segment A′A in which the focii S′ & S lie is of length 2a & is called the
major axis (a > b) of the ellipse. Point of intersection of major axis with directrix is called the
foot of the directrix (Z).
Minor Axis: The y−axis intersects the ellipse in the points B′ ≡ (0, − b) & B ≡ (0, b). The line
segment B′B is of length 2b (b < a) is called the minor axis of the ellipse.
Principal Axis : The major & minor axes together are called principal axis of the ellipse.
Vertices: Point of intersection of ellipse with major axis. A′ ≡ (− a, 0) & A ≡ (a, 0) .
Focal Chord: A chord which passes through a focus is called a focal chord.
Double Ordinate: A chord perpendicular to the major axis is called a double ordinate.
Latus Rectum: The focal chord perpendicular to the major axis is called the latus rectum.
2
2b2 (minor axis )
Length of latus rectum (LL′) =
a
=
major axis
= 2a 1 − e2 ( )
= 2 e (distance from focus to the corresponding directrix)
Centre: The point which bisects every chord of the conic drawn through it, is called the centre
y2
of the conic. C ≡ (0, 0) the origin is the centre of the ellipse x2 + 2 = 1.
2

a b
y 2
If the equation of the ellipse is given as x2 + 2 = 1 and nothing is mentioned, then the rule is
2
Note : (i)
a b
to assume that a > b.
(ii) If b > a is given, then the y−axis will become major axis and x-axis will become the minor axis
and all other points and lines will change accordingly.

RI
A
x2 y2

UH
Equation : 2
+ =1
a b2
b
Foci (0, ± be) Directrices : y=±
e
a2
JA
a2 = b2 (1 – e2), a < b. ⇒ e= 1–
b2
Vertices (0, ± b) ; L.R. y = ± be
2
2a
 (L·R.) = , centre : (0, 0)
b

Example # 6: Find the equation to the ellipse whose centre is origin, axes are the axes of co-ordinate and
LP

passes through the points (2, 2) and (3, 1).


x2 y2
Solution: Let the equation to the ellipse is 2 + 2 = 1
a b
Since it passes through the points (2, 2) and (3, 1)
A

4 4 9 1
∴ 2 + 2 = 1.......... (i) and 2
+ 2 =1 .........(ii)
a b a b
from (i) – 4 (ii), we get
NK

4 − 36 32
= 1– 4 ⇒ a2 =
a2 3
from (i), we get
1 1 3 8−3 32
2
= – = ⇒ b2 =
b 4 32 32 5
SA

∴Ellipse is 3x + 5y = 32
2 2

1
Example # 7 : Find the equation of the ellipse whose focii are (4, 0) and (– 4, 0) and eccentricity is
3
Solution: Since both focus lies on x-axis, therefore x-axis is major axis and mid point of focii is origin
which is centre and a line perpendicular to major axis and passes through centre is minor axis
which is y-axis.
x2 y2
Let equation of ellipse is + =1
a2 b2
1
 ae = 4 and e = (Given)
3
∴ a = 12 and and b2 = a2 (1 – e2)
 1
⇒ b2 = 144  1 −  ⇒ b2 = 16 × 8 ⇒ b=8 2
 9
x2 y2
Equation of ellipse is + =1
144 128
Example # 8 : In the given figure find the eccentricity of the ellipse if SS’ subtends right angle at B.
B

A� S� O S A

B�

Solution: here b = ae --- (i)


in ellipse b2 = a2 – a2 e2 ----- (ii)

RI
from (i) & (ii) a2e2 = a2–a2 e2
1
2e2 = 1 ⇒ e =
2
Example # 9 : From a point Q on the circle x2 + y2 = a2, perpendicular QM are drawn to x-axis, find the locus

A
of point 'P' dividing QM in ratio 2 : 1.
Solution :

Let Q ≡ (a cosθ, a sinθ)

UH
JA
M ≡ (a cosθ, 0)
Let P ≡ (h, k)
a sin θ
∴h = a cosθ, k =
3
2 2
 3k  h x2 y2
∴  a  +a =1 ⇒ Locus of P is + =1
LP

    a2 (a / 3)2

Example # 10 : Find the equation of axes, directrix, co-ordinate of focii, centre, vertices, length of
latus - rectum and eccentricity of an ellipse 16x2 + 25y2 – 96x – 100 y + 156 = 0.
(x − 3)2 (y − 2)2
A

Solution : The given ellipse is + = 1.


25 16
X2 Y2
Let x – 3 = X, y – 2 = Y, so equation of ellipse becomes as + = 1.
NK

52 42
equation of major axis is Y = 0 ⇒ y = 2.
equation of minor axis is X = 0 ⇒ x = 3.
centre (X = 0, Y = 0) ⇒ x = 3, y = 2
C ≡ (3, 2)
SA

Length of semi-major axis a = 5


Length of major axis 2a = 10
Length of semi-minor axis b = 4
Length of minor axis = 2b = 8.
Let 'e' be eccentricity
∴ b2 = a2 (1 – e2)
a2 − b2 25 − 16 3
e= 2
= = .
a 25 5
2b2 2 × 16 32
Length of latus rectum = LL′ = = =
a 5 5
Co-ordinates focii are X = ± ae, Y = 0
⇒ S ≡ (X = 3, Y = 0) & S′ ≡ (X = –3, Y = 0)
⇒ S ≡ (6, 2) & S′ ≡ (0, 2)
Co-ordinate of vertices

Extremities of major axis A ≡ (X = a, Y = 0) & A′ ≡ (X = – a, Y = 0)


⇒ A ≡ (x = 8, y = 2) & A′ = (x = – 2, 2)
A ≡ (8, 2) & A′ ≡ (– 2, 2)
Extremities of minor axis B ≡ (X = 0, Y = b) & B′ ≡ (X = 0, Y = – b)
B ≡ (x = 3, y = 6) & B′ ≡ (x = 3, y = – 2)
B ≡ (3, 6) & B′ ≡ (3, – 2)
a 25 34 16
Equation of directrix X = ± x–3=± ⇒ x= & x=–
e 3 3 3
Self Practice Problems:

RI
(8) Find the equation to the ellipse whose axes are of lengths 6 and 2 and their equations are
x – 3y + 3 = 0 and 3x + y – 1 = 0 respectively.
(9) Find the co-ordinates of the focii of the ellipse 4x2 + 9y2 = 1.
(10) A point moves so that the sum of the squares of its distances from two intersecting lines is

A
constant (given that the lines are neither perpendicular nor they make complimentry angle).
Prove that its locus is an ellipse.
Hint. : Assume the lines to be y = mx and y = – mx.

UH
Ans. (8) 3(x – 3y + 3)2 + 2(3x+ y – 1)2 = 180, 21x2 – 6xy + 29y2 + 6x – 58y – 151 = 0.
 5 
(9)  ± , 0

 6 
JA
3.3 Auxiliary Circle / Eccentric Angle of Ellipse
A circle described on major axis of ellipse as diameter is called the auxiliary circle.
Let Q be a point on the auxiliary circle x² + y² = a² such that line through Q perpendicular to the
x − axis on the way intersects the ellipse at P, then P & Q are called as the Corresponding
Points on the ellipse & the auxiliary circle respectively. ‘θ’ is called the Eccentric Angle of the
point P on the ellipse (− π < θ ≤ π). Q ≡ (a cosθ , a sinθ)
LP
A
NK

P ≡ (a cosθ , b sinθ)
Note that :
(PN) b Semi minor axis
= =
(QN) a Semi major axis
SA

NOTE : If from each point of a circle perpendiculars are drawn upon a fixed diameter then the locus of
the points dividing these perpendiculars in a given ratio is an ellipse of which the given circle is the
auxiliary circle.
x2 y2
Example # 11 : Find the focal distance of a point P(θ) on the ellipse 2 + 2 = 1 (a > b)
a b
Solution : Let 'e' be the eccentricity of ellipse.
a 
∴ PS = e . PM = e  − acos θ 
e 
PS = (a – a e cosθ)
 a
and PS′ = e. PM′ = e  acos θ + 
 e
PS′ = a + ae cosθ
∴ focal distance are (a ± ae cosθ)
Note : PS + PS′ = 2a
PS + PS′ = AA′
x2 y2
Example # 12 : Find the distance from centre of the point P on the ellipse + = 1 whose radius makes
a2 b2
angle α with y – axis in clockwise direction.

RI
Solution : Let P ≡ (a cosθ, b sinθ)
b a
∴ m(op) = tanθ = tan(π/2 – α) ⇒ tanθ = tan (π/2 – α)
a b

A
a2 + b2 tan2 θ
OP = a2 cos2 θ + b2 sin2 θ =
sec 2 θ

UH
a2
a2 + b2 × tan2 (π /2 − α )
a + b tan θ
2 2 2
b2 ab
= = ⇒ OP=
1 + tan2 θ a2 a cos α + b2 sin2 α
2 2
1 + 2 tan (π /2 − α )
2

b
Self Practice Problems :
JA
x2 y2
(11) Find the distance from centre of the point P on the ellipse + = 1 whose eccentric
a2 b2
angle is α
x2 y2
(12) Find the eccentric angle of a point on the ellipse + = 1whose distance from the
16 9
centre is 3.
LP

(13) Show that the area of triangle inscribed in an ellipse bears a constant ratio to the area of the
triangle formed by joining points on the auxiliary circle corresponding to the vertices of the first
triangle.
π
Ans. (11) r = a2 cos2 α + b2 sin2 α (12) ±
A

3.4 Parametric Representation of Ellipse


NK

y2
The equations x = a cos θ & y = b sin θ together represent the ellipse x2 + 2 = 1.
2

a b
Where θ is a parameter. Note that if P(θ) ≡ (a cos θ, b sin θ) is on the ellipse then;
Q(θ) ≡ (a cos θ, a sin θ) is on the auxiliary circle.
SA

The equation to the chord of the ellipse joining two points with eccentric angles α & β is given
x α+β y α+β α−β
by cos + sin = cos
a 2 b 2 2
x2 y2 π  5π 
Example # 13 : Write the equation of chord of an ellipse + =1 joining two points P   and Q   .
25 16 4  4 
 π 5π  y  π 5π   π 5π 
 +   +  4− 4 
Equation of chord is cos 
4 4  4
. sin 
4 4 
= cos  
x
Solution : +
5 2 2 2
x  3π  y  3π  x y
. cos   + . sin   = 0 ⇒ – + = 0 ⇒ 4x = 5y
5  4  4  4  5 4
Example # 14 : If P(α) and P(β) are extremities of a chord of ellipse which passes through the mid-point of the
line segment joining focus & centre then prove that its eccentricity
 α −β
cos  
e = 2.  2 
α+β
cos  
 2 
x2 y2
Solution : Let the equation of ellipse is +
=1
a2 b2
x α+β y α+β  α −β
∴ equation of chord is cos   + b sin  2  = cos  2 
a  2     
above chord passes through (ae/2, 0) or (– ae/2, 0)
 α −β
cos  
α+β  α −β  2  Ans.
∴ ± e cos   = 2cos   ∴e=2

RI
 2   2  α+β
cos  
 2 
Self Practice Problems :

A
x2 y2
(14) Find the locus of the foot of the perpendicular from the centre of the ellipse + = 1 on the
a2 b2

UH
π
chord joining two points whose eccentric angles differ by .
2
Ans. (14) 2(x2 + y2)2 = a2 x2 + b2 y2.

3.5 Position of a Point w.r.t. an Ellipse :


The point P(x1, y1) lies outside, inside or on the ellipse according as S1 > 0, S1 < 0 or S1 = 0
JA
x12 y12
where S1 = + − 1.
a2 b2
x2 y2
Example # 15 : Check whether the point P(1, –1) lies inside or outside of the ellipse + = 1.
25 16
1 1
S1 ≡
LP

Solution : + –1<0
25 16
∴ Point P ≡ (1, –1) lies inside the ellipse.
x2 y2
Example # 16 : Find the set of value(s) of 'α' for which the point P(2α, – 3α) lies inside the ellipse + = 1.
16 9
A

Solution : If P(2α, – 3α) lies inside the ellipse


∴ S1 < 0
α2 α2 2 2  2 2 
⇒ ⇒ <α< ∴ α∈  −
NK

+ –1<0 – , .
4 1 5 5  5 5

4. Elementary Concepts of Hyperbola


Hyperbolic curves are of special importance in the field of science and technology especially astronomy
and space studies. In this chapter we are going to study the characteristics of such curves.
SA

4.1 Definition of Hyperbola


A hyperbola is defined as the locus of a point moving in a plane in such a way that the ratio of
its distance from a fixed point to that from a fixed line (the point does not lie on the line) is a
fixed constant greater than 1.
PS
=e>1, e – eccentricity
PM
4.2 Standard equation of Hyperbola

x2 y2
Standard equation of hyperbola is − = 1, where b2 = a2 (e2 − 1).
a2 b2
b2

RI
• Eccentricity (e) : e2 = 1 + 2
a
• Foci : S ≡ (ae, 0) & S′ ≡ (− ae, 0).

A
a a
• Equations of directrices : x= & x=− .
e e
• Transverse axis :

UH
The line segment A′A of length 2a in which the foci S′ & S both lie is called the
transverse axis of the hyperbola.
• Conjugate axis :
The line segment B′B of length 2b between the two points B′ ≡ (0, − b) & B ≡ (0, b) is
called as the conjugate axis of the hyperbola.
JA
• Principal axes :
The transverse & conjugate axis together are called principal axes of the hyperbola.
• Vertices :
A ≡ (a, 0) & A′ ≡ (− a, 0)
• Focal chord :
A chord which passes through a focus is called a focal chord.
LP

• Double ordinate :
A chord perpendicular to the transverse axis is called a double ordinate.
• Latus rectum :
Focal chord perpendicular to the transverse axis is called latus rectum. Its length () is
2
A

2b2 ( C.A.)
given by  = = = 2a (e2 − 1).
a T.A.
NK

Note : (i) Length of latus rectum = 2 e × (distance of focus from corresponding directrix)
 b2   b2   b2   b2 
(ii) End points of latus rectum are L ≡  ae,  , L′ ≡  ae, −  , M ≡  −ae,  , M′ ≡  −ae, − 
 a   a   a  a
    

• Centre:
SA

The point which bisects every chord of the conic, drawn through it, is called the centre
x2 y2
of the conic. C ≡ (0,0) the origin is the centre of the hyperbola 2 − 2 = 1 .
a b

General note :
Since the fundamental equation to hyperbola only differs from that to ellipse in having
−b2 instead of b2 it will be found that many propositions for hyperbola are derived from those for
ellipse by simply changing the sign of b2.
Example #17: Find the equation of the hyperbola whose directrix is x + 2y = 1, focus (2,1) and eccentricity
3.
Solution: Let P(x,y) be any point on the hyperbola.
Draw PM perpendicular from P on the directrix.
Then by definition SP = e PM
⇒ (SP)2 = e2 (PM)2
2
 x + 2y − 1
⇒ (x – 2)2 + (y – 1)2 = 3   ⇒ 2x2 – 7y2 – 12xy – 14x + 2y + 22 = 0
 4 +1 
Which is the required hyperbola.
Example # 18: Find the eccentricity of the hyperbola whose latus rectum is half of its transverse axis.
x2 y2
Solution: Let the equation of hyperbola be 2 – 2 = 1.
a b
2b2 2b2 1
Then transverse axis = 2a and latus–rectum = . According to question = (2a)
a a 2
⇒ 2b2 = a2 ( b2 = a2 (e2 – 1))
3
⇒ ⇒ ⇒

RI
2a2 (e2 – 1) = a2 2e2 – 2 = 1 e2 =
2
3 3
∴ e= Hence the required eccentricity is .
2 2

A
4.3 Conjugate hyperbola :
Two hyperbolas such that transverse & conjugate axes of one hyperbola are respectively the

UH
conjugate & the transverse axes of the other are called conjugate hyperbolas of each other.

x2 y2 x2 y2
eg. − =1 & − + = 1 are conjugate hyperbolas of each other.
a2 b2 a2 b2
JA
LP

y2 x2
Equation : 2
– =1
b a2
a2
a2 = b2 (e2 – 1) ⇒ e= 1+
b2
A

2a2
Vertices(0, ± b) ;  (L.R.) =
b
NK

Note : (a) If e1 & e2 are the eccentrcities of the hyperbola & its conjugate then e1−2 + e2−2 = 1.

(b) The foci of a hyperbola and its conjugate are concyclic and form the vertices of a
square.
(c) Two hyperbolas are said to be similar if they have the same eccentricity.
SA

(d) Two similar hyperbolas are said to be equal if they have same latus rectum.
(e) If a hyperbola is equilateral then the conjugate hyperbola is also equilateral.
Example # 19 : Find the lengths of transverse axis and conjugate axis, eccentricity, the co-ordinates of foci,
vertices, length of the latus-rectum and equations of the directrices of the following hyperbola
16x2 – 9y2 = – 144.
x2 y2
Solution : The equation 16x2 – 9y2 = –144 can be written as – =–1
9 16
x2 y2
This is of the form – =–1
a2 b2
∴ a2 = 9, b2 = 16 ⇒ a = 3, b = 4
Length of transverse axis : The length of transverse axis = 2b = 8
Length of conjugate axis : The length of conjugate axis = 2a = 6
 a2   9  5
Eccentricity : e =  1 + 2  =  1 + 16  = 4
 b   
Foci : The co-ordinates of the foci are (0, + be) i.e., (0, + 5)

Vertices : The co–ordinates of the vertices are (0, + b) i.e., (0, + 4)


2a2 2(3)2 9
Length of latus–rectum : The length of latus–rectum = = =
b 4 2
Equation of directrices : The equation of directrices are
b 4 16
y=+ ⇒ y=+ ⇒ y=+
e (5 / 4) 5

RI
Self Practice Problems :
(15) Find the equation of the hyperbola whose foci are (6, 4) and (– 4, 4) and eccentricity is 2.
(16) Obtain the equation of a hyperbola with coordinates axes as principal axes given that the

A
distances of one of its vertices from the foci are 9 and 1 units.
x2 y2
(17) The foci of a hyperbola coincide with the foci of the ellipse + = 1. Find the equation of
25 9

UH
the hyperbola if its eccentricity is 2.
x2 y2 y2 x2
Ans. (15) 12x2 – 4y2 – 24x + 32y – 127 = 0 (16) – = 1, – =1
16 9 16 9
(17) 3x2 – y2 – 12 = 0.
4.4 Auxiliary Circle of Hyperbola
JA
A circle drawn with centre C and transverse axis as a diameter is called the auxiliary circle of the
hyperbola. Equation of the auxiliary circle is x2 + y2 = a2.

Note from the following figure that P & Q are called the "corresponding points" of the hyperbola & the
auxiliary circle.
LP
A
NK

4.5 Parametric representation of Hyperbola


x2 y2
The equations x = a sec θ & y = b tan θ together represent the hyperbola 2
− = 1 where
a b2
θ is a parameter.
SA

Note that if P(θ) ≡ (a sec θ, b tan θ) is on the hyperbola then,


Q(θ) ≡ (a cos θ, a sin θ) is on the auxiliary circle.
The equation to the chord of the hyperbola joining the two points P(α) & Q(β) is given by
x α −β y α+β α+β
. cos − sin = cos
a 2 b 2 2
4.6 Position of a point 'P' w.r.t. a hyperbola :
2 2
x1 y1
The quantity S1 ≡ − − 1 is positive, zero or negative according as the point (x1, y1) lies
a2 b2
inside, on or outside the curve.

Example # 20 : Find the position of the point (5, – 4) relative to the hyperbola 9x 2 – y2 = 1.
Solution : Since 9 (5)2 – (–4)2 – 1 = 225 – 16 – 1 = 208 > 0,
So the point (5,–4) lies inside the hyperbola 9x2 – y2 = 1..
5. Rectangular hyperbola (equilateral hyperbola) :
The particular kind of hyperbola in which the lengths of the transverse & conjugate axis are equal is
called an Equilateral Hyperbola. Note that the eccentricity of the rectangular hyperbola is.

Since a = b
equation becomes x2 – y2 = a2
whose asymptotes are y = ± x.
b2
e= 1+ = 1+ 1 = 2
a2

RI
A
Rotation of this system through an angle of 45° in clockwise direction gives another form to the
equation of rectangular hyperbola.
a2
which is xy = c2 where c2 = .

UH
2
It is referred to its asymptotes as axes of co−ordinates.
Vertices : (c, c) & (− c, − c);
Foci : ( ) (
2 c, 2 c & − 2 c,− 2 c , )
JA
Directrices : x + y = ± 2c
Latus Rectum (l ) :  = 2 2 c = T.A. = C.A.
Parametric equation x = ct, y = c/t, t ∈ R – {0}

Example # 21 : A triangle has its vertices on a rectangular hyperbola. Prove that the orthocentre of the triangle
also lies on the same hyperbola.
Solution : Let "t1", "t2" and "t3" are the vertices of the triangle ABC, described on the rectangular hyperbola
LP

xy = c2.
 c  c   c 
∴ Co–ordinates of A,B and C are  ct1,  ,  ct 2 ,  and  ct 3 ,  respectively
 t1  t 2   t 3 
A

c(t 3 − t 2 ) 1
Now slope of BC is =–
c (t 2 − t 3 ) t 2 t 3 t2 t3
NK

∴ Slope of AD is t2t3
c
Equation of Altitude AD is y – = t2t3(x – ct1)
t1
or t1y – c = x t1t2t3 – ct12t2t3 .....(1)
SA

Similarly equation of altitude BE is


t2y – c = x t1t2t3 – ct1t22t3 ......(2)
Solving (1) and (2),
 c 
we get the orthocentre  − , −ct1t 2 t 3  Which lies on xy = c2.
t t
 123 t 
6. Line & a parabola :
The line y = mx + c meets the parabola y² = 4ax in two points real, coincident or imaginary according as
a > cm, a = cm, a < cm respectively.
⇒ condition of tangency is, c = a/m.
Tangent
Secant
A

Length of the chord intercepted by the parabola


on the line y = m x + c is :
 4 
 2 a(1 + m2 )(a − mc)
m 
NOTE :
1. The equation of a chord joining t1 & t2 is 2x − (t1 + t2) y + 2 at1 t2 = 0.

RI
2. If t1 & t2 are the ends of a focal chord of the parabola y² = 4ax then t1t2 = −1.Hence the
a 2a 
co−ordinates at the extremities of a focal chord can be taken as (at², 2at) &  2 , −
t t 

A
Focal chord
A

S (focus)
B

UH
3. Length of the focal chord making an angle α with the x− axis is 4acosec² α.

Example # 22 : Discuss the position of line y = x + 3 with respect to parabola y 2 = 4(x + 2).
Solution : Solving we get (x + 3)2 = 4(x + 2) ⇒ (x – 1)2 = 0
so y = x + 3 is tangent to the parabola.
JA
Example # 23 : Prove that focal distance of a point P(at2, 2at) on parabola y2 = 4ax (a > 0) is a(1 + t2).
Solution :
LP

 PS = PM = a + at2 PS = a (1 + t2).
Example # 24 : If t1, t2 are end points of a focal chord then show that t1 t2 = –1.

Solution : Let parabola is y2 = 4ax


A

2
P (at1 , 2at1)

S(a, 0)
NK

2
Q (at2 , 2at2)

since P, S & Q are collinear ∴ mPQ = mPS


2 2t
⇒ = 2 1 ⇒ t12 – 1 = t12 + t1t2 ⇒ t1t2 = – 1
t1 + t 2 t1 − 1
SA

Example # 25 : If the endpoint t1, t2 of a chord satisfy the relation t1 t2 = –3, then prove that the chord of y2 = 4x
always passes through a fixed point. Find the point ?
Solution : Equation of chord joining (at12, 2at1) and (at22, 2at2) is
2
y – 2at1 = (x – at12) ⇒ (t1 + t2) y – 2at12 – 2at1t2 = 2x – 2at12
t1 + t 2
2
y= (x – 3) ( t1t2 = –3) ∴ This line passes through a fixed point (3, 0).
t1 + t 2
Self Practice Problems :
(18) If the line y = 3x + λ intersect the parabola y2 = 4x at two distinct point's then set of value's of
'λ' is
(19) Find the midpoint of the chord x + y = 2 of the parabola y 2 = 4x.
(20) If one end of focal chord of parabola y2 = 16x is (16, 16) then coordinate of other end is.
1 1 1
(21) If PSQ is focal chord of parabola y2 = 4ax (a > 0), where S is focus then prove that + =
PS SQ a
(22) Find the length of focal chord whose one end point is (ap2, 2ap)
2
 1
Ans. (18) (– ∞, 1/3) (19) (4, – 2) (20) (1, – 4) (22) a  p + 
 p
6.1 Tangents to the parabola y² = 4ax :
Equation of tangent at a point on the parabola can be obtained by replacement method or using
derivatives.

In replacement method, following changes are made to the second degree equation to obtain T.
x2 → x x1, y2 → y y1, 2xy → xy1 + x1y, 2x → x + x1, 2y → y + y1

RI
So, it follows that the targents are :
(i) y y1 = 2 a (x + x1) at the point (x1, y1) ;
a  a 2a 
(ii) y = mx + (m ≠ 0) at  2 ,
m m 

A
m
(iii) t y = x + a t² at (at², 2at).
(iv) Point of intersection of the tangents at the point t1 & t2 is { at1 t2 , a(t1 + t2) }.

UH
a
Example # 26 : Prove that the straight line y = mx + c touches the parabola y 2 = 4a (x + a) if c = ma +
m
Solution: Equation of tangent of slope ‘m’ to the parabola y 2 = 4a(x + a) is
a  1
y = m(x + a) + ⇒ y = mx + a  m + 
m  m 
JA
a
But the given tangent is y = mx + c ∴ c = am +
m

Example # 27 : A tangent to the parabola y2 = 8x makes an angle of 45° with the straight line y = 3x + 5. Find
its equation and its point of contact.
3 ±1 1
⇒ m1 = – 2, m2 =
LP

Solution : Slope of required tangent’s are m =


1 3 2
a
 Equation of tangent of slope m to the parabola y 2 = 4ax is y = mx + .
m
1  1
∴ tangent’s y = – 2x – 1 at  , − 2  ⇒ y = x + 4 at (8, 8)
A

 2  2

Example # 28 : Find the equation to the tangents to the parabola y 2 = 9x which goes through the point (4, 10).
NK

9
Solution : Equation of tangent to parabola y 2 = 9x is y = mx +
4m
Since it passes through (4, 10)
9 1 9
∴ 10 = 4m + ⇒ 16 m2 – 40 m + 9 = 0 m= ,
4m 4 4
x 9
SA

∴ equation of tangent’s are y = +9 & y= x + 1.


4 4
Example # 29 : Find the equations to the common tangents of the parabolas (y – 1) 2 = 4ax and x2 = 4b(y – 1).
Solution : Equation of tangent to (y – 1)2 = 4ax is
a
(y – 1) = mx + ........(i)
m
Equation of tangent to x2 = 4b(y – 1) is
b 1 b
x = m1(y – 1) + ⇒ (y – 1) =
x– ........(ii)
m1 m1 (m1 )2
for common tangent, (i) & (ii) must represent same line.
1/ 3
1 a b a  a
∴ =m & = – 2 ⇒ = – bm2 ⇒ m =  − 
m1 m m1 m  b
1/ 3 1/ 3
 a  a
∴equation of common tangent is y =  −  x + a −  + 1.
 b  b

Self Practice Problems:


(23) Find equation tangent to parabola y2 = 4x whose intercept on y–axis is 2.
(24) Prove that perpendicular drawn from focus upon any tangent of a parabola lies on the tangent
at the vertex.
(25) Prove that image of focus in any tangent to parabola lies on its directrix.
(26) Prove that the area of triangle formed by three tangents to the parabola y 2 = 4ax is half the area
of triangle formed by their points of contacts..

RI
x
Ans. (23) y = +2
2
7. Line and an Ellipse :

A
x2 y2
The line y = mx + c meets the ellipse +
= 1 in two points real, coincident or imaginary
a2 b2
according as c² < a²m² + b², c2 = a²m² + b² or c2 > a²m² + b²

UH
y2
Hence y = mx + c is tangent to the ellipse x2 + 2 = 1 if c² = a²m² + b².
2

a b
2
x y2
NOTE: The equation to the chord of the ellipse = 1 joining two points with eccentric angles α & β is
+
b2 a2
α+β α+β α−β
x y
JA
given by cos + sin = cos
a 2 b 2 2

Example # 30 : Find the set of value(s) of 'λ' for which the line x + y + λ = 0 intersect the ellipse
x2 y2
+ = 1 at two distinct points.
16 9
LP

2
x2 ( x + λ )
Solution : Solving given line with ellipse, we get + =1
16 9
25x2 + 39λ x + 16λ2 – 144 = 0
Since, line intersect the parabola at two distinct points,
A

∴ roots of above equation are real & distinct


∴ D>0
∴ (32λ)2 – 4.25(16λ2 – 144) > 0 ⇒ λ∈ (–5, 5)
NK

Self Practice Problems :


x2 y2
(27) Find the value of 'λ' for which 2x – y + 109 λ = 0 touches the ellipse + =1
25 9
Ans. (27) λ=±1
SA

x2 y2
7.1 Tangents to ellipse + =1
a2 b2
x2 y2
(a) Slope form: y = mx ± a2m2 + b2 is tangent to the ellipse + =1 for all values of m.
a2 b2
x x1 y y1 x2 y2
(b) Point form : + = 1 is tangent to the ellipse + = 1 at (x1, y1).
a2 b2 a2 b2
x cos θ y sin θ x2 y2
(c) Parametric form: + = 1 is tangent to the ellipse 2 + 2 = 1 at the point
a b a b
(a cos θ, b sin θ).
Note : (i) There are two tangents to the ellipse having the same m, i.e. there are two tangents parallel to
any given direction.These tangents touches the ellipse at extremities of a diameter.
 cos α + β sin α 2+ β 
(ii) Point of intersection of the tangents at the point α & β is,  a 2
, b 
 cos α − β cos α −2 β 
 2 
(iii) The eccentric angles of the points of contact of two parallel tangents differ by π.

Example # 31 : Find the equations of the tangents to the ellipse 3x 2 + 4y2 = 12 which are parallel to the
line x – 2y + 7 = 0
1
Solution: Slope of tangent = m =
2
2 2
x y
Given ellipse is + =1

RI
4 3
Equation of tangent whose slope is 'm' is y = mx ± 4m2 + 3
1 1
 m= ∴y= x ± 1+ 3 ⇒ 2y = x ± 4
2 2

A
Example # 32 : A tangent to the ellipse 9x2 + 16y2 – 144 = 0 touches at the point P on it in the first quadrant
and meets the co-ordinate axes in A and B respectively. If P divides AB in the ratio 3 : 1, find
the equation of the tangent.

UH
x2 y2
Solution: The given ellipse is 2 + 2 = 1⇒ a = 4, b = 3
4 3
Let P ≡ ( a cosθ, b sinθ) ∴ equation of tangent is
x y
cosθ + sinθ = 1
JA
a b
A ≡ (a secθ, 0)
B ≡ (0, b cosecθ)
 P divide AB internally in the ratio 3 : 1
a sec θ 1 1
∴ a cosθ = ⇒ cos2θ = ⇒ cosθ =
4 4 2
LP

3bcos ecθ 3
and b sin θ = ⇒ sinθ =
4 2
x 3y
∴ tangent is + =1 ⇒ bx + 3 ay = 2ab ⇒ 3x + 4 3 y = 24
2a 2b
A

Example # 33 : Prove that the locus of the point of intersection of tangents to an ellipse at two points whose
π
eccentric angle differ by is an ellipse having the same eccentricity.
3
NK

Solution : Let P (h, k) be the point of intersection of tangents at A(θ) and B(β) to the ellipse.
θ+β θ+β
acos  b sin 
2  2 
2 2
  h k  θ−β
∴ h= &k= ⇒   +   = sec2  
 θ−β  θ−β a b  2 
cos   cos  2 
 2   
SA

π
but given that θ – β =
3
x2 y2
∴ locus is + = 1 which is ellipse having same eccentricity.
π π
a2 sec 2   b2 sec 2  
6 6
Example # 34 : If the locus of foot of perpendicular drawn from centre to any tangent to the ellipse
3x2 + 4y2 = 12 is (x2 + y2)2 = ax2 + by2, then find a + b.
Solution : Let P(h, k) be the foot of perpendicular to a tangent y = mx + 4m2 + 3 .......(i)
from centre
k h
∴ .m=–1 ⇒ m=– .......(ii)
h k
 P(h, k) lies on tangent
∴ k = mh + 4m2 + 3 .......(iii)

RI
from equation (ii) & (iii), we get

A
2

UH
 h2  4h2
 k +  = +3 ⇒ locus is (x2 + y2)2 = 4x2 + 3y2
 k  k2

Self Practice Problems :


(28) Show that the locus of the point of intersection of the tangents at the extremities of any focal
JA
chord of an ellipse is the directrix corresponding to the focus.
(29) Show that the locus of the foot of the perpendicular on a varying tangent to an ellipse from
either of its foci is a concentric circle.
(30) Prove that the portion of the tangent to an ellipse intercepted between the ellipse and the
directrix subtends a right angle at the corresponding focus.
LP

(31) Find the area of parallelogram formed by tangents at the extremities of latera recta of the
x2 y2
ellipse 2 + 2 = 1.
a b
(32) If y1 is ordinate of a point P on the ellipse then show that the angle between its focal radius and
A

 b2 
tangent at it, is tan–1 
 aey 
.
 1
NK

x2 y2
(33) Find the eccentric angle of the point P on the ellipse + =1 tangent at which, is
a2 b2
equally inclined to the axes.
2a3 b b b
Ans. (31) (33) θ = ± tan–1   , π – tan–1   , – π + tan–1  
a −b
2 2  
a  
a a
SA

8. Line and a hyperbola :


x2 y2
The straight line y = mx + c is a secant, a tangent or passes outside the hyperbola − =1
a2 b2
according as : c2 > a2 m2 − b2 or c2 = a2 m2 − b2 or c2 < a2 m2 − b2, respectively.

x2 y2
NOTE: The equation to the chord of the hyperbola − = 1 joining the two points P(α) & Q(β) is given by
a2 b2
x α −β y α+β α+β
cos − sin = cos
a 2 b 2 2
xy22
8.1 Tangents to hyperbola 22
-=1 :
ab
(i) Slope form : y = m x ± a 2 m2 − b 2 can be taken as the tangent to the hyperbola
2 2
x y
2
− = 1 , having slope 'm'.
a b2
x2 y2
(ii) Point form: Equation of tangent to the hyperbola − = 1 at the point (x1, y1) is
a2 b2
xx1 yy1
− 2 =1
a2 b
x2 y2
(iii) Parametric form: Equation of the tangent to the hyperbola − = 1 at the point.
a2 b2

RI
x sec θ y tan θ
(a sec θ, b tan θ) is − =1
a b
 θ1 − θ2 
 cos 2

A
 θ + θ2 
Note : (i) Point of intersection of the tangents at P(θ1) & Q(θ2) is  a , b tan  1 
 cos θ1 + θ2  2 

 2 

UH
(ii) If |θ1 + θ2| = π, then tangents at these points (θ1 & θ2) are parallel.
(iii) There are two parallel tangents having the same slope m. These tangent touches the hyperbola
at the extremities of a diameter.
x2
Example # 35 : Find c, if x + y = c touch the hyperbola – y2 = 1.
4
JA
Solution: Solving line and hyperbola we get
x2 – 4 (c–x)2 = 4
3x2 + 8cx + 4c2 + 4 = 0
D=0
64c2 – 4.3.4 (c2–1) = 0
c2 – 3 = 0
LP

c=± 3
Example # 36 : Find the equation of the tangent to the hyperbola x 2 – 4y2 = 36 which is perpendicular to the
line 3 x + y + 5 = 0
1
Solution : y = mx ± 36m2 – 9 , where m=
A

3
x
∴ equation of tangents are y = ± 3 ⇒ 3y=x±3
3
NK

x2 y2
Example # 37 : Find the point of contact if 3x – 7 y – 9 = 0 is tangent to − = 1.
16 9
Solution : Let the point of contact is (x1, y1). The equation of tangent is
xx1 yy1

16 9 – 1 = 0...........(i)
SA

The given equation of tangent is 3x – 7 y – 9 = 0...........(ii)


From Equ (i) & (ii)
x1 y 1  16 
= 1 = ⇒ (x1, y1) =  , 7 
16 × 3 9 7 9  3 
9. Line and Rectangular hyperbola :
Equation of a chord joining the points P (t1) & Q(t2) is x + t1 t2 y = c (t1 + t2).
x y x
Equation of the tangent at P (x1, y1) is + = 2 & at P (t) is + t y = 2 c.
x1 y1 t
Example # 38 : A, B, C are three points on the rectangular hyperbola xy = c 2, find
(i) The area of the triangle ABC
(ii) The area of the triangle formed by the tangents at A, B and C.
 c  c 
Solution : Let co–ordinates of A,B and C on the hyperbola xy = c 2 are  ct1,  ,  ct 2 ,  , and
 t1  t 2 
 c
 ct 3 ,  respectively.
 t3 
c c c
ct1 ct 2 ct 3
1 t1 t2 t3
(i) ∴ Area of triangle ABC = [ + + ]
2 c c c
ct 2 ct 3 ct1
t2 t3 t1
c 2 t1 t 2 t 2 t3 t 3 t1 c2
= − + − + − = t12 t 3 − t 22 t 3 + t1t 22 − t 32 t1 + t 2 t 32 − t12 t 2
2 t 2 t1 t3 t2 t1 t 3 2t1t 2 t 3

RI
c2
= | (t1 – t2) (t2 – t3) (t3 – t1) |
2t1t 2 t 3
(ii) Equations of tangents at A,B,C are
x + yt12 – 2ct1 = 0

A
x + yt22 – 2ct2 = 0
and x + yt32 – 2ct3 = 0

UH
2
1 t12 −2ct1
1
∴ Required Area = 1 t 22 −2ct 2 .........(1)
2 | C1C2C3 |
1 t 32 −2ct 3

1 t 22 1 t12 1 t12
JA
where C1 = , C2 = – and C3 =
1 t 32 1 t 32 1 t 22
∴ C1 = t32 – t22, C2 = t12 – t32 and C3 = t22 – t12
1
From (1) = 4c2.(t1 – t2)2 (t2 – t3)2 (t3 – t1)2
2 (t 32 − t 22 ) (t12 − t 32 )(t 22 − t12 )
LP

(t1 − t 2 )(t 2 − t 3 )(t 3 − t1 )


= 2c2
(t1 + t 2 ) (t 2 + t 3 ) (t 3 + t1)

( t1 − t 2 ) (t 2 − t 3 )(t 3 − t1 )
∴ Required area is, 2c2
A

(t1 + t 2 ) (t 2 + t 3 ) (t 3 + t1 )

Example # 39 : Prove that the perpendicular focal chords of a rectangular hyperbola are equal.
NK

Solution : Let rectangular hyperbola is x2 – y2 = a2


Let equations of PQ and DE are
y = mx + c ......(1)
and y = m1x + c1 ......(2)
respectively.
SA

Be any two focal chords of any rectangular hyperbola x2 – y2 = a2 through its focus. We have to
prove PQ = DE. Since PQ ⊥ DE.
∴ mm1 = –1 ......(3)
Also PQ passes through S (a 2 , 0) then from (1),
0 = ma 2 + c
or c2 = 2a2m2 ......(4)
Let (x1,y1) and (x2,y2) be the co–ordinates of P and Q then
(PQ)2 = (x1 – x2)2 + (y1 – y2)2 ......(5)
Since (x1,y1) and (x2,y2) lie on (1)
∴ y1 = mx1 + c and y2 = mx2 + c
∴ (y1 – y2) = m (x1 – x2) .......(6)
From (5) and (6)
(PQ)2 = (x1 – x2)2 (1 + m2) .......(7)
Now solving y = mx + c and x2 – y2 = a2 then x2 – (mx + c)2 = a2
or (m2 – 1) x2 + 2mcx + (a2 + c2) = 0

2mc a2 + c 2
∴ x1 + x2 = – and x1x2 = =
m −1 2
m2 − 1

RI
4m2c 2 4(a2 + c 2 )
⇒ (x1 – x2)2 = (x1 + x2)2 – 4x1x2 = –
(m2 − 1)2 (m2 − 1)
4{a2 + c 2 − a2m2 } 4a2 (m2 + 1)

A
= = { c2 = 2a2m2}
(m2 − 1)2 (m2 − 1)2
2
 m2 + 1 

UH
From (7), (PQ) = 4a  2
 m − 1 
2 2

 
2
  1 2 
2   −  + 1 2
 m + 1
2
 m   m2 + 1 
Similarly, (DE)2 = 4a2  12 = 4a2  
 m − 1   = 4a  2 2
 = (PQ)2

 m − 1
2
JA
 1    − 1  − 1
 m 
  
( mm1 = – 1) Thus (PQ)2 = (DE)2 ⇒ PQ = DE.
Hence perpendicular focal chords of a rectangular hyperbola are equal.

Self Practice Problems :


LP

x2 y2
(34) Show that the line x cos α + y sin α = p touches the hyperbola – =1
a2 b2
if a2 cos2 α – b2 sin2 α = p2.

For what value of λ does the line y = 2x + λ touches the hyperbola 16x 2 – 9y2 = 144 ?
A

(35)

(36) Find the equation of the tangent to the hyperbola x 2 – y2 = 1which is parallel to the line
4y = 5x + 7.
NK

Ans. (35) λ=±2 5 (36) 4y = 5x ± 3


10 Pair of tangents : The equation to the pair of tangents which can be drawn from any point (x 1, y1) to the
curve S = 0 is SS1 = T²
Curve(S=0) T for point (x1, y1) & S = 0 S1 for point (x1, y1) & S = 0 Combined equation of
SA

tangents from external


point (x1, y1) to S=0
Parabola T ≡ y y1 − 2a(x + x1) S1 = y1² − 4ax1 SS1 = T²
(y2 – 4ax = 0)
Ellipse xx1 yy1 x1
2 2
y1 SS1 = T²
T≡ + –1 S1 = + –1
x2
y 2  a 2
b 2
a2 b2
 2 + 2 − 1 = 0 
a b 
Hyperbola xx1 yy1 x1
2
y1
2 SS1 = T²
T≡ – –1 S1 = − –1
 x2 y2  a 2
b 2
a2 b2
 2 − 2 − 1 = 0 
a b 
Example # 40 : Write the equation of pair of tangents to the parabola y 2 = 4x drawn from a point P(–1, 2)
Solution : We know the equation of pair of tangents are given by SS 1 = T²
∴ (y2 – 4x) (4 + 4) = (2y – 2 (x – 1))2
⇒ 8y2 – 32x = 4y2 + 4x2 + 4 – 8xy + 8y – 8x ⇒ y2 – x2 + 2xy – 6x – 2y = 1

Example # 41 : Find the locus of the point P from which tangents are drawn to parabola y 2 = 4ax having slopes
m1, m2 such that
(i) |m1 – m2| = 2 (ii) θ1 + θ2 = π/3
a
Solution : Equation of tangent to y2 = 4ax, is y = mx +
m
Let it passes through P(h, k)

RI
m2h – mk + a = 0
k a
(i) m 1 + m2 = and m1 . m2= ⇒ |m1 – m2| = 2 ⇒ (m1 + m2)2 – 4 m1m2 = 4
h h
k2 a
– 4 = 4 ⇒ 4ax = 4x2

A
2
h h
m + m2 k /h
(ii) tan π/3 = 1 = ⇒ y = (x – a) 3
1 − m1 m2 1 − a / h

UH
x2 y2
Example # 42 : How many real tangents can be drawn from the point (–2, –5) to the ellipse + = 1. Find
4 25
the equation of these tangents & angle between them.
JA
Solution :

x = –2

y = –5
(–2, – 5)
LP

By direct observation
x = –2, y = –5 are tangents.
x2 y2
Example # 43 : Find the locus of point of intersection of perpendicular tangents to the ellipse + =1
a2 b2
A

Solution : Let P(h, k) be the point of intersection of two perpendicular tangents


equation of pair of tangents is SS1 = T2
2
 x2 y2   h2 k 2   hx ky 
⇒  2 + 2 − 1  2 + 2 − 1 =  2 + 2 − 1
NK

a  a  a b 
 b  b 
x2  k 2  y 2  h2 
⇒ 
2  2
− 1 + 2  2 − 1 + ........ = 0 .........(i)
a b  b a 
Since equation (i) represents two perpendicular lines
SA

1  k2  1  h2 
∴ 2  2 − 1 + 2  2 − 1 = 0
a b    
 b a 
⇒ k – b + h – a = 0 ⇒ locus is x2 + y2 = a2 + b2
2 2 2 2

x2 y2
Example # 44 : How many real tangents can be drawn from the point (2, 1) to the hyperbola – =1.
16 9
Find the equation of these tangents.
x2 y2
Solution : Given point P ≡ (2, 1) Hyperbola S≡ – –1=0
16 9
4 1 31
 S1 ≡ – – 1 =− <0 ⇒ Point P ≡ (2, 1) lies outside the hyperbola.
16 9 36
∴ Two tangents can be drawn from the point P(2, 1).
Equation of pair of tangents is SS1 = T2
2
x 2
y2   1 1   2x y 
⇒  − − 1  − − 1 =  − − 1 ⇒ 144 (9x2 – 16y2 – 144) + (9x – 9y – 72)2 = 0
 16 9  4 9   16 9 
 
x2 y2
Example # 45 : Find the locus of point of intersection of perpendicular tangents to the hyperbola =1 −
a2 b2
Solution : Let P(h, k) be the point of intersection of two perpendicular tangents. Equation of pair of
tangents is SS1 = T2

2
 x2 y2   h2 k 2   hx ky 
⇒  2 − 2 − 1  2 − 2 − 1 =  2 − 2 − 1
a b  a b  a b 

RI
x2  k 2  y2  h2 
⇒  − − 1 –  2 − 1 + ........ = 0 .........(i)
a2  b2 
 b2 a 
Since equation (i) represents two perpendicular lines

A
1  k2  1  h2 
∴  − 2 − 1
2 
– 
2  2
− 1 = 0 ⇒ – k2 – b2 – h2 + a2 = 0 ⇒ locus is x2 + y2 = a2 – b2
a  b 
 b a 

UH
Self Practice Problems :
(37) If two tangents to the parabola y2 = 4ax from a point P make angles θ1 and θ2 with the axis of
the parabola, then find the locus of P in each of the following cases.
(i) tan2θ1 + tan2θ2 = λ (a constant) (ii) cos θ1 cos θ2 = λ (a constant)
(38) Find the locus of point of intersection of the tangents drawn at the extremities of a focal chord
JA
x2 y2
of the ellipse 2 + 2 = 1.
a b
a
Ans. (37) (i) y2 – 2ax = λx2 , (ii) x2 = λ2 {(x – a)2 + y2} (38) x=±
e

11. Director circle: Locus of the point of intersection of the tangents which meet at right angles is called
LP

the Director Circle.


Curve(S=0) Locus of Director Circle Figure
of (S=0)
Parabola x+a=0 2
y =4ax
A

2
(y – 4ax = 0) x=0
NK

y=0

Director Circle
(x=–a)
SA

Ellipse x 2 + y 2 = a2 + b2 x= 0
Director circle
x2
y 2  2 2 2
(x +y = a +b )
2
 2 + 2 − 1 = 0 
a b 
y= 0

x2 y2
2
+ =1
a b2
Hyperbola x2 + y2 = a2 – b2
x=0
 x2 y2 
 2 − 2 − 1 = 0  x2

y2
=1
a b  a2 b2

y=0

Director Circle
(x 2 + y 2 = a2 − b2 )

RI
Note: For hyperbola, if b2 < a2 , then the director circle is real.

If b2 = a2 (i.e. rectangular hyperbola), then the radius of the director circle is zero and it reduces to a
point circle at the origin. In this case centre is the only point from which two perpendicular tangents can

A
be drawn on the curve.

If b2 > a2, then the radius of the director circle is imaginary, so that there is no such circle and so no pair

UH
of tangents at right angle can be drawn to the curve.

Example # 46 : Find the point of the line x – y = 0 for from where perpendicular tangent can be drawn to
x2
+ y2 = 1
JA
9
Solution : Solving director circle x2 + y2 = 10 & x – y = 0 ⇒ ( 5 , 5 ), (– 5 , – 5 )
Self Practice Problems :

(39) Find the angle between the tangent drawn from (–2, 1) to x 2 + 4y2 = 4
π
Ans. (39)
LP

12. Chord of contact: Equation to the chord of contact of tangents drawn from a point P(x 1, y1) to the curve
S = 0 is T = 0
A

Curve(S=0) T for point (x1, y1) & S = 0 equation of chord of contact from external
point (x1, y1) to S=0 is T = 0
Parabola T ≡ y y1 − 2a(x + x1) yy1 – 2a (x + x1) = 0
NK

(y2 – 4ax = 0)
Ellipse xx1 yy1 xx1 yy1
T≡ + –1 + – 1= 0
 x2 y2  a 2
b 2
a 2
b2
 2 + − 1 = 0 
a b2 
Hyperbola xx1 yy1 xx1 yy1
SA

T≡ – 2 –1 – 2 –1=0
x 2
y 2  a 2
b a2 b
 2 − 2 − 1 = 0 
a b 
Rectangular xy1 + yx1 xy1 + yx1
Hyperbola T= – c2 – c2 = 0
2 2
(xy – c2 = 0)

NOTE : The area of the triangle formed by the tangents from the point (x 1, y1) & the chord of contact is
1
(y ² − 4ax1)3/2
2a 1

Example # 47 : Find the length of chord of contact of the tangents drawn from point (–2, 3) to the parabola
y2 = 8x.
Solution : Let tangent at P(t1) & Q(t2) meet at (–2, 3)
∴ 2t1t2 = –2 & 2(t1 + t2) = 3
 PQ = (2t − 2t ) + (4(t1 − t 2 ))2
2
1
2 2
2

(32 − 4.2(–2))(32 + 4.22 )


= 2 ((t1 + t 2 )2 − 4t1t 2 )((t1 + t 2 )2 + 4) = = 25/2
22
Example # 48 : If the line x – y – 1 = 0 intersect the parabola y 2 = 8x at P & Q, then find the point of intersection
of tangents at P & Q.
Solution : Let (h, k) be point of intersection of tangents then chord of contact is
yk = 4(x + h)
4x – yk + 4h = 0 .....(i)
But given is x – y – 1 = 0

RI
4 −k 4h
∴ = = ⇒ h = – 1, k = 4 ∴ point ≡ (–1, 4)
1 −1 −1
Example # 49 : Find the locus of point whose chord of contact w.r.t to the parabola y 2 = 4bx is the tangents of

A
the circle x2 + y2 = a2.
Solution : Let it is chord of contact for parabola y 2 = 4bx w.r.t. the point P(h, k)
∴ Equation of chord of contact is yk = 2b(x + h)

UH
2b 2bh
y= x+ .....(i)
k k
2bh
(i) is tangents to x + y = a ⇒
2 2 2 k = a ⇒ 4b2x2 = a2 (y2 + 4b2)
JA 2
4b
1+ 2
k
x2 y2
Example # 50 : If tangents to the circle x2 + y2 = b2 intersect the ellipse + = 1 at A and B, then find the
a2 b2
locus of point of intersection of tangents at A and B.
Let P ≡ (h, k) be the point of intersection of tangents at A & B
LP

Solution :
xh yk
∴equation of chord of contact AB is + 2 =1 .............(i)
a2 b
which touches the circle x2 + y2 = b2
x2 y2 1
A

1
∴ = b ⇒ required locus is 4
+ 4 = 2
h2 k 2 a b b
+
a4 b4
NK

x2 y2
Example # 51: If tangents to the parabola y2 = 4ax intersect the hyperbola – = 1 at A and B, then find
a2 b2
the locus of point of intersection of tangents at A and B.

Solution: Let P ≡ (h, k) be the point of intersection of tangents at A & B


SA

xh yk
∴ Equation of chord of contact AB is 2
– =1 ......(i)
a b2
Which touches the parabola
Equation of tangent to parabola y2 = 4ax
a a
y = mx + ⇒ mx – y = – ......(ii)
m m
equation (i) & (ii) as must be same
a

m −1 m h b2 ak
∴ = = ⇒m= &m=– 2
 h   k  1 k a2 b
 2 − 2 
a   b 
hb2 ak b4
∴ =– ⇒ locus of P is y2 = – .x
ka2 b2 a3

Self Practice Problems :

(40) Prove that locus of a point whose chord of contact w.r.t. parabola passes through focus is
directrix

(41) If from a variable point ‘P’ on the line 2x – y – 1 = 0 pair of tangent’s are drawn to the parabola
x2 = 8y then prove that chord of contact passes through a fixed point, also find that point.

(42) Find the locus of point of intersection of tangents at the extremities of normal chords of the
x2 y2

RI
ellipse + = 1.
a2 b2

(43) Find the locus of point of intersection of tangents at the extremities of the chords of the
x2 y2

A
ellipse 2
+ = 1 subtending a right angle at its centre.
a b2
a6 b6 x2 y2 1 1
Ans. (41) (8,1) (42) + = (a2 – b2) 2 (43) + = +

UH
2 2 4 4 2
x y a b a b2

13. Chord with a given middle point:


Equation of the chord of the curve S = 0 whose middle point is (x 1, y1) is T = S1.
Curve(S=0) T for point (x1, y1) S1 for point (x1, y1) Chord with middle point (x1, y1) for
JA
&S=0 &S=0 S=0 is T = S1
Parabola T = y y1 − 2a(x + x1) S1 = y1² − 4ax1 y y1 − 2a(x + x1) = y1² − 4ax1
(y2 – 4ax = 0)
Ellipse xx1 yy1 x12 y12 xx1 yy1 xx1 yy1
T= + –1 S1 = + –1 + = +
x 2
y 2  a 2
b 2
a 2
b 2 a2 b2 a2 b2
 2 + 2 − 1 = 0 
LP

a b 
Hyperbola xx1 yy1 x1
2
y1
2
xx1 yy1 x1
2 2
y1
T= – –1 S1 = − –1 – = −
 x2 y2  a2 b2 a2 b2 a2 b2 a2 b2
 2 − 2 − 1 = 0 
a b 
A

Rectangular xy1 + yx1 S1 = x1y1 – c2 xy1 + yx1 = 2x1y1


T= – c2
Hyperbola 2
(xy – c2 = 0)
NK

Example # 52 : Find the locus of middle point of the chord of the parabola y 2 = 16x which pass through a given
point (7, –2).
Solution : Let P(h, k) be the mid point of chord of parabola y 2 = 16x
so equation of chord is yk – 8(x + h) = k2 – 16h.
SA

Since it passes through (7, –2)


–2k – 8 (7 + h) = k2 – 16h
∴ Required locus is
y + 2y – 8x + 56 = 0
2

Example # 53 : Find the locus of middle point of the chord of the parabola y 2 = 4ax which is parallel to line
y = mx + c
Solution : Let P(h, k) be the mid point of chord of parabola y 2 = 4ax,
so equation of chord is yk – 2a(x + h) = k2 – 4ah.
2a 2a
but slope = =m ∴ locus is y =
k m
x2 y2
Example # 54 : Find the locus of the mid - point of chords of the ellipse + = 1 Which are focal chords
a2 b2
of y2 = 4ax
Solution : Let P ≡ (h, k) be the mid-point
xh yk h2 k2
∴ equation of chord whose mid-point is given + –1= + –1
a2 b2 a2 b2
since it is a focal chord,
∴ it passes through focus (a, 0)
h h2 k 2 x2 y2 x
⇒ = 2 + 2 ⇒ required locus is 2 + 2 =
a a b a b a
Example # 55 : Find the mid point of chord x + 2y = 4 of ellipse 9x 2 + 36y2 = 324

RI
Solution : Let (h,k) be mid point of chord . So T = S1
9xh + 36yk = 9h2 + 36k2 ------ (i) x + 2y = 4 ------ (ii)
From (i) and (ii)
9h 36k 9h2 + 36k 2

A
= =
1 2 4 ⇒ (h, k) = (2,1)
x2 y2
Example # 56 : Find the locus of the mid - point of focal chords of the hyperbola – = 1.
a2 b2

UH
Solution : Let P ≡ (h, k) be the mid-point
xh yk h2 k2
∴ equation of chord whose mid-point is given is – –1= – –1
a2 b2 a2 b2
since it is a focal chord,
JA
∴ it passes through focus, either (ae, 0) or (–ae, 0)

If it passes through (ae, 0)


LP

ex x2 y2
∴ locus is = 2 – 2
a a b
If it passes through (–ae, 0)
ex x2 y2
∴ locus is – = 2 – 2
A

a a b
x2 y2
Example # 57 : Find the condition on 'a' and 'b' for which two distinct chords of the hyperbola – =1
2a2 2b2
NK

passing through (a, b) are bisected by the line x + y = b.


Solution : Let the line x + y = b bisect the chord at P(α, b – α)
∴ equation of chord whose mid-point is P(α, b – α)
xα y(b − α ) α2 (b − α )2
– = –
SA

2a2 2b2 2a2 2b2


Since it passes through (a, b)
α (b − α ) α2 (b − α )2
∴ – = 2

2a 2b 2a 2b2
 1 1   1 1 1
α2  2 − 2  + α  −  = 0 α = 0, α= ∴ a≠±b
a b  b a 1 1
+
a b
x2 y2
Example # 58 : Find the locus of the mid point of the chords of the hyperbola – = 1 which subtend a
a2 b2
right angle at the origin.
Solution : let (h,k) be the mid–point of the chord of the hyperbola. Then its equation is
hx ky h2 k2 hx ky h2 k2
– –1= – –1 or –
........(1) = –
a2 b2 b2 b2 a2 b2 a2 b2
The equation of the lines joining the origin to the points of intersection of the hyperbola and the
chord (1) is obtained by making homogeneous hyperbola with the help of (1)
2
 hx ky 
x2 y2  2 − 2
∴ – =
a b 
2 2
a b2 h 2
k2 
 2 − 2 
a b 
2 2
1  h2 k 2  1  h2 k 2  h2 k2 2hk
⇒ 2  2 − 2  x2 – 2  2 − 2  y2 = 4 x2 + 4 y2 – 2 2 xy .......(2)
a a  
b  
b a b  a b a b

RI
The lines represented by (2) will be at right angle if coefficient of x 2 + coefficient of y2 = 0
2 2 2
1  h2 k 2  h2 1  h2 k 2  k 2  h2 k 2   1 1  h2 h 2
⇒ 2  2 − 2 – 4 – 2  2 − 2 – 4 =0 ⇒  2 − 2  2 − 2 = 4 + 4
  
b a 
b  b  
b  a  a
a a b  a a b a

A
2
 x2 y2   1 1  x2 y2
hence, the locus of (h,k) is  2 − 2   2 − 2  = 4 + 4
a b   a b 
 a b

UH
Self Practice Problems :
(44) Find the mid point of chord x – y – 2 = 0 of parabola y 2 = 4x.

(45) Find the locus of mid - point of chord of parabola y 2 = 4ax which touches the parabola x2 = 4by.
JA
x2 y2
(46) Find the equation of the chord + = 1 which is bisected at (2, 1).
36 9
x2 y2
(47) Find the locus of the mid-points of normal chords of the ellipse + = 1.
a2 b2
LP

x2 y2
(48) Find the equation of the chord – = 1 which is bisected at (2, 1).
36 9
(49) Find the point 'P' from which pair of tangents PA & PB are drawn to the hyperbola
x2 y2
– = 1 in such a way that (5, 2) bisect AB
25 16
A

(50) From the points on the circle x2 + y2 = a2, tangent are drawn to the hyperbola x2 – y2 = a2, prove
that the locus of the middle points of the chords of contact is the curve (x 2 – y2)2 = a2 (x2 + y2).
Ans. (44) (4, 2) (45) y (2ax – y2) = 4a2b (46) x + 2y = 4
NK

2
 x2 y2   a6 b6   20 8 
(47)  2 + 2   2 + 2  = (a2 – b2)2 (48) x = 2y (49)  , 
a  3 3
 b  x y 

14 NORMAL
SA

14.1 Normal to the parabola :


Normal is obtained using the slope of tangent.
P

Normal

2a y
Slope of tangent at (x1 , y1) = ⇒ Slope of normal = – 1
y1 2a
y1
(i) y − y1 = – (x − x1) at (x1, y1) ; (ii) y = mx − 2am − am3 at (am2, − 2am)
2a
(iii) y + tx = 2at + at3 at (at2, 2at).
NOTE :
(i) Point of intersection of normals at t1 & t2 is (a (t + t + t1t2 + 2), − a t1 t2 (t1 + t2)).
(ii) If the normals to the parabola y² = 4ax at the point t1, meets the parabola again at the point

P(t1)

Q(t2)

 2
t2, then t2 = –  t1 +  .
 t1 
(iii) If the normals to the parabola y² = 4ax at the points t 1 & t2 intersect again on the parabola at the
point 't3' then t1 t2 = 2; t3 = − (t1 + t2) and the line joining t1 & t2 passes through a fixed point

RI
(−2a, 0)
Example # 59 : If the normal at point ‘t1’ intersects the parabola again at ‘t2’ then find value of |t1.t2 + t12|.
2

A
Solution : Slope of normal at P = – t1 and slope of chord PQ =
t1 + t 2
2 2 2
∴ – t1 = ⇒ t2 = – t 1 – ⇒ t2 = – t 1 – ⇒ t2.t1 = –t12 – 2 ⇒|t1.t2 + t12| = 2

UH
t1 + t 2 t1 t1
Example # 60 : If the normals at points t1, t2 meet at the point t3 on the parabola then find value of
(t1 + t2 + t3)2 + (t1 . t2)2
Solution : Since normal at t1 & t2 meet the curve at t3
2
JA
∴ t3 = – t 1 – – .....(i)
t1
2
t3 = – t 2 – .....(ii)
t2
⇒ (t12 + 2) t2 = t1 (t22 + 2) ⇒ t1t2 (t1 – t2) + 2 (t2 – t1) = 0
 t1 ≠ t2 , t1t2 = 2 ......(iii)
LP

Hence (i) t1 t2 = 2
from equation (i) & (iii), we get t3 = – t1 – t2
Hence (ii) t1 + t2 + t3 = 0 ....(iv)
from (iii) & (iv) (t1 + t2 + t3) + (t1 . t2) = 4
2 2
A

Example # 61 : Find the locus of the point N from which 3 normals are drawn to the parabola y 2 = 4ax are
such that
(i) Two of them are equally inclined to y-axis
NK

(ii) Two of them have product of their slops is equal to 2.


Solution : Equation of normal to y2 = 4ax is
y = mx – 2am – am3
Let the normal passes through N(h, k)
∴ k = mh – 2am – am3 ⇒ am3 + (2a – h) m + k = 0
For given value’s of (h, k) it is cubic in ‘m’.
SA

Let m1, m2 & m3 are root’s of above equation


∴ m 1 + m2 + m3 = 0 ......(i)
2a − h
m1m2 + m2m3 + m3m1 = ......(ii)
a
k
m1m2m3 = – ......(iii)
a
(i) If two normal are equally inclined to x-axis, then m1 + m2 = 0
∴ m3 = 0 ⇒ y=0
(ii) If two normal’s have product of their slops = 2
∴ m1 m2 = 2
k
from (3) m3 = – .....(iv)
2a
k 2a − h
from (2) 2– (m1 + m2) = .....(v)
2a a
k
from (1) m 1 + m2 = .....(vi)
2a
from (5) & (6), we get
k2 = 4ax

Self Practice Problems:


(51) Find the points of the parabola y2 = 4ax at which the normal is inclined at 45° to the axis.
(52) If chord drawn from point P(9, –6) on the parabola y 2 = 4x is normal at point Q then Q = ?
(53) Find the length of normal chord at point ‘t’ to the parabola y 2 = 4ax.
(54) If the normals at 3 points P, Q & R on the parabola (x – 3) 2 = y + 2 are concurrent, then show
that

RI
(i) The sum of slopes of normals is zero,
(ii) The locus of centroid of ∆PQR is x – 3 = 0.
3
4a(t 2 + 1) 2

A
Ans. (51) (a, –2a), (a, 2a) (52) (9, –6) (53)  =
t2
14.2 Normal to Ellipse
x2 y2 a2 x b2 y

UH
(i) Equation of the normal at (x1, y1) to the ellipse + = 1 is − = a² − b².
a2 b2 x1 y1
x2 y2
(ii) Equation of the normal at the point (acos θ, bsin θ) to the ellipse + = 1 is;
a2 b2
ax. sec θ − by. cosec θ = (a² − b²).
JA
(iii) Equation of a normal in terms of its slope 'm' is y = mx −
(a 2
− b2 m ) .
a + b m2
2 2

x2 y2
Example # 62 : A and B are corresponding points on the ellipse +
=1 and the auxiliary circles
a2 b2
LP

respectively. The normal at A to the ellipse meets CB in R, where C is the centre of the ellipse.
Prove that locus of R is a circle of radius a + b.

Solution : Let A ≡ (acos θ, b sinθ)


∴ B ≡ (a cosθ, a sinθ)
A

A
NK

Equation of normal at A is
(a secθ) x – (b cosec θ) y = a2 – b2 ..........(i)
equation of CB is y = tanθ . x .........(ii)
SA

Solving equation (i) & (ii), we get (a – b) x = (a2 – b2) cosθ


x = (a + b) cosθ, & y = (a + b) sinθ
∴ R ≡ ((a + b) cosθ, (a + b) sinθ) = (h, k)
h2 + k2 = (a + b)2⇒ x2 + y2 = (a + b)2

x2 y2
Example # 63 : Find the shortest distance between the line 3x + 4y = 12 and the ellipse + =1
16 9

Solution : Shortest distance occurs between two non-intersecting curve always along common normal.
Let 'P' be a point on ellipse and Q is a point on given line for which PQ is common normal.
∴ Tangent at 'P' is parallel to given line
∴ Equation of tangent parallel to given line is (y = mx ± a2m2 + b2 )
3x 3 2
y= ±
4

∴ minimum distance = distance between


3x + 4y = 12 & 3x + 4y = 3 2
12 − 3 2
⇒ shortest distance =
5

RI
Example # 64 : Prove that, in an ellipse, the distance between the centre and any normal does not exceed the
difference between the semi-axes of the ellipse.
x2 y2
Solution : Let the equation of ellipse is 2 + 2 = 1
a b

A
Equation of normal at P (θ) is (a secθ)x – (bcosec θ)y – a2 + b2 = 0
distance of normal from centre

UH
| a2 − b2 | | a2 − b2 |
= OR = =
a2 + b2 + (a tan θ)2 + (b cot θ)2 (a + b)2 + (a tan θ − b cot θ)2
JA
| a2 − b2 |
 (a + b)2 + (a tanθ – b cotθ)2 ≥ (a + b)2 or ≤ ⇒ |OR| ≤ (a – b)
(a + b)2
Self Practice Problems :
(55) Find the value(s) of 'a' for which the line x + y = a is a normal to the ellipse 3x 2 + 4y2 = 12
LP

x2 y2
(56) If the normal at the point P(θ) to the ellipse + = 1 intersects it again at the point Q(2θ)
14 5
then find the value of cosθ
1 2
Ans. (55) a = ± (56) –
7 3
A

14.3 Normal to Hyperbola


x2 y2
NK

(a) The equation of the normal to the hyperbola 2


− = 1 at the point P (x1, y1) on it is
a b2
a2 x b2 y
+ = a2 + b2 = a2 e2.
x1 y1
x2 y2
(b) The equation of the normal at the point P (a sec θ, b tan θ) on the hyperbola − = 1 is
SA

a2 b2
ax by
+ = a2 + b2 = a2 e2.
sec θ tan θ
(a2 + b2 )m
(c) Equation of normals in terms of its slope 'm' are y = mx ± .
a 2 − b 2 m2
14.4 Normal to Rectangular hyperbola
Equation of the normal at P (t) is x t3 − y t = c (t4 − 1)

x2 y2
Example # 65 : A normal to the hyperbola –
= 1 meets the axes in M and N. find a locus of point R
a2 b2
on segment MN such that NR : RM = 2 :1.
x2 y2
Solution : The equation of normal at the point Q (a sec φ, b tan φ) to the hyperbola 2 – 2 = 1 is
a b
ax cos φ + by cot φ = a2 + b2 ........(1)
The normal (1) meets the x–axis in
 a2 + b2   a2 + b2 
M sec φ , 0  and y–axis in N  0, tan φ 
 a   b 
   
Let R (h, k) is point whose locus we have to find. as NR : RM = 2 :1.

RI
2 (a2 + b2 ) (a2 + b2 )
⇒ h= sec φ , k = tan φ
3 a 3b
we know that
9a2 9b2 a2 x 2 (a2 + b2 )2
sec2φ – tan2φ = 1 ⇒ x 2
− y 2
= 1 ⇒ − b 2 2
y =

A
4(a2 + b2 )2 (a2 + b2 )2 4 9
Self Practice Problems :
x2 y2

UH
(57) Prove that the line lx + my – n = 0 will be a normal to the hyperbola – =1
a2 b2
a2 b2 (a2 + b2 )2
if – = .
2 m2 n2
JA
15. Important Highlights of Parabola :

(i) If the tangent & normal at any point ‘P’ of the parabola intersect the axis at T & G then
ST = SG = SP where ‘S’ is the focus. In other words the tangent and the normal at a point P on
the parabola are the bisectors of the angle between the focal radius SP & the perpendicular
from P on the directrix. From this we conclude that all rays emanating from S will become
parallel to the axis of theparabola after reflection.
LP
A
NK

(ii) The portion of a tangent to a parabola cut off between the directrix & the curve subtends a right
angle at the focus.See figure above.
(iii) The tangents at the extremities of a focal chord intersect at right angles on the directrix, and
hence a circle on any focal chord as diameter touches the directrix. Also a circle on any focal
SA

radii of a point P (at2, 2at) as diameter touches the tangent at the vertex and intercepts a chord
of length a 1 + t 2 on a normal at the point P.

(iv) Any tangent to a parabola & the perpendicular on it from the focus meet on the tangent at the
vertex.
(v) If the tangents at P and Q meet in T, then:
⇒ TP and TQ subtend equal angles at the focus S.
⇒ ST2 = SP. SQ & ⇒ The triangles SPT and STQ are similar.
P

T
S ∠TSP = ∠TSQ

RI
P

A
S

Q 2(PS)(SQ)
= 2a
PS + SQ

UH
(vii) The area of the triangle formed by three points on a parabola is twice the area of the triangle
formed by the tangents at these points.
(viii) If normal are drawn from a point P(h, k) to the parabola y 2 = 4ax then
k = mh − 2am − am3 i.e. am3 + m(2a − h) + k = 0.
2a − h k
; m1 m2 m3 = − .
JA
m1 + m2 + m3 = 0 ; m1m2 + m2m3 + m3m1 =
a a
Where m1, m2, & m3 are the slopes of the three concurrent normals. Note that
A

P(h, k)

B
C A, B, C → Conormal points
LP

⇒ algebraic sum of the ordinates of the three conormal points on the parabola is zero
⇒ Centroid of the ∆ formed by three co−normal points lies on the x−axis.
⇒ Condition for three real and distinct normals to be drawn froma point P (h, k) is
4
A

h > 2a & k2 < (h – 2a)3.


27a
(ix) Length of subtangent at any point P(x, y) on the parabola y² = 4ax equals twice the abscissa of
the point P. Note that the subtangent is bisected at the vertex..
NK

O x
T D N
SA

(x) Length of subnormal is constant for all points on the parabola & is equal to the semi latus
rectum. See figure above.

16. Important Highlights of Ellipse :


x2 y2
Refering to the ellipse 2 + 2 = 1
a b
(i) If P be any point on the ellipse with S & S′ as its foci then  (SP) +  (S′P) = 2a.
(ii) The tangent & normal at a point P on the ellipse bisect the external & internal angles between
the focal distances of P. This refers to the well known reflection property of the ellipse which
states that rays from one focus are reflected through other focus & vice−versa. Hence we can
deduce that the straight lines joining each focus to the foot of the perpendicular from the other
focus upon the tangent at any point P meet on the normal PG and bisects it where G is the
point where normal at P meets the major axis.

RI
Intersection is a similiar ellipse as that of the original one.

A
UH
JA
LP

(iii) PG. Pg = SP. S′ P (iv) CG. CT = CS2


(v) Locus of the mid point of Gg is another ellipse having the same eccentricity as that of
A

the original ellipse.


[Where S and S′ are the focii of the ellipse and T is the point where tangent at P meet the major
axis]
NK

(vi) The circle on any focal distance as diameter touches the auxiliary circle. Perpendiculars from
the centre upon all chords which join the ends of any perpendicular diameters of the ellipse are
of constant length.
(vii) If the tangent at the point P of a standard ellipse meets the axis in
T and t and CY is the perpendicular on it from the centre then,
SA

(ii) least value of T t is a + b.

17. Important Highlights of Hyperbola:


(i) Difference of focal distances is a constant, i.e. |PS – PS′| = 2a
x2 y2
(ii) Locus of the feet of the perpendicular drawn from focus of the hyperbola 2
− = 1 upon any
a b2
tangent is its auxiliary circle i.e. x2 + y2 = a2 & the product of these perpendiculars is b2.
P
T
S′ S

T′

S
T

RI
A
UH
JA
x2 y2 x2 y2
+ = 1& the hyperbola − = 1 (a > k > b > 0) are
a2 b2 a2 − k 2 k 2 − b2
confocal and therefore orthogonal.
LP

(v) The foci of the hyperbola and the points P and Q in which any tangent meets the tangents at
the vertices are concyclic with PQ as diameter of the circle.
A

P A

S′ S
NK

(vi) A rectangular hyperbola circumscribing a triangle also passes through the orthocentre of this
SA

 
triangle. If  c ti , c  i = 1, 2, 3 be the angular points P, Q, R then orthocentre is
 ti 

 −c 
 t t t , − c t1 t 2 t 3  .
 1 2 3 
(vii) If a circle and the rectangular hyperbola xy = c2 meet in the four points t1, t2, t3 & t4,, then
(a) t1 t2 t3 t4 = 1
(b) the centre of the mean position of the four points bisects the distance between the
centres of the two curves.
(c) the centre of the circle through the points t1, t2 & t3 is :
c  1  c 1 1 1 
2  t1 + t 2 + t 3 + t t t  , 2  t + t + t + t1 t 2 t 3  
  1 2 3   1 2 3 
Example # 66 : A ray originating from the point (5, 0) is incident on the hyperbola 9x 2 – 16y2 = 144 at the point
P with abscissa 8. Find the equation of the reflected ray after first reflection and point P lying
in first quadrant.
x2 y2
Solution : Given hyperbola is 9x2 – 16y2 = 144. This equation can be rewritten as – =1 ....(1)
16 9
Since x co–ordinate of P is 8. Let y co–ordinate of P is α.  (8,α) lies on (1)

RI
A
UH
64 α 2
– =1 ⇒ α2 = 27 ⇒ α = 3 3 ( P lies in first quadrant)
16 9
Hence co-ordinate of point P is (8,3 3 ).

 Equation of reflected ray passes through P (8,3 3 ) and S′(–5,0)


JA
0−3 3
∴ Its equation is y–3 3 = (x – 8) or 13y – 39 3 = 3 3 x – 24 3
−5 − 8

or 3 3 x – 13y + 15 3 = 0.
LP
A
NK
SA
Marked questions are recommended for Revision.

PART - I : SUBJECTIVE QUESTIONS


Section (A) : Elementary concepts of Parabola
A-1. Find the value of λ for which the equation λx2 + 4xy + y2 + λx + 3y + 2 = 0 represents a parabola

A-2. Find
(i) The vertex, axis, focus, directrix, length of latusrectum of the parabola x 2 + 2y – 3x + 5 = 0.
(ii) The equation of the parabola whose focus is (1, 1) and the directrix is x + y + 1 = 0.
(iii) The equation to the parabola whose focus is (1, –1) and vertex is (2, 1).

RI
(iv) The equation of the directrix of the parabola x 2 – 4x – 3y + 10 = 0.

A-3. Find the equation of the parabola the extremities of whose latus rectum are (1, 2) and (1, –4).

A
A-4. Find the axis, vertex, focus, directrix and equation of latus rectum of the parabola
9y2 – 16x – 12y – 57 = 0

A-5. Find the locus of a point whose sum of the distances from the origin and the line x = 2 is 4 units.

UH
A-6. Find the value of α for which point (α, 2α + 1) doesn't lie outside the parabola y = x 2 + x + 1.

A-7. Find the set of values of α in the interval [π/2, 3π/2], for which the point (sinα, cosα) does not lie outside
the parabola 2y2 + x – 2 = 0.
JA
A-8. If a circle be drawn so as always to touch a given straight line and also a given circle externally then
prove that the locus of its centre is a parabola.(given line and given circle are non intersecting)

Section (B) : Elementary concepts of Ellipse & Hyperbola


B-1. Find the eccentricity of an ellipse of which distance between the focii is 10 and that of focus and
LP

corresponding directrix is 15.

B-2 If focus and corresponding directrix of an ellipse are (3, 4) and x + y – 1 = 0 respectively and
1
eccentricity is then find the co-ordinates of extremities of major axis.
2
A

 5  x2 y2
B-3 Find the set of those value(s) of 'α' for which the point  7 − α, α  lies inside the ellipse +
 4  25 16
NK

=1.

(x − 3)2 (y + 2)2
B-4. Write the parametric equation of ellipse + = 1.
25 16
SA

B-5. Find the set of possible value of α for which point P(α, 3α) lies on the smaller region of the ellipse
9x2 + 16y2 = 144 divided by the line 3x + 4y = 12.

B-6. Find the equation of the ellipse having its centre at the point (2, –3), one focus at (3, –3) and one vertex
at (4, –3).

B-7. Find the equation of the ellipse whose foci are (2, 3), (–2, 3) and whose semi-minor axis is 5 .

B-8. Find
(i) The centre, eccentricity, foci and directrices of the hyperbola 16x 2 – 9y2 + 32x + 36y – 164 = 0.
(ii) The equation of the hyperbola whose directrix is 2x + y = 1, focus (1, 2) and
eccentricity 3 .

B-9. For the hyperbola x2/100 − y2/25 = 1, prove that


(i) eccentricity = 5 / 2
(ii) SA . S′A = 25, where S & S′ are the foci & A is the vertex .
x2 y2
B-10. The foci of a hyperbola coincide with the foci of the ellipse + = 1. Find the equation of the
25 9
hyperbola if its eccentricity is 2.

B-11. Find
(i) The foci of the hyperbola 9x2 – 16y2 + 18x + 32y – 151 = 0
(ii) Equation of the hyperbola if vertex and focus of hyperbola are (2, 3) and (6, 3) respectively and
eccentricity e of the hyperbola is 2

B-12. Find the position of the point (2, 5) relative to the hyperbola 9x 2 – y2 = 1.

RI
B-13. Find the equation of auxiliary circle, of conic which passes through (1, 1) & is having foci (4, 5) & (2, 3).

B-14. Find the eccentricity of the hyperbola with its principal axes along the co-ordinate axes and which
(
passes through (3, 0) & 3 2 , 2 . )

A
x y
B-15. If m is a variable, then prove that the locus of the point of intersection of the lines – = m and

UH
3 2
x y 1
+ = is a hyperbola.
3 2 m

B-16. Given the base of a triangle and the ratio of the tangent of half the base angles. Show that the vertex
JA
moves on a hyperbola whose foci are the extremities of the base.

B-17. Show that for rectangular hyperbola xy = c2 , length of transverse axis, length of conjugate axis and
length of latus rectum are equal to 2 2 c

B-18. Prove that the distance of the point ( 6 cos θ , 2 sin θ) on the ellipse x2/6 + y2/2 = 1 from the centre of
the ellipse is 2, if θ = 5π/4
LP

B-19. Find the eccentricity of the ellipse which meets the straight line 2x – 3y = 6 on the x-axis and the
straight line 4x + 5y = 20 on the y-axis and whose axes lie along the coordinates axes.

x2 y2 x2 y2 1
A

B-20. If the foci of the ellipse + 2 = 1 & the hyperbola − = coincide then find the value of b2.
25 b 144 81 25

Section (C) : Position of line, Equation of chord and various forms of tangents of
NK

parabola

C-1. A line y = x + 5 intersect the parabola (y – 3)2 = 8(x + 2) at A & B. Find the length of chord AB.

 16 
SA

C-2. Chord joining two distinct points P(α2, k1) and Q  k 2 , −  on the parabola y2 = 16x always passes
 α 
through a fixed point. Find the co-ordinate of fixed point.

C-3. Find the locus of the mid-points of the chords of the parabola y 2 = 4ax which subtend a right angle at
the vertex of the parabola.

C-4. Two perpendicular chords are drawn from the origin ‘O’ to the parabola y = x 2, which meet the parabola
at P and Q Rectangle POQR is completed. Find the locus of vertex R.

C-5. Prove that the straight line x + my + n = 0 touches the parabola y 2 = 4ax if n = am2.

C-6. Find the range of c for which the line y = mx + c touches the parabola y 2 = 8 (x + 2).
C-7. Find the equation of that tangent to the parabola y 2 = 7x which is parallel to the straight line
4y – x + 3 = 0. Find also its point of contact.

C-8. A parabola y = ax2 + bx + c crosses the x − axis at (α, 0) (β, 0) both to the right of the origin. A circle also
passes through these two points. Find the length of a tangent from the origin to the circle.

C-9. If tangent at P and Q to the parabola y 2 = 4ax intersect at R then prove that mid point of R and M lies on
the parabola, where M is the mid point of P and Q.

Section (D) : Position of line, Equation of chord and various forms of tangents of
Ellipse & Hyperbola
D-1. Find the length of chord x – 2y – 2 = 0 of the ellipse 4x 2 + 16y2 = 64.

RI
x2 y2
D-2. Find the locus of the middle points of chords of an ellipse + = 1 which are drawn through the
a2 b2
positive end of the minor axis.

A
x2 y2
D-3. Check whether the line 4x + 5y = 40 touches the ellipse + = 1 or not. If yes, then also find its
50 32

UH
point of contact.

D-4. An ellipse passes through the point (4, − 1) and touches the line x + 4 y − 10 = 0. Find its equation if its
axes coincide with co−ordinate axes.

x2 y2
JA
D-5. Find the equation of the tangents at the ends of the latus rectum of the ellipse + = 1 and also
a2 b2
show that they pass through the points of intersection of the major axis and directrices.

D-6. Any tangent to an ellipse is cut by the tangents at the ends of major axis in the points T and T ′. Prove
that the circle, whose diameter is T T ′ will pass through the focii of the ellipse.
LP

x2 y2 25
D-7. If 'P' be a moving point on the ellipse + = 1 in such a way that tangent at 'P' intersect x =
25 16 3
at Q then circle on PQ as diameter passes through a fixed point. Find that fixed point.
A

x2 y2
D-8. AB is a chord to the curve S ≡ + – 1 = 0 with A (3, 0) and C is a point on line AB such that
9 16
AC : AB = 2 : 1 then find the locus of C.
NK

x2 y2
D-9. Find the length of chord x – 3y – 3 = 0 of hyperbola − = 1.
9 4

D-10. For what value of λ, does the line y = 3x + λ touch the hyperbola 9x 2 – 5y2 = 45 ?
SA

x2 y2
D-11. If the straight line 2x + 2 y + n = 0 touches the hyperbola – = 1, then find the value of n.
9 16

D-12. Find the equation of the tangent to the hyperbola x² − 4y² = 36 which is perpendicular to the line
x−y+4=0.

D-13. AB is a chord to the curve S ≡ x2 – y2 – 16 = 0 with A (4, 0) and C is a point on line segment AB such
that AC : AB = 1 : 2 then find the locus of C.

D-14. The curve xy = c(c > 0) and the circle x2 + y2 = 25 touch at two points, then find the distance between
the points of contact.
x2 y2
D-15. If the tangent on the point (3 sec φ, 4 tan φ) (which is in first quadrant) of the hyperbola – = 1 is
9 16
perpendicular to 3x + 8y – 12 = 0, then find the value of φ is (in degree).

Section (E) : Pair of tangents, Director circle, chord of contact and chord with given
middle point of Parabola

E-1. Find the equation of tangents to the parabola y 2 = 9x, which pass through the point (4, 10).

E-2. If two tangents to the parabola y2 = 4ax from a point P make angles θ1 and θ2 with the axis of the
parabola, then find the locus of P in each of the following cases.
(i) θ1 + θ2 = α (a constant)
π

RI
(ii) θ1 + θ2 =
2
(iii) tan θ1 + tanθ2 = λ (is constant)

A
E-3. The equation of a tangent to the parabola y 2 = 8x is y = x + 2. Find the point on this line from which the
other tangents to the parabola is perpendicular to the given tangent.

E-4. From the point (α, β) two perpendicular tangents are drawn to the parabola (x – 7) 2 = 8y. Then find the

UH
value of β.

E-5. Find the locus of the middle point of the focal chord of the parabola y 2 = 4x.

Section (F): Pair of tangents, Director circle, chord of contact and chord with given
JA
middle point of Ellipse & Hyperbola
x2 y2
F-1. Find the equation of tangents to the ellipse + = 1 which passes through a point (15, – 4).
50 32

x2 y2
F-2. If 3x + 4y = 12 intersect the ellipse + = 1 at P and Q, then find the point of intersection of
25 16
LP

tangents at P and Q.
x2 y2
F-3. Find the equation of chord of ellipse + = 1 whose mid point is (3, 1).
25 16
A

F-4. If m1 & m2 are the slopes of the tangents to the hyperbola x2/25 – y2/16 = 1 which passes through the
point (4, 2), find the value of (i) m1 + m2 & (ii) m1 m2 .
NK

F-5. Find the equations of the tangents to the hyperbola x 2 − 9 y2 = 9 that are drawn from (3, 2) . Find the
area of the triangle that these tangents form with their chord of contact.
F-6. Find the locus of the mid points of the chords of the circle x 2 + y2 = 16, which are tangent to the
hyperbola 9 x2 − 16 y2 = 144.
SA

F-7. Chords of the hyperbola, x2 − y2 = a2 touch the parabola, y2 = 4 a x. Prove that the locus of their middle
points is the curve, y2 (x − a) = x3.

x2 y2
F-8. Find the condition so that the line px + qy = r intersects the ellipse + = 1 in points whose
a2 b2
π
eccentric angles differ by .
4

Section (G) : Equation of normal, co-normal points of parabola


G-1. Find equation of all possible normals to the parabola x 2 = 4y drawn from point (1, 2).

G-2. If ax + by = 1 is a normal to the parabola y 2 = 4Px, then prove that Pa3 + 2aPb2 = b2.
G-3. Find the equation of normal to the parabola x2 = 4y at (6, 9).

G-4. The normal at the point P(ap2, 2ap) meets the parabola y2 = 4ax again at Q(aq2 , 2aq) such that the
lines joining the origin to P and Q are at right angle. Then prove that p 2 = 2.

G-5. If a line x + y = 1 cut the parabola y2 = 4ax in points A and B and normals drawn at A and B meet at C
(C does not lies on parabola). The normal to the parabola from C other, than above two meet the
parabola in D, then find D

G-6. If normal of circle x2 + y2 + 6x + 8y + 9 = 0 intersect the parabola y2 = 4x at P and Q then find the locus
of point of intersection of tangent’s at P and Q.

Section (H) : Equation of normal, co-normal points of Ellipse & Hyperbola

RI
x2 y2
H-1. If the normal at an end of a latus-rectum of an ellipse + = 1 passes through one extremity of
a2 b2
the minor axis, show that the eccentricity of the ellipse is given by e4 + e2 – 1 = 0

A
H-2. A ray emanating from the point (− 4, 0) is incident on the ellipse 9x² + 25y² = 225 at the point P with
abscissa 3. Find the equation of the reflected ray after first reflection.

UH
H-3. The tangent & normal at a point on x2/a2 − y2/b2 = 1 cut the y − axis respectively at A & B. Prove that the
circle on AB as diameter passes through the focii of the hyperbola .

H-4. The normal at P to a hyperbola of eccentricity e, intersects its transverse and conjugate axes at L and
JA
e
M respectively. Show that the locus of the middle point of LM is a hyperbola of eccentricity .
(e − 1)
2

Section (I) : Miscelleneous problems


I-1. Find the equation of a circle touching the parabola y 2 = 8x at (2, 4) and passes through (0, 4).
LP

I-2. An ellipse and a hyperbola have the same centre origin, the same foci and the minor-axis of the one is
the same as the conjugate axis of the other. If e1, e2 be their eccentricities respectively, then find value
1 1
of + .
e12 e22
A

x2 y2
I-3. x − 2y + 4 = 0 is a common tangent to y2 = 4x & + = 1. Then find the value of ‘b’ and the other
4 b2
common tangent.
NK

x2 y2
I-4. The line y = x intersects the hyperbola – = 1 at the points P and Q. Then find eccentricity of
9 25
5
ellipse with PQ as major axis and miror axis of length .
SA

I-5. Find the equation of common tangent to circle x2 + y2 = 5 and ellipse x2 + 9y2 = 9.

x2 y2
I-6. If latus rectum of ellipse + = 1 is double ordinate of parabola y2 = 4ax, then find the value of a.
25 16

PART - II : ONLY ONE OPTION CORRECT TYPE


Section (A) : Elementary concepts of Parabola

A-1. The equation of the parabola whose focus is (− 3, 0) and the directrix is x + 5 = 0 is:
(A) y2 = 4 (x − 4) (B) y2 = 2 (x + 4) (C) y2 = 4 (x − 3) (D) y2 = 4 (x + 4)
A-2. If (2, 0) is the vertex & y − axis is the directrix of a parabola, then its focus is:
(A) (2, 0) (B) (− 2, 0) (C) (4, 0) (D) (− 4, 0)

A-3. Length of the latus rectum of the parabola 25 [(x − 2)2 + (y − 3)2] = (3x − 4y + 7)2 is:
(A) 4 (B) 2 (C) 1/5 (D) 2/5

A-4. A parabola is drawn with its focus at (3, 4) and vertex at the focus of the parabola y 2 − 12 x − 4 y + 4 =0.
The equation of the parabola is:
(A) x2 − 6 x − 8 y + 25 = 0 (B) y2 − 8 x − 6 y + 25 = 0
(C) x − 6 x + 8 y − 25 = 0
2
(D) x2 + 6 x − 8 y − 25 = 0

A-5. Which one of the following equations parametrically represents equation to a parabolic profile?
t
(A) x = 3 cos t; y = 4 sin t (B) x2 − 2 = − 2 cos t; y = 4 cos2
2

RI
t t
(C) x = tan t; y = sec t (D) x = 1 − sin t ; y = sin + cos
2 2

A-6. The points on the parabola y2 = 12x whose focal distance is 4, are

A
( ) (
(A) 2, 3 , 2, − 3 ) ( ) (
(B) 1, 2 3 , 1, − 2 3 ) (C) (1, 2), (2, 1) ( ) (
(D) 2, 2 3 , 3, −2 3 )

UH
A-7. Find the all possible values of α such that point P(α, α) is outside the parabola y = x 2 + x + 1 and inside
the circle x2 + y2 = 50.
(A) (–5, ∞) (B) (–∞, ∞) (C) (–1, 5) (D) (–5, 5)

A-8. If on a given base, a triangle be described such that the sum of the tangents of the base angles is a
constant, then the locus of the vertex is :
JA
(A) a circle (B) a parabola (C) an ellipse (D) a hyperbola

A-9. Statement-1 : For triangle whose two vertices are ends of a double ordinate for a parabola and third
vertex lies on axis of same parabola incentre, circumcentre, centroid are collinear.
Statement-2 : In isosceles triangle incentre, circumcentre; orthocentre, centroid all lie on same line.
(A) STATEMENT-1 is true, STATEMENT-2 is true and STATEMENT-2 is correct explanation for
STATEMENT-1
LP

(B) STATEMENT-1 is true, STATEMENT-2 is true and STATEMENT-2 is not correct explanation
for STATEMENT-1
(C) STATEMENT-1 is true, STATEMENT-2 is false
(D) STATEMENT-1 is false, STATEMENT-2 is true
(E) Both STATEMENTS are false
A

 u2 u2 
A-10. The length of the latus rectum of the parabola whose focus is  sin 2α, − cos 2α  and directrix is
 2g 2g 
 
NK

u2
y= , is
2g
u2 u2 2u2 2u2
(A) cos2α (B) cos2α (C) cos2α (D) cos2α
g g g g
SA

2
A-11. The distance between the focus and directrix of the conic ( 3x − y ) ( )
= 48 x + 3y is :
(A) 24 (B) 48 (C) 6 (D) 12

A-12. If one end of a focal chord of the parabola y 2 = 4x is (1, 2), the other end doesn't lie on
(A) x2 y + 2 = 0 (B) xy + 2 = 0 (C) xy – 2 = 0 (D) x2 + xy – y – 1 = 0

A-13. The angle made by a double ordinate of length 8a at the vertex of the parabola y 2 = 4ax is :
(A) π/3 (B) π/2 (C) π/4 (D) π/6
Section (B) : Elementary concepts of Ellipse & Hyperbola
B-1. The equation of the ellipse whose focus is (1, –1), directrix is the line x – y – 3 = 0 and the eccentricity
1
is , is
2
(A) 7x2 + 2xy + 7y2 – 10x + 10y + 7 = 0 (B) 7x2 + 2xy + 7y2 + 7 = 0
(C) 7x + 2xy + 7y + 10x – 10y – 7 = 0
2 2
(D) 7x2 + 4xy + 7y2 – 10x + 10y + 7 = 0

B-2. The eccentricity of the ellipse 4x2 + 9y2 + 8x + 36y + 4 = 0 is


5 3 2 5
(A) (B) (C) (D)
6 5 3 3

x2 y2
B-3. The equation + + 1 = 0 represents an ellipse, if

RI
2−r r −5
(A) r > 2 (B) 2< r < 5 (C) r > 5 (D) r ∈ (2, 5) – {3.5}

B-4. The length of the latus rectum of the ellipse 9x 2 + 4y2 = 1, is

A
3 8 4 8
(A) (B) (C) (D)
2 3 9 9

UH
B-5. The equation of the ellipse with its centre at (1, 2), focus at (6, 2) and passing through the point (4, 6) is
(x − 1)2 (y − 2)2 (x − 1)2 (y − 2)2
(A) + =1 (B) + =1
45 20 20 45
(x − 1)2 (y − 2)2 (x − 1)2 (y − 2)2
(C) + =1 (D) + =1
25 16 16 25
JA
B-6. The position of the point (1, 3) with respect to the ellipse 4x 2 + 9y2 – 16x – 54y + 61 = 0
(A) outside the ellispe (B) on the ellipse
(C) on the major axis (D) on the minor axis

B-7. With respect to the hyperbola (3x – 3y)2 – (2x + 2y)2 = 36


(A) (3,2) lies on conjugate axis (B) (3,2) lies on tranverse axis
LP

(C) (3,2) lies inside hyperbola (D) (3,2) lies outside hyperbola

B-8. Equation of auxilliary circle of the ellipse 2x2 + 6xy + 5y2 = 1 is


7+3 5
(A) (x – 1)2 + y2 = 7–3 5 (B) x2 + y2 =
2
A

2 4
(C) x2 + y2 = (D) (x – 1)2 + y2 =
7+3 5 7+3 5
NK

B-9. Statement-1 : Eccentricity of ellipse whose length of latus rectum is same as distance between foci is
2sin18°.
x2 y2 b2
Statement-2 : For 2
+ 2
= 1, eccentricity e =
` 1−
a b a2
(A) STATEMENT-1 is true, STATEMENT-2 is true and STATEMENT-2 is correct explanation for
SA

STATEMENT-1
(B) STATEMENT-1 is true, STATEMENT-2 is true and STATEMENT-2 is not correct explanation
for STATEMENT-1
(C) STATEMENT-1 is true, STATEMENT-2 is false
(D) STATEMENT-1 is false, STATEMENT-2 is true
(E) Both STATEMENTS are false

B-10. The curve represented by x = 3 (cos t + sin t), y = 4 (cos t – sin t), is
(A) ellipse (B) parabola (C) hyperbola (D) circle

B-11. The eccentricity of the conic represented by x 2 – y2 – 4x + 4y + 16 = 0 is


(A) 1 (B) 2 (C) 2 (D) 1/2
B-12. Which of the following pair, may represent the eccentricities of two conjugate hyperbolas, for all
α ∈ (0, π/2) ?
(A) sin α, cos α (B) tan α, cot α (C) sec α, cosec α (D) 1 + sin α, 1 + cos α

B-13. For hyperbola represented by 16x2 – 3y2 – 32x + 12y – 44 = 0, which of the following statement is
INCORRECT
(A) the length of whose transverse axis is 4 3 (B) the length of whose conjugate axis is 8
19
(C) whose centre is (1, 2) (D) whose eccentricity is
3

 −π π 
B-14. Statement-1 : If secθ, θ ∈  ,  represent eccentricity of a hyperbola then eccentricity of its
 2 2

RI
conjugate hyperbola is given by cosecθ.
Statement-2 : If e1, e2 are eccentricities of two hyperbolas which are conjugate to each other then
e1–2 + e2–2 = 1
(A) STATEMENT-1 is true, STATEMENT-2 is true and STATEMENT-2 is correct explanation for

A
STATEMENT-1
(B) STATEMENT-1 is true, STATEMENT-2 is true and STATEMENT-2 is not correct explanation
for STATEMENT-1
(C) STATEMENT-1 is true, STATEMENT-2 is false

UH
(D) STATEMENT-1 is false, STATEMENT-2 is true
(E) Both STATEMENTS are false

B-15. The eccentricity of the hyperbola whose conjugate axis is equal to half the distance between the foci, is:
4 4 2 5
(A) (B) (C) (D)
JA
3 3 3 3

B-16. Identify the following statements for true/false (T/F) in order


S1 : A latus rectum of an ellipse is a line passing through a focus
S2 : A latus rectum of an ellipse is a line through the centre
S3 : A latus rectum of an ellipse is a line perpendicular to the major axis
S4 : A latus rectum of an ellipse is a line parallel to the minor axis
LP

(A) TFTF (B) TTFF (C) TFTT (D) FFFF

B-17. If P ( 2 sec θ, 2 tan θ) is a point on the hyperbola whose distance from the origin is 6 where P is
in the first quadrant then θ =
π π π
A

π
(A) (B) (C) (D)
4 3 6 15
NK

B-18. A rectangular hyperbola circumscribe a triangle ABC, then it will always pass through its
(A) orthocenter (B) circum centre (C) centroid (D) incentre

B-19. If P(x1, y1), Q(x2, y2), R(x3, y3) and S(x4, y4) are four concyclic points on the rectangular hyperbola
xy = c2, the coordinates of orthocentre of the ∆PQR are
(A) (x4, y4) (B) (x4, – y4) (C) (–x4, – x4) (D) (– x4, – y4)
SA

B-20. The co-ordinates of a focus of the hyperbola 9x2 – 16y2 + 18 x + 32y – 151 = 0 is
(A) (–1, 1) (B) (6, 1) (C) (4, 1) (D) (– 6, –1)

B-21. The set of values of ‘a’ for which (13x – 1)2 + (13y – 2)2 = a(5x + 12y – 1)2 represents an ellipse is
(A) 1 < a < 2 (B) 0 < a < 1 (C) 2 < a < 3 (D) 3 < a < 4

B-22. Find the equation of latus rectum of rectangular hyperbola xy = c 2


(A) x – y ± 2 2 c = 0 (B) x – y ± 2 c = 0 (C) x + y ± 2 2 c = 0 (D) x +y ± 2c = 0
Section (C) : Position of line, Equation of chord and various forms of tangents of
parabola

C-1. The locus of point of trisections of the focal chords of the parabola, y 2 = 4x is:
(A) y2 = x − 1 (B) 9y2 = 4.(3x – 4) (C) y2 = 2 (1 − x) (D) None of these

C-2. The latus rectum of a parabola whose focal chord is PSQ such that SP = 3 and SQ = 2 is given by:
(A) 24/5 (B) 12/5 (C) 6/5 (D) 23/5

C-3. Identify following statements for true/false (T/F) in order


S1 : The circles on focal radii of a parabola as diameter touch the tangent at the vertex
S2 : The circles on focal radii of a parabola as diameter touch the axis
S3 : A circle described on any focal chord of the parabola as its diameter will touch the directrix of the
parabola

RI
S4 : A circle described on any focal chord of the parabola as its diameter will touch the axis of the
parabola
(A) TTFF (B) TFTF (C) FFTT (D) FTFT

A
C-4. The length of the chord y = 3 x – 2 3 intercepted by the parabola y2 = 4(x – 1) is
16 8 4
(A) 4 3 (B) (C) (D)
3 3

UH
3

C-5. If y = 2 x − 3 is a tangent to the parabola y2= 4a  x − 1  , then ' a ' is equal to, where a ≠ 0 :
 3 
14 − 14
(A) 1 (B) − 1 (C) (D)
JA
3 3

C-6. An equation of a tangent common to the parabolas y2 = 4x and x2 = 4y is


(A) x – y + 1 = 0 (B) x + y – 1 = 0 (C) x + y + 1 = 0 (D) y = 0

C-7. Equation of a tangent to the parabola y2 = 12x which make an angle of 45° with line
y = 3x + 77 is
LP

(A) 2x – 4y + 3 = 0 (B) x – 2y + 12 = 0 (C) 4x + 2y + 5 = 0 (D) 2x + y – 12 = 0

C-8. Identify the following statements for true/false (T/F) in order


S1 : The tangents at the extremities of a focal chord of a parabola are perpendicular
S2 : The tangents at the extremities of a focal chord of a parabola are parallel
A

S3 : The tangents at the extremities of a focal chord of a parabola intersect on the directrix
S4 : The tangents at the extremities of a focal chord of a parabola intersect at the vertex
(A) TFTF (B) TTFF (C) TTTT (D) FFFF
NK

Section (D) : Position of line, Equation of chord and various forms of tangents of
Ellipse & Hyperbola
x2 y2
D-1. If the line y = 2x + c be a tangent to the ellipse + = 1, then c is equal to
8 4
SA

(A) ± 4 (B) ± 6 (C) ± 1 (D) ± 8

D-2. The distance of the point of contact from the origin of the line y = x – 7 with the ellipse 3x2 + 4y2 = 12,
is
5 5
(A) 3 (B) 2 (C) (D)
7 7
x y x2 y2
D-3. If + = 2 touches the ellipse 2 + 2 = 1 at a point P, then eccentric angle of P is
a b a b
(A) 0 (B) 45° (C) 60° (D) 90°

x2 y2
D-4. The point of intersection of the tangents at the point P on the ellipse + = 1 and its
a2 b2
corresponding point Q on the auxiliary circle, lies on the line :
b
(A) x = a/e (B) x = 0 (C) y = 0 (D) y =
e

D-5. A chord is drawn to the hyperbola xy = 4 from a point A(2, 2) which cuts it again at point B. The locus
of point P such that AP : PB = 2 :1
(A) (3x – 2) (3y – 2) = 16 (B) (2x – 3)(2y – 3) = 16
(C) xy = 2 (D) (3x – 2)(2y – 3) = 16

D-6. The number of possible tangents which can be drawn to the curve 4x 2 − 9y2 = 36, which are
perpendicular to the straight line 5x + 2y −10 = 0 is :
(A) zero (B) 1 (C) 2 (D) 4

D-7. The tangent at any point P(x1, y1) on the hyperbola xy = c2 meets the co-ordinate axes at points Q & R.

RI
The circumcentre of ∆OQR has co-ordinates.
x y   2x 2y 
(A) (0, 0) (B) (x1, y1) (C)  1 , 1  (D)  1 , 1 
 2 2  3 3 

A
D-8. The equation of the tangent lines to the hyperbola x 2 − 2y2 = 18 which are perpendicular to the line
y = x are :
(B) y = − x ± 3

UH
(A) y = – x ± 7 (C) y = – x ± 4 (D) none of these

Section (E) : Pair of tangents, Director circle, chord of contact and chord with given
middle point of Parabola
JA
E-1. The angle between the tangents drawn from a point ( – a, 2a) to y 2 = 4 ax is
π π π π
(A) (B) (C) (D)
4 2 3 6

E-2. The line 4x − 7y + 10 = 0 intersects the parabola, y 2 = 4x at the points A & B. The co-ordinates of the
point of intersection of the tangents drawn at the points A & B are:
7 5  5 7 5 7  7 5
LP

(A)  ,  (B)  − , −  (C)  ,  (D)  − , − 


2 2  2 2 2 2  2 2

E-3. The locus of the middle points of the focal chords of the parabola, y 2 = 4x is:
(A) y2 = x − 1 (B) y2 = 2 (x − 1) (C) y2 = 2 (1 − x) (D) y2 = 2(x + 1)
A

Section (F) : Pair of tangents, Director circle, chord of contact and chord with given
middle point of Ellipse & Hyperbola
NK

x2 y2
F-1. The equation of the locus of the middle point of the portion of the tangent to the ellipse + =1
16 9
included between the co-ordinate axes is the curve:
(A) 9x2 + 16y2 = 4 x2y2 (B) 16x2 + 9y2 = 4 x2y2
(C) 3x + 4y = 4 x y
2 2 2 2
(D) 9x2 + 16y2 = x2y2
SA

F-2. The equation of the chord of the ellipse 2x 2 + 5y2 = 20 which is bisected at the point (2, 1) is
(A) 4x + 5y + 13 = 0 (B) 4x + 5y = 13 (C) 5x + 4y + 13 = 0 (D) 5x + 4y = 13

F-3 Point, from which tangents to the ellipse 5x2 + 4y2 = 20 are not perpendicular, is:
(A) (1, 2 2 ) (B) (2 2 , 1) (C) (2, 5 ) (D) ( 5 , 3)

F-4. The locus of the middle points of chords of hyperbola 3x 2 – 2y2 + 4x – 6y = 0 parallel to y = 2x is
(A) 3x – 4y = 4 (B) 3y – 4x + 4 = 0 (C) 4x – 4y = 3 (D) 3x – 4y = 2

x2 y2
F-5. The chords passing through L(2, 1) intersects the hyperbola – = 1 at P and Q. If the tangents
16 9
at P and Q intersects at R then Locus of R is
(A) x – y = 1 (B) 9x – 8y = 72 (C) x + y = 3 (D) 9x + 8y = 7
F-6. The number of points from where a pair of perpendicular tangents can be drawn to the hyperbola,
x2 sec2 α − y2 cosec2 α = 1, α ∈ (0, π/4), is :
(A) 0 (B) 1 (C) 2 (D) infinite

F-7. Locus of the middle points of the parallel chords with gradient m of the rectangular hyperbola xy = c 2 is:
(A) y + mx = 0 (B) y − mx = 0 (C) my − x = 0 (D) my + x = 0

F-8. The tangents from (1, 2 2 ) to the hyperbola 16x2 – 25y2 = 400 include between them an angle equal
to:
π π π π
(A) (B) (C) (D)
6 4 3 2

RI
F-9. The locus of the mid points of the chords passing through a fixed point (α, β) of the hyperbola
x2 y2
− = 1 is :
a2 b2
α β α β

A
(A) a circle with centre  ,  (B) an ellipse with centre  , 
 2 2  2 2
α β
(C) a hyperbola with centre  ,  (D) straight line passing through

UH
 2 2

Section (G) : Equation of normal, co-normal points of parabola


G-1. The subtangent, ordinate and subnormal to the parabola y 2 = 4ax at a point (different from the origin)
JA
are in
(A) AP (B) GP (C) HP (D) none of these

G-2. Equation of the normal to the parabola, y 2 = 4ax at its point (am2, 2 am) is:
(A) y = − mx + 2am + am3 (B) y = mx − 2am − am3
(C) y = mx + 2am + am 3
(D) none
LP

G-3. At what point on the parabola y2 = 4x the normal makes equal angles with the axes?

(A) (4, 4) (B) (9, 6) (C) (4, – 1) (D) (1, 2)

G-4. The line 2x + y + λ = 0 is a normal to the parabola y 2 = – 8x, then λ is


A

(A) 12 (B) – 12 (C) 24 (D) – 24

G-5. The equation of the other normal to the parabola y 2 = 4ax which passes through the intersection of
those at (4a, − 4a) & (9a, − 6a) is:
NK

(A) 5x − y + 115 a = 0 (B) 5x + y − 135 a = 0 (C) 5x − y − 115 a = 0 (D) 5x + y + 115 = 0


G-6. The normal chord of a parabola y2 = 4ax at the point P(x1, x1) does not subtends a right angle at the
(A) focus (B) point (12a, 0)
(C) one of the end of the latus rectum (D) (a, 2a)
SA

G-7. If three normals can be drawn to the curve y 2 = x from point (c, 0) then 'c' can be equal to
1 1
(A) 0 (B) – (C) (D) 2
2 2

G-8. The locus of the middle points of normal chords of the parabola y 2 = 4a x is
(A) y4 − 2 a (x − 2 a). y2 + 8 a4 = 0 (B) y4 + 2 a (x − 2 a). y2 + 8 a4 = 0
(C) y4 − 2 a (x + 2 a). y2 + 8 a4 = 0 (D) y4 − 2 a (x − 2 a). y2 – 8 a4 = 0

Section (H) : Equation of normal, co-normal points of Ellipse & Hyperbola


x2 y2
H-1. If the line x cosα + y sinα = p be normal to the ellipse + = 1, then
a2 b2
(A) p2 (a2 cos2 α + b2 sin2 α) = a2 – b2 (B) p2 (a2 cos2 α + b2 sin2 α) = (a2 – b2)2
(C) p2 (a2 sec2 α + b2 cosec2 α) = a2 – b2 (D) p2 (a2 sec2 α + b2 cosec2 α) = (a2 – b2)2

 c
H-2. If the normal at  ct,  on the curve xy = c2 meets the curve again at t′, then
 t
1 1 1 1
(A) t′ = – 3 (B) t′ = (C) t′ = 2 (D) t′2 = – 2
t t t t
x2 y2
H-3. If the focal chord of the ellipse 2
+ = 1, (a > b) is normal at (acosθ, bsinθ) then eccentricity of the
a b2
ellipse is (it is given that sinθ ≠ 0)
(A) |secθ| (B) |cosθ| (C) |sinθ| (D) None of these

H-4. The locus of the foot of perpendicular drawn from the centre of the hyperbola

RI
x2 – y2 = 25 to its normal.
(A) 100x2y2 = (x2 + y2)2 (y2 – x2) (B) 10x2y2 = (x2 + y2)2 (y2 – x2)
(C) 200x2y2 = (x2 – y2)2 (y2 + x2) (D) 100x2y2 = (x2 – y2)2 (y2 + x2)

A
8 y2
H-5. The value of |λ|, for which the line 2x – λy = – 3 is a normal to the conic x2 + = 1 is
3 4
3 1 3 3
(A) (B) (C) (D)

UH
2 2 4 8

Section (I) : Miscelleneous problems

I-1. The feet of the perpendicular drawn from focus upon any tangent to the parabola,
JA
y = x2 − 2x − 3 lies on
(A) y + 4 = 0 (B) y = 0 (C) y = – 2 (D) y + 1 = 0

x2 y2
I-2. If F1 & F2 are the feet of the perpendiculars from the focii S1 & S2 of an ellipse + = 1 on the
5 3
tangent at any point P on the ellipse, then (S1F1). (S2F2) is equal to :
LP

(A) 2 (B) 3 (C) 4 (D) 5

x2 y2
I-3. P & Q are corresponding points on the ellipse + = 1, and the auxiliary circle respectively. The
16 9
normal at P to the ellipse meets CQ in R where C is centre of the ellipse. Then  (CR) is
A

(A) 5 units (B) 6 units (C) 7 units (D) 8 units

I-4. The equation of common tangent of x2 + y2 = 2 and y2 = 8x is


NK

(A) x – y + 2 = 0 (B) x + y + 1 = 0 (C) x – y + 1 = 0 (D) x + y – 2 = 0

I-5. The equation of common normal to the circle x2 + y2 – 12x + 16 = 0 and parabola y2 = 4x is
(A) y = 0 (B) 2x – y = 12 (C) 2x + y = 12 (D) All of the above

I-6. Equation of common tangent to ellipse 5x2 + 2y2 = 10 , and hyperbola 11x2 – 3y2 = 33 is
SA

(A) y = ± 3x ± 21 (B) y = ± x ± 3
(C) y = ± 4x ± 37 (D) 3x ± y = 12

I-7. The equation of common tangent to the parabola y2 = 8x and hyperbola 3x2 – y2= 3 is
(A) 2x ± y + 1 = 0 (B) 2x ± y – 1 = 0
(C) x ± 2y + 1 = 0 (D) x ± 2y – 1 = 0

x2 y2
I-8. x − 2y + 4 = 0 is a common tangent to y2 = 4x & + = 1. Then the value of ‘b’ and the other
4 b2
common tangent are given by :
(A) b = 3 ; x + 2y + 4 = 0 (B) b = 3; x + 2y + 4 = 0
(C) b = 3 ; x + 2y − 4 = 0 (D) b = 3 ; x − 2y − 4 = 0
PART - III : MATCH THE COLUMN
1. Match the column
Column – I Column – II
2 2
x y
(A) If the mid point of a chord of the ellipse + = 1 is (0, 3), (p) 6
16 25
4k
then length of the chord is , then k is
5
(B) Eccentric angle of one of the points on the ellipse x 2 + 3y2 = 6 at (q) 8

a distance 2 units from the centre of the ellipse is , then k is
4
(C) If ‘e’ is eccentricity and  is the length of latus rectum of the ellipse (r) 3
9x2 + 5y2 – 30y = 0, then 4 (e + ) is

RI
x2 y2
(D) Sum of distances of a point on the ellipse + = 1 from (s) 16
9 16
the focii

A
2. AB is a chord of the parabola y2 = 4ax joining A(at12, 2at1) and B (at22, 2at2). Match the following
Column – I Column – II
(A) AB is a normal chord (p) t 2 = – t1 + 2

UH
4
(B) AB is a focal chord (q) t2 = –
t1
1
(C) AB subtends 90º at (0, 0) (r) t2 = –
t1
2
JA
(D) AB is inclined at 45º to the axis of parabola (s) t2 = – t1 –
t1
4 x2 y2
3. A tangent having slope – touches the ellipse + = 1 at point P and intersects the major and
3 18 32
minor axes at A & B respectively, O is the centre of the ellipse
Column - I Column - II
LP

4
(A) Distance between the parallel tangents having slopes – , is (p) 24
3
(B) Area of ∆AOB is (q) 7/24
(C) If the tangent in first quadrant touches (r) 48/5
the ellipse at (h, k) then value of hk is
A

(D) If equation of the tangent intersecting positive axes (s) 12


is x + my = 1, then  + m is equal to
4. Column - I Column - II
NK

(A) Number of positive integral values of b for which tangent (p) 16


x2 y2
parallel to line y = x + 1 can be drawn to hyperbola − = 1 is
5 b2
(B) The equation of the hyperbola with vertices (3, 0) and (q) 2
(–3, 0) and semi-latusrectum 4, is given by is 4x2 – 3y2 = 4k, then k =
SA

(C) The product of the lengths of the perpendiculars (r) 4


from the two focii on any tangent to the hyperbola
x2 y2
– = 1 is k , then k is
25 3
(D) An equation of a tangent to the hyperbola, (s) 9
16x2 – 25y2 – 96x + 100y – 356 = 0 which makes an
π
angle with the transverse axis is y = x + λ , (λ > 0), then 2λ is
4
 Marked questions are recommended for Revision.

PART - I : ONLY ONE OPTION CORRECT TYPE


1. The vertex of a parabola is the point (a, b) and latus rectum is of length 1. If the axis of the parabola is
along the positive direction of y-axis, then its equation is :
1 1
(A) (x + a)2 = (2y – 2b) (B) (x – a)2 = (2y – 2b)
2 2
1 1
(C) (x + a)2 = (2y – 2b) (D) (x – a)2 = (2y – 2b)
4 8

RI
2. Length of the focal chord of the parabola y2 = 4ax at a distance p from the vertex is:
2a2 a3 4 a3 p2
(A) (B) (C) (D)
p p2 p2 a

A
3. The triangle PQR of area 'A' is inscribed in the parabola y 2 = 4ax such that the vertex P lies at the
vertex of the parabola and the base QR is a focal chord. The modulus of the difference of the ordinates
of the points Q and R is:

UH
A A 2A 4A
(A) (B) (C) (D)
2a a a a

4. AB is a chord of the parabola y2 = 4ax with vertex at A. BC is drawn perpendicular to AB meeting the
axis at C. The projection of BC on the axis of the parabola is
JA
(A) a (B) 2a (C) 4a (D) 8a

5. If P1Q1 and P2Q2 are two focal chords of the parabola y2 = 4ax. then the chords P1P2 and Q1Q2
1intersect on
(A) tangent at the vertex of the parabola (B) the directrix of the parabola
(C) at x = –2a (D) y = 2a and x = –2a
LP

6. The vertex of the locus of a point that divides a chord of slope 2 of the parabola y 2 = 4x internally in the
ratio 1 : 2 is

1 2 8 1 2 8  1 1
(A)  9,9  (B)  ,  (C)  ,  (D)  , 
A

  9 9 9 9 9 9

7. AB, AC are tangents to a parabola y2 = 4ax. p1, p2 & p3 are the lengths of the perpendiculars from A, B
NK

& C respectively on any tangent to the curve, then p2, p1, p3 are in:
(A) A.P. (B) G.P. (C) H.P. (D) none of these

8. The mirror image of the parabola y2 = 4x in the tangent to the parabola at the point (1, 2) is
(A) (x – 1)2 = 4(y – 2) (B) (x + 3)2 = 4(y + 2)
(C) (x + 1)2 = 4(y – 1) (D) (x – 1)2 = 4 (y – 1)
SA

9. A normal chord of the parabola subtending a right angle at the vertex makes an acute angle θ with the
x − axis, then θ =
(A) arc tan 2 (B) arc tan 2 (C) arc cot 2 (D) arc cot2

10. If two normals to a parabola y2 = 4ax intersect at right angles then the chord joining their feet passes
through a fixed point whose co-ordinates are:
(A) (− 2a, 0) (B) (a, 0) (C) (2a, 0) (D) (–a, 0)

11. If a parabola whose length of latus rectum is 4a touches both the coordinate axes then the locus of its
focus is
(A) xy = a2 (x2 + y2) (B) x2y2 = a2 (x2 + y2)
(C) x2 – y2 = a2 (x2 + y2) (D) x2y2 = a2 (x2 – y2)
12. T is a point on the tangent to a parabola y2 = 4ax at its point P. TL and TN are the perpendiculars on
the focal radius SP and the directrix of the parabola respectively. Then:
(A) SL = 2 (TN) (B) 3 (SL) = 2 (TN) (C) SL = TN (D) 2 (SL) = 3 (TN)

13. In the parabola y2 = 4ax, the locus of middle points of all chords of constant length c is
(A) (4ax – y2)(y2 – 4a2) = a2c2 (B) (4ax + y2)(y2 + 4a2) = a2c2
(C) (4ax + y )(y – 4a ) = a c
2 2 2 2 2
(D) (4ax – y2)(y2 + 4a2) = a2c2
14. Through the vertex 'O' of the parabola y2 = 4ax, variable chords OP and OQ are drawn at right angles.
If the variable chord PQ intersects the axis of x at R, then the distance OR is equal to
(A) 2a (B) 3a (C) 4a (D) 8a

15. From the focus of the parabola, y2 = 8x as centre, a circle is described so that a common chord of the

RI
curves is equidistant from the vertex & focus of the parabola. The equation of the circle is
(A) (x – 2)2 + y2 = 9 (B) (x – 2)2 + y2 = 3 (C) (x – 2)2 + y2 = 2 (D) (x – 2)2 + y2 = 1

16. If from the vertex of a parabola y2 = 4ax, a pair of chords be drawn at right angles to one another and

A
with these chords as adjacent sides a rectangle be made, locus of the further angle of the rectangle is
(A) y2 = 8a (x – 8a). (B) y2 = 4a (x + 8a). (C) y2 = – 4a (x – 8a). (D) y2 = 4a (x – 8a).

UH
17. A line of fixed length (a + b) moves so that its ends are always on two fixed perpendicular straight lines.
The locus of the point which divided this line into portions of lengths a & b, is :
(A) an ellipse (B) an hyperbola (C) a circle (D) a straight line

18. Coordinates of the vertices B and C of a triangle ABC are (2, 0) and (8, 0) respectively. The vertex A is
B C
JA
varying in such a way that 4 tan . tan = 1. Then locus of A is
2 2
(x − 5)2 y2 (x − 5)2 y2
(A) + =1 (B) + =1
25 16 16 25
(x − 5)2 y2 (x − 5)2 y2
(C) + =1 (D) + =1
25 9 9 25
LP

x2 y2
19. The locus of point of intersection of tangents to an ellipse + = 1 at the points, the sum of
a2 b2
whose eccentric angles is constant, is :
(A) a hyperbola (B) an ellipse (C) a circle (D) a straight line
A

20. An ellipse with major axis 4 and minor axis 2 touches both the coordinate axis, then Locus of its centre
is
NK

x2
(A) x2 – y2 = 5 (B) x2.y2 = 5 (C) + y2 = 5 (D) x2 + y2 = 5
4

21. An ellipse with major axis 4 and minor axis 2 touches both the coordinate axes, then locus of its focus
is
(A) (x2 – y2) (1 + x2y2) = 16 x2 y2 (B) (x2 – y2) (1 – x2y2) = 16 x2 y2.
SA

(C) (x2 + y2) (1 + x2y2) = 16 x2 y2 (D) (x2 + y2) (1 – x2y2) = 16 x2 y2.

22. A series of concentric ellipses E1, E2, ......, En are drawn such that En touches the extremities of the
major axis of En –1 and the foci of En coincide with the extremities of minor axis of En – 1. If the eccentricity
of the ellipses is independent of n, then the value of the eccentricity is
5 −1 5 −1 5 −1 5 −1
(A) (B) (C) (D)
4 16 2 8
x2 y2 y2 x2
23. If e and e′ are the eccentricities of the hyperbola – = 1 and – = 1, then the point
a2 b2 b2 a2
1 1 
 e , e′  lies on the circle :
 
(A) x2 + y2 = 1 (B) x2 + y2 = 2 (C) x2 + y2 = 3 (D) x2 + y2 = 4

x2 y2
24. P is a point on the hyperbola −
= 1, N is the foot of the perpendicular from P on the transverse
a2 b2
axis. The tangent to the hyperbola at P meets the transverse axis at T. If O is the centre of the
hyperbola, then OT. ON is equal to :
(A) e2 (B) a2 (C) b2 (D) b2/a2

RI
x2 y2
25. Tangent at any point on the hyperbola –
= 1 cut the axes at A and B respectively. If the
a2 b2
rectangle OAPB (where O is origin) is completed then locus of point P is given by
a2 b2 a2 b2 a2 b2

A
(A) 2 – 2 = 1 (B) 2 + 2 = 1 (C) 2 – 2 = 1 (D) none of these
x y x y y x

UH
x2 y2
26. If the chord of contact of tangents from two points (x 1, y1) and (x2, y2) to the hyperbola – = 1 are
a2 b2
x1 x 2
at right angles, then is equal to
y1 y 2
a2 b2 b4 a4
JA
(A) – (B) – (C) – (D) –
b2 a2 a4 b4

x2 y2
27^. The sides AC and AB of a triangle ABC touch the conjugate hyperbola of the hyperbola − = 1 at
a2 b2
x2 y2
C and B respectively. If the vertex A lies on the ellipse = 1 , the side BC
LP

+
a2 b2
(A) must touch the ellipse
(B) must cut the ellipse at two distinct points
(C) may not touch the ellipse
(D) may cut the ellipse at two distinct points
A

x2 y2
28. Tangents are drawn from any point on the hyperbola – = 1 to the circle x2 + y2 = 9, then the
9 4
NK

locus of mid-point of the chord of contact is


2 2
x2 y2  x2 + y2  x2 y2  x2 + y2 
(A) − =  (B) + = 
9 4  9 9 4  9
   
2 2
x2 y2  x2 − y2  x2 y2  x2 − y2 
SA

(C) + =   (D) − =  
9 4  9 9 4  9
   

x2 y2
29. If AB is a double ordinate of the hyperbola = 1 such that ∆OAB (O is the origin) is an

a2 b2
equilateral triangle, then the eccentricity ‘e’ of the hyperbola
2 2
(A) is greater than (B) is less than
3 3
2 1
(C) is equal to (D) is less than
3 3
30. Let two variable ellipse E1 and E2 touches each other externally at (0, 0). Their common tangent at
(0, 0) is y = x. If one of the focus at E1 & one of the focus of E2 always lies or line y = 2x then find locus
of other focus of E1 & E2.
(A) y = 4x (B) y = –2x (C) y = x/2 (D) y = –x/2

PART - II : SINGLE AND DOUBLE VALUE INTEGER TYPE


1. If (a2, a – 2) be a point interior to the region of the parabola y 2 = 2x bounded by the chord joining the
points (2, 2) and (8, – 4), then the number of all possible integral values of a is :

2. If  is the distance between focus and directrix of the parabola 9x 2 – 24xy + 16y2 – 20x – 15y – 60 = 0
then 6 is :

3. The number of integral values of a for which the point (–2a, a + 1) will be an interior point of the smaller

RI
region bounded by the circle x2 + y2 = 4 and the parabola y2 = 4x, is :

4. A variable chord PQ of the parabola, y2 = 4x is drawn parallel to the line y = x. If the parameters of the
points P & Q on the parabola be p & q respectively, then (p + q) equal to.

A
5. The parabola whose axis is parallel to the y-axis and which passes through the points (0, 4), (1, 9) and
(–2,6), also passes through (2, α) then the value of α is :

UH
6. Through the vertex O of the parabola y² = 8x, a perpendicular is drawn to any tangent meeting it at
P & the parabola at Q, then the value of OP. OQ is

7. The centre of the circle which passes through the focus of the parabola x 2 = 4 y & touches it at the point
(6, 9) is (α, β) then |α – β| is
JA
8. Points A, B & C lie on the parabola y² = 4ax. The tangents to the parabola at A, B & C, taken in pairs,
λ
intersect at points P, Q & R. the ratio of the areas of the triangles ABC & PQR is where λ and µ are
µ
co-prime number then λ + µ is
9. A normal is drawn to a parabola y2 = 4ax at any point other than the vertex and it cuts the parabola
LP

again at a point whose distance from the vertex is not less than λ 6 a , then the value of λ is

10. If three normal are drawn through (c, 0) to y2 = 4x and two of which of perpendicular then the value of c
is
x2 y2
P & Q are the points with eccentric angles θ & θ + π/6 on the ellipse
A

11. + = 1 , then the area of the


16 4
triangle OPQ is :
NK

x2 y2
12. If P is a variable point on the ellipse + =1 whose focii are S and S′ and e1 is the eccentricity and
a2 b2
 1
the locus of the incentre of ∆PSS′ is an ellipse whose eccentricity is e2 , then the value of  1 +  e22 is:
 e1 
SA

13. If (0, 3 + 5 ) is a point on the ellipse whose foci are (2, 3), (–2, 3) then the length of semimajor axis is :
14. A circle has the same centre as an ellipse & passes through the focii F 1 & F2 of the ellipse, such that
the two curves intersect at 4 points. Let 'P' be any one of their points of intersection. If the major axis of
the ellipse is 17 & the area of the triangle PF1F2 is 30, then the distance between the focii is :

15. Point 'O' is the centre of the ellipse with major axis AB & minor axis CD. Point F is one focus of the
ellipse. If OF = 6 & the diameter of the inscribed circle of triangle OCF is 2, then the product (AB) (CD)
is

16. If 'r' be the radius of largest circle with centre (3, 0) that can be inscribed in the ellipse
9x2 + 25y2 = 225, then 4 7 r is equal to
17. Minimum length of the intercept made by the axes on the tangent to the ellipse
x2 y2
+ = 1 is equal to
81 36

18. If the distance of the centre of the ellipse 4(x – 2y + 1)2 + 9(2x + y + 2)2 = 25 from the origin is λ times
its eccentricity, then 5λ2 is :

α
19. The radius of the largest circle with centre (1, 0) that can be inscribed in the ellipse x 2 + 4y2 =16 is
β
where α and β are prime number, then α + β is

20. Common tangents are drawn to the parabola y2 = 4x & the ellipse 3x2 + 8y2 = 48 touching the parabola
at A & B and the ellipse at C & D, then the area of the quadrilateral ABCD is λ 2 the λ is equal to

RI
x2 y2
21. A circle of radius r is concentric with the ellipse + = 1 and the common tangent is inclined to the
42 32

A
 r 2 − β2 
major axis at an angle of tan–1  2 2  ; r ∈ (b, a) then the value of |α| + |β| is
 α −r 

UH
x2 y2
22. If CF is perpendicular from the centre of the ellipse = 1 to the tangent at P, and G is the point
+
42 32
where the normal at P meets the major axis, then the product CF ⋅ PG is

λ
23. The eccentricity of an ellipse whose focii are (2, 4) & (14, 9) and touches x-axis is then the value
JA
313
of λ is

24. If two points P & Q on the hyperbola x2/a2 − y2/b2 = 1 whose centre is C be such that CP is
1 1  1 1 
perpendicular to CQ & a < b, then + = λ  2 − 2  where λ is :
CP 2
CQ 2
a b 
LP

25. If 7x2 + pxy + qy2 + rx – sy + t = 0 is the eqaution of the hyperbola whose one focus is (–1, 1),
eccentricity = 3 and the equation of the corresponding directrix is x – y + 3 = 0, then the value of 't' is :

x2 y2
A

26. The hyperbola − = 1 passes through the point of intersection of the lines, 7x + 13y − 87 = 0 &
a2 b2
5x − 8y + 7 = 0 & the latus rectum is 32 2 /5. The value of 2(a2 + b2) is :
NK

x2 y2
27. If m1 and m2 are slopes of the tangents to the hyperbola – = 1 which passes through the point
25 16
of contact of 3x – 4y = 5 and x2 – 4y2 = 5 then 32(m1 + m2 – m1m2) = ...................
SA

28. Tangents are drawn from the point (α, 2) to the hyperbola 3x 2 − 2y2 = 6 and are inclined at angles θ & φ
to the x −axis . If tan θ . tan φ = 2, then the value of 2α2 − 7 is

x2 y2
29. C the centre of the hyperbola – = 1. The tangents at any point P on this hyperbola meets the
9 16
striaght lines 4x – 3y = 0 and 4x + 3y = 0 in the points Q and R respectively. Then CQ . CR =

x2 y2 x2 y2
30. If radii of director circles of + = 1 and –
= 1 are 2r and r respectively and ee and eh be
a2 b2 a2 b2
the eccentricities of the ellipse and the hyperbola respectively then 4eh2 – ee2 is equal to

31. The length of that focal chord of the hyperbola xy = 8 which touches the circle x 2 + y2 = 8 is.
x2 y2
32. The sum of lengths of perpendiculars drawn from focii to any real tangent to the hyperbola – =1
16 9
is always greater than a, then find maximum value of a.

33. Let tangent at point A, B and vertex (V) of parabola is x – 2y + 1 = 0, 3x + y + 4 = 0 and y = x


a b
respectively. If focus of parabola is  ,  then find the value of (a + 5b).
7 7
x2 y2
34. If common tangent of x2 + y2 = r2 and + = 1 forms square then find its area.
16 9

35. Let x2 + y2 = r2 and xy = 1 intersect at A & B in first quadrant, If AB = 14 then find the value of r.

RI
PART - III : ONE OR MORE THAN ONE OPTIONS CORRECT TYPE
1. Let A be the vertex and L the length of the latus rectum of the parabola, y 2 − 2 y − 4 x − 7 = 0. The
equation of the parabola with A as vertex, 2 L the length of the latus rectum and the axis at right angles

A
to that of the given curve is:
(A) x2 + 4 x + 8 y − 4 = 0 (B) x2 + 4 x − 8 y + 12 = 0
(C) x + 4 x + 8 y + 12 = 0
2
(D) x2 + 8 x − 4 y + 8 = 0

UH
2. The locus of the mid point of the focal radii of a variable point moving on the parabola, y 2 = 4ax is a
parabola whose
(A) Latus rectum is half the latus rectum of the original parabola
(B) Vertex is (a/2, 0)
(C) Directrix is y-axis
JA
(D) Focus has the co-ordinates (a,0)

3. P is a point on the parabola y2 = 4ax (a > 0) whose vertex is A. PA is produced to meet the directrix in D
and M is the foot of the perpendicular from P on the directrix. If a circle is described on MD as a
diameter then it intersects the x−axis at a point whose co−ordinates are:
(A) (− 3a, 0) (B) (− a, 0) (C) (− 2a, 0) (D) (a, 0)
LP

4. Let y2 = 4ax be a parabola and x2 + y2 + 2 bx = 0 be a circle. If parabola and circle touch each other
externally then:
(A) a > 0, b > 0 (B) a > 0, b < 0 (C) a < 0, b > 0 (D) a < 0, b < 0
A

5. P is a point on the parabola y2 = 4x where abscissa and ordinate are equal. Equation of a circle passing
through the focus and touching the parabola at P is:
(A) x2 + y2 − 13x + 2y + 12 = 0 (B) x2 + y2 − 3x − 18y + 2 = 0
NK

(C) x + y + 13x − 2y − 14 = 0
2 2
(D) x2 + y2 – x = 0

6^. Subset of complete set of values of m for which a chord of slope m of the circle x 2 + y2 = 4 touches
parabola y2 = 4x, can be
 2 −1   2 −1 
(A)  −∞, −  (B) (0 , 1/2) (C)  ,∞  (D) (–1/2, 0)
SA

 2   2 
   

7. Locus of the centre of the circle passing through the vertex and the mid-points of perpendicular chords
from the vertex of the parabola y2 = 4ax is.
(A) is a parabola with vertex (–a, a) (B) is a parabola with latus rectum a
a
(C) is a parabola with vertex (2a,0) (D) is a parabola with latus rectum
2

8. The equation, 3x2 + 4y2 − 18x + 16y + 43 = C.


(A) cannot represent a real pair of straight lines for any value of C
(B) represents an ellipse, if C > 0
(C) no locus, if C < 0
(D) a point, if C = 0
x2 y2
9. If P is a point of the ellipse + = 1, whose focii are S and S′. Let ∠PSS′ = α and ∠PS′S = β, then
a2 b2
(A) PS + PS′ = 2a, if a > b
(B) PS + PS′ = 2b, if a < b
α β 1− e
(C) tan tan =
2 2 1+ e
α β a2 − b2
(D) tan tan = [a – a2 − b2 ] when a > b
2 2 b2

10. Let A(α) and B(β) be the extremeties of a chord of an ellipse . If the slope of AB is equal to the slope of
the tangent at a point C(θ) on the ellipse, then the value of θ, is
α+β α −β α+β α −β
+π –π

RI
(A) (B) (C) (D)
2 2 2 2

11. Let F1, F2 be two focii of the ellipse and PT and PN be the tangent and the normal respectively to the
ellipse at point P then

A
(A) PN bisects ∠ F1 PF2 (B) PT bisects ∠F1PF2
(C) PT bisects angle (180° – ∠ F1PF2) (D) None of these

UH
12. If 1 be the equation of the common tangent in 1st quadrant to the circle x2 + y2 = 16 and ellipse
x2 y2
+ = 1 and λ1 be the length of the intercept of the common tangent between the coordinate axes
25 4
then
JA
14
(A) λ1 = (B) Equation of 1 is 2x + 3y = 4 7
3
4
(C) λ1 = (D) Equation of 1 is x + 3y = 4 7
3
LP

x2 2 2 y2
13. Let E1 and E2 be two ellipses + y = 1 and x + = 1 (where a is parameter) the locus of points
a2 a2
of intersection of the ellipses E1 and E2 is a set of curves
(A) y = x, y = − x, x2 + y2 = 1 (B) y = 2x, y = − 2x, x2 + y2 = 4
A

2 2 2 2
(C) (4x – y ) (x + y –4) = 0 (D) (x2 – y2) (x2 + y2 –1) = 0
14. If (5, 12) and (24, 7) are the foci of a conic, passing through the origin then the eccentricity of conic is
(A) 386 /12 (B) 386 /13 (C) 386 /25 (D) 386 /38
NK

15. The equation of a hyperbola with co-ordinate axes as principal axes, if the distances of one of its
vertices from the foci are 3 & 1 can be :
(A) 3x2 − y2 = 3 (B) x2 − 3y2 + 3 = 0 (C) x2 − 3y2 − 3 = 0 (D) x2 − 3y2 − 6 = 0

16. A point moves such that the sum of the squares of its distances from the two sides of length 'a' of a
SA

rectangle is twice the sum of the squares of its distances from the other two sides of length 'b'. The
locus of the point can be :
(A) a circle (B) an ellipse (C) a hyperbola (D) a pair of lines

17. If (3sinα, 2cosα) lies on the same side as that of origin w.r.t conic 2x 2 – 3y2 = 6, then sinα may be
4 2 1 2
(A) – (B) (C) (D)
5 5 5 15

18. Which of the following equations in parametric form can represent a hyperbolic profile, where 't' is a
parameter.
a  1 b  1 tx y x ty
(A) x =  t +  & y = t −  (B) − +t=0& + −1=0
2  t 2  t a b a b
t
(C) x = et + e−t & y = et − e−t (D) x2 − 6 = 2 cos t & y2 + 2 = 4 cos2
2
19. If two distinct tangents can be drawn from the point (α, 2) on different branches of the hyperbola
x2 y2
− = 1, then the range of α is subset of
9 16
 3 3  1 1
(A)  – ,  (B) [–2,2] (C) [–1,1] (D)  – , 
 2 2  2 2

20. A rectangular hyperbola whose centre is C is cut by any circle of radius r in four points P, Q, R and S.
Then CP2 + CQ2 + CR2 + CS2 =
(A) 16 if r = 2 (B) 16 if r = 2 (C) 2 if r = 1 (D) 4 if r = 1

21. x2 + y2 = 16 is the auxilliary circle of

RI
(A) 9x2 – 16y2 – 144 = 0 (B) 16x2 – 9y2 + 144 = 0
(C) 9(x – y)2 – 16 (x + y)2 – 288 = 0 (D) 16(x – y)2 – 9(x + y)2 + 288 = 0
x2 y2
22. If the chord joining the points whose eccentric angles are ‘α’ and ‘β’ on the hyperbola − = 1 is a

A
a2 b2
focal chord then
 α −β α+β

UH
(A) ± ecos   = cos  2 
 2   
α+β  α −β
(B) ± ecos   = cos  2 
 2   
 ke − 1 
(C) tan(α/2) tan(β/2) +   = 0 where k = ± 1
JA
 ke + 1 
 ke + 1 
(D) tan(α/2) tan(β/2) +   = 0 where k = ± 1
 ke − 1 
23. If the normal at P to the rectangular hyperbola x2 − y2 = 4 meets the axes in G and g and C is the centre
of the hyperbola, then
(A) PG = PC (B) Pg = PC (C) PG = Pg (D) Gg = PC
LP

24. Circles are drawn on chords of rectangular hyperbola xy = c2 parallel to the line y = x as diameters. All
such circles pass through two fixed points whose co-ordinates are :
(A) (c, c) (B) (c, − c) (C) (− c, c) (D) (− c, − c)
A

25. If a circle and the rectangular hyperbola xy = c2 meet in the four points t1, t2, t3 & t4, then
(A) t1 t2 t3 t4 = 1
(B) The arthmetic mean of the four points bisects the distance between the centres of the two curves.
NK

(C) The geometrical mean of the four points bisects the distance between the centres of the two curves.
(D) the centre of the circle through the points t1, t2 & t3 is :
c  1  c 1 1 1 
2  t1 + t 2 + t 3 + t t t  , 2  t + t + t + t1 t 2 t 3  
  1 2 3   1 2 3 
26. Two confocal parabola intersect at A and B. If their axis are parallel to x-axis and y-axis respectively,
SA

then slope of chord AB can be :


(A) 1 (B) –1 (C) 2 (D) –2

PART - IV : COMPREHENSION
Comprehension # 1 (For Q.No. 1 to 3)

Consider three lines y axis, y = 2 and x + my = 1 where (, m) lies on y 2 = 4x. answer the following :

1. Locus of circum centre of triangle formed by given three lines is a parabola whose vertex is
(A) (–2, 3/2) (B) (2, –3/2) (C) (–2, –3/2) (D) (2, –5/2)

2. Area of triangle formed by vertex and end points of latus rectum of parabola obtained in questions (1)
is
1 1 1 1
(A) 8
(unit)2 (B) 9
(unit)2 (C) 10
(unit)2 (D) (unit)2
2 2 2 27

3. Any point on the parabola obtained in question (1) can be represented as


 1 2 3 t   t 2 –3 1 2   1 2 3 t   1 2 3 t
(A)  2 + t , +  (B)  2 + , + t  (C)  –2 + t , +  (D)  –2 + 16 t , 2 + 5 
 32 2 16   32 2 16   32 2 16   
 

Comprehension # 2
Let PQ be a variable focal chord of the parabola y2 = 4ax where vertex is A. Locus of, centroid of
triangle APQ is a parabola ‘P1’

4. Latus rectum of parabola P1 is

RI
2a 4a 8a 16a
(A) (B) (C) (D)
3 3 3 3
5. Vertex of parabola P1 is
 2a   4a   8a  a 

A
(A)  , 0  (B)  , 0 (C)  , 0  (D)  , 0 
 3   3   3  3 
6. Let ∆1 is the area of triangle formed by joining points T1, T2 and T3 on parabola P1 and ∆2 be the area of

UH
triangle T formed by tangents at T1, T2 and T3, then
(A) ∆2 = 2∆1
(B) ∆1 = 4∆2
(C) orthocentre of triangle T lies on x = a/3.
(D) Both (A) and (C) are correct.
JA
Comprehension # 3 
x2 y2
Two tangents PA and PB are drawn from a point P(h, k) to the ellipse E : = 1 (a > b). Angle of
+
a2 b2
the tangents with the positive x - axis are θ1 and θ2. Normals at A and B are intersecting at Q point.
On the basis of above information answer the following questions.
LP

7. Locus of P, if tan θ1 . tan θ2 = 4, is


y–b y+b 4(x – a) y+b x+a
(A) = 2(x + a) (B) y2 – b2 = 2(x2 + a2) (C) = (D) =
x–a x+a y–b x–b y–b
A

8. Circumcentre of ∆QAB is


(A) mid point of AB (B) mid point of PQ (C) orthocentre of ∆PAB (D) can't say
NK

9. Locus of P, if cot θ1 + cotθ2 = λ, is


(A) 2xy = λ(y2 – b2) (B) 2xy – λ(b2 – y2) = 0 (C) xy = λ (D) x2 + xy = λ

Comprehension # 4
SA

Asymptotes are lines whose distance from the curve at infinity tends to zero Let y = mx + c is
x2 y2
asymptote of H − = – 1. Solving the two equations, we have (b2 – a2m2) x2 – 2a2mcx – a2
a2 b2
b
(b2 + c2) = 0. Both roots of this equation must be infinite so m = ± and c = 0 which implies that
a
b x2 y2
y = ±x are asymptotes of − = 1. Note that no real tangent can be drawn to the
a a2 b2
hyperbola from its centre and only one real tangent can be drawn from a point lying on its
x2 y2
asymptote other than centre. Further combined equation of asymptotes is A = 2 − 2 = 0 and
a b
x2 y2
conjugate hyperbola C = − + 1 = 0 . Hence 2A = H + C, as we can see, equation of A, H
a2 b2
and C vary only by a constant, for asymptotes which can be evaluated by applying condition of
pair of lines.
y
y=
– b b x
a
a x y=

x
O

(b2 + c2)

x2 y2
− = 1 from point (2, 1) are
3 2
9 2
(A) (3, 2), (1, 5) (B) (3, 2),  ,  (C) (1, 2), (3, 4) (D) (3, 2), (3, 4)
5 5

RI
11. The number of real distinct tangents drawn to hyperbola 4x2 – y2 = 4 from point (1, 2) is
(A) 1 (B) 2 (C) 3 (D) 4

A
12. The number of real distinct tangents drawn from point (1, 2) to hyperbola x 2 – y2 – 2x + 4y – 4 = 0 is
(A) 1 (B) 2 (C) 3 (D) None of these

13. The asymptotes of xy – 3y – 2x = 0 is

UH
(A) x + 2 = 0 and y + 3 = 0 (B) x – 2 = 0 and y – 3 = 0
(C) x – 3 = 0 and y – 2 = 0 (D) x + 3 = 0 and y + 2 = 0

Comprehension # 5
JA
Equation of the transverse and conjugate axis of a hyperbola are respectively x + 2y – 3 = 0,
2
2x – y + 4 = 0 and their respectively lengths are 2 and then answer following :
3
14. If x2 + 2hxy + by2 + 2gx + 2fy + c = 0 is equation of given hyperbola where h, b, g, f, c all are integers
then the sum h + b + g + f + c =
(A) 3 (B) 4 (C) 5 (D) 6
LP

15. Equation of one of the directrix is


3 2
(A) 2x – y + 4 + =0 (B) x + 2y + 4 – =0
2 3
3 3
A

(C) 2x – y = (D) 2x – y + 4 + = 3
2 2

16. Coordinates of one of possible focus of hyperbola is


NK

 2 1   2   1 
(A)  –1 + ,2–  (B)   –1 + ,  2 – 
 6 6  5  5  
 2   1   2   1 
(C)   –1–
 ,  2 +  (D)   –1 −  , 2 – 
 5  5    5  5  
SA
* Marked Questions may have more than one correct option.

PART - I : JEE (ADVANCED) / IIT-JEE PROBLEMS (PREVIOUS YEARS)


1*. The tangent PT and the normal PN to the parabola y 2 = 4ax at a point P on it meet its axis at points T
and N, respectively. The locus of the centroid of the triangle PTN is a parabola whose
[IIT-JEE 2009, Paper-2, (4, –1), 80]
 2a  2a
(A) vertex is  , 0  (B) directrix is x = 0 (C) latus rectum is (D) focus is (a, 0)
 3  3

RI
2. The line passing through the extremity A of the major axis and extremity B of the minor axis of the
ellipse
x2 + 9y2 = 9 meets its auxiliary circle at the point M. Then the area of the triangle with vertices at A, M
and the origin O is [IIT-JEE 2009, Paper-1, (3, –1)/ 80]

A
31 29 21 27
(A) (B) (C) (D)
10 10 10 10

UH
A
3*. In a triangle ABC with fixed base BC, the vertex A moves such that cos B + cos C = 4 sin 2 . If a, b
2
and c denote the lengths of the sides of the triangle opposite to the angles A, B and C respectively,
then [IIT-JEE 2009, Paper-1, (4, –1)/ 80]
(A) b + c = 4a (B) b + c = 2a
(C) locus of points A is an ellipse
JA (D) locus of point A is a pair of straight lines

4. The normal at a point P on the ellipse x2 + 4y2 = 16 meets the x-axis at Q. If M is the mid point of the
line segment PQ, then the locus of M intersects the latus rectum of the given ellipse at the points
[IIT-JEE 2009, Paper-2, (3, –1)/ 80]
 3 5 2  3 5 19   1  4 3
(A)  ± , ±  (B)  ± , ±  (C)  ±2 3, ±  (D)  ±2 3, ± 
    7  7 
LP

 2 7  2 4  
5. Match the conics in Column - I with the statements/expressions in Column - II.
[IIT-JEE-2009, Paper-1, (8, 0), 80]

Column - I Column - II
A

(A) Circle (p) The locus of the point (h, k) for which
the line hx + ky = 1 touches the
circle x 2 + y2 = 4
NK

(B) Parabola (q) Points z in the complex plane


satisfying |z + 2| – |z – 2| = ± 3

(C) Ellipse (r) Points of the conic have parametric


 1– t 2 
representation x = 3 
 1 + t 2 
,
SA

 
2t
y=
1 + t2
(D) Hyperbola (s) The eccentricity of the conic lies in the
interval 1 ≤ x < ∞

(t) Points z in the complex plane


satisfying Re(z + 1)2 = |z|2 + 1

6*. An ellipse intersects the hyperbola 2x2 – 2y2 = 1 orthogonally. The eccentricity of the ellipse is reciprocal
of that of the hyperbola. If the axes of the ellipse are along the coordinate axes, then
[IIT-JEE 2009, Paper-2, (4, –1), 80]
(A) Equation of ellipse is x2 + 2y2 = 2 (B) The foci of ellipse are (±1, 0)
(C) Equation of ellipse is x2 + 2y2 = 4 (D) The foci of ellipse are (±, 2 0)

7*. Let A and B be two distinct points on the parabola y 2 = 4x. If the axis of the parabola touches a circle of
radius r having AB as its diameter, then the slope of the line joining A and B can be
[IIT-JEE-2010, Paper-1(3, 0)/84]
1 1 2 2
(A) – (B) (C) (D) –
r r r r

Comprehension # 1 (Q.8 - 10)


x2 y2
Tangents are drawn from the point P(3, 4) to the ellipse + = 1 touching the ellipse at point A and
9 4
B.

RI
8. The coordinates of A and B are [IIT-JEE 2010, Paper-2, (3, –1), 79]
 8 2 161   9 8
(A) (3, 0) and (0, 2) (B)  – ,  and  – , 
 5 15   5 5

 8 2 161   9 8

A
(C)  – ,  and (0, 2) (D) (3, 0) and  – , 
 5 15   5 5

UH
9. The orthocentre of the triangle PAB is [IIT-JEE 2010, Paper-2, (3, –1), 79]
 8 7 25   11 8  8 7
(A)  5 ,  (B)  , (C)  , (D)  ,
 7   5 8   5 5 
  25 5 

10. The equation of the locus of the point whose distances from the point P and the line AB are equal, is
JA
(A) 9x2 + y2 – 6xy – 54x – 62y + 241 = 0 (B) x2 + 9y2 + 6xy – 54x + 62y – 241 = 0
2 2
(C) 9x + 9y – 6xy – 54x – 62y – 241 = 0 (D) x2 + y2 – 2xy + 27x + 31y – 120 = 0
[IIT-JEE 2010, Paper-2, (3, –1), 79]

Comprehension
x2 y2
The circle x2 + y2 – 8x = 0 and hyperbola – = 1 intersect at the points A and B.
LP

9 4
11. Equation of a common tangent with positive slope to the circle as well as to the hyperbola is
(A) 2x – 5 y – 20 = 0 (B) 2x – 5 y + 4 = 0
(C) 3x – 4y + 8 = 0 (D) 4x – 3y + 4 = 0 [IIT-JEE-2010, Paper-1(3, –1)/84]
A

12. Equation of the circle with AB as its diameter is [IIT-JEE-2010, Paper-1(3, –1)/84]
(A) x2 + y2 – 12x + 24 = 0 (B) x2 + y2 + 12x + 24 = 0
(C) x2 + y2 + 24x – 12 = 0 (D) x2 + y2 – 24x – 12 = 0
NK

x2 y2
13. The line 2x + y = 1 is tangent to the hyperbola – = 1 . If this line passes through the point of
a2 b2
intersection of the nearest directrix and the x-axis, then find the eccentricity of the hyperbola.
[IIT-JEE-2010, Paper-1(3, 0)/84]
SA

14. Consider the parabola y2 = 8x. Let ∆1 be the area of the triangle formed by the end points of its latus
1 
rectum and the point P  , 2  on the parabola, and ∆2 be the area of the triangle formed by drawing
2 

tangents at P and at the end points of the latus rectum. Then 1 is [IIT-JEE 2011, Paper-1, (4, 0), 80]
∆2

15. Let (x, y) be any point on the parabola y2 = 4x. Let P be the point that divides the line segment from
(0, 0) to (x, y) in the ratio 1 : 3. Then the locus of P is [IIT-JEE 2011, Paper-2, (3, –1), 80]
(A) x2 = y (B) y2 = 2x (C) y2 = x (D) x2 = 2y

16*. Let L be a normal to the parabola y2 = 4x. If L passes through the point (9, 6), then L is given by
(A) y – x + 3 = 0 (B) y + 3x – 33 = 0 (C) y + x – 15 = 0 (D) y – 2x + 12 = 0
[IIT-JEE 2011, Paper-2, (4, 0), 80]
x2 y2
17*. Let the eccentricity of the hyperbola – 2 = 1 be reciprocal to that of the ellipse x2 + 4y2 = 4. If the
a2 b
hyperbola passes through a focus of the ellipse, then [IIT-JEE 2011, Paper-1, (4, 0), 80]
2 2
x y
(A) the equation of the hyperbola is – =1
3 2
(B) a focus of the hyperbola is (2, 0)
5
(C) the eccentricity of the hyperbola is
3
(D) the equation of the hyperbola is x2 – 3y2 = 3

x2 y2
18. Let P(6, 3) be a point on the hyperbola −= 1 . If the normal at the point P intersects the x-axis at

RI
a2 b2
(9, 0), then the eccentricity of the hyperbola is [IIT-JEE 2011, Paper-2, (3, –1), 80]
5 3
(A) (B) (C) 2 (D) 3

A
2 2

x2 y2
19. The ellipse E1 : + = 1 is inscribed in a rectangle R whose sides are parallel to the coordinate

UH
9 4
axes. Another ellipse E2 passing through the point (0, 4) circumscribes the rectangle R. The eccentricity
of the ellipse E2 is [IIT-JEE 2012, Paper-1, (3, –1), 70]
2 3 1 3
(A) (B) (C) (D)
2 2 2 4
JA
x2 y2
20. Tangents are drawn to the hyperbola – = 1 , parallel to the straight line 2x – y = 1. The points of
9 4
contacts of the tangents on the hyperbola are [IIT-JEE 2012, Paper-1, (4, 0), 70]
 9 1   9 1 
(A)  ,
2 2 2 
 (B)  –
 2 2
,–
2
 (
(C) 3 3, –2 2 ) (
(D) –3 3, 2 2 )
LP

21. Let S be the focus of the parabola y2 = 8x and let PQ be the common chord of the circle
x2 + y2 – 2x – 4y = 0 and the given parabola. The area of the triangle PQS is.
[IIT-JEE 2012, Paper-1, (4, 0), 70]

Paragraph for Question Nos. 22 to 23


A

Let PQ be a focal chord of the parabola y2 = 4ax. The tangents to the parabola at P and Q meet at a
point lying on the line y = 2x + a, a > 0. [IIT-JEE - 2013, Paper-2, (3,–1), 60]
NK

22. Length of chord PQ is


(A) 7a (B) 5a (C) 2a (D) 3a

23. If chord PQ subtends an angle θ at the vertex of y2 = 4ax, then tan θ =


SA

2 –2 2 –2
(A) 7 (B) 7 (C) 5 (D) 5
3 3 3 3

24. The common tangents to the circle x2 + y2 = 2 and the parabola y2 = 8x touch the circle at the points P,
Q and the parabola at the points R, S. Then the area of the quadrilateral PQRS is
[JEE (Advanced) 2014, Paper-2, (3, –1)/60]
(A) 3 (B) 6 (C) 9 (D) 15

Paragraph For Questions 25 and 26

Let a, r, s, t be nonzero real numbers. Let P(at2, 2at), Q, R (ar2, 2ar) and S(as2, 2as) be distinct points
on the parabola y2 = 4ax. Suppose that PQ is the focal chord and lines QR and PK are parallel, where
K is the point (2a, 0) [JEE (Advanced) 2014, Paper-2, (3, –1)/60]
25. The value of r is
1 t2 + 1 1 t2 – 1
(A) – (B) (C) (D)
t t t t

26. If st = 1, then the tangent at P and the normal at S to the parabola meet at a point whose ordinate is
(t 2 + 1)2 a(t 2 + 1)2 a(t 2 + 1)2 a(t 2 + 2)2
(A) (B) (C) 3
(D)
t3
3 3
2t 2t t

27. Let the curve C be the mirror image of the parabola y2 = 4x with respect to the line x + y + 4 = 0. If A
and B are the points of intersection of C with the line y= – 5, then the distance between A and B is
[JEE (Advanced) 2015, P-1 (4, 0) /88]

28. If the normals of the parabola y2 = 4x drawn at the end points of its latus rectum are tangents to the

RI
circle (x – 3)2 + (y + 2)2 = r2, then the value of r2 is [JEE (Advanced) 2015, P-1 (4, 0) /88]

29*. Let P and Q be distinct points on the parabola y2 = 2x such that a circle with PQ as diameter passes
through the vertex O of the parabola, if P lies in the first quadrant and the area of the triangle ∆OPQ is

A
3 2 , then which of the following is (are) the coordinates of P ?
[JEE (Advanced) 2015, P-1 (4, –2)/ 88]
1 1 
(A) (4, 2 2 ) (B) (9, 3 2 ) (C)  ,  (D) (1, 2 )

UH
4 2
x2 y2
30. Suppose that the foci of the ellipse + = 1 are (f1, 0) and (f2, 0) where f1 > 0 and f2 < 0. Let P1 and
9 5
P2 be two parabolas with a common vertex at (0, 0) and with foci at (f 1, 0) and (2f2, 0), respectively. Let
T1 be a tangent to P1 which passes through (2f2, 0) and T2 be a tangent to P2 which passes through
JA
 1 
(f1,0). If m1 is the slope of T1 and m2 is the slope of T2, then the value of  2 + m22  is.
 1
m 
[JEE (Advanced) 2015, P-2 (4, 0) / 80]

31*. Let E1 and E2 be two ellipses whose centers are at the origin. The major axes of E1 and E2 lie along the
x-axis and the y-axis, respectively. Let S be the circle x2 + (y – 1)2 = 2. The straight line x + y = 3
LP

2 2
touches the curves S, E1 and E2 at P,Q and R, respectively. Suppose that PQ = PR = . If e1 and e2
3
are the eccentricities of E1 and E2, respectively, then the correct expression(s) is (are)
[JEE (Advanced) 2015, P-2 (4, –2)/ 80]
A

43 7
(A) e12 + e22 = (B) e1e2 =
40 2 10
5
NK

3
(C) | e12 – e22 | = (D) e1e2 =
8 4

32*. Consider the hyperbola H : x2 – y2 = 1 and a circle S with center N(x2, 0). Suppose that H and S touch
each other at a point P(x1, y1) with x1 > 1 and y1 > 0. The common tangent to H and S at P intersects the
x-axis at point M. If (l, m) is the centroid of the triangle ∆PMN, then the correct expression(s) is(are)
SA

[JEE (Advanced) 2015, P-2 (4, –2)/ 80]


dl 1 dm x1
(A) =1– for x1 > 1 (B) = for x1 > 1
( )
2
dx1 3x1 dx1 3 x 2 − 1
1

dl 1 dm 1
(C) = 1+ for x1 > 1 (D) = for y1 > 0
dx1 3x12 dy1 3

33. Let P be the point on the parabola y2 = 4x which is at the shortest distance from the center S of the
circle x2 + y2 – 4x –16y +64 = 0. Let Q be the point on the circle dividing the line segment SP internally.
Then [JEE (Advanced) 2016, Paper-2, (3, –1)/60]
(A) SP = 2 5
(B) SQ : QP = ( )
5+ 1 :2
(C) the x-intercept of the normal to the parabola at P is 6
1
(D) the slope of the tangent to the circle at Q is
2

x2 y2
34. If 2x – y + 1 = 0 is a tangent to the hyperbola 2
– = 1, then which of the following CANNOT be
a 16
sides of a right angled triangle ? [JEE(Advanced) 2017, Paper-1,(4, –2)/61]
(A) a, 4, 1 (B) 2a, 4, 1 (C) a, 4, 2 (D) 2a, 8, 1

35. If a chord, which is not a tangent, of the parabola y 2 = 16x has the equation 2x + y = p, and midpoint
(h, k), then which of the following is(are) possible value(s) of p, h and k?
[JEE(Advanced) 2017, Paper-1,(4, –2)/61]
(A) p = –1, h = 1, k = –3 (B) p = 2, h = 3, k = –4

RI
(C) p = –2, h = 2, k = –4 (D) p = 5, h = 4, k = –3

Answer Q.36, Q.37 and Q.38 by appropriately matching the information given in the three
columns of the following table. [JEE(Advanced) 2017, Paper-1,(3, –1)/61]

A
Columns 1, 2 and 3 contain conics, equations of tangents to the conics and points of contact,
repectively.

UH
Column-1 Column-2 Column-3
 a 2a 
(I) x 2 + y 2 = a2 (i) my = m2 x + a (P)  m2 , m 
 
 −ma a 
(Q)  , 
(II) x2 + a2y2 = a2
JA
(ii) y = mx + a m2 + 1
 m +1 m +1
2 2

 −a2m 1 
(III) y2 = 4ax a2m2 − 1 (R)  2 2 , 
a2m2 + 1 
(iii) y = mx +
 a m +1
 −a2m −1 
(IV) x2 – a2y2 = a2 a2m2 + 1 (S)  2 2 , 
a m − 1 
(iv) y = mx +
 a m −1
LP

2 2

36. For a = 2 , if a tangent is drawn to a suitable conic (Column 1) at the point of contact (–1, 1), then
which of the following options is the only CORRECT combination for obtaining its equation ?
A

[JEE(Advanced) 2017, Paper-1,(3, –1)/61]


(A) (I) (ii) (Q) (B) (I) (i) (P) (C) (III) (i) (P) (D) (II) (ii) (Q)
NK

 1
37. The tangent to a suitable conic (Column 1) at  3,  is found to be 3 x + 2y = 4, then which of the
 2
following options is the only CORRECT combination? [JEE(Advanced) 2017, Paper-1,(3, –1)/61]
(A) (IV) (iv) (S) (B) (II) (iv) (R) (C) (IV) (iii) (S) (D) (II) (iii) (R)
SA

38. If a tangent to a suitable conic (Column 1) is found to be y = x + 8 and its point of contact is (8, 16),
then which of the following options is the only CORRECT combination?
[JEE(Advanced) 2017, Paper-1,(3, –1)/61]
(A) (III ) (i) (P) (B) (I) (ii) (Q) (C) (II) (iv) (R) (D) (III) (ii) (Q)

1
39. Consider two straight lines, each of which is tangent to both the circle x 2 + y2 = and the parabola
2
y2 = 4x. Let these lines intersect at the point Q. Consider the ellipse whose center is at the origin O(0,
0) and whose semi-major axis is OQ. If the length of the minor axis of this ellipse is 2 , then which of
the following statement(s) is (are) TRUE ? [JEE(Advanced) 2018, Paper-2,(3, –1)/60]
1
(A) For the ellipse, the eccentricity is and the length of the latus rectum is 1
2
1 1
and the length of the latus rectum is
(B) For the ellipse, the eccentricity is
2 2
1 1
(C) The area of the region bounded by the ellipse between the lines x = and x = 1 is (π – 2)
2 4 2
1 1
(D) The area of the region bounded by the ellipse between the lines x = and x = 1 is (π – 2)
2 16
2 2
x y
40. Let H : 2
– 2 = 1, where a > b > 0, be a hyperbola in the xy-plane whose conjugate axis LM
a b
subtends an angle of 60° at one of its vertices N. Let the area of the triangle LMN be 4 3 .
[JEE(Advanced) 2018, Paper-2,(3, –1)/60]
LIST-I LIST-II

RI
(P) The length of the conjugate axis of H is (1) 8
4
(Q) The eccentricity of H is (2)
3
2

A
(R) The distance between the foci of H is (3)
3
(S) The length of the latus rectum of H is (4) 4

UH
The correct option is:
(A) P → 4; Q → 2; R → 1; S→3
(B) P → 4; Q → 3; R → 1; S→2
(C) P → 4; Q → 1; R → 3; S→2
(D) P → 3; Q → 4; R → 2; S→1
JA
PART - II : JEE (MAIN) / AIEEE PROBLEMS (PREVIOUS YEARS)
1. The ellipse x2 + 4y2 = 4 is inscribed in a rectangle alingent with the coordinate axes, which in turn is
inscribed in another ellipse that passes through the point (4, 0). Then the equation of the ellipse is :
[AIEEE 2009 (4, –1), 144]
(1) x2 + 12y2 = 16 (2) 4x2 + 48y2 = 48 (3) 4x2 + 64y2 = 48 (4) x2 + 16y2 = 16
LP

2*. Equation of the ellipse whose axes are the axes of coordinates and which passes through the point
2
(–3, 1) and has eccentricity is : [AIEEE 2011, I, (4, –1), 120]
5
(1) 3x2 + 5y2 – 32 = 0 (2) 5x2 + 3y2 – 48 = 0 (3) 3x2 + 5y2 – 15 = 0 (4) 5x2 + 3y2 – 32 = 0
A

3. The equation of the hyperbola whose foci are (–2, 0) and (2, 0) and eccentricity is 2 is given by :
[AIEEE 2011, II, (4, –1), 120]
NK

(1) x2 – 3y2 = 3 (2) 3x2 – y2 = 3 (3) – x2 + 3y2 = 3 (4) – 3x2 + y2 = 3

4. Statement-1 : An equation of a common tangent to the parabola y 2 = 16 3 x and the ellipse
2x2 + y2 = 4 is y = 2x + 2 3 . [AIEEE - 2013, (4, – 1) 120 ]
4 3
, (m ≠ 0) is a common tangent to the parabola y2 = 16 3 x and
SA

Statement-2 : If the line y = mx +


m
the ellipse 2x2 + y2 = 4, then m satisfies m4 + 2m2 = 24.
(1) Statement-1 is false, Statement-2 is true.
(2) Statement-1 is true, statement-2 is true; statement-2 is a correct explanation for Statement-1.
(3) Statement-1 is true, statement-2 is true; statement-2 is not a correct explanation for Statement-1.
(4) Statement-1 is true, statement-2 is false.

5. An ellipse is drawn by taking a diameter of the circle (x – 1) 2 + y2 = 1 as its semi-minor axis and a
diameter of the circle x2 + (y – 2)2 = 4 is semi-major axis. If the centre of the ellipse is at the origin and
its axes are the coordinate axes, then the equation of the ellipse is : [AIEEE-2012, (4, –1)/120]
(1) 4x2 + y2 = 4 (2) x2 + 4y2 = 8 (3) 4x2 + y2 = 8 (4) x2 + 4y2 = 16
x2 y2
6. The equation of the circle passing through the foci of the ellipse + = 1, and having centre at
16 9
(0, 3) is [AIEEE - 2013, (4, – 1) ]
(1) x2 + y2 – 6y – 7 = 0 (2) x2 + y2 – 6y + 7 = 0
(3) x2 + y2 – 6y – 5 = 0 (4) x2 + y2 – 6y + 5 = 0
7. The locus of the foot of perpendicular drawn from the centre of the ellipse x 2 + 3y2 = 6 on any tangent to
it is: [JEE(Main) 2014, (4, – 1), 120]
(1) (x2 + y2)2 = 6x2 + 2y2 (2) (x2 + y2)2 = 6x2 – 2y2 (3) (x2 – y2)2 = 6x2 + 2y2 (4) (x2 – y2)2 = 6x2 – 2y2

8. The slope of the line touching both the parabolas y2 = 4x and x2 = – 32y is :
[JEE(Main) 2014, (4, – 1), 120]
1 2 1 3
(1) (2) (3) (4)
8 3 2 2

RI
9. The area (in sq.units) of the quadrilateral formed by the tangents at the end points of the latera recta to
x2 y2
the ellipse , + = 1 is [JEE(Main) 2015, (4, – 1), 120]
9 5

A
27 27
(1) (2) 18 (3) (4) 27
4 2

UH
10. Let O be the vertex and Q be any point on the parabola, x 2 = 8y. If the point P divides the line segment
OQ internally in the ratio 1 : 3, then the locus of P is [JEE(Main) 2015, (4, – 1), 120]
(1) x2 = y (2) y2 = x (3) y2 = 2x (4) x2 = 2y

11. Let P be the point on the parabola, y2 = 8x which is at a minimum distance from the centre C of the
circle, x2 + (y + 6)2 = 1. Then the equation of the circle, passing through C and having its centre at P is :
JA
[JEE(Main) 2016, (4, – 1), 120]
x
(1) x2 + y2 – x + 4y –12 = 0 (2) x2 + y2 – + 2y –24 = 0
4
(3) x2 + y2 –4 x + 9y + 18 = 0 (4) x2 + y2 – 4x + 8y +12 = 0

12. The eccentricity of the hyperbola whose length of the latus rectum is equal to 8 and the length of its
LP

conjugate axis is equal to half of the distance between its foci, is : [JEE(Main) 2016, (4, – 1), 120]
4 2 4
(1) (2) (3) 3 (4)
3 3 3

1
A

13. The eccentricity of an ellipse whose centre is at the origin is . If one of its directrices is x = – 4, then
2
 3
the equation of the normal to it at 1,  is [JEE(Main) 2017, (4, – 1), 120]
 2
NK

(1) 2y – x = 2 (2) 4x – 2y = 1 (3) 4x + 2y = 7 (4) x + 2y = 4

14. A hyperbola passes through the point P( 2 , 3 ) and has foci at (±2, 0). Then the tangent to this
hyperbola at P also passes through the point : [JEE(Main) 2017, (4, – 1), 120]
SA

(1) (3 2 , 2 3 ) (2) (2 2 , 3 3 ) (3) ( 3 , 2) (4) (– 2 , – 3 )

15. If the tangent at (1, 7) to the curve x2 = y – 6 touches the circle x2 + y2 + 16x + 12y + c = 0 then the
value of c is : [JEE(Main) 2018, (4, – 1), 120]
(1) 85 (2) 95 (3) 195 (4) 185

16. Tangents are drawn to the hyperbola 4x2 – y2 = 36 at the points P and Q. If these tangents intersect at
the point T(0, 3) then the area (in sq. units) of ∆PTQ is : [JEE(Main) 2018, (4, – 1), 120]
(1) 60 3 (2) 36 5 (3) 45 5 (4) 54 3

17. Tangent and normal are drawn at P(16,16) on the parabola y 2 = 16x, which intersect the axis of the
parabola at A and B, respectively. If C is the centre of the circle through the points P, A and B and
∠CPB = θ , then a value of tan θ is : [JEE(Main) 2018, (4, – 1), 120]
4 1
(1) 3 (2) (3) (4) 2
3 2
18. Equation of a common tangent to the parabola y 2 = 4x and the hyperbola xy = 2 is :
[JEE(Main) 2019, Online (11-01-19),P-1 (4, – 1), 120]
(1) x + 2y + 4 = 0 (2) x – 2y + 4 = 0 (3) 4x + 2y + 1 = 0 (4) x + y + 1 = 0

19. If the parabolas y2 = 4b(x – c) and y2 = 8ax have a common normal, then which one of the following is a
valid choice for the ordered triad (a, b, c) ?
[JEE(Main) 2019, Online (10-01-19),P-1 (4, – 1), 120]
1  1 
(1) (1, 1, 3) (2)  ,2,3  (3)  ,2,0  (4) (1, 1, 0)
2  2 

20. The length of the chord of the parabola x2 = 4y having equation x – 2 y + 4 2 = 0 is :

RI
[JEE(Main) 2019, Online (10-01-19),P-2 (4, – 1), 120]
(1) 6 3 (2) 8 2 (3) 3 2 (4) 2 11

 y2 x2 

A
21. Let S = (x,y) ∈ R2 : − = 1 , where r ≠ ± 1. Then S represents :
 1+ r 1− r 
2
(1) a hyperbola whose eccentricity , when 0 < r < 1.

UH
1− r
2
(2) an ellipse whose eccentricity is , when r > 1.
r +1
2
(3) a hyperbola whose eccentricity is , when 0 < r < 1.
JAr +1
1
(4) an ellipse whose eccentricity is , when r > 1.
r +1
[JEE(Main) 2019, Online (10-01-19),P-2 (4, – 1), 120]

22. If tangents are drawn to the ellipse x2+ 2y2 = 2 at all points on the ellipse other than its four vertices
then the mid points of the tangents intercepted between the coordinate axes lie on the curve :
LP

[JEE(Main) 2019, Online (11-01-19),P-1 (4, – 1), 120]


2 2
x y 1 1 1 1 x2 y2
(1) + =1 (2) 2
+ 2
=1 (3) 2
+ 2
=1 (4) + =1
4 2 2x 4y 4x 2y 2 4

Let S and S' be the foci of an ellipse and B be any one of the extremities of its minor axis. If ∆S'BS is a
A

23.
right angled triangle with right angle at B and are (∆S'BS) = 8 sq. units, then the length of a latus
[JEE(Main) 2019, Online (12-01-19),P-2 (4, – 1), 120]
NK

(1) 2 (2) 4 2 (3) 4 (4) 2 2


SA
EXERCISE # 1
PART-I
Section (A) :
A-1. 4

 3 11   3 15 
A-2. (i) vertex ≡  , −  , focus  , −
 2 8  2 8 
3 7

RI
axis x = , directrix y = – , length of latus rectum = 2.
2 8
(ii) x2 – 2xy + y2 – 6x – 6y + 3 = 0
(iii) 4x2 – 4xy + y2 + 8x + 46y – 71 =0
(iv) y = 5/4

A
A-3. (y + 1)2 = 3(2x + 1) & (y + 1)2 = –3(2x – 5)

UH
2  61 2   485 2  −613 485
A-4. y= ,  − 16 , 3  ,  − 144 , 3  , x = 144 ,x = − 144 A-5. x 2 + y 2 + |x – 2| = 4
3    

A-6. α∈ [0, 1] A-7. α ∈ [π/2, 5π/6] ∪ [π, 3π/2]


JA
Section (B) :

 1  12 16 
B-1. e = 2  B-2 ((2, 3) & (6, 7)) B-3  5, 5 
   

4 4
B-4. (x = 3 + 5cosθ, y = – 2 + 4sinθ) B-5. <α< B-6. 3x2 + 4y2 – 12x + 24y + 36 = 0
LP

5 17

B-7. 5x2 + 9y2 – 54y + 36 = 0


5 14 4
B-8. (i) Centre (–1, 2), e = , foci = (4, 2), (–6, 2), x = – and x =
A

3 5 5
(ii) 7x2 – 2y2 + 12xy – 2x + 14y – 22 = 0 B-10. 3x2 – y2 – 12 = 0
NK

(x + 2)2 (y − 3)2
B-11. (i) (4, 1), (–6, 1) (ii) − =1
16 48
2
5± 5 
B-13. (x – 3) + (y – 4) = 
 2 
B-12. Inside 2 2

 
SA

13 7
B-14. B-19. B-20. 16
3 4
Section (C) :

C-1. 8 2 C-2. (4, 0), (–4, 0) C-3. y2 – 2ax + 8a2 = 0 C-4. y = x2 + 2


c
C-6. (– ∞, – 4] ∪ [4, ∞) C-7. x – 4y + 28 = 0 at (28, 14) C-8.
a
Section (D) :
x2 y2 y
D-1. 35 D-2. 2
+ 2
= D-3. (yes, (5, 4))
a b b

D-4. x2 + 64 y2 = 80 & x2 + 4 y2 = 20 D-5. ex ± y = a, – ex ± y = a

D-7. (3, 0) D-8. 16x2 + 9y2 + 96x = 432 or 16x2 + 9y2 – 288x + 720 = 0

8
D-9. 10 D-10. λ=±6 D-11. n=±2 D-12. x+y±3 3 =0
3

D-13. x2 – y2 – 4x = 0 D-14. 10 D-15. 30

RI
Section (E) :

E-1. 4y = 9x + 4, 4y = x + 36 E-2. (i) y = (x – a) tan α, (ii) x = a, (iii) y = λx

A
E-3. (–2, 0) E-4. –2 E-5. y2 = 2x – 2

Section (F):

UH
 25 16 
F-1. 4x + 5y = 40, 4x – 35y = 200. F-2.  4 , 3  F-3. 48x + 25y – 169 = 0
 
JA
5 3
F-4. (i) –16/9 (ii) –20/9 F-5. y= x + ; x – 3 = 0 ; 8 sq. unit
12 4

π
F-6. (x2 + y2)2 = 16 x2 - 9 y2 F-8. a2p2 + b2q2 = r2sec2 = (4 – 2 2 ) r2
8
LP

Section (G) :
G-1. x + y = 3 G-3. x + 3y = 33 G-5. (4a, 4a) G-6. x + 2y – 3 = 0

Section (H) :
A

H-2. 12 x + 5 y = 48; 12 x − 5 y = 48
NK

Section (I) :

I-1. (x – 2)2 + (y – 4)2 + 2(x – y + 2) = 0 I-2. 2 I-3. b = 3 ; x + 2y + 4 = 0

2 2 64
I-4. I-5. y=x± 10 , y = – x ± 10 I-6.
SA

3 75
PART - II
Section (A) :
A-1. (D) A-2. (C) A-3. (D) A-4. (A) A-5. (B) A-6. (B) A-7. (D)

A-8. (B) A-9. (A) A-10. (D) A-11. (D) A-12. (C) A-13. (B)
Section (B) :
B-1. (A) B-2. (B) B-3. (D) B-4. (C) B-5. (A) B-6. (C) B-7. (D)
B-8. (B) B-9. (C) B-10. (A) B-11. (B) B-12. (C) B-13. (A) B-14. (D)

B-15. (C) B-16. (C) B-17. (A) B-18. (A) B-19. (D) B-20. (C) B-21. (B)
B-22. (C)

Section (C) :
C-1. (D) C-2. (A) C-3. (B) C-4. (B) C-5. (D) C-6. (C) C-7. (B)
C-8. (A)

RI
Section (D) :
D-1. (B) D-2. (C) D-3. (B) D-4. (C) D-5. (A) D-6. (A) D-7. (B)
D-8. (B)

A
Section (E) :
E-1. (B) E-2. (C) E-3. (B)

UH
Section (F) :
F-1. (A) F-2. (B) F-3 (D) F-4. (A) F-5. (B) F-6. (D) F-7. (A)
F-8. (D) F-9. (C)
JA
Section (G) :
G-1. (B) G-2. (A) G-3. (D) G-4. (C) G-5. (B) G-6. (D) G-7. (D)
G-8. (A)

Section (H) :
LP

H-1. (D) H-2. (A) H-3. (D) H-4.(A) H-5. (A)

Section (I) :
I-1. (A) I-2. (B) I-3. (C) I-4. (A) I-5. (D) I-6. (C) I-7. (A)
A

I-8. (A)

PART - III
NK

1. (A) → (q), (B) → (r), (C) → (s), (D) → (q)

2. (A) → (s), (B) → (r), (C) → (q), (D) → (p)

3. (A) → (r), (B) → (p), (C) → (s), (D) → (q)


SA

4. (A) → (q), (B) → (s), (C) → (s), (D) → (r)

EXERCISE # 2
PART-I
1. (B) 2. (C) 3. (C) 4. (C) 5. (B) 6. (C) 7. (B)
8. (C) 9. (B) 10. (B) 11. (B) 12. (C) 13. (D) 14. (C)
15. (A) 16. (D) 17. (A) 18. (A) 19. (D) 20. (D) 21. (C)
22. (C) 23. (A) 24. (B) 25. (A) 26. (D) 27. (A) 28. (A)
29. (A) 30. (C)
PART-II
1. 1 2. 3 3. 0 4. 2 5. 18 6. 16 7. 23
8. 3 9. 4 10. 3 11. 2 12. 2 13. 3 14. 13
15. 65 16. 21 17. 15 18. 9 19. 14 20. 55 21. 7
22. 9 23. 13 24. 1 25. 77 26. 57 27. 22 28. 4
29. 0025 30. 6 31. 16 32. 6 33. 6 34. 50 35. 3

PART-III
1. (AB) 2. (ABCD) 3. (AD) 4. (AD) 5. (AD) 6. (AC) 7. (BC)

RI
8. (ABCD) 9. (ABC) 10. (AC) 11. (AC) 12. (AB) 13. (AD) 14. (AD)
15. (AB) 16. (CD) 17. (BCD) 18. (ACD) 19. (AB) 20. (BD)
21. (ABCD) 22. (ACD) 23. (ABC) 24. (AD) 25. (ABD) 26. (AB)

A
PART - IV
1. (A) 2. (B) 3. (C) 4. (B) 5. (A) 6. (C) 7. (C)

UH
8. (B) 9. (A) 10. (B) 11. (A) 12. (D) 13. (C) 14. (A)

15. (A) 16. (A)

EXERCISE # 3
JA
PART-I
1*. (AD) 2. (D) 3*. (BC) 4. (C)

5. (A) → (p), (B) → (s, t), (C) → (r), (D) → (q, s) 6*. (AB) 7*. (CD)
8. (D) 9. (C) 10. (A) 11. (B) 12. (A) 13. 2 14. (2)
LP

15. (C) 16*. (ABD) 17*. (BD) 18. (B) 19. (C) 20. (AB) 21. (4)
22. (B) 23. (D) 24. (D) 25. (D) 26. (B) 27. 4 28. 2
29*. (AD) 30. 4 31*. (AB) 32*. (ABD) 33. (ACD) 34. (ACD) 35. (B)
A

36. (A) 37. (B) 38. (A) 39. (AC) 40. (B)

PART - II
NK

1. (1) 2*. (1,2) 3. (2) 4. (2) 5. (4) 6. (1) 7. (1)

8. (3) 9. (4) 10. (4) 11. (4) 12. (2) 13. (2) 14. (2)

15. (2) 16. (3) 17. (4) 18. (1) 19. (1) 20. (1) 21. (2)
SA

22. (2) 23. (3)


SUBJECTIVE QUESTIONS
1. Prove that in a parabola the angle θ that the latus rectum subtends at the vertex of the parabola
2π 3π
isindependent of the latus rectum and lies between &
3 4

2. A parabola is drawn to pass through A and B, the ends of a diameter of a given circle of radius a, and
to have as directrix a tangent to a concentric circle of radius b; then axes being AB and a perpendicular
x2 y2
diameter, prove that the locus of the focus of the parabola is 2 + 2 =1

RI
b b − a2

3. Find the points of intersection of the curves whose parametric equations are x = t 2 + 1, y = 2t and x =
2s, y = 2/s.

A
4. If r1, r2 be the length of the perpendicular chords of the parabola y 2 = 4ax drawn through the vertex, then
show that (r1r2)4/3 = 16a2(r12/3 + r22/3).

UH
5. Prove that the circle circumscribing the triangle formed by any three tangents to a parabola passes
through the focus.

6. A chord is a normal to a parabola and is inclined at an angle θ to the axis; prove that the area of the
triangle formed by it and the tangents at its extremities is 4a2 sec3 θ cosec3 θ
JA
7. From an external point P, pair of tangent lines are drawn to the parabola, y 2 = 4x. If θ1 & θ2 are the
π
inclinations of these tangents with the axis of x such that, θ1 + θ2 = , then find the locus of P .
4

8. TP and TQ are tangents to the parabola and the normals at P and Q meet at a point R on the curve ;
LP

prove that the centre of the circle circumscribing the triangle TPQ lies on the parabola 2y 2 = a(x – a).

9. From an external point P, tangents are drawn to the parabola; find the equation of the locus of P when
these tangents make angles θ1 and θ2 with the axis, such that cos θ1
cos θ2 = µ, which is constant.
A
NK

11. Prove that the normals at the points, where the straight line x + my = 1 meets the parabola y 2 = 4ax,
 4am2 4am 
meet on the normal at the point  2 ,  on the parabola.
   

SA

12. Prove that the equation to the circle, which passes through the focus and touches the parabola y 2 =4ax
at the point (at2, 2at), is x2 + y2 – ax(3t2 + 1) – ay (3t – t3) + 3a2t2 = 0.
Prove also that the locus of its centre is the curve 27ay 2 = (2x – a) (x – 5a)2.

13. Two tangents to the parabola y² = 8x meet the tangent at its vertex in the points P & Q. If
PQ = 4 units, prove that the locus of the point of the intersection of the two tangents is
y² = 8 (x + 2).

14. Find locus of a point P if the three normals drawn from it to the parabola y 2 = 4ax are such that two of
them make complementry angles with the axis of the parabola

15. Prove that the orthocentre of any triangle formed by three tangents to a parabola lies on the directrix.
16. If tangent drawn at a point (t², 2t) on the parabola y² = 4x is same as the normal drawn at a point ( 5
cos φ, 2 sin φ) on the ellipse 4x² + 5y² = 20. Find the values of t & φ.

17. Find the locus of centre of a family of circles passing through the vertex of the parabola y 2 = 4ax, and
cutting the parabola orthogonally at the other point of intersection.

18. Let A, B, C be three points on the parabola y 2 = 4ax. If the orthocentre of the triangle ABC is at the
focus then show that the circumcircle of ∆ABC touches the y-axis.

19. If α, β are eccentric angles of the extremities of a focal chord of an ellipse, then eccentricity of the
ellipse is

x2 y2
20. If circumcentre of an equilateral triangle inscribed in +
= 1, with vertices having eccentric

RI
2
a b2
angles α, β, γ respectively is (x1, y1), then find Σ cosα cosβ + Σ sin α sin β.

21. Find the locus of extremities of latus rectum of the family of ellipse b 2x2 + y2 = a2b2 where b is a

A
parameter (b2 < 1).

22. A point moves such that the sum of the square of the distances from two fixed straight lines intersecting

UH
at angle 2α is a constant. Prove that the locus is an ellipse of eccentricity
cos 2α π − cos 2α π
if α < and if α >
cos α 4 sin α 4

x2 y2
23. A straight line PQ touches the ellipse = 1 and the circle x2 + y2 = r2(b < r < a). RS is a focal
+
JA
2
a b2
chord of the ellipse. If RS is parallel to PQ and meets the circle at points R and S. Find the length of
RS.

24. Prove that the sum of the eccentric angles of the extremities of a chord of an ellipse, which is drawn in
a given direction is constant and is equal to twice the eccentric angle of the point at which the tangent is
parallel to the given direction.
LP

25. If the normals at α, β, γ , δ on an ellipse are concurrent, prove that (∑cosα)(∑sec α) = 4


(∑cosα)(∑sec α) = 4

x2 y2
A

26. Show that the equation of the pair of tangents to the ellipse + = 1 at the points of intersection
a2 b2
 x2 y2 
with the line, p x + q y + 1 = 0 is  2 + − 1 , (p2 a2 + q2 b2 − 1) = (p x + q y + 1)2.
NK

a 2 
 b 

x2 y2 x2 y2
27. A tangent to the ellipse + = 1meets the ellipse
+ = a+b
a2 b2 a b
at the points P and Q; prove that the tangents at P and Q are at right angles.
SA

28. Find the locus of the point, the chord of contact of the tangents drawn from which to the ellipse
x2 y2
+ = 1 touches the circle x2 + y2 = c2, where c < b < a.
a2 b2
x2 y2
29. A chord of ellipse + = 1 whose eccentric angles of extremities are α and β, intersects its director
a2 b2
circle at point A and B. Tangents at A and B intersect at point P. Find the equation of circumcircle of
triangle ABP.
x2 y2
30. A tangent is drawn at any fixed point P on the ellipse + = 1 and if chord of contact of the ellipse
16 9
x2 y2
+ = 1 with respect to any point on this tangent passes through a fixed point, then prove that the
9 16
line joining this fixed point to the point P never subtends right angle at the origin.

31. If the parabola y2 = 4ax cuts the ellipse


(x − a)2 +
y2
= 1 in three distinct points then show that the
a2 b2
 1 
eccentricity of the ellipse e belongs to  , 1 .
 2 

RI
32. Find the number of integral points lying on or inside the ellipse 2x 2 + 6xy + 6y2 –1 = 0.

33. The equations of the transverse and conjugate axes of a hyperbola are respectively x + 2y – 3 = 0,
2x – y + 4 = 0, and their respective lengths are 2 and 2/ 3 . Then find the equation of the hyperbola .

A
x2 y2
34. If P is any point common to the hyperbola − = 1 and the circle having line segment joining its
16 25

UH
focii as diameter then find the sum of focal distances of point P.

35. The transverse axis of a hyperbola is of length 2a and a vertex divides the segment of the axis between
the centre and the corresponding focus in the ratio 2 : 1. Find the equation of the hyperbola.

x2 y2
JA
36. If x cos α + y sin α = p, a variable chord of the hyperbola –
= 1 subtends a right angle at the
a2 2a2
centre of the hyperbola, then the chords touch a fixed circle, find the radius of the circle.

37. If the distance between the centres of the hyperbolas :


x2 – 16xy – 11y2 – 12x + 6y + 21 = 0 ..... (i)
9x2 – 16y2 – 18x – 32y – 151 = 0 ..... (ii) is d then 125 d2 = .................
LP

38. Find an equation of the hyperbola whose directrix is the normal to circle x 2 + y2 – 4x – 6y + 9 = 0 having
slope is 2 and eccentricity is equal to radius of given circle where focus of hyperbola is point of contact
of given circle with y-axis.
A

x2 y2
39. PQ is the chord joining the points whose eccentric angles are φ1 and φ2on the hyperbola − = 1, If
a2 b2
x2 y2
NK

φ1 – φ2 = 2α, where α is constant, prove that PQ touches the hyperbola cos 2α – = 1.


a2 b2

x2 y2
40. Find the locus of the mid-points of the chord of the hyperbola − = 1 which subtend a right angle
a2 b2
SA

at the origin is

41. If a chord joining the points P (a sec θ, a tan θ) & Q (a sec φ, a tan φ) on the hyperbola x 2 − y2 = a2 is a
normal to it at P, then show that tan φ = tan θ (4 sec 2 θ − 1) .
42. Chords of the hyperbola x2/a2 − y2/b2 = 1 are tangents to the circle drawn on the line joining the foci as
diameter . Find the locus of the point of intersection of tangents at the extremities of the chords .

x2 y2 x2 y2
43. From any point on the hyperbola H1 : − = 1 tangents are drawn to the hyperbola H2 : − = 2.
a2 b2 a2 b2
Then find the area cut-off by the chord of contact on the asymptotes of H 2 .

44. The chord PQ of the rectangular hyperbola xy = a2 meets the x-axis at A; C is the mid point of PQ & 'O'
is the origin. Then prove that the ∆ ACO is isosceles.
45. If the normals at (xi, yi), i = 1, 2, 3, 4 on the rectangular hyperbola, xy = c 2, meet at the point (α, β) show
that
(i) Σxi = α (ii) Σyi = β (iii) Πxi = Πyi = –c4,
(iv) Σxi2 = α2 (v) Σyi2 = β2

2 y 2
46. If α, β, γ & δ be the eccentric angles of feet of four co−normal points of a hyperbola x2 − 2 = 1 from
a b
any point in its plane then prove that α + β + γ + δ is odd integral multiple of π.

x2 y2
47. Prove that a normal to the hyperbola − = 1 cannot be normal to its conjugate hyperbola.
a2 b2

RI
48. Let P be a point from where perpendicular tangents are drawn to the circle 2x 2 + 2y2 – a2 = 0. Let a line
from P perpendicular to OP is drawn which intersect hyperbola x 2 – y2 = a2 at Q and R. Find number of
all possible positions of P such that product of ordinates of points Q and R is.

A
3 a2
(i) a 2 (ii) a2 (iii)
2 2

UH
3. (2, 2) 7. x−y−1=0 9. x2 = µ2 {(x – a)2 + y2} 14. y2 = a (x − a)
JA
1 1 π
16. φ = π – tan–1 2, t = − ; φ = π + tan−12, t = ; φ =± ,t=0
5 5 2

2 2
sin α + sin β 9x1 9y1 3
17. 2y2(2y2 + x2 – 12ax) = ax(3x – 4a)2 19. 20. + –
sin(α + β) 2a 2
2b 2 2
LP

x2 y2 1
21. x2 ± ay = a2 23. RS = 2b 28. 4
+ 4
=
a b c2

2 3
33. (2x – y + 4)2 – (x + 2y – 3)2 = 1 34. 2 66 35. 5x2 – 4y2 = 5a2
A

5 5

36. 2a 37. 0025 38. 11x2 – y2 – 16xy –16x + 38y – 41 = 0


NK

x2 y2 1
42. 4
+ 4
= 43. 4 ab
a b a + b2
2

48. (i) 4 (ii) 2 (iii) 0


SA
MATHEMATICAL INDUCTION

1 INTRODUCTION

The word induction means the method of reasoning about a general statement from the conclusion
of particular cases. Inductions starts with observations. It may be true but then it must be so proved by the
process of reasoning. Else it may be false but then it must be shown by finding a counter example where the
conjecture fails.
In mathematics there are some results or statements that are formulated in terms of n, where
n ∈ N . To prove such statements we use a well suited method, based on the specific technique, which in known

RI
as principle of mathematical induction.
2 PROPOSITION

A
A statement which is either true or false is called a proposition or statement.
P (n ) denotes a proposition whose truth value depends on natural variable ‘n’.
n (n + 1)(2n + 1)

UH
For example 12 + 2 2 + 3 2 + ......n 2 = is a proposition whose truth value depends
6
on natural number n.
n (n + 1)(2n + 1)
We write P (n ) : 12 + 2 2 + 3 2 + .......n 2 = ,
6
JA
4(4 + 1) (8 + 1)
where P(4) means 12 + 22 + 32 + 42 =
6
To prove the truth of proposition P (n ) depending on natural variable n, we use mathematical
induction.
3 FIRST PRINCIPLE OF MATHEMATICAL INDUCTION
LP

The statement P (n ) is true for all n ∈ N if


(i) P(1) is true.
(ii) P(m) is true ⇒ P (m + 1) is true.
The above statement can be generalized as P (n ) is true for all n ∈ N and n ≥ k if
A

(i) P (k ) is true.
(ii) P (m ) is true (m > k ) ⇒ P (m + 1) is true.
NK

4 APPLICATION OF FIRST PRINCIPLE OF MATHEMATICAL INDUCTION

WORKING RULE
To prove any statement P (n ) to be true for all n ≥ k with the help of first principle of mathematical
induction we follow the following procedure:
SA

Step (i) Check if the statement is true or false for n = k.


Step (ii) Assume the statement is true for n = m.
Step (iii) Prove the statement is true for n = m + 1.

Illustration 1
Question: Prove by the principle of mathematical induction that for all n ∈ N :
1 1 1 1 n
+ + + ...... + = .
1 .2 2 .3 3 .4 n(n + 1) n + 1
Solution: Let P (n ) be the statement given by
1 1 1 1 n
P (n ) : + + + ...... + =
1.2 2.3 3.4 n (n + 1) n + 1
1 1
Step I: We have, P (1) : =
1 .2 1 + 1
1 1 1
Since, = =
1 .2 1 + 1 2
So, P(1) is true.
1 1 1 1 m
Step II: Let P (m ) be true, then + + + ...... + = (i)
1 .2 2 .3 3 .4 m(m + 1) m + 1
We shall now show that P (m + 1) is true. If P(m) is true.
For this we have to show that
1 1 1 1 1 m +1
+ + + ...... + + =
1.2 2.3 3.4 m(m + 1) (m + 1)(m + 1 + 1) (m + 1) + 1
1 1 1 1 1
Now, + + + ...... +
1 .2 2 .3 3 .4 m(m + 1) (m + 1) (m + 1 + 1)
 1 1 1 1  1

RI
=  + + + ...... +
 1 . 2 2 . 3 3 . 4 m (m + 1)  (m + 1) (m + 1 + 1)
m 1
= + [using (i)]
m + 1 (m + 1) ((m + 1) + 1)

A
=
1 m
 +
1 
=
1 (
m 2 + 2m + 1
=
)
(m + 1) = m + 1 2

(m + 1)  1 m + 2  (m + 1) (m + 2) (m + 1)(m + 2) m + 2
∴ P (m + 1) is true.

UH
Thus, P (m ) is true ⇒ P (m + 1) is true.
Hence, by the principle of mathematical induction, the given statement is true for all n ∈ N .
Illustration 2
JA
Question: For every positive integer n, prove that 7 n − 3 n is divisible by 4.

Solution: We have P (n ) : 7n − 3n is divisible by 4

We note that P (1) : 71 − 31 = 4 , which is divisible by 4. Thus P (n ) is true for n = 1

Let P (k ) be true for some natural number k.


LP

i.e., P (k ) : 7 k − 3 k is divisible by 4.

We get 7 k − 3 k = 4d, where d ∈ N …(i)


A

Now, we wish to prove that P (k + 1) is true whenever P (k ) is true.

i.e., we have show 7k +1 − 3k +1 = 4m


NK

Now, 7 (k +1) − 3 (k +1) = 7 (k +1) − 7 . 3 k + 7 3 k − 3 (k +1)

( )
= 7 7 k − 3 k + (7 − 3 )3 k = 7(4d ) + (7 − 3 )3 k

= 7(4d ) + 4 . 3k [using (i)]


SA

( )
= 4 7d + 3 k = 4m

From the last line, we see that 7 (k +1) − 3 (k +1) is divisible by 4.

Thus, P (k + 1) is true when P (k ) is true.

Therefore, by principle of mathematical induction the statement is true for every positive integer n.
Illustration 3
Question: Prove that 2.7 n + 3.5 n − 5 is divisible by 24 for all n ∈ N .
Solution: Let the statement P (n ) be defined as
P (n ) : 2.7 n + 3.5 n − 5 is divisible by 24.
Now, P (n ) is true for n = 1, since 2.7 + 3.5 − 5 = 24 , which is divisible by 24.
Assume that P (k ) is true. i.e. 2.7 k + 3.5 k − 5 = 24q , when q ∈ N …(i)
Now, we wish to prove that P (k + 1) is true whenever P (k ) is true.
We have, 2.7 k +1 + 3.5 k +1 − 5 = 2.7 k .7 1 + 3.5 k .5 1 − 5
= 7[2.7 k + 3.5 k − 5 − 3.5 k + 5] + 3.5 k .5 − 5
= 7[24q − 3.5 k + 5] + 15.5 k − 5 [using (i)]
= 7 × 24q − 21.5 k + 35 + 15.5 k − 5
= 7 × 24q − 6.5 k + 30
(
= 7 × 24q − 30 5k −1 − 1 = 7 × 24q − 30 (4 p ) )
[since 5 ( k −1
)
− 1 is a multiple of 4 as x n − y n is divisible by x = y]
= 7 × 24q − 120 p = 24(7q − 5 p )
= 24 × r , r = 7q − p , is some natural number …(ii)

RI
The expression on the R.H.S. of (i) is divisible by 24.
Thus P (k + 1) is true whenever P (k ) is true.
Hence, by principle of mathematical induction, P (n ) is true for all n ∈ N

A
Illustration 4
n
Question: Prove the rule of exponents (ab ) = a n b n by using principle of mathematical induction for every

UH
natural number.
n
Solution: Let P (n ) be the given statement i.e, P (n ) : (ab ) = a n b n
1
We note that P (n ) is true for n = 1 since (ab ) = a 1b 1
k
Let P (k ) be true, i.e., (ab ) = a k b k …(i)
JA
We shall now prove that P (k + 1) is true whenever P (k ) is true.
k +1 k
Now, we have (ab ) = (ab ) (ab )
(
= ak bk ) (ab ) [by (i)]
= (a k
. a )(b . b ) = a . b
1 k 1 k +1 k +1

Therefore, P (k + 1) is also true whenever P (k ) is true.


LP

Hence, by principle of mathematical induction, P (n ) is true for all n ∈ N .


Illustration 5
Question: Using mathematical induction, show that
A

sin 2n θ
cos θ cos 2θ cos 4θ...... cos 2n − 1θ = ( ) 2n sin θ
, ∀ n∈N .

sin 2n θ
Let P (n ) : cos θ cos 2θ cos 4θ...... cos 2n −1θ = ( )
NK

Solution:
2n sin θ
Step I: For n = 1
sin 2θ
L.H.S. of (i) = cos θ and R.H.S. of (i) = = cos θ
2 sin θ
Therefore, P(1) is true.
SA

Step II: Assume it is true for n = k, then


sin 2k θ
p(k ) : cos θ cos 2θ cos 4θ...... cos 2k −1θ = ( ) 2k sin θ
…(i)

Step III: For n = k+ 1


sin 2 k +1 θ
) ( )
P (k + 1) : cos θ cos 2θ cos 4θ...... cos 2 k −1 θ cos 2 k θ = ( 2 k +1 sin θ
L.H.S. = cos θ cos 2θ cos 4θ...... cos(2 θ) cos(2 θ) k −1 k

sin(2 θ) k
2 sin(2 θ) cos(2 θ) k k
= . cos(2 θ) = k
[using (i)]
2k sin θ 2k +1 sin θ

=
(
sin 2 . 2k θ ) = sin(2 θ) = R.H.S.k +1

2 sin θ
k +1
2 sin θ k +1

This shows that the P (k + 1) is true if P (k ) is true.


Hence by the principle of mathematical induction, the result is true for all n ∈ N .

Illustration 6
Question: Prove by induction that the sum Sn = n 3 + 3n 2 + 5n + 3 is divisible by 3 for all n ∈ N .
Solution: Let P (n ) be the statement given by
P (n ) : Sn = n 3 + 3n 2 + 5n + 3 is divisible by 3
2
Step I: We have, P (1) : S1 = 13 + 3(1) + 5(1) + 3 is divisible by 3
2
Since 13 + 3(1) + 5(1) + 3 = 12 , which is divisible by 3
∴ P(1) is true.
Step II: Let P (m ) be true. Then
Sm = m 3 + 3m 2 + 5m + 3 is divisible by 3

RI
⇒ Sm = m 3 + 3m 2 + 5m + 3 = 3λ, for some λ ∈ N …(i)
We now wish to show that P (m + 1) is true. For this we have to show that
(m + 1)3 + 3(m + 1)2 + 5(m + 1) + 3 is divisible by 3

A
Now, (m + 1) + 3(m + 1) + 5(m + 1) + 3 = (m 3 + 3m 2 + 5m + 3 ) + 3m 2 + 9m + 9
3 2

( )
= 3λ + 3 m 2 + 3m + 3 = 3[λ + m 2 + 3m + 3] [using (i)]

UH
= 3µ, where µ = λ + m + 3m + 3 ∈ N
2

∴ P (m + 1) is true.
Thus, P (m ) is true ⇒ P (m + 1) is true
Hence, by the principle of mathematical induction the statement is true for all n ∈ N .
JA
Illustration 7
Question: Show by using principle of mathematical induction that

1.3 + 2.3 2 + 3.3 3 + ...... + n . 3 n =


(2n − 1)3 n +1 + 3 .
4

Let P (n ) : 1.3 + 2.32 + 3.33 + ...... + n . 3n =


(2n − 1)3n +1 + 3
LP

Solution:
4
Step I: When n = 1, L.H.S. =1.3 = 3

and R.H.S. =
(2n − 1)3n +1 + 3 = (2.1 − 1)32 + 3 =
12
=3
4 4 4
A

Hence P(1) is true.


Let P (m ) be true

⇒ 1.3 + 2.32 + 3.33 + ...... + m.3m =


(2m − 1)3m +1 + 3 ….(i)
NK

4
To prove P (m + 1) is true i.e.,

1.3 + 2.32 + ...... + m.3m + (m + 1).3m +1 =


(2m + 1) 3m + 2 + 3
4
Adding (m + 1) . 3m +1 to both sides of (i), we get
SA

1.3 + 2.32 + ...... + m.3m + (m + 1) . 3m +1

=
(2m − 1) 3m +1 + 3 + (m + 1). 3m +1
4

=
{2m − 1 + 4(m + 1)}. 3m +1 + 3 = (2m + 1) 3m + 2 + 3
4 4
Hence P (m + 1) is true whenever P (m ) is true.
It follows that P (n ) is true for all natural numbers n.
Illustration 8
Question: Using the principle of mathematical induction, show that 11n + 2 + 12 2n + 1 , where n is a natural
number, is divisible by 133.
Solution: Let f (n ) = 11n + 2 + 122n +1
Let P (n ) : f (n ) i.e. 11n + 2 + 122n +1 is divisible by 133
Now, f (1) = 113 + 123 = (11 + 12)(121 − 11× 12 + 144 )
= 23 × 133 , where is divisible by 133
∴ P(1) is true. …(A)
Let P (m ) be true ⇒ f (m ) = 11m + 2 + 122m +1 is divisible by 133
⇒ f (m ) = 11m + 2 + 122m +1 = 133k , where k is an integer …(i)
Now, f (m + 1) = 11 m +3
+ 12 2m + 3

m+2
= 11.11m + 2 + 122m +1.122 = 11(11) + 122m +1.144
Now we divide f (m + 1) by f (m )
)
11m + 2 + 122m +1 11.(11)
m+2
+ 144.122m +1(11

RI
m+2
11.(11) + 11.122m +1
− −
133.122m +1

A
∴ f (m + 1) = 11.f (m ) + 133.122m +1 = 11× 133k + 133.122m +1 [from (i)]
( )
= 133 11k + 122m +1 , which is divisible by 133
∴ P (m + 1) is true whenever P (m ) is true

UH
…(B)
From (A) and (B) it follows that P (n ) is true for every natural number n i.e. 11n + 2 + 122n +1 is divisible by
133 for every natural number n.

Illustration 9
JA
n3
Question: Prove that 12 + 2 2 + ....... + n 2 > , n∈N.
3
Solution: Let P (n ) be the given statement.
n3
i.e., P (n ) : 12 + 2 2 + ...... + n 2 > , n∈N
3
LP

13
when note that P (n ) is true for n = 1 since 12 >
3
Assume that P (k ) is true
k3
A

i.e., P (k ) : 12 + 2 2 + ....... + k 2 > …(i)


3
We shall now prove that P(k + 1) is true whenever P(k) is true.
2
We have, 12 + 2 2 + 3 2 + ....... + k 2 + (k + 1)
NK

k3
( )
= 12 + 2 2 + ....... + k 2 + (k + 1)2 >
3
+ (k + 1)
2
[by (i)]

1 3
= [ k + 3k 2 + 6k + 3 ]
3
SA

1 3 1 3
= [(k + 1) + 3k + 2] > (k + 1)
3 3
Therefore, P (k + 1) is also true whenever P(k) is true.
Hence, by mathematical induction P(n) is true for all n ∈ N

Illustration 10
Question: Prove that 2 n > n for all positive integers n.
Solution: Let P (n ) : 2 n > n
When n = 1, 21 > 1
Hence P(1) is true.
Assume that P (k ) is true for any positive integers k i.e.,
2k > k …(i)
We shall now prove that P (k + 1) is true whenever P (k ) is true.
Multiplying both sides of (i) by 2, we get
2 . 2 k > 2k
i.e., 2 k +1 > 2k = k + k > k + 1
∴ P (k + 1) is true when P (k ) is true.
Hence, by principle of mathematical induction, P (n ) is true for every positive integer n.

RI
A
UH
JA
LP
A
NK
SA
EXERCISE
1. Prove by the principle of mathematical induction:
1 1 1 1 n
+ + + ...... + = for all n ∈ N .
1.4 4.7 7.10 (3n − 2)(3n + 1) (3n + 1)
2. Prove by the principle of mathematical induction:
1 1 1 1 n (n + 3 )
+ + + ...... + = for all n ∈ N .
1 .2 .3 2 .3 .4 3 .4 .5 n (n + 1)(n + 2) 4(n + 1)(n + 2)

3. Prove by the principle of mathematical induction that


n (2n − 1)(2n + 1)
12 + 3 2 + 5 2 + ...... + (2n − 1) =
2
for all n ∈ N .

RI
3
4. Prove by the principle of mathematical induction that 3 2n + 2 − 8n − 9 for all n ∈ N is divisible by
8.

A
5. Prove by the principle of mathematical induction that a + ar + ar 2 + ...... + ar n −1 =
(
a r n −1 )
for
r −1
all n ∈ N .

UH
1 1 1 1 1
6. Prove by the principle of mathematical induction that + + + ....... + n = 1 − n for all
2 4 8 2 2
n∈N .
7. Prove by the principle of mathematical induction that
JA
1 1 1 2n
1+ + + ...... + = for all n ∈ N .
(1 + 2) (1 + 2 + 3) (1 + 2 + 3 + ......n ) (n + 1)
8. Prove by the principle of mathematical induction that n (n + 1)(n + 5 ) for all n ∈ N is a multiple
of 3.
9. Prove by the principle of mathematical induction that (r + 1)(r + 2)(r + 3 )(r + 4 )(r + 5 ) in divisible
LP

by 120 ∀ n ∈ N .
10. Prove by the principle of mathematical induction that
1 1 1 1 n
+ + + ...... + = for all n ∈ N .
2.5 5.8 8.11 (3n − 1) (3n + 2) (6n + 4)
A

11. Prove by the principle of mathematical induction that 41n − 14 n for all n ∈ N is a multiple of 27.
NK

12. Prove by the principle of mathematical induction that (2n + 7 ) < (n + 3 )2 for all n ∈ N .
13. Prove by the principle of mathematical induction that 15 2n −1 + 1 for all n ∈ N is a multiple of 16.
14. Prove by the principle of mathematical induction that (1 + x )n > 1 + nx, n > 1 , n ∈ N and
x > −1, x ≠ 0 .
SA

1 2
15. Prove by the principle of mathematical induction that 1 + 2 + 3 + ...... + n < (2n + 1) for all
8
n∈N .
MATHEMATICAL LOGIC

1 MATHEMATICAL LOGIC

The dictionary meaning of the word ‘Logic’ is “the science of reasoning”. Logic is the study and
analysis of the nature of valid arguments. It is the reasoning tool by which philosophers and mathematicians
draw valid inferences from a given set of acts or premises. In fact, in any study, reasoning logic has a role to
play. In the process of reasoning we communicate our ideas or thoughts with the help of sentences in a
particular language. The following types of sentences are normally used in our every day communication:

RI
Assertive Sentence
A sentence that makes an assertion is called an assertive sentence or a declarative sentence.
For example. “Mars supports life” is an assertive or a declarative sentence. “Any two individuals are
always related” is also a declarative sentence.

A
Imperative Sentence
A sentence that expresses a request or a command is called an imperative sentence.
For example. “Please bring me a cup of tea” is an imperative sentence.

UH
Exclamatory Sentence
A sentence that expresses some strong feelling is called an exclamatory sentence.
For example, “How big is the whale fish !” is an exclamatory sentence.
Interrogative Sentence
A sentence that asks some question is called an interrogative sentence.
JA
For example, “What is your age?” is an interrogative sentence.
In this chapter, we shall be discussing about a specific type of sentences which will be called as
statements or propositions.
2 STATEMENTS OR PROPOSITIONS
LP

2.1 DEFINITION
A statement or a proposition is an assertive (or a declarative) sentence which is either true or false but
not both.
A statement is assumed to be either true or false. A true statement is also known as a valid statement.
If a statement is false, we say that it is an invalid statement. A statement cannot be both true and false at the
A

same time.
A sentence which is both true and false simultaneously is not a statement, rather it is a paradox.
Consider the following sentences:
NK

(i) Three plus four is 6.


(ii) The earth is a star.
(iii) Every rectangle is a square.
(iv) New Delhi is in Nepal.
(v) Every relation is a function
SA

Each of these sentences is a false declarative sentence and hence each of them is a statement.

2.2 NEGATION OF A STATEMENT


The denial of a statement is called the negation of the statement.
Let us consider the statement:
P : New Delhi is a city
The negation of this statement is
It is not the case that New Delhi is a city
This can also be written as
It is false that New Delhi is a city
This can simply be expressed as
New Delhi is not a city
Definition:
If p is a statement, then the negation of p is also a statement and is denoted by ~p and read as ‘not p’.
Note: While forming the negation of statement, phrases like, “It is not the case” or “It is false that”
are also used.
Illustration 1
Question: Write the negation of the following statements.
(i) Both the diagonals of a rectangle have the same length.
(ii) 7 is rational
Solution: (i) This statement says that in a rectangle, both the diagonals have the same length. This means that if
you take any rectangle, then both the diagonals have the same length. The negation of this
statement is

RI
It is false that both the diagonals in a rectangle have the same length.
This means the statement
There is atleast one rectangle whose both diagonals do not have the same length.
(ii) The negation of this statement is

A
It is not the case that 7 is rational.
This can also be rewritten as
7 is not rational.

UH
2.3. COMPOUND STATEMENTS
If a statement is combination of two or more simple statements, then it is said to be a compound
statement or a compound proposition.
The statement “The school works or a holiday is declared” is a compound statement as it is a
JA
combination of the statements: “The school woks” and “A holiday is declared”.
3 BASIC LOGICAL CONNECTIVES OR LOGICAL OPERATORS

The phrases or words which connect simple statements are called logical connectives or sentential
connectives or simply connectives or logical operators.
LP

In the following table, we list some possible connectives, their symbols and the nature of the
compound statement formed by them.

Connective Symbol Nature of compound statement formed by using the


connective
A

and ∧ Conjunction
or ∨ Disjunction
If ……then ⇒ or → Implication or conditional
NK

If and only if (iff) ⇔ or ↔ Equivalence or bi-conditional


not ~ or ¬ Negation

Remark: Negation is called a connective although it does not combine two or more statements.
SA

3.1 THE WORD “AND”


Let us look at a compound statement with “And”.
P : A point occupies a position and its location can be determined.
The statement can be broken into two component statements as
q: A point occupies a position.
r: Its location can be determined.
Here, we observe that both statement are true.
Let us look at another statement.
p: 42 is divisible by 5, 6 and 7
This statement has following component statements
q: 42 is divisible by 5.
r: 42 is divisible by 6.
s: 42 is divisible by 7.
Here, we know that the first is false while the other two are true.
(i) A compound statement is true only if both statements connected with ‘AND’ are true.
Otherwise it is false.
(ii) ~ (p and q) = ~p or ~q

3.2 THE WORD “OR”


Let us look at the following statement.
p: Two lines in a plane either intersect at one point or they are parallel.

We know that this is a true statement. What does this mean? This means that if two lines in a plane
intersect, then they are not parallel. Alternatively, if the two lines are not parallel, then they intersect
at a point. That is, this statement is true in both the situations.

RI
p: An ice cream or Pepsi is available with a Thali in a restaurant.

This means that a person who does not want ice cream can have a Pepsi along with Thali or one does

A
not want Pepsi can have an ice cream along with Thali. This is called an exclusive “Or”.
(i) A compound statement is true if at least one of the statements connected with ‘Or’ is true.
(ii) ~ (p or q) = ~p and ~q

UH
3.3 IMPLICATIONS
In this Section, we shall discuss the implications of “if-then”, “only if” and “if and only if”
The statements with “if-then” are very common in mathematics. For example, consider the
statement.
JA
r: If you are born in some country, then you are a citizen of that country.
When we look at this statement, we observe that it corresponds to two statements p and q given by
p: you are born in some country.
q: you are citizen of that country.

Then the sentence “if p then q” says that in the event if p is true, then q must be true.
LP

One of the most important facts about the sentence “if p then q” is that it does not say any thing (or
phaces no demand) on q when p is false. For example, if you are not born in the country, then you
cannot say anything about q. To put it in other words” not happening of p has no effect on happening
of q.
A

Another point to be noted for the statement “if p then q” statements does not imply that p happens.
Then, if p then q is the same as the following:
NK

1. ‘p implies q’ is denoted by p ⇒ q. The symbol ⇒ stands for implies.


This says that ‘A number is a multiple of 9 implies that it is a multiple of 3’.
2. p is a sufficient condition for q.
This says that ‘Knowing that a number as a multiple of 9 is sufficient to conclude that it is a multiple of
3’.
SA

3. p only if q.
This says that ‘A number is a multiple of 9 only if it is a multiple of 3’.
4. q is a necessary condition for p.
This says that ‘When a number is a multiple of 9, it is necessarily a multiple of 3’.
5. ~q implies ~p.
This says that ‘If a number is not a multiple of 3, then it is not a multiple of 9’.
6. If p and q are both true, then p ⇒ q is also true.
If p is false, q is true then p ⇒ q is true.
If p is true, q is false then p ⇒ q is false.
If p is false, q is false then p ⇒ q is true.
3.4 CONTRAPOSITIVE AND CONVERSE
Contrapositive and converse are certain other statements which can be formed from a given
statement with “if-then”.
For example, let us consider the following “if-then” statement.
“If the physical environment changes, then the biological environment changes”.

Then the contrapositive of this statement is


“If the biological environment does not change, then the physical environment does not change”.

Note that both these statement convey the same meaning.

Also, the converse of the statement is ‘If the biological environment changes then the physical
environment changes’.

Contrapositive of p ⇒ q is ~q ⇒ ~p

RI
Converse of p ⇒ q is q ⇒ p
Illustration 2

A
Question: Write the contrapositive of the following statement:
(i) If a number is divisible by 9, then it is divisible by 3.
(ii) If you are born in India, then you are a citizen of India.

UH
(iii) If a triangle is equilateral, it is isosceles.
Solution: The contrapositive of the these statements are
(i) If a number is not divisible by 3, it is not divisible by 9.
(ii) If you are not a citizen of India, then you were not born in India.
(iii) If a triangle is not isosceles, then it is not equilateral.
JA
3.5 CONDITIONAL AND BI-CONDITIONAL STATEMENTS
In Mathematics we come across many statements of the form “if p then q” and “p if and only if q”
such statements are called conditional statements. In this section, we shall discuss about such
statements.
Illustration 3
LP

Question: Let p be the statement “He is rich” and let q denote “He is happy”. Write each of the following
statements in symbolic form by using p and q.
(i) If he is rich, then he is happy.
(ii) It is necessary to be poor in order to be happy.
(iii) To be poor is to be unhappy.
A

Solution: (i) p⇒q (ii) q ⇒ ~p (iii) ~p ⇔ ~q.


NK

Illustration 4
Question: Let p represent the statement “It is raining”; let q represent “the game is cancelled” and let r
represent “Monu is sad”. Then, express each of the following in words:
(i) p ⇒ q (ii) q⇒r (iii) r⇒p (iv) q ⇒ p
Solution: (i) p ⇒ q : If it is raining, then the game is cancelled.
SA

(ii) q ⇒ r : If the game is cancelled, then Monu is sad.


(iii) r ⇒ p : If Monu is sad, then it is raining.
(iv) q ⇒ p : If the game is cancelled, then it is raining.
Illustration 5
Question: Write down the statement ‘A complex number is a real number’ in the form of a compound
statement.
Solution: Let p be the statement “x is a complex number” and let q be the statement “x is a real number”. Then,
the desired compound statement is p ⇒ q.

Important formulae/points
All kind of statements are important and stress should be given on the illustrations followed by each
definition.

RI
A
UH
JA
LP
A
NK
SA
EXERCISE

1. Which of the following sentences are statements or propositions? Justify your answer.
(i) The set of prime integers is infinite.
(ii) Paris is in England.

2. Which of the following is a statement (or proposition)?


(i) x+2=9
(ii) 6 has three prime factors

3. Write the negation of each of the following statements:


(i) Ahmad is cruel or he is strict.

RI
(ii) Rohit is smart or he is healthy
(iii) a < −7 or a > 7

4. State the contra positive of each of the following statements:

A
(i) P : A positive integer is prime only if it has no divisors other that one and itself.
(ii) q : I go to a beach only if it is a sunny day.
(iii) r : If it is hot outside, then you feel thirsty.

UH
5. Given below are two statements:
P : 25 is a multiple of 5.
q : 25 is a multiple of 8.
Write the compound statements connecting these two statements with “And” and “or”. In both
JA
cases check the validity of the compound statement.

6. Rewrite the following statements in the form “p if and only if q”.


(i) If you watch television, then your mind is free and if your mind is free then you watch
television.
(ii) If a quadrilateral is equiangular, then it is a rectangle and if a quadrilateral is a rectangle, then
LP

it is equiangular.

7. By giving a counter example, show that the following statement is false:


If n is an odd integer, then n is prime.
A

8. Verify by the method of contradiction P : 7 is irrational.


NK

9. Write the following statement in five different ways, conveying the same meaning.
P: If a natural number is odd, then its square is also odd.

10. Write the negation of the following statements:


(i) p: For every positive real number x, the number x − 1 is also positive.
(ii) q: All cats scratch.
SA

(iii) r: For every real number x, either x > 1 or x < 1.


(iv) s: There exists a number x such that 0 < x < 1.

11. For the given statements identify the necessary and sufficient conditions.
t: If you drive over 80 km per hour, then you will get a fine.

12. Which of the following statements are true and which are false? In each case give a valid reason
for saying so.
(i) p: Each radius of a circle is a chord of the circle.
(ii) q: The centre of circle bisects each chord of the circle.
(iii) r: Circle is a particular case of an ellipse.
(iv) s: If x and y are integers such that x > y , then − x < − y .
(v) t: 11 is a rational number.
13. Given statements in (a) and (b). Identify the statements given below as contrapositive or
converse of each other.
(a) If you live in Delhi, then you have winter clothes.
(i) If you do not have winter cloths, then you do not live in Delhi.
(ii) If you have winter clothes, then you live in Delhi.
(b) If a quadrilateral is a parallelogram, then its diagonals bisect each other.
(i) If the diagonals of a quadrilateral do not bisect each other, then the quadrilateral is not a
parallelogram.
(ii) If the diagonals of a quadrilateral bisect each other, then it is a parallelogram.

14. Write down the negation of the following statements.


(i) All integers are rational numbers.

RI
(ii) All squares are rectangles.
(iii) 2 is an irrational number.
(iv) Some even numbers are prime numbers.
(v) Every rational number is a real number.

A
15. Write down the following statements as negation of some statement into symbolic form:
(i) If is false that the sky is not blue.

UH
(ii) It is not the case that Roses are not red.
(iii) It is false that the grass is green.
(iv) Some prime numbers are not odd numbers.
JA
LP
A
NK
SA

ANSWERS TO EXERCISE
1. (i) and (ii)
2. (ii)

3. (i) Neither Ahmad is cruel nor he is strict.


(ii) Neither Rohit is smart nor he is healthy.
(iii) –7 < a < 7

4. (i) If a positive integer has divisors other than 1 and itself then it is not prime.
(ii) If it is not a sunny day I won’t go to beach.
(iii) If you don’t feel thirsty then it is not hot outside.

5. 25 is a multiple of 5 and 8. (False)


25 is a multiple of 5 or 8 (True)

RI
6. (i) Your mind is free if and only if you watch television.
(ii) A quadrilateral is a rectangle if and only if it is equiangular.

A
9. A natural number is odd implies that its square is odd.
Or A natural number is odd only if its square is odd.
Or For a natural number to be odd it is necessary that its square is odd.
Or For the square of a natural number to be odd, it is sufficient that the number is odd.

UH
Or If the square of a natural number is not odd, then the natural number is not odd.

10. (i) There exists a positive real number x such that x − 1 is not positive.
(ii) There exists a cat which does not scratch.
(iii) There exists a real number x such that neither x > 1 nor x < 1 .
JA
(iv) There does not exist a number x such that 0 < x < 1.

11. Sufficient condition: “driving over 80 km per hour.”


Necessary condition: “getting a fine.”

12. (i) False: By definition of the chord, it should intersect the circle in two points.
LP

(ii) False: This can be shown by giving a counter example. A chord which is not a diameter
gives the counter example.
(iii) True: In the equation of an ellipse if we put a = b, then it is a circle.
(iv) True: by the rule of inequality.
A

(v) False: Since 11 is a prime number, therefore 11 is irrational.

13. (a) (i) Contrapositive


(ii) Converse
NK

(b) (i) Contrapositive


(ii) Converse

14. (i) Not all integers are rational numbers.


Or There exists an integer which is not a rational number.
SA

Or Some integers are not rational numbers.


Or It is not the case that all integers are rational numbers.
Or It is false that all integers are rational numbers.
(ii) Not all squares are rectangles.
Or There exists a square which is not a rectangle.
Or Some squares are not rectangles.
Or It is not the case that all squares are rectangles.
Or It is false that all squares are rectangle.
(iii) 2 is not an irrational number.
Or It is false that 2 is an irrational number.
Or It is not the case that 2 is an irrational number.
(iv) No even number is a prime number.
(v) Not every rational number is a real number.
Or It is false that every rational number is a real number.
Or It is not the case that every rational number is a real number.

15. (i) Let P : The sky is blue.


~(~P): It is false that the sky is not blue.
(ii) Let P : Roses are red.
~(~P): It is not the case that roses are not red.
(iii) Let P : The grass is green.
~P: It is false that the grass is green.
(iv) Let P : All prime numbers are odd.
~P: Some prime numbers are not odd numbers.

RI
A
UH
JA
LP
A
NK
SA
PERMUTATION & COMBINATIONS

JEE (Advanced) Syllabus

Fundamental principle of counting, permutation as an arrangement and combination as selection, Meaning of P


(n, r) and C (n, r), applications, Multinomial theorem, circular permutation, geometry problems, Dearrangement

JEE (Main) Syllabus

Fundamental principle of counting, permutation as an arrangement and combination as selection, Meaning of P

RI
(n, r) and C (n, r), simple applications.

There can never be surprises in logic...Wittgenstein, Ludwig


The most fundamental application of mathematics is counting. There are many natural methods used

A
for counting
This chapter is dealing with various known techniques those are much faster than the usual counting
methods.
We mainly focus, our methods, on counting the number of arrangements (Permutations) and the

UH
number of selections (combinations), even although we may use these techniques for counting in some
other situations also .
Let us start with a simple problem
A group G1 of 3 circles C1, C2, C3 having different centers are situated in such a way that C2 lie entirely
inside C1 ; C3 lie entirely inside C2. Another group G2 of 4 circles C1′, C2′, C3′, C4′ are also situated in a
JA
similar fashion. The two groups of circles are in such a way that each member of G 1 intersect with every
member of G2, as shown in the following figure
LP

(i) How many centres the circles altogether has ?


(ii) How many common chords are obtained ?
A

The answer to the first part is "3 + 4 = 7" and answer to the second part is "3 × 4 = 12" . The method in
which we calculated first part of the problem is called as "addition rule" and the method we used to
calculate its second part is called as the "multiplication rule". These rules altogether are the most
important tools in counting, popularly known as "the fundamental counting principle".
NK

Fundamental counting principle :


Suppose that an operation O1 can be done in m different ways and another operation O2 can be done in
n different ways.
(i) Addition rule : The number of ways in which we can do exactly one of the operations O 1, O2
SA

is m + n
(ii) Multiplication rule : The number of ways in which we can do both the operations O 1, O2 is mn.
Note : The addition rule is true only when O1 & O2 are mutually exclusive and multiplication rule is true only
when O1 & O2 are independent (The reader will understand the concepts of mutual exclusiveness and
independence, in the due course)
Example # 1 : There are 8 buses running from Kota to Jaipur and 10 buses running from Jaipur to Delhi. In
how many ways a person can travel from Kota to Delhi via Jaipur by bus?
Solution : Let E1 be the event of travelling from Kota to Jaipur & E2 be the event of travelling from Jaipur
to Delhi by the person.
E1 can happen in 8 ways and E2 can happen in 10 ways.
Since both the events E1 and E2 are to be happened in order, simultaneously,
the number of ways = 8 × 10 = 80.
Example # 2 : How many numbers between 10 and 10,000 can be formed by using the digits 1, 2, 3, 4, 5 if
(i) No digit is repeated in any number. (ii) Digits can be repeated.
Solution : (i) Number of two digit numbers = 5 × 4 = 20
Number of three digit numbers = 5 × 4 × 3 = 60
Number of four digit numbers = 5 × 4 × 3 × 2 = 120
Total = 200
(ii) Number of two digit numbers = 5 × 5 = 25
Number of three digit numbers = 5 × 5 × 5 = 125
Number of four digit numbers = 5 × 5 × 5 × 5 = 625
Total = 775

Self Practice Problems :


(1) How many 4 digit numbers are there, without repetition of digits, if each number is divisible
by 5 ?
(2) Using 6 different flags, how many different signals can be made by using atleast three flags,
arranging one above the other?

RI
Ans. (1) 952 (2) 1920
Arrangements :
If nPr denotes the number of permutations (arrangements) of n different things, taking r at a time, then

A
nP = n (n − 1) (n − 2)..... (n − r + 1) = n!
r
(n − r)!
NOTE : (i) Factorials of negative integers are not defined.

UH
(ii) 0 ! = 1 ! = 1
(iii) nPn = n ! = n. (n − 1) !
(iv) (2n) ! = 2n. n ! [1. 3. 5. 7... (2n − 1)]
Example # 3 : How many three digit can be formed using the digits 1, 2, 3, 4, 5, without repetition of digits?
How many of these are even?
JA
Solution : Three places are to be filled with 5 different objects.
∴ Number of ways = 5P3 = 5 × 4 × 3 = 60
For the 2nd part, unit digit can be filled in two ways & the remaining two digits can be filled
in 4P2 ways.
∴ Number of even numbers = 2 × 4P2 = 24.
LP

Example # 4 : If all the letters of the word 'QUEST' are arranged in all possible ways and put in dictionary
order, then find the rank of the given word.
Solution : Number of words beginning with E = 4P4 = 24
Number of words beginning with QE = 3P3 = 6
Number of words beginning with QS = 6
A

Number of words beginning withQT = 6.


Next word is 'QUEST'
∴ its rank is 24 + 6 + 6 + 6 + 1 = 43.
NK

Self Practice Problems :


(3) Find the sum of all four digit numbers (without repetition of digits) formed using the digits
1, 2, 3, 4, 5.
(4) Find 'n', if n – 1P3 : nP4 = 1 : 9.
SA

(5) Six horses take part in a race. In how many ways can these horses come in the first, second
and third place, if a particular horse is among the three winners (Assume No Ties)?
(6) Find the sum of all three digit numbers those can be formed by using the digits. 0, 1, 2, 3, 4.
Ans. (3) 399960 (4) 9 (5) 60 (6) 27200
Result : Let there be 'n' types of objects, with each type containing atleast r objects. Then the number of ways of
arranging r objects in a row is nr.

Example # 5 : How many 3 digit numbers can be formed by using the digits 0, 1, 2, 3, 4, 5. In how many of
these we have atleast one digit repeated?
Solution : We have to fill three places using 6 objects (repetition allowed), 0 cannot be at 100 th place.
The number of numbers = 180.
Number of numbers in which no digit is repeated = 100
∴ Number of numbers in which atleast one digit is repeated = 180 – 100 = 80
Example # 6 : How many functions can be defined from a set A containing 5 elements to a set B having 3
elements? How many of these are surjective functions?
Solution : Image of each element of A can be taken in 3 ways.
∴ Number of functions from A to B = 35 = 243.
Number of into functions from A to B = 25 + 25 + 25 – 3 = 93.
∴ Number of onto functions = 150.

Self Practice Problems :


(7) How many functions can be defined from a set A containing 4 elements to a set B containing 5
elements? How many of these are injective functions?
(8) In how many ways 5 persons can enter into a auditorium having 4 entries?
Ans. (7) 625, 120 (8) 1024.

Combination :

RI
If nCr denotes the number of combinations (selections) of n different things taken r at a time, then
n
n ! Pr
nC
r = = where r ≤ n ; n ∈ N and r ∈ W.
r! (n − r)! r!

A
NOTE : (i) nCr = nCn – r
(ii) nCr + nCr – 1 = n + 1Cr
(iii) nCr = 0 if r ∉ {0, 1, 2, 3........, n}

UH
Example # 7 : There are fifteen players for a cricket match.
(i) In how many ways the 11 players can be selected?
(ii) In how many ways the 11 players can be selected including a particular player?
(iii) In how many ways the 11 players can be selected excluding two particular players?
JA
Solution : (i) 11 players are to be selected from 15
Number of ways = 15C11 = 1365.
(ii) Since one player is already included, we have to select 10 from the remaining 14
Number of ways = 14C10 = 1001.
(iii) Since two players are to be excluded, we have to select 11 from the remaining 13.
Number of ways = 13C11 = 78.
LP

Example # 8 : If 49C3r – 2 = 49C2r + 1, find 'r'.


Solution : n
Cr = nCs if either r = s or r + s = n.
Thus 3r – 2 = 2r + 1 ⇒ r=3
or 3r – 2 + 2r + 1 = 49 ⇒ 5r – 1 = 49 ⇒ r = 10
∴ r = 3, 10
A

Example # 9 : A regular polygon has 20 sides. How many triangles can be drawn by using the vertices, but
not using the sides?
NK

Solution : The first vertex can be selected in 20 ways. The remaining two are to be selected from 17
vertices so that they are not consecutive. This can be done in 17C2 – 16 ways.
∴ The total number of ways = 20 × (17C2 – 16)
But in this method, each selection is repeated thrice.
20 × (17 C2 − 16)

SA

Number of triangles = = 800.


3

Example # 10 : 15 persons are sitting in a row. In how many ways we can select three of them if adjacent
persons are not selected ?
Solution : Let P1, P2, P3, P4, P5, P6, P7, P8, P9, P10 ,P11 , P12 , P13 ,P14 ,P15 be the persons sitting in this order.
If three are selected (non consecutive) then 12 are left out.
Let P,P,P,P,P,P,P,P,P,P,P,P be the left out & q, q, q be the selected. The number of ways in
which these 3 q's can be placed into the 13 positions between the P's (including extremes) is
the number ways of required selection.
Thus number of ways = 13C3 = 286.
Example # 11 : In how many ways we can select 4 letters from the letters of the word MΙSSΙSSΙPPΙ?
Solution : M
ΙΙΙΙ
SSSS
PP
Number of ways of selecting 4 alike letters = 2C1 = 2.
Number of ways of selecting 3 alike and 1 different letters = 2C1 × 3C1 = 6
Number of ways of selecting 2 alike and 2 alike letters = 3C2 = 3
Number of ways of selecting 2 alike & 2 different = 3C1 × 3C2 = 9
Number of ways of selecting 4 different = 4C4 = 1
Total number of ways = 2 + 6 + 3 + 9 + 1 = 21

Self Practice Problems :


(9) In how many ways 7 persons can be selected from among 5 Indian, 4 British & 2 Chinese, if
atleast two are to be selected from each country ?
(10) Find a number of different seven digit numbers that can be written using only three digits 1,2&3
under the condition that the digit 2 occurs exactly twice in each number ?

RI
(11) In how many ways 6 boys & 6 girls can sit at a round table so that girls & boys sit alternate?
(12) In how many ways 4 persons can occupy 10 chairs in a row, if no two sit on adjacent chairs?
(13) In how many ways we can select 3 letters of the word PROPORTION ?

A
Ans. (9) 100 (10) 672 (11) 86400 (12) 840 (13) 36

Arrangement of n things, those are not all different :

UH
The number of permutations of 'n' things, taken all at a time, when 'p' of them are same & of one type, q
of them are same & of second type, 'r' of them are same & of a third type & the remaining
n!
n − (p + q + r) things are all different, is .
p ! q !r !
JA
Example # 12 : In how many ways we can arrange 3 red flowers, 4 yellow flowers and 5 white flowers in a row?
In how many ways this is possible if the white flowers are to be separated in any arrangement?
(Flowers of same colour are identical).
Solution : Total we have 12 flowers 3 red, 4 yellow and 5 white.
12 !
Number of arrangements = = 27720.
3 ! 4 ! 5 !
For the second part, first arrange 3 red & 4 yellow
LP

7 !
This can be done in = 35 ways
3 ! 4 !
Now select 5 places from among 8 places (including extremes) & put the white flowers there.
This can be done in 8C5 = 56.
A

∴ The number of ways for the 2nd part = 35 × 56 = 1960.

Example # 13 : In how many ways the letters of the word "ARRANGE" can be arranged without altering the
NK

relative positions of vowels & consonants?


4!
Solution : The consonants in their positions can be arranged in = 12 ways.
2!
3!
The vowels in their positions can be arranged in = 3 ways
2!
∴ Total number of arrangements = 12 × 3 = 36
SA

Self Practice Problems :


(14) How many words can be formed using the letters of the word ASSESSMENT if each word
begin with A and end with T?
(15) If all the letters of the word ARRANGE are arranged in all possible ways, in how many of words
we will have the A's not together and also the R's not together?
(16) How many arrangements can be made by taking four letters of the word MISSISSIPPI?
Ans. (14) 840 (15) 660 (16) 176.
Formation of Groups :
Number of ways in which (m + n + p) different things can be divided into three different groups

containing m, n & p things respectively is


(m + n + p ) ! ,
m !n !p !
If m = n = p and the groups have identical qualitative characteristic then the number of groups =
(3n)!
.
n! n! n! 3!
(3n)!
Note : If 3n different things are to be distributed equally among three people then the number of ways = .
(n!)3
Example # 14 : 12 different toys are to be distributed to three children equally. In how many ways this can be
done ?

RI
Solution : The problem is to divide 12 different things into three different groups.
12!
Number of ways = = 34650.
4! 4! 4!

A
Example # 15 : In how many ways 10 persons can be divided into 5 pairs?
Solution : We have each group having 2 persons and the qualitative characteristic are same (Since there
is no purpose mentioned or names for each pair).

UH
10!
Thus the number of ways = = 945.
(2!)5 5!

Self Practice Problems :


(17) 9 persons enter a lift from ground floor of a building which stops in 10 floors (excluding ground
JA
floor), if it is known that persons will leave the lift in groups of 2, 3, & 4 in different floors. In how
many ways this can happen?
(18) In how many ways one can make four equal heaps using a pack of 52 playing cards?
(19) In how many ways 11 different books can be parcelled into four packets so that three of the
packets contain 3 books each and one of 2 books, if all packets have the same destination?
52! 11!
LP

Ans. (17) 907200 (18) 4


(19)
(13!) 4! (3!)4 2

Circular Permutation :
The number of circular permutations of n different things taken all at a time is (n − 1) !.
A

(n − 1)!
If clockwise & anti−clockwise circular permutations are considered to be same, then it is .
2
NK

Note : Number of circular permutations of n things when p are alike and the rest are different, taken all at a
(n − 1) !
time, distinguishing clockwise and anticlockwise arrangement is .
p!

Example # 16 : In how many ways can we arrange 6 different flowers in a circle? In how many ways we
SA

can form a garland using these flowers?


Solution : The number of circular arrangements of 6 different flowers = (6 – 1)! = 120
When we form a garland, clockwise and anticlockwise arrangements are similar. Therefore, the
1
number of ways of forming garland = (6 – 1) ! = 60.
2
Example # 17 : In how many ways 6 persons can sit at a round table, if two of them prefer to sit together?
Solution : Let P1, P2, P3, P4, P5, P6 be the persons, where P1, P2 want to sit together.
Regard these person as 5 objects. They can be arranged in a circle in (5 – 1)! = 24 ways. Now
P1, P2 can be arranged in 2! ways. Thus the total number of ways = 24 × 2 = 48.
Self Practice Problems :
(20) In how many ways letters of the word 'MONDAY' can be written around a circle, if vowels
are to be separated in any arrangement ?
(21) In how many ways we can form a garland using 3 different red flowers,5 different yellow flowers
and 4 different blue flowers, if flowers of same colour must be together?
Ans. (20) 72 (21) 17280

Selection of one or more objects


(a) Number of ways in which atleast one object may be selected out of 'n' distinct objects, is
n
C1 + nC2 + nC3 +...............+ nCn = 2n – 1
(b) Number of ways in which atleast one object may be selected out of 'p' alike objects of one
type, 'q' alike objects of second type and 'r' alike objects of third type, is
(p + 1) (q + 1) (r + 1) – 1
(c) Number of ways in which atleast one object may be selected from 'n' objects where 'p' alike of
one type, 'q' alike of second type and 'r' alike of third type and rest n – (p + q + r) are different,
is (p + 1) (q + 1) (r + 1) 2n – (p + q + r) – 1
Example # 18 : There are 12 different books in a shelf. In how many ways we can select atleast one of them?

RI
Solution : We may select 1 book, 2 books,........, 12 books.
∴ The number of ways = 12C1 + 12C2 + ....... + 12C12 = 212 – 1. = 4095

Example # 19 : There are 11 fruits in a basket of which 6 are apples, 3 mangoes and 2 bananas (fruits of same

A
species are identical). How many ways are there to select atleast one fruit?
Solution : Let x be the number of apples being selected
y be the number of mangoes being selected and
z be the number of bananas being selected.

UH
Then x = 0, 1, 2, 3, 4, 5,6
y = 0, 1, 2, 3
z = 0, 1, 2
Total number of triplets (x, y, z) is 7 × 4 × 3 = 84
Exclude (0, 0, 0)
∴ Number of combinations = 84 – 1 = 83.
JA
Self Practice Problems
(22) In a shelf there are 6 physics, 4 chemistry and 3 mathematics books. How many combinations
are there if (i) books of same subject are different? (ii) books of same subject are identical?
(23) From 5 apples, 4 mangoes & 3 bananas, in how many ways we can select atleast two fruits of
LP

each variety if (i) fruits of same species are identical? (ii) fruits of same species are different?
Ans. (22) (i) 8191 (ii) 139 (23) (i) 24 (ii) 212 – 4
Results : Let N = p q r ..... where p, q, r...... are distinct primes & a, b, c..... are natural numbers then:
a. b. c.

(a) The total numbers of divisors of N including 1 & N is = (a + 1) (b + 1) (c + 1)........


A

(b) The sum of these divisors is =


(p0 + p1 + p2 +.... + pa) (q0 + q1 + q2 +.... + qb) (r0 + r1 + r2 +.... + rc)........
(c) Number of ways in which N can be resolved as a product of two factors is
 1 (a + 1) (b + 1) (c + 1) ....
NK

if N is not a perfect square


=  12
 2 (a + 1) (b + 1) (c + 1) .... + 1 if N is a perfect square
(d) Number of ways in which a composite number N can be resolved into two factors which are
relatively prime (or coprime) to each other is equal to 2n−1 where n is the number of different
prime factors in N.
SA

Example # 20 : Find the number of divisors of 1350. Also find the sum of all divisors.
Solution : 1350 = 2 × 33 × 52
∴ Number of divisors = (1+ 1) (3 + 1) (2 + 1) = 24
sum of divisors = (1 + 2) (1 + 3 + 32 + 33) (1 + 5 + 52) = 3720.

Example # 21 : In how many ways 8100 can be resolved into product of two factors?
Solution : 8100 = 22 × 34 × 52
1
Number of ways = [(2 + 1) (4 + 1) (2 + 1) + 1] = 23
2
Self Practice Problems :
(24) How many divisors of 9000 are even but not divisible by 4? Also find the sum of all such
divisors.
(25) In how many ways the number 8100 can be written as product of two coprime factors?
Ans. (24) 12, 4056 (25) 4

Negative binomial expansion :


(1 – x)– n = 1 + nC1x + n + 1C2 x2 + n + 2C3x3 + ........ to ∞ , if – 1 < x < 1.

Coefficient of xr in this expansion = n+r−1


Cr (n ∈ N)

Result : Number of ways in which it is possible to make a selection from m + n + p = N things, where p are alike
of one kind, m alike of second kind & n alike of third kind, taken r at a time is given by coefficient of x r in
the expansion of
(1 + x + x2 +...... + xp) (1 + x + x2 +...... + xm) (1 + x + x2 +...... + xn).
For example the number of ways in which a selection of four letters can be made from the letters of the
word PROPORTION is given by coefficient of x4 in

RI
(1 + x + x2 + x3) (1 + x + x2) (1 + x + x2) (1 + x) (1 + x) (1 + x).

Method of fictious partition :


Number of ways in which n identical things may be distributed among p persons if each person may

A
receive none, one or more things is n+p−1Cn.

Example # 22 : Find the number of solutions of the equation x + y + z = 6, where x, y, z ∈ W.

UH
Solution : Number of solutions = coefficient of x6 in (1 + x + x2 + ....... x6)3
= coefficient of x6 in (1 – x7)3 (1 – x)–3
= coefficient of x6 in (1 – x)–3
= 3+6–1C6 = 8C2 = 28.

Example # 23 : In a bakery four types of biscuits are available. In how many ways a person can buy 10
JA
biscuits if he decide to take atleast one biscuit of each variety?
Solution : Let the person select x biscuits from first variety, y from the second, z from the third and w
from the fourth variety. Then the number of ways = number of solutions of the equation
x + y + z + w = 10.
where x = 1, 2, .........,7
y = 1, 2, .........,7
z = 1, 2, .........,7
LP

w = 1, 2, .........,7

So, number of ways = coefficient of x10 in (x + x2 + ...... + x7)4


= coefficient of x6 in (1 + x + ....... + x6)4
= coefficient of x6 in (1 – x7)4 (1 – x)–4
A

= coefficient x6 in (1 – x)–4
= 4+6–1C6 = 9C3 = 84
Self Practice Problems:
NK

(26) Three distinguishable dice are rolled. In how many ways we can get a total 15?
(27) In how many ways we can give 5 apples, 4 mangoes and 3 oranges (fruits of same species are
similar) to three persons if each may receive none, one or more?
Ans. (26) 10 (27) 3150
SA

Derrangements :
Number of ways in which 'n' letters can be put in 'n' corresponding envelopes such that no letter goes to
 1 1 1 1 1 
correct envelope is n !  1 − + − + − .......... + ( −1)n 
 1 ! 2 ! 3 ! 4 ! n !

Example # 24 : In how many ways we can put 5 writings into 5 corresponding envelopes so that no writing go
to the corresponding envelope?
Solution : The problem is the number of dearragements of 5 digits.
 1 1 1 1 
This is equal to 5!  − + −  = 44.
 2 ! 3 ! 4 ! 5 !

Example # 25 : Four slip of papers with the numbers 1, 2, 3, 4 written on them are put in a box. They are drawn
one by one (without replacement) at random. In how many ways it can happen that the ordinal
number of atleast one slip coincide with its own number?
Solution : Total number of ways = 4 ! = 24.
The number of ways in which ordinal number of any slip does not coincide with its own number
 1 1 1 
is the number of dearrangements of 4 objects = 4 !  − + =9
2 ! 3 ! 4 !
Thus the required number of ways. = 24 – 9 = 15

Self Practice Problems:


(28) In a match the column question, Column Ι contain 10 questions and Column II contain 10
answers written in some arbitrary order. In how many ways a student can answer this question
so that exactly 6 of his matching are correct ?
(29) In how many ways we can put 5 letters into 5 corresponding envelopes so that atleast one
letter go to wrong envelope ?

RI
Ans. (28) 1890 (29) 119

Exponent of prime p in n! :

A
Let p be a prime number, n be a positive integer and Let Ep(n) denote the exponent of the prime p in the
positive integer n. Then,
n   n   n  n

UH
Ep (n!) =   +  2  +  3  + ...... +  s 
     
p p p p 
where s is the largest positive integer such that ps ≤ n < ps+1

Example # 26 : Find exponent 2 and 3 in 100!


 100   100   100   100 
JA
Solution : Exponent of 2 in 100! is represented by E2 (100!) =   +  2  +  3  + ...... +  6 
 2   2   2   2 
= 50 + 25 + 12 + 6 + 3 + 1 = 97

 100   100   100   100 


Exponent of 3 in 100! is represented by E3 (100!) =  + 2 + 3 + 4 
 3   3   3   3 
LP

= 33 +11 + 3 + 1 = 48

Example # 27 : If 100! is divided by (24)k (where k ∈ n), then find maximum value of k.
A

 100   100   100   100 


Solution : Exponent of 2 in 100! is represented by E2 (100!) =   +  2  +  3  + ...... +  6 
 2   2   2   2 
= 50 + 25 + 12 + 6 + 3 + 1 = 97
NK

⇒ Exponent of 23 in 100! is 32.

 100   100   100   100 


Exponent of 3 in 100! is represented by E3 (100!) =  + 2 + 3 + 4 
 3   3   3   3 
SA

= 33 +11 + 3 + 1 = 48
⇒ Exponent of (23 × 3) in 100! is min{48, 32} = 32
⇒ Exponent of (24) in 100! is = 32
⇒ maximum value of k is 32.

Self Practice Problems:


(30) Find the number of zeros at the end of 50C25.
(31) Find the last non zero digits of 25!.
Ans (30) 0 (31) 4
 Marked questions are recommended for Revision.

PART - I : SUBJECTIVE QUESTIONS


Section (A) : Fundamental principle of counting, problem based on selection of given
object & arrangement of given object.
A-1. There are nine students (5 boys & 4 girls) in the class. In how many ways
(i) One student (either girl or boy) can be selected to represent the class.
(ii) A team of two students (one girl & one boy) can be selected.
(iii) Two medals can be distributed. (no one get both)

RI
(iv) One prize for Maths, two prizes for Physics and three prizes for Chemistry can be distributed.
(No student can get more than one prize in same subject & prizes are distinct)

A-2. There are 10 buses operating between places A and B. In how many ways a person can go from place
A to place B and return to place A, if he returns in a different bus?

A
A-3. There are 4 boys and 4 girls. In how many ways they can sit in a row
(i) there is no restriction.

UH
(ii) not all girls sit together.
(iii) no two girls sit together.
(iv) all boys sit together and all girls sit together .
(v) boys and girls sit alternatively.

A-4. Find the number of words those can be formed by using all letters of the word ‘DAUGHTER’. If
JA
(i) Vowels occurs in first and last place.
(ii) Start with letter G and end with letters H.
(iii) Letters G,H,T always occurs together.
(iv) No two letters of G,H,T are consecutive
(v) No vowel occurs together
(vi) Vowels always occupy even place.
(vii) Order of vowels remains same.
LP

(viii) Relative order of vowels and consonants remains same.


(ix) Number of words are possible by selecting 2 vowels and 3 consonants.

A-5. Words are formed by arranging the letters of the word "STRANGE" in all possible manner. Let m be the
number of words in which vowels do not come together and 'n' be the number of words in which vowels
A

come together. Then find the ratio of m: n.(where m and n are coprime natural number)

A-6. In a question paper there are two parts part A and part B each consisting of 5 questions. In how many
ways a student can answer 6 questions, by selecting atleast two from each part?
NK

A-7. How many 3 digit even numbers can be formed using the digits 1, 2, 3, 4, 5 (repetition allowed)?

A-8. Find the number of 6 digit numbers that ends with 21 (eg. 537621), without repetition of digits.
SA

A-9. The digits from 0 to 9 are written on slips of paper and placed in a box. Four of the slips are drawn at
random and placed in the order. How many out comes are possible?

A-10. Find the number of natural numbers from 1 to 1000 having none of their digits repeated.

A-11. A number lock has 4 dials, each dial has the digits 0, 1, 2, ........, 9. What is the maximum unsuccessful
attempts to open the lock?

A-12. In how many ways we can select a committee of 6 persons from 6 boys and 3 girls, if atleast two boys
& atleast two girls must be there in the committee?

A-13. In how many ways 11 players can be selected from 15 players, if only 6 of these players can bowl and
the 11 players must include atleast 4 bowlers?

A-14. A committee of 6 is to be chosen from 10 persons with the condition that if a particular person 'A' is
chosen, then another particular person B must be chosen.
A-15. In how many ways we can select 5 cards from a deck of 52 cards, if each selection must include atleast
one king.

A-16. How many four digit natural numbers not exceeding the number 4321 can be formed using the digits
1, 2, 3, 4, if repetition is allowed?

A-17. How many different permutations are possible using all the letters of the word MISSISSIPPI, if no two
Ι's are together?

A-18. If A = {1, 2, 3, 4 .....n} and B ⊂ A ; C ⊂ A, then the find number of ways of selecting
(i) Sets B and C
(ii) Order pair of B and C such that B ∩ C = φ
(iii) Unordered pair of B and C such that B ∩ C = φ

RI
(iv) Ordered pair of B and C such that B ∪ C = A and B ∩ C = φ
(v) Unordered pair of B and C such that B ∪ C = A, B ∩ C = φ
(vi) Ordered pair of B and C such that B ∩ C is singleton

A
A-19. For a set of six true or false statements, no student in a class has written all correct answers and no
two students in the class have written the same sequence of answers. What is the maximum number of
students in the class, for this to be possible.

UH
A-20. How many arithmetic progressions with 10 terms are there, whose first term is in the set {1, 2, 3, 4}
and whose common difference is in the set {3, 4, 5, 6, 7} ?

A-21. Find the number of all five digit numbers which have atleast one digit repeated.
JA
A-22. There are 3 white, 4 blue and 1 red flowers. All of them are taken out one by one and arranged in a row
in the order. How many different arrangements are possible (flowers of same colurs are similar)?

Section (B) : Grouping and Circular Permutation


B-1. In how many ways 18 different objects can be divided into 7groups such that four groups contains
3 objects each and three groups contains 2 objects each.
LP

B-2. In how many ways fifteen different items may be given to A, B, C such that A gets 3, B gets 5 and
remaining goes to C.

B-3. Find number of ways of distributing 8 different items equally among two children.
A

B-4. (a) In how many ways can five people be divided into three groups?
(b) In how many ways can five people be distributed in three different rooms if no room must be
NK

empty?
(c) In how many ways can five people be arranged in three different rooms if no room must be
empty and each room has 5 seats in a single row.

200!
B-5. Prove that : is an integer
(10!)20 19!
SA

B-6. In how many ways 5 persons can sit at a round table, if two of the persons do not sit together?
B-7. In how many ways four men and three women may sit around a round table if all the women are
together?

B-8. Seven persons including A, B, C are seated on a circular table. How many arrangements are possible if
B is always between A and C?

B-9. In how many ways four '+' and five '–' sign can be arranged in a circles so that no two '+' sign are
together.

Section (C) : Problem based on distinct and identical objects and divisors
C-1. Let N = 24500, then find
(i) The number of ways by which N can be resolved into two factors.
(ii) The number of ways by which 5N can be resolved into two factors.
(iii) The number of ways by which N can be resolved into two coprime factors.
C-2. Find number of ways of selection of one or more letters from AAAABBCCCDEF

(i) there is no restriction.


(ii) the letters A & B are selected atleast once.
(iii) only one letter is selected.
(iv) atleast two letters are selected

C-3. Find number of ways of selection of atleast one vowel and atleast one consonant from the word
TRIPLE

C-4. Find number of divisiors of 1980.


(i) How many of them are multiple of 11 ? find their sum
(ii) How many of them are divisible by 4 but not by 15.

RI
Section (D) : Multinomial theorem & Dearrangement

A
D-1. Find number of negative integral solution of equation x + y + z = – 12

D-2. In how many ways it is possible to divide six identical green, six identical blue and six identical red
among two persons such that each gets equal number of item?

UH
D-3. Find the number of solutions of x + y + z + w = 20 under the following conditions:
(i) x, y, z, w are whole number
(ii) x, y, z, w are natural number
(iii) x, y, z, w ∈ {1, 2, 3, .........., 10}
(iv) x, y, z, w are odd natural number
JA
D-4. A person has 4 distinct regular tetrahedron dice. The number printed on 4 four faces of dice are –3, –1,
1 and 3. The person throws all the 4 dice. Find the total number of ways of getting sum of number
appearing on the bottom face of dice equal to 0.

D-5. Five balls are to be placed in three boxes in how many diff. ways can be placed the balls so that no box
LP

remains empty if
(i) balls and boxes are diff, (ii) balls identical and boxes diff.
(iii) balls diff. and boxes identical (iv) balls as well as boxes are identical
b
D-6. Let Dn represents derangement of 'n' objects. If D n+2 = a Dn+1 + b Dn ∀ n ∈ N, then find
a
A

D-7. A person writes letters to five friends and addresses on the corresponding envelopes. In how many
ways can the letters be placed in the envelopes so that
(a) all letters are in the wrong envelopes?
NK

(b) at least three of them are in the wrong envelopes?

Section (E) : Miscellaneous


E-1. (i) Find exponent of 3 in 20 !
(ii) Find number of zeros at the end of 45!.
SA

E-2. Find the total number of ways of selecting two number from the set of first 100 natural number such that
difference of their square is divisible by 3

E-3. A four digit number plate of car is said to be lucky if sum of first two digit is equal to sum of last two
digit. Then find the total number of such lucky plate. (Assume 0000, 0011, 0111, ......... all are four digit
number)

E-4. Let each side of smallest square of chess board is one unit in length.
(i) Find the total number of squares of side length equal to 3 and whose side parallel to side of
chess board.
(ii) Find the sum of area of all possible squares whose side parallel to side of chess board.
(iii) Find the total number of rectangles (including squares) whose side parallel to side of chess
board.
E-5. A person is to walk from A to B. However, he is restricted to walk only to the right of A or upwards of A.
but not necessarily in the order shown in the figure. Then find the number of paths from A to B.

PART - II : ONLY ONE OPTION CORRECT TYPE


Section (A) : Fundamental principle of counting, problem based on selection of given
object & arrangement of given object, rank of word
A-1. The number of signals that can be made with 3 flags each of different colour by hoisting 1 or 2 or 3
above the other, is:
(A) 3 (B) 7 (C) 15 (D) 16

A-2. 8 chairs are numbered from 1 to 8. Two women & 3 men wish to occupy one chair each. First the
women choose the chairs from amongst the chairs marked 1 to 4, then the men select the chairs from

RI
among the remaining. The number of possible arrangements is:
(A) 6C3. 4C4 (B) P2. 4P3 (C) 4C3. 4P3 (D) 4P2. 6P3

A-3. Number of words that can be made with the letters of the word "GENIUS" if each word neither begins

A
with G nor ends in S, is:
(A) 24 (B) 240 (C) 480 (D) 504

UH
A-4. The number of words that can be formed by using the letters of the word ‘MATHEMATICS’ that start as
well as end with T, is
(A) 80720 (B) 90720 (C) 20860 (D) 37528

A-5. 5 boys & 3 girls are sitting in a row of 8 seats. Number of ways in which they can be seated so that not
all the girls sit side by side, is:
JA
(A) 36000 (B) 9080 (C) 3960 (D) 11600

A-6. Out of 16 players of a cricket team, 4 are bowlers and 2 are wicket keepers. A team of 11 players is to
be chosen so as to contain at least 3 bowlers and at least 1 wicketkeeper. The number of ways in
which the team be selected, is
(A) 2400 (B) 2472 (C) 2500 (D) 960
LP

A-7 Passengers are to travel by a double decked bus which can accommodate 13 in the upper deck and 7
in the lower deck. The number of ways that they can be divided if 5 refuse to sit in the upper deck and 8
refuse to sit in the lower deck, is
(A) 25 (B) 21 (C) 18 (D) 15
A

A-8. The number of permutations that can be formed by arranging all the letters of the word ‘NINETEEN’ in
which no two E’s occur together. is
8! 5! 5! 6 8! 6
(A) (B) (C) × C3 (D) × C3 .
NK

3! 3! 3! × 6 C2 3! 5!

A-9. 10 different letters of an alphabet are given. Words with 5 letters are formed from these given letters,
then the number of words which have atleast one letter repeated is:
(A) 69760 (B) 30240 (C) 99748 (D) none
SA

A-10. In a conference 10 speakers are present. If S1 wants to speak before S2 & S2 wants to speak after
S3, then the number of ways all the 10 speakers can give their speeches with the above restriction if the
remaining seven speakers have no objection to speak at any number is :
10 !
(A) 10C3 (B) 10P8 (C) 10P3 (D)
3

A-11. If all the letters of the word "QUEUE" are arranged in all possible manner as they are in a dictionary,
then the rank of the word QUEUE is:
(A) 15th (B) 16th (C) 17th (D) 18th

A-12. The sum of all the numbers which can be formed by using the digits 1, 3, 5, 7 all at a time and which
have no digit repeated, is
(A) 16 × 4! (B) 1111 × 3! (C) 16 × 1111 × 3! (D) 16 × 1111 × 4!.
A-13. How many nine digit numbers can be formed using the digits 2, 2, 3, 3, 5, 5, 8, 8, 8 so that the odd
digits occupy even positions?
(A) 7560 (B) 180 (C) 16 (D) 60

A-14. There are 2 identical white balls, 3 identical red balls and 4 green balls of different shades. The number
of ways in which they can be arranged in a row so that atleast one ball is separated from the balls of
the same colour, is :
(A) 6 (7 ! – 4!) (B) 7 (6 ! – 4 !) (C) 8 ! – 5 ! (D) none

A-15. A box contains 2 white balls, 3 black balls & 4 red balls. In how many ways can three balls be drawn
from the box if atleast one black ball is to be included in draw (the balls of the same colour are
different).
(A) 60 (B) 64 (C) 56 (D) none

RI
A-16. Eight cards bearing number 1, 2, 3, 4, 5, 6, 7, 8 are well shuffled. Then in how many cases the top
2 cards will form a pair of twin prime equals
(A) 720 (B) 1440 (C) 2880 (D) 2160

A-17. Number of natural number upto one lakh, which contains 1,2,3, exactly once and remaining digits any

A
time is -
(A) 2940 (B) 2850 (C) 2775 (D) 2680

UH
A-18. The sum of all the four digit numbers which can be formed using the digits 6,7,8,9 (repetition is allowed)
(A) 2133120 (B) 2133140 (C) 2133150 (D) 2133122

A-19. If the different permutations of the word ‘EXAMINATION’ are listed as in a dictionary, then how many
words (with or without meaning) are there in this list before the first word starting with M.
(A) 2268000 (B) 870200 (C) 807400 (D) 839440
JA
A-20. The number of ways in which a mixed double tennis game can be arranged from amongst 9 married
couple if no husband & wife plays in the same game is:
(A) 756 (B) 3024 (C) 1512 (D) 6048

Section (B) : Grouping and circular Permutation


LP

B-1. Number of ways in which 9 different toys be distributed among 4 children belonging to different age
groups in such a way that distribution among the 3 elder children is even and the youngest one is to
receive one toy more, is:

(A)
( 5 ! )2 (B)
9!
(C)
9!
(D) none
A

8 2 3
3 ! ( 2 !)

B-2. In an eleven storeyed building (Ground floor + ten floor), 9 people enter a lift cabin from ground floor. It
NK

is know that they will leave the lift in groups of 2, 3 and 4 at different residential storeys. Find the
number of ways in which they can get down.
9 × 9! 8 × 9! 2 × 10! 10!
(A) (B) (C) (D)
4 4 9 4
B-3. The number of ways in which 8 different flowers can be strung to form a garland so that 4 particulars
flowers are never separated, is:
SA

8!
(A) 4 !. 4 ! (B) (C) 288 (D) none
4!

B-4. The number of ways in which 6 red roses and 3 white roses (all roses different) can form a garland so
that all the white roses come together, is
(A) 2170 (B) 2165 (C) 2160 (D) 2155

B-5. The number of ways in which 4 boys & 4 girls can stand in a circle so that each boy and each girl is one
after the other, is:
(A) 3 !. 4 ! (B) 4 !. 4 ! (C) 8 ! (D) 7 !

B-6. The number of ways in which 5 beads, chosen from 8 different beads be threaded on to a ring, is:
(A) 672 (B) 1344 (C) 336 (D) none
B-7. Number of ways in which 2 Indians, 3 Americans, 3 Italians and 4 Frenchmen can be seated on a
circle, if the people of the same nationality sit together, is:
(A) 2. (4 !)2 (3 !)2 (B) 2. (3 !)3. 4 ! (C) 2. (3 !) (4 !)3 (D) 2. (3 !)2 (4 !)3

Section (C) : Problem based on distinct and identical objects and divisors
C-1. The number of proper divisors of apbqcrds where a, b, c, d are primes & p, q, r, s ∈ N, is
(A) p q r s (B) (p + 1) (q + 1) (r + 1) (s + 1) − 4
(C) p q r s − 2 (D) (p + 1) (q + 1) (r + 1) (s + 1) − 1

C-2. N is a least natural number having 24 divisors. Then the number of ways N can be resolved into two
factors is
(A) 12 (B) 24 (C) 6 (D) None of these

RI
C-3. How many divisors of 21600 are divisible by 10 but not by 15?
(A) 10 (B) 30 (C) 40 (D) none

C-4. The number of ways in which the number 27720 can be split into two factors which are co-primes, is:
(A) 15 (B) 16 (C) 25 (D) 49

A
C-5. The number of words of 5 letters that can be made with the letters of the word "PROPOSITION".
(A) 6890 (B) 7000 (C) 6800 (D) 6900

UH
C-6. Let fruits of same kind are identical then how many ways can atleast 2 fruit be selected out of 5
Mangoes, 4 Apples, 3 Bananas and three different fruits.
(A) 959 (B) 953 (C) 960 (D) 954
JA
Section (D) : Multinomial theorem and Dearrangement
D-1. The number of ways in which 10 identical apples can be distributed among 6 children so that each child
receives atleast one apple is :
(A) 126 (B) 252 (C) 378 (D) none of these
LP

D-2. Number of ways in which 3 persons throw a normal die to have a total score of 11, is
(A) 27 (B) 25 (C) 29 (D) 18

D-3. If chocolates of a particular brand are all identical then the number of ways in which we can choose
6 chocolates out of 8 different brands available in the market, is:.
A

(A) 13C6 (B) 13C8 (C) 86 (D) none

D-4. Number of positive integral solutions of x1 . x2 . x3 = 30, is


NK

(A) 25 (B) 26 (C) 27 (D) 28

D-5. There are six letters L1, L2, L3, L4, L5, L6 and their corresponding six envelopes E1, E2, E3,
E4, E5, E6. Letters having odd value can be put into odd value envelopes and even value
letters can be put into even value envelopes, so that no letter go into the right envelopes, then
SA

number of arrangement equals.


(A) 6 (B) 9 (C) 44 (D) 4

D-6. Seven cards and seven envelopes are numbered 1, 2, 3, 4, 5, 6, 7 and cards are to be placed in
envelopes so that each envelope contains exactly one card and no card is placed in the envelope
bearing the same number and moreover the card number 1 is always placed in envelope number 2 and
2 is always placed in envelope numbered 3, then the number of ways it can be done is
(A) 53 (B) 44 (C) 9 (D) 62

Section (E) : Miscellaneous


E-1. The number of ways of choosing triplets (x, y, z) such that z ≥ max {x, y} and
x, y, z ∈ {1, 2, 3 ......, n} is
n
 n(n + 1)2 
(A) ∑t
t =1
2
(B) n+1C3 – n+2C3 (D) 
(C) 2 (n+2C3) + n+1C2

 2


E-2. The streets of a city are arranged like the lines of a chess board. There are m streets running North to
South & 'n' streets running East to West. The number of ways in which a man can travel from NW to SE
corner going the shortest possible distance is:
(m + n) ! (m + n − 2) !
(A) m2 + n2 (B) (m − 1)2 . (n − 1)2 (C) (D)
m! . n! (m − 1) ! . (n − 1) !

E-3. Number of ways of selecting pair of black squares in chessboard such that they have exactly one
common corner is equal to :
(A) 64 (B) 56 (C) 49 (D) 50

RI
PART - III : MATCH THE COLUMN

A
1. Match the column
Column – I Column – II
(A) The total number of selections of fruits which can be made (p) 120

UH
from, 3 bananas, 4 apples and 2 oranges is, it is given that
fruits of one kind are identical
(B) There are 10 true-false statements in a question paper. (q) 286
How many sequences of answers are possible in which exactly
three are correct ?
JA
(C) The number of ways of selecting 10 balls from unlimited (r) 59
number of red, black, white and green balls is, it is given
that balls of same colours are identical
(D) The number of words which can be made from the letters of the (s) 75600
word ‘MATHEMATICS’ so that consonants occur together ?
LP

2. Match the column


Column-I Column-II
(A) There are 12 points in a plane of which 5 are collinear. (p) 185
The maximum number of distinct convex quadrilaterals which
can be formed with vertices at these points is:
A

(B) If 7 points out of 12 are in the same straight line, then (q) 420
the number of triangles formed is
NK

(C) If AB and AC be two line segemets and there are 5, 4 points on (r) 126
AB and AC (other than A), then the number of quadrilateral, with
vertices on these points equals

(D) The maximum number of points of intersection of 8 unequal (s) 60


circles and 4 straight lines.
SA
 Marked questions are recommended for Revision.

PART - I : ONLY ONE OPTION CORRECT TYPE


1. A train is going from London to Cambridge stops at 12 intermediate stations. 75 persons enter the train
after London with 75 different tickets of the same class. Number of different sets of tickets they may be
holding is:
(A) 78C3 (B) 91C75 (C) 84C75 (D) 78C74

2. A family consists of a grandfather, m sons and daughters and 2n grand children. They are to be seated
in a row for dinner. The grand children wish to occupy the n seats at each end and the grandfather
refuses to have a grand children on either side of him. In how many ways can the family be made to sit.

RI
(A) (2n)! m! (m − 1) (B) (2n)! m! m (C) (2n)! (m – 1)! (m − 1) (D) (2n – 1)! m! (m − 1)
3. A bouquet from 11 different flowers is to be made so that it contains not less than three flowers. Then
then number of different ways of selecting flowers to form the bouquet.

A
(A) 1972 (B) 1952 (C) 1981 (D) 1947

4. If α = x1 x2 x3 and β = y1 y2 y3 be two three digit numbers, then the number of pairs of α and β that can
be formed so that α can be subtracted from β without borrowing.

UH
(A) 55 . (45)2 (B) 45 . (55)2 (C) 36 . (45)2 (D) 553

5. 'n' digits positive integers formed such that each digit is 1, 2, or 3. How many of these contain all three
of the digits 1, 2 and 3 atleast once ?
(A) 3(n –1) (B) 3n – 2.2n + 3 (C) 3n – 3.2n – 3 (D) 3n – 3.2n + 3
JA
6. There are ' n ' straight line in a plane, no two of which are parallel and no three pass through the same
point. Their points of intersection are joined. Then the maximum number of fresh lines thus introduced
is
(A) 1 n (n − 1)2 (n − 3) (B) 1 n (n − 1) (n + 2) (n − 3)
12 8
(C) 1 n (n − 1) (n − 2) (n − 3) (D) 1 n (n + 1) (n + 2) (n − 3)
LP

8 8

7. X = {1, 2, 3, 4, ...... 2017} and A ⊂ X ; B ⊂ X ; A ∪ B ⊂ X here P ⊂ Q denotes that P is subset of


Q(P ≠ Q). Then number of ways of selecting unordered pair of sets A and B such that A ∪ B ⊂ X.
A

(42017 − 32017 ) + (22017 − 1) (42017 − 32017 )


(A) (B)
2 2
4 2017
−3 2017
+ 22017
(C) (D) None of these
NK

8. The number of ways in which 15 identical apples & 10 identical oranges can be distributed among three
persons, each receiving none, one or more is:
(A) 5670 (B) 7200 (C) 8976 (D) 7296
SA

9. Two variants of a test paper are distributed among 12 students. Number of ways of seating of the
students in two rows so that the students sitting side by side do not have identical papers & those
sitting in the same column have the same paper is:
12! (12)!
(A) (B) 5 (C) (6 !)2. 2 (D) 12 ! × 2
6! 6! 2 . 6!

10. How many ways are there to invite one of three friends for dinner on 6 successive nights such that no
friend is invited more than three times ?
6 × 6! 6! 6! 6 × 6! 6! 6!
(A) + 3× + (B) + 6× +
1!2!3! 3!3! 2!2!2! 1!2!3! 3!3! 2!2!2!
6 × 6! 6! 6! 3 × 6! 6! 6!
(C) + + (D) + 3× +
1!2!3! 3!3! 2!2!2! 1!2!3! 3!3! 2!2!2!

11. If n identical dice are rolled, then number of possible out comes are.
6n (n + 5)
(A) 6n (B) (C) c5 (D) None of these
n!

12. Number of ways in which a pack of 52 playing cards be distributed equally among four players so that
each have the Ace, King, Queen and Jack of the same suit, is
36 ! . 4 ! 36 ! 52 ! . 4 ! 52 !
(A) 4
(B) 4
(C) 4
(D)
( 9 !) ( 9 !) (13 !) (13 !)4
13. Find total number of positive integral solutions of 15 < x 1 + x2 + x3 ≤ 20.
(A) 685 (B) 1140 (C) 455 (D) 1595

14. Seven person P1, P2, ........., P7 initially seated at chairs C1, C2 ........ ,C7 respectively. They all left their
chairs symultaneously for hand wash. Now in how many ways they can again take seats such that no

RI
one sits on his own seat and P1 sits on C2 and P2 sits on C3 ?
(A) 52 (B) 53 (C) 54 (D) 55

15. Given six line segments of length 2, 3, 4, 5, 6, 7 units, the number of triangles that can be formed by

A
these segments is
(A) 6C3 – 7 (B) 6C3 – 6 (C) 6C3 – 5 (D) 6C3 – 4

UH
16. There are m apples and n oranges to be placed in a line such that the two extreme fruits being both
oranges. Let P denotes the number of arrangements if the fruits of the same species are different and
Q the corresponding figure when the fruits of the same species are alike, then the ratio P/Q has the
value equal to :
(A) nP2. mPm. (n − 2) ! (B) mP2. nPn. (n − 2) ! (C) nP2. nPn. (m − 2) ! (D) none
JA
17. The number of intersection points of diagonals of 2009 sides regular polygon, which lie inside the
polygon.
(A) 2009C4 (B) 2009C2 (C) 2008C4 (D) 2008C2

18. A rectangle with sides 2m – 1 and 2n – 1 is divided into squares of unit length by drawing parallel lines
as shown in the diagram, then the number of rectangles possible with odd side lengths is
LP
A
NK

(A) (m + n – 1)2 (B) 4m+n–1 (C) m2 n2 (D) m(m + 1)n(n + 1)

m
19. Find the number of all rational number such that
n
m
(i) 0 < < 1, (ii) m and n are relatively prime (iii) m n = 25!
SA

n
(A) 256 (B) 128 (C) 512 (D) None of these

PART - II : SINGLE AND DOUBLE VALUE INTEGER TYPE


1. Number of five digits numbers divisible by 3 that can be formed using the digits 0, 1, 2, 3, 4, 7 and 8 if,
N
each digit is to be used atmost one is N then   is equal to
8

2. The sides AB, BC & CA of a triangle ABC have 3, 4 & 5 interior points respectively on them. If the
k
number of triangles that can be constructed using these interior points as vertices is k, then   is
5
equal to
3. Shubham has to make a telephone call to his friend Nisheeth, Unfortunately he does not remember the
7 digit phone number. But he remembers that the first three digits are 635 or 674, the number is odd
and there is exactly one 9 in the number. The maximum number of trials that Shubham has to make to
be successful is N then (N–3400) is equal to

4. Seven different coins are to be divided amongst three persons. If no two of the persons receive the
same number of coins but each receives atleast one coin & none is left over, then the number of ways
 k 
in which the division may be made is k, then   is equal to.
 10 

5. Number of ways in which five vowels of English alphabets and ten decimal digits can be placed
in a row such that between any two vowels odd number of digits are placed and both end
places are occupied by vowels is 20(b!)(5!) then b equals to

RI
6. The number of integers which lie between 1 and 106 and which have the sum of the digits equal to 12 is
N then (N–6000) is equal to

7. The number of ways in which 8 non-identical apples can be distributed among 3 boys such that every

A
 N 
boy should get atleast 1 apple & atmost 4 apples is N then   is equal to
 60 

UH
8. In a hockey series between team X and Y, they decide to play till a team wins ‘10’ match. Then the
20
Cm
number of ways in which team X wins is then m is equal to
2

9. Three ladies have brought one chlid each for admission to a school. The principal wants to
JA
interview the six persons one by one subject to the condition that no mother is interviewed
before her chlid. Then find the number of ways in which interviews can be arranged

10. In a shooting competition a man can score 0, 2 or 4 points for each shot. Then the number of different
ways in which he can score 14 points in 5 shots is

11. Six persons A, B, C, D, E and F are to be seated at a circular table. The number of ways this can be
LP

done if A must have either B or C on his right and B must have either C or D on his right is

12. The number of permutations which can be formed out of the letters of the word "SERIES" taking three
letters together is
A

13. A box contains 6 balls which may be all of different colours or three each of two colours or two each of
three different colours. The number of ways of selecting 3 balls from the box (if ball of same colour are
identical) is
NK

14. Five friends F1, F2, F3, F4, F5 book five seats C1, C2 , C3 ,C4 , C5 respectively of movie KABIL
independently (i.e. F1 books C1, F2 books C2 and so on). In how many different ways can they sit on
these seats if no one wants to sit on his booked seat, more over F 1 and F2 want to sit adjacent to each
other.
SA

15. The number of ways in which 5 X's can be placed in the squares of the figure so that no row remains
empty is:

 N 
 1111110  is
 
equal to
17. Six married couple are sitting in a room. Number of ways in which 4 people can be selected so that
there is exactly one married couple among the four is N then (N–225) is equal to

18. Let Pn denotes the number of ways of selecting 3 people out of ' n ' sitting in a row, if no two of them are
consecutive and Qn is the corresponding figure when they are in a circle. If Pn − Qn = 6, then ' n ' is equal
to:

19. The number of ways selecting 8 books from a library which has 10 books each of Mathematics,
Physics, Chemistry and English, if books of the same subject are alike, is (N 2 – 4) then N is equal to

20. The number of three digit numbers of the form xyz such that x < y and z ≤ y is N then (N–225) is equal
to

PART - III : ONE OR MORE THAN ONE OPTIONS CORRECT TYPE

RI
1. In an examination, a candidate is required to pass in all the four subjects he is studying. The number of
ways in which he can fail is
(A) 4P1 + 4P2 + 4P3 + 4P4 (B) 44 – 1

A
(C) 2 – 1
4
(D) 4C1 + 4C2 + 4C3 + 4C4

UH
2. The kindergarten teacher has 25 kids in her class. She takes 5 of them at a time, to zoological garden
as often as she can, without taking the same 5 kids more than once. Then the number of visits, the
teacher makes to the garden exceeds that of a kid by:
(A) 25C5 − 24C4 (B) 24C5 (C) 25C5 − 24C5 (D) 24C4

3. A student has to answer 10 out of 13 questions in an examination. The number of ways in which he can
JA
answer if he must answer atleast 3 of the first five questions is:
(A) 276 (B) 267
(C) 13C10 – 5C3 (D) 5C3 . 8C7 + 5C4 . 8C6 + 8C5

4. Number of ways in which 3 different numbers in A.P. can be selected from 1, 2, 3,...... n is:
(n − 2)(n − 4) n2 − 4n + 5
(A) if n is even (B) if n is odd
LP

4 2

(C)
(n − 1)2 if n is odd (D)
n (n − 2)
if n is even
4 4

5. 2m white identical coins and 2n red identical coins are arranged in a straight line with (m + n) identical
A

coins on each side of a central mark. The number of ways of arranging the identical coins , so that the
arrangements are symmetrical with respect to the central mark.
(A) m+nCm (B) m+nCn (C) m+nC|m–n| (D) m+nC|n–m|
NK

6. The number of ways in which 10 students can be divided into three teams, one containing 4 and others
3 each, is
10 ! 10 ! 1
(A) (B) 2100 (C) 10C4 . 5C3 (D) .
4 ! 3 ! 3 ! 6 ! 3 ! 3 ! 2
SA

7. If all the letters of the word 'AGAIN' are arranged in all possible ways & put in dictionary order, then
(A) The 50th word is NAAIG (B) The 49th word is NAAGI
(C) The 51 word is NAGAI
st
(D) The 47th word is INAGA

8. You are given 8 balls of different colour (black, white,...). The number of ways in which these balls can
be arranged in a row so that the two balls of particular colour (say red & white) may never come
together is:
(A) 8 ! − 2.7 ! (B) 6. 7 ! (C) 2. 6 !. 7C2 (D) none

9. Consider the word 'MULTIPLE' then in how many other ways can the letters of the word 'MULTIPLE' be
arranged ;
(A) without changing the order of the vowels equals 3359
(B) keeping the position of each vowel fixed equals 59
(C) without changing the relative order/position of vowels & consonants is 359
(D) using all the letters equals 4⋅7! – 1
10. The number of ways of arranging the letters AAAAA, BBB, CCC, D, EE & F in a row if the letter C are
separated from one another is:
12! 13! 14! 13!
(A) 13C3. (B) (C) (D) 11.
5! 3! 2! 5 ! 3! 3 ! 2 ! 3 ! 3! 2 ! 6!

11. The number of non-negative integral solutions of x1 + x2 + x3 + x4 ≤ n (where n is a positive integer) is


(A) n+3C3 (B) n+4C4 (C) n+5C5 (D) n+4Cn

12. There are 10 seats in the first row of a theatre of which 4 are to be occupied. The number of ways of
arranging 4 persons so that no two persons sit side by side is:
(A) 7C4 (B) 4. 7P3 (C) 7C3. 4 ! (D) 840

RI
13. C36 is divisible by
50

(A) 19 (B) 52 (C) 192 (D) 53

14. Pn is equal to
2n

A
(A) (n + 1) ( n + 2) ..... (2n) (B) 2n [1 . 3 . 5 .....(2n – 1)]
(C) (2) . (6) . (10) .... (4n – 2) (D) n! (2nCn)

UH
15. The number of ways in which 200 different things can be divided into groups of 100 pairs, is:
200 !  101   102   103   200 
(A) 100 (B)    2   2  ....  2 
2  2     
200 !
(C) 100
(D) (1. 3. 5...... 199)
2 (100) !
JA
PART - IV : COMPREHENSION
Comprehension # 1
There are 8 official and 4 non-official members, out of these 12 members a committee of 5 members is
to be formed, then answer the following questions.
LP

1. Number of committees consisting of at least two non-official members, are


(A) 456 (B) 546 (C) 654 (D) 466

2. Number of committees in which a particular official member is never included, are


(A) 264 (B) 642 (C) 266 (D) 462
A

Comprehenssion # 2
Let n be the number of ways in which the letters of the word "RESONANCE" can be arranged so that
NK

vowels appear at the even places and m be the number of ways in which "RESONANCE" can be
arrange so that letters R, S, O, A, appear in the order same as in the word RESONANCE, then answer
the following questions.
3. The value of n is
(A) 360 (B) 720 (C) 240 (D) 840
SA

4. The value of m is
(A) 3780 (B) 3870 (C) 3670 (D) 3760

Comprehension # 3

A mega pizza is to be sliced n times, and S n denotes maximum possible number of pieces.

5. Relation between Sn & Sn– 1


(A) Sn = Sn–1 + n + 3 (B) Sn = Sn–1 + n + 2 (C) Sn = Sn–1 + n + 2 (D) Sn = Sn–1 + n

6. If the mega pizza is to be distributed among 60 person, each one of them get atleast one piece then
minimum number of ways of slicing the mega pizza is :
(A) 10 (B) 9 (C) 8 (D) 11
 Marked questions are recommended for Revision.

PART - I : JEE (ADVANCED) / IIT-JEE PROBLEMS (PREVIOUS YEARS)

* Marked Questions may have more than one correct option.


1. The number of seven digit integers, with sum of the digits equal to 10 and formed by using the digits 1,
2 and 3 only, is [IIT-JEE-2009, Paper-I, (3, – 1), 240]
(A) 55 (B) 66 (C) 77 (D) 88

2. Let S = {1, 2, 3, 4}. The total number of unordered pairs of disjoint subsets of S is equal to

RI
(A) 25 (B) 34 (C) 42 (D) 41
[IIT-JEE-2010, Paper-2, (5, –2), 79]

3. The total number of ways in which 5 balls of different colours can be distributed among 3 persons so

A
that each person gets at least one ball is [IIT-JEE 2012, Paper-1, (3, –1), 70]
(A) 75 (B) 150 (C) 210 (D) 243
Paragraph for Question Nos. 4 to 5

UH
Let an denote the number of all n-digit positive integers formed by the digits 0,1 or both such that no
consecutive digits in them are 0. Let bn = the number of such n-digit integers ending with digit 1 and
cn = the number of such n-digit integers ending with digit 0. [IIT-JEE 2012, Paper-2, (3, –1), 66]

4. Which of the following is correct ?


JA
(A) a17 = a16 + a15 (B) c17 ≠ c16 + c15 (C) b17 ≠ b16 + c16 (D) a17 = c17 + b16

5. The value of b6 is


(A) 7 (B) 8 (C) 9 (D) 11

6. Let n1 < n2 < n3 < n4 < n5 be positive integers such that n1 + n2 + n3 + n4 + n5 = 20. Then the number of
LP

such distinct arrangements (n1, n2, n3, n4, n5) is


[JEE (Advanced) 2014, Paper-1, (3, 0)/60]

7. Let n ≥ 2 be an integer. Take n distinct points on a circle and join each pair of points by a line segment.
Colour the line segment joining every pair of adjacent points by blue and the rest by red. If the number
A

of red and blue line segments are equal, then the value of n is
[JEE (Advanced) 2014, Paper-1, (3, 0)/60]

8. Six cards and six envelopes are numbered 1, 2, 3, 4, 5, 6 and cards are to be placed in envelopes so
NK

that each envelope contains exactly one card and no card is placed in the envelope bearing the same
number and moreover the card numbered 1 is always placed in envelope numbered 2. Then the
number of ways it can be done is [JEE (Advanced) 2014, Paper-2, (3, –1)/60]
(A) 264 (B) 265 (C) 53 (D) 67
SA

9. Let n be the number of ways in which 5 boys and 5 girls can stand in a queue in such a way that all the
girls stand consecutively in the queue. Let m be the number of ways in which 5 boys and 5 girls can
stand in a queue in such a way that exactly four girls stand consecutively in the queue. Then the value
m
of is [JEE (Advanced) 2015, P-1 (4, 0) /88]
n

10. A debate club consists of 6 girls and 4 boys. A team of 4 members is to be selected from this club
including the selection of a captain (from among these 4 members) for the team. If the team has to
include at most one boy. Then the number of ways of selecting the team is
[JEE (Advanced) 2016, Paper-1, (3, –1)/62]
(A) 380 (B) 320 (C) 260 (D) 95
11. Words of length 10 are formed using the letters A, B, C, D, E, F, G, H, I, J. Let x be the number of such
words where no letter is repeated; and let y be the number of such words where exactly one letter is
y
repeated twice and no other letter is repeated. Then, = [JEE(Advanced) 2017, Paper-1,(3, 0)/61]
9x

12. Let S = {1, 2, 3, ......, 9}. For k = 1, 2,......,5, let N k be the number of subsets of S, each containing five
elements out of which exactly k are odd. Then N1 + N2 + N3 + N4 + N5 =
[JEE(Advanced) 2017, Paper-2,(3, –1)/61]
(A) 210 (B) 252 (C) 126 (D) 125

13. The number of 5 digit numbers which are divisible by 4, with digits from the set {1, 2, 3, 4, 5} and the
repetition of digits is allowed, is _______. [JEE(Advanced) 2018, Paper-1,(3, 0)/60]

14. In a high school, a committee has to be formed from a group of 6 boys M 1, M2, M3, M4, M5, M6 and 5

RI
girls G1, G2, G3, G4, G5. [JEE(Advanced) 2018, Paper-2,(3, –1)/60]
(i) Let α1 be the total number of ways in which the committee can be formed such that the committee
has 5 members, having exactly 3 boys and 2 girls.
(ii) Let α2 be the total number of ways in which the committee can be formed such that the committee

A
has at least 2 members, and having an equal number of boys and girls.
(iii) Let α3 be the total number of ways in which the committee can be formed such that the committee
has 5 members, at least 2 of them being girls.
(iv) Let α4 be the total number of ways in which the committee can be formed such that the committee

UH
has 4 members, having at least 2 girls and such that both M1 and G1 are NOT in the committee
together.
LIST-I LIST-II
(P) The value of α1 is (1) 136
(Q) The value of α2 is (2) 189
JA
(R) The value of α3 is (3) 192
(S) The value of α4 is (4) 200
(5) 381
(6) 461
The correct option is
(A) P → 4; Q → 6; R → 2; S → 1
(B) P → 1; Q → 4; R → 2; S → 3
LP

(C) P → 4; Q → 6; R → 5; S → 2
(D) P → 4; Q → 2; R → 3; S → 1

PART - II : JEE (MAIN) / AIEEE PROBLEMS (PREVIOUS YEARS)


A

1. From 6 different novels and 3 different dictionaries, 4 novels and 1 dictionary are to be selected and
arranged in a row on the shelf so that the dictionary is always in the middle. Then the number of such
arrangements is- [AIEEE 2009 (4, –1), 144]
NK

(1) atleast 500 but less than 750 (2) atleast 750 but less than 1000
(3) atleast 1000 (4) less than 500

2. Statement-1 : The number of ways of distributing 10 identical balls in 4 distinct boxes such that no box
is empty is 9 C3 . [AIEEE 2011, I, (4, –1), 120]
SA

Statement-2 : The number of ways of choosing any 3 places from 9 different places is 9 C3 .
(1) Statement-1 is true, Statement-2 is true; Statement-2 is a correct explanation for Statement-1.
(2) Statement-1 is true, Statement-2 is true; Statement-2 is not a correct explanation for Statement-1.
(3) Statement-1 is true, Statement-2 is false.
(4) Statement-1 is false, Statement-2 is true.
3. There are 10 points in a plane, out of these 6 are collinear. If N is the number of triangles formed by
joining these points. then : [AIEEE 2011, II, (4, –1), 120]
(1) N ≤ 100 (2) 100 < N ≤ 140 (3) 140 < N ≤ 190 (4) N > 190

4. Assuming the balls to be identical except for difference in colours, the number of ways in which one or
more balls can be selected from 10 white, 9 green and 7 black balls is : [AIEEE-2012, (4, –1)/120]
(1) 880 (2) 629 (3) 630 (4) 879

5. Let Tn be the number of all possible triangles formed by joining vertices of an n-sided regular polygon. If
Tn+1 – Tn = 10, then the value of n is : [AIEEE - 2013, (4, –1),360]
(1) 7 (2) 5 (3) 10 (4) 8

6. The number of integers greater than 6,000 that can be formed, using the digits 3, 5, 6, 7 and 8, without
repetition, is : [JEE(Main)2015,(4, – 1), 120]
(1) 216 (2) 192 (3) 120 (4) 72

7. If all the words (with or without meaning) having five letters, formed using the letters of the word SMALL
and arranged as in a dictionary; then the position of the word SMALL is :
[JEE(Main)2016,(4, – 1), 120]

8. A man X has 7 friends, 4 of them are ladies and 3 are men. His wife Y also has 7 friends, 3 of them are
ladies and 4 are men. Assume X and Y have no common friends. Then the total number of ways in
which X and Y together can throw a party inviting 3 ladies and 3 men, so that 3 friends of each of X and
Y are in this party, is [JEE(Main)2017,(4, – 1), 120]

RI
(1) 485 (2) 468 (3) 469 (4) 484

9. From 6 different novels and 3 different dictionaries, 4 novels and 1 dictionary are to be selected and
arranged in a row on a shelf so that the dictionary is always in the middle. The number of such

A
arrangements is : [JEE(Main)2018,(4, – 1), 120]
(1) at least 500 but less than 750 (2) at least 750 but less than 1000
(3) at least 1000 (4) less than 500

UH
10. Let S be the set of all triangles in the xy-plane, each having one vertex at the origin and the other two
vertices lie on coordinate axes with integral coordinates. If each triangle in S has area 50 sq. units, then
the number of elements in the set S is : [JEE(Main) 2019, Online (09-01-19),P-2 (4, – 1), 120]
(1) 32 (2) 36 (3) 18 (4) 9

11. Consider three boxes, each containing 10 balls labelled 1,2,....,10. Suppose one ball is randomly drawn
JA
from each of the boxes. Denote by ni, the label of the ball drawn from the ith box, (i = 1, 2, 3). Then, the
number of ways in which the balls can be chosen such that n1 < n2 < n3 is :
[JEE(Main) 2019, Online (12-01-19),P-1 (4, – 1), 120]
(1) 120 (2) 164 (3) 240 (4) 82

12. Let S = {1, 2, 3....., 100}. The number of non-empty subsets A of S such that the product of element in
LP

A is even is : [JEE(Main) 2019, Online (12-01-19),P-1 (4, – 1), 120]

(1) 250 + 1 (2) 250(250–1) (3) 2100 – 1 (4) 250 –1


A
NK
SA
EXERCISE - 1

PART - I
Section (A) :
A-1. (i) 9 (ii) 20 (iii) 72 (iv) 326592 A-2. 90
A-3. (i) 40320 (ii) 37440 (iii) 2880 (iv) 1152 (v) 1152
A-4. (i) 4320 (ii) 720 (iii) 4320 (iv) 14400 (v) 14400 (vi) 2880

RI
(vii) 6720 (viii) 720 (ix) 3600
A-5. 5: 2 A-6. 200 A-7. 50 A-8. 7. 7P3 A-9. 10
P4 A-10. 738 A-11. 9999
A-12. 65 A-13. 1170 A-14. 154 A-15. 886656 A-16. 229 A-17. 7350

A
3n – 1
A-18. (i) 4n (ii) 3n (iii) +1 (iv) 2n (v) 2n–1 (vi) nC1 . 3n–1
2

UH
A-19. 63 A-20. 20 A-21. 62784 A-22. 280
Section (B) :
18!
B-1. 4
B-2. 360360 B-3. 70 B-4. (a) 25 (b) 150. (c) 270000
(3!) . (2!)3 4! 3!
JA
B-6. 12 B-7. 144 B-8. 48 B-9. 1
Section (C) :
C-1. (i) 18 (ii) 23 (iii) 4
C-2. (i) 479 (ii) 256 (iii) 6 (iv) 473 C-3. 45 C-4. 36
LP

Section (D) :
D-1. 55 D-2. 37 D-3. (i) 23C3 (ii) 19C3 (iii) 19C3 – 4.9C3 (iv) 11C8 D-4. 9C – 4 × 5C3 = 44
3

D-5. (i) 150, (ii) 6, (iii) 25, (iv) 2 D-6. 1 D-7. (a) 44 (b) 109
A

Section (E) :
E-1. (i) 8 (ii) 10 E-2. 34C + 33C2 + 33C2 + 34C1 . 33C1 E-3. 670
2
NK

E-4. (i) 36 (ii) 1968 (iii) 1296


E-5. 126

PART - II
SA

Section (A) :
A-1. (C) A-2. (D) A-3. (D) A-4. (B) A-5. (A) A-6. (B) A-7 (B)
A-8. (C) A-9. (A) A-10. (D) A-11. (C) A-12. (C) A-13. (D) A-14. (A)
A-15. (B) A-16. (C) A-17. (A) A-18. (A) A-19. (A) A-20. (C)
Section (B) :
B-1. (C) B-2. (D) B-3. (C) B-4. (C) B-5. (A) B-6. (A) B-7. (B)
Section (C) :
C-1. (D) C-2. (A) C-3. (A) C-4. (B) C-5. (A) C-6. (B)
Section (D) :
D-1. (A) D-2. (A) D-3. (A) D-4. (C) D-5. (D) D-6. (A)
Section (E) :
E-1. (A) E-2. (D) E-3. (C)

PART – III
1. (A) → (r), (B) → (p), (C) → (q), (D) → (s) 2. (A) - (q) ; (B) - (p) ; (C) - (s) ; (D) - (r)

EXERCISE - 2
PART – I

1. (A) 2. (A) 3. (C) 4. (B) 5. (D) 6. (C) 7. (A)

RI
8. (C) 9. (D) 10. (A) 11. (C) 12. (A) 13. (A) 14. (B)
15. (A) 16. (A) 17. (A) 18. (C) 19. (A)

A
PART - II
1. 93 2. 41 3. 2 4. 63 5. 10 6. 62 7. 77

UH
8. 10 9. 90 10. 30 11. 18 12. 42 13. 31 14. 21
15. 98 16. 20 17. 15 18. 10 19. 13 20. 51

PART - III
JA
1. (CD) 2. (AB) 3. (ACD) 4. (CD) 5. (AB) 6. (BC)
7. (ABCD) 8. (ABC) 9. (ABCD) 10. (AD) 11. (BD) 12. (BCD)
13. (AB) 14. (ABCD) 15. (BCD)

PART - IV
1. (A) 2. (D) 3. (B) 4. (A) 5. (D) 6. (D)
LP

EXERCISE - 3
A

PART - I
1. (C) 2. (D) 3. (B) 4. (A) 5. (B) 6. 7 7. 5
NK

8. (C) 9. 5 10. (A) 11. 5 12. (C) 13. 625 14. (C)

PART - II
1. (3) 2. (1) 3. (1) 4. (4) 5. (2) 6. (2) 7. (3)
SA

8. (1) 9. (3) 10. (2) 11. (1) 12. (2)

1. How many positive integers are there such that n is a divisor of one of the numbers 10 40, 2030?

2. Six cards are drawn one by one from a set of unlimited number of cards, each card is marked with
numbers − 1, 0 or 1. Number of different ways in which they can be drawn if the sum of the numbers
shown by them vanishes, is:

3. A five letter word is to be formed such that the letters appearing in the odd numbered positions are
taken from the letters which appear without repetition in the word "MATHEMATICS". Further the letters
appearing in the even numbered positions are taken from the letters which appear with repetition in the
same word "MATHEMATICS". The number of ways in which the five letter word can be formed is:

4. In how many ways 4 square are can be chosen on a chess-board, such that all the squares lie in a
diagonal line.

5. Find the number of functions f : A → B where n(A) = m , n(B) = t , which are non decreasing,

6. Find the number of ways of selecting 3 vertices from a regular polygon of sides ‘2n+1’ with vertices
A1, A2, A3 ,......, A2n+1 such that centre of polygon lie inside the triangle.

7. A operation * on a set A is said to be binary, if x * y ∈ A , for all x, y ∈ A, and it is said to be

RI
commutative ,
if x * y = y * x for all x, y ∈ A. Now if A = {a1, a2, ......, an}, then find the following -
(i) Total number of binary operations of A
(ii) Total number of binary operation on A such that

A
ai * aj ≠ ai * ak, if j ≠ k.
(iii) Total number of binary operations on A such that a i * aj < ai * aj + 1 ∀ i, j

UH
8. The integers from 1 to 1000 are written in order around a circle. Starting at 1, every fifteenth number is
marked (that is 1, 16, 31, .... etc.). This process in continued untill a number is reached which has
already been marked, then find number of unmarked numbers.

9.
JA
Find the number of ways in which n ‘1’ and n ‘2’ can be arranged in a row so that upto any point in the
row no. of ‘1’ is more than or equal to no. of ‘2’

10. Find the number of positive integers less than 2310 which are relatively prime with 2310.

11. In maths paper there is a question on "Match the column" in which column A contains 6 entries & each
LP

entry of column A corresponds to exactly one of the 6 entries given in column B (and vice versa) written
randomly. 2 marks are awarded for each correct matching & 1 mark is deducted from each incorrect
matching. A student having no subjective knowledge decides to match all the 6 entries randomly. Find
the number of ways in which he can answer, to get atleast 25 % marks in this question.
A

12. Find the number of positive unequal integral solution of the equation x + y + z = 20.

13. If we have 3 identical white flowers and 6m identical red flowers. Find the number of ways in which a
NK

garland can be made using all the flowers.

14. Number of times is the digit 5 written when listing all numbers from 1 to 10 5 ?
15. The number of combinations of n letters together out of 3n letters of which n are a and n are b and the
rest unlike.
SA

16. In a row, there are 81 rooms, whose door no. are 1,2,.......,81, initially all the door are closed. A person
takes 81 round of the row, numbers as 1st round, 2nd round ........ 81th round. In each round, he
interchage the position of those door number, whose number is multiple of the round number. Find out
after 81st round,How many doors will be open.

17. Mr. Sibbal walk up 16 steps, going up either 1 or 2 steps with each stride there is explosive material on
the 8th step so he cannot step there. Then number of ways in which Mr. Sibbal can go up.

18. Number of numbers of the form xxyy which are perfect squares of a natural number.

19. A batsman scores exactly a century by hitting fours and sixes in twenty consecutive balls. In how many
different ways can he hit either six or four or play a dot ball?
20. In how many ways can two distinct subsets of the set A of k(k ≥ 2) elements be selected so that they
have exactly two common elements.

21. How many 5 digit numbers can be made having exactly two identical digit.

22. Find the number of 3-digit numbers. (including all numbers) which have any one digit is the average of
the other two digits.

23. In how many ways can(2n + 1) identical balls be placed in 3 distinct boxes so that any two boxes
together will contain more balls than the third box.

24. Let f(n) denote the number of different ways in which the positive integer 'n' can be expressed as sum
of 1s and 2s.

RI
for example f(4) = 5 {2 + 2, 2 + 1 + 1, 1 + 2 + 1, 1 + 1 + 2, 1 + 1 + 1 + 1}. Now that order of 1s and 2s is
important. Then determine f(f(6))

A
25. Prove that (n!)! is divisible by (n!)(n–1)!

26. A user of facebook which is two or more days older can send a friend request to some one to join

UH
facebook.
If initially there is one user on day one then find a recurrence relation for an where an is number of users
after n days.

27. Let X = {1, 2, 3,.....,10}. Find the the number of pairs {A, B} such A⊆X. B ⊆X. A ≠ B and A ∩ B = {5,7,8}.
JA
28. Consider a 20-sided convex polygon K, with vertices A1, A2, . . . . , A20 in that order. Find the number of
ways in which three sides of K can be chosen so that every pair among them has at least two sides of
K between them. (For example (A1A2, A4A5, A11A12) is an admissible triple while (A1A2, A4A5, A19A20) is
not).
LP

29. Find the number of 4-digit numbers (in base 10) having non-zero digits and which are divisible by 4 but
not by 8.

30. Find the number of all integer-sided isosceles obtuse-angled triangles with perimeter 2008.
A

31. Let ABC be a triangle. An interior point P of ABC is said to be good if we can find exactly 27 rays
emanating from P intersecting the sides of the triangle ABC such that the triangle is divided by these
rays into 27 smaller triangles of equal area. Determine the number of good points for a given triangle
NK

ABC.

32. Let σ = (a1, a2, a3, ..., an) be a permutation of (1, 2, 3, ..., n). A pair (ai,aj) is said to correspond to an
inversion of σ, if i < j but ai > aj. (Example : In the permutation (2, 4, 5, 3, 1), there are 6 inversions
corresponding to the pairs (2, 1), (4, 3), (4, 1), (5, 3) (5, 1), (3, 1) . ) How many permutations of
SA

(1, 2, 3, ... n), (n ≥ 3), have exactly two inversions?


1. 2301 2. 141 3. 540 4. 3645. (t + m − 1)c m ways

2n
2n + 1 2n 2 Cn
6. ( C2 – 3.nC2) 7. (i) nn (ii) (n!)n (iii) 1 8. 800 9.
3 n +1

10. 480 11. 56 ways 12. 144 13. 3m2 + 3m + 1 14. 50000

15. (n + 2). 2n − 1 16. 9 17. 441 18. 1

20! 20! 20! 20! k ( k − 1)

RI
19. + + + 20. ((3)k–2 –1) 21. 45360
10! 10! 7! 12! 4! 14! 2! 16! 3! 4

n ( n + 1)
22. 121 23. 24. 377 26. a n = an – 1 + an – 2 27. 2186

A
2

26 (n + 1)(n − 2)
28. 520 29. 729 30. 86 31. C2 32.

UH
2

JA
LP
A
NK
SA
SEQUENCE & SERIES

JEE (Advanced) Syllabus

Arithmetic, geometric and harmonic progressions, arithmetic, geometric and harmonic means, sum of finite
arithmetic and geometric progressions, infinite geometric series, sums of squares and cubes of the first ‘n’
natural numbers.

JEE (Main) Syllabus

Arithmetic and Geometric progressions, insertion of arithmetic, geometric means between two given numbers.

RI
Relation between A.M. and G.M. Sum upto n terms of special series: Sn, Sn2, Sn3. Arithmetico - Geometric
progression.

"1729 is a very interesting number; it is the smallest number expressible as the sum of two cubes in two different ways." ...... S.Ramanujan

A
Sequence :
A sequence is a function whose domain is the set N of natural numbers. Since the domain for every
sequence is the set N of natural numbers, therefore a sequence is represented by its range. If

UH
f : N → R, then f(n) = tn, n ∈ N is called a sequence and is denoted by
{f(1), f(2), f(3), ...............} = {t1, t2, t3, ...............} = {tn}
Real sequence :
A sequence whose range is a subset of R is called a real sequence.
e.g. (i) 2, 5, 8, 11, .......................
JA
(ii) 4, 1, – 2, – 5, ......................
Types of sequence :
On the basis of the number of terms there are two types of sequence.
(i) Finite sequences : A sequence is said to be finite if it has finite number of terms.
(ii) Infinite sequences : A sequence is said to be infinite if it has infinitely many terms.
( −2 )n
LP

Example # 1 : Write down the sequence whose n term is th

( −1)n + 2
Solution : Let tn =
( −2 )n
( −1)n + 2
A

put n = 1, 2, 3, 4, .............. we get


4 16
t1 = –2, t2 = , t3 = –8, t4 =
NK

3 3
16
so the sequence is –2, , –8, , ........
3
Series :
By adding or subtracting the terms of a sequence, we get an expression which is called a series.
SA

If a1, a2, a3,........an is a sequence, then the expression a1 + a2 + a3 + ...... + an is a series.


e.g. (i) 1 + 2 + 3 + 4 + ............... + n
(ii) 2 + 4 + 8 + 16 + ...............
(iii) – 1 + 3 – 9 + 27 – ...............
Progression :
The word progression refers to sequence or series – finite or infinite

Arithmetic progression (A.P.) :


A.P. is a sequence whose successive terms are obtained by adding a fixed number 'd' to the preceding
terms. This fixed number 'd' is called the common difference. If a is the first term & d the common
difference, then A.P. can be written as a, a + d, a + 2 d,....... ,a + (n − 1) d,........

e.g. – 4, – 1, 2, 5 ...........
nth term of an A.P. :
Let 'a' be the first term and 'd' be the common difference of an A.P., then
tn = a + (n – 1) d, where d = tn – tn – 1

Example # 2 : Find the number of terms in the sequence 4, 7, 10, 13, ........,82.
Solution : Let a be the first term and d be the common difference
a = 4, d = 3 so 82 = 4 + (n – 1)3
⇒ n = 27

The sum of first n terms of an A.P. :


If a is first term and d is common difference, then sum of the first n terms of AP is
n
Sn = [2a + (n – 1) d]
2

RI
n
= [a +  ] ≡ nt  n +1 , for n is odd. (Where  is the last term and t  n +1 is the middle term.)
2  
 2 

 2 

Note : For any sequence {tn}, whose sum of first r terms is Sr, rth term, tr = Sr – Sr – 1.

A
Example # 3 : If in an A.P., 3rd term is 18 and 7 term is 30, then find sum of its first 17 terms

UH
Solution : Let a be the first term and d be the common difference
a + 2d = 18
a + 6d = 30
d = 3 , a = 12
17
s17 = [2 × 12 + 16 × 3] = 612
2
JA
Example # 4 : Find the sum of all odd numbers between 1 and 1000 which are divisible by 3
Solution : Odd numbers between 1 and 1000 are
3, 5, 7, 9, 11, 13, ------ 993, 995, 997, 999.
Those numbers which are divisible by 3 are
3, 9, 15, 21, ------- 993, 999
LP

They form an A.P. of which a = 3 , d = 6,  = 999 ∴ n = 167


n
S= [a + ] = 83667
2
A

Example # 5 : The ratio between the sum of n term of two A.P.’s is 3n + 8 : 7n + 15. Then find the ratio
between their 12 th term
Sn (n / 2)[2a + (n – 1)d] 3n + 8 a + {(n – 1) / 2}d 3n + 8
Solution : = = or = ----- (i)
NK

Sn ' (n / 2)[2a '+ (n – 1)d'] 7n + 15 a'+ (n – 1) / 2d' 7n + 15


T12 a + 11d
we have to find =
T12 ' a'+ 11d'
choosing (n – 1)/2 = 11 or n = 23 in (1),
T a + 11d 3(23) + 8 77 7
SA

we get 12 = = = =
T12 ' a'+ 11d' (23) × 7 + 15 176 16

Example # 6 : If sum of n terms of a sequence is given by Sn = 3n2 – 4n, find its 50th term.
Solution : Let tn is nth term of the sequence so tn = Sn – Sn – 1.
= 3n2 – 4n – 3(n – 1)2 + 4(n – 1) = 6n – 7
so t50 = 293.

Self practice problems :

(1) Which term of the sequence 2005, 2000, 1995, 1990, 1985, ............. is the first negative term
(2) For an A.P. show that tm + t2n + m = 2 tm + n
(3) Find the maximum sum of the A.P. 40 + 38 + 36 + 34 + 32 + ..............
(4) Find the sum of first 16 terms of an A.P. a1, a2, a3..........
If it is known that a1 + a4 + a7 + a10 + a13 + a16 = 147

Ans. (1) 403 (3) 420 (4) 392

Remarks :
(i) The first term and common difference can be zero, positive or negative (or any complex
number.)
(ii) If a, b, c are in A.P. ⇒ 2 b = a + c & if a, b, c, d are in A.P. ⇒ a + d = b + c.
(iii) Three numbers in A.P. can be taken as a − d, a, a + d ; four numbers in A.P. can be taken as a
− 3d, a − d, a + d, a + 3d; five numbers in A.P. are a − 2d, a − d, a, a + d, a + 2d ;
six terms in A.P. are a − 5d, a − 3d, a − d, a + d, a + 3d, a + 5d etc.
(iv) The sum of the terms of an A.P. equidistant from the beginning & end is constant and equal to

RI
the sum of first & last terms.
(v) Any term of an A.P. (except the first) is equal to half the sum of terms which are equidistant
from it. an = 1/2 (an−k + an+k), k < n. For k = 1, an = (1/2) (an−1+ an+1);
For k = 2, an = (1/2) (an−2+ an+2) and so on.

A
(vi) If each term of an A.P. is increased, decreased, multiplied or divided by the same non-zero
number, then the resulting sequence is also an AP.

UH
(vii) The sum and difference of two AP's is an AP.
t2
Example # 7 : The numbers t (t2 + 1), − and 6 are three consecutive terms of an A.P. If t be real, then find
2
the the next two term of A.P.
Solution : 2b = a + c ⇒ –t2 = t3 + t + 6
JA
or t +t +t+6=0
3 2

or (t + 2) (t2 – t + 3) = 0 ∴ t2 – t + 3 ≠ 0 ⇒ t = –2
the given numbers are – 10, – 2, 6
which are in an A.P. with d = 8. The next two numbers are 14, 22
5
Example # 8 : If a1, a2, a3, a4, a5 are in A.P. with common difference ≠ 0, then find the value of ∑ ai , when
i =1
LP

a3 = 2.
Solution : As a1, a2, a3, a4, a5 are in A.P., we have a1 + a5 = a2 + a4 = 2a3.
5
Hence ∑a
i =1
i = 10.

1 1 1
A

Example # 9 : If a(b + c), b(c + a), c(a + b) are in A.P., prove that , , are also in A.P.
a b c
Solution :  a(b + c), b(c + a), c(a + b) are in A.P. ⇒ subtract ab + bc + ca from each
– bc, – ca, – ab are in A.P.
NK

divide by –abc
1 1 1
, , are in A.P.
a b c

a+b b+c 1 1
SA

Example # 10 : If , b, are in A.P. then prove that ,b are in A.P.


1 − ab 1 − bc a c
a+b b+c
Solution : ∴ , b, are in A.P.
1 − ab 1 − bc
a+b b+c
b– = –b
1 − ab 1 − bc
−a b2 + 1
( ) (
c 1 + b2 )
=
1 − ab 1 − bc
⇒ –a + abc = c – abc
a + c = 2abc
divide by ac
1 1 1 1
+ = 2b ⇒ , b, are in A.P.
c a a c
Arithmetic mean (mean or average) (A.M.) :
If three terms are in A.P. then the middle term is called the A.M. between the other two, so if a, b, c are
in A.P., b is A.M. of a & c.
a + a + a + ..... + an
A.M. for any n numbers a1, a2,..., an is; A = 1 2 3 .
n
n - Arithmetic means between two numbers :
If a, b are any two given numbers & a, A1, A2,...., An, b are in A.P., then A1, A2,... An are the
n A.M.’s between a & b.
b−a 2 (b − a) n (b − a)
A1 = a + ,A =a+ ,......, An = a +
n+1 2 n+1 n+1

Note : Sum of n A.M.’s inserted between a & b is equal to n times the single A.M. between a & b

RI
n
a+b
i.e. ∑ Ar = nA, where A is the single A.M. between a & b i.e. A =
r =1 2

Example # 11 : If a, b, c,d,e, f are A. M’s between 2 and 12, then find a + b + c + d + e + f.

A
6(2 + 12)
Solution : Sum of A.M.s = 6 single A.M. = = 42
2

UH
Example # 12 : Insert 10 A.M. between 3 and 80.
Solution : Here 3 is the first term and 80 is the 12th term of A.P. so 80 = 3 + (11)d
⇒ d=7
so the series is 3, 10, 17, 24, ........, 73, 80
∴ required means are 10, 17, 24, ........, 73.
JA
Self practice problems :
(5) There are n A.M.’s between 3 and 29 such that 6th mean : (n – 1)th mean : : 3 : 5 then find the
value of n.
an+ 3 + bn+ 3
(6) For what value of n, n+ 2 , a ≠ b is the A.M. of a and b.
a + bn+ 2
LP

Ans. (5) n = 12 (6) n = –2

Geometric progression (G.P.) :


G.P. is a sequence of numbers whose first term is non zero & each of the succeeding terms is equal to
the preceeding terms multiplied by a constant. Thus in a G.P. the ratio of successive terms is constant.
A

This constant factor is called the common ratio of the series & is obtained by dividing any term by that
which immediately preceeds it. Therefore a, ar, ar2, ar3, ar4,...... is a G.P. with 'a' as the first term & 'r' as
common ratio.
NK

1 1 1 1
e.g. (i) 2, 4, 8, 16, ....... (ii) , , , , .......
3 9 27 81

Results : (i) nth term of GP = a rn−1


SA

(ii) Sum of the first n terms of GP


 a rn − 1
( )
 , r ≠1
Sn =  r − 1
 na
 , r =1
(iii) Sum of an infinite terms of GP when r < 1. When n → ∞, rn → 0 if r < 1 therefore,
a
S∞ =
1− r
( r < 1)

1
Example # 13 : The nth term of the series 3, 3 , 1 , ------ is , then find n
243
n–1
 1  1
Solution : 3.   = ⇒ n = 13
 3 243
Example # 14 : The first term of an infinite G.P. is 1 and any term is equal to the sum of all the succeeding
terms. Find the series.
Solution : Let the G.P. be 1, r, r2, r3, .........
r2 1
given condition ⇒ r= ⇒ r= ,
1− r 2
1 1 1
Hence series is 1, , , , ..............∞
2 4 8

Example # 15 : In a G.P.., T2 + T5 = 216 and T4 : T6 = 1 : 4 and all terms are integers, then find its first term :
ar 3 1
Solution : ar (1 + r3) = 216 and =
ar 5 4
⇒ r2 = 4 ⇒ r = ± 2

RI
when r = 2 then 2a(9) = 216 ⇒ a = 12
when r = – 2, then – 2a (1–8) = 216
216 108
∴ a= = , which is not an integer.
14 7

A
Self practice problems :
(7) Find the G.P. if the common ratio of G.P. is 3, nth term is 486 and sum of first n terms is 728.

UH
(8) If x, 2y, 3z are in A.P. where the distinct numbers x, y, z are in G.P. Then find the common ratio
of G.P.
(9) A G.P. consist of 2n terms. If the sum of the terms occupying the odd places is S 1 and that of
the terms occupying the even places is S2 , then find the common ratio of the progression.
JA
(10) If continued prodcut of three number in G.P. is 216 and sum of there product in pairs is 156.
Find the numbers.
1 S
Ans. (7) 2, 6, 18, 54, 162, 486 (8) (9) 2 .
3 S1
(10) 2, 6, 18
LP

Remarks :

(i) If a, b, c are in G.P. ⇒ b2 = ac, in general if a1, a2, a3, a4,......... an – 1 , an are in G.P.,
then a1an = a2an – 1 = a3 an – 2 = ..........................
A

a
(ii) Any three consecutive terms of a G.P. can be taken as , a , ar.
r
a a
(iii) Any four consecutive terms of a G.P. can be taken as, 3 , ar, ar3.
NK

r r

(iv) If each term of a G.P. be multiplied or divided or raised to power by the same non−zero
quantity, the resulting sequence is also a G.P..
(v) If a1, a2, a3,........ and b1, b2, b3,......... are two G.P’s with common ratio r1 and r2 respectively, then
SA

the sequence a1b1, a2b2, a3b3, ..... is also a G.P. with common ratio r1 r2.

(vi) If a1, a2, a3,..........are in G.P. where each ai > 0, then log a1, loga2, loga3,..........are in A.P. and its
converse is also true.

Example # 16 : Three numbers form an increasing G.P. If the middle number is doubled, then the new
numbers are in A.P. The common ratio of G.P. is :
a
Solution : Three number in G.P. are , a, ar
r
a
then , 2a ar are in A.P. as given.
r
 1
∴ 2(2a) = a  r + 
 r
or r2 – 4r + 1 = 0
or r=2± 3
or r=2+ 3 as r > 1 for an increasing G.P.

Example # 17 : The sum of an infinite geometric progression is 2 and the sum of the geometric progression
made from the cubes of this infinite series is 24. Then find its first term and common ratio :
Solution : Let a be the first term and r be the common ratio of G.P.
a a3
= 2, = 24 , –1 < r < 1
1– r 1– r 3
1
Solving we get a = 3, r = –
2
Example # 18 : Express 0.423  in the form of p , (where p, q ∈ Ι, q ≠ 0)
q
4 23 23 4 a 4 23 419

RI
Solution : S= + 3 + 5 + ....... ∞ = + = + =
10 10 10 10 1– r 10 990 990

Example # 19 : Evaluate 9 + 99 + 999 + ........... upto n terms.

A
Solution : Let S = 9 + 99 + 999 + ..........upto n terms.
= [9 + 99 + 999 + .......]
= [(10 – 1) + (102 – 1) + (103 – 1) + ........ + upto n terms]
 10(10n − 1) 

UH
= [10 + 102 + 103 + ...........+ 10n – n] =  − n
 9 
Geometric means (mean proportional) (G.M.):
If a, b, c are in G.P., b is called as the G.M. of a & c.
If a and c are both positive, then b = ac and if a and c are both negative, then b = – ac .
JA
b² = ac, therefore b = ac ; a > 0, c > 0.
n-Geometric means between a, b :
If a, b are two given numbers & a, G1, G2,....., Gn, b are in G.P.. Then
G1, G2, G3,...., Gn are n G.M.s between a & b.
LP

G1 = a(b/a)1/n+1, G2 = a(b/a)2/n+1,......, Gn = a(b/a)n/n+1


Note : The product of n G.M.s between a & b is equal to the nth power of the single G.M. between a & b
n


n
i.e.
r =1
Gr = ( ab ) = Gn , where G is the single G.M. between a & b.
A

Example # 20 : Between 4 and 2916 are inserted odd number (2n + 1) G.M’s. Then the (n + 1)th G.M. is
Solution : 4, G1.G2, ....... Gn+1, .... G2n, G2n+1, 2916
Gn+1 will be the middle mean of (2n +1) odd means and it will be equidistant from Ist and last
NK

term
∴ 4,Gn+1 , 2916 will also be in G.P.
∴ Gn2+1 = 4 × 2916 = 4 × 9 × 324 = 4 × 9 × 4 × 81
Gn+1 = 2 × 3 × 2 × 9 = 108.
Self practice problems :
SA

a n +1 + b n + 1
(11) Find the value of n so that may be the G.M. between a and b.
an + bn
(12) If a = 111 ..........1, b = 1 + 10 + 102 + 103 + 104 and c = 1 + 105 + 1010 + ..... + 1050, then prove
  
55

that
(i) ‘a’ is a composite number (ii) a = bc.
1
Ans. (11) n=–
2
Harmonic progression (H.P.)
A sequence is said to be in H.P if the reciprocals of its terms are in A.P.. If the sequence a 1, a2, a3,....,
an is in H.P. then 1/a1, 1/a2,...., 1/an is in A.P.
Note : (i) Here we do not have the formula for the sum of the n terms of an H.P.. For H.P. whose first
ab
term is a and second term is b, the nth term is tn = .
b + (n − 1)(a − b)
2ac a a−b
(ii) If a, b, c are in H.P. ⇒ b = or = .
a+c c b−c
a−b a
(iii) If a, b, c are in A.P. ⇒ =
b−c a
a−b a
(iv) If a, b, c are in G.P. ⇒ =
b−c b

Harmonic mean (H.M.):


2ac

RI
If a, b, c are in H.P., b is called as the H.M. between a & c, then b =
a+c
If a1, a2 , ........ an are ‘n’ non-zero numbers then H.M. 'H' of these numbers is given by
1 11 1 1
=  + + ....... + 

A
H n  a1 a2 an 

1 1
Example # 21 : The 7th term of a H.P. is and 12th term is , find the 20th term of H.P.

UH
10 25
Solution : Let a be the first term and d be the common difference of corresponding A.P.
a + 6d = 10
a + 11d = 25
5d = 15
d = 3, a = – 8
JA
T20 = a + 19d
= – 8 + 19 × 3 = 49
1
20 term of H.P. =
49

3 3
LP

Example # 22 : Insert 4 H.M between and .


4 19
Solution : Let 'd' be the common difference of corresponding A.P..
19 4

so d = 3 3 = 1.
A

5
1 4 7 3
∴ = +1= or H1 =
H1 3 3 7
NK

1 4 10 3
= +2= or H2 =
H2 3 3 10
1 4 13 3
= +3= or H3 =
H3 3 3 13
1 4 16 3
SA

= +4= or H4 = .
H4 3 3 16
2 12
Example # 23 : Find the largest positive term of the H.P., whose first two term are and .
5 23
5 23 30 23 16 9 2 –5
Solution : The corresponding A.P. is , ........ or , , , , , .......
2 12 12 12 12 12 12 12
12 12 12 12 12 12
The H.P. is , , , , , – , .......
30 23 16 9 2 5
12
Largest positive term = =6
2
Self practice problems :
(13) If a, b, c, d, e are five numbers such that a, b, c are in A.P., b, c, d are in G.P. and c, d, e are in
H.P. prove that a, c, e are in G.P.
(14) If the ratio of H.M. between two positive numbers 'a' and 'b' (a > b) is to their G.M. as 12 to 13,
prove that a : b is 9 : 4.
b+a b+c
(15) a, b, c are in H.P. then prove that + =2
b−a b−c
(16) If a, b, c, d are in H.P., then show that ab + bc + cd = 3ad

Arithmetico-geometric series :
A series, each term of which is formed by multiplying the corresponding terms of an A.P. & G.P. is
called the Arithmetico-Geometric Series. e.g. 1 + 3x + 5x 2 + 7x3 +.....
Here 1, 3, 5,.... are in A.P. & 1, x, x2, x3..... are in G.P..
Sum of n terms of an arithmetico-geometric series:
Let Sn = a + (a + d) r + (a + 2 d) r² +..... + [a + (n − 1)d] rn−1, then

RI
a dr 1− r n −1
( )
[a + (n − 1)d] r n , r ≠ 1.
Sn = + −
1− r (1− r )
2
1− r

Sum to infinity: If r < 1 & n → ∞ , then Limit rn = 0 and Limit n.rn = 0

A
n→∞ n→∞

a dr
∴ S∞ = + .
1− r (1− r )2

UH
2 3
 4n + 1   4n + 1   4n + 1 
Example # 24 : The sum to n terms of the series 1 + 5  +9   + 13  4n – 3  + ....... is .
 4n – 3   4n – 3   
4n + 1
Solution : Let x= , then
4n – 3
JA
–4 1 (4n – 3)
1–x= , =–
4n – 3 1– x 4
x (4n + 1)
=–
1– x 4
S = 1 + 5x + 9x2 + ....... + (4n – 3)xn–1
Sx = x + 5x2 + ........ (4n – 3)xn
LP

S – Sx = 1 + 4x + 4x2 + ......... + 4xn–1 – (4n – 3)xn.


4x
S(1 – x) = 1 + [1 –xn–1] – (4n – 3)xn
1– x
1  4x 4xn  (4n – 3)
S= 1 + – – (4n – 3)x n  = – [1– (4n + 1) + (4n – 3)x n – (4n – 3)x n ] = n (4n – 3).
1– x  1– x 1– x  4
A

Example # 25 : Find sum to infinite terms of the series 1 + 2x + 3x 2 + 4x3 + ....., –1 < x < 1
Solution : let S = 1 + 2x + 3x2 + 4x3 + ..... ........(i)
NK

xS = x + 2x2 + 3x3 + ......... ........(ii)


(i) - (ii) ⇒ (1 – x) S = 1 + x + x 2 + x3 + ..........
1
or S=
(1 − x)2
SA

Example # 26 : Evaluate : 12 + 22x + 32x2 + 42x3 ....... upto infinite terms for | x | < 1.
Solution : Let s = 12 + 22x + 32x2 + 42x3 ............ ∞ ...(i)
xs = 12x + 22x2 + 32x3............... ∞ ...(ii)
(i) – (ii)
(1 – x) s = 1 + 3x + 5x2 + 7x3 + ........
1 2x
(1 – x) s = +
1− x (1 − x )
2

1 2x
s= 2
+ 3
(1 − x ) (1 − x )
1 − x + 2x
s= 3
(1 − x )
1+ x
s= 3
(1 − x )
Self practice problems :

4+d 4 + 2d
(17) If 4 + + ......... = 1, then find d.
5 52

(18) Evaluate : 1 + 3x + 6x2 + 10x3 + ...... upto infinite term, where | x | < 1.
2
 1  1
(19) Sum to n terms of the series : 1 + 2  1 +  + 3  1 +  + ......
 n   n
64

RI
Ans. (17) –
5
1
(18)
(1 − x)3

A
(19) n2

Relation between means :

UH
(i) If A, G, H are respectively A.M., G.M., H.M. between a & b both being positive, then G² = AH
(i.e. A, G, H are in G.P.) and A ≥ G ≥ H.

8
Example # 27 : The A.M. of two numbers exceeds the G.M. by 2 and the G.M. exceeds the H.M. by ; find the
5
JA
numbers.
Solution : Let the numbers be a and b, now using the relation
 8
G2 = AH = (G + 2)  G −  ⇒ G = 8 ; A = 10
 5
i.e. ab = 64
also a + b = 20
LP

Hence the two numbers are 4 and 16.

A.M. ≥ G.M. ≥ H.M.


Let a1, a2, a3, .......an be n positive real numbers, then we define their
A

a + a2 + a3 + ....... + an
A.M. = 1 , their
n
G.M. = (a1 a2 a3 .........an)1/n and their
NK

n
H.M. = .
1 1 1
+ + ....... +
a1 a2 an
It can be shown that A.M. ≥ G.M. ≥ H.M. and equality holds at either places iff a 1 = a2 = a3 = .........= an
SA

ab bc ca
Example # 28 : If a, b, c > 0, prove that 2
+ 2
≥3
+
c a b2
Solution : Using the relation A.M. ≥ G.M. we have
ab bc ca 1
+ +
c 2 a2 b2 ≥  ab . bc . ca  3 ⇒ ab bc ca
+ 2+ 2 ≥3
 2 2 2 2
3 c a b  c a b
1 1 1 1
Example # 29 : If ai > 0 ∀ i = 1, 2, 3, ....... prove that (a1 + a2 + a3 .... + an)  + + ..... +  ≥ n2
 a1 a2 a3 an 
Solution : Using the relation A.M. ≥ H.M.
a1 + a2 + a3 ....... + an n

n 1 1 1 1
+ + ..... +
a1 a2 a3 an
1 1 1 1 
⇒ (a1 + a2 + a3 .... + an)  + + ..... +  ≥ n2
 1
a a 2 a 3 a n 

 1 1  1 1   1 1 
Example # 30 : If x, y, z are positive then prove that (x + y)(y + z)(z + x)  +  +   +  ≥ 64
 x y  y z   z x 
Solution : Using the relation A.M. ≥ H.M.
x+y 2  1 1
≥ ⇒ (x + y)  +  ≥ 4 .....(i)
2 1 1 x y
+
x y
 1 1
similarly (y + z)  +  ≥ 4 .....(ii)
y z

RI
 1 1
(z + x) ≥ 4  +  .....(iii)
z x
 1 1  1 1   1 1 
by (i), (ii) & (iii) (x + y)(y + z)(z + x)  +  +   +  ≥ 64
 x y  y z   z x 

A
Example # 31 : If n > 0, prove that 2n > 1 + n 2n −1

UH
Solution : Using the relation A.M. ≥ G.M. on the numbers 1, 2, 2 2, 23,..........., 2n–1 , we have
1 + 2 + 22 + ....... + 2n −1
> (1.2. 22. 23. ......... .2n–1)1/n
n
Equality does not hold as all the numbers are not equal.
1
JA
2n − 1  (n −1) n  n (n −1)
⇒ > n 2 2  ⇒ 2n – 1 > n 2 2
2 −1  
(n −1)
⇒ 2n > 1 + n 2 2
.

Example # 32 : If x, y, z are positive and x + y + z = 7 then find greatest value of x 2 y3 z2.


LP

Solution : Using the relation A.M. ≥ G.M.


x x y y y z z 1
+ + + + + +
2 2 3 3 3 2 2 ≥  x . y . z 7
2 3 2

 
7  4 27 4 
A

1
 x 2 y3 z2  7
⇒ 1≥  . .  ⇒ 432 ≥ x2 y3 z2
 4 27 4 
NK

Self practice problems :

(20) If a, b, c are real and distinct, then show that a2 (1 + b2) + b2 (1 + c2) + c2 (1 + a2) > 6abc
SA

(21) Prove that 2.4.6.8.......2n < (n + 1)n. (n ∈ N)

bcd cda dab abc


(22) If a, b, c, d are positive real numbers prove that + 2 + 2 + 2 >a+b+c+d
a2 b c d

(23) If x6 – 12x5 + ax4 + bx3 + cx2 + dx + 64 = 0 has positive roots then find a, b, c, d,

(24) If a, b > 0, prove that [(1 + a) (1 + b)]3 > 33 a2 b2

Ans. (23) a = 60, b = –160, c = 240, d = –192

Results :
n n n n n
(i) ∑
r =1
(ar ± br) = ∑
r =1
ar ± ∑
r =1
br. (ii) ∑
r =1
k ar = ∑
r =1
k ar.

n
(iii) ∑
r =1
k = k + k + k +...............n times = nk; where k is a constant.

n
n (n + 1)
(iv) ∑
r =1
r = 1 + 2 + 3 +...........+ n =
2
n
n (n + 1) (2n + 1)
(v) ∑
r =1
r² = 12 + 22 + 32 +...........+ n2 =
6
n
n2 (n + 1)2
(vi) ∑
r =1
r3 = 13 + 23 + 33 +...........+ n3 =
4

RI
Example # 33 : Find the sum of the series to n terms whose nth term is 3n + 2.
3(n + 1) n n
Solution : Sn = ΣTn = ∑(3n + 2) = 3Σn + Σ2 = + 2n = (3n + 7)
2 2

A
n
Example # 34 : Tk = k3 + 3k , then find ∑T
k =1
k .

UH
2 2
n n n
 n(n + 1)  3(3n − 1)  n(n + 1) 
∑ T = ∑ k3 ∑
3 n
Solution : k + 3k =   + =   + (3 –1)
k =1 k =1 k =1  2  3 −1  2  2

Method of difference for finding nth term :


JA
Let u1, u2, u3 ........ be a sequence, such that u2 – u1, u3 – u2, ......... is either an A.P. or a G.P. then nth
term un of this sequence is obtained as follows
S = u1 + u2 + u3 + ........... + un ................(i)
S= u1 + u2 + ........... + un–1 + un ................(ii)
LP

(i) – (ii) ⇒ un = u1 + (u2 – u1) + (u3 – u2) + ........... + (un – un–1)

Where the series (u2 – u1) + (u3 – u2) + .......... + (un – un–1) is
n


A

either in A.P. or in G.P. then we can find un. So sum of series S = ur


r =1

Note : The above method can be generalised as follows :


NK

Let u1, u2, u3, ......... be a given sequence.


The first differences are ∆1u1, ∆1u2, ∆1u3, ........ where ∆1u1 = u2 – u1, ∆1u2 = u3 – u2 etc.
The second differences are ∆2u1, ∆2u2, ∆2u3, ......., where ∆2u1 = ∆1u2 – ∆1u1, ∆2u2 = ∆1u3 – ∆1u2 etc.
This process is continued untill the kth differences ∆ku1 , ∆ku2 , ........ are obtained, where the kth
SA

differences are all equal or they form a GP with common ratio different from 1.
Case - 1 : The kth differences are all equal.
In this case the nth term, un is given by
un = a0nk + a1nk–1 + .....+ ak , where a0, a1, ...., ak are calculated by using first 'k + 1' terms of the
sequence.

Case - 2 : The kth differences are in GP with common ratio r (r ≠ 1)


The nth term is given by un = λ rn – 1 + a0 nk–1 + a1 nk–2 + ..... + ak–1

Example # 35 : Find the nth term of the series 1, 3, 8, 16, 27, 41, ..........

Solution : s = 1 + 3 + 8 + 16 + 27 + 41 + ...... T n .....(i)


s= 1 + 3 + 8 + 16 + 27 .......Tn–1 + Tn .....(ii)
(i) – (ii)
Tn = 1 + 2 + 5 + 8 + 11 + ....... (Tn – Tn – 1)
 n − 1 1
Tn = 1 +   [2 × 2 + (n – 2)3] = 2 [3n – 5n + 4]
2

 2 

Example # 36 : Find the sum to n terms of the series 5, 7, 13, 31, 85 + ......

Solution : Successive difference of terms are in G.P. with common ratio 3.


Tn = a(3)n –1 + b
a+b=5

RI
3a + b = 7 ⇒ a = 1, b = 4
Tn = 3 n–1
+4
Sn = ΣTn = Σ(3n – 1 + 4) = (1 + 3 + 32 + ...... + 3n – 1) + 4n

A
1 n
[3 + 8n – 1]
2

UH
Method of difference for finding sn :
If possible express rth term as difference of two terms as tr = ± (f(r) – f(r ± 1)). This can be explained with
the help of examples given below.
JA
t1 = f(1) – f(0),
t2 = f(2) – f(1),
  
tn = f(n) – f(n-1)
⇒ Sn = f(n) – f(0)
LP

Example # 37 : Find the sum of n-terms of the series 2.5 + 5.8 + 8.11 + ...........

Solution : Tr = (3r – 1) (3r + 2) = 9r2 + 3r – 2


A

n n n n
Sn = ∑r =1
Tr = 9 ∑
r =1
Tr + 3 ∑ ∑2
r =1
r –
r =1
NK

 n ( n + 1)( 2n + 1)   n ( n + 1) 
=9  + 3  – 2n
 6   2 
   
= 3n(n + 1)2 – 2n
SA

1 1 1
Example # 38 : Sum to n terms of the series + + + .........
(1 + x)(1 + 3x) (1 + 3x)(1 + 5x) (1 + 5x)(1 + 7x)
Solution : Let Tr be the general term of the series
1
Tr =
1 + ( 2r − 1) x  1 + (2r + 1)x 

1  (1 + (2r + 1)x ) − (1 + (2r − 1)x )   1 1 


So Tr =   =  − 
2x  (1 + (2r − 1)x ) (1 + (2r + 1)x)   (1 + (2r − 1)x ) (1 + (2r + 1)x ) 
∴ Sn = ∑ Tr = T1 + T2 + T3 + .......... + Tn

1  1 1  n
=  − =
2x  1 + x (1 + (2n + 1)x )  (1 + x)[1 + (2n + 1)x]
1 1 1
Example # 39 : Sum to n terms of the series + + +............
1.4.7 4.7.10 7.10.13
1 1  1 1 
Solution : Tn = =  − 
( 3n − 2 )( 3n + 1)( 3n + 4 ) 6  ( 3n − 2 )( 3n + 1) ( 3n + 1)( 3n + 4 ) 
1  1 1   1 1  1 1 
=  −  + −  + ...... + − 
6  1.4 4.7   4.7 7.10  ( 3n − 2 )( 3n + 1) ( 3n + 1)( 3n + 4 ) 
1 1 1 
=  − 
6  4 ( 3n + 1)( 3n + 4 ) 

RI
Example # 40 : Find the general term and sum of n terms of the series
1 + 5 + 19 + 49 + 101 + 181 + 295 + .........
Solution : The sequence of difference between successive term 4, 14, 30, 52, 80 .....

A
The sequence of the second order difference is 10, 16, 22, 28, ...... clearly it is an A.P>
so let nth term

UH
Tn = an3 + bn2 + cn + d
a+b+c+d =1 ....(i)
8a + 4b + 2c + d = 5 ....(ii)
27a + 9b + 3c + d = 19 ....(iii)
64a + 16b + 4c + d = 49 ....(iv)
JA
from (i), (ii), (iii) & (iv)
a = 1, b = –1, c = 0, d = 1 ⇒ T n = n3 – n2 + 1

n n2 − 1 ( 3n + 2 ) ( )
2
 n ( n + 1)  n ( n + 1)( 2n + 1)
sn = Σ(n – n + 1 ) = 
3 2
 – +n= +n
 2  6 12

LP

Self practice problems :

(25) Sum to n terms the following series


A

3 5 7
(i) + + +...........
12.22 22.32 32.42
(ii) 1 + (1 + 2) + (1 + 2 + 3) + (1 + 2 + 3 + 4) ........
NK

(iii) 4 + 14 + 30 + 52 + 82 + 114 + ..........

n n
1
(26) If ∑ T = (n + 1)(n + 2)(n + 3) then find ∑ T
SA

r
r =1 r =1 r

2n + n2 n ( n + 1)( n + 2 ) n
Ans. (25) (i) (ii) (iii) n(n + 1)2 (26)
(n + 1) 2 6 6 (n + 2)
 Marked questions are recommended for Revision.

PART - I : SUBJECTIVE QUESTIONS

Section (A) : Arithmetic Progression


A-1. In an A.P. the third term is four times the first term, and the sixth term is 17 ; find the series.

p
A-2. Find the sum of first 35 terms of the series whose pth term is + 2.

RI
7

A-3. Find the number of integers between 100 & 1000 that are divisible by 7

A
A-4. Find the sum of all those integers between 100 and 800 each of which on division by 16 leaves the
remainder 7.

UH
A-5. The sum of first p-terms of an A.P. is q and the sum of first q terms is p, find the sum of first (p + q)
terms.

A-6. The sum of three consecutive numbers in A.P. is 27, and their product is 504, find them.

A-7. The fourth power of the common difference of an arithmetic progression with integer entries is added to
JA
the product of any four consecutive terms of it. Prove that the resulting sum is the square of an integer.

A-8. If a, b, c are in A.P., then show that:


(i) a2 (b + c), b2 (c + a), c2 (a + b) are also in A.P.
(ii) b + c − a, c + a − b, a + b − c are in A.P.

There are n A.M's between 3 and 54, such that the 8th mean: (n − 2) th mean:: 3: 5. The value of n is.
LP

A-9.

Section (B) : Geometric Progression


B-1. The third term of a G.P. is the square of the first term. If the second term is 8, find its sixth term.
A

B-2. The continued product of three numbers in G.P. is 216, and the sum of the products of them in pairs is
156; find the numbers
NK

B-3. The sum of infinite number of terms of a G.P. is 4 and the sum of their cubes is 192. Find the series.

B-4. The sum of three numbers which are consecutive terms of an A.P. is 21. If the second number is
reduced by 1 & the third is increased by 1, we obtain three consecutive terms of a G.P., find the
numbers.
SA

B-5. If the pth, qth & rth terms of an AP are in GP. Find the common ratio of the GP.

B-6. If a, b, c, d are in G.P., prove that :

(i) (a2 − b2), (b2 − c2), (c2 − d2) are in G.P.


1 1 1
(ii) , , are in G.P.
a2 + b2 b2 + c 2 c 2 + d2
32 243
B-7. Let five geometric means are inserted between and then find sum of all the geometric
3 2
means.
Section (C) : Harmonic and Arithmetic Geometric Progression

1 1
C-1. Find the 4th term of an H.P. whose 7th term is and 13th term is .
20 38

C-2. Insert three harmonic means between 1 and 7.

a–x a–y a–z


C-3. If = = and p, q, r are in A.P. then prove that x, y, z are in H.P.
px qy rz

C-4. If a2, b2, c2 are in A.P. show that b + c , c + a, a + b are in H.P.

RI
1 1 1 1
C-5. If b is the harmonic mean between a and c, then prove that + = + .
b−a b−c a c

C-6. Sum the following series

A
2 3 4
(i) 1+ + 2 + 3 + ........ to n terms.
2 2 2
3 7 15 31

UH
(ii) 1+ + + + + ..... to infinity.
4 16 64 256

C-7. Find the sum of n terms of the series the rth term of which is (2r + 1)2r.
JA
Section (D) : Relation between A.M., G.M., H.M

D-1. Using the relation A.M. ≥ G.M. prove that


(i) (x2y + y2z + z2x) (xy2 + yz2 + zx2) ≥ 9x2 y2 z2. (x, y, z are positive real number)
(ii) (a + b) . (b + c) . (c + a) > abc ; if a, b, c are positive real numbers

x100
LP

D-2. If x > 0, then find greatest value of the expression .


1 + x + x + x 3 + ..... + x 200
2

16
D-3. The H.M. between two numbers is , their A.M. is A and G.M. is G. If 2A + G 2 = 26, then find the
5
A

numbers.

D-4. If a, b, c are positive real numbers and sides of the triangle, then prove that
(a + b + c)3 ≥ 27 (a + b – c) (c + a – b) (b + c – a)
NK

D-5. If ai >0 for all i = 1, 2, 3 ........ n then prove that


(1 + a1 + a21) (1 + a2 + a22) ........ (1 + an + a2n) ≥ 3n(a1 a2 a3 ........ an)
SA

Section (E) : Summation of series


E-1. Find the sum to n-terms of the sequence.
(i) 1 + 5 + 13 + 29 + 61 + ........ up to n terms
(ii) 3 + 33 + 333 + 3333 + ........ up to n terms

1 3 7 15
E-2. + + + + .... to n terms.
2 4 8 16
k
E-3. (i) If tn = 3n – 2n then find ∑t
n =1
n .

k
(ii) If tn = n(n + 2) then find ∑t
n =1
n .
(iii) Find the sum to n terms of the series 12 – 22 + 32 – 42 + 52 – 62 + .....

(iv) 102 + 132 + 162 + ...... upto 10 terms


n n
(v) If ∑ Ι(r) = n(2n
r =1
2
+ 9n + 13) , then find the ∑
r =1
Ι(r)

E-4. Find the sum to n-terms of the sequence.


1 1 1
(i) + + + ....
1.3.5 3.5.7 5.7.9
(ii) 1 . 3 . 22 + 2 . 4 . 32 + 3 . 5 . 42 + ....

PART - II : ONLY ONE OPTION CORRECT TYPE

RI
Section (A) : Arithmetic Progression
A-1. The first term of an A.P. of consecutive integer is p2 + 1. The sum of (2p + 1) terms of this series can be

A
expressed as
(A) (p + 1)2 (B) (2p + 1) (p + 1)2 (C) (p + 1)3 (D) p3 + (p + 1)3
A-2. If a1, a2, a3, .... are in A.P. such that a1 + a5 + a10 + a15 + a20 + a24 = 225, then

UH
a1 + a2 + a3 + ..... + a23 + a24 is equal to
(A) 909 (B) 75 (C) 750 (D) 900

A-3. If the sum of the first 2n terms of the A.P. 2, 5, 8, ....., is equal to the sum of the first n terms of the
A.P. 57, 59, 61, ..., then n equals
JA
(A) 10 (B) 12 (C) 11 (D) 13

A-4. The sum of integers from 1 to 100 that are divisible by 2 or 5 is


(A) 2550 (B) 1050 (C) 3050 (D) none of these

A-5. Let 6 Arithmetic means A1, A2, A3, A4, A5, A6 are inserted between two consecutive natural number a
and b (a > b). If A12 – A22 + A32 – A42 + A52 – A62 is equal to prime number then 'b' is equal to
LP

(A) 1 (B) 2 (C) 3 (D) 4

Section (B) : Geometric Progression


B-1. The third term of a G.P is 4. The product of the first five terms is
A

(A) 43 (B) 45 (C) 44 (D) 4

B-2. If S is the sum to infinity of a G.P. whose first term is ‘a’, then the sum of the first n terms is
  S n 
NK

 a
n
  a 
n
  a 
n

(A) S  1 −  (B) S 1 −  1 −   (C) a 1 −  1 −   (D) S 1 −  1 −  


 S   S     S     a 

an + 1 1 20
B-3. For a sequence {an}, a1 = 2 and = . Then ∑ ar is
an 3 r =1

 1   1 
SA

20
(A) [4 + 19 × 3] (B) 3  1 − 20  (C) 2 (1 – 320) (D)  1 − 20 
2  3   3 
B-4. α, β be the roots of the equation x2 – 3x + a = 0 and γ, δ the roots of x2 – 12x + b = 0 and numbers
α, β, γ, δ (in this order) form an increasing G.P., then
(A) a = 3, b = 12 (B) a = 12, b = 3 (C) a = 2, b = 32 (D) a = 4, b = 16

B-5. One side of an equilateral triangle is 24 cm. The mid−points of its sides are joined to form another
triangle whose mid − points are in turn joined to form still another triangle. This process continues
indefinitely. Then the sum of the perimeters of all the triangles is
(A) 144 cm (B) 212 cm (C) 288 cm (D) 172 cm

B-6. Let 3 geometric means G1, G2, G3 are inserted between two positive number a and b such that
G3 − G2 b
= 2, then equal to
G2 − G1 a
(A) 2 (B) 4 (C) 8 (D) 16
Section (C) : Harmonic and Arithmetic Geometric Progression

1 1 3
C-1. If the 3rd, 6th and last term of a H.P. are , , , then the number of terms is equal to
3 5 203
(A) 100 (B) 102 (C) 99 (D) 101

b+a b+c
C-2. If a, b, c are in H.P. then the value of + is
b–a b–c
(A) 1 (B) 3 (C) 4 (D) 2

C-3. If the roots of the equation x3 – 11x2 + 36x – 36 = 0 are in H.P. then the middle root is
(A) an even number (B) a perfect square of an integer

RI
(C) a prime number (D) a composite number

C-4. Let the positive numbers a, b, c, d be in A.P. Then abc, abd, acd, bcd are:
(A) not in A.P./G.P./H.P. (B) in A.P.

A
(C) in G.P. (D) in H.P.

1 1
C-5. If 3 + (3 + d) + 2 (3 + 2d) +...... + upto ∞ = 8, then the value of d is :

UH
4 4
(A) 9 (B) 5 (C) 1 (D) 4

C-6. Let 'n' Arithmetic Means and 'n' Harmonic Means are inserted between two positive number 'a' and 'b'.
If sum of all Arithmetic Means is equal to sum of reciprocal all Harmonic means, then product of
numbers is
JA
1
(A) 1 (B) 2 (C) (D) 3
2

C-7. Let a1 , a2 a3 ........ be in A.P. and h1, h2, h3,...... in H.P. If a1 = 2 = h1 and a30 = 25 = h30 then
(a7 h24 + a14 h17 ) equal to :
(A) 50 (B) 100 (C) 200 (D) 400
LP

C-8. Statement 1 : 3,6,12 are in G.P., then 9,12,18 are in H.P.


Statement 2 : If three consecutive terms of a G.P. are positive and if middle term is added in these
terms, then resultant will be in H.P.
(A) STATEMENT-1 is true, STATEMENT-2 is true and STATEMENT-2 is correct explanation for
A

STATEMENT-1
(B) STATEMENT-1 is true, STATEMENT-2 is true and STATEMENT-2 is not correct explanation
for STATEMENT-1
(C) STATEMENT-1 is true, STATEMENT-2 is false
NK

(D) STATEMENT-1 is false, STATEMENT-2 is true

39 37 5.36 36 7.34  25 
C-9. S = 310 + 39 + + + + + + ........ upto infinite terms, then   S equals to
4 2 16 16 64  36 
(A) 69 (B) 310 (C) 311 (D) 2. 310
SA

1.3 3.5 5.7 7.9


C-10 The sum of infinite series + 2 + 3 + 4 + ......∞
2 2 2 2
(A) 21 (B) 22 (C) 23 (D) 24

Section (D) : Relation between A.M., G.M., H.M

D-1. If x ∈ R, the numbers 51+x + 51–x, a/2, 25x + 25–x form an A.P. then 'a' must lie in the interval:
(A) [1, 5] (B) [2, 5] (C) [5, 12] (D) [12, ∞)

D-2. If A, G & H are respectively the A.M., G.M. & H.M. of three positive numbers a, b, & c, then the
equation whose roots are a, b, & c is given by :
(A) x3 − 3 Ax2 + 3 G3x − G3 = 0 (B) x3 − 3 Ax2 + 3 (G3/H)x − G3 = 0
(C) x3 + 3 Ax2 + 3 (G3/H) x − G3 = 0 (D) x3 − 3 Ax2 − 3 (G3/H) x + G3 = 0
D-3. If a, b, c, d are positive real numbers such that a + b + c + d = 2, then M = (a + b) (c + d) satisfies the
relation:
(A) 0 ≤ M ≤ 1 (B) 1 ≤ M ≤ 2 (C) 2 ≤ M ≤ 3 (D) 3 ≤ M ≤ 4

D-4. If a + b + c = 3 and a > 0, b > 0, c > 0, the greatest value of a 2b3c2.


310.24 39.24 39.25 310.25
(A) (B) (C) (D)
77 77 77 77

D-5. If P, Q be the A.M., G.M. respectively between any two rational numbers a and b, then P – Q is equal
to
2
a−b a+b 2ab  a− b
(A) (B) (C) (D)  
a 2 a+b  2 

RI
Section (E) : Summation of series

1 1 1 3 5 2n − 1

A
E-1 If Hn = 1 + + + ...........+ , then value of 1 + + + ......... + is
2 3 n 2 3 n
(A) 2n – Hn (B) 2n + Hn (C) Hn – 2n (D) Hn + n

UH
E-2. Statement 1 : The sum of the first 30 terms of the sequence 1,2,4,7,11,16, 22,...... is 4520.
Statement 2 : If the successive differences of the terms of a sequence form an A.P., then general term
of sequence is of the form an2 + bn + c.
(A) STATEMENT-1 is true, STATEMENT-2 is true and STATEMENT-2 is correct explanation for
STATEMENT-1
JA
(B) STATEMENT-1 is true, STATEMENT-2 is true and STATEMENT-2 is not correct explanation
for STATEMENT-1
(C) STATEMENT-1 is true, STATEMENT-2 is false
(D) STATEMENT-1 is false, STATEMENT-2 is true

n
1
E-3. The value of ∑ a + r x + a + (r − 1) x
is
LP

r =1

n n a + nx − a a + a+n x
(A) (B) (C) (D)
a + a + nx a − a + nx 2x x

E-4. The value of (1.12 + 3.22 + 5.32 + ........ + upto 10 terms) is equal to :
A

(A) 6050 (B) 5965 (C) 5665 (D) 5385

PART - III : MATCH THE COLUMN


NK

1. Column – Ι Column – ΙΙ
(A) The cofficient of x in the product
49

(x – 1) (x – 3) (x – 5) (x – 7) ....... (x – 99) (p) –2500


SA

(B) Let Sn denote sum of first n terms of an A.P. If S2n = 3Sn, (q) 9
S
then 3n is
Sn
∞ 1
(C) The sum ∑ r2 − 1
is equal to: (r) 3/4
r =2

(D) The length,breadth, height of a rectangular box are in G.P. (s) 6


(length > breadth > height) The volume is 27, the total surface
area is 78. Then the length is

2. Column – Ι Column – ΙΙ

(A) The value of xyz is 15/2 or 18/5 according as the series (p) 2
a, x, y, z, b are in an A.P. or H.P. then 'a + b' equals
where a, b are positive integers.
1 1 1
(B) The value of . 2 4 4 8 816 . - - - ∞ is equal to (q) 1

(C) If x, y, z are in A.P., then (r) 3


(x + 2y – z) (2y + z – x) (z + x – y) = kxyz,
where k ∈ N, then k is equal to

(D) There are m A.M. between 1 and 31. If the ratio of the (s) 4
m
7th and (m –1)th means is 5 : 9, then is equal to
7

RI
 Marked questions are recommended for Revision.
PART - I : ONLY ONE OPTION CORRECT TYPE

A
1. Given the sequence of numbers x1, x2, x3, ...... x2013
x1 x2 x3 x 2013
which satisfy = = = ....... = , nature of the sequence is
x1 + 1 x2 + 3 x3 + 5 x 2013 + 4025

UH
(A) A.P. (B) G.P. (C) H.P. (D) A.G.P.

3
2. Suppose a, b, c are in A.P. and a 2, b2, c2 are in G.P. if a < b < c and a + b + c = , then the value of a
2
is
JA
1 1 1 1 1 1
(A) (B) (C) – (D) –
2 2 2 3 2 3 2 2

3. If 1, 2, 3 ... are first terms; 1, 3, 5 .... are common differences and S 1, S2, S3 .... are sums of n terms of
given p AP’s; then S1 + S2 + S3 + ... + Sp is equal to
LP

np(np + 1) n(np + 1) np(p + 1) np(np − 1)


(A) (B) (C) (D)
2 2 2 2

4. If the sum of n terms of a G.P. (with common ratio r) beginning with the p th term is k times the sum of an
equal number of terms of the same series beginning with the qth term, then the value of k is:
A

(A) rp/q (B) rq/p (C) r p − q (D) rp + q

5. Consider the sequence 2, 3, 5, 6, 7, 8, 10, 11, ..... of all positive integer, then 2011 th term of this
sequence is
NK

(A) 2056 (B) 2011 (C) 2013 (D) 2060

∞ ∞ ∞
6. If x = ∑a
n=0
n
,y= ∑b
n=0
n
,z= ∑c
n=0
n
where a,b,c are in AP and |a| < 1, |b| < 1, |c| < 1, then x,y,z are in :

(A) HP (B) Arithmetico–Geometric Progression


SA

(C) AP (D) GP

n
a1 a2 a
7. If a1, a2 , ........ are in H.P. and f(k) = ∑ (a
r =1
r – ak ) , then ,
f(1) f(2)
,....... n are in
f(n)
(A) A.P. (B) G.P. (C) H.P. (D) None of these

8. If a1, a2, a3, ........., an are positive real numbers whose product is a fixed number c, then the minimum
value of a1 + a2 + a3 + .... + an – 1 + 2an is
(A) n(2c)1/n (B) (n + 1) c1/n (C) 2nc1/n (D) (n + 1)(2c)1/n

n(n + 1)2
9. The sum of the first n-terms of the series 12 + 2.22 + 32 + 2.42 + 52 + 2.62 + .......... is , when n
2
is even. When n is odd, the sum is
n(n + 1)2 n2 (n + 2) n2 (n + 1) n(n + 2)2
(A) (B) (C) (D)
4 4 2 4

10. Let Tr and Sr be the rth term and sum up to rth term of a series respectively. If for an odd number n, Sn =
Tn−1
n and Tn = then Tm (m being even) is
n2
2 2m2 (m + 1)2 2(m + 1)2
(A) (B) (C) (D)
1 + m2 1 + m2 2 + (m + 1)2 1 + (m + 1)2

11. If 12 + 22 + 32 + ....... + 20032 = (2003) (4007) (334) and


(1) (2003) + (2) (2002) + (3) (2001) + ..... + (2003) (1) = (2003) (334) (x)., then x equals
(A) 2005 (B) 2004 (C) 2003 (D) 2001

RI
n n ( n + 1)( n + 2 )( n + 3 ) n
1
12. If ∑t
r =1
r =
8
, then ∑t
r =1 r
equals

 1 1  1 1

A
(A)  −  (B)  − 
 ( n + 1)( n + 2 ) 2   ( n + 1)( n + 2 ) 2 
   
 1 1  1 1
(C)  +  (D)  + 

UH
 (n + 1) (n + 2) 2   (n − 1) (n − 2) 2 

1 1 1 π2 1 1 1
13. If 2
+ 2 + 2 +...... upto ∞ = , then 2 + 2 + 2 +...... =
1 2 3 6 1 3 5
(A) π2/12 (B) π2/24 (C) π2/8 (D) π2/4
JA
PART - II : SINGLE AND DOUBLE VALUE INTEGER TYPE
LP

1. A man arranges to pay off a debt of Rs. 3600 by 40 annual installments which form an arithmetic
series. When 30 of the installments are paid he dies leaving a third of the debt unpaid. Find the value of
the first installment.
A

2. In a circle of radius R a square is inscribed, then a circle is inscribed in the square, a new square in the
circle and so on for n times. If the ratio of the limit of the sum of areas of all the circles to the limit of the
4k
sum of areas of all the squares as n → ∞ is k, then find the value of .
π
NK

3. If the common difference of the A.P. in which T7 = 9 and T1T2T7 is least, is ‘d’ then 20d is–

4. The number of terms in an A.P. is even ; the sum of the odd terms is 24, sum of the even terms is 30,
and the last term exceeds the first by 10½; find the number of terms.
SA

5. If x > 0, and log2 x + log2 ( x ) + log ( x ) + log ( x ) + log ( x ) + ...... = 4, then find x.
2
4
2
8
2
16

6. Given that α, γ are roots of the equation Ax2 − 4 x + 1 = 0 and β, δ the roots of the equation
B x2 − 6 x + 1 = 0, then find value of (A + B), such that α, β, γ & δ are in H.P.

7. Find the sum of the infinitely decreasing G.P. whose third term, three times the product of the first and
fourth term and second term form an A.P. in the indicated order, with common difference equal to
1/8.

8. If a, b, c are in GP, a – b, c – a, b – c are in HP, then the value of a + 4b + c is


9. a, a1, a2, a3,..., a2n , b are in A.P. and a, g1, g2, g3,.....g2n, b are in G.P. and h is the harmonic mean of a
a + a2n a2 + a2n − 1 an + an + 1 Kn
and b, if 1 + + ... + is equal to , then find value of K.
g1g2n g2 g2n − 1 gn gn + 1 20h

10. If the arithmetic mean of two numbers a & b (0 < a < b) is 6 and their geometric mean G and harmonic
mean H satisfy the relation G2 + 3 H = 48. Then find the value of (2a – b)

5 55 555
11. If S = + 2
+ + + ... up to ∞, then find the value of 36S.
13 (13) (13)3

25 22 32 42 52 62
12. If = 12 − + − + − +........ ∞, then find the value of k
k 5 52 53 54 55

RI
1 1 1
13. If xi > 0, i = 1, 2, ..., 50 and x1 + x2 + .. + x50 = 50, then find the minimum value of + + .....+ .
x1 x 2 x 50

A
14. If a1 , a2 , a3 , a4 are positive real numbers such that a1 + a2 + a3 + a4 = 16 then find maximum value of
(a1 + a2)(a3 + a4).

UH
15. If S1 , S2, S3 are the sums of first n natural numbers, their squares, their cubes respectively, then is
S3 (1 + 8S1 )
equal to
S22
JA
1 2 3
16. If S = + + +........∞, then find the value of 14S.
1 + 12 + 14 1 + 22 + 24 1 + 32 + 3 4

PART - III : ONE OR MORE THAN ONE OPTIONS CORRECT TYPE


1. The interior angles of a polygon are in A.P. If the smallest angle is 120º & the common difference is 5º,
LP

then the number of sides in the polygon is :


(A) 7 (B) 9 (C) 16 (D) 5

2. If 1, logy x, logz y, –15 logx z are in A.P., then


(A) z3 = x (B) x = y–1 (C) z–3 = y (D) x = y–1 = z3
A

3. If a1, a2 , ......., an are distinct terms of an A.P., then


(A) a1+ 2a2 + a3 = 0 (B) a1 − 2a2 + a3 = 0
NK

(C) a1 + 3a2 − 3a3 − a4 = 0 (D) a1 − 4a2 + 6a3 − 4a4 + a5 = 0

4. First three terms of the sequence 1/16, a, b, 1/6 are in geometric series and last three terms are in
harmonic series if
1 1 1 1
SA

(A) a = , b = (B) a = ,b=


9 12 12 9
1 1
(C) a = 1, b = − (D) a = − , b = 1
4 4

5. Which of the following numbers is/are composite


(A)1111......1 (91 digits) (B)1111......1 (81 digits)
(C)1111......1 (75 digits) (D)1111......1 (105 digits)

6. Three numbers a, b, c between 2 and 18 are such that


(i) their sum is 25 (ii) the numbers 2, a, b, are in A.P.
(iii) the number b, c, 18 are in G.P.
then which of the following options are correct.
(A) a = 5 (B) b = 8 (C) b + c = 20 (D) a + b + c = 25
7. Consider an infinite geometric series with first term 'a' and common ratio r. If the sum is 4 and the
second term is 3/4, then:
7 3 3 3 1 1
(A) a = , r = (B) a = 2, r = (C) a = , r = (D) a = 3, r =
4 7 8 2 2 4

 1   1  1 
8. For the series 2 +  2 +

 +  (2 2 − 1) +  +  3 2 – 2 +
2  2  2 2
(
 + ..... )
  n −1
 2n / 2 − 1  ( )  1 2
(
(A) Sn = 2 2 + n − 1 −n +  )
 2 − 1 2 2 
n −1 
(B) Tn = 2 2 + n − 1 −n +  
2
( )
 (
 )
 
 (2n/2
−1 )

RI
n n
(C) Sn =
2
(3 + (n − 1) 2 −n +

) n −1 
(D) Sn = (3 + (n − 1) 2 −n +)
2 − 1 2 2 
2
 ( ) 
p
1 S2p
If ak ak–1 + ak–1 ak–2 = 2ak ak–2 , k ≥ 3 and a1 = 1, here Sp = ∑a

A
9. and given that does not depend on
k =1 k Sp
1
p then may be
a2016

UH
(A) 4031 (B) 1 (C) 2016 (D) 2017/2

n
a k +1
10. If
ak
is constant for every k ≥ 1. If n > m ⇒ an > am and a1 + an = 66, a2an–1 = 128 and ∑a
i =1
i = 126
JA
then
a k +1 a k +1
(A) n = 6 (B) n = 5 (C) =2 (D) =4
ak ak

11. The sides of a right triangle form a G.P. The tangent of the smallest angle is
5 +1 5 −1 2 2
(A) (B) (C) (D)
LP

2 2 5 + 1 5 − 1

12. If b1, b2, b3 (bi > 0) are three successive terms of a G.P. with common ratio r, the value of r for which the
inequality b3 > 4b2 – 3b1 holds is given by
A

(A) r > 3 (B) 0 < r < 1 (C) r = 3.5 (D) r = 5.2

13. If a satisfies the equation a2017 – 2a + 1 = 0 and S = 1 + a + a2 + ..... + a2016. then posible value(s) of S
NK

is/are
(A) 2016 (B) 2018 (C) 2017 (D) 2

14. Let a, x, b be in A.P; a, y, b be in G.P and a, z, b be in H.P. If x = y + 2 and a = 5z, then


(A) y2 = xz (B) x > y > z (C) a = 9, b = 1 (D) a = 1/4, b = 9/4
SA

15. Which of the following is/are TRUE


(A) Equal numbers are always in A.P. , G.P. and H.P.
b b b
(B) If a, b, c be in H.P., then a − , , c − will be in AP
2 2 2
(C) If G1 and G2 are two geometric means and A is the arithmetic mean inserted between two
G12 G22
positive numbers, then the value of + is 2A.
G2 G1
(D) Let general term of a G.P. (with positive terms) with common ratio r be T k + 1 and general term
of another G.P. (with positive terms) with common ratio r be T′ k + 1, then the series whose
general term T′′k + 1 = Tk + 1 + T′k + 1 is also a G.P. with common ratio r.

16. If the arithmetic mean of two positive numbers a & b (a > b) is twice their geometric mean, then a: b is:
(A) 2 + 3 : 2 − 3 (B) 7 + 4 3 : 1 (C) 1: 7 − 4 3 (D) 2: 3
n
17. If ∑ r(r + 1)
r =1
(2r + 3) = an4 + bn3 + cn2 + dn + e, then

(A) a + c = b + d (B) e = 0
(C) a, b – 2/3, c – 1 are in A.P. (D) c/a is an integer

18. The roots of the equation x4 – 8x3 + ax2 – bx + 16 = 0, are positive, if


(A) a = 24 (B) a = 12 (C) b = 8 (D) b = 32

19. Let a1, a2, a3, ……..an is the sequence whose sum of first 'n' terms is represented by
a + a3 − xa2
Sn = an3 + bn2 + cn, n∈N. If a = 1 then
y
(A) H.C.F of (x,y) is 2 (B) H.C.F. of (x,y) is 3

RI
(C) L.C.M of (x,y) is 6 (D) x + y = 8

PART - IV : COMPREHENSION

A
Comprehension # 1 (Q.1 & 2)
n(n + 1)
We know that 1 + 2 + 3 + ....... = = f(n),
2

UH
n(n + 1) (2n + 1)
12 + 22 + 32 + .......... + n2 = = g(n),
6
2
 n(n + 1) 
13 + 23 + 33 + .......... + n3 =   = h(n)
 2 
JA
1. Even natural number which divides g(n) – f(n), for every n ≥ 2, is
(A) 2 (B) 4 (C) 6 (D) none of these

2. f(n) + 3 g(n) + h(n) is divisible by 1 + 2 + 3 + ........... + n


(A) only if n = 1 (B) only if n is odd (C) only if n is even (D) for all n ∈ N
LP

Comprehension # 2 (Q.3 & 4)


In a sequence of (4n + 1) terms the first (2n + 1) terms are in AP whose common difference is 2, and
the last (2n + 1) terms are in GP whose common ratio 0.5. If the middle terms of the AP and GP are
equal, then
A

3. Middle term of the sequence is


n . 2n + 1 n . 2n + 1
(A) (B) (C) n . 2n (D) None of these
2 −1
n
22n − 1
NK

4. First term of the sequence is


4n + 2n . 2n 4n − 2n . 2n 2n − n . 2n 2n + n . 2n
(A) (B) (C) (D)
2n − 1 2n − 1 2n − 1 2n − 1
SA

Comprehension # 3 (Q.5 to 7)
Let ∆1Tn = Tn+1 –Tn, ∆2 Tn = ∆1Tn+1 – ∆1Tn , ∆3Tn = ∆2Tn+1 – ∆2Tn , ........ , and so on, where
T1, T2, T3, …..Tn–1, Tn, Tn+1,…… are the terms of infinite G.P. whose first term is a natural
number and common ratio is equal to 'r'.

5. If ∆2 T1 = 36, then sum of all possible integral values of r is equal to :


(A) 8 (B) 4 (C) 5 (D) –2

∑T
7 p
6. Let n = and r = then sum of squares of all possible value of p is equal to :
n =1
3 7
(A) 42 (B) 46 (C) 45 (D) 30

7. If ∆7 Tn = ∆3 Tn , then 'r' can be equal to


(A) 2 (B) 4 (C) 7 (D) –2
 Marked questions are recommended for Revision.
* Marked Questions may have more than one correct option.

PART - I : JEE (ADVANCED) / IIT-JEE PROBLEMS (PREVIOUS YEARS)


1. If the sum of first n terms of an A.P. is cn2, then the sum of squares of these n terms is
[IIT-JEE - 2009, Paper-2, (3, –1), 80]
n(4n2 − 1) c 2 n(4n2 + 1) c 2 n(4n2 − 1) c 2 n(4n2 + 1) c 2
(A) (B) (C) (D)
6 3 3 6

RI
k –1
2. Let Sk, k = 1, 2,...., 100, denote the sum of the infinite geometric series whose first term is and the
k !
1 1002 100
common ratio is . Then the value of + ∑ (k 2
– 3k + 1) Sk is

A
k 100 ! k =1

[IIT-JEE - 2010, Paper-1, (3, 0), 84]

UH
3. Let a1, a2, a3, ....., a11 be real numbers satisfying a1 = 15, 27 – 2a2 > 0 and ak = 2ak–1 – ak–2 for
a12 + a22 + .... + a11
2
a + a2 + ... + a11
k = 3, 4, ...., 11. If = 90, then the value of 1 is equal to
11 11
[IIT-JEE - 2010, Paper-2, (3, 0), 79]
JA
p
4. Let a1, a2, a3,....., a100 be an arithmetic progression with a1 = 3 and Sp = ∑a
i =1
i , 1 ≤ p ≤ 100.

Sm
For any integer n with 1 ≤ n ≤ 20, let m = 5n. If does not depend on n, then a2 is
Sn
[IIT-JEE 2011, Paper-1, (4, 0), 80]
LP

5. The minimum value of the sum of real numbers a –5, a–4, 3a–3, 1, a8 and a10 where a > 0 is

[IIT-JEE 2011, Paper-1, (4, 0), 80]

6. Let a1, a2, a3,.... be in harmonic progression with a1 = 5 and a20 = 25. The least positive integer n for
A

which an < 0 is [IIT-JEE 2012, Paper-2, (3, –1), 66]


(A) 22 (B) 23 (C) 24 (D) 25
NK

4n k(k +1)
7.* Let Sn = ∑ (–1)
k =1
2
k2 . Then Sn can take value(s) [JEE (Advanced) 2013, Paper-1, (4, – 1)/60]

(A) 1056 (B) 1088 (C) 1120 (D) 1332

8. A pack contains n card numbered from 1 to n. Two consecutive numbered card are removed from the
pack and the sum of the numbers on the remaining cards is 1224. If the smaller of the numbers on the
SA

removed cards is k, then k – 20 = [JEE (Advanced) 2013, Paper-1, (4, – 1)/60]

b
9. Let a,b,c be positive integers such that is an integer. If a,b,c are in geometric progression and the
a
a2 + a – 14
arithmetic mean of a,b,c is b + 2, then the value of is
a +1
[JEE (Advanced) 2014, Paper-1, (3, 0)/60]

10. Suppose that all the terms of an arithmetic progression (A.P.) are natural numbers. If the ratio of the
sum of the first seven terms to the sum of the first eleven terms is 6 : 11 and the seventh term lies in
between 130 and 140, then the common difference of this A.P. is
[JEE (Advanced) 2015, P-2 (4, 0) / 80]
1
11. The least value of α∈R for which 4αx2 + ≥ 1, for all x > 0, is
x
[JEE (Advanced) 2016, Paper-1, (3, –1)/62]
1 1 1 1
(A) (B) (C) (D)
64 32 27 25

12. Let bi > 1 for i = 1,2,….,101. Suppose logeb1,logeb2,…,logeb101 are in Arithmetic progression (A.P.) with
the common difference loge 2. Suppose a1, a2,…,a101 are in A.P. such that a1 = b1 and a51= b51. If
t = b1 + b2 + …. + b51 and s = a1 + a2 + … + a51, then [JEE (Advanced) 2016, Paper-2, (3, –1)/62]
(A) s > t and a101 > b101 (B) s > t and a101 < b101
(C) s < t and a101 > b101 (D) s < t and a101 < b101

13. The sides of a right angled triangle are in arithmetic progression. If the triangle has area 24, then what

RI
is the length of its smallest side? [JEE(Advanced) 2017, Paper-1,(3, 0)/61]

14. Let X be the set consisting of the first 2018 terms of the arithmetic progression 1, 6, 11, …., and Y be
the set consisting of the first 2018 terms of the arithmetic progression 9, 16, 23, ….. Then, the number

A
of elements in the set X ∪ Y is _____. [JEE(Advanced) 2018, Paper-1,(3, 0)/60]

PART - II : JEE (MAIN) / AIEEE PROBLEMS (PREVIOUS YEARS)

UH
1. A person is to count 4500 currency notes. Let an denote the number of notes he counts in the nth
minute. If a1 = a2 = .....= a10 = 150 and a10, a11,....are in an AP with common difference –2, then the time
taken by him to count all notes is [AIEEE 2010 (8, –2), 144]
(1) 34 minutes (2) 125 minutes (3) 135 minutes (4) 24 minutes
JA
2. A man saves Rs. 200 in each of the first three months of his service. In each of the subsequent months
his saving increases by Rs. 40 more than the saving of immediately previous month. His total saving
from the start of service will be Rs. 11040 after : [AIEEE 2011, I, (4, –1), 120]
(1) 18 months (2) 19 months (3) 20 months (4) 21 months

100 100

∑a ∑a
LP

3. Let an be the nth term of an A.P. If 2r = α and 2r −1 =β, then the common difference of the A.P. is :
r =1 r =1
[AIEEE 2011, II, (4, –1), 120]
α −β α −β
(1) α – β (2) (3) β – α (4)
100 200
A

4. The sum of first 20 terms of the sequence 0.7, 0.77, 0.777,....., is [AIEEE - 2013, (4, –1),360]
7 7 7 7
(1) (179 – 10–20) (2) (99 – 10–20) (3) (179 + 10–20) (4) (99 + 10–20)
NK

81 9 81 9

5. If (10)9 + 2(11)1 (10)8 + 3(11)2 (10)7 + . . . . . . . . + 10 (11)9 = k(10)9, then k is equal to


[JEE(Main) 2014, (4, – 1), 120]
121 441
(1) 100 (2) 110 (3) (4)
SA

10 100

6. Three positive numbers form an increasing G.P. If the middle term in this G.P. is doubled, the new
numbers are in A.P. Then the common ratio of the G.P. is [JEE(Main) 2014, (4, – 1), 120]
(1) 2 – 3 (2) 2 + 3 (3) 2 + 3 (4) 3 + 2

7. If m is the A. M. of two distinct real numbers l and n(l, n > 1) and G 1, G2 and G3 are three geometric
means between l and n, then G14 + 2 G24 + G34 equals : [JEE(Main) 2015, (4, – 1), 120]
(1) 4 l2 mm (2) 4 lm2 n (3) 4 lmn2 (4) 4 l2m2n2

13 13 + 23 13 + 23 + 33
8. The sum of first 9 terms of the series + + + ......... is :
1 1+ 3 1+ 3 + 5
[JEE(Main) 2015, (4, – 1), 120]
(1) 71 (2) 96 (3) 142 (4) 192
9. If the 2nd, 5th and 9th terms of a non-constant A.P. are in G.P., then the common ratio of this G.P. is:
[JEE(Main) 2016, (4, – 1), 120]
4 7 8
(1) (2) 1 (3) (4)
3 4 5
2 2 2 2
 3   2  1  4 16
10. If the sum of the first ten terms of the series  1  +  2  +  3  + 42 +  4  + ...... is m, then m
 5  5  5  5 5
is equal to : [JEE(Main) 2016, (4, – 1), 120]
(1) 101 (2) 100 (3) 99 (4) 102

11. For any three positive real numbers a, b and c, 9(25a2 + b2) + 25(c2 – 3ac) = 15b(3a + c), Then
[JEE(Main) 2017, (4, – 1), 120]
(1) b , c and a are in G.P. (2) b, c and a are in A.P.
(3) a, b and c are in A.P. (4) a, b and c are in G.P.

RI
12. Let a,b,c ∈ R. If f(x) = ax2 + bx + c is such that a + b + c = 3 and f(x + y) = f(x) + f(y) + xy, ∀ x,y ∈ R,
10
then ∑ f(n) is equal to
n =1
[JEE(Main) 2017, (4, – 1), 120]

(1) 330 (2) 165 (3) 190 (4) 225

A
13. If, for a positive integer n, the quadratic equation, x(x + 1) + (x + 1)(x + 2) +.....+ (x + n – 1)(x + n) = 10n
has two consecutive integral solutions, then n is equal to [JEE(Main) 2017, (4, – 1), 120]

UH
(1) 12 (2) 9 (3) 10 (4) 11
12
14. Let a1, a2, a3,……, a49 be in A.P. such that ∑a
k =0
4k +1 = 416 and a9 + a43 = 66 . If

a12 + a22 + ...... + a17


2
= 140 m , then m is equal to : [JEE(Main) 2018, (4, – 1), 120]
(1) 34 (2) 33
JA (3) 66 (4) 68

15. Let A be the sum of the first 20 terms and B be sum of the first 40 terms of the series
12+ 2.22 + 32 + 2.42 + 52 + 2. 62 + ..... If B – 2A = 100 λ, then λ is equal to :
[JEE(Main) 2018, (4, – 1), 120]
(1) 464 (2) 496 (3) 232 (4) 248
16. The sum of the following series
LP

9(12 + 2 2 + 3 2 ) 12(12 + 2 2 + 3 2 + 4 2 ) 15(12 + 2 2 + ..... + 5 2 )


1+ 6 + + + +…. up to 15 terms, is :
7 9 11
[JEE(Main) 2019, Online (09-01-19),P-2 (4, – 1), 120]
(1) 7510 (2) 7830 (3) 7520 (4) 7820
A

17. If 5, 5r, 5r2 are the lengths of the sides of a triangle, then r cannot be equal to :
[JEE(Main) 2019, Online (10-01-19),P-1 (4, – 1), 120]
NK

3 3 7 5
(1) (2) (3) (4)
2 4 4 4

18. The sum of all two digit positive numbers which when divided by 7 yield 2 or 5 as remainder is :
[JEE(Main) 2019, Online (10-01-19),P-1 (4, – 1), 120]
(1) 1356 (2) 1256 (3) 1365 (4) 1465
SA
EXERCISE # 1
PART-I
Section (A) :
A-1. 2, 5, 8,..... A-2. 160 A-3. 128 A-4. 19668 A-5. –(p + q) A-6. 4, 9, 14
A-9. 16
Section (B) :
B-1. 128 B-2. 2, 6, 18 or 18, 6, 2 B-3. 6, –3, 3/2, ...... B-4. 3, 7, 11 or 12, 7, 2

RI
q−r
B-5. B-7. 211
p−q
Section (C) :

A
1 14 14 14 2+n 8
C-1. C-2. , , C-6. (i) 4– (ii)
11 11 8 5 2n −1 3

UH
C-7. n.2n + 2 – 2n + 1 + 2.

Section (D) :
1
D-2. D-3. 2, 8
201
JA
Section (E) :
1 n . 2n – 2n + 1
E-1. (i) 2n + 2 – 3n – 4(ii)
(10n + 1 – 9n – 10) E-2.
27 2n
1 k+1 1 n(n + 1) n(n + 1)
E-3. (i) (3 + 1) – 2k+1 (ii) k(k + 1) (2k + 7) (iii) – if n is even, if n is odd
2 6 2 2
LP

3 2
(iv) 6265 (v) (n + 3n)
2
1 1 n
E-4. (i) – (ii) (n + 1) (n + 2) (n + 3) (2n + 3)
12 4(2n + 1)(2n + 3) 10
A

PART-II
NK

Section (A) :
A-1. (D) A-2. (D) A-3. (C) A-4. (C) A-5. (C)
Section (B) :
B-1. (B) B-2. (B) B-3. (B) B-4. (C) B-5. (A) B-6. (D)
SA

Section (C) :
C-1. (A) C-2. (D) C-3. (C) C-4. (D) C-5. (A) C-6. (A) C-7. (B)
C-8. (A) C-9. (B) C-10 (C)
Section (D) :
D-1. (D) D-2. (B) D-3. (A) D-4. (A) D-5. (D)
Section (E) :
E-1 (A) E-2. (D) E-3. (A) E-4. (C)
PART-III
1. (A) → (p), (B) → (s), (C) → (q), (D) → (q)
2. (A) → (s), (B) → (p), (C) → (s), (D) → (p)
EXERCISE # 2
PART-I
1. (A) 2. (D) 3. (A) 4. (C) 5. (A) 6. (A) 7. (C)
8. (A) 9. (C) 10. (D) 11. (A) 12. (A) 13. (C)

PART-II
1. 51 2. 2 3. 33 4. 8 5. 4 6. 11 7. 2
8. 0 9. 40 10. 0 11. 65 12. 54 13. 50 14. 64
15. 9 16. 7

RI
PART-III
1. (B) 2. (ABCD) 3. (BD) 4. (BD) 5. (ABCD) 6. (ABCD) 7. (D)

A
8. (BC) 9. (AB) 10. (AC) 11. (BC) 12. (ABCD) 13. (CD) 14. (ABC)
15. (CD) 16. (ABC) 17. (ABCD) 18. (AD) 19. (ACD)

UH
PART-IV
1. (A) 2. (D) 3. (A) 4. (B) 5. (A) 6. (B) 7. (A)

EXERCISE # 3
JA
PART – I
1. (C) 2. 3 3. 0
4. 3 or 9, both 3 and 9 (The common difference of the arithmatic progression can be either 0 or 6, and
accordingly the second term can be either 3, or 9 ; thus the answers 3, or 9, or both 3 and 9 are
acceptable.)

5. 8 6. (D) 7.* (AD) 8. 5 9. 4 10. 9 11. (C)


LP

12. (B) 13. 6 14. 3748

PART - II
A

1. (1) 2. (4) 3. (2) 4. (3) 5. (1) 6. (2) 7. (2)

8. (2) 9. (1) 10. (1) 11. (2) 12. (1) 13. (4) 14. (1)
NK

15. (4) 16. (4) 17. (3) 18. (1)


SA
1. Prove that 2, 3, 5 cannot be terms of a single A.P.

2. If the sum of the first m terms of an A.P. is equal to the sum of either the next n terms or the next p
 1 1  1 1
terms, then prove that (m + n)  −  = (m + p)  −  .
m p m n

3. If a and b are pth and qth terms of an AP, then find the sum of its (p + q) terms

4. In an A.P. of which ‘a’ is the Ist term, if the sum of the Ist ' p ' terms is equal to zero, show that the sum
a (p + q) q

RI
of the next ' q ' terms is − .
p −1

a + be y b + ce y c + de y
5. If = = , then show that a,b,c,d are in G.P.

A
y y
a – be b – ce c – de y

6. The sum of the first ten terms of an AP is 155 & the sum of first two terms of a GP is 9. The first term of

UH
the AP is equal to the common ratio of the GP & the first term of the GP is equal to the common
difference of the AP. Find the two progressions.

7. Find the sum in the nth group of sequence,


(i) (1), (2, 3); (4, 5, 6, 7); (8, 9,........, 15); ............
(ii) (1), (2, 3, 4), (5, 6, 7, 8, 9),........
JA
8. Let a, b be positive real numbers. If a, A1, A2, b are in arithmetic progression, a, G1, G2, b are in
geometric progression and a, H1, H2, b are in harmonic progression, show that
G1 G2 A + A 2 (2a + b) (a + 2b)
= 1 = .
H1 H2 H1 + H2 9 ab
LP

 n + 1  n+1
9. If total number of runs scored in n matches is   (2 – n – 2) where n > 1 and the runs scored in
 4 
the kth match are given by k. 2n+1–k, where 1 ≤ k ≤ n, find n
A

10. Let a1, a2,........, an be positive real numbers in geometric progression. For each n, let An, Gn, Hn be
respectively the arithmetic mean, geometric mean & harmonic mean of a 1, a2,......, an. Prove that
n 1
G= ∏ (A k Hk ) 2n , Where G is geometric mean between G1, G2, ........., Gn.
NK

K =1

p r
11. If a, b, c are in A.P., p, q, r are in H.P. and ap, bq, cr are in G.P., then find + .
r p
SA

12. If the sum of the roots of the quadratic equation, ax 2 + bx + c = 0 is equal to sum of the squares of their
a b c
reciprocals, then prove that , , are in H.P.
c a b

13. If a, b, c are in H.P.; b, c, d are in G.P.; and c, d, e are in A.P. such that (ka – b) 2 e = ab2 then value of k.

14. The value of x + y + z is 15 if a, x, y, z, b are in AP while the value of (1/x) + (1/y) + (1/z) is 5/3 if
a, x, y, z, b are in HP. Find a and b.

15. If n is a root of the equation x² (1 − ac) − x (a² + c²) − (1 + ac) = 0 and if n HM’s are inserted between a
and c, show that the difference between the first and the last mean is equal to ac(a − c).

16. If a, b, c are positive real numbers, then prove that


(i) b2c2 + c2a2 + a2b2 ≥ abc (a + b + c).
a b c 3
(ii) + + ≥
b+c c+a a+b 2
2 2 2 9
(iii) + + ≥
a+b b+c c+a a+b+c

17. Solve the equation (2 + x1 + x2 + x3 + x4)5 = 6250 x1 x2 x3 x4 .where x1, x2, x3, x4 > 0.

18. Let a1, a2, ........an, be real numbers such that


1 n(n − 3)
a1 + a2 − 1 + a3 − 2 + ...... + an − (n − 1) = (a + a2 + ......+ an) –
2 1 4
100
then find the value of ∑a
i =1
i

RI
 n −1   n −1  n
19. If ai ∈ R, i = 1, 2, 3, .......n and all ai 's are distinct such that  ∑ ai2  + 6  ∑ ai ai+1  + 9∑ ai2 ≤ 0
 i=1   i=1  i= 2

and a1 = 8 then find the sum of first five terms.

A
n n n n
20. Let {an} and {bn} are two sequences given by an = (x)1/ 2 + (y)1/ 2 and bn = (x)1/ 2 – (y)1/ 2 for all n ∈

UH
N. Then find a1a2a3........an.

10
aiai+1ai+ 2
21. Given that a1, a2, a3, .....an form an A.P. find then following sum ∑a
i =1 i + ai + 2

Given that a1 = 1 ; a2 = 2
JA
n n −1 n−2
22. Find sum of the series + + + . . . . up to n terms..
1. 2 . 3 2 . 3 . 4 3 . 4 . 5

n
3n . 5n
23. Find the value of Sn = ∑
n =1 (5n − 3n )(5n +1 − 3n +1 )
and hence S∞.
LP

24. Circles are inscribed in the acute angle α so that every neighbouring circles touch each other. If the
radius of the first circle is R, then find the sum of the radii of the first n circles in terms of R and α.

25. Let A, G, H be A.M., G.M. and H.M. of three positive real numbers a, b, c respectively such that
A

G2 = AH, then prove that a, b, c are terms of a GP.

n ∞
1 1
∑ ∑r
NK

26. If Sn = tr = n (2n2 + 9n + 13) , then equals


r =1 6 r =1 . tr

27. In the quadratic equation ax2 + bx + c = 0, a ≠ 0, ∆ = b2 – 4ac and α + β, α2 + β2, α3 + β3 are in G.P.
where α, β are the root of ax2 + bx + c = 0, then prove that c∆ = 0
SA

28. If sum of first n terms of an A.P. (having positive terms) is given by Sn = (1 + 2Tn) (1 – Tn) where Tn is
a− b
the nth term of series, then T22 = , (a ∈ N, b ∈ N), then find the value of (a + b)
4
p+q  a − b
3.  a+b+ 6. (3 + 6 + 12 +......) ; (2/3 + 25/3 + 625/6 +......) G.P.
2  p − q 

 25 79 83 
(2 + 5 + 8 + .....) ;  + + + ...........  A.P.
 2 6 6 

a c
7. (i) 2n − 2 (2n + 2n − 1 − 1) (ii) (n − 1)3 + n3 9. 7 11. + 13. 2
c a

RI
488 x−y 495
14. a = 1, b = 9 OR b = 1, a = 9 18. 5050 19. 20. 21.

A
81 bn 2

UH
n(n + 1) 3 R 1 − sin α
( 
) α n
2  1 + sin 2 

22. 23. 24. − 1 26. 1 28. 6
4(n + 2) 4 2 sin α  1 − sin α  
2  2 
JA
LP
A
NK
SA
SOLUTION OF TRIANGLES
JEE (Advanced) Syllabus
Relations between sides and angles of a triangle, sine rule, cosine rule, half-angle formula and
the area of a triangle.

According to most accounts, geometry was first discovered among the Egyptians, taking its origin from the measurement of
areas. For they found it necessary by reason of the flooding of the Nile, which wiped out everybody's proper boundaries. Nor
is there anything surprising in that the discovery both of this and of the other sciences should have had its origin in a
practical need, since everything which is in process of becoming progresses from the imperfect to the perfect.
...................Proclus

RI
Sine Rule :
In any triangle ABC, the sines of the angles are proportional to the opposite sides
a b c

A
i.e. = = .
sin A sin B sin C

UH
JA
Example # 1 : How many triangles can be constructed with the data : a = 5, b = 7, sin A = 3/4
a b 5 7
Solution : Since = ⇒ =
sin A sinB 3/4 sinB
21
⇒ sinB = > 1 not possible
20
∴ no triangle can be constructed.
LP

sin A sin(A – B)
Example # 2 : If in a triangle ABC, = , then show that a2, b2, c2 are in A.P.
sinC sin(B – C)
sin A sin(A – B)
A

Solution : We have =
sinC sin(B – C)
⇒ sin (B + C) sin (B–C) = sin (A + B) sin (A–B) ⇒ sin2B – sin2 C = sin2A – sin2B
⇒ b2 – c2 = a2 – b2 ⇒ a2, b2, c2 are in A.P.
NK

Self Practice Problems :


 A C B
(1) In a ∆ABC, the sides a, b and c are in A.P. , then prove that  tan + tan  : cot = 2 : 3
 2 2 2
(2) If the angles of ∆ABC are in the ratio 1 : 2 : 3, then find the ratio of their corresponding sides
SA

A B
tan + tan
c 2 2.
(3) In a ∆ABC prove that =
a−b A B
tan − tan
2 2
Ans. (2) 1 : 3 : 2

Cosine Formula :
In any ∆ABC
b2 + c 2 − a2
(i) cos A = or a² = b² + c² − 2bc cos A = b2 + c2 + 2bc cos (B + C)
2bc
c 2 + a2 − b2 a2 + b2 − c 2
(ii) cos B = (iii) cos C =
2c a 2ab
A –C a+c
Example # 3 : In a triangle ABC, A, B, C are in A.P. Show that 2cos   = .
 2  a – ac + c 2
2

Solution : A + C = 2B ⇒ A + B + C = 3B ⇒ B = 60º
a2 + c 2 – b2
∴ cos60º = ⇒ a2 – ac + c2 = b2
2ac
A+C A –C
2sin   cos  
a+c a+c  sin A + sinC   2   2 
⇒ = =   =
a2 – ac + c 2 b  sinB  sinB
A–C
= 2cos (  A + C = 2B)
2

Example # 4 : In a ∆ABC , prove that a (bcos C – c cosB) = b2 – c2

RI
a2 + b2 − c 2 a2 + c 2 − b2
Solution : Since cosC = & cos B =
2ab 2ac
  a2 + b2 − c 2   a2 + c 2 − b2  
∴ L.H.S. = a b   − c 

A
  2ab   2ac  
a2 + b2 − c 2 (a2 + c 2 − b2 )
= – = (b2 – c2) = R.H.S.
2 2

UH
Hence L.H.S. = R.H.S. Proved

Example # 5 : The sides of ∆ABC are AB = 13 cm, BC = 4 3 cm and CA = 7 cm. Then find the value of
sinθ where θ is the smallest angle of the triangle.
Solution : Angle opposite to AB is smallest . Therefore,
JA
49 + 48 – 13 3 1
cos θ = = ⇒ sinθ =
2.7.4 3 2 2

Self Practice Problems :


(4) If in a triangle ABC, 3 sinA = 6 sinB = 2 3 sinC, Then find the angle A.
LP

(5) If two sides a, b and angle A be such that two triangles are formed, then find the sum of two
values of the third side.
Ans. (4) 90º (5) 2b cosA

Projection Formula :
A

In any ∆ABC
(i) a = b cosC + c cosB (ii) b = c cosA + a cosC (iii) c = a cosB + b cosA
NK

A C 3b
Example # 6 : If in a ∆ABC, c cos2 + a cos2 = , then show that a, b, c are in A.P.
2 2 2
Solution : c (1 + cosA) + a (1 + cosC) = 3b
⇒ a + c + (c cosA + acosC) = 3b
⇒ a + c + b = 3b
SA

⇒ a + c = 2b

Example # 7 : In a ∆ABC, prove that (b + c) cos A + (c + a) cos B + (a + b) cos C = a + b + c.


Solution :  L.H.S. = (b + c) cos A + (c + a) cos B + (a + b) cos C
= b cos A + c cos A + c cos B + a cos B + a cos C + b cos C
= (b cos A + a cos B) + (c cos A + a cos C) + (c cos B + b cos C)
=a+b+c
= R.H.S.
Hence L.H.S. = R.H.S. Proved
Self Practice Problems :

(6) The roots of x2 – 2 3x + 2 = 0 represent two sides of a triangle. If the angle between them is
π
, then find the perimeter of triangle.
3
(7) In a triangle ABC, if cos A + cosB + cos C = 3/2, then show that the triangle is an equilateral
triangle.

cos A cosB cosC a2 + b2 + c 2


(8) In a ∆ABC, prove that + + = .
c cosB + b cosC a cosC + c cos A a cosB + b cos A 2abc
Ans. (6) 2 3 + 6

Napier’s Analogy - tangent rule :

RI
In any ∆ABC
B−C b − c A C − A c − a B
(i) tan = cot (ii) tan = cot
2 b+ c 2 2 c +a 2

A
A −B a−b C
(iii) tan = cot
2 a+b 2

UH
Example # 8 : Find the unknown elements of the ∆ABC in which a = 3 + 1, b = 3 – 1, C = 90°.

Solution :  a= 3 + 1, b = 3 – 1, C = 90°
 A + B + C = 180°
JA
∴ A + B = 90° .......(i)
 A −B  a−b C
 From law of tangent, we know that tan   = cot
 2  a+b 2
( 3 + 1) − ( 3 − 1) 2  A −B  1
= cot 45° = cot 45° ⇒ tan   =
( 3 + 1) + ( 3 − 1) 2 3  2  3
A −B π
LP

∴ =
2 6
π
⇒ A–B= .......(ii)
3
5π π
A

From equation (i) and (ii), we get A = and B=


12 12
Now, c= a2 + b2 = 2 2
5π π
NK

∴ c= 2 2,A= ,B= Ans.


12 12

Self Practice Problems :


7 A
(9) In a ∆ABC if b = 3, c = 5 and cos (B – C) = , then find the value of sin .
SA

25 2

B−C A C−A B
(10) If in a ∆ABC, we define x = tan   tan 2 , y = tan  2  tan 2 and
 2   
 A −B  C
z = tan   tan 2 , then show that x + y + z = – xyz.
 2 
1
Ans. (9)
10

Trigonometric Functions of Half Angles :


A (s − b) (s − c) B (s − c) (s − a) C (s − a) (s − b)
(i) sin = , sin = , sin =
2 bc 2 ca 2 ab

A s (s − a) B s (s − b) C s (s − c)
(ii) cos = , cos = , cos =
2 bc 2 ca 2 ab

A (s − b) (s − c) ∆ (s − b)(s − c) a+b+c
(iii) tan = = = , where s = is semi perimeter and
2 s(s − a) s(s − a) ∆ 2
∆ is the area of triangle.
2 2∆
(iv) sinA = s(s − a)(s − b)(s − c) =
bc bc

RI
Area of Triangle (∆)
1 1 1
∆= ab sin C = bc sin A = ca sin B = s(s − a) (s − b) (s − c)
2 2 2

A
Example # 9 : If p1, p2, p3 are the altitudes of a triangle ABC from the vertices A, B, C and ∆ is the area of the
s–c
triangle, then show that p1–1 + p2–1 – p3–1 =

UH
Solution : We have
1 1 1 a b c
+ – = + –
p1 p2 p3 2∆ 2∆ 2∆
a + b – c 2(s – c) s – c
= = =
2∆ 2∆ ∆
JA
Example # 10 : In a ∆ABC if b sinC(b cosC + c cosB) = 64, then find the area of the ∆ABC.
Solution :  b sinC (b cosC + c cosB) = 64 ........(i) given
 From projection rule, we know that
a = b cosC + c cosB put in (i), we get
ab sinC = 64 ........(ii)
LP

1
 ∆ = ab sinC ∴ from equation (ii), we get
2
∴ ∆ = 32 sq. unit

s2
A

A B C
Example # 11 : If A,B,C are the angle of a triangle, then prove that cot + cot + cot =
2 2 2 ∆
A B C
Solution : cot + cot + cot
NK

2 2 2
s(s – a) s(s – b) s(s – c)
= + +
(s – b)(s – c) (s – c)(s – a) (s – a)(s – b)
s(s – a + s – b + s – c) s s2
= = (3s – 2s) =
(s – a)(s – b)(s – c) ∆ ∆
SA

m - n Rule : In any triangle ABC if D be any point on the base BC, such that BD : DC :: m : n and if
∠BAD= α, ∠DAC = β, ∠ CDA = θ , then
(m + n) cot θ = m cot α − n cot β
n cotB − m cot C
Example # 12 : In a ∆ABC . AD divides BC in the ratio 2 : 1 such that at ∠BAD = 90º then prove that
tanA + 3tanB = 0
Solution : From the figure , we see that θ = 90º + B (as θ is external angle of ∆ABD)

Now if we apply m-n rule in ∆ABC, we get


(2 + 1) cot (90º + B) = 2. cot 90° – 1.cot (A – 90°)
⇒ – 3 tan B = cot (90° – A)

RI
⇒ – 3 tan B = tan A
⇒ tan A + 3 tan B = 0 Hence proved.

Example # 13 : The base of a ∆ is divided into three equal parts . If α, β, γ be the angles subtended by these

A
parts at the vertex, prove that :
(cotα + cotβ) (cotβ+ cotγ) = 4cosec2β
Solution : Let point D and E divides the base BC into three equal parts i.e. BD = DE = EC = d (Let) and
let α, β and γ be the angles subtended by BD, DE and EC respectively at their opposite vertex.

UH
Now in ∆ABC
 BE : EC = 2d : d = 2 : 1
∴ from m-n rule, we get
(2 + 1) cotθ = 2 cot (α + β) – cotγ
⇒ 3cotθ = 2 cot (α + β) – cotγ .........(i)
JA
LP
A

again
 in ∆ADC
 DE : EC = d : d = 1 : 1
∴ if we apply m-n rule in ∆ADC, we get
NK

(1 + 1) cotθ = 1. cotβ – 1 cotγ


2cotθ = cotβ – cotγ .........(ii)
3 cot θ 2cot(α + β) − cot γ
from (i) and (ii), we get =
2cot θ cot β − cot γ
⇒ 3cotβ – 3cotγ = 4cot (α + β) – 2 cotγ
SA

⇒ 3cotβ – cotγ = 4 cot (α + β)


 cot α.cot β − 1
⇒ 3cotβ – cotγ = 4  
 cot β + cot α 
⇒ 3cot2β + 3cotα cotβ – cotβ cotγ – cotα cotγ = 4 cotα cotβ – 4
⇒ 4 + 3cot2β = cotα cotβ + cotβ cotγ + cotα cotγ
⇒ 4 + 4cot2β = cotα cotβ + cotα cotγ + cotβ cotγ + cot2β
⇒ 4(1 + cot2β) = (cotα + cotβ) (cotβ + cotγ)
⇒ (cotα + cotβ) (cotβ+ cotγ) = 4cosec2β

Self Practice Problems :


1
(11) In a ∆ABC, the median to the side BC is of length unit and it divides angle A into
11 − 6 3
the angles of 30° and 45°. Prove that the side BC is of length 2 unit.
Radius of Circumcirlce :
a b c abc
If R be the circumradius of ∆ABC, then R = = = =
2 sin A 2 sinB 2 sinC 4∆

Example # 14 : In a ∆ABC , prove that sin2A + sin2B + sin2C = 2∆/R2


a b c
Solution : In a ∆ABC, we know that = = = 2R
sin A sinB sinC
and sin2A + sin2B + sin2C = 4sinAsinBsinC
4abc 16∆R 2∆
= = = 2
8R3 8R3 R

RI
Example # 15 : In a ∆ABC if a = 22 cm, b = 28 cm and c = 36 cm, then find its circumradius.
abc
Solution :  R= .......(i)
4∆

A
 ∆ = s(s − a)(s − b)(s − c)
a+b+c
 s= = 43 cm

UH
2
∴ ∆ = 43 × 21× 15 × 7 = 21 215
22 × 28 × 36 264
∴ R= = cm
4 × 21 215 215
JA
Example # 16 : In a ∆ABC, if 8R2 = a2 + b2 + c2 , show that the triangle is right angled.
Solution : We have : 8R2 = a2 + b2 + c2
⇒ 8R2 = [4R2 sin2A + 4R2 sin2B + 4R2 sinC] [ a = 2R sin A etc.]
⇒ 2 = sin2A + sin2B + sin2C ⇒ (1 – sin2A) – sin2 B + (1 – sin2C) = 0
⇒ (cos2 A – sin2B) + cos2 C = 0 ⇒ cos (A + B) cos (A – B) + cos2C = 0
⇒ –cos C cos (A – B) + cos2 C = 0 ⇒ – cos C {cos (A – B) – cos C} = 0
⇒ –cos C[cos (A – B) + cos(A + B)] = 0 ⇒ – 2cos A cosB cos C = 0
LP

⇒ cos A = 0 or cos B = 0 or cos C = 0


π π π
⇒A= or B = or C =
2 2 2
⇒ ∆ABC is a right angled triangle.
A

b2 – c 2
Example # 17 : = R sin (B – C)
2a
NK

b2 – c 2 4R2 (sin2 B – sin2 C) R sin(B + C)sin(B – C)


Solution : = = = R sin (B – C)
2a 4R sin A sin A

Self Practice Problems :


SA

 A –B C
(12) In a ∆ABC, prove that (a + b) = 4R cos   cos 2
 2 
4
(13) In a ∆ABC , if b = 15 cm and cos B = , find R.
5

(14) In a triangle ABC if α, β, γ are the distances of the vertices of triangle from the corresponding
αβγ
points of contact with the incircle, then prove that = r2
α+β+ γ
Ans. (13) 12.5

Radius of The Incircle :


If ‘r’ be the inradius of ∆ABC, then
∆ A B C
(i) r= (ii) r = (s − a) tan = (s − b) tan = (s − c) tan
s 2 2 2
B C
a sin sin
2 2 and so on A B C
(iii) r= (iv) r = 4R sin sin sin
A 2 2 2
cos
2
Radius of The Ex- Circles :
If r1, r2 , r3 are the radii of the ex-circles of ∆ABC opposite to the vertex A, B, C respectively, then
∆ ∆ ∆
(i) r1 = ; r2 = ; r3 = ;
s−a s−b s−c
A B C
(ii) r1 = s tan ; r2 = s tan ; r3 = s tan

RI
2 2 2
a cos B2 cos C2 A B C
(iii) r1 = and so on (iv) r1 = 4 R sin . cos . cos
cos A2 2 2 2
 r 

A
Example # 18 : cos A + cosB + cos C =  1 + 
 R
Solution : LHS = cosA + cosB + cosC
 A +B  A –B C

UH
= 2 cos   cos  2  + 1 – 2 sin 2
2

 2   
C   A –B C C   A –B  A + B 
= 2 sin cos   – sin 2  + 1 = 2sin 2 cos  2  – cos  2   + 1
2   2       
C  A B A B C
JA
= 2sin 2sin sin  + 1 = 1 + 4 sin sin sin
2  2 2 2 2 2
1  A B C r
=1+  4R sin sin sin  = 1 + = RHS
R  2 2 2 R

b–c c–a a–b


Example # 19 : In a triangle ABC, find the value of + + .
r1 r2 r3
LP

b–c c–a a–b


Solution : + +
 ∆   ∆   ∆ 
s–a s–b s–c
     
1
A

= [ (b – c) (s – a) + (c – a) (s – b) + (a – b) (s – c)]

1
= [s(b – c + c – a + a – b) –a (b – c) – b(c – a) – c (a – b)] = 0

NK

Self Practice Problems :

(15) In a triangle ABC, r1 , r2, r3 are in HP. If its area is 24 cm2 and its perimeter is 24 cm. then find
lengths of its sides.
SA

(16) In a triangle ABC, a : b : c = 4 : 5 : 6 . Find the ratio of the radius of the circumcircle to that of
the incircle.
r1 − r r −r c
(17) In a ∆ABC, prove that + 2 = .
a b r3

(18) If A, A1, A2 and A3 are the areas of the inscribed and escribed circles respectively of a ∆ABC,
1 1 1 1
then prove that = + + .
A A1 A2 A3
Ans. (15) 6, 8 , 10 (16) 16 : 7
Length of Angle Bisectors, Medians & Altitudes :
2bc cos A
(i) Length of an angle bisector from the angle A = βa = 2 ;
b+c
1
(ii) Length of median from the angle A = ma = 2b2 + 2c 2 − a2
2
2∆
& (iii) Length of altitude from the angle A = Aa =
a
3 2
NOTE : ma2 + mb2 + mc2 = (a + b2 + c2)
4

RI
π π
Example # 20 : In ∆ABC, AD & BE are its two median . If AD = 4 , ∠DAB = and ∠ABE = then find the
6 3

A
length of BE and area of ∆ABC.
2 8 4
Solution : AP = ; AD = ; PD = ; Let PB = x
3 3 3

UH
8/3 8
tan 60º = or x=
x 3 3
1 8 8 32
Area of ∆ABP = × × =
2 3 3 3 9 3
JA
32 32
∴ Area of ∆ABC = 3 × =
9 3 3 3
3 4
Also, BE = x=
2 3

Self Practice Problem :


LP

(19) In a ∆ABC if ∠A = 90º, b = 5 cm, c = 12 cm. If ‘G’ is the centroid of triangle, then find
circumradius of ∆GAB.
13 601
A

Ans. (19) cm
30
NK

The Distances of The Special Points from Vertices and Sides of Triangle :
(i) Circumcentre (O) : OA = R and Oa = R cos A

A
(ii) Incentre (Ι) : ΙA = r cosec and Ιa = r
2
SA

A
(iii) Excentre (Ι1) : Ι1 A = r1 cosec and Ι1a = r1
2
(iv) Orthocentre (H) : HA = 2R cos A and Ha = 2R cos B cos C

1 2∆
(v) Centroid (G) : GA = 2b2 + 2c 2 − a2 and Ga =
3 3a

Example # 21 : If p1,p2, p3 are respectively the lengths of perpendiculars from the vertices of a triangle ABC to
the opposite sides, prove that :
cos A cosB cosC 1 bp1 cp2 ap3 a2 + b2 + c 2
(i) + + = (ii) + + =
p1 p2 p3 R c a b 2R
1 a 1 b 1 c
Solution : (i) use = , = , =
p1 2∆ p2 2∆ p3 2∆
1
∴ LHS = (a cosA + b cosB + c cosC)
2∆
R 4R sin A sinB sinC
= (sin 2A + sin 2B + sin 2C) =
2∆ 2∆
4R a b c 1 1 1
= . . . = abc = .(4R∆ ) = = RHS
2∆ 2R 2R 2R 4∆R2 4∆R2 R

bp1 cp2 ap3 a2 + b2 + c 2 2b∆ 2c∆ 2a∆ 2∆(a2 + b2 + c 2 )


(ii) LHS = + + = = + + =
c a b 2R ac ab bc abc
2∆(a + b + c )
2 2 2 2 2
a +b +c 2
= =
4∆R 2R

RI
Self Practice Problems :

A B C
(20) If Ι be the incentre of ∆ABC, then prove that ΙA . ΙB . ΙC = abc tan tan tan .

A
2 2 2

(21) If x, y, z are respectively be the perpendiculars from the circumcentre to the sides of ∆ABC,

UH
a b c abc
then prove that + + = .
x y z 4xyzJA
LP
A
NK
SA
 Marked questions are recommended for Revision.

SUBJECTIVE QUESTIONS
Section (A) : Sine rule, Cosine rule, Napier's Analogy, Projection rule

A-1. In a ∆ABC, prove that :


(i) a sin (B – C) + b sin (C – A) + c sin (A – B) = 0
a2 sin(B − C) b2 sin(C − A) c 2 sin(A − B)
(ii) + + =0
sin A sinB sinC

RI
(iii) 2(bc cos A + ca cos B + ab cos C) = a2 + b2 + c2
C C
(iv) (a – b)2 cos2 + (a + b)2 sin2 = c2
2 2
(v) b2 sin 2C + c2 sin 2B = 2bc sin A
c − a cosB

A
sinB
(vi) =
sinC b − a cosC

A-2. Find the real value of x such that x2 + 2x, 2x + 3 and x2 + 3x + 8 are lengths of the sides of a triangle.

UH
A-3. The angles of a ∆ABC are in A.P. (order being A, B, C) and it is being given that b : c = 3 : 2 , then
find ∠A.

C
JA
A-4. If cos A + cos B = 4 sin2   , prove that sides a, c, b of the triangle ABC are in A.P.
2

sin A sin(A − B)
A-5. If in a ∆ABC, = , then prove that a2, b2, c2 are in A.P.
sinC sin(B − C)

In a triangle ABC, prove that for any angle θ, b cos (A − θ) + a cos (B + θ) = c cos θ.
LP

A-6.

b + c c + a a + b cos A cosB cosC


A-7. With usual notations, if in a ∆ ABC, = = , then prove that = = .
11 12 13 7 19 25
A

A-8. Let a, b and c be the sides of a ∆ABC. If a2, b2 and c2 are the roots of the equation
cos A cosB cosC
x3 – Px2 + Qx – R = 0, where P, Q & R are constants, then find the value of + + in
a b c
NK

terms of P, Q and R.

A-9. If in a triangle ABC, the altitude AM be the bisector of ∠BAD, where D is the mid point of side BC, then
prove that (b2 − c2) = a2/2.

1 1 3
If in a triangle ABC, ∠C = 60º , then prove that
SA

A-10. + =
a+c b+c a+b+c

A-11. In a triangle ABC, ∠C = 60º and ∠A = 75º. If D is a point on AC such that the area of the ∆ABD is
3 times the area of the ∆BCD, find the ∠ABD.

C
A-12. In a scalene triangle ABC, D is a point on the side AB such that CD 2 = AD. DB, if sinA. sinB = sin2
2
then prove that CD is internal bisector of ∠C.
A-13. In triangle ABC, D is on AC such that AD = BC , BD = DC, ∠DBC = 2x, and ∠BAD = 3x, all angles are
in degrees, then find the value of x.

Section (B) Trigonometric ratios of Half Angles, Area of triangle and circumradius

B-1. In a ∆ABC, prove that


 C A
(i) 2 a sin2 + c sin2  = c + a – b.
 2 2
A B C
cos2 cos2 cos2 2
(ii) 2 + 2 + 2 = s
a b c abc
 A B C
(iii) 4  bc.cos2 + ca.cos2 + ab.cos2  = (a + b + c)2
 2 2 2

A B C
(iv) (b – c) cot + (c – a) cot + (a – b) cot =0
2 2 2

(v) 4∆ (cot A + cot B + cot C) = a2 + b2 + c2

RI
 2abc  A B C
(vi)  a + b + c  . cos 2 .cos 2 . cos 2 = ∆
 

A C

A
B-2. If the sides a, b, c of a triangle are in A.P., then find the value of tan + tan in terms of
2 2
cot(B/2).

UH
B-3. If in a ∆ ABC, a = 6, b = 3 and cos(A − B) = 4/5, then find its area.

3 a2
B-4. If in a triangle ABC, ∠A = 30º and the area of triangle is , then prove that either
4
B = 4 C or C = 4 B.
JA
Section (C) Inradius and Exradius

C-1. In any ∆ABC, prove that



(i) R r (sin A + sin B + sin C) = ∆ (ii) a cos B cos C + b cos C cos A + c cos A cos B =
R
LP

1 1 1 1 A B C r
(iii) + + = . (iv) cos2 + cos2 + cos2 =2+
ab bc ca 2Rr 2 2 2 2R

(v) a cot A + b cot B + c cot C = 2(R + r)


A

C-2. In any ∆ABC, prove that


(i) r. r1 .r2 .r3 = ∆2
1 1 1 1 a2 + b2 + c 2
(ii) r1 + r2 – r3 + r = 4R cos C. (iii) + 2 + 2 + 2 =
NK

r 2
r1 r2 r3 ∆2
2
1 1 1 1  4 1 1 1 b c − r2 r3 c a − r3 r1 ab − r1 r2
(iv)  + + +  =  + +  (v) = = =r
 r r1 r2 r3  r  r1 r2 r3  r1 r2 r3
SA

C-3. Show that the radii of the three escribed circles of a triangle are roots of the equation
x3 − x2 (4 R + r) + x s2 − r s2 = 0.

C-4. The radii r1, r2, r3 of escribed circles of a triangle ABC are in harmonic progression. If its area is 24 sq.
cm and its perimeter is 24 cm, find the lengths of its sides.
C-5. If the area of a triangle is 100 sq.cm, r1 = 10 cm and r2 = 50 cm, then find the value of (b − a).

Section (D) Miscellaneous

D-1. If α, β, γ are the respective altitudes of a triangle ABC, prove that


1 1 1 cot A + cot B + cot C 1 1 1 2ab C
(i) + 2 + 2 = (ii) + − = cos2
α 2
β γ ∆ α β γ (a + b + c) ∆ 2

D-2. If in an acute angled ∆ABC, line joining the circumcentre and orthocentre is parallel to side AC, then
find the value of tan A.tan C.
D-3. A regular hexagon & a regular dodecagon are inscribed in the same circle. If the side of the dodecagon
is ( )
3 − 1 , if the side of the hexagon is 4
k , then find value of k.

D-4. If D is the mid point of CA in triangle ABC and ∆ is the area of triangle, then show that
4∆
tan (∠ADB) = 2 2 .
a −c

 Marked questions are recommended for Revision.

RI
PART-I (OBJECTIVE QUESTIONS)
Section (A) : Sine rule, Cosine rule, Napier's Analogy, Projection rule

A
A-1. In a ∆ABC, A : B : C = 3 : 5 : 4. Then a + b + c 2 is equal to
(A) 2b (B) 2c (C) 3b (D) 3a

UH
A-2*. In a triangle ABC, the altitude from A is not less than BC and the altitude from B is not less than AC.
The triangle is
(A) right angled (B) isosceles (C) obtuse angled (D) equilateral

cos A cosB cosC


JA
A-3. If in a ∆ ABC, = = , then the triangle is :
a b c
(A) right angled (B) isosceles (C) equilateral (D) obtuse angled

bc sin2 A
A-4. In a ∆ABC is equal to
cos A + cosBcosC
(A) b2 + c2 (B) bc (C) a2 (D) a2 + bc
LP

2(tan A + tanC).tan2 B
A-5 Given a triangle ∆ABC such that sin2A + sin2C = 1001.sin2B. Then the value of is
tan A + tanB + tanC
1 1 1 1
(A) (B) (C) (D)
A

2000 1000 500 250

A-6. If in a triangle ABC, (a + b + c) (b + c − a) = k. b c, then :


NK

(A) k < 0 (B) k > 6 (C) 0 < k < 4 (D) k > 4

A-7. In a triangle ABC, a: b: c = 4: 5: 6. Then 3A + B equals to :


(A) 4C (B) 2π (C) π − C (D) π
SA

A-8. The distance between the middle point of BC and the foot of the perpendicular from A is :
−a2 + b2 + c 2 b2 − c 2 b2 + c 2 b2 + c 2
(A) (B) (C) (D)
2a 2a bc 2a

A-9*. If in a triangle ABC, cos A cos B + sin A sin B sin C = 1, then the triangle is
(A) isosceles (B) right angled (C) equilateral (D) None of these

A-10.Triangle ABC is right angle at A. The points P and Q are on hypotenuse BC such that BP = PQ = QC.
If AP = 3 and AQ = 4, then length BC is equal to
(A) 3 5 (B) 5 3 (C) 4 5 (D) 7

A-11. In ∆ABC, bc = 2b2 cosA + 2c2 cosA – 4bc cos2 A, then ∆ABC is
(A) isosceles but not necessarily equilaterial
(B) equilateral
(C) right angled but not neccessarily isosceles
(D) right angled isosceles

Section (B) Trigonometric ratios of Half Angles, Area of triangle and circumradius

B-1. If in a triangle ABC, right angle at B, s − a = 3 and s − c = 2, then


(A) a = 2, c = 3 (B) a = 3, c = 4 (C) a = 4, c = 3 (D) a = 6, c = 8

A B 3
B-2. If in a triangle ABC, b cos2 + a cos2 = c, then a, c, b are :
2 2 2
(A) in A.P. (B) in G.P. (C) in H.P. (D) None

B-3. If H is the orthocentre of a triangle ABC, then the radii of the circle circumscribing the triangles BHC,
CHA and AHB are respectively equal to :
R R R

RI
(A) R, R, R (B) 2 R , 2 R , 2 R (C) 2R, 2R, 2R (D) , ,
2 2 2

B C
B-4. In a ∆ ABC if b + c = 3a, then cot · cot has the value equal to:
2 2

A
(A) 4 (B) 3 (C) 2 (D) 1

2π 9 3

UH
B-5. In a ∆ABC, A = , b – c = 3 3 cm and area (∆ABC) = cm2. Then ‘a’ is
3 2
(A) 6 3 cm (B) 9 cm (C) 18 cm (D) 7 cm

B-6.* The diagonals of a parallelogram are inclined to each other at an angle of 45º, while its sides a and
a
JA
b(a > b) are inclined to each other at an angle of 30º, then the value of is
b
3+ 5 3+ 5 5 +1
(A) 2cos36º (B) (C) (D)
4 4 2

B-7. If in a ∆ABC, ∆ = a2 – (b – c)2, then tan A is equal to


LP

(A) 15/16 (B) 8/15 (C) 8/17 (D) 1/2

31
B-8*. If in a ∆ABC, a = 5, b = 4 and cos (A – B) = , then
32
5 7 
(B) sin A =  
A

(A) c = 6
 16
 
15 7
(C) area of ∆ABC= (D) c = 8
NK

4
b2 − c 2
B-9. If R denotes circumradius, then in ∆ABC, is equal to
2a R
(A) cos (B – C) (B) sin (B – C) (C) cos B – cos C (D) sin(B + C)
SA

B-10*. Which of the following holds good for any triangle ABC?
cos A cosB cosC a2 + b2 + c 2 sin A sinB sinC 3
(A) + + = (B) + + =
a b c 2abc a b c 2R
cos A cosB cosC sin 2A sin 2B sin 2C
(C) = = (D) = =
a b c a2 b2 c2

B-11. A triangle is inscribed in a circle. The vertices of the triangle divide the circle into three arcs of length 3,
4 and 5 units. Then area of the triangle is equal to:
9 3 (1 + 3 ) 9 3 ( 3 − 1) 9 3 (1 + 3 ) 9 3 ( 3 − 1)
(A) (B) (C) (D)
π2 π2 2 π2 2 π2

B-12. In a ∆ABC, a = 1 and the perimeter is six times the arithmetic mean of the sines of the angles. Then
measure of ∠ A is
π π π π
(A) (B) (C) (D)
3 2 6 4

B-13*. Three equal circles of radius unity touches one another. Radius of the circle touching all the three
circles is :
2− 3 3 − 2 2+ 3 3 + 2
(A) (B) (C) (D)
3 2 3 2

B-14. Triangle ABC is isosceles with AB = AC and BC = 65 cm. P is a point on BC such that the
perpendicular distances from P to AB and AC are 24 cm and 36 cm, respectively. The area of triangle
ABC (in sq. cm is)
(A) 1254 (B) 1950 (C) 2535 (D) 5070

RI
Section (C) Inradius and Exradius

A
a cos A + b cosB + c cosC
C-1. In a ∆ ABC, the value of is equal to:
a+b+c
r R R 2r
(A) (B) (C) (D)

UH
R 2r r R

C-2. In a triangle ABC, if a : b : c = 3 : 7 : 8, then R : r is equal to


(A) 2 : 7 (B) 7 : 2 (C) 3 : 7 (D) 7 : 3
JA
C-3*. If r1 = 2r2 = 3r3 , then
a 4 a 5 a 3 a 5
(A) = (B) = (C) = (D) =
b 5 b 4 c 5 c 3

C-4*. In a ∆ABC, following relations hold good. In which case(s) the triangle is a right angled triangle?
(A) r2 + r3 = r1 − r (B) a2 + b2 + c2 = 8 R2 (C) r1 = s (D) 2 R = r1 − r
LP

C-5. The perimeter of a triangle ABC right angled at C is 70, and the inradius is 6, then |a – b| equals
(A) 1 (B) 2 (C) 8 (D) 9
a−b s−a
A

C-6. In a triangle ABC, if = , then r1, r2, r3 are in:


b−c s−c
(A) A.P. (B) G.P. (C) H.P. (D) none of these
NK

C-7. If the incircle of the ∆ ABC touches its sides at L, M and N as shown in the figure and if x, y, z be the
circumradii of the triangles MIN, NIL and LIM respectively, where Ι is the incentre, then the product xyz
is equal to :
SA

1 2 1 2
(A) R r2 (B) r R2 (C) Rr (D) rR
2 2

r 1 A  B C
C-8. If in a ∆ABC, = , then the value of tan  tan + tan  is equal to :
r1 2 2  2 2
1
(A) 2 (B) (C) 1 (D) 3
2
π C
C-9. If in a ∆ABC, ∠A = , then tan is equal to
2 2
a−c a−b a−c a−b
(A) (B) (C) (D)
2b 2c b c

(r1 + r2 ) (r2 +r3 )(r3 +r1 )


C-10. In any ∆ABC, is always equal to
Rs2
(A) 8 (B) 27 (C) 16 (D) 4

C-11*. In a triangle ABC, right angled at B, then


AB + BC − AC AB + AC − BC AB + BC + AC s–r
(A) r = (B) r = (C) r = (D) R =
2 2 2 2

RI
C-.12*. With usual notations, in a ∆ ABC the value of Π (r 1 − r) can be simplified as:
2
A ( abc )
(A) abc Π tan (B) 4 r R2 (C) 2
(D) 4 R r2
2

A
R (a + b + c )

C-13. STATEMENT-1 : In a triangle ABC, the harmonic mean of the three exradii is three times the inradius.

UH
STATEMENT-2 : In any triangle ABC, r1 + r2 + r3 = 4R.
(A) STATEMENT-1 is true, STATEMENT-2 is true and STATEMENT-2 is correct explanation for
STATEMENT-1
(B) STATEMENT-1 is true, STATEMENT-2 is true and STATEMENT-2 is not correct explanation for
STATEMENT-1
JA
(C) STATEMENT-1 is true, STATEMENT-2 is false
(D) STATEMENT-1 is false, STATEMENT-2 is true
(E) Both STATEMENTS are false

Section (D) Miscellaneous


LP

D-1. If in a triangle ABC, the line joining the circumcentre and incentre is parallel to BC, then
cos B + cos C is equal to :
(A) 0 (B) 1 (C) 2 (D) 1/2
A

D-2. In a ∆ABC, if AB = 5 cm, BC = 13 cm and CA = 12 cm, then the distance of vertex ‘A’ from the side BC
is (in cm)
25 60 65 144
NK

(A) (B) (C) (D)


13 13 12 13

D-3. If AD, BE and CF are the medians of a ∆ABC, then (AD2 + BE2 + CF2) : (BC2 + CA2 + AB2) is equal to
(A) 4 : 3 (B) 3 : 2 (C) 3 : 4 (D) 2 : 3
SA

D-4*. In a triangle ABC, with usual notations the length of the bisector of internal angle A is :
A
2bc cos A 2bc sin A abc cos ec
2∆ .
(A) 2 (B) 2 (C) 2 (D) cos ec A
b+c b+c 2R (b + c) b+c 2

D-5. Let f, g, h be the lengths of the perpendiculars from the circumcentre of the ∆ ABC on the sides BC, CA
a b c abc
and AB respectively. If + + = λ , then the value of 'λ' is:
f g h f gh
(A) 1/4 (B) 1/2 (C) 1 (D) 2

D-6. In an acute angled triangle ABC, AP is the altitude. Circle drawn with AP as its diameter cuts the sides
AB and AC at D and E respectively, then length DE is equal to
∆ ∆ ∆ ∆
(A) (B) (C) (D)
2R 3R 4R R

D-7. AA1, BB1 and CC1 are the medians of triangle ABC whose centroid is G. If points A, C 1, G and B1 are
concyclic, then
(A) 2b2 = a2 + c2 (B) 2c2 = a2 + b2 (C) 2a2 = b2 + c2 (D) 3a2 = b2 + c2

D-8. If '' is the length of median from the vertex A to the side BC of a ∆ABC, then
(A) 42 = b2 + 4ac cos B (B) 42 = a2 + 4bc cos A (C) 42 = c2 + 4ab cos C (D) 42 = b2 + 2c2 – 2a2

D-9*. The product of the distances of the incentre from the angular points of a ∆ ABC is:

(A) 4 R2 r (B) 4 Rr2 (C)


(a b c ) R (D)
(a b c ) r

RI
s s

D-10. In a triangle ABC, B = 60° and C = 45°. Let D divides BC internally in the ratio 1 : 3,
sin ∠BAD

A
then value of is
sin ∠CAD
2 1 1 1
(A) (B) (C) (D)

UH
3 3 6 3

D-11*. In a triangle ABC, points D and E are taken on side BC such that BD = DE = EC. If angle
ADE = angle AED = θ, then:
6 tan θ
(A) tanθ = 3 tan B (B) 3 tanθ = tanC (C) = tanA (D) angle B = angle C
tan2 θ − 9
JA
D-12. STATEMENT-1 : If R be the circumradius of a ∆ABC, then circumradius of its excentral ∆Ι 1Ι2Ι3 is 2R.
R
STATEMENT-2 : If circumradius of a triangle be R, then circumradius of its pedal triangle is .
2
(A) STATEMENT-1 is true, STATEMENT-2 is true and STATEMENT-2 is correct explanation for
STATEMENT-1
(B) STATEMENT-1 is true, STATEMENT-2 is true and STATEMENT-2 is not correct explanation
LP

for STATEMENT-1
(C) STATEMENT-1 is true, STATEMENT-2 is false
(D) STATEMENT-1 is false, STATEMENT-2 is true
(E) Both STATEMENTS are false
A

PART-II (COMPREHENSION)
Comprehension # 1 (Q. No. 1 to 4)
NK

The triangle DEF which is formed by joining the feet of the altitudes
of triangle ABC is called the Pedal Triangle.
Answer The Following Questions :
SA

1. Angle of triangle DEF are


(A) π − 2A, π − 2B and π − 2C (B) π + 2A, π + 2B and π + 2C
(C) π − A, π − B and π − C (D) 2π − A, 2π − B and 2π − C

2*. Sides of triangle DEF are


(A) b cosA, a cosB, c cosC (B) a cosA, b cosB, c cosC
(C) R sin 2A, R sin 2B, R sin 2C (D) a cotA, b cotB, c cotC
3. Circumraii of the triangle PBC, PCA and PAB are respectively
(A) R, R, R (B) 2R, 2R, 2R (C) R/2, R/2, R/2 (D) 3R, 3R, 3R

4*. Which of the following is/are correct


Perimeter of ∆DEF r
(A) = (B) Area of ∆DEF = 2 ∆ cosA cosB cosC
Perimeter of ∆ABC R
R
(C) Area of ∆AEF = ∆ cos2A (D) Circum-radius of ∆DEF =
2
Comprehension # 2 (Q. 5 to 8)
The triangle formed by joining the three excentres Ι 1, Ι2 and Ι3 of ∆ ABC is called the excentral or
excentric triangle and in this case internal angle bisector of triangle ABC are the altitudes of triangles
I1I2I3

RI
5. Incentre Ι of ∆ ABC is the ......... of the excentral ∆ Ι 1 Ι2 Ι3.
(A) Circumcentre (B) Orthocentre (C) Centroid (D) None of these

Angles of the ∆ Ι1 Ι2 Ι3 are

A
6.
π A π B π C π A π B π C
(A) − , − and − (B) + , + and +
2 2 2 2 2 2 2 2 2 2 2 2

UH
π π π
(C) – A, – B and –C (D) None of these
2 2 2
7. Sides of the ∆ Ι1 Ι2 Ι3 are
A B C A B C
(A) Rcos , Rcos and Rcos (B) 4R cos , 4R cos and 4R cos
2 2 2 2 2 2
JA
A B C
(C) 2Rcos , 2Rcos and 2Rcos (D) None of these
2 2 2

8. Value of ΙΙ12 + Ι2Ι32 = ΙΙ22 + Ι3Ι12 = ΙΙ32 + Ι1Ι22 =


(A) 4R2 (B) 16R2 (C) 32R2 (D) 64R2
LP

PART-III (MATCH THE COLUMN)


1. Match the column

Column– Ι Column–ΙΙ
A

(A) In a ∆ABC, 2B = A + C and b2 = ac. (p) 8


NK

2
a (a + b + c)
Then the value of is equal to
3abc
a2 + b2 + c 2
(B) In any right angled triangle ABC, the value of (q) 1
R2
is always equal to (where R is the circumradius of ∆ABC)
SA

(C) In a ∆ABC if a = 2, bc = 9, then the value of 2R∆ is equal to (r) 5

(D) In a ∆ABC, a = 5, b = 3 and c = 7, then the value of (s) 9


3 cos C + 7 cos B is equal to

2. Match the column


Column – Ι Column – ΙΙ
(A) In a ∆ABC, a = 4, b = 3 and the medians AA1 and BB1 are (p) 27
mutually perpendicular, then square of area of the ∆ABC
is equal to
r1 r2 r3
(B) In any ∆ABC, minimum value of is equal to (q) 7
r3

C 7
(C) In a ∆ABC, a = 5, b = 4 and tan = , then side ‘c’ (r) 6
2 9
is equal to

(D) In a ∆ABC, 2a2 + 4b2 + c2 = 4ab + 2ac, then value of (8 cos B) (s) 11
is equal to

RI
A
UH
JA
LP
A
NK
SA
 Marked Questions may have for Revision Questions.

PART - I : JEE (ADVANCED) / IIT-JEE PROBLEMS (PREVIOUS YEARS)


* Marked Questions may have more than one correct option.

1. Given an isosceles triangle, whose one angle is 120° and radius of its incircle is 3 unit. Then the area
of the triangle in sq. units is [IIT-JEE-2006, Main.,(3, –1)/184]
(A) 7 + 12 3 (B) 12 – 7 3 (C) 12 + 7 3 (D) 4π

RI
2.* Internal bisector of ∠A of triangle ABC meets side BC at D. A line drawn through D perpendicular to AD
intersects the side AC at E and the side AB at F. If a, b, c represent sides of ∆ABC, then
[IIT-JEE-2006, Main.,(5, –1)/184]
2bc A

A
(A) AE is HM of b and c (B) AD = cos
b+c 2
4bc A
(C) EF = sin (D) the triangle AEF is isosceles
b+c 2

UH
3. Let ABC and ABC′ be two non-congruent triangles with sides AB = 4, AC = AC′ = 2 2 and angle
B = 30º. Find the absolute value of the difference between the areas of these triangles.
[IIT-JEE 2009, Paper-2, (4, –1), 80]
JA
A
4*. In a triangle ABC with fixed base BC, the vertex A moves such that cos B + cos C = 4 sin 2 . If a, b
2
and c denote the lengths of the sides of the triangle opposite to the angles A, B and C respectively,
then [IIT-JEE 2009, Paper-1, (4, –1)/ 80]
(A) b + c = 4a (B) b + c = 2a
(C) locus of points A is an ellipse (D) locus of point A is a pair of straight lines
LP

5. If the angle A, B and C of a triangle are in arithmetic progression and if a, b and c denote the lengths of
a c
the sides opposite to A, B and C respectively, then the value of the expression sin 2C + sin 2A is
c a
A

[IIT-JEE 2010, Paper-1, (3, –1), 84]


1 3
(A) (B) (C) 1 (D) 3
2 2
NK

π
6. Let ABC be a triangle such that ∠ACB = and let a, b and c denote the lengths of the sides opposite
6
to A, B and C respectively. The value(s) of x for which a = x 2 + x + 1, b = x2 – 1 and c = 2x + 1 is (are)

(
(A) – 2 + 3 ) (B) 1 + 3 (C) 2 + 3 (D) 4 3
SA

[IIT-JEE 2010, Paper-1, (3, 0), 84]

7. Consider a triangle ABC and let a, b and c denote the lengths of the sides opposite to vertices A, B and
C respectively. Suppose a = 6, b = 10 and the area of the triangle is 15 3 . If ∠ACB is obtuse and if r
denotes the radius of the incircle of the triangle, then r2 is equal to [IIT-JEE 2010, Paper-2, (3, 0), 79]

7 5
8. Let PQR be a triangle of area ∆ with a = 2, b = and c = , where a, b and c are the lengths of the
2 2
2 sinP – sin 2P
sides of the triangle opposite to the angles at P, Q and R respectively. Then equals
2 sinP + sin 2P
[IIT-JEE 2012, Paper-2, (3, –1), 66]
2 2
3 45  3   45 
(A) (B) (C)   (D)  
4∆ 4∆  4∆   4∆ 

1
9.* In a triangle PQR, P is the largest angle and cosP = . Further the incircle of the triangle touches the
3
sides PQ, QR and RP at N, L and M respectively, such that the lengths of PN, QL and RM are
consecutive even integers. Then possible length(s) of the side(s) of the triangle is (are)
[JEE (Advanced) 2013, Paper-2, (3, –1)/60]
(A) 16 (B) 18 (C) 24 (D) 22

10. In a triangle the sum of two sides is x and the product of the same two sides is y. If x 2 – c2 = y, where c

RI
is the third side of the triangle, then the ratio of the in-radius to the circum-radius of the triangle is
[JEE (Advanced) 2014, Paper-2, (3, –1)/60]
3y 3y 3y 3y
(A) (B) (C) (D)
2x(x + c) 2c(x + c) 4x(x + c) 4c(x + c)

A
11*. In a triangle XYZ, let x, y, z be the lengths of sides opposite to the angles X, Y, Z, respectively, and
s−x s−y s−z 8π

UH
2s = x + y + z. If = = and area of incircle of the triangle XYZ is , then
4 3 2 3
(A) area of the triangle XYZ is 6 6 [JEE (Advanced) 2016, Paper-1, (4, –2)/62]
35
(B) the radius of circumcircle of the triangle XYZ is 6
6
JA
X Y Z 4
(C) sin sin sin =
2 2 2 35
 X+Y 3
(D) sin2   =
 2  5
LP

12*. In a triangle PQR, let ∠PQR = 30º and the sides PQ and QR have lengths 10 3 and 10, respectively.
Then, which of the following statement(s) is (are) TRUE?
(A) ∠QPR = 45º [JEE(Advanced) 2018, Paper-1,(4, –2)/60]
(B) The area of the triangle PQR is 25 3 and ∠QRP = 120º
A

(C) The radius of the incircle of the triangle PQR is 10 3 – 15


(D) The area of the circumcircle of the triangle PQR is 100π
NK

PART - II : JEE (MAIN) / AIEEE PROBLEMS (PREVIOUS YEARS)

1. The sum of the radii of inscribed and circumscribed circles for an n sided regular polygon of side 'a', is :
SA

[AIEEE – 2003 (3, 0), 225]


π a  π   π  a  π 
(1) a cot   (2) cot   (3) a cot   (4) cot  
 
n 2  
2n  
2n 4  2n 

C A 3b
2. If in a triangle ABC, a cos2   + c cos2   = , then the sides a, b and c :
2 2 2
[AIEEE – 2003 (3, 0), 225]
(1) are in A.P. (2) are in G.P. (3) are in H.P. (4) satisfy a + b = c.

π π
3. In a triangle ABC, medians AD and BE are drawn. If AD = 4, ∠DAB = and ∠ABE = , then the area
6 3
of the ∆ABC is : [AIEEE – 2003 (3, 0), 225]
8 16 32 64
(1) (2) (3) (4) .
3 3 3 3 3

π
4. The sides of a triangle are sinα, cosα and 1 + sin α cos α for some 0 < α < . Then the greatest angle
2
of the triangle is : [AIEEE – 2004 (3, 0), 225]
(1) 60º (2) 90º (3) 120º (4) 150º

5. In a triangle ABC, let ∠C = π/2, if r is the inradius and R is the circumradius of the triangle ABC, then
2(r+R) equals : [AIEEE - 2005 (3, 0), 225]
(1) c + a (2) a + b + c (3) a + b (4) b + c

6. If in a ∆ABC, the altitudes from the vertices A,B,C on opposite sides are in H.P., then sinA, sinB, sinC
are in : [AIEEE - 2005 (3, 0), 225]

RI
(1) HP (2) Arithemetico-Geometric Progression
(3) AP (4) GP
7. For a regular polygon, let r and R be the radii of the inscribed and the circumscribed circles. A false

A
statement among the following is [AIEEE - 2010 (4, –1), 144]
r 1 r 2
(1) There is a regular polygon with = . (2) There is a regular polygon with = .
R 2 R 3

UH
r 3 r 1
(3) There is a regular polygon with = . (4) There is a regular polygon with = .
R 2 R 2

8. ABCD is a trapezium such that AB and CD are parallel and BC ⊥ CD. If ∠ADB = θ , BC = p and CD = q,
then AB is equal to : [AIEEE - 2013, (4, –1),120]
JA
(p2 + q2 )sin θ p2 + q2 cos θ p2 + q2 (p2 + q2 )sin θ
(1) (2) (3) 2 (4)
p cos θ + qsin θ p cos θ + qsin θ p cos θ + q2 sin θ (p cos θ + qsin θ)2
9. With the usual notation, in ∆ABC, if ∠A + ∠B = 120°, a = 3 + 1 and b = 3 –1, then the ratio ∠A : ∠B,
is : [JEE(Main) 2019, Online (10-01-19),P-2 (4, – 1), 120]
(1) 9 : 7 (2) 7 : 1 (3) 3 : 1 (4) 5 : 3
LP

10. In a triangle, the sum of lengths of two sides is x and the product of the lengths of the same two sides is
y. If x2 – c2 = y, where c is the length of the third side of the triangle, then the circumradius of the
triangle is [JEE(Main) 2019, Online (11-01-19),P-1 (4, – 1), 120]
c 3 c y
(1) (2) y (3) (4)
A

3 2 3 3
NK
SA
EXERCISE - 1

Section (A) :

P
A-2. x>5 A-3. 75° A-8. A-11. 30° A-13. 10°
2 R

Section (B)

RI
2 B
B-2. cot B-3. 9 sq. unit
3 2

A
Section (C) :

UH
C-4. 6, 8, 10 cm C-5. 8

Section (D) :
JA
D-2. 3 D-3. 2

EXERCISE - 2

Section (A) :
LP

A-1. (C) A-2*. (AB) A-3. (C) A-4. (C) A-5. (D) A-6. (C) A-7. (D)
A

A-8. (B) A-9. (AB) A-10. (A) A-11. (A)

Section (B) :
NK

B-1. (B) B-2. (A) B-3. (A) B-4. (C) B-5. (B) B-6. (AD) B-7. (B)

B-8*. (ABC) B-9. (B) B-10*. (AB) B-11. (A) B-12. (C) B-13*. (AC) B-14. (C)
SA

Section (C) :

C-1. (A) C-2. (B) C-3*. (BD) C-4*. (ABCD) C-5. (A) C-6. (A) C-7. (C)

C-8. (B) C-9. (D) C-10. (D) C-11*. (AD) C-.12*. (ACD) C-13. (C)

Section (D)

D-1. (B) D-2. (B) D-3. (C) D-4*. (ACD) D-5. (A) D-6. (D)
D-7. (C) D-8. (B) D-9*. (BD) D-10. (C) D-11*. (ACD) D-12. (A)

PART-II

1. (A) 2*. (BC) 3. (A) 4*. (ABCD) 5. (B) 6. (A) 7. (B)

8. (B)

PART-III

(A) → (q), (B) → (p), (C) → (s), (D) → (r)

RI
1.

2. (A) → (s), (B) → (p), (C) → (r), (D) → (q)

A
EXERCISE - 3

UH
PART - I

1. (C) 2.* (ABCD) 3. 4 4*. (BC) 5. (D) 6. (B) 7. 3


JA
8. (C) 9.* (BD) 10. (B) 11. (ACD) 12. (BCD)

PART - II
LP

1. (2) 2. (1) 3. (3) 4. (3) 5. (3) 6. (3) 7. (2)

8. (1) 9. (2) 10. (1)


A
NK
SA
 Marked questions are recommended for Revision.

1. In ∆ABC , P is an interior point such that ∠PAB = 10º ∠PBA = 20º, ∠PCA = 30º, ∠PAC = 40º then
prove that ∆ABC is isosceles

 A +B
2. In a triangle ABC, if a tan A + b tan B = (a + b) tan   , prove that triangle is isosceles.
 2 

3. In any triangle ABC, if 2∆a – b2c = c3 , (where ∆ is is the area of triangle), then prove that ∠A is obtuse

RI
cos A + 2cosC sinB
4. If in a triangle ABC, = prove that the triangle ABC is either isosceles or
cos A + 2cosB sinC
right angled.

A
5. In a ∆ ABC, ∠ C = 60° and ∠ A = 75°. If D is a point on AC such that the area of the ∆ BAD is 3
times the area of the ∆ BCD, find the ∠ ABD.

UH
A B C
6. In a ∆ABC, if a, b and c are in A.P., prove that cos A.cot , cos B.cot , and cos C.cot are in A.P.
2 2 2

7.
JA
In a triangle ABC, prove that the area of the incircle is to the area of triangle itself is,
A B C
π : cot   . cot   . cot   .
2 2 2

 A B C
8. In ∆ABC, prove that a2 (s –a) + b2 (s – b) + c2 (s – c) = 4R∆  1 + 4 sin sin sin 
 2 2 2
LP

9. In any ∆ABC, prove that

(i) (r3+ r1) (r3+ r2) sin C = 2 r3 r2 r3 + r3 r1 + r1 r2


A

tan A2 tan B2 tan C2 1


(ii) + + =
NK

(a − b) (a − c) (b − a) (b − c) (c − a) (c − b) ∆

B − C C−A A −B
(iii) (r + r1) tan + (r + r2) tan + (r + r3) tan =0
2 2 2
(iv) r 2 + r12 + r22 + r32 = 16R2 – a2 –b2 – c2.
SA

π
10. In an acute angled triangle ABC, r + r1 = r2 + r3 and ∠B > , then prove that b + 3c < 3a < 3b + 3c
3

11. If the inradius in a right angled triangle with integer sides is r. Prove that
(i) If r = 4, the greatest perimeter (in units) is 90
(ii) If r = 5, the greatest area (in sq. units) is 330
 r   r 
12. If  1 − 1   1 − 1  = 2, then prove that the triangle is right angled.
 r2   r3 

13. DEF is the triangle formed by joning the points of contact of the incircle with the sides of the triangle
ABC; prove that
A B C
(i) its sides are 2r cos , 2r cos and 2r cos ,
2 2 2
π A π B π C
(ii) its angles are – , – and –
2 2 2 2 2 2
and
2∆ 3 1 r
(iii) its area is , i.e. ∆.
(abc)s 2 R

14. Three circles, whose radii are a, b and c, touch one another externally and the tangents at their points
of contact meet in a point, prove that the distance of this point from either of their points of contact is
1
 abc  2
a+b+c  .

RI
 

15. OA and OB are the equal sides of an isoscles triangle lying in the first quadrant making angles θ and φ
respectively with x-axis. Show that the gradient of the bisector of acute angle AOB is cosec β – cot β

A
where β = φ + θ. (Where O is origin)

UH
16. The hypotenuse BC = a of a right-angled triangle ABC is divided into n equal segments where n is odd.
The segment containing the midpoint of BC subtends angle α at A. Also h is the altitude of the triangle
4nh
through A. Prove that tan α = .
(
a n2 − 1 )
JA
LP
A
NK
SA
EXERCISE # 01
Section (A) :

A-2. x>5

A-3. 75°

RI
A-8.
2 R

A-11. 30°

A
A-13. 10°

Section (B)

UH
2 B
B-2. cot
3 2

B-3. 9 sq. unit


JA
Section (C) :

C-4. 6, 8, 10 cm lseh

C-5. 8
LP

Section (D) :
D-2. 3

D-3. 2
A

EXERCISE # 02
NK

Section (A) :
A-1. (C)

A-2*. (AB)
SA

A-3. (C)

A-4. (C)

A-5. (D)

A-6. (C)

A-7. (D)

A-8. (B)

A-9. (AB)

A-10. (A)
A-11. (A)

Section (B) :

B-1. (B)

B-2. (A)

B-3. (A)

B-4. (C)

RI
B-5. (B)

B-6. (AD)

A
B-7. (B)

B-8*. (ABC)

UH
B-9. (B)

B-10*. (AB)

B-11. (A)
JA
B-12. (C)

B-13*. (AC)

B-14. (C)
LP

Section (C) :
C-1. (A)
A

C-2. (B)

C-3*. (BD)
NK

C-4*. (ABCD)

C-5. (A)

C-6. (A)
SA

C-7. (C)
C-8. (B)

C-9. (D)

C-10. (D)

C-11*. (AD)

C-.12*. (ACD)
C-13. (C)

Section (D)
D-1. (B)

D-2. (B)

D-3. (C)

D-4*. (ACD)

D-5. (A)

D-6. (D)

D-7. (C)

RI
D-8. (B)

D-9*. (BD)

D-10. (C)

A
D-11*. (ACD)

UH
D-12. (A)

PART-II
1. (A)
JA
2*. (BC)

3. (A)

4*. (ABCD)
LP

5. (B)

6. (A)

7. (B)
A

8. (B)
NK

PART-III
1. (A) → (q), (B) → (p), (C) → (s), (D) → (r)

2. (A) → (s), (B) → (p), (C) → (r), (D) → (q)


SA

EXERCISE # 03
1. (C)

2.* (ABCD)

3. 4

4*. (BC)

5. (D)

6. (B)
7. 3

8. (C)

9.* (BD)

10. (B)

11. (ACD)

12. (BCD)

PART - II

RI
1. (2)

2. (1)

A
3. (3)

UH
4. (3)

5. (3)

6. (3)
JA
7. (2)

8. (1)
LP

5. ∠ ABD = 30°
A
NK
SA
STRAIGHT LINE

JEE (ADVANCED) SYLLABUS


Straight Line : Two dimensions: Cartesian coordinates, distance between two points, section formulae, shift
of origin. Equation of a straight line in various forms, angle between two lines, distance of a point from a line.
Lines through the point of intersection of two given lines, equation of the bisector of the angle between two
lines, concurrency of lines, centroid, orthocentre, incentre and circumcentre of a triangle.

JEE (MAIN) SYLLABUS

RI
Straight Line : Various forms of equations of a line, intersection of lines, angles between two lines, conditions of for
concurrence of three lines, distance of a point from a line, equations of internal and external bisectors of angles between

A
two lines, coordinates of centroid, orthocenter and circumcentre of a triangle, equation of family of lines passing through the
point of intersection of two lines.

UH
The knowledge of which geometry aims is the knowledge of the eternal...... Plato
_______________________________________________________________________________________________________________
A french mathematician and a greatest philosopher named Rene Descartes, pioneered the use of
algebra in Geometry. He suggested methods to study geometry by algebraic methods without making
direct reference to the actual figures
This geometry was called co-ordinate geometry or analytical geometry and it is the branch of geometry
JA
in which algebraic equations are used to denote points, lines and curves.

Rectangular cartesian co-ordinate systems :


We shall right now focus on two-dimensional co-ordinate geometry in which two perpendicular lines
called co-ordinate axes (x-axis and y-axis) are used to locate a point in the plane.
LP
A
NK

O is called origin. Any point P in this plane can be represented by a unique ordered pair (x, y), which
are called co-ordinates of that point. x is called x co-ordinate or abscissa and y is called y co-ordinate or
ordinate. The two perpendicular lines xox′ and yoy′ divide the plane in four regions which are called
quadrants, numbered as shown in the figure.
SA

Let us look at some of the formulae linked with points now.

Distance Formula :
In rectangular Cartesian coordinate system
The distance between the points A(x1,y1) and B(x2,y2) is = ( x1 − x2 )2 + ( y1 − y2 )2 .

Example # 1 : Find the value of x, if the distance between the points (x, 8) and (4, 3) is 13
Solution : Let P(x ,8) and Q(4, 3) be the given points. Then PQ = 13 (given)
(x − 4)2 + (8 − 3)2 = 13 ⇒ (x – 4)2 + 25 = 169 ⇒ x = 16 or x = – 8
Self practice problems :
(1) Show that four points (0, –1), (6, 7) (–2, 3) and (8, 3) are the vertices of a rectangle.
(2) Find the co-ordinates of the circumcentre of the triangle whose vertices are (8, 6), (8, –2) and
(2, –2). Also find its circumradius.
Ans. (2) (5, 2), 5

Section Formula :
If P(x, y) divides the line joining A(x1, y1) & B(x2, y2) in the ratio m : n, then;
m x 2 + n x1 m y 2 + n y1
x= ;y= .
m + n m + n
m m
Notes : (i) If is positive, the division is internal, but if is negative, the division is external.

RI
n n
(ii) If P divides AB internally in the ratio m : n & Q divides AB externally in the ratio m : n then
P & Q are said to be harmonic conjugate of each other w.r.t. AB.
2 1 1

A
Mathematically, = + i.e. AP, AB & AQ are in H.P.
AB AP AQ

UH
Example # 2 : Find the co-ordinates of the point which divides the line segment joining the points (2, 5) and
(– 3, 7) in the ratio 2 : 3 (i) internally and (ii) externally.
Solution : Let P (x, y) be the required point.
(i) For internal division :
JA
 −6 + 6 14 + 15   29  (2, 5) (–3, 7)
(x, y) =  ,  =  0,
 2+3 2+3   5 
(ii) For external division
3
2
LP

P A B
 −6 − 6 14 − 15  (x, y) (2, 5) (–3, 7)
(x, y) =  ,  = (12, 1)
 2−3 2−3 
Example # 3 : Find the co-ordinates of points which trisect the line segment joining (2, – 3) and (4, 5).
Solution : Let A (2, –3) and B(4, 5) be the given points. Let the points of trisection be P and Q. Then
A

AP = PQ = QB = λ (say)
∴ PB = PQ + QB = 2λ and AQ = AP + PQ = 2λ
⇒ AP : PB = λ : 2λ = 1 : 2 and AQ : QB = 2λ : λ = 2 : 1
NK

So P divides AB internally in the ratio 1 : 2 while Q divides internally in the ratio 2 : 1

(2, –3) (4, 5)


So P divides AB internally in the ratio 1:2 while Q divides internally in the ratio 2: 1
SA

 4+ 4 5−6 8 1
∴ the co-ordinates of P are  ,  =  ,− 
 1+ 2 1+ 2   3 3 
 8 + 2 10 − 3   10 7 
and the co-ordinates of Q are  ,  = , 
 1+ 2 1+ 2   3 3 
8 1  10 7 
Hence, the points of trisection are  , −  and  , 
3 3  3 3

Self practice problems :


(3) In what ratio does the point (4, 1) divide the line segment joining the points
(1, –2) and (5, 2).
(4) The three vertices of a parallelogram taken in order are (–2, 0), (4, 2) and (5, 3) respectively.
Find the co-ordinates of the fourth vertex.
Ans. (3) 3 : 1 internally (4) (–1, 1)

The ratio in which a given line divides the line segment joining two points :
Let the given line ax + by + c = 0 divide the line segment joining A(x1, y1) & B(x2, y2) in the ratio m : n,
m a x1 + b y1 + c
then =− . If A & B are on the same side of the given line then m/n is negative but if A &
n a x2 + b y2 + c
B are on opposite sides of the given line, then m/n is positive

Example # 4 : Find the ratio in which the line joining the points A (1, 2) and B(– 3, 4) is divided by the line
x + y – 5 = 0.
Solution : Let the line x + y = 5 divides AB in the ratio k : 1 at P
 −3k + 1 4k + 2 
∴ co-ordinate of P are  , 

RI
 k +1 k +1 
Since P lies on x + y – 5 = 0
−3k + 1 4k + 2 1
∴ + –5=0 ⇒ k=–
k +1 k +1 2

A
∴ Required ratio is 1 : 2 externally. .
Aliter : Let the ratio is m : n
m (1× 1 + 1× 2 − 5) 1
∴ ∴

UH
=– =– ratio is 1 : 2 externally.
n 1× ( −3) + 1× 4 − 5 2

Self practice problem :


(5) If the line 2x– 3y + λ = 0 divides the line joining the points A (– 1, 2) & B(– 3, – 3) internally in
the ratio 2 : 3, find λ.
JA
18
Ans.
5
Slope Formula :
If θ is the angle at which a straight line is inclined to the positive direction of x −axis, &
0° ≤ θ < 180°, θ ≠ 90°, then the slope of the line, denoted by m, is defined by m = tan θ. If θ is 90°, m
LP

does not exist, but the line is parallel to the y−axis. If θ = 0, then m = 0 & the line is parallel to the
x-axis. If A (x1, y1) & B (x2, y2), x1 ≠ x2, are points on a straight line, then the slope m of the line is given
 y − y2 
by : m =  1 .
 x1 − x 2 
A

Example # 5 : What is the slope of a line whose inclination with the positive direction of x-axis is :
(i) 30º (ii) 90º (iii) 135º
NK

Solution : (i) Here θ = 30º


1
Slope = tan θ = tan 30º = .
3
(ii) Here θ = 90º

SA

The slope of line is not defined.


(iii) Here θ = 135º
∴ Slope = tan θ = tan 135º = tan (180º – 45º) = – tan 45º = – 1.

Example # 6 : Find the slope of the line passing through the points :
(i) (2, 7) and (– 3, 4) (ii) (6, 9) and (–2, 7)
Solution : (i) Let A = (2, 7) and B = (– 3, 4)
4−7 3  y 2 − y1 
∴ Slope of AB = =  Using slope = 
−3 − 2 5  x 2 − x1 
7−9 1
(ii) Let A = (6, 9), B = (–2, 7) ∴ Slope of AB = =
−2 − 6 4
Self practice problems :
(6) Find the value of x, if the slope of the line joining (1, 5) and (x, –7) is 4.
(7) What is the inclination of a line whose slope is
(i) 0 (ii) 1 (iii) –1 (iv) –1/ 3
Ans. (6) –2 (7) (i) 0º, (ii) 45º, (iii) 135º, (iv) 150º

Condition of collinearity of three points :


Points A (x1, y1), B (x2, y2), C(x3, y3) are collinear if
 y − y2   y2 − y3 
(i) mAB = mBC = mCA i.e.  1 =  
 x1 − x 2   x 2 − x 3 
x1 y1 1
(ii) ∆ ABC = 0 i.e. x2 y2 1 = 0
x3 y3 1

RI
(iii) AC = AB + BC or AB ~ BC
(iv) A divides the line segment BC in some ratio.

A
Example # 7 : Show that the points (–2, –1), (2, 7) and (5, 13) are collinear.
Solution : Let (–2, –1) (2, 7) and (5, 13) be the co-ordinates of the points A, B and C respectively.
7 +1 13 − 7
Slope of AB = = 2 and Slope of BC = =2

UH
2+2 5−2
∴ Slope of AB = slope of BC
∴ AB & BC are parallel
∴ A, B, C are collinear because B is on both lines AB and BC.
Self practice problem :
JA
1 1
(8) Prove that the points (a, 0), (0, b) and (1, 1) are collinear if + =1
a b

Area of a Triangle :
If A(x1, y1), B(x2, y2), C(x3, y3) are the vertices of triangle ABC, then its area is equal to
LP

x1 y1 1
1
∆ ABC = x2 y2 1 , provided the vertices are considered in the counter clockwise sense. The
2
x3 y3 1
A

above formula will give a (−) ve area if the vertices (xi, yi), i = 1, 2, 3 are placed in the clockwise sense.

Note: Area of n-sided polygon formed by points (x1, y1) ; (x2, y2); ........;(xn, yn) is given by
NK

1  x1 x 2 x 2 x3 x xn xn x1 
 + + ............... n−1 + .
2 y
 1 y 2 y 2 y 3 y n −1 yn yn y1 
Here vertices are taken in order.

Example # 8 : If the co-ordinates of two points A and B are (2, 1) and (4, –3)respectively. Find the co-
SA

ordinates of any point P if PA = PB and Area of ∆PAB = 6.


Solution : Let the co-ordinates of P be (x, y). Then
PA = PB ⇒ PA2 = PB2
⇒ (x – 2) + (y – 1) = (x – 4)2 + (y + 3)2
2 2
⇒ x – 2y = 5 ....(i)
x y 1
1
Now, Area of ∆PAB = 6 ⇒ 2 1 1 =±6 ⇒ 4x + 2y – 10 = ± 12
2
4 −3 1
⇒ 4x + 2y = 22 or 4x + 2y = –2 ⇒
2x + y = 11 or 2x + y = –1
27 1
Solving 2x + y = 11 and x – 2y = 5 we get x = ,y= .
5 5
3 11
Solving 2x + y = –1 and x – 2y = 5, we get x = , y = – .
5 5
 27 1   3 11 
Thus, the co-ordinates of P are  ,  or  , − 
 5 5 5 5 
Self practice problems :
(9) The area of a triangle is 5. Two of its vertices are (2, 1) and (3, –2). The third vertex lies on
y = x + 3. Find the third vertex.
(10) The coordinates of A, B, C are (6, 3), (–3, 5) & (4,–2) respectively and p is any point
x+y−2
(x, y).Show that the ratio of the areas of the triangles PBC and ABC is
7
 7 13   3 3
Ans. (9)  2 , 2  or  − 2 , 2 
   
Equation of a Straight Line in various forms :

RI
Now let us understand, how a line can be represented with the help of an algebraic equation. A moving
point (point with variable co-ordinates) is assumed on the line and a link is established between its
co-ordinates with the help of some given parameters. There are various forms of lines depending on
the specified parameter

A
Point - Slope form :
y − y1 = m (x − x1) is the equation of a straight line whose slope is m & which passes through the point

UH
(x1, y1).

Example # 9 : Find the equation of a line passing through (3, –4) and inclined at an angle of 150º with the
positive direction of x-axis.
1
Solution : Here, m = slope of the line = tan 150º = tan (90º + 60º) =– cot 60º = – , x1 = 3, y1 = – 4
3
JA
So, the equation of the line is y – y1 = m (x – x1)
1
i.e. y + 4 =– (x – 3)
3
x+ 3y+4 3–3=0

Self practice problem :


LP

(11) Find the equation of s a line passing through P(–3, 5) and whose slope is –2.
Ans. 2x + y + 1 = 0

Slope-intercept form :
A

y = mx + c is the equation of a straight line whose slope is m & which makes an intercept c on the
y−axis.

Example # 10 : Find the equation of a line with slope –3 and cutting off an intercept of 5 units on negative
NK

direction of y-axis.
Solution : Here m = –3 and c = – 5. So, the equation of the line is y = mx + c
i.e. y = –3x – 5 or 3x + y + 5 =0

Self practice problem :


SA

(12) Find the equation of a straight line which cuts off an intercept of length 3 on y- axis and whose
slope is – 3.
Ans. 3x + y – 3 = 0

y 2 − y1
Two point form : y − y1 = (x − x1) is the equation of a straight line which passes through the
x 2 − x1
points (x1, y1) & (x2, y2).

Example # 11 : Find the equation of the line joining the points (3, 4) and (–2, 5)
Solution : Here the two points are (x1, y1) = (3, 4) and (x2, y2) = (–2, 5).
So, the equation of the line in two-point form is
5−4
y–4= (x – 3) ⇒ x + 5y = 23
−2 − 3
Self practice problem :
(13) Find the equations of the sides of the triangle whose vertices are (–1, 8), (4, –2) and (–5, –3).
Also find the equation of the median through (–1, 8)
Ans. 2x + y – 6 = 0, x – 9y – 22 = 0, 11x – 4y + 43 = 0, 21x + y + 13 = 0

x y 1
Determinant form : Equation of line passing through (x1, y1) and (x2, y2) is x1 y1 1 = 0
x2 y2 1
Example # 12 : Find the equation of line passing through (2, 4) & (– 1, 3).
x y 1
Solution : 2 4 1 =0 ⇒ x – 3y + 10 = 0
−1 3 1

RI
Self practice problem :
(14) Find the equation of the passing through (– 2, 3) & (– 1, – 1).
Ans. 4x + y + 5 = 0

A
x y
Intercept form : + = 1 is the equation of a straight line which makes intercepts a & b on OX & OY
a b

UH
respectively.

Example # 13 : Find the equation of the line which passes through the point (–3, 8) and the sum of its
intercepts on the axes is 7.
x y
Solution : Let the equation of the line be + =1 ....(i)
JA
a b
3 8
This passes through (–3, 8), therefore – + =1 ....(ii)
a b
It is given that a + b = 7 ⇒ b = 7 – a.
3 8
Putting b = 7 – a in (ii), we get – + =1
a 7−a
LP

⇒ a2 + 4a – 21 = 0 ⇒ a = 3, – 7
For a = 3, b = 4 and for a = –7, b = 14
Putting the values of a and b in (i), we get the equations of the lines
x y x y
+ = 1 and + = 1 or 4x + 3y = 12 and 2x – y + 14 = 0
3 4 –7 14
A

Self practice problem :


(15) Find the equation of the line through (2, 3) so that the segment of the line intercepted between
the axes is bisected at this point.
NK

Ans. 3x + 2y = 12.

Perpendicular/Normal form :
xcos α + ysin α = p (where p > 0, 0 ≤ α < 2 π) is the equation of the straight line where the length of
the perpendicular from the origin O on the line is p and this perpendicular makes an angle α with
SA

positive x−axis.

Example # 14 : Find the equation of the line which is at a distance 3 from the origin and the perpendicular from
the origin to the line makes an angle of 30º with the positive direction of the x-axis.
Solution : Here p = 3, α = 30º
∴ Equation of the line in the normal form is
3 y
x cos 30º + y sin 30º = 3 or x + = 3 or 3x+y=6
2 2
Self practice problem :
(16) The length of the perpendicular from the origin to a line is 7 and the line makes an angle of
150º with the positive direction (clock-wise) of y-axis. Find the equation of the line.
Ans. 3 x – y + 14 = 0
General Form : ax + by + c = 0 is the equation of a straight line in the general form
a
In this case, slope of line = –
b
c c
x - intercept = – , y - intercept = –
a b

Example # 15 : Find slope, x-intercept & y-intercept of the line 2x – 3y + 5 = 0.


Solution : Here, a = 2, b = – 3, c = 5
a 2
∴ slope = – =
b 3
c 5
x-intercept = – =–
a 2

RI
5
y-intercept =
3

A
Self practice problem :
(17) Find the slope, x-intercept & y-intercept of the line 3x – 5y – 8 = 0.
3 8 8

UH
Ans. (17) , , –
5 3 5
Parametric form :
x − x1 y − y1
P(r) = (x, y) = (x1 + r cos θ, y1 + r sin θ) or = = r is the equation of the line in
cos θ sin θ
JA
parametric form, where ‘r’ is the parameter whose absolute value is the distance of any point (x, y) on
the line from the fixed point (x1, y1) on the line.

Remark : The above form is derived from point-slope form of line.


sin θ
y – y1 = m (x – x1) where m = tan θ ⇒ y – y1 =
(x – x1)
cos θ
LP

Example # 16 : Find the equation of the line through the point A(1, 4) and making an angle of 45º with the
positive direction of x-axis. Also determine the length of intercept on it between A and the line
x + y – 10 = 0
Solution : The equation of a line through A and making an angle of 45º with the x-axis is
x −1 y−4 x −1 y − 4
A

= or = or x–y+3=0
cos 45º sin 45º 1 1
2 2
Suppose this line meets the line x + y – 10 = 0 at P such that AP = r. Then the co-ordinates of
NK

P are given by
x −1 y−4
= =r ⇒ x = 1 + r cos 45º, y = 4 + r sin 45º
cos 45º sin 45º
r r
⇒ x=1+ ,y=4+
2 2
SA

 r r 
Thus, the co-ordinates of P are  1 + , 4+ 
 2 2
r r
Since P lies on x + y – 10 = 0, so 1 + +4+ = 10
2 2
5 5
⇒ 5+ 2 r = 10 ⇒ r = ⇒ length AP = | r | =
2 2
5
Thus, the length of the intercept = .
2
Self practice problem :
(18) A straight line is drawn through the point A ( 3, 2 ) making an angle of π/6 with positive

direction of the x-axis. If it meets the straight line 3x – 4y + 8 = 0 in B, find the distance
between A and B.
Ans. 6 units

Angle between two straight lines in terms of their slopes:

If m1 & m2 are the slopes of two intersecting straight lines (m1 m2 ≠ −1) & θ is the acute angle between
m1 − m2
them, then tan θ = .
1 + m1 m2

RI
Notes : (i) Let m1, m2, m3 are the slopes of three lines L1 = 0;L2 = 0;L3 = 0 where m1 > m2 > m3 , then the
m1 − m2
tangent of interior angles of the ∆ ABC formed by these lines are given by, tan A = ; tan
1 + m1 m2

A
m2 − m3 m3 − m1
B= & tan C =
1 + m2 m3 1 + m3 m1
α with the line y = mx + c are

UH
(ii) The equation of lines passing through point (x1, y1) and making angle
given by :
(y − y1) = tan (θ − α) (x − x1) & (y − y1) = tan (θ + α) (x − x1), where tan θ = m.
JA
Example # 17 : The acute angle between two lines is π/4 and slope of one of them is –1/3. Find the slope of
LP

the other line.


m1 − m2
Solution : If θ be the acute angle between the lines with slopes m 1 and m2, then tan θ =
1 + m1m2
A

π
Let θ = and m1 –1/3
4
1
− m2
NK


π 3 3m2 + 1 3m2 + 1
∴ tan = ⇒ 1= ⇒ = 1 or – 1
4 1 3 – m2 3 – m2
1– m2
3
3m2 + 1 1 3m2 + 1
Now =1 ⇒ m2 = and =–1 ⇒ m2 = –2.
3 – m2 2 3 – m2
SA

∴ The slope of the other line is either 1/2 or –2

Example # 18 : Find the equation of the straight line which passes through the point (3,–2) and making angle
60º with the line 3 x + y =1.
Solution : Given line is 3 x + y =1..
⇒ Slope of (1) = – 3 .
Let slope of the required line be m. Also between these lines is given to be 60º.

⇒ tan 60º =
m− − 3 ( ) ⇒ 3=
m+ 3

m+ 3
=± 3
1+ m (− 3 ) 1– 3m 1– 3m
m+ 3
= 3 ⇒ m + 3 = 3 – 3m ⇒ m=0
1– 3m
the equation of the required line is y + 2 = 0
m+ 3
=– 3 ⇒ m= 3
1– 3m

∴ The equation of the required line is y +2 = 3 (x–3) ⇒ y 3 = x – 2 –3 3

Self practice problem :

(19) A vertex of an equilateral triangle is (2, 3) and the equation of the opposite side is x + y = 2.
Find the equation of the other sides of the triangle.

RI
Ans. (19) (2 – 3 )x – y + 2 3 – 1 = 0 and (2 + 3 ) x – y – 2 3 – 1 = 0.

Parallel Lines :

A
(i) When two straight lines are parallel their slopes are equal. Thus any line parallel to y = mx + c
is of the type y = mx + d, where ‘d’ is a parameter.

UH
a b c
(ii) Two lines ax + by + c = 0 and a′x + b′y + c′ = 0 are parallel if = ≠ .
a′ b′ c ′
Thus any line parallel to ax + by + c = 0 is of the type ax + by + k = 0, where k is a parameter.

(iii) The distance between two parallel lines with equations ax + by + c 1 = 0 &
JA
c1 − c 2
ax + by + c2 = 0 is = .
a2 + b2

Note that coefficients of x & y in both the equations must be same.


LP

p1 p2
(iv) The area of the parallelogram = , where p1 & p2 are distances between two pairs of
sin θ
opposite sides & θ is the angle between any two adjacent sides. Note that area of the
parallelogram bounded by the lines y = m1x + c1, y = m1x + c2 and y = m2x + d1, y = m2x + d2 is
A

(c1 − c 2 ) (d1 − d2 )
given by .
m1 − m2
NK
SA

Example # 19 : Find the equation of the straight line that has y-intercept 5 and is parallel to the straight line
3x – 7y = 8.
Solution : Given line is 3x – 7y = 8
3
∴ Slope of (1) is
7
The required line is parallel to (1), so its slope is also 3/7, y-intercept of required line = 5
∴ By using y = mx + c form, the equation of the required line is
3
y= x+5 or 3x – 7y + 35 = 0
7

Example # 20 : Two sides of a square lie on the lines 5x – 12y + 6 = 0 and 5x – 12y = 20. What is its area ?
Solution : Clearly the length of the side of the square is equal to the distance between the parallel lines
5x – 12y + 6 = 0 ........(i) and 5x – 12y = 20 ........(ii)
Now, Distance between the parallel lines
| 6 + 20 |
= =2
52 + (–12)2
Thus, the length of the side of the square is 2 and hence its area = 4

Example # 21 : Find the area of the parallelogram whose sides are x + 2y + 3 = 0, 3x + 4y – 5 = 0,


2x + 4y + 5 = 0 and 3x + 4y – 10 = 0
Solution :

RI
A
UH
3 5 10 5 1 3
Here, c1 = – , c2 = – , d1 = , d2 = , m1 = – , m2 = –
2 4 4 4 2 4
 3 5   10 5 
− 2 + 4   4 − 4 
∴ Area =     =
5
sq. units
 1 3 4
JA
− 2 + 4 
 
Self practice problem :
(20) Find the area of parallelogram whose sides are given by 4x – 5y + 1 = 0, x – 3y – 6 = 0,
4x – 5y – 2 = 0 and 2x – 6y + 5 = 0
51
Ans. (20) sq. units
LP

14
Perpendicular Lines:
(i) When two lines of slopes m1 & m2 are at right angles, the product of their slopes is −1,
i.e. m1 m2 = −1. Thus any line perpendicular to y = mx + c is of the form
A

1
y=− x + d, where ‘d’ is any parameter.
m
Two lines ax + by + c = 0 and a′x + b′y + c′ = 0 are perpendicular if aa′ + bb′ = 0. Thus any line
NK

(ii)
perpendicular to ax + by + c = 0 is of the form bx − ay + k = 0, where ‘k’ is any parameter.
Example # 22 : Find the equation of the straight line that passes through the point (3, 4) and perpendicular to
the line 3x + 2y + 5 = 0
SA

Solution : The equation of a line perpendicular to 3x + 2y + 5 = 0 is


2x – 3y + λ = 0 ...........(i)
This passes through the point (3, 4)
∴ 3×2–3×4+λ=0⇒λ=6
Putting λ = 6 in (i), we get 2x – 3y + 6 = 0, which is the required equation.

Aliter The slope of the given line is –3/2. Since the required line is perpendicular to the given line. So, the
2
slope of the required line is 2/3. As it passes through (3, 4). So, its equation is y – 4 = (x – 3)
3
or 2x – 3y + 6 = 0
Self practice problem :

(21) The vertices of a triangle are A(10, 4), B (–4, 9) and C(–2, –1). Find the equation of its
altitudes. Also find its orthocentre.
 9
Ans. (21) x – 5y + 10 = 0, 12x + 5y + 3 = 0, 14x – 5y + 23 = 0,  −1, 
 5

Position of the point (x1, y1) relative of the line ax + by + c = 0 :


If ax1 + by1 + c is of the same sign as c, then the point (x1, y1) lie on the origin side of ax + by + c = 0. But
if the sign of ax1 + by1 + c is opposite to that of c, the point (x1, y1) will lie on the non−origin side of
ax + by + c = 0.

RI
In general two points (x1, y1) and (x2, y2) will lie on same side or opposite side of ax + by + c = 0
according as ax1 + by1 + c and ax2 + by2 + c are of same or opposite sign respectively.

A
Example # 23 : Show that (2, –1) and (–3, 3) lie on the opposite sides of the line 2x – 3y + 5 = 0.
Solution : At (2, –1), the value of 2x – 3y + 5 = 4 + 3 + 5 = 12 > 0.
At (–3, 3), the value of 2x – 3y + 5 = –6 – 9 + 5 = –10 < 0

UH
∴ The points (2, –1) and (–3, 3) are on the opposite sides of the given line.

Self practice problems :


(22) Are the points (3, – 4) and (2, 6) on the same or opposite side of the line 3x – 4y = 8 ?
(23) Which one of the points (1, 1), (–1, 2) and (2, 3) lies on the side of the line 4x + 3y – 5 = 0 on
JA
which the origin lies?
Ans. (22) Opposite sides (23) (–1, 2)

Length of perpendicular from a point on a line :


a x1 + b y1 + c
The length of perpendicular from P(x1, y1) on ax + by + c = 0 is .
LP

a2 + b2
A
NK

Example # 24 : Find the distance between the line 4x – 3y + 8 = 0 and the point (–2, 3)
(–2) × 4 − 3 × 3 + 8 9
Solution : The required distance = =
4 + ( −3)
2 2 5
SA

Example # 25 : Find all points on x – y + 2 = 0 that lie at a unit distance from the line 12x – 5y + 9 = 0.
Solution : Note that the co-ordinates of an arbitrary point on x – y + 2 = 0 can be obtained by putting
x = t (or y = t) and then obtaining y (or x) from the equation of the line, where t is a parameter.
Putting x = t in the equation x – y + 2 = 0 of the given line, we obtain y = 2 + t.
So, co-ordinates of an arbitrary point on the given line are P(t, 2 + t). Let P(t, 2 + t) be the
required point. Then, distance of P from the line 12x – 5y + 9 = 0 is unity i.e.
12t – 5(t + 2) + 9
⇒ =1 ⇒ |7t – 1| = 13
122 + 52
⇒ 7t – 1 = ± 13 ⇒ t = 2 or t = –12/7
Hence, required points are (2,4) or (–12/7,2/7)
Self practice problem :
(24) Find the length of the altitudes from the vertices of the triangle with vertices :
(–1, 1), (5, 2) and (3, –1).
16 8 16
Ans. (24) , ,
13 5 37

Reflection of a point about a line :


(i) Foot of the perpendicular from a point (x1, y1) on the line ax + by + c = 0 is
x − x1 y − y1  ax + by1 + c 
= = −  1 2 
a b  a + b2
 
(ii) The image of a point (x1, y1) about the line ax + by + c = 0 is
x − x1 y − y1  ax + by1 + c 
= =−2  1 2 
a b  a + b2
 

RI
Example # 26 : Find the foot of perpendicular of the line drawn from P (2, –3) on the line x – 2y + 5 = 0.
Solution : Slope of PM = – 2
∴ Equation of PM is
2x + y = 1 .........(i)
solving equation (i) with x – 2y + 5 = 0, we get

A
co-ordinates of M (–3/5, 11/5)
(2,–3)

UH
x – 2y+5 = 0

x–2 y+3 2+6+5 x–2 y+3 13


Here, = =– ⇒ = =– ⇒ x = –3/5 , y = 11/5
Aliter
JA
1 −2 1+ 4 1 −2 5
Example # 27 : Find the image of the point P(–1, 2) in the line mirror 2x – 3y + 4 = 0.
Solution : Let image of P is Q.
∴ PM = MQ & PQ ⊥ AB
Let Q is (h, k)
 h −1 k + 2 
LP

∴ M is  ,
 2 2 
It lies on 2x – 3y + 4 = 0.
 h − 1 k+2
∴ 2   –3   + 4 = 0.
A

 2   2 
or 2h – 3k = 0 ...........(i)
k−2
NK

slope of PQ =
h +1
PQ ⊥ AB
k−2 2
∴ × = – 1. ⇒ 3h + 2k – 1 = 0. ........(ii)
h +1 3
SA

3 2
soving (i) & (ii), we get h = ,k=
13 13
 3 2 
∴ Image of P(– 1, 2) is Q  , 
 13 13 
Aliter The image of P (– 1, 2) about the line
x +1 y − 2 [2( −1) − 3(2) + 4]
2x – 3y + 4 = 0 is = =–2
2 −3 22 + ( −3)2
x +1 y − 2 8
= =
2 −3 13
3 2
⇒ 13x + 13 = 16 ⇒ x = & 13y – 26 = – 24 ⇒y=
13 13
 3 2 
∴ image is  , 
 13 13 
Self practice problems :
(25) Find the foot of perpendicular of the line drawn from (– 2, – 3) on the line 3x – 2y – 1 = 0.

(26) Find the image of the point (1, 2) in y-axis.


 −23 −41 
Ans. (25)  ,  (26) (– 1, 2)
 13 13 
Centroid, Incentre & Excentre :
If A (x1, y1), B(x2, y2), C(x3, y3) are the vertices of triangle ABC, whose sides BC, CA, AB are of lengths a,
b, c respectively, then the co-ordinates of the special points of triangle ABC are as follows :

RI
 x + x 2 + x3 y + y2 + y3 
Centroid G ≡  1 , 1 
 3 3 
 ax + bx 2 + cx 3 ay1 + by 2 + cy 3 
Incentre I ≡  1

A
,  , and
 a + b + c a + b + c 

 − ax1 + bx 2 + cx 3 − ay1 + by 2 + cy 3 
Excentre (to A) I1 ≡  ,  and so on.

UH
 − a + b + c − a + b + c 

Notes : (i) Incentre divides the angle bisectors in the ratio, (b + c) : a; (c + a) : b & (a + b) : c.
(ii) Incentre and excentre are harmonic conjugate of each other w.r.t. the angle bisector on which
they lie.
(iii) Orthocentre, Centroid & Circumcentre are always collinear & centroid divides the line joining
JA
orthocentre & circumcentre in the ratio 2 : 1.
(iv) In an isosceles triangle G, O, Ι & C lie on the same line and in an equilateral triangle, all these
four points coincide.
(v) In a right angled triangle orthocentre is at right angled vertex and circumcentre is mid point of
hypotenuse
(vi) In case of an obtuse angled triangle circumcentre and orthocentre both are out side the
triangle.
LP

Example # 28: Find the co-ordinates of (i) centroid (ii) in-centre of the triangle whose vertices are (0, 0), (5, 0)
and (0, 12).
Solution : (i) We know that the co-ordinates of the centroid of a triangle whose angular points are
A

 x + x 2 + x 3 y1 + y 2 + y 3 
(x1, y1), (x2, y2) (x3, y3) are  1 , 
 3 3 
So the co-ordinates of the centroid of a triangle whose vertices are (0, 0), (5, 0) and
NK

 0 + 5 + 0 0 + 0 + 12  5 
(0, 12) are  ,  or  ,4  .
 3 3  3 
(ii) Let A (0, 0), B (5, 0) and C(0, 12) be the vertices of triangle ABC.
Then c = AB = 5, b = CA =12
and a = BC = 13.
SA

 ax + bx 2 + cx 3 ay1 + by 2 + cy 3 
The co-ordinates of the in-centre are  1 , 
 a+b+c a+b+c 
 13 × 0 + 12 × 5 + 5 × 0 13 × 0 + 12 × 0 + 5 × 12 
or  ,  or (2, 2)
 5 + 12 + 13 5 + 12 + 13 

Self practice problems :


(27) Two vertices of a triangle are (3, –5) and (–7, 4). If the centroid is (2, –1), find the third vertex.
(28) Find the co-ordinates of the centre of the circle inscribed in a triangle whose vertices are
(– 36, 7), (20, 7) and (0, – 8)
Ans. (27) (10, – 2) (28) (–1, 0)

Bisectors of the angles between two lines:


Equations of the bisectors of angles between the lines ax + by + c = 0 &
ax + by + c a′ x + b′ y + c ′
a′x + b′y + c′ = 0 (ab′ ≠ a′b) are : =±
2
a + b 2
a′2 + b′2
Note : Equation of straight lines passing through P(x1, y1) & equally inclined with the lines
a1x + b1y + c1 = 0 & a2x + b2y + c2 = 0 are those which are parallel to the bisectors between these two
lines & passing through the point P.
Example # 29 : Find the equations of the bisectors of the angle between the straight lines
3x + y + 1 = 0 and x + 3y + 1 = 0.
3x + y + 1
Solution : The equations of the bisectors of the angles between 3x+y+1= 0 and x+3y+1=0 are
32 + 12
x + 3y + 1
=± or 3x + y + 1 = ± (x + 3y + 1)
12 + 32

RI
Taking the positive sign, we get x = y as one bisector
Taking the negative sign, we get 2x + 2y + 1 = 0 as the other bisector.

Self practice problem :

A
(29) Find the equations of the bisectors of the angles between the following pairs of straight lines
3x + 4y + 13 = 0 and 12x – 5y + 32 = 0

UH
Ans. (29) 21x – 77y – 9 = 0 and 99x + 27y + 329 = 0

Methods to discriminate between the acute angle bisector & the obtuse angle bisector:
(i) If θ be the angle between one of the lines & one of the bisectors, find tan θ.
If tan θ < 1, then 2 θ < 90° so that this bisector is the acute angle bisector.
JA
If tan θ > 1, then we get the bisector to be the obtuse angle bisector.

(ii) Let L1 = 0 & L2 = 0 are the given lines & u1 = 0 and u2 = 0 are the bisectors between
L1 = 0 & L2 = 0. Take a point P on any one of the lines
L1 = 0 or L2 = 0 and drop perpendicular on u1 = 0 & u2 = 0 as shown in figure. If,
LP

p < q ⇒ u1 is the acute angle bisector.


p > q ⇒ u1 is the obtuse angle bisector.
p = q ⇒ the lines L1 & L2 are perpendicular.
A
NK

(iii) If aa′ + bb′ < 0, then the equation of the bisector of this acute angle is
SA

ax + by + c a′ x + b′ y + c ′
=+
2
a +b 2
a′2 + b′2
If, however, aa′ + bb′ > 0, the equation of the bisector of the obtuse angle is :
ax + by + c a′ x + b′ y + c ′
=+
2
a +b 2
a′2 + b′2

Example # 30 : For the straight lines 2x – y + 1 = 0 and x – 2y – 2 = 0, find the equation of the
(i) bisector of the obtuse angle between them;
(ii) bisector of the acute angle between them;
Solution : 2x – y + 1 = 0 ........(1)
and x – 2y – 2 = 0 ........(2)
Here a1 = 2, a2 = 1, b1 = –1, b2 = –2
Now a1a2 + b1b2 = 4 > 0
∴ bisector of the obtuse angle between lines (1) and (2) will be
2x – y + 1 x – 2y – 2
=
2 2
2 + (–1) 12 + (–2)2
or x+y+3=0
and the equation of the bisector of the acute angle will be
2x – y + 1 x – 2y – 2
=–
2 2
2 + (–1) 12 + (–2)2
or 3x – 3y = 1
Self practice problem :

(30) Find the equations of the bisectors of the angles between the lines x + y – 3 = 0 and

RI
7x – y + 5 = 0 and state which of them bisects the acute angle between the lines.
Ans. (30) x – 3y + 10 = 0 (bisector of the obtuse angle);
6x + 2y – 5 = 0 (bisector of the acute angle)

A
Condition of Concurrency :
a1 b1 c1

UH
Three lines a1x + b1y + c1 = 0, a2x + b2y + c2 = 0 & a3x + b3y + c3 = 0 are concurrent if a2 b2 c 2 = 0.
a3 b3 c3

Alternatively : If three constants A, B & C (not all zero) can be found such that
A(a1x + b1y + c1) + B(a2x + b2y + c2) + C(a3x + b3y + c3) ≡ 0, then the three straight lines are concurrent.
JA
Example # 31 : If the straight lines x + 2y = 9, 3x + 5y = 5 and ax – by = 1 are concurrent. Then find the value
of 35a + 22b
Solution : Given lines are
x + 2y = 9 ........(1)
3x + 5y = 5 ........(2)
LP

and ax – by = 1 ........(3)
Lines will be concurrent if ∆ = 0
1 2 −9
∆ = 3 5 –5 = 0 ⇒ 35a + 22b = –1
a −b –1
A

Self practice problem :


NK

(31) Find the value of m so that the lines 4x – 3y + 2 = 0, 3x + 4y – 4 = 0 and


x + my + 6 = 0 may be concurrent.
Ans. (31) –7

Family of Straight Lines :


SA

The equation of a family of straight lines passing through the point of intersection of the lines,
L1 ≡ a1x + b1y + c1 = 0 & L2 ≡ a2x + b2y + c2 = 0 is given by L1 + k L2 = 0 i.e.
(a1x + b1y + c1) + k(a2x + b2y + c2) = 0, where k is an arbitrary real number.

Note : (i) If u1 = ax + by + c, u2 = a′x + b′y + d, u3 = ax + by + c′, u4 = a′x + b′y + d′ then


u1=0; u2= 0; u3= 0 ; u4= 0 form a parallelogram. The diagonal BD can be given by u 2u3 – u1u4= 0.

(ii) The diagonal AC is also given by u1 + λu4 = 0 and u2 + µu3 = 0, if the two equations are
identical for some real λ and µ.
[For getting the values of λ & µ compare the coefficients of x, y & the constant terms].
Example # 32 : If 3a + 2b + 5c = 0 and the set of lines ax + by + c = 0 passes through a fixed point . Find
co-ordinates of that point.
Solution : 3a + 2b + 5c = 0 ....... (i)
ax + by + c = 0 ....... (ii)
Eliminating c, we get.
1  3  2  3 b  2
ax + by – (3a + 2b) = 0 ⇒ a  x –  + b  y –  = 0 ⇒  x –  +  y –  = 0
5  5  5  5 a  5
It is of the form L1 + λL2 = 0
3 2
Which passes through the point of intersection  ,  of L1= 0 & L2= 0 for all real values of a & b
5 5
3 2
Aliter : 3a + 2b + 5c = 0 ⇒ a + b +c=0

RI
5 5
3 2 3 2
 5 , 5  lies on the line ax + by + c = 0 Hence fixed point  5 , 5 
   

A
Example # 33 : Obtain the equations of the lines passing through the intersection of lines

UH
3x + 7y = 17 and x + 2y = 5 and is prependicular to the straight line 3x + 4y = 10.
Solution : The equation of any line through the intersection of the given lines is
(x + 2y – 5) + λ (3x + 7y – 17) = 0
or x (3 λ + 1) + y (7λ + 2) – 17λ – 5 = 0 .......(i)
This is prependicular to the line 3x + 4y = 10
JA
 3λ + 1   3  11
∴ –   – 4  = –1 ⇒ λ = –
 7 λ + 2   37
Putting this value of λ in (i), the equation of required line 4x – 3y + 2 = 0

Self practice problem :


(32) Find the equation of the lines through the point of intersection of the lines
LP

x – 3y + 1 = 0 and 2x + 5y – 9 = 0 and whose distance from the origin is 5


Ans. (32 ) 2x + y – 5 = 0

A Pair of straight lines through origin:


A

(i) A homogeneous equation of degree two,


"ax² + 2hxy + by² = 0" always represents a pair of
straight lines passing through the origin if :
NK

(a) h² > ab ⇒ lines are real & distinct .


(b) h² = ab ⇒ lines are coincident .
(c) h² < ab ⇒ lines are imaginary with real point of intersection i.e. (0, 0)
SA

This equation is obtained by multiplying the two equations of lines


(m1x – y) (m2x – y) = 0
⇒ m1m2x2 – (m1 + m2) xy + y2 = 0

(ii) If y = m1x & y = m2x be the two equations represented by ax² + 2hxy + by² = 0, then;
2h a
m 1 + m2 = − & m1 m2 = .
b b

(iii) If θ is the acute angle between the pair of straight lines represented by,
2 h2 − ab
ax² + 2hxy + by² = 0, then tan θ = .
a + b

(iv) The condition that these lines are :


(a) at right angles to each other is a + b = 0. i.e. co−efficient of x² +co−efficient of y² = 0.
(b) coincident is h² = ab .
(c) equally inclined to the axis of x is h = 0.i.e. coeff. of xy = 0 .
Note that a homogeneous equation of degree n represents n straight lines passing through
origin.

(v) The equation to the pair of straight lines bisecting the angles between the straight lines
x2 − y2 x y

RI
ax² + 2hxy + by² = 0 is = .
a−b h

Example # 34 : Show that the equation 18x2 – 9xy + y2 = 0 represents a pair of distinct straight lines, each
passing through the origin. Find the separate equations of these lines.

A
Solution : The given equation is a homogeneous equation of second degree. So, it represents a pair of
straight lines passing through the origin. Comparing the given equation with
ax2 + 2hxy + by2 = 0, we obtain a = 18, b = 1 and 2h = – 9.

UH
81 9
∴ h2 – ab = – 18 = >0 ⇒ h2 > ab
4 4
Hence, the given equation represents a pair of distinct lines passing through the origin.
2
y y y  y 
Now, 18x2 – 9xy + y2 = 0 ⇒   – 9   + 18 = 0 ⇒  − 6   − 3  = 0
x x x  x 
JA
⇒ – 6 = 0 or – 3 = 0 ⇒ y – 6x = 0 or y – 3x = 0
So the given equation represents the straight lines y – 3x = 0 and y – 6x = 0.

Example # 35 : Find the equations to the pair of lines through the origin which are perpendicular to the lines
represented by 6x2 – xy – 12y2 = 0.
Solution : We have 6x2 – xy – 12y2 = 0.
⇒ 6x2 – 9xy + 8xy – 12y2 = 0 ⇒ (3x + 4y)(2x – 3y) = 0
LP

⇒ 3x + 4y = 0 & 2x – 3y = 0
Thus the given equation represents the lines 3x + 4y = 0 and 2x – 3y = 0. The equations of the
lines passing through the origin and perpendicular to the given lines are 4x – 3y = 0 & 3x + 2y =
0 there combined equations 12x2 – xy – 6y2 = 0
A

Example # 36 : Find the angle between the pair of straight lines x2 – 3xy + 2y2 = 0
Solution : Given equation is x2 – 3xy + 2y2 = 0
Here a = coeff. of x2 = 1, b = coeff. of y2 = 2
NK

3
and 2h = coeff. of xy = –3 ∴ h=–
2
9
2 h2 − ab 2 −2 1
Now tan θ = = 4 =
a+b 3
SA

1+ 2

Where θ is the acute angle between the lines.


 1
∴ acute angle between the lines is tan–1   and obtuse angle between them is
3
 1
π – tan–1  
3
Example # 37 : Find the equation of the bisectors of the angle between the lines represented by
3x2 – 5xy + 4y2 = 0
Solution : Given equation is 3x2 – 5xy + 4y2 = 0 .......(1)
comparing it with the equation ax2 + 2hxy + by2 = 0 .......(2)
we have a = 3, 2h = –5; and b = 4
Now the equation of the bisectors of the angle between the pair of lines (1) is
x2 − y2 xy x2 − y2 xy x2 − y2 2xy
= or = ; or = or 5x2 – 2xy – 5y2 = 0
a−b h 3−4 5 −1 −5

2

Self practice problems :


(33) Find the area of the triangle formed by the lines y 2 – 9xy + 18x2 = 0 and y = 9.
(34) If the pairs of straight lines x2 – 2pxy – y2 = 0 and x2 – 2qxy – y2 = 0 be such that each
pair bisects the angle between the other pair, prove that pq = –1.
27
Ans. (33) sq. units
4
General equation of second degree representing a pair of Straight lines :
(i) ax² + 2hxy + by² + 2gx + 2fy + c = 0 represents a pair of straight lines if :

RI
a h g
abc + 2fgh − af² − bg² − ch² = 0, i.e. if h b f = 0.
g f c

A
Such an equation is obtained again by multiplying the two equation of lines
(a1x + b1y + c1) (a2x + b2y + c2) = 0
(ii) The angle θ between the two lines representing by a general equation is the same as that

UH
between the two lines represented by its homogeneous part only.
Example # 38 : Prove that the equation 6x2 + 13xy + 6y2 + 8x + 7y + 2 = 0 represents a pair of straight lines.
Find the co-ordinates of their point of intersection.
Solution : Given equation is 6x2 + 13xy + 6y2 + 8x + 7y + 2 = 0
Writing the equation (1) as a quadratic equation in x we have
JA
6x2 + (13y + 8) x + 6y2 + 7y + 2 = 0
−(13y + 8) ± (13y + 8)2 − 4.6(6y 2 + 7y + 2)
∴ x=
12
−(13y + 8) ± 169y + 208y + 64 − 144y 2 − 168y − 48
2
=
12
LP

−(13y + 8) ± 25y + 40y + 16


2
−(13y + 8) ± (5y + 4)
= =
12 12
⇒ 12x = –13y – 8 + 5y + 4 or 12x = –13y – 8 – 5y – 4
⇒ 3x + 2y + 1 = 0 or 2x + 3y + 2 = 0
Hence equation (1) represents a pair of straight lines whose equation are
A

3x + 2y + 1 = 0 .....(1) and 2x + 3y + 2 = 0 .....(2)


Solving these two equations, the required point of intersection is
NK

Self practice problem :


(35) Find the combined equation of the straight lines passing through the point (1, 1) and parallel
to the lines represented by the equation x2– 5xy+4y2+x+2y–2 = 0 and find the angle between
them.
3
Ans. (35) x2 – 5xy + 4y2 + 3x – 3y = 0, tan–1  
5
SA

Homogenization :
This method is used to write the joint equation of two lines connecting origin to the points of intersection
of a given line and a given second degree curve. The equation of a pair of straight lines joining origin to
the points of intersection of the line L ≡ x + my + n = 0 and a second degree curve
S ≡ ax² + 2hxy + by² + 2gx + 2fy + c = 0
2
 x + my   x + my   x + my 
is ax² + 2hxy + by² + 2gx   + 2fy   + c   = 0.
 −n   −n   −n 
The equation is obtained by homogenizing the equation of curve with the help of equation of line.
x + my
Notes : (i) Here we have written 1 as and converted all terms of the curve to
−n
second degree expressions
(ii) Equation of any curve passing through the points of intersection of two curves
C1 = 0 and C2 = 0 is given by λ C1 + µ C2 = 0, where λ & µ are parameters.
Example # 39 : All chords of the curve 2x2 – 3y2 + 4x – 2y = 0 which subtend a right angle at the origin passes

RI
through a fixed point. Find that point

Solution : Let Ax + By = 1 be a chord of the curve 2x 2 – 3y2 + 4x – 2y = 0

A
Equation of the given curve is 2x2 – 3y2 + 4x – 2y = 0

and equation of the chord is Ax + By = 1 ....(i)

UH
Making equation (1) homogeneous equation of the second degree in x any y with the help of

(1), we have 2x2 – 3y2 + 4x (Ax + By) – 2y(Ax + By) = 0

x2(2 + 4A) + y2 (–3 – 2B) + xy (4B – 2A) = 0 ....(ii)


JA
since line represented by (ii) are at right angle

⇒ coefficient of x2+ coefficient of y2 = 0

⇒ 2 + 4A + (–3 – 2B) = 0
LP

4A – 2B = 1

chord Ax + By = 1

x=4 y = –2
A

fixed point (4, – 2)


NK

Self practice problems :

(36) Find the equation of the straight lines joining the origin to the points of intersection of the line

3x + 4y – 5 = 0 and the curve 2x2 + 3y2 = 5.


SA

(37) Find the equation of the straight lines joining the origin to the points of intersection of the line

lx + my + n = 0 and the curve y2 = 4ax. Also, find the condition of their perpendicularity.

Ans. (36) x2 – y2 – 24xy = 0 (37) 4alx2 + 4amxy + ny2 = 0; 4al + n = 0


 Marked questions are recommended for Revision.

PART - I : SUBJECTIVE QUESTIONS


Section (A) : Distance formula, section formula, Area of Triangle & polygon,
Collinearity, slope
A-1. (i) Prove that the points (2a, 4a), (2a, 6a) and (2a + 3a , 5a) are the vertices of an equilateral
triangle whose side is 2a.
(ii) Find the points which trisect the line segment joining the points (0, 0) and (9, 12).
1 
A-2. (i) In what ratio does the point  , 6  divide the line segment joining the points (3, 5) and (–7,9)?
2 

RI
(ii) In which ratio P(2a – 2, 4a – 6)) divides Q (2a – 3, 3a – 7) and R(2a, 6a – 4).

A-3. (i) Find the value of λ such that points P(1, 2), Q(– 2,3) and R(λ + 1, λ) are not forming a triangle?
(ii) Find the ratio in which the line segment joining of the points (1, 2) and (– 2, 3) is divided by the

A
line 3x + 4y = 7
(iii) Find the harmonic conjugate of the point R (5, 1) with respect to points P (2, 10) and Q (6, – 2).

UH
A-4. A and B are the points (3, 4) and (5, – 2) respectively. Find the co-ordinates of a point P such that
PA = PB and the area of the triangle PAB = 10.

A-5. Find the area of the quadrilateral with vertices as the points given in each of the following :
(i) (0, 0), (4, 3), (6, 0), (0, 3) (ii) (0, 0), (a, 0), (a, b), (0, b)
JA
Section (B) : Different forms of straight lines and Angle between lines

B-1. Reduce x + 3 y + 4 = 0 to the :


(i) Slope intercepts form and find its slope and y-intercept.
(ii) Intercepts form and find its intercepts on the axes.
(iii) Normal form and find values of P and α .
LP

B-2. Find number of straight line passing through (2, 4) and forming a triangle of 16 sq. cm with the
coordinate axis.

B-3. Find the equation of the straight line that passes through the point A(– 5, – 4) and is such that the
A

portion intercepted between the axes is divided by the point A in the ratio 1 : 2 (internally).

B-4. The co-ordinates of the mid-points of the sides of a triangle are (2, 1), (5, 3) and (3, 7). Find the length
NK

and equation of its sides.

B-5. Find the straight line cutting an intercept of one unit on negative x-axis and inclined at 45°
(in anticlockwise direction) with positive direction of x-axis

11
B-6. Through the point P(4, 1) a line is drawn to meet the line 3x – y = 0 at Q where PQ = . Determine
SA

2 2
the equation of line.

B-7. A line having slope '1' is drawn from a point A(–3, 0) cuts a curve y = x 2 + x + 1 at P & Q. Find (AP)
(AQ).

B-8. Find the direction in which a straight line must be drawn through the point (1, 2), so that its point of
6
intersection with the line x + y = 4 may be at a distance from this point.
3

B-9. Through the point (3, 4) are drawn two straight lines each inclined at 45° to the straight line
x – y = 2. Find their equations and also find the area of triangle bounded by the three lines.

B-10. Find the equation of a straight lines which passes through the point (2, 1) and makes an angle of π/4
with the straight line 2x + 3y + 4 = 0
B-11. From (1, 4) you travel 5 2 units by making 135° angle with positive x-axis (anticlockwise) and then
4 units by making 120° angle with positive x-axis (clockwise) to reach Q. Find co-ordinates of point Q.

B-12. The ends of the hypotenuse of a right angled triangle are (6, 0) and (0, 6), then find the locus of third
vertex of triangle.

B-13. A point moves in the x-y plane such that the sum of its distances from two mutually perpendicular lines
is always equal to 3, then find the area enclosed by the locus of the point.

B-14. One side of a rectangle lies along the line 4x + 7y + 5 = 0. Two of its vertices are (–3, 1) and (1, 1).
Then find the equations of other sides.

Section (C) : Position of point, linear inequation, perpendicular distance, image & foot,

RI
Area of Parallelogram
C-1. Plot the region
(i) 6x + 2y ≥ 31 (ii) 2x + 5y ≤ 10 (iii) 8x + 3y + 6 > 0 (iv) x>2

A
C-2. Find coordinates of the foot of perpendicular, image and equation of perpendicular drawn from the point
(2, 3) to the line y = 3x – 4.

UH
C-3. Starting at the origin, a beam of light hits a mirror (in the form of a line) at the point A(4, 8) and reflected
line passes through the point B (8, 12). Compute the slope of the mirror.

C-4. Find the nearest point on the line 3x + 4y – 1 = 0 from the origin.

C-5. Find the position of the origin with respect to the triangle whose sides are x + 1 = 0, 3x – 4y – 5 = 0,
JA
5x + 12y – 27 = 0.

C-6. Find the area of parallelogram whose two sides are y = x + 3, 2x – y + 1 = 0 and remaining two sides
are passing through (0, 0).

C-7. Is there a real value of λ for which the image of the point (λ, λ − 1) by the line mirror 3 x + y = 6 λ is
LP

the point (λ2 + 1, λ) ? If so, find λ.

C-8. Find the equations of two straight lines which are parallel to x + 7y + 2 = 0 and at 2 distance away
from it.
A

C-9. Prove that the area of the parallelogram contained by the lines 4y – 3x – a = 0, 3y – 4x + a = 0,
2 2
4y – 3x – 3a = 0 and 3y – 4x + 2a = 0 is a.
7
NK

Section (D) : Centroid, orthocentre, circumcentre, incentre, excentre, Locus


D-1. For triangle whose vertices are (0, 0), (5, 12) and (16, 12). Find coordinates of
(i) Centroid (ii) Circumcentre
(iii) Incentre (iv) Excentre opposite to vertex (5, 12)
SA

D-2. Find the sum of coordinates of the orthocentre of the triangle whose sides are x = 3, y = 4 and
3x + 4y = 6.

D-3. Find equations of altitudes and the co-ordinates of the othocentre of the triangle whose sides are
3x – 2y = 6, 3x + 4y + 12 = 0 and 3x – 8y + 12 = 0.

D-4. A triangle has the lines y = m1x and y = m2x for two of its sides, where m1, m2 are the roots of the
equation x2 + ax – 1 = 0, then find the orthocentre of triangle.

D-5. Prove that the circumcentre, orthocentre, incentre & centroid of the triangle formed by the points
A(−1, 11); B (−9, −8); C (15, −2) are collinear, without actually finding any of them.

D-6. Find locus of centroid of ∆AOB if line AB passes through (3, 2), A and B are on coordinate axes.

D-7. Find the locus of the centroid of a triangle whose vertices are (a cos t, a sin t), (b sin t, –b cos t) and
(1, 0), where ‘t’ is the parameter.
D-8. Show that equation of the locus of a point which moves so that difference of its distance from two given
x2 y2
points (ae, 0) and (–ae, 0) is equal to 2a is 2 – 2 2 = 1.
a a (e − 1)

D-9. Find the locus of point of intersection of the lines x cos α + y sin α = a and x sin α – y cos α = b, where
α is a parameter.

D-10. The area of the triangle formed by the intersection of a line parallel to x-axis and passing through
P(h, k) with the lines y = x and x + y = 2 is 4h2. Find the locus of the point P.

Section (E) : Angle Bisector, condition of concurrency, family of straight lines

RI
E-1. Find equations of acute and obtuse angle bisectors of the angle between the lines 4x + 3y – 7 = 0 and
24x + 7y – 31 = 0.

E-2. Find the equation of a straight line passing through the point (4, 5) and equally inclined to the lines

A
3x = 4y + 7 and 5y = 12x + 6.

E-3. The line x + 3y − 2 = 0 bisects the angle between a pair of straight lines of which one has equation

UH
x − 7y + 5 = 0,then find equation of other line

E-4. Find the value of λ such that lines x + 2y = 3, 3x – y = 1 and λx + y = 2 can not form a triangle.

E-5. Find values of λ for which line y = x + 1, y = λx + 2 and y = (λ 2 + λ –1) x + 3 are con-current.
JA
E-6. Find the equation to the straight line passing through
(i) The point (3, 2) and the point of intersection of the lines 2x + 3y = 1 and 3x – 4y = 6.
(ii) The intersection of the lines x + 2y + 3 = 0 and 3x + 4y + 7 = 0 and perpendicular to the straight
line y – x = 8.

E-7. Find the locus of the circumcentre of a triangle whose two sides are along the co-ordinate axes and
LP

third side passes through the point of intersection of the lines ax + by + c = 0 and x + my + n = 0.

Section (F) : Pair of straight lines, Homogenization


F-1. If the slope of one of the lines represented by ax2 + 2hxy + by2 = 0 be the nth power of the other, then
A

1 1
prove that (abn ) n+1 + (anb) n+1 + 2h = 0
F-2. For what value of λ does the equation 12x2 – 10xy + 2y2 + 11x – 5y + λ = 0 represent a pair of straight
NK

lines ? Find their equations, point of intersection, acute angle between them and pair of angle bisector.

F-3. (i) Find the integral values of 'h' for which hx 2 – 5xy + 4hy2 + x + 2y – 2 = 0 represents two real
straight lines.
(ii) If the pair of lines represented by equation k(k – 3) x2 + 16xy + (k + 1)y2 = 0 are perpendicular
to each other, then find k.
SA

F-4. Find the equation of the straight lines joining the origin to the points of intersection of the line
lx + my + n = 0 and the curve y2 = 4ax. Also, find the condition of their perpendicularity.

F-5. Find the condition that the diagonals of the parallelogram formed by the lines ax + by + c =0 ;
ax + by + c′ = 0; a'x + b'y + c = 0, a'x + b'y + c' = 0 are at right angles. Also find the equation to the
diagonals of the parallelogram.

PART - II : ONLY ONE OPTION CORRECT TYPE


Section (A): Distance formula, section formula, Area of Triangle & polygon,
Collinearity, slope
A-1. Mid point of A(0, 0) and B(1024, 2048) is A1. mid point of A1 and B is A2 and so on. Coordinates of A10
are.
(A) (1022, 2044) (B) (1025, 2050) (C) (1023, 2046) (D) (1, 2)
A-2. If the points (k, 2 – 2k), (1 – k, 2k) and (–k –4, 6 – 2k) be collinear, the number of possible values of k
are
(A) 4 (B) 2 (C) 1 (D) 3

A-3. Given a ∆ABC with unequal sides. P is the set of all points which is equidistant from B & C and Q is the
set of all point which is equidistant from sides AB and AC. Then n(P ∩ Q) equals :
(A) 1 (B) 2 (C) 3 (D) Infinite

A-4. A line segment AB is divided internally and externally in the same ratio (> 1) at P and Q respectively
and M is mid point of AB.
Statement-1: MP, MB, MQ are in G.P.
Statement-2 AP, AB and AQ are in HP.
(A) STATEMENT-1 is true, STATEMENT-2 is true and STATEMENT-2 is correct explanation for

RI
STATEMENT-1
(B) STATEMENT-1 is true, STATEMENT-2 is true and STATEMENT-2 is not correct explanation
for STATEMENT-1
(C) STATEMENT-1 is true, STATEMENT-2 is false

A
(D) STATEMENT-1 is false, STATEMENT-2 is true
(E) Both STATEMENTS are false

A-5. Find the area of the triangle formed by the mid points of sides of the triangle whose vertices are

UH
(2, 1), (– 2, 3), (4, – 3)
(A) 1.5 sq. units (B) 3 sq. units (C) 6 sq. units (D) 12 sq. units

Section (B) : Different forms of straight lines and Angle between lines
JA
B-1. A straight line through P (1, 2) is such that its intercept between the axes is bisected at P. Its equation
is :
(A) x + 2y = 5 (B) x – y + 1 = 0 (C) x + y – 3 = 0 (D) 2x + y – 4 = 0

B-2. The number of integral points (integral point means both the coordinates should be integer) exactly in
the interior of the triangle with vertices (0, 0), (0, 21) and (21, 0), is
LP

(A) 133 (B) 190 (C) 233 (D) 105


B-3. The line joining two points A (2, 0) and B (3, 1) is rotated about A in the anticlock wise direction through
an angle of 15º. The equation of the line in the new position is :
(A) x − 3 y − 2 = 0 (B) x − 2y − 2 = 0 (C) 3 x − y − 2 3 = 0 (D) 2 x – y – 2 2 = 0
A

B-4. In a ∆ABC, side AB has the equation 2x + 3y = 29 and the side AC has the equation
x + 2y = 16. If the mid point of BC is (5, 6), then the equation of BC is
(A) 2x + y = 16 (B) x + y = 11 (C) 2x – y = 4 (D) x + y = 10
NK

B-5. A square of side 'a' lies above the x-axis and has one vertex at the origin. The side passing through the
 π
origin makes an angle α  0 < α <  with the positive direction of x-axis. The equation of its diagonal
 4
not passing through the origin is :
(A) y (cos α – sin α) – x (sin α – cos α) = a (B) y (cos α + sin α) + x (sin α – cos α) = a
SA

(C) y (cos α + sin α) + x (sin α + cos α) = a (D) y (cos α + sin α) + x (cos α – sin α) = a

B-6. Find equation of a straight line on which length of perpendicular from the origin is four units and the line
makes an angle of 120º with the positive direction of x-axis.
(A) 3 x – y = 0 (B) 3 x + y = 8 (C) x + 3 y = 8 (D) x – 3 y = 8

B-7. The distance of the point (2, 3) from the line 2 x − 3 y + 9 = 0 measured along a line x – y + 1 = 0 is :
(A) 5 3 (B) 4 2 (C) 3 2 (D) 2 2

B-8. If a point P(x, y) from where line drawn cuts coordinate axes at A and B (with A on x-axis and B on
x2 y2
y-axis) satisfies α. + β = 1 , then α + β is
PB 2 PA 2
(A) 1 (B) 2 (C) 3 (D) 4
B-9. Two particles start from the point (2, – 1), one moving 2 units along the line x + y = 1 and the other 5
units along the line x – 2y = 4. If the particles move towards increasing y, then their new positions are
(
(A) 2 − 2, 2 − 1 , ) ( 2 5 + 2, 5 − 1) (
(B) 2 5 + 2, 5 − 1 , ) (2 2, 2 + 1)
(C) ( 2 + 2, 2 + 1) , ( 2 5 + 2, 5 + 1) (D) none of these

B-10. Equation of a straight line passing through the origin and making with x − axis an acute angle twice the
size of the angle made by the line y = (0.2) x with the x − axis, is :
(A) y = (0.4) x (B) y = (5/12) x (C) 6y − 5x = 0 (D) 6y + 5x = 0

B-11. The points A (1, 3) and C (5,1) are the oppositive vertices of rectangle. The equation of line passing
through other two vertices and of gradient 2, is
(A) 2x+y–8 = 0 (B) 2x–y–4 = 0 (C) 2x–y+4 = 0 (D) 2x+y = 0

RI
B-12. The point (–4,5) is the vertex of a square and one of its digonals is 7x–y+8 = 0. The equation of the
other diagonal is
(A) 7x–y+23 = 0 (B) 7y + x =30 (C) 7y + x =31 (D) x –7y =30

A
B-13. Two straight lines x + 2y = 2 and x + 2y = 6 are given, then find the equation of the line parallel to given
lines and divided distance between lines in the ratio 2 : 1 internally
(A) 3x + 6y + 8 = 0 (B) 3x + 6y = 14 (C) 3x + 6y + 14 = 0 (D) 3x + 2y = 10

UH
Section (C) : Position of point, linear inequation, perpendicular distance, image & foot
C-1. The set of values of 'b' for which the origin and the point (1, 1) lie on the same side of the straight line,
a2x + a by + 1 = 0 ∀ a ∈ R, b > 0 are :
(A) b ∈ (2, 4) (B) b ∈ (0, 2) (C) b ∈ [0, 2] (D) (2, ∞)
JA
C-2. The point (a2, a + 1) is a point in the angle between the lines 3x − y + 1 = 0 and x + 2y − 5 = 0
containing the origin, then
(A) a ≥ 1 or a ≤ − 3 (B) a ∈ (− 3, 0)∪(1/3, 1) (C) a ∈ (0, 1) (D) a ∈ (– ∞, 0)

C-3. Find area of region represented by 3x + 4y > 12, 4x + 3y > 12 and x + y < 4
8 4 7 8
LP

(A) (B) (C) (D)


7 7 8 7

C-4. The image of the point A (1, 2) by the line mirror y = x is the point B and the image of B by the line
mirror y = 0 is the point (α, β), then :
(A) α = 1,β = − 2 (B) α = 0,β = 0 (C) α = 2,β = − 1 (D) α = 1,β = − 1
A

C-5. The equations of the perpendicular bisector of the sides AB and AC of a ∆ABC are
x – y + 5 = 0 and x + 2y = 0 respectively. If the point A is (1, –2), then the equation of the line BC is :
NK

(A) 14x + 23y = 40 (B) 14x – 23y = 40 (C) 23x + 14y = 40 (D) 23x – 14y = 40

C-6. A light beam emanating from the point A(3, 10) reflects from the straight line 2x + y − 6 = 0 and then
passes through the point B(4, 3). The equation of the reflected beam is x + 3y – λ = 0, then the value of
λ is
(A) 11 (B) 12 (C) 13 (D) 14
SA

Section (D) : Centroid, orthocentre, circumcentre, incentre, excentre, Locus


D-1. The orthocentre of the triangle ABC is 'B' and the circumcentre is 'S' (a, b). If A is the origin, then the
co−ordinates of C are :
a b
(A) (2a, 2b) (B)  , 
2 2
(C) (a2 + b2 , 0 ) (D) none

D-2. A triangle ABC with vertices A (− 1, 0),B (− 2, 3/4) & C (− 3, − 7/6) has its orthocentre H. Then the
orthocentre of triangle BCH will be :
(A) (− 3, − 2) (B) (1, 3) (C) (− 1, 2) (D) none of these

D-3. Find locus of centroid of ∆ABC, if B(1, 1), C(4, 2) and A lies on the line y = x + 3.
(A) 3x + 3y + 1 = 0 (B) x + y = 3 (C) 3x – 3y + 1 = 0 (D) x – y = 3
D-4. The locus of the mid-point of the distance between the axes of the variable line x cos α + y sin α = p,
where p is constant, is
1 1 4 4 1 1 2
(A) x2 + y2 = 4p2 (B) 2 + 2 = 2 (C) x2 + y2 = 2 (D) 2 – 2 = 2
x y p p x y p

D-5. Find the locus of a point which moves so that sum of the squares of its distance from the axes is equal
to 3.
(A) x2 + y2 = 9 (B) x2 +y2 = 3 (C) |x|+|y|=3 (D) x2 – y2 = 3

D-6. A variable straight line passes through a fixed point (a, b) intersecting the co −ordinates axes at A & B. If
'O' is the origin, then the locus of the centroid of the triangle OAB is :
(A) bx + ay − 3xy = 0 (B) bx + ay − 2xy = 0 (C) ax + by − 3xy = 0 (D) ax + by − 2xy = 0

RI
D-7. Consider a triangle ABC, whose vertices are A(–2, 1), B(1, 3) and C(x, y). If C is a moving point such
that area of ∆ABC is constant, then locus of C is :
(A) Straight line (B) Circle (C) Ray (D) Parabola

A
D-8. If the equation of the locus of a point equidistant from the points (a 1, b1) and (a2, b2) is
(a1 – a2) x + (b1 – b2) y + c = 0, then the value of ‘c’ is :

UH
1
(A) (a 2 + b22 – a12 – b12) (B) a12 – a22 + b12 – b22
2 2
1
(C) (a 2 + a22 + b12 + b22) (D) a12 + b12 − a22 − b22
2 1
JA
Section (E) : Angle Bisector, condition of concurrency, family of straight lines
E-1. The equation of bisectors of two lines L1 & L2 are 2 x − 16 y − 5 = 0 and 64 x + 8 y + 35 = 0. If the line
L1 passes through (− 11, 4), the equation of acute angle bisector of L1 & L2 is :
(A) 2 x − 16 y − 5 = 0 (B) 64 x + 8 y + 35 = 0
(C) 2x + 16y + 5 = 0 (D) 2x + 16y − 5 = 0
LP

E-2. The equation of the internal bisector of ∠BAC of ∆ABC with vertices A(5, 2), B(2, 3) and
C(6, 5) is
(A) 2x + y + 12 = 0 (B) x + 2y – 12 = 0 (C) 2x + y – 12 = 0 (D) 2x – y – 12 = 0

E-3. The equation of the bisector of the angle between two lines 3 x − 4 y + 12 = 0 and
A

12 x − 5 y + 7 = 0 which contains the point (– 1, 4) is :


(A) 21x + 27y − 121 = 0 (B) 21x − 27y + 121 = 0
− 3 x + 4 y − 12 12 x − 5 y + 7
NK

(C) 21x + 27y + 191 = 0 (D) =


5 13
E-4. The least positive value of t so that the lines x = t + a, y + 16 = 0 and y =ax (where a is real variable)
are concurrent is
(A) 2 (B) 4 (C) 16 (D) 8

Consider the family of lines 5x + 3y – 2 + λ1 (3x – y – 4) = 0 and x – y + 1 + λ2(2x – y – 2) = 0. Equation


SA

E-5.
of a straight line that belong to both families is -
(A) 25x – 62y + 86 = 0 (B) 62x – 25y + 86 = 0 (C) 25x – 62y = 86 (D) 5x – 2y – 7 = 0

E-6. The equation of a line of the system 2x + y + 4 + λ (x – 2y – 3) = 0 which is at a distance 10 units from
point A(2, –3) is
(A) 3x + y + 1 = 0 (B) 3x – y + 1 = 0 (C) y – 3x + 1 = 0 (D) y + 3x – 2 = 0

E-7. The lines ax + by + c = 0, where 3a + 2b + 4c = 0, are concurrent at the point


1 3  3 1
(A)  ,  (B) (1, 3) (C) (3, 1) (D)  4, 2
2 4  

E-8. Find the equation of a straight line which passes through the point of intersection of the straight lines
x + y – 5 = 0 and x – y + 3 = 0 and perpendicular to the straight line intersecting x-axis at the point
(–2, 0) and the y-axis at the point (0, –3),
(A) 2x + 3y + 10 = 0 (B) 2x – 3y + 10 = 0 (C) 2x – 5y + 10 = 0 (D) 2x + 5y + 10 = 0

E-9. The line parallel to the x-axis and passing through the intersection of the lines ax + 2by + 3b = 0
and bx – 2ay –3a = 0, where (a, b) ≠ (0, 0) is
(A) Above the x-axis at a distance of 3/2 from it
(B) Above the x-axis at a distance of 2/3 from it
(C) Below the x-axis at a distance of 3/2 from it
(D) Below the x-axis at a distance of 2/3 from it

Section (F) : Pair of straight lines, Homogenization


F-1. If the slope of one line of the pair of lines represented by ax 2 + 10xy + y2 = 0 is four times the slope of
the other line, then a =
(A) 1 (B) 2 (C) 4 (D) 16

RI
F-2. The combined equation of the bisectors of the angle between the lines represented by
(x2 + y2) 3 = 4xy is
x2 − y2 xy

A
(A) y2 – x2 = 0 (B) xy = 0 (C) x2 + y2 = 2xy (D) =
3 2
F-3. The equation of second degree x + 2 2 xy + 2y + 4x + 4 2 y + 1 = 0 represents a pair of straight
2 2

lines. The distance between them is

UH
4
(A) 4 (B) (C) 2 (D) 2 3
3
F-4. The straight lines joining the origin to the points of intersection of the line 2x + y = 1 and curve
3x2 + 4xy – 4x + 1 = 0 include an angle :
π π π π
JA
(A) (B) (C) (D)
2 3 4 6

PART - III : MATCH THE COLUMN


1. Column –Ι Column – ΙΙ
(A) The points A(–4, –1), B (–2, –4), C (4, 0) (p) Square
LP

and D(2, 3) are the vertices of

(B) The figure formed by the lines 2x + 5y + 4 = 0, (q) Rectangle


5x + 2y + 7 = 0, 2x + 5y + 3 = 0 and 5x + 2y + 6 = 0 is
A

(C) A quadrilateral whose diagonal are (r) Rhombus


perpendicular bisectors of each other must be

(D) A quadrilateral whose diagonals are angle (s) Parallelogram


NK

bisector of sides and bisect each other must be

2. Match The Followings :


Column –Ι Column – ΙΙ
SA

(A)  P lies on the line y = x and Q lies on y = 2x. The equation for (p) 36
the locus of the mid point of PQ, if |PQ| = 4, is
25x2 – λxy + 13y2 = 4, then λ equals

(B) The line (K + 1)2 x + ky – 2K2 – 2 = 0 passes through a


point (α, β) regardless of value K. Then (α − β) equals : (q) 6

(C) Let two lines be C1 : 3x – 4y + 1 = 0 and C2 : 8x + 6y + 1 = 0


suppose (α, β) is a point which is at unit distance from
each of the given lines.
then sum of all possible value of α is (r) –4/5

(D) If distance between the pair of parallel lines (s) –1


x2 + 2xy + y2 – 8ax – 8ay – 9a2 = 0 is 25 2 ,
then ‘a/5' is equal to
 Marked questions are recommended for Revision.

PART - I : ONLY ONE OPTION CORRECT TYPE


1. On the portion of the straight line x + 2y = 4 intercepted between the axes, a square is constructed on
the side of the line away from the origin. Then the point of intersection of its diagonals has co−ordinates
:
(A) (2, 3) (B) (3, 2) (C) (3, 3) (D) (2,2)

π
2. If the straight line ax + by + p = 0 and xcosα + ysinα = p enclosed an angle of and the line
4

RI
xsinα – ycosα = 0 meets them at the same point, then a2 + b2 is
(A) 4 (B) 3 (C) 2 (D) 1

3. A ∆ABC is formed by the lines 2x – 3y – 6 = 0 , 3x – y + 3 = 0 and 3x + 4y – 12 = 0. If the points P(α, 0)

A
and Q(0, β) always lie on or inside the ∆ABC, then ;
(A) α ∈ [–1, 2] & β∈ [–2, 3] (B) α ∈ [–1, 3] & β∈ [–2, 4]
(C) α ∈ [–2, 4] & β∈ [–3, 4] (D) α ∈ [–1, 3] & β∈ [–2, 3]

UH
 t t 
4. If P  1 + ,2 +  be any point on a line, then the range of values of t for which the point P lies
 2 2
between the parallel lines x + 2y = 1 and 2x + 4y = 15 is
4 2 5 2 5 2 4 2 4 2 2
(A) – <t< (B) 0 < t < (C) – <t<0 (D) – <t<
JA
3 6 6 5 3 6

5. The point A(4, 1) undergoes following transformations successively :


(i) reflection about line y = x
(ii) translation through a distance of 3 units in the positive direction of x-axis
(iii) rotation through an angle 105° in anti-clockwise direction about origin O.
Then the final position of point A is
LP

 1 7   1 7 
(A) 
 2
,
2
 (
(B) −2, 7 2 ) (C)  −

,
2 2
 (
(D) −2 6, 2 2 )
6. Given two points A ≡ (–2, 0) and B ≡ (0, 4), then find coordinate of a point P lying on the line
A

2x – 3y = 9 so that perimeter of ∆ APB is least.


 42 11   84 74   21 37 
(A)  ,–  (B)  ,–  (C)  ,–  (D) (0, – 3)
 13 3   13 13   17 17 
NK

7. A ray of light is sent from the point (1, 4). Upon reaching the x-axis, the ray is reflected from the point
(3, 0). This reflected ray is again reflected by the line x + y = 5 and intersect y-axis at P. Find the
co-ordinate of P.
1   − 1  1  1 
(A)  ,0  (B)  0,  (C)  0,  (D)  2, 
2   2   3  −2 
SA

8. AB is a variable line sliding between the co-ordinate axes in such a way that A lies on X-axis and B lies
on Y-axis. If P is a variable point on AB such that PA = b, PB = a and AB = a + b, then equation of locus
of P is
x2 y2 x2 y2
(A) 2 + 2 = 1 (B) 2 – 2 =1 (C) x2 + y2 = a2 + b2 (D) x2 – y2 = a2 + b2
a b a b

9. If the distance of any point (x, y) from the origin is defined as d(x, y) = max { |x|, |y|}, d(x, y) = a (where
'a' is non-zero constant), then the locus is
(A) A circle (B) Straight line (C) A square (D) A triangle

10. Two ends A & B of a straight line segment of constant length 'c' slide upon the fixed rectangular axes
OX & OY respectively. If the rectangle OAPB is completed. Then find locus of the foot of the
perpendicular drawn from P to AB.
(A) x2/3 + y2/3 = c2/3 (B) x2/3 + y2/3 = c1/3 (C) x1/3 + y1/3 = c2/3 (D) x1/3 + y1/3 = c1/3

x y
11. Let the line + = 1 cuts the x and y axes at A and B respectively. Now a line parallel to the given
a b
line cuts the coordinate axis at P and Q and points P and Q are joined to B and A respectively. The
locus of intersection of the joining lines is
x y x y x y x y
(A) − =0 (B) + = 0 (C) − = 0 (D) + = 0
a b a b b a b a

12. A variable line whose slope is –2 cuts the x and y axes respectively at points A and C. A rhombus
ABCD is completed such that vertex B lies on the line y = x. Then the locus of vertex D is
(A) 2x + y = 1 (B) x – y = 0 (C) x + y = 0 (D) x + 2y = 0

RI
13. ABCD is a square away from origin of side length ‘a’. Its side AB slides between x and y−axes in first
quadrant with A on x-axis and B on y-axis. The locus of the foot of perpendicular dropped from the point
E on the diagonal AC (where E is the midpoint of the side AD), is
a2
(A) (y – x)2 + (x – 3y)2 = a2 (B) (y – x)2 + (x – 3y)2 =

A
2
a2
(C) (y – x)2 + (x – 3y)2 = (D) None of these
4

UH
14_. The locus of circumcentre of the triangle formed by vertices A((–pq – p – q), –(1 + p)(1 + q)),
B(pq + p – q, (1 + p)(1 + q)), C(pq + q – p, (1 + p)(1 + q)) is
(A) y + x = 0 (B) y – x = 0 (C) x2 + y2 = 1 (D) xy = 1

15_.
JA
Let two sides of rectangle of area 20 units are along lines x – y = 0 and x + y = 2, then the locus of point
of intersection of diagnals is
(A) (x –1)2 + (y –1)2 = 10 or (y –1)2 + (x –1)2 = 10
(B) (x –1)2 – (y –1)2 = 10 or (y –1)2 – (x –1)2 = 10
(C) (x +1)2 – (y +1)2 = 10 or (y +1)2 – (x +1)2 = 10
(D) (x –1)2 + (y –1)2 = 10 or (y +1)2 – (x +1)2 = 10
16. Area of the triangle formed by the line x + y = 3 and angle bisectors of the pair of straight lines
LP

x2 – y2 + 2y = 1 is
(A) 2 sq units (B) 4 sq. units (C) 6 sq. units (D) 8 sq. units
17. Equation of the line pair through the origin and perpendicular to the line pair
xy −3y2 + y − 2x + 10 = 0 is :
(A) xy − 3y2 = 0 (D) x2 − y2 = 0
A

(B) xy + 3x2 = 0 (C) xy + 3y2 = 0


18. Find the equation of the two straight lines which together with those given by the equation
6x2 – xy – y2 + x + 12y – 35 = 0 will make a parallelogram whose diagonals intersect in the origin.
NK

(A) 6x2 – xy – y2 – x – 12y – 35 = 0 (B) 6x2 – xy – y2 – x – 12y + 35 = 0


(C) 6x – xy – y – x + 12y – 35 = 0
2 2
(D) 6x2 – xy – y2 + x – 12y – 35 = 0

x2 y2
19. The curve passing through the points of intersection of S1 ≡ – 1 = 0 and
+
a2 b2
S2 ≡ x2 + y2 + 2gx + 2fy + c = 0 represents a pair of straight lines which are
SA

(A) equally inclined to the x - axis (B) perpendicular to each other


(C) parallel to each other (D) Not equally inclined to y-axis

PART - II : SINGLE AND DOUBLE VALUE INTEGER TYPE


 a3 a 2 − 3   b3 b 2 − 3   c3 c2 − 3 
If the points   ,   and 
 c − 1 c − 1 
1. , , , are collinear for three distinct values a,
 a −1 a −1   b −1 b −1 
     
b, c and a ≠ 1, b ≠ 1 and c ≠ 1, then find the value of abc– (ab + bc + ac) + 3 (a + b + c).

2. Find number of integral values of λ if (λ, λ + 1) is an interior points of ∆ABC, where


A ≡ (0, 3), B ≡ (–2, 0) and C ≡ (6, 1).
3. Let ABC be a triangle such that the coordinates of the vertex A are (– 3, 1). Equation of the median
through B is 2x + y – 3 = 0 and equation of the angular bisector of C is 7x – 4y – 1 = 0. Find the slope of
line BC.

4. A(3, 4), B(0, 0) and C(3, 0) are vertices of ∆ABC. If 'P' is the point inside the ∆ABC, such that
d(P, BC) ≤ min. {d(P, AB), d (P, AC)}. Then the maximum of d (P, BC) is.
(where d(P, BC) represent distance between P and BC).

5. Drawn from the origin are two mutually perpendicular straight lines forming an isosceles triangle
together with the straight line 2x + y = 5. Then find the area of the triangle.

6. On the straight line y = x + 2, a point (a, b) is such that the sum of the square of distances from the
straight lines 3x – 4y + 8 = 0 and 3x – y – 1 = 0 is least, then find value of 11 (a + b).

RI
7. Parallelogram ABCD is cut by (2n –1) number of parallel lines in which one is diagnal AC. Distance
between any two nearest lines is same which is also equal to distance of B, D from respective nearest
1
line anong these. Ratio of area of smallest triangle so formed to area of parallelogram is . Find n.
32

A
8. A is a variable point on x-axis and B(0,b) is a fixied point. A equilateral triangle ABC is completed with
vertex C away from origin. If the locus of the point C is αx + βy = b, then
α2 + β2 is

UH
9. Two lines (L1 and L2) are drawn from point (α, α) making an angle 450 with the lines
L3 ≡ x + y – f (α) = 0 and L4 ≡ x + y + f (α) = 0. L1 intersects L3 and L4 at A and B and L2 intersects L3
and L4 at C and D respectively (|2α| > |f (α)|). If the area of trapezium ABDC is independent of α. if
f(α) = λαq , where λ is a constant, then |q| is
JA
10. The portion of the line ax + by – 1 = 0, intercepted between the lines ax + y + 1 = 0 and x + by = 0
subtends a right angle at the origin and the condition in a and b is λa + b + b2 = 0, then find value of λ.

11. If the straight lines joining the origin and the points of intersection of the curve
5x2 + 12xy − 6y2 + 4x − 2y + 3 = 0 and x + ky − 1 = 0 are equally inclined to the x-axis, then find the
value of | k |.
LP

12. If the points of intersection of curves C1 = 4 y2 – λx2 − 2x y − 9 x + 3 and C2 = 2 x2 + 3 y2 − 4 x y + 3 x −1


subtends a right angle at origin, then find the the value of λ.
A

13. A parallelogram is formed by the lines ax2 + 2hxy + by2 = 0 and the lines through (p, q) parallel to them
and the equation of the diagonal of the parallelogram which doesn’t pass through origin is
(λx – p)(ap + hq) + (µy – q)(hp + bq) = 0, then find the value of λ 3 + µ3.
NK

14. The equation 9x3 + 9x2 y – 45x2 = 4y3 + 4xy2 – 20y2 represents 3 straight lines, two of which passes
through origin. Then find the area of the triangle formed by these lines

15. Let the integral points inside or on the boundary of region bounded by straight lines as shown in figure
is equal to k, then k − 7 is equal to
SA

PART - III : ONE OR MORE THAN ONE OPTIONS CORRECT TYPE


1. Point P(2, 3) lies on the line 4x + 3y = 17. Then find the co-ordinates of points farthest from the line
which are at 5 units distance from the P.
(A) (6, 6) (B) (6, –6) (C) (2, 0) (D) (–2, 0)

2. Find the equation of the line passing through the point (2, 3) & making intercept of length
2 units between the lines y + 2x = 3 & y + 2x = 5.
(A) 3x – 4y = 18 (B) x = 2 (C) 3x + 4y = 18 (D) x + 2 = 0

3. In a triangle ABC, co-ordinates of A are (1, 2) and the equations to the medians through B and C are
x + y = 5 and x = 4 respectively. Then the co-ordinates of B and C will be
(A) (− 2, 7), (4, 3) (B) (7, − 2), (4, 3) (C) (2, 7), (− 4, 3) (D) (2, − 7), (3, − 4)

4
4. A is a point on either of two rays y + 3 x = 2 at a distance of units from their point of
3
intersection. The co−ordinates of the foot of perpendicular from A on the bisector of the angle between
them is/are
 2   2 
(A)  − , 2 (B) (0, 0) (C)  , 2 (D) (0, 4)
 3   3 

RI
5. If one side of a square is parallel to 3x – 4y = 0 & its area being 16 while centre being (1, 1), then find
equation of sides of square.
(A) 3x − 4y + 11 = 0 (B) 3x − 4y − 9 = 0 (C) 4x + 3y + 3 = 0 (D)4x + 3y − 17 = 0

A
6. Find the equations of the sides of a triangle having (4, –1) as a vertex, if the lines
x – 1 = 0 and x – y – 1 = 0 are the equations of two internal bisectors of its angles.
(A) 2x – y + 3 = 0 (B) x + 2y – 6 = 0 (C) 2x + y – 7 = 0 (D) x – 2y – 6 = 0

UH
7. A straight line L with negative slope passes through the point (8, 2) and cuts the positive coordinate
axes at points P and Q, then the correct statement(s) among the following is/are (O is origin)
(A) The absolute minimum value of OP + OQ, where O is origin is 18 2
(B) Minimum area of ∆OPQ is 32
(C) The absolute minimum value of OP + OQ, where O is origin is 18
JA
 −1 
(D) Area of ∆OPQ is minimum for slope   .
 4 

8. The equation of the diagonals of a rectangle are y + 8x – 17 = 0 and y – 8x + 7 = 0. If the area of the
rectangle is 8 sq. units, find the equation of the sides of the rectangle.
(A) y = 1 (B) y = 9 (C) x = 1 (D) x = 2.
LP

9. Two adjacent sides of a rhombus are 2x + 3y = a – 5 and 3x + 2y = 4 – 2a and its diagonals intersect at
the point (1, 2), then a can be –
10 10
(A) – 16 (B) 16 (C) – (D)
3 3
A

10. A line L1 ≡ 3y − 2x − 6 = 0 is rotated about its point of intersection with y−axis in clockwise direction to
49
make it L2 such that the area formed by L1, L2, x−axis and line x = 5 is sq units if its point of
3
NK

intersection with x = 5 lies below x–axis then points lying on the equation of L2 are
(A) (3, – 1) (B) (4,2) (C) (1,1) (D) (3,3)

11. Let D(x4, y4) be a point such that ABCD is a square & M & P are the midpoints of the sides BC & CD
respectively, then
SA

(A) Ratio of the areas of ∆AMP and the square is 3 : 8


(B) Ratio of the areas of ∆MCP & ∆AMD is 1 : 1
(C) Ratio of the areas of ∆ABM & ∆ADP is 1 : 1
(D) Ratio of the areas of the quandrilateral AMCP and the square is 1 : 3

12. The equations of perpendicular of the sides AB & AC of ∆ ABC are x − y − 4 = 0 and 2x − y − 5 = 0
3 5
respectively. If the vertex A is (− 2, 3) and circumcenter is  ,  , then which of the following is true.
2 2
(A) equation of median of side AB is x – y + 1 = 0 (B) centroid of triangle ABC is (3, 1)
(C) vertex C is (4, 0) (D) Area of triangle ABC is 12.

13. Triangle ABC lies in the cartesian plane and has an area of 70 sq. units. The coordinates of B and C
are (12, 19) and (23, 20) respecitvely and the coordinates of A are (p, q). The median to the side BC
has slope – 5, then which can be corrected.
(A) p + q = 47 (B) p + q = 27 (C) p – q = 17 (D) p – q = 13
14. All the points lying on or inside the triangle formed by the points (1, 3), (5, 6) and (–1, 2) satisfy
(A) 3x + 2y ≥ 0 (B) 2x + y + 1 ≥ 0 (C) 2x + 3y – 12 ≥ 0 (D) 2x + 11 ≥ 0

15. A ≡ (4, 2) and B ≡ (2, 4) are two given points and a point P on the line 3x + 2y + 10 = 0 is given then
which of the following is/are true.
 −14 −4   −14 −4 
(A) (PA + PB) is minimum when P  ,  (B) (PA + PB) is maximum when P  5 , 5 
 5 5   
(C) |PA – PB | is maximum when P(– 22, 28) (D) (PA – PB) is minimum when P(– 22, 28).

16. A line passing through P = ( 3, 0 ) and making an angle of 60° with positive direction of x-axis cuts the
parabola y2 = x + 2 at A and B, then :
2 2

RI
(A) PA + PB = (B) |PA – PB| =
3 3
4(2 + 3 ) 1 1 2− 3
(C) (PA) (PB) = (D) + =
3 PA PB 2

A
17. Let u ≡ ax + by + a 3 b = 0, v ≡ bx − ay + b 3 b = 0, where a, b ∈ R be two straight lines, then find the
equations of the bisectors of the angles formed by k1u − k2v = 0 & k1u + k2v = 0 for non zero real k1 & k2

UH
are :
(A) u = 0 (B) k2u + k1v = 0 (C) k2u − k1v = 0 (D) v = 0

18. The sides of a triangle are the straight line x + y = 1, 7y = x and 3 y + x = 0. Then which of the
following is an interior points of triangle ?
JA
(A) circumcentre (B) centroid (C) incentre (D) orthocentre

19. The line ‘1’ passing through the point (1, 1) and the ‘2’ passes through the point (– 1, 1). If the
difference of the slope of lines is 2. Find the locus of the point of intersection of the  1 and 2.
(A) x2 = y (B) y = 2 – x2 (C) y2 = x (D) x = 2 – y2

20. The two lines pairs y2 – 4y + 3 = 0 and x2 + 4xy + 4y2 – 5x – 10y + 4 = 0 enclose a 4 sided convex
LP

polygon, then the correct statement among the following is/are


(A) Area of polygon is 6 (B) Length of its diagonals are 5 & 53
(C) Point of intersection of diagonals is (– 2, 2) (D) Polygon is parallelogram.

21. If the distance between the lines represented 9x2 - 24xy + 16y2 + k(6x - 8y) = 0 is 4, then k may be
A

(A) 3 (B) 10 (C) –10 (D) 7

PART - IV : COMPREHENSION
NK

Comprehenssion # 1 (Q. NO. 1 to 3)


Let ABC be an acute angled triangle and AD, BE and CF are its medians, where E and F are the points
(3, 4) and (1, 2) respectively and centroid of ∆ ABC is G(3, 2), then answer the following questions :
SA

1. The equation of side AB is


(A) 2x + y = 4 (B) x + y – 3 = 0 (C) 4x – 2y = 0 (D) none of these

2. Co-ordinates of D are
(A) (7, –4) (B) (5, 0) (C) (7, 4) (D) (– 3, 0)

3. Height of altitude drawn from point A is (in units)


(A) 4 2 (B) 3 2 (C) 6 2 (D) 2 3

Comprehension # 2 (Q. No. 4 to 6)

Given two straight lines AB and AC whose equations are 3x + 4y = 5 and 4x – 3y = 15 respectively.
Then the possible equation of line BC through (1, 2), such that ∆ABC is isosceles, is L 1 = 0 or L2 = 0,
then answer the following questions
4. If L1 ≡ ax + by + c = 0 & L2 ≡ dx + ey + f = 0 where a, b, c, d, e, f ∈ I, and a, b, d, f > 0 and
HCF(a, b) = HCF(d, f) = 1, then c + f =
(A) 1 (B) 2 (C) 3 (D) 4

5. A straight line through P(2, c + f – 1), inclined at an angle of 60° with positive Y-axis in clockwise
direction. The co-ordinates of one of the points on it at a distance (c + f) units from point P is
(c, f obtained from previous question)
(A) (2 + 2 3 , 5) (B) (3 + 2 3 , 3) (C) (2 + 3 3 , 4) (D) (2 + 3 3 , 3)

6. If (a, b) is the co-ordinates of the point obtained in previous question, then the equation of line which is
at the distance |b – 2a – 1| units from origin and make equal intercept on co-ordinate axes in first
quadrant, is
(A) x + y + 4 6 = 0 (B) x + y + 2 6 = 0 (C) x + y – 4 6 = 0 (D) x + y – 2 6 = 0

RI
Comprehension # 3 (Q.No. 7 to 9)
p1 p2 1
1
If vertices of triangle are P(p1, p2), Q(q1, q2), R(r1, r2), then area of ∆PQR =

A
q1 q2 1 and if P, Q, R
2
r1 r1 1
p1 p2 1

UH
are collinear, then q1 q2 1 = 0.
r1 r1 1
On the basis of above answer the following question.
JA
7. If A(x1, y1), B(x2, y2), C(x3, y3) are the vertices of the triangle then find equation of median through A.
x y 1 x y 1 x y 1 x y 1
(A) x1 y1 1 − x1 y1 1 = 0. (B) x1 y1 1 + x1 y1 1 = 0
x2 y2 1 x3 y3 1 x2 y2 1 x3 y3 1
x y 1 x y 1
LP

(C) x1 y1 1 + x3 y3 1 = 0 (D) None of these


x2 y2 1 x1 y1 1
8. If A(x1, y1), B(x2, y2), C(x3, y3) are the vertices of the triangle then find equation of line through A and
parallel to BC
A

x y 1 x y 1 x y 1 x y 1
(A) x1 y1 1 − x1 y1 1 = 0. (B) x1 y1 1 + x1 y1 1 = 0
x2 y2 1 x3 y3 1 x2 y2 1 x3 y3 1
NK

x y 1 x y 1 x y 1 x y 1
(C) x2 y2 1 + x3 y3 1 = 0 (D) x1 y1 1 = x 3 y3 1
x1 y1 1 x1 y1 1 x2 y2 1 x1 y1 1
9. If A(x1, y1), B(x2, y2), C(x3, y3) are the vertices of the triangle then find the equation of internal angle
bisector through A
SA

x y 1 x y 1 x y 1 x y 1
(A) b x1 y1 1 − c x1 y1 1 = 0 (B) c x1 y1 1 + b x1 y1 1 =0
x2 y2 1 x3 y3 1 x2 y2 1 x3 y3 1
x y 1 x y 1 x y 1 x y 1
(C) b x1 y1 1 + c x1 y1 1 = 0 (D) c x1 y1 1 − b x1 y1 1 = 0
x2 y2 1 x3 y3 1 x2 y2 1 x3 y3 1

Comprehension # 4 (10 & 11)


Origin of coordinate system xy is shifted to (h, k) to make new coordinate system XY. X and Y are
parallel to x and y. New co-ordinates of point P(x, y) are P(X, Y). x, y, X, Y are related as given below.
X=x–h
Y=y–K
Y
y
X P(x, y) ≡ (X, Y)

y Y

X
(h, k)

x
x

RI
10. Co-ordinates of (–7, 9) if origin is shifted to (2, 4) without changing direction of axes, are
(A) (–5,13) (B) (–7,4) (C) (–9,5) (D) (–9,13)

A
11. If co-ordinate axes are so translated such that ordinate of (4, 12) becomes zero while abscissa remains
same. Then new coordinates of point (–8, –2) are

UH
(A) (–8, 14) (B) (–8, 10) (C) (–8, –14) (D) (–8, –10)

JA
LP
A
NK
SA
* Marked Questions may have more than one correct option.
1. Let O(0, 0), P(3, 4), Q(6, 0) be the vertices of the triangle OPQ. The point R inside the triangle OPQ is
such that the triangles OPR, PQR, OQR are of equal area. The co-ordinates of R are
[IIT-JEE - 2007, P-II, (3, – 1), 81]

4   2  4 4 2
(A)  , 3  (B)  3,  (C)  3,  (D)  , 
3   3  3 3 3

RI
2. Lines L1 : y – x = 0 and L2 : 2x + y = 0 intersect the line L3 : y + 2 = 0 at P and Q, respectively. The
bisector of the acute angle between L1 and L2 intersects L3 at R. [IIT-JEE - 2007, P-II, (3, – 1), 81]

A
STATEMENT – 1 : The ratio PR : RQ equals 2 2 : 5.

UH
because
STATEMENT – 2 : In any triangle, bisector of an angle divides the triangle into two similar triangles.
(A) Statement - 1 is True, Statement - 2 is True; Statement - 2 is a correct explanation for
Statement - 1
JA
(B) Statement - 1 is True, Statement - 2 is True; Statement - 2 is NOT a correct explanation for
Statement - 1
(C) Statement - 1 is True, Statement - 2 is False
(D) Statement - 1 is False, Statement - 2 is True
LP

3. Consider three points [IIT-JEE - 2008, P-II, (3, – 1), 81]


P = (– sin (β – α), – cos β), Q = (cos(β – α), sin β) and R = (cos (β – α + θ), sin (β – θ)), where
π
0 < α, β , θ < . Then,
A

4
(A) P lies on the line segment RQ (B) Q lies on the line segment PR
(C) R lies on the line segment QP (D) P, Q, R are non-collinear
NK

4. The locus of the orthocentre of the triangle formed by the lines [IIT-JEE - 2009, Paper-2, (3, –1), 80]
(1 + p) x – py + p (1 + p) = 0, (1 + q) x – qy + q(1 + q) = 0 and y = 0, where p ≠ q, is
(A) a hyperbola (B) a parabola (C) an ellipse (D) a straight line
SA

5. A straight line L through the point (3, –2) is inclined at an angle 60º to the line 3 x + y = 1. If L also

intersects the x-axis, then the equation of L is [IIT-JEE 2011, Paper-1, (3, –1), 80]
(A) y + 3x+2–3 3=0 (B) y – 3x+2+3 3 =0

(C) 3y–x+3+2 3 =0 (D) 3y+x–3+2 3 =0


6. For a > b > c > 0, the distance between (1, 1) and the point of intersection of the lines ax + by + c = 0
and bx + ay + c = 0 is less than 2 2 . Then [JEE (Advanced) 2013, Paper-1, (2, 0)/60]
(A) a + b – c > 0 (B) a – b + c < 0 (C) a – b + c > 0 (D) a + b – c < 0
7. For a point P in the plane, let d1(P) and d2(P) be the distance of the point P from the lines x – y = 0 and
x + y = 0 respectively. The area of the region R consisting of all points P lying in the first quadrant of the
plane and satisfying 2 ≤ d1(P) + d2(P) ≤ 4, is [JEE (Advanced) 2014, Paper-1, (3, 0)/60]

PART - II : JEE (MAIN) / AIEEE PROBLEMS (PREVIOUS YEARS)


1. The lines p(p2 + 1) x – y + q = 0 and (p2 + 1)2 x + (p2 + 1) y + 2q = 0 are perpendicular to a common line
for: [AIEEE - 2009 (4, –1), 144]
(1) exactly one value of p (2) exactly two values of p
(3) more than two values of p (4) no value of p

RI
2. Three distinct points A, B and C are given in the 2-dimensional coordinate plane such that the ratio of
1

A
the distance of any one of them from the point (1, 0) to the distance from the point (–1, 0) is equal to .
3
Then the circumcentre of the triangle ABC is at the point : [AIEEE - 2009 (4, –1), 144]

UH
5  5  5 
(1)  , 0  (2)  , 0  (3)  , 0  (4) 0, 0
4  2  3 

x y
JA
3. The line L given by + = 1 passes through the point (13, 32). The line K is parallel to L and has the
5 b
x y
equation + = 1. Then the distance between L and K is [AIEEE - 2010 (8, –2), 144]
c 3

17 23 23
(1) 17 (2) (3) (4)
15 17 15
LP

4. The line L1 : y – x = 0 and L2 : 2x + y = 0 intersect the line L3 : y + 2 = 0 at P and Q respectively. The


bisector of the acute angle between L1 and L2 intersects L3 at R. [AIEEE - 2011, I(4, –1), 120]
A

Statement-1 : The ratio PR : RQ equals 2 2 : 5


NK

Statement-2 : In any triangle, bisector of an angle divides the triangle into two similar triangles.
(1) Statement-1 is true, Statement-2 is true ; Statement-2 is correct explanation for Statement-1
(2) Statement-1 is true, Statement-2 is true ; Statement-2 is not a correct explanation for Statement-1
(3) Statement-1 is true, Statement-2 is false
SA

(4) Statement-1 is false, Statement-2 is true

5. The lines x + y = | a | and ax – y = 1 intersect each other in the first quadrant. Then the set of all
possible values of a is the interval : [AIEEE - 2011, II(4, –1), 120]
(1) (0, ∞) (2) [1, ∞) (3) (–1, ∞) (4) (–1, 1]

6. If A(2, –3) and B(–2, 1) are two vertices of a triangle and third vertex moves on the line 2x + 3y = 9 ,
then the locus of the centroid of the triangle is : [AIEEE - 2011, II(4, –1), 120]
(1) x – y = 1 (2) 2x + 3y = 1 (3) 2x + 3y = 3 (4) 2x – 3y = 1

7. If the line 2x + y = k passes through the point which divides the line segment joining the points (1, 1)
and (2, 4) in the ratio 3 : 2, then k equals : [AIEEE-2012, (4, –1)/120]
29 11
(1) (2) 5 (3) 6 (4)
5 5

8. A line is drawn through the point (1, 2) to meet the coordinate axes at P and Q such that it forms a
triangle OPQ, where O is the origin. if the area of the triangle OPQ is least, then the slope of the line
PQ is : [AIEEE-2012, (4, –1)/120]
1 1
(1) – (2) – 4 (3) – 2 (4) –
4 2

9. A ray of light along x + 3 y = 3 gets reflected upon reaching x-axis, the equation of the reflected ray

RI
is [AIEEE - 2013, (4, –1),360]

(1) y = x + 3 (2) 3y=x– 3 (3) y = 3x– 3 (4) 3 y = x –1

A
10. The x-coordinate of the incentre of the triangle that has the coordinates of mid points of its sides as

UH
(0, 1) (1, 1) and (1, 0) is : [AIEEE - 2013, (4, –1),360]
(1) 2 + 2 (2) 2 – 2 (3) 1 + 2 (4) 1 – 2
11. Let PS be the median of the triangle with vertices P(2, 2), Q (6, – 1), and R (7, 3). The equation of the
line passing through (1, – 1) and parallel to PS is : [JEE(Main) 2014, (4, – 1), 120]
JA
(1) 4x + 7y + 3 = 0 (2) 2x – 9y – 11 = 0 (3) 4x – 7y – 11 = 0 (4) 2x + 9y + 7 = 0

12. Let a, b, c and d be non-zero numbers. If the point of intersection of the lines 4ax + 2ay + c = 0 and
5bx + 2by + d = 0 lies in the fourth quadrant and is equidistant from the two axes then :
[JEE(Main) 2014, (4, – 1), 120]
LP

(1) 3bc – 2ad = 0 (2) 3bc + 2ad = 0 (3) 2bc – 3ad = 0 (4) 2bc + 3ad = 0

13. The number of points, having both co-ordinates as integers, that lie in the interior of the triangle with
vertices (0, 0), (0, 41) and (41, 0) is [JEE(Main) 2015, (4, – 1), 120]
A

(1) 901 (2) 861 (3) 820 (4) 780


NK

14. Two sides of a rhombus are along the lines, x – y + 1 = 0 and 7x – y – 5 = 0. if its diagonals intersect at
(–1, –2), then which one of the following is a vertex of this rhombus ? [JEE(Main) 2016, (4, – 1), 120]
1 8  10 7
(1) (–3, –8) (2)  3 ,– 3  (3)  – 3 ,– 3  (4) (–3, –9)
SA

15. Let k be an integer such that the triangle with vertices (k, –3k), (5, k) and (–k, 2) has area 28 sq. units.
Then the orthocentre of this triangle is at the point : [JEE(Main) 2017, (4, – 1), 120]

 1  3  3  1
(1)  2, −  (2)  1,  (3)  1, −  (4)  2, 
 2   4  4  2

16. A straight line through a fixed point (2,3) intersects the coordinate axes at distinct points P and Q. If O
is the origin and the rectangle OPRQ is completed, then the locus of R is
[JEE(Main) 2018, (4, – 1), 120]
(1) 3x + 2y = xy (2) 3x + 2y = 6xy (3) 3x + 2y = 6 (4) 2x + 3y = xy
17. Consider the set all lines px + qy + r = 0 such that 3p + 2q + 4r = 0. Which one of the following
statements is true ? [JEE(Main) 2019, Online (09-01-19),P-1 (4, – 1), 120]
(1) The lines are not concurrent
(2) The lines are all parallel

 3 1
(3) The lines are concurrent at the point  , 
4 2
(4) Each the line passes through the origin.

18. Let the equations of two sides of a triangle be 3x – 2y + 6 = 0 and 4x + 5y – 20 = 0 . If the orthocentre

RI
of this triangle is at (1, 1), then the equation of its third side is :
[JEE(Main) 2019, Online (09-01-19),P-2 (4, – 1), 120]
(1) 26x – 122y – 1675 = 0 (2) 26x + 61y + 1675 = 0

A
(3) 122y – 26x – 1675 = 0 (4) 122y + 26x + 1675 = 0

UH
19. Two vertices of a triangle are (0, 2) and (4, 3). If its orthocentre is at the origin, then its third vertex lies
in which quadrant ?
(1) third (2) second (3) first (4) fourth
[JEE(Main) 2019, Online (10-01-19),P-2 (4, – 1), 120]
JA
LP
A
NK
SA
PART - I

Section (A) :

A-1. (ii) (3, 4) (6, 8) A-2. (i) 1:3 internally (ii) 1:2

3
A-3. (i) (ii) 4 : 1 internally (iii) (8, – 8). A-4. (7, 2) or (1, 0)

RI
2

A-5. (i) 15 sq. units (ii) |ab| sq. units

A
Section (B) :

1 4 1 4
B-1. (i) y=– x– , slope = – , y-intercept = –

UH
3 3 3 3

x y 4
(ii) + = 1 , x-intercept = – 4, y-intercept = –
−4 −4 / 3 3
JA
(iii) x cos 240º + y sin 240º = 2, P = 2, α = 240º

B-2. 3 B-3. 8x + 5y + 60 = 0, 2x + 5y + 30 = 0

B-4. Equations are 2x – 3y + 15 = 0, 2x + y – 5 = 0 , 6x – y – 27 = 0

Length of sides are 2 13 , 4 5 , 2 37


LP

B-5. x–y+1=0 B-6. x + y = 5, x – 7y + 3 = 0 B-7. 14 B-8. π/12, 5π/12

B-9. x = 3, y = 4, 9/2 sq. units B-10. x – 5y + 3 = 0, 5x + y – 11 = 0 B-11. ( −6, 9 − 2 3 )


A

B-12. x2 + y2 – 6x – 6y = 0. B-13. 18 B-14. 7x – 4y + 25 = 0, 7x – 4y – 3 = 0, 4x + 7y = 11


NK

Section (C) :

 23 29   13 14  1 + 10  3 4 
C-2. Foot  ,  , Image  ,  , x + 3y – 11 = 0 C-3. C-4.  25 , 25 
 10 10   5 5  3  
SA

C-5. Inside C-6. 3 sq. unit C-7. 2 C-8. x + 7y + 12 = 0, x + 7y – 8 = 0

Section (D) :

 21 8 
D-1. (i) (7, 8) (ii)  2 , 3 (iii) (7, 9) (iv) (27, –21) D-2. 7
 

 1 23 
D-3. 2x + 3y + 8 = 0, 4x – 3y – 7 = 0, 8x + 3y + 9 = 0 , orthocentre  − , − . D-4. (0, 0)
 6 9 

3 2
D-6. + =3 D-7. (3x – 1)2 + 9y2 = a2 + b2 D-9. x2 + y2 = a2 + b2
x y
D-10. y = 2x + 1 or y = –2x + 1

Section (E) :

E-1. acute 2x + y – 3 = 0, obtuse x – 2y + 1 = 0, origin lies in obtuse angle bisector.

6 1
E-2. 9 x − 7 y = 1, 7 x + 9 y = 73 E-3. 5x + 5y − 3 = 0 E-4. , ,–3
5 2

E-5. 0 E-6. (i) 43x – 29y = 71 (ii) x+y+2=0

E-7. 2xy(ma-b) + x(an – c) + y (mc – bn) = 0.

RI
Section (F) :

 3 5
F-2. λ = 2, 3x – y + 2 =0, 4x – 2y + 1 = 0, point of intersection  − , −  ,
 2 2

A
 1
tan–1   , 2x2 + 4xy – 2y2 + 16x – 4y + 7 = 0.
7

UH
F-3. (i) h = 1 (ii) k = 1 F-4. 4alx2 + 4amxy + ny2 = 0; 4al + n = 0

F-5. a2 + b2 = (a')2 + (b')2 ⇒ (a + a')x + (b + b')y + c + c' = 0 ⇒ (a – a') x + (b – b')y = 0


JA
PART - II

Section (A) :

A-1. (C) A-2. (B) A-3. (B) A-4. (A) A-5. (A)
LP

Section (B) :

B-1. (D) B-2. (B) B-3. (C) B-4. (B) B-5. (D) B-6. (B) B-7. (B)

B-8. (B) B-9. (A) B-10. (B) B-11. (B) B-12. (C) B-13. (B)
A

Section (C) :
NK

C-1. (B) C-2. (B) C-3. (A) C-4. (C) C-5. (A) C-6. (C)

Section (D) :

D-1. (A) D-2. (D) D-3. (C) D-4. (B) D-5. (B) D-6. (A) D-7. (A)
SA

D-8. (A)

Section (E) :

E-1. (A) E-2. (C) E-3. (A) E-4. (D) E-5. (D) E-6. (B) E-7. (D)

E-8. (B) E-9. (C)

Section (F) :

F-1. (D) F-2. (A) F-3. (C) F-4. (A)


PART - III

1. (A)→(q, s); (B)→(r, s); (C)→(r, s); (D)→(r, s) 2. (A) → (p); (B) → (q); (C) → (r); (D) → (s)

EXERCISE - 2

PART - I
1. (C) 2. (C) 3. (D) 4. (A) 5. (D) 6. (C) 7. (B)
8. (A) 9. (C) 10. (A) 11. (A) 12. (C) 13. (C) 14. (A)
15. (B) 16. (A) 17. (B) 18. (A) 19. (A)

RI
PART - II
1. 0 2. 2 3. 18 4. 1 5. 5 6. 52 7. 4
8. 4 9. 1 10. 2 11. 1 12. 19 13. 16 14. 30

A
15. 18
PART - III

UH
1. (AD) 2. (BC) 3. (B) 4. (B) 5. (ABCD) 6. (ACD) 7. (BCD)
8. (ABCD) 9. (AD) 10. (AC) 11. (AC) 12. (C) 13. (ABD) 14. (ABD)
15. (AC) 16. (BC) 17. (AD) 18. (BC) 19. (AB) 20. (ABD) 21. (BC)
PART - IV
JA
1. (A) 2. (B) 3. (C) 4. (D) 5. (A) 6. (C) 7. (B)
8. (A) 9. (C) 10. (C) 11. (C)

EXERCISE - 3
LP

PART - I
1. (C) 2. (C) 3. (D) 4. (D) 5. (B) 6. (A) or (C) or Bonus
7. (6)
A

PART - II
1. (1) 2. (1) 3. (3) 4. (3) 5. (2) 6. (2) 7. (3)
8. (3) 9. (2) 10. (2) 11. (4) 12. (1) 13. (4) 14. (2)
NK

15. (4) 16. (1) 17. (3) 18. (1) 19. (2)
SA
 Marked questions are recommended for Revision.
1. The vertices of a triangle OBC are O(0,0) B(–3,–1) and C(–1,–3). Find the equation of line parallel to
1
BC and intersecting the sides OB and OC, whose perpendicular distance from the point (0,0) is .
2

2. A variable line, drawn through the point of intersection of the straight lines
x y x y
+ = 1 and + = 1, meets the coordinate axes in A & B. Show that the locus of the mid point of
a b b a
AB is the curve 2xy(a + b) = ab(x + y).

RI
3. From the vertices A, B, C of a triangle ABC, perpendiculars AD, BE, CF are drawn to any straight line.
Show that the perpendiculars from D, E, F to BC, CA, AB respectively are concurrent.

A
4. A triangle is formed by the lines whose equations are AB : x + y – 5 = 0, BC :
x + 7y – 7 = 0 and CA : 7x + y + 14 = 0. Find the bisector of the interior angle at B and the exterior
angle at C. Determine the nature of the interior angle at A and find the equation of the bisector.

UH
5. The coordinates of the feet of ⊥ from the vertices of a ∆ on the opposite sides are (20, 25), (8, 16) and
(8, 9). Then find the coordinates of a vertices of the ∆

6. Let P is any point inside the triangle ABC of side lengths 6, 5, 5 units and p 1, p2, p3 be the lengths of
JA
perpendiculars drawn from P to the sides of triangle. Find the maximum value of p 1.p2.p3.

7. Let in ∆PAB, A is (0, 0), B is (a, 0) and P is variable such that ∠PBA is equal to three times ∠PAB the,
find the locus of P.

8. Through a fixed point any straight line is drawn meeting two given parallel straight lines in P and Q,
LP

through P and Q straight lines are drawn in fixed directions, meeting in R. Prove that the locus of R is
straight line.

9. Through the origin O a straight line is drawn to cut the lines y = m 1 x + C1 and y = m2 x + C2 at Q and R.
A

respectively. Find the locus of the point P on this variable line, such that OP is the
geometric mean of OQ and OR.
NK

10. The sides of a triangle are Lr ≡ xcos αr + y sin αr – pr = 0 for r = 1, 2, 3.


Show that its orthocentre is given by
L1 cos (α2 – α3) = L2cos (α3 – α1) = L3 cos (α1 – α2).
SA

RP
11. A line passes through a fixed point R intersecting a fixed line at P. A point Q on RP such that is
RQ
constant. Then show that locus of Q is a straight line.

12. A triangle ABC with a = 8, b = 6 and c = 10 slides on the coordinate axes with vertices A and B on the
x−axis and the y−axis respectively. Find the locus of the vertex C.
13. The line L1 ≡ 4x + 3y – 12 = 0 intersects the x and the y-axis at A and B respectively.
A variable line perpendicular to L1 intersects the x and the y-axis at P and Q respectively. Find the locus
of the circumcentre of triangle ABQ.
14. Show that the orthocentre of ∆ formed by the straight lines, ax 2 + 2hxy + by2 = 0 and x + my =1 is a
x' y' a+b
point (x', y') such that = = .
 m am2 − 2hm + b2

15. Show that the lines joining the origin to the other two points of intersection of the curves
ax2+2hxy+by2+2gx = 0 and a′x2 + 2h′xy + b′y2 + 2g′x = 0 will be at right angles to one another if
g(a′ + b′) = g′(a + b)

16. The distance of a point (x1, y1) from each of two straight lines which passes through the origin of

co−ordinates is δ, find the combined equation of these straight lines.

RI
17. If the equation ax2 + 2hxy + by2 + 2gx + 2fy + c = 0, represents a pair of straight lines, prove that the

A
third pair of straight lines (excluding xy = 0) passing through the points where these meet the axes is
4fg
ax2 – 2hxy + by2 +2gx + 2fy + c + · xy = 0.
c

UH
18. A point moves so that the distance between the feet of the perpendiculars from it on the lines
ax2 + 2h xy + by2 = 0 is a constant 2 d. Show that the equation to its locus is,
(x2 + y2) (h2 − ab) = d2 {(a − b)2 + 4 h2}.
JA
19. Show that the pair of lines given by a2 x2 + 2h(a + b) xy + b2 y2 = 0 is equally inclined to the pair given by
ax2 + 2hxy + by2 = 0.
LP

20. All the chords of the curve 3x² − y² − 2x + 4y = 0 which subtend a right angle at the origin are
concurrent. Does this result also hold for the curve, 3x² + 3y² − 2x + 4y = 0 ? If yes, what is the point of
concurrence.
A

21. The straight lines (A2 – 3B2)x2 + 8AB xy + (B2 – 3A2) y2 = 0 form a ∆ with the line Ax + By + C = 0, then
NK

prove that

C2
(i) Area of ∆ is
(
3 A 2 + B2 )
SA

(ii) ∆ is equilateral
(iii) The orthocentre of ∆ does not lie on one of its vertexs
Comprehension (Q. 22 & 23)
If coordinate system xy is being rotated through an angle θ in anti clock wise direction about the origin
as shown in the diagram, Coordinates of P(x, y) has been change to P(X, Y) in new coordinate system
XY, then x, y, X, Y are related as given below.
y

Y P(x,y) ≡ (X, Y)
X
y y – x tan θ
θ

Y
X x tan θ
θ
x
X
X = x sec θ + (y – x tan θ) sin θ and Y = (y – x tan θ) cos θ

RI
x sin θ
2
= x sec θ + y sin θ – = y cos θ – x sin θ
cos θ
X = x cos θ + y sin θ Y = – x sin θ + y cos θ

A
22. If the axes are rotated through 60º in anticlockwise direction about origin. Find co-ordinats of point (2, 6)
in new co-ordinate axes.

UH
23. If axes are rotated through an acute angle in clockwise direction about origin so that equation x 2 + 2xy +
y2 – 2x + 2y = 0 becomes free from xy in its new position, then find equation in new position
24. Find the acute angle between two straight lines passing through the point M( − 6, − 8) and the points in
which the line segment 2x + y + 10 = 0 enclosed between the co−ordinate axes is divided in the ratio 1
JA
: 2 : 2 in the direction from the point of its intersection with the x − axis to the point of intersection with
the y − axis.

25_. Let A lies on 3x – 4y + 1 = 0. B lies on 4x + 3y – 7= 0 and C is (–2, 5). If ABCD is rhombus, then find
locus of D.
LP

26_. Let D is point on line 1 : x + y – 2 = 0 and S(3, 3) is fixed point. 2 is the line perpendicular to DS and
passing through S. If M is another point on line 1 (other than D), then find locus of point of intersection
of  2 and angle bisector of ∠MDS.
A

27_. A variable line cuts the line 2y = x – 2 and 2y = – x + 2 in points A and B respectively. If A lies in first
quadrant, B lies in 4th quadrant and area of ∆AOB is 4, then find locus of
NK

(i) mid point of AB (ii) centroid of ∆OAB

28. An equilateral triangle PQR is formed where P (1, 3) is fixed point and Q is moving point on line x = 3.
Find the locus of R.
SA
1
= 0. 4. 3x + 6y – 16 = 0; 8x + 8y + 7 = 0 ; 12x + 6y – 11 = 0
2

256
5. (5, 10). (50, – 5), (15, 30) 6. p1p2 p3 ≤ 7. 4x3 – 4xy2 – 3ax2 + ay2 = 0.
75

y 3
= or 3x − 4y = 0.

RI
9. (y – m1x) ( y – m2x) = c1c2 12. vertex C lies on the line
x 4

13. 6x – 8y + 7 = 0 16. (y12 − δ2) x2 − 2 x1y1 xy + (x12 − δ2) y2 = 0

A
(1, − 2), yes gk¡ (1/3, − 2/3) (1 + 3 3 , −

UH
20. 22. 3 + 3) 23. x2 + 2y = 0

24. π/4 25. 25((x + 2)2 + (y – 5)2) = (3x – 4y + 1)2


JA
2
 x + y −2
26. (x – 3)2 + (y – 3)2 =   27. (i) (x –1)2 – 4y2 = 9 (ii) (x – 2/3)2 + 4y2 = 4
 2 

28. (x – 2) = ± 3 (y – 3 ± 3)
LP
A
NK
SA
TRIGONOMETRY

JEE (Advanced) Syllabus


Trigonometry : Trigonometric functions, their periodicity and graphs, addition and subtraction formulae,
formulae involving multiple and sub-multiple angles.
General solution of trigonometric equations.

JEE (Main) Syllabus

RI
Trigonometrical identities. Trigonometrical functions.

When writing about transcendental issues, be transcendentally clear....... Descartes, Rene

A
The word 'trigonometry' is derived from the Greek words 'trigon' and ' metron' and it means 'measuring the sides
and angles of a triangle'.
Angle :

UH
Angle is a measure of rotation of a given ray about its initial point. The original ray is called the initial
side and the final position of the ray after rotation is called the terminal side of the angle. The point of
rotation is called the vertex. If the direction of rotation is anticlockwise, the angle is said to be positive
and if the direction of rotation is clockwise, then the angle is negative.
B
Initial Side
Vertex O A
e
JA
l Sid
na
mi Te
Ter rm
ina
lS
Vertex O A ide
Initial Side
B
(i) Positive angle (ii) Negative angle
(anticlockwise measurement) (clockwise measurement)
LP

Systems For Measurement of Angles :


An angle can be measured in the following systems.
One complete rotation is equal to 360 degree = 400 grade = 2 π radian

Relation between radian, degree and grade :


A

To Sexagesimal System Centesimal System (French Circular System


(British system) system) (Radian Measurement)
NK

From

Sexagesimal 400 π
1 degree = grade 1 degree (1°) = radian
System 360 180
(British system)
1
1min(1′)= degree (1°=60′)
SA

60

1
1 sec(1′′) = min (1′ = 60′′)
60

Centesimal 360
1 grade = degree
System (French 400
system)

Circular System 180 200


1 radian = degree 1 radian = grade
(Radian π π
Measurement)
1 degree = 60 min (1°=60′) 1 grade=100 min(1g = 100')

1 min = 60 sec (1′ = 60′′) 1 min = 100 sec(1' =100'')


Note : # The minutes and seconds in the Sexagesimal system are different with the minutes and
seconds respectively in the Centesimal System. Symbols in both systems are also different.
# If no symbol is mentioned while showing measurement of angle, then it is considered to be
measured in radians.
e.g. θ = 15 implies 15 radian

Arc length AB =  = rθ
1 2
Area of circular sector = r θ sq. units
2
Trigonometric Ratios for Acute Angles :

RI
Let a revolving ray OP starts from OA and revolves into the position OP, thus tracing out the angle
AOP.
In the revolving ray take any point P and draw PM perpendicular to the initial ray OA.

A
In the right angle triangle MOP, OP is the hypotenuse, PM is the perpendicular, and OM is the base.
The trigonometrical ratios, or functions, of the angle AOP are defined as follows :

UH
sin(∠AOP) cos(∠AOP) tan(∠AOP) cot(∠AOP) sec(∠AOP) cosec(∠AOP)

Perp MP Base OM Perp MP Base OM Hyp OP Hyp OP


= = = = = =
Hyp OP Hyp OP Base OM Perp MP Base OM Prep MP
JA
It can be noted that the trigonometrical ratios are all real numbers.

Trigonometric ratios for angle θ ∈ R :


We will now extend the definition of trigonometric ratios to any angle in terms of radian
measure and study them as trigonometric functions. (also called circular functions) Consider a unit
LP

circle (radius 1 unit) with centre at origin of the coordinate axes. Let at origin of the coordinate axes. Let
P(a, b) be any point on the circle with angle AOP = x radian, i.e., length of arc AP = x We define
cos x = a and sin x = b Since ∆ OMP is a right triangle, we have OM2 + MP2 = OP2 or a2 + b2 =1 Thus,
for every point on the unit circle, we have a2 + b2 = 1 or cos2x + sin2 x = 1
A
NK
SA

π
Since one complete revolution subtends an angle of 2π radian at the centre of the circle, ∠ AOB = ,
2
3π π
∠ AOC = π and ∠AOD = . All angles which are integral multiples of are called quadrantal angles.
2 2
The coordinates of the points A, B, C and D are, respectively, (1, 0), (0, 1), (–1, 0) and (0, –1).
Therefore, for quadrantal angles, we have
cos 0 = 1 sin 0 = 0,
π π
cos =0 sin =1
2 2
cos π = –1 sin π = 0
3π 3π
cos =0 sin =–1
2 2
cos 2π = 1 sin 2π = 0
Now if we take one complete revolution from the position OP, we again come back to same position
OP. Thus, we also observe that if x increases (or decreases) by any integral multiple of 2π , the values
of sine and cosine functions do not change. Thus, sin (2nπ + x) = sin x , n ∈ Z, cos (2nπ + x) = cos x,
n ∈ Z. Further, sin x = 0, if x = 0, ± π , ± 2π , ± 3π ....., i.e., when x is an integral multiple of π and
π 3π 5π π
cos x = 0, if x = ± , ± ,± , .....i.e., cos x vanishes when x is an odd multiple of . Thus sin x =
2 2 2 2

RI
π
0 implies x = nπ , where n is any integer cos x = 0 implies x = (2n + 1) , where n is any integer.
2
We now define other trigonometric functions in terms of sine and cosine functions :
1

A
cosec x = , x ≠ nπ , where n is any integer.
sin x
1 π
sec x = , x ≠ (2n + 1) , where n is any integer.

UH
cos x 2
sin x π
tan x = , x ≠ (2n + 1) , where n is any integer.
cos x 2
cos x
cot x = , x ≠ n π, where n is any integer.
sin x
JA
We have shown that for all real x, sin2x + cos2x = 1
π
It follows that 1 + tan2x = sec2x (Think ! ) {x ≠ (2n + 1) ; n ∈ Z}
2
1 + cot2x = cosec2x (Think !) {x ≠ nπ ; n ∈ Z}

Sign of The Trigonometric Functions


LP

(i) If θ is in the first quadrant then P(a, b) lies in the first quadrant. Therefore a > 0, b > 0
and hence the values of all the trigonometric functions are positive.

(ii) If θ is in the ΙΙ quadrant then P(a, b) lies in the ΙΙ quadrant. Therefore a < 0, b > 0 and
hence the values sin, cosec are positive and the remaining are negative.
A

(iii) If θ is in the ΙΙΙ quadrant then P(a, b) lies in the ΙΙΙ quadrant. Therefore a < 0, b < 0 and
hence the values of tan, cot are positive and the remaining are negative.
NK

(iv) If θ is in the ΙV quadrant then P(a, b) lies in the IV quadrant. Therefore a > 0, b < 0 and
hence the values of cos, sec are positive and the remaining are negative.

sinθ cosθ tanθ cotθ secθ cosecθ


SA

Ιst Quadrant + + + + + +

ΙΙnd Quadrant + – – – – +

ΙΙΙrd Quadrant – – + + – –

ΙVth Quadrant – + – – + –

Values of trigonometric functions of certain popular angles are shown in the following table :
π π π π
0
6 4 3 2
0 1 1 2 1 3 3 4
sin =0 = = = =1
4 4 2 4 2 4 2 4
3 1 1
cos 1 0
2 2 2
1
tan 0 1 3 N.D.
3

N.D. implies not defined

The values of cosec x, sec x and cot x are the reciprocal of the values of sin x, cosx and tan x,
respectively.

RI
Trigonometric Ratios of allied angles
π 3π
If θ is any angle, then − θ, ± θ, π ± θ, ± θ, 2π ± θ etc. are called allied angles.
2 2

A
–θ π π π–θ π+θ 3π 3π 2π – θ 2π + θ
–θ +θ –θ +θ
2 2 2 2

UH
sin –sin θ cos θ sin θ sin θ –sin θ –cos θ –cos θ –sin θ sinθ

cos cos θ sin θ –cos θ –cos θ –cos θ –sin θ sin θ cos θ cosθ
JA
tan –tan θ cot θ –tan θ –tan θ tan θ cot θ –cot θ –tan θ tanθ

cot –cot θ tan θ –cot θ –cot θ cot θ tan θ –tan θ –cot θ cotθ

sec sec θ cosec θ –secθ –sec θ –sec θ –cosec θ cosec θ sec θ secθ
LP

cosec –cosecθ secθ cosecθ cosecθ –cosecθ –secθ –secθ –cosecθ cosecθ

Think, and fill up the blank blocks in following table.


A

π π π π 2π 5π 7π 4π 3π 5π 11π
0 π 2π
6 4 3 2 3 6 6 3 2 3 6
NK

1 1 3
sin 0 1
2 2 2
3 1 1
cos 1 0
2 2 2
1
tan 0 1 3 N.D.
SA

3
Trigonometric functions :
Domain Range Graph
y
y = sinx R [–1, 1]
1

0
3π π π π 3π
x
–2π –π 2π
2 2 2 2
–1

y
y = cosx R [–1, 1]
1

0
π
x
–2π 3π –π π π 3π 2π
2 2 2 2
–1

RI
y
y = tanx R– R
 π 
(2n + 1) , n ∈ Ι  π π
 2 

A
–π 2 0 2 π
x
– 32π 3π
2

UH
y = cotx R – {nπ , n ∈ Ι } R y

–π π π π 3π 2π x
0
JA
2 2 2

y = secx R– (− ∞, − 1] ∪ [1,
 π  ∞)
(2n + 1) , n ∈ Ι 
LP

 2 
A
NK

y=cosecx R – {nπ, n ∈ Ι} (− ∞, − 1] ∪ [1,


∞)
SA
Trigonometric functions of sum or difference of two angles:
(a) sin (A ± B) = sinA cosB ± cosA sinB
(b) cos (A ± B) = cosA cosB  sinA sinB
(c) sin²A − sin²B = cos²B − cos²A = sin (A+B). sin (A− B)
(d) cos²A − sin²B = cos²B − sin²A = cos (A+B). cos (A − B)
tan A ± tan B cot A cot B  1
(e) tan (A ± B) = (f) cot (A ± B) =
1  tan A tan B cot B ± cot A
(g) sin (A + B + C) = sin A cos B cos C + sin B cos A cos C + sin C cos A cos B – sin A sin B sin C
(h) cos (A + B + C) = cos A cos B cos C – cos A sin B sin C – sin A cos B sin C – sin A sin B cos C
tan A + tan B + tanC − tan A tan B tan C
(i) tan (A + B + C) = .
1 − tan A tan B − tan B tan C − tan C tan A
S1 − S3 + S5 − ......
(j) tan (θ1 + θ2 + θ3 + ....... + θn) =

RI
1 − S2 + S4 − .......
where Si denotes sum of product of tangent of angles taken i at a time
Example # 1 : Prove that

A
(i) sin (45º + A) cos (45º – B) + cos (45º + A) sin (45º – B) = cos (A – B)
π   3π 
(ii) tan  + θ  tan  + θ  = –1
4   4 

UH
Solution : (i) Clearly sin (45º + A) cos (45º – B) + cos (45º + A) sin (45º – B)
= sin (45º + A + 45º – B) = sin (90º + A – B) = cos (A – B)
π   3π  1 + tan θ −1 + tan θ
(ii) tan  + θ  × tan  + θ = × =–1
4   4  1 − tan θ 1 + tan θ
JA
Self practice problems :
2 2 4
(1) If cos α =, sin β = , then find cos (α + β) (2) Find the value of cos 375º
3 5
A A
(3) Prove that 1 + tan A tan = tan A cot – 1 = sec A
2 2
±6 2 ± 4 3 +1
LP

Answers : (1) (2)


15 2 2
Transformation formulae :
C+D C−D
(i) sin(A+B) + sin(A − B) = 2 sinA cosB (a) sinC + sinD = 2 sin cos
2 2
A

C+D C−D
(ii) sin(A+B) − sin(A − B) = 2 cosA sinB (b) sinC − sinD = 2 cos sin
2 2
C+D C−D
(iii) cos(A+B) + cos(A − B) = 2 cosA cosB (c) cosC + cosD= 2 cos cos
NK

2 2
C+D D −C
(iv) cos(A − B) − cos(A+B) = 2 sinA sinB (d) cosC − cosD = 2 sin sin
2 2
 15A  A
Example # 2 : Prove that cos7A + cos8A = 2cos   cos  
 2  2
SA

 15A  A
Solution : L.H.S. cos7A + cos8A = 2cos   cos  2 
 2   
C+D C−D
[ cos C+ cos D = 2 cos cos ]
2 2

Example # 3 : Find the value of 2sin3θ sinθ – cos2θ + cos4θ


Solution : 2sin3θ sinθ – cos2θ + cos4θ = 2 sin 3θ sin θ – 2sin3θ sinθ = 0
Example # 4 : Prove that
sin8θ cos θ − sin 6θ cos3θ
(i) = tan 2θ
cos 2θ cos θ − sin3θ sin 4θ
(ii) If A + B = 45º then prove that (1 + tanA) (1 + tanB) = 2
2 sin8θ cos θ − 2 sin 6θ cos3θ sin9θ + sin7θ − sin9θ − sin3θ 2 sin 2θ cos5θ
Solution : (i) = = = tan 2θ
2cos 2θ cos θ − 2 sin3θ sin 4θ cos3θ + cos θ − cos θ + cos7θ 2cos5θ cos 2θ
(ii) A + B = 45º
tan A + tanB
tan (A + B) = 1 ⇒ =1
1– tan A tanB
tanA + tanB = 1 – tanA tanB ⇒ tanA + tanB + tanA tanB + 1 = 2
(1 + tanA) (1 + tanB) = 2

Self practice problems

(4) Prove that


13x 3x
(i) cos 8x – cos 5x = – 2 sin sin
2 2
cos A − cos3A
(ii) = – tan2A
sin A − sin3A

RI
sin 2A + sin 4A + sin 6A + sin8A
(iii) = tan 5A
cos 2A + cos 4A + cos 6A + cos8A
sin A + 2 sin3A + sin5A sin3A
(iv) =

A
sin3A + 2 sin5A + sin7A sin5A
sin A − sin5A + sin9A − sin13A
(v) = cot 4A
cos A − cos5A − cos9A + cos13A

UH
θ 7θ 3θ 11θ
(5) Prove that sin sin + sin sin = sin 2θ sin 5θ
2 2 2 2
(6) Prove that cos A sin (B – C) + cos B sin (C – A) + cos C sin (A – B) = 0
π 9π 3π 5π
JA
(7) Prove that 2 cos cos + cos + cos =0
13 13 13 13

Multiple and sub-multiple angles :


θ θ
(a) sin 2A = 2 sinA cosA Note : sin θ = 2 sin cos etc.
2 2
(b) cos 2A = cos²A − sin²A = 2cos²A − 1 = 1 − 2 sin²A
LP

θ θ
Note : 2 cos² = 1 + cos θ, 2 sin² = 1 − cos θ.
2 2
θ
2 tan A 2 tan
(c) tan 2A = Note : tan θ = 2
2 θ
1 − tan2 A 1 − tan
A

2 tan A 1 − tan2 A
(d) sin 2A = , cos 2A =
1 + tan2 A 1 + tan2 A
NK

(e) sin 3A = 3 sinA − 4 sin3A


(f) cos 3A = 4 cos3A − 3 cosA
3 tan A − tan3 A
(g) tan 3A =
1 − 3 tan2 A
SA

Example # 5 : Prove that


sin 2A
(i) = tan A (ii) tan A + cot A = 2 cosec 2 A
1 + cos 2A
1 − cos A + cosB − cos(A + B) A B
(iii) = tan cot
1 + cos A − cosB − cos(A + B) 2 2
sin 2A 2 sin A cos A
Solution : (i) L.H.S. = = tan A
1 + cos 2A 2cos2 A
1 + tan2 A  1 + tan2 A  2
(ii) L.H.S. tan A + cot A = =2   = = 2 cosec 2 A
tan A  2 tan A  sin 2A
A A A 
2 sin2
+ 2 sin sin  + B 
1 − cos A + cosB − cos(A + B) 2 2 2 
(iii) L.H.S. =
1 + cos A − cosB − cos(A + B) A A  A 
2cos2 − 2cos cos  + B 
2 2 2 
 A A    A +B B 
 sin 2 + sin  2 + B    2sin 2 cos  2  
= tan
A     = tan A     = tan A cot B
2  A A  2  A +B B  2 2
 cos 2 − cos  2 + B    2sin 2 sin  2  
     

Self practice problems


sin 4θ + sin 2θ 2 tan θ
(8) Prove that =
1 + cos 4θ + cos 2θ 1 − tan2 θ

RI
π 3π 5π 7π 1
(9) Prove that sin sin sin sin =
18 18 18 18 16

A
(10) Prove that tan 3A tan 2A tan A = tan 3A – tan 2A – tan A
 A
(11) Prove that tan  45º +  = sec A + tan A
 2

UH
Important trigonometric ratios of standard angles :
(a) sin n π = 0 ; cos n π = (−1)n ; tan n π = 0, where n ∈ Ι
π 3 −1 5π
(b) sin 15° or sin = = cos 75° or cos ;
12 12
2 2
JA
π 3 +1 5π
cos 15° or cos = = sin 75° or sin ;
12 2 2 12
3 −1 3 +1
tan 15° = = 2 − 3 = cot 75° ; tan 75° = = 2 + 3 = cot 15°
3 +1 3 −1
LP

π 5 −1
(c) sin or sin 18° = = cos 72°
10 4
π 5 +1
cos 36° or cos = = sin 54°
5 4
A

Conditional identities:
If A + B + C = π then :
NK

(i) sin2A + sin2B + sin2C = 4 sinA sinB sinC


A B C
(ii) sinA + sinB + sinC = 4 cos cos cos
2 2 2
(iii) cos 2 A + cos 2 B + cos 2 C = − 1 − 4 cos A cos B cos C
A B C
(iv) cos A + cos B + cos C = 1 + 4 sin sin sin
SA

2 2 2
(v) tanA + tanB + tanC = tanA tanB tanC
A B B C C A
(vi) tan tan + tan tan + tan tan =1
2 2 2 2 2 2
A B C A B C
(vii) cot + cot + cot = cot . cot . cot
2 2 2 2 2 2
(viii) cot A cot B + cot B cot C + cot C cot A = 1

Example # 6 : If A + B + C = 90°, Prove that, tan A tan B + tan B tan C + tan C tan A = 1
Solution : A + B = 90º – C
tan A + tanB
= cot C
1– tan A tanB
tan A tan B + tan B tan C + tan C tan A = 1
2x 2y 2z 2x 2y 2z
Example # 7 : If x + y + z = xyz, Prove that + + = . . .
1− x 1− y
2 2
1− z 2
1− x 2
1− y 2
1 − z2
Solution : Put x = tanA, y = tanB and z = tanC,
so that we have
tanA + tanB + tanC = tanA tanB tanC ⇒ A + B + C = nπ, where n ∈ Ι
Hence L.H.S.
2x 2y 2z 2 tan A 2 tanB 2 tanC
∴ + + = + + .
1 − x2 1 − y2 1 − z2 1 − tan2 A 1 − tan2 B 1 − tan2 C
= tan2A + tan2B + tan2C [ A + B + C = nπ ]
2x 2y 2z
= tan2A tan2B tan2C = . .
1 − x2 1− y 2
1 − z2
Self practice problem

RI
(12) If A + B + C = 180°, prove that
B−C C−A A −B
(i) sin(B + 2C) + sin(C + 2A) + sin(A + 2B) = 4sin sin sin
2 2 2

A
sin 2A + sin 2B + sin 2C A B C
(ii) = 8 sin sin sin .
sin A + sinB + sinC 2 2 2
(13) If A + B + C = 2S, prove that
(i) sin(S – A) sin(S – B) + sinS sin (S – C) = sinA sinB.

UH
A B C
(ii) sin(S – A) + sin (S – B) + sin(S – C) – sin S = 4sin sin sin .
2 2 2
Sine and Cosine series:

sin 2  n − 1 
(i) sin α + sin (α + β) + sin (α + 2β ) +...... + sin {α + (n − 1) β} = sin  α + β
JA
β  2 
2
sin

sin 2  n − 1 
(ii) cos α + cos (α + β) + cos (α + 2β ) +.... + cos {α + (n − 1) β} = cos  α + β
β  2 
sin 2
where : β ≠ 2mπ, m ∈ Ι
sin2 nθ
LP

Example # 8 : (i) Prove that sinθ + sin3θ + sin5θ + .... + sin(2n–1)θ =


sin θ
(ii) Find the average of sin2º, sin4º, sin6º , ...... , sin180º
π 3π 5π 7π 9π 1
(iii) Prove that cos + cos + cos + cos + cos =
11 11 11 11 11 2
A

 2θ   θ + (2n – 1)θ 
sinn   sin  
 = sin nθ
2
Solution : (i) sinθ + sin3θ + sin5θ + .... + sin(2n–1)θ =  2   2
 2θ  sin θ
NK

sin  
 2 
sin 2º + sin 4º +.... + sin180º sin90º (sin91º ) cos1º cot1º
(ii) = = = =
90 90 sin1º 90 sin1º 90
10π 5π 10π
cos sin sin
π 3π 5π 7π 9π 22 11 = 11 = 1
SA

(iii) cos + cos + cos + cos + cos =


11 11 11 11 11 π π 2
sin 2 sin
11 11
Self practice problem
Find sum of the following series :
π 3π 5π
(14) cos + cos + cos + ...... up to n terms.
2n + 1 2n + 1 2n + 1

(15) sin2α + sin3α + sin4α + ..... + sin nα, where (n + 2)α = 2π


1
Answers : (14) – (15) 0.
2

Product series of cosine angles


sin2n θ
cos θ . cos 2θ . cos22θ . cos23θ ...... cos2n–1θ =
2n sin θ

Range of trigonometric expression:

E = a sin θ + b cos θ
 a b 
⇒ E = a2 + b2  sin θ + cos θ 
 a2 + b2 a2 + b2 
b a
Let = sin α & = cos α
a2 + b2 a2 + b2
b
⇒ E = a2 + b2 sin (θ + α), where tan α =

RI
a
Hence for any real value of θ,
− a2 + b2 ≤ E ≤ a2 + b2

A
Example # 9 : (i) If α+ β = 90º then find the maximum value of sinα sinβ
(ii) Find maximum and minimum value of 1 + 2sinx + 3cos 2x

UH
1
Solution : (i) sinα sin(90º – α) = sinαcosα = × sin2α
2
1
maximum value =
2
2
 2sin x   1  13
JA
(ii) 1 + 2sinx + 3cos2x = – 3sin2x + 2sinx + 4=– 3  sin2 x −  +4=–3  sin x − 3  + 3
 3   
2 2
 1 16 16  1
Now 0 ≤  sin x −  ≤ ⇒ – ≤ – 3  sin x −  ≤ 0
 3 9 9  3
2
 1 13 13
– 1 ≤ – 3  sin x −  + ≤
LP

 3 3 3

Self practice problems


(16) Find maximum and minimum values of following
A

(i) 3 + (sinx – 2)2


(ii) 9cos2x + 48sinx cosx – 5sin2x – 2
 π  π
(iii) 2 sin  θ +  + 3 cos  θ − 
   
NK

6 6
Answers : (i) max = 12, min = 4. (ii) max = 25, min = –25
(iii) max = 13 , min = – 13

Trigonometric Equation :
SA

An equation involving one or more trigonometric ratios of an unknown angle is called a trigonometric
equation.

Solution of Trigonometric Equation :


A solution of trigonometric equation is the value of the unknown angle that satisfies the equation.
1 π 3π 9π 11π
e.g. if sinθ = ⇒ θ= , , , , ...........
2 4 4 4 4
Thus, the trigonometric equation may have infinite number of solutions (because of their periodic
nature) and can be classified as :
(i) Principal solution (ii) General solution.
Principal solutions :
The solutions of a trigonometric equation which lie in the interval [0, 2π) are called Principal solutions.
1
e.g. Find the Principal solutions of the equation sinx = .
2
Solution :
1
 sinx =
2
 there exists two values
π 5π 1
i.e. and which lie in [0, 2π) and whose sine is
6 6 2
1 π 5π
∴ Principal solutions of the equation sinx = are ,

RI
2 6 6
General Solution :
The expression involving an integer 'n' which gives all solutions of a trigonometric equation is called
General solution. General solution of some standard trigonometric equations are given below.

A
General Solution of Some Standard Trigonometric Equations :
 π π
If sin θ = sin α ⇒ θ = n π + (−1)n α where α ∈  − ,  , n ∈ Ι.

UH
(i)
 2 2
(ii) If cos θ = cos α ⇒ θ = 2nπ ± α where α ∈ [0, π], n ∈ Ι.
 π π
(iii) If tan θ = tan α ⇒ θ = nπ + α where α ∈  − ,  , n ∈ Ι.
 2 2
JA
(iv) If sin² θ = sin² α ⇒ θ = n π ± α, n ∈ Ι.
(v) If cos² θ = cos² α ⇒ θ = n π ± α, n ∈ Ι.
(vi) If tan² θ = tan² α ⇒ θ = n π ± α, n ∈ Ι. [Note: α is called the principal angle ]

Some Important deductions :


LP

(i) sinθ = 0 ⇒ θ = nπ, n∈Ι


π
(ii) sinθ = 1 ⇒ θ = (4n + 1) ,n ∈ Ι
2
π
(iii) sinθ = – 1 ⇒ θ = (4n – 1) , n ∈ Ι
A

2
π
(iv) cosθ = 0 ⇒ θ = (2n + 1) , n ∈ Ι
2
NK

(v) cosθ = 1 ⇒ θ = 2nπ,n ∈ Ι

(vi) cosθ = – 1 ⇒ θ = (2n + 1)π, n∈Ι


(vii) tanθ = 0 ⇒ θ = nπ, n∈Ι
SA

1
Example # 10: Solve cos θ =
2
1 π π
Solution :  cos θ = ⇒ cosθ = cos ∴ θ = 2nπ ± ,n∈Ι
2 3 3

2
Example # 11 : Solve : sec 2θ = –
3
2 3
Solution :  sec 2θ = – ⇒ cos2θ = –
3 2
5π 5π 5π
⇒ cos2θ = cos ⇒ 2θ = 2nπ ± ,n∈Ι ⇒ θ = nπ ± ,n∈Ι
6 6 12
3
Example # 12 : Solve tanθ =
4
3
Solution :  tanθ = ............(i)
4
3
Let = tanα ⇒ tanθ = tanα
4
3
⇒ θ = nπ + α, where α = tan–1   , n ∈ Ι
4
Self Practice Problems :
3
(17) Solve cotθ = – 1 (18) Solve cos4θ = –
2
π nπ π
Answers : (17) θ = nπ – ,n∈Ι (18) ± ,n∈Ι
4 2 24

Example # 13 : Solve tan2θ = 1

RI
Solution :  tan2θ = 1 ⇒ tan2θ = (1)2
π π
⇒ tan2θ = tan2 ⇒ θ = nπ ± , n ∈ Ι
4 4

A
Example # 14 : Solve 4 sec2θ = 5 + tan2θ
Solution :  4 sec2θ = 5 + tan2θ .............(i)
π

UH
For equation (i) to be defined θ ≠ (2n + 1) , n ∈ Ι
2
 equation (i) can be written as:

4(1 + tan2θ) = 5 + tan2θ


3tan2θ = 1
JA
tan2θ = tan2π/6
π
θ = nπ ± , n ∈ Ι
6
Self Practice Problems :
tan3x − tan 2x
(19) Solve =1 (20) Solve 2 cos2x + sin22x = 2
LP

1 + tan3x tan 2x
π
Answers : (19) no Solution (20) nπ , n ∈ Ι or nπ ± ,n∈Ι
4
Types of Trigonometric Equations :
A

Type -1

Trigonometric equations which can be solved by use of factorization.


NK

x x
sin3 – cos3
2 2 cos x
Example # 15 : =
2 + sin x 3

x x  x x  x x x x
sin3 – cos3  sin – cos  sin2 + cos2 + sin cos 
2 = cos x ⇒  2 2  2 2 2 2  cos x
SA

Solution : 2 =
2 + sin x 3 2 + sin x 3
 x x
 sin 2 – cos 2  ( 2 + sin x ) cos x
   x x  x x
= ⇒ 3  sin – cos  – 2  cos2 – sin2  = 0
2 ( 2 + sin x ) 3  2 2  2 2
 x x  x x x x x
 sin 2 – cos 2   3 + 2 sin 2 + 2cos 2  = 0 ⇒ sin 2 = cos 2 ⇒ tan 2 = 1
  
x π π
= nπ + ,n ∈ Ι ⇒ x = 2nπ + ,n ∈ Ι
2 4 2

Self Practice Problems :


x
(21) Solve cos3x + cos2x – 4cos2 =0 (22) Solve tan2θ + 3secθ + 3 = 0
2
Answers : (21) (2n + 1)π, n ∈ Ι

(22) 2nπ ± , n ∈ Ι or (2n + 1)π , n ∈ Ι
3
Type - 2

Trigonometric equations which can be solved by reducing them in quadratic equations.


cos x 1
Example # 16 : Solve sin2x – =
4 4
cos x 1
Solution : sin2x – =
4 4
4(1 – cos2x) – cosx = 1

RI
4cos2x + cosx – 3 = 0
(cosx + 1) (4cosx –3) = 0
3
cosx = – 1 , cosx =
4

A
3
x = (2n+1)π , x = (2mπ±α) where α = cos –1 , m, n ∈ Ι
4

UH
Self Practice Problems :
(23) Solve 4sin2θ + 2sinθ ( )
3 −1 – 3 = 0
(24) Solve 4cosθ – 3secθ = tanθ
π  −π 
Answers : (23) nπ + (–1)n , n ∈ Ι or nπ + (–1)n  , n ∈Ι
JA
6  3 
 −1 − 17 
(24) nπ + (– 1)n α where α = sin–1   , n ∈ Ι
 8
 
 −1 + 17 
or nπ + (–1)n β where β = sin–1   , n ∈ Ι
 8
 
LP

Type - 3
Trigonometric equations which can be solved by transforming a sum or difference of trigonometric
ratios into their product.

Example # 17 : Solve cosx + cos3x – 2cos2x = 0


A

Solution : cosx + cos3x – 2cos2x = 0


2cos2x cosx – 2cos2x = 0
2cos2x (cosx–1) = 0
NK

cos2x = 0, cosx = 1
π
x = (2n + 1), x = 2mπ , m, n ∈ Ι
2

Self Practice Problems :


SA

(25) Solve sin7θ = sin3θ + sinθ (26) Solve 1 + cos3x = 2cos 2x


sin 6x
(27) Solve 8cosx cos 2x cos4x =
sin x
nπ nπ π
Answers : (25) ,n∈Ι or ± ,n∈Ι
3 2 12
π nπ π
(26) nπ ± , n ∈ Ι or 2nπ, n ∈ Ι (27) + ,n∈Ι
6 7 14

Type - 4

Trigonometric equations which can be solved by transforming a product of trigonometric ratios into their
sum or difference.
Example # 18 : Solve sec4θ – sec2θ = 2
1 1
Solution : – =2
cos 4θ cos2θ
cos2θ – cos4θ = 2 cos4θ cos2θ
cos2θ – cos4θ = cos6θ + cos2θ
cos6θ + cos4θ = 0
2cos5θ cosθ = 0
cos5θ = 0 or cosθ = 0
π π
5θ = (2n + 1) θ = (2m + 1) m, n ∈ Ι
2 2
Type - 5
Trigonometric Equations of the form a sinx + b cosx = c, where a, b, c ∈ R, can be solved by dividing
a2 + b2 .

RI
both sides of the equation by

Example # 19 : Solve sinx + 2cosx = 5


Solution :  sinx + 2cosx = 5 ..........(i)

A
Here a = 1, b = 2.
∴ divide both sides of equation (i) by 5 , we get
1 1
⇒ ⇒ cos (x– α)= 1

UH
sinx . + 2cosx. =1 sinx.sinα + cosx.cosα = 1
5 5
⇒ x – α = 2nπ, n ∈ Ι ⇒ x = 2nπ + α, n ∈ Ι
 1
∴ Solution of given equation is 2nπ + α, n ∈ Ι where α = tan–1  
2
JA
Note : Trigonometric equation of the form a sinx + b cosx = c can also be solved by changing sinx and cosx
into their corresponding tangent of half the angle.

Example # 20 : Solve 3cosx + 4sinx = 5


Solution :  3cosx + 4sinx = 5 .........(i)
2 x x
LP

1 − tan 2 tan
 cosx = 2 & sinx = 2
x x
1 + tan2 1 + tan2
2 2
∴ equation (i) becomes
 2 x   x 
A

 1 − tan 2   2 tan 2 
⇒ 3   +4  =5 ........(ii)
 1 + tan2 x   1 + tan2 x 
 2  2
NK

x
Let tan =t
2
 1 − t2   2t 
∴ equation (ii) becomes 3 
 1 + t 2 
+4   =5
   1 + t2 
SA

⇒ 4t2 – 4t + 1 = 0 ⇒ (2t – 1)2 = 0


1 x
⇒ t=  t = tan
2 2
x 1 x 1
⇒ tan = ⇒ tan = tanα, where tanα =
2 2 2 2
x  1
⇒ = nπ + α ⇒ x = 2nπ + 2α where α = tan–1   , n ∈ Ι
2 2
Self Practice Problems :
(28) Solve 2 2 cosx + sinx = 3
x
(29) Solve sinx + tan =0
2
 1 
Answers : (28) 2nπ + α, n ∈ Ι where α = tan–1  
2 2 
(29) x = 2nπ, n ∈ Ι
Type - 6
Trigonometric equations of the form P(sinx ± cosx, sinx cosx) = 0, where p(y, z) is a polynomial, can be
solved by using the substitution sinx ± cosx = t.

Example # 21 : Solve sin2x + 3sinx = 1 + 3 cosx


Solution : sin2x + 3sinx = 1 + 3 cosx
sin2x + 3(sinx – cosx) = 1 ......... (i)
Let sinx – cosx = t
⇒ sin2x + cos2x – 2 sinx.cosx = t2 ⇒ sin2x = 1– t2
Now put sinx – cosx = t and sin2x = 1– t2 in (i)

RI
1 – t2 + 3t = 1
t2 – 3t = 0
t =0 or t=3 (not possible)
sinx – cosx = 0

A
π
tanx = 1 ⇒ x = nπ + , n∈Ι
4
Self Practice Problems:

UH
(30) Solve 1– sin2x + 2sinx – 2cosx = 0 (31) Solve 2cosx + 2sinx + sin3x – cos3x = 0

(32) Solve (1 – sin2x) (cosx – sinx) = 1 – 2sin2x.


π π nπ π
Answers : (30) nπ + , n ∈ Ι (31) nπ – or + (–1)n ,n∈Ι
4 4 2 12
JA
π π
(32) 2nπ + , n ∈ Ι or 2nπ, n ∈ Ι or nπ + , n ∈ Ι
2 4
Type - 7
Trigonometric equations which can be solved by the use of boundness of the trigonometric ratios
sinx and cosx.

Example # 22 : Solve sin2x + cos4x = 2


LP

Solution : sin2x + cos4x = 2

Now equation will be true if sin2x = 1 and cos4x = 1


π
⇒ 2x = (4n + 1) , n ∈ Ι and 4x = 2mπ, m ∈ Ι
A

2
π mπ π mπ 4n + 1
⇒ x = (4n + 1) , n ∈ Ι and x= , m ∈ Ι ⇒ (4n + 1) = ⇒m=
4 2 4 2 2
Which is not possible for m, n ∈ Ι
NK

Self Practice Problems :


(33) Solve cos50x – sin50x = 1
(34) Solve 12 sin x + 5cosx = 2y2 – 8y + 21 for x & y
SA

 5 
Answers : (33) nπ, n ∈ Ι (34) x = 2nπ + α where α = cos –1   , n ∈ Ι y = 2
 13 
IMPORTANT POINTS :
1. Many trigonometrical equations can be solved by different methods. The form of solution obtained in
different methods may be different. From these different forms of solutions, the students should not
think that the answer obtained by one method are wrong and those obtained by another method are
correct. The solutions obtained by different methods may be shown to be equivalent by some
supplementary transformations.

To test the equivalence of two solutions obtained from two methods, the simplest way is to put values
of
n = .......–2, –1, 0, 1, 2, 3....... etc. and then to find the angles in [0, 2π]. If all the angles in both solutions
are same, the solutions are equivalent.
2. While manipulating the trigonometrical equation, avoid the danger of losing roots. Generally, some
roots are lost by cancelling a common factor from the two sides of an equation. For example, suppose
1

RI
we have the equation tanx = 2 sinx. Here by dividing both sides by sinx, we get cosx = . This is not
2
equivalent to the original equation. Here the roots obtained by sinx = 0, are lost. Thus in place of
dividing an equation by a common factor, the students are advised to take this factor out as a common
factor from all terms of the equation.

A
3. While equating one of the factors to zero, take care of the other factor that it should not become infinite.
For example, if we have the equation sinx = 0, which can be written as cos x tan x = 0. Here we cannot
put

UH
cosx = 0, since for cos x = 0, tanx = sinx/ cosx is infinite.
4. Avoid squaring : When we square both sides of an equation, some extraneous roots appear. Hence it is
necessary to check all the solutions found by substituting them in the given equation and omit the
solutions not satisfying the given equation.
For example : Consider the equation,
JA
sin θ + cos θ = 1 .....(1)
Squaring we get
1 + sin 2θ = 1 or sin 2θ = 0 .....(2)
i.e. 2θ = nπ or θ = nπ/2,
π 3π
This gives θ = 0, , π, , ......
2 2

LP

Verification shows that π and do not satisfy the equation as sin π + cosπ = –1, ≠ 1
2
3π 3π
and sin + cos = – 1, ≠ 1.
2 2
The reason for this is simple.
A

The equation (2) is not equivalent to (1) and (2) contains two equations : sin θ + cos θ = 1
and sinθ + cos θ = – 1. Therefore we get extra solutions.
Thus if squaring is must, verify each of the solution.
NK

5. Some necessary restrictions :


If the equation involves tanx, secx, take cosx ≠ 0. If cot x or cosec x appear, take sinx ≠ 0.
If log appear in the equation, i.e. log [f(θ)] appear in the equation, use f(θ) > 0 and base of log > 0, ≠ 1.
Also note that [f(θ)] is always positive, for example sin2 θ = |sin θ|, not ± sin θ.
6. Verification : Student are advice to check whether all the roots obtained by them satisfy the equation
SA

and lie in the domain of the variable of the given equation.

Trigonometric Inequalities :
To solve a trigonometric inequality, transform it into many basic trigonometric inequalities. The transformation
process proceeds exactly the same as in solving trigonometric equations. The common period of a
trigonometric inequality is the least common multiple of all periods of the trigonometric functions presented in
the inequality. For example : the trigonometric inequality sinx + sin2x + cos x/2 < 1 has 4π as common period.
Unless specified, the solution set of a trigonometric inequality must be solved, at least, within one whole
common period.
Example : Find the solution set of inequality sinx > 1/2.
Solution : When sinx = 1/2, the two values of x between 0 and 2π are π/6 and 5π/6.
y

0 π
3π x
–2π 3π –π −
π π π 5π 2π
− 2 6 2
2 6 2
–1

y = sinx

From, the graph of y = sinx, it is obvious that, between 0 and 2π, sinx > 1/2 ⇒ π/6 < x < 5π/6.

RI
Hence sinx >1/2 ⇒ 2nπ + π/6 < x < 2nπ + 5π/6, n ∈ Ι .

The required solution set is  (2nπ + π/6, 2nπ + 5π/6)

A
n∈I

Self practice problems

UH
(35) Solve the following inequations

(i) (sinx – 2) (2sinx–1) < 0 (ii) sinx + 3 cosx ≥ 1

π 5π  π π
x∈   6 + 2nπ, 6 + 2nπ 
Ans. (i)
JA (ii) x∈  [– +2nπ, 2nπ+ ]
n∈Ι  n∈I 6 2

Heights and distances :

Angle of elevation and depression :


LP

Let OX be a horizontal line and P be a point which is above point O. If an observer (eye of observer) is
at point O and an object is lying at point P then ∠XOP is called angle of elevation as shown in figure. If
an observer (eye of observer) is at point P and object is at point O then ∠QPO is called angle of
A

depression.
Q P
NK

O X
SA
 Marked questions are recommended for Revision.

PART - I : SUBJECTIVE QUESTIONS

Section (A) : Measurement of Angles & Allied angles

A-1. Find the radian measures corresponding to the following degree measures
(i) 15° (ii) 240° (iii) 530°

RI
A-2. Find the degree measures corresponding to the following radian measures
3π 5π 7π
(i) (ii) – 4π (iii) (iv)
4 3 6

A
A-3. Prove that :
π π π 1 π 7π π
(i) sin2 + cos2 – tan2 =– (ii) 2 sin2 + cosec cos2 =0
6 3 4 2 6 6 3

UH
π 2 π π 29 π 5π π
(iii) 3 cos2 + sec + 5 tan2 = (iv) cot2 + cosec + 3 tan2 =6
4 3 3 2 6 6 6

A-4. Find the value of :


(i) cos 210° (ii) sin 225° (iii) tan 330° (iv) cot (– 315°)
JA
A-5. Prove that
cos( π + θ)cos( −θ)
(i) = cot2 θ.
π 
sin( π − θ)cos  + θ 
2 
(ii) cosθ + sin (270° + θ) – sin (270° – θ) + cos (180° + θ) = 0.
LP

 3π    3π  
(iii) cos  + θ  cos (2π + θ) cot  − θ  + cot (2π + θ) = 1.
 2    2  

A-6. If tan θ = – 5/12, θ is not in the second quadrant, then show that
A

sin(3600 − θ) + tan(900 + θ) 181


=
− sec(270 + θ) + cos ec( −θ)
0
338
NK

Section (B) : Graphs and Basic Identites (sin(A±B), cos(A±B), tan(A±B))

B-1. Sketch the following graphs :


(i) y = 3 sin 2x (ii) y = 2 tan x (iii) y = cosπx

B-2. Find number of solutions of equation sinx = –4x + 1


SA

2
B-3. If tanθ + secθ = then secθ is
3

B-4. Show that : (i) sin 20° . cos 40° + cos 20° . sin 40° = 3 /2
(ii) cos 100° . cos 40° + sin 100° . sin 40° = 1/2

θ 9θ 5θ
B-5. Show that : cos 2 θ cos – cos 3 θ cos = sin 5 θ sin .
2 2 2

0
1
B-6. If A + B = 450, prove that (1 + tanA)(1 + tan B) = 2 and hence deduce that tan 22 = 2–1
2

B-7. Eliminate θ from the relations a sec θ = 1 – b tan θ , a2 sec2θ = 5 + b2 tan2θ


Section (C) : sin2A – sin2B, Multiple angles upto 3A, 2sinA cosB, sinC – sinD

C-1. Show that :


(i) sin2 750 – sin2 150 = 3 /2 (ii) sin2 450 – sin2 150 = 3 /4

C-2. Find the value of


(i) 4 sin 18° cos 36° (ii) cos2 72° – sin2 54°
(iii) cos2 48º – sin2 12º

a2 – b2
C-3. If α and β are the solution of a cosθ + b sinθ = c, then show that cos(α + β) =
a2 + b2

π A π A  1 

RI
C-4. Show that : sin2  +  – sin2  −  =   sin A
8 2  8 2   2

C-5. Show that : cos²α + cos² (α + β) − 2cos α cos β cos (α + β) = sin²β .

A
C-6. Prove that
sin2 A − sin2 B
(i) = tan (A + B)
sin A cos A − sinBcosB

UH
4 cos 2A
(ii) cot (A + 15º) – tan (A – 15º) =
1 + 2 sin 2A
JA
C-7. If 0 < θ < π/4, then show that 2 + 2(1 + cos 4θ) = 2 cos θ

cos3 A − cos3A sin3 A + sin3A


C-8. Prove that + =3
cos A sin A
LP

C-9. Prove that


 2α − π 
 1 − cot  4  
   + cos α cot 4α  sec 9α = cosec 4α.
(i)   `
 1 + cot 2  α − π  2  2
 
 
A

 4 
1 1 sec 8A − 1 tan8A
(ii) − = cot 2α. (iii) =
tan3α − tan α cot 3α − cot α sec 4A − 1 tan 2A
NK

cos A + sin A cos A − sin A


(iv) – = 2 tan 2A
cos A − sin A cos A + sin A

sin3θ
C-10. Prove that sin θ = and hence deduce the value of sin 15°.
SA

1 + 2cos 2θ

C-11. Prove that 4(cos3 20° + cos3 40°) = 3(cos 20° + cos 40°)

C-12. Prove that :


tan3x 2cos 2x + 1 2 sin x tan x
(i) = (ii) + =1
tan x 2cos 2x − 1 sin3x tan3x

C-13. Prove that :


tan θ tan (600 + θ) tan(600 – θ) = tan 3θ and hence deduce that tan 200 tan 400 tan 600 tan 800 = 3.

C-14. Prove that :


(i) (cosec θ – sin θ) (sec θ – cos θ) (tan θ + cot θ) = 1
2sin θ tan θ (1 − tan θ) + 2sin θ sec 2 θ 2 sin θ
(ii) =
(1 + tan θ)2 (1 + tan θ)
1 − sin A
(iii) = ± (sec A – tan A)
1 + sin A
cos A cos ecA − sin A sec A
(iv) = cosec A – sec A
cos A + sin A
1 1 1 1
(v) – = –
sec α − tan α cos α cos α sec α + tan α
cos3 A + sin3 A cos3 A − sin3 A
(vi) + =2
cos A + sin A cos A − sin A

RI
Section (D) : Conditional Identities & Trigonometric Series

D-1. For all values of α, β, γ prove that,


α+β β+γ γ+α
cos α + cos β + cos γ + cos (α + β + γ) = 4 cos . cos . cos .

A
2 2 2

π
D-2. If x + y + z = show that, sin 2x + sin 2y + sin 2z = 4cosx cosy cosz.

UH
2

D-3. If x + y = π + z, then prove that sin2x + sin2y – sin2z = 2 sin x sin y cos z.

D-4. If A + B + C = 2S then prove that


JA
A B C
cos (S – A) + cos(S – B) + cos (S – C) + cos S = 4 cos cos cos
2 2 2

D-5. If A + B + C = 0° then prove that sin 2A + sin 2B + sin 2C = – 4 sin A sin B sin C

D-6. If φ is the exterior angle of a regular polygon of n sides and θ is any constant, then prove that
sin θ + sin (θ + φ) + sin (θ +2φ) + ............ up to n terms = 0
LP

n sinnθ cos(n + 1)θ


D-7. Prove that sin2 θ + sin2 2θ + sin2 3θ + ....... + sin2 nθ = –
2 2 sin θ
A

D-8. Prove that :


2π 4π 6π 1
(i) cos cos cos =
7 7 7 8
NK

π 2π 3π 4π 5π 1
(ii) cos cos cos cos cos =
11 11 11 11 11 32

Section (E) : Range of Trigonometric Expressions


 2π   2π 
E-1. Find the extreme values of cos x cos  + x  cos  − x
SA

 3   3 
E-2. Find the maximum and minimum values of following trigonometric functions
π 
(i) cos 2x + cos2x (ii) cos2  + x  + (sinx – cos x)2
 4 

E-3. Find the greatest and least value of y


 π
(i) y = 10 cos²x − 6 sin x cos x + 2 sin²x (ii) y = 3 cos  θ +  + 5 cos θ + 3
 3

Section (F) : Trigonometric Equations

F-1. What are the most general values of θ which satisfy the equations :
1
(i) sinθ = (ii) tan (x – 1) = 3
2
2
(iii) tanθ = – 1 (iv) cosecθ = .
3
(v) 2cot2θ = cosec2θ

F-2. Solve %
(i) sin9θ = sinθ
(ii) cotθ + tanθ = 2cosecθ
(iii) sin2θ = cos3θ
(iv) cotθ = tan8θ
(v) cotθ – tanθ = 2.

RI
(vi) cosecθ = cotθ + 3
(vii) tan2θ tanθ = 1
(viii) tanθ + tan2θ + 3 tanθ tan2θ = 3 .

A
F-3. Solve
(i) sinθ + sin3θ + sin5θ = 0.
(ii) cos θ + sin θ = cos 2 θ + sin 2 θ.

UH
(iii) cos2 x + cos2 2 x + cos2 3 x = 1 .
(iv) sin2nθ – sin2(n – 1)θ = sin2θ, where n is constant and n ≠ 0, 1

F-4. Solve
(i) tan2θ – (1 + 3 ) tanθ + 3 = 0
(ii) 4 cosθ – 3 secθ = 2 tanθ
JA
 π  2 π
(iii) tan x . tan  x +  . tan  x + = 3.
 3   3 

F-5. Solve
(i) 3 sinθ – cosθ = 2
LP

(ii) 5 sinθ + 2 cosθ = 5

Section (G) : Trigonometric Inquations and Height & Distance


G-1. Solve tan2 x ≤ 1
A

G-2. Solve 2sin2x – sinx – 1 > 0

G-3. Solve 3 cot θ < 1


NK

G-4. Two pillars of equal height stand on either side of a roadway which is 60 m wide. At a point in the
roadway between the pillars, the angle of elevation of the top of pillars are 60º and 30º. Then find height
of pillars -
G-5. If the angles of elevation of the top of a tower from two points distance a and b from the base and in the
same straight line with it are complementary, then find the height of the tower :
SA

G-6. From the top of a cliff 25 m high the angle of elevation of a tower is found to be equal to the angle of
depression of the foot of the tower. Then find height of the tower -

PART - II : ONLY ONE OPTION CORRECT TYPE


Section (A) : Measurement of Angles & Allied angles

A-1. cos (540° – θ) – sin (630° – θ) is equal to


(A) 0 (B) 2 cos θ (C) 2 sin θ (D) sin θ – cos θ

A-2. The value of tan 1° tan 2° tan 3° ... tan 89° is


1
(A) 1 (B) 0 (C) ∞ (D)
2
2π 4π
A-3. If x = y cos = z cos , then xy + yz + zx is equal to
3 3
(A) – 1 (B) 0 (C) 1 (D) 2

3
A-4. If 0° < x < 90° & cosx = , then the value of
10
log10 sin x + log10 cos x + log10tan x is
(A) 0 (B) 1 (C) – 1 (D) 2

A-5. If tanα + cot α = a then the value of tan4 α + cot4 α =


(A) a4 + 4a2 + 2 (B) a4 − 4a2 + 2 (C) a4 − 4a2 − 2 (D) a4 − 2a2 + 2

RI
Section (B) : Graphs and Basic Identites (sin(A±B), cos(A±B), tan(A±B))
B-1. STATEMENT-1 : sin 2 > sin 3
π 
STATEMENT-2 : If x, y ∈  , π  , x < y, then sin x > sin y

A
2 
(A) STATEMENT-1 is true, STATEMENT-2 is true and STATEMENT-2 is correct explanation for
STATEMENT-1

UH
(B) STATEMENT-1 is true, STATEMENT-2 is true and STATEMENT-2 is not correct explanation
for STATEMENT-1
(C) STATEMENT-1 is true, STATEMENT-2 is false
(D) STATEMENT-1 is false, STATEMENT-2 is true

If cosecθ – cotθ = α, then cotθ is :


B-2.
JA
1 1  1 1  1  1 
(A)  + α  (B)  − α  (C)  + α  (D)  – α 
2α  2α  α  α 

B-3. If a cos θ + b sin θ = 3 & a sin θ − b cos θ = 4 then a2 + b2 has the value =
(A) 25 (B) 14 (C) 7 (D) 10
LP

 π  3π   7π 
tan  x −  . cos  + x  − sin3  − x
B-4.  2  2   2  when simplified reduces to:
 π  3π 
cos  x − + x
2 
. tan 
  2 
A

(A) sin x cos x (B) − sin2 x (C) − sin x cos x (D) sin2x
  3π    π  
B-5. The expression 3 sin4  − α  + sin4 (3π + α ) – 2 sin6  + α  + sin6 (5π + α ) is equal to
  2    2  
NK

(A) 0 (B) 1 (C) 3 (D) sin 4α + sin 6α

B-6. The value of the expression


 π  3π   7π   9π 
 1 + cos 10   1 + cos 10   1 + cos 10   1 + cos 10  is
    
SA

1 1 1
(A) (B) (C) (D) 0
8 16 4

sin 24° cos 6° − sin 6° sin 66°


B-7. The value of is
sin 21° cos 39° − cos51° sin 69°
(A) −1 (B) 1 (C) 2 (D) 0

B-8. If tan A and tan B are the roots of the quadratic equation x2 − ax + b = 0, then the value of sin2 (A + B).
a2 a2 a2 a2
(A) (B) (C) (D)
a2 + (1− b)2 a2 + b2 (b + c)2 b2 (1 − a)2

B-9. If tan A – tan B = x and cot B – cot A = y, then cot (A – B) is equal to


1 1 1 1 1 1 1
(A) – (B) – (C) + (D)
y x x y x y x+y

tan155° − tan115°
B-10. If tan 25° = x, then is equal to
1 + tan155° tan115°
1 − x2 1 + x2 1 + x2 1 − x2
(A) (B) (C) (D)
2x 2x 1 − x2 1 + x2

 cot A   cot B 
B-11. If A + B = 225°, then the value of  .   is
 1 + cot A   1 + cot B 
1 1
(A) 2 (B) (C) 3 (D) –

RI
2 2

B-12. The value of tan 203° + tan 22° + tan 203° tan 22° is
(A) – 1 (B) 0 (C) 1 (D) 2

A
Section (C) : sin2A – sin2B, Multiple angles upto 3A, 2sinA cosB, sinC – sinD

1 − tan2 15°

UH
C-1. The value of is
1 + tan2 15°
3
(A) 1 (B) 3 (C) (D) 2
2
JA
C-2. If A lies in the third quadrant and 3 tan A – 4 = 0, then 5 sin 2A + 3sinA + 4 cosA is equal to
24 24 48
(A) 0 (B) – (C) (D)
5 5 5

C-3. If cos A = 3/4, then the value of 16cos2 (A/2) – 32 sin (A/2) sin (5A/2) is
(A) – 4 (B) – 3 (C) 3 (D) 4
LP

C-4. If tan2 θ = 2 tan2 φ + 1, then the value of cos 2θ + sin2 φ is


(A) 1 (B) 2 (C) – 1 (D) Independent of φ
π 
C-5. If α ∈  , π  then the value of 1 + sin α − 1 − sin α is equal to:
2 
A

α α
(A) 2 cos (B) 2 sin (C) 2 (D) none of these
2 2
NK

1 1
C-6. The value of + is
cos 290° 3 sin 250°
2 3 4 3
(A) (B) (C) 3 (D) none
3 3
SA

C-7. The value of tan 3A – tan 2A – tan A is equal to


(A) tan 3A tan 2A tan A (B) – tan 3A tan 2A tan A
(C) tan A tan 2A – tan 2A tan 3A – tan 3A tan A (D) none of these

cos 20° + 8 sin70° sin50° sin10°


C-8. is equal to:
sin2 80°
(A) 1 (B) 2 (C) 3/4 (D) 0

C-9. The numerical value of sin 12° . sin48° . sin 54° is equal to
1 1 1 1
(A) (B) (C) (D)
2 4 16 8

C-10. If A = tan 6° tan 42° and B = cot 66° cot 78°, then
1
(A) A = 2B (B) A = B (C) A = B (D) 3A = 2B
3

Section (D) : Conditional Identities & Trigonometric Series


D-1. In a triangle tan A + tan B + tan C = 6 and tan A tan B = 2, then the values of tan A, tan B and tan C are
respectively
(A) 1, 2, 3 (B) 2, 3, 1 (C) 1, 2, 0 (D) none of these

D-2. tan α + 2 tan 2α + 4 tan 4α + 8 cot 8 α =


(A) tan α (B) cot α (C) cot 16α (D) 16 cot α

π 2π 3π 4π 5π 6π
D-3. The value of cos 0 + cos + cos + cos + cos + cos + cos is

RI
7 7 7 7 7 7
(A) 1/2 (B) − 1/2 (C) 0 (D) 1

π 2π 4π 8π 16π
D-4. The value of cos cos cos cos cos is :

A
10 10 10 10 10
10 + 2 5 cos ( π /10 ) cos ( π /10 ) 10 + 2 5
(A) (B) – (C) (D) –
64 16 16 16

UH
π 3π 5π 17π
D-5. The value of cos + cos + cos +...... + cos is equal to :
19 19 19 19
(A) 1/2 (B) 0 (C) 1 (D) 2
JA
Section (E) : Range of Trigonometric Expressions
E-1. If f(θ) = sin4 θ + cos2 θ, then range of f(θ) is
1  1 3 3 
(A)  , 1 (B)  ,  (C)  , 1 (D) None of these
 2  2 4  4 
E-2. Range of function f(x) = cos x + 4sec2x is
2
LP

(A) [4, ∞) (B) [0, ∞) (C) [5, ∞) (D) (0, ∞)

E-3. The difference between maximum and minimum value of the expression y = 1 + 2 sin x + 3 cos2 x is
16 13
(A) (B) (C) 7 (D) 8
3 3
A

E-4. The maximum value of 12 sin θ – 9 sin2 θ is -


(A) 3 (B) 4 (C) 5 (D) None of these
NK

E-5. The greatest and least value of y = 10 cos²x − 6 sin x cos x + 2 sin²x are respectively
(A) 11, 1 (B) 10, 2 (C) 12, – 4 (D) 11, – 1

Section (F) : Trigonometric Equations


SA

F-1. The solution set of the equation 4sinθ.cosθ – 2cosθ – 2 3 sinθ + 3 = 0 in the interval (0, 2π) is
 3π 7π   π 5π   3π π 5π   π 5π 11π 
(A)  ,  (B)  ,  (C)  , π, ,  (D)  , , 
4 4  3 3  4 3 3  6 6 6 

F-2. All solutions of the equation 2 sinθ + tanθ = 0 are obtained by taking all integral values of m and n in:

2π 2π
(A) 2nπ + ,n∈Ι (B) nπ or 2m π ± where n, m ∈ Ι
3 3
π π
(C) nπ or m π ± where n, m ∈ Ι (D) nπ or 2m π ± where n, m ∈ Ι
3 3

F-3. Total number of solutions of equation sinx . tan4x = cosx belonging to (–π, 2π) are :
(A) 4 (B) 7 (C) 8 (D) 15
 π
F-4. If x ∈ 0 ,  , the number of solutions of the equation sin 7x + sin 4x + sin x = 0 is:
 2
(A) 3 (B) 5 (C) 6 (D) 4

F-5. The general solution of equation sinx + sin5x = sin2x + sin4x is :


nπ nπ nπ 2 nπ
(A) ; n∈Ι (B) ;n∈Ι (C) ;n∈Ι (D) ;n∈Ι
2 5 3 3

F-6. The general solution of the equation 2cos2x = 3.2cos 2x − 4 is


(A) x = 2nπ, n ∈ Ι (B) x = nπ , n ∈ Ι (C) x = nπ/4 , n ∈ Ι (D) x = nπ/2, n ∈ Ι

F-7. If 2 cos2 (π + x) + 3 sin (π + x) vanishes then the values of x lying in the interval from 0 to 2π are

RI
(A) x = π/6 or 5π/6 (B) x = π/3 or 5π/3 (C) x = π/4 or 5π/4 (D) x = π/2 or 5π/2

cos 3θ 1
F-8. = if
2 cos 2θ − 1 2

A
π π π π
(A) θ = nπ + ,n∈Ι (B) θ = 2nπ± ,n∈Ι (C) θ = 2nπ ± , n ∈ Ι (D) θ = nπ + , n ∈ Ι
3 3 6 6

UH
F-9. If cos 2θ + 3 cos θ = 0, then
 17 − 3   − 17 − 3 
(A) θ = 2nπ ± α where α = cos–1   (B) θ = 2nπ ± α where α = cos–1  
 4  4
   
 ± 17 − 3   17 + 3 
(C) θ = 2nπ ± α where α = cos–1   (D) θ = 2nπ ± α where α = cos–1  
  
JA
 4   4 
F-10. If sin θ + 7 cos θ = 5, then tan (θ/2) is a root of the equation
(A) x2 − 6x + 1 = 0 (B) 6x2 − x − 1 = 0 (C) 6x2 + x + 1 = 0 (D) x2 − x + 6 = 0

1
F-11. The most general solution of tanθ = – 1 and cosθ = is :
2
LP

7π 7π 7π 3π
(A) nπ + ,n∈Ι (B) nπ + (– 1)n , n ∈ Ι (C) 2n π + ,n∈Ι (D) 2n π + ,n∈Ι
4 4 4 4

1
F-12. A triangle ABC is such that sin(2A + B) = . If A, B, C are in A.P. then the angle A, B, C are
2
A

respectively.
5π π π π π 5π π π 5π π 5π π
(A) , , (B) , , (C) , , (D) , ,
12 4 3 4 3 12 3 4 12 3 12 4
NK

Section (G) : Trigonometric Inquations and Height & Distance

G-1. The complete solution of inequality sec23x < 2 is


 n π π nπ π   n π π nπ π 
(A) x ∈  ,n∈Ι (B) x ∈  3 – 12 , 3 + 6  , n ∈ Ι
SA

– , +
 3 12 3 12   
 π π   n π π n π π 
(C) x ∈  nπ – ,nπ + ,n∈Ι (D) x ∈  3 – 6, 3 + 6  , n ∈ Ι
 12 12   

G-2. The complete solution of inequality 2cos2x – 7 cosx + 3 < 0 is


π π π π
(A) nπ – < x < + nπ (B) 2nπ – < x < + 2nπ
3 3 6 6
π π π π
(C) 2nπ – < x < + 2nπ (D) nπ – < x < + nπ
3 3 6 6

G-3. The complete solution of inequality cos 2x ≤ cos x is


 π π  2π 2π 
(A) x ∈ 2nπ – , 2nπ +  (B) x ∈ 2nπ – , 2nπ +
 3 3  3 3 
 2π   2π 
(C) x ∈ 2nπ, 2nπ + (D) x ∈ 2nπ – , 2nπ 
 3   3 

G-4. Which of the following set of values of x satisfy the inequation tan2x – (1 + 3 ) tan x + 3<0

 ( 4n + 1) π ( 3n + 1) π   ( 2n + 1) π ( 2n + 1) π 
(A)  ,  , (n ∈ Z) (B)  ,  , (n ∈ Z)
 4 3   4 3 
   
 ( 4n + 1) π ( 4n + 1) π   π π
(C)  ,  , (n ∈ Z) (D) x ∈  , 
 4 3   4 2
 

G-5. A tree 12 m high, is broken by the wind in such a way that its top touches the ground and makes an

RI
angle 60º with the ground. The height from the bottom of the tree from where it is broken by the wind is
appoximately
(A) 5.57 m (B) 5.21 (C) 5.36 (D) 5.9

A
G-6. AB is a vertical pole and C is the middle point. The end A is on the level ground and P is any point on
the level ground other than A. The portion CB subtends an angle β at P. If AP : AB = 2 : 1, then β is
equal to-
 1 4 5 2

UH
(A) tan–1   (B) tan–1   (C) tan–1   (D) tan–1  
 
9  
9  
9 9
G-7. A round ballon of radius r subtends an angle α at the eye of the observer, while the angle of elevation
of its centre is β. The height of the centre of ballon is-
β β α α
(A) r cosec α sin (B) r sin α cosec (C) r sin cosec β (D) r cosec sin β
JA
2 2 2 2

G-8. If the angle of elevation of a cloud from a point 200 m above a lake is 30º and the angle of depression
of its reflection in the lake is 60º, then the height of the cloud above the lake, is
(A) 200 m (B) 500 m (C) 30 m (D) 400 m

G-9. A man on the top of a vertical tower observes a car moving at a uniform speed coming directly towards
LP

it. If it takes 12 minutes for the angle of depression to change from 30º to 45º, then the car will reach
the tower in
(A) 17 minutes 23 seconds (B) 16 minutes 23 seconds
(C) 16 minutes 18 seconds (D) 18 minutes 22 seconds
A

PART - III : MATCH THE COLUMN


1. Column - I Column - II
NK

(A) tan 9° − tan 27° − tan 63° + tan 81° (p) 1

(B) cosec 10° – 3 sec 10° (q) 2


SA

 sec 5° cos 40° 


(C) 2 2 sin10°  + – 2 sin 35° (r) 3
 2 sin 5° 

(D) 3 (cot 70º + 4 cos 70º) (s) 4

2. Column – Ι Column – ΙΙ
1
(A) If for some real x, the equation x + = 2 cos θ holds, (p) 2
x
then cos θ is equal to

(B) If sin θ + cosec θ = 2, then sin2008 θ + cosec2008θ is equal to (q) 1

(C) Maximum value of sin4θ + cos4θ is (r) 0


(D) Least value of 2 sin2θ + 3 cos2θ is (s) –1

3. Column – Ι Column – ΙΙ

(A) Number of solutions of sin2θ + 3 cos θ = 3 (p) 0


in [– π, π]

(B) Number of solutions of sin x . tan 4x = cos x (q) 1


in (0, π)

(C) Number of solutions of equation (r) 4


2
θ  π π
(1 – tan θ) (1 + tan θ) sec2θ + 2tan = 0 where θ ∈  − ,

RI

 2 2 
(D) If [sin x] + [ 2 cosx] = – 3, where x ∈ [0, 2π] (s) 5
then [sin 2x] equals (Here [.] denotes G.I.F.)

A
UH
JA
LP
A
NK
SA
 Marked questions are recommended for Revision.

PART - I : ONLY ONE OPTION CORRECT TYPE


1. In a triangle ABC if tan A < 0 then:
(A) tan B. tan C > 1 (B) tan B. tan C < 1 (C) tan B. tan C = 1 (D) Data insufficient

2. If sin α = 1/2 and cos θ = 1/3, then the values of α + θ (if θ, α are both acute) will lie in the interval

π π  π 2π   2π 5 π   5π 
(A)  ,  (B)  ,  (C)  , (D)  , π 
6 

RI
3 2 2 3  3 6 

sin A 3 cos A 5
3. If = and = , 0 < A, B < π/2, then tan A + tan B is equal to

A
sinB 2 cosB 2
(A) 3/ 5 (B) 5/ 3 (C) 1 (D) ( 5 + 3 ) / 5

UH
4. In a right angled triangle the hypotenuse is 2 2 times the perpendicular drawn from the opposite
vertex. Then the other acute angles of the triangle are
π π π 3π π π π 3π
(A) & (B) & (C) & (D) &
3 6 8 8 4 4 5 10
JA
5. If 3 cos x + 2 cos 3x = cos y, 3 sin x + 2 sin 3x = sin y, then the value of cos 2x is
1 1 7
(A) – 1 (B) (C) – (D)
8 8 8

cos3θ
6. If cos α + cos β = a, sin α + sin β = b and α – β = 2θ, then =
LP

cos θ
(A) a2 + b2 – 2 (B) a2 + b2 – 3 (C) 3 – a2 – b2 (D) (a2 + b2) /4

3π 1
7. If < α < π, then 2cot α + is equal to
A

4 sin2 α
(A) 1 + cot α (B) – 1 – cot α (C) 1 – cot α (D) – 1 + cot α
NK

π π sin θ + sin 2θ
8. For − < θ< , lies in the interval
2 2 1 + cos θ + cos 2θ
(A) (−∞, ∞) (B) (−2, 2) (C) (0, ∞) (D) (−1, 1)

9. The number of all possible triplets (a1, a2, a3) such that a1 + a2 cos 2x + a3 sin2x = 0 for all x is
SA

(A) 0 (B) 1 (C) 2 (D) infinite


10. If A + B + C = , then cos 2A + cos2B + cos2C is equal to
2
(A) 1 – 4cos A cosB cosC (B) 4 sin A sin B sin C
(C) 1 + 2 cos A cos B cos C (D) 1– 4 sin A sin B sin C

11. If A + B + C = π & cosA = cosB. cosC then tanB. tanC has the value equal to:
(A) 1 (B) 1/2 (C) 2 (D) 3
12. The general solution of the equation tan2 α +2 3 tan α = 1 is given by:
nπ π
(A) α = ,n∈Ι (B) α = (2n + 1) , n ∈ Ι
2 2
π nπ
(C) α = (6n + 1) ,n∈Ι (D) α = ,n∈Ι
12 12

 π  2π 
13. The general solution of the equation tan x + tan  x +  + tan  x + = 3 is
 3  3 
nπ π nπ π nπ π nπ π
(A) + ,n∈Ι (B) + ,n∈Ι (C) + ,n∈Ι (D) + ,n∈Ι
4 12 3 6 3 12 3 4

sec 2 x
14. The complete solution set of the equation 1 + 2 cosec x = − 2
is
2
π π π π
(A) 2 n π − ,n∈Ι (B) n π − ,n∈Ι (C) 2 n π + ,n∈Ι (D) n π + ,n∈Ι
2 2 2 2

RI
15. The principal solution set of the equation 2 cos x = 2 + 2sin2 x is
 π 13 π   π 13 π   π 13 π   π 13π 
(A)  ,  (B)  ,  (C)  ,  (D)  , 

A
8 8  4 8   4 10   8 10 

16. The solution of |cosx| = cosx – 2sinx is


π

UH
(A) x = nπ, n ∈ Ι (B) x = nπ + ,n∈Ι
4
π π
(C) x = nπ + (–1)n ,n∈Ι (D) x = (2n + 1)π + , n ∈ Ι
4 4

The solution of inequality 4tanx – 3.2tanx + 2 ≤ 0 is


17.
JA
 π  π
(A) x ∈ nπ, nπ +  ; n ∈ Ι (B) x ∈ nπ, nπ –  ; n ∈ Ι
 4  4
 π  π
(C) x ∈ nπ, nπ +  ; n ∈ Ι (D) x ∈ nπ, nπ –  ; n ∈ Ι
 6  6
LP

18. The solution of inequality 5 − 2sin x ≥ 6 sin x – 1 is


(A) [π (12n – 7)/6, π (12n + 7)/6] (n ∈ Z) (B) [π (12n – 7)/6, π (12n + 1)/6] (n ∈ Z)
(C) [π (2n – 7)/6, π (2n + 1)/6] (n ∈ Z) (D) [π (12n – 7)/3, π (12n + 1)/3] (n ∈ Z)
A

PART - II : SINGLE AND DOUBLE VALUE INTEGER TYPE


α+β
tan
NK

1. If 3 sinα =5 sinβ, then find the value of 2 .


α −β
tan
2

2. If α, β (α – β ≠ 2nπ, n ∈ Ι) are different values of θ satisfying the equation 5 cos θ – 12 sin θ = 11. If the
5k
SA

value of sin (α + β) = – , then find the value of k.


169

 3π  π x
3. If x ∈  π,  , then 4cos  4 − 2  +
2
4 sin4 x + sin2 2x is always equal to
 2   

4. If three angles A, B, C are such that


cos A + cos B + cos C = 0 and if
λ cos A cos B cos C = cos 3A + cos 3B + cos 3C, then value of λ is :

5. Find the number of integral values of λ, for which equation 4cos x + 3 sin x = 2λ + 1 has a solution.

6. If a cos3 α + 3a cos α sin2 α = m and a sin3 α + 3a cos2 α sin α = n. if (m+n)2/3 + (m − n)2/3 = 2λa2/3, then
find value of λ.
7. If 2 cos x + sin x = 1, then find the sum of all possible values of 7 cos x + 6 sin x.

π  3π 
8. The number of roots of the equation cot x = + x in  −π, is,
2  2 

 nπ 
9. If 2tan2x – 5 secx – 1 = 0 has 7 different roots in 0, , n ∈ N, then find the greatest value of n.
 2 

10. Find the number of integral values of a for which the equation cos 2x + a sin x = 2a − 7 possesses a
solution.

11. The number of solutions of the equation |sinx| = | cos3x| in [–2π, 2π] is

RI
12. In any triangle ABC, which is not right angled ∑cosA .cosecB.cosecC is equal to

13. If A + B + C = π, then find value of tan B tan C + tan C tan A + tan A tan B – sec A sec B sec C.

A
14. If the arithmetic mean of the roots of the equation 4cos 3x – 4cos2x – cos(π + x) – 1 = 0 in the interval
[0, 315] is equal to kπ , then find the value of k

UH
1
15. cos (α – β) = 1 and cos (α + β) = , where α, β ∈ [–π, π]. Then number of ordered pairs (α, β) which
e
satisfy both the equations.

Number of values of θ between 0° and 90° which satisfy the equation sec 2θ .cosec2θ + 2 cosec2θ = 8
16.
JA
17. Find the number of all values of θ ∈ [0, 10.5] satisfying the equation
cos 6 θ + cos 4 θ + cos 2 θ + 1 = 0 .

18. In (0, 6π), find the number of solutions of the equation tanθ + tan 2θ + tan 3θ = tan θ.tan2θ.tan3θ
LP

19. If 0 ≤ x ≤ 3π , 0 ≤ y ≤ 3π and cos x . sin y = 1, then find the possible number of values of the ordered
pair (x, y)

20. Find the number of values of θ satisfying the equation sin3 θ = 4sin θ. sin 2θ. sin 4θ in 0 ≤ θ ≤ 2π
A

21. Find the number of solutions of the equation cos 6x + tan2 x + cos 6x . tan2 x = 1 in the interval [0, 2π].

3 7 12(θ + φ)
22. Consider tan θ + sin φ = & tan² θ + cos² φ = , find maximum value of if θ + φ ∈ (0, 2π).
NK

2 4 π

23. Find the number of integral values of n so that sinx(sinx + cosx) = n has at least one solution.

24. Find the number of values of x in (0, 2π) satisfying the equation cot x − 2 sin 2x = 1 .
SA

25. Find the number of solutions of sinθ + 2sin2θ + 3sin3θ + 4sin4θ = 10 in(0, π).

26. Find the values of x satisfying the equation 2 sin x = 3 x 2 + 2 x + 3.

27. Find the number of solution of sinx cosx – 3 cosx + 4 sinx – 13 > 0 in [0,2π] .

PART - III : ONE OR MORE THAN ONE OPTIONS CORRECT TYPE


(cos11° + sin11°)
1. The value of is
(cos11° − sin11°)
(A) –tan 304° (B) tan 56° (C) cot 214° (D) cot 34°

1 t
2. If sin t + cos t = then tan can be
5 2
1 1
(A) −1 (B) – (C) 2 (D) –
3 6

sin x + cos x
3. The value of =
cos3 x
(A) 1 + tan x + tan2x − tan3x (B) 1 + tan x + tan2x + tan3x
(C) 1 − tan x + tan2x + tan3x (D) (1 + tan x) sec2x

4. If (sec A + tan A) (sec B + tan B) (sec C + tan C) = (sec A – tan A) (sec B – tan B) (sec C – tan C) then
each side can be
(A) 1 (B) –1 (C) 0 (D) none

5. Which of the following is correct ?

RI
(A) sin 1° > sin 1 (B) sin 1° < sin 1 (C) cos 1° > cos 1 (D) cos 1° < cos 1

6. If sin x + sin y = a & cos x + cos y = b, then which of the following may be true.
2 ab x−y 4 − a2 − b2
(A) sin (x + y) = (B) tan =

A
a2 + b2 2 a2 + b2

x−y 4 − a2 − b2 2ab
(C) tan =– (D) cos (x + y) =

UH
2 a2 + b2 a2 + b2

3
7. If cos (A – B) = and tan A tan B = 2, then which of the following is/are correct
5
1 2 1 4
(A) cos A cos B = – (B) sin A sin B =
JA (C) cos (A + B) = – (D) sin A cos B =
5 5 5 5

8. If Pn = cosnθ + sinnθ and Qn = cosnθ – sinnθ, then which of the following is/are true.
(A) Pn – Pn–2 = – sin2θ cos2θ P n–4 (B) Q n – Q n–2 = – sin2θ cos2θ Q n–4
(C) P4 = 1 – 2 sin θ cos θ
2 2
(D) Q4 = cos2θ – sin2θ

tan2α + 2tanα. tan2β = tan2β + 2tanβ. tan2α, if


LP

9.
(A) tan2α + 2tanα. tan2β = 0 (B) tan α + tan β =0
(C) tan2β + 2tanβ. tan2α = 1 (D) tan α = tan β

10. If the sides of a right angled triangle are {cos2α + cos2β + 2cos(α + β)} and
{sin2α + sin2β + 2sin(α + β)}, then the length of the hypotenuse is :
A

α − β α + β
(A) 2[1+cos(α − β)] (B) 2[1 − cos(α + β)] (C) 4 cos 2 (D) 4sin2
2 2
NK

11. For 0 < θ < π/2, tan θ + tan 2θ + tan 3θ = 0 if


(A) tan θ = 0 (B) tan 2θ = 0 (C) tan 3θ = 0 (D) tan θ tan 2θ = 2
12. (a + 2) sin α + (2a – 1) cos α = (2a + 1) if tan α =
3 4 2a 2a
(A) (B) (C) 2 (D)
4 3 a +1 a −1
2
SA

2b
13. If tan x = , (a ≠ c)
a−c
y = a cos2x + 2b sin x cos x + c sin2x
z = a sin2x – 2b sin x cos x + c cos2x, then
(A) y = z (B) y + z = a + c (C) y – z = a – c (D) y – z = (a – c)2 + 4b2

n n
 cos A + cosB   sin A + sinB 
14. The value of   +   is
 sin A − sinB   cos A − cosB 
A −B A −B
(A) 2 tann (B) 2 cotn : n is even
2 2
(C) 0 : n is odd (D) 0 : n is even

15. The equation sin6x + cos6x = a2 has real solution if


 1  1 1 1 
(A) a ∈ (–1, 1) (B) a ∈  −1, −  (C) a ∈  −  (D) a ∈  , 1
 2  2 2 2 

16. If sin(x − y) = cos(x + y) = 1/2 then the values of x & y lying between 0 and π are given by:
(A) x = π/4, y = 3π/4 (B) x = π/4, y = π/12
(C) x = 5π/4, y = 5π/12 (D) x = 11π/12, y = 3π/4

17. If 2 sec2 α – sec4 α – 2 cosec2 α + cosec4 α = 15/4, then tan α can be


(A) 1/ 2 (B) 1/2 (C) 1/2 2 (D) –1/ 2

18. If 3 sin β = sin (2α + β), then tan (α + β) – 2 tan α is


(A) independent of α (B) independent of β
(C) dependent of both α and β (D) independent of α but dependent of β

RI
19. If α + β + γ = 2π, then
α β γ α β γ
(A) tan + tan + tan = tan tan tan
2 2 2 2 2 2

A
α β β γ γ α
(B) tan tan + tan tan + tan tan =1
2 2 2 2 2 2
α β γ α β γ

UH
(C) tan + tan + tan = – tan tan tan
2 2 2 2 2 2
α β β γ γ α
(D) tan tan + tan tan + tan tan = 1
4 4 4 4 4 4

20. If x + y = z, then cos2 x + cos2 y + cos2 z – 2 cos x cos y cos z is equal to


JA
(A) cos2 z (B) sin2 z (C) cos (x + y – z) (D) 1

21. If tanA + tan B + tan C = tan A. tan B. tan C, then


(A) A, B, C may be angles of a triangle (B) A + B + C is an integral multiple of π
(C) sum of any two of A, B, C is equal to third (D) none of these

1 − 2x + 5x 2  π π
LP

22. Which of the following values of ' t ' may satisfy the condition 2 sin t = , t ∈ − ,  .
3x − 2x − 1
2
 2 2
 π π  π  3π π   −π 3π 
(A)  − , −  (B) 0,  (C)  ,  (D)  , 
 2 10   2  10 2   10 10 
A

23. sinx, sin2x, sin3x are in A.P if


(A) x = nπ/2, n ∈ Ι (B) x = nπ, n ∈ Ι (C) x = 2nπ, n ∈ Ι (D) x = (2n +1)π, n ∈ Ι
NK

24. sin x + sin2x + sin 3x = 0 if


(A) sin x = 1/2 (B) sin 2x = 0 (C) sin 3x = 3 /2 (D) cos x = − 1/2

25. cos4x cos8x − cos5x cos9x = 0 if


(A) cos12x = cos 14 x (B) sin13 x = 0
SA

(C) sinx = 0 (D) cosx = 0

26. sinx − cos2x − 1 assumes the least value for the set of values of x given by:
(A) x = nπ + (−1)n+1 (π/6) , n ∈ Ι (B) x = nπ + (−1)n (π/6) , n ∈ Ι
(C) x = nπ + (−1) (π/3), n ∈ Ι
n
(D) x = nπ − (−1)n (π/6) , n ∈ Ι

π
27. Let 0 ≤ θ ≤ and x = X cosθ + Ysinθ , y = Xsinθ – Ycosθ such that x 2 + 4xy + y2 = aX2 + bY2,
2
where a, b are constants then
π
(A) a = –1, b = 3 (B) θ = π/4 (C) a = 3, b = – 1 (D) θ =
3

28. If the equation sin (πx2) – sin(πx2 + 2πx) = 0 is solved for positive roots, then in the increasing sequence
of positive root
−1 + 7 −1 + 3
(A) first term is (B) first term is
2 2
−1 + 11
(C) third term is 1 (D) third term is
2

29. The general solution of the equation cosx . cos6x = – 1, is :


(A) x = (2n + 1)π, n ∈ Ι (B) x = 2nπ, n ∈ Ι
(C) x = (2n – 1)π, n ∈ Ι (D) none of these

30. Which of the following set of values of x satisfy the inequation sin 3x < sin x.
 ( 8n − 1) π   ( 8n − 1) π (8n + 1)π 
(A)  , 2nπ  , n ∈ Ι (B)  ,  ,n∈Ι
 4   4 4 
   

RI
 ( 8n + 1) π (8n + 3)π   ( 8n + 5 ) π  , n ∈ Ι
(C)  , , n ∈ Ι (D)  ( 2n + 1) π, 
 4 4   4 
   

A
x x
31. The equation 2sin . cos2x + sin2x = 2 sin . sin2x + cos2x has a root for which
2 2
1 1
(A) sin2x = 1 (B) sin2x = – 1 (C) cosx = (D) cos2x = –

UH
2 2

32. cos 15 x = sin 5x if


π nπ π nπ
(A) x = − + ,n ∈ Ι (B) x = + ,n∈Ι
20 5 40 10
JA
3 π nπ 3 π nπ
(C) x = + ,n∈Ι (D) x = − + ,n∈Ι
20 5 40 10

33. 5 sin2 x + 3 sinx cosx + 6 cos2x = 5 if


(A) tan x = − 1/ 3 (B) sin x = 0
(C) x = nπ + π/2, n ∈ Ι (D) x = nπ + π/6, n ∈ Ι
LP

34. sin2x + 2 sin x cos x − 3cos2x = 0 if


(A) tan x = 3 (B) tanx = − 1
(C) x = nπ + π/4, n ∈ Ι (D) x = nπ + tan−1 (−3), n ∈ Ι
A

35. Solution set of inequality sin3 x cos x > cos3 x sin x , where x ∈ (0, π), is
π π  3π   π  π 3π 
(A)  ,  (B)  , π  (C)  0 ,  (D)  , 
NK

4 2  4   4 2 4 

36. 4 sin4x + cos4x = 1 if


1  1
(A) x = nπ ; (n ∈ I) (B) x = nπ ± cos–1   ; (n ∈ I)
2 5
SA


(C) x = ; (n ∈ I) (D) x = – nπ ; (n ∈ I)
2

37. sin x + sin 2x + sin 3x = cos x + cos 2x + cos 3x if


1
(A) cos x = – (B) sin 2x = cos 2x
2
nπ π 2π
(C) x = + (D) x = 2nπ ± , (n ∈ Ι)
2 8 3
PART - IV : COMPREHENSION
Comprehenssion # 1

Let p be the product of the sines of the angles of a triangle ABC and q is the product of the cosines
of the angles.

1. In this triangle tan A + tan B + tan C is equal to


p
(A) p + q (B) p – q (C) (D) none of these
q

2. tan A tan B + tan B tan C + tan C tan A is equal to


1+ q 1+ p
(A) 1 + q (B) (C) 1 + p (D)

RI
q p

3. The value of tan3 A + tan3 B + tan3 C is


p3 − 3pq2 q3 p3 p3 − 3pq

A
(A) (B) (C) (D)
q3 p3 q3 q3
Comprehension # 2

UH
Let a, b, c, d ∈ R. Then the cubic equation of the type ax 3 + bx2 + cx + d = 0 has either one root real or
all three roots are real. But in case of trigonometric equations of the type a sin 3 x + b sin2 x + c sinx + d
= 0 can possess several solutions depending upon the domain of x.
To solve an equation of the type a cosθ + b sinθ = c. The equation can be written as
cos (θ – α) = c/ (a2 + b2 ) .
JA
The solution is θ = 2nπ + α ± β, where tan α = b/a, cos β = c/ (a2 + b2 ) .
4. On the domain [–π, π] the equation 4sin 3 x + 2 sin2 x – 2sinx – 1 = 0 possess
(A) only one real root (B) three real roots
(C) four real roots (D) six real roots

10 tan x
LP

5. In the interval [–π/4, π/2], the equation, cos 4x + = 3 has


1 + tan2 x
(A) no solution (B) one solution (C) two solutions (D) three solutions
1
6. |tan x| = tan x + (0 ≤ x ≤ 2π) has
cos x
A

(A) no solution (B) one solution (C) two solutions (D) three solutions

Comprehension # 3
NK

To solve a trigonometric inequation of the type sin x ≥ a where |a| ≤ 1, we take a hill of length 2π in the
sine curve and write the solution within that hill. For the general solution, we add 2nπ. For instance, to
1  π 3π  π 7π
solve sinx ≥ – , we take the hill  – ,  over which solution is – <x< . The general
2  2 2  6 6
π 7π
SA

solution is 2nπ – < x < 2nπ + , n is any integer. Again to solve an inequation of the type sin x ≤ a,
6 6
where |a| ≤ 1, we take a hollow of length 2π in the sine curve. (since on a hill, sinx ≤ a is satisfied over
two intervals). Similarly cos x ≥ a or cosx ≤ a, |a| ≤ 1 are solved.

7
7. Solution to the inequation sin6x + cos6x < must be
16
π π π π
(A) nπ + < x < nπ + (B) 2nπ + < x < 2nπ +
3 2 3 2
nπ π nπ π
(C) + <x< + (D) none of these
2 6 2 3

8. Solution to inequality cos 2x + 5 cos x + 3 ≥ 0 over [– π, π] is


 –5π 5π   –2π 2π 
(A) [– π, π] (B)  , (C) [0, π] (D)  , 
 6 6   3 3 
 π
9. Over [– π, π], the solution of 2 sin 2  x +  + 3 cos 2x ≥ 0 is
 4
 – 5π 5π 
(A) [– π, π] (B)  ,
 6 6 
 –7π   π 5π   3π 
(C) [0, π] (D)  – π,  ∪  – 4 , 12  ∪  4 , π 
 12     

RI
A
UH
JA
LP
A
NK
SA
* Marked Questions may have more than one correct option.

PART - I : JEE (ADVANCED) / IIT-JEE PROBLEMS (PREVIOUS YEARS)


sin4 x cos4 x 1
1*. If + = , then [IIT-JEE - 2009 ,Paper-1, (4, –1), 80]
2 3 5
2 sin8 x cos8 x 1
(A) tan2x = (B) + =
3 8 27 125
1 sin8 x cos8 x 1
(C) tan2 x = (D) + =

RI
3 8 27 125

6
π  (m − 1) π   mπ 
2*. For 0 < θ < , the solution(s) of ∑ cos ec  θ +

 cos ec  θ + 4  = 4 2 is(are)
  

A
2 m =1
4
[IIT-JEE - 2009, Paper-2, (4, –1), 80]
π π π 5π
(A) (B) (C) (D)

UH
4 6 12 12
1
3. The maximum value of the expression is
sin2 θ + 3 sin θ cos θ + 5 cos2 θ
[IIT-JEE-2010, Paper-1, (3, 0)/84]
4.
JA
The positive integer value of n > 3 satisfying the equation
1 1 1
= + is [IIT-JEE 2011, Paper-1, (4, 0), 80]
π  2π   3π 
sin   sin   sin  n 
n  n   

 π π nπ
5. The number of values of θ in the interval  – ,  such that θ ≠ for n = 0, ±1, ± 2 and
 2 2  5
LP

tanθ = cot 5θ as well as sin 2θ = cos 4θ is [IIT-JEE-2010, Paper-1, (3, 0)/84]

6. Let P = {θ : sin θ – cos θ = 2 cos θ} and Q = {θ : sin θ + cos θ = 2 sin θ} be two sets. Then
(A) P ⊂ Q and Q – P ≠ ∅ (B) Q ⊂ / P
A

(C) P ⊂/ Q (D) P = Q [IIT-JEE 2011, Paper-1, (3, –1), 80]

 θ θ
Let θ, φ ∈ [0, 2π] be such that 2cosθ(1 – sinφ) = sin2θ  tan + cot  cosφ – 1, tan(2π – θ) > 0 and
NK

7*.
 2 2 
3
–1 < sinθ < – . Then φ cannot satisfy [IIT-JEE 2012, Paper-1, (4, 0), 70]
2
π π 4π 4π 3π 3π
(A) 0 < φ < (B) < φ< (C) < φ< (D) < φ < 2π
2 2 3 3 2 2
SA

8. For x ∈ (0, π), the equation sinx + 2 sin 2x – sin 3x = 3 has


[JEE (Advanced) 2014, Paper-2, (3, –1)/60]
(A) infinitely many solutions (B) three solutions
(C) one solution (D) no solution

9. The number of distinct solutions of the equation


5
cos2 2x + cos4 x + sin4 x + cos6 x + sin6x = 2 in the interval [0, 2π] is
4
[JEE (Advanced) 2015, P-1 (4, 0) /88]
13


1
10. The value of is equal to [JEE (Advanced) 2016, Paper-2, (3, –1)/62]

k = 1 sin
π (k − 1)π  π kπ 
+  sin + 
4 6  4 6 
(A) 3 − 3 (
(B) 2 3 − 3 ) (C) 2 ( 3 −1) (
(D) 2 2 + 3 )
 π
11. Let S = x ∈ (– π, π) : x ≠ 0,±  . The sum of all distinct solutions of the equation
 2
3 sec x + cosec x + 2(tan x – cot x) = 0 in the set S is equal to
[JEE (Advanced) 2016, Paper-1, (3, –1)/62]
7π 2π 5π
(A) – (B) – (C) 0 (D)
9 9 9

12. Let α and β be nonzero real numbers such that 2(cos β – cos α) + cos α cos β = 1. Then which of the
following is/are true? [JEE(Advanced) 2017, Paper-2,(4, –2)/61]
α β α β
(A) 3 tan   – tan   = 0 (B) tan   – 3 tan   = 0

RI
 
2  
2  
2 2
α β α β
(C) tan   + 3 tan   = 0 (D) 3 tan   + tan   = 0
2 2 2 2

A
13. Let a, b, c be three non-zero real numbers such that the equation 3 a cos x + 2b sinx = c,
 π π π b
x ∈  − ,  , has two distinct real roots α and β with α + β = . Then, the value of is ______.
 2 2 3 a

UH
[JEE(Advanced) 2018, Paper-1,(4, –2)/60]

PART - II : JEE (MAIN) / AIEEE PROBLEMS (PREVIOUS YEARS)


1. Let A and B denote the statements [AIEEE 2009 (4, –1), 144]
JA
A : cos α + cos β + cos γ = 0
B : sin α + sin β + sin γ = 0
3
If cos (β – γ) + cos (γ – α) + cos (α – β) = – , then :
2
(1) A is false and B is true (2) both A and B are true
(3) both A and B are false (4) A is true and B is false
LP

4 5 π
2. Let cos(α + β) = and let sin(α – β) = , where 0 ≤ α, β ≤ . Then tan 2α =
5 13 4
[AIEEE 2010 (4, –1), 144]
A

56 19 20 25
(1) (2) (3) (4)
33 12 7 16
NK

3. If A = sin2 x + cos4 x, then for all real x : [AIEEE 2011 (4, –1), 120]
3 13 3 13
(1) ≤ A ≤ 1 (2) ≤ A ≤1 (3) 1 ≤ A ≤ 2 (4) ≤A≤
4 16 4 16
SA

4. In a ∆PQR, if 3 sin P + 4 cos Q = 6 and 4 sin Q + 3 cos P = 1, then the angle R is equal to :
[AIEEE-2012, (4, –1)/120]
5π π π 3π
(1) (2) (3) (4)
6 6 4 4

tan A cot A
5. The expression + can be written as : [AIEEE - 2013, (4, –1),360]
1 − cot A 1 − tan A
(1) sinA cosA + 1 (2) secA cosecA + 1 (3) tanA + cotA (4) secA + cosecA

1
6. Let fk (x) = (sinkx + coskx) where x ∈ R and k ≥ 1. Then f4(x) – f6(x) equals
k
[JEE(Main)2014,(4, – 1), 120]
1 1 1 1
(1) (2) (3) (4)
4 12 6 3
7. If the angles of elevation of the top of a tower from three collinear points A, B and C, on a line leading to
the foot of the tower, are 30º, 45º and 60º respectively, then the ratio, AB : BC , is
[JEE(Main)2015,(4, – 1), 120]
(1) 3 : 1 (2) 3 : 2 (3) 1 : 3 (4) 2 : 3
8. A man is walking towards a vertical pillar in a straight path, at a uniform speed. At a certain point A on
the path, he observes that the angle of elevation of the top of the pillar is 30°, After walking for 10
minutes from A in the same direction, at a point B, he observes that the angle of elevation of the top of
the pillar is 60°. Then the time taken (in minutes) by him, from B to reach the pillar, is :
[JEE(Main)2016,(4, – 1), 120]
(1) 10 (2) 20 (3) 5 (4) 6

9. If 0 ≤ x < 2π, then the number of real values of x, which satisfy the equation
cosx + cos2x + cos3x + cos4x = 0, is [JEE(Main)2016,(4, – 1), 120]

RI
(1) 5 (2) 7 (3) 9 (4) 3

10. If 5(tan2 x – cos2x) = 2cos2x + 9, then the value of cos4x is : [JEE(Main)2017,(4, – 1), 120]
−3 1 2 7
(4) −

A
(1) (2) (3)
5 3 9 9

11. Let a vertical tower AB have its end A on the level ground. Let C be the mid-point of AB and P be a

UH
point on the ground such that AP = 2AB. If ∠BPC = β, then tanβ is equal to
[JEE(Main)2017,(4, – 1), 120]
6 1 2 4
(1) (2) (3) (4)
7 4 9 9
JA
 π  π  1
12. If sum of all the solutions of the equation 8 cosx.  cos  + x  .cos  − x  −  = 1 in [0, π] is kπ, then k
 6  6  2
is equal to : [JEE(Main)2018,(4, – 1), 120]
8 20 2 13
(1) (2) (3) (4)
9 9 3 9
LP

13. PQR is a triangular park with PQ = PR = 200 m. A T.V. tower stands at the mid-point of QR. If the
angles of elevation of the top of the tower at P, Q and R are respectively 45º, 30º and 30º, then the
height of the tower (in m) is : [JEE(Main)2018,(4, – 1), 120]
(1) 100 3 (2) 50 2 (3) 100 (4) 50
A

 π 3
14. The sum of all values of θ∈  0,  satisfying sin2 2θ + cos4 2θ = is :
 2 4
[JEE(Main) 2019, Online (10-01-19),P-1 (4, – 1), 120]
NK

π 3π 5π
(1) π (2) (3) (4)
2 8 4
π π π π
15. The value of cos 2 . cos 3 . .... . cos 10 . sin 10 is :
2 2 2 2
[JEE(Main) 2019, Online (10-01-19),P-2 (4, – 1), 120]
SA

1 1 1 1
(1) (2) (3) (4)
1024 2 512 256

16. If sin 4 θ + 4 cos 4 β + 2 = 4 2 sin α cos β ; α, β ∈ [0, π] , then cos(α + β) – cos(α – β) is equal to
[JEE(Main) 2019, Online (12-01-19),P-2 (4, – 1), 120]
(1) – 2 (2) 0 (3) 2 (4) –1
PART - I
Section (A) :
π 4π 53π
A-1. (i) (ii) (iii)
12 3 18

A-2. (i) 135° (ii) – 720° (iii) 300° (iv) 210°

− 3 1 1
(i) 
 2 
A-4. (ii) – (iii) – (iv) 1
  2 3

RI
Section (B) :

A
B-1. (i) (ii)

UH
(iii)
JA
1 2

13
B-2. 1 B-3. B-7. a2b2 + 4a2 = 9b2
12
LP

Section (C) :
5 +1 3 −1
C-2. (i) 1 (ii) – 5 /4 (iii) C-10.
8 2 2
A

Section (E) :
1 1
E-1. – , E-2. (i) 2, –1 (ii) 3, 0 E-3. (i) ymax = 11; ymin = 1 (ii) ymax = 10; ymin=–4
4 4
NK

Section (F) :
π π
F-1. (i) nπ + (– 1)n ,n∈Ι (ii) + 1, n ∈ Ι
nπ +
4 3
π π π
(iii) nπ – ,n∈Ι (iv) nπ + (– 1)n , n ∈ Ι (v) nπ ± ,n∈Ι
4 3 4
SA

mπ (2m + 1)π π
F-2. (i) ,m∈Ι or , m∈Ι (ii) 2nπ ± , n∈Ι
4 10 3
 1 π π  1 π
(iii)  2n + 2  , n ∈ Ι or 2nπ – ,n∈Ι (iv) n + 2  , n ∈ Ι
  5 2   9
 1 π 2π π
(v) n + 4  2 , n ∈ Ι (vi) 2nπ + ,n∈Ι (vii) nπ ±
  3 6
 1 π
(viii) n + 3  , n ∈ Ι
  3
nπ  1 2n π π
F-3. (i) , n ∈ Ι or  n ±  π, n ∈ Ι (ii) 2 n π, n ∈ Ι or + , n∈Ι
3  3  3 6
π π π
(iii) x = (2n + 1) , n ∈ Ι or x = (2n + 1) , n ∈ Ι or x = n π ± ,n∈Ι
4 2 6
mπ  1 π
(iv) mπ, m ∈ Ι or , m ∈ Ι or  m +  ,m∈Ι
n −1  2 n

π π π 3π
F-4. (i) nπ + , n ∈ Ι or nπ + ,n∈Ι (ii) nπ + (– 1)n , n ∈ Ι or ;k nπ – (– 1)n ,n∈Ι
3 4 10 10

nπ π π π
(iii) x= − , n∈Ι F-5. (i) nπ + + (– 1)n , n ∈ Ι
3 9 6 4
π 3
(ii) 2nπ + , n ∈ Ι or 2nπ + 2α where α = tan–1 ,n ∈ Ι
2 7

Section (G) :

RI
 π π  7π 11π 
G-1. x ∈ nπ − ,nπ +  : n ∈ I G-2.  2nπ + 6 , 2nπ + 6  G-3. θ ∈ (nπ + π/3, nπ + π/2]
 4 4  

G-4. 15 3 m G-5. ab G-6. 50 m

A
PART - II
Section (A) :

UH
A-1. (A) A-2. (A) A-3. (B) A-4. (C) A-5. (B)

Section (B) :
B-1. (A) B-2. (B) B-3. (A) B-4. (D) B-5. (B) B-6. (B) B-7. (A)
JA
B-8. (A) B-9. (C) B-10. (A) B-11. (B) B-12. (C)

Section (C) :
C-1. (C) C-2. (A) C-3. (C) C-4. (D) C-5. (A) C-6. (B) C-7. (A)
C-8. (B) C-9. (D) C-10. (C)
LP

Section (D) :
D-1. (A) D-2. (B) D-3. (D) D-4. (B) D-5. (A)

Section (E) :
A

E-1. (C) E-2. (C) E-3. (A) E-4. (B) E-5. (A)

Section (F) :
NK

F-1. (D) F-2. (B) F-3. (D) F-4. (B) F-5. (C) F-6. (B) F-7. (A)

F-8. (B) F-9. (A) F-10. (B) F-11. (C) F-12. (B)

Section (G) :
SA

G-1. (A) G-2. (C) G-3. (B) G-4. (A) G-5. (A) G-6. (D) G-7. (D)

G-8. (D) G-9. (B)

PART - III
1. (A) → (s), (B) → (s), (C) → (s), (D) → (r) 2. (A) → (q, s), (B) → (p), (C) → (q), (D) → (p)

3. (A) → (q), (B) → (s), (C) → (r), (D) → (p)

EXERCISE - 2
PART – I
1. (B) 2. (B) 3. (D) 4. (B) 5. (A) 6. (B) 7. (B)
8. (A) 9. (D) 10. (D) 11. (C) 12. (C) 13. (C) 14. (A)
15. (A) 16. (D) 17. (A) 18. (B)

PART - II
1. 4 2. 24 3. 2 4. 12 5. 6 6. 1 7. 8
8. 3 9. 15 10. 5 11. 24 12. 2 13. 1 14. 50
15. 4 16. 2 17. 17 18. 17 19. 6 20. 15 21. 07
22. 17 23. 2 24. 6 25. 0 26. 0 27. 0

RI
PART - III
1. (ABCD) 2. (BC) 3. (BD) 4. (AB) 5. (BC) 6. (ABC) 7. (BC)
8. (ABCD) 9. (BCD) 10. (AC) 11. (CD) 12. (BD) 13. (BC) 14. (BC)

A
15. (BD) 16. (BD) 17. (AD) 18. (AB) 19. (AD) 20. (CD) 21. (AB)
22. (AC) 23. (ABCD) 24. (BD) 25. (ABC) 26. (AD) 27. (BC) 28. (BC)

UH
29. (AC) 30. (ACD) 31. (ABCD) 32. (ABCD) 33. (AC) 34. (CD) 35. (AB)
36. (ABD) 37. (ABCD)

PART - IV
JA
1. (C) 2. (B) 3. (D) 4. (D) 5. (C) 6. (B) 7. (C)
8. (D) 9. (D)

EXERCISE - 3
LP

PART – I
1.* (AB) 2*. (CD) 3. 2 4. 7 5. 3 6. (D)

7.* (ACD) 8. (D) 9. 8 10. (C) 11. (C) 12. Bonus


A

13. (0.5)
PART – II
NK

1. (2) 2. (1) 3. (1) 4. (2) 5. (2) 6. (2) 7. (1)

8. (3) 9. (2) 10. (4) 11. (3) 12. (4) 13. (3) 14. (2)

15. (3) 16. (1)


SA

1. Prove that :
cot 2 θ (sec θ − 1) 1 − sin θ
(i) sec4 A (1 – sin4 A) – 2 tan2 A = 1 (ii) = sec2 θ .
1 + sin θ 1 + sec θ

2. Simplify the expression


sin4 x + 4 cos2 x – cos4 x + 4 sin2 x

3. Let a, b, c, d be numbers in the interval [0, π] such that


sin a + 7 sin b = 4(sin c + 2sin d),
cos a + 7 cos b = 4(cos c + 2cos d)
Prove that 2 cos (a – d ) = 7 cos (b – c ).
4. Prove that (4cos2 9º – 3) (4cos2 27º – 3) = tan 9º

4 5 π
5. If cos (α + β) = ; sin (α − β) = & α, β lie between 0 & , then find the value of tan 2 α.
5 13 4
2ac
6. If α & β are two distinct roots of the equation a tan θ + b sec θ = c, then prove that tan (α + β) = .
a2 − c 2
p
7. If tan α = where α = 6 β, α being an acute angle, prove that;
q
1
(p cosec 2 β − q sec 2 β) = p2 + q2 .
2

8. If sin (θ + α) = a & sin (θ + β) = b (0 < α, β, θ < π/2) then find the value of

RI
cos2 (α − β) − 4 ab cos(α − β)

9. Show that:
1° 1°
(i) cot 7 or tan 82 = (
3 + 2 )( )
2 + 1 or 2 + 3 + 4 + 6

A
2 2

(ii) tan 142 =2+ 2 − 3 − 6.
2

UH
tan α + tan γ sin2 α + sin2 γ
10. If tan β = , prove that sin 2β = .
1 + tan α . tan γ 1+ sin2 α .sin2 γ
a2 + ac + b2
11. If α & β satisfy the equation a cos 2 θ + b sin 2 θ = c then prove that: cos² α + cos² β = .
a2 + b2
JA
12. Show that : 4 sin 27° = (5 + 5 )1/ 2 − (3 − 5 )1/ 2

13. If xy + yz + xz = 1, then prove that


x y z 4xyz
+ + = .
1− x 2
1− y 2
1− z 2
(1 − x )(1 − y 2 )(1 − z2 )
2

π
LP

14. Let a =
7
(a) Show that sin2 3a – sin2 a = sin 2a sin 3a
(b) Show that cosec a = cosec 2a + cosec 4a
(c) Evaluate cos a – cos 2a + cos 3a
(d) Prove that cos a is a root of the equation 8x 3 – 4x2 – 4x + 1 = 0
A

(e) Evaluate tan a tan 2a tan 3a


(f) Evaluate tan2 a + tan2 2a + tan2 3a
(g) Evaluate tan2 a tan2 2a + tan2 2a tan2 3a + tan2 3a tan2 a
NK

(h) Evaluate cot2 a + cot2 2a + cot2 3a


A B C π−A  π−B  π−C
15. In a ∆ABC, prove that sin + sin + sin = 1 + 4 sin   sin   sin  
2 2 2  4   4   4 
16. Evaluate

SA

cos a cos 2a cos 3a..........cos 999a, where a =


1999
17. Prove that the average of the numbers
2 sin 2º, 4 sin 4º, 6 sin 6º, .........180 sin 180º is cot 1º
2
18. Solve tan2θ = tan .
θ
19. Find the general values of x and y satisfying the equations
5 sin x cos y = 1, 4 tan x = tan y
x x
sin3 − cos3
20. Solve 2 2 = cos x .
2 + sin x 3

21. Solve the system of equations :


2π 3  π
x+y= , sin x + sin y = and x, y ∈ 0, 2 
3 2  
22. Solve the following system of simultaneous equations for x and y:
4sinx + 31/cos y = 11
5.16sinx – 2.31/cos y = 2
23. Solve : cos θ + sin θ = cos 2 θ + sin 2 θ .

3 1
24. Solve 8 sin x = +
cos x sin x
25. Solve the equation sin3x cos 3x + cos3x sin 3x + 0.375 = 0.

RI
26. Solve the equation sin x – cos x = cos2x.
2
3
27. Solve the equation sin4x + cos4x – 2 sin2x + sin22x = 0
4

A
28. Solve for x, the equation 13 − 18 tan x = 6 tan x − 3, where − 2 π < x < 2 π .

UH
29. Solve the equation 3 – 2cos θ – 4 sin θ – cos 2θ + sin 2θ = 0

30. Solve the equation sin2 4x + cos2x = 2 sin 4x . cos4x

31. Prove that : cos 5A = 16 cos5 A – 20 cos3 A + 5 cos A


JA
1  1 1  k 1 
32. If cos θ = a +  and cos 3θ =  a + k  then number of natural numbers 'k' less than 50 is
2  a 2  a 
(given a ∈ R)

2 π 
33. (
Consider the equation for 0 ≤ θ ≤ 2 π; sin 2θ + 3 cos 2 θ ) − 5 = cos  − 2 θ  . If greatest value of
LP

6 

θ is (k, p are coprime), then find the value of (k + p).
p
A
NK

56
2. cos2 x – sin2x = cos 2x 5. 8. 1 − 2a2 − 2b2
33
SA

1 1
14. (c) (e) 7 (f) 21 (g) 35 (h) 5 16. 999
2 2

nπ n2 π 2 π π π π π
18. ± 1+ , n ∈Ι 20. x = (4 n + 1) ,n∈Ι 21. x= ,y= or x = , y =
4 16 2 2 6 6 2

π π 2nπ π
22. x = nπ + (– 1)n , y = 2nπ ± 23. 2 n π, n ∈ Ι or + , n∈Ι
6 3 3 6
π nπ π nπ π
24. x = nπ + ,n∈Ι,x= – ,n∈Ι 25. x= + (–1)n+1 . , n ∈ Ι
6 2 12 4 6

π 1
26. x = (2n + 1) π : n ∈ Ι , x = 2nπ ±
3
,n∈Ι 7. x = nπ ±
2
( )
cos–1 2 − 5 , n ∈ Ι

2
28. α − 2 π; α − π, α, α + π, where tan α = 29. θ = (4n + 1) π/2, θ = 2nπ , n ∈ Ι
3

RI
π
30. x = (2 n + 1) , n∈I 32. 25 33. 31
2

A
UH
JA
LP
A
NK
SA
APPLICATION IF DERIVATIVES

JEE (ADVANCED) SYLLABUS

Tangents and normals, increasing and decreasing functions, maximum and minimum values of a function,
Rolle’s theorem and Lagrange’s mean value theorem.

RI
JEE (MAIN) SYLLABUS

Rolle’s and Lagrange’s Mean Value Theorems. Applications of derivatives: Rate of change of quantities,

A
monotonic - increasing and decreasing functions, Maxima and minima of functions of one variable, tangents
and normals.

UH
Tangent and Normal
Let y = f(x) be function with graph as shown in figure. Consider secant PQ. If Q tends to P along the curve
passing through the points Q1, Q2, .....I.e. Q → P, secant PQ will become tangent at P. A line through P
JA
perpendicular to tangent is called normal at P.
dy
LP

Geometrical Meaning of
dx
As Q → P, h → 0 and slope of chord PQ tends to slope of tangent at P (see figure).
f(x + h) − f(x)
Slope of chord PQ =
h
A

f(x + h) − f(x)
lim slope of chord PQ = lim
Q →P h→0 h
dy

NK

slope of tangent at P = f(x) =


dx
Equation of tangent and normal
dy 
= f(x1) denotes the slope of tangent at point (x 1, y1) on the curve y = f(x). Hence the equation
dx  ( x1, y1 )
of tangent at (x1, y1) is given by
SA

(y – y1) = f(x1) (x – x1) ; when, f(x1) is real.

Also, since normal is a line perpendicular to tangent at (x 1 , y1) so its equation is given by
1
(y – y1) = – (x – x1), when f(x1) is nonzero real.

f (x1 )
If f(x1) = 0, then tangent is the line y = y 1 and normal is the line x = x1.
f(x1 + h) − f(x1 )
If lim =  or – , then x = x1 is tangent (VERTICAL TANGENT) and y = y1 is normal.
h→0 h
Example # 1 Find equation of tangent to y = e x at x = 0. Hence draw graph
Solution : At x = 0  y = e0 = 1
dy dy
= ex  =1
dx dx x =0

RI
Hence equation of tangent is
1 (x – 0) = (y – 1)
 y=x+1
1
Example # 2 Find the equation of all straight lines which are tangent to curve y = and which are

A
x −1
parallel to the line x + y = 0.
Solution : Suppose the tangent is at (x 1, y1) and it has slope – 1.

UH
JA
dy
 = – 1.
dx ( x1 , y1 )

1
 – = – 1.
(x1 − 1)2
LP

 x1 = 0 or 2

 y1 = – 1 or 1
A

Hence tangent at (0, – 1) and (2, 1) are the required lines (see figure) with equations
– 1(x – 0) = (y + 1) and – 1 (x – 2) = (y – 1)

NK

x+y+1=0 and y+x=3

Example # 3 Find equation of normal to the curve y = |x 2 – | x | | at x = – 2.


Solution : In the neighborhood of x = – 2, y = x 2 + x.
Hence the point of contact is (– 2, 2)
SA

dy dy
= 2x + 1  = – 3.
dx dx x =−2

So the slope of normal at (– 2, 2) is .


Hence equation of normal is
1
(x + 2) = y – 2  3y = x + 8
3

Example # 4 Prove that sum of intercepts of the tangent at any point to the curve represented by x = 3cos 4
& y = 3sin4 on the coordinate axis is constant.
Solution : Let P(3cos4 , 3sin4) be a variable point on the given curve.
dy
3.4 sin3 .cos 
= d =
dy
 m= = –tan
dx dx −3.4 cos3  sin 
d
 equation of tangent at point P is
y – 3sin4 = –tan2 (x – 3cos4)
x y
 + =1
3cos2  3 sin2 
 sum of x-axis intercept and y-axis intercept = 3cos 2 + 3sin2 = 3 (which is constant)

RI
Self Practice Problems :

(1) Find the slope of the normal to the curve x = 1 – a sin , y = b cos 2  at  =.
2

A
(2) Find the equation of the tangent and normal to the given curves at the given points.
(i) y = x4 – 6x3 + 13x2 – 10x + 5 at (1, 3)
x3

UH
(ii) y2 = at (2, – 2).
4−x
(3) Prove that area of the triangle formed by any tangent to the curve xy = c 2 and coordinate axes is
constant.
(4) A curve is given by the equations x = at2 & y = at3 . A variable pair of perpendicular lines through the
origin 'O' meet the curve at P & Q . Show that the locus of the point of intersection of the tangents at P
JA
& Q is 4y2 = 3ax - a2 .

a
Ans. (1) – (2) (i) Tangent : y = 2x + 1, Normal :x + 2y = 7
2b
(ii) Tangent : 2x + y = 2, Normal :x – 2y = 6
LP

Tangent and Normal from an external point


Given a point P(a, b) which does not lie on the curve y = f(x), then the equation of possible tangents to
the curve y = f(x), passing through (a, b) can be found by solving for the point of contact Q.
f(h) − b
A

f(h) =
h−a
NK

f(h) − b
And equation of tangent is y – b = (x – a)
SA

h−a

Example # 5 Tangent at P(2, 8) on the curve y = x 3 meets the curve again at Q.


Find coordinates of Q.
Solution : Equation of tangent at (2, 8) is y = 12x – 16
Solving this with y = x3
x3 – 12x + 16 = 0
This cubic will give all points of intersection of line and
curve y = x3 i.e., point P and Q. (see figure)
But, since line is tangent at P so x = 2 will be a repeated root of equation x 3 – 12x + 16 = 0 and
another root will be x = h. Using theory of equations :
sum of roots  2+2+h=0  h=–4
Hence coordinates of Q are (– 4, – 64)

Self Practice Problems :

RI
(5) How many tangents are possible from (1, 1) to the curve y – 1 = x 3. Also find the equation of
these tangents.
x+9

A
(6) Find the equation of tangent to the hyperbola y = which passes through (0, 0) origin
x+5

UH
Ans. (5) y = 1, 4y = 27x – 23 (6) x + y = 0; 25y + x = 0

Derivative as rate of change


In various fields of applied mathematics one has the quest to know the rate at which one variable is
changing, with respect to other. The rate of change naturally refers to time. But we can have rate of
JA
change with respect to other variables also.
An economist may want to study how the investment changes with respect to variations in interest
rates.
A physician may want to know, how small changes in dosage can affect the body's response to a drug.
A physicist may want to know the rate of charge of distance with respect to time.
LP

All questions of the above type can be interpreted and represented using derivatives.
Definition : The average rate of change of a function f(x) with respect to x over an interval [a, a + h] is
f(a + h) − f(a)
defined as
h
A

f(a + h) − f(a)
Definition : The instantaneous rate of change of f(x) with respect to x is defined as f(x) = lim
h→0 h
NK

, provided the limit exists.

Note : To use the word 'instantaneous', x may not be representing time. We usually use the word 'rate
of change' to mean 'instantaneous rate of change'.

Example # 6 How fast the area of a circle increases when its radius is 5cm;
SA

(i) with respect to radius (ii) with respect to diameter


dA
Solution : (i) A = r2 , = 2r
dr
dA 
 = 10 cm2/cm.
dr  r = 5

 dA 
(ii) A= D2 , = D
4 dD 2
dA  
 = . 10 = 5 cm2/cm.
dD D = 10 2
Example # 7 If area of circle increases at a rate of 2cm 2/sec, then find the rate at which area of the inscribed
square increases.
Solution : Area of circle, A1 = r2. Area of square, A2 = 2r2 (see figure)

dA1 dr dA 2 dr
= 2r , = 4r .
dt dt dt dt

RI
dr dr 1
 2 = 2r .  r =
dt dt 
dA 2 1 4
 =4. = cm2/sec
 

A
dt
4
 Area of square increases at the rate cm2/sec.

UH
Example # 8 The volume of a cube is increasing at a rate of 7 cm 3/sec. How fast is the surface area
increasing when the length of an edge is 4 cm?
Solution. Let at some time t, the length of edge is x cm.
dv dx dv
v = x3  = 3x2 (but = 7)
dt dt dt
JA
dx 7
 = cm/sec.
dt 3x 2
Now S = 6x2
dS dx dS 7 28
= 12x  = 12x. =
dt dt dt 3x 2 x
dS
LP

when x = 4 cm, = 7 cm2/sec.


dt

Example # 9 Sand is pouring from pipe at the rate of 12 cm 3/s. The falling sand forms a cone on the ground
in such a way that the height of the cone is always one - sixth of radius of base. How fast is the
A

height of the sand cone increasing when height is 4 cm?


1 2
Solution. V= r h
3
NK

r
but h=
6
1
 V=  (6h)2 h
3
SA

 V = 12 h3
dV dh
= 36 h2.
dt dt
dV
when, = 12 cm3/s and h = 4 cm
dt
dh 12 1
= = cm/sec.
dt 36.(4) 2
48
Self Practice Problems :

(7) Radius of a circle is increasing at rate of 3 cm/sec. Find the rate at which the area of circle is
increasing at the instant when radius is 10 cm.
(8) A ladder of length 5 m is leaning against a wall. The bottom of ladder is being pulled along the
ground away from wall at rate of 2cm/sec. How fast is the top part of ladder sliding on the wall
when foot of ladder is 4 m away from wall.

(9) Water is dripping out of a conical funnel of semi-vertical angle 45° at rate of 2cm 3/s. Find the
rate at which slant height of water is decreasing when the height of water is 2 cm.

RI
(10) A hot air balloon rising straight up from a level field is tracked by a range finder 500 ft from the
lift-off point. At the moment the range finder's elevation angle is /4, the angle is increasing at
the rate of 0.14 rad/min. How fast is the balloon rising at that moment.

A
8 1
Ans. (7) 60 cm2/sec (8) cm/sec (9) cm/sec. (10) 140 ft/min.
3 2
Error and Approximation :

UH
Let y = f(x) be a function. If these is an error x in x then corresponding error in y is y = f(x + x) – f(x).
f(x + x) − f(x) dy
We have lim = = f(x)
x →0 x dx
We define the differential of y, at point x, corresponding to the increment x as f(x) x and denote it by
JA
dy.
i.e. dy = f(x) x.
Let P(x, f(x)), Q((x + x), f(x + x)) (as shown in figure)
y = QS,
x = PS,
dy = RS
In many practical situations, it is easier to evaluate dy but not y.
LP
A

Example # 10. Find the approximate value of 25 1/3.


Sol. Let y = x1/3
Let x = 27 and x = –2
NK

Now y = (x + x)1/3 – x1/3 = (25)1/3 – 3


dy
x = 251/3 – 3
dx
At x = 27, 251/3 = 3 – 0.074 = 2.926
SA

Monotonicity of a function :
Let f be a real valued function having domain D(DR) and S be a subset of D. f is said to be
monotonically increasing (non decreasing) (increasing) in S if for every x 1, x2  S, x1 < x2  f(x1)  f(x2). f
is said to be monotonically decreasing (non increasing) (decreasing) in S if for every x 1, x2  S, x1 < x2
 f(x1)  f(x2)

f is said to be strictly increasing in S if for x 1, x2  S, x1 < x2  f(x1) < f(x2). Similarly, f is said to be strictly
decreasing in S if for x1, x2  S, x1 < x2  f(x1) > f(x2) .
Notes : (i) f is strictly increasing  f is monotonically increasing (non decreasing). But converse need not
be true.
(ii) f is strictly decreasing  f is monotonically decreasing (non increasing). Again, converse need
not be true.
(iii) If f(x) = constant in S, then f is increasing as well as decreasing in S
(iv) A function f is said to be an increasing function if it is increasing in the domain. Similarly, if f is
decreasing in the domain, we say that f is monotonically decreasing
(v) f is said to be a monotonic function if either it is monotonically increasing or monotonically
decreasing
(vi) If f is increasing in a subset of S and decreasing in another subset of S, then f is non monotonic
in S.

RI
Application of differentiation for detecting monotonicity :
Let  be an interval (open or closed or semi open and semi closed)
(i) If f(x) > 0  x  , then f is strictly increasing in 
If f(x) < 0  x  , then f is strictly decreasing in 

A
(ii)

Note : Let I be an interval (or ray) which is a subset of domain of f. If f (x) > 0,  x   except for countably
many points where f (x) = 0, then f(x) is strictly increasing in .

UH
{f (x) = 0 at countably many points  f (x) = 0 does not occur on an interval which is a subset of  }
Let us consider another function whose graph is shown below for x  (a, b).
JA
Here also f(x)  0 for all x  (a, b). But, note that in this case, f(x) = 0 holds for all x  (c, d) and (e,b).
LP

Thus the given function is increasing (monotonically increasing) in (a, b), but not strictly increasing.

Example # 11 : Let f(x) = x3. Find the intervals of monotonicity.


A

Solution : f(x) = 3x2


f(x) > 0 everywhere except at x = 0. Hence f(x) will be strictly increasing function for
x  R {see figure}
NK
SA

Example # 12 : Let f(x) = x – sinx. Find the intervals of monotonicity.

Solution : f(x) = 1 – cosx


Now, f(x) > 0 every where, except at x = 0, ± 2, ± 4 etc. But all these points are discrete
(countable) and do not form an interval. Hence we can conclude that f(x) is strictly increasing in
R. In fact we can also see it graphically.
RI
Example # 13 : Find the intervals in which f(x) = x 3 – 2x2 – 4x + 7 is increasing.

A
Solution : f(x) = x3 – 2x2 – 4x + 7
f(x) = 3x2 – 4x – 4
f(x) = (x – 2) (3x + 2)

UH
 2
for M.. f(x)  0  x   −, −   [2, )
 3

Example # 14 : Find the intervals of monotonicity of the following functions.


(i) f(x) = x2 (x – 2)2 (ii) f(x) = x n x
JA
Solution : (i) f(x) = x2 (x – 2)2  f(x) = 4x (x – 1) (x – 2)
observing the sign change of f(x)

Hence increasing in [0, 1] and in [2, )


and decreasing for x  (– , 0] and [1, 2]
LP

(ii) f(x) = x n x
f(x) = 1 + n x
1
f(x)  0  n x  – 1  x
e
A

1   1
 increasing for x   ,   and decreasing for x   0 ,  .
e   e
NK

Note : If a function f(x) is increasing in (a, b) and f(x) is continuous in [a, b], then f(x) is increasing on
[a, b]

Example # 15 : f(x) = [x] is a step up function. Is it a strictly increasing function for x  R.


Solution : No, f(x) = [x] is increasing (monotonically increasing) (non-decreasing), but not strictly
SA

increasing function as illustrated by its graph.

Example # 16 : If f(x) = sin4x + cos4x + bx + c, then find possible values of b and c such that f(x) is monotonic
for all x  R
Solution : f(x) = sin4x + cos4x + bx + c
f(x) = 4 sin3x cosx – 4cos3x sinx + b = – sin4x + b.
Case - (i) : for M.I. f(x)  0 for all x  R
 b  sin4x for all x  R  b1
Case - (ii) : for M.D. f(x)  0for all x  R
 b  sin4x for all x  R  b–1
Hence for f(x) to be monotonic b  (– , – 1]  [1, ) and c  R.

Example # 17 : Find possible values of 'a' such that f(x) = e 2x – 2(a2 – 21) ex + 8x + 5 is monotonically
increasing for x  R

Solution : f(x) = e2x – 2(a2 – 21) ex + 8x + 5

RI
f(x) = 2e2x – 2(a2 – 21) ex + 8  0 ;  c x  R
4
 ex +  a2 – 21
ex

A
 4 
4  a2 – 21  ex +  4
 
x
e
 a  [–5, 5]

UH
Self Practice Problems :

(11) Find the intervals of monotonicity of the following functions.


(i) f(x) = – x3 + 6x2 – 9x – 2
1
JA
(ii) f(x) = x +
x +1
f(x) = x . ex − x
2
(iii)
(iv) f(x) = x – cosx

(12) Let f(x) = x – tan–1x. Prove that f(x) is monotonically increasing for x  R.
LP

(13) If f(x) = 2ex – ae–x + (2a + 1) x – 3 monotonically increases for  x  R, then find range of
values of a
A

(14) Let f(x) = e2x – aex + 1. Prove that f(x) cannot be monotonically decreasing for  x  R for any
value of 'a'.
NK

(15) The values of 'a' for which function f(x) = (a + 2) x 3 – ax2 + 9ax – 1 monotonically decreasing
for  x  R.

Ans. (11) (i)  in [1, 3] ; D in (– , 1]  (3, )


SA

(ii)  in (– , – 2]  [0, ) ; D in [– 2, – 1)  (–1, 0]


 1   1
(iii)  in  − , 1 ; D in  −, −   [1, )
 2   2
(iv) I for x  R

(13) a0 (15) – < a  – 3


Monotonicity of function about a point :
1. A function f(x) is called as a strictly increasing function about a point (or at a point) a  D f if it is strictly
increasing in an open interval containing a (as shown in figure).

RI
2. A function f(x) is called a strictly decreasing function about a point x = a, if it is strictly decreasing in an

A
open interval containing a (as shown in figure).

UH
JA
Note : If x = a is a boundary point then use the appropriate one sided inequality to test monotonicity of f(x).
LP
A
NK
SA

e.g. : Which of the following functions (as shown in figure) is increasing, decreasing or neither increasing nor
decreasing at x = a.

(i) (ii)
(iii) (iv)

Test for increasing and decreasing functions about a point


Let f(x) be differentiable.
(1) If f(a) > 0 then f(x) is increasing at x = a.

RI
(2) If f(a) < 0 then f(x) is decreasing at x = a.
(3) If f(a) = 0 then examine the sign of f(x) on the left neighbourhood and the right neighbourhood
of a.

A
(i) If f(x) is positive on both the neighbourhoods, then f is increasing at x = a.
(ii) If f(x) is negative on both the neighbourhoods, then f is decreasing at x = a.

UH
(iii) If f(x) have opposite signs on these neighbourhoods, then f is non-monotonic at x = a.

Example # 18 : Let f(x) = x3 – 3x + 2. Examine the monotonicity of function at points x = 0, 1, 2.


Solution : f(x) = x3 – 3x + 2
JA
f(x) = 3(x2 – 1)
(i) f(0) = – 3  decreasing at x = 0
(ii) f(1) = 0
also, f(x) is positive on left neighbourhood and f(x) is negative in right neighbourhood.
 neither increasing nor decreasing at x = 1.
LP

(iii) f(2) = 9  increasing at x = 2

Note : Above method is applicable only for functions those are continuous at x = a.
A

Self Practice Problems :

(16) For each of the following graph comment on monotonicity of f(x) at x = a.


NK

(i) (ii) (iii)


SA

(17) Let f(x) = x3 – 3x2 + 3x + 4, comment on the monotonic behaviour of f(x) at (i) x = 0 (ii) x = 1.
 x 0  x 1
(18) Draw the graph of function f(x) =  . Graphically comment on the monotonic
[x] 1  x  2
behaviour of f(x) at x = 1. Is f(x) M.. for x  [0, 2] ?
Ans. (16) (i) neither M.. nor M.D. (ii) M.D. (iii) M.
(17) M.. both at x = 0 and x = 1.
(18) M.. at x = 1; f(x) is M.. for x  [0, 2].
Global Maximum :
A function f(x) is said to have global maximum on a set E if there exists at least one c  E such that
f(x)  f(c) for all x  E.

We say global maximum occurs at x = c and global maximum (or global maximum value) is f(c).

Local Maxima :
A function f(x) is said to have a local maximum at x = c if f(c) is the greatest value of the function in a

RI
small neighbourhood (c – h, c + h), h > 0 of c.
i.e. for all x  (c – h, c + h), x  c, we have f(x)  f(c).

Global Minimum :

A
A function f(x) is said to have a global minimum on a set E if there exists at least one c  E such that
f(x)  f(c) for all x  E.

UH
Local Minima :

A function f(x) is said to have a local minimum at x = c if f(c) is the least value of the function in a small
neighbourhood (c – h, c + h), h > 0 of c.
JA
i.e. for all x  (c –h, c + h), x  c, we have f(x)  f(c).

Extrema :

A maxima or a minima is called an extrema.


LP

Explanation : Consider graph of y = f(x), x  [a, b]


A
NK

x = c2, x = c4 are points of local maxima, with maximum values f(c 2), f(c4) respectively.

x = c1, x = c3 are points of local minima, with minimum values f(c 1), f(c3) respectively
SA

x = c2 is a point of global maximum

x = c3 is a point of global minimum

Consider the graph of y = h(x), x  [a, b)


h(c4)

h(c1)
h(c3)
h(c2)
h(a)

a c1 c2 c3 c4 b

x = c1, x = c4 are points of local maxima, with maximum values h(c1), h(c4) respectively.

RI
x = c2 are points of local minima, with minimum values h(c 2) respectively.
x = c3 is neither a point of maxima nor a minima.
Global maximum is h(c4)

A
Global minimum is h(a)

| x | 0  | x |  2
Example # 19 : Let f(x) =  . Examine the behaviour of f(x) at x = 0.

UH
 1 x=0
Solution : f(x) has local maxima at x = 0 (see figure).
JA
 3 (b3 − b2 + b − 1)
 –x + 0  x 1
Example # 20 : Let f(x) =  (b2 + 3b + 2)
 2x − 3 1 x  3

Find all possible values of b such that f(x) has the smallest value at x = 1.
LP

Solution. Such problems can easily be solved by graphical approach (as in figure).
A

Hence the limiting value of f(x) from left of x = 1 should be either greater or equal to the value of
NK

function at x = 1.
lim− f(x)  f(1)
x →1

(b3 − b2 + b − 1)
 –1+ –1
(b2 + 3b + 2)
SA

(b2 + 1)(b − 1)
 0
(b + 1) (b + 2)
 b  (– 2, –1)  [1, + )

Self Practice Problems :


(19) In each of following graphs identify if x = a is point of local maxima, minima or neither

(i) (ii) (iii)


(20) Examine the graph of following functions in each case identify the points of global
maximum/minimum and local maximum / minimum.

(i) (ii) (iii)

Ans. (19) (i) Maxima (ii) Neither maxima nor minima


(iii) Minima

RI
(20) (i) Local maxima at x = 2, Local minima at x = 3, Global maximum at x = 2. No
global minimum
(ii) Local minima at x = – 1, No point of Global minimum, no point of local or
Global maxima
(iii) Local & Global maximum at x = 1, Local & Global minimum at x = 0.

A
Maxima, Minima for differentiable functions :

UH
Mere definition of maxima, minima becomes tedious in solving problems. We use derivative as a tool to
overcome this difficulty.

A necessary condition for an extrema :


JA
Let f(x) be differentiable at x = c.

Theorem : A necessary condition for f(c) to be an extremum of f(x) is that f(c) = 0.

i.e. f(c) is extremum  f(c) = 0

Note : f(c) = 0 is only a necessary condition but not sufficient


LP

i.e. f(c) = 0 f(c) is extremum.


A

Consider f(x) = x3
NK

f(0) = 0
but f(0) is not an extremum (see figure).

Sufficient condition for an extrema :


SA

Let f(x) be a differentiable function.

Theorem : A sufficient condition for f(c) to be an extremum of f(x) is that f(x) changes sign as x passes through c.

i.e. f(c) is an extrema (see figure)  f(x) changes sign as x passes through c.
x = c is a point of maxima. f(x) changes sign from positive to negative.

RI
A
UH
x = c is a point of local minima (see figure), f(x) changes sign from negative to positive.
Stationary points :
The points on graph of function f(x) where f(x) = 0 are called stationary points.
JA
Rate of change of f(x) is zero at a stationary point.
Example # 21 : Find stationary points of the function f(x) = 4x 3 – 6x2 – 24x + 9.
Solution : f(x) = 12x2 – 12x – 24
f(x) = 0  x = – 1, 2
LP

f(– 1) = 23, f(2) = – 31


(– 1, 23), (2, – 31) are stationary points

Example # 22 : If f(x) = x3 + ax2 + bx + c has extreme values at x = – 1 and x = 3. Find a, b, c.


A

Solution. Extreme values basically mean maximum or minimum values, since f(x) is differentiable
function so
NK

f(– 1) = 0 = f(3)
f(x) = 3x2 + 2ax + b
f(3) = 27 + 6a + b = 0
f(– 1) = 3 – 2a + b = 0
SA

 a = – 3, b = – 9, c  R

First Derivative Test :

Let f(x) be continuous and differentiable function.


Step -  Find f(x)
Step - . Solve f(x) = 0, let x = c be a solution. (i.e. Find stationary points)
Step - . Observe change of sign
(i) If f(x) changes sign from negative to positive as x crosses c from left to right then x = c is a
point of local minima
(ii) If f(x) changes sign from positive to negative as x crosses c from left to right then x = c is a
point of local maxima.
(iii) If f(x) does not changes sign as x crosses c then x = c is neither a point of maxima nor minima.

Example # 23 : Find the points of maxima or minima of f(x) = x 2 (x – 2)2.

Solution. f(x) = x2 (x – 2)2


f(x) = 4x (x – 1) (x – 2)

RI
f(x) = 0  x = 0, 1, 2
examining the sign change of f(x)

A
UH
Hence x = 1 is point of maxima, x = 0, 2 are points of minima.

Note : In case of continuous functions points of maxima and minima are alternate.
JA
Example # 24 : Find the points of maxima, minima of f(x) = x 3 – 12x. Also draw the graph of this functions.

Solution. f(x) = x3 – 12x


f(x) = 3(x2 – 4) = 3(x – 2) (x + 2)
f(x) = 0  x=±2
LP

For tracing the graph let us find maximum and minimum values of f(x).
A
NK

x f(x)
2 −16
−2 +16
SA

Example # 25 : Show that f(x) = (x3 – 6x2 + 12x – 8) does not have any point of local maxima or minima. Hence
draw graph
Solution. f(x) = x3 – 6x2 + 12x – 8
f(x) = 3(x2 – 4x + 4)
f(x) = 3(x – 2)2
f(x) = 0  x=2
but clearly f(x) does not change sign about x = 2. f(2+) > 0 and f(2–) > 0. So f(x) has no point
of maxima or minima. In fact f(x) is a monotonically increasing function for x  R.
Example # 26 : Let f(x) = x3 + 3(a – 7)x2 + 3(a2 – 9) x – 1. If f(x) has positive point of maxima, then find possible
values of 'a'.
Solution. f(x) = 3 [x2 + 2(a – 7)x + (a2 – 9)]
Let ,  be roots of f(x) = 0 and let  be the smaller root. Examining sign change of f(x).

RI
Maxima occurs at smaller root  which has to be positive. This basically implies that both roots

A
of f(x) = 0 must be positive and distinct.

29

UH
(i) D>0  a<
7
b
(ii) – >0  a<7
2a
(iii) f(0) > 0  a  (– , – 3)  (3, )
JA
 29 
from (i), (ii) and (iii)  a  (– , – 3)   3,
 7 
Self Practice Problems :
(21) Find the points of local maxima or minima of following functions
(i) f(x) = (x – 1)3 (x + 2)2
(ii) f(x) = x3 + x2 + x + 1.
LP

4
Ans. (i) Maxima at x = – 2, Minima at x = –
5
(ii) No point of local maxima or minima.
A

Maxima, Minima for continuous functions :


Let f(x) be a continuous function.
NK

Critical points :

The points where f(x) = 0 or f(x) is not differentiable are called critical points.

Stationary points  Critical points.


SA

Example # 27 : Find critical points of f(x) = max (sinx, cosx) , x  (0, 2).

Solution :

  5
From the figure it is clear that f(x) has three critical points x = , , .
4 2 4

Important Note :
For f(x) defined on a subset of R, points of extrema (if exists) occur at critical points
Example # 28 : Find the possible points of Maxima/Minima for f(x) = |x 2 – 2x| (x  R)
 x 2 − 2x x  2

Solution. f(x) = 2x − x 2 0  x  2
 x 2 − 2x x  0

2(x − 1) x  2

f(x) = 2(1 − x) 0  x  2
2(x − 1) x  0

f(x) = 0 at x = 1 and f(x) does not exist at x = 0, 2. Thus these are critical points.

RI
 x 3 + x 2 − 10x x  0
Example # 29 : Let f(x) =  . Examine the behaviour of f(x) at x = 0.
 3 sin x x0
Solution : f(x) is continuous at x = 0.
3x 2 + 2x − 10 x  0

A
f(x) = 
 3cos x x0
f(0+) = 3 and f(0–) = – 10 thus f(x) is non-differentiable at x = 0  x = 0 is a critical

UH
point.
Also derivative changes sign from negative to positive, so x = 0 is a point of local minima.

Example # 30 : Find the critical points of the function f(x) = 4x 3 – 6x2 – 24x + 9 if (i) x  [0, 3] (ii) x  [–3, 3]
(iii) x  [– 1, 2].
JA
Solution : f(x) = 12(x2 – x – 2)
= 12(x – 2) (x + 1)
f(x) = 0  x = – 1 or 2
(i) if x  [0, 3] , x = 2 is critical point.
(ii) if x  [– 3, 3], then we have two critical points x = – 1, 2.
(iii) If x  [– 1, 2], then no critical point as both x = –1 and x = 2 become boundary points.
LP

Note : Critical points are always interior points of an interval.

Global extrema for continuous functions :


(i) Function defined on closed interval
A

Let f(x), x  [a, b] be a continuous function


Step - I : Find critical points. Let it be c 1, c2 ......., cn
NK

Step - II : Find f(a), f(c1).........., f(cn), f(b)


Let M = max· { f(a), f(c1),..........., f(cn), f(b)}
m = min · {f(a), f(c1), ........f(cn), f(b)}
Step -   M is global maximum.
SA

m is global minimum.
(ii) Function defined on open interval.
Let f(x), x  (a, b) be continuous function.
Step - I Find critical points . Let it be c 1, c2, .......cn
Step - II Find f(c1), f(c2), ........., f(cn)
Let M = max · {f(c1), .......f(c n)}
m = min· {f(c1),............,f(cn)}
Step - III Lim f(x) = 1 (say), Lim– f(x) = 2 (say).
x → a+ x → b

Let  = min. {1, 2}, L = max. {1, 2}


Step - IV
(i) If m   then m is global minimum
(ii) If m >  then f(x) has no global minimum

(iii) If M  L then M is global maximum


(iv) If M < L , then f(x) has no global maximum

Example # 31 : Find the greatest and least values of f(x) = x 3 – 12x x  [– 1, 3]


Solution : The possible points of maxima/minima are critical points and the boundary points.
for x  [– 1, 3] and f(x) = x 3 – 12x
x = 2 is the only critical point.
Examining the value of f(x) at points x = –1, 2, 3. We can find greatest and least values.

RI
x f(x)
−1 11
2 −16
3 −9

A
 Minimum f(x) = – 16 & Maximum f(x) = 11.

UH
Self Practice Problems :
(22) Let f(x) = x3 + x2 – x – 4
(i) Find the possible points of Maxima/Minima of f(x) for x  R.
(ii) Find the number of critical points of f(x) for x  [1, 3].
JA
(iii) Discuss absolute (global) maxima/minima value of f(x) for x  [–2, 2]
(iv) Prove that for x  (1, 3), the function does not has a Global maximum.
1
Ans. (i) x = –1, (ii) zero
3
(iii) f(–2) = –6 is global maximum, f(2) = 6 is global maximum
LP


 x2 + x ; −1  x  0

Example # 32 : Let f(x) =   ; x=0

log1/ 2  x + 1  ; 0  x  3
A

  2 2
Discuss global maxima, minima for  = 0 and  = 1. For what values of  does f(x) has global
NK

maxima
Solution : Graph of y = f(x) for  = 0

1
SA

–1/2 3/2
–1

–1
No global maxima, minima
Graph of y = f(x) for  = 1
1

–1/2 3/2
–1

–1

Global maxima is 1, which occurs at x = 0

RI
Global minima does not exists

Lim− f(x) = 0, Lim+ f(x) = 1, f(0) = 


x →0 x →0

A
For global maxima to exists
f(0)  1    1.

UH
x2 + 4 1
Example # 33 : Find extrema of f(x) = . Draw graph of g(x) = and comment on its local and
2x + 3 f(x)
global extrema.
2(x 2 + 3x − 4) 2(x + 4)(x − 1)
JA
Solution : f(x) = = =0
(2x + 3)2 (2x + 3)2

1
3

–4 4
3
LP


2
1
1

4
local minima occurs at x = –4
A

local maxima occurs at x = 1

1  2x + 3 
NK

f(x)  x 2 + 4 
g(x) = =

−2(x + 4)(x − 1)
g'(x) =
(x 2 + 4)2
local maxima at x = –4
SA

local minima at x = 1
global maxima & minima do not exists

–4 –3/2 1
–4
Self Practice Problems :
1
(23) Let f(x) = x + . Find local maximum and local minimum value of f(x). Can you explain this
x
discrepancy of locally minimum value being greater than locally maximum value.
(x +  )2 x0
(24) If f(x) =  , find possible values of  such that f(x) has local maxima at x = 0.
 cos x x0

Answers : (23) Local maxima at x = –1, f(–1) = – 2 ; Local minima at x = 1, f(1) = 2.


(24)   [ –1, 1)

RI
Maxima, Minima by higher order derivatives :
Second derivative test :

A
Let f(x) have derivatives up to second order
Step - I. Find f(x)
Step - II. Solve f(x) = 0. Let x = c be a solution

UH
Step - III. Find f(c)
Step - IV.
(i) If f(c) = 0 then further investigation is required
(ii) If f(c) > 0 then x = c is a point of minima.
(iii) If f(c) < 0 then x = c is a point of maxima.
JA
LP

For maxima f(x) changes from positive to negative (as shown in figure).
 f(x) is decreasing hence f(c) < 0

Example # 34 : Find the points of local maxima or minima for f(x) = sin2x – x, x  (0, ).
A

Solution : f(x) = sin2x – x


f(x) = 2cos2x – 1
 5
NK

1
f(x) = 0  cos 2x =  x= ,
2 6 6
f(x) = – 4 sin 2x
 
f   < 0  Maxima at x =
6 6
SA

 5  5
f   >0  Minima at x =
 6  6

Self Practice Problems :


(25) Let f(x) = sinx (1 + cosx) ; x  (0, 2). Find the number of critical points of f(x). Also identify
which of these critical points are points of Maxima/Minima.
Ans. Three

x= is point of maxima.
3
x =  is not a point of extrema.
5
x= is point of minima.
3
nth Derivative test :
Let f(x) have derivatives up to nth order
If f(c) = f(c) = ..........= fn–1(c) = 0 and
fn(c)  0 then we have following possibilities
(i) n is even, f(n)(c) < 0  x = c is point of maxima
(ii) n is even, f(n)(c) > 0  x = c is point of minima.
(iii) n is odd, f(n)(c) < 0  f(x) is decreasing about x = c
(iv) n is odd, f(n) > 0  f(x) is increasing about x = c.

RI
Example # 35 : Find points of local maxima or minima of f(x) = x 5 – 5x4 + 5x3 – 1

Solution. f(x) = x5 – 5x4 + 5x3 – 1

A
f(x) = 5x2 (x – 1) (x – 3)
f(x) = 0  x = 0, 1, 3
f(x) = 10x (2x – 6x + 3)
2

UH
Now, f(1) < 0  Maxima at x = 1
f(3) > 0  Minima at x = 3
and, f(0) = 0  nd derivative test fails
so, f(x) = 30 (2x2 – 4x + 1)
JA
f(0) = 30
 Neither maxima nor minima at x = 0.
Note : It was very convenient to check maxima/minima at first step by examining the sign change of
f(x) no sign change of f(x) at x = 0
f(x) = 5x2 (x – 1) (x – 3)
LP

Application of Maxima, Minima :


A

For a given problem, an objective function can be constructed in terms of one parameter and then
NK

extremum value can be evaluated by equating the differential to zero. As discussed in n th derivative test
maxima/minima can be identified.

Useful Formulae of Mensuration to Remember :


1 2
SA

Area of a circular sector = r , when  is in radians.


2
volume of cube = 3 , Total Surface area of cube = 6 2
volume of cuboid = bh, Total Surface area of cube = 2( b + bh + h)

3-D Figures Volume Total Surface area Curved/lateral Surface area


Cone 1 2 r + r2 Curved Surface area = r
r h
3
Cylinder r2h 2rh + 2r2 Curved Surface area =2rh
Sphere 4 3 4r2
r
3
Prism (area of base) × (height) lateral surface area lateral Surface area =
+ 2 (area of base) (perimeter of base) × (height)
Right 1 Curved surface area Curved Surface area =
×(area of base) × (height)
Pyramid 3 + (area of base) 
×(perimeter of base) ×(slant height)
2

(Note that lateral surfaces of a prism are all rectangle).


(Note that slant surfaces of a pyramid are triangles).

Example # 36: If the equation x3 + px + q = 0 has three real roots, then show that 4p 3 + 27q2 < 0.

RI
Solution: f(x) = x3 + px + q, f(x) = 3x2 + p
 f(x) must have one maximum > 0 and one minimum < 0. f(x) = 0
−p
 x=± , p0

A
3
−p −p
f is maximum at x = – and minimum at x =
3 3

UH
 −p   −p 
f–
  f    0
 3   3 
 2p −p   2p −p 
 q −   q + 0
 3 3   3 3 
JA
4p3
q2 + < 0, 4p3 + 27q2 < 0.
27

Example # 37 : Find two positive numbers x and y such that x + y = 60 and xy 3 is maximum.
Solution : x + y = 60
LP

 x = 60 – y  xy3 = (60 – y)y3


Let f(y) = (60 – y) y 3
; y  (0, 60)
for maximizing f(y) let us find critical points
f(y) = 3y2 (60 – y) – y3 = 0
A

f(y) = y2 (180 – 4y) = 0


 y = 45
f(45+) < 0 and f(45–) > 0. Hence local maxima at y = 45.
NK

So x = 15 and y = 45.
Example # 38 : Rectangles are inscribed inside a semicircle of radius r. Find the rectangle with maximum area.
Solution : Let sides of rectangle be x and y (as shown in figure).
 A = xy.
Here x and y are not independent variables and are related by Pythogorus theorem with r.
SA

x2 x2
+ y2 = r2  y= r2 −
4 4
x2
 A(x) = x r2 −
4
x4
 A(x) = x 2r 2 −
4
x4
Let f(x) = r2x2 – ; x  (0, r)
4
A(x) is maximum when f(x) is maximum
Hence f(x) = x(2r2 – x2) = 0  x=r 2
also f(r 2+ ) < 0 and f(r 2− ) > 0
r
confirming at f(x) is maximum when x = r 2 & y = .

RI
2

Aliter Let us choose coordinate system with origin as centre of circle (as shown in figure).

A
UH
A = xy
JA
 
 A = 2 (rcos) (rsin)  A = r2 sin2    0, 
 2
 r
Clearly A is maximum when  =  x=r 2 and y= .
4 2
LP

Example # 39. Show that the least perimeter of an isosceles triangle circumscribed about a circle of radius ‘r’
is 6 3 r .
Solution : AQ = r cot  = AP
AO = r cosec
A

A
(
NK

Q P

r r
x O x

B x C
x N
SA

x
= tan
AO + ON
x = (r cosec + r) tan
x = r(sec + tan)

Perimeter = p = 4x + 2AQ
p = 4r(sec + tan) + 2rcot
p = r(4sec + 4tan + 2cot)
dp
= r[4sec tan + 4sec2  – 2cosec2]
d
dp
for max or min =0  2sin3  + 3sin2 – 1 = 0
d
 (sin + 1) (2sin2 + sin – 1) = 0
(sin + 1)2 (2sin – 1) = 0  sin = 1/2   = 30° = /6

 4.2 4
pleast = r 
 8 + 4 + 6 
+ 2 3 = r 
(
6 3 3 )
+  = r=6 3 r
 3 3   3  3

Example # 40 : Let A(1, 2) and B(– 2, – 4) be two fixed points. A variable point P is chosen on the straight line
y = x such that perimeter of PAB is minimum. Find coordinates of P.

RI
Solution. Since distance AB is fixed so for minimizing the perimeter of PAB, we basically have to
minimize (PA + PB)
Let A be the mirror image of A in the line y = x (see figure).
F(P) = PA + PB

A
F(P) = PA + PB
But for PAB

UH
JA
PA + PB  AB and equality hold when P, A and B becomes collinear. Thus for minimum path
length point P is that special point for which PA and PB become incident and reflected rays
with respect to the mirror y = x.
Equation of line joining A and B is y = 2x intersection of this line with y = x is the point P.
Hence P  (0, 0).
LP
A
NK

Note : Above concept is very useful because such problems become very lengthy by making
perimeter as a function of position of P and then minimizing it.
Self Practice Problems :
SA

(26) Find the two positive numbers x and y whose sum is 35 and the product x 2 y5 maximum.
(27) A square piece of tin of side 18 cm is to be made into a box without top by cutting a square
from each corner and folding up the slops to form a box. What should be the side of the square
to be cut off such that volume of the box is maximum possible.
(28) Prove that a right circular cylinder of given surface area and maximum volume is such that the
height is equal to the diameter of the base.
x2 y2
(29) A normal is drawn to the ellipse + = 1. Find the maximum distance of this normal from
25 16
the centre.
(30) A line is drawn passing through point P(1, 2) to cut positive coordinate axes at A and B. Find
minimum area of PAB.
(31) Two towns A and B are situated on the same side of a straight road at distances a and b
respectively perpendiculars drawn from A and B meet the road at point C and D respectively.
The distance between C and D is c. A hospital is to be built at a point P on the road such that
the distance APB is minimum. Find position of P.
Ans. (26) x = 25, y = 10. (27) 3 cm (29) 1 unit
ac
(30) 4 units (31) P is at distance of from C.
a+b

Use of monotonicity for proving inequalities


Comparison of two functions f(x) and g(x) can be done by analysing the monotonic behaviour of

RI
h(x) = f(x) – g(x)

 
Example # 41 : For x   0,  prove that sin2 x < x2 < tan2 x
 2

A
Solution : Let f(x) = x2 – sin2x
= (x + sinx) (x – sinx) > 0

UH
 
for x   0,   sin2 x < x2 .....(i)
 2
Let g(x) = x2 – tan2x
= (x + tanx) (x + tanx) < 0
JA
 
for x   0,   x2 < tan2 x .....(ii)
 2
From (i) & (ii) sin2 x < x2 < tan2 x
x3 x3  tan−1 x 
Example # 42 : For x  (0, 1) prove that x – < tan–1 x < x – hence or otherwise find lim  
x →0
3 6  x 
LP

x3 1 x4
Solution : Let f(x) = x – – tan–1x f(x) = 1 – x2 – f(x) = –
3 1 + x2 1 + x2
f(x) < 0 for x  (0, 1)  f(x) is M.D.
x3
 f(x) < f(0)  x– – tan–1x < 0
A

3
x3
 x– < tan–1x ...........(i)
3
NK

x3 x2 1 x 2 (1 − x 2 )
Similarly g(x) = x – – tan–1x , g(x) = 1 – – g(x) =
6 2 1 + x2 2(1 + x 2 )
g(x) > 0 for x  (0, 1)  g(x) is M.I.
3
x
 g(x) > g(0) x– – tan–1x > 0
6
SA

x3
x– > tan–1x ........(ii)
6

x3 x3
from (i) and (ii), we get x– < tan–1x < x – Hence Proved
3 6
x2 tan−1 x x2
Also, 1– < <1– , for x > 0
3 x 6
tan−1 x
Hence by sandwich theorem we can prove that lim = 1 but it must also be noted that as x → 0,
x →0 x
tan−1 x tan−1 x
value of ⎯→ 1 from left hand side i.e. <1
x x
 tan−1 x 
 lim   =0
x →0
 x 
NOTE : In proving inequalities, we must always check when does the equality takes place because the
point of equality is very important in this method. Normally point of equality occur at end point of
the interval or will be easily predicted by hit and trial.

RI
  x3
Example # 43 : For x   0,  , prove that sin x > x –
 2 6

x3

A
Solution : Let f(x) = sin x – x +
6

x2

UH
f(x) = cos x – 1 +
2

we cannot decide at this point whether f(x) is positive or negative, hence let us check for

monotonic nature of f(x)


JA
f(x) = x – sinx

 
Since f(x) > 0  f(x) is M.I. for x   0, 
 2

 f(x) > f(0)  f(x) > 0


LP

 f(x) is M..  f(x) > f(0)

x3 x3
 sin x – x + >0  sin x > x – . Hence proved
6 6
A

 sin x tan x   
Example # 44 : Examine which is greater : sin x tan x or x2. Hence evaluate lim   , where x   0, 2 
x →0
 x2   
NK

Solution : Let f(x) = sinx tanx – x2

f(x) = cos x . tan x + sin x . sec 2x – 2x

 f(x) = sin x + sin x sec 2x – 2x


SA

 f(x) = cos x + cos x sec 2x + 2sec2x sin x tan x – 2

 f(x) = (cos x + sec x – 2) + 2 sec2x sin x tan x

2  
Now cos x + sec x – 2 = ( cos x − sec x ) and 2 sec2x tan x . sin x > 0 because x   0, 
 2

 f(x) > 0  f(x) is M.I.

Hence f(x) > f(0)

 f(x) > 0  f(x) is M.I.  f(x) > 0


 sin x tan x – x2 > 0

sin x tan x  sin x tan x 


Hence sin x tan x > x2  > 1  lim   = 1.
x2 x →0
 x2 
x
 1
Example # 45 : Prove that f(x) =  1 +  is monotonically increasing in its domain. Hence or otherwise draw
 x
graph of f(x) and find its range
x
 1 1
Solution : f(x) =  1 +  , for Domain of f(x), 1 + >0
 x x

RI
x +1
 >0  (–, –1)  (0, )
x
 
1   −1
x
 1 x

A
Consider f(x) =  1 +   n  1 +  + 
 x   x 1 x2 
1+
 x 
1   1 
x
 1

UH
 f(x) =  1 +   n  1 +  −
 x   x  x + 1
x
 1  1 1
Now  1 +  is always positive, hence the sign of f(x) depends on sign of n  1 +  –
 x  x  1+ x
 1
JA
i.e. we have to compare n  1 +  and
 x
 1 1
So lets assume g(x) = n  1 +  –
 x  x +1
1 −1 1 −1
g(x) = 2
+  g(x) =
1 x (x + 1)2 x(x + 1)2
1+
LP

x
(i) for x  (0, ), g(x) < 0  g(x) is M.D. for x  (0, )
g(x) > lim g(x)
x →

g(x) > 0. and since g(x) > 0  f(x) > 0


(ii) for x  (– , – 1), g(x) > 0  g(x) is M.I. for x  (– , –1)
A

 g(x) > lim g(x)  g(x) > 0  f(x) > 0


x →−

Hence from (i) and (ii) we get f(x) > 0 for all x  (– , –1)  (0, )
 f(x) is M.I. in its Domain
NK

For drawing the graph of f(x), its important to find the value of f(x) at boundary points
i.e. ± , 0, –1
x
 1
lim 1 +  =e
x → 
 x
SA

x x
 1  1
lim  1 +  =1 and lim  1 +  = 
x →0+  x x →−1
 x
so the graph of f(x) is

Range is y  (1, ) – {e}

Example # 46 : Compare which of the two is greater (100)1/100 or (101)1/101.


Solution : Assume f(x) = x1/x and let us examine monotonic nature of f(x)
 1 − nx 
f(x) = x1/x .  
 x 
2

f(x) > 0  x  (0,e)


and f(x) < 0  x  (e,)

RI
Hence f(x) is M.D. for x  e

A
UH
and since 100 < 101
 f(100) > f(101)
 (100)1/100 > (101)1/101
JA
Self Practice Problems :
(32) Prove the following inequalities
(i) x > tan–1(x) for x  (0, )
(ii) ex > x + 1 for x  (0, )
x
(iii)  n (1 + x)  x for x  (0, )
1+ x
LP

Rolle’s Theorem :
If a function f defined on [a, b] is
(i) continuous on [a, b]
A

(ii) derivable on (a, b) and


(iii) f(a) = f(b),
then there exists at least one real number c between a and b (a < c < b) such that f(c) = 0
NK

Geometrical Explanation of Rolle’s Theorem :


Let the curve y = f(x), which is continuous on [a, b] and derivable on (a, b), be drawn (as shown in
figure).
SA

A(a, f(a)), B(b, f(b)), f(a) = f(b), C(c, f(c)), f(c) = 0.

C1 (c1, f(c1)), f(c1) = 0


C2 (c2, f(c2)), f(c2) = 0
C3 (c3, f(c3)), f(c3) = 0
The theorem simply states that between two points with equal ordinates on the graph of f(x), there
exists at least one point where the tangent is parallel to x-axis.

Algebraic Interpretation of Rolle’s Theorem :


Between two zeros a and b of f(x) (i.e. between two roots a and b of f(x) = 0) there exists at least one
zero of f(x)

Example # 47 : If 2a + 3b + 6c = 0 then prove that the equation ax 2 + bx + c = 0 has at least one real root
between 0 and 1.

RI
ax 3 bx 2
Solution : Let f(x) = + + cx
3 2
a b
f(0) = 0 and f(1) = + + c = 2a + 3b + 6c = 0

A
3 2
If f(0) = f(1) then f(x) = 0 for some value of x  (0, 1)
 ax2 + bx + c = 0 for at least one x  (0, 1)

UH
Self Practice Problems :

(33) If f(x) satisfies condition in Rolle’s theorem then show that between two consecutive zeros of
f(x) there lies at most one zero of f(x).
JA
(34) Show that for any real numbers , the polynomial P(x) = x 7 + x3 +  , has exactly one real root.

Lagrange’s Mean Value Theorem (LMVT) :


LP

If a function f defined on [a, b] is


(i) continuous on [a, b] and
(ii) derivable on (a, b)
A

f(b) − f(a)
then there exists at least one real numbers between a and b (a < c < b) such that = f(c)
b−a

Proof : Let us consider a function g(x) = f(x) + x, x  [a, b]


NK

where  is a constant to b determined such that g(a) = g(b).


f(b) − f(a)
 =–
b−a
SA

Now the function g(x), being the sum of two continuous and derivable functions it self
(i) continuous on [a, b]
(ii) derivable on (a, b) and
(iii) g(a) = g(b).
Therefore, by Rolle’s theorem there exists a real number c  (a, b) such that g(c) = 0
But g(x) = f(x) + 
 0 = g(c) = f(c) + 
f(b) − f(a)
f(c) = –  =
b−a
Geometrical Interpretation of LMVT :

The theorem simply states that between two points A and B of the graph of f(x) there exists at least one
point where tangent is parallel to chord AB.

RI
C(c, f(c)), f(c) = slope of AB.

A
Alternative Statement : If in the statement of LMVT, b is replaced by a + h, then number c between a

and b may be written as a + h, where 0 <  < 1. Thus

UH
f(a + h) − f(a)
= f(a + h) or f(a + h) = f(a) + hf (a + h), 0 <  < 1
h
JA
Example # 48 : Verify LMVT for f(x) = – x2 + 4x – 5 and x  [–1, 1]

Solution : f(1) = –2 ; f(–1) = –10

f(1) − f( −1)
LP

 f(c) =  –2c + 4 = 4  c=0


1 − ( −1)

Example # 49 : Using Lagrange’s mean value theorem, prove that if b > a > 0,
A

b−a b−a
then < tan–1 b – tan–1 a <
1 + b2 1 + a2
NK

Solution : Let f(x) = tan–1 x ; x  [a, b] applying LMVT

tan−1 b − tan−1 a 
f(c) = for a < c < b and f(x) = ,
b−a 1 + x2
SA

Now f(x) is a monotonically decreasing function

Hence if a < c < b

 f(b) < f(c) < f(a)

1 tan−1 b − tan−1 a 1
 < < Hence proved
1+ b 2
b − a 1 + a2
  5 
Example # 50 : Let f : R → R be a twice differentiable function such that f   = 0, f   = 0 & f(3) = 4 then
4  4 

show that there exists a c  (0, 2) such that f(c) + sinc – cosc < 0.

Solution : Consider g(x) = f(x) – sinx + cosx


 g'(x) = f'(x) – cosx – sinx

 g"(x) = f"(x) + sinx – cosx


By LMVT

RI
  5 
g(3) − g   g − g(3)
 4  = g(c ) ,  < c < 3  4  
and = g(c2), 3 < c2 <
 1
4 1
5 4

A
3− −3
4 4
g(c1) > 0 , g(c2) < 0

UH
By LMVT

g (c 2 ) − g (c1 )
= g(c), c1 < c < c 2  g(c) < 0  f(c) + sinc – cosc < 0
c 2 − c1
JA
for some c  (c1 , c2), c  (0, 2)

Self Practice Problems :


(35) Using LMVT, prove that if two functions have equal derivatives at all points of (a, b), then they
LP

differ by a constant

(36) If a function f is
(i) continuous on [a, b],
A

(ii) derivable on (a, b) and


(iii) f(x) > 0, x  (a, b), then show that f(x) is strictly increasing on [a, b].
NK
SA
 Marked questions are recommended for Revision.

PART - I : SUBJECTIVE QUESTIONS

Section (A) : Equation of Tangent / Normal and Common Tangents / Normals


 
A-1. (i)_ Find the equation of tangent to curve y = 3x 2 + 4x + 5 at (0, 5).

RI
(ii) Find the equation of tangent and normal to the curve x 2 + 3xy + y2 = 5 at point (1, 1) on it.
2at 2 2at 3
(iii)_ Find the equation of tangent and normal to the curve x = , y = at the point for
1 + t2 1 + t2
1

A
which t =
2


 x 2 sin1/ x x0

UH
(iv) Find the equation of tangent to the curve y =  at (0,0)

0 x=0

A-2 (i). Find equations of tangents drawn to the curve y 2 – 2x2 – 4y+ 8 = 0 from the point (1, 2).
JA
(ii). Find the equation of all possible normals to the curve x 2 = 4y drawn from the point (1,2)

A-3. (i) Find the point on the curve 9y 2 = x3 where normal to the curve has non zero x-intercept and
both the x intercept and y-intercept are equal.
LP

(ii) If the tangent at (1, 1) on y2 = x(2 – x)2 meets the curve again at P, then find coordinates of P

(iii) The normal to the curve 5x 5 – 10x3 + x + 2y + 6 = 0 at the point P(0, –3) is tangent to the curve
at some other point(s). Find those point(s)?
A

A-4_. (i) Find common tangent between curves y = x 3 and 112x2 + y2 = 112
NK

1
(ii) Find common normals of the curves y = and x2 + y2 – y = 0
x2
 
A-5. (i) If the tangent to the curve xy + ax + by = 0 at (1, 1) is inclined at an angle tan –1 2 with positive
x-axis in anticlockwise, then find a and b ?
SA

a
(ii) The curve y = ax3 + bx2 + 3x + 5 touches y = (x + 2) 2 at (–2, 0) then + b is
2

Section (B) : Angle between curves, Orthogonal curves, Shortest/Maximum distance


between two curves
 
B-1. Find the cosine of angle of intersection of curves y = 2 x nx and y = x2x–1 at (1, 0).

B-2. Find the angle between the curves y = lnx and y = (lnx) 2 at their point of intersections.

B-3. Find the angle between the curves y 2 = 4x + 4 and y2 = 36 (9 – x).


B-4. Show that if the curves ax2 + by2 = 1 and Ax2 + By2 = 1 are orthogonal then ab(A – B) = AB(a – b).
 
B-5. Find the shortest distance between line y = x – 2 and y = x 2 + 3x + 2

B-6. Find shortest distance between y2 = 4x and (x – 6)2 + y2 = 1

Section (C) : Rate of change and approximation


C-1. The length x of rectangle is decreasing at a rate of 3 cm/min and width y is increasing at a rate of 2
cm/min. When x = 10 cm and y = 6 cm, find the rate of change of (i) the perimeter, (ii) the area of
rectangle.

RI
C-2. x and y are the sides of two squares such that y = x – x2 . Find the rate of change of the area of the
second square with respect to the first square.

C-3. A man 1.5 m tall walks away from a lamp post 4.5 m high at a rate of 4 km/hr.

A
(i) How fast is his shadow lengthening?
(ii) How fast is the farther end of shadow moving on the pavement?

UH
C-4. Find the approximate change in volume V of a cube of side 5m caused by increasing its side length by
2%.

Section (D) : Monotonicity on an interval, about a point and inequalities, local


maxima/minima
 
JA
x
D-1. Show that f(x) = – n (1 + x) is an increasing function for x > – 1.
1+ x
 
D-2. Find the intervals of monotonicity for the following functions.
x 4 x3
LP

(i) + – 3x2 + 5 (ii) log32 x + log3x


4 3
 
D-3. If g(x) is monotonically increasing and f(x) is monotonically decreasing for x  R and if (gof) (x) is
defined for x  R, then prove that (gof)(x) will be monotonically decreasing function. Hence prove that
(gof) (x + 1)  (gof) (x – 1).
A

x2 ; x  0

NK

D-4. Let f(x) =  . Find real values of 'a' such that f(x) is strictly monotonically increasing at x = 0.
ax ; x  0

D-5. Check monotonocity at following points for


(i) f(x) = x3 – 3x + 1 at x = –1, 2
SA

(ii) f(x) = | x – 1 | + 2 | x – 3 | – | x + 2 | at x = – 2, 0, 3, 5
(iii) f(x) = x1/3 at x=0
1
(iv) f(x) = x2 + 2 at x = 1, 2
x
 x 3 + 2x 2 + 5x , x  0
(v) f(x) =  at x=0
 3 sin x , x0
 
  1    1
 sin     sin   
D-6. Prove that    
10 9.
 1   1 
 10   9 
   
D-7. Let f and g be differentiable on R and suppose f(0) = g(0) and f(x)  g (x) for all x  0. Then show that
f(x)  g(x) for all x  0.

 3−x 0  x 1
D-8. Let f(x) =  2 . Find the set of values of b such that f(x) has a local minima at x = 1.
 x + nb x 1

D-9. Find the points of local maxima/minima of following functions


(i) f(x) = 2x3 – 21x2 + 36x – 20 (ii) f(x) = – (x – 1)3 (x + 1)2
(iii) f(x) = x nx

D-10. Find points of local maxima / minima of  

RI
(i) f(x) = (2x – 1)(2x – 2)2
(ii) f(x) = x2e–x
(iii) f(x) = 3cos4x + 10cos3x + 6cos2x – 3, x  [0, ]
(iv) f(x) = 2x + 3x2/3

A
x2 – 2
(v) f(x) = 2
x –1
 

UH
D-11. Draw graph of f(x) = x|x – 2| and, hence find points of local maxima/minima.
 
Section (E) : Global maxima, Global minima, Application of Maxima and Minima
E-1. Find the absolute maximum/minimum value of following functions
JA
(i) f(x) = x3 ; x  [–2, 2]
(ii) f(x) = sinx + cosx ; x  [0, ]
x2  9
(iii) f(x) = 4x – ; x   −2, 
2  2
(iv) f(x) = 3x – 8x + 12x – 48x + 25 ;
4 3 2
x  [0, 3]
1  
x  0, 
LP

(v) f (x) = sin x + cos 2 x ;


2  2

E-2. Let f(x) = x2 ; x  (– 1, 2). Then show that f(x) has exactly one point of local minima but global maximum
is not defined.
A

E-3. Find the minimum and maximum values of y in 4x 2 + 12xy + 10y2 – 4y + 3 = 0.

E-4. John has 'x' children by his first wife and Anglina has 'x + 1' children by her first husband. They both
NK

marry and have their own children. The whole family has 24 children. It is given that the children of the
same parents don't fight. Then find then maximum number of fights that can take place in the family.

E-5. If the sum of the lengths of the hypotenuse and another side of a right angled triangle is given, show
that the area of the triangle is a maximum when the angle between these sides is /3.
 
SA

E-6. Find the volume of the largest cylinder that can be inscribed in a sphere of radius ' r ' cm.

E-7. Show that the semi vertical angle of a right circular cone of maximum volume, of a given slant height is
tan−1 2 .

E-8. A running track of 440 m. is to be laid out enclosing a football field, the shape of which is a rectangle
with semi circle at each end . If the area of the rectangular portion is to be maximum, find the length of
its sides.
 
E-9. Find the area of the largest rectangle with lower base on the x-axis and upper vertices on the curve
y = 12 − x².
E-10. Find the dimensions of the rectangle of perimeter 36 cm which will sweep out a volume as large as
possible when revolved around one of its side .

1 1 1
E-11. The combined resistance R of two resistors R 1 & R2 (R1, R2 > 0) is given by, = + . If
R R1 R2
R1 + R2 = constant. Prove that the maximum resistance R is obtained by choosing R 1 = R2.

Section (F) : Rolle's Theorem, LMVT

F-1. Let f : [1, 2] → [1, 4] and g : [1, 2] → [2, 7] be two continuous bijective functions such thaf f(1) = 4 &
g(2) = 7. The number of solutions of the equation f(x) = g(x) in (1, 2), is :

RI
 x 2 + ab 
F-2. Verify Rolle’s theorem for the function, f(x) = log e   + p, for [a, b] where 0 < a < b.
 x(a + b) 
 

A
F-3. Using Rolle’s theorem prove that the equation 3x 2 + px – 1 = 0 has at least one real root in the interval
(– 1, 1).
 

UH
 
 x sin for x0
F-4. Using Rolle's theorem show that the derivative of the function f(x) =  x vanishes at an

 0 for x=0
infinite set of points of the interval (0, 1).
JA
F-5. Let f(x) be differentiable function and g(x) be twice differentiable function. Zeros of f(x), g(x) be a, b
respectively (a < b). Show that there exists at least one root of equation f(x) g(x) + f(x) g(x) = 0 on
(a, b).

      2
F-6. If f(x) = tanx, x  0,  then show that  f  
 5 5 5 5
LP

F-7. If f(x) and g(x) are differentiable functions for 0  x  23 such that f(0) = 2, g(0) = 0, f(23) = 22,
g(23)=10, then show that f'(x) = 2g'(x) for at least one x in the interval (0, 23).

sin3 x sin3 a sin3 b


A

F-8. If f(x) = xe x aea beb


x a b
NK

1 + x 1 + a 1 + b2
2 2

where 0 < a < b < 2, then show that the equation f (x) = 0 has atleast one root in the interval (a, b)

 –8x ; 0  x 1

A function y = f(x) is defined on [0, 6] as f(x) = (x – 3) ; 1  x  4
SA

3
F-9.
 2 ; 4x6

Show that for the function y = f(x), all the three conditions of Rolle's theorem are violated on [0, 6] but
still f'(x) vanishes at a point in (0, 6)

 
PART - II : ONLY ONE OPTION CORRECT TYPE
Section (A) : Equation of Tangent / Normal and Common Tangents / Normals
 

A-1. Equation of the normal to the curve y = − x + 2 at the point (1, 1)


(A) 2x − y − 1 = 0 (B) 2x − y + 1 = 0 (C) 2x + y − 3 = 0 (D) none of these
 
A-2. The angle between x-axis and tangent of the curve y = (x+1) (x–3) at the point (3, 0) is
 8   15 
(A) tan–1   (B) tan–1   (C) tan–1 4 (D) none of these
 15   8 

RI
A-3. The numbers of tangent to the curve y – 2 = x5 which are drawn from point (2,2) is / are
(A) 3 (B) 1 (C) 2 (D) 5

A-4. The equation of tangent drawn to the curve xy = 4 from point (0, 1) is

A
1 1 1 1
(A) y– = – (x + 8) (B) y– = – (x – 8)
2 16 2 16
1  1

UH
1 1
(C) y+ = – (x – 8) (D) y–8= –  x– 
2 16 16  2

A-5. The curve y − exy + x = 0 has a vertical tangent at point


(A) (1, 1) (B) (0, 1) (C) (1, 0) (D) no point

JA
A-6. If the tangent to the curve x = a ( + sin ), y = a (1 + cos ) at  = makes an angle  (0  < ) with
3
x-axis, then  =
 2  5
(A) (B) (C) (D)
3 3 6 6

A-7. If the normal at the point (3t, 4/t) of the curve xy = 12 cuts the curve again at (3t 1, 4/t1) then find 't1' in
LP

terms of 't'
−9 −16 9 16
(A) 3
(B) 3
(C) 3
(D) 3
16t 9t 16t 9t
1
A-8_. The common tangent of the curves y = x 2 + and y2 = 4x is
A

x
(A) y = x + 1 (B) y = x –1 (C) y = – x + 1 (D) y = – x –1
NK

A-9_. The area of triangle formed by tangent at (1,1) on y = x 2 + bx + c with coordinate axis is equal to 1, then
the integral value of b is
(A) –3 (B) 3 (C) 2 (D) –2

Section (B) : Angle between curves, Orthogonal curves, Shortest/Maximum distance


SA

between two curves


 
B-1. The angle of intersection of y = ax and y = bx is given by

log(ab) log(a / b) log(a / b)


(A) tan = (B) (C) (D) None 
1 − log(ab) 1 + logalogb 1 − log(a / b)

B-2. The angle between curves x2 + 4y2 = 32 and x2 – y2 = 12 is


   
(A) (B) (C) (D)
3 4 6 2

B-3. Find the angle at which two curves x 3 – 3xy2 + 2 = 0 and 3x2y – y3 – 2 = 0 intersect
  
(A) 0 (B) (C) (D)
6 3 2
x2 y2
B-4. The value of a2 if the curves 2
+ = 1 and y3 = 16x cut orthogonally is
a 4
(A) 3/4 (B) 1 (C) 4/3 (D) 4

B-5. The shortest distance between curves y 2 = 8x and y2 = 4 (x–3) is


(A) 2 (B) 2 2 (C) 3 2 (D) 4 2

2
x2 y2  7
B-6. The shortest distance between curves = 1 and  x −  + y 2 = 1

RI
+
32 18  4
11 15 11
(A) 15 (B) (C) (D)
2 4 4

A
Section (C) : Rate of change and approximation
 
C-1. Water is poured into an inverted conical vessel of which the radius of the base is 2 m and height 4 m, at

UH
the rate of 77 litre/minute. The rate at which the water level is rising at the instant when the depth is 70
cm is (use  = 22/7)
(A) 10 cm/min (B) 20 cm/min (C) 40 cm/min (D) 30 cm/min

C-2
JA
On the curve x3 = 12y. The interval in which abscissa changes at a faster rate then its ordinate
(A) (–3, 0) (B) (–, –2)  (2, ) (C) (–2, 2) (D) (–3, 3)

C-3. A kite is 300 m high and there are 500 m of cord out. If the wind moves the kite horizontally at the rate
of 5 km/hr. directly away from the person who is flying it, find the rate at which the cord is being paid?
(A) 4 (B) 8
LP

(C) 3 (D) cannot be determined

C-4. The approximate value of tan 46° is (take  = 22/7) :


(A) 3 (B) 1.035 (C) 1.033 (D) 1.135
A

C-5. A spherical iron ball 10 cm in radius is coated with a layer of ice of uniform thickness that melts at a rate
of 50 cm3/min. When the thickness of ice is 5 cm, then the rate at which the thickness of ice decreases ,
is-
NK

5 1 1 1
(A) cm/min (B) cm/min (C) cm/min (D) cm/min
6 54 18 36

Section (D) : Monotonicity on an interval, about a point and inequalities, local


maxima/minima
SA

D-1. The complete set of values of ‘a’ for which the function f(x) = (a + 2) x 3 – 3ax2 + 9ax – 1 decreases for
all real values of x is.
(A) (– , – 3] (B) (– , 0] (C) [– 3, 0] (D) [– 3, )

D-2. Let f(x) = x3 + ax2 + bx + 5 sin2 x be an increasing function in the set of real numbers R. Then a & b
satisfy the condition :
(A) a2 − 3b − 15 > 0 (B) a2 − 3b + 15  0 (C) a2 + 3b − 15 < 0 (D) a > 0 & b > 0
 x − 1|
D-3. The function is monotonically decreasing at the point
x2
(A) x = 3 (B) x = 1 (C) x = 2 (D) none of these

D-4. If f(x) = 1 + 2 x2 + 4 x4 + 6 x6 +...... + 100 x100 is a polynomial in a real variable x, then f(x) has:
(A) neither a maximum nor a minimum (B) only one maximum
(C) only one minimum (D) one maximum and one minimum

D-5_. Which of the following statement is/are true ?


 −  
(1) f(x) = sin x is increasing in interval  , 

RI
 2 2
 −  
(2) f(x) = sin x is increasing at all point of the interval  , 
 2 2
 −    3 5 

A
(3) f(x) = sin x is increasing in interval  ,    , 
 2 2  2 2 
 −    3 5 
(4) f(x) = sin x is increasing at all point of the interval  ,    , 

UH
 2 2  2 2 
 −    3 5 
(5) f(x) = sin x is increasing in intervals  ,    , 
 2 2  2 2 
(A) all are correct (B) all are false
(C) (3) and (4) are correct (D) (1), (4) & (5) are correct
JA
x x  [1,2]

 5 − x x  (2,4)
D-6. Let f(x) =  then which of the following statement is / are correct about f(x) ?
2 x=4
7 − x x  (4,6]
LP

(A) Function is strictly increasing at point x = 2 (B) Function is strictly increasing at point x = 4
(C) Function is not increasing at point x = 2 and x = 4 (D) None of these
D-7. STATEMENT-1 : e is bigger than e.
STATEMENT-2 : f(x) = x1/x is a increasing function when x  [e, )
(A) Statement-1 is True, Statement-2 is True; Statement-2 is a correct explanation for Statement-1.
A

(B) Statement-1 is True,Statement-2 is True; Statement-2 is NOT a correct explanation for Statement-1
(C) Statement-1 is True, Statement-2 is False
(D) Statement-1 is False, Statement-2 is True
NK

(E) Statement-1 is False, Statement-2 is False

Section (E) : Global maxima, Global minima, Application of Maxima and Minima
E-1. The greatest, the least values of the function, f(x) = 2 − 1 + 2x + x 2 , x  [− 2 1] are respectively
(A) 2, 1 (B) 2, − 1 (C) 2, 0 (D) –2, 3
SA

E-2. Let f(x) = (1 + b2)x2 + 2bx + 1 and let m(b) be the minimum value of f(x). As b varies, the range of m(b)
is
 1 1 
(A) [0, 1] (B)  0,  (C)  , 1 (D) (0, 1]
 2   2 
E-3. The radius of a right circular cylinder of greatest curved surface which can be inscribed in a given right
circular cone is
(A) one third that of the cone (B) 1/ 2 times that of the cone
(C) 2/3 that of the cone (D) 1/2 that of the cone
E-4. The dimensions of the rectangle of maximum area that can be inscribed in the ellipse
(x/4)2 + (y/3)2 = 1 are
(A) 8, 2 (B) 4, 3 (C) 2 8, 3 2 (D) 2, 6

E-5. The largest area of a rectangle which has one side on the x−axis and the two vertices on the curve
e−x
2
y= is
(A) 2 e −1/2 (B) 2 e −1/2 (C) e −1/2 (D) none of these
E-6. The maximum distance of the point (k, 0) from the curve 2x 2 + y2 – 2x = 0 is equal to
(A) 1 + 2k − k2 (B) 1 + 2k + 2k 2 (C) 1 − 2k + 2k 2 (D) 1 − 2k + k2

Section (F) : Rolle's Theorem, LMVT

RI
F-1. The function f(x) = x 3 – 6x2 + ax + b satisfy the conditions of Rolle's theorem on [1, 3]. Which of these
are correct ?
(A) a =11, b  R (B) a = 11, b = – 6 (C) a = –11, b = 6 (D) a = – 11, b  R

A
F-2. The function f(x) = x(x + 3)e –x/2 satisfies all the conditions of Rolle's theorem on [–3, 0]. The value of c
which verifies Rolle's theorem, is
(A) 0 (B) – 1 (C) – 2 (D) 3

UH
F-3. If f(x) satisfies the requirements of Lagrange’s mean value theorem on [0, 2] and if f(0) = 0 and
1
f(x)   x  [0, 2], then
2
(A) | f(x) |  2 (B) f(x)  1
(C) f(x) = 2x (D) f(x) = 3 for at least one x in [0, 2]
JA
F-4_. If ab > 0 and 3a + 5b + 15 c = 0 then which of the following statement is “INCORRECT”?
(A) there exist exactly one root of equation ax4 + bx2 + c = 0 in (–1,0)
(B) there exist exactly one root of equation ax4 + bx2 + c = 0 in (0,1)
(C) there exist exactly two root of equation ax4 + bx2 + c = 0 in (–1,1)
(D) number of roots of equation ax4 + bx2 + c = 0 can be two in (–1,0)
LP

F-5. Consider the function for x  [–2, 3]


 –6 ; x =1
 3
f(x) =  x – 2x – 5x + 6
2 . The value of c obtained by applying Rolle's theorem for which
 ; x 1
 x –1
A

f(c) = 0 is
(A) 0 (B) 1 (C) 1/2 (D) 'c' does not exist  
 
NK

 
 
 
 
 
SA

 
 
 
 
 
 
 
 
 
 
PART - III : MATCH THE COLUMN
1. Column – I Column – II
x

2a
(A) If curves y2 = 4ax and y = e are orthogonal then ‘a’ (p) 3
can take value
(B) If  is angle between the curves y = [| sin x | + | cos x|], (q) 1
([  ] denote GIF) and x2 + y2 = 5 then cosec2 is
(C)_ If curves y2 = 4a (x + a) and y2 = 4b (x + b) intersects
a
each other orthogonally then can be equal to____ (r) 5/4

RI
b
(D)_ If y = x2 + 3x + c and x =y2 + 3y + c touches each other
at (h, k) then |h + k + c| is equal to…… (s) 2

2. Column-I Column-II

A
1
(A) The number of point (s) of maxima of f(x) = x 2 + 2 is (p) 0
x

UH
(B) (sin– 1 x)3 + (cos– 1 x)3 is maximum at x = (q) 2
8
(C) If [a, b], (b < 1) is largest interval in which (r)
3
f(x) = 3x4 + 8x3 – 6x2 – 24x + 19 is strictly increasing
a
then is
JA
b
a3 + b3
(D) If a + b = 8, a, b > 0 then minimum value of is (s) –1
48
3. Column –  Column – 
sin x
(A) f(x) = , x  [0,] (p) Conditions in Rolle's theorem are satisfied.
ex
LP

1 3
(B) f(x) = sgn ((ex – 1)nx), x   ,  (q) Conditions in LMVT are satisfied.
2 2
(C) f(x) = (x–1)2/5, x  [0,3] (r) At least one condition in Rolle's theorem is not
satisfied.
A

  1 
  e x – 1
 x , x [–1,1] – {0}
(D) f(x) =   1  (s) At least one condition in LMVT is not satisfied.
  e x + 1 
NK


 0 , x =0

4. Column – I Column – II
(A) A rectangle is inscribed in an equilateral triangle of side 4cm. (p) 65
Square of maximum area of such a rectangle is
SA

(B) The volume of a rectangular closed box is 72 and the base (q) 45
sides are in the ratio 1 : 2. The least total surface area is
(C) If x and y are two positive numbers such that x + y = 60 and x 3y is (r) 12
maximum then value of x is

(D) The sides of a rectangle of greatest perimeter which is inscribed (s) 108
in a semicircle of radius 5 are a and b. Then a3 + b3 =
 Marked questions are recommended for Revision.

PART - I : ONLY ONE OPTION CORRECT TYPE


sin x 2
1. Equation of normal drawn to the graph of the function defined as f(x) = , x  0 and f(0) = 0 at the
x
origin is
(A) x + y = 0 (B) x − y = 0 (C) y = 0 (D) x = 0

RI
x y
2. The line + = 1 touches the curve y = be–x/a at the point
a b
a b  b
(A) (– a, 2b) (B)  ,  (C)  a,  (D) (0, b)

A
 2 2   e

3. The equation of normal to the curve x 3 + y3 = 8xy at point where it is meet by the curve y 2 = 4x, other

UH
than origin is
(A) y = x (B) y = –x + 4 (C) y = 2x (D) y = –2x

4. The length of segment of all tangents to curve x 2/3 + y2/3 = a2/3 intercepted between coordinate axes is
|a| 3|a|
JA
(A) 2|a| (B) |a| (C) (D)
2 2
5. If tangents are drawn from the origin to the curve y = sin x, then their points of contact lie on the curve
(A) x – y = xy (B) x + y = xy (C) x2 – y2 = x2y2 (D) x2 + y2 = x2y2

6. Number of tangents drawn from the point (–1/2, 0) to the curve y = e{x}. (Here { } denotes fractional part
function).
LP

(A) 2 (B) 1 (C) 3 (D) 4


 −x2 , x  0
7. Let f(x) =  2 Equation of tangent line touching both branches of y = f(x) is
x + 8 , x  0
(A) y = 4x + 1 (B) y = 4x + 4 (C) y = x + 4 (D) y = x + 1
A

8. Minimum distance between the curves f(x) = e x & g(x) = n x is


  (A) 1 (B) 2 (C) 2 (D) e
NK

9. The point(s) on the parabola y2 = 4x which are closest to the circle,


x2 + y2 − 24y + 128 = 0 is/are:
(A) (0, 0) (
(B) 2 , 2 2 ) (C) (4, 4) (D) none

ax  ; g(x) = a a x
sgn x  sgn x 
SA


10. If f(x) = a for a > 1, a  1and x  R, where { } & [ ] denote the fractional part

and integral part functions respectively, then which of the following statements holds good for the
function h(x), where (n a) h(x) = (n f(x) + n g(x)).
(A) ‘h’ is even and increasing (B) ‘h’ is odd and decreasing
(C) ‘h’ is even and decreasing (D) ‘h’ is odd and increasing
 
11. If f : [1, 10] → [1, 10] is a non-decreasing function and g : [1, 10] → [1, 10] is a non-increasing function.
Let h(x) = f(g(x)) with h(1) = 1, then h(2)
(A) lies in (1, 2) (B) is more than 2 (C) is equal to 1 (D) is not defined

12. If f(x) = |ax – b| + c|x| is stricly increasing at atleast one point of non differentiability of the function
where a > 0, b > 0, c > 0 then
(A) c > a (B) a > c (C) b > a + c (D) a = b
ex − e− x
13. If g(x) is a curve which is obtained by the reflection of f(x) = by the line y = x then
2
(A) g(x) has more than one tangent parallel to x-axis
(B) g(x) has more than one tangent parallel to y-axis
(C) y = –x is a tangent to g(x) at (0, 0)
(D) g(x) has no extremum

14. The set of values of p for which all the points of extremum of the function
f(x) = x3 − 3 px2 + 3 (p2 − 1) x + 1 lie in the interval (− 2, 4), is:
(A) (− 3, 5) (B) (− 3, 3) (C) (− 1, 3) (D) (− 1, 4)

RI
15. The complete set of values of the parameter ‘a’ for which the point of minimum of the function
x2 + x + 2
f(x) = 1 + a2 x – x3 satisfies the inequality 2 < 0 is
x + 5x + 6
(A) (−3 3, −2 3)  (2 3, 3 3) (B) (−3 3, −2 3)

A
(C) (−3 3, −2 3) (D) (−3 2, 2 3)

16. Consider the following statements :

UH
2x 2 – 1
S1 : The function y = is neither increasing nor decreasing.
x4
S2 : If f(x) is strictly increasing real function defined on R and c is a real constant, then
number of Solutions of f(x) = c is always equal to one.
S3 : Let f(x) = x ; x  (0, 1). f(x) does not has any point of local maxima/minima
JA
    S4 : f(x) = {x} has maximum at x = 6 (here {.} denotes fractional part function).
State, in order, whether S1, S2, S3, S4 are true or false
(A) TTFT (B) FTFT (C) TFTF (D) TFFT

17. If f(x) = sin3x +  sin2 x ; –/2 < x < /2, then the interval in which  should lie in order that f(x) has
LP

exactly one minima and one maxima


(A) (–3/2, 3/2) (B) (–2/3, 2/3) – {0} (C) R (D) none of these

 x 3 − x 2 + 10 x − 5 , x 1
18. Let f(x) =  the set of values of b for which f(x) has greatest value at x =
− 2 x + log2 b − 2
( ) , x 1
2
A

1 is given by :
(A) 1  b  2 (B) b = {1, 2}
NK

(C) b  (− , − 1) (D)  − 130, − 2 ) U ( 2 , 130 


 

19. Four points A, B, C, D lie in that order on the parabola y = ax 2 + bx + c. The coordinates of A, B & D are
known as A(− 2, 3); B(− 1, 1) and D(2, 7). The coordinates of C for which the area of the quadrilateral
ABCD is greatest, is
SA

(A) (1/2, 7/4) (B) (1/2, − 7/4) (C) ( −1/2, 7/4) (D) ( −1/2, –7/4)

20. In a regular triangular prism the distance from the centre of one base to one of the vertices of the other
base is . The altitude of the prism for which the volume is greatest, is :
(A) (B) (C) (D)
2 3 3 4

21. The maximum area of the rectangle whose sides pass through the angular points of a given rectangle
of sides a and b is
1 1 2 a3
(A) 2 (ab) (B) (a + b)2 (C) (a + b2) (D)
2 2 b
22. Let ABC is given triangle having respective sides a,b,c. D,E,F are points of the sides BC,CA,AB
respectively so that AFDE is a parallelogram. The maximum area of the parallelogram is
1 1 1
(A) bcsinA (B) bcsinA (C) bcsinA (D) bcsinA
4 2 8

23. If f(x) = (x – 4) (x – 5) (x – 6) (x – 7) then,


(A) f(x) = 0 has four roots.
(B) three roots of f(x) = 0 lie in (4, 5)  (5, 6)  (6, 7).
(C) the equation f(x) = 0 has only one real root.
(D) three roots of f(x) = 0 lie in (3, 4)  (4, 5)  (5, 6).

RI
24. Square roots of 2 consecutive natural number greater than N 2 is differ by

1 1 1 1
(A) > (B)  (C) < (D) >
2N 2N 2N N

A
nx
25. If Rolle's theorem is applicable to the function f(x) = , (x > 0) over the interval [a, b] where a  ,
x
b  , then the value of a2 + b2 can be

UH
  (A) 20 (B) 25 (C) 45 (D) 10

26. If f(x) be a twice differentiable function such that f(x) = x 2 for x = 1, 2, 3, then
(A) f (x) = 2  x  [1, 3] (B) f(x) = 2 for some x  (1, 3)
(C) f(x) = 2  x  (1, 3) (D) f (x) = 2x  x  (1, 3)
JA
PART - II : SINGLE AND DOUBLE VALUE INTEGER TYPE
1. The number of distinct line(s) which is/are tangent at a point on curve 4x 3 = 27 y2 and normal at other
point, is :
LP

2. The sum of the ordinates of point of contacts of the common tangent to the parabolas y = x 2 + 4x + 8
and y = x2 + 8x + 4, is

3. If p  (0, 1/e) then the number of the distinct roots of the equationn x − px = 0 is:
A

4. A light shines from the top of a pole 50 ft. high. A ball is dropped from the same height from a point
30 ft. away from the light. If the shadow of the ball moving at the rate of 100 ft/sec along the ground
1/2 sec. later [Assume the ball falls a distance s = 16 t 2 ft. in ' t ' sec.], then || is :
NK

5. A variable ABC in the xy plane has its orthocentre at vertex 'B' , a fixed vertex 'A' at the origin and the
7 x2
third vertex 'C' restricted to lie on the parabola y = 1 + . The point B starts at the point (0, 1) at time
36
t = 0 and moves upward along the y axis at a constant velocity of 2 cm/sec. If the area of the triangle
SA

7
increasing at the rate of 'p' cm2/sec when t = sec, then 7p is.
2

ex − e− x
2 2

6. Function defined by f(x) = is injective in [ – 2, ), the least value of  is


ex + e− x
2 2

 
 3x 
7. Find lim+   where [ . ] denotes the GIF.
x → 0  2sin x + tan x 

 
8. If f(x) = 2ex – ae–x + (2a + 1) x – 3 monotonically increases for  x  R, then the minimum value of 'a' is
9. If the set of all values of the parameter 'a' for which the function
f(x) = sin2x – 8(a + 1) sin x + (4a2 + 8a – 14) x increases for all x  R and has no critical points for all
x  R, is (–, –m – n )  ( n , ) then (m2 + n2) is (where m, n are prime numbers) :

10. ( )
If n2 < log2 2 + 3 <n3, then number of roots of the equation 4cos (e x) = 2x + 2–x, is

1 
11. For –1  p  1, the equation 4x 3 – 3x – p = 0 has ‘n’ distinct real roots in the interval  , 1 and one of
 2 
1
its root is cos(kcos–1p), then the value of n + is :
k

RI
2 2
12. Least value of the function, f(x) = 2x − 1 + is:
x2
2 +1

A
13. Real root of the equation
(x – 1)2013 + (x – 2)2013 + (x – 3)2013 +...............+(x – 2013)2013 = 0 is a four digit number. Then the sum of

UH
the digits is :

a 3
14. The exhaustive set of values of 'a' for which the function f(x) = x + (a + 2) x2 + (a − 1) x + 2 possess
3
a negative point of minimum is (q, ). The value of q is :
1/ x
JA
 f (x) 
15. If f(x) is a polynomial of degree 6, which satisfies Lim  1 + 3  = e2 and has local maximum at x = 1
x→0
 x 
4
 5   18 
and local minimum at x = 0 and x = 2, then the value of   f   is :
 9  5 

( ) + (x – 5)2 (where 1  x  3) is
LP

16. Maximum value of –3 + 4x – x 2 + 4

17. The three sides of a trapezium are equal each being 6 cms long. Let  cm2 be the maximum area of the
trapezium. The value of 3  is :
A

18. A sheet of poster has its area 18 m². The margin at the top & bottom are 75 cms. and at the sides 50
cms. Let , n are the dimensions of the poster in meters when the area of the printed space is
maximum. The value of 2 + n2 is :
NK

19. The fuel charges for running a train are proportional to the square of the speed generated in m.p.h. and
costs Rs. 48/- per hour at 16 mph. What is the most economical speed if the fixed charges i.e. salaries
etc. amount to Rs. 300/- per hour.

20. Let f(x) = Max. {x2, (1 – x)2, 2x(1 – x)} where x  [0, 1] If Rolle's theorem is applicable for f(x) on largest
SA

possible interval [a, b] then the value of 2(a + b + c) when c  (a, b) such that f'(c) = 0, is ___

21_. For every twice differentiable function f(x) the value of |f(x)|  3  x  R and for some 
f() + (f'())2 = 80. Number of integral values that (f'(x)) 2 can take between (0, 77) are equal to ___
PART - III : ONE OR MORE THAN ONE OPTIONS CORRECT TYPE
1. If tangent to curve 2y3 = ax2 + x3 at point (a, a) cuts off intercepts ,  on co-ordinate axes, where 2 +
2 = 61, then the value of 'a' is equal to
(A) 20 (B) 25 (C) 30 (D) − 30

2. For the curve x = t2 + 3t – 8, y = 2t2 – 2t – 5, at point (2, – 1)


(A) length of subtangent is 7/6. (B) slope of tangent = 6/7
(C) length of tangent = (85) / 6 (D) none of these

RI
3. Which of the following statements is/are correct ?
(A) x + sinx is increasing function
(B) sec x is neither increasing nor decreasing function
(C) x + sinx is decreasing function

A
(D) sec x is an increasing function
 
4. If f(x) = 2x + cot–1 x + n ( )
1 + x 2 − x , then f(x) :

UH
(A) increases in [0, ) (B) decreases in [0, )
(C) neither increases nor decreases in [0, ) (D) increases in (– , )

5. Let g(x) = 2f(x/2) + f(1 – x) and f(x) < 0 in 0  x  1 then g(x)
 2 2 
JA
(A) decreases in 0,  (B) decreases  , 1
 3 3 
 2   2 
(C) increases in 0,  (D) increases in  , 1
 3 3 

6. Let f(x) = xm/n for x  R where m and n are integers, m even and n odd and 0 < m < n. Then
(A) f(x) decreases on (– , 0] (B) f(x) increases on [0, )
LP

(C) f(x) increases on (– , 0] (D) f(x) decreases on [0, )

7. Let f and g be two differentiable functions defined on an interval  such that f(x)  0 and g(x)  0 for all
x   and f is strictly decreasing on  while g is strictly increasing on  then
(A) the product function fg is strictly increasing on 
A

(B) the product function fg is strictly decreasing on I


(C) fog(x) is monotonically increasing on 
(D) fog (x) is monotonically decreasing on 
NK

 
8. Let (x) = (f(x))3 – 3(f(x))2 + 4f(x) + 5x + 3 sin x + 4 cos x  x  R, where f(x) is a differentiable function
xR, then
(A)  is increasing whenever f is increasing (B)  is increasing whenever f is decreasing
(C)  is decreasing whenever f is decreasing (D)  is decreasing if f(x) = – 11
SA

x + p2 pq pr
2
9. If p, q, r be real, then the intervals in which, f(x) = pq x+q qr ,
pr qr x + r2
2 2 2
(A) increase is x < – (p + q2 + r2), x > 0 (B) decrease is (– (p2 + q2 + r2), 0)
3 3
2 2
(C) decrease is x < – (p2 + q2 + r2), x > 0 (D) increase is (– (p2 + q2 + r2), 0)
3 3
x2 x2
10. If f(x) = ; g(x) = where 0 < x < 1, then
2 − 2cos x 6x − 6 sin x
(A) 'f' is increasing function (B) 'g' is decreasing function
f(x)
(C) is increasing function (D) g(f(x)) is decreasing function
g(x)

x  
11. Let f(x) = & x   0, 
sin x  2 
2 3 4
Then the interval in which at least one root of equaiton lie + + =0
     5 
x−f  x−f  x−f 
 12  4  12 
             5      5  
(A)  f   , f    (B)  0, f    (C) f  ,   (D)  f   , f   
  12   4   12     12    4  12  

RI

12. Let f(x) = (x2 − 1)n (x2 + x + 1). f(x) has local extremum at x = 1 if
(A) n = 2 (B) n = 3 (C) n = 4 (D) n = 6

A
x  
13. If f(x) = , x   0,  , then
1 + x tan x  2

UH
(A) f(x) has exactly one point of minima (B) f(x) has exactly one point of maxima
 
(C) f(x) is increasing in  0,  (D) maxima occurs at x0 where x0 = cosx0
 2

14. If f(x) = a n |x| + bx2 + x has its extremum values at x = – 1 and x = 2, then
JA
(A) a = 2 (B) b = – 1/2 (C) a = – 2 (D) b = 1/2

− 1 − x2 , 0  x  1
15. If f(x) =  , then
 − x , x 1
(A) Maximum of f(x) exist at x = 1 (B) Maximum of f (x) doesn't exists
(C) Minimum of f–1(x) exist at x = – 1 (D) Minimum of f–1(x) exist at x = 1
16. If f(x) = tan–1x – (1/2) n x. Then
LP

(A) the greatest value of f(x) on 1/ 3, 3  is /6 + (1/4) n 3

(B) the least value of f(x) on 1/ 3, 3  is /3 – (1/4) n 3

(C) f(x) decreases on (0, )


A

(D) f(x) increases on (– , 0)


NK

40
17. Let f(x) = . Which of the following statement(s) about f(x) is (are) correct ?
3x 4 + 8x 3 − 18x 2 + 60
(A) f(x) has local minima at x = 0. (B) f(x) has local maxima at x = 0.
(C) Absolute maximum value of f(x) is not defined. (D) f(x) is local maxima at x = – 3, x = 1.
 
SA

x 2 − 3x + 2
18. A function f(x) = is -
x 2 + 2x − 3
(A) 1 is not in its domain (B) minimum at x = – 3 and maximum at x = 1
(C) no point of maxima and minima (D) increasing in its domain

19. For the function f(x) = x cot–1x, x  0


x
(A) there is atleast one x  (0, 1) for which cot–1x =
1 + x2
 2
(B) for atleast one x in the interval (0, ), f  x +  –f(x) < 1
 
(C) number of solution of the equation f(x) = sec x is 1
(D) f’(x) is strictly decreasing in the interval (0, )

20. Which of the following statements are true :


(A) If f(x) is differentiable function such that f(a)  f(b) then there exist no c(a, b) such that f’(c) = 0
(B) The function x100 + sinx – 1 is strictly increasing in [0, 1]
(C) If a, b, c are in A.P, then at least one root of the equation 3ax 2 – 4bx + c = 0 is positive
(D) The number of solution(s) of equation 3 tanx + x 3 = 2 in (0, /4) is 2

21. Let f(x) be a differentiable function and f() = f() = 0 ( < ), then in the interval (, )

RI
(A) f(x) + f '(x) = 0 has at least one root (B) f(x) – f '(x) = 0 has at least one real root
(C) f(x) . f '(x) = 0 has at least one real root (D) none of these
 
22. Which of the following inequalities are valid –

A
(A) |tan–1 x – tan–1y|  |x – y|  x, y  R (B) |tan–1 x – tan–1y|  |x – y|
(C) |sin x – sin y|  |x – y| (D) |sin x – sin y|  |x – y|

UH
23. For all x in [1, 2]
Let f"(x) of a non-constant function f(x) exist and satisfy |f(x)|  2. If f(1) = f(2), then
(A) There exist some a  (1, 2) such that f(a) = 0
(B) f(x) is strictly increasing in (1, 2)
(C) There exist atleast one c  (1, 2) such that f(c) > 0
JA
(D) |f(x)| < 2  x  [1, 2]

PART - IV : COMPREHENSION
Comprehension # 1
LP

Lengths of tangent, normal, subtangent and subnormal :


Let P (h, k) be any point on curve y = f(x). Let tangent drawn at point P meets x-axis at T & normal at
dy 
point P meets x-axis at N (as shown in figure) and m = = slope of tangent.
dx  (h, k )
A
NK
SA

1
(i) Length of Tangent = PT = | k | 1 +
m2
(ii) Length of Normal = PN = | k | 1 + m2
k
(iii) Length of subtangent = Projection of segment PT on x-axis = TM =
m
(iv) Length of subnormal = projection of line segment PN on x axis =MN = |km|

1. Find the product of length of tangent and length of normal for the curve y = x 3 + 3x2 + 4x – 1 at point
x = 0.
17 15 4
(A) (B) (C) 17 (D)
4 4 17
2. Determine 'p' such that the length of the subtangent and subnormal is equal for the curve
y = epx + px at the point (0, 1).

RI
1 1
(A) ±1 (B) ±2 (C) ± (D) ±
2 4
−
3. Find length of subnormal to x = 2 cos t, y = – 3sin t at t = .

A
4
2 7 9
(A) (B) 1 (C) (D)
9 2 2

UH
Comprehension # 2.
 x + sin x 
Consider a function f defined by f(x) = sin –1 sin   ,  x  [0, ], which satisfies
 2 
f(x) + f(2 – x) = ,  x  [, 2] and f(x) = f(4 – x) for all x  [2, 4], then
JA
4. If  is the length of the largest interval on which f(x) is increasing, then =

(A) (B)  (C) 2 (D) 4
2
5. If f(x) is symmetric about x = , then  =
 
(A) (B)  (C) (D) 2
LP

2 4
6. Maximum value of f(x) on [0, 4] is :
 
(A) (B)  (C) (D) 2
2 4
A

Comprehension # 3.
Concavity and convexity :
If f(x) > 0  x  (a, b), then the curve y = f(x) is concave up (or convex down) in (a,b) and
NK

If f(x) < 0  x  (a, b) then the curve y = f(x) is concave down (or convex up) in (a, b).

concave upward concave downward


or or
convex downward convex upward

   
SA

a b a b

 
Inflection point :
The point where concavity of the curve changes is known as point of inflection (at inflection
point f(x) is equal to 0 or undefined).
inflection point

c
  y>0 y< 0
 
7. Number of point of inflection for f(x) = (x – 1)3 (x– 2)2 , is
(A) 1 (B) 2 (C) 3 (D) 4
3x 2
8. Exhaustive set of values of ‘a’ for which the function f(x) = x 4 + ax3 + + 1 will be concave upward
2
along the entire real line, is :
(A) [–1,1] (B) [–2,2] (C) [0,2] (D) [0,4]

Comprehension # 4
For a double differentiable function f(x) if f(x)  0 then f(x) is concave upward and if f(x)  0 then f(x) is
concave downward

RI
A
 k  + k 2 
Here M  1 , 0
 k1 + k 2 

UH
k1f( ) + k 2 f()  k  + k 2 
If f(x) is a concave upward in [a, b] and ,  [a, b] then  f 1 ,
k1 + k 2  k1 + k 2 
where k1, k2 R+
k1f( ) + k 2 f()  k  + k 2 
If f(x) is a concave downward in [a, b] and ,   [a, b] then  f 1  ,
JA
k1 + k 2  k1 + k 2 
where k1, k2 R+

then answer the following :


9. Which of the following is true
sin  + sin  + sin  + sin  +
(A)  sin   ; ,  (0, ) (B)  sin   ; ,   (, 2)
LP

2  2  2  2 
sin  + sin  +
(C)  sin   ;  ,  (0, ) (D) None of these
2  2 
A

10. Which of the following is true


2 + 2+1
+ 2
2 n  + n  2 +  
(A)  2 3 (B)  n  
3 3  3 
NK

tan−1  + tan−1  +


  + 2
– (D) e + 2e   e 3
(C)  tan−1   ) a, b  R
2  2  3

11. Let  ,  and  are three distinct real numbers and f (x) < 0. Also f(x) is increasing function and let
SA

f –1( ) + f –1() + f –1(  ) ++ 


A= and B = f –1   , then order relation between A and B is ?
3  3 
(A) A > B (B) A < B (C) A = B (D) none of these
 Marked questions are recommended for Revision.
* Marked Questions may have more than one correct option.
PART - I : JEE (ADVANCED) / IIT-JEE PROBLEMS (PREVIOUS YEARS)

1
1*. For the function f(x) = x cos , x  1, [IIT-JEE 2009, Paper-2, (4, –1)/ 80]
x
(A) for at least one x in the interval [1, ), f(x + 2) – f(x) < 2

RI
(B) lim f(x) = 1
x →

(C) for all x in the interval [1, ), f(x + 2) – f(x) > 2
(D) f(x) is strictly decreasing in the interval [1, )

A
 p(x) 

UH
2. Let p(x) be a polynomial of degree 4 having extremum at x = 1, 2 and lim  1 + 2  = 2. Then the
x →0
 x 
value of p(2) is [IIT-JEE 2009, Paper-2, (4, –1)/ 80]

3. Let f be a function defined on R (the set of all real numbers) such that
JA
f(x) = 2010 (x – 2009) (x – 2010)2 (x – 2011)3 (x – 2012)4, for all x  R.
If g is a function defined on R with values in the interval (0, ) such that f(x) = n (g(x)), for all x  R,
then the number of points in R at which g has a local maximum is
[IIT-JEE 2010, Paper-2, (3, 0)/ 79]
LP

2 2
4. Let f, g and h be real-valued functions defined on the interval [0, 1] by f(x) = ex + e– x ,
2 2 2 2
g(x) = xex + e– x and h(x) = x2 ex + e– x . If a, b and c denote, respectively, the absolute maximum
of f, g and h on [0, 1], then [IIT-JEE 2010, Paper-1, (3, –1)/ 84]
A

(A) a = b and c  b (B) a = c and a  b (C) a  b and c  b (D) a = b = c


5. Match the statements given in Column-I with the intervals/union of intervals given in Column-II
[IIT-JEE 2011, Paper-2, (8, 0), 80]
NK

Column-I Column-II
(A) The set
  2iz  
Re  2 
:z is a complex number, | z |= 1, z  1 is (p) (–, –1)  (1, )
  1− z  
SA

 8(3)x − 2 
(B) The domain of the function f(x) = sin –1  2( x −1) 
is (q) (–, 0)  (0, )
 1− 3 

1 tan  1
(C) If f() = − tan  1 tan  , (r) [2, )
−1 − tan  1
 
then the set f( ) : 0     is
 2 

(D) If f(x) = x3/2 (3x – 10), x  0, then f(x) is increasing in (s) (–, –1]  [1, )
(t) (–, 0]  [2, )

6. The number of distinct real roots of x 4 – 4x3 + 12x2 + x – 1 = 0 is [IIT-JEE 2011, Paper-2, (4, 0), 80]

7. Let p(x) be a real polynomial of least degree which has a local maximum at x = 1 and a local minimum
at x = 3. If p(1) = 6 p(3) = 2 , then p(0) is       [IIT-JEE 2012, Paper-1, (4, 0), 70]
 
8. Let f : R → R be defined as f(x) = |x| + |x 2 – 1|.  The total number of points at which f attains either a
local maximum or a local minimum is         [IIT-JEE 2012, Paper-1, (4, 0), 70]  

9. The number of points in (– , ), for which x2 – x sinx – cosx = 0, is

RI
[JEE (Advanced) 2013, Paper-1, (2, 0)/60]
(A) 6 (B) 4 (C) 2 (D) 0

10. A rectangular sheet of fixed perimeter with sides having their lengths in the ratio 8 : 15 is converted into

A
an open rectangular box by folding after removing squares of equal area from all four corners. If the
total area of removed squares is 100, the resulting box has maximum volume. The lengths of the sides
of the rectangular sheet are [JEE (Advanced) 2013, Paper-1, (4, – 1)/60]

UH
(A) 24 (B) 32 (C) 45 (D) 60

x2 y2
11. A vertical line passing through the point (h, 0) intersects the ellipse + = 1 at the points P and Q.
4 3
Let the tangents to the ellipse at P and Q meet at the point R. If (h) = area of the triangle PQR,  1 =
JA
8
max (h) and 2 = min (h), then 1 – 82 = [JEE (Advanced) 2013, Paper-1, (4, – 1)/60]
1/ 2  h 1 1/ 2  h 1
5

12*. The function f(x) = 2|x| + |x + 2| – ||x + 2| – 2|x|| has a local minimum or a local maximum at x =
[JEE (Advanced) 2013, Paper-2, (3, –1)/60]
–2 2
(A) – 2 (B) (C) 2 (D)
LP

3 3

Paragraph for Question Nos. 13 to 14.

Let f : [0, 1] → R (the set of all real numbers) be a function. Suppose the function f is twice
A

differentiable,
f(0) = f(1) = 0 and satisfies f(x) – 2f(x) + f(x)  ex, x  [0, 1].
NK

13. Which of the following is true for 0 < x < 1 ? [JEE (Advanced) 2013, Paper-2, (3, –1)/60]

1 1 1
(A) 0 < f(x) <  (B) – < f(x) < (C) – < f(x) < 1 (D) –  < f(x) < 0
2 2 4
SA

1
14. If the function e–xf(x) assumes its minimum in the interval [0, 1] at x = , which of the following is true ?
4
[JEE (Advanced) 2013, Paper-2, (3, –1)/60]
1
(A) f (x) < f(x), (B) f (x) > f(x), 0 < x <
4
1 3
(C) f (x) < f(x), 0 < x < (D) f (x) < f(x), <x<1
4 4

15. A line L : y = mx + 3 meets y - axis at E(0, 3) and the arc of the parabola y 2 = 16x, 0  y  6 at the point
F(x0 , y0). The tangent to the parabola at F(x 0, y0) intersects the y-axis at G(0, y1). The slope m of the line
L is chosen such that the area of the triangle EFG has a local maximum
Match List I with List II and select the correct answer using the code given below the lists :
[JEE (Advanced) 2013, Paper-2, (3, –1)/60]
List - I List - II
1
P. m= 1.
2
Q. Maximum area of EFG is 2. 4
R. y0 = 3. 2
S. y1 = 4. 1

Codes :
P Q R S

RI
(A) 4 1 2 3
(B) 3 4 1 2
(C) 1 3 2 4
(D) 1 3 4 2

A
16*. Let a  R and let f : R → R be given by f(x) = x 5 – 5x + a. Then
[JEE (Advanced) 2014, Paper-1, (3, 0)/60]
(A) f(x) has three real roots if a > 4 (B) f(x) has only one real root if a > 4

UH
(C) f(x) has three real roots if a < – 4 (D) f(x) has three real roots if – 4 < a < 4

17. The slope of the tangent to the curve (y – x5)2 = x(1 + x2)2 at the point (1, 3) is
[JEE (Advanced) 2014, Paper-1, (3, 0)/60]

18. A cylindrical container is to be made from certain solid material with the following constraints: It has
JA
fixed inner volume of V mm3, has a 2 mm thick solid wall and is open at the top. The bottom of the
container is solid circular disc of thickness 2 mm and is of radius equal to the outer radius of the
container.
If the volume of the material used to make the container is minimum when the inner radius of the
V
container is 10 mm, then the value of is
250
LP

[JEE (Advanced) 2015, P-1 (4, 0) /88]  

19*. Let f, g : [–1, 2] → R be continuous function which are twice differentiable on the interval (–1, 2). Let the
values of f and g at the points –1, 0 and 2 be as given in the following table :
x = –1 x = 0 x = 2
A

f ( x) 3 6 0
g( x ) 0 1 –1
NK

In each of the intervals (–1, 0) and (0, 2) the function (f – 3g)" never vanishes. Then the correct
statement(s) is (are) [JEE (Advanced) 2015, P-2 (4, –2)/ 80]
(A) f'(x) – 3g'(x) = 0 has exactly three solutions in (–1, 0)  (0, 2)
(B) f'(x) – 3g'(x) = 0 has exactly one solution in (–1, 0)
(C) f'(x) – 3g'(x) = 0 has exactly one solution in (0, 2)
(D) f'(x) – 3g'(x)= 0 has exactly two solutions in (–1, 0) and exactly two solutions in (0,2)
SA

 
20. Let f : R → (0, ) and g : R → R be twice differentiable functions such that f " and g" are continuous
f ( x )g( x )
functions on R. Suppose f '(2) = g(2) = 0, f "(2)  0 and g'(2)  0, If lim = 1, then
x → 2 f ' ( x )g' ( x )

(A) f has a local minimum at x = 2 (B) f has a local maximum at x = 2


(C) f "(2) > f(2) (D) f(x) – f "(x) = 0 for at least one x  R
[JEE (Advanced) 2016, Paper-2, (4, –2)/62]  

Answer Q.21, Q.22 and Q.23 by appropriately matching the information given in the three
columns of the following table.  
Let f(x) = x + logex – xlogex, x  (0, )
• Column1 contains information about zeros of f(x), f(x) and f(x).
• Column2 contains information about the limiting behavior of f(x), f(x) and f(x) at infinity.
• Column3 contains information about increasing/decreasing nature of f(x) and f(x).
Column-1 Column-2 Column-3
() f(x) = 0 for some x  (1, e2) (i) limx→ f(x) = 0 (P) f is increasing in (0, 1)
() f(x) = 0 for some x  (1, e) (ii) limx→ f(x) = –  (Q) f is decreasing in (e, e2)
() f(x) = 0 for some x  (0, 1) (iii) limx→ f(x) = –  (R) f is increasing in (0, 1)
(V) f(x) = 0 for some x  (1, e) (iv) limx→ f(x) = 0 (S) f is decreasing in (e, e2)
 

21. Which of the following options is the only INCORRECT combination?


[JEE(Advanced) 2017, Paper-1,(3, –1)/61]

RI
(A) () (iii) (P) (B) () (iv) (Q) (C) () (iii) (P) (D) () (i) (R)

22. Which of the following options is the only CORRECT combination?


[JEE(Advanced) 2017, Paper-1,(3, –1)/61]

A
(A) () (ii) (R) (B) () (iv) (P) (C) () (iii) (S) (D) (V) (i) (S)

23. Which of the following options is the only CORRECT combination?

UH
[JEE(Advanced) 2017, Paper-1,(3, –1)/61]
(A) () (iii) (R) (B) (V) (iv) (S) (C) () (ii) (Q) (D) () (i) (P)

 1 1
24. If f : R → R is a twice differentiable function such that f (x) > 0 for all x  R, and f   = , f(1) = 1,
2 2
JA
then [JEE(Advanced) 2017, Paper-2,(3, –1)/61]
1 1
(A) f (1)  0 (B) f (1) > 1 (C) 0 < f (1)  (D) < f (1)  1
2 2

25*. If f : R → R is a differentiable function such that f(x) > 2f(x) for all x  R, and f(0) = 1, then
[JEE(Advanced) 2017, Paper-2,(4, –2)/61]
(A) f(x) > e2x in (0, ) (B) f(x) < e2x in (0, )
LP

(C) f(x) is increasing in (0, ) (D) f(x) is decreasing in (0, )


cos(2x) cos(2x) sin(2x)
26*. If f(x) = − cos x cos x − sin x , then [JEE(Advanced) 2017, Paper-2,(4, –2)/61] 
sin x sin x cos x
A

(A) f (x) attains its minimum at x = 0


(B) f (x) attains its maximum at x = 0
(C) f '(x) = 0 at more than three points in (–, )
NK

(D) f '(x) = 0 at exactly three points in (–, )


 
27*. For every twice differentiable function f : R → [–2, 2] with (f(0)) 2 + (f(0))2 = 85, which of the following
statement(s) is (are) TRUE? [JEE(Advanced) 2018, Paper-1,(4, –2)/60]
(A) There exist r, s  R, where r < s, such that f is one-one on the open interval (r, s)
SA

(B) There exists x0  (–4, 0) such that |f(x0)|  1


(C) lim f(x) = 1
x →
(D) There exists   (–4, 4) such that f() + f() = 0 and f()  0
PART - II : JEE (MAIN) / AIEEE PROBLEMS (PREVIOUS YEARS)

1. Given P(x) = x4 + ax3 + bx2 + cx + d such that x = 0 is the only real root of P(x) = 0. If P(–1) < P(1), then
in the interval [–1, 1] [AIEEE 2009(8, –2), 144]
(1) P (–1) is the minimum and P(1) is the maximum of P
(2) P (–1) is not minimum but P(1) is the maximum of P
(3) P (–1) is the minimum and P(1) is not the maximum of P
(4) neither P (–1) is the minimum nor P(1) is the maximum of P
 
2. The shortest distance between the line y – x = 1 and the curve x = y 2 is [AIEEE 2009(4, –1), 144]

RI
3 2 2 3 3 2 3
(1) (2) (3) (4)
8 8 5 4

3. Let f : R → R be defined by [AIEEE 2010(8, –2), 144]

A
k – 2x , if x  –1
f(x) = 
2x + 3 , if x  –1

UH
If f has a local minimum at x = – 1, then a possible value of k is
1
(1) 0 (2) – (3) –1 (4) 1
2

1
4. Let f : R → R be a continuous function defined by f(x) = [AIEEE 2010(8, –2), 144]
JA
e + 2e – x
x

1
Statement -1 : f(c) = , for some c  R.
3
1
Statement -2 : 0 < f(x)  , for all x  R.
2 2
(1) Statement -1 is true, Statement-2 is true ;Statement -2 is not a correct explanation for Statement -1.
LP

(2) Statement-1 is true, Statement-2 is false.


(3) Statement -1 is false, Statement -2 is true.
(4) Statement -1 is true, Statement -2 is true; Statement-2 is a correct explanation for Statement-1.

4
5. The equation of the tangent to the curve y = x + , that is parallel to the x-axis, is
A

x2
[AIEEE 2010 (4, –1), 144]
(1) y = 1 (2) y = 2 (3) y = 3 (4) y = 0
 
NK

6. Let f be a function defined by - [AIEEE 2011 II(4, –1), 120]


 tan x
 , x 0
f(x) =  x

 1 , x=0
SA

Statement - 1 : x = 0 is point of minima of f


Statement - 2 : f '(0) = 0.
(1) Statement-1 is true, statement-2 is true; statement-2 is a correct explanation for statement-1.
(2) Statement-1 is true, statement-2 is true; statement-2 is not a correct explanation for statement-1
(3) Statement-1 is true, statement-2 is false.
(4) Statement-1 is false, statement-2 is true.

7. The shortest distance between line y – x = 1 and curve x = y 2 is : [AIEEE 2011 (4, –1), 120]
3 3 2 8 4
(1) (2) (3) (4)
4 8 3 2 3
8. A spherical balloon is filled with 4500 cubic meters of helium gas. If a leak in the balloon causes the
gas to escape at the rate of 72 cubic meters per minute, then the rate (in meters per minute) at which
the radius of the balloon decreases 49 minutes after the leakage began is : [AIEEE 2012(4, –1), 120]
9 7 2 9
(1) (2) (3) (4)
7 9 9 2

9. Let a, b  R be such that the function f given by f(x) = n |x| + bx 2 + ax, x  0 has extreme values at
x = – 1 and x = 2.
Statement-1 : f has local maximum at x = – 1 and at x = 2. [AIEEE 2012 (4, –1), 120]
1 −1
Statement-2 : a = and b = .
2 4

RI
(1) Statement-1 is false, Statement-2 is true.
(2) Statement-1 is true, statement-2 is true; statement-2 is a correct explanation for Statement-1.
(3) Statement-1 is true, statement-2 is true; statement-2 is not a correct explanation for Statement-1.
(4) Statement-1 is true, statement-2 is false.

A
10. The real number k for which the equation, 2x 3 + 3x + k = 0 has two distinct real roots in [0, 1]
(1) lies between 1 and 2 (2) lies between 2 and 3
(3) lies between –1 and 0 (4) does not exist. [AIEEE - 2013, (4, –1),120]
 

UH
11. If f and g are differentiable functions in [0, 1] satisfying f(0) = 2 = g(1), g(0) = 0 and f(1) = 6, then for
some
c]0, 1[ : [JEE(Main)2014,(4, – 1), 120]
(1) f'(c) = g'(c) (2) f'(c) = 2g'(c) (3) 2f'(c) = g'(c) (4) 2f'(c) = 3g'(c)
JA
12. If x = –1 and x = 2 are extreme points of f(x) = log|x| + x 2 + x then :
[JEE(Main)2014, (4, –1), 120]
1 1 1 1
(1)  = 2,  = − (2)  = 2,  = (3)  = –6,  = (4)  = –6,  = –
2 2 2 2
 
13. A wire of length 2 units is cut into two parts which are bent respectively to form a square of side
LP

= x units and a circle of radius = r units. If the sum of the areas of the square and the circle so formed is
minimum, then [JEE(Main)2016,(4, – 1), 120]
(1) (4 – ) x = r (2) x = 2r (3) 2x = r (4) 2x = ( + 4) r

 1 + sin x    
A

Consider f(x) = tan–1  , x   0,  . A normal to y = f(x) at x = also passes through the


 1– sin x 
14.
   2  6
point : [JEE(Main)2016,(4, – 1), 120]
 2     
NK

(1)  0, (2)  , 0  (3)  , 0 


3 
(4) (0, 0)
 6  4 
15. Twenty meters of wire is available for fencing off a flower-bed in the form of a circular sector. Then the
maximum area (in sq. m) of the flower-bed, is : [JEE(Main)2017,(4, – 1), 120]
(1) 12.5 (2) 10 (3) 25 (4) 30
SA

16. The normal to the curve y(x – 2)(x – 3) = x + 6 at the point where the curve intersects the y-axis passes
through the point : [JEE(Main)2017,(4, – 1), 120]
 1 1  1 1  1 1  1 1
(1)  – ,–  (2)  ,  (3)  ,–  (4)  , 
 2 2   2 2   2 3  2 3

17. The radius of a circle, having minimum area, which touches the curve y = 4 – x 2 and the lines, y = |x| is
[JEE(Main)2017,(4, – 1), 120]
(1) 2 ( )
2 +1 (2) 2 ( )
2 –1 (3) 4 ( 2 –1) (4) 4 ( )
2 +1

 
18. If the curves y2 = 6x, 9x2 + by2 = 16 intersect each other at right angels , then the value of b is :
                    [JEE(Main)2018,(4, – 1), 120]
9 7
(1) 4 (2) (3) 6 (4)
2 2

1 1 f(x)
19. Let f(x) = x2 + and g(x) = x – , x  R – {–1, 0, 1}. If h(x) = , then the local minimum value of
x2 x g(x)
h(x) is : [JEE(Main)2018,(4, – 1), 120] 
(1) – 2 2 (2) 2 2 (3) 3 (4) – 3

20. Let A(4, –4) and B(9, 6) be points on the parabola, y 2 = 4x. Let C be chosen on the arc AOB of the
parabola, where O is the origin, such that the area of ACB is maximum. Then, the area (in sq. units) of

RI
ACB, is : [JEE(Main) 2019, Online (09-01-19),P-2 (4, – 1), 120]

1 3 1
(1) 30 (2) 31 (3) 31 (4) 32
2 4 4

A
A helicopter is flying along the curve given by y – x3/2 = 7, (x  0). A soldier positioned at the point

UH
21.
1 
 2 ,7  wants to shoot down the helicopter when it is nearest to him. Then this nearest distance is :
 

[JEE(Main) 2019, Online (10-01-19),P-2 (4, – 1), 120]


JA
1 1 7 1 7 5
(1) (2) (3) (4)
2 3 3 6 3 6

x (d − x)
22. Let f(x) = – , x  R, where a, b and d are non-zero real constant. Then :
2
a +x 2
b + (d − x)2
2
LP

(1) f is neither increasing nor decreasing function of x

(2) f is an increasing function of x

(3) f is not a continuous function of x


A

(4) f is a decreasing function of x [JEE(Main) 2019, Online (11-01-19),P-2 (4, – 1), 120]
NK
SA
EXERCISE # 1

PART - I
Section (A) :
 
A-1. (i)_ y = 4x + 5 (ii) y + x = 2 (tangent), y = x (normal) (iii)_ 16x + 13y = 9a
(iv) y=0

RI
A-2 (i) 2x + y = 4, y = 2x (ii) x+y=3
 
 8
A-3. (i)  4, 3  (ii). (9/4, 3/8) (iii) (1, –1) , (–1, –5)
 

A
 
A-4_. (i) y = 12x – 16 or y = 12x + 16 (ii) x – 2y + 1 = 0 or 2y + x – 1 = 0
 

UH
A-5. (i) a = 1, b = –2 (ii) 1

Section (B) :
 
 e  3
B-1. 1 B-2. 45° at (1, 0) and tan–1  2  at (e, 1) B-3. 90° B-5.
JA
e +2 2
B-6. 20 − 1

Section (C) :
C-1. (i) – 2 cm/min (ii) 2 cm2/min C-2. 2x2 – 3x + 1
LP

C-3. (i) 2 km/hr (ii) 6 km/h C-4. 7.5 m3

Section (D) :
 

D-2. (i) M.D. in (– , –3]


A

M.I. in [– 3 , 0]
M.D. in [0 , 2]
M.I. in [2 , ) 
NK

 1   1 
(ii) M.D. in  0,  ; M.I. in  ,  D-4. a  R+
 3  3 

D-5. (i) Neither increasing nor decreasing at x = –1 and increasing at x = 2


SA

(ii) at x = – 2 decreasing
at x = 0 decreasing
at x = 3 neither increasing nor decreasing
at x = 5 increasing  
  (iii) Strictly increasing at x = 0 (iv) Strictly increasing at x = 2, neither I nor D at x = 1
(v) Strictly increasing at x = 0  
 
D-8. b  (0, e]
 
1
D-9. (i) local max at x = 1, local min at x = 6 (ii) local max. at x = – , local min. at x = – 1
5
1
(iii) local mini at x = , No local maxima
e
4
D-10. (i) local maxima at x = log2 and local minima at x = 1 (ii) local min at 0, local max at 2
3
2 
(iii) local max at x = 0, , local min at x = , 
3 2
(iv) local maxima at –1 and local minma at 0 (v) local minima at x = ± 2 , 0
 
D-11. local max at x = 1, local min at x = 2.
 
Section (E) :
 

RI
E-1. (i) max = 8, min. = – 8 (ii) max = 2 , min = – 1
(iii) max. = 8, min. = – 10 (iv) max. = 25, min = – 39
(v) max. at x = /6, max. value = 3/4; min. at x = 0 and /2, min. value = 1/2

A
E-3. 1, 3 (respective)
4  r3 220
E-4. F = 191 E-6. E-8. 110 m , m E-9. 32 sq. units
3 3 

UH
E-10. 12cm, 6 cm

Section (F) :
F-1. 1
JA
 
PART - II
 
Section (A) :
A-1. (A) A-2. (C) A-3. (C) A-4. (B) A-5. (C) A-6. (D) A-7. (B)
A-8. (A) A-9. (A)
LP

Section (B) :
B-1. (B) B-2. (D) B-3. (D) B-4. (C) B-5. (B) B-6. (D)
A

Section (C) :
C-1. (B) C-2 (C) C-3. (A) C-4. (B) C-5. (C)
NK

Section (D) :
D-1. (A) D-2. (B) D-3. (A) D-4. (C) D-5_. (D) D-6. (C) D-7. (C)

Section (E) :
E-1. (C) E-2. (D) E-3. (D) E-4. (C) E-5. (A) E-6. (C)
SA

Section (F) :
F-1. (A) F-2. (C) F-3. (B) F-4. (D) F-5. (C)  
 
PART – III
1. (A → p, q, r,s); (B → r) ; (C → p,q,r,s); (D →q) 2. (A) → (p), (B) → (s), (C) → (q), (D) → (r)
3. (A) → (p,q), (B) → (r,s), (C) → (r,s), (D) → (r,s)
4. (A) → (r), (B) → (s), (C) → (q), (D) → (p)
EXERCISE # 2
PART-I
1. (A) 2. (D) 3. (A) 4. (B) 5. (C) 6. (B) 7. (B)
8. (B) 9. (C) 10. (D) 11. (C) 12. (A) 13. (D) 14. (C)
15. (A) 16. (C) 17. (D) 18. (D) 19. (A) 20. (B) 21. (B)
22. (A) 23. (B) 24. (C) 25. (A) 26. (B)

PART - II

RI
1. 2 2. 24 3. 3 4. 15 5. 66 6. 2 7. 0
8. 0 9. 29 10. 4 11. 4 12. 1 13. 8 14. 1

A
15. 32 16. 36 17. 81 18. 39 19. 40 20. 3 21. 76

PART - III

UH
1. (CD) 2. (ABC) 3. (AB) 4. (AD) 5. (BC) 6. (AB) 7. (AD)
8. (AD) 9. (AB) 10. (ABC) 11. (AD) 12. (ACD) 13. (BD) 14. (AB)
15. (AC) 16. (ABC) 17. (ACD) 18. (AC) 19. (BD) 20. (BC)
JA
21. (ABC) 22. (AC) 23. (ACD)

PART - IV
1. (A) 2. (C) 3. (D) 4. (C) 5. (B) 6. (A) 7. (C)
8. (B) 9. (C) 10. (D) 11. (A)
LP

EXERCISE # 3

PART - I
1*. (BCD) 2. 0 3. 1 4. (D) 5. (A)→(s), (B)→(t), (C)→(r), (D)→(r)
A

6. 2 7. 9 8. 5 9. (C) 10. (AC) 11. 9 12*. (AB)

13. (D) 14. (C) 15. (A) 16*. (BD) 17. 8 18. 4 19*. (BC)
NK

 
20. (A,D) 21 . (D) 22. (C) 23. (C) 24. (B) 25. (A,C) 26. (BC)

27. (ABD)
PART - II
SA

1. (2) 2. (1) 3. (3) 4. (4) 5. (3) 6. (2) 7. (2)

8. (3) 9. (2) 10. (4) 11. (2) 12. (1) 13. (2) 14. (1)

15. (3) 16. (2) 17. (3) 18. (2) 19. (2) 20. (3) 21. (3)

22. (2)
1. A particle moving on a curve has the position at time t given by
x = f'(t) sin t + f''(t) cos t, y = f'(t) cos t - f''(t) sin t, where f is a thrice differentiable function. Then prove
that the velocity of the particle at time t is f'(t) + f'''(t).

2. Find the interval in which f(x) = x 4ax − x2 (a < 0) is decreasing


 
3. f : [0, 4] → R is a differentiable function. Then prove that for some a, b  (0, 4) , f 2(4) – f2(0) = 8f(a) .
f(b)

RI
a 2
4. If all the extreme value of function f(x) = a 2x3 –
x – 2x – b are positive and the minimum is at the
2
1 −11 1

A
point x0 = then show that when a = –2  b < and when a = 3  b < –
3 27 2

 3+ | x − k | , x  k

UH

5. If f(x) =  2 sin (x − k) has minimum at x = k , then show that |a| > 2
a − 2 + x −k
, x  k

6. The equation x3 – 3x + [a] = 0, where [.] denotes the greatest integer function, will have three real and
distinct roots then find the set of all posible values of a.
JA
{x} {x}
7. Let f(x) = sin + cos where a > 0 and { . } denotes the fractional part function. Then find the set
a a
of values of 'a' for which f can attain its maximum values.

8. Find the values of the parameter ‘k’ for which the equation x 4 + 4x3 – 8x2 + k = 0 has all roots real.
LP

Comprehension (Q. No. 9 to 11)

A function f(x) having the following properties;


(i) f(x) is continuous except at x = 3
A

(ii) f(x) is differentiable except at x = – 2 and x = 3


(iii) f(0) = 0, lim f(x) → – , lim f(x) = 3, lim f(x) = 0
x →3 x →−  x→ 

(iv) f (x) > 0  x  (–, – 2)  (3, ) and f (x)  0  x  (– 2, 3)


NK

(v) f (x) > 0  x  (– , – 2)  (– 2, 0) and f (x)  0  x  (0, 3)  (3, )


then answer the following questions

9. Find the Maximum possible number of solutions of f(x) = | x |

10. Show that graph of function y = f (– | x |) is continuous but not differentiable at two points, if f (0) = 0
SA

11. Show that f(x) + 3x = 0 has five solutions if f (0) > – 3 and f(– 2) > 6

12. Let F(x) = (f(x))2 + (f(x))2, F(0) = 7, where f(x) is thrice differentiable function such that
|f(x)|  1  x  [–1, 1], then prove the followings.
(i) there is atleast one point in each of the intervals (–1, 0) and (0, 1) where |f(x)|  2
(ii) there is atleast one point in each of the intervals (–1, 0) and (0, 1) where F(x)  5
(iii) there exits atleast one maxima of F(x) in (–1, 1)
(iv) for some c  (–1, 1), F(c)  7, F(c) = 0 and F(c)  0

13. A figure is bounded by the curves, y = x 2 + 1, y = 0, x = 0 and x = 1. At what point (a, b), a tangent
should be drawn to the curve y = x 2 + 1 for it to cut off a trapezium of the greatest area from the figure.
ax + b
14. If y = has a turning value at (2, −1) find a and b, show that the turning value is a
(x − 1) (x − 4)
maximum.

15. With the usual meaning for a, b, c and s, if  be the area of a triangle, prove that the error in  resulting
from a small error in the measurement of c, is given by
 1 1 1 1 
d =  + + −  dc
4 s s−a s−b s−c

16. Find the possible values of 'a' such that the inequality 3 – x 2 > |x – a| has atleast one negative solution

RI
17. If (m – 1) a12 – 2m a2 < 0, then prove that xm + a1 xm – 1 + a2 xm – 2 + ..... + am–1 x + a0 = 0 has at least one
non real root (a1, a2, ....., am  R)
 

A
18. If f '(x) > 0, f''(x) > 0  x  (0, 1) and f(0) = 0, f(1) = 1, then prove that f(x) f –1 (x) < x2  x  (0, 1)

 x2   27 

UH
19. Find the interval of increasing and decreasing for the function g(x) = 2f   + f  − x 2  , where
 2  2 
f "(x) < 0 for all x  R.
 
20. Using calculus prove that H.M  G.M.  A.M for positive real numbers.
JA
21. Prove the following inequalities
(i) 1 + x2 > (x sinx + cosx) for x  [0, ).
 
(ii) sin x – sin 2x  2x for all x  0,
 3 
LP

x2
(iii) + 2x + 3  (3 – x)ex for all x  0
2
sin2 x 1 
(iv) 0 < x sinx – < ( – 1) for 0 < x <
2 2 2
A

 21 − 4b − b2 
22. Find the interval to which b may belong so that the function f(x) =  1 −  x3 + 5x + 6 is
 b +1 
 
NK

increasing at every point of its domain.

x2 1 d2 y
23. If 0 < x < 1 prove that y = x n x – + is a function such that > 0. Deduce that
2 2 dx 2
x2 1
SA

x n x > – .
2 2

24. Find positive real numbers ‘a’ and ‘b’ such that f(x) = ax – bx3 has four extrema on [–1 , 1] at each of
which | f(x) | = 1

sin A sinB sin C


25. For any acute angled ABC , find the maximum value of + +
A B C

26. Suppose p,q,r,s are fixed real numbers such that a quadrilateral can be formed with sides p,q,r,s in
clockwise order. Prove that the vertices of the quadrilateral of maximum area lie on a circle .
 
5a2 3
27. For what real values of ‘a’ and ‘b’ all the extrema of the function f(x) = x + 2ax2 – 9x + b
3
−5
are positive and the maximum is at the point x 0 =
9

28. Find the minimum value of f(x) = 8 x + 8– x – 4(4x + 4– x),  x  R

29. Using calculus , prove that log23 > log35 > log47.

30. Show that the volume of the greatest cylinder which can be inscribed in a cone of height ' h ' and semi −

RI
4
vertical angle  is  h3 tan2 .
27

31. Find the cosine of the angle at the vertex of an isosceles triangle having the greatest area for the given
constant length  of the median drawn to its lateral side .

A
32. A tangent to the curve y = 1 − x2 is drawn so that the abscissa x 0 of the point of tangency belongs to the
interval (0, 1]. The tangent at x 0 meets the x−axis and y−axis at A & B respectively. Then find the

UH
minimum area of the triangle OAB, where O is the origin
 
33. A cone is made from a circular sheet of radius 3 by cutting out a sector and keeping the cut edges of
the remaining piece together. Then find the maximum volume attainable for the cone
JA
    a  
34. Suppose velocity of waves of wave length  in the Atlantic ocean is k   +    , where k and a
 a     
are constants. Show that minimum velocity attained by the waves is independent of the constant a.
 
35. Find the minimum distance of origin from the curve ax 2 + 2bxy + ay2 = c where a > b > c > 0
LP

36. Prove that ex + 1 + e2x  (1 + x) + 2 + 2x + x2  x  R

tan−1 x
37. Find which of the two is larger n (1 + x) or .
1+ x
A

 
 
38. Let f (sinx) < 0 and f (sin x) > 0,  x   0,  and g(x) = f(sin x) + f(cos x), then find the intervals of
 2
NK

monotonicity of g(x).

39. If f(x) = (2013)x2012 – (2012)x2011 – 2014x + 1007, then show that for x  [0, 10071/2011], f(x) = 0 has at
least one real root.
 
SA

f(x)
40. A function f is differentiable in the interval 0  x  5 such that f(0) = 4 & f(5) = – 1. If g(x) = , then
x +1
5
prove that there exists some c  (0, 5) such that g(c) = – .
6

41. Let f(x) and g(x) be differentiable functions having no common zeros so that f(x) g(x)  f(x) g(x). Prove
that between any two zeros of f(x), there exist atleast one zero of g(x).
 
f(a) f(b)
42. f is continuous in [a, b] and differentiable in (a, b) (where a > 0 ) such that = . Prove that there
a b
f(x 0 )
exist x0  (a, b) such that f(x0) = .
x0

43. If (x) is a differentiable function x  R and a  R+ such that (0) = (2a), (a) = (3a) and (0)  (a)
then show that there is at least one root of equation (x + a) = (x) in (0, 2a)
 
44. Find the set of values of the parameter 'a' for which the function ;
f(x) = 8ax – a sin 6x – 7x – sin 5x increases & has no critical points for all x  R, is
 

RI
45. Let h be a twice differentiable positive function on an open interval J. Let
g(x) = n (h(x))  x  J
Suppose (h'(x)) > h''(x) h(x) for each x  J. Then prove that g is concave downward on J.
2

A
a x3
46. If the complete set of value(s) of 'a' for which the function f (x) = + (a + 2) x2 + (a − 1) x + 2
3
possess a negative point of inflection is (− , )  (, ), then || + || is :

UH
5
47. If two curves y = 2sin x and y = x2 – 3x + 2 + 1 touch each other at some point then the value
6
3  18 
is  0  x 
of
JA
5  5 
LP

 4
2. [4a, 3a] 6. a  [–1, 2) 7.  0,   8. k  [0,3] 9. 3
 
1 5  13 
13. 2 , 4 14. a = 1, b = 0 16. a  − , 3
   4 
g(x) is increasing if x  (–, 3]  [0, 3] ; g(x) is decreasing if x  [–3, 0]  [3, )
A

19.
 
9 3
22. [– 7, – 1)  [2, 3] 24. a=3 , b=4 25.
NK

2

−9 36 81 400
27. If a = , then b > ; If a = then b >
5 5 25 243
SA

4 3 c
28. –10 31. cos A = 0.8 32. 33. 2/3 35.
9 a+b

   
37. n (1 + x) 38. Increasing when x   ,  , decreasing when x   0,  .
 4 2   4

44. a  (6, ) 46. 2 47. ½


DEFINITE INTEGRATION & ITS APPLICATION

Newton-Leibnitz formula.
b
d
Let
dx
(F(x)) = f(x)  x  (a, b). Then  f(x)
a
dx = lim− F(x) – lim+ F(x).
x →b x →a

b
Note : 1. If a > b, then  f(x)
a
dx = lim+ F(x) – lim− F(x).
x →b x →a

2. If F(x) is continuous at a and b, then = F(b) – F(a)

RI
2
dx
Example # 1 : Evaluate 
1
(x + 1)(x + 2)
1 1 1
Solution :  = – (by partial fractions)

A
(x + 1)(x + 2) x +1 x+2
9
2
dx
  n(x + 1) − n(x + 2)1 = n3 – n4 – n2 + n3 =
2
= n  
(x + 1)(x + 2) 8

UH
1

Self practice problems :


Evaluate the following
 
2
5x 2 4 3
x
  ( 2sec ) 
(1) dx (2)
JA 2
x + x 3 + 2 dx (3) dx
1 x 2 + 4x + 3 0 0
1 + sec x
5  5 3 4  2   2 
Ans. (1) 5 –  9 n − n  (2) + +2 (3) – + 2 n  
2  4 2 1024 2 18 3 3  3

b b

Property (1)  f(x) dx =  f(t) dt


LP

a a
i.e. definite integral is independent of variable of integration.

b a

Property (2)  f(x) dx = –  f(x) dx


A

a b

b c b

Property (3)  f(x) dx =  f(x) dx +  f(x) dx, where c may lie inside or outside the interval [a, b].
NK

a a c

 x+3 : x 3 5

Example # 2 : If f(x) =  2
3x + 1 : x  3
, then find  f(x)
2
dx.

3
5 3 5 3
 x2  5
5
2 2 3 2 3  + 3x  +  x + x  3
SA

3 2
Solution f(x) dx = f(x) dx + f(x) dx = (x + 3) dx + (3x + 1) dx =
2 2
9−4 211
= + 3 (3 – 2) + 53 – 33 + 5 – 3 =
2 2

Example # 3 : Evaluate  | x − 5 | dx.


2
8 5 8

Solution :  | x − 5 | dx =  (− x + 5) dx +
2 2
 (x – 5) dx
5
=9
2 5 2

Example # 4 : Show that  (2x + 1) dx =  (2x + 1) +  (2x + 1)


0 0 5
Solution : L.H.S. = x2 + x ]20 = 4 + 2 = 6 ; R.H.S. = 25 + 5 – 0 + (4 + 2) – (25 + 5) = 6
 L.H.S. = R.H.S

Self practice problems :


Evaluate the following
4 4
(4)  (| x – 1| + | x – 3 |)dx
0
(5)  [x]dx , where [x] is integral part of x.
–2
9
 
(6) 
0
 t  dt.
Ans. (4) 10 (5) 3 (6) 13

RI
 a
−a f(x) dx = 0 (f(x) + f( −x)) dx =  0
a a
2 f ( x ) dx , if f( −x) = f(x) i.e. f(x) is even
Property (4)

 0 , if f( − x) = −f(x) i.e. f(x) is odd

A
3x + 3− x
1

Example # 5 : Evaluate 
−1 1 + 3x
dx

UH
1
3x + 3− x
1
 3x + 3− x 3− x + 3x  1
 3 x + 3− x 3x (3− x + 3 x ) 
Solution : 
−1 1 + 3x
dx = 
0

 1+ 3
x
+  dx =
1 + 3− x  
0

 1+ 3
x
+
1 + 3x


1
1
 3 3   3 x
3   1 –x
1  1  1 8
–1
= 
0
(3 x + 3− x ) dx = 
 n3
–  = 
n3 0  n3
–  – 
n3   n3
– 
n3 
= 
n3 
3– =
3  3 n3
JA

2
Example # 6 : Evaluate 

cos x dx.

2
 
2 2


LP

Solution : cos x dx = 2  cos x dx = 2 ( cos x is even function)


 0

2

2−x
1

Example # 7 : Evaluate 
loge   dx.
A

−1 2+x
2−x 2+x 2−x
Solution : Let f(x) = loge    f(–x) = loge   = – loge   = – f(x)
2+x 2−x 2+x
NK

2−x
1

i.e. f(x) is odd function  −1loge  2 + x  dx = 0

Self practice problems :


Evaluate the following
SA

 
1 2 4
sec xdx
| x |  sin 
7
(7) dx. (8) x dx. (9) dx.
−1   1 + 2x
− −
2 4

Ans. (7) 1 (8) 0 (9) n ( )


2 +1

b b a a

Property (5) 
a
f(x) dx = 
a
f(a + b − x) dx. Further 
0
f(x) dx = 
0
f(a − x) dx
 
2
g (sin x) 2
g (cos x) 
Example # 8 : Prove that 
0
g (sin x) + g (cos x)
dx = 
0
g (sin x) + g (cos x)
dx =
4
.

   

g  sin  − x  
 2 
2 2
g (sin x)
Solution : Let  =  g (sin x) + g (cos x)
dx   =       
dx
0 0
g  sin  − x   + g  cos  − x  
 2   2 

2
g (cos x)
0
= 
g (cos x) + g (sin x)
dx

on adding, we obtain

RI
 
2
 g (sin x) g (cos x)  2

2 = 
0

 g (sin x) + g (cos x)
+
g (cos x) + g (sin x)
 dx =


0
dx =
4

A
Self practice problems:
Evaluate the following

 2
x x
0 1 + sin x dx.  sin x + cos x

UH
(10) (11) dx.
0
 5
2 12
x sin x cos x dx
(12) 0 sin4 x + cos4 x dx. (13)  1+
 cot x
12
JA
 2 
Ans. (10)  (11)
2 2
(
loge 1 + 2 ) (12)
16
(13)
6
 a
0 f(x) dx = 0 (f(x) + f(2a − x)) dx =  0
2a a
2 f ( x ) dx , if f(2a − x) = f(x)
Property (6)

 0 , if f(2a − x) = −f(x)
LP

Example # 9 : Evaluate 
0
cot x.cos 2x dx

Solution : Let f(x) = cotx cos2x


 f( – x) = cot(–x) cos2(–x) = –cotx cos2x = – f(x)

A

 
0
cot x cos 2x dx = 0
NK


dx
Example # 10 : Evaluate 
0 1 + 3cos2 x
dx.

1 dx
Solution : Let f(x) =
1 + 3cos2 x
 f( – x) = f(x)  
0 1 + 3cos2 x
   
SA

sec x dx  −1  tan x  
2 2 2 2 2 2
dx sec x dx
=2 
0 1 + 3cos2 x
=2 
0 1 + tan2 x + 3
=2 
0 4 + tan2 x
=  tan 

 
 2  0
  tan x  –1  tan0 
  tan is undefined, we take limit = lim tan–1   – tan  2  = /2 – 0 = /2
2 x → / 2–
 2   

 (cot
–1
Example # 11 : Evaluate : x)2 dx
0

Let  =  (cot   dx = – cosec2 d


–1
Solution : x)2 dx Let x = cot
0

0 2
 =   (–co sec )d  =   (co sec )d
2 2 2 2

 0
2
 
/2 2 2
= (  (– cot ))
2
+ 2  cot  d   = 0 + 2  cot  d
0
0 0

  S tandard result
/2
2    
= ( 2 n sin  )0 –2  nsin  d 2 –
= 0 – 2 ×  –  n2 = n2.
 2
0   nsin  d = n2
 0 2
Self practice problems :

RI
Evaluate the following
  n
(1 + x 2 )  dx. 
tan−1 x
1
 
(14) 0  1 + x 2 
 (15)  x(1 + x 2 )
dx (16)  n sin  x 
2 
dx.
  0 0

A

Ans. (14)  n2 (15) n2 (16) – n2
2

UH
Property (7) If f(x) is a periodic function with period T, then
nT T
(i)  f(x)
0
dx = n  f(x) dx, n  z
0
a + nT T
(ii)  f(x) dx = n  f(x) dx, n  z, a  R
JA
a 0
nT T
(iii) 
mT
f(x) dx = (n – m)  f(x)
0
dx, m, n  z
a + nT a
(iv)
nT
 f(x) dx =  f(x)
0
dx, n  z, a  R
LP

b + nT b

(v) 
a + nT
f(x) dx =  f(x)
a
dx, n  z, a, b  R

e
{x}
Example # 12 : Evaluate dx , where {.} denotes the fractional part function.
A

−3
5 1 1
1
e dx = (5 – (–3)) e {x} dx = 8  e x dx = 8 e x ( )
{x}
Solution : = 8 (e –1)
0
−3 0 0
NK

1000 n

Example # 13 : Evaluate 
n =1
 | cos 2x |dx
n–1
1 2 1000 1000

Solution :  | cos 2x | dx +  | cos 2x | dx + ........+


0 1

999
| cos 2x | dx = 
0
| cos 2x | dx
SA

Now |cos2x| is a periodic function of period 1/2


1
2
I = 2000  | cos 2x | dx
0
 I = 2000 × 2 = 4000

Self practice problems :


Evaluate the following

41
2
2x – 2x 
(17) e
−1
dx , where [•] denotes the greatest integer function.
14  3
3 2
(18) 
0
| sin x | dx (19) 

(sin4 x + cos4 x)dx

43 19 3
Ans. (17) (e – 1) (18) (19)
2 2 8

h( x )
dF(x)
Leibnitz Theorem : If F(x) = 
g( x )
f(t) dt , then
dx
= h(x) f(h(x)) – g(x) f(g(x))

h( x )

Proof : Let P(t) =  f(t) dt  F(x) = 


g( x )
f(t) dt = P(h(x)) – P(g(x))

dF(x)

RI
 = P(h(x)) h(x) – P(g(x)) g(x) = f(h(x)) h(x) – f (g(x)) g(x)
dx

x2

Example # 14 : If F(x) =  tan t dt , then find F(x).

A
x

Solution : F(x) = 2x . tan x2 – 1. tan x

UH
x3
1
Example # 15 : If F(x) =  2 nt
dt then find F’(e)
x

3x 2 2x x2 x x(x – 1) e(e – 1)
Solution : F’(x) = – = – = now F’(e) = = e(e–1)
nx 3 nx 2
nx nx nx ne
JA
x2
sin t tan tdt
Example # 16 : Evaluate : Lim+
0 x4 x →0

Solution : Applying L’ hospital rule
2x sin x tan x 1  sin x   tan x  1
Lim  Lim   =
2  x  x  2
LP

3 →
x →0 +
4x x 0

Example # 17 : Let f(x) =  (t – 1)(t – 2)2 dt, then find a point of minimum
0
x
A

 (t – 1)(t – 2)
2
Solution : f(x) = dt
0

f’(x) = (x – 1) (x – 2)2
NK

– + +
– 1 2 
 x = 1 is the point of minimum
1
1 5 17 17
f(1) =  (t 3 – 5t 2 + 8t – 4)dt = – +4 –4 = – . Hence (1, – ) is a point of minimum
4 3 12 12
SA

xb − 1
1

Example # 18 : Evaluate ,  0
nx
‘b’ being parameter.

xb − 1 d (b)
1 1
xb nx
Solution : Let (b) = 
0
nx
dx 
db
= 
0
nx
dx + 0 – 0 (using modified Leibnitz Theorem)

1
x 
1 b +1
= 
0
x b dx =  =  (b) = n (b + 1) + c
b + 1 0
b=0  (0) = 0  c = 0  (b) = n (b+1)
tan−1(ax)
1
Example # 19 : Evaluate 
0 x 1 − x2
dx , ‘a’ being parameter.

tan−1(ax)
1
d (a)
1 1
x 1 dx
Solution : Let (a) =  dx  =  dx = 
x 1− x 0
2 da 0 (1 + a2 x 2 ) x 1 − x 2 0 (1 + a x 2 ) 1 − x 2
2

Put x = sin t  dx = cos t dt


L.L. : x = 0  t=0

U.L. : x = 1  t=
2
 

d(a) 2
1 1 2
dt
da
= 
0 1 + a2 sin2 t cos t
cos t dt = 
0 1 + a2 sin2 t

RI


2
sec 2 t dt 1 
tan t  =
1
= 
0 1 + (1 + a2 )tan2 t
=
1+ a 2
tan
–1
( 1+ a 2 2
 0) 1+ a 2
.
2
 

A
 (a) =
2
( )
n a + 1 + a2 + c But (0) = 0  c = 0  (a) =
2
(
n a + 1 + a2 )
Self Practice Problems :

UH
x3

(20) If f(x) = 
0
cos t dt, find f(x).

x3
dt
(21) Find the equation of tangent to the y = F(x) at x = 1 , where F(x) =  1+ t4
JA x

x x3
f(t)
(22) If  f(t)dt
0
=x – 2
 x
t
dt then find f(1)

x2

x
2
(23) If f(x) = nt dt then find f’(e)
x
4x 2
d2 y
LP

te
4 4t
(24) If y = dt , Find
4 dx 2
x2
d2 y
(25) If y = 
0
n(1 + t)dt , then find
dx 2
A

2
x (1+ x) 

(26) If 
0
f(t)dt = x then find f(2) (27) Evaluate 
0
n (1 + bcos x) dx, ‘b’ being parameter.
NK

Ans. (20) 3x2 cos x3 (21) 2x–y= 2 (22) 2/3


16x2
(23) e (6e–1)
2
(24) 2048 e
2 1  1 + 1 − b2 
(25) [2x2 + (1 + x2)n(1 + x2)] (26) (27)  n 
1 + x2 5  2 
 
SA

Reduction formulae in definite Integrals:



2
 n − 1
1. If n = 
0
sinn x dx , then show that n = 
 n 
 n–2

2
Proof : n = 
0
sinn x dx

 
 2 2
n =  − sinn −1 x cos x  +  (n − 1) sinn − 2 x . cos2 x dx = (n – 1)  sinn − 2 x . (1 − sin2 x) dx
2
0
0 0
 
2 2

  sinn x dx  n + (n – 1) n = (n – 1) n–2
n−2
= (n – 1) sin x dx − (n − 1)
0 0

 n − 1
n =   n–2
 n 
 
2 2

 sin x dx =  cosn x dx
n
Note : 1.
0 0

 n − 1  n − 3   n − 5  
2. n =       ..... 0 or 1 according as n is even or odd. 0 = , 1 = 1
 n  n−2 n−4 2
 n − 1  n − 3  n − 5   1 
 n  n − 2  n − 4  ........  2  . 2 , if n is even

RI
     
Hence n = 
 n −1 n − 3 n − 5
  





2
........   . 1 , if n is odd
  n  n − 2  n − 4 

A
3

4
1
2. If n =  tann x dx , then show that n + n–2 =
n −1

UH
0
 
4 4
Proof : n = 
0
(tan x)n − 2 . tan2x dx = 
0
(tan x)n − 2 (sec2x – 1) dx

  
4 4
 (tan x)n −1  4
JA
= 
0
(tan x)n − 2 sec2x dx – 
0
(tan x)n − 2 dx = 
 n − 1 0
 – n–2

1 1
n = – n–2  n + n–2 =
n −1 n −1

2
m −1
3. If m,n =  sinm x . cosn x dx , then show that m,n = 
LP

0
m + n m–2, n
  
2
 sinm −1 x . cosn +1 x  2 2
cosn+1 x
Proof : m,n = 
0
sinm −1 x (sin x cosn x) dx =  −
 n +1
 +
0

0
n +1
(m – 1) sinm–2 x cos x dx

 
A

 m − 1 2
 m − 1 2

 sinm − 2 x.cosn x.cos2 x dx =    ( sin x.cos x − sin x.cos x )


m−2 n m n
=   dx
 n +1  0  n +1  0
 m − 1  m − 1  m − 1  m − 1
NK

=   m–2,n –  n + 1  m,n   1 + n + 1  m,n =  n + 1  m–2,n


 n + 1       
 m −1
m,n =   m–2,n
m+n
 m −1  m−3   m−5 
Note : 1. m,n =   m+n−2 m+n−4 ........ 0,n or 1,n according as m is even or odd.
SA

m+n    
 
2 2
1
0,n = 
0
cosn xdx and 1,n = 
0
sin x.cosn xdx =
n +1

2. Walli’s Formula
 (m − 1) (m − 3) (m − 5) .........(n − 1) (n − 3) (n − 5)....... 
 when both m, n are even
(m + n) (m + n − 2) (m + n − 4)........ 2


m,n = 
 (m − 1) (m − 3) (m − 5) .........(n − 1) (n − 3) (n − 5).......
 otherwise

 (m + n) (m + n − 2) (m + n − 4)........


2
Example # 20 : Evaluate

 sin2 x cos2 x(sin x + cos x) dx .

2
 
2 2

 

RI
Solution : Given integral = sin3 x cos2 x dx + sin2 x cos3 x dx
 
− −
2 2

2
= 0 + 2  sin2 x cos3 x dx

A
( sin3x cos2x is odd and sin2x cos3x is even)
0

1.2 4
= 2. =
5.3.1 15

UH

Example # 21 : Evaluate 
0
x sin7 x cos6 x dx

Solution : Let  =  x sin7 x cos6 x dx
JA
0
  
= 
0
(  − x) sin7 (  − x) cos6 (  − x) dx =  
0
sin7 x cos6 x dx – 
0
x sin7 x cos6 x dx

2
6.4.2.5.3.1 48
 2 =  . 2  sin7 x.cos6 xdx   = 
13.11.9.7.5.3.1
 =
9009
LP

x
5/2
Example # 22 : Evaluate : a – xdx
0

Solution : Put x = a sin2  dx = 2a sin cos d


A

Lower limit : x = 0  =0



Upper limit x = a = .
2
NK


a 2
 (5.31)(1) 5a 4
 x a – xdx =  2a sin  cos  d = 2a x
5/2 4 6 2 4
. =
0 0
2 8.6.4.2 128
SA

Self Practice Problems:


Evaluate the following
 
2 2 1

(28) 
0
sin11 x dx . (29) 
0
sin5 x cos4 x dx . (30) 
0
x 6 sin−1 x dx

.
a 7 2

 (a − x2 )  2 − x dx.
2
(31) x 2
dx . (32) x3 / 2
0 0

128 8  16 a9 
Ans. (28) (29) (30) – (31) (32)
693 315 14 245 9 2
Property (8) If (x)  f(x)   (x) for a  x  b, then
b b b

 (x)
a
dx   f(x)
a
dx   (x)
a
dx

b
Property (9) If m  f(x)  M for a  x  b, then m (b – a)   f(x) dx  M (b – a)
a
b
Further if f(x) is monotonically decreasing in (a, b), then f(b) (b – a) <  f(x)
a
dx < f(a) (b – a) and if f(x) is
b
monotonically increasing in (a, b), then f(a) (b – a) <  f(x) dx < f(b) (b – a)
a

RI
b b

Property (10)  f(x) dx


a
 
a
f(x) dx

A
b

Property (11) If f(x)  0 on [a, b], then  f(x)


a
dx  0

UH
1 1 1
Example # 23 : For x  (0, 1) arrange f1(x) = , f2(x) = and f3(x) = in ascending order
9 – x2 9 – 2x 2 9 − x2 − x3
1
1 1 1
and hence prove that n2 <  dx < n5
0 9−x −x
2 3
6 6 2
Solution :  0 < x3 < x2 , for all x  (0,1)  x2 < x2 + x3 < 2x2
JA
 –2x2 < – x2 – x3 < –x2  9 – 2x2 < 9 –x2 – x3 < 9 – x2
1 1 1
  
9 – x 2 9 – x 2 – x 3 9 – 2x 2
f1(x) < f3(x) < f2(x) for x  (0, 1)
1 1 1

  f1(x) dx <  f3 (x) dx <  f (x) dx


2
LP

0 0 0
1 1 1
dx dx dx
 0 9 – x2  0 9 – x 2 – x3  0 9 – 2x 2
1 1
1 3 + x  1  3 + 2x 
1
dx
  n  < 0 9 – x2 – x3 < 6 2  n 3 – 2x 
A

6  3 – x 0 0
1
1 1 1
 n2 <  dx < n5
0 9−x −x
2 3
6 6 2
NK

 5–x 
2
6
Example # 24 : Prove that 1 <   2 
dx 
0  
9 – x 5
SA

5–x
Solution : Let f(x) =
9 – x2
(x – 9)(x – 1)
 f(x) = –  f(x) = 0 or not defined  x = 1
(9 – x 2 )2
1
Then f(0) = 5/9, f(1) = , f(2) = 3/5 The greatest and least values of the integrand in the
2
1
interval [0,2] are respectively , equal to f(2) = 3/5 and f(1) =
2
   x 
2 2
1 5 – x 5 – 6
(2– 0) <  2 
dx < (2 – 0) 3/5 Hence 1 <   2 
dx 
09– x  0 9– x 
2 5
1

e
x2
Example # 25 : Estimate the value of dx using (i) rectangle, (ii) triangle.
0
Solution : (i) By using rectangle
1

e
x2
Area OAED < dx < Area OABC
0

e
x2
1< dx < 1 . e
0
1

e
x2
1< dx < e
0

RI
(ii) By using triangle
1

e
x2
Area OAED < dx < Area OAED + Area of triangle DEB
0
1 1
1 e +1
 e dx < 1 + e
2
x2

A
x
1< .1.(e – 1) 1< dx <
0
2 0
2

1 1

UH
e  e dx .
x2 x
Example # 26 : Estimate the value of dx by using
0 0
x2
Solution : For x  (0, 1), e <e x

1 1

 e dx <  e dx
2
 1×1< x x

0 0
JA
1

e
2
x
1< dx < e – 1
0

Self practice problems :


1 1

 e cos x dx < e
−x 2 − x2
(33) Prove the following : cos2 x dx
LP

0 0
 
2 2

 sin  sin
n +1 2
(34) Prove the following : 0 < x dx < x dx , n > 1
0 0
1 1

<  x x dx  1
A

e
(35) Prove the following : e
0
1
1 x 3 cos x 1
(36) Prove the following : –  0 2 + x 2 dx < 2
NK

2

2

(37) Prove the following : 1 < 
0
sin x dx <
2
/3
4 tan x 3 3

SA

(38) Prove the following : < dx <


 / 4
x 

Definite Integral as a limit of sum


Let f(x) be a continuous real valued function defined on the closed interval [a, b] which is divided into n
parts as shown in figure.
b−a
The point of division on x-axis are a, a + h, a + 2h ..........a + (n – 1)h, a + nh, where = h.
n
Let Sn denotes the area of these n rectangles.
Then, Sn = hf(a) + hf(a + h) + hf(a + 2h) + ........+hf(a + (n – 1)h)

RI
Clearly, Sn is area very close to the area of the region bounded by curve y = f(x), x–axis and the
ordinates x = a, x = b.
b
Hence  f(x) dx = Lt Sn

A
n→
a
b n −1 n −1
b−a  (b − a) r 
 f(x) dx = Lt
n→

r =0
h f(a + rh) = Lt
n→

r =0
 n  f a +
   n 

UH
a

Note :
1. We can also write
b−a  b−a 
b n
Sn = hf(a + h) + hf (a + 2h) + .........+ hf(a + nh) and  f(x) dx = Lt
n→

r =1
 n  f a +  n  r 
     
a
JA
n −1
r
1
1
2. If a = 0, b = 1,  f(x) dx = Lt
n→

r =0 n
f  
n 
0

Steps to express the limit of sum as definite integral :


LP

r 1
Step 1. Replace by x, by dx and Lt by 
n n n→

r
Step 2. Evaluate Lt   by putting least and greatest values of r as lower and upper limits
n→
n
respectively.
A

p
pn
1 r r r
For example Lt
n→

r =1 n
f   =
n
 f(x) dx ( Lt
n→
 
 n  r =1
= 0, Lt  
n→
 n  r = np
= p)
0
NK

 1 1 1 1 
Example # 27 : Evaluate Lt  + + + ......... +
n→
1 + n 2 + n 3 + n 10n 

 1 1 1 1  9n
SA

1
Solution : Lt
n →  1 + n

+ +
2+n 3+n
+ ......... +
10n 
= Lt
n→

r =1 r +n
9n 9
1 dx

1
  n (x + 1)0 = n 10
9
= Lt = =
n→
r =1 n r 0
x +1
n + 1
 

 n +1 n+2 n+3 1
Example # 28 : Evaluate Lt  2 2 + 2 + 2 + ......... +  .
 
2 2
n→ n + 1 n + 2 n + 3 n
r
1+
n
n+r n
1 r
Solution : Lt
n→
 n2 + r 2
= Lt
n→
 n
2
 Lt  
n→
n
= 0, when r = 1, lower limit = 0
r =1 r =1 n r
1+  
n
r n
and Lt   = Lt   = 1, when r = n, upper limit = 1
n→
n n→
n
1

1 1 1
1+ x 1 1 2x 1

0 1 + x2
dx = 
0 1+ x 2
dx +
2 
0 1 + x2
dx = [ tan–1x ]10 +  loge (1 + x 2 )
2 0
 1
= + n 2
4 2

RI
1
 (2n) !  n
Example # 29 : Evaluate : lim  n 
n→
n ! n 
1
 (2n)!  n 1  (2n)! 
 n y = lim

A
Solution : Let y = lim  n 
n  n 
n→ →
 n! n  n n  n! n 
1  2n(2n – 1)(2n – 2)....(n + 1) 
= lim n  
 nn 

UH
n→ n

n 1 1
1 x
= lim
n→
n
r =1
[n(1 + r/n)] =  n(1 + x)dx = (x n(1 + x))10 – 
1+ x
dx
0 0

= (x n(1 + x))10 – (x – n(1 + x))10 = n2 – (1– n2) =n4/e  y = 4/e


JA
Self Practice Problems :
Evaluate the following limits

1 n2 n2 1
(39) lim  + 3
+ 3
+ ..... + 
n → n
 (n + 1) (n + 2) 8n 
LP

 1 1 1 1
(40) lim  + + + ....... + 
n→ 1 + n 2 + n 3 + n 5n 

1  3  2 3 n 
(41) lim sin + 2sin3 + 3 sin3 + ........ + nsin3 
n2 
A

n→ 4n 4n 4n 4n 
n −1
1
(42) lim 
NK

n→
r =0 n − r2
2

1
  2 3 n  n
(43) lim tan tan tan ......tan 
n→ 
 2n 2n 2n 2n 
SA

3 2
Ans. (39) (40) n 5 (41) (52 – 15)
8 92

(42) (43) 1
2

Area Between The Curve :

Area included between the curve y = f(x), x-axis and the ordinates x = a, x = b
b

(a) If f(x)  0 for x  [a, b], then area bounded by curve y = f(x), x-axis, x = a and x = b is    f(x) dx
a
Example # 30 : Find the area enclosed between the curve y = x 2 + 2, x-axis, x = 1 and x = 2.

Solution :

RI
Graph of y = x2 + 2
2

A
2
 x3  13
( )
2
Area = x + 2 dx =  + 2x  =
1 3 1 3

UH
Example # 31 : Find area bounded by the curve y = n x + tan–1 x and x-axis between ordinates x = 1 and x = 2.
Solution : y = n x + tan–1x
dy 1 1
Domain  x > 0, = + >0
dx x 1 + x2

JA
y is increasing and x = 1, y =  y is positive in [1, 2]
4
2
 
2
1
 Required area = 
1
( n x + tan−1 x) dx =  x nx − x + x tan−1 x − n(1 + x 2 )
 2 1
1 1
= 2 n 2 – 2 + 2 tan–12 – n 5 – 0 + 1 – tan–1 1 + n 2
2 2
LP

5 1 
= n 2 – n 5 + 2 tan–12 – –1
2 2 4
Note : If a function is known to be positive valued then graph is not necessary.
A

Example # 32 : The area cut off from a parabola by any double ordinate is k times the corresponding rectangle
contained by the double ordinate and its distance from the vertex. Find the value of k ?
Solution : Consider y2 = 4ax, a > 0 and x = c
NK

c
8
Area by double ordin ate = 2 2 a x dx = a c3 / 2
0
3

Area by double ordinate = k (Area of rectangle)


8 2
a c 3 / 2 = k 4 a c3 / 2  k =
SA

3 3

(b) If f(x) < 0 for x  [a, b], then area bounded by curve y = f(x), x-axis, x = a and x = b is –
b


a
f(x) dx
Example # 33 : Find area bounded by y = log 1 x and x-axis between x = 1 and x = 2
2

Solution : A rough graph of y = log 1 x is as follows


2
2 2
Area = – 
1
log 1 x dx = –
2

1
loge x . log 1 e dx
2

= – log 1 e . [x loge x − x]12 = – log 1 e . (2 loge2 – 2 – 0 + 1) = – log 1 e . (2 loge 2 – 1)

RI
2 2 2

Note :  If y = f(x) does not change sign in [a, b], then area bounded by y = f(x), x-axis between
b

A
ordinates x = a, x = b is 
a
f(x) dx

(c) If f(x) > 0 for x  [a,c] and f(x) < 0 for x  [c,b] (a < c < b) then area bounded by curve y = f(x)

UH
c b

and x-axis between x = a and x = b is  f(x) dx − f(x) dx


a c

Example # 34 : Find the area bounded by y = x 3 and x- axis between ordinates x = –1 and x = 1
0 1
JA
 −x dx +  x dx
3 3
Solution : Required area =
−1 0
0 1
 x4   x3 
= −  +  
 4  −1  4 0

 1 1 1
= 0 – −  +
LP

–0=
 4 4 2

Note :   Most general formula for area bounded by curve y = f(x) and x- axis between ordinates x = a and x = b
b

is  | f(x) | dx
A

Area included between the curve x = g(y), y-axis and the abscissas y = c, y = d
(a) If g (y)  0 for y  [c,d] then area bounded by curve x = g(y) and y–axis between abscissa y = c
NK

and
d

y = d is
y =c
 g(y)dy
SA


Example # 35 : Find area bounded between y = cos –1x and y–axis between y = and y = .
2
Solution : y = cos–1 x  x = cos y

Required area = –  cos y dy

2
y


/2

–1 0 1 x

–1
Graph of y = cos x

= – sin y   = 1

RI
2

Example # 36 : Find the area bounded by the parabola x 2 = y, y-axis and the line y = 1.

A
Solution : Graph of y = x2

UH
1 1
2
Area OEBO = Area OAEO =  | x | dy =
0

0
y dy = 3

Example # 37 : Find the area bounded by the parabola x 2 = y and line y = 1.


JA
Solution :

Graph of y = x2
Required area is area OABO
1 1
4
= 2 area (OAEO) = 2 | x | dy = 2 y dy = .
LP

0 0 3
1 1 t
Example # 38 : For any real t, x = (et + e–t), y = (e – e–t) is point on the hyperbola x 2 – y2 =1.Show that the
2 2
area bounded by the hyperbola and the lines joining its centre to the points corresponding to t 1
and – t1 is t1.
A

Solution : It is a point on hyperbola x 2 – y2 = 1.


et1 + e – t1 et1 + e – t1
2 2
Area (PQRP) = 2  ydx = 2  x 2 – 1dx
NK

1 1
et1 + e– t1
x 1  2 e2t1 – e –2t1
=2  x2 – 1 – n(x + x 2 – 1) = – t1
2 2 1 4
1  et1 + e– t1   et1 – e – t1  e2t1 – e –2t1
SA

Area of OPQ = 2×     =
2 2   2  4
 Required area = area OPQ – area (PQRP) = t1

(b) If g (y)  0 for y  [c,d] then area bounded by curve x = g(y) and y–axis between abscissa y = c
d

and y = d is –  g(y)dy
y =c

Note : General formula for area bounded by curve x = g(y) and y–axis between abscissa y = c and
d
y = d is y = c
| g(y) | dy
Curve-tracing :
To find approximate shape of a curve, the following phrases are suggested :
(a) Symmetry:
(i) Symmetry about x-axis :
If all the powers of 'y' in the equation are even then the curve (graph) is symmetrical about the
x-axis.

E.g. : y2 = 4 a x.

RI
(ii) Symmetry about y-axis :
If all the powers of 'x' in the equation are even then the curve (graph) is symmetrical about the
y-axis.

A
UH
E.g. : x2 = 4 a y.
(iii) Symmetry about both axis :
If all the powers of 'x' and 'y' in the equation are even, then the curve (graph) is symmetrical
about the axis of 'x' as well as 'y' .
JA
E.g. : x2 + y2 = a2.
(iv) Symmetry about the line y = x :
If the equation of the curve remain unchanged on interchanging 'x' and 'y', then the curve
LP

(graph) is symmetrical about the line y = x.


A

E.g. : x3 + y3 = 3 a x y.
(v) Symmetry in opposite quadrants :
NK

If the equation of the curve (graph) remain unaltered when 'x' and 'y' are replaced by '–x' and '–
y' respectively, then there is symmetry in opposite quadrants.
SA

E.g. : xy = c2
(b) Find the points where the curve crosses the x-axis and the y-axis.
dy
(c) Find and equate it to zero to find the points on the curve where you have horizontal
dx
tangents.
(d) Examine intervals when f(x) is increasing or decreasing
(e) Examine what happens to 'y' when x →  or x → −    
Area between two curves
If f(x) > g(x) for x[a,b] then area bounded by curves (graph) y = f(x) and y = g(x) between ordinates x =
b
a and x = b is  ( f(x) − g(x) ) dx .
a

Example # 39 : Find the area enclosed by curve (graph) y = x 2 + x + 1 and its tangent at (1,3) between

RI
ordinates x = – 2 and x = 1.
dy
Solution : = 2x + 1
dx
dy

A
= 3 at x = 1
dx
Equation of tangent is
y – 3 = 3 (x – 1)

UH
y = 3x
1 –2
Required area = 
−2
(x 2 + x + 1 − 3x) dx

1 1
x3  1   8 
 (x − 2x + 1) − x 2 + x  =  − 1 + 1 –  − 3 − 4 − 2 = 9
2
=
JA
dx =
−2
3  −2 3   

Note : Area bounded by curves y = f(x) and y = g (x) between ordinates x = a and x = b is  | f(x) − g(x) | dx .
a

Example # 40 : Find the area of the region bounded by y = sin x, y = cos x and ordinates x = 0, x = /2
LP

/2

Solution : 
0
| sin x − cos x | dx
/ 4 /2


0
(cos x − sin x) dx + 
/ 4
(sin x − cos x) dx = 2 ( 2 − 1)
A

Example # 41 : Find the area contained by ellipse 5x 2 + 6xy + 2y2 + 7x + 6y + 6 = 0


Solution : 5x2 + 6xy + 2y2 + 7x + 6y + 6 = 0
–3(1 + x)  (3 – x)(x – 1)
2y2 + 6(1 + x) y + 5x2 + 7x + 6 = 0 y=
NK

2
Clearly, the values of y are real for x [1,3]
y
SA

(1,0) (3,0)
x' 0 x

Q(x,y2)
(0,–3) ••
• y = f(x)
••
•••
(0,–6)

P (x,y1) y = g(x)
y'
when x = 1 , we get y = – 3 and, x = 3  y = – 6
–3(1 + x) + (3 – x)(x – 1) –3(1 + x) – (3 – x)(x – 1)
Let f(x) = and , g(x) =
2 2
3

required area =  {g(x) – f(x)}dx


1
3
1 1 –1  x – 2  
3 3

= 
1
–x 2 + 4x – 3 dx = 
1
12 – (x – 2)2 dx =  (x – 2)
2
–x 2 + 4x – 3 +
2
sin  
 1  1
 1  1  
=   sin–1 1  –  sin–1(–1) = sq. unit
 2  2  2
Miscellaneous examples

RI
Example # 42 : Find the area contained between the two arms of curves (y – x)2 = x5 between x = 0 and x = 1.
Solution : (y – x)2 = x5  y = x ± x 5/2
For arm

A
dy 5 3/2
y = x + x5/2  =1+ x >0 x > 0.
dx 2
5/2
x

UH
1 5/2
 4 3 x
 25 
 

y is increasing function.
JA
For arm
dy 5 3/2
y = x – x5/2  =1– x
dx 2
1/ 3 1 1/ 3
5  4  d2 y 5  4 
=0  x=   , =− x 2 < 0 at x =  
2  25  dx 2 4  25 
LP

1/ 3
 4 
 at x =   , y = x – x5/2 has maxima.
 25 
1 1 1
x7 / 2  4
 (x + x
2
−x+x
x
5/2 5/2 5/2
Required area = ) dx = 2 dx =  =
A

7 / 2  7
0 0 0

Example # 43 : Let A (m) be area bounded by parabola y = x 2 + 2x – 3 and the line y = mx + 1. Find the least
NK

area A(m).
Solution : Solving we obtain
x2 + (2 – m) x – 4 = 0
Let  be roots   +  = m – 2,  = – 4
 

 (mx + 1 − x − 2x + 3)dx  (−x + (m − 2)x + 4)dx


2 2
A (m) = =
SA

 


 x3 x2   3 − 3 m − 2 2
=  − + (m − 2) + 4x  = + ( −  2 ) + 4( −  )
 3 2  3 2
1 2 (m − 2)
= | – |. − ( +  +  ) + ( +  ) + 4
2

3 2
1 (m − 2) 1 8
= (m − 2)2 + 16 − (m − 2)2 + 4 +
( (m − 2) + 4 =
) (m − 2)2 + 16 (m − 2)2 +
3 2 6 3
3/2 1 1 32
A(m) ( (m − 2)2 + 16 ) =  Least A(m) = (16)3/2 = .
6 6 3
Example # 44 : A curve y = f(x) passes through the origin and lies entirely in the first quadrant. Through any
point P(x, y) on the curve, lines are drawn parallel to the coordinate axes. If the curve divides
the area formed by these lines and coordinate axes in m : n, then show that f(x) = cx m/n or f(x) =
cxn/m (c-being arbitrary).
x

Solution : Area (OAPB) = xy  Area (OAPO) =  f(t)


0
dt

RI
Area (OAPO) m
Area (OPBO) = xy –  f(t) dt  =
0 Area (OPBO) n
x
 x
 x x
n f(t) dt = m  xy –  f(t) dt   n f(t) dt = mx f(x) – m  f(t) dt
 

A
0 0 0 0

Differentiating w.r.t. x
f (x) n 1
nf(x) = m f(x) + mx f¢(x) – m f(x)  =

UH
f(x) m x
f(x) = cxn/m
similarly f(x) = cxm/n

Self practice problems :


JA
(44) Find the area bounded by the curves y = ex, y = |x – 1| and x = 2.
(45) Compute the area of the region bounded by the parabolas y 2 + 8x = 16 and y2 – 24x = 48.

(46) Find the area between the x-axis and the curve y = 1 + cos4x , 0  x  .

(47) What is geometrical significance of


LP

3
 2
(i)  cos x
0
dx , (ii)  cos x
0
dx

(48) Find the area of the region bounded by the x-axis and the curves defined by y = tan x,
A

 –    2 
 where 3  x  3  and y = cot x .  where 6  x  3 
   
NK

(49) Find the area bounded by the curves x = y 2 and x = 3 – 2y2.

(50) Find the area bounded by the curve y = x 2 – 2x + 5, the tangent to it at the point (2, 5) and the
axes of coordinates.

(51) Find the area of the region bounded by y = x – 1 and (y – 1)2 = 4(x + 1)
SA

(52) Find the area of the region lying in the first quadrant and included between the curves
x2 + y2 = 3a2. x2 = 2ay and y2 = 2ax. a > 0

(53) Find the area enclosed by the curves y = –x2 + 6x – 5, y = –x2 + 4x – 3 and the straight line
y = 3x – 15.

(54) Find the area bounded by the curves 4y = |4 – x2|, y = 7 – |x|

(55) Find the area bounded by the curves x = |y 2 – 1| and y = x – 5.

(56) Find the area of the region formed by x 2 + y2 – 6x – 4y + 12  0, y  x and 2x  5.


2
Ans. 44. (e2 – 2) sq. units 45. 32 sq. units 46. 2 2 sq. units
3
47. (i) Area bounded by y = cos x, x-axis between x = 0, x = .

(ii) Difference of area bounded by y = cos x, x-axis between x = 0, x = and area
2
 3
bounded by y = cos x, x-axis between x = ,x= .
2 2
3
48. n 49. 4 sq. units
2
50. 8/3 sq. units
 2 3 −1 1  73
51. 64/3 sq. units  52. a2  + sin  sq. units 53. sq. units

RI
 3 2 3  6
109  1 3
54. 32 sq. units 55. sq. units 56.  + −  sq. units
6 6 8 8 

A
UH
JA
LP
A
NK
SA
 Marked questions are recommended for Revision.

PART - I : SUBJECTIVE QUESTIONS


Section (A) : Definite Integration in terms of Indefinite Integration, using substitution
and By parts  
A-1. Evaluate :
x2 − 4 x
1 3 1

  x cos(tan
−1
(i) dx (ii) x)dx
x

RI
0 0

A-2. Evaluate :
  4
dx dx x2
  (i) − x2 + 2x + 2 (ii) x (iii) 0 1 + x dx

A
2 x2 − 1
/2

(iv)  cos  sin3  d

UH
0

A-3. Evaluate :
1 2 1
n x
 sin x  x
−1
(i) dx (ii) dx (iii) 2
sin−1 x dx .
0 1 x2 0
JA
A-4. Evaluate
/3
 
(i)  f(x) dx where f(x) = Minimum {tanx, cot x}  x 
0
 0, 2 
 
1
(ii) 
−1
f(x) dx where f(x) = min {x + 1, 1 − x}
LP

(iii)  f(x)
−1
dx where f(x) = minimum (|x| , 1 – |x|, 1/4)
A

A-5. Evaluate
−1  2x  x tan−1 x
1 1

(i) 0 sin  1 + x2  dx (ii)  (1 + x


0
2 3/2
)
dx
NK

 2x 
b 3

  tan
−1
(iii) (x − a)(b − x) dx, a > b (iv)  2 
dx
a 0  1− x 

A-6. Evaluate :
 1 /2
dx x sin x cos x
0 ex + e− x  1+ 
SA

(i) (ii) dx (iii) dx


0 x 0 cos2 x + 3cos x + 2
/2 / 4
sin2 d sin x + cos x
(iv) 
0 sin4  + cos4 
(v) 
0
9 + 16 sin2x
dx

a
1
e
3
A-7 (i) Find the value of a such that dx = n 2.
0
x
+ 4e− x + 5
(  / 2)1/ 3

(ii) Find the value of 


0
x 5 . sin x 3 dx
Section (B) : Definite Integration using Properties

 1 − sin x   1 + sin x 
b a
B-1. Let f(x) = n   , then show that
 1 + sin x 

a
f(x) dx = 
b
n   dx.
 1 − sin x 

B-2. Evaluate :
2

 [x
2
(i) ] dx (where [.] denotes greatest integer function)
0
 2
(ii) 
0
1 + sin2x dx (iii)  f(x)dx
0
where

4 

RI
| x  [cot
−1
(iv) 2
− 4x + 3 |dx (v) x]dx (where [.] denotes greatest integer function)
0 0
5 1

 | x + 2 | dx  [cos
−1
(vi) (vii) x]dx (where [.] denotes greatest integer function)

A
−5 −1

B-3. Evaluate :
/ 4 / 4
x + /4
1

 e dx  

UH
|x|
(i) (ii) | sin x | dx (iii) dx
−1 − / 4 − / 4
2 − cos 2x
/2
g(x) − g( − x)
1

 sin 
5
(iv) x cos4 x dx (v) dx
−1 − / 2
f( − x) + f(x)
JA
B-4. Evaluate
/2 /2 a
sin x esin x x
(i) 
0 sin x + cos x
dx (ii) 
0 e + ecos x
sin x
dx (iii) 
0 x + a−x
dx
/2 /2
a sin x + bcos x sin x − cos x
(iv) 
0
sin x + cos x
dx (v) 
0 (sin x + cos x)2
dx
LP

B-5. Evaluate :
2 
dx
(i)  sin(sin x) + sin(cos x) dx
0
(ii) 
0
5 + 4cos 2x
/2 
A

 1  dx
(iii) 
0
(2 nsin x − n sin2x) dx (iv) 
0
n  x + .
 x  1 + x2
NK

B-6. Evaluate :
2

(i)  {2x}
−1
dx (where function {.} denotes fractional part function)
10 

(ii)  (| sin x | + | cos x |)


0
dx
SA

 [x] dx
(iii) 0
n
, where [x] and {x} are integral and fractional parts of x and n  N

0
{x} dx

2n
  sin x 
(iv) 
0
 | sin x | − 
  2
  dx (where [ ] denotes the greatest integer function and n  )

 T T
B-7. If f(x) is a function defined  xR and f(x) + f(–x) = 0  x  − ,  and has period T, then prove that
 2 2
x
(x) =  f(t)
a
dt is also periodic with period T.

Section (C) : Leibnitz formula and Wallis' formula


x2
t
C-1. (i) If f(x) = 5 g(x)
and g(x) = 
2 n (1 + t 2 )
dt, then find the value of f( 2 ).

x3
d
dx 
0
cos t dt
(ii) The value of Lim

RI
x →0
1 − cos x
x2
1
 cos
−1 2
(iii) Find the slope of the tangent to the curve y = t dt at x = 4
x 2

A
sin2 x cos2 x

 
−1
C-2. (i) If f(x) = sin t dt + cos−1 t dt , then prove that f(x) = 0  x  R.

UH
0 0
x

  (ii) Find the value of x for which function f(x) =  t(et – 1) (t – 1) (t – 2)3 (t – 3)5 dt has a local
−1

minimum

x
JA
C-3. If y = x
1
n t dt

d2 y
then find the value of at x = e
dx 2
LP

1/ n


1/(n +1)
tan–1(nx)dx
C-4. lim 1/ n
is equal to
n →


–1
sin (nx)dx
1/(n +1)
A

C-5. Let f be a differentiable function on R and satisfying the integral equation


x x
x  f(t)dt –  tf(x – t)dt = ex –1  x  R, then f(1) equals to
NK

0 0

C-6. Evaluate :

/2 

 sin2 x cos2 x(sin x + cos x)  x sin


5
(i) dx (ii) x dx
SA

− / 2 0

2 2

x 2−x 
3/2
(iii) dx (iv) x (sin2 x cos2 x) dx
0 0
SECTION (D) : ESTIMATION & MEAN VALUE THEOREM
 
D-1. Prove the following inequalities :
/3 3
3 sin x 2
(i) <  dx < (ii) 4  (3 + x 3 ) dx  2 30
8 / 4
x 6 1

D-2. Show that  


1 1
1 x9 1 1 tan x 
  (i)
10 2
 
0
1+ x
dx 
10
(ii)
2
n2 
0
1+ x 2dx 
2

RI
D-3. (i) Show that  sin x.cos
0
x dx = 2sinc.cos c for some c(0, 2)

4
f(x) is a continuous function x  R, then show that  f(x)dx = 2f( ) some   (1, 2)
2
(ii)

A
1

Section (E) : Integration as a limit of sum and reduction formula

UH
E-1. Evaluate :
n −1
1 3  n n n n 
(i) Lim
n→
 (ii) Lim
n→
1 +

+ + + .......... + 
n + 3(n − 1) 
r =0 n −r
2 2 n n+3 n+6 n+9
1  2n 
JA
4 
(iii) lim (3nr 2 + 2n2r) 
n→ n  r =1 

/ 4
1
If n =  tan dx , then show that n + n – 2 =
n
E-2. (i) x
0
n −1
/2
n −1
 (sin x) dx, n  N . Show that 
LP

(ii) n = n
n
= n–2 n  2
n
0

Section (F) : Area Under Curve


A

F-1. Find the area enclosed between the curve y = x 3 + 3, y = 0, x = – 1, x = 2.

x
F-2. (i) Find the area bounded by x² + y² − 2 x = 0 and y = sin in the upper half of the circle.
NK

2
(ii) Find the area bounded by the curve y = 2x 4 – x2, x-axis and the two ordinates corresponding to
the minima of the function.
(iii) Find area of the curve y² = (7 − x) (5 + x) above x−axis and between the ordinates x = − 5 and
x = 1.
SA

F-3. Find the area of the region bounded by the curve y 2 = 2y – x and the y-axis.

F-4. Find the area bounded by the y-axis and the curve x = ey sin y, y = 0, y = 1.
x2 y2
F-5. (i) Find the area bounded in the first quadrant between the ellipse + = 1 and the line 3x + 4y =12
16 9
(ii) Find the area of the region bounded by y = {x} and 2x – 1 = 0, y = 0, ({ } stands for fraction part)

F-6. Compute the area of the figure bounded by straight lines x = 0, x = 2 and the curves y = 2 x and
y = 2x – x2

F-7. Let f(x) = tan x . Show that area bounded by y = f(x), y = f(c), x = 0 and x = a, 0 < c < a < is
2
a
minimum when c =
2

F-8. Find the area included between the parabolas y 2 = x and x = 3 – 2y2.

F-9. A tangent is drawn to the curve x 2 + 2x – 4ky + 3 = 0 at a point whose abscissa is 3. This tangent is
perpendicular to x + 3 = 2y. Find the area bounded by the curve, this tangent and ordinate x = – 1

   
F-10. (i) Draw graph of y = (tan x)n , n  0,  N, x  0,  . Hence show
 4  4

RI
 
0 < (tan x)n+1 < (tan x)n, x   0, 
 4
(ii) Let An be the area bounded by the curve y = (tan x)n and the lines x = 0, y = 0 and x = /4.
Prove that for n > 2, An + An−2 = 1/(n − 1) and deduce that 1/(2n + 2) < An < 1/(2n − 2).

A
 
PART - II : ONLY ONE OPTION CORRECT TYPE

UH
SECTION (A) : D.I. IN TERMS OF INDEFINITE INTIGRATION, USING SUBSTITUTION AND BY PARTS
 
x
dt 
A-1. If | t | t −1
2
=
6
, then x can be equal to :
1
JA
2 4
(A) (B) 3 (C) 2 (D)
3 3


1
dx
A-2. The value of the integral x
0
2
+ 2x cos  + 1
, where 0 <  < , is equal to:
2
LP

 
(A) sin  (B)  sin  (C) (D) sin 
2 sin  2

 x x 1 2

A-3. If f(x) =  , then  x 2 f(x) dx is equal to :


A

 x − 1 x 1 0

4 5 5
(A) 1 (B) (C) (D)
3 3 2
NK

A-4. If f(0) = 1, f(2) = 3, f(2) = 5 and f(0) is finite, then x


0
. f  (2x) dx is equal to

(A) zero (B) 1 (C) 2 (D) 3


SA

A-5. 
0
1 + 2cos x dx is equal to :

2 
(A) (B)  (C) 2 (D) +2 3
3 3

3
A-6. The value of  (| x − 2 | +[x]) dx is ([x] stands for greatest integer less than or equal to x)
−1
(A) 7 (B) 5 (C) 4 (D) 3

A-7. 
0
[2e− x ] dx , where [ . ] denotes the greatest integer function, is equal to :
(A) 0 (B) n 2 (C) e2 (D) 2e–1

n
ex
A-8.  2 x
dx is equal to
n− n2
1 − cos  e 
3 
1 1
(A) 3 (B) – 3 (C) (D) –
3 3

e2 2
dx ex
A-9. If 1= 
e
n x
and 2 = 1 x dx, then
(A) 1 = 2 (B) 2 1 = 2 (C) 1 = 2 2 (D) 1 + 2 = 0

RI
/ 4
x . sin x
A-10. 
0 cos3 x
dx equals to :

 1  1  

A
(A) + (B) − (C) (D) +1
4 2 4 2 4 4

UH
4
 2x − 5(4x − 5) + 2x + 5(4x − 5)  dx is equal to
A-11. The value of the definite integral

3
 
2

2 2 4 3 5
(A) 4 5 − (B) 4 5 (C) 4 3 − (D)
JA
5 3 8

x
dx 
A-12. If 
n2 e −1
x
=
6
, then x is equal to

(A) 4 (B) n 8 (C) n 4 (D) n 2


LP


x2 + 1
A-13. x
0
4
+ 7x 2 + 1
dx =

  
(A)  (B) (C) (D)
2 3 6
A

Section (B) : Definite Integration using Properties


NK

n +1 4

B-1. Suppose for every integer n, . 


n
f(x)dx = n2 The value of  f(x)dx
−2
is :

(A) 16 (B) 14 (C) 19 (D) 21

B-2. Let f : R → R, g : R → R be continuous functions. Then the value of integeral


SA

 x2 
n1/  
f  [f(x) − f( − x)]
 4 
n  x 2  dx is :
g   [g(x) + g( − x)
 4 
(A) depend on  (B) a non-zero constant (C) zero (D) 2

−1  x + x 
1 3
B-3.  cot  4 
 1+ x 
dx is equal to
−1

(A) 2 (B) (C) 0 (D) 
2
0

 {x
3
B-4. + 3x 2 + 3x + 3 + (x + 1)cos(x + 1)} dx is equal to
−2
(A) – 4 (B) 0 (C) 4 (D) 6

 x n(1 + e )dx
x
B-5. =
−1
(A) 0 (B) n(1 + e) (C) n(1 + e) – 1 (D) 1/3

3/2
k
B-6. If 
−1
| x sin x | dx =
2
, then the value of k is :

(A) 3 + 1 (B) 2 + 1 (C) 1 (D) 4

RI
2
4
dx
B-7. The value of definite integral is 

A
0 1 + sin x + cos x
 ln2  ln2
(A)  ln 2 (B) (C) (D) 2 ln2
2 4

UH
3+ n 3
n (4 + x)
B-8.
2− n3
 n (4 + x) + n (9 − x)
dx is equal to :

5
(A) cannot be evaluated (B) is equal to
JA
2
1
(C) is equal to 1 + 2 n 3 (D) is equal to + n 3
2

B-9. The value of the definite integral I = x


0
1+ | cos x | dx is equal to
LP

(A) 2 2  (B) 2  (C) 2  (D) 4


/2

B-10. The value of 


0
n | tan x + cot x | dx is equal to :

 
A

(A)  n 2 (B) – n 2 (C) n 2 (D) – n 2


2 2

1
1 1
e x dx x 2 dx
0 1 + x and 2 = e
NK

B-11. Let 1 = , then is


0
x3
(2 − x ) 3
2
(A) 3/e (B) e/3 (C) 3e (D) 1/3e
[ x]

B-12. The value of  {x} dx (where [ . ] and { . } denotes greatest integer and fraction part function
SA

respectively) is
1 1
(A) [x] (B) 2[x] (C) (D) [x]
2 2[x]
11
11x k
B-13. If
0 11 
[ x]
dx =
log11
, (where [ ] denotes greatest integer function) then value of k is

(A) 11 (B) 101 (C) 110 (D)121


Section (C) : Leibnitz formula and Wallis' formula
x2
et
C-1. f(x) =  t
dt , then f ' (1) is equal to:
x

(A) e (B) 2e (C) 2e2 – 2 (D) e2 – e

 (t − 1)(t − 2) (t − 3) (t − 4)
2 3 5
C-2. f(x) = dt (x > 0) then number of points of extremum of f(x) is
0
(A) 4 (B) 3 (C) 2 (D) 1
 
x +h x

RI

a
n2 t dt −  n2 t dt
a
C-3. Limit equals to :
h→0 h
2 nx
(A) 0 (B) n2 x (C) (D) does not exist

A
x

C-4. The value of the function f (x) = 1 + x +  (n2t + 2 nt)dt, where f  (x) vanishes is:

UH
1

(A) e−1 (B) 0 (C) 2 e−1 (D) 1 + 2 e−1

y x2
sin t dy
 cos t dt = 
2
C-5. If dt , then the value of is
t dx
a a
JA
2 sin x 2
2 sin x 2 2sin x 2 sin x 2
(A) (B) (C) (D)
x cos2 y x cos y 2  y2  2y
x  1 − 2sin 
 2 

1
 1 

LP

C-6. If t 2 (f(t)) dt = (1 – sinx), then f   is


sin x  3
(A) 1/3 (B) 1/ 3 (C) 3 (D) 3

a
1
 ln(1 + e
x
A

C-7. The value of Lim ) dx equals


a→ a2 0
1
(A) 0 (B) 1 (C) (D) non-existent
NK

x
sin x cos y
C-8. f(x) =  dy , then   
1 y2 + y + 1
n 
(A) f ' (x) = 0  x = ,nZ (B) f ' (x) = 0  x = (2n + 1) ,nZ
SA

2 2
(C) f ' (x) = 0  x = n, n  Z (D) f ' (x)  0  x  R

/2
C-9.  sin4 x cos3 x dx is equal to :
0
6 2 2 2
(A) (B) (C) (D)
35 21 15 35

1
C-10.  x 2 (1 − x)3 dx is equal to
0
1 1 2 
(A) (B) (C) (D)
60 30 15 120
 
SECTION (D) : ESTIMATION & MEAN VALUE THEOREM
 
3
D-1. Let   = 
1
x 4 + x 2 dx, then

(A)  > 6 10 (B)  < 2 2 (C) 2 2   < 6 10 (D)  < 1

2


2
D-2.  = esin x + sin x +1
dx, then

RI
0
(A) e3 <  < 2e5 (B) 2e3/4 <  < 2e3 (C) 2e3 <  < 2e4 (D) 0 <  < 2

D-3. Let f "(x)    f '(x)   f(0) = 3 & f(x) is defined in [–2, 2]. If f(x) is non-negative, then

A
0 2 2 1
(A)  f(x)dx  6 (B)  f(x)dx  12
−1 −2
(C)  f(x)dx  12
−2
(D)  f(x)dx  12
−1

UH
 
1 1
D-4. Let mean value of f(x) = over interval (0, 2) is n3 then positive value of c is
x+c 2
1 3
(A) 1 (B) (C) 2 (D)
2 2
JA
SECTION (E) : INTEGRATION AS A LIMIT OF SUM AND REDUCTION FORMULA
 
n
 r3 
E-1. lim
n→
  4 4 
r =1  r + n 
equals to :

1 1 1
LP

(A) n 2 (B) n 2 (C) n 2 (D) n 2


2 3 4

3n
n
E-2. Lt
n→

r = 2n + 1 r − n2
2
is equal to :
A

2 3 2 3
(A) n (B) n (C) n (D) n
3 2 3 2
NK

1/ n
 1   22   n2  
E-3. lim  1 + 2   1 + 2  ...  1 + 2   is equal to :
 n   n 
n→
 n 
/2
e 2 /2
(A) (B) 2 e2 e/2 (C) e (D) 2 e
2e2 e2
SA

   2 (n − 1) 
E-4. lim sin n + sin n + ..... + sin n  is equals to :
n→ n  
(A) 0 (B)  (C) 2 (D) 3

1
Let n =  (1 − x ) dx, (n  N) then
3 n
E-5.
0
(A) 3n n = (3n – 1) n–1  n  2 (B) (3n – 1)n = 3n n–1 n  2
(C) (3n – 1)n = (3n + 1) n–1  n  2 (D) (3n + 1)n = 3n n–1  n  2
Section (F) : Area Under Curve
F-1. The area bounded by the x-axis and the curve y = 4x – x2 – 3 is
1 2 4 8
(A) (B) (C) (D)
3 3 3 3

F-2. The area of the figure bounded by right of the line y = x + 1, y = cos x and x−axis is:
1 2 5 3
(A) (B) (C) (D)
2 3 6 2

F-3. Area bounded by curve y3 – 9y+ x = 0 and y-axis is


9 81

RI
(A) (B) 9 (C) (D) 81
2 2

x
F-4. Let f:[0, ) → R be a continuous and strictly increasing function such that f 3 (x) = t f 2 (t) dt , x  0.

A
0
The area enclosed by y = f(x), the x-axis and the ordinate at x = 3 is –––––––
3 5 7 1
(A) (B) (C) (D)

UH
2 2 2 2

F-5. The area bounded by the curve y = ex and the lines y = x − 1, x = 2 is given by:
(A) e² + 1 (B) e² − 1 (C) e² − 2 (D) e – 2

3
JA
F-6. The area bounded by y = 2 – 2 – x and y = is:
x
4+3 n 3 4−3 n 3 3 1
(A) (B) (C) + n3 (D) + n3
2 2 2 2

F-7. The area bounded by the curve y2 = 4x and the line 2x – 3y + 4 = 0 is


1 2 4 5
LP

(A) (B) (C) (D)


3 3 3 3

F-8. The area of the region bounded by x = 0, y = 0, x = 2, y = 2, y  e x and y  n x, is


(A) 6 – 4 n 2 (B) 4 n 2 – 2 (C) 2 n 2 – 4 (D) 6 – 2 n 2
A

F-9. The area between two arms of the curve |y| = x3 from x = 0 to x = 2 is
(A) 2 (B) 4 (C) 8 (D) 16
NK

1
F-10. The area bounded by the parabolas y = (x + 1) 2 and y = (x – 1)2 and the line y = is
4
1 4 1
(A) 4 sq. units (B) sq. units (C) sq. units (D) sq. units
6 3 3
SA

PART - III : MATCH THE COLUMN


T
1
T → T 
1. Let Lim (sin x + sinax)2 dx = L then
0
Column - I Column- II
(A) for a = 0, the value of L is (p) 0
(B) for a = 1 the value of L is (q) 1/2
(C) for a = – 1 the value of L is (r) 3/2
(D)  a  R – {–1, 0, 1} the value of L is (s) 2
(t) 1
2. Column –  Column – 
(A) Area bounded by region 0  y  4x – x2 – 3 is (p) 32/3
(B) The area of figure formed by all the points satisfying the (q) 1/2
inequality y2  4 (1 – |x|) is
(C) The area bounded by |x| + |y|  1 and |x|  1/2 is (r) 4/3
(D) Area bounded by x  4 – y2 and x  0 is (s) 16/3

RI
A
UH
JA
LP
A
NK
SA
 Marked questions are recommended for Revision.

PART - I : ONLY ONE OPTION CORRECT TYPE


1

1. The value of  ( 2 x − 1) ( 3 x − 1)
0
dx, (where {. } denotes fractional part of x) is equal to :

19 19 19 19
(A) (B) (C) (D)
36 144 72 18

100 100 1 
    f (r − 1 + x ) dx  =

RI
2. If f(x) dx = a, then
0 
r =1 0 
(A) 100 a (B) a (C) 0 (D) 10 a
t

A
3. lim −

t → 
 tan 
0
cos  n(cos )d is equal to:
2
(A) –4 (B) 4 (C) –2 (D) Does not exists

UH
 n 2
0 , where x = , n = 1, 2, 3.....
4. If f(x) =  n +1 , then the value of  f(x) dx .

 1 , else where 0

(A) 1 (B) 0 (C) 2 (D) 


JA
 

e e
− x2 − ax 2
5. If dx = , then dx where a > 0 is :
0
2 0

   1 
(A) (B) (C) 2 (D)
2 2a a 2 a
LP

4
 yi
4 i=1  ex 
6. If  ( sin–1 xi + cos–1 yi ) = 6, then  x ln(1 + x 2 ) 
 1 + e2x
 dx is equal to

i=1 4  
 xi
i=1
A

 17 
(A) 0 (B) e4 + e–4 (C) ln   (D) e4 – e–4
 12 
NK

7. The tangent to the graph of the function y = f(x) at the point with abscissa x =1 form an angle of  / 6
and at the point x = 2, an angle of  / 3 and at the point x = 3, an angle of  / 4 with positive x-axis. The
3 3

value of 
1
f ' ( x ) f '' ( x ) dx + 
2
f '' (x) dx (f''(x) is supposed to be continuous) is :
SA

4 3 −1 3 3 −1 4− 3 4
(A) (B) (C) (D) − 3
3 3 2 3 3

e− t
1 a
et
8. Let A = 
0 1+ t
dt, then
a −1

t − a −1
dt has the value :

(A) Ae–a (B) – Ae–a (C) – ae–a (D) Aea

x (1 + 2 n x)dx is equal to         


2x2 +1
9.
1
255
(A) 256 (B) 255 (C) (D) 128
2
cos ec
 1
10. If f(x) is a function satisfying f   + x2 f(x) = 0 for all non-zero x, then
x

sin 
f(x) dx equals to :

(A) sin + cosec (B) sin2  (C) cosec2  (D) none of these

C0 C1 C2
11. If + + = 0 , where C0, C1, C2 are all real, the equation C2x2 + C1x + C0 = 0 has:
1 2 3
(A) atleast one root in (0, 1) (B) one root in (1, 2) & other in (3, 4)
(C) one root in (−1, 1) & the other in (− 5, − 2) (D) both roots imaginary
 
x

 (2cos 3t + 3 sin2 3t) dt, f(x + ) is equal to :


2
12. If f(x) =

RI
0

 
(A) f(x) + 2f() (B) f(x) + 2f   (C) f(x) + 4f   (D) 2f(x)
2 4
 

A
x
dt
13. Let f (x) = 
0 1 + t3
and g (x) be the inverse of f (x), then which one of the following holds good?

UH
(A) 2g'' = g2 (B) 2g'' = 3g2 (C) 3g'' = 2g2 (D) 3g'' = g2

2 1

14. Let f(x) is differentiable function satisfying 2 f(tx)dt = x + 2 ,  x  R Then  (8f(8x) – f(x) – 21x) dx
1 0
equals to
JA
(A) 3 (B) 5 (C) 7 (D) 9

1
15. Let  n =  xn (tan−1 x)dx , n  N, then
0
 1  1
(A) (n + 1)n + (n – 1)n–2 = +  n3 (B) (n + 1)n + (n – 1)n–2 = −  n3
LP

4 n 2 n
 1  1
(C) (n + 1)n – (n – 1)n–2 = +  n3 (D) (n + 1)n – (n – 1)n–2 = −  n3
4 n 2 n

/2

x
A

n
16. If , un = sin x dx , then the value of u10 + 90 u8 is :
0
8 9 9 9
   
(A) 9   (B)   (C) 10   (D) 9  
NK

2 2 2 2

tan x cot x
t 1
17. The value of 
1/ e 1+ t2
dt + 
1/ e t(1 + t 2 )
dt , where x  (/6, /3), is equal to :

(A) 0 (B) 2 (C) 1 (D) cannot be determined


SA

x
u  x
A1
18. Let A1 =    f (t) dt  du and A2 =  f (u).(x − u) du then is equal to :
0 0  0 A2
1
(A) (B) 1 (C) 2 (D) –1
2

1/ n
  2 3 (n − 1) 
lim  sin . sin . sin .......sin
n 
19. is equal to :
n→
 2n 2n 2n
1 1 1 3
(A) (B) (C) (D)
2 3 4 4
20. Area bounded by the region consisting of points (x, y) satisfying y  2 − x 2 , y2  x, y  − x is

(A) (B)  (C) 2 (D) /4
2

21. The area enclosed between the curves


 1
y = loge(x + e), x = loge   and the x-axis is
y
(A) 2 (B) 1 (C) 4 (D) 3
 

22. The area bounded by the curve x = acos 3t, y = a sin3 t is


3a2 3a2 3a2

RI
  (A) (B) (C) (D) 3a2
8 16 32
 
23. The area bounded by the curve f(x) = x + sin x and its inverse function between the ordinates x = 0 and

A
x = 2 is
(A) 4 (B) 8 (C) 4 (D) 8

24. P(2, 2), Q(–2, 2), R(–2, –2) & S(2, –2) are vertices of a square. A parabola passes through P, S & its

UH
vertex lies on x-axis. If this parabola bisects the area of the square PQRS, then vertex of the parabola
is
 3 
(A) (–2, 0) (B) (0, 0) (C)  − , 0  (D) (–1, 0)
 2 
 x 
JA
25. The ratio in which the curve y = x² divides the region bounded by the curve; y = sin  
 2 
and the x−axis as x varies from 0 to 1, is :
(A) 2:  (B) 1: 3 (C) 3:  (D) (6 − ): 

   
26. If f(x) = sin x,  x  0,  , f(x) + f( – x) = 2.  x   ,   and f(x) = f(2 – x), x  ( , 2 , then
LP

 2 2 
the area enclosed by y = f(x) and x-axis is
(A)  (B) 2 (C) 2 (D) 4
A

27. The area bounded by the curves y = x e x, y = x e–x and the line x = 1
2 2 1 1
(A) (B) 1 – (C) (D) 1 –
e e e e
NK

28. Obtain the area enclosed by region bounded by the curves y = x n x and y = 2x – 2x 2.
(A) 7/6 (B) 7/24 (C) 12/7 (D) 7/12

1
29. The area of the region on plane bounded by max (|x|, |y|)  1 and xy  is
SA

2
(A) 1/2 + n 2 (B) 3 + n 2 (C) 31/4 (D) 1 + 2 n 2

30. Consider the following statements :


2

 cos
−1
S1 : The value of (cos x) dx is 2
0

S2 : Area enclosed by the curve |x – 2| + |y + 1| = 1 is equal to 3 sq. unit


d b
[f(b)]2 − [f(a)]2
S3 : If
dx
f(x) = g(x) for a  x  b, then  f(x)g(x)dx equals to
a
2
.
1
S4 : Area of the region R  {(x, y) ; x2  y  x} is
6
  State, in order, whether S1, S2, S3, S4 are true or false
(A) TFTT (B) TTTT (C) FFFF (D) TFTF

PART - II : SINGLE AND DOUBLE VALUE INTEGER TYPE


4
3x 2 + 1 
1.  (x
2
2
− 1) 3
dx =
n2
where , n  N and gcd(, n) = 1, then find the value of  + n

RI

2 5
 min . ( 3 sinx,cos x) dx and V = x
2
2. Let U = sgn (x – 1)dx. If V = U, then find the value of .

A
−3
6
[Note : sgn k denotes the signum function of k.]

UH
10 100
 100  f(x) f(x)
3. Let f(x) be a function satisfying f(x) = f 
 x 
  x  0. If  x
dx = 5 then find the value of  x
dx
1 1

4. Evaluate
JA
1002 1003
dx
2005 
0 1002 − x + 1003 − x
2 2 2 2
+
1002
 10032 − x 2 dx

1
= k, then find the sum of squares of digits of


0
1 − x 2 dx
LP

natural number k.

/2

5. If 
0
sin2.sin  d =
n
then find n
A

/4 1
dx
6. Let I1 =  (1 + tan x)2 dx , 2 =  (1 + x) (1 + x
2 2
)
NK

0 0

then find the value of


1 2 
7. Find the value of

n  et + t (2t 2 + t + 1)dt 
SA

 
0 

0
x
8. If  x + 1+ e
−1
x
dx is equal to –nk , then find the value of k.

9. If f, g, h be continuous functions on [0, a] such that f (a − x) = f (x), g (a − x) = − g (x)


a

and 3 h (x) − 4 h (a − x) = 5, then find the value of  f (x) g (x) h (x) dx


0
/2 
sin x   
10. If f(x) =
x
 x  (0, ], If
k 
0
f(x) f  − x  dx =
 2 
0
f(x) dx then find the value of k.

11. Evaluate: 3 z

0
a2 sin2 x + b2 cos2 x
a4 sin2 x + b4 cos2 x
dx , where a2 + b2 =
3
, a2  b2 and ab  0.
4

2
12. |
0
15 sin x + cos x | dx

3 + a
x
 | x − a −  | sin   dx = –16, then determine

RI
13. Let a be a real number in the interval [0, 314] such that
−+ a
2
number of such values of a.
 

A

 1 1    
14.   4n − 3 − 4n − 1 = n , find 'n'  Note that tan−1 x + c =
 
1
1+ x 2
dx 

n=1  

UH
1 1

15. If f (x) = x +  t(x + t) f(t) dt , then the value of the definite integral  f(x) dx can be expressed in
0 0

p
the form of rational as (where p and q are coprime). Find (p + q).
q
JA
x
16. If f(x) = (ax + b) ex satisfies the equation : f(x) =  e x − y f '(y)dy − (x 2 − x + 1)e x , find (a2 + b2)
0


17. If the minimum of the following function f(x) defined at 0 < x < .
LP

2

x 2
d d
f(x) =  cos  +  sin  is equal to
0 x
n(a + b) where a, b  N and b is not a perfect square then find the

value of (a + b)
A

18. If f() = 2 and  (f(x) + f (x)) sin x dx = 5, then find the value of f(0)
NK

(it is given that f(x) is continuous in [0, ])

x 5 – x

19.  If f(x) = 2x3 – 15x2 + 24x and g(x) =  f(t) dt +


0
 0
f(t) dt (0 < x < 5). Find the number of integers for
SA

which g(x) is increasing.

 1− x if 0  x  1 x
0 if 1  x  2 and function F(x) =  f(t) dt. If number of points of discontinuity in
20. Let f(x) = 
(2 − x)2 if 2  x  3 0

[0, 3] and non-differentiablity in (0, 3) of F(x) are  and  respectively, then ( – ) is equal to.

21. Find the value of m (m > 0) for which the area bounded by the line y = mx + 2 and
x = 2y − y2 is 9/2 square units.

22. Find area bounded by y = f–1(x), x = 10, x = 4 and x-axis


given that area bounded by y = f(x), x = 2, x = 6 and x-axis is 30 sq. units, where f(2) = 4 and f(6) = 10.
(given f(x) is an invertible function)

1  
23. Consider a line  : 2x – 3 y = 0 and a parameterized C : x = tan t, y =  0t 
cos t  2
1
If the area of the part bounded by , C and the y-axis is equal to n(a + b ) , where a, b,  N, b, is
4
not perfect square then find the value of (a + b)

PART - III : ONE OR MORE THAN ONE OPTIONS CORRECT TYPE

RI

x
1.  (1 + x ) (1 + x )
0
2
dx equals to :

 

A
(A) (B)
4 2

dx
(C) is same as  (1 + x ) (1 + x ) 2
(D) cannot be evaluated

UH
0

b
|x|
2. The value of integral 
a
x
dx, a < b is :

(A) b – a if a > 0 (B) a – b if b < 0 (C) b + a if a < 0 < b (D) | b | – | a |


JA
1
dx
3. If n =  n
; n  N, then which of the following statements hold good?
0 (1 + x ) 2

 1
(A) 2n n + 1 = 2 −n + (2n − 1) In (B) 2 = +
8 4
 1  5
(C) 2 = − (D) 3 = −
LP

8 4 16 48

4. The value of integral 


0
xf (sin x) dx is :
A

 /2 /4

(A)
2  f(sin x) dx
0
(B)   f(sin x)dx
0
(C) 0 (D) 2  f(sin x) dx
0
2
NK

If  =  sin
2
5. xdx, then
0
 /2 2 / 4

(A)  = 2  sin xdx (B)  = 4  (C)  =  cos xdx (D)  = 8  sin


2
sin2 xdx 2 2
xdx
0 0 0 0
SA

6. Given f is an odd function defined everywhere, periodic with period 2 and integrable on every interval.
x

Let g(x) = 
0
f(t) dt. Then :

(A) g(2n) = 0 for every integer n (B) g(x) is an even function


(C) g(x) and f(x) have the same period (D) g(x) is an odd function

ex e− x
dt dt
7. Let f : R → R be defined as f(x) =  1 + t2 +  1 + t 2
, then
−1 1
(A) f(x) is periodic (B) f(f(x)) = f(x)  x  R

(C) f(1) = f '(1) = (D) f(x) is unbounded
2
n
n
8. If a, b  R+ then Lim
n →
 (k + an)(k + bn)
k =1
is equal to

1 b(b + 1) 1 a(b + 1)
(A) ln if a  b (B) ln if a  b
a−b a(a + 1) a−b b(a + 1)
1
(C) non existent if a = b (D) if a = b
a(1 + a)

x+
3
9. Let f(x) =  | sin  |d
x
(x  [0, ])

RI
(A) f(x) is strictly increasing in this interval (B) f(x) is differentiable in this interval

(C) Range of f(x) is 2 − 3, 1 (D) f(x) has a maxima at x =
  3

A
2

10. If f(x) is integrable over [1, 2], then 


1
f(x) dx is equal to :

UH
1 n
r 1 2n
r
  (A) lim
n→ n

r =1
f 
n
(B)
n→
lim
n

r =n+1
f 
n
1 n
r +n 1 2n
r
(C) lim
n→ n

r =1
f
 n 
 (D) lim  f  
n→ n
r =1 n
JA
1
2
1
11. Let n = 
0 1 − xn
dx where n > 2 , then

  1 1
(A) n < (B) n > (C) n < (D) n >
6 6 2 2
LP

12. If f(x) = 2{x}, where {x} denotes the fractional part of x. Then which of the following is true ?
1 1 100
1
(A) f is periodic (B)  2{x} dx = (C)  2{x} dx = log2 e (D)  2{x} dx = 100log2 e
n2
A

0 0 0
x

13. Let f(x) =  | 2t − 3 | dt, then f is


0
NK

(A) continuous at x = 3/2 (B) continuous at x = 3


(C) differentiable at x = 3/2 (D) differentiable at x = 0
 

Let n =  (sin x) dx, n  N, then
n
14.
0
SA

(A) n is rational if n is odd (B) n is irrational if n is even


(C) n is an increasing sequence (D) n is a decreasing sequence

15. Let f(x) be a function satisfying f(x) + f(x + 2) = 10 xR, then


(A) f(x) is a periodic function (B) f(x) is aperiodic function
7 7
(C) 
−1
f(x)dx = 20 (D)  f(x)dx = 40
−1


sin2 (nx)
16. Let n = 0
sin2 x
dx , n  N, then
(A) n+2 + n = 2n+1 (B) n = n+1
(C) n = n (D) 1, 2, 3,n are in Harmonic progression

9
17. Let f(x) be a continuous function and  = 
1
x f(x) dx , then

(A) There exists some c  (1, 9) such that  = 8 c f(c)


(B) There exists some p, q  (1, 3) such that  = 2[p2 f(p2) + q2 f(q2)]
(C) There exists some   (1, 9) such that  = 9  f()
(D) If f(x)  0 x  [1, 9]  >0
 

RI
e2
nx
18. Let A = 
1 x
dx , then

A
 1  1   1  2
 (D) A < ( e – 1)
2
(A) A > 2  e –  (B) A < (e – 1)  2 +  (C) A > (e – 1)  2 +
 e  e  e e
b

 (x

UH
19. Let f(a, b) = 2
− 4x + 3)dx , (b > a) then
a
(A) f(a, 3) is least when a = 1 (B) f(4, b) is an increasing function b  4
4
(C) f(0, b) is least for b = 2 (D) min{f(a, b)} = −
3
JA

 x 1 
20. Let  =   x
2
2
−  dx &  is a finite real number, then
+ 1 2x + 1 
1 1 5 1 5
(A)  = (B)  = 1 (C)  = n (D)  = n
2 2  2  4  4 
LP


21. Let f(x) be a strictly increasing, non-negative function such that f"(x) < 0 x  (, ) &  =  f(x)dx

( > ), then
+ +
A

(A)  < f   ( −  ) (B)  > f   ( −  )


 2   2 
1 1
(C)  > (f() + f())( – ) (D) I < (f() + f())( – )
NK

2 2

 
x sin x x3 sin x
22. 1 =  1 + cos
0
2
x
dx , 2 =  (
0
2
− 3x + 3x 2 )(1 + cos2 x)
dx, then

2 2
SA

(A) 1 = (B) 1 = (C) 1 = 2 (D) 1 > 2


8 4

x2 x2

 sin
0
t dt  sin
0
t dt
23. Let L1 = lim+ , L2 = lim , then identify the correct option(s).
x →0 x − sin x x →0 − x − sin x

(A) L1 = 4 (B) L1 + L2 = 8 (C) L1 + L2 = 0 (D) |L2| = |L1|


 
(1k + 2k + 3k + ..... + nk )
24. lim = F(k), then (k  N)
n→ (12 + 22 + ..... + n2 )(13 + 23 + ........ + n3 )
12 5
(A) F(k) is finite for k  6 (B) F(5) = 0 (C) F(6) = (D) F(6) =
7 7

n n−1

r r
n n
25. Let Tn = , Sn = , then  
r =1
2
− 2r.n + 2n 2
r =0
2
− 2r.n + 2n2
  
(A) Tn > Sn  n  N (B) Tn > (C) Sn < (D) lim Sn =
4 4 n→ 4

1
26. f(x) =  f(tx)dt, where f ' (x) is a continuous function such that f(1) = 2, then
0

RI
(A) f(x) is a periodic function (B) f '(x) = 0
(C) f(x) is an even function (D) f(x) is an odd function

27. Area bounded by y = sin–1x, y = cos–1x, y = 0 in first quadrant is equal to :

A
1/ 2 1 /2
(A)

0
−1
(sin x)dx + 
−1
(cos x)dx (B)  (sin y − cos y)dy
/4
1/ 2
/4

UH
(C)  (cos y − sin y)dy
0
(D) ( 2 − 1) sq.unit  

28. Let f(x) be a non-negative, continuous and even function such that area bounded by x-axis, y-axis & y =
f(x) is equal to (x2 + x3) sq. units x  0, then
JA
n n
(A)  r =1
f '(r) = 3n2 + 5n  n  N (B)  f '(r) = 6n2 + 5n  n  N
r =1
(C) f(x) = 3x2 + 2x  x  0 (D) f(x) = 3x2 – 2x  x  0

29. Let 'c' be a positive real number such that area bounded by y = 0 y = [tan –1x] from x = 0 to x = c is equal
LP

to area bounded by y = 0, y = [cot –1 x], from x = 0 to x = c (where [*] represents greatest integer
function), then
(A) c = tan1 + cot1 (B) c = 2cosec2 (C) c = tan1 – cot1 (D) c = –2 cot2

30. Area bounded by y = x2 – 2|x| and y = –1 is equal to


A

 (2x − x
2
(A) 2 )dx
0
NK

2
(B) sq. units
3
2
(C) (Area of rectangle ABCD) where points A, B, C, D are (–1, –1), (–1, 0), (1, 0) & (1, –1)
3
2
(D) (Area of rectangle ABCD) where points A, B, C, D are (–1, –1), (–1, 0), (1, 0) & (1, –1)
SA

3
 
PART - IV : COMPREHENSION
Comprehension # 1
v( x)
dy dy
If y = 
u( x)
f(t) dt , let us define
dx
in a different manner as
dx
= v(x) f 2 (v(x)) – u(x) f2 (u(x)) and the

 dy 
equation of the tangent at (a, b) as y – b =   (x – a)
 dx (a, b)
 
x2

1. If y = 
x
t 2 dt , then equation of tangent at x = 1 is

(A) y = x + 1 (B) x + y = 1 (C) y = x – 1 (D) y = x

x
d
If F(x) =  e t
2
/2
2. (1 – t2) dt, then F(x) at x = 1 is
1
dx
(A) 0 (B) 1 (C) 2 (D) – 1
x4
dy
3. If y =  nt dt , then lim+
x → 0 dx
is
x3

(A) 0 (B) 1 (C) 2 (D) – 1

RI
Comprehension # 2

 n (1 + x cos )
x2 x2

Let g(t) =  f(t, x)


x1
dx . Then g(t) = x t (f(t, x)) dx. Consider f(x) = 
0
cos 
d.
1

A
 
4. Range of f(x) is
 −    −2 2 
(A) (0, ) (B) (0, 2) (D)  

UH
(C)  ,
2 
,
 2  2 2

5. The number of critical points of f(x), in the interior of its domain, is


(A) 0 (B) 1 (C) 2 (D) infinitely many
JA
6. f(x) is
(A) discontinuous at x = 0 (B) differentiable at x = 1
(C) continuous at x = 0 (D) None of these
 
Comprehension # 3
If length of perpendicular drawn from points of a curve to a straight line approaches zero along an
LP

infinite branch of the curve, the line is said to be an asymptote to the curve. For example, y-axis is an
asymptote to y = nx & x-axis is an asymptote to y = e–x.

Asymptotes parallel to x-axis :

If lim f(x) = e (a finite number) then y = e is an asymptote to y = f(x). Similarly if lim f(x) =  , then y =
A

x → x →−
 is also an asymptote.

Asymptotes parallel to y-axis :


NK

If lim f(x) =  or lim f(x) = − , then x = a is an asymptote to y = f(x).


x →a x →a

(x + 1)(x + 2)
7. Number of asymptotes parallel to co-ordinate axes for the function f(x) = is equal to :
(x − 1)(x − 2)
SA

(A) 1 (B) 2 (C) 3 (D) 4

2x
8. Area bounded by y = 2
, it's asymptote and ordinates at points of extremum is equal to (in square
x +1
unit)
(A) n2 (B) 2n2 (C) n3 (D) 2n3

9. Area bounded by y = x2e–x and it's asymptote in first quadrant is equal to (in square unit)
(A) 2e (B) e (C) 1 (D) 2
 Marked questions are recommended for Revision.

PART - I : JEE (ADVANCED) / IIT-JEE PROBLEMS (PREVIOUS YEARS)


* Marked Questions may have more than one correct option.

sinnx
1*. If n = 
− (1 + x )sin x
dx, n = 0, 1, 2, ..., then [IIT-JEE - 2009,Paper-2, (4, –1), 80]

RI
10 10
(A) n = n+2 (B) 
m =1
 2m +1 = 10 (C) 
m =1
 2m = 0 (D) n = n+1

Let f : R → R be a continuous function which satisfies f(x) = 

A
2. f(t) dt . Then the value of f(n 5) is
0

[IIT-JEE - 2009,Paper-2, (4, –1), 80]

UH
3*. Area of the region bounded by the curve y = ex and lines x = 0 and y = e is
[IIT-JEE 2009, P-1, (4, –1), 80]
e 1 e

(A) e – 1 (B) 
1
n (e + 1 − y) dy (C) e – 
0
e x dx (D) 
1
n y dy
JA
x
1 t n (1 + t)
4. The value of lim
x →0 x3 
0 t4 + 4
dt is [IIT-JEE-2010, Paper-1 (3, –1)/84]

1 1 1
(A) 0 (B) (C) (D)
12 24 64

x 4 (1 − x)4
1
LP

5. The value(s) of 


0 1 + x2
dx is (are) [IIT-JEE-2010, Paper-1 (3, 0)/84]

22 2 71 3
(A) – (B) (C) 0 (D) –
7 105 15 2
A

6*. Let f be a real-valued function defined on the interval (0, ) by f(x) = n x +  1 + sin t dt. Then which
0
NK

of the following statement(s) is (are) true? [IIT-JEE-2010, Paper-1 (3, 0)/84]


(A) f(x) exists for all x  (0, )
(B) f(x) exists for all x  (0, ) and f is continuous on (0, ), but not differentiable on (0, )
(C) there exists  > 1 such that |f(x)| < |f(x)| for all x  (, )
(D) there exists  > 0 such that |f(x)| + |f(x)|   for all x  (0, )
 
SA

7. For any real number, let [x] denote the largest integer less than or equal to x. Let f be a real valued
function defined on the interval [–10, 10] by [IIT-JEE-2010, Paper-1 (3, 0)/84]
 x – [x] if [x] is odd,
f(x) = 
1 + [x] – x if [x] is even
2
10

Then the value of  f(x) cos x dx is


10 –10

8. Let f be a real-valued function defined on the interval (–1, 1) such that e –x f(x) = 2 + 
0
t 4 + 1 dt , for all

x  (–1, 1) and let f–1 be the inverse function of f. Then (f –1) (2) is equal to
[IIT-JEE-2010, Paper-2 (5, –2)/84]
1 1 1
(A) 1 (B) (C) (D)
3 2 e
Comprehension (9 to 11)
Consider the polynomial
f(x) = 1 + 2x + 3x2 + 4x3
Let s be the sum of all distinct real roots of f(x) and let t = |s|

9. The real number s lies in the interval. [IIT-JEE 2010, Paper-2, (3, –1), 79]
 1   3  3 1  1
(A)  – , 0  (B)  –11, − (C)  – , – (D)  0 ,
 4   4   4 2
 4 

10. The area bounded by the curve y = f(x) and the lines x = 0, y = 0 and x = t, lies in the interval

RI
[IIT-JEE 2010, Paper-2, (3, –1), 79]
 3   21 11   21 
(A)  , 3  (B)  ,  (D)  0 ,
64 
(C) (9, 10)
4   64 16  

A
11. The function f(x) is [IIT-JEE 2010, Paper-2, (3, –1), 79]
 1  1 
(A) increasing in  –t ,  and decreasing in  – 4 , t 
 4   

UH
 1  1 
(B) decreasing in  –t , –  and increasing in  – , t 
 4  4 
(C) increasing in (–t, t)
(D) decreasing in (–t, t)
JA
n3
x sin x 2
12. The value of  sin x + sin( n6 − x 2 )
2
dx is [IIT-JEE 2011, Paper-1, (3, –1), 80]
n2

1 3 1 3 3 1 3
(A) n (B) n (C) n (D) n
4 2 2 2 2 6 2
LP

13. Let the straight line x = b divide the area enclosed by y = (1 – x) 2, y = 0, and x = 0 into two parts R 1 (0 
1
x  b) and R2(b  x  1) such that R1 – R2 = . Then b equals [IIT-JEE 2011, Paper-1, (3, –1), 80]
4
3 1 1 1
(A) (B) (C) (D)
A

4 2 3 4

14. Let f : [–1, 2] → [0, ) be a continuous function such that f(x) = f(1 – x) for all x  [–1, 2].
2
NK

Let R1 = x
−1
f(x) dx , and R2 be the area of the region bounded by y = f(x), x = –1, x = 2, and the x-

axis. Then [IIT-JEE 2011, Paper-2, (3, –1), 80]


(A) R1 = 2R2 (B) R1 = 3R2 (C) 2R1 = R2 (D) 3R1 = R2

If S be the area of the region enclosed by y = e− x , y = 0, x = 0, and x = 1. Then


2
SA

15*.
[IIT-JEE 2012, Paper-1, (4, 0), 70]
1 1 1 1  1 1  1 
  (A) S  (B) S  1 − (C) S   1 +  (D) S  + 1 − 
e e 4 e 2 e 2

/2
 2 +x
16. The value of the integral   x + n  – x  cos x dx is [IIT-JEE 2012, Paper-2, (3, –1), 66]
–/ 2  
2 2 2
(A) 0 (B) –4 (C) +4 (D)
2 2 2
Paragraph for Question Nos. 17 to 18 
 2(t – 1) 
x
  Let f(x) = (1 – x)2 sin2x + x2 for all x  R and let g(x) =   – nt  f(t) dt for all x  (1, ).
1
t + 1 

17. Which of the following is true ? [IIT-JEE 2012, Paper-2, (3, –1), 66]
(A) g is increasing on (1, )
(B) g is decreasing on (1, )
(C) g is increasing on (1, 2) and decreasing on (2, )
(D) g is decreasing on (1, 2) and increasing on (2, )
 
18. Consider the statements :
P : There exists some x  R such that f(x) + 2x = 2(1 + x 2)

RI
Q : There exists some x  R such that 2f(x) + 1 = 2x(1 + x)
Then
(A) both P and Q are true (B) P is true and Q is false
(C) P is false and Q is true (D) both P and Q are false

A
 
x
19.* If f(x) = e
t2
(t − 2) (t − 3) dt for all x  (0, ), then      
0

UH
  (A) f has a local maximum at x = 2 [IIT-JEE 2012, Paper-2, (4, 0), 66]
(B) f is decreasing on (2, 3)
(C) there exists some c  (0, ) such that f(c) = 0
(D) f has a local minimum at x = 3  

 
JA
20. The area enclosed by the curves y = sinx + cosx and y = |cosx – sinx| over the interval 0,  is
 2 
[JEE (Advanced) 2013, Paper-1, (2, 0)/60]
(A) 4 ( 2 –1 ) (B) 2 2 ( 2 –1) (C) 2 ( 2 +1 ) (D) 2 2 ( )
2 +1

1 
Let f :  , 1 → R (the set of all real numbers) be a positive, non-constant and differentiable function
LP

21.
2 
 1
1

such that f(x) < 2 f(x) and f   = 1. Then the value of  f(x) dx lies in the interval
2 1/ 2

                [JEE (Advanced) 2013, Paper-1, (2, 0)/60]


A

 e – 1   e – 1
(C)  , e – 1 (D)  0 ,
2 
(A) (2e – 1, 2e) (B) (e – 1, 2e – 1)
 2  
NK

(1a + 2a + .... + na ) 1
22. For a  R (the set of all real numbers), a  –1, lim a–1
= . Then
n → (n + 1) [(na + 1) + (na + 2) + ... + (na + n)] 60
a= [JEE (Advanced) 2013, Paper-2, (3, –1)/60]
–15 –17
(A) 5 (B) 7 (C) (D)
2 2
SA

23*. Let f:[a, b] → [1, ) be a continuous function and let g : R → R be defined as




 0 if x  a,
x

g(x) =   f(t)dt if a  x  b, , Then [JEE (Advanced) 2014, Paper-1, (3, 0)/60]
a
b
 f(t)dt if
 a
x  b.

(A) g(x) is continuous but not differentiable at a


(B) g (x) is differentiable on R
(C) g(x) is continuous but not differentiable at b
(D) g(x) is continuous and differentiable at either a or b but not bot h

x  1
– t+  dt
24*. Let f: (0, ) → R be given by f(x) = e
1
 t
t
. Then
x

(A) f(x) is monotonically increasing on [1, ) [JEE (Advanced) 2014, Paper-1, (3, 0)/60]

(B) f(x) is monotonically decreasing on (0, 1)


 1
(C) f(x) + f   = 0, for all x  (0, )
x

RI
(D) f(2 ) is an odd function of x on R
x

1
 d2 2 5
 4x  2 (1 − x )  dx is
3
25. The value of [JEE (Advanced) 2014, Paper-1, (3, 0)/60]
 dx 

A
0

π
2

 (2cosecx)
17

UH
26. The following integral dx is equal to [JEE (Advanced) 2014, Paper-2, (3, –1)/60]
π
4
log(1+ 2 ) log(1+ 2 )

(A) 
0
2(eu + e −u )16 du (B) 
0
(eu + e−u )17 du

log(1+ 2 )
JA log(1+ 2 )

(C) 
0
(eu − e−u )17 du (D) 
0
2(eu − e−u )16 du

Paragraph For Questions 27 and 28

Given that for each a  (0, 1)


1–h

t
LP

–a
lim+ (1– t)a–1 dt
h→0
h

exists. Let this limit be g(a). In addition, it is given that the function g(a) is differentiable
on (0, 1). [JEE (Advanced) 2014, Paper-2, (3, –1)/60]
 1
A

27. The value of g   is


2
 
(A)  (B) 2 (C) (D)
2 4
NK

 1
28. The value of g   is
2
 
(A) (B)  (C) – (D) 0
2 2
SA

29. List I List II


[JEE (Advanced) 2014, Paper-2, (3, –1)/60]
P. The number of polynomials f(x) with non -negative integer 1. 8
1

coefficients of degree  2, satisfying f(0) = 0 and  f(x)dx = 1 , is


0

Q. The number of points in the interval  – 13, 13  at which 2. 2

f(x) = sin(x 2 ) + cos(x 2 ) attains its maximum value, is


2
3x 2
R.  x
–2 (1 + e )
dx equals 3. 4
 1/ 2  1+ x  
  cos 2x log   dx 
 –1/ 2  1– x  
S. equals 4. 0
 1/ 2  1+ x  
  cos 2x log   dx 
0  1– x  

[x], x  2
30. Let f: R → R be a function defined by f(x) =  where [x] is the greatest integer
 0, x  2
2
xf(x 2 )
less than or equal to x. If I =  2 + f(x + 1) dx , then the value of (4 – 1) is
–1

                [JEE (Advanced) 2015, P-1 (4, 0) /88]  

RI
1
 12 + 9x 2 
( )
–1
31. If  = e9x + 3 tan x
 2  dx where tan – 1 x takes only principal values, then the value of
0  1+ x 
 3 

A
 loge | 1 +  | – 4  is [JEE (Advanced) 2015, P-2 (4, 0) / 80]
 

Let f: R → R be a continuous odd function, which vanishes exactly at one point and

UH
32.
x x
1
f(1) = . Suppose that F(x) =  f(t) dt for all x  [–1, 2] and G(x) =  t | f(f(t)) | dt for all
2 –1 –1

F(x) 1  1
x  [–1, 2]. If lim = , then the value of f   is.
x →1 G(x) 14 2
JA
[JEE (Advanced) 2015, P-2 (4, 0) / 80]

  
33*. Let f(x) = 7tan 8 x + 7tan 6 x – 3tan 4 x – 3tan 2 x for all x   – ,  . Then the correct
 2 2
expression(s) is (are)       [JEE (Advanced) 2015, P-2 (4, –2)/ 80]  
/ 4 / 4
1
  0 0 f(x) dx = 0
LP

(A) xf(x) dx = (B)


12
/ 4 / 4
1
(C) 
0
xf(x) dx =
6
(D)  f(x)
0
dx = 1
A

192x 3  1
1

34. Let f'(x) =


2 + sin4 x
for all x  R with f 
2
 = 0. If m   f(x) dx  M, then the possible
1/ 2
values of m and M are [JEE (Advanced) 2015, P-2 (4, –2)/ 80]
NK

1 1
(A) m = 13, M = 24 (B) m = , M =
4 2
(C) m = – 11, M = 0 (D) m = 1, M = 12
SA

35*. The option(s) with the values of a and L that satisfy the following equation is(are)
4

 e (sin
t 6
at + cos 4 at)dt
0

= L ? [JEE (Advanced) 2015, P-2 (4, –2)/ 80]
 e (sin
t 6 4
at + cos at)dt
0

e4  − 1 e4  + 1
(A) a = 2, L = (B) a = 2, L =
e − 1 e + 1
e4  − 1 e4  + 1
(C) a = 4, L = (D) a = 4, L =
e − 1 e + 1
Paragraph For Questions 36 and 37

Let F : R → R be a thrice differentiable function. Supose that F(1) = 0, F(3) = –4 and


F'(x) < 0 for all x  (1/2, 3). Let f(x) = xF(x) for all x  R.
[JEE (Advanced) 2015, P-2 (4, –2)/ 80]
36*. The correct statement(s) is(are)
(A) f '(1) < 0 (B) f (2) < 0
(C) f '(x)  0 for any x  (1, 3) (D) f '(x) = 0 for some x  (1, 3)

3 3

 x F'(x)dx = −12 and  x F"(x)dx = 40 , then the correct expression(s) is(are)


2 3
37*. If
1 1
3

 f(x)dx = 12

RI
(A) 9f '(3) + f '(1) – 32 = 0 (B)
1
3
(C) 9f '(3) – f '(1) + 32 = 0 (D)  f(x)dx = −12

A
1


x2 +
6
 1

UH
38. Let F(x) = 2cos2 t dt for all x  R and f: 0,  → [0, ) be a continuous function. For
x  2
 1
a , 0,  if F(a) + 2 is the area of the region bounded by x = 0, y = 0, y = f(x) and x =
 2
a, then f(0) is [JEE (Advanced) 2015, P-1 (4, 0) /88]  
JA
x
t2
39. The total number of distinct x  (0, 1] for which  1+ t
0
4
dt = 2x – 1 is

[JEE (Advanced) 2016, Paper-1, (3, 0)/62]



2
x 2 cos x
 1 + e x dx is equal to
LP

40. The value of [JEE (Advanced) 2016, Paper-2, (3, –1)/62]



2

 2
2
(A) –2 (B) +2 (C) 2 – e/2 (D) 2 + e/2
4 4
A

41. 
Area of the region ( x, y )  R 2 : y  
x + 3 , 5 y  x + 9  15 is equal to
NK

[JEE (Advanced) 2016, Paper-2, (3, 1)/62]


1 4 3 5
(A) (B) (C) (D)
6 3 2 3

x
 n
 nn ( x + n) x + n ..... x + n  
SA

  2  n 
Let f(x) = lim
2 
42. , for all x > 0. Then
n→ 
2 2  2 n 2
 
 n! ( x + n ) x + ..... x +  
2 n 
  4   n2  
 
                [JEE (Advanced) 2016, Paper-2, (4, –2)/62]
 1  1  2 f (3) f (2)
(A) f    f (1) (B) f    f   (C) f(2)  0 (D) 
2 3 3 f ( 3 ) f ( 2)
43*. Let f : R → (0, 1) be a continuous function. Then, which of the following function(s) has (have) the value
zero at some point in the interval (0, 1) ? [JEE(Advanced) 2017, Paper-1,(4, –2)/61]

x 2
(A) ex –  f (t) sin t dt
0
(B) f(x) +  f (t) sin t dt
0

–x
2
(C) x – 
0
f(t) cos t dt (D) x9 – f(x)


44. Let f : R → R be a differentiable function such that f(0) = 0, f   = 3 and f(0) = 1. If
2

2
 
g(x) =  [f (t)cos ec t − cot t cosec t f(t)] dt for x   0, 2  , then lim
  x →0
g(x) =

RI
x

[JEE(Advanced) 2017, Paper-1,(3, 0)/61]  

k +1
 
98 k +1
45*. If I = dx, then [JEE(Advanced) 2017, Paper-2,(4, –2)/61]

A
k =1 k
x(x + 1)
49 49
(A) I > loge 99 (B) I < loge 99 (C) I < (D) I >
50 50

UH
46*. If the line x =  divides the area of region R = {(x, y)  R2 : x3  y  x, 0  x  1} into two equal parts,
then [JEE(Advanced) 2017, Paper-2,(4, –2)/61]
1 1
(A) 24 – 42 + 1 = 0 (B) 4 + 42 – 1 = 0 (C) <  < 1 (D) 0 <  
2 2
JA
sin(2x )
47. If g(x) =  sin x
sin−1(t)dt, then [JEE(Advanced) 2017, Paper-2,(4, –2)/61]
     
(A) g'  −  = 2 (B) g'  −  = −2 (C) g'   = 2 (D) g'   = −2
 2  2   
2 2
1
48. For each positive integer n, let y n = ((n + 1) (n + 2) … (n + n))1/n.
LP

n
For x  R, let [x] be the greatest integer less than or equal to x. If lim yn = L, then the value of [L] is
n→
________. [JEE(Advanced) 2018, Paper-1,(3, 0)/60]

49. A farmer F1 has a land in the shape of a triangle with vertices at P(0, 0), Q(1, 1) and R(2, 0). From this
A

land, a neighbouring farmer F2 takes away the region which lies between the side PQ and a curve of
the form y = xn (n > 1). If the area of the region taken away by the farmer F 2 is exactly 30% of the area
of PQR, then the value of n is [JEE(Advanced) 2018, Paper-1,(3, 0)/60]
NK

1
2
1+ 3
50. The value of the integral  1
dx is ____. [JEE(Advanced) 2018, Paper-2,(3, 0)/60]
0
((x + 1)2 (1– x)6 ) 4
SA

PART - II : JEE (MAIN) / AIEEE PROBLEMS (PREVIOUS YEARS)


1.  [cot x]dx , where [  ] denotes the greatest integer function, is equal to :
0
[AIEEE 2009 (4, –1), 144]

 
(1) 1 (2) – 1 (3) – (4)
2 2

2. The area of the region bounded by the parabola (y – 2)2 = x – 1, the tangent to the parabola at the point
(2, 3) and the x-axis is [AIEEE 2009 (8, –2), 144]
(1) 6 sq unit (2) 9 sq unit (3) 12 sq unit (4) 3 sq unit
 
3. Let p(x) be a function defined on R such that p(x) = p(1 – x), for all x  [0, 1], p(0) = 1 and p(1) = 41.
1
Then  p(x)dx
0
equals [AIEEE 2010 (8, –2), 144]

(1) 21 (2) 41 (3) 42 (4) 41


 
3
4. The area bounded by the curves y = cos x and y = sinx between the ordinates x = 0 and x = is
2
[AIEEE 2010 (4, –1), 144]
(1) 4 2 + 2 (2) 4 2 – 1 (3) 4 2 + 1 (4) 4 2 – 2

 5 
x
For x   0, , define f(x) =  t sin t dt. Then f has :
2 

RI
5. [AIEEE 2011, I, (4, –1), 120]
 0

(1) local maximum at  and 2.


(2) local minimum at  and 2
(3) local minimum at  and local maximum at 2.

A
(4) local maximum at  and local minimum at 2.

1.5

x

UH
6. Let [.] denote the greatest integer function then the value of [x 2 ] dx is :
0

[AIEEE 2011, II, (4, –1), 120]


3 3 5
(1) 0 (2) (3) (4)
2 4 4
JA
1
7. The area of the region enclosed by the curves y = x, x = e, y = and the positive x-axis is
x
[AIEEE 2011, I, (4, –1), 120]
1 3 5
(1) square units (2) 1 square units (3) square units (4) square units
2 2 2
LP

8. The area bounded by the curves y 2 = 4x and x2 = 4y is : [AIEEE 2011, II, (4, –1), 120]
32 16 8
(1) (2) (3) (4) 0
3 3 3

y
A

9. The area bounded between the parabolas x 2 = and x2 = 9y and the straight line y = 2 is :
4
[AIEEE-2012, (4, –1)/120]
10 2
NK

20 2
(1) 20 2 (2) (3) (4) 10 2
3 3

10.* If g(x) = 
0
cos 4t dt , then g(x + ) equals [AIEEE-2012, (4, –1)/120]
SA

g(x)
(1) (2) g(x) + g() (3) g(x) – g() (4) g(x) . g()
g( )
 
/3
dx
11. Statement-I : The value of the integral  1+
/6 tan x
is equal to /6. [AIEEE - 2013, (4, –1),360]
b b

Statement-II : .  f(x)dx = f(a + b − x) dx


a a

(1) Statement-I is true; Statement-II is true; Statement-II is a correct explanation for Statement-I.
(2) Statement-I is true; Statement-II is true; Statement-II is not a correct explanation for Statement-I.
(3) Statement-I is true; Statement-II is false.
(4) Statement-I is false; Statement-II is true.
12. The area (in square units) bounded by the curves y = x , 2y – x + 3 = 0, x-axis, and lying in the first
quadrant is : [AIEEE - 2013, (4, –1),360]
27
(1) 9 (2) 36 (3) 18 (4)
4


x x
13. The integral 
0
1 + 4 sin2
2
− 4 sin dx equals :
2
[JEE(Main)2014,(4, – 1), 120]

 2
(1) 4 3 − 4 (2) 4 3 − 4 − (3)  – 4 (4) −4−4 3
3 3

RI
14. The area of the region described by A = {(x, y) : x 2 + y2  1 and y2  1 – x} is :
[JEE(Main)2014,(4, – 1), 120]
 2  2  4  4
(1) – (2) + (3) + (4) –
2 3 2 3 2 3 2 3

A
4
log x 2
15. The integral 2 log x 2 + log(36 – 12x + x 2 ) dx is equal to [JEE(Main)2015,(4, – 1), 120]

UH
(1) 2 (2) 4 (3) 1 (4) 6

16. The area (in sq. units) of the region described by {(x, y); y 2  2x and y  4x – 1} is
[JEE(Main)2015,(4, – 1), 120]
7 5 15 9
(1) (2) (3) (4)
JA
32 64 64 32

17. The area (in sq.units) of the region {(x,y) : y 2  2x and x2 + y2  4x, x  0, y  0} is
[JEE(Main)2016,(4, – 1), 120]
8 4 2  2 2 4
(1)  – (2)  – (3) – (4)  –
3 3 2 3 3
LP

1/ n
 (n + 1)(n + 2).......3n 
18. lim   is equal to : [JEE(Main)2016,(4, – 1), 120]
n→
 n 2n 
27 9 18
(1) (2) (3) 3 log3 – 2 (4)
A

e2 e2 e4

19. The area (in sq. units) of the region {(x, y) : x  0 , x + y  3 , x 2  4y and y  1 + x } is :
NK

[JEE(Main) 2017, (4, – 1), 120]


59 3 7 5
(1) (2) (3) (4)
12 2 3 2
3
4
dx
 1 + cos x
SA

20. The integral is equal to [JEE(Main) 2017, (4, – 1), 120]



4
(1) –2 (2) 2 (3) 4 (4) –1

2
sin 2 x
21. The value of

 1+ 2 x
dx is :     [JEE(Main) 2018, (4, – 1), 120]

2
  
(1) 4 (2) (3) (4)
4 8 2
22. Let g(x) = cosx2, f(x) = x , and ,  ( < ) be the roots of the quadratic equation 18x 2 – 9x + 2 = 0.
Then the area (in sq. units) bounded by the curve y = (gof) (x) and the lines x=, x =  and y = 0, is
[JEE(Main) 2018, (4, – 1), 120]
1 1 1 1
(1)
2
( 3− 2 )
(2)
2
2 −1 ( (3)
2
) 3 −1 ( (4)
2
) 3 +1 ( )
b

Let  =  (x − 2x 2 )dx . If  is minimum then the ordered pair (a, b) is :


4
23.
a

              [JEE(Main) 2019, Online (10-01-19),P-1 (4, – 1), 120]


(1) (0, 2) (2) ( 2 ,– 2 ) (3) (– 2 , 2 ) (4) (– 2 , 0)

/2

RI
dx
24. The value of

− / 2
[ x] + [sin x] + 4
, where [t] denotes the greatest less than or equal to t, is :

              [JEE(Main) 2019, Online (10-01-19),P-2 (4, – 1), 120]

A
1 3 3 1
(1) (7 – 5) (2) (4 – 3) (3) (4 – 3) (4) (7 + 5)
12 10 20 12

 x  2 x  e  x 
e

UH
25. The integral
1 
e 
  –    log e x dx is equal to
 x  
[JEE(Main) 2019, Online (12-01-19),P-2 (4, – 1), 120]
3 1 1 3 1 1 1 1 1 1
(1) – – 2 (2) – e – 2 (3) – e – 2 (4) – + –
2 e 2e 2 2e 2 e 2 e 2e 2
JA
LP
A
NK
SA
EXERCISE - 1

PART - I
Section (A) :
10  8
A-1. (i) − (ii) 2 −1 A-2. (i)  (ii) (iii) 4 + n 5 (iv)
21 4 21
−2 1 e  2 3

RI
A-3. (i) (ii) n   (iii) – A-4. (i) n ( 3 ) (ii) 7/6 (iii)
2 2 2 6 9 8

 4−   1 
A-5. (i) – n 2 (ii) (iii) – (b – a)2 (iv)  1 −  – n 4
2 4 2 8  3

A
 5 9  1
A-6. (i) (ii) − 2 n2 (iii) n   (iv) (v) n 3
4 3 8 2 20

UH
1
A-7 (i) n 11 (ii)
3
Section (B) :
JA
B-2. (i) 5 – 2 – 3 (ii) 2 2 (iii) 9 (iv) 4 (v) cot 1 (vi) 29
(vii) cos 1 + cos 2 + cos 3 + 3
2
B-3. (i) 2e – 2 (ii) 2 – 2 (iii) (iv) 0 (v) 0
6 3
  a 
LP

B-4. (i) (ii) (iii) (iv) (a + b) (v) 0


4 4 2 4
 
B-5. (i) 0 (ii) (iii) – n 2 (iv) n2
3 2
A

3
B-6. (i) (ii) 40 (iii) n – 1 (iv) 4n
2
Section (C) :
NK

48 1 1
C-1. (i) 4 2 (ii) 12 (iii)  −   C-2. (ii) 1, 3 C-3. 5/2 C-4.
 3 4  2

4 8  2
C-5. e C-6. (i) (ii) (iii) (iv)
SA

15 15 2 4
Section (E) :

E-1. (i) (ii) 2 (iii) 12
2
Section (F) :
51  4 7
F-1. sq. unit F-2. (i) − (ii) (iii) 9 
4 2  120
(e + 1)  1  3 4
F-3. 4/3 sq. units F-4. F-5. (i) 3 ( − 2) (ii) F-6.  −  sq. units
1 + 2 8  loge 2 3 

16
F-8. 4 sq. units. F-9. sq. units.
3
PART - II

SECTION (A) :  
A-1. (A) A-2. (C) A-3. (C) A-4. (C) A-5. (D) A-6. (A) A-7. (B)
A-8. (A) A-9. (A) A-10. (B) A-11. (D) A-12. (C) A-13. (C)

RI
Section (B) :
B-1. (C) B-2. (C) B-3. (D) B-4. (C) B-5. (D) B-6. (A) B-7. (B)
B-8. (D) B-9. (C) B-10. (A) B-11. (C) B-12. (A) B-13. (C)

A
Section (C) :
C-1. (A) C-2. (B) C-3. (B) C-4. (D) C-5. (B) C-6. (C) C-7. (C)

UH
C-8. (B) C-9. (D) C-10. (A)
SECTION (D) :
D-1. (C) D-2. (B) D-3. (C) D-4. (A)
JA
SECTION (E) :
E-1. (D) E-2. (B) E-3. (C) E-4. (C) E-5. (D)
Section (F) :
F-1. (C) F-2. (D) F-3. (C) F-4. (A) F-5. (C) F-6. (B) F-7. (A)
LP

F-8. (A) F-9. (C) F-10. (D)


 
PART - III

1. A-q, B-s, C-p, D-t 2. (A)→(r), (B) → (s), (C) → (q), (D) → (p)
A

EXERCISE - 2
NK

PART - I
1. (C) 2. (B) 3. (A) 4. (C) 5. (D) 6. (A) 7. (D)
8. (B) 9. (C) 10. (D) 11. (A) 12. (B) 13. (B) 14. (C)
15. (B) 16. (C) 17. (C) 18. (B) 19. (C) 20. (A) 21. (A)
SA

22. (A) 23. (D) 24. (D) 25. (D) 26. (B) 27. (A) 28. (D)
29. (B) 30. (A)
PART - II
1. 61 2. 64 3. 10 4. 29 5. 4 6. 4 7. 2
8. 2 9. 0 10. 2 11. 8 12. 16 13. 25 14. 4
15. 65 16. 5 17. 11 18. 3 19. 2 20. 0 21. 1
22. 22 sq.unit 23. 55
PART - III
1. (AC) 2. (ABCD) 3. (AB) 4. (AB) 5. (ABC) 6. (ABC) 7. (AB)
8. (BD) 9. (BCD) 10. (BC) 11. (AD) 12. (ABCD) 13. (ABCD)
14. (ABD) 15. (AD) 16. (AC) 17. (AB) 18. (BD) 19. (ABD) 20. (AD)
21. (AC) 22. (BC) 23. (ACD) 24. (ABC) 25. (ABCD) 26. (ABC)
27. (ABCD) 28. (AD) 29. (AB) 30. (BD)
PART - IV
1. (C) 2. (A) 3. (A) 4. (D) 5. (A) 6. (BC) 7. (C)
8. (B) 9. (D)

RI
EXERCISE - 3

PART - I

A
1*. (ABC) 2. 0 3*. (BCD) 4. (B) 5. (A) 6*. (BC)

7. 4 8. (B) 9. (C) 10. (A) 11. (B) 12. (A) 13. (B)

UH
14. (C) 15.* (ABD) 16. (B) 17. (B) 18. (C) 19. (ABCD) 20. (B)

21. (D) 22. (B) 23.* (AC) 24.* (ACD) 25. 2 26. (A) 27. (A)

28. (D) 29. (D) 30. 0 31. 9 32. 7  33*. (AB) 34*. (D)
JA
35*. (AC) 36*. (ABC) 37*. (CD) 38. 3 39. 1 40. (A) 41. (C)

42*. (BC) 43. (CD) 44. (2) 45. (BD) 46. (AC) 47. (BONUS)

48. (1) 49. (4) 50. (2)


LP

PART - II
1. (3) 2. (2) 3. (1) 4. (4) 5. (4) 6. (3) 7. (3)
A

8. (2) 9. (3) 10.* (2) or (3) 11. (4) 12. (1) 13. (2) 14. (3)
15. (3) 16. (4) 17. (1) 18. (1) 19. (4) 20. (2) 21. (2)
22. (3) 23. (3) 24. (3) 25. (2)
NK
SA
 
2 2
4a
 (sin x + acos x) dx −  − 2  x cos xdx = 2
3
1. Find the integral value of a for which
0 0

2. Evaluate :


0
(cos x + cos2x + cos3x)2 + (sin x + sin2x + sin3x)2 dx

RI
1
3. Let  &  be distinct positive roots of the equation tanx = 2x, then evaluate  sin(x).sin(x)dx
0

A
4. Evaluate %
a
lim

a→ 
−  (cos x)ln(cos x)dx

UH
0
2

5. Find the value of a(0 < a < 1) for which the following definite integral is minimized.

 | sin x − ax | dx
0
JA
1
 3n C n
Find the Lim  2n n

6.
n→ 
 Cn 
i.(i − 1)....(i − j + 1)
where iCj is a binomial coefficient which means
j .(j − 1)....2.1
LP

 
a x nx a x dx
7. Show that 
0
f x + a .
  x
dx = na .  f 
0  x
+  .
a  x

1
A

n
8. Evaluate lim n2
n→  (2014 sin x + 2015 cos x) | x | dx
1

NK

9. Let sequence {an} be defined as


1
2

a1 =
4
, an =  (cos(x) + a n −1 ) cosx dx, (n = 2, 3, 4, .......)
SA

0
then evaluate lim an
n→

 (x + ye
x
10. Find f(x) if it satisfies the relation f(x) = e x + ) f(y) dy.
0

 
1 3
x4  2x 
11. Evaluate :  1− x 4
cos−1  2 
 1+ x 
dx.
−1 3
1
1
12. Evaluate  (2 − 4x )
dx
0 ( 5 + 2x − 2x ) (1 + e
2
)
13. Prove that for any positive integer k ;
sin2k x
= 2 [cos x + cos 3x +..... + cos (2k − 1)x]. Hence prove that;
sin x
/2


0
sin (2kx). cot x dx =
2
.

 2p  2p 2 2p 3 2p   2 r −1
p
1
14. Prove that Limit 

cos + cos + cos + ...... + cos = 
2  r = 1 2 r
,

RI
n→ n 2n 2n 2n
where  denotes the continued product and p  N.

dx

A
15. If n > 1, evaluate n
0
(x + 1+ x 2 )

UH
a
16. Let f(x) be a continuous function  x  R, except at x = 0 such that  f(x)dx ,
0
a  R+ exists. If
a a a
f(t)
g(x) = 
x
t
dt, prove that 
0
g(x) dx = 
0
f(x) dx
JA
loge (n2 + r 2 ) – 2loge n
n

17. Given that lim
r =1n→
 n
= loge2 + – 2, then
2
1
evaluate : lim 2m [(n2 + 12)m (n2 + 22)m ....... (2n2)m]1/n.
n→ n
/ 4

 (tan x)
2n
18. For a natural number n, let an = dx
LP

Now answer the following questions :


(1) Express an+1 in terms of an
(2) Find lim an
n→
A

n
(3) Find lim
n→  (–1)
k =1
k –1
(ak + ak–1 )
NK

t2 x
0 a + t dt
19. Given that lim = 1, then find the values of a and b
x →0 bx − sin x

x
 
SA

 sec dt  (m + 1)x  x   0,  mN


m
20. Prove that m sin x + t
0  2 

21. f(x) is differentiable function: g(x) is double differentiable function such that |f(x)|  1 and g(x) = f ’(x). If
f2(0) + g2(0) = 9 then show that there exists some C  (–3, 3) such that g(c) g" (c) < 0

22. Draw a graph of the function f (x) = cos −1 (4x3 − 3x), x  [−1, 1] and find the area enclosed between the
graph of the function and the x−axis as x varies from 0 to 1.

23. Consider a square with vertices at (1, 1), (− 1, 1), (−1, −1) and (1, −1). Let S be the region consisting of
all points inside the square which are nearer to the origin than to any edge. Sketch the region S and
find its area.
24. If [x] denotes the greatest integer function. Draw a rough sketch of the portions of the curves
x 2 = 4  x  y and y 2 = 4  y  x that lie within the square {(x, y) | 1 < x < 4, 1 < y < 4} Find the
   
area of the part of the square that is enclosessd by the two curves and the line x + y = 3
 
25. Find the area of the region bounded by y = f(x) , y = | g(x) | and the lines x = 0, x = 2, where ‘f’ , ‘g’ are
continuous functions satisfying f(x + y) = f(x) + f(y) – 8xy  x, y  R and g(x + y) = g(x) + g(y) + 3xy(x+y)
x, y  R also f’(0) = 8 and g’(0) = – 4.

 −2 , −3  x  0
26. Let f(x) =  , where g(x) = min {f(|x|) + |f(x)|, f(|x|) – |f(x)|}
x − 2 , 0  x  3
Find the area bounded by the curve g(x) and the x-axis between the ordinates x = 3 and x = –3.

RI
27. Find the area of region {(x, y) : 0  y  x2 + 1, 0  y  x + 1, 0  x  2} .
 
28. A curve y = f(x) passes through the point P(1, 1), the normal to the curve at P is a(y – 1) + (x – 1) = 0. If

A
the slope of the tangent at any point on the curve is proportional to the ordinate of that point, determine
the equation of the curve. Also obtain the area bounded by the y-axis, the curve and the normal to the
curve at P.

UH
29. Find the area bounded by y = [– 0. 01 x4 – 0.02 x2], (where [ . ] G.I.F.) and curve 3x2 + 4y2 = 12, which
lies below y = – 1.

30. Let ABC be a triangle with vertices A(6, 2( 3 + 1)), B(4, 2) and C(8, 2). If R be the region consisting of
all these points and point P inside ABC which satisfy d(P, BC)  max. {d(P, AB), d(P, AC)}
JA
where d(P, L) denotes the distance of the point P from the line L. Sketch the region R and find its area.
 
31. Find the area of the region which contains all the points satisfying the condition |x – 2y| + |x + 2y|  8
and xy  2.

32. Find the area of the region which is inside the parabola y = – x 2 + 6 x – 5, out side the parabola
LP

y = − x2 + 4 x − 3 and left of the straight line y = 3 x − 15.

33. Consider the curve C: y = sin 2x − 3 sin x, C cuts the x − axis at (a, 0), x  (− , ).
A1 : The area bounded by the curve C and the positive x − axis between the origin and the line x = a.
A2 : The area bounded by the curve C and the negative x − axis between the line x = a and the
A

origin.
Prove that A1 + A2 + 8 A1 A2 = 4.
NK

34. Area bounded by the line y = x, curve y = f(x), (f(x) > x  x > 1) and the lines x = 1, x = t is
(t + 1 + t 2 – (1 + 2 )  t > 1. Find f(x) for x > 1.

35. Consider the two curves y = 1/x² and y = 1/[4 (x − 1)].


SA

(i) At what value of ‘a’ (a > 2) is the reciprocal of the area of the figure bounded by the curves, the
lines x = 2 and x = a equal to ‘a’ itself ?

(ii) At what value of ‘b’ (1 < b < 2) the area of the figure bounded by these curves, the lines
x = b and x = 2 equal to 1 − 1/b.

36. Let C1 and C2 be the graphs of the functions y = x 2 and y = 2x, 0  x  1 respectively. Let C 3 be the
graph of a function y = f(x), 0  x  1, f(0) = 0. For a point P on C 1, let the lines through P, parallel to the
axes, meet C2 and C3 at Q and R respectively (see figure). If for every position of P (on C 1), the areas of
the shaded regions OPQ and ORP are equal, determine the function f(x).
37. Given the parabola C : y = x 2. If the circle centred at y axis with radius 1 touches parabola C at two

RI
distinct points, then find the coordinate of the center of the circle K and the area of the figure
surrounded by C and K.

 4a2 4a 1   f( −1) 3a2 + 3a 

A
     
38. If  4b2 4b 1   f(1)  = 3b2 + 3b  , f(x) is a quadratic function and its maximum value occurs at a
 4c 2 4c 1   f(2)  3c 2 + 3c 
   

UH
point V. A is a point of intersection of y = f(x) with x-axis and point B is such that chord AB subtends a
right angle at V. Find the area enclosed by f(x) and chord AB.

a2 b2
39_. f(x) and g(x) are polynomials of degree 2 such that
 (f (x) − 1)dx =  (g(x) − 1)dx
JA
a1 b1

where a1, a2 (a2 > a1) are roots of equation f(x) = 1 and b 1, b2 (b2 > b1) are roots of equation g(x) = 1. If
f(x) and g(x) are positive constant and
a2 b2 b2 b2

 f(x))dx = (a2 − a1 ) −
a1
 (f(x) − 1)dx but
b1
 g(x))dx  (b2 − b1 ) −
b1
 (g(x) − 1)dx
b1
then
LP

(A) |f(x)| < |g(x)| (B) |f(x)| > |g(x)| (C) a2 – a1 > b2 – b1 (D) a2 – a1 > b2 – b1

x y i x y i x2 y2
40_. Let L = 4x – 5y, Li = + – , Li = + + , and E = + –1.
10 8 n 10 8 n 50 32
A

Let Ai represents the area of region common between L i–1 > 0, Li < 0, E < 0 and L < 0;
A'i represents the area of region common between L' i–1 < 0, L'i > 0, E < 0 and L < 0;
Bi represents the area of region common between L i–1 > 0, Li < 0, E < 0 and L > 0;
NK

B'i represents the area of region common between L' i–1< 0, L'i > 0, E < 0 and L > 0, then value of
(A1 + A'2 + A3 + A'4 + …..) + (B1 + B'2 + B3 + B'4 + …..) is equal to.
SA
 2   
1. –1 2. +2 3 3. 0 4. n2 – 1 5. a= sin  
3   2
27  −3e x
6. 8. 2015 9. 10. – 3x
16 4(  − 1) 2(e − 1)

 2  1 1 11 + 1 n
11.
4
(
n 2 + 3 +
12

3
) 12.
2 11
n
11 − 1
15.
n −1
2

RI
m
 2 e  1 
17.  2  18. (1) – an (2) 0 (3)
 e  2n + 1 4
 

A
1
19. a = 4, b = 1 22. 3( 3 − 1) sq. units     23. (
16 2 − 20 )
3

UH
19 4 23 23  e−a 1 
24. 25. 26. 27. 28. y = ea(x – 1) ,  1 + − 
6 3 2 JA 6  a 2a 

2 2 2 4 3 73 x
29. 2 3 sin–1 – 30. 31. 2(6 – 2 log 4) 32. 34. 1 + x + .
3 3 3 6 1 + x2

35. (i) a = 1 + e2 (ii) b = 1 + e −2 36. f(x) = x3 − x2


LP

 5 3 3  125
37. centre  0,  and area = − 38. square units. 39. (AC) 40. 20 
 4 4 3 3
A
NK
SA
DIFFERENTIAL EQUATION

It is not certain that everything is uncertain ...........Pascal, Blaise


Introduction :
An equation involving independent and dependent variables and the derivatives of the dependent
variables is called a differential equation. There are two kinds of differential equation:
1.1Ordinary Differential Equation : If the dependent variables depend on one independent variable x,
then the differential equation is said to be ordinary.
dy dz
for example + = y + z,
dx dx

RI
dy d3 y dy
+ xy = sin x , +2 + y = ex ,
dx dx3 dx
3/2
d2 y   dy 2  dy   dy 2 
k = 1 +   , y=x +k 1 +   
dx 

A
dx 2    
dx   dx  

1.2 Partial differential equation : If the dependent variables depend on two or more indepen

UH
dent variables, then it is known as partial differential equation
 z 2z 2z 2z
for example y2 +y = ax, + =0
 x  y2  x2  y2
Order and Degree of a Differential Equation:
JA
2.1 Order : Order is the highest differential appearing in a differential equation.
2.2 Degree :
It is determined by the highest degree of the highest order derivative present in it after the differential
equation is cleared of radicals and fractions so far as the derivatives are concerned.
Note : In the differential equation, all the derivatives should be expressed in the polynomial form
n n
 dm y  1  dm −1y  2 n
 dy  k
LP

f1 (x, y)  m  + f2 (x, y)  m −1  + ........ fk(x, y)   = 0


 dx   dx   dx 
The above differential equation has the order m and degree n 1.

Example # 1:Find the order & degree of following differential equations.


A

1/ 4
d2 y   dy  
6
 d3 y  dy 2 
(i) = y +    (ii) y = loge  3 +   
 dx  dx  
dx 2   dx   
NK

 dy d2 y 
tan–1  x
 dx dx 2 
(iii) + =y (iv) ey – xy + y = 0
 
4
 d2 y   dy 
6
Solution : (i)  2  = y +    order = 2, degree = 4
 dx   dx 
SA

2
d3 y  dy 
(ii) +   = ey  order = 3, degree = 1
dx 3  dx 
d2 y dy
(iii) +x = tan y  order = 2, degree = 1
dx 2 dx
d3 y
3 d2 y
(iv) e dx –x+y=0
dx 2
equation can not be expressed as a polynomial in differential coefficients, so degree is not
applicable but order is 3.

Self Practice Problems :


(1) Find order and degree of the following differential equations.
dy 1 dy dy
(i) +y= (ii) –4 – 7x = 0
dx dy dx dx
dx
2
 dy 1/ 2  d2 y
(iii)   + y  =
 dx   dx 2
Ans. (1) (i) order = 1, degree = 2 (ii) order = 1, degree = 2
(iii) order = 2, degree = 2

Formation of Differential Equation:


Differential equation corresponding to a family of curve will have :
(a) Order exactly same as number of essential arbitrary constants in the equation of curve.
(b) No arbitrary constant present in it.

RI
The differential equation corresponding to a family of curve can be obtained by using the following
steps:
(i) Identify the number of essential arbitrary constants in equation of curve.

A
NOTE : If arbitrary constants appear in addition, subtraction, multiplication or division, then we can club
them to reduce into one new arbitrary constant.
(ii) Differentiate the equation of curve till the required order.
(iii) Eliminate the arbitrary constant from the equation of curve and additional

UH
equations obtained in step (ii) above.

Example # 2 : Form a differential equation of family of straight lines passing through (0,2)
Solution : Family of straight lines passing through (0,2) is y = mx + 2 where’m’ is a
parameter.
Differentiating w.r.t. x
JA
dy
=m
dx
Eliminating ‘m’ from both equations, we obtain
dy
y=x +2 which is the required differential equation.
dx
LP

Example # 3 :Form a differential equation of family of parabolas having x axis as line of symmetry and tangent
at vertex is y-axes
Solution : Let equation of parabola
y2 = 4ax ..........(i)
dy
A

2y = 4a .........(ii)
dx
by (i) and (ii)
dy dy
NK

 y2 = 2yx  y = 2x
dx dx
Self Practice Problems :
(2) Obtain a differential equation of the family of curves y = a sin (bx + c) where a and c being
arbitrary constant.
SA

(3) Show that the differential equation of the system of parabolas y 2 = 4a(x – b) is given by
2
d2 y  dy 
y 2
+   =0
dx  dx 
(4) Form a differential equation of family of parabolas with focus as origin and axis of symmetry
along the x-axis.
2
d2 y  dy  dy
Ans. (2) 2
+ b2y = 0 (4) y2 = y2   + 2xy
dx  dx  dx
Solution of a Differential Equation:
Finding the dependent variable from the differential equation is called solving or integrating it. The
solution or the integral of a differential equation is, therefore, a relation between dependent and
independent variables (free from derivatives) such that it satisfies the given differential equation
NOTE : The solution of the differential equation is also called its primitive, because the differential
equation can be regarded as a relation derived from it.
There can be three types of solution of a differential equation:
(i) General solution (or complete integral or complete primitive) : A relation in x and y satisfying a
given differential equation and involving exactly same number of arbitrary constants as order of
differential equation.
(ii) Particular Solution : A solution obtained by assigning values to one or more than one arbitrary
constant of general solution.
(iii) Singular Solution : It is not obtainable from general solution. Geometrically, Singular solution
acts as an envelope to General solution.
4.1. Differential Equation of First Order and First Degree :
dy
A differential equation of first order and first degree is of the type
+ f(x, y) = 0, which can also be
dx
written as : Mdx + Ndy = 0, where M and N are functions of x and y.

Solution methods of First Order and First Degree Differential Equations :

RI
5.1 Variables separable : If the differential equation can be put in the form, f(x) dx = (y) dywe say that
variables are separable and solution can be obtained by integrating each side separately. A general

 
solution of this will be f(x) dx= (y) dy + c, where c is an arbitrary constant.

A
Example # 4 : Solve the differential equation (1 + x) y dx = (y – 1) x dy

UH
Solution : The equation can be written as -
 1+ x   y − 1 1   1
 x  dx =  y  dy
   


 x + 1 dx =
  1 − y  dy
n x + x = y – ny + c  ny + nx = y – x + c  xy = ce y–x
x x
Example # 5 : Solve : (e + 1) y dy = (y + 1) e dx
JA
Solution : The given differential equation is (e x + 1) y dy = (y + 1) ex dx
ydy ex
= x
(y + 1) (e + 1)
Integrating both sides
 y – log |y + 1|= log (ex + 1) + log k  y = log |(y + 1)(ex + 1)| + log k  (y + 1)(ex + 1)=eyc
LP

dy x(2 nx + 1)
Example # 6 : = Solve :
dx sin y + y cos y
dy x(2 nx + 1)
Solution : =
dx sin y + y cos y
A

(siny + ycosy)dy = x(2nx + 1)dx


Integrating both sides

x
2
1 x   x
2
 – cosy + {(ysiny) + cosy} = 2 ×  ln x − 
dx  +  y sin y = x2 n x
NK


 2 2 1 
 2
5.1.1 Polar coordinates transformations :
Sometimes transformation to the polar co-ordinates facilitates separation of variables. In this
connection it is convenient to remember the following differentials:
(a) If x = r cos  ; y = r sin  then,
SA

(i) x dx + y dy = r dr (ii) dx2 + dy2 = dr2 (iii) x dy – y dx = r2d


(b) If x = r sec  & y = r tan  then
(i) x dx – y dy = r dr (ii) x dy – y dx = r 2 sec d.

Example # 7 : Solve the differential equation xdx + ydy = x (xdy – ydx)


Solution : Taking x = r cos, y = r sin
x2 + y2 = r2
2x dx + 2ydy = 2rdr
xdx + ydy = rdr .........(i)
dy
x −y
y dx d
= tan  2
= sec2 .
x x dx
xdy – y dx = x2 sec2 . d
xdy – ydx = r2 d ........(ii)
Using (i) & (ii) in the given differential equation then it becomes
r dr = r cos. r2 d
dr 1 1 y y +1
2
= cos d – = sin +   – = +  =c
r r x +y2 2 2
x +y 2
x2 + y2
where –  = c  (y + 1)2 = c(x2 + y2)

5.1.2 Equations Reducible to the Variables Separable form : If a differential equation can be reduced
into a variables separable form by a proper substitution, then it is said to be
dy
“Reducible to the variables separable type”. Its general form is = f(ax + by + c) a, b  0. To solve
dx

RI
this, put ax + by + c = t.

dy
Example # 8 : Solve = (4x + y + 1)2

A
dx
dy dt dy dt
Solution : Putting 4x + y + 1 = t  4+ =  = –4
dx dx dx dx

UH
Given equation becomes
dt dt
– 4 = t2  2
= dx (Variables are separated)
dx t +4
Integrating both sides,
 4x + y + 1 
dt
JA
 dx 
1 t 1
4+t 2
=
2
tan–1
2
=x+c 
2
tan–1 
 2  =x+c

 dy 
Example # 9 : Solve sin–1   =x+y
 dx 
dy
LP

Solution : = sin (x + y)
dx
putting x + y = t
dy dt dt dt dt
= –1 – 1 = sin t  = 1 + sin t  = dx
dx dx dx dx 1 + sin t
A

Integrating both sides,


1 − sin t
 1 + sin t =  dx
dt
  cos   (sec t − sec t tan t) dt = x + c
2
2
dt = x + c
NK

t
1 − sin t
tan t – sec t = x + c  – =x+c  sin t – 1 = x cos t + c cos t
cos t
substituting the value of t
sin (x + y) = x cos (x + y) + c cos (x + y) + 1
SA

Self Practice Problems :


dy
(5) Solve the differential equation x 2 y = (x + 1) (y + 1)
dx
xdx + ydy ydx − xdy
(6) Solve the differential equation =
2
x +y 2 x2
dy
(7) Solve : = ex + y + x2ey
dx
dy
(8) Solve : xy = 1 + x + y + xy
dx
dy
(9) Solve = 1 + ex – y
dx
dy
(10) = sin(x + y) + cos (x + y)
dx
dy
(11) Find the solution of the differential equation (x + y) 2 = 1, satisfying the condition y(1) = 0
dx
1 y
Ans. (5) y – n (y + 1) = nx – +c (6) x2 + y2 + =c
x x
1 x3
(7) – = ex + +c (8) y = x + n |x (1 + y)| + c
ey 3
x+y
(9) ey – x = x + c (10) log tan +1 =x+c
2

(11) y+ = tan–1(x + y)
4

RI
5.2 Homogeneous Differential Equations :
dy f(x,y)
A differential equation of the form = where f and g are homogeneous function of
dx g(x,y)

A
x and y, and of the same degree, is called homogeneous differential equaiton and can be solved easily
by putting y = vx.
y

UH
dy
Example # 10 : Solve x = y + x tan  
dx x
dy y y
Solution : = + tan (Homogeneous differential equation)
dx x x
dv dx
put y = vx  v+x = v + tan v  cot v.dv =
JA
dx x
Integrating both sides we have
y
n sin v = nx + nc  sinv = cx  sin   = cx
x
Example # 11 : Solve : x2dy + y (x + y)dx = 0 given that y = 1 when x = 1
dy y y2
=– − 2
LP

Solution : put y = vx
dx x x
dy dv dv dv dx
=v+x  v+x = – v – v2  =–
dx dx dx (2 + v)v x
1 1 1  dx 1  v 
   v – v + 2  –  n + nx = nC
A

2  v + 2 
dv =
2 x
v y
 x =C  x =C
v+2 y + 2x
NK

1
When x = 1 then y = 1  C =  3x2y = (y + 2x)
3
5.2.1 Equations Reducible to the Homogeneous form
dy ax + by + c
Equations of the form = .........(1)
SA

dx Ax + By + C
can be made homogeneous (in new variables X and Y) by substituting x = X + h and y = Y + k, where h
dY aX + bY + (ah + bk + c)
and k are constants to obtain, = ........(2)
dX AX + BY + (Ah + Bk + C)
These constants are chosen such that ah + bk + c = 0, and Ah + Bk + C = 0. Thus we obtain the
dY aX + bY
following differential equation =
dX AX + BY
The differential equation can now be solved by substituting Y = vX.
dy x + 2y − 5
Example # 12 : Solve the differential equation =
dx 2x + y − 4
Solution : Let x = X + h, y=Y+k
dy d
= (Y + k)
dX dX
dy dY dx
= ...........(i) =1+0 ...........(ii)
dX dX dX
dy dY
on dividing (i) by (ii) =
dx dX
dY X + h + 2(Y + k) − 5 X + 2Y + (h + 2k − 5)
= =
dX 2X + 2h + Y + k − 4 2X + Y + (2h + k − 4)
h & k are such that h + 2k – 5 = 0 & 2h + k – 4 = 0
h = 1, k = 2
dY X + 2Y
= which is homogeneous differential equation.
dX 2X + Y
Now, substituting Y = vX
1 + 2v dX

dY dv dv 2+v
=v+X X = –v   dv =

RI
dX dX dX 2+v 1− v 2 X
 1 3  1 3
  +  dv = n X + c
 2(v + 1) 2(1 − v) 
 n (v + 1) –
2 2
n (1 – v) = n X + c

v +1 (Y + X) X2

A
n = nX2 + 2c  = e2c
(1 − v)3 (X − Y)3 X2
X + Y = c(X – Y)3 where e2c = c

UH
x – 1 + y – 2 = c (x – 1– y + 2)3
x + y – 3 = c (x – y + 1)3
Special case :
a b
Case - 1 In equation (1) if = , then the substitution ax + by = v will reduce it to the form in which variables
A B
JA
are separable.
dy 2x + 3y − 1
Example # 13 : Solve =
dx 4x + 6y − 5
Solution : Putting u = 2x + 3y
du dy 1  du  u −1 du 3u − 3 + 4u − 10
=2+3.   − 2 =  =
dx dx 3  dx  2u − 5 dx 2u − 5
LP

2u − 5

2

9 1
 7u − 13
dx = dx 
7
1.du –
7 7u − 13 
. du = x + c

2 9 1 9
 u– . n (7u – 13) = x + c  4x + 6y – n (14x + 21y – 13) = 7x + 7c
7 7 7 7
A

9
 – 3x + 6y – n (14x + 21y – 13) = c
7
Case - 2 In equation (1), if b + A = 0, then by a simple cross multiplication equation (1) becomes an
NK

exact differential equation.

dy x − 2y + 5
Example # 14 : Solve =
dx 2x + y − 1
Solution : Cross multiplying,
SA

2xdy + y dy – dy = xdx – 2ydx + 5dx


2 (xdy + y dx) + ydy – dy = xdx + 5 dx
2 d(xy) + y dy – dy = xdx + 5dx
On integrating,
y2 x2
2xy + –y= + 5x + c  x2 – 4xy – y2 + 10x + 2y = c where c = – 2c
2 2

Case - 3 If the homogeneous equation is of the form :


yf(xy) dx + xg(xy)dy = 0, the variables can be separated by the substitution xy = v.

Self Practice Problems :


Solve the following differential equations
 dy  y dy y
(12)  x dx − y  tan x = x given that y = 0 at x = 1 (13) x = y – x tan
–1

  dx x
dy 2x – y + 3
(14) = (15) (3x – 2y + 1) dy + (4y – 6x + 3)dx = 0
dx x + 2y + 4
dy 3x + 2y − 5
(16) =
dx 3y − 2x + 5
y y
tan−1 y
Ans. (12) x2 + y2 = e x x (13) x sin =C
x
(14) y2 – x2 + xy + 4y – 3x + C = 0 (15) 10 n |3x – 2y – 9| = 2y – 4x + C
(16) 3x + 4xy – 3y – 10x – 10y = C
2 2

5.3 Exact Differential Equation :


dy
The differential equation M + N =0 ...........(1)

RI
dx
Where M and N are functions of x and y is said to be exact if it can be derived by direct differentiation
(without any subsequent multiplication, elimination etc.) of an equation of the form f(x, y) = c
dx
e.g. y2 dy + x dx + = 0 is an exact differential equation.

A
x
M N
NOTE : (i) The necessary condition for (1) to be exact is = .
y x

UH
(ii) For finding the solution of exact differential equation, following results on exact differentials
should be remembered:
xdy − ydx y
(a) xdy + y dx = d(xy) (b) = d 
x 2
x
JA
xdy − ydx  y
(c) 2(x dx + y dy) = d (x 2 + y2) (d) = d  ln 
xy  x
xdy − ydx  y xdy + ydx
(e) = d  tan−1  (f) = d(ln xy)
2
x +y 2
 x xy
xdy + ydx  1 
(g) = d− 
LP

2 2
x y  xy 

Example # 15 : Solve : y2x dx + ydx – xdy = 0


y 2 xdx + ydx – xdy x
Solution : = 0  xdx + d   = 0
A

2
y y
x2 x
on integrating + +c=0
2 y
NK

Example # 16 : Solve : (x – y)dy + (x + y)dx = dx + dy


Solution : The given equation can be written as
(xdy + ydx) – ydy + xdx = dx + dy  d(x.y) – ydy + xdx = dx + dy
2 2
y x
Also integrating each term we get xy – + =x+y+C
SA

2 2
Self Practice Problems :
(17) Solve : xdy + ydx + xy ey dy = 0
(18) Solve : ye–x/y dx – (xe–x/y + y3) dy = 0
Ans. (17) n (xy ) + ey = c (18) 2e–x/y + y2 = c

Linear Differential Equation :


A linear differential equation has the following characteristics :
(i) Dependent variable and its derivative in first degree only and are not multiplied
together
(ii) All the derivatives should be in a polynomial form
(iii) The order may be more than one
The mth order linear differential equation is of the form.
dm y dm −1y dy
P0(x) + P1(x) + .................... + Pm–1 (x) + Pm (x) y = (x),
dxm dxm −1 dx
whereP0(x), P1(x)...............Pm(x) are called the coefficients of the differential equation.
dy
NOTE : + y2 sinx = lnx is not a Linear differential equation.
dx

6.1 Linear differential equations of first order :


dy
The differential equation + Py = Q , is linear in y.
dx
(Where P and Q are functions of x only).

Integrating Factor (I.F.) : It is an expression which when multiplied to a differential equation converts it
into an exact form.

RI
I.F for linear differential equation = e Pdx (constant of integration will not be considered)
 after multiplying above equation by .F it becomes;
dy  Pdx
e + Py . e Pdx = Q. e Pdx

A
dx


d
 (y.e Pdx ) = Q.e Pdx  y.e Pdx = Q.e  Pdx + C .
dx

UH
NOTE : Some times differential equation becomes linear, if x is taken as the dependent variable, and y
dx
as independent variable. The differential equation has then the following form : + P1 x = Q1.
dy

where P1 and Q1 are functions of y. The .F. now is e 


P1 dy
JA
dy
Example # 17 : Solve the differential equation f(x) = f 2(x) + f(x) y + f(x) y
dx
dy
Solution : f(x) = f 2(x) + f(x) y + f(x) y
dx
dy  f (x) 
LP

Given DE can be written as – 1 +  y = f(x)


dx  f(x) 
Which is L.D.E.
e– x
I.F. = e–x -n f(x) =
f(x)
A

e– x e– x
General solution y
f(x)
=  f(x)
f(x)
dx + c = – e–x + c  y = – f(x) + cex f(x)
NK

dy
Example # 18 : Solve : x nx + y = 2 n x
dx
dy 1 2 1 2
Solution : + y=  P= ,Q=
dx x nx x x nx x
SA

1
IF = e 
P.dx
=e
 x nx dx = e n( nx) = n x
2
 General solution is y. (n x) =  x . nx.dx + c  y (n x) = (n x)2 + c

Example # 19 : Solve the differential equation


t (1 + t2) dx = (x + xt2 – t2) dt and it given that x = – /4 at t = 1
Solution : t (1 + t2) dx = [x (1 + t2) – t2] dt
dx x t
= −
dt t (1 + t 2 )
dx x t dx
− =− which is linear in
dt t 1 + t2 dt
1
1
Here, P = – , Q=–
t
IF = e
−  t dt = e–nt = 1
t 1 + t2 t
 General solution is -
1 t 

1 x
x: = . − dt + c  = – tan–1 t + c
t t  1 + t 2  t
putting x = – /4, t = 1
– /4 = – /4 + c  c=0
 x = – t tan–1 t

6.2 Equations reducible to linear form


6.2.1 By change of variable.
Often differential equation can be reduced to linear form by appropriate substitution of the

RI
non-linear term
dy
Example # 20 : Solve : y sinx = cos x (sinx – y2)
dx

A
Solution : The given differential equation can be reduced to linear form by change of variable by a
suitable subtitution.
Substituting y2 = z

UH
dy dz
2y =
dx dx
differential equation becomes
sin x dz
+ cos x.z = sin x cos x
2 dx
JA
dz dz
+ 2 cot x . z = 2 cos x which is linear in
dx dx

IF = e 
2 cot x dx
= e2 n sin x = sin2 x
 General solution is -
2
 2cos x.sin x.dx + c  y2 sin2x =
2
z. sin2 x = sin3 x + c
3
LP

6.2.2 Bernoulli’s equation :


dy
Equations of the form + Py = Q.yn, n  0 and n  1
dx
where P and Q are functions of x, is called Bernoulli’s equation and can be made linear in v by dividing
A

by yn and putting y –n+1 = v. Now its solution can be obtained as in (v).


dy
e.g. : 2 sin x – y cos x = xy3 ex .
dx
NK

dy
Example # 21 : Solve : = x3y3 – xy (Bernoulli's equation)
dx
Solution : Dividing both sides by y 3
1 dy x
+ 2 = x3
y3 dx y
SA

2 dy dt dt
Putting t = 1/y2  – 3 dx
=  – 2x t = –2x3
y dx dx

.F. = e−  2xdx = e − x
2

General solution is
2
e– x 1
 –2x e
2
–x2 3 – x2 2
te = dx + C  2
= – e–x (–x2 – 1) + C  2
= (x2 + 1) + C e x
y y

Self Practice Problems :


dy
(19) Solve : x (x2 + 1) = y (1 – x2) + x2 nx
dx
dy
(20) Solve : (x + 2y3) =y
dx
dy
(21) Solve : x + y = y2 log x
dx

 dy 
(22) Solve the differential equation xy2   – 2y = 2x given y = 1 at x = 1
3 3

 dx 
 x2 + 1 
Ans. (19)   y = x n x – x + c (20) x = y (c + y2)
 x 
(21) y (1 + cx + log x) = 1 (22) y3 + 2x3 = 3x6

Higher Degree Equation :


dy
The differential equation y = mx + f(m) (where m= ) ............(1),

RI
dx
is known as Clairaut’s Equation.
dy dm d f(m) dm
To solve (1), differentiate it w.r.t. x, which gives =m+x +
dx dx dm dx

A
dm d f(m) dm
x + =0
dx dm dx
dm

UH
either =0m=c ...........(2) or x + f(m) = 0 ............(3)
dx
NOTE : (i)If m is eliminated between (1)and(2),thesolutionobtained is a general solution of(1)
(ii) If m is eliminated between (1) and (3), then solution obtained does not contain any arbitrary
constant and is not particular solution of (1). This solution is called singular solution of (1).
dy
JA
Example # 22 : Solve : y = mx + m – m3 where, m =
dx
Solution : y = mx + m – m3 ..... (i)
The given equation is in clairaut's form.
Now, differentiating wrt. x -
dy dm dm dm dm dm dm
=m+x + − 3m2  m=m+x + − 3m2
dx dx dx dx dx dx dx
LP

dm dm
(x + 1 – 3m2) = 0  =0 m=c ..... (ii)
dx dx
x +1
or x + 1 – 3m2 = 0  m2 = ..... (iii)
3
A

Eliminating 'm' between (i) & (ii) is called the general solution of the given equation.
y = cx + c – c3 where, 'c' is an arbitrary constant.
Again, eliminating 'm' between (i) & (iii) is called singular solution of the given equation.
y = m (x + 1 – m2)
NK

1/ 2 1/ 2
 x + 1  x + 1  x + 1 2
y = ±   x + 1− 3   y = ±  (x + 1)
 3     3  3
3/2
 x + 1 4
y = ± 2   y2 = (x + 1)3  27y2 = 4 (x + 1)3
 3  27
SA

Self Practice Problems :


dy
(23) Solve the differential equation y = mx + 2/m where, m =
dx
dy
(24) Solve : sin px cos y = cos px sin y + p where p =
dx
Ans. (23) General solution : y = cx + 2/c where c is an arbitrary constant
Singular solution : y2 = 8x
(24) General solution : y = cx – sin–1 (c) where c is an arbitrary constant.
x2 − 1
Singular solution : y = x 2 − 1 − sin−1
x2

Example # 23 : The normal to a given curve at each point (x,y) on the curve passes through the point (3,0). If
the curve contains, the point (3,4) find its equation.
Solution : Equation of normal at any point (x,y) is
dy
(Y – y) + (X – x) = 0
dx
Passes through (3,0)
dy y2 x2
 (3 – x) – y = 0  ydy = (3 – x)dx  = 3x – +C .......... (i)
dx 2 2
The curve contains the point (3,4)
9
 8 = 9 – +C  C = 7/2
2
By equation (i)
y2 x2 7
= 3x – +  x2 + y2 – 6x – 7 =0
2 2 2

RI
y y y
Example # 24 : The slope of the tangent to a curve at any point (x,y) on it given by – cot . cos ,
x x x
(x > 0, y > 0) and curve passes through the point(1,/4). Find the equation of the curve.

A
Solution : Let y = f(x) be the curve
dy y y y
given that = – cot . cos (homogeneous differential equation)
dx x x x

UH
put y = vx
dv –dx
 v+x = v – cotv. cosv  tan v. sec v dv =
dx x
Integrating both sides, we have
y
JA
 secv = – n|x| + C  sec + n|x| = C
x
 
Passes through  1,   C = 2
 4
y
The curve is  sec +n|x| = 2
x
LP

Example # 25 : Assume that a spherical rain drop evaporates at a rate proportional to its surface area. if its

radius originally is 3mm and 1 hr later has been reduced to 2mm, find an expression for radius
A

of the rain drop at any time.

Solution : Let r be radius , V be volume and S be surface area of rain drop at any time t.
NK

4 3
Then V = r and S = 4 r2
3

dV dV
given S  = kS, k is constant of proportionality
dt dt
SA

4 dr dr
 .3r2 = k4r2  =k  dr = kdt
3 dt dt

Integrating both sides we have r = kt + C

when t = 0, r = 3  C = 3

when t = 1 hr, r = 2  k = – 1

Hence r = 3 – t Ans.
Self Practice Problems :

(25) The decay rate of radium at any time t is proportional to its mass at that time. Find the time

when the mass will be halved of its initial mass.

Ans. (25) k log 2 , where k is constant of proportionality

RI
A
UH
JA
LP
A
NK
SA
 Marked questions are recommended for Revision.

PART - I : SUBJECTIVE QUESTIONS

Section (A) : Degree & Order, Differential equation formation


A-1. Find the order and degree of the following differential equations -
2 3 2 4
 d2 y   dy   d3 y  d3 y  dy 
(i)  2  +   +y =0
4
(ii)  3  + 3 +   =y
 dx   dx   dx  dx  dx 
 dy   dy  1

RI
(iii) sin–1   =x+y (iv)  dx  + y = dy
 dx   
dx
d3 y 5/2
d2 y   dy 2  d3 y
1 +   
3
(v) e dx –x +y=0 (vi) =x

A
dx 2   dx   dx 3
d2 y  dy 
(vii) = sin  x + 
2
 

UH
dx dx

A-2. Identify the order of the following equations, (where a, b, c, d are parameters)
(i) (sin a) x + (cos a) y =  (ii) y2 = 4a ex+b
   
(iii) n (ay) = bex + c (iv) y = tan  + ax  tan  − ax  + cebx+d
JA
4  4 
 
A-3. Form differential equations to the curves
(i) y2 = m (n2 – x2), where m, n are arbitrary constants.
(ii) ax2 + by2 = 1, where a & b are arbitrary constants.
(iii) xy = ae–x + bex
LP

A-4. (i) Form diffrential equation of all circles touching both positive co-ordinate axes.
(ii) Form diffrential equation of all straight lines at a distance unity from (2, 0)
(iii) Form D.E of locus of a point whose distance from origin is equal to distance from line
x + y +  = 0 where  is a variable parameter.
A

Section (B) : Variable separable, Homogeneous equation, polar substitution


 
B-1. Solve the following differential equations
NK

dy 1
(i) (1 + cosx) dy = (1 – cosx) dx (ii) – x sin2x =
dx x log x
dy x(2 nx + 1)
(iii) =
dx sin y + y cos y
SA

B-2. Solve :
dy dy
(i) = sin(x + y) + cos(x + y) (ii) + ex–y + ey–x = 1
dx dx

B-3. Solve :
x dx − y dy 1 + x2 − y2 xdx + ydy xdy − ydx
(i) = (ii) =
x dy − y dx x2 − y2 2
x +y 2 x2

B-4. Solve :
(i) x2 dy + y(x + y) dx = 0, given that y = 1, when x = 1
y y 
(ii) y cos ( xdy − ydx ) + x sin ( xdy + ydx ) = 0 , when y(1) = .
x x 2
dy y 2 − 2xy − x 2
B-5. Find the equation of the curve satisfying = and passing through (1, –1).
dx x 2 + 2xy − y 2

B-6. Identify the conic whose differential equation is (1 + y 2) dx – xydy = 0 and passing through (1, 0). Also
find its focii and eccentricity

B–7. If a curve passes through the point (1, /4) and its slope at any point (x, y) on it is given by y/x –
cos2(y/x), then find the equation of the curve.

B-8. (i) The temperature T of a cooling object drops at a rate which is proportional to the difference
T – S, where S is constant temperature of the surrounding medium.
dT
Thus, = –k (T – S), where k > 0 is a constant and t is the time. Solve the differential
dt

RI
equation if it is given that T(0) = 150.
(ii) The surface area of a spherical balloon, being inflated changes at a rate proportional to time t.
If initially its radius is 3 units and after 2 seconds it is 5 units, find the radius after t seconds.
(iii) The slope of the tangent at any point of a curve is  times the slope of the straight line joining

A
the point of contact to the origin. Formulate the differential equation representing the problem
and hence find the equation of the curve.

UH
B-9. Find the curve such that the distance between the origin and the tangent at an arbitrary point is equal to
the distance between the origin and the normal at the same point.

B-10. Find the curve such that the ordinate of any of its points is the geometric mean between the abscissa
and the sum of the abscissa and subnormal at the point.
JA
Section (C) : Linear upon linear, Linear diff. eq. & bernaullis diff. eq.

C-1. Solve :
(i) (2x – y + 1) dx + (2y – x – 1) dy = 0
dy 4x + 6y + 5
(ii) =
dx 3y + 2x + 4
LP

(iii) (2x + 3y – 5) dy + (3x + 2y – 5) dx = 0


dy 3x − 4y + 7
(iv) 4 =
dx x−y

C-2. Solve :
A

dy
(i) = y tanx – 2sinx (ii) (1 + y + x2y) dx + (x + x3)dy = 0
dx
NK

dy dy
(iii)  (x + 3y2) = y, y > 0 (iv) (1 + x2) + 2xy = cosx
dx dx

C-3 Solve :
dy dy y2 − x
SA

(i) x + y = x2y4 (ii) 2 =


dx dx xy + y
dy
(iii) = ex−y (ex − ey) (iv) y y sin x = cos x (sin x − y2)
dx
C-4. (a) Find the integrating factor of the following equations
dy
(i) (x log x) + y = 2 log x
dx
dy
(ii) = y tan x – y2 sec x, is
dx
(b) If the integrating factor of x(1 – x2) dy + (2x2 y – y – ax3) dx = 0 is e p . dx , then P is equal to
 
Section (D) : Exact differential equation, Higher degree & Higher Order differential equation
 
D-1 Solve the following differential equations
(i) xdy – ydx= x3dy + x2ydx
(ii) x2y(2xdy + 3ydx)=dy
(iii) x dy – ydx = x10y4 (3ydx + 4x dy)

D-2. Solve
(i) y(x2y + ex) dx = ex dy
dy
(ii) 2y sinx + y2 cosx + 2x = 0
dx
(iii) (1 + x x 2 + y 2 ) dx + y (–1 + x 2 + y 2 ) dy = 0

RI
D-3. Solve

2
 dy  dy  dy dy  dy 
(i)  y − x dx  dx − 1 = dx (ii) y + x. = x4  

A
   dx  dx 
 
dy d2 y

UH
D-4. Solve (here y1 = and y2 = 2 )
dx dx
d2 y dy
(i) 2
=
dx dx
d3 y d2 y 1
(ii) =8 satisfying y(0) = , y (0) = 0 and y2(0) = 1.
JA
dx 3
dx 2 8 1

PART - II : ONLY ONE OPTION CORRECT TYPE


Section (A) : Degree & Order, Differential equation formation
LP

A-1. The order and degree of the differential equation


3/2
  dy 2 
1 +   
  dx  
A

r= are respectively
d2 y
dx 2
(A) 2, 2 (B) 2, 3 (C) 2, 1 (D) none of these
NK

A-2. The order of the differential equation whose general solution is given by
y = (C1 + C2) sin (x + C3) – C4 ex +C5 is
(A) 5 (B) 4 (C) 2 (D) 3
SA

A-3. The order and degree of differential equation of all tangent lines to parabola x 2 = 4y is
(A) 1, 2 (B) 2, 2 (C) 3, 1 (D) 4, 1
2 1/ 3
 d2 y   dy 
A-4. If p and q are order and degree of differential equation y  2  +3x  
2
+x2y2 = sin x, then :
 dx   dx 
 
p 1
(A) p > q (B) = (C) p = q (D) p < q
q 2

A-5. Family y = Ax + A3 of curve represented by the differential equation of degree


(A) three (B) two (C) one (D) four

A-6. The differential equation whose solution is (x – h)2 + (y – k)2 = a2 is (a is a constant)


3 3
  dy 2  d2 y   dy 2   2 
2
2 d y
(A) 1 +    = a2 2 (B) 1 +    = a  2 
  dx   dx   dx    dx 
2
  dy    2 
3 2
  dy 2   2 
3
2 d y 2 d y
(C) 1 +   = a  2  (D) 1 +   a  
dx  
 =
 dx 2 
  dx    dx      

A-7. The differential equations of all conics whose centre lie at the origin is of order :
(A) 2 (B) 3 (C) 4 (D) none of these

A-8. The differential equation for all the straight lines which are at a unit distance from the origin is
2 2 2 2
 dy   dy   dy   dy 
(A)  y − x  = 1 –  (B)  y + x  =1+  
 dx   dx   dx   dx 

RI
2 2 2 2
 dy   dy   dy   dy 
(C)  y − x  = 1 +  (D)  y + x  = 1 – 
 dx   dx   dx   dx 

A
Section (B) : Variable separable, Homogeneous equation, polar substitution
dy
B-1. If = e–2y and y = 0 when x = 5, the value of x for y = 3 is
dx

UH
e6 + 9
(A) e5 (B) e6 + 1 (C) (D) loge 6
2
B-2. If (x) = (x) and (1) = 2, then (3) equals
(A) e2 (B) 2 e2 (C) 3 e2 (D) 2 e3
JA
dy
B-3. If = 1 + x + y + xy and y (– 1) = 0, then function y is
dx
(A) e(1− x)
2 2
/2
(B) e(1+ x) /2
−1 (C) loge (1 + x) – 1 (D) 1 + x

dy
LP

B-4. The value of lim y(x) obtained from the differential equation = y – y2, where y (0) = 2 is
x → dx
2
(A) 1 (B) –1 (C) 0 (D)
2−e
dy 1 − y2
A

B-5. The solution of + = 0 {where x, y  (–1, 1)} is


dx 1 − x2
(A) sin–1 x sin–1 y = C (B) sin–1 x = C sin–1 y (C) sin–1 x – sin–1 y = C (D) sin–1x + sin–1 y = C
NK

x2 + y2
B-6. Integral curve satisfying y = , y(1) = 2, has the slope at the point (1, 2) of the curve, equal to
x2 − y2
5 5
(A) – (B) – 1 (C) 1 (D)
3 3
SA

B-7. Solution of differential equation xdy – y dx = 0 represents :


(A) rectangular hyperbola (B) straight line passing through origin
(C) parabola whose vertex is at origin (D) circle whose centre is at origin
 
B-8. The slope of a curve at any point is the reciprocal of twice the ordinate at that point and it passes
through the point (4, 3). The equation of the curve is
(A) x2 = y + 5 (B) y2 = x – 5 (C) y2 = x + 5 (D) x2 = y + 5

B-9. Solution of differential equation x(xdx – ydy) = 4 x 2 − y 2 (xdy – ydx) is


x
4 sin−1   −1
(A) x −y =
2 2
Ae  y
(B) x 2 + y 2 = Ae4cos x

y y
4 tan−1   4 sin−1  
(C) x 2 − y 2 = Ae x (D) x 2 − y 2 = Ae x
B-10. Let normal at point P on curve intersect on x-axis at N and foot of P on x-axis is P'. If P'N is always
constant for any point P on curve, then equation of curve is
(A) y = ax + b (B) y2 = 2ax + b (C) ay2 – x2 = a (D) ay2 + x2 = a

Section (C) : Linear upon linear, Linear diff. eq. & bernaullis diff. eq.
dy 2x + 5y
C-1. Solution of D.E. = is, (if (y(0) = 0)
dx 2y − 5x + 3
(A) x2 – y2 + 5xy – 3y = 0 (B) x2 + y2 + 5xy – 3y = 0
(C) x – y + 5xy + 3y = 0
2 2
(D) x2 – y2 – 5xy – 3y = 0

dy 3x + 4y + 3
C-2. Solution of D.E. = is  
dx 12x + 16y − 4
(A) y=4x + n|3x + 4y| + C (B) 4y=x + n|3x + 4y| + C

RI
(C) y=n|3x + 4y| + C (D) x +y =n|3x +4y| + C

dv k
C-3. Solution of D.E. + v = – g is
dt m

A
k k k k
− t mg mg − m t −
m
t mg t mg
(A) v = ce m – (B) v = c – e (C) v e =c– (D) v e m =c–
k k k k

UH
dy y 4
C-4. Solution of differential equation 4y 3 + = x3 is
dx x
x x5
(A) y4.x5 = +C (B) y4 = +C
5 5
x5
JA
(C) y4.x = x5 + C (D) y4.x = +C
5
dy 1
C-5. Solution of differential equation sin y. + cos y = x4 cos2y is
dx x
(A) x secy = x6 + C (B) 6x secy = x + C (C) 6x secy = x 6 + C (D) 6x secy = 6x6 + C

Section (D) : Exact differential equation, Higher degree & Higher Order differential equation
LP

dy 2x 3 y + 3x 4 + y
D-1. Solution of differential equation = is
dx x – x4
y y
(A) x2y + x3 = +C (B) x2y + 2x3 = +C
x x
A

2y y
(C) x2y + x3 = +C (D) y + x3 = +C
x x
xy
NK

D-2. Solution of differential equation xdy = dx – ydx is


1– x 2
(A) n(x+y) = sin–1x + C (B) n(xy) = sin–1x + C (C) 2n(xy) = sin–1x + C (D) n(xy) = 2sin–1x + C

D-3. Solution of differential equation x 6dy + 3x5ydx = xdy – 2y dx is


y 2y
(A) x3y = 2 + C (B) x3y = 2 + C
SA

x x
y y
(C) x3y2 = 2 + C (D) x3 = 2 + C
x x

2
 dy  dy
D-4. Solution of   +x dx – y = 0 is
 
dx
4 2
(A) y = 3x2 + 9 (B) y = 3x + 9 (C) y = x (D) y = 9x + 3
3

D-5. The equation of the curve satisfying the differential equation y 2 (x2 + 1) = 2xy1 passing through the point
dy d2 y
(0, 1) and having slope of tangnet at x = 0 as 3, is (Here y 1 = and y2 = )
dx dx 2
(A) y = x2 + 3x + 2 (B) y2 = x2 + 3x + 1 (C) y = x3 + 3x + 1 (D) none of these
PART - III : MATCH THE COLUMN
1. Match the following

Column -  Column - 

xdy dy
(A) Solution of y – = y2 + is (p) xy2 = 2y5 + c
dx dx

dy
(B) Solution of (2x – 10y3) + y = 0 is (q) sec y = x + 1 + ce x
dx

(C) Solution of sec2 y dy + tan y dx = dx is (r) (x + 1) (1 – y) = cy

RI
dy
(D) Solution of sin y = cos y (1 – x cos y) is (s) tan y = 1 + ce–x
dx

A
2. Match the following

Column -  Column - 

UH
1
(A) xdy = y(dx + ydy), y(1) = 1 and y(x 0) = –3, then x0 = (p)
4
dy
(B) If y(t) is solution of (t + 1) – ty = 1, (q) – 15
dt
JA
y (0) = –1, then y (1) =
1
(C) (x2 + y2) dy = xydx and y(1) = 1 and (r) –
2
y(x0) = e, then x0 =
dy 2y
(D) + = 0, y (1) = 1, then y(2) = (s) 3e
dx x
LP
A
NK
SA
 Marked questions are recommended for Revision.

PART - I : ONLY ONE OPTION CORRECT TYPE

1. The differential equation of all parabola having their axis of symmetry coinciding with the x-axis is
2 2
d2 y  dy  d2 x  dx  d2 y dy
(A) y +  = 0 (B) y +  =0 (C) y + =0 (D) none of these
dx 2
 dx  dy 2
 dy  dx 2 dx

dy
2. If y1(x) and y2(x) are two solutions of + f(x) y = r(x) then y1(x) + y2(x) is solution of :
dx

RI
dy dy
(A) + f(x) y = 0 (B) + 2f(x) y = r(x)
dx dx
dy dy
(C) + f(x) y = 2 r(x) (D) + 2f (x) y = 2r(x)
dx dx

A
dy
3. If y1(x) is a solution of the differential equation + f(x) y = 0, then a solution of differential equation

UH
dx
dy
+ f(x) y = r (x) is
dx
1

r(x)
(A)
y (x)
y1(x) dx (B) y1(x)  y (x)
1
dx (C)  r(x)y (x) dx 1 (D) none of these
JA
3
4. The solution of y dx – x dy + 3x2 y2 e x dx = 0 is
x 3 x 3 x 3 y 3
(A) + ex = C (B) – ex = 0 (C) – + ex = C (D) + ex = c
y y y x

5. The solution of the differential equation (x 2 sin3 y – y2 cos x) dx + (x3 cos y sin2 y – 2y sin x) dy = 0 is
LP

(A) x3 sin3 y = 3y2 sin x + C (B) x3 sin3 y + 3y2 sin x = C


(C) x2 sin3 y + y3 sin x = C (D) 2x2 sin y + y2 sin x = C
−y
xdy x3 e x
6. Solve : − (  + 2) = y + 2x 2
dx 2
A

y y
 +1  +1
  (A) e x = − (1 + 2x) + ce2x (B) e x = − (1 + 2x) + ce3x
8 8
x y
 +1 +2
(C) e y = − (1 + 2x) + ce3x (D) . e x = − (1 + 2x) + ce2x
8
NK

xdy + ydx
7. Solution of differential equation xy(my dx + nx dy) = xm yn , given m + n = 1 , is
(A) x .y + 1 = c(x/y)
m+1 n+1
(B) x .y + 1 = cxy
m+1 n+1

(C) xm+1.yn+1– 1 = cxy (D) xm.yn + 1 = cxy .


SA

8. The equation of the curve which is such that the portion of the axis of x cut off between the origin and
tangent at any point is proportional to the ordinate of that point is
(A) x = y (b – a log y) (B) log x = by2 + a
(C) x = y (a – b log y)
2
(D) y = x (b – a log x)
(a is constant of proportionality)

9. A curve passing through the point (1, 1) has the property that the perpendicular distance of the origin
from the normal at any point P of the curve is equal to the distance of P from the x-axis. Then equation
of the curve is.
(A) x2 + y2 = 2x (B) 2x2 + y2 = 3x (C) x2 + 2y2 = 3x (D) x2 – y2 = x – 1

10. f(x) is a continuous and differentiable function defined in  [0, ). If f(0) = 1 and f'(x) > 3f(x)  x  0
then
(A) f(x)  e3x x  0 (B) f(x)  e–3x x  0 (C) f(x) > e3x x  0 (D) f(x)  e3x x  0
x x
11. Solution of the equation x  y(t)dt = (x + 1)  t y(t)dt , x > 0 is
0 0
1 1 1
c c c c −
(A) y = ex (B) y = e−1/ x (C) x = ey (D) x = e y
x3 x3 y3 y3

dy
12. The solution of diffferntial equation (1 – x2) + xy = ax is
dx
(y − a)2 + c 2 x 2 (y + a)2 + c 2 x 2
(A) =1 (B) =1
c2 c2
(y + a)2 − c 2 x 2 (y + a)2 + c 2 x 2
(C) =1 (D) = –1

RI
2
c c2

13. Find the curve which passes through the point (2, 0) such that the segment of the tangent between the
point of tangency & the y–axis has a constant length equal to 2.
 2 − 4 − x2   2 − 4 − x2 

A
(A) y =   4 − x 2 − 2 n  (B) y =   4 − x 2 + 2 n 
 x   x 
   
 2   2 

UH
2+ 4−x 2+ 4−x
(C) y =   4 − x 2 + 2 n  (D) y =   4 − x 2 − 2 n 
 x   x 
   

14. A & B are two separate reservoirs of water. Capacity of reservoir A is double the capacity of reservoir
B. Both the reservoirs are filled completely with water, their inlet are closed and then the water is
JA
released simultaneously from both the reservoirs. The rate of flow of water out of each reservoir at any
instant of time is proportional to the quantity of water in the reservior at that time. One hour after the
water is released, the quantity of water in reservoir A is 1.5 times the quantity of water in reservior B.
After how many hours do both the reservoirs have the same quantity of water?
(A) 2log4/3 2 (B) – log2/3 2 (C) log3 2 (D) log4/3 2

15. A tank contiains 20 kg of salt dissolved in 5000 L of water. Brine that contains .03 kg of salt per litre of
LP

water enters the tank at a rate of 25 L/min. The solution is kept thoroughly mixed and drains from the
tank at the same rate. How much salt remains in the tank after half an hour ?
(A) 150 – 130 e–50/200 (B) 130 – 150 e–30/200 (C) 130 – 150 e–50/200 (D) 150 – 130 e–30/200

________________________________________________________________________________________  
A

PART - II : SINGLE AND DOUBLE VALUE INTEGER TYPE


1. If differential equations of the curves c(y + c) 2 = x3, where ‘c’ is any arbitrary constant is 12y(y’) 2 + ax =
NK

bx(y')3 then (a + b) is equal to

2. The order of the differential equation of the family of ellipse having fixed centre and given eccentricity ,
is :

1
 11 − 3e 
SA

3. If y(x) satisfies the equation y'(x) = y(x) +  y dx & y(0) = 1 then value of y 
0
n
2 

dy x 2 − y 2
4. Let c1 and c2 be two integral curves of the differential equation = . A line passing through
dx x 2 + y 2
origin meets c1 at P(x1, y1) and c2 at Q(x2, y2). If c1 : y = f(x) and c2 : y = g(x) then find the value of
f ' ( x1 )
g' ( x 2 )

dy 1
5. If solution of the differential equation = is x = cesin y – k (1 + sin y), then k =
dx x cos y + sin2y

6. If y(x) satisfies the differential equation ; cos 2x (dy/dx) – (tan 2x) y = cos4x, | x | , and y(0) = 0 then
4

64y  
 6  is equal to
3 3

7. Let y1 and y2 are two different solutions of the equation y + P(x) . y = Q(x).
Such that the linear combination y1 + y2 is also solution of given differential equation . Then value of
 +  is

8. Let the curve y = f(x) passes through (4, –2) satisfy the differential equation,
sin2 x cos2 x

 
−1
y (x + y3) dx = x(y3 – x) dy & let y = g (x) = sin tdt + cos−1 tdt , 0  x  , If the area of the

RI
2
1/ 8 1/ 8
4
1  3 
where a  N then a is equal to
8  a 
region bounded by curves y = f(x), y = g(x) and x = 0 is

A
9. If the equation of curve passing through (3, 4) and satisfying the differential equation
2
 dy  dy
y  + (x – y) – x = 0 is Ax + By + 2 = 0 then value of A – B is

UH
 dx  dx

10. The perpendicular from the origin to the tangent at any point on a curve is equal to the abscissa of the
point of contact. If equation of tangent to the curve at (1, 3) is ax + by + 5 = 0 then value of a 2 + b2 is
equal to
 
JA
11. A curve passing through point (1, 2) possessing the following property; the segment of the tangent
between the point of tangency & the x−axis is bisected at the point of intersection with the y−axis. If A is
area bounded by the curve & line x = 1 then 9A 2 is equal to

12. Two cylindrical tanks in which initially one is filled with water to the height of 1 m and other is empty, are
connected by a pipe at the bottom. Water is allowed to flow from filled tank to the empty tank through
LP

the pipe. The rate of flow of water through the pipe at any time is a 2g (h1 − h2 ) , where ‘h 1’ and ‘h2’ are
the heights of water level (above pipe) in the tanks at that time and ‘g’ is acceleration due to gravity. If
the cross sectional area of the filled and empty tanks be A and A/2 and that of the pipe be ‘a’, and if
1A 2
 a g is the time when the level of water in both tanks will be same (neglect the volume of the water in
A

pipe), then  is :
dn f ( x )  1
13. If f (x) = e−1/x, x > 0. Let for each positive integer n, P n be the polynomial such that = Pn  e−1 / x for
dx n x
NK

 d 
all x > 0 and if Pn+1 (x) = x2 .Pn (x) − . dx Pn (x) ,
 
then  +  is :
14. If y = f (x) be a curve passing through (e, e e) and which satisfy the differential equation
e
(2ny + xy logx)dx − x log x dy = 0, value of  g(x) dx where g (x) =
lim
n → f (x), is :
1/ e
SA

PART - III : ONE OR MORE THAN ONE OPTIONS CORRECT TYPE


 dy d2 y 
1. The differential equation of all circles in a plane must be  y1 = ,y 2 = 2 ,.........etc. 
 dx 
 dx 
( )
(A) y 3 1 + y12 − 3y1y 22 = 0 (B) of order 3 and degree 1

(C) of order 3 and degree 2 ( )


(D) y 32 1 − y12 − 3y1y 22 = 0

2. Correct statement is/are


(A) The differential equation of all conics whose axes coincide with the axes of co-ordinates
is of order 2.
(B) The differential equation of all staright lines which are at a fixed distance p from origin is
of degree 2.
(C) The differential equation of all parabola each of which has a latus rectum 4a &
whose axes are parallel to y-axis is of order 2.
(D) The differential equation of all parabolas of given vertex, is of order 3.
 
dy 1 + y2
3. Solution of the differential equation + = 0 is
dx 1 − x2
(A) tan–1 y + sin–1 x = c (B) tan–1 x + sin–1 y = c
1
(C) tan–1 y . sin–1 x = c (D) cot–1 + cos–1 1 − x 2 = c
y

4. The solution of (x + y + 1) dy = dx are


(A) x + y + 2 = Cey (B) x + y + 4= = C log y
(C) log (x + y + 2) = Cy (D) log (x + y + 2) = C + y

RI
dx
5. The solution of + y = ye(n – 1)x , (n  1)
dy

A
1  e(n−1) x − 1  y2
y2
( n −1) 2
(A) n   = +C (B) e (1 – n)x
= 1 + ce
n −1  e(n−1) x  2
 

UH
y2 (n−1) y2
( n −1) 2 (n−1) +
(C) n (1 + ce ) + nx + 1 = 0 (D) e (n –1)x
= ce 2 +1

6. Correct statement is/are


x
y3 y
JA
(A) f(x, y) = x 2 e y + + y 2 n   is a homogenous function of degree two.
x x
sin y + x
(B) f(x, y) = is homogenous function of degree one.
sin2y + x cos y
y  y 
(C) xsin   dy +  y sin x − x  dx = 0 is a homogenous differential equation.
x  
LP

y
y
(D) f(x, y) = e x + tan is homogenous function of degree zero.
x

dy
A

7. Solution of differential equation f(x) = f2 (x) + f(x) y + f(x) y is


dx
(A) y = f(x) + cex (B) y = – f(x) + cex (C) y = – f(x) + cex f(x) (D) y = cf(x) + ex
NK

8. The solution of x2 y12 + xy y1 – 6y2 = 0 are (here y1 =dy/dx)


1
(A) y = Cx2 (B) x2 y = C (C) ny=C+ log x (D) x3 y = C
2
2
 dy  dy x
9. The solution of differential equation   − dx (e + e ) + 1 = 0 is
–x
SA

 
dx
(A) y = ex + c (B) y = –e–x + c
(C) y = 2e + 3e + c
x –x
(D) yex + 1 = cex

dn y
10. If y = e–x cos x and yn + kn y = 0, where yn = and kn, n  N are constants.
dxn
(A) k4 = 4 (B) k8 = –16 (C) k12 = 20 (D) k16 = –24

dn y
11. A solution of the differential equation y 1 y3 = 3y22 can be (where yn = )
dxn
(A) x = A1y2 + A2 y + A3 (B) x = A1 y + A2
(C) x = A1 y2 + A2 y (D) y = A1 x2 + A2x + A3
x
12. A differentiable function satisfies equation f(x) =  ( f ( t ) cos t − cos ( t − x )) dt
0
then


(A) f"   = e (B) lim f(x) = 1
2 x →−

(C) f(x) has minimum value 1 – e–1 (D) f'(0) = –1

x
13. Let f(x) is a continuous function which takes positive values for x  0 and satisfy  f ( t ) dt = x
0
f ( x ) with

1
f(1) = then
2
1

RI
(A) f(x) = 2
1 + 1 − 2 x 
( )
 
  1
(B) f  cot  =

A
 8 4
1
(C) Area bounded by f(x) and x-axis between x = 0 to x = 2 + 1 is square units.
2 ( 2 −1 )

UH
 
(D) f  sin  = 2
 4 
x
14. 
Let f(x), x  0 be a non negative continuous function & let F(x) = f ( t ) dt, x  0. If for some c > 0,
JA
0
f(x)  c F(x) for all x  0 then
(A) f(x) = 0  x  0
(B) f(0) = 0
(C) e–cx F(x) is a non-increasing function on [0, )
(D) F(x)  0  x  0
15. A curve passing through (1 , 0) such that the ratio of the square of the intercept cut by any tangent off
LP

the y−axis to the subnormal is equal to the ratio of the product of the co−ordinates of the point of
tangency to the product of square of the slope of the tangent and the subtangent at the same point.
Determine all such possible curves.
2 y/x 2 x/y − 2 y/x − 2 x/y
(A) x = e (B) y = e (C) x = e (D) x = e
A

d2 y
16. The differential equation + y + cot2 x = 0 must be satisfied by
dx 2
NK

(A) y = 2 + c1 cos x + c 2 sin x


 x
(B) y = cos x . n  tan  + 2
 2
 x
(C) y = 2 + c1 cos x + c2 sin x + cos x log  tan 
SA

 2
(D) all the above
 
 
PART - IV : COMPREHENSION
Comprehension # 1

Differential equations are solved by reducing them to the exact differential of an expression in x & y
i.e., they are reduced to the form d(f(x, y)) = 0
e.g. :

xdx + ydy ydx − xdy


=
x2 + y2 x2
1 2xdx + 2ydy xdy − ydx y

2 x2 + y2
=–
x2
 d ( x2 + y2 ) = – d 
x
 y
 d  x2 + y2 +  = 0
 x
y
 solution is x 2 + y 2 +
= c.
x
Use the above method to answer the following question (3 to 5)

1. The general solution of (2x3 – xy2) dx + (2y3 – x2y) dy = 0 is


(A) x4 + x2y2 – y4 = c (B) x4 – x2y2 + y4 = c
(C) x – x y – y = c
4 2 2 4
(D) x4 + x2y2 + y4 = c

RI
xdy  y 
2. General solution of the differential equation + 1 − 2  dx = 0 is
2 2  x + y 2 
x +y 
y x y
(A) x + tan–1   = c (B) x + tan–1 =c (C) x – tan–1   = c (D) none of these

A
 
x y x

3. General solution of the differential equation e y dx + (xey – 2y) dy = 0 is

UH
(A) xey – y2 = c (B) yex – x2 = c (C) yey + x = c (D) xey – 1 = cy2

Comprehension # 2
dn y dn−1y
In order to solve the differential equation of the form a 0 + a1 +..... + any = 0, where
dxn dxn−1
JA
a0, a1, a2 are constants.
We take the auxiliary equation a0Dn + a1 Dn–1 + ....+ an = 0
Find the roots of this equation and then solution of the given differential equation will be as given in the
following table.

Roots of the auxiliary equation Corresponding complementary


LP

function
1. One real root 1 c1e1x
2. Two real and different roots 1 and 2 c1e1x + c 2e2 x
3. Two real and equal roots 1 and 1 (c1 + c2x) e1x
A

4. Three real and equal roots 1, 1, 1 (c1 + c2x + c3x2) e1x
5. One pair of imaginary roots  ± i (c1 cos x + c2 sin x) ex
NK

6. Two pair of equal imaginary roots  ± i and  ± i [(c 1 + c2x) cos x + (c1 + c2x) sin x] ex
Solution of the given differential equation will be y = sum of all the corresponding parts of the
complementary functions.

d2 y dy
4. Solve –2+ y = 0.
SA

2 dx
dx
(A) y = (c1 + c2x)ex (B) y = (c1ex + c2ex) (C) y = (c1x)ex (D) none of these

d2 y
5. Solve + a2y = 0.
dx 2
(A) y = (c1 cos ax + c2 sin ax)eax (B) y = c1 cos ax + c2 sin ax
(C) y = c1 eax + c2 e–ax (D) none of these

d3 y d2 y dy
6. Solve 3
–6 2
+ 11 – 6y = 0
dx dx dx
(A) y = (c1 + c2 x + c3 x2) ex (B) y = x (c1 ex + c2 e2x + c3 e3x)
(C) y = c1 ex + c2 e2x + c3 e3x (D) none of these
 
Comprehension # 3 (Q.No. 7 to 9)

Let f(x) be a differentiable function, satisfying f (0) = 2, f (0) = 3 and f (x) = f(x)
7. Graph of y = f(x) cuts x -axis at
1 1
(A) x = – n5 (B) x = n5 (C) x = – n5 (D) x = n5
2 2
8. Area enclosed by y = f(x) in the second quadrant is
1 1
(A) 3 + n 5 (B) 2 + n5 (C) 3 – 5 (D) 3
2 2
1
9. Area enclosed by y = f(x), y = f–1 (x), x + y = 2 and x + y = – n5 is
2
1 2 1 2 1 1
(A) 8 + ( n5) (B) 8 – 2 5 + ( n5) (C) 2 5 – ( n5)2 (D) 8 + 2 5 – ( n5)2
8 8 8 8

RI
A
UH
JA
LP
A
NK
SA
 Marked questions are recommended for Revision.
* Marked Questions may have more than one correct option.

PART - I : JEE (ADVANCED) / IIT-JEE PROBLEMS (PREVIOUS YEARS)


x x

 1 − (f (t)) dt =  f(t) dt, 0  x  1


2
1. Let f be a non-negative function defined on the interval [0, 1]. If
0 0
and f(0) = 0, then [IIT-JEE-2009, Paper-1, (3, –1), 80]
 1 1  1 1  1 1  1 1
(A) f   < and f   > (B) f   > and a f   >

RI
2 2 3 3 2 2 3 3
 1 1  1 1  1 1  1 1
(C) f   < and f   < (D) f   > and f   <
2 2 3 3 2 2 3 3

A
2. Match the statements/expressions in Column - I with the open intervals in Column - II
[IIT-JEE-2009, Paper-1, (8, 0), 80]
Column - I Column - II

UH
  
(A) Interval contained in the domain of definition of (p) – 2, 2 
 
non-zero solutions of the differential
equation (x – 3)2 y + y = 0
 
JA
(B) Interval containing the value of the integral (q)  0, 2 
 
5

 (x – 1)(x – 2)(x – 3)(x – 4)(x – 5)dx


1

  5 
(C) Interval in which at least one of the points of local (r) 8, 4 
LP

 
maximum of cos2x + sinx lies
 
(D) Interval in which tan–1 (sinx + cosx) is increasing (s)  0, 8 
 
(– , )
A

(t)

3. Match the statements/expressions given in Column - I with the values given in Column - II
[IIT-JEE-2009, Paper-2, (8, 0), 80]
NK

Column - I Column – II

(A) The number of solutions of the equation xe sinx – cos x = 0 (p) 1


 
in the interval  0, 
 2
SA

(B) Value(s) of k for which the planes kx + 4y + z = 0, (q) 2


4x + ky + 2z = 0 and 2x + 2y + z=0 intersect in a straight line

(C) Value(s) of k for which |x – 1| + |x – 2| + |x + 1| + |x + 2| = 4k (r) 3


has integer solution(s)

(D) If y = y + 1 and y(0) = 1, then value(s) of y (n 2) (s) 4


(t) 5
4. Let f be a real-valued differentiable function on R (the set of all real numbers) such that
f(1) = 1. If the y-intercept of the tangent at any point P(x, y) on the curve y = f(x) is equal to the cube of
the abscissa of P, then the value of f(–3) is equal to [IIT-JEE 2010, Paper-1, (3, 0), 84]
x
5. 
Let f : [1, ) → [2, ) be a differentiable function such that f(1) = 2. If 6 f(t)dt = 3xf(x) − x 3
1
for all

x  1, then the value of f(2) is [IIT-JEE 2011, Paper-1, (4, 0), 80]
df(x)
6. Let y(x) + y(x) g(x) = g(x) g(x), y(0) = 0, x  R , where f(x) denotes and g(x) is a given
dx
non-constant differentiable function on R with g(0) = g(2) = 0. Then the value of y(2) is
[IIT-JEE 2011, Paper-2, (4, 0), 80]

7.* If y(x) satisfies the differential equation  y – y tan x = 2x sec x and y(0) = 0, then
[IIT-JEE 2012, Paper-1, (4, 0), 70]
 2    2
  (A) y   = (B) y   =
4 8 2  4  18
   2    4 22
(C) y   = (D) y   = +
3 9 3 3 3 3

RI
  y y
8. A curve passes through the point  1,  . Let the slope of the curve at each point (x, y) be + sec   ,
 6  x x

A
x > 0. Then the equation of the curve is [JEE (Advanced) 2013, Paper-1, (2, 0)/60]
y 1 y
(A) sin   = logx + (B) cosec   = logx + 2
x 2 x

UH
 2y   2y  1
(C) sec   = log x + 2 (D) cos   = log x +
 x   x  2

dy xy x 4 + 2x
9. The function y = f(x) is the solution of the differential equation + 2 = in (–1, 1) satisfying
JA
dx x − 1 1 − x2
3
2
f(0) = 0. Then  f(x)dx is [JEE (Advanced) 2014, Paper-2, (3, –1)/60]
3

2
LP

 3  3  3  3
(A) − (B) − (C) − (D) −
3 2 3 4 6 4 6 2

10. Let f : [0, 2] → R be a function which is continuous on [0, 2] and is differentiable on (0, 2) with f(0) = 1.
x2
A

Let F(x) =  f(
0
t )dt for x  [0, 2]. If F'(x) = f'(x) for all x  (0, 2), then F(2) equals

[JEE (Advanced) 2014, Paper-2, (3, –1)/60]


NK

(A) e2 – 1 (B) e4 – 1 (C) e – 1 (D) e4

11*. Let y(x) be a solution of the differential equation (1 + e x)y' + yex = 1. If y(0) = 2, then which of the
following statements is (are) true ? [JEE (Advanced) 2015, P-1 (4, –2)/ 88]
(A) y(–4) = 0
(B) y(–2) = 0
SA

(C) y(x) has a critical point in the interval (–1, 0)


(D) y(x) has no critical point in the interval (–1, 0)
12*. Consider the family of all circles whose centers lie on the straight line y = x. If this family of circles is
represented by the differential equation Py" + Qy' + 1 = 0, where P, Q are functions of x, y and y' (here
y' = , y" = ), then which of the following statements is (are) true?
[JEE (Advanced) 2015, P-1 (4, –2)/ 88]
(A) P = y + x (B) P = y – x
(C) P + Q = 1 – x + y + y' + (y') 2 (D) P – Q = x + y – y' – (y')2
f (x)
13. Let f : (0, ) → R be a differentiable function such that f(x) = 2 – for all x  (0, ) and f(1)  1.
x
Then [JEE (Advanced) 2016, Paper-1, (3, –1)/62]
 1  1
(A) Lim f '   = 1
x →0+  x 
(B) Lim x f   = 2
x →0+ x
(C) Lim x 2 f ' ( x ) = 0 (D) f ( x )  2 for all x 
x →0+

(0, 2)
dy
14. A solution curve of the differential equation (x 2 + xy + 4x + 2y + 4) dx –y2 = 0, x > 0, passes through the
point
(1, 3). Then the solution curve [JEE (Advanced) 2016, Paper-1, (4, –2)/62]
(A) intersects y = x + 2 exactly at one point
(B) intersects y = x + 2 exactly at two points
(C) intersects y = (x + 2)2
(D) does NOT intersect y = (x + 3) 2
 
–1
 
15. If y = y(x) satisfies the differential equation 8 x  9 + x dy =  4 + 9 + x  dx, x > 0 and
   
y (0) = 7 , then y(256) = [JEE(Advanced) 2017, Paper-2,(3, –1)/61]

RI
(A) 16 (B) 3 (C) 9 (D) 80

16*. Let f : R → R and g : R → R be two non-constant differentiable functions. If f(x) = (e (f(x) – g(x)) g(x) for all
x  R, and f(1) = g(2) = 1, then which of the following statement(s) is (are) TRUE?
[JEE(Advanced) 2018, Paper-1,(4, –2)/60]

A
(A) f(2) < 1 – loge2 (B) f(2) > 1 – loge2
(C) g(1) > 1 – loge2 (D) g(1) < 1 – loge2

UH
x
17*. Let f : [0, ) → R be a continuous function such that f(x) = 1 – 2x + e
0
x−t
f(t) dt

for all x  [0, ). Then, which of the following statement(s) is (are)) TRUE?
(A) The curve y = f(x) passes through the point (1, 2)
JA
(B) The curve y = f(x) passes through the point (2, –1) [JEE(Advanced) 2018, Paper-1,(4, –2)/60]
−2
(C) The area of the region {(x, y)  [0, 1] × R : f(x)  y  1 − x2 is
4
 −1
(D) The area of the region {(x, y)  [0, 1] × R : f(x)  y  1 − x2 is
4

f(x)sin t – f(t)sin x
LP

18*. Let f : (0, ) → R be a twice differentiable function such that lim = sin2x for all
t→x t–x
 
x  (0, ) . If f   = – , then which of the following statement(s) is (are) TRUE ?
 
6 12
[JEE(Advanced) 2018, Paper-2,(4, –2)/60]
A

  x4
(A) f   = (B) f(x) < – x2 for all x  (0, )
4 4 2 6
 
NK

(C) There exists   (0, ) such that f () = 0 (D) f    + f   = 0


2 2

19. Let f : R → R be a differentiable function with f(0) = 0. If y = f(x) satisfies the differential equation
dy
= (2 + 5y)(5y – 2),
SA

dx
then the value of lim f(x) is _______ . [JEE(Advanced) 2018, Paper-2,(3, 0)/60]
x→– 

20. Let f : R → R be a differentiable function with f(0) = 1 and satisfying the equation
f(x + y) = f(x) f (y) + f (x)f(y) for all x, y  R.
Then, the value of loge(f(4)) is ______ . [JEE(Advanced) 2018, Paper-2,(3, 0)/60]

 
PART - II : JEE (MAIN) / AIEEE PROBLEMS (PREVIOUS YEARS)

1. The differential equation which represents the family of curves y = c 1, ec2 x where c1 and c2 are arbitary
constants is [AIEEE 2009 (4, –1), 144]
(1) y = y 2 (2) y  = y y (3) y.y  = y (4) y.y  = (y)2


2. Solution of the differential equation cosx dy = y(sinx – y) dx, 0 < x < is [AIEEE 2010 (4, –1),
2
144]
(1) y sec x = tan x + c (2) y tan x = sec x + c (3) tanx = (sec x + c)y (4) secx = (tanx + c) y
 
3. Let  be the purchase value of an equipment and V(t) be the value after it has been used for t years.

RI
dV(t)
The value V(t) depreciates at a rate given by differential equation = – k(T – t), where k > 0 is a
dt
constant and T is the total life in years of the equipment. Then the scrap value V(T) of the equipment is
[AIEEE 2011, I, (4, –1), 120]

A
1 kT 2
k(T − t)2
(1) T2 – (2)  – (3)  – (4) e–kT
k 2 2

UH
dy
4. If = y + 3 > 0 and y(0) = 2, then y(n2) is equal to : [AIEEE 2011, I, (4, –1), 120]
dx
(1) 7 (2) 5 (3) 13 (4) –2

5. The curve that passes through the point (2, 3), and has the property that the segment of any tangent to
it lying between the coordinate axes is bisected by the point of contact is given by :
JA
[AIEEE 2011, II, (4, –1), 120]
2 2
6 x y
(1) 2y – 3x = 0 (2) y = (3) x2 + y2 = 13 (4)   +   =2
x 2 3

 1
6. Consider the differential equation y 2dx +  x −  dy = 0 . If y (1) = 1, then x is given by :
LP

 y
[AIEEE 2011, II, (4, –1), 120]
1 1 1 1
2 ey 1 ey 1 ey 1 ey
(1) 4 – – (2) 3 – + (3) 1 + – (4) 1– +
y e y e y e y e
A

7. The population p(t) at time t of a certain mouse species satisfies the differential equation
dp(t)
NK

= 0.5 p(t) – 450. If p(0) = 850, then the time at which the population becomes zero is :
dt
[AIEEE-2012, (4, –1)/120]
1
(1) 2 n 18 (2) n 9 (3) n 18 (4) n 18
2
SA

8. At present, a firm is manufacturing 2000 items. It is estimated that the rate of change of production P
dP
w.r.t. additional number of workers x is given by = 100 – 12 x . If the firm employs 25 more
dx
workers, then the new level of production of items is [AIEEE - 2013, (4, –1),360]
(1) 2500 (2) 3000 (3) 3500 (4) 4500
 
dp(t) 1
9. Let the population of rabbits surviving at a time t be governed by the differential equation =  p(t)
dt 2
– 200 . If p(0) = 100, then p(t) equals : [JEE(Main)2014,(4, – 1), 120]
(1) 600 – 500 et/2 (2) 400 – 300 e–t/2 (3) 400 – 300 et/2 (4) 300 – 200 e– t/2
 
dy
10. Let y(x) be the solution of the differential equal (x log x) + y = 2x log x, (x  1). Then y(e) is equal
dx
to [JEE(Main)2015,(4, – 1), 120]
(1) e (2) 0 (3) 2 (4) 2e
 
11. If a curve y = f(x) passes through the point (1, –1) and satisfies the differential equation,
 1
y(1 + xy) dx = xdy, then f–  is equal to [JEE(Main)2016,(4, – 1), 120]
 2
4 2 4 2
(1) – (2) (3) (4) –
5 5 5 5
 
dy 
12. If (2 + sin x) + (y + 1) cos x = 0 and y(0) = 1, then y   is equal to : [JEE(Main)2017,(4, – 1), 120]
dx 2
1 2 1 4
(1) (2) – (3) – (4)
3 3 3 3

13. Let y = y(x) be the solution of the differential equation

RI
dy  
sinx + y cosx = 4x, x  (0,). If y   = 0, then y   is equal to [JEE(Main)2018,(4, – 1), 120]
dx  
2 6
8 4 4 2 –8
(1) – 2 (2) – 2 (3)  (4) 2

A
9 9 9 3 9 3

14. Let f : [0, 1] → R be such that f(xy) = f(x).f(y), for all x, y  [0, 1], and f(0)  0. if y = y(x) satisfies the
 1 3

UH
dy
differential equation, = f(x) with y(0) = 1, then y   + y  is equal to :
dx  
4 4
[JEE(Main) 2019, Online (09-01-19),P-2 (4, – 1), 120]
(1) 5 (2) 2 (3) 3 (4) 4

The curve amongst the family of curves represented by the differential equation, (x 2 – y2)dx + 2xy dy= 0
15.
JA
which passes through (1, 1), is : [JEE(Main) 2019, Online (10-01-19),P-2 (4, – 1), 120]
(1) a circle with centre on the x-axis.
(2) a hyperbola with transverse axis along the x-axis
(3) an ellipse with major axis along the y-axis.
(4) a circle with centre on the y-axis
LP

dy  2x + 1  1
16. If y(x) is the solution of the differential equation +   y = e–2x, x > 0, where y(1) = e–2, then
dx  x  2
[JEE(Main) 2019, Online (11-01-19),P-1 (4, – 1), 120]
loge 2
(1) y(loge 2) = loge4 (2) y(loge 2) =
4
A

1 
(3) y(x) is decreasing in  ,1 (4) y(x) is decreasing in (0, 1)
2 
NK
SA
EXERCISE - 1
PART - I
Section (A) :
A-1. (i) (2, 2) (ii) (3, 2) (iii) 1, 1
(iv) 1, 2 (v) 3, degree is not applicable
(vi) 3, 2 (vii) 2, degree is not applicable
A-2. (i) 1 (ii) 1 (iii) 2 (iv) 2

RI
2
d2 y  dy  dy d2 y dy
A-3. (i) xyy2 + (xy1 – y) y1 = 0 (ii) xy + x  − y =0 (iii) x +2 = xy
dx 2
 
dx dx dx 2 dx
2
x + yy ' ( x + yy ' ) = 0

A
A-4. (i) x2 + y2 – 2 (x+y) + (ii) (1 + y’2) = (y – (x –2) y’)2
1+ y ' (1 + y ' )2
(iii) (1 + y’) x2 + y2 = ± 2(x + yy')

UH
Section (B) :
x2 1 1
B-1. (i) y = 2 tan x/2 – x + c (ii) y= − x sin2x – cos2x + log |logx| + c
4 4 8
(iii) y siny = x2nx + c
JA
x+y
B-2. (i) log tan   +1 = x + c (ii) tan–1 (ey–x) + x = c
 2 

c(x + y) y
B-3. (i) x2 − y2 + 1 + x2 − y2 = (ii) x2 + y2 = +c
x −y
2 2 x
LP


B-4. (i) 3x2y = 2x + y (ii) xy sin (y/x) =
2

B-5. x+y=0 B-6. Conic: x2 – y2 = 1 (hyperbola) focii : (± 2 , 0), e = 2


A

B–7. tan y/x = 1 – log x.


NK

T−S
B-8. (i) = e–kt (ii) r = 4t 2 + 9 units (iii) y = kx where, k is some constant
150 − S
y
 tan−1 x4 + c
B-9. x 2 + y 2 = ce x B-10. y2 = or y2 + 2x2nx = cx2
2x 2
SA

Section (C) :
3
C-1. (i) x2 + y2 – xy + x – y = c (ii) y – 2x + n (24y + 16x + 23) = c
8
(iii) 4xy + 3 (x2 + y2) – 10 (x + y) = c (iv) 8xy – 4y2 = 3 x2 +14x + C

C-2. (i) y = cosx + k secx (ii) yx = – tan–1x + c


x
(iii) = 3y + C (iv) (1 + x2)y = sin x + C
y
1
C-3 (i) = 3x2 + kx3 (ii) y2 + (1 + x) n(1 + x) + 1 + c(1 + x)
y3
x 2
(iii) e y = ce– e + ex – 1 (iv) y2 sin2 x = sin3 x + c
3
(2x 2 − 1)
C-4. (a) (i) ±n x (ii) ±sec x (b)
x(1 − x 2 )
 
Section (D) :
4
1 y  x 6 .y8
+C
4  x 
D-1 (i) (y/x) = xy + C (ii) x3y2 = y + c (iii) =
2

1 x x3 y2 1
D-2. (i) e =– +c (ii) y2 sinx = –x2 + c (iii) x– + (x2 + y2)3/2 + c = 0
y 3 2 3

RI
c
D-3. (i) General solution : y = cx +
c −1
Singular solution : y = ( x ± 1)2
(ii) General solution : xy + c = c2x

A
Singular solution : 4x2y + 1 = 0
 
D-4. (i) c1ex + c2 (ii) 64y = (e8x – 8x) + 7

UH
PART - II
Section (A) :
A-1. (A) A-2. (D) A-3. (A) A-4. (D) A-5. (A) A-6. (B)
JA
A-7. (B) A-8. (C)
Section (B) :
B-1. (C) B-2. (B) B-3. (B) B-4. (A) B-5. (D)

B-6. (A) B-7. (B) B-8. (C) B-9. (D) B-10. (B)
LP

Section (C) :
C-1. (A) C-2. (B) C-3. (A) C-4. (D) C-5. (C)
A

Section (D) :
D-1. (A) D-2. (B) D-3. (A) D-4. (B) D-5. (C)
NK

PART - III
1. (A) → (r), (B) → (p), (C) → (s), (D) → (q) 2. (A) → (q), (B) → (r), (C) → (s), (D) → (p)

EXERCISE - 2
SA

PART - I

1. (A) 2. (C) 3. (B) 4. (A) 5. (A) 6. (D) 

7. (B) 8. (A) 9. (A) 10. (D) 11. (B) 12. (A)

13. (B) 14. (D) 15. (D) 


PART – II

1. 35 2. 2 3. 4 4. 1 5. 2 6. 8

7.  +  = 1 8. 16 9. 4 10. 25 11. 64 12. 3


13. 2 14. 0
PART - III
1. (AB) 2. (ABC) 3. (AD) 4. (AD) 5. (AB) 6. (ACD) 7. (C)

8. (ACD) 9. (ABD) 10. (AB) 11. (ABC) 12. (AD) 13. (BC)

14. (ABC) 15. (AC) 16. (BC)


 
PART - IV

1. (B) 2. (A) 3. (A) 4. (A) 5. (B) 6. (C) 7. (A)


8. (C) 9. (B)

RI
EXERCISE - 3

A
PART - I

1. (C) 2. (A) → (p, q, s), (B) → (p, t), (C) → (p, q, r, t), (D) → (s)

UH
3. (A) → (p), (B) → (q, s), (C) → (q, r, s, t), (D) → (r) 4. 9

5. Bonus (Taking x = 1, the integral becomes zero, whereas the right side of the equation gives 5.
Therefore, the function f does not exist.)
JA
6. 0 7.* (AD) 8. (A) 9. (B) 10. (B) 11*. (AC)

12*. (BC) 13. (A) 14. (A,D) 15. (B) 16. (BC) 17. (BC)

18. (BCD) 19. (0.4) 20. 2


LP

 
PART - II
A

1. (4) 2. (4) 3. (2) 4. (1) 5. (2) 6. (3) 7. (1)

8. (3) 9. (3) 10. (3) 11. (3) 12. (1) 13. (1) 14. (3)
NK

15. (1) 16. (3)


SA
dy cos x
1. Solve the differential equation = (sinx – siny)
dx cos y
dy
2. Solve : (1 + xy) y + (1 – xy) x =0
dx
dy
3. Use the substitution y2 = a – x to reduce the equation y3 + x + y2 = 0 to homogeneous form and
dx
hence solve it. (where ‘a’ is variable)

RI
dy
5. Solve : − y n2 = 2 sin x
(cos x − 1) n2 , y being bounded when x →  .
dx

dy  2x  1

A
6. Solve : y = given that y = 0, when x = 1.
dx  1 + x 2
+
 2
(
1 + x2 )
7. Solve the differential equation, (x 2 + 4y2 + 4xy) dy = (2x + 4y + 1) dx.

UH
8. Solve the following differential equations.
dy 2y x3 dy
(i) 3 + = 2 (ii) x 2 y − x3 = y4 cos x
dx x +1 y dx
dy
JA
9. Find the integral curve of the differential equation x(1 – xny) + y = 0 which passes through (1, 1/e).
dx

10. Solve the following differential equations.


dy
(i) (x² + y² + a²) y + x (x² + y² − a²) = 0 (ii) (1 + tany) (dx − dy) + 2x dy = 0
dx
LP

dy
11. If y1 & y2 be solutions of the differential equation + Py = Q, where P & Q are functions of x alone,
dx
−  yQ1 dx
and y2 = y1 z, then prove that z = 1 + ae ,'a' being an arbitrary constant.
A

12. Let y1 and y2 are two different solutions of the equation y + P(x) . y = Q(x).
Prove that y = y1 + C(y2 – y1) is the general solution of the same equation (C is a constant)
NK

13. Find the equation of the curve which passes through the origin and the tangent to which at every point
x 4 + 2xy − 1
(x, y) has slope equal to .
1 + x2

14. A curve y = f(x) passes through the point p (1, 1). The normal to the curve at P is; a (y – 1) + (x – 1) = 0.
SA

If the slope of the tangent at any point on the curve is proportional to the ordinate of the point,
determine the equation of the curve. Also obtain the area bounded by the y-axis, the curve & the
normal to the curve at P.

15. Consider a curved mirror y = f(x) passing through (8, 6) having the property that all rays emerging from
origin after getting reflected from the mirror becomes parallel to x - axis. Find the equation of curve (s)

16. Find the curve for which sum of the lengths of the tangent and subtangent at any of its point is
proportional to the product of the co-ordinates of the point of tangency, the proportionality factor is
equal to k.

17. Find the curve y = f(x) where f(x)  0, f(0) = 0, bounding a curvilinear trapezoid with the base [0, x]
whose area is proportional to (n + 1)th power of f(x). It is known that f(1) = 1
18. Find the nature of the curve for which the length of the normal at the point P is equal to the radius
vector of the point P.
19. A country has a food deficit of 10 %. Its population grows continuously at a rate of 3 % per year. Its
annual food production every year is 4 % more than that of the last year. Assuming that the average
food requirement per person remains constant, prove that the country will become self−sufficient in
n 10 − n 9
food after ' n ' years , where ' n ' is the smallest integer bigger than or equal to, .
n (1.04) − 0.03

 y2 1 1 x2 
   
20. Solution of Differential equation  2
−  dx +  − 2
dy = 0 is

 ( x − y ) x 
 

y ( x − y ) 

21. Solution of the differential equation (xdy – ydx) (x+y)2= 4xy(x2+y2) (xdx –ydy) is

RI
x(y − x)
22. Solve . x2 dy + y2 e y
= 2y(x − y)
dx


y  −y 

A
23. Solution of differential equation xe x dy −  ye x + x 3  dx = 0 is
 
 

UH
x 1
1. siny = sinx – 1 + c e–sinx 2. n – =c
y xy
JA
1 a
3. n | x 2 + a2 | − tan−1   = c where, a = x + y2 5. y = 2sinx
2 x
 3  x + 2y + 2 − 2 
6. y(1 + x2) = tan–1x –
4
7. ( 2
y = n ( x + 2y ) + 4 ( x + 2y ) + 2 − ) n +c
2 2  x + 2y + 2 + 2 
LP

x6 2 5 1 4 −3
8. (i) y 3 (x + 1) 2 = + x + x + c (ii) x3 y = 3sin x + c
6 5 4
9. x(ey + ny + 1) = 1

10. (i) (x² + y²)² + 2a² (y² − x²) = c (ii) x e y (cosy + siny) = e y siny + C
A

1 1 
13. y = (x – 2tan–1 x) (1 + x2) 14. ea(x – 1), a − + e−a  sq. unit
a 2 
1
NK

15. y2 = 4(1 + x) or y2 = 36(9 – x) 16. n | c(k2x2 – 1) |


y=±
k
17. y = x1/n 18. Rectangular hyperbola or circle.   19. 19

y xy y 1 x 2 x(x − y) = y ln(cex − 1)


20. n   + = C 21. tan–1   = n   + x – y2 + c 22.
x y−x x 2 y
SA

y

23. 2e x + x2 = C
DIFFERENTIATION

In the previous chapter, we have studied the differentiability of f (x ) at a point. We know that f is
differentiable at ‘a’ if f (a) exists. We say that f is a differentiable function, if it is differentiable at each
point in its domain. 
dy d
For y = f (x ) derivative function is denoted by or by [f ( x )] or by Df (x ) or by y  ,
dx dx
dy
hence is a symbol for the derivative. The process of taking the derivative is known as
dx
differentiation.

RI
 
1 DIFFERENTIATION FROM FIRST PRINCIPLE

When the derivative of a function is calculated directly by using the definition of derivative,

A
it is called differentiation from first principle. This method is also known as ab-initio method or
delta method.

UH
Illustration 1

3

Question: Differentiate x 2 , with respect to x, from first principle.
3

JA
Solution: Let f (x ) = x 2

3
3 −

 f (x ) = lim
(x + h ) − 2 −x 2
, h is small increment in x.
h →0 h

  3  5  
LP

3  − −  2 
3  −
3
 −
1 − 3 . h +  2   2  h + ...... − 1
1 + h   2
− x 2
x 2 − 1  2 x 2! x2 
 x   
   
= lim = lim
A

h →0 h h →0 h

  3  5  
 − −  
NK

3 3 5
− 3 +  2   2  . h + ...... = x 2   − 3  = − 3 x 2
− − −
= lim x 2
h →0  2x 2! x2   2x  2
 
 
SA

Illustration 2

Question: Differentiate tan 3 x , with respect to x, from first principle.

Solution: Let f (x ) = tan 3 x

tan 3(x + h ) − tan 3 x


 f (x ) = lim , h is small increment in x.
h →0 h

Multiplying and dividing by ( tan(3 x + 3h ) + tan 3 x , we get )


tan(3 x + 3h ) − tan 3 x
= lim
h →0 h[ tan(3 x + 3h ) + tan 3 x ]
tan(3 x + 3h − 3 x )[1 + tan(3 x + 3h ) tan 3 x ]
= lim
h →0 h[ tan(3 x + 3h ) + tan 3 x

 tan A − tan B 
= tan(A − B )(1 + tan A tan B )
 

tan 3h [1 + tan(3 x + 3h ) tan 3 x ] 1 + tan 2 3 x 3 sec 2 3 x


= lim .3. = 1 3  =  
h →0 3h tan(3 x + 3h ) + tan 3 x 2 tan 3 x 2 tan 3 x

Illustration 3

RI
x
Question: Differentiate e , with respect to x, from first principle.

Solution: Let f (x ) = e x

A
e x +h
−e x
 f (x ) = lim , h is small increment in x
h →0 h

UH
x +h − x x +h − x
e x
[e − 1] x e −1 x+h − x
= lim = lim e . .
h →0 h h →0 x+h − x h

x+h−x 1 e x
x
JA x
= e .1. lim =e . lim =
h →0 h[ x + h + x ] h →0 x+h + x 2 x

Illustration 4

Question: Find the derivative of tan−1x with first principle.


LP

  
Solution: Let tan−1x = ,   − , 
 2 2

 x = tan …(i)
A

and tan −1
(x + h) =  + 

 x + h = tan ( + ) ..(ii)


NK

tan −1 ( x + h) − tan −1 x
Let lim =L
h →0 h
 +  −  
  L =  lim = lim   (from (i) and (ii)) 
h →0 h h →0 h
SA

 
= lim = lim
h →0 tan(  + ) − tan  →0 tan(  + ) − tan 
( ) cos( + ) cos  1 1
= lim = cos2 = =
→0 sin  sec 2
1+ x 2
 
2 DERIVATIVES OF SOME OF THE FREQUENTLY USED FUNCTIONS

Function Derivative
c (constant) 0
sinx cosx
cos x –sinx
tanx sec2x
cosecx −cosecx cotx
cotx −cosec2x
secx secx tanx
−1
sin x 1
1− x 2
cos−1x −1
1− x 2
tan−1x 1
1+ x 2

RI
cot−1x −1
1+ x 2
sec−1x 1

A
| x | x2 −1
cosec−1 x −1

UH
| x | x2 −1
loge x 1/x
loga x (1/x) loga e
xn
nxn−1
JA
ax ax logea
ex ex
The above written derivatives can be easily found by using the definition of differentiation.
LP

 
3 RULES TO FIND OUT DERIVATIVES

Let u and v are differentiable functions of ‘x’.


A

(i) The sum rule:


d du dv
(u + v ) = +
dx dx dx
NK

d de x d (log e x ) 3
e.g. (2e x + 3 log e x ) = 2 +3 = 2e x +
dx dx dx x
SA

d
in general (u  v  w  ......) = du  dv  dw ......
dx dx dx dx

Illustration 5

Question: Differentiate 5 sin x − 2 loge x .


d
Solution: (5 sin x − 2 loge x ) = d (5 sin x ) − d (2 loge x ) = 5 cos x − 2
dx dx dx x

(ii) Product rule:


d (uv ) dv du
=u +v
dx dx dx
d ((sin x ) e x ) de x d (sin x )
e.g. = sin x + ex = (sinx) ex + (cosx) ex.
dx dx dx

 du 
in general
d
(u1u 2 u 3 ......) = (u1u 2 u 3 ......) 1 du1 + 1 du 2 + 1 3 ........
dx  u1 dx u 2 dx u 3 dx 

Illustration 6

Question: Differentiate x 2e x sin x .

Solution: First we differentiate x 2 e x

RI
d 2 x d x d 2
dx
(
x e = x2
dx
)e + ex ( )
dx
x = x 2 e x + 2xe x( )

A
d 2 x d
Now, ( )
x e sin x = x 2 e x (sin x ) + sin x d x 2 e x ( )
dx dx dx

(
= x 2e x cos x + sin x x 2 + 2x e x )

UH
( )
= e x x 2 cos x + x 2 sin x + 2x sin x = xe x (x cos x + x sin x + 2 sin x )

(iii)  The quotient rule: 
Here v(x)  0
JA
du dv
v −u
d u  dx dx
 =
dx  v  v2

d (tan x ) dx
LP

x − (tan x )
d  tan x  dx dx
e.g.  =
dx  x  x2

x sec 2 x − tan x
=
A

x2
NK

Illustration 7

ex
Question: Differentiate .
1 + sin x

(1 + sin x ) d e x − e x d (1 + sin x )
( )
SA

d  e x  dx dx
Solution: =
dx  1 + sin x  (1 + sin x )2

=
(1 + sin x )e x − e x cos x =
e x (1 + sin x − cos x )
(1 + sin x )2 (1 + sin x )2
(iv) Chain rule:
The chain rule is probably the most widely used differentiation rule in mathematics. Chain
rule says that the derivative of the composition of two differentiable functions is the product of their
derivatives evaluated at appropriate points.
The formula is [f (g (x ))] = f (g (x )). g (x )
Illustration 8

Question: Differentiate sin x 2 .

Solution: Put y = x 2 and z = sin y

dy dz
Then = 2 x and = cos y
dx dy


d
dx
(
sin x 2 =
dz dz dy
)= .
dx dy dx
(
= (cos y ) (2 x ) = cos x 2 (2x ) = 2x cos x 2 )
This solution can be rewritten using a more convenient notation in the following manner:

RI
d
(
sin x 2 =
d sin x 2 d x 2
) .
( ) ( )
= cos x 2 .2 x = 2x cos x 2
dx dx 2
dx ( )

A
(v) Differentiation of parametrically defined functions:

Working rule:

UH
dx dy
(a) If x and y are functions of parameter ‘’, then find and respectively.
d d
dy dy d
(b) Then = 
dx d dx
JA
e.g. x = sint + cost
y = cost
dx dy
= cos t − sin t ; = − sin t
dt dt
LP

dy − sin t
 =
dx cos t − sin t
A

Illustration 9

dy
Question: Find , if x = a( − sin ); y = a(1 − cos ) .
NK

dx

Solution: Consider x = a( − sin ); y = a(1 − cos )

dx dy
 = a(1 − cos ); = a(0 + sin ) = a sin 
d d
SA

 
2 sin cos
dy dy dx a sin  2 2 = cot 
 =  = =
dx d d a(1 − cos )  2
2 sin 2
2
Illustration 10

dy
If y = e − ax sin( x loge x ) , find
2
Question: .
dx
d  e −ax 
 
2

2 d sin(x log x )
+ sin(x loge x ).  
dy d −ax 2
Solution: = e sin(x loge x ) = e −ax e
dx dx dx dx
d  e −ax  d
2

d sin(x log e x ) d  .
= e −ax 2
.
d (x log e x ) dx
(x log e x ) + sin(x loge x )
d − ax 2 dx
− ax 2
( ) ( )
 1
= e −ax cos(x loge x ). 1 . loge x + x .  + sin(x loge x )e −ax (− 2ax )
2 2

x 
− ax 2
cos(x loge x )(loge x + 1) − 2axe −ax sin(x loge x )
2
= e

Illustration 11

cot x dy
Question: If y = 1 − x 2 + , find .
x dx

d  cot x 

RI
dy d
Solution: = 1− x 2 +  
dx dx dx  x 
d
d 1− x 2 d 1− x 2 ( ) x (cot x ) − cot x . d (x )

A
= . + dx dx
(
d 1− x 2 dx ) ( x) 2

1
( )
x − cosec 2 x − cot x .

UH
1 2 x
= (− 2x ) +
2 1− x 2 x

x 2 x cosec 2 x + cot x
= − −
1− x 2 2x 3 2
JA
(vi) Differentiation of Implicit Function:

If a relation between x and y is such that y cannot be expressed in terms of x, then y is


dy
called an implicit function of x. Here we will give method to find if y is an implicit function of x.
dx
LP

Working Rule:
(i) Differentiate the given relation between x and y w.r.t. x.
dy
(ii) Bring all the terms containing on left hand side and remaining terms on right
dx
A

dy
hand side and then find .
dx
(iii) Use the given relation between x and y to get the result in simplified form.
NK

Illustration 12

dy
If y = tan(x + y ), find
SA

Question: .
dx
Solution: y = tan(x + y ) …(i)
Differentiating w.r.t. x, we get
dy
=
d
tan(x + y ) = d tan(x + y ). d (x + y )
dx dx d (x + y ) dx
 dy  dy
= sec 2 (x + y ). 1 +  = sec 2 (x + y ) + sec 2 (x + y )
 dx  dx
dy
or [1 − sec 2 (x + y )] = sec 2 (x + y )
dx
dy sec 2 (x + y ) 1 + tan 2 (x + y ) 1+ y 2
 = = =

dx 1 − sec 2 (x + y ) 1 − 1 + tan 2 (x + y ) 1 − 1 + y 2  ( )
1+ y 2
= − [from (i), y = tan(x + y ) ]
y2

Illustration 13

1 dy
Question: If x = y + , prove that = 2 x 2 + y 2 − 3 xy .
y +1 dx
y+1
y + ...... to 

1
Solution: x=y+
y +1
y +1
y +...... to 

RI
1
 x=y+ …(i)
x
Differentiating w.r.t. x, we get
dy 1 dy 1
− 2

A
1= or = 1+ 2
dx x dx x
dy 2  1 
or = 1 + (x − y ) ;  from (i), x = x − y 

UH
dx  
= 1 + x 2 + y 2 − 2xy …(ii)
From (i), x 2 = xy + 1, 1 = x 2 − xy
dy
Putting in (ii), we get = x 2 − xy + x 2 + y 2 − 2 xy
JA
dx
dy
Hence = 2 x 2 + y 2 − 3 xy
dx

Illustration 14
LP

dy
Question: If x 3 y 2 = log e (x + y ) + sin e x , find ( ) dx
.

Solution: x 3 y 2 = loge (x + y ) + sin e x ( ) …(i)


A

Differentiating w.r.t. x, we get


dy 1  dy 
3 x 2 y 2 + x 3 .2 y = 1 + ( )
 + cos e x . e x
NK

dx x + y  dx 
dy 1 1 dy
or 3 x 2 y 2 + 2x 3 y = + .
dx x + y x + y dx
+ e x cos e x ( )
or
 3
 2 x y −  =
( )
1  dy 1 + (x + y )e x cos e x − 3 x 2 y 2 .(x + y )
SA

 x + y  dx x+y

or =
( )
dy 1 + (x + y )e x cos e x − 3 x 3 y 2 − 3 x 2 y 3
dx 2x 4 y + 2x 3 y 2 − 1
(vii) Logarithmic differentiation:
When u and v both are functions of x, then derivative of a function of the form u v cannot
be found directly by using standard formula and hence in such cases both sides are differentiated
after taking logarithm. This process is called logarithmic differentiation.
Illustration 15

dy
Question: If y = x x , find
dx
Solution: [Here power is variable]
Given, y = x x …(i)
Taking logarithm we get, loge y = loge x x ( )
or loge y = x loge x
d
Differentiating w.r.t. x, we get (loge y ) = d (x loge x )
dx dx

RI
d d (x ) d
or (loge y ) = . loge x + x (loge x )
dy dx dx
1 dy 1
or . = 1 . loge x + x . = loge x + 1
y dx x

A
dy
 = y (1 + loge x ) = x x (1 + loge x ) [from (i), y = x x ]
dx

UH
Illustration 16

dy
Question: If x m y n = (x + y )m + n , find .
dx
m+n
JA
Solution: x m y n = (x + y )
Taking logarithm, we get
( ) ( )
loge x m + loge y n = (m + n ) loge (x + y ) or m loge x + n loge y = (m + n ) loge (x + y )
m n dy m + n  dy  m + n m + n dy
Differentiating w.r.t. x, we get, + = 1 + = +
x y dx x+y  dx  x + y x + y dx
LP

 n m + n  dy m + n m
or  −  = −
 y x + y  dx x+y x
nx + ny − my − ny dy mx + nx − mx − my nx − my dy nx − my
= or =
y (x + y ) dx x (x + y ) y (x + y ) dx x (x + y )
A

dy y
 =
dx x
NK

Illustration 17

dy
Question: If y = e x sin x 3 + (tan x )x , find .
dx
x
Solution: Let u = e x sin x 3 and v = (tan x )
SA

Now, u = e x sin x 3

Differentiating w.r.t. x, we get


du
= ex .

d sin(x )
3

+ sin x 3 .
d x
e( )
dx dx dx
= e x . cos x 3 . 3x 2 + sin x 3 . e x
du
Hence = 3 x 2 e x cos x 3 + e x sin x 3
dx
x
And v = (tan x )  loge v = x loge (tan x )
Differentiating w.r.t. x, we get
1 dv 1
= 1 . loge (tan x ) + x . sec 2 x
v dx tan x
dv
 = v [loge (tan x ) + x cot x. sec 2 x ] = (tan x )x [loge (tan x ) + x cot x sec 2 x ]
dx
Now y = u +v
dy du dv
 = +
dx dx dx
( ) x
( )
= 3x 2e x cos x 3 + e x sin x 3 + (tan x ) [loge (tan x ) + x cot x sec2 x]

(viii) Differentiation by Substitution:


Sometimes, it is easier to differentiate, by making substitutions. Usually these examples
involve inverse trigonometric functions.

Illustration 18

RI
 5 x + 12 1 − x 2  5  1
Question: If f ( x ) = cos−1 ,  x  1 , then find f   .
 13  13 2
 

A
Solution: Put x = cos 

 5 cos  + 12 sin    5 
f (x ) = cos −1   > 0 = cos −1 (cos( − )) > 0, where  = cos −1 

UH
 13   13 

 5   5 
=  –  as x   , 1 = cos−1  − cos−1 x
 13   13 

 1
JA2
1
 f (x ) =  f   =
1− x 2  
2 3

Illustration 19
LP

Question: ( )
Differentiate cos−1 4 x 3 − 3 x , where 0  x  1 .

Solution: Put x = cos 

  
 3 ,   0, 
A

  3
cos −1 (
4x − 3x = 
3
)
2 − 3 ,     ,  

 
 3 2
NK

d
dx
(
[cos−1 4 x 3 − 3 x ] = −
3
) if
1
2
 x 1
1− x2

3 1
= if 0  x 
SA

1− x 2 2

 
4 DERIVATIVE OF SECOND ORDER y’’ or y2

d 2y d  dy 
=   is derivative of second order and is denoted by y   or y 2
dx 2
dx  dx 
Illustration 20

Question: If y = loge (loge x ) , find y 2 .

Solution: Let y = loge (loge x )


1 1 1
y1 = . =
loge x x x loge x

d  1  d −1 −2 d
 
y2 =
dx  x log x  = dx (x loge x ) = −1(x loge x ) . dx (x loge x )
 e 

1  1  − (1 + loge x )
= − 2
.  x. + loge x . 1 =
(x loge x )  x  (x loge x )2
Illustration 21

Question: If y = a cos(loge x ) + b sin(loge x ) , prove that x 2y  + xy  + y = 0 .

RI
Solution: Consider y = a cos(loge x ) + b sin(loge x ) …(i)

Differentiating w.r.t. x, we get

A
 1  1
y  = a . − sin(loge x ).  + b cos(loge x ). 
 x   x 

xy  = −a sin(loge x ) + b cos(loge x )

UH

Again differentiating w.r.t. x, we get

 1  1
xy  + y  . 1 = −a . cos(loge x ).  + b . − sin(loge x ). 
 x   x 
JA
 x 2y  + xy  = −a cos(loge x ) + b sin(loge x ) = −y [from (i)]

 x 2 y  + xy  + y = 0

 
LP

5 DIFFERENTIATION OF A FUNCTON WITH RESPECT TO ANOTHER


FUNCTION

Let u = f (x ) and v = g (x ) be two functions of ‘x’, then to find derivative of f (x ) w.r.t. g (x )


A

du
i.e. to find , we use the formula
dv
NK

du du dx
=
dv dv dx
Illustration 22
SA

Question: Differentiate loge sin x w.r.t. cos x .


Solution: Let u = loge sin x and v = cos x
du dv − sin x
Then = cot x and =
dx dx 2 cos x
du du dx
 = = −2 cos x cot x. cosecx
dv dv dx
SOLVED EXAMPLES

Example 1:
dy
If x 1 + y + y 1 + x = 0 , prove that = − (1 + x ) − 2 .
dx

Solution:

Given, x 1 + y = −y 1 + x or x 2 (1 + y ) = y 2 (1 + x ) or x 2 − y 2 = xy (y − x )

(x − y ) (x + y + xy ) = 0

RI
or

but x − y = 0 , i.e., x = y doesn’t satisfy given equation, hence x + y + xy = 0

or x + y = −xy or y (1 + x ) = −x …(i)

A
dy
Differentiating both sides w.r.t. x, we get y .1 + (1 + x ) = −1
dx

UH
 −x 
 + 1
 y + 1  
or (1 + x ) dy = −(y + 1) or
dy
= − =−
1 + x
[from (i)]
dx dx  1+ x  1+ x

−x + 1+ x
JA
−2
= − 2
= −(1 + x )
(1 + x )

Example 2:

d  3 d 2y 
If y 2 = P (x ) is a polynomial of degree 3, then show that 2 y  = P ( x ) . P ( x ) .
LP

dx  dx 2 
 

Solution:
dy
Given, y 2 = P (x ) …(i);  2y = P (x ) …(ii)
A

dx

dy dy d 2y
Differentiating again w.r.t. to x, we get 2 . + 2y . 2 = P (x )
NK

dx dx dx
2 2
 dy  d 2y d 2y  dy 
or 2y 2   + 2y 3 = y 2 P (x ) or 2y 3 = y 2 P (x ) − 2y 2  
 
dx dx 2
dx 2
 dx 

d 2y
SA

1
 2y 3 2
= y 2 P (x ) − [P (x )] 2 [from (ii)]
dx 2

d  3 d 2 y  dy 1
Differentiating w.r.t. x, we get  2 y = 2y P (x ) + y 2 P (x ) − 2(P (x ). P (x ))
dx  dx 2  dx 2

 dy 
= P (x )P (x ) + y 2 P (x ) − P (x )P (x )  2y dx = P (x )
 

= y 2P (x ) = P (x )P (x ) [ y 2 = P (x )]
Example 3:

c1 c2 x c3 x 2
If y = 1 + + + , show that
x − c1 ( x − c1 )( x − c2 ) ( x − c1 )( x − c2 ) (x − c3 )

dy y  c1 c2 c3 
=  + + .
dx x  c1 − x c2 − x c3 − x 

Solution:

 c1  c2 x c3 x 2
y = 1 +  + +
 x − c1  (x − c1 )(x − c 2 ) (x − c1 )(x − c 2 )(x − c 3 )

RI
 x c2 x  c3 x 2
=  +  +
 x − c1 (x − c1 )(x − c 2 )  (x − c1 )(x − c 2 )(x − c 3 )

x2 c3 x 2 x3 x x x

A
= + = = . .
(x − c1 )(x − c 2 ) (x − c1 )(x − c 2 )(x − c 3 ) (x − c1 )(x − c 2 )(x − c 3 ) x − c1 x − c 2 x − c 3
 x   x   x 

UH
 log y = log  + log  + log 

 x − c 1  x − c2   x − c3 

Differentiating w.r.t. x, we get

1 dy x − c1 x − c1 − x x − c 2 x − c 2 − x x − c 3 x − c 3 − x
= . + . + .
JA
y dx x (x − c1 )2 x (x − c 2 )2 x (x − c 3 )2
1  c1 c2 c3 
= −  + + 
x  x − c1 x − c 2 x − c 3 

dy y  c1 c2 c3 
 =  
LP

+ +
dx x  c1 − x c 2 − x c 3 − x 

Example 4:
A

If x 2 + y 2 = 1, then

(a) yy  − 2(y  ) + 1 = 0 (b) yy   + (y  )2 + 1 = 0


NK

(c) yy  + (y  ) − 1 = 0 (d) yy   + 2(y  ) + 1 = 0


2 2

Solution:

We have, x 2 + y 2 = 1
SA

Differentiating w.r.t. x, we get 2x + 2yy  = 0  x + yy  = 0

Again differentiating w.r.t. x, we get

1 + (y ) + yy  = 0
2

Hence (b) is correct.


Example 5:
If y is a function of x and log e (x + y ) − 2 xy = 0 , then the value of y (0) is equal to

(a) 1 (b) –1 (c) 2 (d) 0

Solution:
When x = 0, we have

log e (0 + y ) − 2  0  y = 0  y = 1

Now, loge (x + y ) − 2xy = 0

Differentiating with respect to x, we get

RI
1  dy  dy
1 +  − 2y − 2 x =0
x+y dx  dx

 dy 

A
Putting x = 0 and y = 1, we get 1 +  − 2 − 20 = 0
 dx 

dy

UH
 =1
dx
Hence (a) is correct.
 
 
JA
 
 
 
 
 
LP

 
 
 
 
A

 
 
 
NK

 
 
 
 
SA

 
 
 
 
 
 
 
 
 
 
 
 
 

MIND MAP

DIFFERENTIATION

 
 
 

RI
  Function Derivative
• The sum rule
  c (constant) 0 d du dv
(u + v ) = +
  dx dx dx
sinx cosx

A
 
  cos x –sinx
  tanx sec2x

UH
• Product rule
 
cosecx −cosecx cotx d (uv ) dv du
  =u +v
  dx dx dx
cotx −cosec2x
 
secx secx tanx
JA
 
  sin−1x 1 • The quotient rule 
  1− x 2
Here v(x)  0
 
cos−1x −1
  v du u dv

1− x 2
d u 
LP

   = dx dx
  dx  v  v 2
tan−1x 1
  1+ x 2
 
cot−1x −1
A

  • Chain rule
  1+ x 2 [f (g (x ))] = f (g (x )). g (x )
  sec−1x
NK

1
 
| x | x2 −1
 
  cosec−1 x −1
• If x and y are functions of parameter
  | x | x2 −1 dy dy d
SA

  ‘’, then = 
loge x 1/x dx d dx
 
  loga x (1/x) loga e
 
xn nxn−1
  • Let u = f (x ) and v = g (x ) be two
  ax ax logea du
functions of ‘x’, then to find ,
  ex ex dv
  use formula
du du dx
=  
  dv dv dx
 
 
EXERCISE – I

SINGLE CHOICE CORRECT

f ( x ) − f ( 4)
1. If f ( x ) = x 2 + 9 , then lim has the value
x →4 x−4
(a) 5/4 (b) –4/5 (c) 4/5 (d) none of these

2. If f ( x ) = log | x |, x  0 , then f ' ( x ) equals


1 1 −1

RI
(a) (b) (c) (d) none of these
|x| x x

3. If f ( x ) =| x − 2 | and g( x ) = f (f ( x )), then for x  20, g ' ( x ) equals

A
(a) –1 (b) 1 (c) 0 (d) none of these

4. If f ( x ) = 1 − sin 2 x , then f ' ( x ) equals

UH
(a) − (cos x + sin x ), for x  ( / 4,  / 2) (b) cos x + sin x, for x  (0,  / 4)
(c) − (cos x + sin x ), for x  (0,  / 4) (d) cos x − sin x, for x  ( / 4,  / 2)

5. If f ( x ) =| x 2 − 5 x + 6 |, then f ' ( x ) equals


(a) 2x − 5 for 2  x  3 (b) 5 − 2x for 2  x  3
JA
(c) 2x − 5 for 2  x  3 (d) 5 − 2x for 2  x  3
dy
6. If 2 x + 2 y = 2 x + y , then the value of at x = y = 1 is
dx
(a) 0 (b) –1 (c) 1 (d) 2
LP

7. The differential coefficient of f (loge x ), where f ( x ) = loge x is


x loge x
(a) (b) ( x loge x ) −1 (c) (d) none of these
log e x x
A

8. If f ( x ) = ( x + 1) tan −1(e −2 x ), then f´ (0) is


  
(a) +1 (b) −1 (c) +5 (d) none of these
NK

2 4 6

9. Differential coefficient of sec (tan–1 x) is


x 1 x
(a) (b) x 1 + x 2 (c) (d)
1+ x 2
1+ x 2 1+ x 2
SA

1− t 2 2t dy
10. If x = 2
and y = 2
, then  is equal to
1+ t 1+ t dx
y y x x
(a) − (b) (c) − (d)
x x y y

11. If f ( x ) = (cos x + i sin x )(cos3x + i sin 3x )....(cos(2n − 1)x + i sin( 2n − 1)x ), then f"(x) is equal to
(a) n 2 f ( x ) (b) − n 4 f ( x ) (c) − n 2f ( x ) (d) n 4 f ( x )

d 2x
12. is equal to
dy 2
d 2y  dy   d 2 y 
(a) (b)    2 
dx 2  dx   dx 
−3
 dy  d 2y
(c) −  (d) none of these
 dx  dx 2

b b
13. If a curve is given by x = a cos t + cos 2t and y = a sin t + sin 2t , then the points for which
2 2
d 2y
= 0 are given by
dx 2
2a 2 + b 2 a 2 + 2b 2
(a) sin t = (b) cos t = −
3ab 3ab

RI
(c) tan t = a / b (d) none of these

d 3y
14. If y = sin2  + cos2 ( + ) + 2 sin  sin cos( + ), then , is
d 3

A
sin3 ( + )
(a) (b) cos( + 3) (c) 0 (d) none of these
cos 

UH
d 2y
15. If x + y + y − x = c, then equals
dx 2
(a) 2/c (b) –2/c2 (c) 2/c2 (d) none of these
JA
d
16. {log e (ax ) x }, where a is a constant, is equal to
dx
1
(a) 1 (b) loge (ax ) (c) (d) loge (ax ) + 1
a
 1 + sin x 
LP

17. If f ( x ) = tan −1  ,0  x   , then f '    is



 1 − sin x  2 6
 1 1 1
(a) − (b) − (c) (d)
4 2 4 2
A

18. If f ( x ) = log e | x | , then f ' ( x ) equals


NK

1 1 1
(a) ,x0 (b) for | x | 1 and − for | x | 1 , x  0
|x| x x
1 1 1 1
(c) − for | x | 1 and for | x | 1 , x  0 (d) for x  0 and − for x  0
x x x x
SA

19. The functions u = e x sin x; v = e x cos x satisfy the equation

du dv d 2u
(a) v −u = u2 + v 2 (b) = 2v
dx dx dx 2
d 2v
(c) = −2u (d) all of these
dx 2

−1  2 cos x − 3 sin x 
20. If y = cos   , then dy is
 13  dx
(a) zero (b) constant = 1
(c) constant  1 (d) none of these
2x 2x
21. Derivative of tan −1 w.r.t. sin −1 is
1− x 2
1+ x 2
1 1
(a) (b)
1+ x 2 1− x 2
(c) 0 (d) 1

tan x dy
22. If y = (sin x ) , then is equal to
dx
(a) (sin x )tan x .(1 + sec 2 x. log e sin x ) (b) tan x(sin x ) tan x −1. cos x
tan x −1
(sin x )tan x . sec 2 x. log e . sin x (d) tan x.(sin x )

RI
(c)

cos2 x  


23. If f ( x ) = , then f   − 3f '   is
  4
2

A
1 + sin x 4
(a) –1 (b) 1
(c) 2 (d) 3

UH
x x2 x3
dy
24. If f ( x ) = 1 2 x 3 x 2 , then is
dx
0 2 6x
JA
(a) 12 (b) 6x2
(c) 6x (d) 4x3

x a a
x a
If  1 = b x a and  2 =
LP

25. , then
b x
b b x
d d
(a) 1 =  2 (b)  1 = 3 2
dx dx
A

d
(c)  2 = 1 −  2 (d) none of these
dx
NK
SA
EXERCISE – II
IIT-JEE – SINGLE CHOICE CORRECT

1. Let f(x) be a polynomial in x. Then the second order derivative of f(ex), is


(a) f " (e x ).e x + f ' (e x ) (b) f " (e x ).e 2 x + f ' (e x ).e 2 x
(c) f " (e x )e 2 x (d) f " (e x )e 2 x + f ' (e x ).e x

2. Let f ( x ) = sin x, g( x ) = x 2 and h( x ) = loge x. If F ( x ) = (hogof )( x ), then F " ( x ) is equal to


(a) 2 cosec3 x (b) 2 cot x2 –4x2 cosec2 x2

RI
(c) 2x cot x2 (d) –2 cosec2 x
d 2x
3. If y = sin x + e x , then equals
dy 2

A
sin x − e x
(a) (− sin x + e x ) −1 (b)
(cos x + e x ) 2

UH
sin x − e x sin x + e x
(c) (d)
(cos x + e x ) 3 (cos x + e x ) 3

dy
If y = sin x + sin x + sin x + .... to  , then the value of
4.
JA is equal to
dx
sin x sin x
(a) (b)
y +1 y +1
cos x cos x
(c) (d)
2y + 1 2y − 1
LP

f ' (1) f " (1) f " ' (1) f ( n ) (1)


5. If f ( x ) = x n , then the value of f (1) + + + + .... + is
1! 2! 3! n!
(a) n (b) 2n
A

n(n + 1)
(c) 2n −1 (d)
2
dy
NK

6. If y = log (x 2 + 4 ) (7 x 2 − 5 x + 1), then is equal to


dx
 14 x − 5 2 xy 
(a) loge ( x 2 + 4). 2 − 2 
 7x − 5x + 1 x + 4 
 14 x − 5 2 xy 
SA

1
(b)  2 − 2 
loge ( x + 4)  7 x − 5 x + 1 x + 4 
2

 2x (14 x − 5)y 
(c) loge (7 x 2 − 5 x + 1) 2 − 2 
 x + 4 7 x − 5 x + 1
1  2x (14 x − 5)y 
(d)  2 − 
log e (7 x − 5 x + 1)  x + 4 7 x 2 − 5 x + 1
2

 
7. If f ( x ) = sin [ x ] − x 2  for 1 < x < 3 and [x] denotes the greatest integer less than or equal to
 3 
x, then f (  / 3 ) is equal to
(a) /3 (b) −  / 3

(c) −  (d) − 2
3

d  
x3
8. Let f (t ) = ln( t ). Then f (t )dt 

dx  x 2 
 
(a) has value 0 when x = 2
(b) has a value 0 when x = 1and x = 4/9
(c) has a value 9e2 + 4 e when x = e
(d) has a value 27 e + 8 for x = e

RI
 
9. If f ( x ) =| x ||sin x| , then f '  −  equals
 4
 
1/ 2
 2 4 2 2 
1/ 2
 2 4 2 2

A
(a)    ln −  (b)    ln + 
4  2    4  2   
   
 
1/ 2
 2  2 2 
1/ 2
 2  2 2

UH
(c)    ln −  (d)    ln + 
4  2 4   4  2 4  
   

y dy
10. If x = b cos −1 + by − y 2 , then is equal to
JA
b dx
b
(a) −1 (b) 0
y
by − y 2
(c) (d) none of these
y
LP

dy
11. If x y .y x = 1, then is equal to
dx
y ( x loge y − y ) y ( x loge y − y )
(a) (b)
A

x( y loge x − y ) x( y loge x + x )
y ( x loge y + y ) − y ( x loge y + y )
(c) (d)
NK

x( y loge x − x ) x( y loge x + x )

  e 
 log 2  
−1   x   + tan −1  3 + 2 log x , d 2y
12. If y = tan   then is equal to
SA

 log(ex 2 )   1 − 6 log x  dx 2
 
 
(a) 2 (b) 1
(c) 0 (d) –1

a+b b +c c +a
 xa   xb   xc  dy
13. If y =  b  
 xc



 xa

 , then is equal to
x      dx
(a) 0 (b) 1
(c) a + b + c (d) none of these
14. Let f(x) and g(x) be two functions having finite non-zero 3rd derivatives f ' " ( x ) and g ' " ( x ) for
f  g 
all x  R . If f ( x ) g( x ) = 1 for all x  R, then − is equal to
f g
 f " g"   f " g" 
(a) 3 −  (b) 3 − 
g f  f f 
 g" f "   f " g" 
(c) 3 −  (d) 3 − 
g g f g 

 1+ x 2 + 1− x 2 
15. If y = tan −1   , then dy equals
 1+ x 2 − 1− x 2  dx
 

RI
1 1
(a) (b) −
1− x 4 1− x 4

A
x x
(c) (d) −
1− x 4
1− x 4

UH
dy a x ....
16. The expression of of the function y = a x is
dx
y2 y 2 loge y
(a) (b)
x(1 − y loge x ) x(1 − y loge x )
JA
y 2 loge y y 2 loge y
(c) (d)
x (1 − y log e x log e y ) x(1 + y loge x loge y )

d 2y
17. If y 2 = ax 2 + bx + c, then y 3 . is
dx 2
LP

(a) a constant (b) a function of x only


(c) a function y only (d) a function of x and y
A

nx
x  x
18. If y =   1 + loge  , then y ' (n) is given by
n  n

NK

n2 + 1
(a) (b)
n n
n
 1  1
n
 n 2 + 1
(c)   (d)    
n n  n 
 
SA

x
 x  dy
19. If y = 2 log 2 x 2x
−  tan  , then at x = 1 is equal to
 4  dx
4 1
(a) −2 (b) −4
loge 2 loge 2
(c) 4 loge 2 (d) none of these

dy
20. If sin y = x cos(a + y ), then is equal to
dx
cos2 (a + y ) 3 cos(a + y )
(a) (b)
cos a x sin( a + y ) + cos y
2 cos a
(c) 2
(d) none of these
x + 2 x sin a

ONE OR MORE THAN ONE CHOICE CORRECT

1. Let f (x ) = x 2 + xg (1) + g (2) and g (x ) = f (1)x 2 + xf (x ) + f (x ) , then


(a) f (1) + f (2) = 0 (b) g (2) = g (1)
(c) g (2) + 3f (2) = 6 (d) none of these

RI
2. Let f (x ) = (ax + b) cos x + (cx + d ) sin x and f (x ) = x cos x be an identity in x, then
(a) a = 5 (b) b = 1
(c) c = 1 (d) d = −5

A
2x
3. Let f (x ) = 2 tan −1 x + sin −1 , then
1+ x 2

UH
(a) f (2) = f (3) (b) f (2) = 0
 1  16  1
(c) f   = (d) f   = 0
2 5 2
JA
4. If f (x − y ), f (x ).f (y ) and f (x + y ) are in A.P. for all x, y and f (0)  0 , then
(a) f (2) = f (− 2) (b) f (3) + f (− 3) = 0
(c) f (2) + f (− 2) = 0 (d) f (3) + f (− 3) = 0
LP

dy
5. If y = log2 (log2 x ) , then is equal to
dx
log 2 e 1
(a) (b)
x log e x x log e x log e 2
A

1
(c) (d) none of these
log e (2 x )x
NK

6. If y =
(1 + t ) −
2
1− t 2
and x = 1 − t 4 , then
dy
is equal to
1+ t 2 + 1− t 2 dx
SA

1 1− t 4 − 1
(a) − (b)
t2 4
1 + 1 − t 
t6
 

1 1− 1− t 4
(c) (d)
t 2 1 + 1 − t 4  t6
 

xn sin x cos x
n n dn
7. If f (x ) = n ! sin cos , then the value of (f (x )) at x = 0 for n = 2m + 1 is
2 2 dx n
a a2 a3
(a) –1 (b) 0
(c) 1 (d) independent of a

 2x 
8. If f (x ) = sin −1   , then
1+ x 2 
(a) f is derivable for all x, with x  1 (b) f is not derivable at x = 1
(c) f is not derivable at x = −1 (d) f is derivable for all x, with x  1

9. Let f (x ) = x 2 + xg (1) + g (2) and g (x ) = x 2 + xf (2) + f (3 ) , then


(a) f (1) = 4 + f (2) (b) g (2) = 8 + g (1)

RI
(c) g (2) + f (3) = 4 (d) none of these

If f (x ) == sin −1 (sin x ) , then

A
10.
 3   5 
(a) f   = 1 (b) f   = −1
 4   4 

UH

(c) f   does not exist (d) f () does not exist.
2 JA
LP
A
NK
SA
EXERCISE – III
MATCH THE FOLLOWING

Note: Each statement in column – I has one or more than one match in column - II
1.

Column I Column II
 3x − x 3  3 1 1
I. If y = tan −1   , then dy is equal to A. ;− x
 1 − 3x 2  2

RI
  dx 1+ x 3 3

dy −3 1
II. ( )
If y = sin −1 3 x − 4 x 3 , then is equal to B. ;
2
|x|1
dx 1− x 2

A
 1− x 2  3 1
III. If y = cos −1   , then dy is equal to C. ;x
1+ x 2  dx 1+ x 2
3
 

UH
3 1
D. ; x 
dy 1− x 2 2
IV. ( )
If y = cos cos −1 x , then
dx
is equal to

E. 1; x = 1
JA
REASONING TYPE

Direction: Read the following questions and choose:


(A) If both the statements are true and statement-2 is the correct explanation of
LP

statement-1.
(B) If both the statements are true but statement-2 is not the correct explanation of
statement-1.
A

(C) If statement-1 is True and statement-2 is False.


(D) If statement-1 is False and statement-2 is True.
NK

du
1. Statement-1 : If u = e x and v = log x , then = xe x .
dv
Statement-2 : If u = f (x ), v = g (x ) , then the derivative of f with respect to g (x ) is
du du dx
SA

= .
dv dv dx
(a) A (b) B (c) C (d) D

2x 1− x 2
2. Statement-1 : Differentiation of sin −1 with respect to cos −1 is 1 when
1+ x 2 1+ x 2
0  x  1.
 2x  1− x 2 
Statement-2 : sin −1   = cos −1   when − 1  x  1 .
1+ x 2  1+ x 2 
 
(a) A (b) B (c) C (d) D

3. Statement-1 : The function f (x ) = min  x , x − 1 , x + 1  is not differentiable at 5 points in


domain.
Statement-2 : f (x ) is not differentiable at a point where the sharp corner is occurred.
(a) A (b) B (c) C (d) D

4. Statement-1 : Let f be a function such that f (xy ) = f (x ).f (y ) for all x, y  R and
f (1 + x ) = 1 + x (1 + g (x )) and xf (x ) = f (x ) , if and only if lim g (x ) = 1.
x →0

f (x + h ) − f ( x )

RI
Statement-2 : lim = l exists if l is finite.
x →0 h
(a) A (b) B (c) C (d) D

A
d
5. Statement-1 : If x  0 , then (ln | x |) = − 1 .
dx x

UH
Statement-2 : x = − x if x  0 .
(a) A (b) B JA (c) C (d) D

LINKED COMPREHENSION TYPE


The function f (x ) = x is not differentiable at x = 0 but for every x  0 , f (x ) is differentiable
x
and f (x ) =
x

 1
LP

| x| − 1
1. If f (x ) = e , then f   is
2
(a) e 1 2 (b) e −1 2 (c) − e1 2 (d) − e −1 2
A

2. For f (x ) = tan −1 tan x , f (− ) is equal to


(a) 0 (b) 1 (c) –1 (d) none of these
NK

 3 
3. If f (x ) = sin x − cos x , then f   equals to
 4 
(a) –1 (b) 1 (c) 0 (d) does not exist
SA
EXERCISE – IV

SUBJECTIVE PROBLEMS

d 2y 24 y
1. If y 3 − y = 2 x , then prove that =− .
dx 2
(3y 2
−1) 3

 1  d 2y dy 1
Hence, show that  x 2 −  2 + x. = y.
 27  dx dx 9

RI
 x + y  f (x ) + f (y )
2. Let f  = and f (0) = a and f (0) = b . Find f (x ) (where y is independent
 2  2

A
of x), when f (x ) is differentiable.
 

d  3 d 2 y 

UH
3. If y 2 = p(x ) , a polynomial of degree n  3 , show that 2 y = p(x ) p (x ) .
dx  dx 2 
y x
4. If (a + bx )e = x , show that x 3 y  = (xy  − y )2 .
JA
x x x x sin x
5. If cos . cos 2 . cos 3 . cos 4 ........ = , then prove that
2 2 2 2 x
1 x 1 x 1 x 1
sec 2 + sec 2 + sec 2 + ...... = cosec 2 x − .
22 2 24 22 26 23 x2

dy x 1 − y
4
LP

6. If ( )
1 − x 4 + 1 − y 4 = k x 2 − y 2 , prove that =
dx y 1 − x 4
.

x2 −1 x +1 x − 2
A

7. Let  (x ) = 2 x 2 − 1 3 x 3 x − 3 . Prove, by using calculus, that (x ) is a first degree


x + 4 2x − 1 2x − 1
2
NK

polynomial.

8. Let f be twice differentiable function such that f (x ) = −f (x ) and f (x ) = g (x ) .

If h(x ) = f (x )2 + g (x )2 , where h(5) = 11, find h(10) .


SA

dy x
sec −1 (2 x ) + 2 x tanlog e (x + 2) = 0
3
9. Find at x = −1 when (sin y )sin 2 +
dx 2

10. Find the sum of sin x + 3 sin 3x + 5 sin 5x + .... + (2k − 1) sin(2k − 1)x .
ANSWERS

EXERCISE – I

SINGLE CHOICE CORRECT

1. (c) 2. (b) 3. (b) 4. (c) 5. (b)

6. (b) 7. (b) 8. (b) 9. (a) 10. (c)

RI
11. (b) 12. (c) 13. (b) 14. (c) 15. (c)

16. (d) 17. (d) 18. (b) 19. (d) 20. (b)

A
21. (d) 22. (a) 23. (d) 24. (b) 25. (b)

UH
EXERCISE – II

IIT-JEE – SINGLE CHOICE CORRECT


JA
1. (d) 2. (d) 3. (c) 4. (d) 5. (b)

6. (b) 7. (d) 8. (b) 9. (b) 10. (d)

11. (d) 12. (c) 13. (a) 14. (d) 15. (d)
LP

16. (c) 17. (a) 18. (a) 19. (a) 20. (a)

ONE OR MORE THAN ONE CHOICE CORRECT


A

1. (a), (b), (c) 2. (b), (c) 3. (a), (b), (c) 4. (a), (c), (d) 5. (a), (b)
NK

6. (a), (b) 7. (b), (d) 8. (a), (b), (c), (d) 9. (a), (b), (c) 10. (b), (c)
 

EXERCISE – III
SA

MATCH THE FOLLOWING

1. I-A, C; II-B, D; III-E; IV-E

REASONING TYPE

1. (a) 2. (c) 3. (a) 4. (b) 5. (d)


LINKED COMPREHENSION TYPE

1. (c) 2. (d) 3. (d)

 
EXERCISE – IV
 
SUBJECTIVE PROBLEMS

RI
2. 0

8. 11

A
3
9.

UH
 2 − 3

10.
1
(2k + 1) sin(2x − 1)x − (2k − 1) sin(2k + 1)x 
4 sin 2 x
JA
 
 
LP
A
NK
SA
INDEFINITE INTEGRATION

JEE (ADVANCED) SYLLABUS

Integration as the inverse process of differentiation, indefinite integrals of standard functions. Integration by
parts, integration by the methods of substitution and partial fractions.

RI
JEE (MAIN) SYLLABUS

A
UH
Integral as an anti - derivative. Fundamental integrals involving algebraic, trigonometric, exponential and
logarithmic functions. Integration by substitution, by parts and by partial fractions. Integration using
trigonometric identities

But just as much as it is easy to find the differential of a given quantity, so it is difficult to find the integral of a given differential. Moreover,
JA
sometimes we cannot say with certainty whether the integral of a given quantity can be found or not. Bernoulli, Johann

  If f & g are functions of x such that g (x) = f(x), then indefinite integration of f(x) with respect to x is
defined and denoted as  f(x) dx = g(x) + C, where C is called the constant of integration.

Standard Formula:
LP

n +1
( ax + b ) + C, n  −1
(i)  (ax + b) dx =
n
a ( n + 1)
dx 1
(ii)  = n |ax + b| + C
ax + b a
A

1 ax+b
(iii)  eax+b dx =
a
e +C

1 ap x + q
NK

(iv)  apx+q dx =
p na
+ C; a > 0

1
(v)  sin (ax + b) dx = − cos (ax + b) + C
a
1
(vi)  cos (ax + b) dx = sin (ax + b) + C
SA

a
1
(vii)  tan(ax + b) dx = n |sec (ax + b)| + C
a
1
(viii)  cot(ax + b) dx =
a
n |sin(ax + b)| + C

1
(ix)  sec² (ax + b) dx = tan(ax + b) + C
a
1
(x)  cosec²(ax + b) dx = – cot(ax + b)+ C
a
1
(xi)  sec (ax + b). tan (ax + b) dx =
a
sec (ax + b) + C
1
(xii)  cosec (ax + b). cot (ax + b) dx = –
a
cosec (ax + b) + C

 x
(xiii)  secx dx = n |secx + tanx| + C OR n tan 4 + 2 + C
 
x
(xiv)  cosec x dx = n |cosecx − cotx| + C OR n tan + C OR − n |cosecx + cotx| + C
2

dx x
(xv)  a −x2 2
= sin−1
a
+C

RI
dx 1 x
(xvi)  2
a +x 2
=
a
tan−1 + C
a
dx 1 x
(xvii)  x x −a 2 2
=
a
sec−1
a
+C

A
dx x
 = n x + x 2 + a2 sinh−1

UH
(xviii) +C OR +C
x +a2 2 a
dx x
(xix)  x −a2 2
= n x + x 2 − a2 + C OR cosh−1
a
+C

dx 1 a+x

JA
(xx) = ln +C
a2 − x 2 2a a−x
dx 1 x −a
(xxi)  x − a2
2
=
2a
ln
x +a
+C

x a2 x
(xxii)  a 2 − x 2 dx =
2
a2 − x 2 +
2
sin−1 + C
a
LP

x a2 x + x 2 + a2
(xxiii)  x 2 + a 2 dx =
2
x 2 + a2 +
2
n
a
+C

x a2 x + x 2 − a2
(xxiv)  x 2 − a2 dx = x 2 − a2 − n +C
A

2 2 a
eax
(xxv)  eax. sin bx dx = (a sin bx − b cos bx) + C
a + b2
2
NK

eax
(xxvi)  eax. cos bx dx =
a2 + b2
(a cos bx + b sin bx) + C

Theorems on integration
 C f(x).dx = C 
SA

(i) f(x).dx
(ii)  (f(x)  g(x)) dx =  f(x)dx   g(x) dx
g(ax + b)
(iii)  f(x)dx = g(x) +C1   f(ax + b)dx =
a
+ C2

 3x dx
6
Example # 1 Evaluate :

 3x dx =
6 3 7
Solution : x +C
7
 3 7 2 
  x + 5x − 4 + +
2
Example # 2 Evaluate :  dx
 x x
 3 7 2 
  x + 5x − 4 + +
2
Solution :  dx
 x x
7 2
x  5x  4dx  
3 2
= dx + dx – + dx + dx
x x
1
x x dx . – 4 .  1 . dx + 7 .   x −1/ 2 dx
3 2
= dx + 5 dx + 2.
x
 x1/ 2 

RI
x4 x3 x4 5
= + 5. – 4x + 7 n | x | + 2   +C= + x3 – 4x + 7 n | x | + 4 x + C
4 3  1/ 2  4 3

2
x log2 3
Example # 3 Evaluate : dx

A
3x
Solution : We have,  2x log2 3 dx =  3 x dx =
n3
+C

UH
4x + 5x
Example # 4 Evaluate :  7x
dx

4x + 5x  4x 5x    4 x  5 x  (4 / 7)x (5 / 7)x
Solution :  7x
dx=   x + x  dx =
7 7      +    dx =
  7   7   4
+
5
+C
n   n  
JA
7 7
cos7x – cos8x
Example # 5 Evaluate :  1 + 2cos5x
dx

5x 5x
2sin
cos7x – 2sin cos8x
cos7x – cos8x 1

Solution : We have,  dx = 2 2 dx
1 + 2cos5x 2 5x 5x
sin + 2cos5x sin
LP

2 2
 19x 9x   21x 11x 
 sin 2 – sin 2  –  sin 2 – sin 2 
1     dx
=
2  5x 15x 5x
sin + sin – sin
2 2 2
A

 19x 11x   21x 9x 


 sin + sin  –  sin + sin 
1  2 2   2 2 
=
2  15x
dx
NK

sin
2
15x 15x
2sin cos2x – 2sin cos3x
1 1 1
dx =  cos 2x – cos3x)dx =
2 
= 2 2 sin2x – sin3x + C
15x 2 3
sin
2
SA

3
x
Example # 6 Evaluate :  dx
(x + 1)2
x3 x 3 + 1– 1 x3 + 1 1
Solution :  (x + 1)2 dx =  (x + 1)2 dx =  (x + 1)2 dx –  (x + 1)2 dx
(x + 1)(x 2 – x + 1) 1 x 2 – x + 1) 1
=  (x + 1)2 dx –  (x + 1)2 dx =  (x + 1) dx –  (x + 1)2 dx
 3  1 x2 1
= 
 x – 2 + 
x + 1
dx –  (x + 1)2
dx =
2
– 2x + 3 n(x + 1) +
x +1
+C
1
Example # 7 : Evaluate :  4 + 9x 2
dx

Solution : We have
1 1 1 1 1
 4 + 9x 2 dx = dx 9 4 =
9  (2 / 3) 2
+ x2
dx
+ x2
9
1 1  x  1  3x 
= . tan–1   +C= tan–1   +C
9 (2 / 3)  2/3  6  2 

Example # 8 : Evaluate :  cos x cos 2x dx

RI
1 1 1  sin3x 
Solution :  cos x cos 2x dx =
2  2cos x cos 2x dx =
2  (cos3x + cos x) dx = 
2  3
+ sin x  + C

Self Practice Problems :

A
1
 tan  1 + sin x
2
(1) Evaluate : x dx (2) Evaluate : dx

Ans. (1) tanx – x + C (2) tanx – sec x + C

UH
Integration by Substitution
 

  If we substitute (x) = t in an integral then


(i) everywhere x will be replaced in terms of new variable t.
(ii) dx also gets converted in terms of dt.
JA
cos x + x sin x
Example # 9 : Evaluate :  x(x + cos x)
dx

Solution : We have,
cos x + x sin x
 x(x + cos x) dx
LP

(x + cos x) – x + x sin x 1 1– sin x 


  x – x + cos x 
1 1– sin x
=  x(x + cos x)
dx = dx = x dx – 
x + cos x
dx

1 1
= x dx –  x + cos x d (x + cos x) = n|x| – n |x + cosx| + C.
A

( nx)n
Example # 10 : Evaluate :  x
dx
NK

( nx)n 1 ( nx)n+1
Solution : We have  x
dx =  ( nx)n
x
dx =  ( nx)n d( nx) =
n +1
+C

(sin–1 x)3
Example # 11 : Evaluate :  1– x 2
dx
SA

(sin–1 x)3 (sin–1 x)4


Solution : We have ,  dx =  (sin–1 x)3 d(sin–1 x) = +C
1– x 2 4

x
Example # 12 : Evaluate : x 4
+ 2x 2 + 2
dx

Solution : We have,
x dt
 (x
x
= x 4
+ 2x + 2 2
dx = 2 2 2
) + 2x + 2
dx {Put x2 = t  x.dx =
2
}

1 1 1 1 1
 =
2  2
t + 2t + 2
dt =
2  2
(t + 1) + 1
dt =
2
tan–1 (t + 1) + C
1
= tan–1 (x2 + 1) + C
2

(f(x))n +1
Note: (i)  [ f(x)]n f (x) dx = +C
n +1
f  (x) (f(x))1−n
(ii)  n
dx =
1− n
+C,n1
 f (x)
dx
(iii)  ; nN Take xn common & put 1 + x−n = t.
x (xn + 1)
dx
 ; n  N, take xn common & put 1 + x−n = tn

RI
(iv) (n −1)
(
x 2 xn + 1 ) n

dx
(v)   ; take xn common as x and put 1 + x −n = t.

A
1/ n
n
x 1+ x( n
)

UH
Self Practice Problems :
sec 2 x sin( nx)
(3) Evaluate :  1 + tan x dx (4) Evaluate : 
x
dx

Ans. (3) n |1 + tan x| + C (4) –cos (n x) + C


JA
Integration by Parts : Product of two functions f(x) and g(x) can be integrated using formula :
 d 
 ( f(x) g(x) ) dx = f(x)  ( g(x) ) dx –   dx ( f(x))  ( g(x)) dx  dx
(i) when you find integral  g(x) dx then it will not contain arbitarary constant.

 g(x)
LP

(ii) dx should be taken as same at both places.

(iii) The choice of f(x) and g(x) can be decided by ILATE guideline.
the function will come later is taken an integral function (g(x)).
 → Inverse function
L → Logarithmic function
A

A → Algebraic function
T → Trigonometric function
E → Exponential function
NK

 sec
–1
Example # 13 : Evaluate : x dx
Solution : Put sec–1 x = t so that x = sec t and dx = sec t tan t dt
  sec –1 x dx =  t(sec t tan t)dt = t (sect) –  1.sec t dt
 

= t sec t – n |sect + tan t)| + C


SA

= t sec t – n | sec t + sec 2 t – 1 | + C = x (sec–1x) – n |x + x2 – 1 | + C

Example # 14 : Evaluate : x n(1 + x) dx


x2 1 x2
Solution : Let  =  x n(1 + x) dx =
2
.n (x + 1) –  x +1. 2 dx

x2 1 x2 x2 1 x2 − 1 + 1
=
2
n (x + 1) –
2  x +1 dx =
2
n (x + 1) –
2  x +1
dx

x2 1  x2 − 1 1  x2 1  1 
=
2
n (x + 1) –
2   +  dx =
x + 1 x + 1 2
n (x + 1) –
2   (x − 1) + x + 1 dx
x2 1  x2 
= n (x + 1) –  − x + n | x + 1| + C
2 2 2 
e
2x
Example # 15 : Evaluate : sin3x dx
Solution : Let  =  e sin3x dx
2x

 cos3x   cos3x  1 2x 2
= e2x  −  –  2e2x  −  dx = – e cos 3x + e
2x
cos3x dx
 3   3  3 3
1 2  2x sin3x sin3x 
=– e2x cos 3x + e −  2e2x dx 
3 3  3 3 

RI
1 2 2x 4
9 
= – e2x cos 3x + e sin 3x – e2x sin3x dx
3 9
1 2x 2 4 4 e2x
=– e cos 3x + e2x sin 3x –   + = (2 sin 3x – 3 cos 3x)
3 9 9 9 9

A
13 e2x e2x
 = (2 sin 3x – 3 cos 3x)   = (2 sin 3x – 3 cos 3x) + C
9 9 13
Note :

UH
(i)  ex [f(x) + f (x)] dx = ex. f(x) + C (ii)  [f(x) + xf (x)] dx = x f(x) + C

( x − 2x + 2) dx
2

e
x
Example # 16 : Evaluate : 2
( x + 2)
2
JA
 
( x − 2x + 2) dx =
2
x  1 ( −2x )  = ex + C
e   x2 + 2 2 2  x2 + 2
x
Solution : Given integral = 2
e +
( x + 2)
2  x +2 
( )
 

 1 − sin x 
LP

e
x
Example # 17 : Evaluate :  1 − cos x  dx
 
 x x
 1 − 2sin 2 cos 2 
Solution : Given integral =  e 
x
 dx
 x 
2sin2
A

 2 
 1 x x x
=  e x  cos ec − cot  dx = – ex cot
2
+C
2 2 2 2
NK

 1 
Example # 18 : Evaluate :   n ( nx) +  dx
( nx)2 
 1 
Solution : Let  =   n ( nx) + 2 
dx {put x = et  dx = et dt}
SA

 ( nx) 
 1   1 1 1
  =  e t  nt + 2  dt  e t  nt − + + 2  = dt
 t   t t t 
 1  1 
= et  nt −  + C = x  n ( nx) −  +C
 t   nx 

Self Practice Problems :


 x sin x x e
2 x
(5) Evaluate : dx (6) Evaluate : dx

Ans. (5) – x cosx + sin x + C (6) x2 ex – 2xex + 2ex + C


dx dx
Integration of type  ax 2 + bx + c , 
ax + bx + c 2
,  ax 2 + bx + c dx

Express ax2 + bx + c in the form of perfect square & then apply the standard results.

Example # 19 : Evaluate :  x 2 + 2x + 5 dx
Solution : We have,
 x 2 + 2x + 5 =  x 2 + 2x + 1 + 4 dx =  (x + 1)2 + 22
1 1
= (x + 1) (x + 1)2 + 22 + . (2)2 n |(x + 1) + (x + 1)2 + 22 | + C
2 2

RI
1
= (x + 1) x2 + 2x + 5 + 2 n |(x + 1) + x2 + 2x + 5 | + C
2
dx
Example # 20 : Evaluate :  2 − 6x − 9x 2
dx

A
dx 1  3x + 1 
 
1
Solution : dx = dx = sin–1   +C
2 − 6x − 9x 2 3 − ( 3x + 1)
2 3  3 

UH
Self Practice Problems :
8x − 11

1
(7) Evaluate :  2x 2
+ x −1
dx (8) Evaluate : dx
5 + 2x − x 2
1 2x − 1 x −1
5 + 2x − x2 – 3sin–1
Ans. (7) n +C
JA (8) –8 +C
3 2x + 2 6

Integration of type
px + q px + q
 ax 2 + bx + c dx,  ax 2 + bx + c
dx,  (px + q) ax 2 + bx + c dx
LP

Express px + q = A (differential co −efficient of denominator) + B.

2x – 3
Example # 21 : Evaluate : x 2
+ 3x – 18
dx
A

d
Solution : Let 2x – 3 =  (x2 + 3x – 18) + µ
dx
Then 2x – 3 =  (2x + 3) + 
NK

Comparing the coefficients of like power of x, we get.


2 = 2, and 3 + µ = –3   = 1 and µ = – 6
2x – 3 2x + 3 – 6 2x + 3 1
So,  2 dx =  2 dx =  2 dx – 6  2 dx
x + 3x – 18 x + 3x – 18 x + 3x – 18 x + 3x – 18
1 1
= n|x2 + 3x – 18| – 6  dx = n |x2 + 3x – 18| – 6  2 2
dx
9 9
SA

2
x + 3x + – – 18  3   9 
4 4 x + 2 –  2
   
3 9
x+ –
= n|x2 + 3x – 18| – 6.
1
n 2 2 + C = n|x2 + 3x – 18| – 2 n x – 3 + C
9 3 9
x+ +
3 x+6
2 
2 2 2
2x + 3
Example # 22 : Evaluate :  2
x + 4x + 1
dx

2x + 3 (2x + 4) − 1 2x + 4 1
Solution :  2
x + 4x + 1
dx =  2
x + 4x + 1
dx =  2
x + 4x + 1
dx –  2
x + 4x + 1
dx
dt 1
=  t
–  2
dx, where t = (x2 + 4x + 1) for Ist integral
(x + 2)2 − ( 3)
=2 t – n | (x + 2) + x2 + 4x + 1 | + C = 2 x2 + 4x + 1 – n | x + 2 + x2 + 4x + 1 | + C

Example # 23 : Evaluate : x 1 + x – x 2 dx
d
Solution : Let x =  . (1 + x – x 2 ) + µ.
dx
 x =  (1–2x) + µ

RI
Comparing the coefficients of like powers of x, we get
1 1 1 1
1 = – 2 and  + µ = 0   = – and µ =  x = – (1–2x) +
2 2 2 2
so, x 1 + x – x 2 dx

A
 1 1 1 1
=  – 2 (1– 2x) + 2  1 + x – x 2 dx = –
2  (1– 2x) 1 + x – x 2 dx +
2  1 + x – x 2 dx

UH
2
1 1  5  1
2

 
2
= – 1+ x – x d(1 + x – x ) + 2
  –  x –  dx,
2 2  2   2
 2 2 
1  5   1 5 
2
1 1 1
(1 + x –x2)3/2 +   x –  
1 x – 1/ 2 
=–  – x –  +   sin–1 +C
2   2   2  2  5 /2 
JA
3 2 2 2
 
1 1  1 2 5 –1  2x – 1  
= – (1 + x –x2)3/2 +  x –  1 + x – x + sin   + C
3 2  2 8  5 
Self Practice Problems :
3 − 4x 6x − 5
LP

(9) Evaluate :  2x 2
− 3x + 1
dx (10) Evaluate :  3x 2 − 5x + 1
dx

(11) Evaluate :  (x − 1) 1 + x + x 2 dx

Ans. (9) –n|2x2 – 3x + 1| + C (10) 2 3x2 − 5x + 1 + C


A

1 3 9
(11) (x2 + x + 1)3/2 – (2x + 1) 1 + x + x2 – log (2x +1 + 2 x2 + x + 1 ) + C
3 8 16
NK

Integration of Rational Algebraic Functions by using Partial Fractions:


PARTIAL FRACTIONS :
f(x)
If f(x) and g(x) are two polynomials, then defines a rational algebraic function of x.
g(x)
f(x)
SA

If degree of f(x) < degree of g(x), then is called a proper rational function.
g(x)
f(x)
If degree of f(x)  degree of g(x) then is called an improper rational function.
g(x)
f(x) f(x)
If is an improper rational function, we divide f(x) by g(x) so that the rational function is
g(x) g(x)
(x)
expressed in the form (x) + , where (x) and (x) are polynomials such that the degree of (x)
g(x)
f(x)
is less than that of g(x). Thus, is expressible as the sum of a polynomial and a proper rational
g(x)
function.
ax 2 + bx + c A B C
CASE-I = + +
(x –  )(x – )(x –  ) x –  x –  x – 
ax 2 + bx + c A B C
CASE-II = + +
(x –  )(x – )2
x –  x –  (x – )2
ax 2 + bx + c A Bx + C
CASE-III = +
(x – )(x 2 + 2 ) x –  x 2 +  2
where A, B, C can be evaluated by subsitution or by comparing coefficients.

1
Example # 24 : Resolve into partial fractions.
2x + 3x 2 − 3x − 2
3

RI
1 1
Solution : We have, =
2x + 3x − 3x − 2
3 2 (x − 1)(x + 2)(2x + 1)
1 A B C

A
Let = + + . Then,
2x + 3x − 3x − 2
3 2
x −1 x + 2 2x + 1
 1 = A(x + 2)(2x + 1) + B(x – 1) (2x + 1) + C (x – 1)(x + 2) ..(i)
1

UH
Putting x – 1 = 0 or x = 1 in (i), we get  A= ,
9
1
Putting x = –2 in (i), we obtain B=
9
1 4
Putting x = – in (i), we obtain C = –
JA
2 9
1 1 1 1 4
 = = + –
2x + 3x − 3x − 2
3 2 (x − 1)(x + 2)(2x + 1) 9(x − 1) 9(x + 2) 9(2x + 1)

x 3 − 6x 2 + 10x − 2
Example # 25 : Resolve into partial fractions.
LP

x 2 − 5x + 6
Solution : Here the given function is an improper rational function. On dividing we get
x 3 − 6x 2 + 10x − 2 ( − x + 4)
=x–1+ 2 ...........(i)
x − 5x + 6
2
(x − 5x + 6)
−x + 4
A

−x + 4
we have, =
x − 5x + 6
2
(x − 2)(x − 3)
−x + 4 A B
So, let = + , then
NK

(x − 2)(x − 3) x−2 x−3


– x + 4 = A(x – 3) + B(x – 2) ...........(ii)
Putting x – 3 = 0 or x = 3 in (ii), we get
1 = B(1)  B = 1.
Putting x – 2 = 0 or x = 2 in (ii), we get
2 = A (2 – 3)  A = – 2
SA

−x + 4 −2 1
 = +
(x − 2)(x − 3) x−2 x−3
x 3 − 6x 2 + 10x − 2 2 1
Hence =x–1– +
x 2 − 5x + 6 x−2 x−3

3x + 1
Example # 26 : Evaluate :  (x – 1) (x + 1) dx
3

3x + 1 A B C D
Solution : Let = + + 2
+ ......... (i)
3
(x – 1) (x + 1) x + 1 (x – 1) (x – 1) (x – 1)3
Multiplying both sides by (x + 1) and then putting x = – 1, we get
–2 1
A= 3
=
(–2) 4
Multiplying both sides by (x – 1)3 and then putting x = 1, we get
4
D= =2
2
From (i) , we get
3x + 1 = A (x – 1)3 + B (x – 1)2 (x + 1) + C (x – 1) (x + 1) + D (x + 1)
putting x = 0, we get
1 = –A + B – C + D
1 –3
 1=– +B–C+2 B–C=
4 4

RI
Putting x = 2, we get
7 = A +3B + 3C + 3D
1 3 1
 7 = + 3B + 3C + 6  3B + 3C =  B + C =
4 4 4

A
1 –3 1 1
Solving B + C = and B – C = , we get B = – , C =
4 4 4 2
Substituting the values of A, B, C and D in (i), we get

UH
3x + 1 1 1 1 1 2
 = . – + +
(x – 1)3 (x + 1) 4 x + 1 4(x – 1) 2(x – 1)2 (x – 1)3
3x + 1 1 1 1 1 1 1 1
  (x – 1) (x + 1) dx
3
=
4  x + 1 dx – 4  x – 1 dx + 2  (x – 1) 2
dx + 2  (x – 1) 3
dx
JA
1 1 1 1
= n |x + 1|– n|x – 1| – – +C
4 4 2(x – 1) (x – 1)2

1
Example # 27 : Evaluate :  sin x(2cos 2
x – 1)
dx

Solution : Putting cosx = t, we get


LP

1 1 dt 1
I= 
sin x(2cos2 x – 1)
dx =  sin x(2t 2
– 1)
–
sin x
= –
(1– t )(2t 2 – 1)
2
dt

 1 2  1 1
 I=–   1– t 2
+  dt = –
2t – 1 
2  1– t 2
dt – 2  2t 2
–1
dt
A

1 1+ t 2 2t – 1 1 1 + cos x 1 2 cos x – 1
=– n – n +C= – n – n +C
2 1– t 2 2t + 1 2 1– cos x 2 2 cos x + 1
2x − 3
NK

Example # 28 : Resolve into partial fractions.


(x − 1)(x 2 + 1)2
2x − 3 A Bx + C Dx + E
Solution : Let = + + . Then,
(x − 1)(x 2 + 1)2 x −1 x 2 + 1 (x 2 + 1)2
2x – 3 = A(x2 + 1)2 + (Bx + C) (x – 1) (x2 + 1) + (Dx + E) (x – 1) ......(i)
SA

Putting x = 1 in (i), we get – 1 = A (1 + 1) 2  A = –


Comparing coefficients of like powers of x on both side of (i), we have
A + B = 0, C – B = 0, 2A + B – C + D = 0, C + E – B – D = 2 and A – C – E = –3.
1
Putting A = – and solving these equations, we get
4
1 1 5 2x − 3 −1 x +1 x+5
B= = C, D = and E =  = + +
4 4 2 (x − 1)(x 2 + 1)2 4(x − 1) 4(x 2 + 1) 2(x 2 + 1)2

2x
Example # 29 : Resolve into partial fractions.
x3 − 1
2x 2x
Solution : We have, 3 =
x − 1 (x − 1)(x 2 + x + 1)
2x A Bx + C
So, let = + 2 .
(x − 1)(x + x + 1)
2
x –1 x + x +1
Then, 2x = A (x2 + x + 1) + (Bx + C) (x – 1) ...(i)
2
Putting x – 1 = 0 or, x = 1 in (i), we get 2 = 3 A  A =
3
2
Putting x = 0 in (i), we get A – C = 0  C = A =
3
2 4 2
Putting x = – 1 in (i), we get –2 = A + 2B – 2 C. + 2B – B=–  –2=
3 3 3
2x 2 1 (– 2 / 3) x + 2 / 3 2x 2 1 2 1− x

RI
= . + or 3 = +
x3 − 1 3 x − 1 x2 + x + 1 x −1 3 x −1 3 x2 + x + 1
Self Practice Problems :
1 dx
(12) (i) Evaluate :  (x + 2)(x + 3) dx (ii) Evaluate :  (x + 1)(x

A
2
+ 1)
x+2 1 1
Ans. (12) (i) n +C (ii) n |x + 1| – n (x2 + 1) + tan–1 (x) + C
x+3 2 2

UH
Integration of type
x2  1
 x4 + K x2 + 1
dx where K is any constant.

1
Divide Nr & Dr by x² & put x = t.
JA
x
x2 + 4
Example # 30 : Evaluate  x 4 + 16 dx
4
x2 + 4 1+ 2
1  4 dt
Solution :  x 4 + 16 dx =  2 x16 dx =  4
2
d x –  =
 x t 2
+ (2 2)2
,
x + 2
LP

x x – x  +8
 
4 1  t  1  x2 – 4 
where t = x– = tan–1   +C = tan–1   +C
x 2 2 2 2 2 2  2 2x 
x –1
 (x + 1)
A

Example # 31 : Evaluate : dx
x3 + x2 + x
x2 – 1  Multiplying the 
Solution :  =  (x + 1) dx Nr and Dr by (x + 1)
NK

2 3
x +x +x 2
 
(x 2 – 1)
 =  (x 2
+ 2x + 1) x 3 + x 2 + x
dx

1
1– 2
SA

 =  x dx [Dividing Nr and Dr by x2)


 1  1
 x + x + 2 x + x + 1
 
2t dt 1 1
 =  where, x + +1 = t2   = 2  2 dt   = 2tan–1 (t) + C
2
(t + 1) t 2 x t + 1
1
  = 2 tan–1 x+ +1+ C
x
Self Practice Problems :
x2 − 1
(13) Evaluate :  x − 7x 2 + 1
4
dx (14) Evaluate :  tan x dx
1
x+ −3
1 x 1  y  1 y− 2
Ans. (13) n +C (14) tan–1   + n +C
6 1 2  2 2 2 y+ 2
x+ +3
x
1
where y = tan x –
tan x
Integration of type
dx dx
( ax + b) px + q
OR  ( ax 2
+ bx + c ) px + q
.

RI
Put px + q = t2.
dx
Example # 32 : Evaluate :  ( x − 4) x+5

A
dx
Solution : Let  =  ( x − 4) x+5
{Put x + 5 = t2  dx = 2t dt}

UH
2dt t −3 x+5 −3
 (t
2 1
 = = n + C = ln +C
2
−9 ) 6 t+3 3 x+5 +3

dx
Example # 33 : Evaluate :  (x 2
+ 3x + 2 ) x+4
JA
dx
Solution : Let  =  (x 2
+ 3x + 2 ) x+4
2t dt
Putting x + 4 = t2, and dx = 2t dt, we get  =  {(t 2
− 4) + 3(t 2 − 4) + 2} t 2
2
LP

dt  1 1 
t
dt
 2 4
− 5t + 6
2
dt = 2  (t 2
)(
−2 t −32
dt = 2  2 − 2  dt
t − 3 t − 2
) 
1 t− 3 1 t− 2
= n – n + C where t = x+4
A

3 t+ 3 2 t+ 2
Integration of type
dx 1 dx 1
( , put ax + b = ; ( , put x =
NK

2
ax + b) px + qx + r t 2
ax + b) px + q 2 t

dx
Example # 34 : Evaluate ( x −1 ) x2 − x −1
dx
(
1 1
SA

Solution : Let  = {put x – 1 =  dx = – 2 dt }


x −1 ) x − x −1 2 t t
1
− dt
dt dt
 = 1 1  1 
2
t2 = − −t + t + 1
2
= − 2
 5   1 2
 t + 1 −  t + 1 − 1   −  t − 
t      2   2
 1
t− 
= –sin–1  2  + C = –sin–1  2t − 1  + C , where t = 1
 
 5   5  x −1
 
 2 
dx
Example # 35 : Evaluate  (1 + x 2
) 1 − x2
tdt
 (t
1 1
Solution : Put x =  dx = – dt  =– {put t2 – 1 = y2  tdt = ydy }
t t2 2
+ 1) t − 12

y dy 1  y 
 =–  2
(y + 2) y
=–
2
tan–1 
 2
 +C

1  1 − x2 
=– tan–1  + C
2  2x 

RI
Self Practice Problems :
dx dx
 

A
(15) Evaluate : (16) Evaluate : 2
(x + 2) x + 1 (x + 5x + 6) x + 1
dx dx
(17) Evaluate :  (x + 1) 1 + x − x
(18) Evaluate :  (2x + 1) 1 − x 2

UH
2 2

dx
(19) Evaluate :  (x + 2x + 2) x 2 + 2x − 4
2

 x +1
Ans. (15) 2 tan–1 ( x +1 + C ) (16) 2 tan–1 ( )
x + 1 – 2 tan–1 
 + C
2 
JA

3 1 
 −  1  1 − x2 
(17) sin–1  2 x +1 + C (18) – tan–1   +C
 5  3  3 x 
   
 2 
 x 2 + 2x − 4 − 6 (x + 1) 
LP

1
(19) – n   +C
2 6  x 2 + 2x − 4 + 6 (x + 1) 
 
Integration of type
x−α
 dx or  ( x − α) ( β − x ) dx; put x =  cos2  +  sin2 
A

β−x
x−α
 dx or  ( x − α) ( x − β ) dx; put x =  sec2  −  tan2 
x−β
NK

dx
 ( x − α) ( x − β ) ; put x −  = t2 or x −  = t2.

Self Practice Problems


SA

x−3 dx
(20) Evaluate :  x−4
dx (21) Evaluate :  [(x − 1)(2 − x)]3 / 2
dx
(22) Evaluate :  [(x + 2)8 (x − 1)6 ]1/ 7
 x −1 2−x 
Ans. (20) (x − 3)(x − 4) + n ( x−3 + x−4 ) +C (21) 2
 2−x
−  +C
x − 1 

1/ 7
7  x −1
(22)   +C
3  x +2
Integration of trigonometric functions
dx dx dx
(i)  a + b sin2 x
OR 
a + b cos 2 x
OR  a sin2 x + b sinx cos x + c cos 2 x
Multiply Nr & Dr by sec² x & put tan x = t.

dx dx dx
(ii)  a + b sinx
OR  a + b cosx
OR  a + b sinx + c cos x
x
Convert sines & cosines into their respective tangents of half the angles and then, put tan =t
2
a. cos x + b. sinx + c

RI
(iii) dx.
. cos x + m. sinx + n
Express Nr  A(Dr) + B (Dr) + C & proceed.
1 + sin x
Example # 36 : Evaluate:  dx

A
sin x(1 + cos x)
1 + sin x
Solution : Let I =  sin x(1 + cos x) dx

UH
2 tan x / 2 1– tan2 x / 2
Putting sinx = and , cos x = ,
1 + tan2 x / 2 1 + tan2 x / 2
we get
2 tan x / 2
1+
1 + tan2 x / 2 (1 + tan2 x / 2 + 2 tan x / 2)(1 + tan2 x / 2)
JA
I=   2 tan x / 2   1– tan2 x / 2 
dx = 
2 tan x / 2(1 + tan2 x / 2 + 1– tan2 x / 2)
dx
  1 + 
 1 + tan2 x / 2   1 + tan2 x / 2 
(1 + tan x / 2)2 sec 2 x / 2 1 + t 2 + 2t) x
=  4 tan x / 2
dx =

2t
dt , where t = tan
2
1 1  1  t2  1  tan2 x / 2 
LP

=
2 t  + t + 2 

dt = 
2 
n | t | +
2
+ 2t 

+ C = 
2 
n | tan x / 2 | +
2
+ 2 tan x / 2  + C


dx
Example # 37 : Evaluate :  sin x + 3 cos x
A

Solution : Let 1 = rcos and 3 = r sin  r = (1)2 + ( 3 )2 = 2


NK

tan = 3   = /3
dx 1 dx 1 dx
  sin x + 3 cos x
=
r  sin x cos  + cos x sin  =
r  sin(x + )
1 1 x  1  x 
SA

r 
= cos ec(x + )dx = n tan  +  + C = n tan  +  + C
r  2 2  2 2 6

3cos x + 2
Example # 38 : Evaluate :  sin x + 2cos x + 3 dx

Solution : We have,
3cos x + 2
=  sin x + 2cos x + 3 dx

Let 3 cos x + 2 =  (sin x + 2 cos x + 3) +  (cos x – 2 sin x) + 


Comparing the coefficients of sin x, cos x and constant term on both sides, we get
6 3 8
 – 2 = 0, 2 +  = 3, 3 +  = 2  = ,= and  = –
5 5 5
(sin x + 2cos x + 3) + (cos x − 2sin x) + 
 =  sin x + 2cos x + 3
dx

cos x − 2sin x 1
 =  dx +   sin x + 2cos x + 3 dx +   sin x + 2cos x + 3 dx

  =  x +  log | sin x + 2 cos x + 3 | +   1


1
where 1 =  sin x + 2cos x + 3 dx

RI
2 tan x / 2 1 − tan2 x / 2
Putting, sin x = , cos x = , we get
1 + tan2 x / 2 1 + tan2 x / 2

1 1 + tan2 x / 2
 2 tan x / 2 2(1 − tan2 x / 2)  2 tan x / 2 + 2 − 2 tan2 x / 2 + 3(1 + tan2 x / 2) dx

A
1 = dx =
+ +3
1 + tan2 x / 2 1 + tan2 x / 2

UH
sec 2 x / 2
=  tan 2
x / 2 + 2 tan x / 2 + 5
dx

x 1 x x
Putting tan = t and sec2 = dt or sec2
dx = 2 dt, we get
2 2 2 2
 x 
JA
2dt dt 2  t + 1  tan 2 + 1 
1 =  2 =2  = tan 
–1
 = tan 
–1

t + 2t + 5 (t + 1)2 + 22 2  2   2 
 
 x 
 tan 2 + 1 
Hence,  = x +  log | sin x + 2 cos x + 3 | +  tan–1   +C
 2 
LP

 
6 3 8
where  = ,  = and  = –
5 5 5
A

dx
Example # 39 : Evaluate : 
1 + 3cos2 x
Solution : Multiply Nr. & Dr. of given integral by sec 2x, we get
 tan x 
NK

sec 2 x dx 1
=  = tan–1   +C
 2 
2
tan x + 4 2

Self Practice Problems :


4 sin x + 5 cos x
(23) Evaluate :  5 sin x + 4 cos x dx
SA

40 9
Ans. (23) x+ log |5sinx + 4cosx| + C
41 41

Integration of type  sinm x. cosn x dx


Case - 
If m and n are even natural number then converts higher power into higher angles.
Case - 
If at least one of m or n is odd natural number then if m is odd put cosx = t and vice-versa.
Case - 
When m + n is a negative even integer then put tan x = t.
 cos
5
Example # 40 : Evaluate : x sin4 xdx

Let  =  cos 
5
Solution : x sin4 xdx put sinx = t cosx dx = dt

 =  (1 − t  (t − 2t 2 + 1) t4 dt =  (t − 2t 6 + t 4 ) dt
2 2 4 8
) . t4 . dt =

t9 2t 7 t5
= – + + C , where t = sinx
9 7 5
 sec
4/3
Example # 41 : Evaluate : x cos ec 8 / 3 xdx
Solution : We have ,

RI
1
=  sec  cos  cos
4/3
x cos ec 8 / 3 xdx = 4/3
dx = –4 / 3
x sin–8 / 3 xdx
x sin8 / 3 x
divide Nr and Dr by cos4x
sec 4 x (1 + tan2 x) 1 + tan2 x 1+ t2

A
=  tan 8/3
x
dx = 
tan x8/3
sec 2 xdx =  8/3
tan x
d(tan x) =  8 / 3 dt
t
where t = tan x

–3 –3
=  (t –8 / 3 + t –2 / 3 )dt = t–5/3 + 3t1/3 + C = tan–5/3 x + 3 tan1/3 x + C

UH
5 5

 sin
4
Example # 42 : Evaluate : x cos2 x dx

 sin  4 sin  sin


4 1 1
Solution : x cos2 x dx = 2
x cos2 x.2sin2 xdx = 2
2x (1 – cos2x)dx
JA
8 8

 sin
1 1 1 1
 sin  (1 − cos 4x)
2
= 2xdx – 2
2x cos 2x dx = dx − (sin2x)3
8 8 16 48
x sin 4x 1
= − – (sin2x)3 + C
16 64 48
LP

Reduction formula of  tann x dx ,  cotn x dx ,  sec n x dx ,  cos ec n x dx

n =  tan  tan x tann − 2 x  (sec x − 1) tann – 2x dx


n 2 2
1. x dx = dx =
A

tann −1 x
 n =  sec x tann − 2 x dx – n – 2  n = – n – 2 , n  2
2

n −1
NK

2. n =  cot n x dx =  cot 2 x . cot n − 2 x dx =  (cos ec


2
x − 1)cot n − 2 x dx

cot n −1 x
 cos ec x cot x dx – n – 2
n−2
 n = 2
 n = – – n – 2 , n  2
n −1
SA

n =  sec  sec x sec n − 2 x dx


n 2
3. x dx =

 n = tanx secn – 2x –  (tan x)(n − 2) secn – 3 x. secx tanx dx.


 n = tanx secn – 2 x – (n – 2) (sec2 x – 1) secn – 2x dx
tan x sec n − 2 x n − 2
 (n – 1) n = tanx secn – 2x + (n – 2) n – 2  n = + 
n −1 n −1 n –2

n =  cos ec  cos ec
n 2
4. x dx = x cosecn – 2 x dx

 n = – cotx cosecn – 2x +  (cot x)(n − 2) (– cosec n–3


x cosec x cot x) dx
  cot x cos ec x
2 n−2
– cotx cosecn – 2x – (n – 2) dx

 n = – cotx cosecn – 2 x – (n – 2)  (cos ec x − 1) 2


cosecn – 2 x dx
 (n – 1) n = – cotx cosecn – 2 x + (n – 2) n – 2
cot x cos ec n − 2 x n−2
 n = + 
– ( n − 1) n −1 n –2

 cos
n
Example # 43 : Obtain the reduction formula for xdx

 cos

RI
Solution : Let n = n
xdx


n = cos x (cosx)n–1 dx

A
 

UH
 (n – 1)(cos x)
n−2
n = (sinx)(cosx)n–1 – ( − sin x)sin xdx

 (cos x)
n−2
n =(sinx)(cosx)n–1 + (n – 1) (1 − cos2 x)dx

 (cos x) 
n−2
JA
n = (sinx)(cosx)n–1 + (n – 1) dx − (n − 1) (cos x)n dx

n = (sinx)(cosx)n–1 + (n – 1) n–2 – (n – 1) n

n + (n – 1)n = (sinx)(cosx)n–1 + (n – 1)n–2

(sin x)(cos x)n−1 (n − 1)


n = + n − 2 , n  2
LP

n n

Self Practice Problems :


A

dx dx
(24) Deduce the reduction formula for n =  (1 + x 4 )n
and Hence evaluate 2 =  (1 + x 4 )2
.

(25) If m,n =  (sin x)m (cos x)n dx then prove that


NK

(sin x)m +1(cos x)n−1 n −1


m,n = + .
m+n m + n m,n–2
x 4n − 5
Ans. (24) n = + 
SA

4 n −1
4(n − 1)(1 + x ) 4(n − 1) n–1

  1  1 
x  1 −1
x− x 1 x+ x
− 2 
2 = +  tan  − n   + C
4 (1 + x 4 ) 2 2 1
  2  4 2  x + + 2  

    x 
 Marked questions are recommended for Revision.

PART - I : SUBJECTIVE QUESTIONS


Section (A) : Integration using Standard Integral :
A-1. Integrate with respect to x :
(i) (2x + 3)5 (ii) sin 2x (iii) sec2 (4x + 5)

RI
(iv) sec (3x + 2) (v) tan (2x + 1) (vi) 23x+4
1
(vii) (viii) e4x + 5
2x + 1

A
A-2. Integrate with respect to x :
(i) sin2x (ii) cos3x (iii) sin 2x cos 3x

UH
x 3x 1
(iv) 4sin x cos cos (v)
2 2 x+3 − x+2

Section (B) : Integration using Substitution :


 
JA
B-1. Integrate with respect to x :
x ex + 1
(i) x sin x2 (ii) (iii) sec2 x tan x (iv)
x2 + 1 ex + x
1 − sin x e2x cos 2x + x + 1 sec x
(v) (vi) (vii) (viii)
x + cos x e2x − 2 x 2 + sin 2x + 2x n (sec x + tan x)
LP

2
x  x 1 1
(ix) (x) e + x  (xi) (ex + 1)2 ex (xii)
x+2  e  (
x x +1 5
)
1 x2 − 8
(xiii) (xiv)
1
x4
A

x 5 (1 + x ) 5 5

d(x 2 + 1)

NK

B-2. Find the value of .


(x 2 + 2)

B-3. Evaluate the following :


 x 
 x cos x − sin x   x + 1 − ln(x + 1) 
    dx
SA

(i)   dx (ii)
 x sin x   x(ln(x + 1)) 
 
Section (C) : Integration by parts :
 
C-1. Integrate with respect to x :
(i) x n x (ii) x sin2x (iii) x tan–1 x (iv) n x
2 x tan−1 x
2
(v) sec3x (vi) 2x3 ex (vii) sin–1 x (viii)
1 + x2
(ix) ex sin x (x) ex (sec2x + tan x)

C-2. Find the antiderivative of f(x) = ln (ln x) + (ln x) –2 whose graph passes through (e, e).
Section (D) : Algebraic integral :
D-1. Integrate with respect to x :
1 1 1
(i) 2
(ii) 2
(iii) 2
x +4 x +5 x + 2x + 5
2x + 1 x3 −1 1
(iv) (v) (vi)
2
x + 3x + 4 x3 + x x2 − 4
(vii) x2 + 4 (viii) x2 + 2x + 5 (ix) (x – 1) 1 − x − x2
(x)  x5 a3 + x3

D-2. Integrate with respect to x :


1 1

RI
(i) (ii)
(x + 1)(x + 2) (x 2 + 1)(x + 3)
3x + 2 1
(iii) (iv)
(x + 1)2 (x + 2) (x + 1)(x + 2)(x + 3)

A
D-3. Integrate with respect to x :
1 1 + x2 1 − x2

UH
(i) (ii) (iii)
x4 + x2 + 1 1+ x4 1 − x2 + x4

D-4. Integrate with respect to x :


1 1
(i) (ii)
(x + 1) x + 2 (x − 4) x + 1
2
JA
1 1
(iii) (iv)

(x + 1) x + 2 (x + 1) x  + 2
2

D-5. Evaluate the following :


 1+ x   x − 1  x 1+ x 
(i)   x  dx (ii)   x + 1  dx (iii)    dx
1 − x 
LP

   

Section (E) : Integration of trigonometric functions :


E-1. Integrate with respect to x :
A

1 1 2 sin x + 2cos x
(i) (ii) (iii)
2 + cos x 2 − cos x 3 cos x + 2 sin x
1 1 cos ec 2 x.sin x
NK

(iv) (v) (vi)


1 + sin x + cos x 2 + sin2 x (sin x − cos x)
4
sin x
(vii)
cos2 x
SA

E-2. Evaluate the following


 sin x + cos x   cos x − sin x 
(i)   9 + 16 sin2x  dx (ii)   8 − sin2x 
 dx

cos3 x  −9 −5

E-3. If  11
sin x
dx = – 2 

A tan 2
x + B tan 2
x  + C, then find A and B.

Section (F) : Reduction formulae

1 x 2n − 3
F-1. If n =  (x 2 2 n
+a )
dx then prove that In = 2 n −1
2a (n − 1)(x + a )
2 2
+
2(n − 1)a2
n–1

2an 2xn (a − x)3 / 2


F-2. If n = x (a – x)1/2dx then prove that n = n–1 –
n

2n + 3 2n + 3

PART - II : ONLY ONE OPTION CORRECT TYPE

RI
* In each question C is arbitrary constant
Section (A) : Integration using Standard Integral :

A
A-1. Integrate with respect to x : x +1
(x + 1) 3/2
3(x + 1)3 / 2 (x + 1)3 / 2 2(x + 1)3 / 2
  (A) +C (B) +C (C) +C (D) +C
2 2 3 3

UH
1
A-2 Integrate with respect to x :
2x + 1
3/2 1
(A) 2x + 1 + C (B) ( 2x + 1) +C (C) – 2x + 1 + C (D) 3/2
+C
( 2x + 1)
JA
1 x 
A-3. If  1 + sin x
dx = tan  + a  + C, then
 2 
  5 
(A) a = – , C  R (B) a = , C  R (C) a = ,CR (D) a = ,CR
4 4 4 3
LP

1
A-4. If  (sin2x − cos 2x) dx =
2
sin (2x – a) + C, then

5 5  
(A) a = ,CR (B) a = – ,CR (C) a = ,CR (D) a = ,CR
4 4 4 2
A

cos 2x
A-5. The value of
cos x 
dx is equal to
NK

(A) 2 sin x – n |sec x + tan x| + C (B) 2 sin x – n |sec x – tan x| + C


(C) 2 sin x + n |sec x + tan x| + C (D) sin x – n |sec x – tan x| + C

cos 4x + 1
A-6. If  dx = A cos 4x + B; where A & B are constants, then
SA

cot x − tan x
(A) A = − 1/4 & B may have any value (B) A = − 1/8 & B may have any value
(C) A = − 1/2 & B = − 1/4 (D) A = B = 1/2

Section (B) : Integration using Substitution :


x
a
B-1. The value of  x
dx is equal to

a x
2a x x x
(A) +C (B) +C (C) 2a . n a + C (D) 2a +C
x n a
x

5
55 x
B-2. The value of . 55 . 5 x dx is equal to
x 5x 5x
55 55
x
55 55
(A) +C (B) 5 (n 5) + C
3
(C) +C (D) +C
( n 5)3 ( n 5)3 ( n 5)2

tan x
B-3. The value of  sin x cos x
dx is equal to

tan x
(A) 2 tan x + C (B) 2 cot x + C (C) +C (D) tan x + C
2

RI
2x
B-4. If  1 − 4x
dx = K sin–1 (2x) + C, then the value of K is equal to

1 1 1
 

A
(A) n 2 (B) n 2 (C) (D)
2 2 n 2

dx
If y = 

UH
B-5. 3/2
and y = 0 when x = 0, then value of y when x = 1, is:
(1 + x ) 2

2 1
(A) (B) 2 (C) 3 2 (D)
3 2
JA
B-6. The value of  tan3 2 x sec 2 x dx is equal to :
1 1 1 1
  (A) sec3 2 x − sec 2 x + C (B) − sec3 2 x − sec 2 x + C
3 2 6 2
1 1 1 1
(C) sec3 2 x − sec 2 x + C (D) sec3 2 x + sec 2 x + C
6 2 3 2
LP

1/ 2
x ( )
13 / 2
B-7. If . 1 + x5 / 2 dx = P (1 + x5/2)7/2 + Q (1 + x5/2)5/2 + R (1 + x5/2)3/2 + C, then P,Q and R are
4 8 4 4 8 4
  (A) P = ,Q=– ,R= (B) P =
,Q= ,R=
35 25 15 35 25 15
A

4 8 4 4 8 4
(C) P = – ,Q=– ,R= (D) P = ,Q=– ,R=–
35 25 15 35 25 15
NK

1 − x7
B-8. The value of  x (1 + x ) dx is equal to 7

2 2
  (A) n |x| + n |1 + x7| + C (B) n |x| − n |1 − x7| + C
7 7
2 2
SA

(C) n |x| − n |1 + x7| + C (D) n |x| + n |1 − x7| + C


7 7

Section (C) : Integration by parts :


   
C-1. The value of  (x − 1) e − x dx is equal to
  (A) –xex + C (B) xex + C (C) – xe–x + C (D) xe–x + C

 1 + x + x2 
e
−1
tan x
C-2. The value of  2  dx is equal to
 1+ x 
−1 −1 1 −1 −1
(A) x etan x
+C (B) x2 etan x
+C (C) etan x
+C (D) x ecot x
+C
x

C-3. The value of  [f(x)g(x) − f (x)g(x)] dx is equal to


f(x)
(A) +C (B) f(x) g(x) – f(x) g(x) + C
g(x)
(C) f(x) g(x) – f(x) g(x) + C (D) f(x) g(x) + f(x) g(x) + C

x nx
C-4.  3/2
dx equals
(x 2
−1 )

RI
nx nx
  (A) arc sec x − +C (B) sec −1 x + +C
x −1
2
x2 − 1
nx nx
(C) cos−1 x − (D) sec x −

A
+C +C
x2 − 1 x2 − 1

C-5. The value of  (x en sin x − cos x) dx is equal to:

UH
(A) x cos x + C (B) sin x − x cos x + C (C) − en x cos x + C (D) sin x + x cos x + C

Section (D) : Algebraic integral :

dx
JA
D-1. The value of x 2
+ x +1
is equal to

3  2x + 1  2  2x + 1 
(A) tan–1   +C (B) tan–1   +C
2  3  3  3 
1  2x + 1  2  2x − 1 
(C) tan–1   +C (D) tan–1   +C
3  3  3  3 
LP

1
D-2. The value of  x 2 (x 4 + 1)3 / 4
dx is equal to

1/ 4 1/ 4 1/ 4
 1  1  1
 
A

(A)  1 + 4  +C (B) (x + 1)
4 1/4
+C (C)  1 − 4  +C (D) –  1 + 4  +C
 x   x   x 

dx
x
NK

D-3. The value of is equal to


1 − x3
1 1 − x3 − 1 1 1 − x2 + 1
(A) n +C (B) n +C
3 1 − x3 + 1 3 1 − x2 − 1
1 1 1
SA

(C) n +C (D) n |1 – x3 | + C
3 1− x 3 3

ex − 1
D-4. The value of  ex + 1
dx is equal to

(A) n (e + x
)
e2x − 1 – sec–1 (ex) + C (B) n (e + x
)
e2x − 1 + sec–1 (ex) + C

(C) n ( e − x
e2x − 1) – sec –1
(ex) + C (D) n ( e + x
e2x − 1) – sin –1
(ex) + C

dx A B x
D-5. If x 4
+x 3
= 2
x
+
x
+ n
x +1
+ C, then
1 1 1 1 1
(A) A = ,B=1 (B) A = 1, B = – (C) A = – ,B=1 (D) A = – ,B=
2 2 2 2 2

Section (E) : Integration of trigonometric functions :


 
cos 2x
E-1. The value of  (sin x + cos x) 2
dx is equal to

−1
  (A) +C (B) n (sin x + cos x) + C
sin x + cos x
(C) n (sin x – cos x) + C (D) n (sin x + cos x)2 + C

RI
E-2 The value of  [1 + tan x . tan(x +  )] dx is equal to

A
sin x sin x
(A) cos  . n +C (B) tan  . n +C
sin(x +  ) sin(x +  )
sec(x +  ) cos(x +  )
(C) cot  . n +C (D) cot  . n +C

UH
sec x cos x

E-3 The value of  sec x − 1 dx is equal to


 x x 1  x x 1
(A) 2 n  cos + cos2 −  + C (B) n  cos + cos2 −  + C
 2 2   2 2 
JA
 2  2
 x x 1  x x 1
(C) – 2 n  cos + cos2 −  + C (D) – 2 n  sin + cos2 −  + C
 2 2 2   2 2 2 
 

dx
E-4. The value of  cos is equal to
LP

3
x sin2x
 1 5/2   1 5/2 
(A) 2  cos x + tan x  + C (B) 2  tan x + tan x  + C
 5   5 
 1 5/2   1 
(C) 2  tan x − tan x  + C (D) 2  cos x − tan5 / 2 x  + C
A

 5   5 

sin2 x
E-5. Antiderivative of w.r.t. x is :
NK

1 + sin2 x
2 1  tan x 
(A) x −
2
arctan ( )
2 tan x + C (B) x −
2
arctan 
 2 
 +C

 tan x 
(C) x − 2 arctan ( )
2 tan x + C (D) x − 2 arctan   +C
SA

 2 

1
E-6. Integrate
1 − cot x
1 1 1 1
(A) log | sin x − cos x | + x + C (B) log | sin x + cos x | + x + C
2 2 2 2
1 1 1 1
(C) log | sin x + cos x | − x + C (D) log | sin x − cos x | − x + C
2 2 2 2
dx
E-7. I =  is equal to
sin x + sec x
1 3 + sinx - cosx
(A) log + tan−1 (sinx + cosx) + C
2 3 3 − (sinx - cosx)
1 3 + sinx - cosx
(B) log + tan−1 (sinx - cosx) + C
2 3 3 − (sinx - cosx)
1 3 + sinx + cosx
(C) log + tan−1 (sinx + cosx) + C
2 3 3 − (sinx - cosx)
1 3 + sinx - cosx
(D) log + tan−1 (sinx + cosx) + C
2 3 3 − (sinx + cosx)

RI
Section (F) : Reduction formulae
 
−e x

A
ex 1
F-1. If n = x n
dx and n = +
k1xn −1 k 2 − 1
n–1 , then (k2 – k1) is equal to :

(A) 0 (B) 1 (C) 2 (D) 3

UH
 u2 u9 
If n =  cot
dx and 0 + 1 + 2 (2 + .....+ 8) + 9 + 10 = A  u +
n
F-2. x + ..... +  + C, where u = cotx
 2 9
and C is an arbitrary constant, then
(A) A = 2 (B) A = – 1 (C) A = 1 (D) A is dependent on x
JA
PART - III : MATCH THE COLUMN
 
1. Column – I Column – II

LP

x + sin x
(A) If F(x) =  1 + cos x dx and F(0) = 0, then the value of F(/2) is (p)
2

 x  
Let F(x) =  esin
−1

1 −
x
(B)  dx and F(0) = 1, (q)
1 − x2 
A

 3

k 3 e / 6
If F(1/2) = , then the value of k is

NK

dx 
(C) Let F(x) =  (x 2
+ 1) (x 2 + 9)
and F(0) = 0, (r)
4
5
if F( 3 ) = k, then the value of k is
36
SA

tan x
(D) Let F(x) =  sin x cos x dx and F(0) = 0 (s) 

2k
if F(/4) = , then the value of k is

dx
2. If I = a+b
cos x
, where a, b > 0 and a + b = u, a – b = v, then match the following column

Column – I Column – II
x
1 u+ v tan
(A) v=0 (p) = n 2 +C
uv x
u – v tan
2
2  v x 
(B) v>0 (q) = tan–1  tan  +C
uv  u 2 

x
u + –v tan
1 2 +C
(C) v<0 (r) = n
–u v x
u – –v tan
2

RI
2 x
(s) tan +C
u 2

A
UH
JA
LP
A
NK
SA
 Marked questions are recommended for Revision.

PART - I : ONLY ONE OPTION CORRECT TYPE


* In each question C is arbitrary constant

1
1. Value of  sin (x − a) cos (x − b)
dx is equal to

RI
1 sin (x − a) 1 cos (x − b)
(A) n +C (B) n +C
cos (a − b) cos (x − b) cos (a − b) sin (x − a)
1 sin (x − a) 1 cos (x − a)
(C) n +C (D) n +C

A
sin (a − b) cos (x − b) sin (a + b) sin (x − b)

UH
2.  tan x. tan 2x. tan 3x dx =
1 1
(A) − n | cos x | − n | sec 2x | + n | sec 3x | + C
2 3
1 1
(B) − n | sec x | − n | sec 2x | + n | sec 3x | + C
2 3
JA
(C) n | cos x | + n | cos2x | + n | cos 3x | + C
1 1
(D) n | sec x | + n | sec 2x | + | sec 3x | + C
2 3

3. The value of  (sin x. cos x. cos 2x. cos 4x. cos 8x. cos 16 x) dx is equal to
LP

sin 16 x cos 32 x cos 32 x cos 32 x


(A) +C (B) − +C (C) +C (D) − +C
1024 1024 1096 1096

a2 − x 2
4.  x
a2 + x 2
dx =
A

1 2  x2  1 1  x2 
(A) a cos− 1  2  + a4 + x 4 + C (B) sin− 1  2  + a4 + x 4 + C
2 a  2 2 a 
NK

1 2  x2  1 1  x2  1
(C) a sin− 1  2  + a4 − x 4 + C (D) cos− 1  2  + a4 − x 4 + C
2  
a 2 2 a  2

x −1 1
5. The value of  . dx is equal to
SA

x + 1 x2
1 x2 −1 x2 −1 1
(A) sin −1 + +C (B) + cos −1 + C
x x x x
x2 −1 x2 −1
(C) sec −1 x − +C (D) tan −1 x2 +1 − +C
x x

n |x|
6. The value of  x 1 + n |x|
dx equals :

2 2
  (A) 1 + n x (nx − 2) + C (B) 1 + n x (nx + 2) + C
3 3
1
(C) 1+ n x (nx − 2) + C (D) 2 1 + n x (3 nx − 2) + C
3

1
7. The value of  [(x − 1) (x + 2) ]3 5 1/ 4
dx is equal to

1/ 4 1/ 4 1/ 4 1/ 4
4  x −1 4  x +2 1  x −1 1  x + 1
(A)   +C (B)   +C (C)   +C (D)   +C
3  x +2 3  x −1 3  x +2 3  x − 1

1− x
8. The value of  dx is equal to

RI
1+ x
(A) x 1− x − 2 1 − x + cos −1 ( x) + C (B) x 1 − x + 2 1 − x + cos −1 ( x) + C
(C) x 1 − x − 2 1 − x − cos −1 ( x) + C (D) x 1− x + 2 1 − x − cos −1 ( x) + C

A
x
9.  sin−1 dx is equal to

UH
a+x
x x
(A) (a + x) arc tan − ax + C (B) (a + x) arc tan + ax + C
a a
x x
(C) (a – x) arc tan − ax + C (D) (a + x) arc cot − ax + C
a
JA a

x
e
10. The value of  x
( x + x ) dx is equal to :
(A) 2e x
[ x − x +1 ] + C (B) 2e x
[x − 2 x + 1 ] + C
LP

(C) 2e x
[x − x + 1 ] + C (D) 2e x
(x + x + 1 ) + C

2 A
 x (x
nx
11. If I = )( n x)3 dx = Axnx(nx)2 – B xnx + C, then is equal to :
B
(A) 1 (B) –1 (C) 2 (D) –2
A

12. The value of  etan  (sec  – sin ) d is equal to


(A) − etan  sin  + C (B) etan  sin  + C (C) etan  sec  + C (D) etan  cos  + C
NK

  x 
13. The value of  ln(1 + sin x) + x tan  4 − 2  dx is equal to:

(A) x n (1 + sinx) + C (B) n (1 + sin x) + C


(C) – x n (1 + sin x) + C (D) n (1 – sin x) + C
SA

(
n x + 1+ x 2 ) dx equals:
14. The value of  x. 1+ x 2

x x
(A) 1 + x2 n x + 1+ x 2 ( ) −x+C (B)
2
(
. n2 x + 1+ x 2 )− 1 + x2
+C

x x
(C)
2
(
. n2 x + 1+ x 2 )+ 1 + x2
+C (D) (
1 + x2 n x + 1+ x 2 ) + x+C
x tan−1 x
15. If  1+ x 2
dx = 1 + x2 f(x) + A n |x + x2 + 1 | + C, then
(A) f(x) = tan–1 x, A = –1 (B) f(x) = tan–1 x, A = 1
(C) f(x) = 2 tan–1 x , A = –1 (D) f(x) = 2 tan–1 x, A = 1

x + x +1
16.  x+2
dx is equal to

(A) (x+ 1) – 2 x + 1 + 2 nx + 2− 2tan −1 x +1 +C


(B) (x+ 1) + 2 x + 2 − 2 nx + 2− 2tan −1 x + 2+C

RI
(C) (x+ 1) + 2 x + 1 − 2 nx + 2− 2tan −1 x + 1+ C
(D) (x+ 1) + 2 x + 2 − 2 nx + 1+ 2tan −1 x + 2+C

A
1 − cos x
17. The value of  cos  − cos x
dx, where 0 <  < x < , is equal to

  x   x
  (A) 2 n  cos − cos  + C 2 n  cos − cos  + C

UH
(B)
 2 2   2 2 
  x  cos x 
(C) 2 2 n  cos − cos  + C (D) −2sin −1  2 +C
 2 2  cos  
 2
JA
sin x + sin3 x
18. If  = cos 2x dx = A cosx + B n |f(x)| + C, then
1 – 1 2 cos x – 1 1 –3 2 cos x – 1
(A) A = , B = , f(x) = (B) A = – , B = , f(x) =
4 2 2 cos x + 1 2 4 2 2 cos x + 1
1 3 2 cos x + 1 1 – 3 2 cos x – 1
(C) A = – , B = , f(x) = (D) A = , B = , f(x) =
LP

2 2 2 cos x – 1 2 4 2 2 cos x + 1

1
19. The value of  cos6 x + sin6 x
d x is equal to

(B) − tan −1 (tan x + cot x) + C


A

(A) tan −1 (tan x + cot x) + C


(C) tan −1 (tan x − cot x) + C (D) − tan −1 (tan x − cot x) + C

20. Consider the following statements :


NK

S1 : The antiderivative of every even function is an odd function.


3 x4 − 1 x
S2 : Primitive of 2
w.r.t. x is + C.
(x 4
+ x +1 ) x4 + x + 1

1 −2
S3 :  dx = + C.
SA

sin3 x cos x tan x


 a+x a−x 
S4 : The value of   −  dx is equal to – 2 a2 − x2 + C
 a−x a + x 

  State, in order, whether S1, S2, S3, S4 are true or false
(A) FFTT (B) TTTT (C) FFFF (D) TFTF

1 2k
If n =  (sin x + cos x) dx , snd n = n–2 then k =
n
21. (sinx + cosx)n–1 (sinx – cosx) +
n n
(A) (n+ 1) (B) (n – 1) (C) (2n+ 1) (A) (2n– 1)
PART - II : SINGLE AND DOUBLE VALUE INTEGER TYPE
* In each question C is arbitrary constant

2 sin x − sin2x
1. If f(x) =  x3
dx, where x  0, then Limit f  (x) has the value
x→0

1  1 
2. If  sin4 x cos4 x dx = 
128 
a x − sin 4 x + . sin 8 x  + C then value of 'a' equal to :
8 

RI
3x + 2
3. Let f(x) be the primitive of w.r. to x. If f(10) = 60 then twice of sum of digits of the value of f(13)
x−9
is.

A
3/2
4 + x2 a + x2 (
. x2 − b ) ( )
4. If  x6
dx =
120 x5
+ C then a + b equals to :

UH
x d  x3 / 2 
5. If  a3 − x 3
dx =
b
sin −1
 3 / 2  + C, (where b & d are coprime integer) then b + d equals to.
a 

x dx
6. If  =k 1 + 1 + x 2 + C then k equals to :
JA
3
1 + x2 + (1+ x ) 2

x cos3 x − sin x  


7. If  esin x . 2
cos x
dx = esinx f(x) + C such that f(0) = –1 then − f   is equal to :
3 3
LP

1 + 2cos x 
8. Let g(x) =  (cos x + 2) 2
dx and g(0) = 0 then value of 32 g   is.
2
9. If f(x) = x − 1 ; g(x) = ex and  fog(x)dx = Afog(x) + Btan –1
(fog(x)) + C then A3 + B2 equals
A

2 sin 2  − cos 
10. If  6 − cos2  − 4 sin 
d = p n sin2  − 4 sin  + 5 + q tan −1(sin− r) + C then p + q + r equal to :
NK

(x − 1)2 1  x2 − 1 b  2 x2 + 1
11. If  x + x2 + 1
4
dx = tan −1   − tan −1 
 3 
 + C then a2 + b2 equals to :
a x 3  a

1 + x cos x x 2 e2 sin x
12. If  x (1 − x dx = k n + C then k is equal to :
SA

2
e2 sin x ) 1 − x 2 e2 sin x

x4 + 1 B
13. If  2
dx = A n x +
1 + x2
+ C, then A + B equals to :
(
x x +1 2
)
1 1 1 a
14. If  1 − sin x
4
dx =
a b
tan −1 ( a tan x + ) b
tan x + C then
b
is equal to :

cos3 x + cos5 x

q
15. If dx = p sin x – – r tan–1 (sinx) + C then p + 2q + r is equal to :
sin2 x + sin4 x sin x
dx
16. If  = a cot x + b tan3 x + C, where C is an arbitrary constant of integration, then the
sin x cos5 x
3

values of a2 + 9b equals to :

PART - III : ONE OR MORE THAN ONE OPTIONS CORRECT TYPE


* In each question C is arbitrary constant

1. The value of  2mx. 3nx dx (when m, n  N) is equal to :

RI
e( )
(mn ) . 2x . 3 x
m n 2 + n n3 x
2mx + 3nx 2mx . 3nx
(A) + C (B) + C (C) +C (D) +C
m n2 + n n3 m n2 + n n3 n ( 2m .3n ) m n2 + n n3

A
 1− x 
2. If f   = x and g(x) = f(x) dx then
 1+ x  
(A) g(x) is continuous in domain

UH
(B) g(x) is discontinuous at two points in its domain
(C) lim g'(x) = –1
x →

x2  1+ x 
(D)  g(x)dx = −
2
+ (2x + 1) n 
 e 
+C
JA
1
 tan
5
3. If = xdx = A tan4x – tan2x +Bn|secx| + C then
2
1 1
(A) A = (B) A = (C) B = 1 (D) B = – 1
4 2
LP

 {1 + 2 tan x(tan x + sec x)}


1/ 2
4. The value of dx is equal to
(A) n |sec x (sec x – tan x)| + C (B) n |cosec x (sec x + tan x)| + C
(C) n |sec x (sec x + tan x)| + C (D) –n |cosx(sec x – tan x)|+ C

x −1
n 
A


 x + 1
5. The value of  x2 − 1
dx is equal to

1 x −1 1 x −1 1 x +1 1 2 x +1
NK

(A) n2 +C (B) n2 +C (C) n2 +C (D) n +C


2 x +1 4 x +1 2 x −1 4 x −1

n (tan x)
6. The value of  sin x cos x
dx is equal to
SA

1 2 1 2
(A) n (cot x) + C (B) n (sec x) + C
2 2
1 1
(C) n2 (sin x sec x) + C (D) n2 (cos x cosec x) + C
2 2

cos3 x
7. The value of  sin2 x + sin x
d x is equal to :

(A) n sin x + sin x + C (B) n sin x − sin x + C


(C) − n cosec x − sin x + C (D) − n sin x + sin x + C
( x −1) dx f(x)
8. If x 2
2 x − 2x + 1
2
=
g(x)
+ C, where f(x) is a quadratic expression and g(x) is a monic linear

expression.

(A) f(x) = 2x2 – 2x + 1 (B) g(x) = x + 1


(C) g(x) = x (D) f(x) = 2x2 – 2x

9. If  e3x cos 4x dx = e3x (A sin 4x + B cos 4x) + C then :


(A) 4A = 3B (B) 2A = 3B (C) 3A = 4B (D) 4A + 3B = 1

RI
sin−1 x − cos−1 x
10. I=  sin −1
x + cos−1 x
dx equals to

2 2

A
(A) –x + (2x – 1)sin–1 x + x − x2 + C
 
4x 2 2
(B) x – cos–1 x – sin–1 x + x 1− x + C
  

UH
2 2
(C) –x + (2x + 1)cos–1 x + x 1− x + C
 
4x
(D) x – sin–1 x + C

JA
x2 − x + 1
11. If  32
ex dx = exf(x) + C then
(1 + x2 )
(A) f(x) is a an even function (B) f(x) is a bounded function
(C) Range of f(x) is (0, 1] (D) f(x) has two points of exterma.
LP

4e x + 6e− x
12. If  9e
x
− 4e− x
dx = Ax + B n |9e2x – 4| + C, then

(A) A + 18B = 16 (B) 18B – A = 19


(C) A –18B = 17 (D) A + 18B = 32
A

x 2 + cos2 x
13. The value of  1 + x2
cosec2 x dx is equal to:
NK

(A) cot x − cot −1 x + C (B) C − cot x + cot −1 x


cos ecx 1
(C) − tan −1 x − +C (D) – cot x + C
sec x tan−1 x

dx  1
SA

14. The value of  x−x


;  x   is equal to
2 2

(A) 2 sin −1 x+C (B) sin −1 (2x − 1) + C


(C) C − 2 cos −1 (2x − 1) (D) cos −1 2 x − x2 + C

x3 − 1
15.  x3 + x dx is equal to
(A) x – n |x| + n (x2 + 1) – tan–1x + C
1
(B) x – n |x| + n (x2 + 1) – tan–1x + C
2
1
(C) x + n |x| + n (x2 + 1) + tan–1x + C
2
x2 + 1
(D) x + n + cot–1x + C
x2

16. The value of 2  sin x . cos ec4x dx is equal to

1 1 + 2 sin x 1 1 + sin x 1 1 + 2 sin x 1 1 + sin x


(A) n – n + C (B) n – n +C
2 2 1 − 2 sin x 4 1 − sin x 2 2 1 − 2 sin x 2 cos x
1 1 − 2 sin x 1 1 + sin x 1 1 − 2 sin x 1 1 − sin x
+ C (D) −

RI
(C) n – n n + n +C
2 2 1 + 2 sin x 4 1 − sin x 2 2 1 + 2 sin x 4 1 + sin x

3 cot 3x − cot x
17. If  dx = p f(x) + q g(x) + C, then which of the following may be correct?

A
tan x − 3 tan3x
1 3 − tan x
(A) p = 1; q = ; f(x) = x; g(x) = n
3 3 + tan x

UH
1 3 − tan x
(B) p = 1; q = − ; f(x) = x; g(x) = n
3 3 + tan x
2 3 + tan x
(C) p = 1; q = − ; f(x) = x; g(x) = n
3 − tan x
JA
3
1 3 + tan x
(D) p = 1; q = − ; f(x) = x; g(x) = n
3 3 − tan x

dx  x
18. If 
5 + 4cos x
= P tan−1  m tan  + C then :
 2 
LP

(A) P = 2/3 (B) m = 1/3 (C) P = 1/3 (D) m = 2/3

sin2x
19. The value of  sin x + cos4 x
4
dx is equal to:
A

(A) cot −1 (cot2 x) + C (B) − cot −1 (tan2 x) + C


(C) tan −1 (tan2 x) + C (D) − tan −1 (cos 2 x) + C
NK

PART - IV : COMPREHENSION
Comprehension # 1 (Q.No. 1 to 3 )
SA

Let n, m =  sinn x cosm x.dx . Then we can relate n, m with each of the following
(i) In – 2, m (ii) n + 2, m (iii) n, m – 2
(iv) n, m + 2 (v) n – 2, m + 2 (vi) n + 2, m – 2
Suppose we want to establish a relation between n, m and n, m – 2, then we set
P(x) = sinn + 1x cosm – 1x ...........(1)
In n,m and n, m – 2 the exponent of cosx is m and m – 2 respectively, the minimum of the two is m – 2,
adding 1 to the minimum we get m – 2 + 1 = m – 1. Now choose the exponent
m – 1 of cosx in P(x). Similarly choose the exponent of sin x for P (x)
Now differentiating both sides of (1), we get
P(x) = (n + 1) sinnx cosmx – (m – 1) sinn + 2x cosm – 2x
= (n + 1) sinnx cosmx – (m – 1) sinnx (1 – cos2x) cosm – 2x
= (n + 1) sinnx cosmx – (m – 1) sinnx cos m – 2 x + (m – 1) sinnx cosmx
= (n + m) sinnx cosmx – (m – 1) sinnx cosm – 2x
Now integrating both sides, we get
sinn + 1x cosm – 1x = (n + m) n, m – (m – 1) n, m – 2.
Similarly we can establish the other relations.
 
1. The relation between 4, 2 and 2, 2 is
1 1
(A) 4, 2 = (– sin3x cos3x + 32, 2) (B) 4, 2 = (sin3x cos3x + 32, 2)
6 6
1 1
(C) 4, 2 = (sin3x cos3x – 32, 2) (D) 4, 2 = (– sin3x cos3x + 22, 2)
6 6

RI
2. The relation between 4, 2 and 6, 2 is
1 1
(A) 4, 2 = (sin5x cos3x + 86, 2) (B) 4, 2 = (– sin5x cos3x + 86, 2)
5 5

A
1 1
(C) 4, 2 = (sin5x cos3x – 86, 2) (D) 4, 2 = (sin5x cos3x + 86, 2)
5 5

UH
3. The relation between 4, 2 and 4, 4 is
1 1
(A) 4, 2 = (sin5x cos3x + 8 4, 4) (B) 4, 2 = (– sin5x cos3x + 8 4, 4)
3 3
1 1
(C) 4, 2 = (sin5x cos3x – 8 4, 4) (D) 4, 2 = (sin5x cos3x + 6 4, 4)
3 3
JA
Comprehension # 2 (Q. No. 4 to 6)

It is known that
 sin x cos x 
 + if 0x
 cos x sin x 2
cot x = 
LP

tan x + ,
 − sin x − cos x 3
 − cos x + − sin x if x
 2

d 1    3 
( tan x − cot x ) ( )
tan x + cot x (tan x + cot x) ,  x   0,    , 2 
A

=
dx 2  2   
d 1    3 
( ) ( )
tan x − cot x (tan x + cot x) ,  x   0,   ,
2  2 
and tan x + cot x = .
dx 2  
NK

   3 
Value of integral  =  ( tan x + cot x ) dx , where x   0 ,   ,
2  2 
4. is
 
 tan x − cot x   tan x + cot x 
(A) 2 tan–1   + C (B) 2 tan–1   + C
SA

 2   2 
 tan x − cot x   tan x + cot x 
(C) – 2 tan–1   + C (D) – 2 tan–1 
  + C
 2   2 

 
5. Value of the integral  = ( tan x + cot x ) dx , where x   0 ,
 2 
, is

(A) 2 sin–1 (cos x – sin x) + C (B) 2 sin–1 (sin x – cos x) + C


(C) 2 sin–1 (sin x + cos x) + C (D) – 2 sin–1 (sin x + cos x) + C
 3 
6. Value of the integral  = ( tan x + cot x ) dx , where x    ,
 2 
, is

(A) 2 sin–1 (cos x – sin x) + C (B) 2 sin–1 (sin x – cos x) + C


(C) 2 sin–1 (sin x + cos x) + C (D) – 2 sin–1 (sin x + cos x) + C

 Marked questions are recommended for Revision.

RI
* Marked Questions may have more than one correct option.
PART - I : JEE (ADVANCED) / IIT-JEE PROBLEMS (PREVIOUS YEARS)

A
x3 + 3 x + 2
1. Integrate,  2
dx. [IIT-JEE 1999, Part-2, (7, 0), 120]
(x 2
)
+ 1 (x + 1)

UH
2. Let f (x) =  ex (x − 1) (x − 2) d x then f decreases in the interval :[IIT-JEE 2000, Scr, (1, 0), 35]
(A) (− , 2) (B) (− 2, − 1) (C) (1, 2) (D) (2, + )

 2x + 2 
 sin −1   d x.
3. Evaluate,
JA [IIT-JEE 2001, Main, (5, 0), 100]
 4 x 2 + 8 x + 13 
 

4. For any natural number m, evaluate,


1/ m
 (x ) (2 x )
2m
3m
+ x 2m + xm + 3 xm + 6 d x, x > 0. [IIT-JEE 2002, Main, (5, 0), 60]
LP

x2 − 1
5. x 3
2x 4 − 2x 2 + 1
dx is equal to [IIT-JEE 2006, (3, –1), 184]

2x 4 − 2x 2 + 1 2x 4 − 2x 2 + 1
(A) +C (B) +C
x2 x3
A

2x 4 − 2x 2 + 1 2x 4 − 2x 2 + 1
(C) +C (D) +C
x 2x 2
NK

x
x
n−2
6. Let f(x) = for n  2 and g(x) = (f f .... f ) (x). Then g(x) dx equals
(1 + x n )1/ n f occurs n times

[IIT-JEE 2007, Paper-2, (3, – 1), 81]


1 1
1 1− 1 1−
(A)
n(n − 1)
(1 + nxn ) n + K (B)
(n − 1)
(1 + nxn ) n + K
SA

1 1
1 1+ 1 1+
(C)
n(n + 1)
(1 + nxn ) n + K (D)
(n + 1)
(1 + nxn ) n + K

7. Let F(x) be an indefinite integral of sin 2 x. [IIT-JEE 2007, Paper-1, (3, – 1), 81]
STATEMENT-1 : The function F(x) satisfies F(x + ) = F(x) for all real x.
because
STATEMENT-2 : sin2(x + ) = sin2x for all real x.
(A) Statement-1 is True, Statement-2 is True ; Statement-2 is a correct explanation for Statement-1
(B)Statement-1 is True, Statement-2 is True; Statement-2 is NOT a correct explanation for Statement-1
(C) Statement-1 is True, Statement-2 is False
(D) Statement-1 is False, Statement-2 is True
ex e− x
8. Let  = e 4x
+ e2x + 1
dx, J = e −4x
+ e−2x + 1
dx . Then, for an arbitrary constant C, the value of J –  is

equal to : [IIT-JEE 2008, Paper-2, (3, – 1), 81]


1 e4x − e2x + 1 1 e + ex + 1
2x
(A) n 4x +C (B) n +C
2 e + e2x + 1 2 e2x − e x + 1
1 e2x − e x + 1 1 e4x + e2x + 1
(C) n +C (D) n +C
2 e2x + e x + 1 2 e4x − e2x + 1

sec 2 x
9. The integral  (sec x + tan x)9 / 2 dx equals (for some arbitrary constant K)

RI
–1 1 1 
  (A) 11/ 2 
− (sec x + tan x)2  + K [IIT-JEE 2012, Paper-1, (3, –1), 70]
(sec x + tan x) 11 7 

A
1 1 1 
(B) 11/ 2 
− (sec x + tan x)2  + K
(sec x + tan x) 11 7 
–1 1 1 2
 + (sec x + tan x)  + K

UH
(C)
(sec x + tan x)11/ 2 11 7 
1 1 1 2
(D)  + (sec x + tan x)  + K
11 7 
11/ 2
(sec x + tan x)
JA
PART - II : JEE (MAIN) / AIEEE PROBLEMS (PREVIOUS YEARS)
5 tan x
1. If the integral  tan x − 2 dx = x + a n |sin x – 2 cos x| + k, then a is equal to :

[AIEEE-2012, (4, –1)/120]


(1) – 1 (2) – 2 (3) 1 (4) 2
LP

 f(x) dx = (x)  x f(x


5 3
2. If , then ) dx is equal to [AIEEE - 2013, (4, –1),360]
1 3 1 3
(1) x (x 3 ) –  x 2 (x 3 ) dx  + C (2) x (x 3 ) – 3  x 3 (x 3 ) dx + C
3 3
A

1 3 1 3
(3) x (x 3 ) –  x 2 (x 3 ) dx + C (4) x (x 3 ) –  x 3 (x 3 ) dx  + C
3 3

 1  x+
1

 1 + x − x e x dx is equal to :


NK

3. The integral [JEE(Main) 2014, (4, – 1), 120]


1 1 1 1
x+ x+ x+ x+
x x x x
(1) (x + 1) e +c (2) –x e +c (3) (x – 1) e +c (4) x e +c

dx
4. The integral x equals [JEE(Main) 2015, (4, – 1), 120]
SA

3/4
2
(x 4
)
+1
1/ 4 1/ 4
 x 4 + 1  x 4 + 1
(1)  4  +c (2) (x4 + 1)1/4 + c (3) –(x4 + 1)1/4 + c (4) –  4  +c
 x   x 

2x12 + 5x9
5. The integral  (x 5
+ x3 + 1)3
dx is equal to [JEE(Main) 2016, (4, –1), 120]

x10 x5 –x10 –x5


(1) +C (2) +C (3) +C (4) +C
2(x5 + x3 + 1)2 2(x5 + x3 + 1)2 2(x5 + x3 + 1)2 (x5 + x3 + 1)2
where C is an arbitrary constant
Let n =  tan x dx , (n > 1). If 4 + 6 = a tan5x + bx5 + C, where C is a constant of integration, then the
n
6.
ordered pair (a, b) is equal to [JEE(Main) 2017, (4, –1), 120]
 1  1  1   1 
(1)  – ,1 (2)  , 0  (3)  , – 1 (4)  – , 0 
 5  5   5   5 

sin2 x cos2 x
7. The integral  (sin5 x + cos3 x sin2 x + sin3 x cos2 x + cos5 x)2 dx is equal to :
[JEE(Main) 2018, (4, –1), 120]
1 –1 1 –1
(1) +C (2) +C (3) +C (4) +C
1 + cot 3 x 1 + cot 3 x 3(1 + tan3 x) 3(1 + tan3 x)

RI
(where C is a constant of integration)
1
(sinn  − sin )n cos 
8. Let n  2 be a natural number and 0 <  <  / 2. Then  d is equal to :

A
sinn +1 
(where C is a constant of integration) [JEE(Main) 2019, Online (10-01-19),P-1 (4, – 1), 120]
n+1 n+1
n  1  n n  1  n
1− 1−

UH
(1) 2 +C (2) 2 +C
n − 1  sinn+1   n − 1  sinn−1  
n+1 n +1
n  1  n n  1  n
(3) 2 1− +C (4) 2 1 +  +C
n + 1  sinn−1   n − 1 sinn −1  
9. The integral  cos (log x )dx is equal to : (where C is a constant of integration
e
JA
              [JEE(Main) 2019, Online (12-01-19),P-1 (4, – 1), 120]
x
(1) x cos(loge x) − sin ( loge x )  + C (2) sin(loge x) − cos (loge x )  + C
2
x
(3) x[cos(loge x) + sin(log e x)] + C (4) cos(loge x) + sin (loge x )  + C
2
LP
A

EXERCISE - 1

PART - I
NK

Section (A) :

(2x + 3)6 cos 2x tan(4x + 5)


A-1. (i) +C (ii) – +C (iii) +C
12 2 4
SA

1 1
(iv) n |sec (3x + 2) + tan (3x + 2) | + C (v) n |sec(2x + 1)| + C
3 2
23x + 4 1 e 4x + 5
(vi) +C (vii) n |2x + 1| + C (viii) +C
3 n 2 2 4

x 1 sin3x 3
A-2. (i) – sin2x + C (ii) + sin x + C
2 4 12 4
1 1 1 1
(iii) – cos 5x + cosx + C (iv) cos x − cos 2x − cos 3x + C
10 2 2 3
2
(v)
3
((x + 3)3 / 2 + (x + 2)3 / 2 ) + C
Section (B) :
 
1 1
B-1. (i) – cosx2 + C (ii) n |x2 + 1| + C
2 2
1 sec 2 x
(iii) (tanx)2 + C or +C (iv) n |ex + x| + C
2 2
1
(v) n |x + cosx| + C (vi) n |e2x – 2| + C
2
1
(vii) n |x2 + sin2x + 2x| + C (viii) n | n(secx + tanx) | + C

RI
2
2 1 2x
(ix) (x + 2)3/2 – 4(x + 2)1/2 + C (x) (e – e– 2x) + 2x + C
3 2
1 3x 1 1

A
(xi) e + e2x + ex + C (xii) – n 1 + 5 + C
3 5 x
3/2
1  1
4/5
(x 2
− 8 )
(xiii) – 1 + 5  +C (xiv) +C

UH
4  x  24 x3

 sin x   ln(x + 1) 
B-2. 2 (x 2 + 2) + C B-3. (i) ln  +C (ii) ln  +C
 x   x 
Section (C) :
JA
 

x2 x2 x2 x 1
C-1. (i) n x – +C (ii) – sin2x – cos2x + C
2 4 4 4 8
x2 x 1
(iii) tan–1x – + tan–1x + C (iv) x (nx – 1) + C
2 2 2
sec x tan x 1 2
n |sec x + tanx | + C (x2 – 1) ex + C
LP

(v) + (vi)
2 2
x 1– x 1 1
(vii) x sin–1 x+ – sin–1 x+C (viii) x tan–1x – n(1 + x2) –
2 2 2
(tan−1 x)2
+C
A

2
ex
(ix) (sinx – cosx) + C (x) ex tanx + C
2
NK

 1 
C-2. y = x  n( nx) − + 2e
 nx 
Section (D) :

1 x 1 x 1  ( x + 1) 
SA

D-1. (i) tan–1 + C (ii) tan–1 +C (iii) tan–1   +C


2 2 5 5 2  2 
4 2x + 3 1+ x 2
(iv) n |x2 + 3x + 4| – tan–1 +C (v) x − arctan x + n +C
7 7 x
x
(vi) n |x + x2 − 4 | + C (vii) x2 + 4 + 2 n |x + x2 + 4 | + C
2
x +1
(viii) x2 + 2x + 5 + 2 n x + 1 + x 2 + 2x + 5 + C
2
(1 − x − x 2 )3 / 2 3 15  2x + 1 
(ix) – – (2x + 1) 1 − x − x2 – sin–1  + C
3 8 16  5 
2 2a3
(x) (a3 + x3)5/2 – (a3 + x3)3/2 + C
15 9
x +1 1 1 3
D-2. (i) n +C (ii) n |x + 3| – n |x2 + 1| + tan–1 x + C
x+2 10 20 10
1 1 1
(iii) 4n|x + 1| + – 4n|x + 2| + C (iv) n |x + 1| – n |x + 2| + n |x + 3| + C
(x + 1) 2 2
1
1  x2 − 1 x+ −1 1  x2 − 1 
1 x
D-3. (i) tan–1   – n +C (ii) tan–1   + C
2 3  3x  4 1 2  2 x
x + +1
x

RI
1
1 – 3 x+
(iii) – n x +C
2 3 1
x+ + 3

A
x

x + 2 −1 1 t− 3 1
D-4. (i) n +C (ii) n – tan–1(t) + C, where t = x +1

UH
x + 2 +1 4 3 t+ 3 2
2
1  1  1 2 1
(iii) – n  t −  +  t −  + + C, where t =
3  3  3 9 x +1

x2 + 2
JA
(iv) – tan–1 +C
x2
1  1
D-5. (i) ln  x +  + x 2 + x + x2 + x + C
2  2 
1 −1 x
(ii) x 2 − 1 – ln x + x 2 − 1 + C (iii) sin x − 1 − x2 − 1 − x2 + C
2 2
LP

Section (E) :

2  tan x / 2  2  x
E-1. (i) tan–1   +C (ii) tan–1  3 tan  + C
3  3  3  2 
A

10 2 x
(iii) x– n |3cosx + 2sinx| + C (iv) n 1 + tan +C
13 13 2
 3 tan x 
NK

1
(v) tan–1   +C (vi) n | 1 − cot x | + C
 2 
6 
1 3x
(vii) tan x + sin 2x − + C
4 2
SA

1  4(sin x − cos x) + 5   sin x + cos x 


E-2. (i) n +C (ii) sin–1   +C
40  4(sin x + cos x) − 5   3 

1 1
E-3. A= ,B=
9 5
PART - II

Section (A) :
A-1. (D) A-2 (A) A-3. (A) A-4. (B) A-5. (A) A-6. (B)

Section (B) :
B-1. (B) B-2. (C) B-3. (A) B-4. (D) B-5. (D) B-6. (C)
B-7. (A) B-8. (C)

Section (C) :
   
C-1. (C) C-2. (A) C-3. (C) C-4. (A) C-5. (C)

Section (D) :
D-1. (B) D-2. (D) D-3. (A) D-4. (A) D-5. (C)

RI
Section (E) :
 

A
E-1. (B) E-2 (C) E-3. (C) E-4. (B) E-5. (A) E-6. (A) E-7. (A)

Section (F) :

UH
F-1. (B) F-2. (B)

PART - III
1. (A) → (p), (B) → (p), (C) → (r), (D) → (s) 2. (A) → (s) ; (B) → (q) ; (C) → (r)
JA
EXERCISE - 2

PART - I
1. (A) 2. (B) 3. (B) 4. (C) 5. (C) 6. (A) 7. (A)
LP

8. (A) 9. (A) 10. (C) 11. (A) 12. (D) 13. (A) 14. (A)

15. (A) 16. (C) 17. (D) 18. (D) 19. (C) 20. (A) 21. (B)

PART - II
A

1. 1 2. a=3 3. 12 4. 10 5. 5 6. 2 7. 2

8. 16 9. 12 10. 11 11. 13 12. 1 13. 2 14. 1


NK

15. 11 16. 10

PART - III
1. (BC) 2. (AC) 3. (AC) 4. (CD) 5. (BD) 6. (ACD) 7. (BC)
SA

8. (AC) 9. (CD) 10. (AB) 11. (ABC) 12. (AB) 13. (BC) 14. (ABD)

15. (BD) 16. (ABD) 17. (AD) 18. (AB) 19. (ABCD)

PART - IV

1. (A) 2. (A) 3. (B) 4. (A) 5. (B) 6. (A)

EXERCISE - 3

PART - I
3 1 1 x
1. tan-1 x – n (1 + x) + n (1 + x2) + +C 2. (C)
2 2 4 1 + x2

m +1
 2x + 2  3 (2x 3m + 3x 2m + 6xm ) m
3. (x + 1)tan 
–1
 – n (4x2 + 8x + 13) + C 4. +C
 3  4 6(m + 1)

5. (D) 6. (A) 7. (D) 8. (C) 9. (C)

RI
PART - II
1. (4) 2. (3) 3. (4) 4. (4) 5. (1) 6. (2) 7. (4)

A
8. (2) 9. (4)

UH
JA
LP
A
NK
SA
sin8 x − cos8 x
1. Evaluate :  1 − 2sin2 x cos2 x dx
cos5x + cos 4x
2. Evaluate :  1 − 2cos3x
dx

3. Evaluate :  x + x 2 + 2 dx

RI
dx
4. Evaluate :  x 2 + 2x − 3
(x 3 2
+ 3 x + 3x + 1 )

A
(cos 2x − 3)
5. Evaluate :  cos 4
x 4 − cot 2 x
dx

UH
 x 2 + 1 { n(x 2 + 1) − 2 nx} 
6. Evaluate :   dx
 x4 
JA
x
7. Evaluate :  (7x − 10 − x 2 )3 / 2
dx

x cos  + 1 f(x)
8. If  + 2x cos  + 1
3/2
dx = + C then find f(x) and g(x).
(x 2
) g(x)
LP

9. Evaluate : cos x. ex. x2 dx

x  x − x +2
3
10. Evaluate :   (x 2 + 1)2  dx
e
A

x2
11. Evaluate :  (x sin x + cos x)2 dx
NK

 sin 4x.e
tan2 x
12. Evaluate : dx

13. Evaluate :  tan −1


x. n (1 + x 2 ) dx.  
SA

1 + n xn −1 − x 2n
14. Evaluate :  ex dx  
(1 − x ) n
1 − x 2n

15. Evaluate :  cos 2 x n (1 + tan x) dx

dx
16. Evaluate :  ( a + b cos x )2 , (a > b)
 
2 − x − x2
17. Evaluate :  x2
dx

(5 x2 − 12) d x
18. Integrate:  (x2 − 6 x + 13)2

3x 2 + 2x
19. If  x6 + 2x5 + x 4 + 2x3 + 2x 2 + 5 dx = F(x), then find the value of [F(1) – F(0)], where [.] represents
greatest integer function.

RI
n (1 + sin2 x) dx
20. Evaluate :  cos2 x

A
1 + cos  cos x
21. Evaluate :  cos  + cos x
dx

UH
a + b sin x
22. Evaluate :  (b + a sin x ) 2
dx

dx
23. Evaluate :  (x − )
JA
(x −  ) (x − )
 
(cos 2x)1/ 2
24. Evaluate  sin x
dx

x
sin3
LP


25. Evaluate 2 dx
x
cos cos3 x + cos2 x + cos x
2

x2  x2 − 3  B x2 − 3 x + 3
A

26. If  x 4 + 3x 2 + 9 dx = A tan –1

 3x 
+
3
n
x2 + 3 x + 3
+ c, then find the value of 12(A + B).
NK

3cos x + 2
27. Evaluate  sin x + 2cos x + 3 dx
28. Evaluate  3
tan xdx
SA

cosec x − cot x sec x


29. Evaluate :  cosec x + cot x
.
1 + 2sec x
dx
1  sin2x  1 3/2
2
1. –
2
sin 2x + C 2. −  sin x +
 2 
+C 3.
3
(
x+ x2 + 2 ) − 1/ 2
+C
( x+ x2 + 2 )
x 2 + 2x − 3 1  2  1
4. 2
+ . cos−1  + C 5. C– tanx.(2 + tan2x). 4 − cot 2 x
8(x + 1) 16  x + 1 3

2 (x 2 + 1) x 2 + 1  3  1  2(7 x − 20)
6. . 1 − n  1 + 2   + C 7. +C

RI
 2  x 
3
9x 9 7 x − 10 − x 2
1 x 2  x +1
8. x; x2 + 2x cos  + 1 9. e [(x – 1) cos x + (x – 1)2 . sin x] + C 10. ex  2  +C
2  x + 1

A
sin x − x cos x 2
11. +C 12. –2 cos4x. e tan x
+C
x sin x + cos x

UH
2
13. x tan −1
x. n (1 + x 2 ) + (tan −1
x) 2 − 2x tan −1
x + n (1 + x 2 ) − (n 1 + x2 ) +C
1 + xn 1
14. ex +C 15. [ sin2 x.n(1+tan x) – x +  n |sin x + cos x|] + C
1 − xn 2
JA
b sin x 2a a−b x
16. – + 2 tan–1 tan +C
(a − b )(a + bcos x)
2 2
(a − b2 )3 / 2 a+b 2
 
2 − x − x2 2 4 − x + 2 2 2 − x − x2  2x + 1 
17. – + ln – sin–1   +K
x 4 x  3 
LP

13x – 159 53 x−3


18. + tan −1 +C 19. 0
8(x 2 – 6x + 13) 16 2
20. tan x ln (1 + sin2x) – 2x + 2 tan–1 ( 2 .tan x) + C.
1
A

cos ( − x)
2 cos x
21. x cos  + sin  n +C 22. – +C
1 b + a sin x
cos ( + x)
2
NK

−2 x −  cot 2 x − 1 + 2cot 2 x 
23. . +C 24. 2 log   − log cot x + cot 2 x − 1 + c
 −  x−  2   
 cot x + 1 
 1 
25. sec–1  cos x +  + c. 26. 5
 cos x 
SA

 x 
6 3 8 −1
 tan 2 + 1 
27. x + log | sin x + 2cos x + 3 | − tan  +C
5 5 5  2 
 

1 1 3 2 tan2 / 3 x − 1
28. − log(1 + tan2 / 3 x) + log(tan4 / 3 x − tan2 / 3 x + 1) + tan−1 +c
2 4 2 3

1 2 x
29. sin−1  sec  + C
2 2
LIMITS, CONTINUITY & DIFFERENTIABILITY
JEE (ADVANCED) SYLLABUS

Limits : Limit and Continuity of a function, Limit and Continuity of the sum, difference, product and quotient of
two functions, L'Hospital rule of evaluation of limits of functions, Even and Odd functions, Inverse of a function,
Continuity of Composite functions, Intermediate value property of continuous functions.
Continuity : Continuity of a function, Continuity of the sum, difference, product and quotient of two functions,
L'Hospital rule of evaluation of limits of functions. Continuity of composite functions, intermediate value property

RI
of continuous functions.
Derivability : Derivative of a function, derivative of the sum, difference, product and quotient of two functions,
chain rule, derivatives of polynomial, rational, trigonometric, inverse trigonometric, exponential and logarithmic
functions. Derivatives of implicit functions, derivatives up to order two, geometrical interpretation of the

A
derivative.
JEE (MAIN) SYLLABUS

UH
Continuity & Derivability : Sets and their representation; Union, intersection and complement of sets and
their algebraic properties; Power set; Relation, Types of relations, equivalence relations.

 
Calculus  required  continuity,  and  continuity  was  supposed  to  require  the  infinitely    little;  But  nobody  could  discover  what  the  infinitely  little  might 
be............Russell, Bertrand 
JA
 
 

Definition : Limit of a function f(x) is said to exist, as x → a when,


im f (a − h) = im f (a + h) = Finite
h→0 h→0
+ +

(Left hand limit) (Right hand limit)


LP

Note that we are not interested in knowing about what happens at x = a. Also note that if L.H.L. &
R.H.L. are both tending towards '  ' or ‘–’, then it is said to be infinite limit.
Remember, ‘x → a’ means that x is approaching to ‘a’ but not equal to ‘a’.
A

Fundamental theorems on limits :


Let im f (x) = and im g (x) = m. If  & m are finite, then:
x→a x→a
NK

(A) im { f (x) ± g (x) } =  ± m


x→a

(B) im { f(x). g(x) } = . m


x→a

f (x)
, provided m  0
SA

(C) im =
x→a g (x) m

(D) im k f(x) = k im f(x) = k; where k is a constant.


x→a x→a

 
(E) im f (g(x)) = f  im g (x)  = f (m); provided f is continuous at g (x) = m.
x→a
 x→a 

Example # 1 : Evaluate the following limits : -


(i) im (x + 2) (ii) im cos (sin x)
x →2 x →0

Solution : (i) x + 2 being a polynomial in x, its limit as x → 2 is given by im (x + 2) = 2 + 2 = 4


x →2
(ii) im cos (sin x) = cos
x →0
( im sin x ) = cos 0 = 1
x →0

Self practice problems

Evaluate the following limits : -


x2 + 4
(1) im x(x – 1) (2) im
x →2 x →2 x+2
Ans. (1) 2 (2) 2

Indeterminate forms :

RI
0 
If on putting x = a in f(x), any one of , ,0  ,  −  º, 0º, 1 form is obtained, then the limit
0 
has an indeterminate form. All the above forms are interchangeable, i.e. we can change one form to

A
other by suitable substitutions etc.
In such cases im f(x) may exist.
x →a

x2 − 4

UH
Consider f(x) = . Here im x2 – 4 = 0 and im x – 2 = 0
x−2 x →2 x →2

0
 im f(x) has an indeterminate form of the type
x →2 0
nx
im has an indeterminate form of type .
x →
JA
x
im (1 + x)1/x is an indeterminate form of the type 1
x →0

NOTE :
(i) +=
(ii) x=
LP

a
(iii) = 0, if a is finite.

(iv) is not defined for any a  R.
x [x 2 ]
(v) im is an indeterminate form whereas im is not an indeterminate form
A

x →0 x x →0 x2
(where [.] represents greatest integer function) Students may remember these forms along with
tending to zero
the prefix ‘tending to’ i.e. is an indeterminate form where as
NK

tending to zero
exactly zero
is not an indeterminate form, its value is zero.
tending to zero

Similarly (tending to one)tending to  is indeterminate form whereas (exactly one) tending to  is not an
indeterminate form, its value is one.
SA

To evaluate a limit, we must always put the value where ' x ' is approaching to in the function. If we get a
determinate form, then that value becomes the limit otherwise if an indeterminate form comes, we have
to remove the indeterminancy, once the indeterminancy is removed the limit can be evaluated by
putting the value of x, where it is approaching.
Methods of removing indeterminancy
Basic methods of removing indeterminancy are
(A) Factorisation (B) Rationalisation
(C) Using standard limits (D) Substitution
(E) Expansion of functions.
Factorisation method : -
We can cancel out the factors which are leading to indeterminancy and find the limit of the remaining
expression.

x 2 − 2x − 3
Example # 2 im
x →3 x 2 − 4x + 3
x 2 − 2x − 3 (x − 3)(x + 1)
Solution : im 2 = im =2
x →3 x − 4x + 3 x →3 (x − 3)(x − 1)

Rationalisation method :-

RI
We can rationalise the irrational expression in numerator or denominator or in both to remove the
indeterminancy.

Example # 3 : Evaluate :

A
3 − 8x + 1 x
(i) im (ii) im
x →1 5 − 24x + 1 x →0 1+ x − 1− x

UH
3 − 8x + 1 ( 9 − 8x − 1) ( 5 + 24x + 1) 5
Solution : (i) im = im =
x →1 5 − 24x + 1 x →1
(3 + 8x + 1) ( 25 − 24x − 1) 9

(ii) The form of the given limit is when x → 0. Rationalising the


numerator, we get
JA
x  x 1+ x + 1− x 
im = im   
x →1 1+ x − 1− x x →1
 1 + x − 1 − x 1 + x + 1 − x 
 x ( 1+ x + 1− x )   x ( 1+ x + 1− x )   1+ x + 1− x  2
= im   = xim   = xim  = = 1
x →1
 (1 + x) − (1 − x)  →1
 2x  →1
 2  2
LP

Self practice problems


Evaluate the following limits : -
1 − (sin x)1/ 3 x+h − x
(3) im (4) im
x→
 1 − (sin x)2 / 3 h→0 h
A

x −b − a−b x
(5) im (6) im+
x →a x 2 − a2 x →0
4− x − x
NK

1 1 1
Ans. (3) (4) (5) (6) 0
2 2 x 4a a − b
Standard limits :
sin x tan x
SA

(a) (i) im = im =1 [ Where x is measured in radians ]


x→0 x x→0 x
tan−1 x sin−1 x
(ii) im = im =1
x→0 x x→0 x
1 1

(iii) im (1 + x) x = e ; im (1 + ax) x = ea
x →0
x→0

x x
 1 im  1 + a  = ea
(iv) im 1 +  = e ; im x →
x→0   x 
 x x→0

ex − 1 ax − 1
(v) im =1 ; im = logea = na ,a > 0
x→0 x x→0 x
n(1 + x)
(vi) im =1
x→0 x
x n − an
(vii) im = nan – 1
x→ a x−a
(b) If f(x) → 0, when x → a, then
sin f(x)
(i) im =1 (ii) im cos f(x) = 1
x→ a f(x) x→ a

tan f(x) ef ( x ) − 1
(iii) im =1 (iv) im =1
x→ a f(x) x→ a f(x)

RI
b −1
f(x)
n(1 + f(x))
(v) im = n b, (b > 0) (vi) im =1
x→ a f(x) x→ a f(x)
1

(vii) im (1 + f(x)) f ( x ) = e
x→ a

A
(c) im f(x) = A > 0 and im (x) = B(a finite quantity), then im [f(x)](x) = AB.
x →a x →a x →a

UH
(1 + x)n − 1
Example # 4 : Evaluate : im
x →0 x
(1 + x) − 1
n
(1 + x)n − 1
Solution : im = im =n
x →0 x x → a (1 + x) − 1
JA
3x − 1
Example # 5 : Evaluate : im
2x − 1 x →0

3 −1 x
3x − 1 1 n3
Solution : im x = im . x =
x →0 2 − 1 x →0 x 2 −1 n2
x
LP

1 − cos 3x
Example # 6 : Evaluate : im
x →0 x2
2
 3x 
3 sin
1 − cos 3x 1  2  9
Solution : im = im .   =
A

x →0 x2 x →0 2
 3x  2
 2 
sin2 x
NK

Example # 7 : Evaluate : im
x →0 sin 4x.tan x
2
 sin x 
x2  
sin2 x  x  1
Solution : im = im =
x →0 sin 4x.tan x x →0  sin 4x  tan x  4
4x   x  x
 4x  
SA

Example # 8 : Evaluate : im (1 + 2x)1/x


x →0
2
im .x
Solution : im (1 + 2x)1/x = e x → 0 x = e2 .
x →0

Example # 9 : Evaluate :
ex − ey x(e x − 1)
(i) im (ii) im
x→y x−y x →0 1 − cos x
x−y
e −ex y
e (ey
− 1)
Solution : (i) im = im = ey
x→y x−y x→y x−y
 
x(e x − 1) x(e x − 1) 1  ex − 1 x2 
(ii) im
x →0 1 − cos x
= im = . im  . =2
x →0 x 2 x →0  x x
2 sin2 sin2 
2  2 
Self practice problems
Evaluate the following limits : -
sin7x 8  x2 x2 x2 x2 
(7) im (8) im  1– cos – cos + cos cos 
x→0 3x x→0 x8  2 4 2 4 

1 − sin2x 5x − 9x
(9) im (10) im

RI
x→
  − 4x x →0 x
4

b
(11) im (1 + a2)x sin , where a  0
x → (1 + a2 )x

A
7 1 5
Ans. (7) (8) (9) does not exist (10) n (11) b
3 32 9

UH
Use of substitution in solving limit problems
Sometimes in solving limit problem we convert im f(x) into im f(a + h) or im f(a – h) according as
x →a h→0 h→0
need of the problem. (here h is approaching to zero.)

cos x − sin x
JA
Example # 10 : Evaluate : im
x→

4
cosx ( 2 − 2sin x )
cos x − sin x 1 1 − tan x 
Solution : im = im . Put x= +h
x→

4
cosx ( 2 − 2sin x ) x→

4
2 1 − 2 sin x 4


 x→  h→0
LP

4
  1 + tan h −2 tan h
1 − tan  + h  1−
1  4  = 1 1 − tan h 1 1 − tan h
im im = im
h→0 2 1 − 2 sin   + h  h→0 2 1 − sin h − cos h h→0 2 2sin2 h − 2sin h cos h
4 
  2 2 2
A

tanh
−2
−2 tan h
1 1 1 h 1
= im = im
h→0   (1 − tanh) → h (1 − tanh)
NK

2 h h h h 0 2 sin
2sin sin − cos 
2 2 2 2 sin h − cos h 
h  2 2 
2
−2 1
= . = 2
−1 2
SA

Limits using expansion


x n a x 2 n2a x 3 n3 a
(a) ax = 1 + + + + ........., a  0
1! 2! 3!
x x2 x3
(b) ex =1 + + + + ......
1! 2! 3!
x2 x3 x 4
(c) n (1+x) = x − + − + ........., for − 1  x  1
2 3 4
x3 x5 x7
(d) sin x = x − + − + .....
3! 5! 7!
x2 x4 x6
(e) cos x = 1 − + − + .....
2! 4! 6!
x 3 2x 5
(f) tan x = x + + + ......
3 15
n(n − 1) 2 n(n − 1)(n − 2) 3
(g) for |x| < 1, n  R; (1 + x)n = 1 + nx + x + x + ............
1 . 2 1 . 2 . 3
 
1
x 11 2
(h) (1 + x) x
= e 1 − + x − ............. 
 2 24 

tan x − x

RI
Example # 11: Evaluate : im
x3
x→0

x3 2 5
x+ + x + .... − x
tan x − x 3 15 1
Solution : im = im =

A
x→0 3 x→0 3
x x 3

1
(14 + x ) 4 − 2

UH
Example # 12 : Evaluate : im
x→2 x−2
1 1 1

(14 + x ) 4 −2 (14 + x ) 4 − 16 4 1
1
−1 1
Solution : im = im = . 16 4 =
x→2 x−2 x→2 (14 + x ) − 16 4 32
JA
x2
n(1 + x) +  sin x +
Example # 13 : If im 2 = 1 then find .
x→0
x tan2 x 2
 x 2 x3   x3 x5  x2
x2  x − + − .....  +   x − + − .....  +
n(1 + x) +  sin x + 2 3   3! 5!  2
im    
2 = 1
Solution : im =
LP

2
x→0
x tan2 x x→0
tan x 2
x3 .
x2
  = –1
1
e − (1 + x) x
A

Example # 14 : Evaluate : im
x→0 tan x
 x 
1
e − e  1 − + ...... 
NK

e − (1 + x) x  2  = im x e
Solution : im = im × =
x→0 tan x x→0 tan x x→0 tan x 2

e x −  cos x + e − x
Example # 15 : Find the values of  + 2 + 3 if im =2
x →0 x tan x
SA

e x −  cos x + e− x
Solution : im =2 .....(1)
x →0 x tan x
at x → 0 numerator must be equal to zero
 –+=0  =+ .....(2)
−x
e − ( +  ) cos x + e
x
From (1) & (2), im =2
x →0 tan x
x2
x
 x x 2
x 3   x2 x 4   x x 2 x3 
 1 + + + + ......  – ( +  )  1 − + − ......  +  1 − + − + ...... 
 1! 2! 3!   2! 4!   1! 2! 3! 
 im   
2
   =2
x →0 x
 ( −  ) x 
x2  + ( +  ) + ( −  ) + ..... 
 im  x 3!  =2
x →0 2
x

Since R.H.S is finite,


0 + 2 + 0 + ....
 –=0   = , then =2   = 1 then  = 1
1
From (2), =+=1+1=2
So,  + 2 + 3 = 8

Limit when x → 

RI
In these types of problems we usually cancel out the greatest power of x common in numerator and
1
denominator both. Also sometime when x → , we use to substitute y = and in this case y → 0+.
x

A
cos x
Example # 16 : Evaluate : im
x→  x

UH
cos x
Solution : im =0
x→  x
1
Example # 17: Evaluate im x.tan
x→  x
1
tan
JA
1 x =1
Solution : im x.tan = im
x→  x x→  1
x

4x + 3
Example # 18 : Evaluate : im
x→  x−8
LP

3
4+
4x + 3 x = 4.
Solution : im = im
x→  x − 8 x→  8
1−
x
4x 2 − 8
A

Example # 19 : Evaluate im
x→ 
7x + x5 + 1
4 8

4x 2 − 8 3
x5 = 0
NK

Solution : im = im x
x→ 
7x + x5 + 1 x→  7
+ 1+ 5
1
x x
x−8
Example # 20 : Evaluate im
x→ − 
4x 2 + x + 1
SA

Solution : Replace x by –t
8
−1 −
−t − 8 t 1
im = im =–
t→  t→  2
4t − t + 1
2 1 1
4− + 2
t t
Some important notes :
nx x xn
(i) im =0 (ii) im =0 (iii) im =0
x → x x → ex x → ex
n
( nx )
(iv) im =0 (v) im x(nx)n = 0
x → x x →0+
As x → , n x increases much slower than any (positive) power of x where as e x increases much faster
than any (positive) power of x.
(vi) im (1 − h)n = 0 and im (1 + h)n →  where h → 0+.
n→ n→

x10 + 7x 2 + 1
Example # 21 : Evaluate im
x → ex
x10 + 7x 2 + 1
Solution : im =0
x → ex
Limits of form 1, 00, 0
0

RI
(A) All these forms can be converted into form in the following ways
0
(a) If x → 1, y →  , then z = (x) y is of 1 form
 n z = y n x
nx  0  1

A
 n z = form  As y →   → 0 and x → 1  nx → 0
1  0  y
y
(b) If x → 0, y → 0, then z = x y is of (00) form

UH
y 0 
 n z = y n x  n z =  0 form 
1  
nx
(c) If x →  , y → 0, then z = xy is of ()0 form
JA
y 0 
 n z = y n x  0 form   n z =
1  
nx
(B) (1) type of problems can be solved by the following method
1
x
(a) im (1 + x) =e
x →0
LP

(b) im [f(x)]g(x) ; where f(x) → 1 ; g(x) →  as x → a


x →a
(f ( x) −1) g( x)
 
1
{ f ( x )−1} . g( x )
im[f(x) −1] g(x)
= im 1 + f(x) − 1
f ( x )−1 1
= im  [1 + (f(x) − 1)] f ( x)−1  = ex→a
x →a x →a  
 
A

 x2 − 1 
( x +1)
2

Example # 22 : Evaluate im  2 
x →  x + 3 
 
Solution : Since it is in the form of 1
NK

 x2 − 1 
( x +1)
2
  x2 − 1 − x2 − 3  2 
 2 
= exp  lim 
x → x2 + 3
 x +1 ( )  = e –4

x +3    

Example # 23 : Evaluate im (tan x)tan 2x


SA


x→
4
2 tan x
im (tan x −1) tan 2x im (tan x −1)
  1− tan2 x
x→ x→

Solution : Since it is in the form of 1 so im (tan x) tan 2x
=e 4
= e 4


x→
4
tan  / 4
2
−1(1+ tan  / 4) 1
= e = e–1 =
e
x
tan
 a 2a
Example # 24 : Evaluate im  2 − 
x →a
 x
x
tan
 a 2a
Solution : im  2 −  put x=a+h
x →a
 x
  h   h 
tan +  − cot   h  h 
 h   2 2a   h   2a  im − cot
h→0
. 1+
2a  a + h
− 1

= im  1 +  = im  1 + = e
h→0
 (a + h)  h→0  a + h 
 h  2a
 
im −  2a  . 
h→0  tan h  a + h 2
  –
 2a 
= e = e 
Example # 25 : Evaluate im+ (tanx)tanx
x →0

Solution : Let y = im+ (tanx)tanx


x →0

1
n
n y = im+ tanx n tanx = im – tan x = 0, as 1 → y=1

RI
x →0 x →0+ 1 tan x
tan x
Sandwitch theorem or squeeze play theorem:
Suppose that f(x)  g(x)  h(x) for all x in some open interval containing a,

A
except possibly at x = a itself. Suppose also that
h
g
im f(x) = = im h(x),

UH
f
x→a x→a

Then im g(x) = .
x→a

[x] + [4x] + [7x] + .... + (3n − 2)x 


Example # 26 : Evaluate im , where [.] denotes greatest integer function.
JA
n→
n2
Solution : We know that, x – 1 < [x]  x
4x – 1 < [4x]  4x
7x – 1 < [7x]  7x
. . .
. . .
(3n – 2)x – 1 < [(3n – 2)x]  (3n – 2)x
LP

 (x + 4x + 7x + .... + (3n – 2)x) – n < [x] + [4x] + .....


+[(3n – 2)x]  (x + 4x + .... + nx)
n


n n
 (3n – 1)x – n < [(3r − 2)x]  (3n – 1)x
2 r =1
2
A

n ( 3n − 1) 1 [x] + [4x] + .... + [(3n − 2)x] n ( 3n − 1)


 im 2
x– < im  im x
n→ 2 n n n → 
n2 n→ 2 n2
3x [x] + [4x] + .... + [(3n − 2)x] 3x
NK

 < im 2

2 n →  2
n
[x] + [4x] + .... + [(3n − 2)x] 3x
 im =
n→
n2 2
Continuity & Derivability :
SA

A function f(x) is said to be continuous at x = c, if Limit f(x) = f(c)


x→c

i.e. f is continuous at x = c
if Limit f(c − h) = Limit f(c+h) = f(c).
h→ 0
+ +
h→ 0

If a function f (x) is continuous at x = c, the graph of f (x) at the corresponding point ( c, f (c)) will not be
broken. But if f (x) is discontinuous at x = c, the graph will be broken when x = c
(i) (ii) (iii) (iv)
((i), (ii) and (iii) are discontinuous at x = c)
((iv) is continuous at x = c)
A function f can be discontinuous due to any of the following three reasons :
f(x)  Limit

RI
(i) Limit f(x) does not exist i.e. Limit

f (x) [figure (i)]
x→c x→c +
x→c

(ii) f (x) is not defined at x = c [figure (ii)]

Limit f(x)  f (c)

A
(iii) [figure (iii)]
x→c

Geometrically, the graph of the function will exhibit a break at x= c.


cos 2x , x  1

UH
Example # 27 : If f(x) =  , then find whether f(x) is continuous or not at x = 1, where
 [x] , x 1
[ . ] is greatest integer function.
cos 2x , x  1
Solution : f(x) = 
 [x] , x 1
JA
For continuity at x = 1, we determine f(1) , lim– f(x) and lim+ f(x).
x →1 x →1

Now, f(1) = [1] = 1


lim– f(x) = lim– cos 2x = cos 2 = 1
x →1 x →1

and lim+ f(x) = lim+ [x] = 1


x →1 x →1
LP

so f(1) = lim– f(x) = lim+ f(x) = 1


x →1 x →1

 f(x) is continuous at x = 1

Self practice problems :


A


(12) If possible find value of  for which f(x) is continuous at x =
2
NK

 1 − sin x 
 1 + cos 2x , x
2

 
f(x) =  , x=
 2
 (2x − )2 
x
SA

 ,
 tan2x 2

(13) Find the values of p and q such that the function



 x + p sin x ; 0  4x  
  
f(x) =  2x cot x + q ;   4x  2 is continuous at x = and x =
 p 4 2
 cos 2x − qsin x ; 2  4x  4
 2
 −
Ans. (12) discontinuous (13) p= ,q=
3 2 12
Theorems on continuity :
(i) If f & g are two functions which are continuous at x = c, then the functions defined by:
F1(x) = f(x)  g(x) ; F2(x) = K f(x), K is any real number ; F 3(x) = f(x).g(x) are also continuous at
f(x)
x = c. Further, if g (c) is not zero, then F 4(x) = is also continuous at x = c.
g(x)
(ii) If f(x) is continuous & g(x) is discontinuous at x = a, then the product function
 (x) = f(x). g(x) may or may not be continuous but sum or difference function  (x) = f(x)  g(x)
will necessarily be discontinuous at x = a.
sin x x  0
e.g. f (x) = x & g(x) = 

RI
0 x=0
(iii) If f (x) and g(x) both are discontinuous at x = a, then the product function  (x) = f(x). g(x) is not
necessarily be discontinuous at x = a.
 1 , x0

A
e.g. f (x) = g(x) = 
 −1 , x  0
and atmost one out of f(x) + g(x) and f(x) – g(x) is continuous at x = a.

UH

Example # 28 : If f(x) = [sin(x–1)] – {sin(x–1)}. Comment on continuity of f(x) at x = +1
2
(where [ . ] denotes G.I.F. and { . } denotes fractional part function).
Solution : f(x) = [sin (x – 1)] – {sin (x – 1)}
Let g(x) = [ sin (x – 1)] + {sin (x – 1)} = sin (x – 1)

JA
which is continuous at x = +1
2

as [sin (x – 1)] and { sin (x – 1)} both are discontinuous at x = +1
2

 At most one of f(x) or g(x) can be continuous at x = +1
2
LP


As g(x) is continuous at x = + 1, therefore, f(x) must be discontinuous
2
 
Alternatively, check the continuity of f(x) by evaluating lim f(x) and f.  + 1

x → +1
2
2 
A

Continuity of composite functions :


If f is continuous at x = c and g is continuous at x = f(c), then the composite g[f(x)] is continuous at
x sin x
& g(x) = x are continuous at x = 0, hence the composite function
NK

x = c. eg. f(x) = 2
x +2
x sin x
(gof) (x) = 2 will also be continuous at x = 0.
x +2
Self practice problem :
 1
SA

1 + 8x 3 , x  0 (2x − 1) 3
, x0
 
(14) f(x) =  −1 , x=0 and g(x) =  1 , x=0
4x 2 – 1 , x  0 
  2x + 1 , x  0

Then define fog (x) and comment on the continuity of gof(x) at x = 1/2
16x − 7 ; x  0

Ans. [ fog(x) =  3 ; x = 0 and gof(x) is discontinous at x = 1/2]
 8x + 3 ; x  0

Continuity in an Interval : 
(a) A function f is said to be continuous in (a, b) if f is continuous at each & every point (a, b).
(b) A function f is said to be continuous in a closed interval [ a, b ] if:
(i) f is continuous in the open interval (a, b),
(ii) f is right continuous at ‘a’ i.e. Limit f(x) = f(a) = a finite quantity and
x →a
+

(iii) f is left continuous at ‘b’ i.e. Limit



f(x) = f(b) = a finite quantity.
x →b

(c) All Polynomial functions, Trigonometrical functions, Exponential and Logarithmic functions are
continuous at every point of their respective domains.

On the basis of above facts continuity of a function should be checked at the following points
(i) Continuity of a function should be checked at the points where definition of a function
changes.
(ii) Continuity of {f(x)} and [f(x)] should be checked at all points where f(x) becomes integer.

RI
(iii) Continuity of sgn (f(x)) should be checked at the points where f(x) = 0 (if f(x) = 0 in any
open interval containing a, then x = a is not a point of discontinuity)
(iv) In case of composite function f(g(x)) continuity should be checked at all possible points of
discontinuity of g(x) and at the points where g(x) = c, where x = c is a possible point of

A
discontinuity of f(x).
 [2x] 0  x 1
Example # 29: If f(x) =  , where { . } represents fractional part function and
{3x} sgn(–x) 1  x  2

UH
[ . ] is greatest integer function, then comment on the continuity of function in the interval [0, 2].
Solution : The given function is
 1
 0 0x
JA
2

 1 1
 x 1
 2

3(1– x) 1  x  4
f(x) =  3
 4 5
 4 – 3x x
LP

 3 3
 5
 5 – 3x x2
 3
 0 x=2
so discontinous at x = 1/2 , 1,4/3, 5/3, 2
A

x+3 1
Example # 30 : If f(x) = and g(x) = , then discuss the continuity of f(x), g(x) and fog (x).
x −1 x−3
NK

x+3
Solution : f(x) =
x −1
f(x) is a rational function it must be continuous in its domain
and f is not defined at x = 1
 f is discontinuous at x = 1
SA

1
g(x) =
x−3
g(x) is also a rational function. It must be continuous in its domain and g is not defined at
x=3
 g is discontinuous at x = 3
Now fog (x) will be discontinuous at
(i) x=3 (point of discontinuity of g(x))
(ii) g(x) = 1 (when g(x) = point of discontinuity of f(x))
1
if g(x) = 1  =1  x=4
x−3
 discontinuity of fog(x) should be checked at x = 3 and x = 4
at x = 3
1
+1
fog (x) = x − 3
1
−1
x−3
fog (3) is not defined
1
+1
x − 3 1+ x − 3
lim fog (x) = lim = lim = 1  fog (x) is discontinuous at x = 3
x →3 x →3 1 x →2 1 − x + 3
−1
x−3
fog (4) = not defined
lim+ fog (x) = 

RI
x→4

lim fog (x) = –  fog (x) i s discontinuous at x = 4.


x → 4−

Self practice problem :

A
  1
[ n x] . sgn  x −   ; 1  x  3
(15) If f(x) =   2 . Find the pointswhere the continuity of f(x),
 2
3  x  3.5
 {x } ;

UH
should be checked, where [ . ] is greatest integer function and {.} fractional part function.
3 5
Ans. { 1, , , e, 3 , 10 , 11 , 12 ,3.5 }
2 2

Intermediate value theorem : 


JA
A function f which is continuous in a,b  possesses the following properties:

(i) If f(a) & f(b) possess opposite signs, then there exists at least one solution of the equation
f(x) = 0 in the open interval (a, b).
(ii) If K is any real number between f(a) & f(b), then there exists at least one solution of the
equation f(x) = K in the open interval (a, b).
LP

Example # 31: Prove that the equation 3(x – 1) (x – 2) + 4(x + 1) (x – 4) = 0


will have real and distinct roots.

Solution : 3(x – 1) (x – 2) + 4(x + 1) (x – 4) = 0


A

f(x) = 3(x – 1) (x – 2) + 4 (x + 1) (x – 4)
f(–1) = + ve
f(1) = –ve –1 1 2 4
f(2) = –ve
NK

f(4) = +ve
hence 3(x – 1) (x – 2) + 4(x + 1) (x – 4) = 0
have real and distinct roots
Self practice problem :
SA

(16) If f(x) = xnx – 2, then show that f(x) = 0 has exactly one root in the interval 1, e).
1 
Example # 32: Let f(x) = Lim , then find f   and also comment on the continuity at x = 0
n→ 1 + nsin2 x 4
1
Solution : Let f(x) = lim
n→ 1 + nsin2 x

 1 1
f   = lim = lim =0
4 n →  n→  1
1 + n . sin2 1+ n  
4 2
Now
1 1  1 
f(0) = lim 2
= = 1  lim f(x) = lim  lim 2  =0
n→ n . sin (0) + 1 x →0 x → 0 n → 1 + n sin x
1+ 0  
{here sin2x is very small quantity but not zero and very small quantity when multiplied with
 becomes }
 f(x) is not continuous at x = 0
Self practice problem :
1
(17) If f(x) = Lim (1 + x)n. Comment on the continuity of f(x) at x = 0 and explain Lim (1 + x) x = e .
n→ x →0
Ans. Discontinous (non-removable)

Example # 33: f(x) = minimum (2 + cos t, 0  t  x), 0  x  2 discuss the continuity of this function at x= 

RI
Solution : f(x) = minimum (2 + cos t, 0  t  x), 0  x  2
2 + cos x 0  x  
f(x) = 
 1   x  2

A
y

UH
1

x
which is continuous at x = 

f(x)
JA
3

x
LP

Differentiability of a function at a point :


(i) The right hand derivative of f (x) at x = a denoted by f (a+) is defined by:
f (a + h) − f (a)
R.H.D. = f (a+) = Limit , provided the limit exists.
h→ 0 +
h
A

(ii) The left hand derivative of f(x) at x = a denoted by


f (a–) is defined by:
f (a − h) − f (a)
L.H.D. = f  (a– ) = Limit , provided the limit exists.
NK

h → 0+ −h
A function f(x) is said to be differentiable at x = a if f (a+) = f  (a–) = finite
f (a + h) − f (a)
By definition f (a) = Limit
h → 0+ h
 2x + 3 , x  1
SA

Example # 34 : Comment on the differentiability of f(x) =  2 at x = 1.


4x – 1 , x  1
f(1 + h) − f(1)
Solution : R.H.D. = f (1+)= Limit =8
h → 0+ h
f(1 − h) − f(1)
L.H.D. = f(1–)= Limit =2
h → 0+ −h
As L.H.D.  R.H.D. Hence f(x) is not differentiable at x = 1.

 ax + b , x  −1
Example # 35: If f(x) =  3 , then find a and b so that f(x) become differentiable at x = –1.
ax + x + 2b , x  −1
Solution : –a + b = –a – 1 + 2b using continuity  
 b=1
 a , x  −1
f' (x) = 
3ax + 1 , x  −1
2

1
a = 3a + 1  a = –
2

 3
[sin x], x 1
Example # 36 : If f(x) =  2 , then comment on the derivability at x = 1,
 2{x} − 1, x 1

where [ . ] is greatest integer function and {.} is fractional part function.
 3 

RI
f(1 − h) − f(1) sin 2 (1 − h) + 1
Solution : f(1–) = lim+ = lim+   = 0
h→0 −h h→0 −h
f(1 + h) − f(1) 2{1 + h} + 1 − 1 2h

A
f(1+) = lim+ = lim+ = lim+ =2
h→0 h h→0 h h→0 h
 f(1+)  f(1–)
f(x) is not differentiable at x = 1.

UH
Self Practice Problems :

  2x  x
  + +2 , x  3
 3  3
JA
(18) If f(x) =  , then comment on the continuity and differentiable at
 x + 3 , x  3
  3 
x = 3, where [ . ] is greatest integer function and {.} is fractional part function.

 x sin−1 1/ x, x  0

If f(x) = 
LP

(19) , then comment on the derivability of f(x) at x = 0.



 0 , x=0

Ans. (18) Discontinuous and non-differentiable at x = 3


(19) non-differentiable at x = 0
A

Concept of tangent and its association with derivability :


Tangent :- The tangent is defined as the limiting case of a chord or a secant.
NK

f(a + h) − f(a)
slope of the line joining (a,f(a)) and (a + h, f(a + h)) =
h

f(a + h) − f(a)
SA

Slope of tangent at P = f(a) = Lim


h→0 h

The tangent to the graph of a continuous function f at the point P(a, f(a)) is
(i) the line through P with slope f(a) if f(a) exists ;
(ii) the line x = a if L.H.D. and R.H.D. both are either  or – .
If neither (i) nor (ii) holds then the graph of f does not have a tangent at the point P.
In case (i) the equation of tangent is y – f(a) = f(a) (x – a).
In case (ii) it is x = a
Note : (i) tangent is also defined as the line joining two infinitesimally close points on a curve.
(ii) A function is said to be derivable at x = a if there exist a tangent of finite slope at that point.
f(a+) = f(a–) = finite value
(iii) y = x3 has x-axis as tangent at origin.

(iv) y = |x| does not have tangent at x = 0 as L.H.D.  R.H.D.


Example # 37 : Find the equation of tangent to y = (x) 1/3 at x = 1 and x = 0.
Solution : At x = 1 Here f(x) = (x) 1/3
f(1 − h) − f(1) (1 − h)1/ 3 − 1 1
L.H.D = f(1–) = lim+ = lim+ =
h→0 −h h → 0 −h 3
f(1 + h) − f(1) (1 + h)1/ 3
− 1 1
R.H.D. = f(1+) = lim+ = lim+ =
h→0 h h → 0 h 3

RI
1
As R.H.D. = L.H.D. =
3
1 1
 slope of tangent =  y – f(1) = (x – 1)
3 3

A
1
y–1= (x – 1)
3
 3y – x = 2 is tangent to y = x 1/3at (1, 1)

UH
At x = 0
(0 − h)1/ 3 − 0
L.H.D. = f(0–) = lim+ =+
h→0 −h
(0 + h)1/ 3 − 0
R.H.D. = f(0+) = lim+ =+
h→0 h
JA
As L.H.D. and R.H.D are infinite. y = f(x) will have a vertical tangent at origin.
 x = 0 is the tangent to y = x 1/3 at origin.
Self Practice Problems :
(20) If possible find the equation of tangent to the following curves at the given points.
(i) y = x3 + 3x2 + 28x +1 at x = 0.
LP

(ii) y = (x – 8)2/3 at x = 8.
Ans. (i) y = 28x + 1 (ii) x=8
Relation between differentiability & continuity:
(i) If f (a) exists, then f(x) is continuous at x = a.
A

(ii) If f(x) is differentiable at every point of its domain of definition, then it is continuous in that
domain.
NK

Note : The converse of the above result is not true i.e. "If 'f' is continuous at x = a, then 'f' is differentiable at
x = a is not true.
e.g. the functions f(x) =  x − 2  is continuous at x = 2 but not differentiable at x = 2.

If f(x) is a function such that R.H.D = f(a+) =  and L.H.D. = f(a–) = m.


Case - 
SA

If  = m = some finite value, then the function f(x) is differentiable as well as continuous.
Case - 
if   m = but both have some finite value, then the function f(x) is non differentiable but it is continuous.
Case - 
If at least one of the  or m is infinite, then the function is non differentiable but we can not say about
continuity of f(x).

(i) (ii) (iii)


continuous and differentiable continuous but not differentiable neither continuous nor differentiable
Example # 38 : If f(x) is differentiable at x = a, prove that it will be continuous at x = a.
f(a + h) − f(a)
Solution : f(a+) = lim+ =
h→0 h
lim+ [f(a+h) – f(a)] = h
h→0

as h → 0 and  is finite, then lim+ f(a + h) – f(a) = 0


h→0

 lim+ f (a + h) = f(a).
h→0

Similarly lim+ [f(a – h)– f(a)] = – h  lim+ f(a – h) = f(a)


h→0 h→0

 lim+ f(a + h) = f(a) = lim+ f(a – h)

RI
h→0 h→0

Hence, f(x) is continuous.


 + x 2 sgn[x] + {x − 4} , −2  x  2

A
Example # 39 :  If f(x) = , comment on the continuity and differentiability
  − sin(x + )+ | x − 3 |, 2  x  6

of f(x), where [ . ] is greatest integer function and {.} is fractional part function, at x = 1, 2.

UH
Solution : Continuity at x = 1
lim+ f(x) = lim+ ( + x2 sgn[x] + {x – 4}) = 1 + 
x →1 x →1

lim f(x) = lim− ( + x2 sgn [x] + {x – 4})


x → 1− x →1

=1 sgn (0) + 1 +  = 1 + 
JA
  f(1) = 1 + 
 L.H.L = R.H.L = f(1). Hence f(x) is continuous at x = 1.
Now for differentiability,
f(1 + h) − f(1)
R.H.D. = f(1+) = lim+
h→0 h
LP

 + (1 + h)2 sgn[1 + h] + {1 + h − 4} − 1 − 
= lim+
h→0 h
(1 + h)2 + h − 1 1 + h2 + 2h + h − 1 h2 + 3h
= lim = lim = lim =3
h→0 +
h h→0 h +
h→0 h +
A

f(1 − h) − f(1)  + (1 − h)2 sgn[1 − h] + 1 − h − 1 − 


and L.H.D. = f(1–) = lim+ = lim+ =1
h→0 −h h→0 −h
 f(1 )  f(1 ).
+ –
NK

Hence f(x) is non differentiable at x = 1.


Now at x=2
lim− f(x) = lim− ( + x2 sgn [x] + {x – 4}) =  + 4 . 1 + 1 = 5 + 
x →2 x →2

lim f(x) = lim+ ( + sinx + |x – 3|) = 1 +  + sin 2


x → 2+ x →2
SA

Hence L.H.L  R.H.L


Hence f(x) is discontinuous at x = 2 and then f(x) also be non differentaible at
x = 2.
Self Practice Problem :
 e[x] + | x | −1 
  x0
(21) If f(x) =  [x] + {2x}  , comment on the continuity at x = 0 and differentiability at

 1/ 2 x=0
x = 0, where [ . ] is greatest integer function and {.} is fractional part function.
Ans. discontinuous hence non-differentiable at x = 0
Differentiability of sum, product & composition of functions :
(i) If f(x) & g(x) are differentiable at x = a, then the functions f(x)  g(x), f(x). g(x) will also be
differentiable at x = a & if g (a)  0, then the function f(x)/g(x) will also be differentiable at x = a.
(ii) If f(x) is not differentiable at x = a & g(x) is differentiable at x = a, then the product function
F(x) = f(x) . g(x) can still be differentiable at x = a
e.g. f(x) = x and g(x) = x2.
(iii) If f(x) & g(x) both are not differentiable at x = a, then the product function
F(x) = f(x) . g(x) can still be differentiable at x = a e.g. f(x) = x & g(x) = x.
(iv) If f(x) & g(x) both are non-differentiable at x = a, then the sum function F(x) = f(x) + g(x) may be
a differentiable function. e.g. f(x) = x & g(x) = −x.

RI
Example # 40 : Discuss the differentiability of f(x) = x + |x|.

A
Solution :

UH
Non-differentiable at x = 0.

Example # 41 : Discuss the differentiability of f(x) = x|x|


 x2 , x  0
Solution :  f(x) =  2
− x , x  0
JA
LP

Differentiable at x = 0
Example # 42 : If f(x) is differentiable and g(x) is differentiable, then prove that f(x) . g(x) will be differentiable.
Solution : Given, f(x) is differentiable
f(a + h) − f(a)
A

i.e. lim+ = f(a)


h→0 h
g(x) is differentiable
g(a + h) − g(a)
i.e. lim = g(a)
NK

h → 0+ h
let p(x) = f(x) . g(x)
p(a + h) − p(a) f(a + h).g(a + h) − f(a).g(a)
Now, lim+ = lim+
h→0 h h → 0 h
f(a + h)g(a + h) + f(a + h).g(a) − f(a + h).g(a) − f(a).g(a)
= lim+
SA

h→0 h
 f(a + h) (g (a + h) − g(a)) g (a)(f(a + h) − f(a)) 
= lim+  + 
h→0  h h 
 g(a + h) − g(a) f(a + h) − f(a) 
= lim+  f(a + h). + g(a). 
h→0  h h 
= f(a) . g(a) + g(a) f(a) = p (a)
Hence p(x) is differentiable.
 2x − 1 , x0
Example # 43 : If f(x) =  2 then comment on the continuity and differentiability of g(x) by
2x − 4x , x  0
drawing the graph of f(|x|) and, |f(x)| and hence comment on the continuity and differentiability
of g(x) = f(|x|) + |f(x)|.

Solution : 2 x
–1

RI
–2

A
Graph of f(|x|) and |f(x)|

y=f(|x|) y=|f(x)|

UH
2
1
–2 2 x 2 x

–2 –2
JA
If f(|x|) and |f(x)| are continous, then g(x) is continuous. At x = 0 f(|x|) is continuous, and |f(x)| is
discontinuous therefore g(x) is discontineous at x = 0.
 g(x) is non differentiable at x = 0, 2 (find the reason yourself).

Differentiability over an Interval :


LP

f (x) is said to be differentiable over an open interval if it is differentiable at each point of the interval and
f(x) is said to be differentiable over a closed interval [a, b] if:
(i) for the points a and b, f (a+) and f (b–) exist finitely
(ii) for any point c such that a < c < b, f (c+) & f(c–) exist finitely and are equal.
A

All polynomial, exponential, logarithmic and trigonometric (inverse trigonometric not included) functions
are differentiable in their domain.
2x
Graph of y = sin–1 Graph of y = sin–1 x.
NK

1 + x2
SA

Non differentiable at x = 1 & x = – 1 Non differentiable at x = 1 & x = – 1

Note : Derivability should be checked at following points


(i) At all points where continuity is required to be checked.
(ii) At the critical points of modulus and inverse trigonometric function.
  7 1
 2x +  [sin2x] , 0x
  3 2
Example # 44 : If f(x) =  , find those points at which continuity and
 [4x] +  x   .sgn  2x − 4  , 1
 x 1
 4
 

 3  2

differentiability should be checked, where [ . ] is greatest integer function and {.} is fractional
part function. Also check the continuity and differentaibility of f(x) at x = 1/2.
  7 1
 2x +  [sin2x] , 0x
  3 2
Solution : f(x) = 
 [4x] +  x   .sgn  2x − 4  ,

RI
1
 x 1
 4
 

 3  2

The points, where we should check the continuity and

A
1 1 1 2 3
differentiability are x = 0, , , , , ,1
4 3 2 3 4
At x = 1/2

UH
 7
L.H.L. = lim− f(x) = lim− 2x +  [sin 2x] = 0
1 1  3
x→ x→
2 2

 4
R.H.L. = lim+ f(x) = lim+ [4x] sgn 2x −  = 2(– 1) = – 2
JA
x→
1
x→
1  3
2 2

 L.H.L  R.H.L. hence f(x) is discontinuous at x = 1/2 and hence it is non diffferentiable
at x = 1/2.
Self Practice Problems:

 x + 1  1 − x 
LP

(22) If f(x) =  +  , –3  x  5, then draw its graph and comment on the continuity and
 2   2 
differentiability of f(x), where [ . ] is greatest integer function.

 | 4x 2 + 6x + 3 | , –2  x  −1
A

(23) If f(x) =  , then draw the graph of f(x) and comment on the
2
 [x + 2x] , −1  x  0
differentiability and continuity of f(x), where [ . ] is greatest integer function.
NK

Ans. (22) f(x) is discontinuous at x = –3, –1, 1, 3, 5 hence non-differentiable.


(23) f(x) is discontinuous at x = –1, 0 & non differentiable at x = –1, 0.
Problems of finding functions satisfying given conditions :
Example # 45: If f(x) is a function satisfies the relation for all x, y  R, f(x + 2y) = f(x) + f(2y) and if f(0) = 3 and
SA

function is differentiable every where, then find f(x).

f(x + 2h) − f(x) f(x) + f(2h) − f(x) − f(0)


Solution : f(x) = lim+ = lim+ ( f(0) = 0)
h→0 2h h→0 2h
f(2h) − f(0)
= lim+
h→0 2h
= f(0)  f(x) = 3   f (x) 
dx = 3 dx

f(x) = 3x + c
  f(0) = 2.0 + c as f(0) = 0
 c=0  f(x) = 3x
Example # 46 : f(x + ) = f(x) . f()  x,   R and f(x) is a differentiable function and f(0) = 1/3, f(x)  0 for
any x. Find f(x)
Solution : f(x) is a differentiable function
f(x + h) − f(x) f(x).f(h) − f(x).f(0)
 f(x) = lim+ = lim+ ( f(0) = 1)
h→0 h h→0 h
f(x).(f(h) − f(0)) f (x) 1
= lim+
h→0 h
= f(x). f(0) = f(x)  f(x) = f(x)  
f(x)
dx =
 3 dx
x x
 n f(x) = + c  n 1 = 0 + c  c = 0  n f(x) =  f(x) = ex/3
3 3
x+y
Example # 47 : 3f   = f(x) + f(y)  x, y  R and f(0) = 4 and f(0) = 2 and function is differentiable for
 3 
all x, then find f(x).

RI
 3x + 3h   3x + 3.0 
f −f
f(x + h) − f(x)  3   3 

Solution : f(x) = lim = lim
h→0 h h→0 h
f(3h) − f(0)
= lim

A
= f(x) = 2
h→0 3h
f '(x) = 2  f(x) = 2x + c  c = 4  f(x) = 2x + 4
Self Practice Problem:

UH
x
(24) f   = f(x) – f(y)  x , y  R+ and f(1) = 1 , then show that f(x) = nx.
y

Result of Some Known Functional Equation :-


Let x, y are independent variables and f(x) is differentiable function in its domain :
JA
(i) If f(xy) = f (x) + f (y)  x, y  R+, then f(x) = k ln x or f (x) = 0.
(ii) If f(xy) = f (x). f (y)  x, y  R, then f(x) = x k, k  R
(iii) If f(x + y) = f (x). f (y)  x, y  R, then f(x) = akx.
(iv) If f(x + y) = f (x) + f (y)  x, y  R, then f(x) = kx,
where k is a constant in all four parts.
 1  1
Example # 48 : If f(x) is a polynomial function satisfying f(x) . f   = f(x) + f    x  R – {0} and f(2) =9,
LP

 
x x
then find f (3)
Solution : f(x) = 1 ± xn
As f(2) = 9  f(x) = 1 + x3
Hence f(3) = 1 + 33 = 28
A

Self practice problems


 1  1
(25) If f(x) is a polynomial function satisfying f(x) . f   = f(x) + f    x  R – {0} and
 
x x
NK

f(3) = – 8, then find f(4)


f(x)
(26) If f(x + y) = f(x) . f(y) for all real x, y and f(0)  0, then prove that the function, g(x) = is
1 + f 2 (x)
an even function.
Ans. (25) – 15
SA

f( − 9h) − f( + h2 )


Example # 49 : Evaluate lim , if f() = 2
h→0 h
 f( − 9h) − f( + h2 ) 
 .( −9h − h ) = lim
2
Solution :  lim  f'() . (–9 – h) = 2 × –9 = –18
h→0 
 ( −9h − h2
)h 
h→0

Self Practice Problems :


(27) If f(x) and g(x) are differentiable, then prove that f(x) ± g(x) will be differentiable.
f(3 + h) − f(3 + sinh)
(28) If f(3) = 12, then find the value of lim .
h→0
h.tan2h
Ans. (28) 2
 Marked questions are recommended for Revision.

PART - I : SUBJECTIVE QUESTIONS


Section (A) : Definition of LHL/RHL and Indeterminate forms
A-1. Examine the graph of y = f(x) as shown and evaluate the following limits :

RI
A
(i) im f(x) (ii) im f(x) (iii) im f(x)
x →1 x →2 x →3

UH
(iv) im f(x) (v) im f(x)
x →1.99 x →3−

A-2. Evaluate the following limits :

(i) im (x + sin x) (ii) im (tan x – 2x) (iii) im x cos x


x →2 x →3
JA x→
3
4

ex
(iv) im xx (v) im
x →5 x →1 sin x

A-3. Evaluate the following limits,


where [ . ] represents greatest integer function and { . } represents fractional part function
LP

x
(i) im [sin x] (ii) im   (iii) im sgn [tan x]
x→

2
x → 2 2 x →

(iv) im sin–1 (n x)


x →1
A

 x +1 , x 1
A-4. (i) If f(x) =  , evaluate im f(x).
2x − 3 , x 1 x →1

 x+ , x 1
NK

(ii) Let f(x) =  , if im f(x) exist, then find value of .


2x − 3 , x 1 x →1

x2 + 2 , x  2  2x , x  1
A-5. If f(x) =  and g(x) =  , evaluate im f (g(x)).
 1− x , x  2 3 − x , x  1 x →1
SA

A-6. Which of the followings are indeterminate forms. Also state the type.
[x]
(i) im , where [ . ] denotes the greatest integer function
x →0+ x

(ii) im x2 + 1 – x (iii) im − (tan x)tan2x


x →− 
x → 
2
1

(iv) im+ ,
x →1
(x) nx
where { . } denotes the fractional part function

 
SECTION (B) : Evaluation of limits of form 0/0, /,  – , 0 × , Use of L-Hospital
Rule & Expansion
B-1. Evaluate each of the following limits, if exists
x 3 − 3x + 1 4x 3 − x 2 + 2x − 5 a + 2x − 3x
(i) im (ii) im 6 (iii) im ,a0
x →−1 x −1 x →1 x + 5x 3 − 2x − 4 x →a 3a + x − 2 x

B-2. Evaluate the following limits, if exists


1 − cos 4x 3 sin x − cos x
(i) im (ii) im
x →0 1 − cos 5x  
x→
x−

RI
6
6
tan3x − 2x (a + x)2 sin(a + x) − a2 sina
(iii) im (iv) im
x →0 3x − sin2 x x →0 x

A
ebx − eax x (e 2+ x
− e2 )
(v) im , where 0 < a < b (vi) im
x →0 x x →0 1 − cos x

UH
n(1 + 3x) n (2 + x) + n 0.5
(vii) im (viii) im
x →0 3x − 1 x→0 x
1 − cos 2x
x n − 2n
  (ix) Find n  N, if im = 80. (x) im 2
JA
x →2 x−2 x → 0 x

( n (1 + x) − n2)(3.4 x −1 − 3x)
(xi) im 1 1
x →1
[(7 + x) 3 − (1 + 3x) 2 ].sin(x − 1)
LP

B-3. Evaluate the following limits.


 1 2 x  n3 − 2n2 + 1 + 3 n4 + 1
(i) im  2 + 2 + .... + 2  (ii) im , nN
x →  x x x  n →  4
n6 + 6n5 + 2 − 5 n7 + 3n3 + 1
 
x 5 tan  12  + 3 | x |2 + 7
 x 
A

(iii) im
x →
( x 2 − 8x + x ) (iv) im
x → − | x |3 + 7 | x | + 8
NK

B-4. Evaluate the following limits.


5 5
 2 2
 (x + 2) 2 − (a + 2) 2
(i) im  ( x + 1) 3 – (x – 1) 3  (ii) im
x → 
  x →a x −a
(iii)
x→
im cos ( x +1 ) − cos ( x)
SA

 
1

 ( (x + 1) (x + 2) (x + 3) (x + 4) ) −
4
(iv) im x
x → 
 

B-5. Evaluate the following limits using expansions :


1 1 tan2 x
(x + 2) − (15x + 2)
2 5 e x − 1 − sin x −
(i) im (ii) im 2
1
x →2 x →0 x3
(7x + 2) − x4

a + b sin x − cos x + ce x
B-6 If im exists, find the values of a, b, c. Also find the limit
x →0 x3
B-7. Find the values of a and b so that:
1 + a x sin x − b cos x
(i) im may have a finite limit.
x→0 x4
(ii) im
x→
( x 4 + ax 3 + 3x 2 + bx + 2 − x 4 + 2x 3 − cx 2 + 3x − d ) =4

axex − b n (1 + x) + cxe − x
(iii) im = 2
x→0 x 2 sin x

 n (1 + x )
(1+ x)
1
B-8. Find the following limit using expansion im  − :
x→0  x 2
x
 

RI
(cos  )x − (sin  )x − cos 2   
B-9. Prove that im = cos4  n (cos ) − sin4  n (sin ),    0 , 
x→4 x−4  2

A
tan(a + 2h) − 2 tan(a + h) + tana
B-10_. Find the value of lim .
h →0 h2

UH
x n x
SECTION (C) : Limit of form 0 0 , 0 , 1 , im , im , Sandwitch theorem and
x → ex x → x
Miscellaneous problems on limits.
 
C-1 Evaluate the following limits :
JA
x2
(i) im+ ( x ) (ii) im− (tan x)cosx
x →0 
x→
2

(iii) im ([x])1–x , where [ . ] denotes greatest integer function


x →1−

(iv) im+ etanx



LP

x→
2

C-2. Evaluate the following limits :


x
 1 + 2x 
(i) im (tan x)tan2x (ii) im  
 x →  1 + 3x 
A

x→
4

x 1
sec
2    x
(iii) im (1 + nx ) (iv) im  tan  + x  
x →1 x →0
 4 
NK

c
x −1
C-3. If im (1 + ax + bx 2
) = e3 , then find conditions on a, b and c.
x →1

C-4. Evaluate following limits :


SA

 n x  xn 
x n 1 + e x sin  x 
 x  e 
(i) im (ii) im
x→  n x x→  xn

[1 . 2x] + [2 . 3x] + ..... + [n . (n + 1) x]


C-5. Evaluate im , where [ . ] denotes greatest integer function.
n→ n3

x 2n − 1
C-6. If f(x) = im ,nN find range of f(x).
n →  x 2n + 1
Section (D) : Continuity at a point
 sin (a + 1) x + sin x
 x for x  0

D-1. Determine the values of a, b & c for which the function f (x) =  c for x = 0
 (x + bx2 )1/2 − x1/2
 for x  0
 b x3/2
is continuous at x = 0.
 1 − sin3 x , x  /2
 3 cos2 x

D-2. Find the values of ' a ' & ' b ' so that the function, f (x) =  a , x =  / 2 is continuous at

RI
 b (1 − sin x)
 , x  /2
 (  − 2 x)
2

x = /2.

A
D-3. If f(x) = {x} & g(x) = [x] (where {. } & [.] denotes the fractional part and the integral part functions
respectively), then discuss the continuity of :
(i) h(x) = f(x). g(x) at x = 1 and 2 (ii) h(x) = f(x) + g(x) at x = 1

UH
(iii) h(x) = f(x) – g(x) at x = 1 (iv) h(x) = g(x) + f(x) at x = 1 and 2
 f(x)
, x3
D-4. Suppose that f(x) = x − 3x − 4x + 12 and h(x) =  x − 3
3 2
, then

 K , x=3
(a) find all zeros of f
JA
(b) find the value of K that makes h continuous at x = 3
(c) using the value of K found in (b), determine whether h is an even function.

sin3x + A sin 2x + B sin x


D-5. If f(x) = (x  0) is continuous at x = 0. Find A & B. Also find f (0).
x5
LP

D-6_. If graph of function y = f(x) is

1
A

0 1 2 3
and graph of function
NK

y = g(x)is
 
SA

1 2
then discuss the continuity of f(x) g(x) at x = 3 and x = 2.

Section (E) : Continuity in an interval, Continuity of composite functions, IMVT


E-1. Find interval for which the function given by the following expressions are continuous :
3x + 7 1 x2
(i) f(x) = 2 (ii) f(x) = –
x − 5x + 6 | x | −1 2
x2 + 1  x 
(iii) f(x) = (iv) f(x) = tan  
1 + sin2 x  2 
E-2. If f(x) = x + {− x} + [x], where [ . ] is the integral part & { . } is the fractional part function. Discuss the
continuity of f in [ − 2, 2 ]. Also find nature of each discontinuity.
 
x2 + 1
E-3. If f(x) = and g(x) = tan x, then discuss the continuity of fog (x).
x2 − 1

1 + x , 0  x  2
E-4. Let f(x) =  . Determine the composite function g(x) = f (f(x)) & hence find the point
3 − x , 2  x  3
of discontinuity of g , if any.
 

RI
3 1
E-5. Find the point of discontinuity of y = f(u), where f(u) = 2
and u = .
2u + 5u – 3 x+2
 
x3 7

A
E-6. Show that the function f(x) = – sin  x + 3 takes the value within the interval
4 3
[–2, 2].

UH
1 5
E-7_. If g(x) = (|x – 1| + |4x – 11|) [x2 – 2x – 2], then find the number of point of discontinuity of g(x) in  , 
2 2

{where [.] denotes GIF}


JA
Section (F) : Derivability at a point
 
F-1. Test the continuity & differentiability of the function defined as under at x = 1 & x = 2.
 x ; x  1

f (x) =  2 − x ; 1  x  2
− 2 + 3 x − x 2 ;
LP

 x  2


 1 for −   x  0
 
F-2. A function f is defined as follows: f(x) =  1 + sin x for 0  x 
A

 2
  
2

2 +  x − for  x  
  2  2
NK

Discuss the continuity & differentiability at x = 0 & x = /2.


 
F-3. Prove that f (x) = |x| cos x is not differentiable at x = 0

 xm sin 1 ; x  0
 ( )
SA

F-4. Show that the function f (x) =  x is,


 0 ; x = 0
(i) differentiable at x = 0, if m > 1.
(ii) continuous but not differentiable at x = 0, if 0 < m  1.
(iii) neither continuous nor differentiable, if m  0.
 

1 − e − x at x = 0.
2
F-5. Examine the differentiability of f (x) =

 a x2 − b if | x |  1

F-6. If f(x) =  1 is derivable at x = 1. Find the values of a & b.
− | x | if | x |  1

Section (G) : Derivability in an interval
 

G-1. Draw a graph of the function, y = [x] + |1 − x| , −1  x  3. Determine the points, if any, where this
function is not differentiable, where [. ] denotes the greatest integer function.
 
 1
 x − ; 0  x  1
G-2. Discuss the continuity & derivability of f (x) =  2
 x . [x] ; 1 x  2

where [ x ] indicates the greatest integer x.

G-3. Discuss continuity and differentiability of y = f(x) in [–2, 5] where [.] denotes GIF & {.} denotes FPF

RI
 [x] , x  [–2, 0]
 {x} , x  (0, 2)

 x2
f(x) =  , x  [2, 3)

A
 4
 1
 , x  [3, 5]
 log4 (x − 3)

UH
  x2 
G-4. Check differentiability of f(x) = sgn (x 2/3) + cos  2 
+ |x – 1|5/3 in [–2, 2] where [.] denotes GIF.
  1 + x  

G-5_. Discuss the continuity and differentiability of h(x) = f(x) g(x) in (0, 3) if
JA
 | x − 1| + | x − 2 |
 , x  (0, 1)
 2
ex − e 
f(x) = {where [.] denot GIF} and g(x) =  | x − 1| + | x − 2 | , x  [1, 2)
[x] + 1  3 | x − 1| + | x − 2 |
 ( ) , x  [2, 3)

 2
LP

Section (H) : Functional equations and Miscellaneous


7
H-1. If f : R → R satisfies f(x + y) = f(x) + f(y), for all x, y  R and f(1) = 2, then 
r =1
f(r) is :
A

f(1 + cos x) − f(2)


H-2. If f(2) = 4 then, evaluate lim ..
x →0 tan2 x
NK

H-3. Let a function f : R → R be given by f(x + y) = f(x) f(y) for all x, y  R and f(x)  0 for any x  R. If the
function f (x) is differentiable at x = 0, show that f(x) = f(0) f(x) for all x  R. Also, determine f(x).

 1  1
H-4. Let f(x) be a polynomial function satisfying the relation f(x). f   = f(x) + f    x  R – {0} and
 
x x
SA

f(3) = –26. Determine f(1).

H-5. Let function f(x) satisfying the relation f(x + y)+ f(x – y) = 2f(x).f(y), then prove that it is even function
 
H-6. Let f(x) be a bounded function. L 1 = im (f(x) – f(x)) and L2 = im f(x) where  > 0. If L1, L2 both
x →  x → 

L
exist and L1 = L, then prove that L2 = – .

H-7. Let R be the set of real numbers and f: R→R be such that for all x & y in R
|f(x) – f(y)|  |x – y|3. Prove that f(x) is constant.
PART - II : ONLY ONE OPTION CORRECT TYPE

Section (A) : Definition of LHL/RHL and Indeterminate forms


 
A-1. im sin−1 ((sec x)) is equal to
x→0


(A) (B) 1 (C) zero (D) none of these
2
 
A-2. Consider the following statements :

RI
[x]
S1 : im− is an indeterminate form (where [ . ] denotes greatest integer function).
x →0 x
sin (3 x )
S2 : im =0
x →  3x

A
x – sin x
S3 : im does not exist.
x →  x + cos2 x

UH
(n + 2) ! + (n + 1) !
  S4 : im
n→
(n  N) =0
(n + 3) !
(n + 2) ! + (n + 1) !
S4 : im (n  N) =0 
n→ (n + 3) !
JA
  State, in order, whether S1, S2, S3, S4 are true or false
(A) FTFT (B) FTTT (C) FTFF (D) TTFT

A-3. im (1 − x + [x − 1] + [1 − x]) is equal to (where [.] denotes greatest integer function)


x →1

(A) 0 (B) 1 (C) − 1 (D) does not exist


LP

cos−1(cos x)
A-4_. lim− −1
is equal to :
x → 0 sin (sin x)

(A) 0 (B) 1 (C) –1 (D) Does not exist


 
A

SECTION (B) : Evaluation of limits of form 0/0, /,  – , 0 × , Use of L-Hospital


Rule & Expansion
NK

(x3 + 27) n (x − 2)
B-1. im is equal to
x →3 (x 2 − 9)
(A) − 8 (B) 8 (C) 9 (D) − 9
SA

(4x − 1)3
B-2. im is equal to
x →0 x  x2 
sin   n  1 + 
p  3 
(A) 9 p (n 4) (B) 3 p (n 4)3 (C) 12 p (n 4)3 (D) 27 p (n 4)2
sin ( e x − 2 − 1)
B-3. im is equal to
x→2 n (x − 1)
(A) 0 (B) − 1 (C) 2 (D) 1

sin ( n(1 + x) )
B-4. The value of im is equal to
x → 0 n(1 + sin x)
1 1
(A) 0 (B) (C) (D) 1
2 4

1 − cos 2(x − 1)
B-5. im
x →1 x −1
(A) exists and it equals 2
(B) exists and it equals − 2
(C) does not exist because x − 1 → 
(D) does not exist because left hand limit is not equal to right hand limit.

RI
3
1 + x 2 − 4 1 − 2x
B-6. The value of lim is equal to
x →0 x + x2
1 1
(A) (B) 1 (C) –1 (D) –

A
2 2

2 − 1 + cos x
B-7. The value of lim is equal to
sin2 x

UH
x →0

2 2 − 2 2
(A) (B) (C) (D)
2 4 8 8
 
cos−1 (1 − x)
JA
B-8. im + is equal to
x → 0 x
1
(A) (B) 2 (C) 1 (D) 0
2

 100 
  xk  − 100
LP

im  
k =1
B-9. is equal to
x →1 x −1
(A) 0 (B) 5050 (C) 4550 (D) − 5050

1
A

x3 sin + x + 1
B-10. im x is equal to
x→ x2 + x + 1
NK

1
(A) 0 (B) (C) 1 (D) none of these
2
 1
x 2 sin  
B-11. im  x  is equal to
x → −  2
9x + x + 1
SA

1 1
(A) (B) – (C) 0 (D) does not exist
3 3

5n + 1 + 3n − 22n
B-12. im , n  N is equal to
n →  5n + 2n + 3 2n + 3

(A) 5 (B) 3 (C) 1 (D) zero

     
B-13. im n cos   sin   , n  N is equal to:
n → 
 4n   4n 
  
(A) (B) (C) (D) none of these
3 4 6
 
x − 2 
B-14. im   is equal to (where [ . ] represents greatest integer function)
 cos x 
2 
x→

 
(A) – 1 (B) 0 (C) – 2 (D) does not exist

−3n + ( −1)n
B-15. im is equal to (n  N)
n→ 4n − ( −1)n
3 3 3
(A) – (B) – if n is even ; if n is odd
4 4 4

RI
3
(C) not exist if n is even ; – if n is odd (D) 1 if n is even ; does not exist if n is odd
4

A
 2 1 
B-16_. lim 
− 1 
+ is equal to :
x →1 1 − x 2
 x
1 1
(D) Does not exist 

UH
(A) (B) – (C) –1
2 2

  1 
im  x − x 2 n 1 +  is equal to :
x  
B-17.
x→ 
 
1 3
JA 1
(A) (B) (C) (D) 1
2 2 3
x2

2
e – cos x
B-18. im is equal to
x→0 x 3 sin x
1 1 1 1
(A) (B) (C) (D)
LP

4 6 12 8

sin(6x 2 )
B-19. im is equal to
x → 0 ncos(2x 2 − x)
(A) 12 (B) − 12 (C) 6 (D) − 6
A

sin(a + 3h) − 3 sin(a + 2h) + 3 sin(a + h) − sina


B-20_. lim is equal to :
h→0 h3
NK

(A) cosa (B) –cosa (C) sina (D) sina cosa

x n x
SECTION (C) : Limit of form 0 0 , 0 , 1 , im , im , Sandwitch theorem and
x → ex x → x
SA

Miscellaneous problems on limits.


 
x +1
 x + 2
C-1. im   is equal to
x→
 x − 2
(A) e4 (B) e −4 (C) e2 (D) none of these

C-2.
x → 0
(
im + 1 + tan2 x ) x
is equal to
(A) e5 (B) e2 (C) e (D) none of these
1
n (tan x)
C-3. The value of im (1 + [x]) is equal to (where [ . ] denotes the greatest integer function)

x→
4

(A) 0 (B) 1 (C) e (D) e−1

x
 x 2 − 2x + 1 
C-4. im  2  is equal to
x→
 x − 4x + 2 
(A) 1 (B) 2 (C) e2 (D) e

C-5. The limiting value of (cos x) sin x at x = 0 is:

RI
(A) 1 (B) e (C) 0 (D) none of these

 x 
tan  
 a  2a 
C-6. im  2 −  is equal to

A
x→a
 x
a 2a 2
− − −
  
(A) e (B) e (C) e (D) 1

UH
n
 x 
C-7. im  cos  is
n→
 n
− x2 x2
− x2 2 x2 2
(A) e (B) e (C) e (D) e
JA
1
C-8. If [x] denotes greatest integer less than or equal to x, then im
n→ n4
( [13 x] + [23 x] + ... + [n3 x] ) is
equal to
x x x x
(A) (B) (C) (D)
2 3 6 4
LP

Section (D) : Continuity at a point


cos(sin x) − cos x
D-1. A function f(x) is defined as below f(x) = , x  0 and f(0) = a
x2
f(x) is continuous at x = 0 if 'a' equals
A

(A) 0 (B) 4 (C) 5 (D) 6

 1
D-2. Let f(x) =  x +  [x] , when – 2  x  2. where [ . ] represents greatest integer function. Then
NK

 2 
(A) f(x) is continuous at x = 2 (B) f(x) is continuous at x = 1
(C) f(x) is continuous at x = – 1 (D) f(x) is discontinuous at x = 0

log(4x −3) x 2 − 2x + 5 , if
( 3  x  1 or
) x  1
D-3. The function f(x) is defined by f(x) =  4
SA

 4 , if x = 1
(A) is continuous at x = 1
(B) is discontinuous at x = 1 since f(1 +) does not exist though f(1−) exists
(C) is discontinuous at x = 1 since f(1 −) does not exist though f(1+) exists
(D) is discontinuous since neither f(1−) nor f(1+) exists.

 
 
D-4_. If f(x) = x sin  ( x + 2[x] )  , then f(x)is {where [.] denotes GIF}
 2 
(A) Discontinuous at x = 2 (B) Discontinuous at x = 1
(C) Continuous at x = 1 (D) Continuous at x = 3
Section (E) : Continuity in an interval, Continuity of composite functions, IMVT
 
 (1 + p x) − (1 − p x)
 x , −1  x  0
E-1. f (x) =  is continuous in the interval [ − 1, 1 ], then ' p' is
 2x + 1 , 0  x 1
 x − 2
equal to:
(A) − 1 (B) − 1/2 (C) 1/2 (D) 1

RI
E-2. Let f(x) = Sgn(x) and g(x) = x (x 2 – 5x + 6). The function f(g(x)) is discontinuous at
(A) infinitely many points (B) exactly one point
(C) exactly three points (D) no point
 

A
1 1
E-3. If y = where t = , then the number of points of discontinuities of y = f(x), x  R is
t2 + t − 2 x −1
(A) 1 (B) 2 (C) 3 (D) infinite

UH
 
E-4. The equation 2 tan x + 5x – 2 = 0 has
(A) no solution in [0, /4] (B) at least one real solution in [0, /4]
(C) two real solution in [0, /4] (D) None of these
 
JA
Section (F) : Derivability at a point
 
F-1. If f(x) = x ( )
x − x + 1 , then indicate the correct alternative(s):
(A) f(x) is continuous but not differentiable at x = 0
(B) f(x) is differentiable at x = 0
(C) f(x) is not differentiable at x = 0
LP

(D) none

 x(3e1/ x + 4)
 , x0
F-2. If f(x) =  2 − e1/ x , then f(x) is
A


 0 , x=0
(A) continuous as well differentiable at x = 0
(B) continuous but not differentiable at x = 0
NK

(C) neither differentiable at x = 0 nor continuous at x = 0


(D) none of these

x
F-3. If f(x) = be a real valued function, then
x +1− x
SA

(A) f(x) is continuous, but f(0) does not exist (B) f(x) is differentiable at x = 0
(C) f(x) is not continuous at x = 0 (D) f(x) is not differentiable at x = 0
 
F-4. The function f(x) = sin−1 (cos x) is:
(A) discontinuous at x = 0 (B) continuous at x = 0
(C) differentiable at x = 0 (D) none of these
 x + {x} + x sin{x} for x  0
F-5. If f(x) =  , where { . } denotes the fractional part function, then:
0 for x = 0
(A) f is continuous & differentiable at x = 0 (B) f is continuous but not differentiable at x = 0
(C) f is continuous & differentiable at x = 2 (D) none of these.
 
   [x] +2[ − x]  − 5 
   x  
log a [x] + [ − x] x  a  for x  0 ; a  1
F-6.

Given f(x) =  a
( )  1 
  x 
 3 + a 
  

 0 for x = 0
where [.] represents the integral part function, then:
(A) f is continuous but not differentiable at x = 0
(B) f is continuous & differentiable at x = 0
(C) the differentiability of 'f' at x = 0 depends on the value of a
(D) f is continuous & differentiable at x = 0 and for a = e only.  

RI
 
 x2 − 1
 , 0  x  2
x2 + 1

 , 2  x  3

A
F-7. If f (x) = 
4
(
1 x3 − x2 ) , then:

9
4 ( x − 4 + 2 − x ) , 3  x  4

UH

(A) f (x) is differentiable at x = 2 & x = 3 (B) f (x) is non-differentiable at x = 2 & x = 3
(C) f (x) is differentiable at x = 3 but not at x = 2 (D) f (x) is differentiable at x = 2 but not at x = 3.
 
Section (G) : Derivability in an interval
JA
x
G-1. The set of all points where the function f(x) = is differentiable is:
1 +|x|
(A) (−  ) (B) [ 0,  ) (C) (− , 0)  (0, ) (D) (0, )

G-2. If f (x) is differentiable everywhere, then :


(A) f  is differentiable everywhere
2
(B) f is differentiable everywhere
LP

(C) f f is not differentiable at some point (D) f + f is differentiable everywhere

G-3. Let f (x) be defined in [− 2 , 2] by


max 4 − x 2 ,
( )
1 + x2 , − 2  x  0
A


f (x) =  , then f (x) :
 (
 min 4 − x 2 , 1 + x2 ) , 0  x  2
NK

(A) is continuous at all points


(B) is not continuous at more than one point .
(C) is not differentiable only at one point
(D) is not differentiable at more than one point
G-4. The number of points at which the function f(x) = max. {a − x, a + x, b}, −  < x < , 0 < a < b cannot be
differentiable is:
SA

(A) 1 (B) 2 (C) 3 (D) none of these

max f(t), 0  t  x, 0  x  1
G-5. Let f(x) = x – x2 and g(x) =  , then in the interval [0, )
 sin x , x  1
(A) g(x) is everywhere continuous except at two points
(B) g(x) is everywhere differentiable except at two points
(C) g(x) is everywhere differentiable except at x = 1
(D) none of these

G-6. Consider the following statements :


S1 : Number of points where f(x) = | x sgn (1 – x2) | is non-differentiable is 3.
 
a sin 2 (x + 1) , x  0
S2 : Defined f(x) =  , In order that f(x) be continuous at x = 0, 'a' should be
 tan x − sin x , x  0
 x3
1
equal to
2
S3 : The set of all points, where the function 3
x 2 | x | is differentiable is (–, 0)  (0, )
1
  S4 : Number of points where f(x) = –1
is non-differentiable in the interval (0, 3) is 3.
sin (sin x)
State, in order, whether S1, S2, S3, S4 are true or false

RI
(A) TTTF (B) TTTT (C) FTTF (D) TFTT

G-7. Consider the following statements :


sin (  [x − ])

A
S1 : Let f(x) = , where [ . ] stands for the greatest integer function. Then f(x) is
1 + [x]2
discontinuous at x = n + , n  

UH
S2 : The function f(x) = p[x + 1] + q [x – 1], (where [.] denotes the greatest integer function) is
continuous at x = 1 if p + q = 0

S3 : Let f(x) = |[x] x| for – 1  x  2, where [.] is greatest integer function, then f is not differentiable
at x = 2.
JA
  S4 : If f(x) takes only rational values for all real x and is continuous, then f(10) = 10.

(A) FTTT (B) TTTF (C) FTTF (D) FFTF

G-8. For what triplets of real numbers (a, b, c) with a  0 the function
 x , x  1
LP

f(x) =  is differentiable for all real x?


2
ax + bx + c , otherwise
(A) {(a, 1−2a, a)  a  R, a  0 } (B) {(a, 1−2a, c)  a, c  R, a  0 }
(C) {(a, b, c)  a, b, c  R, a + b + c = 1 } (D) {(a, 1−2a, 0)  a  R, a  0}
A

Section (H) : Functional Equations and Miscellaneous

f(2h + 2 + h2 ) − f(2)
H-1. Given that f(2) = 6 and f(1) = 4, then lim =
NK

h→0 f(h − h2 + 1) − f(1)


(A) does not exist (B) is equal to –3/2 (C) is equal to 3/2 (D) is equal to 3

10
H-2. If f(x + y) = f(x) . f(y),  x & y  N and f(1) = 2, then the value of  f(n) is
n =1
SA

(A) 2036 (B) 2046 (C) 2056 (D) 2066

H-3. If f(1) = 1 and f(n + 1) = 2f(n) + 1 if n  1, then f(n) is equal to


(A) 2n + 1 (B) 2n (C) 2n – 1 (D) 2n–1 – 1

1
H-4. ( )
If y = f(x) satisfies the condition f x + 1x = x2 + 2 (x  0), then f(x) is equal to
x
(A) − x2 + 2 (B) − x2 − 2
(C) x2 – 2, x  R – {0} (D) x2 − 2, |x|  [2, )

H-5. A function f : R → R satisfies the condition x 2 f(x) + f(1 − x) = 2x − x4. Then f(x) is:
(A) – x2 – 1 (B) – x2 + 1 (C) x2 − 1 (D) – x4 + 1

H-6. If f: R → R be a differentiable function, such that f(x + 2y) = f(x) + f(2y) + 4xy  x, y  R. then
(A) f(1) = f(0) + 1 (B) f(1) = f(0) – 1 (C) f(0) = f(1) + 2 (D) f(0) = f(1) – 2

RI
PART - III : MATCH THE COLUMN
 
1. Let [.] denotes the greatest integer function.

A
Column – I Column – II

(A) If P(x) = [2 cos x], x  [–, ], then P(x) (p) is discontinuous at exactly 7 points

UH
(B) If Q(x) = [2 sin x], x  [–, ], then Q(x) (q) is discontinuous at exactly 4 points

  
(C) If R(x) = [2 tan x/2], x   − ,  , then R(x) (r) is non differentiable at some points
 2 2 
JA
 x  
(D) If S(x) = 3cosec  , x   , 2  , then S(x) (s) is continuous at infinitely many values
 3 2 

2. Column – I Column – II
(A) f(x) = |x3| is (p) continuous in (–1, 1)
LP

(B) f(x) = | x | is (q) differentiable in (–1, 1)

(C) f(x) = |sin–1 x| is (r) differentiable in (0, 1)

(D) f(x) = cos–1 |x| is (s) not differentiable atleast at one point in (–1, 1)
A
NK
SA
 Marked questions are recommended for Revision.
PART - I : ONLY ONE OPTION CORRECT TYPE
 | x |3  x  3 
1. im−  −    (a < 0), where [x] denotes the greatest integer less than or equal to x,
x →a  a  a  

is equal to
(A) a 2 + 1 (B) – a 2 – 1 (C) a 2 (D) – a 2

RI
x x x x x
2. im cos cos 2 cos 3 cos 4 .......cos n is equal to (x 0)
n →  2 2 2 2 2
sin x x
(A) 1 (B) − 1 (C) (D)
x sin x

A
  nsin    n tan   
3. im    +   , where [.] represents greatest integer function and n  N, is equal to
→0
       

UH
(A) 2n (B) 2n + 1 (C) 2n – 1 (D) does not exist

im  sin x 
x →0 (1 − e )
x
4.  , where [] represents greatest integer function, is equal to
 x 
JA
(A) – 1 (B) 1 (C) log 2 +1
(3 − 2 ) (D) does not exist

cos (sin x) − cos x


5. The value of im is equal to
x →0 x4
tan x 1 sin x − x 1
LP

(A) im (B) (C) im (D)


x →0 x 3 6 x →0 x3 3

sin x – (sin x)sin x


6. The value of im is
 1– sin x + n sin x
x →
2
A

(A) 1 (B) 2 (C) 3 (D) /2


NK

7. The value of im tan2 x


x →

( 2 sin2 x + 3 sin x + 4 − )
sin2 x + 6 sin x + 2 is equal to:
2

1 1 1 1
(A) (B) (C) (D)
10 11 12 8
SA

1
8. If  and  be the roots of equation ax 2 + bx + c = 0, then im (1 + ax 2 + bx + c ) x −  is equal to
x→
a(  − )
(A) a ( − ) (B) n |a ( − ) | (C) e (D) ea| −|

 n 1 1

e x  2x
 ( ) ( ) ex
− 3x
n
ex

9. im   , n  N, is equal to
x → xn
2 3
(A) 0 (B) n   (C) n   (D) none of these
3 2
 
 ay   b y  
10. im 

im
exp x ( n (1 + x )) − exp  x

n 1 + x   
  
is equal to
y→0 x→ y 
 
 
(A) a + b (B) a −b (C) b −a (D) − (a + b)

 2x −1 x 
11. The graph of the function f(x) = im  cot 2  is
t →0   t 

RI
(A) (B) (C) (D)

A
Let [x] denote the integral part of x  R and g(x) = x − [x]. Let f(x) be any continuous function with f(0) =

UH
12.
f(1), then the function h(x) = f(g(x)) :
(A) has finitely many discontinuities (B) is continuous on R
(C) is discontinuous at some x = c (D) is a constant function.


JA
 a(1 − x sin x) + bcos x + 5
 x0
x2

13. Let f(x) =  3 x=0
 1/ x
   cx + dx 3  
  1 +  x 2   x0
   
LP

If f(x) is continuous at x = 0 then find (a – b – c + ed)


(A) 0 (B) 6 (C) –6 (D) 2

x 2 if x is irrational
14. Let f(x) = 
1 if x is rational , then:
A

(A) f(x) is discontinuous for all x


(B) discontinuous for all x except at x = 0
(C) discontinuous for all x except at x = 1 or − 1
NK

(D) none of these

15. A point (x, y), where function f(x) = [sin [x]] in (0, 2) is not continuous, is ([.] denotes greatest integer
 x).
(A) (3, 0) (B) (2, 0) (C) (1, 0) (D) (4, –1)
 
SA

 (1 + sin x)t − 1
16. The function f defined by f(x) = lim   . is
 (1 + sin x)
t→ t
+ 1
(A) everywhere continuous (B) discontinuous at all integer values of x
(C) continuous at x = 0 (D) none of these
 
  1
 x  1 + x sin x  , x  0
  
  1
17. If f(x) = − − x  1 + x sin  , x  0 , then f(x) is
  x
 0 , x=0


(A) continuous as well as diff. at x = 0 (B) continuous at x = 0, but not diff. at = 0
(C) neither continuous at x = 0 nor diff. at x = 0 (D) none of these
 
18. The functions defined by f(x) = max {x2, (x − 1)2, 2x (1 − x)}, 0  x  1

RI
(A) is differentiable for all x
(B) is differentiable for all x except at one point
(C) is differentiable for all x except at two points
(D) is not differentiable at more than two points.

A
19. [x] denotes the greatest integer less than or equal to x. If f(x) = [x] [sin x] in (−1,1), then f(x) is:
(A) continuous at x = 0 (B) continuous in (−1, 0)

UH
(C) differentiable in (−1,1) (D) none
 
20. Let f(x) = [n + p sin x], x  (0, ), n  Z, p is a prime number and [x] is greatest integer less than or
equal to x. The number of points at which f(x) is not differentiable is
(A) p (B) p – 1 (C) 2p + 1 (D) 2p – 1
JA
1
21. Let f: R → R be any function and g (x) = . Then g is
f(x)
(A) onto if f is onto (B) one-one if f is one-one
(C) continuous if f is continuous (D) differentiable if f is differentiable
 
max f (t) for 0  t  x for 0  x  1
LP

22. Let f(x) = x3 − x2 + x + 1 and g(x) =,  then:


 3 −x + x for 1  x  2
2

(A) g(x) is continuous & derivable at x = 1


(B) g(x) is continuous but not derivable at x = 1
(C) g(x) is neither continuous nor derivable at x = 1
A

(D) g(x) is derivable but not continuous at x = 1

 
NK

 x + y  f(x) + f(y)
23. Let f : R → R be a function such that f  = , f(0) = 0 and f(0) = 3, then
 3  3
f(x)
(A) is differentiable in R
x
(B) f(x) is continuous but not differentiable in R
SA

(C) f(x) is continuous in R


(D) f(x) is bounded in R

 x + y  4 − 2(f(x) + f(y))
24. If a differentiable function f satisfies f  =  x, y  R, then f(x) is equal to
 3  3
1 2 8 4
(A) (B) (C) (D)
7 7 7 7
PART - II : SINGLE AND DOUBLE VALUE INTEGER TYPE
2
 im f(x) 
sin−1(1 − {x}) . cos −1(1 − {x})
, then   =
x →0 +
1. Let f(x) =
2{x} (1 − {x}) 
 im− f(x) 
 x →0 
(where {.} denotes the fractional part function)

  1  1
 x sin   + sin  2  , x  0
2. Let f (x) =  x x  , then im f(x) is equal to
x→
0
 , x =0

RI
 1 − cos x cos 2x 1 + cos3 x + 3cos2 x + 3cos x 
3. im  3  is equal to

A
+
x →0  2
x cos x + 63 
 

im f(x) exists and is finite and nonzero and im  f(x) + 3f(x) − 1  = 3, then the value of im f(x)

UH
4. If x→ x→  f 2 (x) 
 x→

is equal to

 x −1 , x  1  x +1 , x  0
5. If f(x) =  2 , g(x) =  2 and h(x) = |x|,
JA
2x − 2 , x  1 − x + 1 , x  0
then im f(g(h(x))) is equal to
x →0

sin x , x  n , n = 0, 1, 2,........


6. If f(x) =  and
2 , otherwise
x + 1 , x  0 , 2
2
LP


g(x) = 4 , x=0 , then im g (f(x)) is equal to
5 x→ 0
 , x=2

 1 1 1 1 
A

7. im  + + + ............. +  is equal to


n→  n 2 2
n +1 2
n +2 n + 2n 
2
NK

 1
8. The value of lim x 2  2  where [.] denotes G..F., is
x →0 x 

 −1
2 −1 ( )  is equal to
sin−1 x − tan−1 x 84x tan
9. im  +
→ 0  3 sin x 

SA

x
x
 

im x3
10. If x →0 = 1, then the value of (a + b) where a > 0, is
a + x (bx − sin x)

n
 5  4 
11. If f(x) =   x −    x −  + 1  , then
  
im f(0) is equal to
=1 
n → 
 ( −1)[x ]
2
if x0

12. Let f(x) =  1 . Then im 5f(x) + im+ 7f(x) equals (where [ . ] represents greatest
 n→
im if x0 x →0− x →0
 1 + xn
integer function)

 1

 e − (1 + x ) x 
13. The value of lim where [.] denotes GIF is
 x →0 tan x 
 

nx
e− n x + en x − 2 cos − k x2

RI
14. If im 2 exists and finite (n, k  N), then the least
x → 0 (sin x − tan x)
value of 4k + n  2 is :
 

A
12 n + 22 (n − 1) + 32 (n − 2) + ..... + n2 . 1 a
15. If im = where a and b are coprime numbers then
n→ 13 + 23 + 33 + ...... + n3 b
2a + 3b =

UH
n98 1
16. If im = , then the value of x equals
n→ n − (n − 1)
x x
99
JA
 4 x − 5 [x] for x  1
17. The number of points of discontinuity of f(x) = 
 cos  x  for x  1
(where [x] is the greatest integer not greater than x) in [0, 2] is

 2x 2 + 12x + 16 , −4  x  −2

18. If f(x) = 2− | x | , −2  x  1 , then the maximum length of interval for which f(|x|) is
LP


 4x − x − 2 , 1  x  13
2

continuous is

1 − sin x n (sin x)  


, x  . The value of f   so that the function is
A

19. Let f(x) = .


(  − 2 x)2 n (1 +  − 4x + 4x )
2 2
2 2

continuous at x = is  and || = 1 where    N then find product of all possible values of 
2
NK

 
(sin x + cos x)
cosecx
− x0
, 2



 a x=0
SA

20. If the function f(x) defined as f(x) =  ,



 1 2 3
 ex + ex + ex 
 1 3
, 0x
 ae−2 + x + be−1+ x 2

is continuous at x = 0 , then the value of loge1/ 7 a + 9b is :

21. The number of points of non differentiability of the function f (x) = |sin x| + sin |x| in
[–4, 4] is
 sin [x 2 ] 
 2 + ax3 + b , 0  x  1
22. If f (x) =  x − 3x + 8 is differentiable in [0 , 2] , then the value of [a + b + 6] is
 2cos  x + tan−1 x , 1  x  2

(Here [ . ] stands for the greatest integer function)

 x 2 e2( x −1) for 0  x  1


23. If f(x) =  is differentiable at x = 1 then
a sgn (x + 1) cos (2x − 2) + bx for 1  x  2
2

a3 + b3 =

{e x }n − 1

RI
24. Find number of points of non-differentiability of f(x) = lim in interval [0, 1] where {.} represents
n → {e x } n + 1

fractional part function

25. Let [x] denote the greatest integer less than or equal to x. The number of integral points in [–1, 1] where

A
f(x) = [x sin x] is differentiable are

2f(x) − 3f(2x) + f(4x)


Let f(x) be continuous at x = 0 and f(0) = 4 then value of lim

UH
26. is
x →0 x2

27. Let f : R → R is a function satisfying f(10 – x) = f(x) and f(2 – x ) = f(2 +x), x  R. If f(0) = 101, then the
minimum possible number of values of x satisfying f(x) = 101 for x  [0,30] is
JA
n
28. Find the natural number 'a' for which  f(a + k)
k =1
= 2048(2n – 1), where the function ‘f’ satisfies the

relation f(x + y) = f(x) . f(y) for all natural numbers x & y and further f(1) = 2

PART - III : ONE OR MORE THAN ONE OPTIONS CORRECT TYPE


LP

x 2 − 9x + 20
1. Let f(x) = (where [x] denotes greatest integer less than or equal to x), then
x − [x]
(A) im− f(x) = 0 (B) im+ f(x) = 1
x→5 x→5
A

(C) im f(x) does not exist (D) none of these


x→5

cos 2 − cos 2x
NK

2. If f(x) = , then
x2 − | x |
(A) im f(x) = 2 sin 2 (B) im f(x) = 2 sin 2
x →−1 x →1

(C) im f(x) = 2 cos 2 (D) im f(x) = 2 cos 2


x →−1 x →1
SA

x(1 + acos x) − b sin x 1 + a cos x b sin x


3. If  = im = im – im , where   R, then
x → 0 x3 x → 0 x2 x →0 x3
(A) (a, b) = (–1, 0) (B) a & b are any real numbers
1
(C)  = 0 (D)  =
2
|x + |
4. Let f(x) = , then
sin x
(A) f(–+) = − 1 (B) f(–−) = 1
(C) im f(x) does not exist (D) im f(x) does not exist
x → − x→
 2x
1 + , 0  x  1
5. Let f(x) =  a , if im   f(x) exists, then value of a is :   
x →1

 ax, 1  x  2
(A) 1 (B) – 1 (C) 2 (D) – 2

ax 2 + bx + c
6. Let ,  be the roots of equation ax 2 + bx + c = 0, where 1 <  <  and xim
→x = 1, then
0 ax 2 + bx + c
which of the following statements is correct
(A) a > 0 and x0 < 1 (B) a > 0 and x0 > 
(C) a < 0 and  < x0 <  (D) a < 0 and x0 < 1

RI
a0 xm + a1xm +1 + ..... + ak xm +k
7. Let  (x) = , where a0  0, b0  0 and m, n  N, then which of the following
b0 xn + b1xn+1 + .... + b x n +
statements is/are correct.

A
(A) If m > n then, im  (x) is equal to 0
x →0

a0
(B) If m = n then, im  (x) is equal to

UH
x →0 b0
a0
(C) If m < n and n – m is even, > 0, then im  (x) is equal to 
b0 x →0

a0
(D) If m < n and n – m is even, < 0, then im  (x) is equal to – 
JA x →0
b0

8. Given a real valued function f such that


 tan2 [x]
 , x0
 (x − [x] )
2 2



f(x) = 
LP

1 , x=0


 {x} cot {x}, x  0
where [.] represents greatest integer function and {.} represents fractional part function, then
A

(A) im f(x) = 1 (B) im− f(x) = cot1


x →0 x →0
2
(C) cot–1 ( im f(x)) = 1 (D) im+ f(x) = 0
NK

x →0− x →0

x2 + 2
9. If f(x) = , then
3x − 6
1 1 1 1
(A) im f(x) = – (B) im f(x) = (C) im f(x) = (D) im f(x) = –
SA

x →− 3 x → 3 x →− 3 x → 3
sin 2x + a sin x
10. If im = p (finite), then
x→0 x3
(A) a = – 2 (B) a = – 1 (C) p = – 2 (D) p = – 1

(ax + 1)n
11. im is equal to
x →  xn + A
(A) a n if n  N (B)  if n  Z – & a = A = 0
1
(C) if n = 0 (D) a n if n  Z – , A = 0 & a  0
1+ A
12. If  = im (sin x + 1 – sin x ) and m = im [sin x + 1 – sin x ], where [.] denotes the greatest
x→ x→−

integer function, then :


(A)  = 0 (B) m = 0
(C) m is undefined (D)  is undefined  

13. If f(x) = | x |sin x , then


(A) im− f(x) = 1 (B) im+ f(x) = 1
x→0 x→0

(C) im f(x) = 1 (D) limit does not exist at x = 0


x→0

RI
1
14. If
x
im
→ 0
( cos x + a sin bx ) x = e2 , then the possible values of ' a ' & ' b ' are :
(A) a = 1 , b = 2 (B) a = 2 , b = 1 (C) a = 3, b = 2/3 (D) a = 2/3 , b = 3

A
2
15. If
x→0
im (1 + ax + bx ) 2 x
= e3, then possible values of a and b is/are :

3 1 3 3 3
(A) a = 3, b = 0 (B) a = ,b= (C) a =

UH
,b= (D) a = ,b=0
2 2 2 2 2

16. im log x sin x is equal to


x → 0+ sin
2

(A) 1 (B) 0 (C) lim x sin x (D) lim+ (tan x)sin x


x →0 x →0
JA
xn
17. im = 0, n  integer number, is true for
x →  ex

(A) no value of n (B) all values of n


(C) negative values of n (D) positive values of n
LP

log (x + 2) − x 2n sin x
18. If f (x) = Limit (n  N), then
n→ x 2n + 1
(A) lim+ f(x) = –sin1 (B) lim f(x) = log3
x →1 x → 1−

log3 − sin1
A

(C) lim f(x) = sin1 (D) f(1) =


x →1 2

19. Which of the following function(s) defined below has/have single point continuity.
NK

 1 if x  Q  x if x  Q
(A) f(x) =  (B) g(x) = 
 0 if x  Q 1− x if x  Q
 x if x  Q  x if x  Q
(C) h(x) =  (D) k(x) = 
0 if x  Q − x if x  Q
SA

 
 | x −3| , x  1
 2 
20. The function f(x) =  x  3x   13  is:
  −   +   , x 1
 4   2   4 
(A) continuous at x = 1 (B) differentiable at x = 1
(C) continuous at x = 3 (D) differentiable at x = 3
1
21. If f(x) = x – 1, then on the interval [0, ]
2
1
(A) tan (f(x)) and are both continuous
f(x)
1
(B) tan (f(x)) and are both discontinuous
f(x)
(C) tan (f(x)) and f–1 (x) are both continuous
1
(D) tan (f(x)) is continuous but is not.
f(x)
 

RI
0 , x
22. Let f(x) and g(x) be defined by f(x) = [x] and g(x) =  2 (where [ . ] denotes the greatest
x , x  R − 
integer function), then

A
(A) lim g(x) exists, but g is not continuous at x = 1
x →1

(B) lim f(x) does not exist and f is not continuous at x = 1


x →1

UH
(C) gof is continuous for all x
(D) fog is continuous for all x

23. Let f(x) = [x] + x − [x] , where [ . ] denotes the greatest integer function. Then
(A) f(x) is continuous on R+ (B) f(x) is continuous on R
(C) f(x) is continuous on R – 
JA
(D) discontinuous at x = 1
 
24. The points at which the function, f(x) = x − 0.5 + x − 1 + tan x does not have a derivative in the
interval (0, 2) are:
(A) 1 (B) /2 (C) /4 (D) 1/2
 
25. f(x) = (sin-1x)². cos (1/x) if x  0; f(0) = 0, f(x) is:
LP

(A) continuous no where in −1  x  1 (B) continuous everywhere in −1  x  1


(C) differentiable no where in −1  x  1 (D) differentiable everywhere in −1 < x < 1

n
26. If f(x) = a0 + a k | x |k , where ai s are real constants, then f(x) is
A

k =1

(A) continuous at x = 0 for all ai (B) differentiable at x = 0 for all a i  R


(C) differentiable at x = 0 for all a 2k – 1 = 0 (D) none of these
NK

27. Let f : R → R be a function such that f(0) = 1 and for any x, y  R, f(xy + 1) = f(x) f(y) – f(y) – x + 2.
Then f is
(A) one-one (B) onto (C) many one (D) into

28. Suppose that f is a differentiable function with the property that f(x + y) = f(x) + f(y) + xy and
SA

1
lim f(h) = 3
h→0 h

where [.] represents greatest integer function, then


(A) f is a linear function (B) 2f(1) =  lim (1 + 2x)1/ x 
 x →0 
x2
(C) f(x) = 3x + (D) f ’(1) = 4
2

29. Let ‘f’ be a real valued function defined for all real numbers x such that for some positive constant ‘a’
1 2
the equation f(x + a) = + f(x) − ( f(x) ) holds for all x. Then f(x) is periodic function with period
2
equal to
(A) 2 a (B) 4 a (C) 6 a (D) 8 a

PART - IV : COMPREHENSION
Comprehension # 1
sin x + ae x + be− x + c n (1 + x)
Consider f(x) = , where a, b, c are real numbers.
x3
 
1. If im f(x) is finite, then the value of a + b + c is
X → 0+

(A) 0 (B) 1 (C) 2 (D) – 2

RI
2. If im f(x) = (finite), then the value of  is
X → 0+

1 1
(A) – 2 (B) – (C) – 1 (D) –

A
2 3

3. Using the values of a, b, c as found in Q.No. 1 or Q. No.2 above, the value of im+ x f(x) is
x →0

UH
1 1
(A) 0 (B) (C) – (D) 2
2 2

Comprehension # 2
JA
If both Lim− f(x) and Lim+ f(x) exist finitely and are equal , then the function f is said to have removable
x →c x →c

discontinuity at x = c
If both the limits i.e. Lim− f(x) and Lim+ f(x) exist finitely and are not equal, then the function f is said to
x →c x →c

have non-removable discontinuity at x = c and in this case | Lim+ f(x) – Lim− f(x) | is called jump of the
x →c x →c

discontinuity.
 
LP

4. Which of the following function has non-removable discontinuity at the origin ?


1  1  | sin x | 
(A) f(x) = (B) f(x) = x sin (C) f(x) = (D) f(x) = cos  
 x 
cot x
n |x| x 1 + 2
A

5. Which of the following function not defined at x = 0 has removable discontinuity at the origin ?
1
1 1 ex − 1 1
(A) f(x) = 1
(B) f(x) = tan–1 (C) f(x) = 1
(D) f(x) =
NK

1 + 2x x ex + 1 n |x|

 −1 
 tan (tan x) ; x  4
6. If f(x) =  , then jump of discontinuity is
  [x] + 1 
; x
 4
SA

(where [ . ] denotes greatest integer function)


   
(A) –1 (B) +1 (C) 1 – (D) – 1 –
4 4 4 4
Comprehension # 3
 x g(x) , x0
Let f(x) =  , where g(t) = lim (1 + a tan x)t/x, a is positive constant, then
 x + ax 2
− x 3
, x0 x → 0

7. If a is even prime number, then g(2) =


(A) e2 (B) e3 (C) e4 (D) none of these

8. Set of all values of a for which function f(x) is continuous at x = 0

RI
(A) (–1, 10) (B) (–, ) (C) (0, ) (D) none of these

9. If f(x) is differentiable at x = 0, then a 

A
(A) (–5, –1) (B) (–10, 3) (C) (0, ) (D) none of these

UH
Comprehension # 4

Let f : R → R be a function defined as,


JA
1 − | x | , | x |  1
f(x) =  and g(x) = f(x – 1) + f(x + 1),  x  R. Then
 0 , | x | 1

10. The value of g(x) is :


 0 , x  −3  0 , x  −2
2 + x 2 + x
LP

 , −3  x  −1  , −2  x  −1
 − x , −1  x  0 
 −x , −1  x  0
(A) g(x) =  (B) g(x) = 
 x , 0  x 1  x , 0  x 1
2 − x , 1 x  3 2 − x , 1 x  2
 
A

 0 , x3 
 0 , x2

 0 , x0
 0  x 1
NK

2 + x ,
 − x , 1 x  2
(C) g(x) =  (D) none of these
 x , 2x3
2 − x , 3x4

 0 , 4x
SA

11. The function g(x) is continuous for, x 


(A) R – {0, 1, 2, 3, 4} (B) R – {–2, –1, 0, 1, 2} (C) R (D) none of these

12. The function g(x) is differentiable for, x 


(A) R (B) R – {–2, –1, 0, 1, 2}
(C) R – {0, 1, 2, 3, 4} (D) none of these
PART - I : JEE (ADVANCED) / IIT-JEE PROBLEMS (PREVIOUS YEARS)
 Marked questions are recommended for Revision.
* Marked Questions may have more than one correct option.

x2
a − a2 − x 2 −
1*. Let L = im 4 , a > 0. If L is finite, then
x →0 x4
[IIT-JEE-2009, Paper-1, (4, –1), 80]

RI
1 1
(A) a = 2 (B) a = 1 (C) L = (D) L =
64 32

1
If lim 1 + x ln(1 + b2 ) x = 2b sin2 , b > 0 and   (–, ], then the value of  is

A
2.
x →0  
[IIT-JEE 2011, Paper-2, (3, –1), 80]
   

UH
(A) ± (B) ± (C) ± (D) ±
4 3 6 2

3*. Let f : R → R be a function such that f(x + y) = f(x) + f(y),  x, y  R. If f(x) is differentiable at x = 0, then
(A) f(x) is differentiable only in a finite interval containing zero [IIT-JEE 2011, Paper-1, (4, 0), 80]
JA
(B) f(x) is continuous  x  R
(C) f(x) is constant x  R
(D) f(x) is differentiable except at finitely many points

  
 −x − 2 , x−
2

 
LP

4*. If f(x) = − cos x , −  x  0 , then [IIT-JEE 2011, Paper-2, (4, 0), 80]
 2
 x −1 , 0  x 1
 x 1
 n x ,

A

(A) f(x) is continuous at x = – (B) f(x) is not differentiable at x = 0


2
3
(C) f(x) is differentiable at x = 1 (D) f(x) is differentiable at x = –
2
NK

b−x
5. Let f : (0, 1) → R be defined by f(x) = , where b is a constant such that 0 < b < 1. Then
1 − bx
[IIT-JEE 2011, Paper-2, (4, 0), 80]
1
(B) f  f–1 on (0, 1) and f(b) =
SA

(A) f is not invertible on (0, 1)


f (0)
1
(C) f = f–1 on (0, 1) and f(b) = (D) f–1 is differentiable on (0, 1)
f (0)

 x2 + x + 1 
6. If lim  − ax − b  = 4, then [IIT-JEE 2012, Paper-1, (3, –1), 70]
x →
 x +1 

  (A) a = 1, b = 4 (B) a = 1, b = –4
(C) a = 2, b = –3 (D) a = 2, b = 3
7. Let (a) and (a) be the roots of the equation ( 3
)
1+ a – 1 x2 + ( ) (
1+ a – 1 x + 6
1+ a – 1 )=0 where
a > –1. Then lim+ (a) and lim+ (a) are [IIT-JEE 2012, Paper-2, (3, –1), 66] 
a →0 a →0

5 1 7 9
(A) – and 1 (B) – and –1 (C) – and 2 (D) – and 3
2 2 2 2

 2 
 x cos , x0
8. Let f(x) =  x , x  R , then f is
0,
 x=0
(A) differentiable both at x = 0 and at x = 2 [IIT-JEE 2012, Paper-1, (3, –1), 70]

RI
(B) differentiable at x = 0 but not differentiable at x = 2
(C) not differentiable at x = 0 but differentiable at x = 2
(D) differentiable neither at x = 0 nor at x = 2
 

A
9*. For every integer n, let an and bn be real numbers. Let function f : R → R be given by  
an + sin  x, for x  [2n, 2n + 1]
  f(x) =  , for all integers n.  
 bn + cos x, for x  (2n − 1, 2n)

UH
  If f is continuous, then which of the following hold(s) for all n ? [IIT-JEE 2012, Paper-2, (4, 0), 66]
 
  (A) an–1 – bn–1 = 0 (B) an – bn = 1 (C) an – bn+1 = 1 (D) an–1 – bn = –1  
 
10*. For every pair of continuous functions f, g:[0, 1] → R such that
JA
max {f(x) : x  [0,1]} = max {g(x) : x  [0, 1]}, [JEE (Advanced) 2014, Paper-1, (3, 0)/60]
the correct statement(s) is (are) :
(A) (f(c))2 + 3f(c) = (g(c))2 + 3g(c) for some c  [0, 1]
(B) (f(c))2 + f(c) = (g(c))2 + 3g(c) for some c  [0, 1]
(C) (f(c))2 + 3f(c) = (g(c))2 + g(c) for some c  [0, 1]
(D) (f(c))2 = (g(c))2 for some c  [0, 1]
LP

1− x
 −ax + sin(x − 1) + a 1− x 1
11. The largest value of the non-negative integer a for which lim   = is
x →1
 x + sin(x − 1) − 1  4
            [JEE (Advanced) 2014, Paper-1, (3, 0)/60]  
A

12. Let f : R → R and g : R → R be respectively given by f(x) = |x| + 1 and g(x) = x 2 + 1. Define h : R → R
max {f(x),g(x)} if x  0,
NK


by h(x) =  The number of points at which h(x) is not differentiable is
 min {f(x),g(x)} if x  0.

[JEE (Advanced) 2014, Paper-1, (3, 0)/60]
SA

13. Let f1 : R → R, f2 : [0, ) → R,f3 : R → R and f4 : R → [0, ) be defined by


| x | if x  0 ,
f1(x) =  x
e if x  0 ;
f2(x) = x2 ;
sin x if x  0,
f3(x) = 
 x if x0
 f2 (f1(x)) if x0 ,
and f4(x) = 
f2 (f1(x)) – 1 if x0

List I List II
P. f4 is 1. onto but not one-one

Q. f3 is 2. neither continuous nor one-one

R. f2o f1 is 3. differentiable but not one-one

S. f2 is 4. continuous and one-one


[JEE (Advanced) 2014, Paper-2, (3, –1)/60]
P Q R S
(A) 3 1 4 2
(B) 1 3 4 2

RI
(C) 3 1 2 4
(D) 1 3 2 4

14*. Let g: R → R be a differentiable function with g(0) = 0, g'(0) = 0 and g'(1)  0. Let
 x

A
 g(x), x  0
f(x) = | x | and h(x) = e|x| for all x  R. Let (foh)(x) denote f(h(x)) and (hof)(x) denote h(f(x)).
 0,
 x=0

UH
Then which of the following is(are) true?
(A) f is differentiable at x = 0 (B) h is differentiable at x = 0
(C) foh is differentiable at x = 0 (D) hof is differentiable at x = 0
[JEE (Advanced) 2015, P-1 (4, –2)/ 88]

π π  
JA
15*. Let f(x) = sin  sin  sinx   for all x  R and g(x) = sin x for all x  R. Let (fog) (x) dentoe f(g(x))
 6  2  2
and (gof) (x) denote g(f(x)). Then which of the following is(are)true?

 1 1  1 1
(A) Range of f is − 2 , 2  (B) Range of fog is  − , 
   2 2
LP

f(x) π
(C) lim = (D) There is an x  R such that (gof)(x) = 1
x →0 g(x) 6

[JEE (Advanced) 2015, P-1 (4, –2)/ 88]


A

 ecos( n ) – e 
 = –  e  , then the value of
m
16. Let m and n be two positive integers greater than 1. If lim 
→0   m  2
NK

n
 
is [JEE (Advanced) 2015, P-2 (4, 0) / 80]

17*. Let f : R → R , g : R → R and h : R → R be differentiable functions such that f(x) = x 3 + 3x + 2,


g(f(x)) = x and h(g(g(x))) = x for all x  R. Then [JEE (Advanced) 2016, Paper-1, (4, –2)/62]
SA

1
(A) g'(2) = (B) h'(1) = 666 (C) h(0) = 16 (D) h(g(3)) = 36
15

x 2 sin(x)
18. Let ,   R be such that lim = 1 . Then 6( + ) equals
x →0 x – sin x

[JEE (Advanced) 2016, Paper-1, (3, 0)/62]


 1   1 
19*. Let f : − , 2 → R and g : − 2 , 2 → R be functions defined by f(x) = [x – 3] and
2
 2   
g(x) = |x| f(x) + |4x – 7| f(x), where [y] denotes the greatest integer less than or equal to y for y  R.
Then
 1 
(A) f is discontinuous exactly at three points in − , 2 [JEE (Advanced) 2016, Paper-2, (4, –2)/62]
 2 
 1 
(B) f is discontinuous exactly at four point in − , 2
 2 
 1 

RI
(C) g is NOT differentiable exactly at four points in  − , 2 
 2 
 1 
(D) g is NOT differentiable exactly at five points in  − , 2  .
 2 

A
20*. Let a, b  R and f : R → R be defined by f(x) = a cos (|x 3 –x|) + b|x| sin(|x3 +x|). Then f is
(A) differentiable at x = 0 if a = 0 and b = 1 [JEE (Advanced) 2016, Paper-2, (4, –2)/62]

UH
(B) differentiable at x = 1 if a = 1 and b = 0
(C) NOT differentiable at x = 0 if a = 1 and b = 0
(D) NOT differentiable at x = 1 if a = 1 and b = 1

21*. Let [x] be the greatest integer less than or equals to x. Then, at which of the following point(s) the
JA
function f(x) = x cos((x + [x])) is discontinuous ? [JEE(Advanced) 2017, Paper-1,(4, –2)/61]
(A) x = – 1 (B) x = 1 (C) x = 0 (D) x = 2

1 − x(1+ | 1 − x |)  1 
22*. Let f (x) = cos   for x  1. Then [JEE(Advanced) 2017, Paper-2,(4, –2)/61] 
| 1− x |  1− x 
(B) limx →1− f (x) does not exist  
LP

(A) limx →1+ f (x) = 0


(C) limx →1− f (x) = 0 (D) limx →1+ f (x) does not exist

23. For any positive integer n, define f n : (0, ) → R as


A

 

1
fn(x) = n
tan−1   for all x  (0, ). [JEE(Advanced) 2018, Paper-2,(4, –2)/60]
 1 + (x + j)(x + j − 1) 
j=1

  
NK

(Here, the inverse trigonometric function tan –1 x assumes values in  − ,  )


 2 2
Then, which of the following statement(s) is (are) TRUE ?
5
(A)  tan (f (0)) = 55
j=1
2
j
SA

10
(B)  (1 + f (0))sec (f (0)) = 10
j=1
j
2
j

1
(C) For any fixed positive integer n, lim tan(fn (x)) =
x → n
(D) For any fixed positive integer n, lim sec 2 (fn (x)) = 1
x →

  
    −1,e 2 − 2  → R and f4: R→R be functions defined by
24. Let f1 : R → R, f2 : ,  − ,  → R , f3 :
 2 2  
 
 2 
(i) f1 ( x ) = sin  1 − e− x  [JEE(Advanced) 2018, Paper-2,(3, –1)/60]
 

 | sin x |
 if x0
(ii) f2 ( x ) =  tan−1 x , where the inverse trigonometric function tan –1 x assumes values in
 1
 if x=0
  
 − 2 , 2 

(iii) f3(x) = [sin(loge(x+2))], where for t  R, [t] denotes the greatest integer less than or equal to t,

RI
 x2 sin 1 
   if x  0
(iv) f4 (x) =  0  x 
 if x = 0

LIST-I LIST-II

A
(P) The function f1 is (1) NOT continuous at x =0
(Q) The function f2 is (2) continuous at x = 0 and NOT differentiable at x = 0
(R) The function f3 is (3) differentiable at x = 0 and its derivative is NOT continuous

UH
at x = 0
(S) The function f4 is (4) differentiable at x= 0 and its derivative is continuous at x= 0
The correct option is:
(A) P → 2; Q → 3; R → 1; S → 4 (B) P → 4; Q → 1; R → 2; S → 3
(C) P → 4; Q → 2; R → 1; S → 3 (D) P → 2; Q → 1; R → 4; S → 3
JA
PART - II : JEE (MAIN) / AIEEE PROBLEMS (PREVIOUS YEARS)
1. Let f(x) = x|x| and g(x) = sin x [AIEEE 2009, (8, –2), 144]
Statement-1 gof is differentiable at x = 0 and its derivative is continuous at that point.
LP

Statement-2 gof is twice differentiable at x = 0.


(1) Statement-1 is True, Statement-2 is True; Statement-2 is a correct explanation for Statement-1.
(2) Statement-1 is True, Statement-2 is True; Statement-2 is NOT a correct explanation for Statement-1
(3) Statement-1 is True, Statement-2 is False
A

(4) Statement-1 is False, Statement-2 is True


NK

f(3x) f(2x)
2. Let f : R → R be a positive increasing function with lim = 1. Then lim .
x → f(x) x → f(x)

[AIEEE– 2010, (8, –2), 144]


2 3
(1) (2) (3) 3 (4) 1
3 2
SA

 1 − cos {2(x − 2)} 


3. lim   [AIEEE– 2011, , (4, –1), 120]
x →2  x−2 
 
1
(1) does not exist (2) equals 2 (3) equals – 2 (4) equals
2

(f(x))2 − 9
4. Let f: R → [0, ) be such that lim f(x) exists and lim =0 [AIEEE– 2011, II,(4, –1), 120]
x →5 x →5
| x −5|
Then lim f(x) equals : 
x →5

(1) 0 (2) 1 (3) 2 (4) 3


 sin(p + 1)x + sin x
 , x0
 x
5. The value of p and q for which the function f(x) =  q , x = 0 is continuous for all x in

 x+x − x
2
, x0
 x3 / 2
R, are : [AIEEE 2011, I,(4, –1), 120] 
1 3 5 1 3 1 1 3
(1) p = , q = – (2) p = , q = (3) p = – , q = (4) p = , q =
2 2 2 2 2 2 2 2

RI
 1
sin , If x  0
6. Define F(x) as the product of two real functions f 1(x) = x, x  R, and f2(x) =  x

 0 , If x = 0

A
as follows :
f1(x) . f2 (x) , If x  0
F(x) =  [AIEEE 2011, ,(4, –1), 120]

UH
 0 , If x = 0
Statement - 1 : F(x) is continuous on R.
Statement - 2 : f1(x) and f2(x) are continuous on R.
(1) Statement-1 is true, Statement-2 is true; Statement-2 is a correct explanation for Statement-1.
(2) Statement-1 is true, Statement-2 is true; Statement-2 is NOT a correct explanation for Statement-1
(3) Statement-1 is true, Statement-2 is false
JA
(4) Statement-1 is false, Statement-2 is true
 
x 2 f(a) − a2 f(x)
7. If function f(x) is differentiable at x = a, then lim is : [AIEEE 2011, ,(4, –1), 120]
x →a x−a
(1) –a2f ’(a) (2) af(a) – a2f ’ (a) (3) 2af(a) – a2f ’ (a) (4) 2af(a) + a2f ’ (a)
 
LP

 2x − 1 
8. If f : R → R is a function defined by f(x) = [x] cos   , where[x] denotes the greatest integer
 2 
function, then f is : [AIEEE- 2012, (4, –1), 120]
(1) continuous for every real x.
A

(2) discontinuous only at x = 0.


(3) discontinuous only at non-zero integral values of x.
(4) continuous only at x = 0.
NK

9. Consider the function, f(x) = |x – 2| + |x – 5|, x  R . [AIEEE- 2012, (4, –1), 120]
Statement-1 : f(4) = 0
Statement-2 : f is continuous in [2, 5], differentiable in (2, 5) and f(2) = f(5).
SA

(1) Statement-1 is false, Statement-2 is true.


(2) Statement-1 is true, statement-2 is true; statement-2 is a correct explanation for Statement-1.
(3) Statement-1 is true, statement-2 is true; statement-2 is not a correct explanation for Statement-1.
(4) Statement-1 is true, statement-2 is false.
 
(1– cos 2x)(3 + cos x)
10. lim is equal to [AIEEE - 2013, (4, –1),360]
x →0 x tan 4x
1 1
(1) – (2) (3) 1 (4) 2
4 2
sin(  cos2 x)
11. lim is equal to : [JEE(Main) 2014, (4, – 1), 120]
x →0 x2
(1) – (2)  (3) /2 (4) 1

(1– cos 2x)(3 + cos x)


12. lim is equal to [JEE(Main) 2015, (4, – 1), 120]
x →0 x tan 4x
1
(1) 4 (2) 3 (3) 2 (4)
2

RI
k x + 1 , 0  x  3
13. If the function g(x) =  is differentiable, then the value of k+ m is;
 mx + 2 , 3  x  5
[JEE(Main) 2015, (4, – 1), 120]

A
16 10
(1) 2 (2) (3) (4) 4
5 3
1

UH
14. Let p = lim
x →0 +
(1 + tan 2
x ) 2x then log p is equal to: [JEE(Main) 2016, (4, – 1), 120]

1 1
(1) 1 (2) (3) (4) 2
2 4
JA
15. For x  R, f(x) = |log2 – sinx| and g(x) = f(f(x)), then [JEE(Main) 2016, (4, – 1), 120]
(1) g(0) = cos(log2)
(2) g(0) = –cos(log2)
(3) g is differentiable at x = 0 and g(0) = –sin(log2)
(4) g is not differentiable at x = 0
LP

cot x – cos x
16. lim equals [JEE(Main) 2017, (4, – 1), 120]
x→
 (  – 2 x )3
2

1 1 1 1
(1) (2) (3) (4)
A

24 16 8 4

  1  2  15  
17. For each tR let [t] be the greatest integer less than or equal to t. Then lim+ x    +   + ...... +   
NK

x →0
   
x x  x 
[JEE(Main) 2018, (4, – 1), 120]
(1) is equal to 120 (2) does not exist (in R) (3) is equal to 0 (4) is equal to 15

18. Let S = {t  R : f(x) = |x – |. (e|x| – 1) sin|x| is not differentiable at t.} Then the set S is equal to :
SA

                  [JEE(Main) 2018, (4, – 1), 120]


(1) {} (2) {0, } (3)  (an empty set) (4) {0}

1+ 1+ y 4 − 2
19. lim [JEE(Main) 2019, Online (09-01-19),P-1 (4, – 1), 120]
y →0
y4
1 1
(1) exists and equals (2) exists and equals
2 2 2 2 ( 2 + 1)
1
(3) exists and equals (4) does not exist
4 2
20. For each tR, let [t] be the greatest integer less than or equal to t. Then ,
 
(1− | x | + sin | 1 − x |)sin  [1 − x] 
lim 2  [JEE(Main) 2019, Online (10-01-19),P-1 (4, – 1), 120]
x →1+ | 1 − x | [1 − x]
(1) does not exist (2) equals 1 (3) equals –1 (4) equals 0

– 1, – 2  x  0
21. Let f(x) =  2 and g(x) = |f(x)| + f(|x|). Then, in the interval (–2, 2), g is :
x – 1, 0  x  2

RI
(1) not differentiable at two point (2) not continuous
(3) not differentiable at one point (4) differentiable at all points
[JEE(Main) 2019, Online (11-01-19),P-1 (4, – 1), 120]

A
UH
JA
LP
A
NK
SA
EXERCISE - 1
PART – I
Section (A) :
A-1. (i) Limit does not exist (ii) 3 (iii) 3 (iv) 3 (v) 3
3 3 e
A-2. (i) 2 + sin 2 (ii) tan 3 – 23 (iii) cos (iv) 55 (v)
4 4 sin1

RI
A-3. (i) 0 (ii) Limit does not exist (iii) Limit does not exist (iv) 0

A-4. (i) Limit does not exist (ii)  = –2 A-5. 6

(iii) Yes, 0 form(iv)

A
A-6. (i) No (ii) No No
 
SECTION (B) :

UH
3 12 2
B-1. (i) – (ii) (iii)
2 19 3 3

16 1
B-2. (i) (ii) 2 (iii) (iv) 2a sina + a2 cos a
JA
25 3
3 1
(v) (b – a) (vi) 2e2 (vii) (viii) (ix) 5
n3 2
9 4
(x) limit does not exist (xi) − n
4 e
LP

3
1 1 5
B-3. (i) (ii) 1 (iii)  (iv) − B-4. (i) 0 (ii) ( a + 2 )2 (iii) 0 (iv) 5/2
2  2

2 1 1
B-5. (i) – (ii) B-6 a = 2, b = 1, c = –1 and limit = –
A

25 3 3

1
B-7. (i) a=− , b = 1 (ii) a = 2, b  R, c = 5, d  R (iii) a = 3, b = 12, c = 9
NK

2
1
B-8. B10. 2(sec2a)tana
2

SECTION (C) :
 
SA

C-1 (i) 1 (ii) 1 (iii) 0 (iv) 0

2


C-2. (i) e–1 (ii) 0 (iii) e (iv) e2 C-3 a + b = 0 and bc = 3

 n x
n 1 +
x 
x
 x
C-4. (i) im 1 (ii) 1 C-5. C-6. {–1, 0, 1}
x→  n x 3
Section (D) :
3 1 1
D-1. a=− , b  0, c = D-2. a = ,b=4
2 2 2
D-3. (i) continuous at x = 1 (ii) continuous (iii) discontinuous (iv) continuous at x = 1, 2

D-4. (a) −2, 2, 3 (b) K=5 (c) even

D-5. A = − 4, B = 5, f(0) = 1 D6. Continuous at x = 1 but discontinuous at x = 2

Section (E) :
E-1. (i) x  R – {2, 3} (ii) x  R – {– 1, 1} (iii) xR (iv) x  R – {(2n + 1), n  }

 

RI
E-2. discontinuous at all integral values in [− 2, 2] E-3. discontinuous at n ± , (2n + 1) ,n
4 2

E-4. g(x) = 2 + x ; 0  x  1,
= 2 − x ; 1 < x  2,

A
= 4 − x ; 2 < x  3,
7
g is discontinuous at x = 1 & x = 2 E-5. – , – 2, 0 E-7_. 2
3

UH
Section (F) :
 
F-1. continuous at both points but differentiable only at x = 2
 
JA
F-2. continuous but not differentiable at x = 0; differentiable & continuous at x = /2
 
F-5. not differentiable at x = 0 F-6. a = 1/2, b = 3/2

Section (G) :
 
LP

G-1. f is not derivable at all integral values in −1 < x  3

G-2. f is continuous but not derivable at x = 1/2, f is neither differentiable nor continuous at x = 1 & x = 2

G-3. discontinuous and non-differentiable at –1, 0, 1, 3, 4


A

G-4. Differentiable in [–2, 2]


NK

G-5_. Continuos everwhere in (0, 3) but non differentiable at x= 2

Section (H) :
H-1. 56 H-2. –2 H-3. f(x) = exf(0)  x  R H-4. – 3
SA

PART – II

Section (A) :
 
A-1. (D) A-2. (A) A-3. (C) A-4. (C)
 
SECTION (B) :

B-1. (C) B-2. (B) B-3. (D) B-4. (D) B-5. (D) B-6. (A) B-7. (D)
 
B-8. (B) B-9. (B) B-10. (C) B-11. (B) B-12. (D) B-13. (B) B-14. (C)
B-15. (A) B16. (A) B-17. (A) B-18. (C) B-19. (B) B-20_. (B)

SECTION (C) :
 
C-1. (A) C-2. (A) C-3. (B) C-4. (C) C-5. (A) C-6. (C) C-7. (B)

C-8. (D)

Section (D) :
D-1. (A) D-2. (D) D3. (D) D-4_. (B)

RI
Section (E) :
 

A
E-1. (B) E-2. (C) E-3. (C) E-4. (B)
 
Section (F) :

UH
 
F-1. (B) F-2. (B) F-3. (B) F-4. (B) F-5. (D) F-6. (B) F-7. (B)

Section (G) :
 
JA
G-1. (A) G-2. (B) G-3. (D) G-4. (B) G-5. (C) G-6. (A) G-7. (C)

G-8. (A)

Section (H) :
LP

H-1. (D) H-2. (B) H-3. (C) H-4. (D) H-5. (B) H-6. (D)
PART – III

1. (A) → (p, r, s), (B) → (p, r, s), (C) → (q, r, s), (D) → (r, s)
(A) → (p, q, r), (B) → (p, r, s), (C) → (p, r, s), (D) → (p, r, s)
A

2.

EXERCISE - 2
NK

PART – I
1. (B) 2. (C) 3. (C) 4. (A) 5. (B) 6. (B) 7. (C)

8. (C) 9. (B) 10. (B) 11. (C) 12. (B) 13_. (B) 14. (C)
 
15. (D) 16. (B) 17. (B) 18. (C) 19. (B) 20. (D) 21. (B)
SA

22. (C) 23. (C) 24. (D)


PART – II
1. 2 2. 1 3. 2 4. 1 5. 0 6. 1 7. 2

8. 1 9. 11 10. 37 11. 20 12. 12 13. 1 14. 21


 
15. 11 16. 99 17. 4 18. 26 19. 36 20. 16 21. 7
 
22. 4 23. 7 24. 0 25. 3 26. 12 27. 11 28. 10
PART – III
1. (ABC) 2. (AB) 3. (AD) 4. (ABCD) 5. (BC) 6. (ABC)

7. (ABCD) 8. (BCD) 9. (AB) 10. (AD) 11. (ABCD) 12. (AC)

13. (ABC) 14. (ABCD) 15. (BCD) 16. (AD) 17. (BCD) 18. (ABD)

19. (BCD) 20. (ABC) 21. (CD) 22. (ABC) 23. (ABC) 24. (ABD)
 
25. (BD)    26. (AC) 27. (AB) 28. (BCD) H-9. (ABCD)

PART – IV

RI
1. (A) 2. (D) 3. (A) 4. (C) 5. (D) 6. (C)

7. (C) 8. (C) 9. (C) 10. (B) 11. (C) 12. (B)

A
EXERCISE - 3
PART – I
1*. (AC) 2. (D) 3. (B, C, D) or (B,C) 4*. (ABCD) 5. (A) 6. (B)

UH
7. (B) 8. (B) 9*. (BD) 10*. (AD) 11. 0 12. 3 13. (D)

14*. (AD) 15*. (ABC) 16. 2 17. (B,C) 18. 7 19. (B,C) 20. (A,B)

21. (ABD) 22*. (CD) 23. (D) 24. (D)


JA
PART - II
1. (3) 2. (4) 3. (1)    4. (4) 5. (3) 6. (3) 7. (3)
 
8. (1) 9. (3) 10. (4) 11. (2) 12. (3) 13. (1) 14. (2)
LP

15. (1) 16. (2) 17. (1) 18. (3) 19. (3) 20. (4) 21. (3)
A
NK
SA
1– cos(a1x) . cos (a 2 x). cos (a3 x).......cos(an x)
1. Evaluate : im , where a1, a2, a3, ...... , anR.
x →0 x2
x
2. f1 (x) = + 10
2
fn (x) = f1 (fn–1 (x)) n2
then evaluate lim fn(x)
n→

RI
3. Let f : R → R be a real function. The function f is derivable and there exists nN and p  R such that
im xn f(x) = p, then evaluate im (xn+1.f(x)).
x → x →

n
x 

A
4. Let <xn> denotes a sequence, x1 = 1, xn+1 = x n2 + 1 , then evaluate im  n +1 
n→
 xn 
 1 2 3 n 
5. Evaluate im  + 2 + + ....... + 2 

UH
n→  n2 + 1 n + 2 n2 + 3 n +n

6. Evaluate : im x3
x→  x2 + 1 + x 4 − x 2
JA 
loge ( loge x )
7. Evaluate im
x → x
e

8. Evaluate im loge ( sin ( 4m + 1) x ) , where m,n Z



x→
2 loge ( sin ( 4n + 1) x )

n
x x
 tan 2 .sec 2
LP

9. f(x) = r r −1
r, n  N
r =1
n
 x   f(x) + tan x    tan x  
loge  f(x) + tan n −
2n    2  
  sin
  2   
 lim x
n→ n

A

 x  4
 1 +  f(x) + tan n 
g(x) =   2 


NK

 
 k x=
4
 
where [ ] denotes greatest integer function and domain of g(x) is  0, 
 2
find 'k' for which g(x) is continuous at x = /4
SA

 
n(1+ nx )

im e x
10. Evaluate −1 +
x →(e )
x − e−1

23 − 1 3 3 − 1 n3 − 1 2
11. Let Pn = 3
. 3
........... . 3
Prove that im Pn = .
2 + 1 3 + 1 n + 1 n →  3
12. Verify the following limits
1
1
 x   
sec 2   a2
 (1 + x)x  –
1
 2      2 – bx  –
b2
(i) im = e 2
(ii) im  sin   = e
x→0  
 e  x→0
  2 − ax  
 
x sinn x
13. f(x) = im . Find domain and range of f(x), where n  N.
n →  sinn x + 1

1
 a x + a2 x  x
14. Evaluate im  1x  where a1, a2, b1 and b2 are positive numbers
x →0  x 
 b1 + b2 

RI
 p q 
15. Evaluate im  −  where p, q  N
x →1  1 − x p 1 − xq 

n
 

A

16. If f(n, ) =  1 − tan
r =1
2
im f (n,) = g(), then find the value im of g()
 and n→
2r  →0

UH
1  4cos2 x 
17. Find the value of lim  − 2
x → (x − )  2 + cos x 

18. im xa
x →
( 3
x +1+ 3
x −1 − 2 3
)
x =  ,   0 then find the value of a + 
JA
(1 + sin x)n + log x
19. Discuss the continuity of the function f(x) = lim
n→ 2 + (1 + sin x)n

 1– a x + xa x . na
 , x0
 x 2ax
LP


20. If g(x) =  k , x=0
 (2a)x – x n 2a − 1
 , x0

 x2
(where a > 0) , then find ‘a’ and g(0) so that g(x) is continuous at x = 0.
A

 
 cos−1(2x 1 − x 2 ) 1
 x
 1
NK

x− 2
 2
21. f(x) = 

 1
 k x=
 2
SA

1
Then find ‘k’ if possible for which function is continuous at x =
2

22. Find the value of f(0) so that the function


cos−1(1 − {x} 2 )sin−1(1 − {x})
f(x) = ,x0
{x} − {x} 3
({x} denotes fractional part of x) becomes continuous at x = 0
 
 1  x2
= ( sine x ) e− x + 2
2
23. Let f be a continuous function on R such that f   , then find the value of
 4x  x +1
f(0).
24. Examine the continuity at x = 0 of the sum function of the infinite series:
x x x
+ + + ..............  .
x +1 (x + 1)(2x + 1) (2x + 1)(3x + 1)

25. If f(x) is continuous in [a, b] such that f(a) = b and f(b) = a, then prove that there exists at least one c 
(a, b) such that f(c) = c.

26. If f(x . y) = f(x). f(y) for all x, y and f(x) is continuous at x = 1. Prove that f(x) is continuous for all x except
possibly at x = 0. Given f(1)  0.
 
xn f(x) + h(x) + 1
27. g(x) = im , x1

RI
n→ 2xn + 3x + 3
sin2 (  2x )
g(1) = im be a continous function at x = 1, then find the value of 4g(1) + 2 f(1) – h(1),
loge sec(  2x )
x →1

assume that f(x) and h(x) are continuous at x = 1

A
28. If f(x) = x2 – 2|x|, then test the derivability of g(x) in the interval [–2, 3], where
min.{f(t); –2  t  x} , –2  x  0

UH
g(x) = 
max. {f(t); 0  t  x} , 0  x  3

29. Discuss the continuity and differentiability of f(x) = [x] + {x} 2 and also draw its graph. Where [.] and {.}
denote the greatest integer function and fractional part function respectively.
 
JA
 x
1+ | x | ; | x |  1

30. Discuss the continuity and differentiability of the function f(x) = 
 x ;| x| 1
1− | x |
LP

 
  cos2n (m! x) − 1  
31. Discuss the continuity and differentiability of the function f(x) =  lim  lim  ,
n → m → cos2n (m! x) + 1
   
A

(where m, n  N) at rational and irrational points.

  2[x]  
NK

32. Given f(x) = cos−1  sgn    , where sgn ( ) denotes the signum function and [ . ] denotes the
  3x − [x]  
greatest integer function. Discuss the continuity and differentiability of f (x) at x = ± 1.
 
33. Discuss the continuity on 0  x  1 & differentiability at x = 0 for the function.
SA

1 1 1
f(x) = x sin sin where x  0, x  & f(0) = f (1/r) = 0, r = 1, 2, 3,.......
x 1 r
x sin
x

34. Let f be a function such that f(xy) = f(x) . f(y)  x > 0 , y > 0 . If f(1 + x) = 1 + x (1+g(x)) ,
f(x)
where lim g(x) = 0 . Find  dx
x →0 f '(x)

35. Let f : R+ → R satisfies the equation


f(xy) = exy – x – y (ey f(x) + ex f(y))  x , y  R+
If f ’ (1) = e , then find f(x) .
36. Let f(x) be a real valued function not identically zero such that
f(x + y3) = f(x) + (f(y))3  x, y  R and f ’(0)  0, then find f(10)

 1  2 (1 − 2x)
37. Determine a function f satisfying the functional relation f(x) + f   = .
1 − x  x (1 − x)

38. If f (x) + f (y) + f (xy) = 2 + f (x) . f (y) , for all real values of x and y and f (x) is a polynomial function
with f (4) = 17 and f(1)  1, then find the value of f (5) .

RI
k
p k(k – 1)
39. If | f(p + q) – f(q)| 
q
for all p and q  Q & q  0, show that | f(2 ) – f(2 ) | 
i =1
k i

A
40. The function f : R → R satisfies x + f(x) = f(f(x)) for every xR. Find all solutions of the equation
f(f(x)) = 0.

UH
41. If 2f (x) = f(xy) + f(x/y)  x, y  R+, f(1) = 0 and f(1) = 1, find f(x).

 1
f(x)
JA
42. If f(x × f(y)) =  x, y  R , y  0, then prove that f(x) . f   = 1
y x

43. Find the period of f(x) satisfying the condition :


(i) f(x + p) = 1 + {1 – 3 f(x) + 3 f2(x) – f3 (x)}1/3, p > 0
LP

(ii) f(x – 1) + f(x + 3) = f(x + 1) + f(x + 5)

44. Let f(x) is defined only for x  (0, 5) and defined as f 2 (x) = 1  x  (0, 5). Function f(x) is continuous for
all x  (0, 5) – {1, 2, 3, 4} (at x = 1, 2, 3, 4 f(x) may or may not be continuous). Find the number of
A

possible function f(x) if it is discontinuous at


(i) One integral points in (0, 5)
NK

(ii) two integral points in (0, 5)


(iii) three integral points in (0, 5)
(iv) four integral points in (0, 5)
SA

45. Let f(x) is increasing and double differentiable function everywhere such that f(x) = x has 3 distinct root
,  and ( <  < ). h(x) = lim ( f(f(....(f(x))) )
n →
n times

(i) If f”(x) > 0  x  (–, ) and f”(x) < 0  x  (, ] and f”() = 0, then find h(x)
(ii) If f(x)  x  x  (–, ]  [, ) and f(x)  x  x  [, ] then find h(x)
n
1 1 1
a
2
1. i 2. 20 3. – np 4. e 5. 6.
2 i =1 2 4 2
2
( 4m + 1)
7. 0 8. 2
9. k=0 10. 0
( 4n + 1)
     
13. Domain = R – 2k − , k  Z  , Range = {0}  k + , k  Z 
 2   4 
a1a2 p−q 13

RI
14. 15. 16. 1 17. 0 18.
b1b2 2 9
1 1
19. f(x) is discontinuous at integral multiples of  20. , (n 2)2
2 8

A
22. no value of f(0) 23. 1 24. Discontinuous 27. 5
28. discontinuous at x = 0 and not differentiable at x = 0, 2
29. f(x) is continuous and non-differentiable for integral points

UH
30. At x = 0 differentiable and at x = ±1 discontinuous
31. discontinuous and non-differentiable
32. f is continuous & derivable at x = − 1 but f is neither continuous nor derivable at x = 1
33. continuous in 0  x  1 & not differentiable at x = 0
x2 x +1
34. +c 35. f(x) = exn|x| 36. f(10) = 10 37. 38. 26
JA
2 x −1
41. f(x) = log(x) 43. (i) 2p (ii) 8 44. (i) 24 (ii) 108 (iii) 216 (iv) 162
 , x  ( −, )  , x  ( −, ]
 
45. (i) h(x) =   , x= (ii) h(x) =   , (,  )
  , x  (, )  , [ , )
 
LP
A
NK
SA
MATRIX & DETERMINANTS
JEE (Advanced) Syllabus
Matrices and Determinant : As a rectangular array of real numbers, equality of matrices, addition,
multiplication by a scalar and product of matrices, transpose of a matrix, determinant of a square matrix of order
up to three, inverse of a square matrix of order up to three, properties of these matrix operations, diagonal,
symmetric and skew-symmetric matrices and their properties, solutions of simultaneous linear equations in two
or three variables.Determinant of a square matrix of order up to three.

JEE (Main) Syllabus

RI
Matrices and Determinant : Algebra of matrices, types of matrices, determinants and matrices of order two
and three. Properties of determinants, evaluation of determinants, area of triangles using determinants. Adjoin
and evaluation of inverse of a square matrix using determinants and elementary transformations, Test of
consistency and solution of simultaneous linear equations in two or three variables using determinants and

A
matrices.

As for everything else, so for a mathematical theory, beauty can be perceived but not explained..... Cayley

UH
Arthur

Introduction :
Any rectangular arrangement of numbers (real or complex) (or of real valued or complex valued
expressions) is called a matrix. If a matrix has m rows and n columns then the order of matrix is
written as m × n and we call it as order m by n
JA
The general m × n matrix is

 a11 a12 a13 ...... a1j ..... a1n 


a a22 a23 ...... a2 j ..... a2n 
 21
..... ..... ..... ..... ..... ..... ..... 
A=  
LP

 ai1 ai2 ai3 ...... aij ...... ain 


..... ..... ..... ..... ..... ..... ..... 
 
am1 am2 am3 ..... amj ..... amn 

where aij denote the element of ith row & jth column. The above matrix is usually denoted as [aij]m × n .
A

Notes :
(i) The elements a11, a22, a33,........ are called as diagonal elements. Their sum is called as trace
NK

of A denoted as tr(A)

(ii) Capital letters of English alphabets are used to denote matrices.

(iii)
Order of a matrix : If a matrix has m rows and n columns, then we say that its order is "m by n",
written as "m × n".
SA

1
Example # 1 : Construct a 3 × 2 matrix whose elements are given by a ij = | i –3j |.
2
a11 a12 
Solution : In general a 3 × 2 matrix is given by A = a21 a22 
a31 a32 
1
aij = | i – 3j |, i = 1, 2, 3 and j = 1, 2
2
1 1 5
Therefore a11 = | 1 – 3 × 1 | = 1 a12 = |1–3×2|=
2 2 2
1 1 1
a21 = |2–3×1|= a22 = |2–3×2|=2
2 2 2
1 1 3
a31 = |3–3×1|=0 a32 = |3–3×2|=
2 2 2

 5
1 2
 
Hence the required matrix is given hy A =  2
1
2 
 
0 3
 2 

Types of Matrices :

RI
Row matrix :
A matrix having only one row is called as row matrix (or row vector).General form of row
matrix is A = [a11, a12, a13, ...., a1n]
This is a matrix of order "1 × n" (or a row matrix of order n)

A
Column matrix :
A matrix having only one column is called as column matrix (or column vector).

UH
 a11 
a 
Column matrix is in the form A =  
21

 ... 
 
am1 
JA
This is a matrix of order "m × 1" (or a column matrix of order m)

Zero matrix :
A = [aij]m × n is called a zero matrix, if aij = 0  i & j.
0 0 0 
LP

0 0 0 
e.g. : (i)   (ii)  0 0 0 

0 0 0 
 0 0 0 

Square matrix :
A

A matrix in which number of rows & columns are equal is called a square matrix. The general
form of a square matrix is
 a11 a12 ....... a1n 
NK

a a22 ........ a2n 


A= 
21
which we denote as A = [aij]n.
....... ....... ....... .......
 
 an1 an2 ....... ann 
This is a matrix of order "n × n" (or a square matrix of order n)
SA

Diagonal matrix :
A square matrix [aij]n is said to be a diagonal matrix if aij = 0 for i  j. (i.e., all the elements of
the square matrix other than diagonal elements are zero)

Note : Diagonal matrix of order n is denoted as Diag (a 11, a22, ......ann).

 a 0 0 0 
a 0 0   
e.g. : (i)  0 b 0  0 b 0 0
 (ii)  
 0 0 0 0 
 0 0 c   
 0 0 0 c 
Scalar matrix :
Scalar matrix is a diagonal matrix in which all the diagonal elements are same. A = [a ij]n is a
scalar matrix, if (i) aij = 0 for i  j and (ii) aij = k for i = j.
a 0 0 
a 0 
e.g. : (i)   (ii)  0 a 0 
0 a   0 0 a 

Unit matrix (identity matrix) :


Unit matrix is a diagonal matrix in which all the diagonal elements are unity. Unit matrix of
order 'n' is denoted by n (or ).
A = [aij]n is a unit matrix when aij = 0 for i  j & aii = 1

RI
i.e.
1 0 0
1 0 0 1 0 
eg. 2 =   , 3 =  
 0 1 0 0 1

A
Upper triangular matrix :

UH
A = [aij]m × n is said to be upper triangular, if aij = 0 for i > j (i.e., all the elements below
the diagonal elements are zero).

a b c d  a b c 
e.g. : (i)  0 x y z  (ii)  0 x y 
JA
 0 0 u v   0 0 z 

Lower triangular matrix :


A = [aij]m × n is said to be a lower triangular matrix, if aij = 0 for i < j. (i.e., all the
elements above
LP

the diagonal elements are zero.)


a 0 0  a 0 0 0 
e.g. : (i)  b c 0  (ii)  b c 0 0 
 x y z   x y z 0 
A

Comparable matrices :
Two matrices A & B are said to be comparable, if they have the same order (i.e., number of rows of A &
B are same and also the number of columns).
NK

 2 3 4  3 4 2
e.g. : (i) A =   & B=  are comparable
 3 −1 2  0 1 3 
 3 0 
 2 3 4 
e.g. : (ii) C =   & D =  4 1  are not comparable
 3 −1 2 
SA

 2 3 
Equality of matrices :
Two matrices A and B are said to be equal if they are comparable and all the corresponding elements
are equal.
Let A = [aij] m × n & B = [bij]p × q
A = B iff (i) m = p, n = q
(ii) aij = bij  i & j.
 sin  1/ 2  1/ 2 sin  
   
Example # 2 : Let A =  −1/ 2 cos   & B =  cos  cos  . Find  so that A = B.
  
 cos  tan    cos  −1 
Solution : By definition A & B are equal if they have the same order and all the corresponding elements
are equal.
1 1
Thus we have sin  = , cos = – & tan  = – 1
2 2

  = (2n + 1)  – .
4
 x + 3 z + 4 2y – 7   0 6 3y – 2
Example # 3 : If  –6 a – 1 0  =  –6 –3 2c + 2  , then find the values of a, b, c, x, y and z.

RI
b – 3 –21 0  2b + 4 –21 0 
Solution : As the given matrices are equal, therefore, their corresponding elements must be equal.
Comparing the corresponding elements, we get

A
x+3=0 z+4=6 2y – 7 = 3y – 2
a–1=–3 0 = 2c + 2 b – 3 = 2b + 4
 a = – 2, b = – 7, c = – 1, x = – 3, y = – 5, z = 2

UH
Multiplication of matrix by scalar :
Let  be a scalar (real or complex number) & A = [a ij]m × n be a matrix. Thus the product A is
defined as A = [bij]m × n where bij = aij  i & j.
 2 −1 3 5   −6 3 −9 −15 
e.g. : A =  0 2 1 −3  & – 3A  (–3) A =  0 −6 −3 9 
JA

 0 0 −1 −2   0 0 3 6 

Note : If A is a scalar matrix, then A = , where  is a diagonal entry of A

Addition of matrices :
LP

Let A and B be two matrices of same order (i.e. comparable matrices). Then A + B is defined to be.
A + B = [aij]m × n + [bij]m × n.
= [cij]m × n where cij = aij + bij  i & j.

 1 −1   −1 2  0 1
A

e.g. : A =  2 3  , B =  −2 −3  , A + B = 0 0 
 
 1 0   5 7   6 7 
NK

Substraction of matrices :
Let A & B be two matrices of same order. Then A – B is defined as A + (– B) where – B is (– 1) B.

Properties of addition & scalar multiplication :


Consider all matrices of order m × n, whose elements are from a set F (F denote Q, R or C).
Let Mm × n (F) denote the set of all such matrices.
SA

Then
(a) A  Mm × n (F) & B  Mm × n (F)  A + B  Mm × n(F)
(b) A+B=B+A
(c) (A + B) + C = A + (B + C)
(d) O = [o]m × n is the additive identity.
(e) For every A  Mm × n(F), – A is the additive inverse.
(f)  (A + B) = A + B
(g) A = A
(h) (1 + 2) A = 1A + 2A
8 0  2 –2
Example # 4 : IF A =  4 –2 and B =  4
  2 , then find the matrix X, such that 2A + 3X = 5B
 3 6   –5 1
Solution : We have 2A + 3X = 5B.
 3X = 5B – 2A
1
 X= (5B – 2A)
3
  2 –2 8 0     10 –10   –16 0  
1   1 
 X=   
5 4 2 – 2  4 –2  =   20 10  +  –8 4  
3   3
  –5 1 3 6     –25 5   –6 –12 
   
 –10 

RI
 –2 3 
 10 – 16 –10 + 0   –6 –10   
X =  20 – 8 10 + 4  =  12 14  =  4
1 1 14 

3 3 3 
 –25 – 6 5 – 12   −31 –7   

A
 –31 –7 
 3 3 

UH
Multiplication of matrices :
Let A and B be two matrices such that the number of columns of A is same as number of rows
of B. i.e., A = [aij]m × p & B = [bij]p × n.
p
Then AB = [cij]m × n where cij =  aik bkj , which is the dot product of i th row vector of A and jth
JA
k =1

column vector of B.
0 1 1 1
 1 2 3   0 0 1 0  , AB =  3 4 9 1 
e.g. : A =   , B =    1 3 7 2 
2 3 1  1 1 2 0   
LP

Notes :
(1) The product AB is defined iff the number of columns of A is equal to the number of rows of B. A
is called as premultiplier & B is called as post multiplier. AB is defined BA is defined.
(2) In general AB  BA, even when both the products are defined.
A

(3) A (BC) = (AB) C, whenever it is defined.

Properties of matrix multiplication :


NK

Consider all square matrices of order 'n'. Let Mn (F) denote the set of all square matrices of
order n. (where F is Q, R or C). Then
(a) A, B  Mn (F)  AB  Mn (F)
(b) In general AB  BA
SA

(c) (AB) C = A(BC)


(d) n, the identity matrix of order n, is the multiplicative identity.
An = A = n A  A  Mn (F)
(e) For every non singular matrix A (i.e., |A|  0) of M n (F) there exist a unique (particular)
matrix B  Mn (F) so that AB = n = BA. In this case we say that A & B are multiplicative
inverse of one another. In notations, we write B = A –1 or A = B–1.
(f) If  is a scalar (A) B = (AB) = A(B).
(g) A(B + C) = AB + AC  A, B, C  Mn (F)

(h) (A + B) C = AC + BC  A, B, C  Mn (F).
Notes : (1) Let A = [aij]m × n. Then An = A & m A = A, where n & m are identity matrices of order
n & m respectively.
(2) For a square matrix A, A2 denotes AA, A3 denotes AAA etc.
 1 2 3
Example # 5 : If A = 3 –2 1 , then show that A3 – 23A – 40 I = O
 4 2 1
 1 2 3  1 2 3 19 4 8 
Solution : We have A2 = A.A = 3 –2 1 3 –2 1 =  1 12 8 
 4 2 1  4 2 1 14 6 15 

RI
 1 2 3  1 2 3 19 4 8 
So A3 = AA2 = 3 –2 1 3 –2 1 =  1 12 8 
   
 4 2 1  4 2 1 14 6 15 
 1 2 3  1 2 3 1 0 0

A
Now A3 – 23A – 40I = 3 –2 1 – 23 3 –2 1 – 40 0 1 0 
   
 4 2 1  4 2 1 0 0 1

UH
63 46 69   –23 –46 –69   –40 0 0 
= 69 –6 23  +  –69 46 –23  +  0 –40 0 
92 46 63   –92 –46 –23   0 0 –40 
JA
63 – 23 – 40 46 – 46 + 0 69 – 69 + 0  0 0 0 
=  69 – 69 + 0 –6 + 46 – 40 23 – 23 + 0  = 0 0 0  = O
 90 – 92 + 0 46 – 46 + 0 63 – 23 – 40  0 0 0 

Self practice problems :


LP

cos  − sin  
(1) If A() =   , verify that A() A() = A( + ).
 sin  cos  
Hence show that in this case A(). A() = A() . A().
A

 4 6 −1  2 4
(2) Let A = 3 0 2  , B =  0 1 and C = [3 1 2].
 1 −2 5   −1 2 
NK

Then which of the products ABC, ACB, BAC, BCA, CAB, CBA are defined. Calculate the
product whichever is defined.
Answer (2) Only CAB is defined. CAB = [25 100]

Transpose of a matrix :
SA

Let A =[aij]m × n. Then the transpose of A is denoted by A( or AT) and is defined as

A = [bij]n × m where bij = aji  i & j.

i.e. A is obtained by rewriting all the rows of A as columns (or by rewriting all the columns of A as
rows).
 1 a x 
 1 2 3 4   2 
e.g. : A =  a b c d  , A = 
b y 
 3 c z 
 x y z w   
 4 d w 
Results : (i) For any matrix A = [aij]m × n, (A) = A

(ii) Let  be a scalar & A be a matrix. Then (A) = A


(iii) (A + B) = A + B & (A – B) = A – B for two comparable matrices A and B.
(iv) (A1 ± A2 ± ..... ± An) = A1 ± A2 ± ..... ± An, where Ai are comparable.

(v) Let A = [aij]m × p & B = [bij]p × n , then (AB) = BA

(vi) (A1 A2 .......An) = An. An – 1 ...........A2 . A1, provided the product is defined.

RI
Symmetric & skew-symmetric matrix : A square matrix A is said to be symmetric if A = A
i.e. Let A = [aij]n. A is symmetric iff aij = aji  i & j. A square matrix A is said to be skew-
symmetric if A = – A
i.e. Let A = [aij]n. A is skew-symmetric iff aij = – aji  i & j.

A
a h g
e.g. A = h b f  is a symmetric matrix.

UH
 g f c 
o x y
B =  − x o z  is a skew-symmetric matrix.

 − y −z 0 
JA
Notes :
(1) In a skew-symmetric matrix all the diagonal elements are zero.
( aii = – aii  aii = 0)
(2) For any square matrix A, A + A is symmetric & A – A is skew-symmetric.

(3) Every square matrix can be uniquely expressed as a sum of two square matrices of
LP

which one is symmetric and the other is skew-symmetric.


1 1
A = B + C, where B = (A + A) & C = (A – A).
2 2

 –2
A

Example # 6 : If A =  4  , B = [1 3 – 6], verify that (AB)' = B'A'.


 5 
NK

Solution : We have
 –2
A =  4  , B = [1 3 –6]
 5 
 –2  –2 –6 12 
SA

Then AB =  4  [1 3 –6] =  4 12 –24 


 5   5 15 –30 
 1
Now A' = [–2 4 5], B' =  3 
 –6 
 1  –2 4 5
B'A' =  3  [–2 4 5] =  –6 12 15  = (AB)'

 –6   12 –24 –30 
Clearly (AB)' = B'A'
 2 –2 –4 
Example # 7 : Express the matrix B =  –1 3 4  as the sum of a symmetric and a skew symmetric
 1 –2 –3 
matrix.
 2 –1 1
Solution : Here B' =  –2 3 –2
 –4 4 –3 
 3 3
 2 –
2
– 
2
 4 –3 –3   
=  – 1 
1 
2
1 3
Let P= (B + B') = –3 6 3
2 2  2 

RI
 –3 2 –6   
– 3
1 –3 
 2 
 –3 –3 
 2 2 

A
2
 
P' =  1  =P
–3
Now 3
2 
 

UH
 –3
1 –3 
 2 
1
Thus P= (B + B') is a symmetric matrix.
2
JA
 –1 –5 
0 2 2 
0 –1 –5   
Also, Let
1
Q = (B – B') =
1 
1 0  
6 = 
1
0 3 
2 2  2 
5 –6 0   
 5 –3 0 
 2 
LP

 1 5
 0 2 3
 
Q' =  – –3  = – Q
1
Now 0
2 
 
A

– 5
3 0
 2 
1
Thus Q= (B – B') is a skew symmetric matrix.
NK

2
 –3 –3   –1 –5 
 2 2 2  0 2 2 
     2 –2 –4 
P + Q =  1  + 3  =  –1 3 4  = B
–3 1
Now 3 0
2  2  
     1 –2 –3 
SA

 –3 1 –3   5 –3 0 
 2   2 
Thus, B is represented as the sum of a symmetric and a skew symmetric matrix.

Thus, B is represented as the sum of a symmetric and a skew symmetric matrix.

Example # 8 : Show that BAB is symmetric or skew-symmetric according as A is symmetric or skew-


symmetric (where B is any square matrix whose order is same as that of A).
Solution : Case -  A is symmetric  A = A
(BAB) = (B)AB = BAB  BAB is symmetric.
Case -  A is skew-symmetric  A = – A
(BAB) = (B)AB
= B ( – A) B
= – (BAB)
 BAB is skew-symmetric

Self practice problems :


(3) For any square matrix A, show that AA & AA are symmetric matrices.
(4) If A & B are symmetric matrices of same order, then show that AB + BA is symmetric and
AB – BA is skew-symmetric.
Submatrix : Let A be a given matrix. The matrix obtained by deleting some rows or columns of A is called
as submatrix of A.
a b c d  a c  a b c 
     a b d  
eg. A =  x y z w  Then  x z  ,   ,  x y z  are all submatrices of A.
p r    
p q s

RI
p q r s  p q r 

Determinant of a square matrix :


To every square matrix A = [aij] of order n, we can associate a number (real or complex) called

A
determinant of the square matrix.
Let A = [a]1×1 be a 1×1 matrix. Determinant A is defined as |A| = a.
e.g. A = [– 3]1×1 |A| = – 3

UH
a b 
Let A =   , then |A| is defined as ad – bc.
 c d
 5 3
e.g. A=   , |A| = 23
 −1 4 
JA
Minors & Cofactors :
Let  be a determinant. Then minor of element a ij, denoted by Mij, is defined as the determinant
of the submatrix obtained by deleting i th row & jth column of . Cofactor of element aij, denoted
by Cij, is defined as Cij = (– 1)i + j Mij.
a b
e.g. 1 =
LP

c d
M11 = d = C11
M12 = c, C12 = – c
M21 = b, C21 = – b
A

M22 = a = C22

a b c
NK

e.g. 2 = p q r
x y z
q r
M11 = = qz – yr = C11.
y z
a b
SA

M23 = = ay – bx, C23 = – (ay – bx) = bx – ay etc.


x y

Determinant of any order :


Let A = [aij]n be a square matrix (n > 1). Determinant of A is defined as the sum of products of elements
of any one row (or any one column) with corresponding cofactors.
 a11 a12 a13 
e.g.1 A = a21 a22 a23 
a31 a32 a33 
|A| = a11C11 + a12 C12 + a13C13 (using first row).
a22 a23 a21 a23 a21 a22
= a11 – a12 + a13
a32 a33 a31 a33 a31 a32
|A| = a12 C12 + a22 C22 + a32C32 (using second column).
a21 a23 a11 a13 a11 a13
= – a12 + a22 – a32
a31 a33 a31 a33 a21 a23

Transpose of a determinant : The transpose of a determinant is the determinant of transpose of the


corresponding matrix.
a1 b1 c1 a1 a2 a3
D = a2 b2 c2  T
D = b1 b2 b3
a3 b3 c3 c1 c 2 c3

RI
Properties of determinant :
(1) |A| = |A| for any square matrix A.

A
i.e. the value of a determinant remains unaltered, if the rows & columns are inter changed,
a1 b1 c1 a1 a2 a3

UH
i.e. D = a2 b2 c 2 = b1 b2 b3 = D
a3 b3 c3 c1 c 2 c3
(2) If any two rows (or columns) of a determinant be interchanged, the value of determinant is
changed in sign only.
a1 b1 c1 a2 b2 c 2
JA
e.g. Let D1 = a2 b2 c 2 & D2 = a1 b1 c1 Then D2 = – D1
a3 b3 c3 a3 b3 c3

(3) Let  be a scalar. Than  |A| is obtained by multiplying any one row (or any one column)
of |A| by 
a1 b1 c1 Ka1 Kb1 Kc1
LP

D = a2 b2 c 2 and E = a2 b2 c2 Then E = KD
a3 b3 c3 a3 b3 c3
(4) | AB | = | A | | B |.
(5) |A| = n |A|, when A = [aij]n.
A

(6) A skew-symmetric matrix of odd order has determinant value zero.


(7) If a determinant has all the elements zero in any row or column, then its value is zero,
NK

0 0 0
i.e. D = a2 b2 c 2 = 0.
a3 b3 c3
(8) If a determinant has any two rows (or columns) identical (or proportional), then its value
is zero,
SA

a1 b1 c1
i.e. D = a1 b1 c1 = 0.
a3 b3 c3
(9) If each element of any row (or column) can be expressed as a sum of two terms then the
determinant can be expressed as the sum of two determinants, i.e.
a1 + x b1 + y c1 + z a1 b1 c1 x y z
a2 b2 c2 = a2 b2 c2 + a2 b2 c2
a3 b3 c3 a3 b3 c3 a3 b3 c3

(10) The value of a determinant is not altered by adding to the elements of any row (or column) a
constant multiple of the corresponding elements of any other row (or column),
a1 b1 c1 a1 + ma2 b1 + mb2 c1 + mc 2
i.e. D1 = a2 b2 c 2 and D2 = a2 b2 c2 . Then D2 = D1
a3 b3 c3 a3 + na1 b3 + nb1 c 3 + nc1

(11) Let A = [aij]n. The sum of the products of elements of any row with corresponding
cofactors of any other row is zero. (Similarly the sum of the products of elements of
any column with corresponding cofactors of any other column is zero).

a b c
Example # 9 Simplify b c a
c a b
Solution : Let R 1 → R 1 + R2 + R3

RI
a+b+c a+b+c a+b+c 1 1 1
 b c a = (a + b + c) b c a
c a b c a b

A
Apply C1 → C 1 – C 2 , C2 → C 2 – C 3
0 0 1
= (a + b + c) b − c c − a a

UH
c −a a −b b
= (a + b + c) ((b – c) (a – b) – (c – a)2)
= (a + b + c) (ab + bc – ca – b2 – c2 + 2ca – a2)
= (a + b + c) (ab + bc + ca – a2 – b2 – c2)  3abc – a3 – b3 – c3
JA
a b c
Example # 10 Simplify a2 b2 c 2
bc ca ab
Solution : Given determinant is equal to
a2 b2 c2 a2 b2 c2
1
a3 b3 c 3 = a3 b3 c3
LP

=
abc
abc abc abc 1 1 1
Apply C1 → C1 – C2, C2 → C 2 – C3
a −b 2 2
b −c
2 2
c2
A

= a3 − b3 b3 − c 3 c3
0 0 1
a+b b+c c2
NK

= (a – b) (b – c) a + ab + b b + bc + c 2
2 2 2
c3
0 0 1
= (a – b) (b – c) [ab2 + abc + ac2 + b3 + b2C + bc2 – a2b – a2c – ab2 – abc – b3 – b2c]
= (a – b) (b – c) [c(ab + bc + ca) – a(ab + bc + ca)]
SA

= (a – b) (b – c) (c – a) (ab + bc + ca)

Self practice problems


0 b−a c −a
(5) Find the value of  = . a−b 0 c −b
a−c b−c 0
b2 − ab b − c bc − ac
(6) Simplify . ab − a2 a − b b2 − ab
bc − ac c − a ab − a2
a−b−c 2 a 2 a
(7) Prove that 2 b b−c −a 2 b = (a + b + c)3.
2 c 2 c c −a−b

1 a bc
(8) Show that 1 b ca = (a – b) (b – c) (c – a) by using factor theorem .
1 c ab
Answers : (5) 0 (6) 0

Application of determinants : Following examples of short hand writing large expressions are:
(i) Area of a triangle whose vertices are (x r, yr); r = 1, 2, 3 is:

RI
x1 y1 1
1
D= x2 y 2 1 If D = 0 then the three points are collinear.
2
x3 y3 1

A
x y 1
(ii) Equation of a straight line passing through (x 1, y1) & (x2, y2) is x1 y1 1 = 0
x2 y2 1

UH
(iii) The lines : a1x + b1y + c1 = 0........ (1)
a2x + b2y + c2 = 0........ (2)
a3x + b3y + c3 = 0........ (3)
a1 b1 c1
JA
are concurrent if, a2 b2 c 2 = 0.
a3 b3 c3
Condition for the consistency of three simultaneous linear equations in 2 variables.
(iv) ax² + 2 hxy + by² + 2 gx + 2 fy + c = 0 represents a pair of straight lines if:
a h g
abc + 2 fgh − af² − bg² − ch² = 0 = h b f
LP

g f c

Singular & non singular matrix : A square matrix A is said to be singular or non-singular according as |A|
is zero or non-zero respectively.
A

Cofactor matrix & adjoint matrix : Let A = [aij]n be a square matrix. The matrix obtained by replacing
each element of A by corresponding cofactor is called as cofactor
matrix of A, denoted as cofactor A. The transpose of cofactor matrix of
NK

A is called as adjoint of A, denoted as adj A.


i.e. if A = [aij]n
then cofactor A = [cij]n when cij is the cofactor of aij  i & j.
Adj A = [dij]n where dij = cji  i & j.
SA
Properties of cofactor A and adj A :
(a) A . adj A = |A| n = (adj A) A where A = [aij]n.
(b) |adj A| = |A|n – 1, where n is order of A. In particular, for 3 × 3 matrix, |adj A| = |A| 2
(c) If A is a symmetric matrix, then adj A are also symmetric matrices.
(d) If A is singular, then adj A is also singular.

Example # 11 : For a 3×3 skew-symmetric matrix A, show that adj A is a symmetric matrix.
 0 a b  c 2 −bc ca 
   
Solution : A =  −a 0 c  cof A =  −bc b2 −ab 
 −b −c 0   ca −ab a2 
 
c 2
−bc ca 
 
adj A = (cof A) =  −bc b2 −ab  which is symmetric.

RI
 ca −ab a2 
 

Inverse of a matrix (reciprocal matrix) :

A
1
Let A be a non-singular matrix. Then the matrix adj A is the multiplicative inverse of A (we call it inverse of
|A|
A) and is denoted by A–1. We have A (adj A) = |A| n = (adj A) A

UH
 1   1 
 A  adj A  = n =  adj A  A, for A is non-singular
| A |  | A | 
1
 A–1 = adj A.
|A|
JA
Remarks :
1. The necessary and sufficient condition for existence of inverse of A is that A is non-singular.
2. A–1 is always non-singular.
3. If A = dia (a11, a22, ....., ann) where aii  0  i, then A–1 = diag (a11– 1, a22–1, ...., ann–1).

4. (A–1) = (A)–1 for any non-singular matrix A. Also adj (A) = (adj A).
LP

5. (A–1)–1 = A if A is non-singular.
1 –1
6. Let k be a non-zero scalar & A be a non-singular matrix. Then (kA) –1 = A .
k
1
A

7. |A–1| = for |A|  0.


|A|
8. Let A be a non-singular matrix. Then AB = AC  B = C & BA = CA  B= C.
NK

9. A is non-singular and symmetric  A–1 is symmetric.


10. (AB)–1 = B–1 A–1 if A and B are non- singular.
11. In general AB = 0 does not imply A = 0 or B = 0. But if A is non-singular and AB = 0, then B = 0.
Similarly B is non-singular and AB = 0  A = 0. Therefore, AB = 0  either both are singular or one of
them is 0.
SA

1 3 3 
Example # 12 : If A = 1 4 3  , then verify that A adj A = | A | . Also find A –1
1 3 4 
Solution : We have | A | = 1 (16 – 9) – 3 (4 – 3) + 3 (3 – 4) = 1  0
Now C11 = 7, C12 = – 1, C13 = – 1, C21 = – 3, C22 = 1, C23 = 0,C31 = – 3, C32 = 0, C33 = 1
 7 –3 –3 
Therefore adj A =  –1 1 0 
 –1 0 1 
1 3 3   7 –3 –3  7 – 3 – 3 –3 + 3 + 0 –3 + 0 + 3 
Now A(adj A) = 1 4 3   –1 1 0  = 7 – 4 – 3 –3 + 4 + 0 –3 + 0 + 3 
   
1 3 4   –1 0 1  7 – 3 – 4 –3 + 3 + 0 –3 + 0 + 4 
1 0 0 1 0 0
= 0 1 0  = (1) 0 1 0  = |A|. I
0 0 1 0 0 1
 7 –3 –3   7 –3 –3 
adj A =  –1 1 0  =  –1 1 0 
1 1  
Also A–1 =
|A| 1
 –1 0 1   –1 0 1 

2 3 

RI
Example # 13 : Show that the matrix A =  2
 satisfies the equation A – 4A + I = O, where I is 2 × 2 identity
 1 2
matrix and O is 2 × 2 zero matrix. Using the equation, find A –1 .
2 3  2 3   7 12 

A
Solution : We have A2 = A.A =    1 2 =  4 7 
 1 2     
 7 12   8 12   1 0  0 0 
Hence A2 – 4A + I =   –   +   =   =0
4 7   4 8   0 1 0 0 

UH
Now A2 – 4A + I = 0
Therefore A A – 4A = – I
or AA(A–1) – 4 A A–1 = – I A–1 (Post multiplying by A–1 because |A|  0)
or A (A A–1) – 4I = – A–1 or AI – 4I = – A–1
4 0  2 3   2 –3 
JA
or A–1 = 4I – A =   –  =  
0 4   1 2  –1 2 
 2 –3 
Hence A–1 =  
 –1 2 

Example # 14 : For two non-singular matrices A & B, show that adj (AB) = (adj B) (adj A)
We have (AB) (adj (AB)) = |AB| n
LP

Solution :
= |A| |B| n
A–1 (AB)(adj (AB)) = |A| |B| A–1
1
 B adj (AB) = |B| adj A ( A–1 = adj A)
|A|
A

 B–1 B adj (AB) = |B| B–1 adj A


 adj (AB) = (adjB) (adj A)
NK

Self practice problems :

(9) If A is non-singular, show that adj (adj A) = |A| n – 2 A.


(10) Prove that adj (A–1) = (adj A)–1.
For any square matrix A, show that |adj (adj A) | = | A |(n−1) .
2
(11)
SA

(12) If A and B are non-singular matrices, show that (AB) –1 = B–1 A–1.

Elementary row transformation of matrix :


The following operations on a matrix are called as elementary row transformations.
(a) Interchanging two rows.
(b) Multiplications of all the elements of row by a nonzero scalar.
(c) Addition of constant multiple of a row to another row.
Note : Similar to above we have elementary column transformations also.
Remarks : Two matrices A & B are said to be equivalent if one is obtained from other using elementary
transformations. We write A  B.
Finding inverse using Elementry operations
(i) Using row transformations :
If A is a matrix such that A–1 exists, then to find A–1 using elementary row operations,
Step I : Write A = IA and

Step II : Apply a sequence of row operation on A = IA till we get, I = BA.


The matrix B will be inverse of A.
Note : In order to apply a sequence of elementary row operations on the matrix equation X = AB, we
will apply these row operations simultaneously on X and on the first matrix A of the product AB on RHS.

(ii) Using column transformations :


If A is a matrix such that A–1 exists, then to find A–1 using elementary column operations,
Step I : Write A = AI and
Step II : Apply a sequence of column operations on A = AI till we get, I = AB.
The matrix B will be inverse of A.
Note : In order to apply a sequence of elementary column operations on the matrix equation X = AB, we

RI
will apply these row operations simultaneously on X and on the second matrix B of the product AB on
RHS.

0 1 2 

A
Example # 15 : Obtain the inverse of the matrix A =  1 2 3  using elementary operations.
 
3 1 1
0 1 2  1 0 0

UH
Solution : Write A = IA, i.e.  1 2 3  , = 0 1 0  A
3 1 1 0 0 1
 1 2 3 0 1 0
or    0  A (applying R1  R2)
0 1 2  =  1 0
JA
3 1 1 0 0 1
1 2 3 0 1 0
or 0 1 2  =  1
 0 0  A (applying R3 → R3 – 3R1)

0 –5 –8  0 –3 1
 1 0 –1  –2 1 0
LP

or 0 1 2  = 1 0 0  A (applying R1 → R1 – 2R2)
  
0 –5 –8   0 –3 1
 1 0 –1  –2 1 0
or 0 1 2  =  1 0 0  A (applying R3 → R3 + 5R2)
A

  
0 0 2   5 –3 1
 
 –2 1 0
NK

 1 0 –1  
0 1 2  = A  1 0  A (applying R3 → R3)
1
or  
0
 2
0 0 1  
5 –3 1
 2 2 2 
SA

1 1 1
2 – 2 2
1 0 0  
or 0 1 2  =  1 0 0  A (Applying R1 → R1 + R3)
   
0 0 1  
5 – 3 1
 2 2 2 
1 –1 1
2 2 2
1 0 0  
or 0 1 0  =  –4 3 –1 A (Applying R2 → R2 – 2R3)
   
0 0 1  
5 –3 1
 2 2 2 
1 1 1
 2 –2 2
 
Hence A =  –4 3 –1
–1 
 
 5 –3 1
 2 2 2 

RI
System of linear equations & matrices : Consider the system
a11 x1 + a12x2 + .......... + a1nxn = b1

A
a21x1 + a22 x2 + ..........+ a2n xn = b2
.................................................
am1x1 + am2x2 + ..........+ amnxn = bn.

UH
 b1 
 a11 a12 .......... a1n  x1  b 
a a22 .......... a 2n 
x   2
A=    & B =  ...  .
21 2
Let ,X=
..... ..... .......... ..... 
....  
 
   ... 
am1 am2 .......... amn 
 xn  bn 
JA
Then the above system can be expressed in the matrix form as AX = B.
The system is said to be consistent if it has atleast one solution.

System of linear equations and matrix inverse:


If the above system consist of n equations in n unknowns, then we have AX = B where A is a square
matrix.
LP

Results :
(1) If A is non-singular, solution is given by X = A –1B.
(2) If A is singular, (adj A) B = 0 and all the columns of A are not proportional, then the system has
infinitely many solutions.
(3) If A is singular and (adj A) B  0, then the system has no solution (we say it is inconsistent).
A

Homogeneous system and matrix inverse :


If the above system is homogeneous, n equations in n unknowns, then in the matrix form it is AX = O.
( in this case b1 = b2 = ....... bn = 0), where A is a square matrix.
NK

Results :
(1) If A is non-singular, the system has only the trivial solution (zero solution) X = 0
(2) If A is singular, then the system has infinitely many solutions (including the trivial solution) and
hence it has non-trivial solutions.
SA

x+y+z=6
Example # 16 : Solve the system x − y + z = 2 using matrix inverse.
2x + y − z = 1
1 1 1  x 6 
Solution : Let A =  1 −1 1  , X =  y  & B =
  2 .
 
2 1 −1  z   1
Then the system is AX = B.
|A| = 6. Hence A is non singular.
0 3 3 
Cofactor A = 2 −3 1 
2 0 −2
0 2 2 
adj A = 3 −3 0 
3 1 −2
0 2 2   0 1/ 3 1/ 3 
1 
3 −3 0   0 
1
–1
A = adj A = = 1/ 2 −1/ 2
|A| 6 
3 1 −2 1/ 2 1/ 6 −1/ 3 
 0 1/ 3 1/ 3  6  x  1
 0  2  y  = 2

RI
X= A–1 B = 1/ 2 −1/ 2   i.e.      x = 1, y = 2, z = 3.
1/ 2 1/ 6 −1/ 3   1  z  3 

Self practice problems:

A
0 1 2 
(13) A =  1 2 3  . Find the inverse of A using |A| and adj A.
3 1 1

UH
(14) Find real values of  and µ so that the following systems has
(i) unique solution (ii) infinitely many solutions (iii) No solution.
x+y+z=6
x + 2y + 3z = 1
x + 2y + z = µ
JA
(15) Find  so that the following homogeneous system have a non zero solution
x + 2y + 3z = x
3x + y + 2z = y
2x + 3y + z = z
1 1 1

2 2 2
LP

Answers : (13) −4 3 −1 (14) (i)   3, µ  R (ii)  = 3, µ = 1 (iii)  = 3, µ  1 (15)  = 6


5 3 1

2 2 2
A
NK
SA
 Marked questions are recommended for Revision.

PART - I : SUBJECTIVE QUESTIONS

Section (A): Matrix, Algebra of Matrix, Transpose, symmetric and skew symmetric
matrix
 
A-1. Construct a 3 × 2 matrix whose elements are given by a ij = 2i – j.

RI
 x − y 1 z   −1 1 4 
A-2. If   =  , find x, y, z, w.
 2x − y 0 w   0 0 5 

A
 4 –1  0 –1 0 –2
A-3. Let A + B + C =   , 4A + 2B + C =  –3 2  and 9A + 3B + C =  2 1  then find A
0 1     

UH
 1 2 
 4 5 6 
If A =  3 −4  and B = 
7 −8 2 
A-4. , will AB be equal to BA. Also find AB & BA.
 5 6   JA
 3 −4   7 −12 
A-5. If A =   , then show that A =  3 −5 
3

 1 −1   
 
  
 0 − tan
2   cos  − sin  
A-6. If A =   show that ( + A) = ( – A)  
LP

 tan  0   sin  cos  


 2 

cos x − sin x 0 
Given F(x) =  sin x cos x 0  . If x  R Then for what values of y, F(x + y) = F(x) F(y).
A

A-7.
 0 0 1
NK

A-8. Let A = [aij]n × n where aij = i2 – j2 . Show that A is skew-symmetric matrix.


 
 1 4 6  0 2 3   1 7 9
A-9. If C = 7 2 5   –2 0 4   4 2 8  , then trace of C + C 3 + C5 + ........ + C99 is
  
SA

9 8 3   –3 –4 0  6 5 3 
 
Section (B) : Determinant of Matrix
0 1 sec 
B-1. If the minor of three-one element (i.e. M31) in the determinant tan  − sec  tan  is 1 then find the
1 0 1
value of . (0    ).
B-2. Using the properties of determinants, evalulate:
23 6 11 0 c b
(i) 36 5 26 (ii) −c 0 a
63 13 37 −b −a 0

103 115 114 113 116 104 13 + 3 2 5 5


(iii) 111 108 106 + 108 106 111 . (iv) 15 + 26 5 10
104 113 116 115 114 103 3 + 65 15 5

B-3. Prove that :


1 1 1
c = (a − b) (b − c) (c − a) (a + b + c)

RI
(i) a b
a3 b3 c 3
a b + c a2

A
(ii) b c + a b2 = − (a + b + c) (a − b) (b − c) (c − a)
c a + b c2
b+c a a

UH
(iii) b c +a b = 4 abc
c c a +b
1 a2 a 4 1 1 1
(iv) If 1 b2 b 4 = (a + b) (b + c) (c + a) a b c .
JA
1 c2 c4 a2 b2 c2

B-4. If a, b, c are positive and are the p th, qth, rth terms respectively of a G.P., show without expanding that,
loga p 1
logb q 1 = 0.
LP

logc r 1

B-5. Find the non − zero roots of the equation,


a b ax + b 15 − 2 x 11 10
(i) = b c bx + c = 0. (ii) 11 − 3 x 17 16 = 0
A

ax + b bx + c c 7 − x 14 13
NK

S0 S1 S2
B-6. If Sr = r + r + r then show that S1 S2 S3 = ( − )2 ( − )2 ( − )2.
S 2 S3 S 4

a1 l1 + b1 m1 a1 l2 + b1 m2 a1 l3 + b1 m3
SA

B-7. Show that a2 l1 + b2 m1 a2 l2 + b2 m2 a2 l3 + b2 m3 = 0.


a3 l1 + b3 m1 a3 l2 + b3 m2 a3 l3 + b3 m3

ex sin x
B-8. If = A + Bx + Cx2 + ....., then find the value of A and B.
cos x n(1 + x)
Section (C) : Cofactor matrix, adj matrix and inverse of matrix

 2 −1  5 2   2 5 
C-1. If A =   ,B=   ,C=   and AB – CD = 0 find D.
3 4  7 4  3 8 

C-2. (i) Prove that (adj adj A) = |A|n–2 A


(ii) Find the value of |adj adj adj A| in terms of |A|

 3 −1 1   1 2 −2 
C-3.
 
If A =  −15 6 −5  & B =
–1  −1 3 0  , find (AB)–1
 
 5 −2 2   0 −2 1 

RI
C-4. If A is a symmetric and B skew symmetric matrix and (A + B) is non-singular and C = (A + B) –1 (A – B),
then prove that
(i) CT (A + B) C = A + B (ii) CT (A – B) C = A – B

A
0 1 2  1/ 2 −1/ 2 1/ 2 
   c  , then find values of a & c.
C-5. If A =  1 2 3  , A–1 =  −4 3
 3 a 1   5 / 2 −3 / 2 1/ 2 

UH
Section (D) : Charactristic equation and system of equations
 
3 2
For the matrix A = 
JA
D-1.  find a & b so that A + aA + b = 0. Hence find A .
2 –1

 1 1

D-2. Find the total number of possible square matrix A of order 3 with all real entries, whose adjoint matrix B
has characterstics polynomial equation as 3 – 2 +  + 1 = 0.

 1 1 2 
  , show that A3 = (5A – ) (A – )
LP

D-3. If A =  0 2 1 
 1 0 2 

D-4. Apply Cramer's rule to solve the following simultaneous equations.


(i) 2 x + y + 6 z = 46
A

5 x − 6 y + 4 z = 15
7 x + 4 y − 3 z = 19
(ii) x + 2y + 3z = 2
NK

x–y+z=3
5x – 11y + z = 17

4 3 6 6
D-5. Solve using Cramer’s rule: + = −1 & − = − 5.
x+5 y+7 x+5 y+7
SA

D-6. Find those values of c for which the equations:


2x+3y = 3
(c + 2) x + (c + 4) y = c + 6
(c + 2)² x + (c + 4)² y = (c + 6)² are consistent.
Also solve above equations for these values of c.

D-7. Solve the following systems of linear equations by matrix method.


(i) 2x − y + 3z = 8 (ii) x+y+z=9
−x + 2y + z = 4 2x + 5y + 7z = 52
3x + y − 4z = 0 2x + y − z = 0

D-8. Investigate for what values of ,  the simultaneous equations


x + y + z = 6; x + 2 y + 3 z = 10 & x + 2 y +  z =  have;
(a) A unique solution
(b) An infinite number of solutions.
(c) No solution.

 − 4 4 4   1 − 1 1 
D-9.

Determine the product  − 7 1 3   1 − 2 − 2  and use it to solve the system of

 5 − 3 − 1   2 1 3 
equations x – y + z = 4, x – 2 y – 2 z = 9, 2 x + y + 3 z = 1.

3 −2 3 
D-10. Compute A−1,
 
if A =  2 1 −1  Hence solve the matrix equations

RI
 4 −3 2 
3 0 3  x 8  2y 
2 1 0  y  =  1 +  z  .
       

A
 4 0 2  z   4  3y 

D-11. Which of the following statement(s) is/are true:

UH
4x − 5y − 2z = 2
S1 : The system of equations 5x − 4y + 2z = 3 is Inconsistent.
2x + 2y + 8z = 1

S2 : A matrix ‘A’ has 6 elements. The number of possible orders of A is 6.


JA
10 0 
S3 : For any 2 × 2 matrix A, if A (adjA) =   , then |A| = 10.
 0 10 
  S4 : If A is skew symmetric, then BAB is also skew symmetric.

PART - II : ONLY ONE OPTION CORRECT TYPE


LP

Section (A): Matrix, Algebra of Matrix, Transpose, symmetric and skew symmetric
matrix,  
 
A

 x2 + x x   0 −1  0 −2 
A-1.   +   =   then x is equal to -
 3 2 −x + 1 x  5 1 
(A) – 1 (B) 2 (C) 1 (D) No value of x
NK

 
 1  −5 4 0 
A-2.
 
If A = 2 and B =
 0 2 −1 , then
 
3   1 −3 2 
SA

 −5 8 0  −1
 −2 (B) AB = [– 2 – 1  
(A) AB =  0 4 4] (C) AB =  1  (D) AB does not exist
 3 −9 6   1 
 
1 0  0 1  cos  sin  
A-3. If  =   ,J=   and B =   , then B =
 0 1  −1 0   − sin  cos  
(A) cos + Jsin (B) cos – Jsin (C) sin + Jcos (D) – cos + Jsin
A-4. In an upper triangular matrix A = [aij]n × n the elements aij = 0 for
(A) i < j (B) i = j (C) i > j (D) i  j

A-5. If A = diag (2, −1, 3), B = diag (−1, 3, 2), then A 2B =


(A) diag (5, 4, 11) (B) diag (− 4, 3, 18) (C) diag (3, 1, 8) (D) B

A-6. If A is a skew- symmetric matrix, then trace of A is


(A) 1 (B) – 1 (C) 0 (D) none of these

p q
A-7. Let A =   such that det(A) = r where p, q, r all prime numbers, then trace of A is equal to
q p 
(A) 6 (B) 5 (C) 2 (D) 3

RI
 0 1  31
A-8. A=   and (A8 + A6 + A4 + A2 + ) V =   .
2 0  62 
(Where  is the (2 × 2) identity matrix), then the product of all elements of matrix V is

A
(A) 2 (B) 1 (C) 3 (D) –2

3x 2  (x + 2)2 5x 2 2x 


   

UH
2
A-9. Let A =  1  , B = [a b c] and C =  5x 2x (x + 1)2 
 6x   (x + 2)2 5x 2 
   2x
Where a, b, c and x  R, Given that tr (AB) = tr(C), then the value of (a + b + c).
(A) 7 (B) 2 (C) 1 (D) 4
JA
Section (B) : Determinant of Matrix

B-1. If A and B are square matrices of order 3 such that |A| = – 1, |B| = 3, then |3AB| is equal to
(A) – 9 (B) – 81 (C) – 27 (D) 81

cos–1 x cos–1 y cos–1 z 


 
LP

B-2. Let A = cos–1 y cos–1 z cos–1 x  such that |A| = 0, then maximum value of x + y + z is
cos–1 z cos–1 x cos–1 y 
 
(A) 3 (B) 0 (C) 1 (D) 2

−1
A

2 1
B-3. The absolute value of the determinant 3 + 2 2 2 + 2 2 1 is:
3 − 2 2 2 − 2 2 1
NK

(A) 16 2 (B) 8 2 (C) 8 (D) none


 
  
B-4. If ,  &  are the roots of the equation x3 + px + q = 0, then the value of the determinant    =
SA

  

(A) p (B) q (C) p2 − 2q (D) none


2 2
(a x
+ a− x ) (a x
− a− x ) 1
2 2
B-5. If a, b, c > 0 & x, y, z  R, then the determinant (b y
+ b− y ) (b y
− b− y ) 1 =
2 2
(c z
+ c −z ) (c z
− c −z ) 1

(A) axbycz (B) a−xb−yc−z (C) a2xb2yc2z (D) zero


b2 c 2 bc b + c
B-6. If a, b & c are non-zero real numbers , then D = c 2a2 ca c + a =
a 2b 2 ab a + b
(A) abc (B) a2 b2 c2 (C) bc + ca + ab (D) zero

b1 + c1 c1 + a1 a1 + b1
B-7. The determinant b2 + c 2 c 2 + a2 a2 + b2 =
b3 + c 3 c 3 + a3 a3 + b3
a1 b1 c1 a1 b1 c1 a1 b1 c1
(A) a2 b2 c 2 (B) 2 a2 b2 c 2 (C) 3 a2 b2 c 2 (D) none of these
a3 b3 c 3 a3 b3 c 3 a3 b3 c 3

RI
x x+y x+y+z
B-8. If x, y, z  R &  = 2x 5x + 2y 7x + 5y + 2z = − 16 then value of x is

A
3x 7x + 3y 9x + 7y + 3z
(A) − 2 (B) − 3 (C) 2 (D) 3

cos ( + ) − sin ( + ) cos 2

UH
B-9. The determinant sin  cos  sin  is:
− cos  sin  cos 
(A) 0 (B) independent of  (C) independent of  (D) independent of  &  both
JA
B-10. Let A be set of all determinants of order 3 with entries 0 or 1, B be the subset of A consisting of all
determinants with value 1 and C be the subset of A consisting of all determinants with value –1. Then
STATEMENT -1 : The number of elements in set B is equal to number of elements in set C.
and
STATEMENT-2 : (B  C)  A
(A) STATEMENT-1 is true, STATEMENT-2 is true and STATEMENT-2 is correct explanation for
STATEMENT-1
LP

(B) STATEMENT-1 is true, STATEMENT-2 is true and STATEMENT-2 is not correct explanation
for STATEMENT-1
(C) STATEMENT-1 is true, STATEMENT-2 is false
(D) STATEMENT-1 is false, STATEMENT-2 is true
(E) Both STATEMENTS are false
A

Section (C) : Cofactor matrix, adj matrix and inverse of matrix.


1 2
NK

If A = 
1
C-1. , then adj A =
2
1 −2  2 1  1 −2   −1 2 
(A) 
1 
(B)   (C)   (D)  
 −2  1 1  −2 −1  2 −1
SA

C-2. Identify statements S1, S2, S3 in order for true(T)/false(F)


cos  − sin  0 
 
S1 : If A =,  sin  cos  0  then adj A = A'
 0 0 1
a 0 0  a 0 0 
0 b 0  −1  
S2 : If A =   , then A = 0 b 0 
0 0 c  0 0 c 
S3 : If B is a non-singular matrix and A is a square matrix, then det (B –1 AB) = det (A)
(A) TTF (B) FTT (C) TFT (D) TTT
C-3. If A, B are two n × n non-singular matrices, then
(A) AB is non-singular (B) AB is singular
(C) (AB)–1 = A–1 B–1 (D) (AB)–1 does not exist
 
1 2  1 0
C-4. Let A =   and B =   and X be a matrix such that A = BX, then X is equal to
 3 −5  0 2 
1 2 4  1  −2 4  2 4 
(A) 
2  3 −5  (B)   (C)   (D) none of these
2  3 5  3 −5 

 –1 2 –3 
C-5.

Let A =  –2 0 3  be a matrix, then (det A) x (adj A– 1) is equal to
 3 –3 1 
 –1 2 –3   3 –3 1 

RI
(A) O3 × 3 (B) 3

(C)  –2 0 3  
(D)  3 0 –2 

 3 –3 1   –1 2 –3 
 a2 + x 2 ab – cx ac + bx  x c –b 

A
  
C-6. STATEMENT-1 : If A = ab + xc b + x bc − ax  and B =  –c x
2 2
a  , then |A| =|B|2.
ac – bx bc + ax c + x 
2 2
 b –a x 
 

UH
STATEMENT-2 : If Ac is cofactor matrix of a square matrix A of order n then |A c| = |A|n–1.
(A) STATEMENT-1 is true, STATEMENT-2 is true and STATEMENT-2 is correct explanation for
STATEMENT-1
(B) STATEMENT-1 is true, STATEMENT-2 is true and STATEMENT-2 is not correct explanation
for STATEMENT-1
(C) STATEMENT-1 is true, STATEMENT-2 is false
JA
(D) STATEMENT-1 is false, STATEMENT-2 is true
(E) Both STATEMENTS are false

Section (D) : Characteristic equation and system of equations


 1 0 2 
D-1.
 
If A =  0 2 1  is a root of polynomial x – 6x2 + 7x + k = 0, then the value of k is
LP

 2 0 3 
(A) 2 (B) 4 (C) –2 (D) 1

a b 
 (where bc  0) satisfies the equations x + k = 0, then
D-2 If A =  2
A

c d
(A) a + d = 0 & k = |A| (B) a – d = 0 & k = |A|
(C) a + d = 0 & k = –|A| (D) a + d  0 & k = |A|
NK

D-3. If the system of equations x + 2y + 3z = 4, x + py + 2z = 3, x + 4y + z = 3 has an infinite number of


solutions and solution triplet is
 −1
(A) p = 2,  = 3 and (5 – 4,  – 1 , ) (B) p = 2,  = 4 and (5 – 4, , 2)
2
 −1
SA

(C) 3 p = 2  and (5 – 4,  – 1, 2) (D) p = 4,  = 2 and (5 – 4, , )


2

D-4. Let  and  be real. Find the set of all values of  for which the system of linear equations have infinite
solution  real values of .
x + (sin )y + (cos ) z = 0
x + (cos  )y + (sin ) z = 0
– x + (sin ) y + (cos ) z = 0
(A) (–  , 2 )  ( 2 ,  ) (B) – 1 (C) (–5, – 2 ) (D) None of these
a o b   x  0 
     
D-5. Let A =  1 e 1  y  = 0  where a,b, c, d, e  {0, 1}
c o d  z  0 
then number of such matrix A for which system of equation AX = O have unique solution.
(A) 16 (B) 6 (C) 5 (D) none

D-6. If the system of equations ax + y + z = 0,. x + by + z = 0 and x + y + cz = 0, where


1 1 1
a, b, c  1, has a non−trivial solution, then the value of + + is
1− a 1− b 1− c
(A) 1 (B) 2 (C) 3 (D) 4

RI
PART - III : MATCH THE COLUMN
1. Column  Column 
 1 2 3   1

A
   2  = 0 then x =
(A)  1 x 1  4 5 6    (p) 2
 3 2 5   3 

UH
(B) If A is a square matrix of order 3 × 3 and (q) –2
k is a scalar, then adj (kA) = k m adj A, then m is

 2    7
(C) If A =  2  and B =   here (A – B) is upper triangular (r) 1
 3   49  
JA
matrix then number of possible values of  are

(b + c)2 a2 a2
9
(D) If b2 (c + a)2 b2 = k abc (a + b + c)3 (s) –
8
c2 c2 (a + b)2
LP

then the value of k is

2. Column –  Column – 
A

(A) If A and B are square matrices of order 3 × 3, where (p) 7


|A| = 2 and |B| = 1, then |(A –1) . adj (B–1) . adj (2A–1)| =
NK

(B) If A is a square matrix such that A2 = A and (I + A)3 = I + kA, (q) 8


then k is equal to

a b (a − b)
 
(C) Matrix b c (b − c) is non invertible (b2  ac) if –2 is (r) 0
2 1 0 
SA

(D) If A = [aij]3×3 is a scalar matrix with a11 = a22 = a33 = 2 (s) –1


and A(adjA) = k then k is
 Marked questions are recommended for Revision.

PART - I : ONLY ONE OPTION CORRECT TYPE

1. Two matrices A and B have in total 6 different elements (none repeated) . How many different matrices
A and B are possible such that product AB is defined.
(A) 5( 6!) (B) 3(6!) (C) 12(6!) (D) 8 ( 6!)

2. If AB = O for the matrices

RI
 cos2  cos  sin    cos2  cos  sin  
A=   and B =   then  –  is
cos  sin  sin2   cos  sin  sin2  

(A) an odd multiple of (B) an odd multiple of 

A
2

(C) an even multiple of (D) 0
2

UH
 3 −4 
3. If X =   , then value of X is, (where n is natural number)
n

 1 −1
3n −4n 2 + n 5 − n 3n ( −4)n  2n + 1 –4n 
(A)   (B) 
−n 
(C)  n  (D) 
–(2n – 1)
JA
 n −n   n 1 ( −1)n   n
 
4. If A and B are two matrices such that AB = B and BA = A, then A 2 + B2 =
(A) 2AB (B) 2BA (C) A + B (D) AB
LP

5. Find number of all possible ordered sets of two (n × n) matrices A and B for which AB – BA = 
(A) infinite (B) n2 (C) n! (D) zero
 
6. If B, C are square matrices of order n and if A = B + C, BC = CB, C 2 = O, then which of following is true
A

for any positive integer N.


(A) AN+1 = B N (B + (N + 1) C) (B) AN = BN (B + (N + 1) C)
N+1
(C) A = B (B + (N + 1) C) (D) A N+1 = B N (B + (N + 2) C)
NK

7. How many 3 × 3 skew symmetric matrices can be formed using numbers –2, –1, 1, 2, 3, 4, 0 (any
number can be used any number of times but 0 can be used at most 3 times)
(A) 8 (B) 27 (C) 64 (D) 54
 
SA

8. If A is a skew - symmetric matrix and n is an even positive integer, then A n is


(A) a symmetric matrix (B) a skew-symmetric matrix
(C) a diagonal matrix (D) none of these

9. Number of 3 × 3 non symmetric matrix A such that A T = A2 –  and |A|  0, equals to
(A) 0 (B) 2 (C) 4 (D) Infinite
 
10. Matrix A is such that A2 = 2A – , where  is the identity matrix. Then for n  2, A n =
(A) nA – (n – 1) (B) nA –  (C) 2n – 1 A – (n – 1) (D) 2n – 1 A – 
 3 1 
   1 1
11. If P =  2 2 ,A=
0 1 and Q = PAP and x = P Q P, then x is equal to
T T 2005
 1 3  
− 
 2 2 
 1 2005   4 + 2005 3 6015 
0 (B)  
1 
(A)
  2005 4 − 2005 3 
1 2 + 3 1  1  2005 2 − 3 
(C)   (D)  
4  −1 2 − 3  4 2 + 3 2005 

sin  cos  sin  sin  cos 

RI
12. Let  = cos  cos  cos  sin  − sin  , then
− sin  sin  sin  cos  0
(A)  is independent of  (B)  is indepedent of 
(C)  is a constant (D) none of these

A
1 + a2 + a4 1 + ab + a2b2 1 + ac + a2c 2
 = 1 + ab + a2b2 1 + b2 + b4 1 + bc + b2c 2

UH
13. is equal to
1 + ac + a2c 2 1 + bc + b2c 2 1 + c 2 + c 4
(A) (a – b)2 (b – c)2 (c – a)2 (B) 2(a – b) (b – c) (c – a)
(C) 4(a – b) (b – c) (c – a) (D) (a + b + c)3
JA
a2 + 1 ab ac
2
14. If D = ba b + 1 bc then D =
2
ca cb c +1
(A) 1 + a2 + b2 + c2 (B) a2 + b2 + c2 (C) (a + b + c)2 (D) none

a3 − x a 4 − x a 5 − x
LP

15. Value of the  = a5 − x a6 − x a7 − x is


a7 − x a 8 − x a 9 − x
(A) 0 (B) (a3 – 1) (a6 – 1) (a9 – 1)
(C) (a3 + 1) (a6 + 1) (a9 + 1) (D) a15 – 1
A

2a b e f 2d e
16. If 1 = 2d e f , 2 = 2z 4x 2y , then the value of 1 – 2 is
NK

4x 2y 2z e 2a b
y
(A) x + +z (B) 2 (C) 0 (D) 3
2
SA

17. From the matrix equation AB = AC, we conclude B = C provided:


(A) A is singular (B) A is non−singular (C) A is symmetric (D) A is a square
 

 –2 7 3
 
18. Let A =  0 0 –2  and A4 = , then  is
0 2 0 

(A) – 16 (B) 16 (C) 8 (D) –8
19. If A is 3 × 3 square matrix whose characterstic polynomial equations is 3 – 32 + 4 = 0 then trace of
adjA is
(A) 0 (B) 3 (C) 4 (D) – 3

20. If a, b, c are non zeros, then the system of equations


( + a) x + y + z = 0
x + ( + b)y + z = 0
x + y + ( + c)z = 0
has a non-trivial solution if
(A) –1 = – (a–1 + b–1 + c–1) (B) –1 = a + b + c
(C)  + a + b + c = 1 (D) none of these

RI
PART - II : SINGLE AND DOUBLE VALUE INTEGER TYPE
1. Let X be the solution set of the equation
0 1 −1

A
 
A = , where A =  4 −3 4  and  is the unit matrix and X  N then the minimum value of
x

3 −3 4 

 (cos

UH
x
 + sinx ) ,   R is :
x
 
2. If A is a diagonal matrix of order 3 × 3 is commutative with every square matrix of order 3 × 3 under
multiplication and tr(A) = 12, then the value of |A| is :
JA
3. A, is a (3×3) diagonal matrix having integral entries such that det(A) = 120, number of such matrices is
10n. Then n is :

b+c c +a a+b
a+b
4. If c +a a+b b+c > 0 , where a, b, c  R+ , then is
c
a+b b+c c +a
LP

a1 a2 a3
5. If a1, a2, a3 , 5, 4, a6, a7, a8, a9 are in H.P. and D = 5 4 a6 , then the value of 21D is
a7 a8 a9
A

a + b + 2c a b
NK

6. If c b + c + 2a b = k(a + b + c)3 , then (2 +  – )k is (   k  z+)


c a c + a + 2b

7. If A is a square matrix of order 3 and A denotes transpose of matrix A, A A =  and det A = 1, then
det (A – ) must be equal to
SA

8. Suppose A is a matrix such that A2 = A and ( + A)6 =  + kA, then k is

− bc b2 + bc c 2 + bc
9. If a2 + ac − ac c 2 + ac = 64, then (ab + bc + ac) is :
2 2
a + ab b + ab − ab

1 + sin2 x cos2 x 4 sin2x


2 2
10. Let f(x) = sin x 1 + cos x 4 sin2x then the maximum value of f(x) is
sin2 x cos2 x 1 + 4 sin2x
n 1 5 N


N
11. If Un = n2 2N + 1 2N + 1 and U
n =1
n = n2 , then  is
n3 3N2 3N + 1 n =1

12. The absolute value of a for which system of equations, a 3x + (a + 1)3y + (a + 2)3z = 0,
ax + (a + 1) y + (a + 2) z = 0, x + y + z = 0, has a non−zero solution is:
 
13. Consider the system of linear equations in x, y, z:
(sin 3) x − y + z = 0
(cos 2) x + 4y + 3z = 0

RI
2x + 7y + 7z = 0
Number of values of   (0, ) for which this system has non − trivial solution, is

14. The value of ‘ 2k ‘ for which the set of equations 3x + ky − 2z = 0, x + ky + 3z = 0, 2 x + 3 y − 4 z = 0 has

A
a non − trivial solution over the set of rational is:

15. A 1 = [ a1 ]

UH
a a3 
A2 =  2
a 4 a5 
 a6 a7 a8 
 
A 3 =  a9 a10 a11  ......................A n = ........ 
JA
a12 a13 a14 
Where ar = [ log2r ] ([.] denotes greatest integer). Then trace of A 10

   
  − 13 −
−1 3 3   −2 3 4
2  
LP

 1 
16. If  ( A − A  +  ) =  − 17 10 − 1  for A =  5 −4 −3  , then  is :

 2   
7 − 11 5   7 2 9 
 
 
2 0 − 
A

 
17. Given A = 5  0  For   R − {a, b}, A–1 exists and A−1 = A2 − 5bA + cI, when  = 1. The value of
0  3 
NK

a + 5b + c is :

18. Let a, b, c positive numbers. Find the number of solution of system of equations in x, y and z
x2 y2 z2 x2 y2 z2 x2 y2 z2
2
+ 2 – 2 = 1 ; 2 – 2 + 2 = 1 ;– 2 + 2 + 2 = 1 has finitely many solutions
a b c a b c a b c
SA

PART - III : ONE OR MORE THAN ONE OPTIONS CORRECT TYPE


1. Which one of the following is wrong ?
(A) The elements on the main diagonal of a symmetric matrix are all zero
(B) The elements on the main diagonal of a skew - symmetric matrix are all zero
(C) For any square matrix A, A A is symmetric
(D) For any square matrix A, (A + A)2 = A2 + (A)2 + 2AA
 1 −1
2. Which of the following is true for matrix A =  
2 3 
(A) A + 4I is a symmetric matrix
(B) A2 − 4A + 5I2 = 0
  −1
(C) A − B is a diagonal matrix for any value of  if B =  
2 5 
(D) A − 4I is a skew symmetric matrix

3. Suppose a1, a2, a3 are in A.P. and b1, b2, b3 are in H.P. and let
a1 − b1 a1 − b2 a1 − b3
 = a2 − b1 a2 − b2 a2 − b3 , then

RI
a3 − b1 a3 − b2 a3 − b3
(A)  is independent of a1, a2, a3, (B) a1 − , a2 − 2, a3 − 3 are in A.P.
(C) b1 + , b2 + 2, b3 +  are in H.P. (D)  is independent of b 1, b2, b3

A
 cos  – sin  
4. Let  = , X =   , O is null maxtrix and  is an identity matrix of order 2 × 2, and if
5  sin  cos  

UH
 + X + X2 + ...... + Xn = O, then n can be
(A) 9 (B) 19 (C) 4 (D) 29

x 2y − z −z
5. If  = y 2x − z −z , then
JA
y 2y − z 2x − 2y − z
(A) x – y is a factor of  (B) (x – y)2 is a factor of 
(C) (x – y)3 is a factor of  (D)  is independent of z

−x a b
Let a, b > 0 and  = b − x a , then
LP

6.
a b −x
(A) a + b – x is a factor of  (B) x2 + (a + b)x + a2 + b2 – ab is a factor of 
(C)  = 0 has three real roots if a = b (D) a + b + x is a factor of 
 
A

b c b + c
7. The determinent  = c d c + d is equal to zero if
b + c c + d a3 − c
NK

(A) b, c, d are in A.P. (B) b, c, d are in G.P.


(C) b, c, d are in H.P. (D)  is a root of ax3 – bx2 – 3cx – d = 0

a2 (1 + x) ab ac
SA

8. The determinant  = ab 2


b (1 + x) bc is divisible by
2
ac bc c (1 + x)
(A) x + 3 (B) (1 + x)2 (C) x2 (D) x2 + 1

9. If A is a non−singular matrix and AT denotes the transpose of A, then:


(A) A  AT (B) A. AT = A2
(C) A . A = A 
T T 2 (D) A+AT  0
2sin x sin2 x 0
10. Let f(x) = 1 2sin x sin2 x , then
0 1 2sin x
(A) f(x) is independent of x (B) f(/2) = 0
/2
(C) 
− / 2
f(x)dx = 0 (D) tangent to the curve y = f(x) at x = 0 is y = 0

1 x x2
11. Let  = x 2 1 x , then
x x2 1
(A) 1 – x3 is a factor of  (B) (1 – x3)2 is factor of 

RI
(C) (x) = 0 has 4 real roots (D) (1) = 0

1/ x log x xn

A
12. Let f(x) = 1 −1/ n ( −1)n , then (where fn(x) denotes nth derivative of f(x))
1 a a2
(A) fn (1) is indepedent of a

UH
(B) fn (1) is indepedent of n
(C) fn (1) depends on a and n
(D) y = a(x – fn (1)) represents a straight line through the origin

13. If D is a determinant of order three and  is a determinant formed by the cofactors of determinant D ;
JA
then
(A)  = D2 (B) D = 0 implies  = 0
(C) if D = 27, then  is perfect cube (D) if D = 27, then  is perfect square

14. Let A, B, C, D be real matrices such that A T = BCD ; BT = CDA ; CT = DAB and DT = ABC for the matrix
M = ABCD, then find M2016 ?
(A) M (B) M2 (C) M3 (D) M4
LP

15. Let A and B be two 2 × 2 matrix with real entries. If AB = O and tr(A) = tr(B) = 0 then
(A) A and B are comutative w.r.t. operation of multiplication.
(B) A and B are not commutative w.r.t. operation of multiplication.
(C) A and B are both null matrices.
A

(D) BA = 0

 1 −1 0 
 
=  0 −2 1  , then
NK

16. IfA–1
 0 0 −1 
(A) | A | = 2 (B) A is non-singular
 1/ 2 −1/ 2 0 
(C) Adj. A =
 0 −1 1/ 2  (D) A is skew symmetric matrix
 
SA

 0 0 −1/ 2 

17. If A and B are square matrices of order 3, then the true statement is/are (where I is unit matrix).
(A) det (− A) = − det A
(B) If AB is singular then atleast one of A or B is singular
(C) det (A + I) = 1 + det A
(D) det (2A) = 23 det A

18. Let M be a 3 × 3 non-singular matrix with det(M) = 4. If M– 1 adj(adj M) = k2, then the value of 'k' may be
(A) +2 (B) 4 (C) –2 (D) –4

19. If AX = B where A is 3 × 3 and X and B are 3×1 matrices then which of the following is correct?
(A) If |A| = 0 then AX = B has infinite solutions
(B) If AX = B has infinite solutions then |A| = 0
(C) If (adj(A)) B = 0 and |A|  0 then AX = B has unique solution
(D) If (adj(A)) B  0 & |A| = 0 then AX = B has no solution

PART - IV : COMPREHENSION
Comprehension # 1

Let be the set of all 3×3 symmetric matrices whose entries are 1,1,1,0,0,0,–1, –1, –1. B is one of the
matrix in set and
x 0   1

RI

X = y
 
U = 0 
  
V = 0  .
 z  0  0 

is , then  lies in the interval

A
1. Number of such matrices B in set
(A) (30, 40) (B) (38, 40) (C) (34, 38) (D) (25, 35)

2. Number of matrices B such that equation BX = U has infinite solutions

UH
(A) is at least 6 (B) is not more than 10 (C) lie between 8 to 16 (D) is zero.

3. The equation BX = V


(A) is inconsistent for atleast 3 matrices B.
(B) is inconsistent for all matrices B.
JA
(C) is inconsistent for at most 12 matrices B.
(D) has infinite number of solutions for at least 3 matrices B.

Comprehension # 2
Some special square matrices are defined as follows :
Nilpotent matrix : A square matrix A is said to be nilpotent ( of order 2) if, A 2 = O. A square matrix is
said to be nilpotent of order p, if p is the least positive integer such that A p = O.
LP

Idempotent matrix : A square matrix A is said to be idempotent if, A2 = A.


1 0
e.g.   is an idempotent matrix.
 0 1
Involutory matrix : A square matrix A is said to be involutory if A 2 = ,  being the identity matrix.
A

1 0
e.g. A =   is an involutory matrix.
 0 1
Orthogonal matrix : A square matrix A is said to be an orthogonal matrix if A A =  = AA.
NK

4. If A and B are two square matrices such that AB = A & BA = B, then A & B are
(A) Idempotent matrices (B) Involutory matrices
(C) Orthogonal matrices (D) Nilpotent matrices

 0 2  
SA

 
5. If the matrix   −  is orthogonal, then
 −  
1 1 1
(A)  = ± (B)  = ± (C)  = ± (D) all of these
2 6 3
1 1 3
 
6. The matrix A =  5 2 6  is
 −2 −1 −3 
(A) idempotent matrix (B) involutory matrix
(C) nilpotent matrix (D) none of these
 Marked questions are recommended for Revision.
* Marked Questions may have more than one correct option.

PART - I : JEE (ADVANCED) / IIT-JEE PROBLEMS (PREVIOUS YEARS)


Comprehension # 1 (Q. No. 1 to Q. No. 3)

Let be the set of all 3 × 3 symmetric matrices all of whose entries are either 0 or 1. Five of these
entries are 1 and four of them are 0.
 

RI
1. The number of matrices in is [IIT-JEE 2009, Paper-1, (4, –1), 80]
(A) 12 (B) 6 (C) 9 (D) 3

A
 x  1 
2. The number of matrices A in
   
for which the system of linear equations A  y  = 0  has a unique
 z  0 

UH
solution, is [IIT-JEE 2009, Paper-1, (4, –1), 80]
(A) less than 4 (B) at least 4 but less than 7
(C) at least 7 but less than 10 (D) at least 10
 
x 1 
  0 
JA
3. The number of matrices A in for which the system of linear equations A y =   is
 z  0 
inconsistent, is
(A) 0 (B) more than 2 (C) 2 (D) 1
[IIT-JEE 2009, Paper-1, (4, –1), 80]
 
LP

Comprehension # 2 (Q. No. 4 to 6)


Let p be an odd prime number and Tp be the following set of 2 × 2 matrices :

 a b  

Tp =  A =   : a, b, c  {0, 1 , 2,.....,p – 1} 

  c a  

A

4. The number of A in T p such that A is either symmetric or skew-symmetric or both, and det (A) divisible
by p is [IIT-JEE 2010, Paper-1, (3, –1), 84]
NK

(A) (p – 1)2 (B) 2 (p – 1) (C) (p – 1)2 + 1 (D) 2p – 1

5. The number of A in Tp such that the trace of A is not divisible by p but det (A) is divisible by p is
[Note : The trace of matrix is the sum of its diagonal entries.] [IIT-JEE 2010, Paper-1, (3, –1), 84]
(A) (p – 1)(p2 – p + 1) (B) p3 – (p – 1)2 (C) (p – 1)2 (D) (p – 1)(p2 – 2)
SA

6. The number of A in Tp such that det (A) is not divisible by p is [IIT-JEE 2010, Paper-1, (3, –1), 84]
(A) 2p2 (B) p3 – 5p (C) p3 – 3p (D) p3 – p2

7. The number of all possible values of , where 0 <  < , for which the system of equations
(y + z) cos 3 = (xyz) sin 3
2cos3 2 sin3
x sin 3 = +
y z
(xyz) sin 3 = (y + 2z) cos 3  + y sin 3

have a solution (x0, y0, z0) with y0 z0  0, is [IIT-JEE 2010, Paper-1, (3, 0), 84]
 

8. Let k be a positive real number and let [IIT-JEE 2010, Paper-2, (3, 0), 79]
 2k − 1 2 k 2 k   0 2k − 1 k 
   
A= 2 k 1 −2k  and B = 1 − 2k 0 2 k  . If det (adj A) + det (adj B) = 10 6, then
   
 −2 k 2k −1 
 − k −2 k 0 
[k] is equal to
(Note : adj M denotes the adjoint of a square matrix M and [k] denotes the largest integer less than or
equal to k].

9. Let M and N be two 2n × 2n non-singular skew-symmetric matrices such that MN = NM. If P T denotes
the transpose of P, then M2 N2 (MT N)–1 (MN–1)T is equal to
(A) M2 (B) – N2 (C) – M2 (D) MN

Comprehension # 3 (10 to 12) [IIT-JEE 2011, Paper-1, (3, –1), 80]

RI
Let a, b and c be three real numbers satisfying
1 9 7
 
[a b c] 8 2 7  = [0 0 0] ...........(E)

A
7 3 7 

UH
10. If the point P(a, b, c), with reference to (E), lies on the plane 2x + y + z = 1, then the value of 7a + b + c
is
(A) 0 (B) 12 (C) 7 (D) 6

11. Let  be a solution of x3 – 1 = 0 with m () > 0. if a = 2 with b and c satisfying (E), then the value of
3 1 3
JA
+ b + c is equal to
a
 
(A) – 2 (B) 2 (C) 3 (D) – 3

12. Let b = 6, with a and c satisfying (E). If  and  are the roots of the quadratic equation ax 2 + bx + c = 0,
n

 1 1
then   +  is
 
LP

n=0

6
(A) 6 (B) 7 (C) (D) 
7
1 a b
13.

Let   1 be a cube root of unity and S be the set of all non-singular matrices of the form,   1 c 
A

2  1

where each of a, b and c is either  or  2. Then the number of distinct matrices in the set S is
NK

[IIT-JEE 2011, Paper-2, (3, –1), 80]


(A) 2 (B) 6 (C) 4 (D) 8

14. Let M be a 3 × 3 matrix satisfying


0   −1 1 1 1 0
 1 2  −1   1 =  0  . Then the sum of the diagonal entries of M is
SA

M   =   , M   =  1  , and M   
0   3   0   −1 1 12
[IIT-JEE 2011, Paper-2, (4, 0), 80]

15. Let P = [aij] be a 3 × 3 matrix and let Q = [b ij], where bij = 2i+jaij for 1  i, j  3. If the determinant of P is 2,
then the determinant of the matrix Q is [IIT-JEE 2012, Paper-1, (3, –1), 70]
  (A) 210 (B) 211 (C) 212 (D) 213
16. If P is a 3 × 3 matrix such that P T = 2P + I, where PT is the transpose of P and I is the 3 × 3 identity
x 0 
   
matrix, then there exists a column matrix X =  y   0  such that
 z  0 
[IIT-JEE 2012, Paper-2, (3, –1), 66]
0 
 
(A) PX = 0  (B) PX = X (C) PX = 2X (D) PX = – X
0 

1 4 4
17*.
 
If the adjoint of a 3 × 3 matrix P is 2 1 7  , then the possible value(s) of the determinant of P is

RI
 1 1 3 
(are)
[IIT-JEE 2012, Paper-2, (4, 0), 66]
  (A) –2 (B) –1 (C) 1 (D) 2  

A
18.* For 3×3 matrices M and N, which of the following statement(s) is (are) NOT correct ?
[JEE (Advanced) 2013, Paper-1, (4, – 1)/60]

UH
(A) NT M N is symmetric or skew symmetric, according as M is symmetric or skew symmetric
(B) M N – N M is skew symmetric for all symmetric matrices M and N
(C) M N is symetric for all symmetric matrices M and N
(D) (adj M) (adj N) = adj(MN) for all invertible matrices M and N

19*. Let M be a 2 × 2 symmetric matrix with integer entries. Then M is invertible if


JA
[JEE (Advanced) 2014, Paper-1, (3, 0)/60]
(A) the first column of M is the transpose of the second row of M
(B) the second row of M is the transpose of first column of M
(C) M is a diagonal matrix with nonzero entries in the main diagonal
(D) the product of entries in the main diagonal of M is not the square of an integer
 
20*. Let M and N be two 3 × 3 matrices such that MN = NM. Further, if M  N 2 and M2 = N4, then
LP

(A) determinant of (M2 + MN2) is 0 [JEE (Advanced) 2014, Paper-1, (3, 0)/60]
(B) there is a 3 × 3 non-zero matrix U such that (M2 + MN2)U is the zero matrix
(C) determinant of (M2 + MN2)  1
(D) for a 3 × 3 matrix U, if (M2 + MN2)U equals the zero matrix then U is the zero matrix
A

21*. Let X and Y be two arbitrary, 3 × 3, non-zero, skew-symmetric matrices and Z be an arbitrary 3 × 3,
non-zero, symmetric matrix. Then which of the following matrices is (are) skew symmetric ?
[JEE (Advanced) 2015, Paper-1 (4, –2)/88]
NK

(A) Y3Z4 – Z4Y3 (B) X44 + Y44 (C) X4Z3 – Z3X4 (D) X23 + Y23

(1 +  )2 (1 + 2 )2 (1+ 3 )2
22*. Which of the following values of  satisfy the equation (2 + ) (2 + 2 )
2 2
(2 + 3 )2 = – 648 ?
(3 + )2 (3 + 2)2 (3 + 3)2
SA

[JEE (Advanced) 2015, Paper-1 (4, –2)/88]


(A) – 4 (B) 9 (C) – 9 (D) 4
3 − 1 − 2
 
23*. Let P = 2 0   , where   R. Suppose Q = [qij] is a matrix such that PQ = k , where k  R, k  0
3 − 5 0 
2
k k
and  is the identity matrix of order 3. If q 23 = – and det (Q) = , then
8 2
[JEE (Advanced) 2016, Paper-1 (4, –2)/62]
(A)  = 0, k = 8 (B) 4 – k + 8 = 0 (C) det (P adj (Q)) = 2 9 (D) det (Q adj (P)) = 213
x x2 1 + x3
24. The total number of distinct x  R for which 2x 4x 2 1 + 8x 3 = 10 is
2 3
3x 9x 1 + 27x
[JEE (Advanced) 2016, Paper-1 (3, 0)/62]
 1 0 0
25.
 
Let P = 4 1 0 and  be the identity matrix of order 3. If Q = [q ij] is a matrix such that P50 – Q= ,
 
16 4 1
q31 + q32
then equals [JEE (Advanced) 2016, Paper-2 (3, –1)/62]
q21

RI
(A) 52 (B) 103 (C) 201 (D) 205

26*. Let a, ,   R. Consider the system of linear equations


ax + 2y = 

A
3x – 2y =  [JEE (Advanced) 2016, Paper-2 (4, –2)/62]
Which of the following statement(s) is(are) correct ?
(A) if a = – 3, then the system has infinitely many solutions for all values of  and 
(B) If a  –3, then the system has a a unique solution for all values of  and 

UH
(C) If  +  = 0, the the system has infinitely many solutions for a = –3
(D) If  +   0, then the system has no solution for a = –3

27*. Which of the following is(are) NOT the square of a 3 × 3 matrix with real entries?
[JEE(Advanced) 2017, Paper-1,(4, –2)/61]
JA
1 0 0  1 0 0   −1 0 0  1 0 0

(A) 0 1 0 
   
(B) 0 −1 0  (C)  0 −1 0 
 
(D) 0 1 0 

0 0 −1 0 0 −1  0 0 −1 0 0 1
 
28. For a real number , if the system [JEE(Advanced) 2017, Paper-1,(3, 0)/61]
 1  2   x   1 
LP

     
  1    y  =  −1
 2  1   z   1 
 
of linear equations, has infinitely many solutions, then 1 +  +  2 =
A

29. How many 3 × 3 matrices M with entries from {0, 1, 2} are there, for which the sum of the diagonal
entries of MT M is 5 ? [JEE(Advanced) 2017, Paper-2,(3, –1)/61]
(A) 198 (B) 162 (C) 126 (D) 135
NK

 b1 
 
30*. Let S be the set of all column matrices b2  such that b1, b2, b2  R and the system of equation
b3 
(in real variables)
SA

– x + 2y + 5z = b1
2x – 4y + 3z = b2
x – 2y + 2z = b3
has at least one solution. Then, which of the following system(s) (in real variables) has (have) at least
 b1 
 
one solution for each b2   S ? [JEE(Advanced) 2018, Paper-2,(4, –2)/60]
b3 
(A) x + 2y + 3z = b1, 4y + 5z = b2 and x + 2y + 6z = b 3
(B) x + y + 3z = b1, 5x + 2y + 6z = b2 and – 2x – y – 3z = b3
(C) – x + 2y – 5z = b1, 2x – 4y + 10z = b2 and x – 2y + 5z = b3
(D) x + 2y + 5z = b1, 2x + 3z = b2 and x + 4y – 5z = b3
31. Let P be a matrix of order 3 × 3 such that all the entries in P are from the set {–1, 0, 1}. Then, the
maximum possible value of the determinant of P is _____ . [JEE(Advanced) 2018, Paper-2,(3, 0)/60]

PART - II : JEE (MAIN) / AIEEE PROBLEMS (PREVIOUS YEARS)


1. Let A be a 2 × 2 matrix.
Statement-1 : adj(adj (A)) = A.
Statement-2 : |adj A| = |A| [AIEEE 2009 (4, –1), 144]
(1) Statement-1 is true, Statement-2 is true; Statement-2 is not a correct explanation for Statement-1.
(2) Statement-1 is true, Statement-2 is false.
(3) Statement-1 is false, Statement-2 is true.
(4) Statement-1 is true, Statement-2 is true; Statement-2 is a correct explanation for Statement-1.

RI
a a +1 a –1 a +1 b +1 c –1
2. Let a, b, c be such that b(a + c)  0. If –b b + 1 b – 1 + a –1 b –1 c + 1 = 0. Then the
c c –1 c +1 (–1) a (–1) b (–1)n c
n+ 2 n +1

value of 'n' is - [AIEEE 2009 (4, –1), 144]

A
(1) zero (2) any even integer (3) any odd integer (4) any integer

3. The number of 3 × 3 non-singular matrices, with four entries as 1 and all other entries as 0, is

UH
(1) 5 (2) 6 (3) at least 7 (4) less than 4
[AIEEE 2010 (8, –2), 144]

4. Let A be a 2 × 2 matrix with non-zero entries and let A2 = I, where I is 2 × 2 identity matrix.
Tr(A) = sum of diagonal elements of A and |A| = determinant of matrix A. [AIEEE 2010 (4, –1), 144]
Statement -1 : Tr(A) = 0
JA
Statement -2 : |A| = 1
(1) Statement -1 is true, Statement-2 is true ; Statement -2 is not a correct explanation for Statement-1.
(2) Statement-1 is true, Statement-2 is false.
(3) Statement -1 is false, Statement -2 is true.
(4) Statement -1 is true, Statement -2 is true; Statement-2 is a correct explanation for Statement-1.
 
5. Consider the system of linear equations : [AIEEE 2010 (4, –1), 144]
LP

x1 + 2x2 + x3 = 3
2x1 + 3x2 + x3 = 3
3x1 + 5x2 + 2x3 = 1
The system has
A

(1) exactly 3 solutions (2) a unique solution (3) no solution (4) infinite number of solutions
 
6. Let A and B be two symmetric matrices of order 3. [AIEEE 2011, I, (4, –1), 120]
NK

Statement-1 : A(BA) and (AB)A are symmetric matrices.


Statement-2 : AB is symmetric matrix if matrix multiplication of A with B is commutative.
(1) Statement-1 is true, Statement-2 is true; Statement-2 is a correct explanation for
Statement-1.
(2) Statement-1 is true, Statement-2 is true; Statement-2 is true; Statement-2 is not a correct
explanation for Statement-1.
SA

(3) Statement-1 is true, Statement-2 is false.


(4) Statement-1 is false, Statement-2 is true.

7. The number of values of k for which the linear equations [AIEEE 2011, I, (4, –1), 120]
4x + ky + 2z = 0
kx + 4y + z = 0
2x + 2y + z = 0
posses a non-zero solution is :
(1) 3 (2) 2 (3) 1 (4) zero

 0 
8. If   1 is the complex cube root of unity and matrix H =   , then H is equal to -
70

 0 
[AIEEE 2011, I, (4, –1), 120]
(1) 0 (2) – H (3) H2 (4) H

9. If the trivial solution is the only solution of the system of equations [AIEEE 2011, II, (4, –1), 120]
x – ky + z = 0
kx + 3y – kz = 0
3x + y – z = 0
then the set of all values of k is :
(1) R – {2, –3} (2) R – { 2 } (3) R – { –3 } (4) {2, –3}

10. Statement - 1 : Determinant of a skew-symmetric matrix of order 3 is zero.


[AIEEE 2011, II, (4, –1), 120]
Statement - 2 : For any matrix A, det (A) T = det(A) and det (–A) = – det(A).
Where det (B) denotes the determinant of matrix B. Then :

RI
(1) Both statements are true (2) Both statements are false
(3) Statement-1 is false and statement-2 is true. (4) Statement-1 is true and statement-2 is false
 
 1 0 0  1 0

A
     
11. Let A =  2 1 0  . If u1 and u2 are column matrices such that Au1 =  0  and Au2 =  1  , then u1 + u2
 3 2 1 0 0
     

UH
is equal to : [AIEEE-2012, (4, –1)/120]
 −1  −1  −1 1
       
(1)  1  (2)  1  (3)  −1 (4)  −1
0  −1 0  −1
       
JA
12. Let P and Q be 3 × 3 matrices P  Q. If P3 = Q3 and P2Q = Q2P, then determinant of (P2 + Q2) is equal
to :
[AIEEE-2012, (4, –1)/120]
(1) – 2 (2) 1 (3) 0 (4) – 1

13. The number of values of k, for which the system of equations :


[AIEEE - 2013, (4, – 1) 120 ]
LP

(k + 1)x + 8y = 4k
kx + (k + 3)y = 3k – 1
has no solution, is
(1) infinite (2) 1 (3) 2 (4) 3
A

1  3
 
14. If P =  1 3 3  is the adjoint of a 3 × 3 matrix A and |A| = 4, then  is equal to :
2 4 4 
NK

[AIEEE - 2013, (4, – 1) 120 ]


(1) 4 (2) 11 (3) 5 (4) 0

3 1 + f(1) 1 + f(2)
15. If ,   0 and f(n) =  +  and 1 + f(1) 1 + f(2) 1 + f(3) = K (1 – )2 (1 – )2 ( – )2 , then K is equal
n n
SA

1 + f(2) 1 + f(3) 1 + f(4)


to [JEE(Main) 2014, (4, – 1), 120] 
1
(1) 1 (2) –1 (3)  (4)

16. If A is an 3 × 3 non-singular matrix such that AA = AA and B = A –1A, then BB equals :

[JEE(Main) 2014, (4, – 1), 120]


(1) B–1 (2) (B–1) (3) I + B (4) I
1 2 2 
17.
 
If A = 2 1 –2 is a matrix satisfying the equation AA T = 9, where  is 3 × 3 identity matrix, then the
a 2 b 
ordered pair (a, b) is equal to : [JEE(Main) 2015, (4, – 1), 120]
(1) (2, – 1) (2) (–2, 1) (3) (2, 1) (4) (–2, – 1)

18. The set of all value of  for which the system of linear equations :
2x1 – 2x2 + x3 = x1
2x1 – 3x2 + 2x3 = x2
– x1 + 2x2 = x3
has a non-trivial solution, [JEE(Main) 2015, (4, –1), 120]
(1) is an empty set (2) is a singleton

RI
(3) contains two elements (4) contains more than two elements

19. The system of linear equations [JEE(Main) 2016, (4, –1), 120]
x + y – z = 0

A
x –y –z = 0
x +y – z = 0
has a non-trivial solution for :
(1) Exactly one value of . (2) Exactly two values of .

UH
(3) Exactly three values of . (4) Infinitely many values of .
 
5a – b T
20. If A =   and A adj A = A A , then 5a + b is equal to [JEE(Main) 2016, (4, –1), 120]
 3 2 
JA
(1) 5 (2) 4 (3) 13 (4) – 1

21. It S is the set of distinct values of ‘b’ for which the following system of linear equations
x+y+z=1 [JEE(Main) 2017, (4, –1), 120]
x + ay + z = 1
ax + by + z = 0
has no solution, then S is :
LP

(1) an empty set (2) an infinite set


(3) a finite set containing two or more elements (4) a singleton

 2 – 3 2
22. If A =   , then adj (3A + 12A) is equal to [JEE(Main) 2017, (4, –1), 120]
– 4 1 
A

 72 – 84  51 63   51 84   72 – 63
(1)   (2)   (3)   (4)  
 – 63 51  84 72  63 72  – 84 51 
NK

23. If the system of linear equations [JEE(Main) 2018, (4, –1), 120]
x + ky + 3z = 0
3x + ky – 2z = 0
2x + 4y – 3z = 0
xz
SA

has a non-zero solution (x, y, z), then is equal to :


y2
(1) – 30 (2) 30 (3) –10 (4) 10

x − 4 2x 2x
24. If 2x x − 4 2x = (A + Bx) (x–A)2 then the ordered pair (A, B) is equal to :
2x 2x x − 4
[JEE(Main) 2018, (4, –1), 120]  
(1) (–4, 5) (2) (4, 5) (3) (–4, –5) (4) (–4, 3)
25. The system of linear equations [JEE(Main) 2019, Online (09-01-19),P-1 (4, – 1), 120]
x+y+z=2
2x + 3y + 2z = 5
2x + 3y + (a2 + 1)z = a + 1
(1) is inconsistent when a = 4 (2) has infinitely many solutions for a = 4
(3) is inconsistent when |a| = 3 (4) has a unique solution for |a| = 3

cos  − sin  
 , then the matrix A when  =
26. If A =  –50 , is equal to :
 sin  cos   12
              [JEE(Main) 2019, Online (09-01-19),P-1 (4, – 1), 120]
 1 3  3 1 

RI
   
(1)  2 2  (2)  2 2 
 3 1   1 3
 2 2  − 2 2 

A
 3 1  1 3
 −   
(3)  2 2 (4)  2 2 
 1 3  3 1 
  − 2 

UH
 2 2  2

 −2 4+d (sin  − 2) 
27.

Let d  R, and A =  1 (sin ) + 2 d  ,   [0, 2]. If the minimum value of det(A) is 8,

 5 (2sin ) − d ( − sin ) + 2 + 2d
JA
then a value of d is : [JEE(Main) 2019, Online (10-01-19),P-1 (4, – 1), 120]
(1) –5 (2) 2( 2 + 2) (3) 2( 2 +1) (4) –7

 0 2q r 
 
Let A =  p q – r  . If AAT = 3, then |p| is  
LP

28. [JEE(Main) 2019, Online (11-01-19),P-1 (4, – 1), 120]


p – q r 
 
1 1 1 1
(1) (2) (3) (4)
5 3 6 2
A
NK
SA
EXERCISE # 1
PART-I
Section (A) :
 1 0 
3 2   2 –1/ 2
A-1.   A-2. (x, y, z, w) = (1, 2, 4, 5) A-3. 4 –1 
 5 4  

 18 −11 10 

AB =  −16 47 10
 , BA =  49 24  yR
A-4.   −7 58  A-7. A-9. Zero

RI
 62 −23 42   

Section (B) :

A
3
B-1. 0, , B-2. (i) 0 (ii) 0 (iii) 0 (iv) 5(3 2 − 5 3)
4

UH
B-5. (i) x = − 2 b/a (ii) 4 B-8. A = 0, B = 0

Section (C) :
 9 −3 5 
 −191 −110  (n −1)3  −2 1 0  C-5.
 77 |A|
44 
C-1. C-2. (ii) C-3.   a = 1, c = – 1
JA
  1 0 2
Section (D) :
 1 −2 
D-1. a = – 4, b = 1, A–1 =   D-2. 0
 −1 3 
5k 8 2k 1
LP

D-4. (i) x = 3, y = 4, z = 6 (ii) x= – + , y =– – , z = k, where k  R


3 3 3 3
1 4
D-5. x = − 7, y = − 4 D-6. for c = 0, x = − 3, y = 3 ; for c = − 10, x = −
,y=
2 3
D-7. (i) x = 2, y = 2, z = 2 (ii) x = 1, y = 3, z = 5 D-8. (a)   3 (b)  = 3,  = 10 (c)  = 3,   10
A

1 5 1
1 
D-9. x = 3, y = – 2, z = – 1 D-10. x = 1, y = 2, z = 3, A =–1
8 6 −9 
17 
10 −1 −7 
NK

D-11. S1, S3, S4


PART-II
Section (A) :  
A-1. (A) A-2. (D) A-3. (A) A-4. (C) A-5. (B) A-6. (C) A-7. (A)
SA

A-8. (A) A-9. (A)


Section (B) :
B-1. (B) B-2. (A) B-3. (A) B-4. (D) B-5. (D) B-6. (D) B-7. (B)
B-8. (C) B-9. (B) B-10. (B) 
Section (C) :
C-1. (A) C-2. (C) C-3. (A) C-4. (A) C-5. (C) C-6. (A)
Section (D) :
D-1. (A) D-2 (A) D-3. (D) D-4. (B) D-5. (B) D-6. (A)

PART-III
1. (A) → (s), (B) → (p), (C) → (p), (D) → (p)

2. (A) → (q), (B) → (p), (C) → (s), (D) → (q)

EXERCISE # 2

PART-I
1. (D) 2. (A) 3. (D) 4. (C) 5. (D) 6. (A) 7. (C)
 
8. (A) 9. (A) 10. (A) 11. (A) 12. (B) 13. (A) 14. (A)

15. (A) 6. (C) 17. (B) 18. (B) 19. (A) 20. (A)

RI
PART - II
1. 2 2. 64 3. 36 4. 2 5. 50 6. 4 7. 0

A
8. 63 9. 4 10. 6 11. 2 12. 1 13. 2 14. 33

15. 80 16. 39 17. 17 18. 8

UH
PART - III
1. (AD) 2. (BC) 3. (ABCD) 4. (ABD) 5. (AB) 6. (ABC) 7. (BD)

8. (AC) 9. (BCD) 10. (BCD) 11. (ABD) 12. (ABD) 13. (ABCD)
JA
14. (BD) 15. (AD) 16. (BC) 17. (ABD) 18. (AC) 19. (BCD)

PART - IV
1*. (AC) 2*. (AC) 3*. (AC) 4. (A) 5. (D) 6. (C)
LP

EXERCISE # 3

PART-I
A

1. (A) 2. (B) 3. (B) 4. (D) 5. (C) 6. (D) 7. 3


 
8. 4 9. (C) 10. (D) 11. (A) 12 . (B) 13. (A) 14. 9
NK

15. (D) 16. (D) 17*. (AD) 18.* (CD) 19*. (CD) 20*. (AB) 21*. (CD)

22*. (BC) 23*. (BC) 24. 2 25. (B) 26*. (BCD) 27*. (AC) 28. 1

29. (A) 30*. (AD) 31. 4


SA

PART - II
1. (1) 2. (3) 3. (3) 4. (2) 5. (3) 6. (2) 7. (2) 

8. (4) 9. (1) 10. (4) 11. (4) 12. (3) 13. (2) 14. (2)

15. (1) 16. (4) 17. (4) 18. (3) 19. (3) 20. (1) 21. (4)

22. (2) 23. (4) 24. (1) 25. (3) 26. (2) 27. (1) 28. (4)
a2 + (b2 + c 2 )cos  ab(1 − cos ) ac (1 − cos )
1. 2 2 2
If a + b + c = 1, then prove that ba(1 − cos ) b + (c + a )cos 
2 2 2
bc (1 − cos )
ca(1 − cos ) cb(1 − cos ) c + (a2 + b2 )cos 
2

is independent of a, b, c

2. If a, b, c, x, y, z  R, then prove that,


(a − x)2 (b − x)2 (c − x)2 (1 + a x)2 (1 + b x)2 (1 + c x)2
(a − y)2 (b − y)2 (c − y)2 = . (1 + a y)2 (1 + b y)2 (1 + c y)2

RI
(a − z)2 (b − z)2 (c − z)2 (1 + a z)2 (1 + b z)2 (1 + c z)2

1 + a1 1 1

A
3. If a1, a2, a3 are distinct real roots of the equation px 3 + px2 + qx + r = 0 such that 1 1 + a2 1 =0,
1 1 1 + a3
r
then Prove that <0

UH
p

  '+  '   '  '


4. Prove that  =   '+  '   '  ' = (' – ') (' – ') (' – ')
  '+  '   '  '
JA
5. If ax1² + by1² + cz12 = d ax2x3 + by2y3 + cz2z3 = f
ax22 + by22 + cz22 = d and ax3x1 + by3y1 + cz3z1 = f
ax32 + by32 + cz32 = d ax1x2 + by1y2 + cz1z2 = f
2
x1 y1 z1
LP

d + 2f
then prove that x 2 y2 z2 = (d − f)2 , where a, b, c  0.
abc
x3 y3 z3

2 2
( x1 − x 2 ) + ( y1 − y 2 ) = a2
A

2 2
6. If ( x2 − x3 ) + ( y 2 − y3 ) = b2 ,
2 2 2
( x3 − x1 ) + ( y3 − y1 ) = c
NK

2
x1 y1 1
prove that 4 x2 y2 1 = (a + b + c) (b + c − a) (c + a − b)(a + b − c).
x3 y3 1

cos–1 x cos–1 y cos–1 z 


SA

 
7. Let A = cos–1 y cos–1 z cos–1 x  such that |A| = 0, then find the maximum value of x + y + z
cos–1 z cos–1 x cos–1 y 
 

u v 0
u d2 y
8. If y = , where u & v are functions of ' x ', show that, v 3 
= u v  v .
v d x2
u v  2 v 
9. If ,  be the real roots of ax 2+ bx+c = 0 and sn= n + n, then prove that as n + bsn–1 + csn–2 = 0 for all
3 1 + s1 1 + s2
n  2, n  N. Hence or otherwise prove that 1 + s1 1 + s2 1 + s3 > 0 for all real a, b, c.
1 + s 2 1 + s3 1 + s 4

1 1 1
a a (a + d) (a + d) (a + 2d)
10. Let a > 0, d > 0. Find the value of determinant 1 1 1
(a + d) (a + d) (a + 2d) (a + 2d) (a + 3 d)
1 1 1
(a + 2d) (a + 2d) (a + 3 d) (a + 3 d) (a + 4 d)

RI

11. Let a r = xr ˆi + yr ˆj + zr k,
ˆ r = 1, 2, 3 be three mutually perpendicular unit vectors, then find the value of
x1 x2 x3
y1 y2 y3

A
z1 z2 z3

UH
xk xk + 2 xk + 3
 1 1 1
12. If yk yk + 2 yk + 3 = (x − y) (y − z) (z − x)  + +  , then find the value of k.
x y z
zk zk + 2 zk + 3

a + p + x u + f
JA
13. If the determinant b + q m + y v + g splits into exactly K determinants of order 3, each
c + r n + z w + h
element of which contains only one term, then find the value of K =
 
a a3 a4 − 1
LP

14. If a, b, c are all different and b b3 b4 − 1 = 0, then find the value of abc (ab + bc + ca) – (a + b + c).
c c3 c 4 −1

0 −b −c
A

15. If a, b, c are complex numbers and z = b 0 −a then show that z is purely imaginary.
c a 0
 
NK

f(x)g(x) [f(x)]g(x) 1
2 2 2 g(x 2 )
16. If f(x) = log10x and g(x) = eix and h(x) = f(x )g(x ) [f(x )] 0 , then find the value of h(10).
3 3 3 g(x3 )
f(x )g(x ) [f(x )] 1
SA

a 1 + 2i 3 − 5i
17. If a, b, c, are real numbers, and D = 1 − 2i b −7 − 3i then show that D is purely real.
3 + 5i −7 + 3i c

 1 a 1
18. If A =   then find lim A n .
0 1 n → n
  
 cos 9 sin 9 
19. Let P =   and    be non-zero real numbers such that p6 + p3 + 
 – sin  cos  
 9 9 
is the zero matrix. Then find value of (2 + 2 +  2 )(  –)( –  )(  –  )

20. Consider an odd order square symmetric matrix A = [a i j]nxn. It's element in any row are 1, 2, ......, n in
some order, then prove that a11, a22, ..........., ann are numbers 1, 2, 3, ......, n in some order.

1 1 1 2 –1 –1
21.

Let A = 1 1 1
 ;

B =  –1 2 –1 and C = 3A + 7B
1 1 1  –1 –1 2 
Prove that
(i) (A + B)2013 = A2013 + B2013
(ii) Prove that An = 3n – 1 A ; Bn = 3n – 1 B ; Cn = 32n – 1 A + 7.21n – 1 B.

RI
22. Let 'A' is (4×4) matrix such that sum of elements in each row is 1. Find out sum of all the elements in
A10.
 x+ x x 
23.

Let A =  x x+ x  , then prove that A–1 exists if 3x +   0,   0

A
 x x x +  

24. Prove that if A and B are n × n matrices, then det ( n – AB) = det (n – BA).

UH
25. Let A be an n × n matrix such that A n = A where  is a real number different from 1 and – 1. Prove that
the matrix A + n is invertible.

26. Let p and q be real numbers such that x 2 + px + q  0 for every real number x. Prove that if n is an odd
JA
positive integer, then X2 + pX + qn  0n for all real matrices X of order n × n.

27. Let A, B, C be three 3 × 3 matrices with real entries. If BA + BC + AC =  and det(A + B) = 0 then find
the value of det(A + B + C – BAC).

−1 −1
z −z2   z1 z2   12 0
If |z1| = |z2| = 1, then prove that  1
LP

28.
z1  −z z1 
= 0 
 z2  2  2
1

29. If A and B are two square matrices such that B = –A–1 BA, then show that (A +B)2 = A2 + B2

a 1 0  a 1 1 f  a 2  x
A

        y
30. If A =  1 b d , B = 0 d c  , U = g , V =  0  , X =  
 1 b c   f g h  h 0  z 
 
NK

and AX = U has infinitely many solution. Prove that BX = V has no unique solution, also prove that if
afd  0, then BX = V has no solution.

31. If the system of equations x = cy + bz, y = az + cx and z = bx + ay has a non-zero solution and at least
one of a, b, c is a proper fraction, prove that a 3 + b3 + c3 < 3 and abc > – 1.
SA

32. If D = diag {d1, d2,........., dn}, then prove that f(D) = diag {f(d 1), f(d2),........, f(dn)}, where f(x) is a
polynomial with scalar coefficient.
 –1 3 5 
 
33. Given the matrix A =  1 −3 −5  and X be the solution set of the equation A x = A, where
 −1 3 5 
x3 + 1
x  N – {1}. Evaluate  3 ; where the continued product extends  x  X.
x –1
Comprehension (Q. NO. 34 to 36)
Any non-zero vector, X, is said to be characteristic vector of a matrix A, if there exist a number  such
that AX = X. And then  is said to be a charactristic root of the matrix A corresponding to the
characteristic vector X and vice versa.
Also AX = X  (A – )X = 0
Since X  0  |A – | = 0
Thus every characteristic root  of a matrix A is a root of its characteristic equation.

34. Prove that the two matrices A and P –1 AP have the same characteristic roots and hence show that
square matrices AB & BA have same characteristic roots if at least one of them is invertible.

|A|
35. If q is a characteristic root of a non singular matrix A, then prove that is a characteristic root of a
q

RI
adj A.

36. Show that if 1, 2, ........, n are n characteristic roots of a square matrix A of order n, then the roots of
the matrix A2 be 12, 22,......... n2.

A
37. IF threre are three square matrices A, B, C of same order satisying the equation A 3 = A–1 and let
then prove that det(B + C) = 0, n  N  
n (n + 4)
B = −A3 and C = A3

UH
38. If A is a non-singular matrix satisfying AB – BA = A, then prove that det.(B + I) = det.(B – I)  
39. If rank is a number associated with a matrix which is the highest order of non-singular sub matrix then
prove that
JA
 1 3 −2
(i)
 
Rank of the matrix A =  4 −1 0  is 2
 2 −7 4 
y + a b c 
(ii)

If the matrix A =  a y+b c  has rank 3, then y  – (a + b + c) and y  0
 a y + c 
LP

(iii) If A & B are two square matrices of order 3 such that rank of matrix AB is two, then atleast one
of A & B is singular.
A
NK

4 d4
7. 3 10. 11. 1 12. −1 13. 8
a(a + d)2 (a + 2d)3 (a + 3 d)2 (a + 4 d)

0 a 
14. 0 16. 0 18. 0 0  19. 1 22. 4 27. 0
SA

 
3
33.
2
PROBABILITY

JEE (Advanced) Syllabus

Probability : Addition and multiplication rules of probability, conditional probability, independence of events,
computation of probability of events using permutations and combinations.

RI
JEE (Main) Syllabus

Probability : Probability of an event, addition and multiplication theorems of probability, Baye’s theorem,
probability distribution of a random variate, Bernoulli trials and Binomial distribution.

A
Lest men suspect your tale untrue Keep probability in view........Gay, John

UH
There are various phenomena in nature, leading to an outcome, which cannot be predicted apriori
e.g. in tossing of a coin, a head or a tail may result. Probability theory aims at measuring the
uncertainties of such outcomes.

(I) Important terminology : 


JA
(i) Random experiment:
It is a process which results in an outcome which is one of the various possible outcomes that are
known to us before hand e.g. throwing of a die is a random experiment as it leads to fall of one of the
outcome from {1, 2, 3, 4, 5, 6}. Similarly taking a card from a pack of 52 cards is also a random
experiment.
(ii) Sample space :
LP

It is the set of all possible outcomes of a random experiment e.g. {H, T} is the sample space associated
with tossing of a coin.

In set notation it can be interpreted as the universal set.


A

Example # 1 : Write the sample space of the experiment ‘A coin is tossed and a die is thrown’.
Solution : The sample space S = {H1, H2, H3, H4, H5, H6, T1, T2, T3, T4, T5, T6}.
NK

Example # 2 : Write the sample space of the experiment ‘A coin is tossed, if it shows head a coin tossed
again else a die is thrown.
Solution : The sample space S = {HH, HT, T1, T2, T3, T4, T5, T6}

Example # 3 : Find the sample space associated with the experiment of rolling a pair of dice (plural of die)
once. Also find the number of elements of the sample space.
SA

Solution : Let one die be blue and the other be green. Suppose ‘1’ appears on blue die and ‘2’ appears
on green die. We denote this outcome by an ordered pair (1, 2). Similarly, if ‘3’ appears on blue
die and ‘5’ appears on green die, we denote this outcome by (3, 5) and so on. Thus, each
outcome can be denoted by an ordered pair (x, y), where x is the number appeared on the first
die (blue die) and y appeared on the second die (green die). Thus, the sample space is given
by
S = {(x, y) x is the number on blue die and y is the number on green die}
We now list all the possible outcomes (figure)
1 2 3 4 5 6
1 (1, 1) (1, 2) (1, 3) (1, 4) (1, 5) (1, 6)
2 (2, 1) (2, 2) (2, 3) (2, 4) (2, 5) (2, 6)
3 (3, 1) (3, 2) (3, 3) (3, 4) (3, 5) (3, 6)
4 (4, 1) (4, 2) (4, 3) (4, 4) (4, 5) (4, 6)
5 (5, 1) (5, 2) (5, 3) (5, 4) (5, 5) (5, 6)
6 (6, 1) (6, 2) (6, 3) (6, 4) (6, 5) (6, 6)

Figure

RI
Number of elements (outcomes) of the above sample space is 6 × 6 i.e., 36

Self practice problems :


(1) A coin is tossed twice, if the second throw results in head, a die is thrown then write sample

A
space of the experiment.
(2) An urn contains 3 red balls and 2 blue balls. Write sample space of the experiment ‘Selection of
a ball from the urn at random’.

UH
Ans. (1) {HT, TT, HH1, HH2, HH3, HH4, HH5, HH6, TH1, TH2, TH3, TH4, TH5, TH6}.
(2) {R1, R2, R3, B1, B2 }. (Here the balls are distinguished from one and other by
naming red balls as R1, R2 and R3 and the blue balls as B1 and B2.)
(iii) Event :
JA
It is subset of sample space. e.g. getting a head in tossing a coin or getting a prime number in throwing
a die. In general if a sample space consists ‘n’ elements, then a maximum of 2 n events can be
associated with it.

(iv) Complement of event :


LP

The complement of an event ‘A’ with respect to a sample space S is the set of all elements of ‘S’ which
are not in A. It is usually denoted by A, or AC.
(v) Simple event :
If an event covers only one point of sample space, then it is called a simple event e.g. getting a head
A

followed by a tail in throwing of a coin 2 times is a simple event.


(vi) Compound event :
When two or more than two events occur simultaneously, the event is said to be a compound event.
NK

Symbolically A  B or AB represent the occurrence of both A & B simultaneously.


Note : “A  B” or A + B represent the occurrence of either A or B.

Example # 4 : Write down all the events of the experiment ‘tossing of a coin’.
Solution : S = {H, T}
SA

the events are  , {H}, {T}, {H, T}

Example # 5 : A die is thrown. Let A be the event ‘ an odd number turns up’ and B be the event ‘a number
divisible by 3 turns up’. Write the events (a) A or B (b) A and B
Solution : A = {1, 3, 5}, B = {3, 6}
 A or B = A  B = {1, 3, 5, 6}
A and B = A  B = {3}
Self practice problems :
(3) A coin is tossed and a die is thrown. Let A be the event ‘H turns up on the coin and odd number
turns up on the die’ and B be the event ‘ T turns up on the coin and an even number turns up
on the die’. Write the events (a) A or B (b) A and B.
(4) In tossing of two coins, let A = {HH, HT} and B = {HT, TT}. Then write the events
(a) A or B (b) A and B.
Ans. (3) (a) {H1, H3, H5, T2, T4, T6} (b) 
(4) (a) {HH, HT, TT} (b) {HT}

(vii) Equally likely events :


If events have same chance of occurrence, then they are said to be equally likely.
e.g
(i) In a single toss of a fair coin, the events {H} and {T} are equally likely.
(ii) In a single throw of an unbiased die the events {1}, {2}, {3} and {4}, are equally likely.
(iii) In tossing a biased coin the events {H} and {T} are not equally likely.

RI
(viii) Mutually exclusive / disjoint / incompatible events :
Two events are said to be mutually exclusive if occurrence of one of them rejects the possibility of
occurrence of the other i.e. both cannot occur simultaneously.

A
In the vein diagram the events A and B are mutually exclusive. Mathematically, we write
AB=
Events A1, A2, A3, ....... An are said to be mutually exclusive events iff

UH
Ai  Aj =   i, j  {1, 2, ..., n} where i  j

Note : If Ai  Aj =   i, j  {1, 2, ..., n} where i  j, then A1  A2  A3 ....  An =  but converse need


JA
not to be true.
Example # 6 : In a single toss of a coin find whether the events {H}, {T} are mutually exclusive or not.
Solution : Since {H}  {T} = ,
 the events are mutually exclusive.
Example # 7 : In a single throw of a die, find whether the events {1, 2}, {2, 3} are mutually exclusive or not.
LP

Solution : Since {1, 2}  {2, 3} = {2}    the events are not mutually exclusive.

Self practice problems :


(5) In throwing of a die write whether the events ‘Coming up of an odd number’ and
‘Coming up of an even number’ are mutually exclusive or not.
A

(6) An experiment involves rolling a pair of dice and recording the numbers that come up.
Describe the following events :
A : the sum is greater than 8.
NK

B : 2 occurs on either die.


C : the sum is at least 7 and a multiple of 3.
Also, find A  B, B  C and A  C.
Are (i) A and B mutually exclusive ?
(ii) B and C mutually exclusive ?
(iii) A and C mutually exclusive ?
SA

Ans. (5) Yes


(6) A = {(3, 6), (4, 5), (5, 4), (6, 3), (4, 6), (5, 5), (6, 4), (5, 6), (6, 5), (6, 6)}
B = {(1, 2), (2, 2), (3, 2), (4, 2), (5, 2), (6, 2), (2, 1), (2, 3), (2, 4). (2, 5), (2, 6)}
C = {(3, 6), (6, 3), (5, 4), (4, 5), (6, 6)}
A  B = , B  C = , A  C = {(3, 6), (6, 3), (5, 4), (4, 5), (6, 6)}
(i) Yes (ii) Yes (iii) No.
(ix) Exhaustive system of events :
If each outcome of an experiment is associated with at least one of the events E 1, E2, E3, .........En, then
collectively the events are said to be exhaustive. Mathematically we write
E1  E2  E3.........En = S. (Sample space)
Example # 8 : In throwing of a die, let A be the event ‘even number turns up’, B be the event ‘an odd prime
turns up’ and C be the event ‘a numbers less than 4 turns up’. Find whether the events
A, B and C form an exhaustive system or not.
Solution : A  {2, 4, 6}, B  {3, 5} and C  {1, 2, 3}.
Clearly A  B  C = {1, 2, 3, 4, 5, 6} = S. Hence the system of events is exhaustive.
Example # 9 : Three coins are tossed. Describe
(i) two events A and B which are mutually exclusive
(ii) three events A, B and C which are mutually exclusive and exhaustive.
(iii) two events A and B which are not mutually exclusive.
(iv) two events A and B which are mutually exclusive but not exhaustive.
(v) three events A, B and C which are mutually exclusive but not exhaustive.

RI
Ans. (i) A : “getting at least two heads” B : “getting at least two tails”
(ii) A : “getting at most one heads” B : “getting exactly two heads”
C : “getting exactly three heads”
(iii) A : “getting at most two tails” B : “getting exactly two heads”

A
(iv) A : “getting exactly one head” B : “getting exactly two heads”
(v) A : “ getting exactly one tail” B : “getting exactly two tails”
C : “getting exactly three tails”

UH
[Note : There may be other cases also]

Self practice problems :


(7) In throwing of a die which of the following pair of events are mutually
exclusive?
JA
(a) the events ‘coming up of an odd number’ and ‘coming up of an even
number’
(b) the events ‘coming up of an odd number’ and ‘coming up of a number  4’
(8) In throwing of a die which of the following system of events are exhaustive ?
(a) the events ‘an odd number turns up’, ‘a number  4 turns up’ and ‘the
number 5 turns up’.
LP

(b) the events ‘a number  4 turns up’, ‘a number > 4 turns up’.
(c) the events ‘an even number turns up’, ‘a number divisible by 3 turns up’,
‘number 1 or 2 turns up’ and ‘the number 6 turns up’.
Ans. (7) (a) (8) (b)
A

(II) Classical (a priori) definition of probability :


If an experiment results in a total of (m + n) outcomes which are equally likely and if ‘m’ outcomes are
NK

favorable to an event ‘A’ while ‘n’ are unfavorable, then the probability of occurrence of the event ‘A’,
m number of favourable outcomes
denoted by P(A), is defined by =
m+n total number of outcomes
m
i.e. P(A) = .
m+n
SA

Here we say that odds in favour of ‘A’ are m : n, while odds against ‘A’ are n : m.
n
Note that P(A) or P(A) or P(AC), i.e. probability of non-occurrence of A = = 1 – P(A)
m+n
In the above we shall denote the number of out comes favourable to the event A by n(A) and the total
number of out comes in the sample space S by n(S).
n(A)
 P(A) = .
n(S)

Example # 10 : In throwing of a fair die find the probability of the event ‘ a number  3 turns up’.
Solution : Sample space S = {1, 2, 3, 4, 5, 6} ; event A = {1, 2, 3}
 n(A) = 3 and n(S) = 6
n(A) 3 1
 P(A) = = = .
n(S) 6 2

Example # 11 : In throwing of a fair die, find the probability of turning up of an odd number  4.
Solution : S = {1, 2, 3, 4, 5, 6}
Let E be the event ‘turning up of an odd number  4’
then E = {5}
n (E) 1
 P(E) = = .
n (S) 6
Example # 12 : In throwing a pair of fair dice, find the probability of getting a total of 10.

RI
Solution : When a pair of dice is thrown the sample space consists
{(1, 1) (1, 2) .......... (1, 6)
(2, 1,) (2, 2,)......... (2, 6)
.... ..... .... ...
.... ... ... ...

A
(6, 1), (6, 2) ........ (6, 6)}
Note that (1, 2) and (2, 1) are considered as separate points to make each outcome as
equally likely.

UH
To get a total of ‘10’, favourable outcomes are, (4, 6) (5, 5) and (6, 4)
3 1
Hence required probability = =
36 12

Example # 13 : A five digit number is formed using the digits 0, 1, 2, 3, 4,5 without repetition. Find the
JA
probability that it is divisible by 4
Solution : Total 5 digit numbers formed

5 5 4 3 2
Total ways =5×5×4×3×2 = 600
LP

Now, A number is divisible by 4, if last two digits of the number is divisible by 4


Hence we can have 0 4 → first 3 places can be filled in 4×3×2 = 24 ways

1,3 2 → first 4 places can be filled in 3×3×2×3 = 54 ways


A

or 5
2 or 0 → first 4 places can be filled in 4×3×2×2 = 48 ways
4

2 → first 3 places can be filled in 3×3×2 = 18 ways


NK

__________
Total number of ways 144 ways
favorable outcomes 144 6
probability = = = Ans.
SA

Total outcomes 600 25

Self practice problems :


(9) A bag contains 4 white, 3 red and 2 blue balls. A ball is drawn at random. Find the
probability of the event the ball drawn is blue or red
(10) In throwing a pair of fair dice find the probability of the events ‘ a total of of less than or
equal to 9”’.
Ans. (9) 5/9 (10) 5/6.

(III) Addition theorem of probability :


If ‘A’ and ‘B’ are any two events associated with an experiment, then
P(AB) = P(A) + P(B) – P(AB)

De Morgan’s laws : If A & B are two subsets of a universal set U, then


(a) (A  B)c = Ac  Bc
(b) (A  B)c = Ac  Bc

Distributive laws : (a) A  (B  C) = (A  B)  (A  C)


(b) A  (B  C) = (A  B)  (A  C)
For any three events A, B and C we have the figure

RI
A
UH
JA
(i) P(A or B or C) = P(A) + P(B) + P(C) – P(A  B) – P(B  C) – P(C  A) + P(A  B  C)
(ii) P (at least two of A, B, C occur) = P(B  C) + P(C  A) + P(A  B) – 2P(A  B  C)
(iii) P(exactly two of A, B, C occur) = P(B  C) + P(C  A) + P(A  B) – 3P(A  B  C)
(iv) P(exactly one of A, B, C occur) =
P(A) + P(B) + P(C) – 2P(B  C) – 2P(C  A) – 2P(A  B) + 3P(A  B  C)
Example # 14 : A bag contains 4 white, 3red and 4 green balls. A ball is drawn at random. Find the probability
LP

of the event ‘the ball drawn is white or green’.


Solution : Let A be the event ‘the ball drawn is white’ and B be the event ‘the ball drawn is green’.
8
P(The ball drawn is white or green) = P (A  B) = P(A) + P(B) – P(A  B) =
11
Example # 15 : In throwing of a die, let A be the event ‘an odd number turns up’, B be the event ‘a number
A

divisible by 3 turns up’ and C be the event ‘a number  4 turns up’. Then find the probability that
atleast one of A, B, C occur.
Solution : Event A = {1, 3, 5}, event B = {3, 6} and event C = {1, 2, 3, 4}
NK

 A  B = {3}, B  C = {3}, A  C = {1, 3} and A  B  C = {3}.


Thus
P(A or B or C) = P(A) + P(B) + P(C) – P(A  B) – P(B  C) – P(C  A) + P(A  B  C)
3 2 4 1 1 2 1
= + + – – – + =1
6 6 6 6 6 6 6
SA

Self practice problems :


(11) In throwing of a die, let A be the event ‘an odd number turns up’, B be the event ‘a number
divisible by 3 turns up’ and C be the event ‘a number  4 turns up’. Then find the probability that
atleast two of A, B and C occur.
(12) In the problem number 11, find the probability that exactly one of A, B and C occurs.
1 2
Ans. (11) (12)
3 3
(IV) Conditional probability
P(A  B)
If A and B are two events, then P(A/B) = .
P(B)
Note that for mutually exclusive events P(A/B) = 0.
Example # 16 : If P(A/B) = 0.2 and P(B) = 0.5 and P(A) = 0.2. Find P(A  B ).
Solution : P(A  B ) = P(A) – P(A  B)
P(A  B)
Also P(A/B) =  P(A  B) = 0.1
P(B)
From given data,
P(A  B ) = 0.1
Example # 17 : If P(A) = 0.25, P(B) = 0.5 and P(A  B) = 0.14, find probability that neither ‘A’ nor ‘B’ occurs.
(
Also find P A  B )

RI
Solution : ( )
We have to find P A  B = 1 – P(A  B) (by De-Morgan’s law)
Also, P(A  B) = P(A) + P(B) – P(A  B)
(
putting data we get, P A  B = 0.39 )

A
The shaded region denotes the simultaneous occurrence of A and B

UH
( )
Hence P A  B = P(A) – P(A  B) = 0.11

Self practice problem :


(13) If P( A / B ) = 0.3, P(A  B) = 0.8, then find P(A  B ) ?
JA
7
Ans.
15

(V) Independent and dependent events


If two events are such that occurence or non-occurence of one does not affect the chances of
occurence or non-occurence of the other event, then the events are said to be independent.
LP

Mathematically : if P(A  B) = P(A) P(B), then A and B are independent.


Note: (i) If A and B are independent, then
(a) A and B are independent,
(b) A and B are independent and
(c) A and B are independent.
A

(ii) If A and B are independent, then P(A / B) = P(A).


If events are not independent then they are said to be dependent.
Independency of three or more events
NK

Three events A, B & C are independent if & only if all the following conditions hold :
P(A  B) = P(A) . P(B) ; P(B  C) = P(B) . P(C)
P (C  A) = P(C) . P(A) ; P(A  B  C) = P(A) . P(B) . P(C)

Example # 18 : A pair of fair coins is tossed yielding the equiprobable space S = {HH, HT, TH, TT}. Consider
SA

the events:
A = {head on first coin} = {HH, HT}, B = {head on second coin} = {HH, TH}
C = {head on exactly one coin} = {HT, TH}
Then check whether A, B, C are independent or not.
2 1
Solution : P(A) = P(B) = P(C) = = .
4 2
1 1 1
Also P(A  B) = = P(A) P(B), P(A  C) = = P(A) P(C), P(B  C) = = P(B) P(C)
4 4 4
but P(A  B  C) = 0  P(A) P(B) P(C)
 A, B & C are not independent

Example # 19 : In drawing two balls from a box containing 7 red and 4 white balls without replacement, which
of the following pairs is independent ?
(a) Red on first draw and red on second draw
(b) Red on first draw and white on second draw
Solution : Let E be the event ‘Red on first draw’, F be the event ‘Red on second draw’ and G be the
event ‘white on second draw’.
7 7 4
P(E) = , P(F) = , P(G) =
11 11 11
7
P 21
(a) P(E  F) = 11 2 =
P2 55
7 7 49 21
P(E) . P(F) = × =   E and F are not independent
11 11 121 55

RI
7 4 28
(b) P(E) . P(G) = × =
11 11 121
7
P1 4 P1 14
P(E  G) = 11
=
P2 55

A
 P(E) . P(G)  P(E  G)  E and G are not independent

Example # 20 : If two switches S1 and S2 have respectively 90% and 80% chances of working. Find the

UH
probabilities that each of the following circuits will work.

JA
Solution : Consider the following events :
A = Switch S1 works,
LP

B = Switch S2 works,
We have,
90 9 80 8
P(A) = = and P(B) = =
100 10 100 10
A

(i) The circuit will work if the current flows in the circuit. This is possible
only when both the switches work together. Therefore, Required probability
= P(A  B) = P(A) P (B) [ A and B are independent events]
9 8 72 18
NK

= × = =
10 10 100 25
(ii) The circuit will work if the current flows in the circuit. This is possible
only when at least one of the two switches S 1, S2 works. Therefore,
Required Probability
= P(A  B) = 1 – P (A) P( B ) [ A, Bare independent events]
SA

 9  8  1 2 49
= 1 – 1 −   1 − 10  =1 – × =
 10   10 10 50
Example # 21 : A speaks truth in 60% of the cases and B in 90% of the cases. In what percentage of cases are
they likely to contradict each other in stating the same fact?
Solution : Let E be the event that A speaks truth and F be the event that B speaks truth. Then E and F
are independent events such that
60 3 90 9
P(E) = = and P(F) = =
100 5 100 10
A and B will contradict each other in narrating the same fact in the following mutually
exclusive ways:
(i) A speaks truth and B tells a lie i.e. E  F
(ii) A tells a lie and B speaks truth lie i.e. E  F
 P(A and B contradict each other)
= P(I or II) = (I  II) = P[(E  F )  ( E  F)]
= P(E  F ) + P ( E  F) [ E  F and E  F are mutually exclusive]
= P(E) P( F ) + P( E ) P(F) [ E and F are independent]
3  9   3 9 3 1 2 9 21
= × 1 −  +  1 −  × 10 = 5 × 10 + 5 × 10 = 50
5  10   5 

Example # 22 : A box contains 5 bulbs of which two are defective. Test is carried on bulbs one by one untill the
two defective bulbs are found out. Find the probability that the process stops after

RI
(i) Second test (ii) Third test
Solution : (i) Process will stop after second test. Only if the first and second bulb are both
found to be defective
2 1 1
probability = × = (Obviously the bulbs drawn are not kept back.)

A
5 4 10
(ii) Process will stop after third test when either
2 3 1 1
(a) DND → × × = Here ‘D’ stands for defective

UH
5 4 3 10
3 2 1 1
or (b) NDD → × × = and ‘N’ is for not defective.
5 4 3 10
3 2 1 1
or (c) NNN × × =
5 4 3 10
JA
3
hence required probability =
10

1 1 E  1
Example # 23 : If E1 and E2 are two events such that P(E1) = ; P(E2) = ; P  1 = , then choose the
4 2  E2  4
LP

correct options.
(i) E1 and E2 are independent (ii) E1 and E2 are exhaustive

(iii) E1 and E2 are mutually exclusive (iv) E1 & E2 are dependent


E   E2 
A

Also find P  1  and  


 E2   E1 
E 
Solution : Since P =  1  P(E1)  E1 and E2 are independent of each other
NK

 E2 
Also since P(E1  E2) = P(E1) + P(E2) – P(E1) . P(E2)  1
Hence events are not exhaustive. Independent events can’t be mutually exclusive.
Hence only (i) is correct
Further since E1 & E2 are independent; E1 and E 2 or E1 , E2 are E1 , E 2 are also independent.
SA

E  3 E  1
Hence P  1 ( )
 = P E1 = and P  2  = P (E2) =
 E2  4  E1  2

Example # 24 : If cards are drawn one by one from a well shuffled pack of 52 cards without replacement, until
an ace appears, find the probability that the fourth card is the first ace to appear.
48
C3 4 C1
Solution : Probability of selecting 3 non-Ace and 1 Ace out of 52 cards is equal to 52
C4
Since we want 4th card to be first ace, we will also have to consider the arrangement, Now 4
cards in sample space can be arranged in 4! ways and, favorable they can be arranged in 3 !
ways as we want 4th position to be occupied by ace
48
C3 4 C1 3 !
Hence required probability = 52
×
C4 4 !

Aliter :
‘NNNA’ is the arrangement then we desire in taking out cards, one by one
48 47 46 4
Hence required chance is × × ×
52 51 50 49
Self practice problems :
(14) An urn contains 7 red and 4 blue balls. Two balls are drawn at random with

RI
replacement. Find the probability of getting
(i) 2 red balls (ii) 2 blue balls (iii) one red and one blue ball
1 1
(15) Probabilities of solving a specific problem independently by A and B are and
2 3

A
respectively. If both try to solve the problem independently, find the probability that
(i) the problem is solved (ii) exactly one of them solves the problem.
(16) In throwing a pair of dies find the probability of getting an odd number on the first die

UH
and a total of 7 on both the dies.
(17) In throwing of a pair of dies, find the probability of getting a doublet or a total of 4.
(18) A bag contains 8 marbles of which 3 are blue and 5 are red. One marble is drawn at
random, its colour is noted and the marble is replaced in the bag. A marble is again
drawn from the bag and its colour is noted. Find the probability that the marbles will be
JA
(i) blue followed by red (ii) blue and red in any order (iii)of the same colour.
(19) A coin is tossed thrice. In which of the following cases are the events E and F
independent ?
(i) E : “the first throw results in head”.
F : “the last throw result in tail”.
(ii) E : “the number of heads is two”.
F : “the last throw result in head”.
LP

(iii) E : “the number of heads is odd ”.


F : “the number of tails is odd”.
49 16 56 2 1
Ans. (14) (i) (ii) (iii) (15) (i) (ii)
121 121 121 3 2
A

1 2 15 15 17
(16) (17) (18) (i) (ii) (iii)
12 9 64 32 32
(19) (i)
NK

(VI) Total probability theorem


If an event A can occur with one of the n mutually exclusive and exhaustive events B 1, B2, ....., Bn and
the probabilities P(A/B1), P(A/B2) .... P(A/Bn) are known, then
n

P(B )
SA

P(A) = i . P(A / Bi )
i =1
Proof :
The event A occurs with one of the n mutually exclusive and exhaustive events
B1, B2, B3,........,Bn
A = (A  B1)  (A  B2)  (A  B3)  ........  (A  Bn)
n
P(A) = P(A  B1) + P(A  B2) + ....... + P(A  Bn) =  P(A
i =1
 Bi )

Now,
P(A  Bi) = P(A) . P(Bi/A) = P(Bi) . P(A/Bi)
n
 P(A) = P(B )
i =1
i . P(A / Bi )

RI
Example # 25 : Box -  contains 6 red and 3 white balls while box -  contains 4 red and 3 white balls. A fair die
is thrown. If it turns up a multiple of 3, a ball is drawn from box -  else a ball is drawn from box -
. Find the probability that the ball drawn is white.

A
Solution : Let A be the event ‘a multiple of 3 turns up on the die’ and R be the event ‘the ball drawn is
white’
then P (ball drawn is white)

UH
2 3  2 3 25
= P(A) . P(R / A) + P (A) P(R / (A) ) = × + 1 −  =
6 9  6 7 63

Example # 26 : Cards of an ordinary deck of playing cards are placed into two heaps. Heap -  consists of all
the red cards and heap -  consists of all the black cards. A heap is chosen at random and a
JA
card is drawn, find the probability that the card drawn is a king.
Solution : Let  and  be the events that heap -  and heap -  are choosen respectively. Then
1
P() = P() =
2
Let K be the event ‘the card drawn is a king’
2 2
 P (K / ) = and P(K / ) =
LP

26 26
1 2 1 2 1
 P(K) = P () P(K / ) + P() P(K / ) = × + × = .
2 26 2 26 13
Self practice problems :
(20) Box -  contains 3 red and 6 blue balls while box - II contains 5 red and 4 blue balls. A
A

fair coin is tossed. If it turns up head, a ball is drawn from box - , else a ball is drawn
from box -  . Find the probability that the ball drawn is red.
NK

(21) There are 5 brilliant students in class XI and 8 brilliant students in class XII. Each class
has 50 students. The odds in favour of choosing the class XI are 2 : 3. If the class XI is
not chosen then the class XII is chosen. Find the probability of selecting a brilliant
student.
4 17
Ans. (20) (21) .
9 125
SA

(VII) Bayes’ theorem :


If an event A can occur with one of the n mutually exclusive and exhaustive events B 1, B2 , ....., Bn and
P(B ) . P(A / Bi )
the probabilities P(A/B1), P(A/B2) .... P(A/Bn) are known, then P(Bi / A) = n i
P(B )
i =1
i . P(A / Bi )

Proof :
The event A occurs with one of the n mutually exclusive and exhaustive events
B1, B2, B3,........,Bn
A = (A  B1)  (A  B2)  (A  B3)  ........  (A  Bn)
n
P(A) = P(A  B1) + P(A  B2) + ....... + P(A  Bn) =  P(A
i =1
 Bi )

Now, P(A  Bi) = P(A) . P(Bi/A) = P(Bi) . P(A/Bi)


P(Bi ) . P(A / Bi ) P(B ) . P(A / Bi )
P (Bi/A) = = ni
P(A)

P(A  Bi )
i =1

P(Bi ) . P(A / Bi )
P(Bi/A) =
P(B ) . P(A / B )
i i

RI
A
UH
2
Example # 27 : Pal’s gardener is not dependable, the probability that he will forget to water the rose bush is .
3
The rose bush is in questionable condition any how, if watered the probability of its withering is
JA
1 3
, if not watered, the probability of its withering is . Pal went out of station and upon
2 4
returning, he finds that the rose bush has withered, what is the probability that the gardener did
not water the bush.
[Here result is known that the rose bush has withered, therefore. Bayes’s theorem should be
used]
Solution : Let A = the event that the rose bush has withered
LP

Let A1 = the event that the gardener did not water.


A2 = the event that the gardener watered.
By Bayes’s theorem required probability,
P(A1 ) . P(A / A1)
P(A1/A) = .....(i)
A

P(A1 ) . P(A / A1) + P(A 2 ) . P(A / A 2 )


2 1
Given, P(A1) =  P(A2) =
3 3
NK

2 3
.
3 1 3 4 6 3
P(A/A1) = , P(A/A2) = From (1), P(A1/A) = = =
4 2 2 3 1 1 6+2 4
. + .
3 4 3 2
Example # 28 : There are 5 brilliant students in class XI and 8 brilliant students in class XII. Each class has 50
SA

students. The odds in favour of choosing the class XI are 2 : 3. If the class XI is not chosen
then the class XII is chosen. A student is chosen and is found to be brilliant, find the probability
that the chosen student is from class XI.
Solution : Let E and F be the events ‘Class XI is chosen’ and ‘Class XII is chosen’ respectively.
2 3
Then P(E) = , P(F) =
5 5
5 8
Let A be the event ‘Student chosen is brilliant’. Then P(A / E) = and P(A / F) = .
50 50
2 5 3 8 34
 P(A) = P(E) . P(A / E) + P(F) . P(A / F) = . + . = .
5 50 5 50 250
P(E) . P(A / E) 5
 P(E / A) = = .
P(E) . P(A / E) + P(F) . P(A / F) 17
Example # 29 : A pack of cards is counted with face downwards. It is found that one card is missing. One card
is drawn and is found to be red. Find the probability that the missing card is red.
Solution : Let A be the event of drawing a red card when one card is drawn out of 51 cards (excluding
missing card.) Let A1 be the event that the missing card is red and A 2 be the event that the
missing card is black.
Now by Bayes’s theorem, required probability,
P(A1 ) . (P(A / A1 )
P(A1/A) = ..........(i)
P(A1 ) . P(A / A1) + P(A 2 ) . P(A / A 2 )
In a pack of 52 cards 26 are red and 26 are black.
26
C 26 1
Now P(A1) = probability that the missing card is red = 52 1 = =
52 2

RI
C1
26 1
P(A2) = probability that the missing card is black = =
52 2
P(A/A1) = probability of drawing a red card when the missing card is red.

A
25
=
51
[Total number of cards left is 51 out of which 25 are red and 26 are black as the

UH
missing card is red]
26
Again P(A/A2) = Probability of drawing a red card when the missing card is black =
51
1 25
.
2 51 25
Now from (i), required probability, P(A1/A) = =
JA
1 25 1 26 51
. + .
2 51 2 51
Example # 30 : A bag contains 6 white and an unknown number of black balls ( 3). Balls are drawn one by
one with replacement from this bag twice and is found to be white on both occassion. Find the
probability that the bag had exactly ‘3’ Black balls.
Solution : A priori, we can think of the following possibilies
LP

(i) E1 6W , 0B
(ii) E2 6W , 1B
(iii) E3 6W , 2B
(iv) E4 6W , 3B
1
A

Clearly P(E1) = P(E2) = P(E3) = P(E4) =


4
Let ‘A’ be the event that two balls drawn one by one with replacement are both white therefore
E 
NK

we have to find P  4 
 A 
 A 
P   P(E4 )
 E4   E4 
By Baye’s theorem P   =
 A  A  A   A   A 
P    P(E1 ) + P 
SA

 . P(E2 ) + P   . P(E3 ) + P   . P(E 4 )


 1
E  2
E E
 3  E4 
 A  6 6  A  6 6  A  6 6 A 6 6
P   = × ; P   = × ; P   = × ; P   = × ;
 4
E 9 9 E
 3 8 8  2
E 7 7  1
E 6 6
1 1
E  
Putting values P  4 =
81 4
 A  1 1 1 1 1
+ + +
4  81 64 49 36 

Self practice problems :
(22) Box- contains 3 red and 2 blue balls while box- contains 2 red and 3 blue balls. A fair
coin is tossed. If it turns up head, a ball is drawn from box-, else a ball is drawn from
box-. If the ball drawn is red, then find the probability that the ball is drawn from
box-.
(23) Cards of an ordinary deck of playing cards are placed into two heaps. Heap -  consists
of all the red cards and heap -  consists of all the black cards. A heap is chosen at
random and a card is drawn, if the card drawn is found to be a king, find the probability
that the card drawn is from the heap - .
2 1
Ans. (22) (23)

RI
5 2

(VIII) Binomial probability theorem :


If an experiment is such that the probability of success or failure does not change with trials, then the

A
probability of getting exactly r success in n trials of such an experiment is nCr pr qn – r, where ‘p’ is the
probability of a success and q is the probability of a failure in one particular experiment. Note that
p + q = 1.

UH
Example 31 : A pair of dice is thrown 7 times. Find the probability of getting a doublet thrice
1
Solution : In a single throw of a pair of dice probability of getting a doublet is
6
1
considering it to be a success, p =
6
JA
1 5
 q=1– =
6 6
number of success r = 3
3 4
 1 5
 P(r = 3) = 7C3 p3 q4 = 35    
6 6
LP

Example # 32 : A pair of dice is thrown 4 times. If getting ‘a total of 7’ in a single throw is considered as a
success then find the probability of getting ‘a total of 7’ thrice.
6 1
Solution : p = probability of getting ‘a total of 7’ = =
36 6
A

1 5
 q=1– =
6 6
r = 3, n = 4
NK

3
 1 5 5
 P(r = 3) = 4C3 p3 q = 4     =
 6   6  324

Example # 33 : In an examination of 10 multiple choice questions (1 or more can be correct out of 3 options). A
student decides to mark the answers at random. Find the probability that he gets exactly two
questions correct.
SA

Solution : A student can mark 7 different answers to a MCQ with 3 option i.e. 3C1 + 3C2 + 3C3 = 7
1
Hence if he marks the answer at random, chance that his answer is correct = and
7
6 1 6
being incorrecting Thus p= ,q=
7 7 7
2 8
 1 6
P (2 success) = 10C2 ×   ×  
7 7
Example # 34 : A family has three children. Event ‘A’ is that family has at most one boy, Event ‘B’ is that family
has at least one boy and one girl, Event ‘C’ is that the family has at most one girl. Find whether
events ‘A’ and ‘B’ are independent. Also find whether A, B, C are independent or not.
Solution : A family of three children can have
(i) All 3 boys (ii) 2 boys + 1 girl (iii) 1 boy + 2 girls (iv) 3 girls
3
 1 1
(i) P (3 boys) = 3C0   = (Since each child is equally likely to be a boy or a girl)
2 8
2
 1 1 3
(ii) P (2 boys +1girl) = 3C1 ×   × =
2 2 8
(Note that there are three cases BBG, BGB, GBB)
1 2
 1  1 3
(iii) P (1 boy + 2 girls) = 3C2 ×   ×   =
 
2  
2 8
1
(iv) P (3 girls) =

RI
8
1
Event ‘A’ is associated with (iii) & (iv). Hence P(A) =
2
3

A
Event ‘B’ is associated with (ii) & (iii). Hence P(B) =
4
1
Event ‘C’ is associated with (i) & (ii). Hence P(C) =
2

UH
3
P(A  B) = P(iii) = = P(A) . P(B) . Hence A and B are independent of each other
8
P(A  C) = 0  P(A) . P(C) . Hence A, B, C are not independent

Self practice problems :


JA
(24) A box contains 4 red and 5 blue balls. Two balls are drawn successively without
replacement. If getting ‘a red ball on first draw and a blue ball on second draw’ is
considered a success, then find the probability of 2 successes in 3 performances.
(25) Probability that a bulb produced by a factory will fuse after an year of use is 0.2. Find
the probability that out of 5 such bulbs exectly 2 bulb will fuse after an year of use.
325 640
Ans. (24) (25)
LP

1944 3125

(IX) Probability distribution :


A probability distribution spells out how a total probability of 1 is distributed over several values of a
A

random variable (i.e. how ............................... possibilities)

(X) Expectation :
NK

If there are n possibilities A1, A2, .... An in an experiment having the probabilities p 1, p2, .........pn
respectively. If value M1, M2, ....., Mn are associated with the respective possibility. Then the expected
n
value of the experiment is given by p
i=1
i . Mi

Note:
SA

(i) Mean of any probability distribution of a random variable is given by :


 pi xi
µ= =  pi x i (Since  pi = 1)
 pi
Its also known as expectation.
(ii) Variance of a random variable is given by, 2 =  (xi – µ)2 . pi
 2 =  pi xi2 – µ2 (Note that SD = + 2 )
(iii) The probability distribution for a binomial variate ‘X’ is given by :
P(X = r) = nCr pr qn – r where P(X = r) is the probability of r successes.
P(r + 1) n − r p
The recurrence formula = . , is very helpful for quickly computing
P(r) r +1 q
P(1) . P(2) . P(3) etc. if P(0) is known.
Mean of Binomial Probability Distribution = np ; variance of Binomial Probability
Distribution = npq.
(iv) If p represents a person’s chance of success in any venture and ‘M’ the sum of money
which he will receive in case of success, then his expectations or probable value = pM
Example # 35 : A random variable X has the following probability distribution :
X 0 1 2 3 4 5 6 7
P(X) 0 2k 3k k 0 k 2+k 2k 2 5k 2
Determine
(i) k (ii) P(X < 3) (iii) P(X  6) (iv) P(0 < X  3)
Hints : Use  P(X) = 1 to determine k, P(X < 3) = P(0) + P(1) + P(2), P(X 6) = P(6) +(P(7),
P(0 < X  3) = P(0) + P(1) + P(2) + P(3)

RI
1 5 7 3
Ans. (i) (ii) (iii) (iv)
8 8 64 4
Example # 36 : A fair die is tossed. If 2, 3 or 5 occurs, the player wins that number of rupees, but if 1, 4, or 6

A
occurs, the player loses that number of rupees. First complete probability distribution table.
Hence find expectation.
Ai 2 3 5 1 4 6

UH
Solution : Mi 2 3 5 –1 –4 –6
Pi 1/6 1/6 1/6 1/6 1/6 1/6
Then expected value E of the game payoffs for the player
 1  1  1  1  1  1  1
JA
= 2   + 3   + 5   – 1   – 4   – 6  = – 
6 6 6 6 6 6 6
Since E is negative therefore game is unfavorable to the player.
Example # 37 : There are 50 tickets in a raffle (Lottery). There is 1 prize each of Rs. 800/-, Rs. 300/- and
Rs. 200/-. Remaining tickets are blank. Find the expected price of one such ticket.
Solution : Expectation =  piMi
LP

1
Probability of 1 ticket =
50
1 1 1 47
expected price = 800 × + 300 × + 200 × + × 0 = 26 Rs.
50 50 50 50
A

Example # 38 : A purse contains four coins each of which is either five rupees or two rupees coin. Find the
expected value of a coin in that purse.
Solution : Various possibilities of coins in the purse can be
5Rs. 2Rs.
NK

0 4 
1 3 
 equally likely 1/ 5
2 2 
3 1 

SA

4 0 
expected value per coin
1 1 1 1 1
= [5× 0+4×2] + [5×1+3×2] + [ 5 × 2 + 2 × 2]+ [ 5 × 3 + 2 ×1] [5 × 4 + 2 × 0] = 14
5 5 5 5 5

Example # 39 : A pair of dice is thrown 7 times. If getting a doublet is considered as a success, then find the
mean and variance of successes.
1
Solution : In a single throw of a pair of dice, probability of getting a doublet =
6
1 1 5
considering it to be a success, p =  q=1– =
6 6 6
1 7 1 5 35
mean = 7 × = , variance = 7 × × =
6 6 6 6 36

Example # 40 : A pair of dice is thrown 4 times. If getting a total of 7 in a single throw is considered as a
success then find the mean and variance of successes.
6 1
Solution : p = probability of getting a total of 7 = =
36 6
1 5
 q=1– =
6 6
1 2
 mean = np = 4 × =
6 3

RI
1 5 5
variance = npq = 4 × × =
6 6 9

Example # 41 : Difference between mean and variance of a Binomial variate is ‘1’ and difference between their

A
squares is ‘11’. Find the probability of getting exactly three success
Solution : Mean = np & variance = npq
therefore, np – npq = 1 ..........(i)

UH
n2p2 – n2p2q2 = 11 ..........(ii)
Also, we know that p + q = 1 ..........(iii)
5 1
Divide equation (ii) by square of (i) and solve, we get, q = , p = & n = 36
6 6
3 33
 1 5
Hence probability of ‘3’ success = 36C3 ×   ×  
JA
6 6
Self practice problems :
(26) From a bag containing 2 one rupee and 3 two rupee coins a person is allowed to draw
2 coins simultaneously ; find the value of his expectation.
(27) A box contains 2 red and 3 blue balls. Two balls are drawn successively without
replacement. If getting ‘a red ball on first draw and a blue ball on second draw’ is
LP

considered a success, then find the mean and variance of successes.


(28) Probability that a bulb produced by a factory will fuse after an year of use is 0.2. If
fusing of a bulb is considered an failure, find the mean and variance of successes for a
sample of 10 bulbs.
A

(29) A random variable X is specified by the following distribution law :

X 2 3 4
NK

P(X = x) 0.3 0.4 0.3


Find the variance of this distribution.

Ans. (26) Rs. 3.20 (27) mean = 2.1, 2 = .63 (28) mean = 8 and variance = 1.6
SA

(29) 0.6
 Marked questions are recommended for Revision.

PART - I : SUBJECTIVE QUESTIONS

Section (A) : Problems based on Classical definition of Probability (PRCD)


A-1. Write the sample space of the following experiment

(i) ‘Three coins are tossed’.

RI
(ii) ‘Selection of two children from a group of 3 boys and 2 girls without replacement’.

A-2. There are three events A, B, C, one of which must, and only one can, happen; the odds are 8 to 3

A
against A, 5 to 2 against B : find the odds against C.

A-3. If the letters of the word BANANA are arranged randomly, then find the probability that the word thus
formed does not contain the pattern BAN.

UH
A-4. Nine cards are labelled 0, 1, 2, 3, 4, 5, 6, 7, 8. Two cards are drawn at random and put on a table in a
sucessive order, and then the resulting number is read say 07(seven), 42(fourty two) and so on. Find
the probability that the number is even.
JA
A-5. Three persons A, B and C speak at a function along with 5 other persons. If the persons speak at
random, find the probability that A speaks before B and B speaks before C

A-6. (i) A rectangle is randomly selected from the grid of equally spaced squares as shown.
LP

Find the probability that the rectangle is a square.


A

(ii) Three of the six vertices of a regular hexagon are chosen at random. Then the probability that
the triangle with three vertices is equilateral is ‘p’ then 100p equals
NK

A-7. In throwing a pair of dice, find whether the two events


(i) E1 : ‘coming up of an odd number on first dice’ and E2 : ‘coming up of a total of 8’.
(ii) E1 : ‘coming up of 4 on first dice’ and E 2 : ‘coming up of 5 on the second dice’.
are mutually exclusive or not

A-8. In throwing of a pair of dice, find the probability of the event : total is ‘not 8’ and ‘not 11’.
SA

A-9. Before a race the chance of three runners, A, B, C were estimated to be proportional to 5, 3, 2, but
during the race A meets with an accident which reduces his chance to 1/3. What are the respective
chance of B and C now?

A-10. Tickets are numbered from 1 to 100. One ticket is picked up at random. Then find the probability that
the ticket picked up has a number which is divisible by 5 or 8.

A-11. Three cards are drawn at random from a pack of well shuffled 52 cards. Find the probability that
(i) all the three cards are of the same suit;
(ii) one is a king, the other is a queen and the third a jack.

A-12. In a throw of a pair of dice, then find the probability of ‘A total of 8 but not 11’.
A-13. Six boys and six girls sit in a row at random. Find the probability that boys and girls sit alternately

A-14. Four persons draw 4 cards from an ordinary pack find the chance
(1) that a card is of each suit
(2) that no two cards are of equal value.

Section (B) : Problems based on venn diagram & set theory (PRVD)
B-1. Prove that
P(A – B) = P(A) – P(A  B) = P(A  B) – P(B) = P(A B ) = 1 – P ( A  B)

RI
B-2 If P(A) = 0.7 and P(AB) = 0.5 find
(i) P (A– B ) (ii) P( A  B)

B-3. If P(A) = 0.4, P(B) = 0.48 and P(A  B) = 0.16, then find the value of each of the following :
P(A  B) P(A  B)

A
(i) (ii)
(iii) P(A  B) (iv) P((A  B)  (A  B))

B-4. There are three clubs A,B,C in a town with 40, 50, 60 members respectively 10 people are members of

UH
all the three clubs, 70 members belong to only one club. A member is randomly selected. Find the
probability that he has membership of exactly two clubs

Section (C) : Problems based on Conditional Probability/ Total probability & Bayes' theorem
JA
C-1. (i) In a two child family, one child is a boy. What is the probability that the other child is a girl ?

(ii) If the older child is a boy, then probability that the second child is a girl is

C-2. A fair dice is thrown untill a score of less than 5 points is obtained. Find the probability of obtaining not
less than 2 points on the last throw.
LP

C-3. A card is drawn from a well shuffled ordinary deck of 52 playing cards. Find the probability that the card
drawn is :
(i) A king or a queen (ii) A king or a spade
A

C-4. The odds against a certain event are 5 to 2, and the odds in favor of another event independent of the
former are 6 to 5 : find the chance that one at least of the events will happen.

C-5. A, B, C in order draws a card from a pack of cards, replacing them after each draw, on condition that
NK

the first who draws a spade shall win a prize : find their respective chances.

C-6. 6 persons A,B,C,D,E,F are arranged in row. Find the conditional probability that C & D are separated
given that A & B are together.

C-7. There are 5 brilliant students in class XI and 8 brilliant students in class XII. Each class has 50
SA

students. The odds in favour of choosing the class XI are 2 : 3. One of the classes is chosen randomly
and then a student is randomly selected. Find the probability of selecting a brilliant student.

C-8. Box – I contains 5 red and 2 blue balls while box – II contains 2 red and 6 blue balls. A fair coin is
tossed. If it turns up head, a ball is drawn from box–I, else a ball is drawn from box–II. Find the
probability of each of the following :
(i) A red ball is drawn
(ii) Ball drawn is from box–I if it is blue

C-9. Two cards are drawn successively from a well-shuffled ordinary deck of 52-playing cards without
replacement and is noted that the second card is a king. Find the probability of the event ‘first card is
also a king’.
C-10. 12 cards, numbered 1 to 12, are placed in a box, mixed up throughly and then a card is drawn at
random from the box. If it is known that the number on the drawn card is more than 3, find the
probability that it is an even number.

C-11. In a building programme the event that all the materials will be delivered at the correct time is M, and
the event that the building programme will be completed on time is F. Given that P(M) = 0.8 &
P(M  F) = 0.65. If P(F) = 0.7, find the probability that the building programme will be completed on
time if all the materials are not delivered at the correct time.

Section (D) : Problem based on Binomial Distribution / Expectation / mean & Variance

RI
D-1. South African cricket captain lost the toss of a coin 13 times out of 14. Then find the chance of this
happening.

D-2. In an examination of 10 multiple choice questions (1 or more can be correct out of 4 options). A student

A
decides to mark the answers at random. Find the probability that he gets exactly two questions correct.
(Assume he attempts all the questions)

UH
D-3. Three cards are drawn successively with replacement from a well shuffled deck of 52 playing cards. If
getting a card of spade is considered a success, find the probability distribution of the number of
successes.

D-4. A had in his pocket a 100 Rupee and four 10 rupee notes; taking out two notes at random he promises
to give them to B and C. What is the worth of C’s expectation?
JA
D-5. A box contains 2 red and 3 blue balls. Two balls are drawn successively with replacement. If getting ‘a
red ball on first draw and a blue ball on second draw’ is considered a success, then write the probability
distribution of successes. It is given that the above experiment is performed 3-times,

D-6. A coin is tossed 5-times. Find the mean and variance of the probability distribution of appearance of
heads on the tosses.
LP

PART - II : ONLY ONE OPTION CORRECT TYPE


Section (A) : Problems based on Classical definition of Probability (PRCD)
A

A-1. In drawing of a card from a well shuffled ordinary deck of playing cards the events ‘card drawn is
spade’ and ‘card drawn is an ace’ are
NK

(A) mutually exclusive (B) equally likely


(C) forming an exhaustive system (D) none of these

A-2. A 9 digit number using the digits 1, 2, 3, 4, 5, 6, 7, 8 & 9 is written randomly without repetition. The
probability that the number will be divisible by 9 is:
(A) 1/9 (B) 1/2 (C) 1 (D) 9!/99
SA

A-3. Entries of a 2 × 2 determinant are chosen from the set {–1, 1}. The probability that determinant has zero
value is
1 1 1
(A) (B) (C) (D) none of these
4 3 2

A-4. A dice is thrown a fixed number of times. If probability of getting even number 3 times is same as the
probability of getting even number 4 times, then probability of getting even number exactly once is
1 3 5 7
(A) (B) (C) (D)
4 128 64 128
A-5. A and B throw with two dice ; if A throws 9, then B’s chance of throwing a higher number equals
1 1 1 2
(A) (B) (C) (D)
6 3 2 3
A-6. If an integer q is chosen at random in the interval – 10  q  10, then the probability that the roots of the
3q
equation x2 + qx + + 1 = 0 are real is
4
16 15 14 17
(A) (B) (C) (D)
21 21 21 21

A-7. The chance that a 13 card combination from a pack of 52 playing cards is dealt to a player in a game of
bridge, in which 9 cards are of the same suit, is
4 . 13 C9 . 39 C4 4! . 13 C9 . 39 C4 13
C9 . 39 C4 13
C9 . 39 C4
(A) 52
(B) 52
(C) 52
(D) 2. 52
C13 C13 C13 C13

RI
A-8. A bag contains 7 tickets marked with the numbers 0, 1, 2, 3, 4, 5, 6 respectively. A ticket is drawn &
replaced. Then the chance that after 4 drawings the sum of the numbers drawn is 8 is:
(A) 165/2401 (B) 149/2401 (C) 3/49 (D) 1/49

A
A-9. A & B having equal skill, are playing a game of best of 5 points. After A has won two points & B has
won one point, the probability that A will win the game is:

UH
(A) 1/2 (B) 2/3 (C) 3/4 (D) 2/5

Section (B) : Problems based on venn diagram & set theory (PRVD)
B-1. If two subsets A and B of set S containing n elements are selected at random, then the probability that
A  B =  and A  B = S is
JA
4
1 1 3 1
(A) (B) (C)   (D)
2 n
2 4 3n

3 2
B-2. If P(A) = and P(B) = then –
5 3
LP

(i) The range of values of P(A  B) is


2 9  2   1  4 3
(A)  ,  (B)  3 ,1 (C) 0,  (D)  , 
 5 10     3  15 5 

The range of values of P(A  B) is


A

(ii)
2 9  2   1  4 3
(A)  ,  (B)  ,1 (C) 0,  (D)  , 
 5 10  3   3  15 5 
NK

(iii) The range of values of P(A  B) is


2 9  2   1  4 3
(A)  ,  (B)  ,1 (C) 0,  (D)  , 
 5 10  3   3  15 5 

Let X = {1,2, ..... , 10}, if set A and B are formed from elements of X the probability that n (A  B) = 2, is
SA

B-3
10 10
C 10
3 3 38
(A) 102 (B) 5.   (C)   (D)
4 4 4 410

B-4 If probability that exactly one of events A, B, C occurs, is 0.6 and probability that none of A,B,C occur
is 0.2, then probability that atleast two of A,B,C occur is
(A) 0.6 (B) 0.4 (C) 0.8 (D) 0.2
Section (C) : Problems based on Conditional Probability/ Total probability & Bayes'
theorem
 
C-1. The odds that a book will be favourably reviewed by three independent critics are 5 to 2, 4 to 3, and 3
to 4 respectively. Then the probability that of the three reviews a majority will be favourable.
163 209 209 208
(A) (B) (C) (D)
343 343 387 387

C-2. In throwing a pair of dice, the events ‘coming up of 6 on Ist dice’ and ‘a total of 7 on both the dice’ are
(A) mutually exclusive (B) forming an exhaustive system
(C) independent (D) dependent

RI
C-3. A dice is thrown twice and the sum of the numbers appearing is observed to be 8. The conditional
probability that the number 5 has appeared at least once is
(A) 1/6 (B) 2/5 (C) 3/5 (D) 1/2

A
C-4. An instrument consists of two units. Each unit must function for the instrument to operate. The reliability
of the first unit is 0.9 and that of the second unit is 0.8. The instrument is tested & fails. The probability
that “only the first unit failed & the second unit is sound” is “

UH
1 2 3 4
(A) (B) (C) (D)
7 7 7 7

C-5. A pack of cards is counted with face downwards. It is found that one card is missing. One card is drawn
and is found to be red. Then the probability that the missing card is red.
JA
25 26 1 25
(A) (B) (C) (D)
51 51 2 52

C-6. Pal’s gardner is not dependable, the probability that he will forgot to water the rose bush is 2/3. The
rose bush is in questionable condition. Any how if watered, the probability of its withering is 1/2 & if not
watered then the probability of its withering is 3/4. Pal went out of station & after returning he finds that
rose bush has withered. Then the probability that the gardner did not water the rose bush is.
LP

(A) 3/4 (B) 2/5 (C) 1/4 (D) 1/2

C-7. A dice is weighted so that the probability of different faces to turn up is as given

Number 1 2 3 4 5 6
A

Probability 0.2 0.1 0.1 0.3 0.1 0.2


NK

If P(A/B) = p1 and P(B/C) = p2 and P(C/A) = p3 then the values of p1, p2, p3 respectively are -
Take the events A, B & C as A = {1, 2, 3}, B = {2, 3, 5} and C = {2, 4, 6}
2 1 1 1 1 1 1 1 1 2 1 1
(A) , , (B) , , (C) , , (D) , ,
3 3 4 3 3 6 4 3 6 3 6 4
SA

Section (D) : Problem based on Binomial Distribution / Expectation / mean & Variance
D-1. A bag contains 2 white & 4 black balls. A ball is drawn 5 times, each being replaced before another is
drawn. The probability that atleast 4 of the balls drawn are white is:
(A) 4/81 (B) 10/243 (C) 11/243 (D) 8/243
D-2. In a series of 3 independent trials the probability of exactly 2 success is 12 times as large as the
probability of 3 successes. The probability of a success in each trial is:
(A) 1/5 (B) 2/5 (C) 3/5 (D) 4/5
D-3. A coin is tossed n times, what is the chance that the head will present itself an odd number of times.
1 2 1 2
(A) (B) (C) (D)
2 3 4 5
D-4. From a bag containing 2 one rupee and 3 two rupee coins a person is allowed to draw 2 coins
randomly then the value of his expectation.
(A) Rs. 5.10 (B) Rs. 2.30 (C) Rs. 4.30 (D) Rs. 3.20

D-5. A & B throw with one dice for a stake of Rs. 99/- which is to be won by the player who first throws 4. If A
has the first throw then their respective expectations of rupees are:
(A) 50 & 49 (B) 54 & 45 (C) 45 & 54 (D) 33 & 66

D-6. A fair coin is tossed 99 times. If X is the number of times heads occur, if P (X = r) is maximum then sum
of possible values of r is
(A) 98 (B) 99 (C) 101 (D) 104

RI
PART - III : MATCH THE COLUMN
 
Column –  Column – 

A
1.

(A) If the probability that units digit in square of an even integer is 4 (p) 1
is p, then the value of 5p is

UH
1
(B) If A and B are independent events and P(A  B) = , (q) 2
6
1 B
P(A) = , then 6P   =
3 A
(C)
JA
One mapping is selected at random from all mappings of (r) 3
the set S = {1, 2, 3, ......, n} into itself. If the probability that
3
the mapping is one-one is , then the value of n is
32

(D) A boy has 20% chance of hitting at a target. Let p denote (s) 4
the probability of hitting the target for the first time at the n th
LP

trial. If p satisfies the inequality 625p 2 – 175p + 12  0, then


value of n is

2. Column –  Column – 
A

(A) A pair of dice is thrown. If total of numbers turned up (p) 5/16


on both the dice is 8, then the probability that the
number turn up on the second dice is 5’ is
NK

(B) A box contains 4 white and 3 black balls. Two balls are (q) 1/3
drawn successively and is found that second ball is
white, then the probability that Ist ball is also white is
1
(C) A biased coin with probability p, 0 < p < 1 of heads is (r)
2
SA

tossed until a head appears for the first time. If the


probability that the number of tosses required is even is 2/5,
then p equals
1
(D) A coin whose faces are marked 3 and 5 is tossed 4 times : what (s)
5
is the probability that the sum of the numbers thrown being less,
than 15?
 Marked questions are recommended for Revision.

PART - I : ONLY ONE OPTION CORRECT TYPE

1. A local post office is to send M telegrams which are distributed at random over N communication
channels, (N > M). Each telegram is sent over any channel with equal probability. Chance that not more
than one telegram will be sent over each channel is:
N N N N
CM . M ! CM . N ! CM . M ! CM . N !
(C) 1 − (D) 1 −

RI
(A) M
(B) N N
N M M NM

2. A cube painted red on all sides, is cut into 125 equal small cubes. A small cube when picked up is
found to show red colour on one of its faces. Then the probability that two more faces also show red

A
colour.
4 4 8 3
(A) (B) (C) (D)
49 120 49 49

UH
3. A car is parked by an owner in a parking lot of 25 cars in a row, including his car not at either end. On
his return he finds that exactly 15 placed are still occupied. The probability that both the neighboring
places are empty is
91 15 15 17
(A) (B) (C) (D)
JA
276 184 92 92

4. A has 3 tickets in a lottery containing 3 prizes and 9 blanks; B has 2 tickets in a lottery containing 2
prizes and 6 blanks. Compare their chances of success
(A) 952 / 715 (B) 950 / 952 (C) 952 / 710 (D) 425/952

5. A 2n digit number starts with 2 and all its digits are prime, then the probability that the sum of all 2
LP

consecutive digits of the number is prime, is


(A) 4 × 23n (B) 4 × 2–3n (C) 23n (D) 22n

6. A fair coin is tossed eight times, then find the probability that resulting sequence of heads and tails
A

looks the same when viewed from the beginning or from the end.
(A) 1/8 (B) 1/16 (C) 1/4 (D) 1/2

7. An urn contains 'm' green and 'n' red balls. K (< m, n) balls are drawn and laid aside, their colur being
NK

ignored. Then one more ball is drawn. Then the probability that it is green.
m n 2n 3n
(A) (B) (C) (D)
m+n m+n m+n m+n

8. In a regular decagon find the probability that the two diagonal chosen at random will intersect inside the
SA

polygon.
6 12 5 3
(A) (B) (C) (D)
17 17 17 17

9. An urn contains m white and n black balls. A ball is drawn at random and is put back into the urn along
with k additional balls of the same colour as that of the ball drawn. A ball is again drawn at random.
Then the probability that the ball drawn now is white.
m n 2n 2m
(A) (B) (C) (D)
m+n m+n m+n m+n

10. There are two urns. There are m white & n black balls in the first urn and p white & q black balls in the
second urn. One ball is taken from the first urn & placed into the second. The probability of drawing a
white ball from the second urn is -
(p + 1)n + pm (p + 1)m + pn
(A) (B)
(m + n) (p + q + 1) (m + n) (p – q + 1)
(p + 1)m + 2pn (p + 1)m + pn
(C) (D)
(m + n) (p + q + 1) (m + n) (p + q + 1)

11. The chance that the top card in the deck is a diamond given that the fourth card from the top is a
eight in well shuffled deck.
1 2 1 2
(A) (B) (C) (D)  
4 3 2 5

RI
12. A fair coin is tossed 9 times the probability that at least 5 consecutive heads occurs is
9
5 3  1 5
(A) (B) (C) 9C5   (D)
64 32 2 29

A
13. A man has 10 coins and one of them is known to have two heads. He takes one at random and tosses
it 5 times and it always falls head. Then the chance that it is the coins with two heads.
32 32 23 19

UH
(A) (B) (C) (D)
41 51 32 32

14. 2 hunters A & B shot at a bear simultaneously. The bear was shot dead with only one hole in its hide.
Probability of A shooting the bear 0.8 & that of B shooting the bear is 0.4. The hide was sold for
Rs. 280/-. If this sum of money is divided between A & B in a fair way, then find the share of A
(A) 130 (B) 240
JA (C) 200 (D) 190

15. A number is chosen at random from the numbers 10 to 99. A number whose product of digits is 12 will
be called a good number. If he choose three numbers with replacement then the probability that he will
choose a good number at least once is
(A) 0.872 (B) 0.127 (C) 0.562 (D) 0.461
LP

PART - II : SINGLE AND DOUBLE VALUE INTEGER TYPE


1. Seven digits from the number 1, 2, 3, 4, 5, 6, 7, 8 & 9 are written in random order. The probability that
p
A

this seven digit number is divisible by 9 is then (p + q) equals. (Where p & q are co-prime natural
q
numbers)
NK

2. Three numbers are chosen at random without replacement from {1, 2, 3,......, 10}. The probability that
p
the minimum of the chosen numbers is 3 or their maximum is 7 is then (q – 2p) equals.
q
(Where p & q are co-prime natural numbers)
SA

3. In a multiple choice question there are 4 alternative answers of which 1, 2, 3 or all may be correct. A
candidate will get marks in the question only if he ticks all the correct answer. The candidate decides to
tick answers at random. If he is allowed upto 5 chances to answer the question, If the probability that he
will get the marks in the question is p then 3p equals

4. 3 firemen X, Y and Z shoot at a common target. The probabilities that X and Y can hit the target are
2/3 and 3/4 respectively. If the probability that exactly two bullets are found on the target is 11/24, then
the probability of Z to hit the target is ‘’ then 6 equals

5. A mapping is selected at random from all the mappings defined on the set A consisting of three
p
distinct elements. Probability that the mapping selected is one to one is (where p and q are co-prime
q
natural numbers) then p + q is
6. A card is drawn from a pack, the card is replaced & the pack shuffled. If this is done 6 times, the
p
probability that the cards drawn are 2 hearts, 2 diamonds & 2 black cards is then total number of
q
proper divisors of (pq). (Where p & q are co-prime natural numbers):

7. There is a three volume dictionary among 40 books arranged on a shelf in random order. Then the
raciprocal of probability of these volumes standing in increasing order from left to right (the volumes are
not necessarily kept side by side) is

8. There are 4 urns. The first urn contains 1 white & 1 black ball, the second urn contains 2 white & 3 black
balls, the third urn contains 3 white & 5 black balls & the fourth urn contains 4 white & 7 black balls. The

RI
i2 + 1
selection of each urn is not equally likely. The probability of selecting ith urn is (i=1,2,3, 4). If we
34
p
randomly select one of the urns & draw a ball, then the probability of ball being white is then sum of

A
q
digits of p is. (Where p & q are co-prime natural numbers) :

UH
9. In a Nigerian hotel, among the english speaking people 40% are English & 60% Americans. The
English & American spellings are "RIGOUR" & "RIGOR" respectively. An English speaking person in the
hotel writes this word. A letter from this word is chosen at random & found to be a vowel. If the
probability that the writer is an Englishman is a/b, then (b – a) equals. (Where a & b are co-prime
natural numbers)
JA
10. Mr. Modi is a professional tea taster. When given a high grade tea, he will identify it with probability 0.9
correctly as high grade and will mistake it for a low grade tea with probability 0.1 . When given a low
grade tea, he will identify it with probability 0.8 correctly as low grade tea and will mistake it for a high
grade tea with probability 0.2. Suppose that Mr. Modi is given ten unlabelled cups of tea, three with high
grade and seven with low grade tea. He randomly picks a cup, tries the tea and solemnly says “high
grade tea”. If the probability that the tea he tasted was low grade tea is express in form of a/b then
(3a – b) equals. (Where a & b are co-prime natural numbers)
LP

11. A gambler has one rupee in his pocket. He tosses an unbiased normal coin unless either he is ruined or
unless the coin has been tossed for a maximum of five times. If for each head he wins a rupee and for
p
each tail he looses a rupee, then if the probability that the gambler is ruined is (where p and q are
A

q
co-prime natural numbers) then p + q is
NK

12. 3 cards are given, one of them is red on both sides, one is blue on both sides & one is blue on one side
and red on the other side. One of them is chosen randomly & put on the table. It shows red colour on
p
the upper side. If probability of the other side of the card being red is (where p and q are co-prime
q
natural numbers) then q – p is
SA

13. In a purse there are 10 coins, all 5 paise except one which is a rupee. In another purse there are
10 coins all 5 paise. 9 coins are taken out from the former purse & put into the latter & then 9 coins are
taken out from the latter & put into the former. Then the chance that the rupee is still in the first purse is
p then 19p – 9 equals :
 
14. A Teacher wrote either of words “PARALLELOGRAM” or “PARALLELOPIPED” on board but due to
malfunction of marker words is not properly written and only two consecutive letters "RA" are visible
p
then the chance that the written word is “PARALLELOGRAM” is then (p + q) equals. (Where p & q
q
are co-prime natural numbers) :
15. The numbers ‘a and b ’ are randomly selected from the set of natural numbers. Probability that the
p
number 3a + 7b has a digit equal to 8 at the units place, is then p + q is : (Where p & q are co-prime
q
natural numbers)

16. Suppose that of all used cars of a particular year, 30% have bad brakes. You are considering buying a
used car of that year. You take the car to a mechanic to have the brakes checked. The chance that the
mechanic will give you wrong report is 20%. Assuming that the car you take to the mechanic is selected
"at random" from the population of cars of that year. The odds in favor of chance that the car's brakes
are good given that the mechanic says its brakes are good is m : n then (m – 7n) equals. (Where m & n
are co-prime natural numbers)

RI
17. A bag contains (n + 1) coins. It is known that one of these coins has a head on both sides, whereas the
other coins are normal. One of these coins is selected at random & tossed. If the probability that the
toss results in head, is 7/12, then the value of n is.

A
18. In a certain factory machines A, B and C produce bolts of their production A, B and C produce 2%, 1%
and 3% defective bolts respectively. Machine A produces 35% of the total output of bolts machine
B produces 25% and machine C produces 40%. A bolts is chosen at random from the factory’s

UH
production and its found to be defective. The odds in favor that it was produced on machine C is m : n
then (m –n) equals. (Where m & n are co-prime natural numbers)

19. In each of a set of games it is 2 to 1 in favor of the winner of the previous game. If the probabilty that
p
the player who wins the first game shall win three at least of the next four is then pq equals :
JA q
(Where p & q are co-prime natural numbers)

1 1 1
20. A couple has one or two or three children with probability , and respectively. Probability of a
4 2 4
p
couple having exactly four grandchildren in such a type of society is then sum of digit of q equals.
LP

q
(Where p & q are co-prime natural numbers)

PART - III : ONE OR MORE THAN ONE OPTIONS CORRECT TYPE


A

1. In throwing a dice let A be the event ‘coming up of an odd number’, B be the event ‘coming up of an even
number’, C be the event ‘coming up of a number  4’ and D be the event ‘coming up of a number < 3’,
NK

then
(A) A and B are mutually exclusive and exhautive
(B) A and C are mutually exclusive and exhautive
(C) A, C and D form an exhautive system
(D) B, C and D form an exhautive system
SA

2. If M & N are any two events, then which one of the following represents the probability of the occurence
of exactly one of them ?
(A) P (M) + P (N) − 2 P (M  N) (B) P (M) + P (N) − P (M  N)
( ) ( )
(C) P M + P N − 2 P M  N ( ) ( ) (
(D) P M  N + P M  N )
3. Let 0 < P(A) < 1, 0 < P(B) < 1 & P(A  B) = P(A) + P(B) − P(A). P(B), then:
(A) P(B/A) = P(B) − P(A) (B) P(AC  BC) = P(AC) + P(BC)
(C) P((A  B)C) = P(AC). P(BC) (D) P(A/B) = P(A)

4. A box contains 11 tickets numbered from 1 to 11. Two tickets are drawn similtaneously at random.
Let E1 denotes the event that the sum of the numbers on the tickets drawn is even
and E2 denotes the event that the sum of the numbers on the tickets drawn is odd
Which of the following hold good?
(A) P(E1/E2) = P(E2/E1) (B) E1 and E2 are exhaustive
(C) P(E2) > P(E1) (D) E1 and E2 are equally likely

5. The probabilities of events, A  B, A, B & A  B are respectively in A.P. with second term equal to the
common difference. Therefore A & B are :
(A) mutually exclusive
(B) independent
(C) such that one of them must occur
(D) such that one is twice as likely as the other
6. A bag contains four tickets marked with numbers 112, 121, 211, 222. One ticket is drawn at random

RI
from the bag. Let Ei(i = 1, 2, 3) denote the event that ith digit on the ticket is 2. Then
(A) E1 and E2 are independent (B) E2 and E3 are independent
(C) E3 and E1 are independent (D) E1, E2, E3 are independent

A
7. In an experimental performance of a single throw of a pair of unbiased normal dice, three events E 1, E2
& E3 are defined as follows:
E1 : getting a prime numbered face on each dice

UH
E2 : getting the same number on each dice
E3 : getting a sum total of dots on two dice equal to 8. Then:
(A) the events E1, E2 & E3 are mutually exclusive
(B) the events E1, E2 & E3 are not pairwise mutually exclusive
(C) the events E1, E2 are independent
JA
(D) P(E3E1) = 2/9.
8. The probability that a bulb produced by a factory will fuse after an year of use is 0.1. Then the
probability that out of 4 such bulbs
94
(A) None of then bulb will fuse after an year of use is 4
10
LP

1
(B) More then three bulbs will fuse after an year of use is 4
10
9999
(C) Not more then three bulbs will fuse after an year of use is
10000
A

1
(D) All the bulbs will fuse after an year of use is
104
NK

9. If 4 whole numbers taken at random are multiplied together


16
(A) Probability that the last digit in the product is 1, 3, 7 or 9 is
625
369
(B) Probability that the last digit in the product is ‘5’ is
104
SA

3727
(C) Probability that the last digit in the product is 0 is
104
2357
(D) Probability that the last digit in the product is 0 is
104
10. The probability that 4th power of a positive integer ends in the digit  is P()
4 4 1 1
(A) P(6) = (B) P(1) = (C) P(5) = (D) P(0) =
10 10 10 10
11. Mean and variance of a Binomial variate of 10 trials of the experiment are in the ratio of 3 : 2.
(A) The most probable number of happening of variable is 3
(B) Sum of the mean and variance is 10
8064
(C) Probability of getting exactly 5 success is
310
(D) The most probable number of happening of variable is 5

12. A student appears for tests I, II & III. The student is successful if he passes either in tests I & II or tests I
& III. The probabilities of the student passing in the tests I, II & III are p, q &
1/2 respectively. If the probability that the student is successful is 1/2, then:
(A) p = 1, q = 0 (B) p = 2/3, q = 1/2
(C) p = 3/5, q = 2/3 (D) there are infinitely many values of p & q.

13. A student has to match three historical events i.e. Dandi March, Quit India Movement and Mahatma

RI
Gandhi’s assasination with the years 1948, 1930 and 1942 and each event happens in different years.
The student has no knowledge of the correct answers and decides to match the events and years
randomly. Let Ei : (0  i  3) denote the event that the student gets exactly i correct answer, then
(A) P(Eo) + P(E3) = P(E1) (B) P(Eo) . P(E1) = P(E3)

A
(C) P(E0  E1) = P(E2) (D) P(E0) + P(E1) + P(E3) = 1

14. For any two events A & B defined on a sample space,


P(A) + P(B) − 1

UH
(A) P ( A B )  , P (B)  0 is always true
P(B)
( )
(B) P A  B = P (A) - P (A  B)
(C) P (A  B) = 1 - P (Ac). P (Bc), if A & B are independent
(D) P (A  B) = 1 - P (Ac). P (Bc), if A & B are disjoint
JA
15. An unbiased coin is tossed n times. Let X denote the number of times head occurs. If
P(X = 4), P (X = 5) and P(X = 6) are in AP, then the value of n can be
(A) 7 (B) 10 (C) 12 (D) 14
x
 1
16. A random variable x takes values 0, 1, 2, 3, ....., with probability proportional to (x + 1)   , then
5
LP

16 112 9 25
(A) P(x = 0) = (B) P(x  1) = (C) P(x  1) = (D) E(x) =
25 125 25 32
17. Let X be a set containing ‘n’ elements. If two subsets A and B of X are picked at random. The
A

probability of A and B having same number of elements


2n
C 1 1.3.5..... ( 2n − 1) 3n
(A) 2nn (B) 2n (C) (D)
2 Ccn 2n.n! 4n
NK

18. A square matrix of order 3 × 3 is formed using the elements of the set {–2016, 0, 2016}
1
(A) Probability of getting a matrix which is symmetric 3 is
3
1
(B) Probability of getting a matrix which is skew symmetric 6 is
3
SA

1
(C) Probability of getting a matrix which has maximum trace is 3
3
1
(D) Probability of getting a matrix which has minimum trace is 3
3
PART - IV : COMPREHENSION
Comprehension # 1

If sample space contains infinite number of points then with the help of geometry that is length, area,
volume many problems of probibility can be solved
For example : A point is selected randomely inside the circle then the probability that it is nearer to
centre then its circumeference
r 2
Favourable Area 1
Probability = = 42 =

RI
Total Area r 4

1. A sphere is circumscribed over a cube. Find the probability that a point lies inside the sphere, lies

A
outside the cube.
2 1 1 2
(A) 1 – (B) 1 – (C) 1 – (D) 1 –
 3  3 2 3 2 3

UH
2. A parallelogram is inscribed inside a circle of radius 10 cm. One side of parallelogram being 12 cms.
Then the probability that a point inside the circle also lies inside the parallelogram.
48 24 42 1
(A) (B) (C) (D)
25 25 25 2
JA
3. The sides of a rectangle are chosen at random, each less than 10 cm, all such lengths being equally
likely. The chance that the diagonal of the rectangle is less than 10 cm is
(A) 1/10 (B) 1/20 (C) /4 (D) /8

Comprehension # 2
LP

A JEE aspirant estimates that he will be successful with an 80 percent chance if he studies 10 hours
per day, with a 60 percent chance if he studies 7 hours per day and with a 40 percent chance if he
studies 4 hours per day. He further believes that he will study 10 hours, 7 hours and 4 hours per day
with probabilities 0.1, 0.2 and 0.7 respectively.
A

4. The chance he will be successfull is


(A) 0.28 (B) 0.38 (C) 0.48 (D) 0.58
NK

5. Given that he is successful the chance that he studied for 4 hours, is


6 7 8 9
(A) (B) (C) (D)
12 12 12 12
SA

6. Given that he does not achieve success, the chance that he studied for 4 hour, is
18 19 20 21
(A) (B) (C) (D)
26 26 26 26
Comprehension # 3
A bag contain 6 Red and 4 White balls. 4 balls are drawn one by one without replacement and were
found to be atleast 2 white.
7. Then the probablity that next draw of a ball from this bag will give a white ball.
34 19 90 24
(A) (B) (C) (D)
115 115 115 115
8. If the next draw of a ball from this bag will give a white ball then the probablity that the drawn of four
balls initially contain two white and two red balls.
8 15 13 13
(A) (B) (C) (D)
17 17 17 34

Comprehension # 4

Eight digit number can be formed using all the digits 1,1,2,2,3,3,4,5.

9. A number is selected at random then the probability such that no two identical digits appear together
37 43 17 23
(A) (B) (C) (D)

RI
84 84 84 84

10. A number is selected at random then the probability that it has exactly two pair of identical digits
occuring together

A
1 2 3 4
(A) (B) (C) (D)
7 9 5 9

UH
JA
LP
A
NK
SA
 Marked questions are recommended for Revision.

PART - I : JEE (ADVANCED) / IIT-JEE PROBLEMS (PREVIOUS YEARS)

* Marked Questions may have more than one correct option.


Comprehension (Q.1 to 3)

RI
A fair die is tossed repeatedly until a six is obtained. Let X denote the number of tosses required.

1. The probability that X = 3 equals [IIT-JEE 2009, Paper-1, (4, –1), 80]

A
25 25 5 125
(A) (B) (C) (D)
216 36 36 216

UH
2. The probability that X  3 equals [IIT-JEE 2009, Paper-1, (4, –1), 80]
125 25 5 25
(A) (B) (C) (D)
216 36 36 216

3. The conditional probability that X  6 given X > 3 equals [IIT-JEE 2009, Paper-1, (4, –1), 80]
JA
125 25 5 25
(A) (B) (C) (D)
216 216 36 36

4. Let  be a complex cube root of unity with   1. A fair die is thrown three times. If r 1, r2 and r3 are the
numbers obtained on the die, then the probability that r1 + r2 + r3 = 0 is
LP

[IIT-JEE 2010, Paper-1, (3, –1), 84]


1 1 2 1
(A) (B) (C) (D)
18 9 9 36
A

4 1
5. A signal which can be green or red with probability and respectively, is received by station A and
5 5
3
then transmitted to station B. The probability of each station receiving the signal correctly is . If the
NK

4
signal received at station B is green, then the probability that the original signal was green is
[IIT-JEE 2010, Paper-2, (5, –2), 79]
3 6 20 9
(A) (B) (C) (D)
5 7 23 20
SA

Comprehension (Q.6 & 7)

Let U1 and U2 be two urns such that U1 contains 3 white and 2 red balls, and U 2 contains only 1 white
ball. A fair coin is tossed. If head appears then 1 ball is drawn at random from U 1 and put into U2.
However, if tail appears then 2 balls are drawn at random from U 1 and put into U2. Now 1 ball is drawn
at random from U2. [IIT-JEE 2011, Paper-1, (3, –1), 80]

6. The probability of the drawn ball from U 2 being white is


13 23 19 11
(A) (B) (C) (D)
30 30 30 30

7. Given that the drawn ball from U 2 is white, the probability that head appeared on the coin is
17 11 15 12
(A) (B) (C) (D)
23 23 23 23

11
8.* Let E and F be two independent events. The probability that exactly one of them occurs is and the
25
2

RI
probability of none of them occurring is . If P(T) denotes the probability of occurrence of the event
25
T, then [IIT-JEE 2011, Paper-2, (4, 0), 80]
4 3 1 2
(A) P(E) = , P(F) = (B) P(E) = , P(F) =

A
5 5 5 5
2 1 3 4
(C) P(E) = , P(F) = (D) P(E) = , P(F) =
5 5 5 5

UH
9*. A ship is fitted with three engines E 1, E2 and E3. The engines function independently of each other with
1 1 1
respective probabilities , and . For the ship to be operational at least two of its engines must
2 4 4
function. Let X denote the event that the ship is operational and let X 1, X2 and X3 denotes respectively
JA
the events that the engines E1 E2 and E3 are functioning. Which of the following is (are) true ?
3
(A) P  X1c | X  =
  16
7
(B) P[ Exactly two engines of the ship are functioning | X] =
8
LP

5
(C) P[X | X2] =
16
7
(D) P[X | X1] = [IIT-JEE 2012, Paper-1, (4, 0), 70]
16
A

10. Four fair dice D1, D2, D3 and D4 each having six faces numbered 1,2,3,4,5 and 6 are rolled
simultaneously. The probability that D4 shows a number appearing on one of D1, D2 and D3 is
NK

[IIT-JEE 2012, Paper-2, (3, –1), 66]


91 108 125 127
(A) (B) (C) (D)
216 216 216 216

1 1 1
11*. Let X and Y be two events such that P(X | Y) = , P(Y | X) = and P(X  Y) = . Which of the
SA

2 3 6
following is (are) correct ?     [IIT-JEE 2012, Paper-2, (4, 0), 66]  
2
  (A) P(X  Y) = (B) X and Y are independent
3
1
(C) X and Y are not independent     (D) P(XC  Y) =  
3
 
1 3 1 1
12. Four persons independently solve a certain problem correctly with probabilities , , , . Then the
2 4 4 8
probability that the problem is solved correctly by at least one of them is
[JEE (Advanced) 2013, Paper-1, (2, 0)/60]
235 21 3 253
(A) (B) (C) (D)
256 256 256 256

13. Of the three independent events E1, E2 and E3, the probability that only E1 occurs is ,only E2 occurs is
 and only E3 occurs is . Let the probability p that none of events E 1, E2 or E3 occurs satisfy the
equations ( – 2) p =  and ( – 3) p = 2 . All the given probabilities are assumed to lie in the
interval (0, 1).
Pr obability of occurrence of E1
Then = [JEE (Advanced) 2013, Paper-1, (4, – 1)/60]
Pr obability of occurrence of E3

RI
Comprehension (Q.14 & 15)

A box B1 contains 1 white ball, 3 red balls and 2 black balls. Another box B 2 contains 2 white balls, 3 red
balls and 4 black balls. A third box B3 contains 3 white balls, 4 red balls and 5 black balls.

A
14. If 1 ball is drawn from each of the boxes B 1, B2 and B3, the probability that all 3 drawn balls are of the
same colour is [JEE (Advanced) 2013, Paper-2, (3, –1)/60]

UH
82 90 558 566
(A) (B) (C) (D)
648 648 648 648

15. If 2 balls are drawn (without replacement) from a randomly selected box and one of the balls is white
and the other ball is red, the probability that these 2 balls are drawn from box B 2 is
                [JEE (Advanced) 2013, Paper-2, (3, –1)/60]
JA
116 126 65 55
(A) (B) (C) (D)
181 181 181 181

16. Three boys and two girls stand in a queue. The probability, that the number of boys ahead of every girl
is at least one more than the number of girls ahead of her, is [JEE (Advanced) 2014, Paper-2, (3, –1)/60]
1 1 2 3
(A) (B) (C) (D)
LP

2 3 3 4

Comprehension (Q.17 & 18) [JEE (Advanced) 2014, Paper-2, (3, –1)/60]

Box 1 contains three cards bearing numbers 1, 2, 3; box 2 contains five cards bearing numbers 1, 2, 3,
A

4, 5; and box 3 contains seven cards bearing numbers 1, 2, 3, 4, 5, 6, 7. A card is drawn from each of
the boxes. Let xi be the number on the card drawn from the ith box, i = 1, 2, 3.
17. The probability that x1 + x2 + x3 is odd, is
NK

29 53 57 1
(A) (B) (C) (D)
105 105 105 2

18. The probability that x1, x2, x3 are in an arithmetic progression, is


9 10 11 7
SA

(A) (B) (C) (D)


105 105 105 105

19. The minimum number of times a fair coin needs to be tossed, so that the probability of getting at least
two heads is at least 0.96, is     [JEE (Advanced) 2015, P-1 (4, 0) /88]  

Comprehension (Q.20 & 21)

Let n1 and n2 be the number of red and black balls, respectively, in box I. Let n3 and n4 be the number
of red and black balls, respectively, in box II. [JEE (Advanced) 2015, P-2 (4, –2)/ 80]
20*. One of the two boxes, box I and box II, was selected at random and a ball was drawn randomly out of
1
this box. The ball was found to be red. If the probability that this red ball was drawn from box II is ,
3
then the correct option(s) with the possible values of n 1, n2, n3 and n4 is(are)
(A) n1 = 3, n2 = 3, n3 = 5, n4 = 15 (B) n1 = 3, n2 = 6, n3 = 10, n4 = 50
(C) n1 = 8, n2 = 6, n3 = 5, n4 = 20 (D) n1 = 6, n2 = 12, n3 = 5, n4 = 20

21*. A ball is drawn at random from box I and transferred to box II. If the probability of drawing a red ball
1
from box I, after this transfer, is , then the correct option(s) with the possible values of n 1 and n2
3
is(are)

RI
(A) n1 = 4 and n2 = 6 (B) n1 = 2 and n2 = 3
(C) n1 = 10 and n2 = 20 (D) n1 = 3 and n2 = 6

22. A computer producing factory has only two plants T 1 and T2. Plant T1 produces 20% and plant T2

A
produces 80% of the total computers produced. 7% of computers produced in the factory turn out to be
defective. It is known that
P(computer turns out to be defective given that it is produced in plant T 1)

UH
= 10 P(computer turns out to be defective given that it is produced in Plant T 2),
where P(E) denotes the probability of an event E. A computer produced in the factory is randomly
selected and it does not turn out to be defective. Then the probability that it is produced in plant T 2 is
[JEE (Advanced) 2016, Paper-1, (3, –1)/62]
36 47 78 75
(A) (B)
JA (C) (D)
73 79 93 83
 

Comprehension (Q.23 & 24)


 

Football teams T1 and T2 have to play two games against each other. It is assumed that the outcomes
of the two games are independent. The probabilities of T 1 winning, drawing and losing a game against
1 1 1
LP

T2 are , and , respectively. Each team gets 3 points for a win, 1 point for a draw and 0 point for a
2 6 3
loss in a game. Let X and Y denote the total points scored by teams T 1 and T2, respectively, after two
games. [JEE (Advanced) 2016, Paper-2, (3, –1)/62]
23. P (X > Y) is
1 5 1 7
A

(A) (B) (C) (D)


4 12 2 12

24. P (X = Y) is
NK

11 1 13 1
(A) (B) (C) (D)
36 3 36 2
 
1 1 2
25*. Let X and Y be two events such that P(X) = , P(X|Y) = and P(Y|X) = . Then
3 2 5
SA

[JEE(Advanced) 2017, Paper-1,(4, –2)/61]


4 1 2 1
(A) P(Y) = (B) P(X|Y) = (C) P(X  Y) = (D) P(X  Y) =
15 2 5 5
 
26. Three randomly chosen nonnegative integers x, y and z are found to satisfy the equation x + y + z = 10.
Then the probability that z is even, is [JEE(Advanced) 2017, Paper-2,(3, –1)/61]  
1 36 6 5
(A) (B) (C) (D)
2 55 11 11

Comprehension (Q.27 & 28)


There are five students S1, S2, S3, S4 and S5 in a music class and for them there are five seats R 1, R2,
R3, R4 and R5 arranged in a row, where initially the seat R i is allotted to the student Si, i = 1, 2, 3, 4, 5.
But, on the examination day, the five students are randomly allotted the five seats.
(There are two questions based on PARAGRAPH “A”, the question given below is one of them)
[JEE(Advanced) 2018, Paper-1,(3, –1)/60]

27. The probability that, on the examination day, the student S 1 gets the previously allotted seat R 1, and
NONE of the remaining students gets the seat previously allotted to him/her, is
3 1 7 1
(A) (B) (C) (D)
40 8 40 5

28. For i = 1, 2, 3,4, let Ti denote the event that the students Si and Si+1 do NOT sit adjacent to each other

RI
on the day of the examination. Then, the probability of the event T 1  T2  T3  T4 is
1 1 7 1
(A) (B) (C) (D)
15 10 60 5

A
PART - II : JEE (MAIN) / AIEEE PROBLEMS (PREVIOUS YEARS)

UH
 1
1. In a binomial distribution B  n, p =  , if the probability of at least one success is greater than or equal
 4
9
to , then n is greater than : [AIEEE 2009 (4, –1), 144]
10
JA
1 9 4 1
(1) (2) (3) (4)
log10 4 + log10 3 log10 4 − log10 3 log10 4 − log10 3 log10 4 − log10 3

2. One ticket is selected at random from 50 tickets numbered 00, 01, 02, ....., 49. Then the probability that
the sum of the digits on the selected ticket is 8, given that the product of these digits is zero, equal :
[AIEEE 2009 (4, –1), 144]
LP

1 5 1 1
(1) (2) (3) (4)
7 14 50 14

3. Four numbers are chosen at random (without replacement) from the set {1,2,3,.....,20}.
Statement -1 : The probability that the chosen numbers when arranged in some order will form an AP
A

1
is . [AIEEE 2010 (8, –2), 144]
85
Statement -2 : If the four chosen numbers form an AP, then the set of all possible values of common
NK

difference is {±1, ±2, ±3, ±4, ±5}


(1) Statement- is true, Statement-2 is true ; Statement -2 is not a correct explanation for Statement -1.
(2) Statement-1 is true, Statement-2 is false.
(3) Statement -1 is false, Statement -2 is true.
(4) Statement -1 is true, Statement -2 is true; Statement-2 is a correct explanation for Statement-1.
 
SA

4. An urn contains nine balls of which three are red, four are blue and two are green. Three balls are
drawn at random without replacement from the urn. The probability that the three balls have different
colours is [AIEEE 2010 (4, –1), 144]
2 1 2 1
(1) (2) (3) (4)
7 21 23 3
 
5. Consider 5 independent Bernoulli’s trials each with probability of success p. If the probability of at least
31
one failure is greater than or equal to , then p lies in the interval : [AIEEE 2011, I, (4, –1), 120]
32
 1 3  3 11   1  11 
(1)  ,  (2)  ,  (3) 0,  (4)  , 1
 2 4  4 12   2  12 
6. If C and D are two events such that C  D and P(D)  0, then the correct statement among the following
is : [AIEEE 2011, I, (4, –1), 120]
P(D)
(1) P(C|D) = P(C) (2) P(C|D)  P(C) (3) P(C|D) < P(C) (4) P(C|D) =
P(C)

7. Let A, B, C be pairwise independent events with P(C) > 0 and P(A  B  C) = 0. Then P(Ac  Bc / C) .
[AIEEE 2011, II, (4, –1), 120]
(1) 1 – P(Bc) (2) P(Ac) + P(Bc) (3) P(Ac) – P(Bc) (4) P(Ac) – P(B)

8. Three numbers are chosen at random without replacement from {1, 2, 3, ..., 8}. The probability that their

RI
minimum is 3, given that their maximum is 6, is : [AIEEE-2012, (4, –1)/120]
3 1 1 2
(1) (2) (3) (4)
8 5 4 5

A
9. A multiple choice examination has 5 questions. Each question has three alternative answers of which
exactly one is correct. The probability that a student will get 4 or more correct answers just by guessing
is :

UH
[AIEEE - 2013, (4, – 1) 120]
17 13 11 10
(1) 5 (2) 5 (3) 5 (4) 5
3 3 3 3
1 1 1
10. (
Let A and B be two event such that P A  B = ) 6
, P(A  B) =
4
( )
and P A =
4
, where A stands for
JA
the complement of the event A. Then the events A and B are : [JEE(Main)2014,(4, – 1), 120]
(1) independent but not equally likely (2) independent and equally likely
(3) mutually exclusive and independent (4) equally likely but not independent
 
11. If 12 identical balls are to be placed in 3 identical boxes, then the probability that one of the boxes
contains exactly 3 balls is [JEE(Main)2015, (4, – 1), 20]
LP

11 10 12 11
55  2  2  1  1
(2) 55   (3) 220   (4) 22  
3  3 
(1)
 
3  
3 3
12. Let two fair six-faced dice A and B be thrown simultaneously. If E 1 is the event that die A shows up four,
E2 is the event that die B shows up two and E3 is the event that the sum of numbers on both dice is odd,
A

then whic h of the following statements is NOT True ? [JEE(Main)2016,(4, – 1), 120]
(1) E2 and E3 are independent (2) E1 and E3 are independent
(3) E1, E2 and E3 are independent (4) E1 and E2 are independent
NK

13. For three events A, B and C, P(Exactly one of A or B occurs) = P(Exactly one of B or C occurs)
1 1
= P(Exactly one of C or A occurs) = and P (All the three events occur simultaneously) = . Then
4 16
the probability that at least one of the events occurs, is : [JEE(Main)2017,(4, – 1), 120]
7 7 7 3
(1) (2) (3) (4)
SA

32 16 64 16

14. It two different numbers are taken from the set {0,1,2,3,....., 10}; then the probability that their sum as
well as absolute difference are both multiple of 4, is [JEE(Main)2017,(4, – 1), 120]
6 12 14 7
(1) (2) (3) (4)
55 55 45 55
15. A box contains 15 green and 10 yellow balls. If 10 balls are randomly drawn, one-by-one, with
replacement, then the variance of the number of green balls drawn is [JEE(Main) 2017, (4, – 1), 120]
12 6
(1) (2) 6 (3) 4 (4)
5 25
16. A bag contains 4 red and 6 balck balls. A ball is drawn at random from the bag, its colour is observed
and this ball along with two additional balls of the same colour are returned to the bag. If now a ball is
drawn at random from the bag, then the probability that this drawn ball is red, is
[JEE(Main)2018,(4, – 1), 120] 
1 3 3 2
(1) (2) (3) (4)
5 4 10 5
17. Two cards are drawn successively with replacement from a well-shuffled deck of 52 cards. Let X
denote the random variable of number of aces obtained in the two drawn cards. Then P(X = 1) + P(X =
2) equals : [JEE(Main) 2019, Online (09-01-19),P-1 (4, – 1), 120]
(1) 52/169 (2) 24/169 (3) 49/169 (4) 25/169

RI
18. Let S = {1,2,…..,20}. A subset B of S is said to be “nice”, if the sum of the elements of B is 203. Then
the probability that a randomly chosen subset of S is “ nice” is :
              [JEE(Main) 2019, Online (11-01-19),P-2 (4, – 1), 120]
4 5 7
(1) 20 (2) 20 (3) 20 (4)

A
2 2 2
6
2 20

UH
19. In a random experiment a fair die is rolled until two fours are obtained in succession the probability that
the experiment will end in the fifth throw of the die is equal
              [JEE(Main) 2019, Online (12-01-19),P-1 (4, – 1), 120]
200 175 150 225
(1) 5 (2) 5 (3) 5 (4) 5
6 6 6 6
JA
20. In a game, a man wins Rs. 100 if he gets 5 or 6 on a throw of a fair die and loses Rs. 50 for getting any
other number on the die. If he decides to throw the die either till he gets a five or a six or to a maximum
of three throws, then his expected gain/loss (in rupees) is –
[JEE(Main) 2019, Online (12-01-19),P-2 (4, – 1), 120] 
400 400 400
(1) 0 (2) loss (3) gain (4) loss
9 3 3
LP
A
NK
SA
EXERCISE - 1
PART - I
Section (A) :

A-1. (i) {HHH, HHT, HTH, THH, HTT, THT, TTH, TTT}
(ii) {B1 B2, B1 B3, B1 G1, B1 G2, B2 B3, B2 G1, B2 G2, B3 G1, B3 G2, G1 G2}

RI
4 5 1 4
A-2. 43 to 34 A-3. A-4. A-5. A-6. (i) (ii) 10
5 9 6 15
29 2 4 3
A-7. (i) No (ii) No A-8. A-9. B= ,C= A-10.

A
36 5 15 10

22 16 5 1 2197
13
C4  4 4
A-11. (i) (ii) A-12. A-13. A-14. (i) (ii) 52
425 5525 36 462 20825

UH
C4

Section (B) :

B-2 (i) 0.5 (ii) 0.8 B-3. (i) 0.72 (ii) 0.32 (iii) 0.28 (iv) 0.56 B-4. 5/21
JA
Section (C) :
2 1 3 2 4 52
C-1. (i) (ii) C-2. C-3. (i) (ii) C-4.
3 2 4 13 13 77
LP

16 12 9 17 27 8 1
C-5. , , C-6. 3/5 C-7. C-8. (i) (ii) C-9.
37 37 37 125 56 29 17

5 1
A

C-10. C-11.
9 4
NK

Section (D) :
7 (14)8
D-1. D-2. 10
C2 . D-3. X 0 1 2 3 D-4. 28 rupees 
213 1510 27 27 9 1
P( X)
64 64 64 64
SA

xi 0 1 2 3
D-5. 3
 19  19 2 19 216
pi   18  108  3
 25  25 3
25 25 3

D-6. mean = 2.5, variance = 1.25

PART - II
Section (A) :
A-1. (D) A-2. (C) A-3. (C) A-4. (D) A-5. (A) A-6. (D)

A-7. (A) A-8. (B) A-9. (C)

Section (B) :
B-1. (B) B-2. (i) (D) (ii) (B) (iii) (C) B-3 (B) B-4 (D)

Section (C) :

RI
C-1. (B) C-2. (C) C-3. (B) C-4. (B) C-5. (A) C-6. (A)

C-7. (D)

A
Section (D) :

D-1. (C) D-2. (A) D-3. (A) D-4. (D) D-5. (B) D-6. (B)

UH
PART - III

1. (A) → (q), (B) → (r), (C) → (s), (D) → (r)


2. (A) → (s), (B) → (r), (C) → (q), (D) → (p)
JA
EXERCISE - 2

PART - I
1. (A) 2. (A) 3. (C) 4. (A) 5. (B) 6. (B) 7. (A)
LP

8. (A) 9. (A) 10. (D) 11. (A)   12. (B) 13. (A) 14. (B)

15. (B)

PART - II
A

1. 10 2. 18 3. 1 4. 3 5. 11 6. 59 7. 6
NK

8. 20 9. 6 10. 1 11. 27 12. 1 13. 1 14. 32

15. 19 16. 7 17. 5 18. 5 19. 36 20. 11

PART - III
SA

1. (AC) 2. (ACD) 3. (CD) 4. (ABC) 5. (AD) 6. (ABC) 7. (BD)

8. (ABCD) 9. (AB) 10. (ABCD) 11. (AC) 12. (ABCD) 13. (ABCD)

14. (AC) 15. (AD) 16. (ABC) 17. (AC) 18. (ABCD)

PART - IV

1. (A) 2. (A) 3. (C) 4. (C) 5. (B) 6. (D) 7. (A)

8. (B) 9. (A) 10. (A)


EXERCISE - 3
PART - I

1. (A) 2. (B) 3. (D) 4. (C) 5. (C) 6. (B) 7. (D)

8.* (AD) 9*. (BD) 10. (A) 11*. (AB) 12. (A) 13. 6 14. (A)

15. (D) 16. (A) 17. (B) 18. (C) 19. 8 20. (AB) 21. (CD)

22. (C) 23. (B) 24. (C) 25. (AB) 26. (C) 27. (A) 28. (C)

RI
PART - II
1. (4) 2. (4) 3. (2) 4. (1) 5. (3) 6. (2) 7. (4)

8. (2) 9. (3) 10. (1) 11. (1) 12. (3) 13. (2) 14. (1)

A
15. (1) 16. (4) 17. (4) 18. (2) 19. (2) 20. (1)
 

UH
JA
LP
A
NK
SA
1. Urn A contains 6 red & 4 black balls and urn B contains 4 red & 6 black balls. One ball is drawn at
random from urn A & placed in urn B. Then one ball is drawn at random from urn B & placed in urn A. If
one ball is now drawn at random from urn A, then find the probability that it is red.

2. Let p be the probability that a man aged x years will die in a year time. Then find the probability that out
of 'n' men A1, A2, A3,......, An each aged 'x' years. A1 will die & will be the first to die.

3. A Sudoku matrix is defined as a 9 × 9 arrary with entries from {1, 2, 3 . . . . . 9} and with the constraint

RI
that each row, each column and each of the nine 3 × 3 boxes that tile the array contains each digit from
1 to 9 exactly once. A Sudoku matrix is chosen at random (so that every Sudoku matrix has equal
probability of being chosen). We know two of square in this matrix as shown. Then find the probability
that the square marked by ? contains the digit 3.

A
4. 5 girls and 10 boys sit at random in a row having 15 chairs numbered as 1 to 15, then find the
probability that end seats are occupied by the girls and between any two girls an odd number of boys
sit.

UH
5. Team A plays with 5 other teams exactly once. Assuming that for each match the probabilities of a win,
draw and loss are equal then find the probability that A wins and losses equal no. of matches.

6. Suppose that S be the set of all the ordered 4-tuples (x, y, z, w) of the +ve integers, which are the
solutions of x + y + z + w = 21. One such ordered tuple of solution is selected at random from S. Then
JA
find the probability that x > y.

7. In a betting game in an exhibition two dice P and Q are being used. Dice P has four red faces and two
white faces where as dice Q has two red and four white faces. A fair coin is tossed once. If it shows
head the game continues by throwing dice P. If it falls tail dice Q is thrown. If first n throws of the die all
turns up red then find the probability that P is being used.
LP

8. On a particular day, six persons pick six different books, one each from different counters at a public
library. At the closing time, they arbitrarily put their books to the vacant counters. Then find the
probability that exactly two books are at their previous places.

9. A dice has one 1, two 2’s and three 3’s on its faces. A player throws it till he gets three consecutive 1’s.
A

If pn is the probability that no 3 consecutive 1's appear in n throws, then prove that
215
(i) p = p = 1 and p =
1 2 3 216
NK

5
(ii) pn =
216
pn−3 + 6pn−2 + 36pn−1 , n  3
10. n students filled their forms for a competitive exam. Probability that exactly r students will not appear in
the exam is proportional to r. If probability that out of remaining n–r students exactly i students are
SA

selected is proportional to i. Prove that the probability of exactly two students finally getting selected is
8   1 1  1 1 1 
n  −  −  + + ...... +  
n(n + 1)   2 n   3 4 n 

11. In an organization number of women are µ times that of men. If n things are to be distributed among
 1  1 n+1 
them then the probability that the number of things received by men are odd is  −    Evaluate
2 2 
 
.
12. The color of a person’s eyes is determined by a single pair of genes. If they are both blue eyed genes,
then the person will have blue eyes ; if they are both brown -eyed genes, then the person will have
brown eyes; and if one of them is a blue-eyed gene and the other a brown-eyed gene, then the person
will have brown eyes. (Because of the latter fact we say that the brown-eyed gene is dominant over the
blue-eyed one.) A newborn child independently receives one eye gene from each of its parents and the
gene it receives from a parent is equally likely to be either of the two eye genes of that parent. Suppose
that Smith and both of his parents have brown eyes, but Smith’s sister has blue eyes. Suppose that
Smith’s wife has blue eyes. Find
(i) What is the probability that both of Smith’s parents has one blue-eyed gene and one brown
eyed gene?
(ii) What is the probability that Smith’s possesses a blue-eyed gene ?
(iii) What is the probability that Smith's first child will have blue eyes ?
(iv) If Smith's first child has brown eyes, what is the probability that both Smith’s genes are brown-
eyed genes?

RI
(v) If Smith's first child has brown-eyes, what is the probability that Smith's next child will also have
brown eyes ?

13. Each square of a 3 × 3 board is coloured either red or blue at random (each having probability 1/2).

A
Then find the probability that there is no 2 × 2 red square.

14. A fair coin is tossed (2m + 1) times, then find the probability of getting at least m consecutive heads.

UH
15. In a single throw of three dice find the probability of the event ‘a total of 8.

3 5
16. Suppose A & B shoot independently until each hits his target. They have probabilities , of hitting
5 7
the target at each shoot. Find the probability that B will require more shots than A.
JA
17. A quadratic equation is chosen by selecting two real numbers as its roots such that the quadratic
equation doesn't change by squaring the numbers. Now find probability that both roots are equal.

18. Five team of equal strength play against each other in a tournament and each match either ends in a
win or loss for a team. Find the probability that no team win all its games or loss all its game.

19. If A and B has (n + 1) and n fair coins respectively. Then find the probability that A gets more heads
LP

then B.

20. Let the probability pn that a family has exactly n children be pn, where n  1 and
p0 = 1 – p(1 + p + p2 + .....) (0 < , p < 1). Giving birth to a boy and girl is equally likely. If k  1, then
find the probability that the family has exactly k boys.
A

9 16
21. A bear hides itself either behind bush A with probability or behind bush B with probability .A
25 25
NK

hunter have 5 bullets each of which can be fired either at bush A or B. Hunter hits each target
independtly with an accuracy of 1/4. How many bullets can be fired at bush A to hit the bear with max.
probability.

22. Set A : {randomly choosen 100 years in which 76 are simple and 24 are leap years}
Set B : {randomly choosen 100 years in which 75 are simple and 25 are leap years}
SA

An year is chosen from either set A or set B and is found to have 53 sundays. Probability that the
chosen year was a leap year.

23. In ten trials of an experiment, if the probability of getting '4 successes’ is maximum, then find the range
of probability of success in each trial.

24. There are two lots of identical articles with different amounts of standard & defective articles. There are
N articles in the first lot, n of which are defective & M articles in the second lot, m of which are
defective. K articles are selected from the first lot & L articles from the second & a new lot results. Find
the probability that an article selected at random from the new lot is defective.
25. Find the chance of throwing 10 exactly in one throw with 3 dice.
26. Two players of equal skill, A and B, are playing a set of games; they leave off playing when A gets 3
points and B gets 2 (in each game winner get one point). If the stake is Rs.1600, what share ought
each to take?

27. A family has three children. Event ‘A’ is that family has at most one boy, Event ‘B’ is that family has at
least one boy and one girl, Event ‘C’ is that the family has at most one girl. Find whether events ‘A’ and
‘B’ are independent. Also find whether A, B, C are independent or not.
 
28. A line segment of length a is divided in two parts at random by taking a point on it, find the probability
that no part is greater than b, where 2b > a

29. Two ants are on the opposite corners of a grid of size 8 × 8 if they move then what is the probability

RI
that they will meet after each travelled eight steps (Assuming that they do not move in backward
direction)

33. Match : A box contains n coins. Let P(Ei) be the probability that exactly i out of n coins are biased.

A
If P(Ei) is directly proportional to i(i + 1) ; 1  i  n.
(3n + 1)
(A) Proportionality constant k is equal to (p)
4n

UH
3
(B) If P be the probability that a coin selected at (q)
n(n + 1)(n + 2)
random is biased then P is
3
(C) If a coin selected at random is found to be (r)
JA n+2
biased then the probability that it is the only
biased coin in the box is
24
(D) P(En) is equal to (s)
n(n + 1)(n + 2)(3n + 1)
LP

32 1 − (1 − p)n 2 20  10! 5! 17 35
1. 2. 3. 4. 5. 6.
A

55 n 21 15! 81 76

2n
NK

7. 8. 3/16 11. 3 12. (i) 1 (ii) 2/3 (iii) 1/3 (iv) 1/2 (v) 2/3
2n + 1
417 (m + 3)2m − 1 6 1 17
13. 14. 2m +1
15. 7/72 16. 17. 18.
512 2 31 2 32

2pk 98 4 5
p  , 
SA

19. ½ 20. 21. 1, 2 22. 23.


(2 − p) k +1
249  11 11

K nM + LmN
24. 25. 1/8 26. 500 Rs. & 1100 Rs.
MN (K + L)
16
2b − a C8
27. A and B are independent but A,B,C are not independent 28. 29. 16
a 2
1 1 9
30. 31. 32. 33. A−q ; B–p, C–s, d–r
256 256 64
RELATIONS, FUNCTIONS & ITF

JEE (ADVANCED) SYLLABUS


Functions : Real valued functions of a real variable, into, onto and one-to-one functions, sum, difference,
product and quotient of two functions, composite functions, absolute value, polynomial, rational, trigonometric,
exponential and logarithmic functions. Even and odd functions, inverse of a function.
Inverse Trigonometric Functions : (principal value only).

RI
Relation : Types of relations, equivalence relations.

JEE (MAIN) SYLLABUS

A
Functions : Real valued functions of a real variable, into, onto and one-to-one functions, sum, difference,
product and quotient of two functions, composite functions, absolute value, polynomial, rational, trigonometric,
exponential and logarithmic functions. Even and odd functions, inverse of a function.

UH
Inverse Trigonometric Functions : Inverse trigonometrical functions and their properties
Relation : Types of relations, equivalence relations.

RELATIONS
JA
ORDERED PAIR :
A pair of objects listed in a specific order is called an ordered pair. It is written by listing the two objects
in specific order separating them by a comma and then enclosing the pair in parentheses.
In the ordered pair (a, b), a is called the first element and b is called the second element.
Two ordered pairs are set to be equal if their corresponding elements are equal.
i.e. (a, b) = (c, d) if a = c and b = d.
LP

CARTESIAN PRODUCT :
The set of all possible ordered pairs (a, b), where a  A and b  B i.e. {(a, b) ; a  A and b  B} is
called the Cartesian product of A to B and is denoted by A × B. Usually A × B  B × A.
Similarly A × B × C = {(a, b, c) : a  A, b  B, c  C} is called ordered triplet.
A

RELATION :
Let A and B be two sets. Then a relation R from A to B is a subset of A × B. Thus, R is a relation from A
NK

to B  R  A × B. The subsets is derived by describing a relationship between the first element and
the second element of ordered pairs in A × B e.g. if A = {1, 2, 3, 4, 5, 6, 7, 8} and B = {1, 2, 3, 4, 5} and
R = {(a, b) : a = b2, a  A, b  B} then R = {(1, 1), (4, 2), (9, 3)}. Here a R b  1 R 1, 4 R 2, 9 R 3.

NOTE :
(i) Let A and B be two non-empty finite sets consisting of m and n elements respectively. Then
SA

A × B consists of mn ordered pairs. So total number of subsets of A × B i.e. number of possible


relations from A to B is 2mn.
(ii) A relation R from A to A is called a relation on A.

DOMAIN AND RANGE OF A RELATION :


Let R be a relation from a set A to a set B. Then the set of all first components of coordinates of the
ordered pairs belonging to R is called to domain of R, while the set of all second components of
coordinates of the ordered pairs in R is called the range of R.
Thus, Dom (R) = {a : (a, b)  R} and Range (R) = {b : (a, b)  R}
It is evident from the definition that the domain of a relation from A to B is a subset of A and its range is
a subset of B.
Example # 1 :If A = {1, 2} and B = {3, 4}, then find A × B.
Solution : A × B = {(1, 3), (1, 4), (2, 3), (2, 4)}

Example # 2 : Let A = {1, 3, 5, 7} and B = {2, 4, 6, 8} be two sets and let R be a relation from A to B defined by
the phrase "(x, y)  R  x > y". Find relation R and its domain and range.
Solution : Under relation R, we have 3R2, 5R2, 5R4, 7R2, 7R4 and 7R6
i.e. R = {(3, 2), (5, 2), (5, 4), (7, 2), (7, 4), (7, 6)}
 Dom (R) = {3, 5, 7} and range (R) = {2, 4, 6}

Example # 3 : Let A = {2, 3, 4, 5, 6, 7, 8, 9}. Let R be the relation on A defined by

{(x, y) : x  A, y  A & x2 = y or x = y2}. Find domain and range of R.

RI
Solution : The relation R is
R = {(2, 4), (3, 9), (4, 2), (9, 3)}

A
Domain of R = {2, 3, 4, 9}
Range of R = {2, 3, 4, 9}

UH
Self Practice Problem :

(1) If (2x + y, 7) = (5, y – 3) then find x and y.

(2) If A × B = {(1, 2), (1, 3), (1, 6), (7, 2), (7, 3), (7, 6)} then find sets A and B.

(3)
JA
If A = {x, y, z} and B = {1, 2} then find number of relations from A to B.

(4) Write R = {(4x + 3, 1 – x) : x  2, x  N}


5
Answers (1) x = – , y = 10 (2) A = {1, 7}, B = {2, 3, 6}
2
(3) 64 (4) {(7, 0), (11, –1)}
LP

TYPES OF RELATIONS :

In this section we intend to define various types of relations on a given set A.


A

(i) Void relation : Let A be a set. Then   A × A and so it is a relation on A. This relation is
called the void or empty relation on A.
NK

(ii) Universal relation : Let A be a set. Then A × A  A × A and so it is a relation on A. This


relation is called the universal relation on A.

(iii) Identity relation : Let A be a set. Then the relation I A = {(a, a) : a  A} on A is called the
SA

identity relation on A. In other words, a relation I A on A is called the identity relation if every
element of A is related to itself only.

(iv) Reflexive relation : A relation R on a set A is said to be reflexive if every element of A is


related to itself. Thus, R on a set A is not reflexive if there exists an element a  A such that
(a, a)  R.

Note : Every identity relation is reflexive but every reflexive relation in not identity.
(v) Symmetric relation : A relation R on a set A is said to be a symmetric relation

iff (a, b)  R  (b ,a)  R for all a, b  A. i.e. a R b  b R a for all a, b  A.


(vi) Transitive relation : Let A be any set. A relation R on A is said to be a transitive relation

iff (a, b)  R and (b, c)  R  (a, c)  R for all a, b, c  A

i.e. a R b and b R c  a R c for all a, b, c  A

(vii) Equivalence relation : A relation R on a set A is said to be an equivalence relation on A iff

(i) it is reflexive i.e. (a, a)  R for all a  A

(ii) it is symmetric i.e. (a, b)  R  (b, a)  R for all a, b  A

RI
(iii) it is transitive i.e. (a, b)  R and (b, c)  R  (a, c)  R for all a, b  A
Example # 4 : Which of the following are identity relations on set A = {1, 2, 3}.
R1 = {(1, 1), (2, 2)}, R2 = {(1, 1), (2, 2), (3, 3), (1, 3)}, R 3 = {(1, 1), (2, 2), (3, 3)}.

A
Solution: The relation R 3 is identity relation on set A.
R1 is not identity relation on set A as (3, 3)  R1.

UH
R2 is not identity relation on set A as (1, 3)  R2

Example # 5 : Which of the following are reflexive relations on set A = {1, 2, 3}.
R1 = {(1, 1), (2, 2), (3, 3), (1, 3), (2, 1)}, R 2 = {(1, 1), (3, 3), (2, 1), (3, 2)}..
JA
Solution : R1 is a reflexive relation on set A.
R2 is not a reflexive relation on A because 2  A but (2, 2)  R2.

Example # 6 : Prove that on the set N of natural numbers, the relation R defined by x R y  x is less than y is
transitive.
LP

Solution : Because for any x, y, z  N x < y and y < z  x < z  x R y and y R z  x R z. so R is


transitive.

Example # 7 : Let T be the set of all triangles in a plane with R a relation in T given by R = {(T 1 , T2) : T1 is
A

congruent to T2}. Show that R is an equivalence relation.


Solution : Since a relation R in T is said to be an equivalence relation if R is reflexive, symmetric and
transitive.
NK

(i) Since every triangle is congruent to itself


 R is reflexive
(ii) (T1 , T2)  R  T1 is congruent to T2  T2 is congruent to T1  (T2, T1)  R
Hence R is symmetric
(iii) Let (T1, T2)  R and (T2, T3)  R  T1 is congruent to T2 and T2 is congruent to T3
SA

 T1 is congruent to T3  (T1, T3)  R


 R is transitive
Hence R is an equivalence relation.
 
Example # 8 : Show that the relation R in R defined as R = {(a, b) : a  b} is transitive.
Solution : Let (a, b)  R and (b, c)  R
 (a  b) and b  c  ac  (a, c)  R Hence R is transitive.

Example # 9 : Show that the relation R in the set {1, 2, 3} given by R = {(1, 2), (2, 1)} is symmetric.
Solution : Let (a, b)  R [ (1, 2)  R]
 (b, a)  R [ (2, 1)  R]
Hence R is symmetric.

Self Practice Problem :


(5) Let L be the set of all lines in a plane and let R be a relation defined on L by the rule (x ,y)  R
 x is perpendicular to y. Then prove that R is a symmetric relation on L.

(6) Let R be a relation on the set of all lines in a plane defined by (1, 2)  R  line 1 is parallel to
line 2. Prove that R is an equivalence relation.

RI
FUNCTION
Definition :
Function is a rule (or correspondence), from a non empty set A to a non empty set B, that associates
each member of A to a unique member of B. Symbolically, we write f: A → B. We read it as "f is a

A
function from A to B".
For example, let A  {–1, 0, 1} and B  {0, 1, 2}.
Then A × B  {(–1, 0), (–1, 1), (–1, 2), (0, 0), (0, 1), (0, 2), (1, 0), (1, 1), (1, 2)}

UH
Now, " f : A → B defined by f(x) = x 2 " is the function such that
f  {(–1, 1), (0, 0), (1, 1)}
f can also be shown diagramatically by following mapping.
A B
JA
Note : Every function say y = f(x) : A → B. Here x is independent variable which takes its values from A while
'y' takes its value from B. A relation will be a function if and only if
(i) x must be able to take each and every value of A and
(ii) one value of x must be related to one and only one value of y in set B.
LP
A

Graphically : If any vertical line cuts the graph at more than one point, then the graph does not represent a
NK

function.
Example # 10 : (i) Which of the following correspondences can be called a function ?
(A) f(x) = x3 ; {–1, 0, 1} → {0, 1, 2, 3}

(B) f(x) = ± x ; {0, 1, 4} → {–2, –1, 0, 1, 2}


SA

(C) f(x) = x ; {0, 1, 4} → {–2, –1, 0, 1, 2}

(D) f(x) = – x ; {0, 1, 4} → {–2, –1, 0, 1, 2}

(ii) Which of the following pictorial diagrams represent the function

(A) (B)

(C) (D)
Solution :
(i) f(x) in (C) and (D) are functions as definition of function is satisfied. while in case of (A) the
given relation is not a function, as f(–1)  2nd set. Hence definition of function is not satisfied.
While in case of (B), the given relation is not a function, as f(1) = ± 1 and f(4) = ± 2 i.e. element
1 as well as 4 in 1st set is related with two elements of 2 nd set.Hence definition of function is not
satisfied.
(ii) B and D. In (A) one element of domain has no image, while in (C) one element of 1 st set has
two images in 2nd set

Self practice problem :


(7) Let g(x) be a function defined on [−1, 1]. If the area of the equilateral triangle with two of its

RI
vertices at (0,0) and (x,g(x)) is 3 / 4 sq. unit, then the function g(x) may be.

(A) g(x) =  (1 − x 2 ) (B) g(x) = (1 − x 2 ) (C) g(x) = − (1 − x 2 ) (D) g(x) = (1 + x 2 )

A
(8) Represent all possible functions defined from { } to {1, 2}.
Answers : (7) B, C

UH
(8) (i) (ii) (iii) (iv)

Domain, Co-domain and Range of a Function :


JA
Let y = f(x) : A → B, then the set A is known as the domain of f and the set B is known as co-domain of
f.
LP

If x1 is mapped to y1, then y1 is called as image of x1 under f. Further x1 is a pre-image of y1 under f.


If only expression of f (x) is given (domain and co-domain are not mentioned), then domain is complete
set of those values of x for which f (x) is real, while codomain is considered to be (– , ) (except in
A

inverse trigonometric functions).


Range is the complete set of values that y takes. Clearly range is a subset of Co-domain.
A function whose domain and range are both subsets of real numbers is called a real function.
NK

Example # 11 : Find the domain of following functions :

(i) f(x) = x2 − 5 (ii) sin (x3 – x)

Solution : (i) f(x) = x 2 − 5 is real iff x2 – 5  0


SA

 |x|  5  x– 5 or x  5

 the domain of f is (–, – 5 ]  [ 5 , )

(ii) x3 – x  R  domain is x  R

Algebraic Operations on Functions :


If f and g are real valued functions of x with domain set A and B respectively, then both f and g are
defined in A  B. Now we define f + g, f − g, (f . g) and (f /g) as follows:
f f (x)
(iii)   (x) = domain is {x  x  A  B such that g(x)  0}.
g g(x)

Note :  For domain of (x) = {f(x)}g(x) , conventionally, the conditions are f(x) > 0 and g(x) must be real.
 For domain of (x) = f(x)Cg(x) or (x) = f(x)Pg(x) conventional conditions of domain are f(x)  g(x)
and f(x)  N and g(x)  W.

3
Example # 12 : Find the domain of function f(x) = log(x3 − x)

RI
4 − x2

Solution : Domain of 4 − x 2 is [−2, 2] but 4 − x 2 = 0 for x = ± 2  x  (–2, 2)


log(x3 − x) is defined for x3 − x > 0 i.e. x(x − 1)(x + 1) > 0.

A
 domain of log(x3 − x) is (−1, 0 )  (1, ).
Hence the domain of the given function is {(−1, 0 )  (1, )} (−2, 2)  (−1, 0 )  (1, 2).

UH
Self practice problems :
(9) Find the domain of following functions.
1 2x − 1
(i) f(x) = + x +1 (ii) f(x) = 1 − x – sin
log(2 − x) 3

Answers : (i) [–1, 1)  (1, 2) (ii) [–1, 1]


JA
Methods of determining range :
(i) Representing x in terms of y
If y = f(x), try to express as x = g(y), then domain of g(y) represents possible values of y, which
LP

is range of f(x).

(ii) Graphical Method :


The set of y– coordinates of the graph of a function is the range.
A

x2 + x + 1
Example # 13 : Find the range of f(x) =
x2 + x − 1

x2 + x + 1
NK

Solution : f(x) = {x2 + x + 1 and x2 + x – 1 have no common factor}


x2 + x − 1

x2 + x + 1
y=
x2 + x − 1
SA

 yx2 + yx – y = x2 + x + 1
 (y – 1) x2 + (y – 1) x – y – 1 = 0
If y = 1, then the above equation reduces to –2 = 0. Which is not true.
Further if y  1, then (y – 1) x2 + (y – 1) x – y – 1 = 0 is a quadratic and has real roots
if
(y – 1)2 – 4 (y – 1) (–y – 1)  0
i.e. if y  –3/5 or y  1 but y  1
Thus the range is (–, –3/5]  (1, )

x2 − 4
Example # 14 : Find the range of f(x) =
x−2
Solution :

x2 − 4
f(x) = = x + 2; x  2
x−2

 graph of f(x) would be


Thus the range of f(x) is R – {4}
Further if f(x) happens to be continuous in its domain then range of f(x) is [min f(x), max. f(x)]. However

RI
for sectionally continuous functions, range will be union of [min f(x), max. f(x)] over all those intervals
where f(x) is continuous, as shown by following example.

A
UH
Example # 15 : Let graph of function y = f(x) is
JA
LP

Then range of above sectionally continuous function is [y 2, y3]  [y7, y6)  (y4, y5]
A

(iii) Using monotonocity : Many of the functions are monotonic increasing or monotonic decreasing. In case
of monotonic continuous functions the minimum and maximum values lie at end points of domain.
Some of the common function which are increasing or decreasing in the interval where they are
continuous is as under.
NK
SA

For monotonic increasing functions in [a, b]


(i) f(x)  0 (ii) range is [f(a), f(b)]
for monotonic decreasing functions in [a, b]
(i) f(x)  0 (ii) range is [f(b), f(a)]

Example # 16 : Find the range of function y = n (2x – x2)


Solution : Step – 1
We have 2x – x2  (–, 1]
Step – 2 Let t = 2x – x2
For nt to be defined accepted values are (0, 1]
Now, using monotonocity of n t,
n (2x – x2)  (–, 0]
 range is (– , 0] Ans.

Self practice problems :


(10) Find domain and range of following functions.

RI
x 2 − 2x + 5 1
(i) y = x3 (ii) y= 2 (iii) y=
x + 2x + 5 x2 − x
3 − 5 3 + 5 

A
Answers : (i) domain R; range R (ii) domain R ; range  , 
 2 2 
(iii) domain R – [0, 1] ; range (0, )

UH
Classification of Functions :
Functions can be classified as "One − One Function (Injective Mapping)" and "Many − One Function" :

One - One Function :


JA
A function f : A → B is said to be a one-one function or injective mapping if different elements of A have
different f images in B.
Thus for x1, x2  A and f(x1), f(x2)  B, f(x1) = f(x2)  x1 = x2 or x1  x2  f(x1)  f(x2).
Diagrammatically an injective mapping can be shown as
LP

OR

Many - One function :


A function f : A → B is said to be a many one function if there exist at least two or more elements of A
A

having the same f image in B.


Thus f : A → B is many one iff there exist atleast two elements x 1, x2  A, such that f(x 1) = f(x2) but x1 
x2.
NK

Diagrammatically a many one mapping can be shown as

OR
Note : 
SA

If a function is one−one, it cannot be many−one and vice versa.

Methods of determining whether a given function is ONE-ONE or MANY-ONE :


(a) If x1, x2  A and f(x1), f(x2)  B, equate f(x1) and f(x2) and if it implies that x 1 = x2, then and only
then function is ONE-ONE otherwise MANY-ONE.
(b) If there exists a straight line parallel to x-axis, which cuts the graph of the function atleast at two
points, then the function is MANY-ONE, otherwise ONE- ONE.
(c) If either f(x)  0,  x  domain or f(x)  0  x  domain, where equality can hold at discrete
point(s) only i.e. strictly monotonic, then function is ONE-ONE, otherwise MANY-ONE.
Note : If f and g both are one-one, then gof and fog would also be one-one (if they exist). Functions can also
be classified as "Onto function (Surjective mapping)" and "Into function":
Onto function :
If the function f : A → B is such that each element in B (co−domain) must have atleast one
pre−image in A, then we say that f is a function of A 'onto' B. Thus f : A → B is surjective iff  b  B,
there exists some a  A such that f (a) = b.
Diagrammatically surjective mapping can be shown as

OR
Into function :
If f : A → B is such that there exists atleast one element in co−domain which is not the image of any
element in domain, then f(x) is into.

RI
Diagrammatically into function can be shown as

A
OR
Note : (i) If range  co−domain, then f(x) is onto, otherwise into

UH
(ii) If a function is onto, it cannot be into and vice versa.
A function can be one of these four types:

(a) one−one onto (injective and surjective)


JA
(b) one−one into (injective but not surjective)

(c) many−one onto (surjective but not injective)

(d) many−one into (neither surjective nor injective)


LP

Note : (i) If f is both injective and surjective, then it is called a bijective mapping. The bijective
functions are also named as invertible, non singular or biuniform functions.
(ii) If a set A contains 'n' distinct elements, then the number of different functions defined
A

from
A → A is nn and out of which n! are one one.
(iii) If f and g both are onto, then gof or fog may or may not be onto.
NK

(iv) The composite of two bijections is a bijection iff f and g are two bijections such that gof
is defined, then gof is also a bijection only when co-domain of f is equal to the domain of g.

Example # 17 : (i) Find whether f(x) = x + cos x is one-one.


3 2
(ii) Identify whether the function f(x) = –x + 3x – 2x + 4 for f : R → R is ONTO or INTO
(iii) f(x) = x2 – 2x; [0, 3] → A. Find whether f(x) is injective or not. Also find the
SA

set A, if f(x) is surjective.


Solution : (i) The domain of f(x) is R. f (x) = 1 − sin x.
 f (x)  0  x  complete domain and equality holds at discrete points only
 f(x) is strictly increasing on R. Hence f(x) is one-one.
(ii) As range  codomain, therefore given function is ONTO
(iii) f(x) = 2(x – 1); 0  x  3
−ve ; 0  x  1
 f(x) = 
+ ve ; 1  x  3
 f(x) is non monotonic. Hence it is not injective.
For f(x) to be surjective, A should be equal to its range. By graph range is [–1, 3]
 A  [–1, 3]

RI
Self practice problems :

(11) For each of the following functions find whether it is one-one or many-one and also into or onto

A
1
(i) f(x) = 2 tan x; (/2, 3/2) → R (ii) f(x) = ; (–, 0) → R
1 + x2
(iii) f(x) = x2 + n x

UH
Answers : (i) one-one onto (ii) one-one into (iii) one-one onto

Equal or dentical Functions :


Two functions f and g are said to be identical (or equal) iff :
(i) The domain of f  the domain of g.
JA
(ii) f(x) = g(x), for every x belonging to their common domain.
1 x
e.g. f(x) = and g(x) = 2 are identical functions. Clearly the graphs of f(x) and g(x) are
x x
exactly same
LP
A

x2
But f(x) = x and g(x) = are not identical functions.
x
Clearly the graphs of f(x) and g(x) are different at x = 0.
NK

Example # 18 : Examine whether following pair of functions are identical or not ?


SA

x2 − 1
(i) f(x) = and g(x) = x + 1
x −1
(ii) f(x) = sin2x + cos2x and g(x) = sec2x – tan2x
Solution : (i) No, as domain of f(x) is R – {1}
while domain of g(x) is R
(ii) No, as domain are not same. Domain of f(x) is R
  
while that of g(x) is R – ( 2n + 1) ; n  I
 2 
Self practice problems
(12) Examine whether the following pair of functions are identical or not :
 x
 x0
(i) f(x) = sgn (x) and g(x) =  | x |
 0 x=0

(ii) f(x) = cosec2x – cot2x and g(x) = 1
Answers : (i) Yes (ii) No

Composite Function :
Let f: X→Y1 and g: Y2→ Z be two functions and D is the set of values of x such that if x  X, then
f(x)  Y2. If D  , then the function h defined on D by h(x) = g{f(x)} is called composite function of g and
f and is denoted by gof. It is also called function of a function.
Note :  Domain of gof is D which is a subset of X (the domain of f ). Range of gof is a subset

RI
of the range of g. If D = X, then f(X)  Y2.

Pictorially gof(x) can be viewed as under

A
Note that gof(x) exists only for those x when range of f(x) is a subset of domain of g(x).

Properties of Composite Functions :


(a) In general gof  fog (i.e. not commutative)

UH
(b) The composition of functions are associative i.e. if three functions f, g, h are such that
fo (goh) and (fog) oh are defined, then fo (goh) = (fog) oh.

Example # 19 : Describe fog and gof wherever is possible for the following functions
(i) f(x) = x + 3 , g(x) = 1 + x2 (ii) f(x) = x , g(x) = x2 − 1.
JA
Solution : (i) Domain of f is [−3, ), range of f is [0, ).
Domain of g is R, range of g is [1, ).
For gof(x)
Since range of f is a subset of domain of g,
 domain of gof is [−3, ) {equal to the domain of f }
gof (x) = g{f(x)} = g ( x + 3 ) = 1 + (x+3) = x + 4. Range of gof is [1, ).
LP

For fog(x)
since range of g is a subset of domain of f,
 domain of fog is R {equal to the domain of g}
x 2 + 4 Range of fog is [2, ).
A

fog (x) = f{g(x)}= f(1+ x 2 ) =


(ii) f(x) = x , g(x) = x2 − 1.
Domain of f is [0, ), range of f is [0, ).
Domain of g is R, range of g is [−1, ).
NK

For gof(x)
Since range of f is a subset of the domain of g,
 domain of gof is [0, ) and g{f(x)}= g(x) = x − 1. Range of gof is [−1, )
For fog(x)
SA

Since range of g is not a subset of the domain of f


i.e. [−1, )  [0, )
 fog is not defined on whole of the domain of g.
Domain of fog is {xR, the domain of g : g(x) [0, ), the domain of f}.
Thus the domain of fog is D = {xR: 0  g(x) < }
i.e. D = { xR: 0  x2 − 1}= { xR: x  −1 or x  1 }= (−, −1]  [1, )
fog (x) = f{g(x)} = f(x 2−1) = x 2 − 1 Its range is [0, ).

  
Example # 20 : Let f(x) = ex ; R+ → R and g(x) = sinx ;  − ,  → [–1, 1]. Find domain and range of fog(x)
 2 2
Solution : Domain of f(x) : (0, ) Range of g(x) : [–1, 1]
 
values in range of g(x) which are accepted by f(x) are  0, 
 2

 0 < g(x)  1  0 < sinx  1  0<x
2

Hence domain of fog(x) is x  (0, ]
2

Therefore Domain : (0, ]
2
Range : (1, e]

Example # 21 : If f (x) = −1 + x − 2, 0  x  4

RI
g (x) = 2 − x, − 1  x  3
Then find fog (x) and gof (x). Also draw their rough sketch.
Solution : fog (x) = { −1 + g(x) − 2, 0  g(x)  4, − 1  x  3
= { −1 + 2 − x − 2, 0  2 − x 4, − 1  x  3

A
= {–1 + x , − 2  x  2, − 1  x  3

UH
−(1 + x) , −1  x  0
=  ;
 x −1 , 0  x  2
JA
gof(x) = {2 − f(x), − 1  f(x)  3, 0  x  4
= {2 − −1 + x − 2, − 1  −1 + x − 2 3, 0  x  4
= {2 − −1 + x − 2, –2  x  6, 0  x  4
LP

x +1 , 0  x 1

A

3 − x , 1 x  2
=  ;
x −1 , 2x3
5 − x , 3x4
NK

Self practice problems


(13) Define fog(x) and gof(x). Also find their domain and range.
(i) f(x) = [x], g(x) = sin x
f(x) = tan x, x  (–/2, /2); g(x) = 1 − x2
SA

(ii)

(14) Let f(x) = ex : R+ → R and g(x) = x2 – x : R → R. Find domain and range of fog (x) and gof (x)
Answers :
(13) (i) gof = sin [x] domain : R range { sin a : a  }
fog = [ sin x] domain : R range : {–1, 0, 1}
  
(ii) gof  1 − tan2 x , domain :  − ,  range : [0, 1]
 4 4
fog  tan 1 − x2 domain : [–1, 1] range [0, tan 1]
(14) fog (x) gof (x)
Domain : (–, 0)  (1, ) Domain : (0, )
Range : (1, ) Range : (0, )

Odd and Even Functions :


(i) If f (−x) = f (x) for all x in the domain of ‘f’, then f is said to be an even function.
e.g. f (x) = cos x; g (x) = x² + 3.
(ii) If f (−x) = −f (x) for all x in the domain of ‘f’, then f is said to be an odd function.
e.g. f (x) = sin x; g (x) = x 3 + x.
Note : (i) A function may neither be odd nor even. (e.g. f(x) = e x , cos–1x)
(ii) If an odd function is defined at x = 0, then f(0) = 0
Properties of Even/Odd Function
(a) The graph of every even function is symmetric about the y−axis and that of every odd

RI
function is symmetric about the origin.
For example graph of y = x 2 is symmetric about y-axis, while graph of y = x 3 is
symmetric about origin

A
UH
(b) All functions (whose domain is symmetrical about origin) can be expressed as the sum of an
even and an odd function, as follows
JA
f(x) =
(c) The only function which is defined on the entire number line and is even and odd at the same
time is f(x) = 0.
(d) If f and g both are even or both are odd, then the function f.g will be even but if any one of
LP

them is odd and the other even then f.g will be odd.
(e) If f(x) is even then f(x) is odd while derivative of odd function is even. Note that same cannot
be said for integral of functions.
A

Example # 22 : Show that ax +a–x is an even function.


Solution : Let f(x) = ax + a–x
Then f(–x) = a–x + a–(–x) = a–x +ax = f(x). Hence f(x) is an even function
NK

 x x
Example # 23 : Prove that f(x) = x  x +  is odd function
 e −1 2 
 x x  −x −x   x x
Solution : Let g(x) =  x +  then g(–x) =  − x +  =  x + 
 e − 1 2   e − 1 2   e − 1 2 
SA

 g(x) is even
 x x
hence f(x) = x.g(x) = x  x +  is odd function.
 e −1 2 

Self practice problems


(15) Determine whether the following functions are even / odd / neither even nor odd?
ex + e− x
(i) f(x) = x
e − e− x
(ii) f : [–2, 3] → 0, 9] , f(x) = x 2
(iii) (
f(x) = x log x + x 2 + 1 )
Answers (i) Odd (ii) neither even nor odd (iii) Even
Periodic Functions :
A function f(x) is called periodic with a period T if there exists a real number T > 0 such that for each x
in the domain of f the numbers x – T and x + T are also in the domain of f and f(x) = f(x + T) for all x in
the domain of f(x). Graph of a periodic function with period T is repeated after every interval of 'T'.
e.g. The function sin x and cos x both are periodic over 2 and tan x is periodic over 
The least positive period is called the principal or fundamental period of f(x) or simply the
period of the function.
Note : Inverse of a periodic function does not exist.
Properties of Periodic Functions :
1
(a) If f(x) has a period T, then and f (x) also have a period T.
f (x)

RI
T
(b) If f(x) has a period T, then f (ax + b) has a period .
|a|
(c) Every constant function defined for all real x, is always periodic, with no fundamental period.
(d) If f (x) has a period T1 and g (x) also has a period T2 then period of f(x) ± g(x) or f(x) . g(x) or

A
f(x)
is L.C.M. of T1 and T2 provided their L.C.M. exists. However that L.C.M. (if exists) need
g(x)

UH
f(x)
not to be fundamental period. If L.C.M. does not exists then f(x) ± g(x) or f(x) . g(x) or is
g(x)
nonperiodic.
a p  L.C.M.( a, p, )
L.C.M. of  , , =
 b q m  H.C.F . ( b, q, m)
JA
e.g. |sinx| has the period , | cosx | also has the period 

 |sinx| + |cosx| also has a period . But the fundamental period of |sinx| + |cosx| is .
2

(e) If g is a function such that gof is defined on the domain of f and f is periodic with T,
then gof is also periodic with T as one of its periods.
LP

Example # 24 : Find period of the following functions


x x
(i) f(x) = sin + cos
2 3
(ii) f(x) = {x} + sin x, where {.}denotes fractional part function
A

3x x 2x
(iii) f(x) = 4 cos x . cos 3x + 2 (iv) f(x) = sin – cos – tan
2 3 3
x x x x
Solution : (i) Period of sin is 4 while period of cos is 6 . Hence period of sin + cos is 12 
NK

2 3 2 3
{L.C.M. of 4 and 6 is 12}
(ii) Period of sin x = 2
Period of {x} = 1
but L.C.M. of 2 and 1 is not possible as their ratio is irrational number

SA

it is aperiodic
(iii) f(x) = 4 cos x . cos 3x + 2
 2 
period of f(x) is L.C.M. of  2,
3 
= 2

2
but 2 may or may not be fundamental periodic, but fundamental period = , where
n
n  N. Hence cross-checking for n = 1, 2, 3, ....we find  to be fundamental period
f( + x) = 4(– cos x) (– cos 3x) + 2 = f(x)
2 2  4 3
(iv) Period of f(x) is L.C.M. of , , = L.C.M. of , 6 , = 12
3 / 2 1/ 3 2 / 3 3 2
Inverse of a Function :
Let y = f(x) : A → B be a one-one and onto function. i.e. bijection, then there will always exist bijective
function x = g(y) : B → A such that if (p, q) is an element of f, (q, p) will be an element of g and the
functions f(x) and g(x) are said to be inverse of each other. g(x) is also denoted by f −1(x) and f(x) is
–1
denoted by g (x)
Note : (i) The inverse of a bijection is unique.
(ii) Inverse of an even function is not defined.

Properties of Inverse Function :


(a) The graphs of f and g are the mirror images of each other in the line y = x. For example
f(x) = ax and g(x) = loga x are inverse of each other, and their graphs are mirror images of each

RI
other on the line y = x as shown below.

A
(b)

UH
Normally points of intersection of f and f –1 lie on the straight line y = x. However it must be noted
JA
–1
that f(x) and f (x) may intersect otherwise also. e.g f(x) = 1/x
(c) In general fog(x) and gof(x) are not equal. But if f and g are inverse of each other, then
gof = fog. fog(x) and gof(x) can be equal even if f and g are not inverse of each other.
–1
e.g. f(x) = x + 1, g(x) = x + 2. However if fog(x) = gof(x) = x, then g(x) = f (x)
(d) If f and g are two bijections f : A → B, g : B → C, then the inverse of gof exists and
LP

(gof)−1 = f−1 o g−1.


1
(e) If f(x) and g(x) are inverse function of each other, then f(g(x)) =
g(x)
2x + 3
Example # 25 : (i) Determine whether f(x) = for f : R → R, is invertible or not? If so find it.
A

4
(ii) Let f(x) = x2 + 2x; x  –1. Draw graph of f–1(x) also find the number of solutions of the
equation, f(x) = f–1(x)
(iii) If y = f(x) = x2 – 3x + 2, x  1. Find the value of g(2)where g is inverse of f
NK

Solution : (i) Given function is one-one and onto, therefore it is invertible.


2x + 3 4y − 3 –1 4x − 3
y=  x=  f (x) =
4 2 2
(ii) f(x) = f–1(x) is equivalent to f(x) = x  x2 + 2x = x  x(x + 1) = 0  x = 0, –1
Hence two solution for f(x) = f–1(x)
SA
(iii) f(x) = x2 – 3x + 2, x  1
f(g(x) = g(x)2 – 3 g(x) + 2
 2 = g(2)2 – 3g(2) + 2
 g(2) = 0, 3  1
so g(2) = 0
f(x) = 2x – 3
1 1 1
f(g(x) = x  f(g(x)) . g(x) = 1  g(2) = = =–
f (g(2)) f (0) 3
Self practice problems :
–1
(16) Determine f (x), if given function is invertible
2
f : (–, –1) → (–, –2) defined by f(x) = –(x + 1) – 2

RI
Answers : –1– −x − 2

Inverse Trigonometry Functions


 
 

A
Introduction : The student may be familiar about trigonometric functions viz sin x, cos x, tan x, cosec x, sec x,
cot x with respective domains R, R, R – {(2n + 1) /2}, R – {n}, R – {(2n + 1) /2}, R – {n} and
respective ranges [–1, 1], [–1, 1], R, R – (–1, 1), R – (–1, 1), R.

UH
Correspondingly, six inverse trigonometric functions (also called inverse circular functions) are defined.

Inverse Domain Range Graph


Trigonometric
Function
[–/2, /2]
f(x) = sin–1x or [–1, 1]
JA
arcsinx
LP

f(x) = cos–1x or [–1, 1] [0, ]


arccosx
A
NK

y
f(x) = tan–1x or R (–/2, /2)
arctanx
SA

/2

o x

−/2
f(x) = cot–1x or R (0, )
arccotx

f(x) = sec–1x or R – (–1, 1) [0, ] – {/2}


arcsecx

RI
A
f(x) = cosec–1x R – (–1, 1) [–/2, /2] – {0}
or arccosecx

UH
JA
  1  1 
Example # 26 : Find the value of tan . cos−1   + tan−1  − 
LP

 2  3  
  1  1        1
Solution : tan cos−1   + tan−1  −   = tan  +  −   = tan   = .
 2  3    3  6  6 3
A

Example # 27 : Find domain of sin–1 (2x2 + 1)


Solution : Let y = sin–1 (2x2 + 1)
For y to be defined – 1  (2x2 + 1)  1  –2  2x2  0  x  {0}
NK

Self practice problems :


  1 
(17) Find the value of (i) cos  − sin−1  −  
3  2 
(ii) cosec [sec–1 ( 2 ) + cot–1 (1)]
SA

(18) Find the domain of


(i) y = sec–1 (x2 + 3x + 1)
 x2 
(ii) y = sin–1 
 1 + x 2 
 
(iii) y = cot–1 ( x 2 − 1)

(19) Find the range of (i) sin–1|x| + sec–1|x|


(ii) sin–1 x2 + x + 1
Answers : (17) (i) 0 (ii) 1
(18) (i) (– , – 3]  [ – 2, – 1]  [0, ) (ii) R
(iii) (– , –1]  [1, )
(19) (i) {/2} (ii) [/3, /2]

Property 1 : T(T–1)
(i) sin (sin−1 x) = x, −1  x  1

Proof : Let  = sin–1x. Then x  [–1, 1] &   [–/2, /2].


 sin  = x, by meaning of the symbol 

RI
sin (sin–1 x) = x

Similar proofs can be carried out to obtain

−1  x  1 tan (tan−1 x) = x, x  R

A
(ii) cos (cos−1 x) = x, (iii)
(iv) cot (cot−1 x) = x, x  R (v) sec (sec−1 x) = x, x  −1, x  1
(vi) cosec (cosec−1 x) = x, |x|  1

UH
Property 2 : T–1(T)
 −2n + x, x  [2n −  / 2, 2n +  / 2]
(i) sin–1 (sin x) = 
JA
(2n + 1)  − x, x  [(2n + 1)  −  / 2, (2n + 1) +  / 2], n  Z

Graph of y = sin–1 (sin x)


LP
A
NK

−2n + x, x  [2n, (2n + 1)]


(ii) cos–1 (cos x) = 
 2n − x, x  [(2n − 1) , 2n], n  
Graph of y = cos–1 (cos x)
SA
RI
A
(iii) tan–1 (tan x) = – n + x, n – /2 < x < n + /2, n  z
Graph of y = tan (tan x)
–1

UH
JA
LP

(iv) cosec–1 (cosec x) is similar to sin–1 (sin x)


Graph of y = cosec–1 (cosec x)
A
NK
SA

(v) sec–1 (sec x) is similar to cos–1 (cos x)


Graph of y = sec–1 (sec x)
RI
A
(vii) cot–1 (cot x) = –n + x, x  (n , (n + 1)  ), n  Z
Graph of y = cot–1 (cot x)

UH
JA
LP

Remark : sin (sin–1x), cos (cos–1x), .... cot (cot–1x) are aperiodic (non periodic) functions where as
sin–1 (sin x), ..., cot–1(cot x) are periodic functions.
A

Property 3 : “–x”
The graphs of sin–1x, tan–1 x, cosec–1x are symmetric about origin.
NK

Hence we get sin–1 (–x) = – sin–1x


tan–1 (–x) = – tan–1x
cosec–1 (–x) = – cosec–1x.
Also the graphs of cos–1x, sec–1x, cot–1x are symmetric about the point (0, /2). From this, we get
cos–1 (–x) =  – cos–1x
SA

sec–1 (–x) =  – sec–1x


cot–1 (–x) =  – cot–1x.

Property 4 : “/2”

(i) sin−1 x + cos−1 x = , −1  x  1
2
Proof : Let A = sin–1x and B = cos–1x  sin A = x and cos B = x
 sin A = cos B  sin A = sin (/2 – B)
 A = /2 – B, because A and /2 – B  [–/2, /2]
 A + B = /2.
Similarly, we can prove
 
(ii) tan−1 x + cot−1 x = , x  R (iii) cosec−1 x + sec−1 x = , x  1
2 2
  3  
Example # 28 : Find the value of cosec cot  cot −1 .
  4  
Solution :
 cot (cot–1 x) = x,  x  R
 3  3
 cot  cot −1 =
 4  4
  3    3 

RI
cosec cot  cot −1   = cosec  =. 2
  4   4 

A
 3 
Example # 29 Find the value of tan–1  tan .
 4 
  

UH
Solution :  tan–1 (tan x) = x if x   − , 
 2 2
3      3  3
As  − ,   tan–1  tan   
4  2 2  4  4
3   3 
JA
     , 
4 2 2 
graph of y = tan–1 (tan x) is as :
LP
A
NK

 3
   from the graph we can see that if < x< ,
2 2
then tan–1 (tan x) = x – 
 3  3 
 tan–1  tan  = 4 – =– 4
 4 
SA

Example # 30 : Find the value of sin–1 (sin7) and sin–1 (sin (–5)).
Solution. Let y = sin–1 (sin 7)
   5
sin–1 (sin 7)  7 as 7   − ,    2 < 7 <
 2 2 2
graph of y = sin (sin x) is as :
–1
RI
5
From the graph we can see that if 2  x  , then
2
y = sin–1(sin x ) can be written as :
y = x – 2

A
 sin–1 (sin 7) = 7 – 2
Similarly if we have to find sin–1 (sin(–5)) then
3

UH
 – 2 < – 5 < –
2
 from the graph of sin–1 (sin x), we can say that sin–1 (sin(–5)) = 2 + (–5) = 2 – 5

Example # 31 : Solve sin–1 (x2 – 2x + 1) + cos–1(x2 – x) =
2

JA
Solution : sin–1(f(x)) + cos–1(g(x)) =  f(x) = g(x) and –1  f(x), g(x)  1
2
x2 – 2x + 1 = x2 – x  x = 1, accepted as a solution

Self practice problems :


   
(20) Find the value of (i) cos sin  sin−1  
  6 
LP

  3  
(ii) sin cos  cos−1 
4  
(iii) cos–1 (cos 13)
 
  7  
(iv) cos–1 (– cos 4) (v) tan–1 tan  −  (vi) tan–1
A

  8 
  1 
cot  −  
  4 
NK

 5 
(21) Find sin–1 (sin ), cos–1(cos), tan–1 (tan ), cot–1(cot) for    , 3 
 2 
(22) Solve the following equations (i)5 tan–1x + 3 cot–1x = 2 (ii) 4 sin–1x =  – cos–1x

(iii) Solve sin–1(x2 – 2x + 3) + cos–1(x2 – x) =
SA

2
3
Answer : (20) (i) (ii) not defined (iii) 13 – 4
2
  1 
(iv) 4 –  (v) (vi) 4 − 2
8  
(21) 3 –  ,  – 2,  – 3,  – 2
1
(22). (i) x = 1 (ii) x= (iii) No solution
2

Interconversion & Simplification


Interconversion of any trigonometric ratio inverse means its conversion in remaining five trigonometric
ratio inverse. Example
 for x  (0,1) for x  (–1,0)
 −1
 cos 1 − x
2
− cos−1 1 − x 2
 x x
 tan−1 tan−1
 1 − x2 1 − x2

sin–1 x =  1 − x2 1 − x2
cot −1 − + cot −1
 x x

sec −1 1 1
− sec −1
 1 − x2 1 − x2

 cos ec −1 1 cos ec −1
1
 x x

RI
Example # 32 : Convert (i) tan–13, (ii) sin–1 (–1/3) in terms of cosine inverse.
1 1
Sol. (i) Let  = tan–13  tan = 3  cos =   = cos–1

A
10 10
(ii) sin–1 (–1/3) = – sin–1 (1/3)
1 2 2 2 2
Let  = sin–1 (1/3)  sin =  cos =   = cos–1

UH
3 3 3
2 2
 sin–1 (–1/3) = – cos–1
3
 tan (1/ x),
−1
x0
Example # 33 : Show that cot–1x = 
−1
JA
 + tan (1/ x), x  0
Sol. Let cot–1x =  (x = cot)    (0, )
  
     0, 
  2
Now tan–1(1/x) = tan–1tan() = 
 −      ,  
 2 
 
LP

 cot −1 x x0
= 
−1
cot x −  x  0
 tan−1(1/ x), x0
A

 cot–1x = 
−1
 + tan (1/ x), x  0
NK

 2 tan−1 x if | x |  1
2x 
Example # 34 : Show that sin−1 =   − 2 tan−1 x if x  1
1 + x2 
 −  + 2 tan−1 x
( ) if x  −1

SA

  
Sol : Let tan–1 x = (x = tan)     − ,   2  (– )
 2 2
     
− − 2 2   −, −  or    − , − 
  2  2 4
2x       
Now sin–1 = sin–1sin2 =  2 2   − ,  or    − , 
1 + x2   2 2   4 4
    
  − 2 2   ,   or    , 
 2  4 2 
 2 tan−1 x if x  [–1, 1]

=   − 2 tan−1 x if x 1

 −  + 2 tan−1 x
( ) if x  −1

Example # 35 : Define y = cos–1 (4x3 – 3x) in terms of cos–1 x


Solution : Let cos–1x = (x = cos)    [0, ]  3 = [0, 3]
Now y = cos–1(4x3 – 3x) = cos–1 cos3
  
 3 3  [0, ] or   0, 
  3
   2 

RI
= 2 − 3 3  [, 2] or    , 
 3 3 
  2 
 3 −  3  [2, 3] or    ,  
 3 

A
 −1 1
 3cos x ;  x 1
2

UH
1 1
y = cos–1 (4x3 – 3x) =  2 − 3cos−1 x ; −  x 
 2 2
 −1 1
−2 + 3cos x ; −1  x  − 2

JA
   −2   
Example # 36 : Simplify (i) sin   tan cot −1   
  3   
 
 1
(ii) sin  2 tan−1
 2 
(iii) cos (2cos–1(1/5) + sin–1(1/5))
LP

  −2  
Solution : (i) Let y = tan cot −1   ........(A)
  3 
 cot–1 (–x) =  – cot–1x, x  R
(A) can be written as
A

  2 
y = tan  − cot −1   
  3 
NK

 2
y = – tan  cot −1 
 3
1
 cot–1 x = tan–1 if x>0
x
 3 3    −2     3 
y = – tan  tan−1  so sin   tan cot −1 
SA

  y =–
    = sin − 2  = 1
 2 2    3     
 1  1  1  1   −1 2 
(ii) sin  2 tan−1  = 2sin  tan−1  cos  tan−1  = 2sin  sin−1  × cos  cos 
 2  2  2  5  5
1 2 4
=2×  =
5 5 5
 1 1  1 1 1
(iii) cos  2cos−1 + sin−1  = cos  cos−1 + sin−1 + cos−1 
 5 5   5 5 5 
 1  −1  1  
= cos  + cos−1  = – sin  cos    .........(i)
 2 5    5 
2
 1 2 6
=– 1−   =– .
5 5

Self practices problem :


 1 − x2 
(23) Define (i) cos−1   in terms of tan–1x
 1 + x2
 
 3x − x3 
(ii) tan–1   in terms of tan–1 x
 1 − 3x 2
 

RI
  2   −1  1  
(24) Find the value of (i) sec  cos−1    , (ii) cosec  sin  −  ,
  3    3  
 41   16  1  −3  

A
(iii) tan  cos ec −1  , (iv) sec  cot −1  , (v) sin  cot −1  
 4   63   4 
 2
  1 
(vi) tan 2 tan−1   −  ,

UH
  5  4

(25) If x  (–1, 1) and 2 tan–1 x = tan–1y then find y in term of x.


JA
1
(26) Find the value of sin (2cos–1x + sin–1x) when x =
5
1 − x2  2 tan−1 x if x  0
Answers : (23) (i) cos−1 =  −1
1 + x2  − 2 tan x if x  0
 −1 1 1
LP

 3 tan x ; − x
 3 3
 3x − x3   −1 1
(ii) tan–1   =   + 3 tan x ; −  x  −
2 
 1 − 3x   3
 −1 1
− + 3 tan x ; x
A

 3
3 4 65 2 5 −7
(24) (i) (ii) – 3 (iii) (iv) (v) (vi)
NK

2 5 16 5 17
2x
(25) y=
1 − x2
1
(26)
5
SA

Identities on addition and subtraction :

S.No. Identities Condition


(1) tan–1x + tan–1y = /2 x, y > 0 & xy = 1
(2)  x+y  x, y  0 & xy < 1
tan–1x + tan–1y = tan−1  
 1 − xy 
(3)  x+y  x, y  0 & xy > 1
tan–1x + tan–1y =  + tan−1  
 1 − xy 
(4)  x−y  x  0, y  0
tan–1x – tan–1y = tan–1  
 1 + xy 
x  0, y  0 and (x2 + y2)  1
(5)
(
sin−1 x + sin−1 y = sin−1 x 1 − y 2 + y 1 − x 2 )
x  0, y  0 and (x2 + y2)  1
(6)
(
sin−1 x + sin−1 y =  − sin−1 x 1 − y 2 + y 1 − x 2 )
x, y  [0, 1]
(7)
(
sin–1x – sin–1y = sin–1 x 1 − y 2 − y 1 − x 2 )
x, y  [0, 1]
(8)
(
cos–1 x + cos–1y = cos–1 xy − 1 − x 2 1 − y 2 )
0x<y1
(9) cos–1 x – cos–1y = cos−1 ( xy + 1− x 1− y )
2 2

RI
0y<x1
(10) cos–1 x – cos–1y = – cos−1 ( xy + 1− x 1− y )
2 2

A
Some useful Results :
(i) If tan−1 x + tan−1 y + tan−1 z =  , then x + y + z = xyz

(ii) If tan−1 x + tan−1 y + tan−1 z = , then xy + yz + zx = 1

UH
2
(iii) tan−1 1 + tan−1 2 + tan−1 3 = 

1 1 
(iv) tan−1 1 + tan−1+ tan−1 =
2 3 2
JA
4 15  84
Example # 37 : Show that cos–1 + sin–1 = + cos–1
5 17 2 85
4 3
Solution : cos–1 = sin–1 
5 5
2 2
3 3  15  8226
LP

15
   > 0, > 0 and  5  +  17  = 7225 > 1
5 17    

3 15 3 225 15 9 
 sin–1 + sin–1 =  – sin–1  1− + 1− 
5 17 5 289 17 25 
A

3 8 15 4  84   84  84
=  – sin–1  . + .  =  – sin–1   =– + cos–1 = + cos–1
5 17 17 5  85  2 85 2 85
NK

1 4
Example # 38 : Evaluate cot–1 + cot–1 + cot–11
9 5
1 4 5
SA

Solution : cot–1 + cot–1 + cot–11 = tan–1 9 + tan–1 + cot–11


9 5 4

5  5
   9 > 0, > 0 and  9   > 1
4  4

 5 
5  9+ 4 
 tan–1 9 + tan–1 + cot–11 =  + tan–1   + cot–11 =  + tan–1 (– 1) + cot–11
4  1 − 9. 5 
 4 

=– + cot–11 =  .
4
Self practice problems:
4 5 16
(27) Evaluate sin–1 + sin–1 + sin–1
5 13 65
(28) If tan–14 + tan–1 5 = cot–1 , then find ‘’
3 16 1 7
(29) Prove that 2 cos–1 + cot–1 + cos–1 =
13 63 2 25

 19 1
Answers. (27) (28) =– (29) x=
2 9 2

RI
A
UH
JA
LP
A
NK
SA
 Marked questions are recommended for Revision.
PART - I : SUBJECTIVE QUESTIONS

Section (A) : Ordered pair , Cartesian product, Relation, Domain and Range of Relation
A-1. If A = {2, 4, 5}, B = {7, 8, 9}, then find n(A × B).

A-2. If A = {x : x2 – 5x + 6 = 0}, B = {2, 4}, C = {4, 5} then find A × (B  C).

RI
A-3. A and B are two sets having 3 and 4 elements respectively and having 2 elements in common. Find the
number of possible relations which can be defined from A to B.

A-4. If A = {2, 3, 4, 5}, B = {1, 3, 5, 7} and a relation R : A →B such that y = 2x –3, xA, yB, then find R.

A
2
A-5. Let R be a relation defined as R = { (x, y) : y = ( x – 1) , x  Z and –3  x  3} then find
(i) Domain of R (ii) Range of R (iii) Relation R

UH
A-6. The Certesian product A × A has 16 elements S = {(a,b)  A × A| a < b}. (–1,2) and (0,1) are two
elements belonging to S. Find the set containing the remaining elements of S.
 
Section (B) : Types of Relation
JA
 
B-1. Identify the type of relation among reflexive, symmetric and transitive.
(i) R = {(1, 1), (2, 2), (3, 3), (1, 2), (2, 3), (1, 3)} on set A = {1, 2, 3}.
(ii) P = {(x, y) | x2 + y2 = 1, x, y  R}

B-2. Prove that the relation ''less than'' in the set of natural number is transitive but not reflexive and
LP

symmetric.

B-3. Let A = {p, q , r}. Which of the following is an equivalence relation on A ?


(i) R = {(p, q), (q, r), (p, r) (p, p)}
(ii) R = {(p, p), (q, q), (r, r) (q, p)}
A

(iii) R = {(p, p), (q, q), (r, r)}


(iv) R = {(p, p), (q, q), (r, r), (p, q), (q, r), (p, r)}
(v) R = {(p, p), (q, q), (r, r), (p, q), (q, p)}
NK

B-4. Let R be a relation on the set N be defined by {(x, y)| x, y  N, 2x + y = 41}. Then prove that R is neither
reflexive nor symmetric and nor transitive.
 
B-5. Let n be a fixed positive integer. Define a relation R on the set of integers Z, aRb  n|(a – b). Then
prove that R is equivalence.
 
SA

B-6. Let S be a set of all square matrices of order 2. If a relation R defined on set S such that
AR B  AB = BA, then identify the type of relation of R (A, B  S) among reflexive, symmetric and
transitive.
 
Section (C) : Definition of function, Domain and Range, Classification of Functions
 
C-1. Check whether the followings represent function or not
(i) x2 + y2 = 36, y  [0, 6] (ii) x2 + y2 = 36, x  [0, 1]
(iii) x2 + y2 = 36, x  [–6, 6] (iv) x2 + y2 = 36

 
C-2. Find the domain of each of the following functions :
x3 − 5x + 3
(i) f(x) = (ii) f(x) = sin(cos x)
x2 − 1
1
(iii) f(x) = (iv) f(x) = ex+sinx
x+ | x |
1 log2 (x − 2)
(v) f(x) = + x+2 (vi) f(x) =
log10 (1 − x) log1/ 2 (3x − 1)

cos x − 1
(vii) f(x) = n [x + x + 1], where [.] GIF.
2
(viii) f (x) = 2
6 + 35 x − 6 x 2

RI
C-3. Find the domain of definitions of the following functions :
  (i) f (x) = 3 − 2x − 21 − x (ii) f (x) = 1 − 1 − x2

A
x−2 1− x
(iii) f (x) = (x2 + x + 1)–3/2 (iv) f (x) = +
x+2 1+ x
1

UH
(v) f(x) = tan x − tan2 x (vi) f(x) =
1 − cos x
 5x − x 2
(vii) f (x) = og1/ 4 
  (viii) f (x) = og10 (1 – og10(x2 – 5x + 16))  
 4 
JA
C-4. Find the range of each of the following functions :
x |x−4|
(i) f(x) = | x – 3 | (ii) f(x) = (iii) f(x) = 16 − x 2 (iv) f(x) =
1 + x2 x−4

C-5. Find the domain and the range of each of the following functions :
1 x2 − 9
LP

(i) f(x) = (ii) f(x) = x ! (iii) f(x) = (iv) f(x) = sin2(x3) + cos2(x3)
4 + 3 sin x x−3

C-6. Find the range of each of the following functions : (where {.} and [.] represent fractional part and
greatest integer part functions respectively )
A

1
(i) f(x) = 5 + 3 sin x + 4 cos x (ii) f(x) =
1+ x
(iii) f(x) = 2 – 3x – 5x2 (iv) f(x) = 3 | sin x| – 4 |cos x|
NK

sin x cos x  8 − x2   1 
(v) f(x) = + (vi) f(x) = n   (vii) f(x) =  
1 + tan2 x 1 + cot 2 x  x−2   sin{x} 
 
C-7. Find the range of the following functions : (where {.} and [.] represent fractional part and greatest
integer part functions respectively )
SA

1
(i) f (x) = 1 – | x – 2 | (ii). f(x) =
16 − 4 x − x
2

1 x+2
(iii) f (x) = (iv) f (x) = 2
2 − cos 3 x x − 8x − 4
x2 − 2 x + 4 2
(v) f (x) = (vi) f (x) = 3 sin − x2
x2 + 2 x + 4 16
(vii) f (x)= x4 − 2 x2 + 5 (viii) f (x) = x3 – 12x, where x  [–3, 1]
(ix) f (x) = sin x + cos x
2 4

(x) f(x) = [sin x + [cos x + [tan x+ [secx]]]] Here x  (0, /4)


(xi) f(x) = sec2x – tan2x + sin (sinx + cos x)
C-8. Find whether the following functions are one-one or many-one & into or onto if f : D → R where D is its
domain.
(i) f(x) = |x2 + 5x + 6 | (ii) f(x) = |nx|
   1
(iii) f(x) = sin 4x :  − ,  → ( −1, 1) (iv) f(x) = x + , x  (0, )
 8 8 x
1 
 −1 3x 2
(v) f(x) = 1 − e x  (vi) f(x) = – cos x
4

1 + x6 1
(vii) f(x) = 3
(viii) f(x) = x cos x (ix) f(x) =
x sin | x |

RI
C-9. Classify the following functions f(x) defined in R → R as injective, surjective, both or none.
2
f(x) = x3 − 6 x2 + 11x − 6
x
(i) f (x) = x |x| (ii) f (x) = (iii)
1 + x2

A
C-10. Check whether the following functions is/are many-one or one-one & into or onto
(i) f(x) = tan (2 sin x) (ii) f(x) = tan (sin x)

UH
 
C-11. Let f : A → A where A = {x : –1  x  1}. Find whether the following functions are bijective.
x
(i) x – sin x (ii) x |x| (iii) tan (iv) x4
4
JA
C-12. Let A be a set of n distinct elements. Then find the total number of distinct functions from A to A ? How
many of them are onto functions ?
 
Section (D) : Identical functions, Composite functions
 
D-1. Check whether following pairs of functions are identical or not ?
1
LP

2
(i) f(x) = x2 and g(x) = ( x) (ii)_ f(x) = tanx and g(x) =
cot x
1 + cos 2x
(iii) f(x) = and g(x) = cos x (iv) f(x) = x and g(x) = enx
2
 
A

D-2. Find for what values of x, the following functions would be identical.
 x − 1
f(x) = log (x − 1) − log (x − 2) and g (x) = log  
x − 2
NK

D-3. Let f(x) = x2 + x + 1 and g(x) = sin x. Show that fog  gof

D-4. Let f(x) = x2, g(x) = sin x, h(x) = x , then verify that [fo (goh)] (x) and [(fog) oh] (x) are equal.
SA

D-5. Find fog and gof, if


(i) f(x) = ex ; g(x) = n x (ii) f(x) = |x| ; g(x) = sin x
1
(iii) f(x) = sin x ; g(x) = x 2 (iv) f(x) = x2 + 2 ; g(x) = 1 – ,x1
1− x

D-6. If f(x) = n(x2 – x + 2) R+ → R and


;
g(x) = {x} + 1 ; [1, 2] → [1, 2] , where {x} denotes fractional part of x.
Find the domain and range of f(g(x)) when defined.

1 + x 2 ; x 1
D-7. If f(x) =  and g(x) = 1 – x ; – 2 < x < 1, then define the function fog(x).
 x + 1; 1  x  2
x+2 x−2
D-8. If f(x) = and g(x) = , then find the domain of
x +1 x
(i) fog(x) (ii) gof(x) (iii) fof(x) (iv) fogof(x)

 2x x  Q − {0}
D-9. If f(x) =  , then define fof(x) and hence define fofof.....f(x) where f is ‘n’ times.
 3x x  Qc

 x +1 x4
  x 2 −1  x  3
D-10. Let f(x)  2x + 1 4  x  9 and g(x) =  then, find f(g(x)).
− x + 7  x + 2 3  x  5
 x9

RI
4x
D-11. If f(x) = , then show that f(x) + f(1 – x) = 1
4x + 2

A
Section (E) : Even/Odd Functions & Periodic Functions

E-1. Determine whether the following functions are even or odd or neither even nor odd :

UH
 ax − 1 
(iii) f(x) = x  x
 a + 1 
(i) sin (x2 + 1) (ii) x + x2
 
(iv) f(x) = sin x + cos x (v) f(x) = (x 2 – 1) | x |
 | ne x | ; x  −1

JA
(vi) f(x) = [2 + x] + [2 − x] ; −1  x  1 , where [.] is GIF.

 e nx ; x 1
 
E-2. Examine whether the following functions are even or odd or neither even nor odd, where [ ] denotes
greatest integer function.
(1 + 2x )7 sec x + x 2 − 9
LP

(i) f (x) = (ii) f (x) =


2x x sin x
 x | x |, x  −1

(iii) f (x) = 1+ x + x 2
– 1− x + x 2
(iv) f (x) = [1 + x] + [1 − x], −1  x  1
 − x | x |, x 1
A

E-3. Which of the following functions are not periodic (where [ . ] denotes greatest integer function) :
NK

(i) f(x) = sin x (ii) f(x) = x + sin x


(iii) f(x) = [sin 3x] + |cos 6x|

E-4. Find the fundamental period of the following functions :

(i) f (x) = 2 + 3cos (x – 2) (ii) f(x) = sin 3x + cos 2x + | tanx |


SA

x x 3 2
(iii) f (x) = sin + sin (iv) f(x)= cos x − sin x.
4 3 5 7

1 sin12x
(v) f(x) = (vi) f(x) =
1 − cos x 1 + cos2 6x

(vii) f(x) = sec3x + cosec3x


Section (F) : Inverse of a function

F-1. Let f : D → R, where D is the domain of f. Find the inverse of f, if it exists


1/ 5
(i) f (x) = 1 − 2 −x (ii) (
f (x) = 4 − (x − 7)3 )
(iii) f(x) = n (x + 1 + x 2 )
(iv) Let f  [0, 3] → [1, 13] is defined by f(x) = x 2 + x + 1, then find f– 1 (x).

e2x − e−2x
F-2. Let f : R → R be defined by f(x) = . Is f(x) invertible ? If yes, then find its inverse.
2

RI
F-3. (a) If f(x) = –x|x|, then find f–1(x) and hence find the number of solutions of f(x) = f –1(x).
5 − 9 + 8x 5
(b) Solve 2x2 – 5x + 2 = , where x <
4 4

A
F-4. If g is inverse of f(x) = x3 + x + cosx, then find the value of g(1).

 ( − 1)x x  Qc  x x  Qc

UH
F-5. If f(x) =  and g(x) =  are inverse to each other then find all
− x +  + 3x − 1 x  Q
2
1 − x x Q
possible values of .

Section (G) : Definition, graphs and fundamentals & Inverse Trigonometry


 
JA
G-1. Find the domain of each of the following functions :
sin−1 x  3x − 1  −1 1
(i) f(x) = (ii) f(x) = 1 − 2x + 3 sin–1   (iii) f(x) = 2sin x +
x  2  x−2

G-2. Find the range of each of the following functions :


LP

 3x 2 + 1 
(i) f(x) = n (sin–1x) (ii) f(x) = sin–1  
 5x 2 + 1 
 
 (x − 1)(x + 5) 
(iii) f(x) = cos–1  
A

 x(x − 2)(x − 3) 

G-3. Find the simplified value of the following expressions :


NK

  1   1  1 
(i) sin  − sin−1  −   (ii) tan cos−1 + tan−1  − 
3  2   2  3  
 
  3 
(iii) sin–1 cos sin−1    
   

  2 
SA

n n
G-4. (i) If 
i =1
cos−1 i = 0, then find the value of  i· 
i =1
i

2n 2n
(ii) If 
i=1
sin−1 xi = n , then show that x
i =1
i = 2n

G-5. Solve the following inequalities:


(i) cos −1 x > cos −1 x2 (ii)  arccot2 x − 5 arccot x + 6 > 0

(iii) sin–1 x > – 1 (iv) cos–1 x < 2 (v) cot–1 x < – 3


  
G–6. Let f :  − ,  → B defined by f (x) = 2 cos 2x + 3 sin2x + 1. Find B such that f –1
exists. Also find
 3 6
f (x).
–1

Section (H) : Trig (trig–1x), trig–1 (trig x), trig–1 (–x) and Property (/2)

H-1. Evaluate the following inverse trigonometric expressions :


 7   2 
(i) sin–1  sin (ii) tan–1  tan 3 
 6   
 5   7 
(iii) cos–1  cos (iv) sec–1  sec
4  4 

RI
 

H-2. Find the value of the following inverse trigonometric expressions :


(i) sin–1 (sin 4) (ii) cos–1 (cos 10)

A
(iii) tan–1 (tan (– 6)) (iv) cot–1 (cot (– 10))
 1  9 9  
(v) cos–1   cos 10 − sin 10  
 2  

UH
H-3. Find the value of following expressions :
(i) cot (tan–1 a + cot–1 a) (ii) sin (sin–1x + cos–1x) , | x |  1

H-4. Solve the inequality tan–1 x > cot–1 x.


JA
Section (I) : Interconversion/Simplification

I-1. Evaluate the following expressions :


 3  1
(i) sin  cos−1  (ii) tan  cos−1 
 5   3 
LP

 41   65 
cosec  sec −1  tan  cos ec −1
63 
(iii) (iv)
 5  

 1  4 2
(v) sin  + cos−1  (vi) cos  sin−1 + cos−1 
6 4  5 3
A

       1
(vii) sec  tan tan−1  −    (viii) cos tan– 1 sin cot–1  
  3    2
 
NK

 3 3 
(ix) tan cos−1   + sin−1   − sec −1 3 
  
4  
4 

I-2. Find the value of sin–1 (cos(sin–1x)) + cos–1 (sin (cos–1x))


SA

1
I-3. If tan–1x + cot–1 + 2tan–1z = , then prove that x + y + 2z = xz 2 + yz2 + 2xyz 
y
1 − x 2 – xy
I-4. If cos–1x + 2sin–1x + 3cot–1y + 4tan–1y = 4sec–1z + 5cosec–1z , then prove that z2 – 1 =  
x + y 1– x 2
2  2 
I-5. Consider, f(x) = tan–1   , g(x) = sin–1   and h(x) = tan (cos–1(sinx)), then show that
x  4+x 
2

0 , x  0
(h(f(x)) + h(g(x)) = 
x , x  0
I-6. Prove each of the following relations :
1 x 1
(i) tan–1 x = –  + cot–1 = sin–1 = – cos–1 when x < 0.
x 1 + x2 1 + x2
1 1 − x2 x
(ii) cos–1x = sec–1 =  – sin–1 1 − x 2 =  + tan–1 = cot –1 when – 1<x< 0
x x 1 − x2
I-7. Express in terms of
2x 1
(i) tan–1 to tan–1 x for x > 1 (ii) sin–1 (2x 1 − x 2 ) to sin–1 x for 1  x >
1 − x2 2
(iii) cos–1 (2x2 – 1) to cos–1x for – 1  x < 0

RI
 1  2x  1 
−1 1 − y
 
2
I-8. Simplify tan  sin−1   2
+ cos    , if x > y > 1.
 1 + x2   1 + y2
 2   

A
I-9. Solve for x
1 
(i) cos (2 sin–1x) = (ii) cot–1 x + tan–1 3 =
3 2

UH
 x −1   x +1  2
(iii) tan–1   + tan  x + 2  = 4
–1
(iv) sin–1x + sin–12x =
 x − 2    3

Section (J) : Addition and Subtraction Rule


JA
J-1. Prove that
3  8  77 3 5 33
  (i) sin–1   + sin–1   = sin–1 (ii) tan–1 + sin–1 = cos–1
5  17  85 4 13 65
 1    1  1  1  1 
(iii) sin–1   + cot–1 3 = (iv) tan–1   + tan–1   + tan–1   + tan–1   =
 5 4 3 5 7 8 4
LP

J-2. Find the sum of each of the following series :


1 1 1 1
(i) tan−1 2 + tan−1 2 + tan−1 2 + tan−1 2 ........ upto
x + x + 1 x + 3x + 3 x + 5x + 7 x + 7 x + 13
n terms.
A

1 2 2n−1
(ii) tan−1 + tan−1 + ..... + tan−1 + .....................upto infinite terms
3 9 1 + 22n − 1
NK

1 2 − 1 n − n − 1
(iii)  sin−1 + sin−1 + ..... + sin−1 + ............ upto infinite terms
2 6 n (n + 1)

PART - II : ONLY ONE OPTION CORRECT TYPE


SA

Section (A) : Ordered pair , Cartesian product, Relation, Domain and Range of Relation
A-1. If A = {a, b}, B = {c, d}, C = {d, e}, then {(a, c), (a, d), (a, e), (b, c), (b, d), (b, e)} is equal to
(A) A  (B  C) (B) A  (B  C) (C) A × (B  C) (D) A × (B  C)

A-2. If A={1, 2, 3} and B = {1, 2} and C = {4, 5, 6}, then what is the number of elements in the set A  B  C ?
(A) 8 (B) 9 (C) 15 (D) 18

A-3. Let A = {a, b, c} and B = {1, 2}. Consider a relation R defined from set A to set B. Then R can equal to
set
(A) A (B) B (C) A × B (D) B × A
A-4. Let R be relation from a set A to a set B, then
(A) R = A  B (B) R = A  B (C) R  A × B (D) R  B × A

A-5. Let X = {1, 2, 3, 4, 5} and Y = {1, 3, 5, 7, 9}. Which of the following is not a relation from X to Y
(A) R1 = {(x, y) | y = 2 + x, x  X, y  Y} (B) R 2 = {(1, 1), (2, 1), (3, 3), (4, 3), (5, 5)}
(C) R3 = {(1, 1), (1, 3) (3, 5), (3, 7), (5, 7)} (D) R4 = {(1, 3), (2, 5), (2, 4), (7, 9)}

A-6. The relation R defined in A = {1, 2, 3} by a R b if –5  a 2 – b2  5. Which of the following is false?


(A) R = {(1, 2), (2, 2), (3, 3), (2, 1), (2, 3), (3, 2)} (B) Co-domain of R = {1, 2, 3}
(C) Domain of R = {1, 2, 3} (D) Range of R = {1, 2, 3}

RI
Section (B) : Types of Relation

B-1. The relation R defined in N as aRb  b is divisible by a is


(A) Reflexive but not symmetric (B) Symmetric but not transitive

A
(C) Symmetric and transitive (D) Equivalence relation

B-2. In the set A = {1, 2, 3, 4, 5} a relation R is defined by R = {(x, y)| x, y  A and x < y}. Then R is
(A) Reflexive (B) Symmetric (C) Transitive (D) Equivalence relation

UH
B-3. Which one of the following relations on R is equivalence relation-
(A) x R1y  x2 = y2 (B) x R2y  x  y (C) x R3y  x | y (x divides y) (D) x R4y  x < y

B-4. Let R1 be a relation defined by R1 = {(a, b)| a  b ; a, b  R} . Then R1 is


JA
(A) An equivalence relation on R (B) Reflexive, transitive but not symmetric
(C) Symmetric, Transitive but not reflexive (D) Neither transitive nor reflexive but symmetric

B-5. Let L denote the set of all straight lines in a plane. Let a relation R be defined by R  ⊥, ,   L.
The R is
(A) Reflexive (B) Symmetric (C) Transitive (D) equivalence relation
LP

B-6. Let S be the set of all real numbers. Then the relation R =
{(a, b) : 1 + ab > 0} on S is
(A) Reflexive and symmetric but not transitive (B) Reflexive, transitive but not symmetric
(C) Symmetric, transitive but not reflexive (D) Reflexive, transitive and symmetric
A

B-7. Consider the following :


1. If R = {(a, b)  N × N : a divides b in N} then the relation R is reflexive and symmetric but not
transitive.
2. If A = {1, 2, 3, 4, 5, 6} and R = {(S 1, S2) : S1, S2 are subsets of A, S1  S2}, then the relation R is
NK

not reflexive, not symmetric and not transitive.


Which of the statements is/are correct ?
(A) 1 only (B) 2 only (C) Both 1 and 2 (D) Neither 1 nor 2

B-8. Let R be a relation over the set N × N and it is defined by (a, b) R (c, d)  a + d = b + c. Then R is
(A) Symmetric only (B) Transitive only (C) Reflexive only (D) Equivalence only
SA

B-9. Let L be the set of all straight lines in the Euclidean plane. Two lines 1 and 2 are said to be related by
the relation R if 1 is parallel to 2. Then R is
(A) Symmetric only (B) Transitive only (C) Reflexive only (D) Equivalence only

B-10. Let R = {(x, y) : x, y  A, x + y = 5} where A = {1, 2, 3, 4, 5} then R is


(A) Reflexive (B) symmetric (C) Transitive (D) Equivalence
 
B-11. Let S be a set of all square matrices of order 2. If a relation R defined on set S such that
AR B  AB = O, where O is zero square matirx of order 2, then relation R is (A, B  S)
(A) Reflexive (B) Transitive
(C) Symmetric (D) Not equivalence
Section (C) : Definition of function, Domain and Range, Classification of Functions
− log0.3 (x − 1)
C-1. The domain of the function f(x) = is
x 2 + 2x + 8
(A) (1, 4) (B) (– 2, 4) (C) (2, 4) (D) [2, )

C-2. Range of f(x) = n (3x2 – 4x + 5) is


 11   11   11 
(A)  n ,   (B) [n 10 , ) (C)  n ,   (D)  n , 
 3   6   12 

C-3. Range of f(x) = 4x + 2x + 1 is


(A) (0, ) (B) (1, ) (C) (2, ) (D) (3, )

RI
C-4. Range of f(x) = log 5
( 2 (sinx – cosx) + 3) is
 3
(C) 0, 

A
(A) [0, 1] (B) [0, 2] (D) [1, 2]
 2

2 x2 − x + 5

UH
C-5. Let f : R → R be a function defined by f(x) = , then f is :
7 x 2 + 2x + 10
(A) one − one but not onto (B) onto but not one − one
(C) onto as well as one − one (D) neither onto nor one − one
 
Let f : R → R be a function defined by f(x) = x 3 + x2 + 3x + sin x. Then f is:
JA
C-6.
(A) one − one and onto (B) one − one and into
(C) many one and onto (D) many one and into

3
C-7. Domain of definition of the function f(x) = + log10(x3 – x), is :
4 – x2
(A) (1,2) (B) (–1,0)  (1,2)
LP

(C) (1,2)  (2, ) (D) (–1,0)  (1,2)  (2, )

x
C-8. If f : [0, ) → [0, ), and f (x) = , then f is:
1+ x
A

(A) one-one and onto (B) one-one but not onto


(C) onto but not one-one (D) neither one-one nor onto
 
NK

(x − 2)2
C-9. Range of the function f(x) = is
(x − 1)(x − 3)
(A) (1, ) (B) (–, 1) (C) R – (0, 1] (D) (0, 1]

x−2
C-10. Range of the function f(x) = is
SA

x 2 − 4x + 3
(A) (–, 0) (B) R (C) (0, ) (D) R – {0}

C-11. Statement - 1 If f (x) and g (x) both are one one and f(g (x)) exists, then f(g (x)) is also one one.
Statement - 2 If f(x1) = f(x2)  x1 = x2 , then f(x) is one-one.
(A) STATEMENT-1 is true, STATEMENT-2 is true and STATEMENT-2 is correct explanation for
STATEMENT-1
(B) STATEMENT-1 is true, STATEMENT-2 is true and STATEMENT-2 is not correct explanation
for STATEMENT-1
(C) STATEMENT-1 is true, STATEMENT-2 is false
(D) STATEMENT-1 is false, STATEMENT-2 is true
(E) Both STATEMENTS are false
C-12. Statement - 1 If y = f (x) is increasing in [ ], then its range is [f (), f () ]
Statement - 2 Every increasing function need not to be continuous.
(A) STATEMENT-1 is true, STATEMENT-2 is true and STATEMENT-2 is correct explanation for
STATEMENT-1
(B) STATEMENT-1 is true, STATEMENT-2 is true and STATEMENT-2 is not correct explanation
for STATEMENT-1
(C) STATEMENT-1 is true, STATEMENT-2 is false
(D) STATEMENT-1 is false, STATEMENT-2 is true
(E) Both STATEMENTS are false
 
0 , x  rational
C-13. If the functions f(x) and g(x) are defined on R→R such that f(x) =  ,
 x , x  irrational

RI
0 , x  irrational
g(x) =  , then (f – g) (x) is
 x , x  rational
(A) one-one and onto (B) neither one-one nor onto

A
(C) one-one but not onto (D) onto but not one-one
 
Section (D) : Identical functions, Composite functions

UH
D-1. Which of the following pair of functions are identical –
(A) f(x) = sin2x + cos2x and g(x) = 1 (B) f(x) = sec2x – tan2x and g(x) = 1
(C) f(x) = cosec2x – cot2 x and g(x) = 1 (D) f(x) = nx2 and g(x) = 2nx
 
D-2. Let f(x) be a function whose domain is [– 5, 7]. Let g(x) = |2x + 5|, then domain of (fog) (x) is
JA
(A) [– 4, 1] (B) [– 5, 1] (C) [– 6, 1] (D) [– 5, 7]

− 1 , x  0

D-3. Let g (x) = 1 + x − [ x ] and f (x) =  0 , x = 0 . Then for all x, f (g (x)) is equal to (where [.] denotes

1 , x  0
LP

greatest integer function)


(A) x (B) 1 (C) f (x) (D) g (x)

Section (E) : Even/Odd Functions & Periodic Functions


 1 + sin x 
E-1. The function f(x) = log   is
A

 1 − sin x 
(A) even (B) odd
(C) neither even nor odd (D) both even and odd
NK

1
E-2. The function f(x) = [x] + , x   is a/an (where [ . ] denotes greatest integer function)
2
(A) Even (B) odd
(C) neither even nor odd (D) Even as well as odd
 
SA

E-3. The graph of the function y = f(x) is symmetrical about the line x = 2, then :
(A) f(x + 2) = f(x – 2) (B) f(2 + x) = f(2 – x) (C) f(x) = f(–x) (D) f(x) = –f(–x)
E-4. Fundamental period of f(x) = sec (sin x) is

(A) (B) 2 (C)  (D) aperiodic
2

E-5. If f (x) = sin ( )


[ a ] x (where [ . ] denotes the greatest integer function) has  as its fundamental period,
then
(A) a = 1 (B) a = 9 (C) a  [1, 2) (D) a  [4, 5)
 
 E-6. Find the area below the curve y =  2 + 2cos 2x  but above the x-axis in [–3, 6] is
 
(where [ . ] denotes the greatest integer function) :
(A) 2 square units (B)  square units (C) 6 square units (D) 8 square units

Section (F) : Inverse of a function

ex − e− x
F-1. The inverse of the function f(x) = is
ex + e− x
1 1+ x 1 2+x 1 1− x
(A) n (B) n (C) n (D) 2 n (1 + x)
2 1− x 2 2−x 2 1+ x

RI
1
F-2. If f: [1, ) → [2, ) is given by f (x) = x + , then f −1 (x) equals:
x
x+ x2 − 4 x x− x2 − 4
(D) 1 − x 2 − 4

A
(A) (B) 2
(C)
2 1+ x 2

F-3. If f : R → R is an invertible function such that f(x) and f –1(x) are also mirror image to each other about

UH
the line y = –x, then
(A) f(x) is odd
(B) f(x) and f–1(x) may not be mirror image about the line y = x
(C) f(x) may not be odd
(D) f(x) is even
JA
ax + b
F-4. If f(x) = , then (fof) (x) = x, provided that
cx + d
(A) d + a = 0 (B) d – a = 0 (C) a = b = c = d = 1 (D) a = b = 1


x −1  x  1
F-5. Let f(x) =  2 the range of h–1(x), where h(x) = fof(x) is
LP


x 1 x  2
(A)  −1, 2  (B) [–1, 2] (C) [–1, 4] (D) [–2, 2]
 
 
F-6. Statement – 1 All points of intersection of y = f (x) and y = f – 1 (x) lies on y = x only.
A

Statement – 2 If point P (, ) lies on y = f (x), then Q (, ) lies on y = f – 1 (x).


Statement – 3 Inverse of invertible function is unique and its range is equal to the function domain.
Which of the following option is correct for above statements in order
NK

(A) T T F (B) F T T (C) T T T (D) T F T

Section (G) : Definition, graphs and fundamentals of Inverse Trigonometric functions


 
G-1. The domain of definition of f(x) = sin −1 (x − 1 − 2) is:
(A) [− 2, 0]  [2, 4] (B) (− 2, 0)  (2, 4) (C) [− 2, 0]  [1, 3] (D) (− 2, 0)  (1, 3)
SA

G-2. The function f(x) = cot−1 (x + 3) x + cos−1 x 2 + 3x + 1 is defined on the set S, where S is equal to:
(A) {0, 3} (B) (0, 3) (C) {0, − 3} (D) [− 3, 0]

G-3. Domain of f(x) = cos–1 x + cot–1 x + cosec–1 x is


(A) [– 1, 1] (B) R (C) (–  − ]  [1, ) (D) {– 1, 1}

G-4. Range of f(x) = sin–1 x + tan–1 x + sec–1 x is


  3    3    3    3 
(A)  ,  (B)  ,  (C)  ,  (D)  , 
4 4  4 4  4 4  4 4 
G-5. cosec–1 (cos x) is real if
(A) x  [– 1, 1] (B) x  R

(C) x is an odd multiple of (D) x is a multiple of 
2


G-6. Domain of definition of the function f (x) = sin−1 (2 x) + for real valued ' x ' is:
6
 1 1  1 1  1 1  1 1
(A) − 4 , 2  (B) − 2 , 2  (C)  − ,  (D)  − , 
     2 9  4 4

   3 

RI
The solution of the equation sin−1  tan  − sin−1  − = 0 is
 x  6
G-7.
 4  
(A) x = 2 (B) x = − 4 (C) x = 4 (D) x = 3 

A
3
G-8. Number of solutions of the equation cot–1 4 − x 2 + cos−1(x 2 − 5) = is :
2
(A) 2 (B) 4 (C) 6 (D) 8

UH
Section (H) : Trig (trig–1x), trig–1 (trig x), trig–1 (–x) and Property (/2)

H-1. If   x  2, then cos–1 (cosx) is equal to


(A) x (B)  – x (C) 2 + x (D) 2 – x
JA
2
H-2. If sin–1 x + sin–1 y = , then cos–1x + cos–1y is equal to
3
2  
(A) (B) (C) (D) 
3 3 6

If x  0 and  = sin−1x + cos−1x − tan−1x, then


LP

H-3.
 3    
(A)  (B) 0    (C) 0   < (D) 
2 4 4 2 4 2

H-4. Number of solutions of equation tan–1(e–x) + cot–1(|nx|) = /2 is :


A

(A) 0 (B) 1 (C) 3 (D) 2

Section (I) : Interconversion/Simplification


NK

 3 3
I-1. The numerical value of cot  2sin−1 + cos−1  is
 5 5
−4 −3
  (A) (B) (C)
3
     (D)
4
3 4 4 3
I-2. STATEMENT-1 : tan2 (sec–1 2) + cot2 (cosec– 1 3) = 11.
SA

STATEMENT-2 : tan2  + sec2  = 1 = cot2  + cosec2 


(A) STATEMENT-1 is true, STATEMENT-2 is true and STATEMENT-2 is correct explanation for
STATEMENT-1
(B) STATEMENT-1 is true, STATEMENT-2 is true and STATEMENT-2 is not correct explanation
for STATEMENT-1
(C) STATEMENT-1 is true, STATEMENT-2 is false
(D) STATEMENT-1 is false, STATEMENT-2 is true
(E) Both STATEMENTS are false
1 
I-3. If  is a real root of the equation x 3 + 3x – tan2 = 0, then cot–1  + cot–1 – can be equal to 
 2
 3
(A) 0 (B) (C)  (D)
2 2
 x  x 2
If sin–1  + sin–1  1–  + tan–1y =
 2 
I-4. , then :
 4  3
   
49
(A) maximum value of x2 + y2 is (B) maximum value of x2 + y2 is 4
3
1
(C) minimum value of x2 + y2 is (D) minimum value of x2 + y2 is 3
2

 1
I-5. If x < 0, then value of tan–1(x) + tan–1   is equal to
x

RI
 
(A) (B) – (C) 0 (D) –
2 2

 1 

A
I-6. If sin–1x + cot–1   = , then x is equal to
2 2
1 2 3
(A) 0 (B) (C) (D)

UH
5 5 2

 1 
I-7. The numerical value of tan  2 tan−1 −  is
 5 4
−7 7 17 2
JA
(A) (B) (C) (D) –
17 17 7 3

Section (J) : Addition and Subtraction Rule

 3x – 3x   x 
LP

J-1. If f(x) = tan–1 


 3 3 + x 2 
+ tan–1   , 0  x  3, then range of f(x) is
   3
       
(A) 0,  (B) 0,  (C)  ,  (D) 0, 
 2  4 6 3  3
A

a b 
J-2. STATEMENT-1 : If a > 0, b > 0, tan– 1   + tan– 1   = . x= ab .
x x 2
NK

m n−m 
STATEMENT-2 : If m, n  N, n  m, then tan– 1   + tan– 1   = .
n n+m 4
(A) STATEMENT-1 is true, STATEMENT-2 is true and STATEMENT-2 is correct explanation for
STATEMENT-1
(B) STATEMENT-1 is true, STATEMENT-2 is true and STATEMENT-2 is not correct explanation
for STATEMENT-1
SA

(C) STATEMENT-1 is true, STATEMENT-2 is false


(D) STATEMENT-1 is false, STATEMENT-2 is true
(E) Both STATEMENTS are false

y
J-3. If cos–1x – cos–1 = , then 4x2 – 4xy cos  + y2 is equal to-
2
(A) 2 sin 2 (B) 4 (C) 4sin2  (D) – 4 sin2 
PART - III : MATCH THE COLUMN
1. Match the relation defined on set A = {a,b,c} in column I with the corresponding type in column II
Column I Column II
(A) {a,b), (b,a) (p) symmetric but not reflexive and transitive
(B) {(a,b), (b,a), (a,a), (b,b)} (q) equivalence
(C) {(a,b), (b,c), (a,c)} (r) symmetric and transitive but not reflexive
(D) {(a,a), (b,b), (c,c)} (s) transitive but not reflexive and symmetric
 

2. Column –  Column – 
(A) If S be set of all triangles and f : S → R+, f() = Area (p) one-one
of , then f is
 3 
(B) f : R →  ,   and f(x) = cot–1(2x – x2 – 2), then f(x) is

RI
(q) many one
 4 
2x 2 − x + 1
(C) If f : R → R such that f(x) = , then f(x) is (r) onto function
7x 2 − 4x + 4

A
(D) f : R → R and f(x) = epx sinqx where p, q  R+, then f(x) is (s) into function

3. Match The column

UH
(A) If f(x) is even & g(x) is odd (p) then fog must be odd
 

(B) If g(x) is periodic (q) then fog must be manyone


 

(C) If f(x) & g(x) are bijective (r) then fog is periodic
 

(D) If f(x) is into (s) then fog is injective


JA
 

(t) then fog is into

4. Let f (x) = sin–1 x, g (x) = cos–1 x and h(x) = tan –1


x. For what complete interval of variation of x the
following are true.
Column –  Column – 
(A) f ( x) + g ( x) = /2 (p) [0, )
LP

(B) f (x) + g ( 1− x )
2
=0 (q) [0, 1]

 1 − x2 
(C) g   = 2 h (x) (r) (– , 1)
 1 + x2
 
A

 1+ x 
(D) h(x) + h(1) = h   (s) [– 1, 0]
 1− x 
NK

5. Match the column


Column -  Column - 
(A) Let a, b, c be three positive real numbers (p) 
a (a + b + c) b (a + b + c) c (a + b + c)
 = tan– 1 + tan– 1 + tan– 1 ,
bc ca ab
SA

then  is equal to
(B) The value of the expression
1  
tan −1  tan 2 A  + tan −1 (cot A) + tan −1 (cot3A) for 0 < A < (/4) (q) –
2  2
is equal to
1
(C) If x < 0, then {cos– 1(2x2 – 1) + 2cos–1 x} is equal to (r) –
2
3  12   16  
(D) The value of sin–1   – cos–1   + cos–1   is equal to (s)
5  13   65  2
 Marked questions are recommended for Revision.
PART - I : ONLY ONE OPTION CORRECT TYPE
1. For real numbers x and y, we write x R y  x – y + 2 is an irrational number. Then the relation R is-
(A) Reflexive (B) Symmetric (C) Transitive (D) Equivalence relation

2. Let A = N × N be the Certesian product of N and N. Let


S = {((m, n), (p, q))  A × A : m + q = n + p}
Consider the following statements:
I.If ((m,n), (p , q))  S, and ((p,q), (r, s))  S then ((r,s), (m,n))  S

RI
II.There exists at least one element ((m,n), (p, q))  S such that ((p , q), (m, n))  S
Which of the statements given above is / are correct ?
(A) I only (B) II only (C) Both I and II (D) Neither I nor II.

A
3. Let A = Z, the set of integers. Let R 1 = {(m, n)  Z × Z : (m + 4n) is divisible by 5 in Z}.
Let R2 = {(m, n)  Z × Z : (m + 9n) is divisible by 5 in Z}.
Which one of the following is correct ?

UH
(A) R1 is a proper subset of R2 (B) R2 is a proper subset of R1
(C) R1 = R2 (D) R1 is not a symmetric relation on Z

4. Let X be the set of all persons living in a state. Elements x, y in X are said to be related if ‘x < y’,
whenever y is 5 years older than x. Which one of the following is correct?
(A) The relation is an equivalence relation
JA
(B) The relation is transitive only
(C) The relation is transitive and symmetric, but not reflexive
(D) The relation is neither reflexive, nor symmetric, nor transitive

  1  
5. The domain of the function f (x) = log 1 / 2  − log2  1 + 4  − 1 is:
  x 
LP

(A) 0 < x < 1 (B) 0 < x  1 (C) x  1 (D) null set

6. If q 2 − 4 p r = 0, p > 0, then the domain of the function f (x) = log (p x 3 + (p + q) x 2 + (q +


r) x + r) is:
 q    q 
(A) R −  − (B) R −  ( −  , − 1]  −
A

 
 2p    2p  
  q 
(C) R −  ( −  , − 1)  −  (D) R
NK

  2p  
x − [x]
7. Let f (x) = , R → A is onto then find set A. (where {.} and [.] represent fractional part and
1 + x − [x]
greatest integer part functions respectively )
 1  1  1  1
SA

(A)  0 ,  (B) 0 ,  (C) 0 ,  (D)  0 , 


 2  2  2  2

−x
ex − e
8. Let f be a real valued function defined by f(x) = , then the range of f(x) is :
x
ex + e
(A) R (B) [0, 1] (C) [0, 1) (D) 0 , 21 )

9. The range of the function f (x) = log


2
( 2 − log (16 sin
2
2
x +1)) is
(A) (−  1) (B) (−  2) (C) (−  1] (D) (−  2]
10. Which of the following pair of functions are identical ?
x x x2
(A) 1 + sin x , sin + cos (B) x,
2 2 x
2
(C) x2 , ( x) (D) n x3 + n x2 , 5 n x

11. If domain of f(x) is (– , 0], then domain of f(6{x} 2 – 5 {x} + 1) is (where {} represents fractional part
function).
 1 1  1   1 1
(A) n + 3 , n + 2  (B) (– , 0) (C) n + 6 , n + 1 (D) n − 2 , n − 3 
n   n    nI  

RI
12. Let f: (e, ) → R be defined by f(x) =n (n(n x)), then
(A) f is one one but not onto (B) f is onto but not one - one
(C) f is one-one and onto (D) f is neither one-one nor onto
 

A
13. If f (x) = 2 [x] + cos x, then f: R → R is: (where [. ] denotes greatest integer function)
(A) one−one and onto (B) one−one and into
(C) many−one and into (D) many−one and onto

UH
 
 x | x | −4 ; x  Q

14. If f : R → R be a function such that f(x) =  , then f(x) is
x | x | − 3 ; x  Q

(A) one-one, onto (B) many one, onto (C) one-one, into (D) many one, into
 
JA
15. f (x) = x − 1, f: R+ → R, g(x) = ex, g: [− 1, ) → R. If the function fog (x) is defined, then its domain
and range respectively are:
(A) (0, ) and [0, ) (B) [− 1, ) and [0, )
 1  1 
(C) [− 1, ) and 1 − ,   (D) [− 1, ) and  − 1,  
 e  e 
LP

x
16. Let f : (2, 4) → (1, 3) be a function defined by f (x) = x −   (where [. ] denotes the greatest integer
2
function), then f −1 (x) is equal to :
x
(A) 2x (B) x +   (C) x + 1 (D) x − 1
A

2

17. The mapping f : R → R given by f (x) = x 3 + ax2 + bx + c is a bijection if


NK

(A) b2  3a (B) a2  3b (C) a2  3b (D) b2  3a

18. If the function f: [1, ) →[1, ) is defined by f(x) = 2 x(x – 1) then f–1 is
1
(A) (1/2)x(x – 1) (B)
2
( 1 + 1 + 4log2 x )
1
SA

(C)
2
( 1 − 1 + 4log2 x ) (D) Not defined

19. Let f : N → N, where f(x) = x + (–1)x – 1 , then the inverse of f is.


(A) f –1(x) = x + (–1)x – 1 , x  N (B) f –1(x) = 3x + (–1)x – 1 , x  N
(C) f (x) = x , x  N
–1
(D) f –1(x) = (–1)x – 1 , x  N

 1   1 
20. tan  + cos−1 x  + tan  − cos−1 x  , x  0 is equal to
4 2  4 2 
2 x
(A) x (B) 2x (C) (D)
x 2
 1 − sin x + 1 + sin x  
21. The value of cot−1   , where < x < , is:
 1 − sin x − 1 + sin x  2
x  x x x
(A)  − (B) + (C) (D) 2  −
2 2 2 2 2
 1+ x 3 
22. The domain of the function f (x) = sin−1  3 / 2  + sin (sin x) + log(3{x} + 1) (x2 + 1),
 2x 
 
where {.} represents fractional part function, is:
(A) x  {1} (B) x  R − {1, − 1} (C) x > 3, x  I (D) x  

23. A function g(x) satisfies the following conditions

RI
(i) Domain of g is (–, ) (ii) Range of g is [–1, 7]
(iii) g has a period  and (iv) g(2) = 3
Then which of the following may be possible.
 3 ; x = n
(A) g(x) = 3 + 4 sin (n + 2x – 4), n  

A
(B) g(x) = 
 3 + 4 sin x ; x  n
(C) g(x) = 3 + 4 cos (n + 2x – 4), n   (D) g(x) = 3 – 8 sin (n + 2x – 4), n  

UH
24. The complete solution set of the inequality [cot –1x]2 – 6 [cot–1 x] + 9  0, where [.] denotes greatest
integer function, is
(A) (– , cot 3] (B) [cot 3, cot 2] (C) [cot 3, ) (D) (– , cot 2]

25. The inequality sin– 1 (sin 5) > x2 – 4x holds for


JA
(
(A) x  2 – 9 − 2 , 2 + 9 − 2 ) (B) x > 2 + 9 − 2

(C) x < 2 – 9 − 2 (D) x  

 x2 x3   2 x 4 x6  
26. If sin-1  x − + − .......  + cos-1  x − + − .......  = for 0 < | x | < 2 , then x equals
 2 4  2 4  2
   
LP

(A) 1/2 (B) 1 (C) – 1/2 (D) –1


 
27. cot–1 ( cos  ) – tan–1 ( )
cos  = x. then sin x is equal to -

  


A

(A) tan2   (B) cot2   (C) tan  (D) cot  


2 2 2
 
The Inverse trigonometric equation sin–1 x = 2 sin–1 , has a solution for
NK

28.
3 3 1 1
(A) – << (B) all real values of  (C) ||  (D) || 
2 2 2 2
 

 
29. If f(x) = cot–1x : R+ →  0, 
 2
SA

and g(x) = 2x – x2 : R → R. Then the range of the function f(g(x)) wherever define is
       
(A)  0,  (B)  0,  (C)  ,  (D)  
 2  4   4 2  4
 

cos−1 sin x + 
( ( )) , g(x) = cosec−1  4
− 2cos x 
3
30. Given the functions f(x) = e   and the function
 3 
h(x) = f(x) defined only for those values of x, which are common to the domains of the functions f(x) and
g(x). The range of the function h(x) is :
    
− −
(A) [e 6 ,e  ] (B) [e 6 ,e  ] (C) (e 6 ,e  ) (D) [e 6 ,e 6 ]  
PART - II : SINGLE AND DOUBLE VALUE INTEGER TYPE
 q   r 
1. The domain of the function y = sin x + cos x + 7x − x 2 − 6 is p, 4    4 , s  then value of
   
p + q + r + s is

 1
x + 2 
2. The domain of f(x) such that the f(x) =   is prime is [x , x ), then the value of 2(x 2 + x 2). [Where
 1 1 2 1 2

x − 2 
 
[.] denotes greatest integer function less than or equal to x]

RI
x3 + 2x 2 + 3x + 2
3. Number of integers in the range of the function f(x) = ; x  R – {0} is :
x3 + 2x 2 + 2x + 1

A
4. Range of the function f(x) = |sin x |cos x| + cos x |sin x|| is [a, b] then (a + b) is equal to

5. If f and g are two distinct linear functions defined on R such that they map[–1, 1] onto [0, 2] and

UH
f(x)
h : R – {–1, 0, 1} → R defined by h(x) = , then |h(h(x)) + h(h(1/x))| > n. Then maximum integral
g(x)
value of n is :

1
, g(x) = f (f(x)), h(x) = f(f(f(x))), then the absolute value of f(x) . g(x) . h(x), where x  0, 1, is
6. If f(x) =
JA
1− x

7. If f(x) = ax7 + bx3 + cx – 5 ; a, b, c are real constants and f(–7) = 7 then maximum value of |f(7)+17cosx|
is

4a − 7 3
8. If f(x) = x + (a – 3) x2 + x + 5 is a one-one function, then number of possible integral values of a
LP

3
is

Number of solutions of the equation e− sin


2
x
9. = tan2x in [0, 10] is
 
A

10. Let f(x)= ([a]2 – 5[a] + 4)x3 – (6{a}2 – 5 {a} + 1)x – (tan x) sgn (x) be an even function  x  R, then the
sum of all possible values of '3a' is
(where [] denotes G.I. F and {} fractional part functional part function)
NK

11. Let f be a one−one function with domain {21, 22, 23} and range {x,y,z}. It is given that exactly one of
the following statements is true and the remaining two are false. f(21) = x; f(22)  x ; f(23)  y. Then
f–1(x) is :
SA

− 2 +1 2 + 1 1− x
12. Let f : [– 2 + 1, 2 +1] →  ,  be a function defined by f(x) = .
 2 2  1 + x2
 −1 + 


If f–1(x) = 
( 4x − 4x 2 + 1 ), x  0 , then  +  is.
 2x

  , x=0

3
13. The number of real solutions of the equation x 3 + 1 = 2 2x – 1 , is :
 
14. If cos−1x + cos−1y + cos−1z = , where − 1  x, y, z  1, then find the value of x 2 + y2 + z2 + 2 x y z
226
15. The sum of absolute value of all possible values of x for which cos tan −1 sin cot −1 x = .
227

n 
16. If cot−1 > , n  N, then the maximum value of ‘ n ‘ is:
 6

  sin(cos−1 x) + cos(sin−1 x)   10
 1
17. If x  (0, 1) and f(x) = sec tan−1  
 cos(cos−1 x) + sin(sin−1 x)  
, then 
f   is
r =2  
r
   

1  3 sin2  
18. If sin−1   = , then tan  is equal to

RI
2  5 + 4cos 2  4

19. The number of real solutions of equation 1 + cos 2x = 2 sin–1 (sin x), −10   x  10 , is/are

A
20. The number of solution(s) of the equation, sin −1x + cos−1 (1 − x) = sin−1 (− x), is/are
 

n
   k 
1 

UH
21. Find the value of 3    
n =1  k =1
cot −1  1 + 2
  r 
3

  r =1  

PART - III : ONE OR MORE THAN ONE OPTIONS CORRECT TYPE


JA
1. Let A = {1, 2, 3, 4} and R be a relation in A given
by R = {(1, 1), (2, 2), (3, 3), (4, 4), (1, 2), (2, 1), (3, 1), (1, 3)}, then relation R is
(A) Reflexive (B) Symmetric (C) Equivalence (D) Reflexive and Symmetric

2. For n, m  N, n | m means that n is a factor of m, then relation | is


(A) Reflexive (B) symmetric (C) Transitive (D) Equivalence
LP

 4−x 
2
3. If f(x) = sin n   , then
 1− x 
 
(A) domain of f(x) is (– 2, 1) (B) domain of f(x) is [–1, 1]
A

(C) range of f(x) is [–1, 1] (D) range of f(x) is [–1, 1)

4. D is domain and R is range of f(x) = x − 1 + 2 3 − x , then


NK

(A) D : [1 , 3] ; (B) D : (– , 1]  [3, ),


(C) R : 1, 3  (D) R :  2 , 10 
   
5. If [ 2 cos x ] + [ sin x ] = − 3, then the range of the function, f (x) = sin x + 3 cos x in [0, 2 ] lies in
(where [. ] denotes greatest integer function)
SA

(A) [– 3 , 3 ) (B) [–2, – 3 ] (C) [–3, –1] (D) [–2, – 3 )

6. Let D  [− 1, 1] is the domain of the following functions, state which of them are injective.
 −1 1
tan x0
(A) f(x) =  x (B) g(x) = x3
 1
 x=0
(C) h(x) = sin 2x (D) k(x) = sin (x/2)
 
7. Let f(x) = x135 + x125 – x115 + x5 + 1. If f(x) divided by x 3 – x, then the remainder is some function of x say
g(x). Then g(x) is an :
(A) one-one function (B) many one function (C) into function (D) onto function
8. The function f : X → Y, defined by f(x) = x 2 − 4x + 5 is both one−one and onto if
(A) X = [2, ) & Y = [1, ) (B) X = (–, 2] & Y = [1, )
(C) X = [3, ) & Y = [2, ) (D) X = (–, 2] & Y = (1, )

9. f : N → N where f(x) = x – (–1)x then f is :


(A) one-one (B) many-one (C) onto (D) into
 
10. Which one of the following pair of functions are NOT identical ?
(A) e(nx)/2 and x
(B) tan (tanx) and cot (cotx)
(C) cos2x + sin4x and sin2x + cos4x
|x|

RI
(D) and sgn (x), where sgn(x) stands for signum function.
x
 
ax − 1
11. If the graph of the function f (x) = is symmetric about y-axis, then n is equal to:

A
xn (a x + 1)
(A) 1/5 (B) 1/3 (C) 1 / 4 (D) – 1 / 3
 

UH
 x 2 x  1
12. If f(x) =  & composite function h(x) = |f(x)| + f(x + 2), then
1 − x x  1
(A) h(x) = 2x2 + 4x + 4  x  –1
(B) h(x) = x2 + x + 1  –1 < x  1
JA
(C) h(x) = x2 – x – 1  –1 < x  1
(D) h(x) = –2 x>1

 0 for x = 0

 2 
13. Let f(x) =  x sin x( ) for − 1  x  1 (x  0) , then:


x x for x  1 or x  − 1
LP

(A) f(x) is an odd function (B) f(x) is an even function


(C) f(x) is neither odd nor even (D) f  (x) is an even function
 
 x 2 + 1
If f : [–2, 2] → R where f(x) = x 3 + tanx + 
A

14.  is a odd function, then the value of parametric P,


 P 
where [.] denotes the greatest integer function, can be
(A) 5 < P < 10 (B) P < 5 (C) P > 5 (D) P = 15
NK

 
15. If f: R → [ − 1, 1], where f (x) = sin   x   , (where [.] denotes the greatest integer function), then
2 
(A) f (x) is onto (B) f (x) is into (C) f (x) is periodic (D) f (x) is many one
SA

2 x (sin x + tan x)
16. If f (x) = then it is, (where [.] denotes the greatest integer function)
x + 2 
2  − 3
  
(A) odd (B) Even (C) many one (D) one-one

17. dentify the statement(s) which is/are incorrect ?


(A) the function f(x) = sinx + cosx is neither odd nor even
(B) the fundamental period of f(x) = cos (sin x) + cos (cos x) is 
(C) the range of the function f(x) = cos (3 sin x) is [− 1, 1]
(D) f(x) = 0 is a periodic function with period 2
 
sin  [x]
18. If F (x) = , then F (x) is: (where { . } denotes fractional part function and [ . . ] denotes greatest
{x}
integer function and sgn (x) is a signum function)
(A) periodic with fundamental period 1 (B) even
 {x} 
(C) range is singleton (D) identical to sgn  sgn − 1
 { x } 

 

19. Let f : R → R and g : R → R be two one-one and onto functions such that they are mirror images of
each other about the line y = a. If h(x) = f(x) + g(x), then h(x) is
(A) one-one (B) into
(C) onto (D) many-one
 

RI
20. Which of following pairs of functions are identical.
−1
(A) f(x) = e nsec x and g(x) = sec−1 x
(B) f(x) = tan (tan−1 x) and g(x) = cot (cot−1 x)

A
(C) f(x) = sgn (x) and g(x) = sgn (sgn (x))
(D) f(x) = cot2 x. cos2 x and g(x) = cot2 x − cos2 x
 

UH
3
21. If sin–1x + sin–1y + sin–1z = , then
2
9
(A) x100 + y100 + z100 – =0 (B) x22 + y42 + z62 – x220 –y420 – z620 = 0
x101 + y101 + z101
JA
x 2008 + y 2008 + z2008
(C) x50 + y25 + z5 = 0 (D = 0)
(xyz)2009
 

22. If X = cosec tan−1 cos cot−1 sec sin−1 a and Y = sec cot−1 sin tan−1 cosec cos−1 a; where 0  a < 1. Find the
relation between X and Y. Then
(A) X = Y (B) Y = 3 − a2
LP

(C) XY (D) X = 2Y

23. If  satisfies the inequation x 2 – x – 2 > 0, then a value exists for


(A) sin–1  (B) cos–1  (C) sec–1  (D) cosec–1 
 
A

24. For the function f(x) = n (sin −1 og2 x),


1   
(A) Domain is  , 2 (B) Range is  −  , n 
2   2
NK

(C) Domain is (1, 2] (D) Range is R

25. In the following functions defined from [–1, 1] to [–1, 1] , then functions which are not bijective are
2
(A) sin (sin–1x) (B) sin–1(sin x) (C) (sgn x) n ex (D) x3 sgn x

SA

 sincot −1 cos tan−1 t 


1    1 + 2t 2 

26. The expression  −1 −1  .  2  can take the value
2 cos tan sincot 2t
  2+t 

(A) 1/2 (B) –5 (C) 1 (D) 3/4

1 − x2
27. If 0 < x < 1, then tan–1 is equal to:
1+ x
1 1+ x 1− x 1
(A) cos−1 x (B) cos−1 (C) cos−1 (D) sin−1x
2 2 2 2
x 1 
28. If f(x) = cos–1x + cos–1  + 3 − 3x 2  , then
2 2 
2  2 
(A) f   = (B) f   =
3 3 3 2
 1   1 1 
(C) f   = (D) f   = 2 cos–1 –
3 3 3 3 3


4n
29. tan −1 is equal to:
n =1 n − 2n2 + 2
4

(A) tan −1 2 + tan −1 3 (B) 4 tan −1 1 (C) /2 (


(D) sec −1 − 2 )

RI
30. If sin2 (2 cos–1 (tan x)) = 1 then x may be
 1   1 
(A) x =  + tan–1   (B) x =  – tan–1  

A
 2  2
 1   1 
(C) x = − + tan–1   (D) x = − – tan–1  
 2  2

UH
 
31. If sin–1 x + 2 cot–1 (y2 – 2y) = 2 , then
(A) x + y = y2 (B) x2 = x + y (C) y = y2 (D) x2 – x + y = y2

 
JA
PART - IV : COMPREHENSION
Comprehension # 1

Given a function f : A → B ; where A = {1, 2, 3, 4, 5} and B = {6, 7, 8}


LP

1. Find number of all such functions y = f(x) which are one-one ?
(A) 0 (B) 35 (C) 5P3 (D) 53

2. Find number of all such functions y = f(x) which are onto
(A) 243 (B) 93 (C) 150 (D) none of these
A

3. The number of mappings of g(x) : B → A such that g(i)  g(j) whenever i < j is
(A) 60 (B) 140 (C) 10 (D) 35
NK

Comprehension # 2
Let the domain and range of inverse circular functions are defined as follows
Domain Range
  3 
sin–1x [–1, 1] 2, 2 
SA

 
cos–1x [–1, 1] [0, ]
  3 
tan–1x R  2, 2 
 
cot–1x R (0, )
  3 
cosec–1x (–, –1]  [1, )  2 , 2  – {}
 

sec–1x (–, –1]  [1, ) [0, ] –  
2
3
4. sin–1x < then solution set of x is
4
 1   1   1 1 
(A)  , 1 (B)  − , − 1 (C)  − ,  (D) none of these
 2   2   2 2

 −  
5. If x  ,  , cosec–1 cosec x is
 2 2
(A) 2 – x (B)  + x (C)  –x (D) –  – x

6. If x  [–1, 1], then range of tan–1(–x) is


 3 7   3 5    

RI
(A)  , (B)  , (C) [–, 0] (D)  − , 
 4 4   4 4   4 4

A
UH
JA
LP
A
NK
SA
PART - I : JEE (ADVANCED) / IIT-JEE PROBLEMS (PREVIOUS YEARS)
* Marked Questions may have more than one correct option.
 Marked questions are recommended for Revision.
1. The maximum value of the function f(x) = 2x 3 – 15x2 + 36x – 48 on the set A = {x |x 2 + 20  9x} is
[IIT-JEE 2009,P-2,(4, –1), 80]

x
2. If the function f(x) = x3 + e 2 and g(x) = f–1(x), then the value of g(1) is [IIT-JEE 2009,P-2,(4, –1), 80]

Let f(x) = x2 and g(x) = sin x for all x  R. Then the set of all x satisfying (f o g o g o f) (x) = (g o g o f)

RI
3.
(x), where (f o g) (x) = f(g(x)), is [IIT-JEE 2011, Paper-2, (3, –1), 80]
(A) ± n , n  {0, 1, 2,....} (B) ± n , n  {1, 2,....}

(C) + 2n, n  {.....–2, –1, 0, 1, 2,....} (D) 2n, n  {...., –2, –1, 0, 1, 2,....}

A
2

  sin      d
Let f() = sin  tan−1    , where – <  < . Then the value of (f()) is

UH
4.
 )
  cos 2   4 4 d(tan
[IIT-JEE 2011, Paper-1, (4, 0), 80]
5. The function f : [0, 3] → [1, 29], defined by f(x) = 2x3 – 15x2 + 36x + 1, is
(A) one-one and onto (B) onto but not one-one
(C) one-one but not onto
JA (D) neither one-one nor onto
[IIT-JEE 2012, Paper-1, (3, –1), 70] 

2    
6*. Let f : (–1, 1) → R be such that f(cos 4) = for    0,    ,  . Then the value(s) of  
2 − sec  2
 4 4 2
 1
f   is (are)               [IIT-JEE 2012, Paper-2, (4, 0), 66]  
LP

 3
3 3 2 2
  (A) 1 – (B) 1 + (C) 1 – (D) 1 +
2 2 3 3
A

 23  n 
7. The value of cot 
 n=1 
cot –1  1 +
 
2k  

is [JEE (Advanced) 2013, Paper-1, (2, 0)/60]
  k =1 
NK

23 25 23 24
(A) (B) (C) (D)
25 23 24 23

8. Match List I with List II and select the correct answer using the code given below the lists :
List - I List - II
1/ 2
 2 
SA

 cos(tan–1 y) + y sin(tan–1 y)  1 5
P  1   + y 4
takes value 1.
 y2 –1 –1   2 3
  cot(sin y) + tan(sin y)  
Q. If cos x + cos y + cox z = 0 = sin x + sin y + sin z then 2. 2
x–y
possible value of cos is
2
  1
R. If cos  – x  cos 2x + sin x sin 2x sec x = cos xsin 2x sec x + 3.
4  2
 
cos  + x  cos 2x then possible value of sec x is
 4 
S. (
If cot sin–1 1– x 2 ) = sin ( tan –1
( x 6 )) , x  0, 4. 1
then possible value of x is [JEE (Advanced) 2013, Paper-2, (3, –1)/60]

Codes :
P Q R S
(A) 4 3 1 2
(B) 4 3 2 1
(C) 3 4 2 1
(D) 3 4 1 2 

  
9*. Let f:  – ,  → R be given by f(x) = (log(sec x + tanx)) 3 . Then

RI
 2 2
[JEE (Advanced) 2014, Paper-1, (3, 0)/60]
(A) f(x) is an odd function (B) f(x) is a one-one function
(C) f(x) is an onto function (D) f(x) is an even function

A
10. Let f : [0, 4] → [0, ] be defined by f(x) = cos –1 (cos x). The number of points x  [0, 4] satisfying the
10 − x
equation f(x) = is [JEE (Advanced) 2014, Paper-1, (3,

UH
10
0)/60]

6 4
11*. If  = 3sin–1   and  = 3cos–1   , where the inverse trigonometric functions take only the principal
 
11 9
JA
values, then the correct option(s) is(are) [JEE (Advanced) 2015, P-2 (4, –2)/ 80]
  (A) cos  > 0 (B) sin  < 0 (C) cos( + ) > 0 (D) cos  < 0

12. The number of real solutions of the equation


  
 x 
i
    x
i  
sin–1   xi+1 − x   2   = – cos –1
   − 2  −  ( − x)i 
LP

 i=1 i=1  2  i = 1 i=1




 1 1
lying in the interval  − ,
2 
is __________. [JEE(Advanced) 2018, Paper-1,(3, 0)/60]
 2
  
A

(Here, the inverse trigonometric functions sin –1x and cos–1x assume values in  − ,  and [0, ],
 2 2
respectively).
NK

13. Let X be a set with exactly 5 elements and Y be a set with exactly 7 elements. If  is the number of
one-one functions from X to Y and  is the number of onto function form Y to X, then the value of
1
( – ) is ______ . [JEE(Advanced) 2018, Paper-2,(3, 0)/60]
5!
 x  
 –1   x  

SA

14. Let E1 =  x  R : x  1 and  0  and E2 =  x  E1 : sin  loge    is a real number  .


 x –1  
   x – 1  

  
(Here, the inverse trigonometric function sin–1x assumes values in  – ,  .)
 2 2
 x 
Let f : E1 → R be the function defined by f(x) = log e  
 x – 1
  x 
and g : E2 → R be the function defined by g(x) = sin –1  loge  
  x – 1 
[JEE(Advanced) 2018, Paper-2,(3, –1)/60]
LIST-I LIST-II
 1   e 
(P) The range of f is (1)  – ,    , 
 1– e  e –1 
(Q) The range of g contains (2) (0, 1)
 1 1
(R) The domain of f contains (3)  – , 
 2 2
(S) The domain of g is (4) (–, 0)  (0, )
 e 
(5)  – ,
 e – 1
1 e 
(6) (–, 0)   , 
 2 e – 1

RI
The correct option is
(A) P → 4; Q → 2; R → 1; S → 1 (B) P → 3; Q → 3; R → 6; S → 5
(C) P → 4; Q → 2; R → 1; S → 6 (D) P → 4; Q → 3; R → 6; S → 5

A
PART - II : JEE (MAIN) / AIEEE PROBLEMS (PREVIOUS YEARS)

UH
1. Consider the following relations : [AIEEE-2010, (4, – 1), 144]
R : {(x, y)| x ,y are real numbers and x = wy for some rational number w}
m p
S= {  ,  | m, n, p and q are integers such that n, q  0 and qm = pn}
 n q
Then
JA
(1) neither R nor S is an equivalence relation
(2) S is an equivalence relation but R is not an equivalence relation
(3) R and S both are equivalence relations
(4) R is an equivalence relation but S is not an equivalence relation

2. Let R be the set of real numbers. [AIEEE-2011(Part-), (4, – 1), 120]
Statement-1 : A = {(x, y)  R × R : y – x is an integer} is an equivalence relation on R.
LP

Statement-2 : B = {(x, y)  R × R : x = y for some rational number } is an equivalence relation on R.


(1) Statement-1 is true, Statement-2 is true; Statement-2 is a correct explanation for Statement-1.
(2) Statement-1 is true, Statement-2 is true; Statement-2 is not a correct explanation for Statement-1.
(3) Statement-1 is true, Statement-2 is false.
(4) Statement-1 is false, Statement-2 is true.
A

 
3. Consider the following relation R on the set of real square matrices of order 3.
R = {(A, B)|A = P–1 BP for some invertible matrix P}. [AIEEE-2011(Part-), (3, – 1), 120]
NK

Statement -1 : R is equivalence relation.


Statement - 2 : For any two invertible 3 × 3 matrices M and N, (MN) –1 = N–1M–1.
(1) Statement-1 is true, statement-2 is a correct explanation for statement-1.
(2) Statement-1 is true, statement-2 is true; statement-2 is not a correct explanation for statement-1.
(3) Statement-1 is true, statement-2 is false.
(4) Statement-1 is false, statement-2 is true.
SA

1
4. The domain of the function f(x) = is : [AIEEE 2011, , (4, –1), 120]
| x | −x
(1) (– , ) (2) (0, ) (3) (–, 0) (4) (–, ) – {0}

5. Let f be a function defined by f(x) = (x–1)2 + 1, (x  1). [AIEEE 2011, , (4, –1), 120]
Statement - 1 : The set {x : f(x) = f –1(x)} = {1, 2}.
Statement - 2 : f is a bijection and f–1(x) = 1 + x − 1 , x  1.
(1) Statement-1 is true, Statement-2 is true; Statement-2 is a correct explanation for Statement-1.
(2) Statement-1 is true, Statement-2 is true; Statement-2 is NOT a correct explanation for Statement-1
(3) Statement-1 is true, Statement-2 is false
(4) Statement-1 is false, Statement-2 is true .
6. If x, y, z are in A.P. and tan –1x, tan–1y and tan–1z are also in A.P., then [AIEEE - 2013, (4, –1),120]
(1) x = y = z (2) 2x = 3y = 6z (3) 6x = 3y = 2z (4) 6x = 4y = 3z

1
7. If g is the inverse of a function f and f'(x) = , then g'(x) equal to :
1 + x5
[JEE(Main)2014,(4, – 1), 120]
1
(1) (2) 1 + {g(x)}5 (3) 1 + x5 (4) 5x4
1 + {g(x)}5

 2x  1
8. Let tan–1y = tan–1x + tan–1  2 
, where |x| < . Then a value of y is
 1– x  3

RI
[JEE(Main)2015,(4, – 1), 120]
3 3 3
3x – x 3x + x 3x – x 3x + x 3
(1) 2
(2) 2
(3) (4)
1– 3x 1– 3x 1 + 3x 2 1 + 3x 2

A
 1
9. If f(x) + 2f   = 3x, x  0, and S = {x  R : f(x) = f (–x)} ; then S : [JEE(Main)2016,(4, – 1), 120]
x

UH
(1) contains exactly one element (2) contains exactly two elements.
(3) contains more than two elements. (4) is an empty set.
 
10. Two sets A and B are as under : A = {(a, b)  R  R : |a –5| < 1 and |b – 5| < 1 } ;
B = {(a, b )  R  R : 4(a– 6)2 + 9(b–5)2  36 }. Then; [JEE(Main)2018,(4, – 1), 120]
(1) A  B =  ( an empty set) (2) Neither A  B nor B  A
JA
(3) B  A (4) A  B

 2   3   3
11. If cos–1   + cos–1   =  x   then x is equal to :
 3x   4x  2 4

[JEE(Main) 2019, Online (09-01-19),P-1 (4, – 1), 120]


LP

145 145 146 145


(1) (2) (3) (4)
12 10 12 11
A

1 1
12. For x  R – {0, 1}, let f1(x) = , f2(x) = 1 – x and f3(x) = be three given functions. If a function, J(x)
x 1− x
NK

satisfies (f2 J f1 ) (x) = f3(x) is equal to : [JEE(Main) 2019, Online (09-01-19),P-1 (4, – 1), 120]

1
(1) f3(x) (2) f1(x) (3) f3(x) (4) f2(x)
x

 19 −1 
n

SA

13. The value of cot   cot  1 +  2p   is : [JEE(Main) 2019, Online (10-01-19),P-2 (4, – 1), 120]
 n =1  p =1  

19 21 22 23
(1) (2) (3) (4)
21 19 23 22

14. The number of functions f from {1, 2, 3, …., 20}, onto {1,2,3,………..,20} such that f(k) is a multiple of 3,
wherever k is a multiple of 4, is : [JEE(Main) 2019, Online (11-01-19),P-2 (4, – 1), 120]

(1) 5!  6! (2) (15)!  6! (3) 65 × (15)! (4) 56 × 15


2
15. Let Z be the set of integers. If A = { x  Z : 2( x + 2)( x –5x +6)
= 1] and B = {x  Z : – 3  2x – 1  9} then the

number of subsets of the set A × B, is – [JEE(Main) 2019, Online (12-01-19),P-2 (4, – 1), 120]

(1) 218 (2) 212 (3) 215 (4) 210

EXERCISE - 1

RI
PART - I
Section (A) :
A-1. 9 A-2. {(2, 4), (3, 4)} A-3. 212 A-4. R = { (2, 1), (3, 3), (4, 5), (5, 7)}

A
A-5. (i) {– 3, – 2, – 1, 0, 1, 2, 3} (ii) {0, 1, 2, 3, 4}
(iii) {(–3, 4), (–2, 3), (–1, 2), (0, 1), (1, 0), (2, 1), (3, 2)}

UH
A-6. {(–1,0), (–1,1), (0,2), (1,2)}
 
Section (B) :
B-1. (i) Reflexive and transitive but not symmetric.
(ii) neither reflexive nor transitive but it is symmetric
 
JA
B-3. (iii) & (v) B-6. Reflexive and symmetric but not transitive
Section (C) :
C-1. (i) yes (ii) no (iii) no (iv) no  
 
C-2. (i) R – {–1, 1} (ii) 2n –  x  2n + , n   (iii) (0, ) (iv) R (v) [–2, 0)  (0, 1)
2 2
 1  5 
(vi) (2, 3] (vii) (–  –1]  [0, ) (viii) − 6 , 3    3 , 6  
LP

   
C-3. (i) [0, 1] (ii) [– 1, 1] (iii) R (iv) 
 
(v) n, n + 4  (vi) R – {2n}, n   (vii) (0, 1]  [4,5) (viii) (2, 3)
nI  
A

 1 1
C-4. (i) [0, ) (ii)  − ,  (iii) [0, 4] (iv) {–1, 1}
 2 2
1
y1 (ii) Domain : N  {0}, Range : (n! : n = 0, 1, 2,...}
NK

C-5. (i) Domain : R, Range :


7
(iii) Domain R – {3}, Range : R – {6} (iv) Domain : R, Range : {1}
49
C-6. (i) [0, 10] (ii) (0, 1] (iii) (–  , ] (iv) [–4, 3] (v) [–1, 1] (vi) R
20
(vii) n  N
SA

 1  1 
C-7. (i) (– , 1] (ii)  ,  (iii)  , 1 
 16 − 1/ 2  3 

 1  1  1   3 
(iv)  −, −    − ,   (v)  , 3  (vi)  0 , 
 4   20   3   2 
3 
(vii) [4, ) (viii) [–11, 16] (ix)  , 1
4 
(x) 1 (xi) 1 − sin 2, 1 + sin 2 
 
C-8. (i) many-one & into (ii) many-one & into (iii) one-one & onto (iv) many-one & into
(v) one – one & into (vi) many-one & into (vii) many-one & into (viii) many-one & onto
(ix) many-one & into
C-9. (i) bijective (injective as well as surjective) (ii) neither surjective nor injective
(iii) surjective but not injective
C-10. (i) many-one & onto (ii) many-one & into  
C-11. (i) No (ii) Yes (iii) Yes (iv) No C-12. nn , n!

Section (D) :
D-1. (i) No (ii) No (iii) No (iv) No D-2. (2, )
D-4. [fo(goh)] (x) = [(fog) oh] (x) = sin 2 x
D-5. (i) fog = x, x > 0 ; gof = x, x  R (ii) |sin x|, sin |x|
3x 2 − 4x + 2 x2 + 2

RI
(iii) sin (x2), (sin x)2 (iv) ,
(x − 1) 2
x2 + 1
2 − 2x + x 2 , 0  x  1
D-6. Domain : [1 , 2 ] ; Range : [n2 , n4) D-7. f(g(x)) = 
 2 − x, −1  x  0

A
D-8. (i) x  R – {0, 1} (ii) x  R – {–2, –1}
 3 

UH
(iii) x  R –  − , − 1 (iv) x  R – {–2, –1}
 2 
3 2 x x  Q − {0} 3
n−1
2x x  Q − {0}
D-9. fof (x) =  , fofof.....f(x) = 
2
 3 x x  Qc n
 3 x x  Qc
JA
 x 2 + 1 x  [ −1, 2]

D-10. f(g(x)) = 2x 2 + 1 x  (2, 3)
 2x + 5 x  [3, 5]

Section (E) :
E-1. (i) even, (ii) neither even nor odd (iii) even, (iv) neither even nor odd
LP

(v) even (vi) even


E-2. (i) neither even nor odd (ii) even (iii) odd (iv) even

E-4. (i) 2 (ii) 2 (iii) 24 (iv) 70  (v) 2 (vi) /6 (vii) 2


Section (F) :
A

F-1. (i) f–1 Does not exists (ii) f–1 : R → R ; f −1 = 7 + (4 − x5)1/3


−x
e −e
x
−1 + 4x − 3
(iii) f–1 : R → R ; f–1 = (iv) f–1(x) =
2 2
NK

1  − x x0 3− 5
F-2. f–1 : R → R, f–1(x) = n (x + x 2 + 1 ) F-3. (a) f–1(x) =  ,3 (b) x =
2 − x x0 2
F-4. 1 F-5. =2
Section (G) :
SA

 1 1
G-1. (i) [–1, 1] – {0} (ii)  − ,  (iii)  G-2. (i) (– , n /2](ii) (0, /2] (iii) [0, ]
 3 2
1   n + 1
G-3. (i) 1 (ii) (iii) G-4. (i) n  
3 6  2 

G-5. (i) [− 1, 0) (ii) (−  cot 3) U (cot 2, )


(iii) – sin 1 < x  1 (iv) cos2 < x  1 (v) no solution

1  −1  x − 2   
G–6. B = [0, 4] ; f –1 (x) =  sin  − 
2   2  6
Section (H) :
  3 
H-1. (i) – (ii) – (iii) (iv)
6 3 4 4
17
H-2. (i)  –4 (ii) 4 – 10 (iii) 2 – 6 (iv) 4 – 10 (v)
20
H-3. (i) 0 (ii) 1 H-4. x>1
Section (I) :
4 41 63 1+ 3 5 6−4 5 5
I-1. (i) (ii) 2 2 (iii) (iv) (v) (vi) (vii) 2 (viii) (ix)
5 4 16 8 15 3
1

RI
2 2

I-2. I-7. (i) 2tan–1x –  (ii)  – 2sin–1x (iii) 2 – 2cos–1x
2
1 + xy

A
I-8.
x−y
1 1 1
I-9 (i) ± (ii) x=3 (iii) ± (iv) x=
3 2 2

UH
Section (J) :
 
J-2. (i) tan–1 (x + n) − tan–1 x (ii) (iii)
4 2
JA
PART - II
Section (A) :
A-1. (C) A-2. (D) A-3. (C) A-4. (C) A-5. (D) A-6. (A)
Section (B) :
B-1. (A) B-2. (C) B-3. (A) B-4. (B) B-5. (B) B-6. (A)
LP

B-7. (B) B-8. (D) B-9. (D) B-10. (B) B-11. (D)
Section (C) :
C-1. (D) C-2. (A) C-3. (B) C-4. (B) C-5. (D) C-6. (A) C-7. (D)
C-8. (B) C-9. (C) C-10. (B) C-11. (A) C-12. (D) C-13. (A) 
Section (D) :
A

D-1. (A) D-2. (C) D-3. (B)


Section (E) :
E-1. (B) E-2. (B) E-3. (B) E-4. (C) E-5. (D) E-6. (C)
NK

Section (F) :
F-1. (A) F-2. (A) F-3. (A) F-4. (A) F-5. (A) F-6. (B)
Section (G) :
G-1. (A) G-2. (C) G-3. (D) G-4. (C) G-5. (D) G-6. (A) G-7. (C)  
G-8. (A)
SA

Section (H) :
H-1. (D) H-2. (B) H-3. (D) H-4. (D)
Section (I) :
I-1. (B) I-2. (C) I-3. (C) I-4. (A) I-5. (B) I-6. (B) I-7. (A)
Section (J) :
J-1. (B) J-2. (B) J-3. (C)
PART – III
1. (1) → (p), (2) → (r), (3) → (s), (4) → (q)

2. (A) → (q,r), (B) → (q,r), (C) → (q,s), (D) → (q,r),

3. (A→ q ; B→ r,q ; C→ s ; D→ t)

4. (A) → (q), (B) → (s), (C) → (p), (D) → (r),

5. (A) → (p), (B) → (p), (C) → (p), (D) → (s)

RI
EXERCISE - 2
PART - I

A
1. (A) 2. (A) 3. (C) 4. (D) 5. (D) 6. (B) 7. (C)
8. (D) 9. (D) 10. (D) 11. (A) 12. (C) 13. (C) 14. (D)

UH
15. (B) 16. (C) 17. (B) 18. (B) 19. (A) 20. (C) 21. (B)
22. (D) 23. (A) 24. (A) 25. (A) 26. (B) 27. (A) 28. (C)
29. (C) 30. (A)

PART - II
JA
1. 17 2. 17 3. 0 4. 1 5. 2 6. 1 7. 34
8. 7 9. 20 10. 35 11. 22 12. 2 13. 3 14. 1
15. 30 16. 5 17. 54 18. 3 19. 20 20. 1 21. 1

PART – III
LP

1. (ABD) 2. (AC) 3. (AC) 4. (AD) 5. (BCD) 6. (BD) 7. (AD)


8. (ABC) 9. (AC) 10. (ABD) 11. (ABD) 12. (ACD) 13. (AD) 14. (ACD)
15. (BCD) 16. (AC) 17. (BC) 18. (ABCD) 19. (BD) 20. (BCD) 21. (AB)
A

22. (AB) 23. (CD) 24. (BC) 25. (BCD) 26. (AD) 27. (AB) 28. (AD)
29. (AD) 30. (ABCD) 31. (CD)
NK

PART - IV
1. (A) 2. (C) 3. (D) 4. (A) 5. (C) 6. (B)
SA

EXERCISE - 3
PART - I

1. 7 2. 2 3. (A) 4. 1 5. (B) 6*. (AB) 7. (B)

8. (B) 9. (ABC) 10. 3 11. (BCD) 12. 2 13. 119 14. (A)

PART – II

1. (2) 2. (3) 3. (2) 4. (3) 5. (1) 6. (1) 7. (2)


8. (1) 9. (2) 10. (4) 11. (1) 12. (3) 13. (2) 14. (2)

15. (3)

SUBJECTIVE QUESTIONS
 Marked Questions may have for Revision Questions.

 2x −1 

RI
1. Find the domain of the function f(x) = − log x + 4  log 
2 3+ x 
 
2

A
2. Let f (x) = (x12 − x9 + x4 − x + 1)−1/2 . The domain of the function is :

UH
3. Find the values of ' a ' in the domain of the definition of the function , f (a) = 2a2 − a for which the

roots of the equation , x2 + (a + 1) x + (a − 1) = 0 lie between − 2 & 1 .


JA
1
4. The domain of the function f (x) = −1
is:
(x − 1 cos
) (2 x + 1) . tan 3 x
LP

5. Find domain of the following functions

(i) f(x) = log1/ 3 log4 ([x]2 − 5) , where [ . ] denotes greatest integer function.
A

1
(ii) f (x) = , where [x] denotes the greatest integer not greater than x.
[| x − 1|] + [| 12 − x |] − 11
NK

x2 + 2 x − 3
log
(iii) f(x) = ( x + 0.5 ) ( 0.5 + x) 4 x2 − 4 x − 3

5 −1 x 2 (7 x + 1) !
(iv) f (x) = − 3sin + , where [ . ] denotes greatest integer function.
 x − 1 x +1
 2 
SA

4 2
−1
(v) 3y + 2x = 24x

 1  1
6. The range of the function f (x) = sin −1  x 2 +  + cos−1  x 2 –  , where [ ] is the greatest integer
 2   2 
function, is:
1
7. Find the range of f(x) = – {x}, (where {.} represents fractional part of x)
2{–x}

x 2 + 1 – 3x
8. If f : R → R ; f(x) = then find the range of f(x).
x2 + 1 + x

9. If a function is defined as f(x) = logh(x) g(x) , where g(x) = |sinx| + sinx , h(x) = sinx + cosx , 0  x  

.Then find the doman of f(x).

RI
10. Find the domain and range of the following functions.

A
x
(i) f (x) = cos−1 log[x] , where [ . ] denotes the greatest integer function .
x

UH
(ii) f (x) = [
log1/ 2 log2 x 2 + 4 x + 5 ] where [ . ] denotes the greatest integer function

  x2 
JA
(iii) f (x) = sin−1  log 2    , where [ . ] denotes greatest integer function .
  2  

(iv) f (x) = log[x − 1] sin x , where [ ] denotes greatest integer function .


LP

1
(v) f(x) = tan–1 ( [x] + [−x] ) + 2− | x | + , (where [ ] denotes greatest integer function)
x2
A

sin2 x + 4 sin x + 5
11. If f(x) = , then range of f(x) is
2sin2 x + 8 sin x + 8
NK

12. Find range of the function f(x) = log 2 3x −  x +  x +  x    


 
(where [· ] is greatest integer function)
SA

1 x
13. If f(x) = 2
and g(x) = sin  x + 8   where {· } denotes fractional part function then
x +1 2
the find range of f(g(x))

x  (1 − 2[x])    [x]      


14. If the range of the function f(x) =   + cos   + sin  2  is 4 ,4  4,4  
 
4  2       

 2 + 1  
 4 , 2  , (where {.} and [.] represent fractional part and greatest integer part functions respectively),
 
then 2 + 2 + 2 + 2 is
 x 
15. The fundamental period of sin [x] + cos + cos [x], where [ . ] denotes the integral part of x, is.
4 2 3

 ( 22011 –1)  2010


16. Consider the function g(x) defined as g(x),  x − 1 = (x + 1)(x 2 + 1)(x 4 + 1)...........(x 2 + 1) – 1
 
(|x|  1). Then the value of g(2) is equal to

17. It is given that f(x) is a function defined on N, satisfying f(1) = 1 and for any x  N

RI
f(x + 5)  f(x) + 5 and f(x + 1)  f(x) + 1
If g(x) = f(x) + 1 – x, then g(2016) equals

A
18. Find the integral solutions to the equation [x] [y] = x + y. Show that all the non-integral solutions lie on
exactly two lines. Determine these lines. Here [ .] denotes greatest integer function.

UH
19. Let f(x) = Ax2 + Bx + C, where A, B, C are real numbers. Prove that if f(x) is an integer whenever x is
integer, then the numbers 2A, A + B and C are all integers. Conversely, prove that if the numbers 2A, A
+ B and C are all integer then f(x) is an integer whenever x is an integer.
JA
20. Suppose X and Y are two sets and f : X → Y is a function. For a subset A of X, define f(A) to be the
subset {f(a) : a  A} of Y. For a subset B of Y, define f –1(B) to be the subset {x  X : f(x)  B} of X. Then
prove the followings
(i) Statement "f–1(f(A)) = A for every A  X" is false
LP

(ii) Statement "f–1(f(A)) = A for every A  X if only if f(X) = Y" is false


(iii) Statement "f(f–1(B)) = B for every B  Y" is false
(iv) Statement "f(f–1(B)) = B for every B  Y if only if f(X) = Y" is true
A

 
 x2 − k 
NK

Let g : R → (0, /3] is defined by g(x) = cos –1 


 1 + x 2 
21. . Then find the possible values of ‘k’ for which g
 
is surjective. 
SA


22. Let 0 <  , ,  < are the solutions of the equations cosx = x, cos(sinx) = x and sin(cosx) = x
2
respectively, then show that  <   

23. Let f(x) = log2log3log4log5(sinx + a2). Find the set of values of a for which domain of f(x) is R.
 3   3 
 + , −
2
−
2 − −  , − 2    − 2
 
     
24. tan (tan ) =  
–1
, −  , sin–1 (sin ) =   , −  ,
 2 2  2 2
  3   3
− +  ,   − , 
 2 2  2 2
 − , −    0

cos (cos ) =  
–1
, 0
 2 −  ,     2

Based on the above results, prove each of the following :

RI
(i) cos–1 x = sin–1 1 − x2 if 0 < x < 1
(ii) sin–1 x = cos–1 1 − x 2 if 0 < x < 1
1 − x2

A
(iii) cos–1 x =  + tan–1 if –1 < x < 0
x
 
25. Express cot (cosec–1x) as an algebraic function of x.

UH
x 1– x 2
26. Express sin–1x in terms of (i) cos–1 1– x2 (ii) tan–1 (iii) cot–1
1– x 2 x
 x , x 1
If f(x) =  x2
JA
27. , 1  x  4 , then find f (x) .
-1


8 x , x4
x2
y  2
 x y 
28. sin–1  +  + cos–1  + – 2  equals to :
 4 9  2 2 3 2 

1 + x   1 − x2 
If  = 2 tan–1   &  = sin–1   for 0 < x < 1, then prove that  +  = . What the value of
LP

29.
1 − x 
2
1 + x 
 +  will be if x > 1 ?

30. Solve {cos–1 x} + [tan–1 x] = 0 for real values of x. Where { . } and [ . ] are fractional part and greatest
A

integer functions respectively.


 
31. Find the set of all real values of x satisfying the inequality sec –1x > tan–1x.
 
NK

x x −1 1
32. Find the solution of sin−1 − sin−1 = sin−1 .
1+ x x +1 1+ x

33. (i) Find all positive integral solutions of the equation, tan −1 x + cot−1 y = tan−1 3.
SA

(ii) If 'k' be a positive integer, then show that the equation:


tan−1 x + tan−1 y = tan−1 k has no non−zero integral solution.

34. Determine the integral values of 'k' for which the system, (tan –1 x)2 + (cos–1 y)2 = 2 k and

tan–1 x + cos–1 y = possess solution and find all the solutions.
2
 1  1    
1. (– 4, –3)  (4, ) 2. (−  , ) 3. a  − , 0    , 1 4. − 6 , 0
 2  2   

5. (i) [–3, –2) U [3, 4) (ii) R – {(0, 1)  {1, 2,......., 12}  (12, 13)}
 1 1  1  3  n 
(iii)  − ,    , 1   ,   (iv)  7 , n  , − 1  n  6 
 2 2 2  2   
 – 3 – 1 – 3 + 1  3 – 1 3 + 1
(v). 

,  , 
 2 2   2 2 

RI
  
6.  7. [ 2 – 1, ) 8. (– 1, ) 9. 6, 2 
 
10. (i) D : [2 , ) ; R : {/2} (ii) D: ( −2 − 2, −3  U ,  −1, −2 + 2 ; R {0}]
)
 

A
  
(iii) D: (− 8 , − 1  1 , 8 ; R :  − , 0 ,  ]
)
 

UH
 2 2

(iv) D : [3 , )  (2n  , 2n  + ) ; R : (−  , 0] }
n =1

1 
(v) D : {–2, –1, 1, 2} ; R :  , 2 
JA
4 
5   1 
11.  9 , 1 12. {0, 1} 13.  65 , 1 14. 15 15. 24
   

16. 2 17. 1 18. Integral solution (0, 0); (2, 2). x + y = 6, x + y = 0


LP

1
21. k= – 23. a (−, − 626)  ( 626, ) 25. cot (cosec–1x) =
2
– x2 – 1 if x  –1


 x 2 – 1 if x  1
A

 – cos–1 1– x 2 , if – 1  x  0
 x
26. (i) sin–1x =  (ii) sin–1x = tan–1 , for all x  (–1, 1)
NK

 cos–1 1– x 2
 if 0  x  1 1– x 2
 1– x 2 
cot –1 – if – 1  x  0  x , x 1
 x 

(iii) sin–1x =  27. f–1(x) =  x , 1  x  16
  x2
SA

2 if 0  x  1 
 –1 1– x , x  16
 cot x  64
3
28. 29. – 30. {1, cos 1} 31. {x : x  (–, –1]} 32. x0
2
 
33. (i) Two solutions (1, 2) (2, 7) 34. k = 1, x = tan (1 – 7 ) , y = cos ( 7 + 1)
4 4
STATISTICS
Introduction:
Data means information or a set of given facts. The data is usually collected through census or
surveys. Survey is the process of collecting information from a selected group of persons. Data
collected by this process is called the raw data. The raw data is classified into two types; the
primary data and the secondary data. Primary data is reliable and the secondary data may or

RI
may not be reliable. Classification of data is essential for analyzing it. Statistics is defined as the
collection, presentation, analysis and interpretation of numerical (statistical) data.

Variable (or variate)

A
The value of each item in a data has certain characteristics. The characteristics like intelligence,
beauty etc are non-measurable but vary from person to person or from item to item. We call such

UH
quantities as qualitative variables. The characteristics like height, weight, marks etc are
measurable and these variables are quantitative variables. We also make a distinction between
the observed value and the possible value of a variable. For example, if we have a variable
defined by the sex of a person, the possible values of the variable are male and female. But, if we
are considering the number of males in the age group 20 − 25 in a village, we are considering the
JA
observed values of the variable defined by male.

• A variable (or variate) which is not capable of assuming all values in a given range is
called a discrete variable.
• A variable which is capable of assuming all the numerical values in a given range is
LP

called a continuous variable.

Example
S.N. Individual (item) Characteristic Type of characteristic
1 A student Height in cms Continuous variable
A

Weight in kgs Continuous variable


Colour of skin Attribute (qualitative)
NK

Age Continuous variable


Sex Attribute (qualitative)
Mother tongue Attribute (qualitative)
Marks in English Discrete variable
2 A bolt Diameter in cms Continuous variable
SA

Defective or not Attribute (qualitative)


3 A family Number of members Discrete variable
Monthly income in rupees Discrete variable

Frequency Distribution:
Let the data regarding the weights (in kgs) of 20 students of a class be given as

50 48 54 49 60 54 61 55 48 49
55 60 50 48 57 62 49 50 52 54
This is called the raw data. This is also called an individual series. We note that some of the
weights (values of the quantitative variable) are repeated. If there are 3 students having weight 50
kg, then we say the frequency of 50 is 3. Therefore, the number of times the value of the item is
repeated is called the frequency of that value. The table containing the weights and the
corresponding frequencies is given as

Weight (in kg) Tally by bars No. of students (frequency)


50 ||| 3

RI
48 ||| 3
54 ||| 3
49 ||| 3
60 || 2

A
61 | 1
55 || 2
57 | 1

UH
62 | 1

Tally bars are used to count the number of times the values of the variable has occurred. Also
denote that the value is repeated 5 times. The table containing the values and its frequencies is
JA
called a frequency distribution. The variable is denoted by x and the frequency by f. In the order
of magnitude, the frequency distribution is written as follows;

Weight (in kg) No. of students


x f
LP

48 3
49 3
50 3
52 1
54 3
A

55 2
57 1
NK

60 2
61 1
62 1
Total 20
SA

We denote the total number of students, that is the total frequency by n i.e. n =  f. Also we
denote different values of the variables x as x i and different frequencies by fi.

Let the data be classified according to different classes of values of the variable. This is an
important tool in condensing a large data. In the above example, the classes may be defined as;
45 and under 50, 50 and under 55, 55 and under 60 etc.
We denote these classes by 45 − 49, 50 − 54, 55 −59 etc. Usually the length of the class is taken
as same. With the length of the class as 5, the above frequency distribution can be displayed as
Weight (in kg) class No. of students
x F
45− 49 6
50 − 54 7
55 − 59 3
60 − 64 4
f = 20 = n

RI
In the above frequency table 45 − 49, 50 − 54 are called class intervals. 45 − 49 is one of the
class intervals in which 45 is lower class limit and 49 is upper class limit.
The classes are written in two forms.

A
(i) Inclusive form: In this case, the lower limit of a class is not equal to the upper limit of the
previous class. For example: 45 − 49, 50 − 54, 55− 59, 60 − 64 are in inclusive form.
However, in the class 45 − 49, all items with values greater or equal to 44.5 but less than 49.5 are

UH
to be taken. Thus actual limits are 44.5 − 49.5, 49.5−54.5, 54.5 − 59.5. 59.5 − 64.5.

(ii) Exclusive form: In this case, the lower limit of a class is equal to the upper limit of the
previous class. For example− we may have classes of the form 45 − 50, 50 − 55, 55 − 60, 60 − 65
JA
etc. The value 50 is counted in the class 50 and under 55 and not in 45 and under 50.

In both the forms, the length of classes (upper limit − lower limit) is same.

RELATIVE AND CUMULATIVE FREQUENCY


Relative Frequency:
LP

The frequency of any class in a frequency table is the number of units of observations for which
the values of the variable belong to that class. Sometimes it is useful to express the frequency as
a fraction of total frequency, usually as a percentage. This fraction expressed as a percentage is
called relative frequency of the class.
A

The relative frequency gives useful information about the data, particularly when the calss
frequencies are large and total frequency is very large.
 class frequency  100 
NK

Relative frequency =  % .
 Total frequency 

Cumulative Frequency of a value (or class of values) is obtained by adding all the frequencies of
all values (or classes of values) less than or equal to that under consideration. Cumulative
SA

frequency is an important concept and is useful is determining the measures of location.

Illustration 1: Represent the following data in the form of frequency distribution


Height (in cms) 90 80 70 60 50 40 30
(above)
No. of plants 2 5 12 22 30 48 52

Solution: From the data we find the number of plants with heights
between 30 & 40 cms = 52 − 48 = 4
between 40 & 50 cms = 48 − 30 = 18
between 80 & 90 cms = 5 − 2 = 3
above 90 cms = 2.
Hence, frequency distribution is

Class (cms) No. of Plants (frequency)


30 − 40 4
40 − 50 18
50 − 60

RI
12
60 − 70 10
70 − 80 7
80 − 90 3

A
90 − 100 2

Exercise 1

UH
(i) The following data gives the number of children in 30 families in a village
2, 3, 0, 1, 2, 4, 3, 0, 1, 2, 1, 3, 0, 2, 2, 3, 1, 1, 1, 0, 2, 4, 1, 2, 3, 2, 1, 2, 2, 1.
Represent the data in the form of a frequency distribution.
(ii) Following are the ages of 360 patients getting medical treatment in a hospital on a
JA
day:
Age (in years) 10 − 20 20 − 30 30 − 40 40 − 50 50 − 60 60 − 70
No. of patients 90 50 60 80 50 30
Construct the cumulative frequency distribution table.
(iii) Following are marks of 75 students in a test:
LP

Marks (upto) 30 40 50 60 70 80
No. of students 0 12 30 47 60 75
Form a frequency distribution from the data. How many students are recurring
more than 60 marks?
A

Measure of Location:
One of the most important objectives of statistical analysis is to get one single value that
NK

describes the characteristics of entire mass of unwieldy data. Such a value is called central value
or the average. It is a single value, which represents a group of values.

TYPES OF AVERAGES
SA

(a) Mean
(i) Arithmetic Mean (ii) Weighted arithmetic mean
(iii) Geometric Mean (iv) Weighted Geometric Mean
(v) Harmonic Mean (vi) Weighted Harmonic Mean
(b) Median
(c) Mode
THE ARITHMETIC MEAN:
The arithmetic mean of a statistical data is defined as the ratio of the sum of all the values of the
variable and the total number of items. It is denoted by A.M.

Calculation of Arithmetic Mean:


(i) For an individual series
(a) Let x1, x2, …, xn be a set of n observed values of a statistical data. We denote the
arithmetic mean or simply the mean by x . Therefore, for this individual data, the arithmetic mean
is defined as

RI
n

n x i

x
1 1
x =  x1 + x 2 + ... + xn  = i = i=1
.
n n n

A
i=1

(b). If the numbers xi, i = 1, 2, …, n are very large, then we can shift the origin to a point so

UH
that the new numbers yi are smaller compared to xi. So it is easier to compute y . This method is
based on the fact that if each observation of the data be changed by an amount a, then its mean
is also changed by the same amount a. This can be easily proved as below:
Let x1, x2, …, xn be the observations. Then new observations are defined by
y1 = x1 + a, y2 = x2 + a, …, yn = xn + a
JA
The mean y of new data is
1
y= ( x1 + a ) + ( x 2 + a ) + ... + ( xn + a ) 
n
1
= ( x1 + x 2 + .... + xn ) + ( a + a + ... + a ) 
n
LP

1
= nx + na
n
= x+a .
Thus, when the observations x i, i = 1, 2, …, n are very large then the arithmetic mean is
A

calculated as follows:

A.M. = A +
 di
where d= xi − A, i = 1, 2, …, n.
n
NK

A is the assumed mean.

(ii) For a Frequency Distribution


(a) Let us consider a frequency distribution. Let x i be the values of the variable and f i be the
corresponding frequencies that is, the grouped data is (x i, fi), i = 1, 2, …, n. If the values of the
SA

variables are given as intervals or classes are taken as x i, then, the arithmetic mean of the
frequency distribution is defined as
x f + x 2 f2 + ... + xn fn
x = 11 =

xi fi
f1 + f2 + ... + fn fi
(b) Short − cut Method
If each value of the variable x i is very large then each xi is shifted by a, i.e. y i = xi − a. The new
frequency distribution is given by (y i, fi). The mean of this frequency distribution is
y=
y fi i  ( x − a)f .
or x − a =
i i

f i f i

Hence, x = a +
d f i i

f i

where di = xi − a, a is assumed mean.

(c) Step Deviation Method

RI
If the class intervals of equal length are defined in the frequency distribution, then the
computations can be simplified further.
x −a
In this case, define, di = i
h

A
or, xi = a + hdi
where h is the length of the class intervals and a is the assumed mean.
Then, xifi = (a + hdi) fi

UH
= a fi + h difi.

Hence, x =
 xi fi
=
1 
a fi + h di fi 
 f
i  f 
i

 d f  .
JA

= a + h
i i

 f  i

 d f 
Thus x = a + h   i i
 f 
  i
LP

where a = assumed mean,


h = length of class interval
fi = frequency of each variable
x −a
di = i .
A

Illustration 2: A group of 10 items has mean 6. If the mean of 4 of these items is 7.5, then find
NK

the mean of remaining items.

Solution: Sum of all the 10 items = 10  6 = 60


Sum of four of these items 4  7.5 = 30
 sum of the remaining six items = 60 –30 = 30.
SA

30
Hence, the mean of remaining six items = = 5.
6

1 1 1
Illustration 3: If the values 1, , , ...., occur at frequencies 1, 2, 3, 4, 5,…., n in a
2 3 n
distribution, then find the mean.

1 1 1 1 1
1 1 +  2 +  3 +  4 +  5 + ...... +  n
Solution: Mean = 2 3 4 5 n
1 + 2 + 3 + ........ + n
2 (1 + 1 + 1 + ....... + 1) 2
= = .
n ( n + 1) n +1

Illustration 4: The salaries of teachers in a school are given below:

Salary (Rs) 1800 2200 2800 3200 4500 6000


No. of teachers 5 6 7 5 3 4
Find the average salary of the teachers.

RI
Solution: Let a = 3000 be the shift in the salaries. Then we have the following distribution
Salary No. of di = xi−3000 fidi

A
xi teachers
fi
1800 5 −1200 −6000

UH
2200 6 −800 −4800
2800 7 −200 −1400
3200 5 200 1000
4500 3 1500 4500
6000 4 3000 12000
JA
fi = 30 fidi = 5300

Hence the average salary of the teachers is x = a +


fdi i

f i
LP

5300
= 3000 +  3176.67 Rs.
30

Illustration 5: A factory manufactures nuts and bolts of various sizes. The measurement of
inner diameters of 1000 nuts gave the following frequency table:
A

Diameter (mm) 43 − 45 46 − 48 49 − 51 52 − 54 55 − 57
No. of nuts 175 236 200 196 193
NK

Determine the mean inner diameter per nut.

Solution: The given table is in inclusive form of frequency distribution. We first convert it
SA

into an exclusive form and write it in the form.

Diameter (mm) 42.5 − 45.5 − 48.5 − 51.5 − 54.5−


45.5 48.5 51.5 54.5 57.5
No. of nuts 175 236 200 196 193

The width of the class interval is h = 3.


Let the shift (assumed mean) be a = 50.
xi − a
Using di = , we construct the following frequency distribution:
h
Diameter Mid value Frequency xi − 50 fi di
di =
(mm) (xi) (fi) h
42.5 − 45.5 44 175 −2 −350
45.5 − 48.5 47 236 −1 −236
48.5 − 51.5 50 200 0 0
51.5 − 54.5 53 196 1 196
54.5− 57.5 56 193 2 286
fi = 1000 fidi = −4

RI
Thus the mean inner diameter is

x =a+
fi di
h = 50 −
( 4 )( 3 )
= 49.998 mm.
 1000

A
fi

Exercise 2

UH
(i) In a factory, the workers each of age 20 years or more are grouped as follows
Age below (in years) 25 30 35 40 45 50 55 60
No. of workers 32 60 190 236 288 350 380 400
Find the average age of the workers.
JA
(ii) A college sends the results of their entrance examination by post. The following
distribution of amount spent and the number of letters dispatched was given:

Amount (Rs) No. of letters


2.00 2000
10.50 1500
LP

15.00 500
30.00 300
Find the mean cost of postage per student.
(iii) The marks obtained by 20 students of a class in a test are 72, 48, 54, 65, 68, 82, 85,
A

47, 40, 76, 74, 73, 53, 56, 67, 65, 49, 52, 61, 36. Find the average marks of class.
(iv). If mean of n observations x1, x2, …, xn be x , then find the mean of n observations
2x1 + 3, 2x2 + 3, 2x3 + 3, …, 2xn + 3.
NK

(v) The mean of a set of observations is x . If each observation is divided by ,   0


and then is increased by 10, then find the mean of the new set.

Weighted Arithmetic Mean


SA

If w1, w2, w3, …, wn are the weights assigned to the values x 1, x2, x3, …, xn respectively, then the
weighted average is defined as:
w 1x1 + w 2 x 2 +  + w n x n
Weighted Arithmetic Mean = .
w1 + w 2 +  + w n

Illustration 6: A school runs in two shifts. The morning shift is from 7:30 am to 12:30 pm. The
afternoon shift is from 1:00 pm to 6:00 pm. There are a total of 60 teachers
working in either of the shifts and their average salary is Rs 3000/−. There are no
teachers working in both the shifts. The average salary of 36 teachers working in
the morning shift is Rs 3200/−. Find the average in the afternoon shift.
Solution: We have:
Total teachers = 60
No. of teachers in morning shift = n1 = 36
No. of teachers in the afternoon shift = n2 = 60 − 36 = 24
Average salary of all teachers = x = 3000
Average salary of teachers in the morning shift = x1 = 3200
Let average salary of teachers in the afternoon shift be x 2 .
n1x1 + n2 x2
We have x =

RI
n1 + n2
36 ( 3200 ) + 24 ( x 2 )
or 3000 =
36 + 24
1

A
 x2 = (180000 − 115200 ) = 2700 .
24

UH
Geometric Mean
If x1, x2, …, xn are n values of a variable x, none of them being zero, then the geometric mean G
is defined as G = (x1x2x3 …. xn)1/n.

Geometric mean for frequency distribution:


JA
Geometric mean of n values x 1, x2, x3, …, xn of a variable x, occurring with frequency f 1, f2, f3, …,
fn respectively is given by
1/ n
f
( f
G = x11 x 22 ....xnfn )
LP

 n 


 f log x 
i=1
i i
or G = antilog  .
n
 
 
 
A

Harmonic Mean
NK

The harmonic mean of n items x1, x2, x3,…, xn is defined as:


n
Harmonic Mean =
1 1 1 1
+ + ++
x1 x 2 x3 xn
SA

Harmonic Mean of Frequency Distribution:

Let x1, x2, x3, …, xn be n items which occur with frequencies f 1, f2, f3, …, fn respectively. Then their
Harmonic Mean is given by:

Harmonic Mean =
f1 + f 2 + f 3 +  + fn
=
 fi .
f1 f2 f3 fn 1
+
x1 x 2 x 3
+ ++
xn
 fi  x
i
Relation between Arithmetic Mean, Geometric Mean and Harmonic Mean:

The arithmetic mean (A. M.), Geometric mean (G.M.) and Harmonic Mean (H.M.) for a given set
of observations of a series are related as under:
A. M  G.M  H.M

Illustration 7: Find the geometric mean of number 7, 72, 73…… 7n.

Solution: G.M. = (7. 72. 73………7n)1/n

RI
n( n +1) n +1
=7 2n
=7 2
.

Illustration 8: Find the harmonic mean of 4, 8, 16.

A
3
Solution: H.M. of 4, 8, 16 = = 6.85.
1 1 1

UH
+ +
4 8 16

Exercise 3
(i) A boy goes to school from his home at a speed of x km/ hr and comes back at a
JA
speed of y km/ h then find the average speed of the boy.
(ii) If the values 2, 8, 16, 128, 512 are given, then find the geometric mean.
(iii) Find the harmonic mean of the following distribution:
Class 4.5−5.5 5.5−6.5 6.5−7.5 7.5−8.5 8.5−9.5
Frequency 8 10 18 6 4
LP

Median:

Median is defined as the middle most or the central value of the variables in a set of
observations, when the observations are arranged either in ascending or in descending order of
A

their magnitudes. It divides the arranged series in two equal parts. Median is a position average,
whereas, the arithmetic mean is the calculated average. When a series consists of an even
NK

number of terms, median is the arithmetic mean of the two central items. It is generally denoted
by M.

Case I: When n is odd.


SA

n +1 n +1
In this case th value is the median i.e. M = th term.
2 2
Case II: When n is even.

n n 
In this case there are two middle terms th and  + 1 th . The median is the average
2  2 
n n 
+  + 1
2 2 
of these two terms, i.e. M = th term
2
Case III: When the series is continuous.

In this case the data is given in the form of a frequency table with class-interval, etc., we
prepare the cumulative frequency table and determine the median class i.e. the class in
th
n
which the   observation lies and the following formula is used to calculate the
2
Median:
n
−C

RI
M=L+ 2  i , where
f
L = lower limit of the class in which the median lies
n = total number of frequencies, i.e., n = f.

A
f = frequency of the class in which the median lies
C = cumulative frequency of the class preceding the median class
i = width of the class-interval of the class in which the median lies.

UH
Illustration 9: The marks obtained by 9 students in a class are 70, 47, 52, 66, 73, 61, 55, 59,
68. Find the median marks of the students.
JA
Solution: We first arrange the marks in ascending order as
47, 52, 55, 59, 61, 66, 68, 70, 73.
th
 9 + 1
Since n = 9, the median is the value of   item = 5th item.
 2 
Hence median is 61.
LP

Illustration 10: Find the median of the following frequency distribution

x 8 5 6 10 9 4 7
f 6 4 5 8 9 6 4
A

Solution: We note that the values of x are not given in ascending order. Hence, we first
arrange the values of x in ascending order and then form the cumulative
NK

frequency table. We have the following table


x f Cumulative
frequency
4 6 6
SA

5 4 10
6 5 15
7 4 19
8 6 25
9 9 34
10 8 42

The total frequency n = 42 is even. We have


n 42 n
= = 21 and + 1 = 22 .
2 2 2
The values of the 21st and 22nd items are 8, 8 since the values of the items from
1
20 to 25 are 8 each. Therefore median = ( 8 + 8 ) = 8.
2

Illustration 11: The wage distribution for the workers in a certain factory is given below:

Monthly wages (in No. of workers


rupees)
1500−1700

RI
35
1700−1900 62
1900−2100 55
2100−2300 72

A
2300− 2500 142
2500−2700 85
2700−2900 50

UH
Find the median of the wage distribution.

Solution: We have the following commutative frequency table:

Monthly wages ( in
JA
No. of workers Cumulative
rupees) frequency
1500−1700 35 35
1700−1900 62 97
1900−2100 55 152
2100−2300 72 224
LP

2300− 2500 142 366


2500−2700 85 451
2700−2900 50 501 = n
th
n
Therefore, median = size of   item
A

2
= size of 250.5 th item.
Hence median class is 2300 − 2500.
NK

We also have L = 2300, i = 200, n = 501, C = 224, f = 142


n
−C
200
 median = L + 2  i = 2300 + ( 250.5 − 224 ) = 2337.32 rupees.
f 142
SA

Mode:
Mode is defined as that value in a series which occurs most frequently. In a frequency distribution
mode is that variate which has maximum frequency. This measure is used when it is important to
know which values occurs most frequently.

Continuous Frequency Distribution:


i) Modal Class: It is that class in grouped frequency distribution in which the mode lies.
fm − f1
Mode = L +  i , where
2fm − f1 − f2
L = the lower limit of the modal class
i = the width of the modal class
f1 = the frequency of the class preceding modal class
fm = the frequency of the modal class
f2 = the frequency of the class succeeding modal class.

Sometimes it so happens that the above formula fails to give the mode. In this case, the modal
value lies in a class other than the one containing maximum frequency. In such cases we take

RI
the help of the following formula:
f2
Mode = L +  i , where L, f1, f2, i have usual meanings.
f1 + f2

A
Illustration 12: Find the mode of the following items 0, 1, 6, 7, 2, 3, 7, 6, 6,2, 6, 0, 5, 6, 0.

UH
Solution: Maximum frequency = 5 of number 6
 Mode = 6

Illustration 13: Find the mode of following data


JA
Marks 1−10 11−20 21−30 21−40 41−50
No. of 8 15 28 16 8
students

Solution: We construct the following table:


LP

Class 1−10 11−20 21−30 21−40 41−50


interval
Modified 0.5−10.5 10.5−20.5 20.5−30.5 30.5−40.5 40.5−50.5
A

class
intervals
NK

No. of 8 15 28 16 8
students
Clearly, modal class is 20.5 − 30.5 as the maximum frequency occurs in this
class.
fm − f1
 Mode = l + i
SA

2fm − f1 − f2
28 − 15
= 20.5 +  10 = 25.7 .
2  28 − 15 − 16
Symmetrical Distribution:
A distribution in which mean, median and mode coincide is called symmetrical distribution.

Relation between Mean, Median and Mode:

Symmetrical distribution:
A distribution in which same number of frequencies is found to be distributed at the same linear
distance on either side of the mode. In this case, mean, median and mode coincide.
Thus, Mean = Median = Mode.

RI
A = M = M0

A
Asymmetrical distribution:
In this distribution, variations do not have symmetry. If the distribution is moderately asymmetrical
then mean, median and mode are connected by the formula

UH
Mode = 3 Median − 2 Mean.

Illustration 14: In a moderately skewed distribution the values of mean and median are 5 and 6
respectively. Find the value of mode in such a situation.
Solution: Here mode = 3 median – 2 mean
JA
= 3(6) – 2(5) = 8.

Exercise 4
(i) Find the median of the data 13, 14, 16, 18, 20, 22.
(ii) The following data gives the distribution of heights of students
LP

Height (in 160 150 152 161 156 154 155


cms)
No. of 12 8 4 4 3 3 7
students
A

Calculate the mean of the distribution.


(iii) If the mode of a data is 18 and the mean is 24, then find median.
(iv). The marks obtained by 60 students in certain test are given below:
NK

Marks No. of students


10−20 2
20−30 3
30−40 4
40−50 5
SA

50−60 6
60−70 12
70−80 14
80−90 10
90−100 4
Calculate median and mode.
(v) Find the mode of following frequency distribution:
x 40 43 46 49 52 55
f 5 8 16 9 7 3
Measure of Dispersion:
We have defined an average (mean, median) as a measure of central tendency. However, it does
not show as to how the variates are scattered about the central value. It is possible that two
distributions may have the same average (or the average may be very close) but they may differ
widely in the scatter of their values.
For example− consider the scores of two cricket players in six innings. Let the scores of the two
players be as follows:
Player A 80 5 6 0 90 20

RI
Player B 35 31 25 38 42 30

Total score of both the players is 201. Therefore arithmetic mean of two players is same but A
shows large variation and B does not show much variation from inning to inning. So, players B is

A
more consistent. The scores of B are bunched together while the scores of A are scattered. This
property is called dispersion.

UH
Dispersion is defined as scatter or spread of the observed valued of a quantitative variable from a
central value.
Normally, the following measures of dispersion are used:
(a) Range
JA
(b) Mean Deviation
(c) Standard Deviation

(a) Range:
It is the simplest form of measuring the variation. The range of a set of values is the difference
between the largest and the smallest values in the set.
LP

For example − range of the values 2, 4, 10, 20, 15, 21, 16, 3 is 21 − 2 = 19.
Range gives very limited information. It tells the difference between the extreme values but
nothing about the variations between other values
A

(b) Mean Deviation:


The mean deviation is defined as the arithmetic mean of the absolute values of the deviations of
NK

the observed values from mean or median.

Method for Calculation of Mean Deviation

Case−I : For ungrouped data


SA

Let x1, x2, x3,… , xn be n observations. Then


n

x −x
1
Mean deviation from mean = i
n i=1
where x = mean value of given observations.
n = total number of observations or items.
n

 x −M
1
Mean deviation from median = i
n i=1
where M = median of the given observations.
Case−2: For grouped data
n
Let x1, x2, x3,…, xn occur with frequencies f1, f2, f3, …, fn respectively and let f = N.
i=1
i

f x − x
i=1
i i
Then Mean deviation from mean = n

f
i=1
i

RI
where x = mean.
n

f x −M
i=1
i i
Mean deviation from median =

A
n

f
i=1
i

where M = median.

UH
Illustration 15: Find the mean deviation from median for the following distribution
xi 15 20 25 30 35 40 45 50
fi 6 4 7 6 5 9 5 8
JA
Solution: We form the cumulative frequency table as

xi fi Cumulative di fidi
frequency =|xi−M|
LP

15 6 6 20 120
20 4 10 15 60
25 7 17 10 70
30 6 23 5 30
35 5 28 0 0
A

40 9 37 5 45
45 5 42 10 50
NK

50 8 50 15 120
495
N N
We have N = 50  = 25  + 1 = 26.
2 2
1
Hence, median = (
value of 25 th term + value of 26 th term )
SA

2
1
= ( 35 + 35 ) = 35 .
2

 Mean deviation from median =


fd i i
=
1 99
 495 = = 9.9 .
N 50 10

In case of frequency distribution when the values of the variable are given in terms of classes,
then their mid-values are taken as the values x i of the variable.
Median is used in calculating the mean deviation, because of the property that the sum of
absolute values of the deviations of the observed values from median is always the least.
This indicates that the amount of dispersion of the observed values about the median is
minimum.

Illustration 16: The scores of a cricketer in 7 innings are given as follows:


67, 56, 38, 45, 52, 58, 69.
Find the mean deviation from median.

RI
Solution: We first arrange the sores in ascending order
38, 45, 52, 56, 58, 67, 69.
(n + 1)

A
Since = 4, the 4th item is the median i.e. M = 56.
2
 Mean deviation from median
1

UH
= [|x1 − M| + |x2 − M| + …. + |xn − M|.
7
1
= [|38 − 56| + |45 − 56| + …. + |69 − 56|.
7
1
= [18 + 11 + 4 + 0 + 2 + 11 + 13]
JA
7
59
=  8.43.
7

Standard Deviation:
Standard deviation of a given set of observations is defined as the positive square root of the
LP

average of squared deviations of all observations taken from their arithmetic mean. It is generally
denoted by Greek alphabet  or s.
Variance
The square of the standard deviation is called variance and is denoted by 2.
A

In computing the mean deviation, the signs of the deviations are ignored. Thus it is inconvient for
further mathematical treatment. So, standard deviation and variance are used which are more
NK

convenient for further mathematical treatment and are based on all the values of the data.

Method of Calculating Standard Deviation:


(a) For ungrouped data
Direct Method:
SA

Let us consider n observations x1, x2, …, xn. Let the arithmetic mean of these
observations be x . Then standard deviation is given by
1
= 

( x1 − x )2 + ( x2 − x )2 + ... + ( xn − x )2 
n
n

(x − x)
1 2
= i .
n i=1

For computational purposes we simplify the above equation so that the number of arithmetic
operations to be carried out is reduced.
n

(x − x)
1 2
We have  = i
n i=1
n

(x
1
=
n
i
2
− 2xi x + x 2 )
i=1

x  x + ( x ) n 1
1 1 2 1
= i
2
− ( 2x ) i
n n

x x
1 2 2 1 2
= 2
− 2(x) + (x) = 2
− (x)

RI
i i
n n
 
( x )
2
1 
x
i
= 2
− .
n
i
n

A
 

UH
Short Cut Method:
This method is applied to calculate standard deviation, when the mean of the data comes out to
be a fraction. In that case it is very difficult and tedious to find the deviations of all observations
from the mean by earlier method.
In this case, we shift xi by A (assumed mean) i.e. define di = xi − A and then find the standard
JA
deviation.
We have d = x − A .
Hence,

 (
1 1 2
 = ( x i − x )2 = di + A − d − A )
n n
LP

 ( d − d)
1 2
= i
n

d  d + n d 1
1 2 1
= i
2
− d i
2
n n
A

d d + d
1 2
= i
2
− d i
2
n n
2
 di   di 
NK

   
1 di
= di − 2 
2
n   n 
+
n  n

( d ) d 
2 2
2  di 
d
1 i
− 
i
= 2
− =
n
i
n n  n 
 
SA

d 
2
2  di 
− 
i
=
n  n 
 
where di = xi − A,
A = assumed mean,
n = total number of observation.

(b) For grouped data


If a variate x takes values x 1, x2, …, xn with respective frequencies fi, f2, …, fn then standard
deviation is given by
n n

 f x
2
fi ( xi − x ) i i
= i=1
n
where x = i=1
n
.
f
i=1
i f
i=1
i

If class intervals are given, then mid values of class intervals give the values of variate x.
But when the mean has a fractional value, then the following formula is applied to calculate

RI
standard deviation
2
n  n 
 2
fi di 

 fi di 
2

A
= i=1
n
− i=1
n 
 fi 


 fi 

UH
i=1 i =1

where di = xi − A, A assumed mean.

Combined Standard Deviation:


Let 1 and 2 be the standard deviations of the two groups containing n 1 and n2 items
JA
respectively. Let x1 and x2 be their respective A.M. Let x and  be the A.M. and S.D. of the
combined group respectively. Then
n x + n2 x2
x= 1 1 ,
n1 + n2
n112 + n2 22 + n1d12 + n2 d22
=
LP

n1 + n2
where d1 = x1 − x and d2 = x2 − x .

Coefficient of Variation:
A

For comparing two or more series for variability, we calculate the coefficient of standard deviation
and the coefficient of variation.

NK

The coefficient of standard deviation is defined as: coefficient of standard deviation = .


x

The coefficient of variation is defined as: coefficient of variation =    100 .
x
Coefficient of variation gives us a measure of scattering (dispersion). Scattering is less if the
SA

coefficient of variation is small.

Illustration 17: Find the variance of first n natural numbers.

Solution: The variance is given by


 
( )
2
1 xi 
V =  =  xi2 −
2
n  n 
 

where xi, i = 1, 2, …, n are the first n natural numbers.


n ( n + 1)
Now, xi = 1 + 2 + 3 + … + n =
2
n ( n + 1)( 2n + 1)
xi2 = 12 + 22 + … + n2 =
6
11 
2
n2 (n + 1)
Thus, V =  n ( n + 1)( 2n + 1) − 
n 6 4n 
 
n ( n + 1)
=  4 ( 2n + 1) − 6 (n + 1) 
24n 

RI
(n + 1) n2 − 1
= ( 2n − 2) = .
24 12

A
Illustration 18: Find the standard deviation of 7 scores 1,2,3,4,5, 6,7.

UH
n2 − 1
Solution: Standard deviation of first n natural numbers is .
12
72 − 1
For n = 7, the value = = 4 = 2.
12
JA
Illustration 19: Find the mean and standard deviation for the following data

Age (years) 25−30 30−35 35−40 40−45 45−50 50−55


No. of 30 23 20 14 10 3
teachers
LP

Solution: We have a grouped data. The distance between two successive mid-values of
the classes is 5, that is h = 5. We choose
x − a xi − 42.5
a = 42.5 and ui = i = .
h 5
A

Class Mid value fi ui fiui ui2 fiui2


(xi)
NK

25−30 27.5 30 −3 −90 9 270


30−35 32.5 23 −2 −46 4 92
35−40 37.5 20 −1 −20 1 20
40−45 42.5 14 0 0 0 0
45−50 47.5 10 1 10 1 10
SA

50−55 52.5 3 2 6 4 12
100 −140 404

 x = a + h
 f u  = 42.5 + 5  − 140  = 35.5
i i

  f  i  100 
 

2
2
h  ( fu ) 

 f 
i i
V= 2 = fu − 2

 f 
i i
i i
25  ( −140 )  = 52.
2
=  404 −
100  100 
 
S.d. =  = 52  7.21.

Illustration 20: The scores of 25 students in an intelligent test are given below: 75, 56, 50, 62,
68, 62, 56, 78, 80, 75, 50, 62, 72, 78, 68, 67, 80, 75, 50, 68, 80, 68, 62, 56, 68.
Find the mean and standard deviation of the data.

RI
Solution: We note that the marks are repeated.
Hence, the data forms a frequency distribution. Since the numbers are large
varying from 50 to 80, we choose the shift value as a = 65.

A
We have following frequency distribution.

Salary Frequency di = xi−a fidi di2 fidi2


= xi − 65

UH
xi fi
50 3 −15 −45 225 675
56 3 −9 −27 81 243
62 4 −3 −12 9 36
67 1 2 2 4 4
JA
68 5 3 15 9 45
72 1 7 7 49 49
75 3 10 30 100 300
78 2 13 26 169 338
80 3 15 45 225 675
LP

25 23 41 2365

41
Mean = x = a + d = 65 + = 66.64
25
 
( f d )
2
1   1  ( 41)2  57444
 
A

i i
2 = fidi 2
− = 2365 − =
fi  f i


25 

25 

625

NK

57444
and S.d. =  =  9.59.
25

Illustration 21: Find the variance of 2, 4, 6, 8, 10.


SA

n2 − 1 5 2 − 1
Solution: Variance of 1, 2, 3, 4, 5, is = =2
12 12
( variance of first n natural number is (n2 –1)/12, when each item is doubled
(i.e. 2, 4, 5, 8, 10) variance is multiplied by 2 2 = 4. Required variance = 4  2 = 8)

Illustration 22: The standard deviations of two samples of sizes 50 and 100 are 8 and 7
respectively. Find the standard deviation of the combined sample.
Solution: S.D. of combined sample
n1 + 1 + n2 + 2 1
= = ( 501064 + 1001049 ) = 7.35
n1 + n2 150

Exercise 5
(i). Which of the following is not a measure of dispersion?
(A) mean (B) variance

RI
(C) mean deviation (D) range
(ii). Find the range of the following set of observations 2, 3, 5, 9, 8, 7, 6, 5, 7, 4, 3.
(iii) Find the mean and standard deviation of the following distribution:

A
Class interval Frequency
31−35 2
36−40 3

UH
41−45 8
46−50 12
51−55 16
56−60 5
61−65 2
JA
66−70 2
(iv) Find the mean deviation about median for the distribution
Marks 50 55 60 65 70
No. of 5 8 6 4 2
students
LP

(v). The batting scores of two cricket players A and B in 10 innings are as follows:
Batsman 15 17 19 27 30 36 40 90 95 110
A
A

Batsman 10 16 21 28 37 41 36 80 82 85
B
NK

Find which of the player is more consistent.


(vi) The weights of 9 men are 76, 74.5, 61, 64, 69, 67.5, 71, 73, 74. Find the variance and
standard deviation of the weights.
SA
ANSWERS TO EXERCISES
Exercise 1
(i)
No. of children No. of families
0 4
1 9
2 10
3 5

RI
4 2
(ii)
Age No, of patients Cumulative frequency
10 − 20 90 90

A
20 − 30 50 140
30 − 40 60 200

UH
40 − 50 80 280
50 − 60 50 330
60 − 70 30 360
(iii)
Age frequency
JA
30 − 40 12
40 − 50 18
50 − 60 17
60 − 70 13
70 − 80 15
LP

There are 28 students getting more than 60 marks.


Exercise 2
(i). 38.3 years (ii).  8.43 rupees (iii). 61.2

x + 10
A

(iv). 2x + 3 (v).

Exercise 3
NK

2xy
(i) km/h (ii). 224/5 (iii). 6.54
x+y
Exercise 4
(i) 17 (ii) 155 (iii) 22
SA

(iv) median = 68.33, mode = 76.33 (v) 46


Exercise 5
(iii) mean = 50, standard deviation = 58
(iv) 5
(v) x ( A ) = 47.9 ,  ( A ) = 34.18 , c.v. for A = 71.35%,
x (B) = 45.6 ,  (B) = 27.06 , c.v. for B = 59.34%.
 B is consistent.
(vi)  = 4.79, variance = 22.94.
SOLVED PROBLEMS

Problem -1: Co-efficient of variation of two series are 75% and 90% and their standard
deviations 15 and 18. Find their mean.

 15
Solution: Co-efficient of variance =  100  for first series 75 =  100  x = 20
x x
18
and for second series 90 =  100  x = 20.

RI
x
Thus both the series have same mean i.e. 20.

Problem -2: Find the mean and standard deviation of the binomial co-efficients of the

A
expansion of (1 + x)n. Find the value for n = 4.

UH
C0 + C1 + C2 + ..... + Cn 2n
Solution: Mean = = .
n +1 n +1

 (C )  (C ) 
2
2

−
i i
2 = .
n  n 
 
JA
We know that  (C ) i
2
= 2 n Cn .
= co-efficient of xn in (1 + x)2n
2
2n
Cn  2n  (2n) ! 22n
 2 = −   =
 − .
n  n + 1 (n !)2 n (n + 1)2
LP

Now if n = 4,
8! 28 363
= − = .
4 ! 4 ! 4 25 50
A

ax + b
Problem -3: The S.D of the variate x is . Find the S.D of the variable ; a, b, c are
c
NK

constant.

ax + b a b
Solution: Let y = y= x +  y = Ax + B
c c c
SA

a b
where A = and  y = Ax + B and hence
c c
y – y = Ax + B –(A x + B) = A (x – x )  (y – y )2 = A2(x – x )2
 (y – y )2 = A2 (x – x )2  ny2 = A2 (nx2)  y = |A|x.
a
Hence S.D is multiplied by |A| = .
c
Problem -4: Find out A.M. of C0, 2 C1, 3 C2, …, (n + 1) Cn,
where (1 + x)n= C0 + C1x+ C2x2 +…+ Cnxn.
Solution: (1 + x)n = C0 + C1x + C2x2 + ... + Cnxn
Multiplying with x, we get x (1 + x)n = C0x + C1x2 + C2x3 + ... + Cnxn+1
Differentiating w. r. t. x, we have
nx (1 + x)n−1 + (1 + x)n = C0 + 2C1x + 3C2x2 + ..... + (n + 1)Cnxn.
Putting x = 1, this gives n (2n−1) + 2n = C0 + 2C1 +3 C2 + ...... + (n + 1) Cn
C0 + 2C1 + 3C2 + .... + (n + 1)Cn 2n −1(n + 2)
so that, A.M. = = .
(n + 1) (n + 1)

RI
Problem -5: The number of observations in a group is 40. If the average of first 10 is 4.5 and
that of the remaining 30 is 3.5, then find the average of the whole group?

x1 + x 2 + ........... + x10
Solution: = 4.5

A
10
x + x12 + ............x 40
and 11 = 3.5
30

UH
x + x 2 + ...........x 40 150 15
 1 = = .
40 40 4

Problem -6: Find the mean of the values 0, 1, 2, …………n having corresponding weight nC0,
JA
nC nC ……………,nC respectively?
1, 2, n

0  n C0 + 1 n C1 + 2  n C2 + ........... + n  n Cn
Solution: Required mean = n
C0 + n C1 + ........ + n Cn
n  2n−1 n
= =
LP

2n 2
( 1 n C1 + 2  n C2 + ........... + n  n Cn = n  2n−1 .
)
A

Problem -7: Calculate mean deviation from


(i) mean, (ii) mode (iii) median
in respect of the marks obtained by nine students given below and show that
NK

mean deviation from median is minimum:


7, 4, 10, 9, 15, 12, 7, 9, 7.

7 + 4 + 10 + 9 + 15 + 12 + 7 + 9 + 7 80
Solution: Mean x = = = 8.9 .
9 9
SA

Mode z = item with maximum frequency = 7.


Arranging the data in ascending order, we get
4, 7, 7, 9, 9, 10, 12, 15.
 n + 1 9 +1
Hence median, M =   th item = = 5 th item = 9.
 2  2

Mean deviation from mean =


 m − 8.9 ,
9
where m denotes the marks obtained by each student.
21.1
Mean deviation from mean = = 2.34 .
9

Mean deviation from mode =


 m − 7 = 23 = 2.56 .
9 9

Mean deviation fro mean =


 m − 9 = 21 = 2.33 .
9 9
Hence mean deviation from median is minimum.

RI
Problem -8: For a set of 100 observations, taking assumed mean as 4, the sum of the
deviations is –11 cm, and the sum of the squares of these deviations is 275 cm 2.
Find the coefficient of variation.

A
fd 11
Solution: x=A+ = 4− = 3.87
n 100

UH
2 2
d 2  d  257  11 
and  = −  = − −  = 1.6.
n  n  100  100 
 1.6
Coefficient of Variation =  100 =  100 = 41.13% .
JA
x 3.89

Problem -9: If a variable takes the discrete value  + 4,  –7/2,  –5/2,  –3,  –2,  + 1/2,
 –1/2,  + 5, ( > 0), then find the median.

Solution: Arranging the data, we have –7/2, –3, –5/2, –2, –1/2,  + 1/2,  + 4,  +5
LP

Median is 1/2(4th observation + 5th observation) = 1/2( –2 +  –1/2) =  –5/4.

Problem -10: A student obtain 75%, 80% and 85% in three subjects. If the marks of another
subject are added, then find the minimum average marks?
A

Solution: Marks obtained from 3 subjects out of 300 = 75 + 80 + 85


i.e. = 240
If the marks of another subjects is added, then the marks will be  240 out
NK

of 400.
240
Minimum average marks = = 60% .
4
SA
ASSIGNMENT PROBLEMS

1. The mean height of 29 male workers is 71 cms and 31 female workers is 48 cms. Find
the combined mean height of all 60 workers in the factory.

2. The price of a commodity is increased by 5% from 1997 to 1998, 8% from 1998 to 1999
and 53% from 1999 to 2000. Find the average from the period 1997 to 2000.

3. The geometric mean of 6 observations was calculated as 13. It was later observed that

RI
one of the observation was recorded as 28 instead of 36. Find the correct geometric
mean.

A
4. The weighted arithmetic mean of 10 observations was 36. However a particular
observation was recorded as 60 instead of 40. In what ratio should be the weights of
correct and incorrect readings so as to have no change in AM.

UH
5(a). The mean of k items is X . If the first item is increased by 1, second by 2 and so on. Find
the new mean.
(b). If each item is increased to make its cube then prove that the mean is ( X )2k.
JA
1 n
6. Let r be the range and S2 = 
n − 1 i =1
(xi − x )2 be the standard deviation of the set of
n
observations x1, x2, ……, xn, then prove that S  r .
n −1
LP

7. Show that the mean deviation from the mean of the A.P a, a + d, a + 2d, ….., a + 2nd is
independent of the common difference of A.P..

8. If the observations x1, x2, x3 …, xn are changed to x1 + y, x2 + y, …, xn + y where y is a


A

positive or a negative number, show that the variance remains unchanged.


NK

9. Prove that the variance and standard deviation are independent of change of origin but
not of scale.

10. The mean and standard deviation of one sample are respectively 54.8 and 8, the mean
and standard deviation of another sample are 50.3 and 7 respectively. The size of the
SA

first sample is 50 and that of the second is 100. Find the mean and standard deviation of
the composite sample (size 150) combining the above two samples.

11. The mean of observations is 4.4 and the variance is 8.24. If three of the five observations
are 1, 2 and 6, find the other two observations.

12. The mean annual salaries paid to 1000 employees of a company was Rs. 5000. The
mean annual salaries paid to male and female employees were Rs. 200 and Rs. 4200
respectively. Determine the percentage of males and females employed by the company.
13. If a vehicle covers the distance along four sides of a square with four speeds x, 2x, 3x
and 4x m/sec respectively, then show that harmonic mean of speeds is better average
than arithmetic mean and hence find the average speed.

14. The mean and standard deviation of a set of 100 observations were worked out as 40
and 5 respectively. But by mistake a value 50 was taken in place of 40 for the
observation. Recalculate the correct mean and standard deviation.

15. Prove that the sum of squares of the deviations of a set of values is minimum when taken

RI
about mean.

A
ANSWERS TO ASSIGNMENT PROBLEMS

k +1
5.(a) X +

UH
2

7.
(n + 1)n
2n + 1
X 12 =51.67 ; 12 = 7.6
10.
JA
11. 9 and 4
12. 80% and 20%

14. X =39.9 ;  = 4.9


LP
A
NK
SA
VECTORS & 3-D

JEE (Advanced) Syllabus


 

Vector & Three Dimensional Geometry : Vectors and scalars, magnitude and direction of a vector.
Direction cosines/ratios of vectors. Types of vectors (equal, unit, zero, parallel and collinear vectors), position
vector of a point, negative of a vector, components of a vector, addition of vectors, multiplication of a vector
by a scalar, position vector of a point dividing a line segment in a given ratio. Scalar (dot) product of vectors,
projection of a vector on a line. Vector (cross) product of vectors, scalar triple product.
Direction cosines/ratios of a line joining two points. Cartesian and vector equation of a line, coplanar and
skew lines, shortest distance between two lines. Cartesian and vector equation of a plane. Angle between (i)

RI
two lines, (ii) two planes, (iii) a line and a plane. Distance of a point from a plane.
 

JEE (Main) Syllabus


Vector : Vectors and scalars, addition of vectors, components of a vector in two dimensions and three

A
dimensional space, scalar and vector products, scalar and vector triple product.
3-D : Coordinates of a point in space, distance between two points, section formula, direction ratios and
direction cosines angle between two intersecting lines. Skew lines, the shortest distance between them and

UH
its equation. Equations of a line and a plane in different forms, intersection of a line and a plane, coplanar
lines.

 
What if angry vectors veer  Round your sleeping head, and from.  There's never need to fear  Violence of the poor world's abstract storm.  ........        Warren,  Robert 
PennNature is an infinite sphere of which the centre is everywhere and the circumference nowhere      ........ Pascal, Blaise 
JA
 
Vectors and their representation :
LP

Vector quantities are specified by definite magnitude and definite direction. A vector is generally
represented by a directed line segment, say AB . A is called the initial point and B is called the
A

terminal point. The magnitude of vector AB is expressed by  AB .


Zero vector :
NK

A vector of zero magnitude i.e. which has the same initial and terminal point, is called a zero vector. It
is denoted by O. The direction of zero vector is indeterminate.
Unit vector :
A vector of unit magnitude in the direction of a vector a is called unit vector along a and is denoted by
SA

a
a , symbolically â = .
|a|

Equal vectors :
Two vectors are said to be equal if they have the same magnitude, direction and represent the same
physical quantity.
Collinear vectors :
Two vectors are said to be collinear if their directed line segments are parallel irrespective of their
directions. Collinear vectors are also called parallel vectors. If they have the same direction ( )
⎯⎯
they are named as like vectors but if they have opposite direction (⎯→) then they are named as

unlike vectors.
Symbolically, two non-zero vectors a and b are collinear if and only if, a = b , where   R
a1 a2 a
(
a = b  a1ˆi + a2 ˆj + a3kˆ ) (
=  b1ˆi + b2 ˆj + b3kˆ )  a1 = b1, a2 = b2, a3 = b3 
b1
=
b2
= 3
b3
(

= )
a 1 a 2 a3
Vectors a = a1 î + a2 ˆj + a3kˆ and b = b1 î + b2 ˆj + b3kˆ are collinear if = =
b1 b 2 b3

Note : If a,b are non zero, non−collinear vectors, such that xa + yb = x'a + y'b  x = x' , y = y' ,
(where x, x’, y, y’ are scalars)
Example # 1 : Find unit vector of ˆi − 2jˆ + 3kˆ

RI
Solution : a = ˆi − 2jˆ + 3kˆ
a = a x ˆi + a y ˆj + azkˆ
2 2 2
if then | a | = ax + ay + az
a 1 2 3
 

A
|a| = 14 â = = î – ĵ + k̂
|a| 14 14 14

Example # 2 : a = (x + 1) î – (2x + y) ĵ + 3 k̂ and b = (2x – 1) î + (2 + 3y) ĵ + k̂

UH
find x and y for which a and b are parallel.
x +1 –(2x + y) 3
Solution : a and b are parallel  = =  x = 4/5 , y = –19/25
2x – 1 2 + 3y 1
JA
Coplanar vectors :
A given number vectors are called coplanar if their line segments are all parallel to the same plane.
Note that “two vectors are always coplanar”.

Multiplication of a vector by a scalar :


If a is a vector and m is a scalar, then m is a vector parallel to a whose magnitude is m times that of
LP

a . This multiplication is called scalar multiplication. If a and a are vectors and m, n are scalars,
then :
(i) m (a) = (a) m = ma (ii) m (na) = n(ma) = (mn)a
(iii) (m + n) a = ma + na (iv) m (a + b) = ma + mb
A

Self Practice Problems :


(1) Given a regular hexagon ABCDEF with centre O, show that
(i) OB – OA = OD – OE (ii) EA = 2 OB + OF (iii) AD + EB + FC = 4 AB
NK

(2) Let ABCDEF be a regular hexagon. If AD = x BC and CF = y AB then find xy.


(3) The sum of the two unit vectors is a unit vector. Show that the magnitude of the their difference
is 3 .
Answers : (2) – 4
SA

Addition of vectors :
(i) If two vectors a and b are represented by OA and OB , then their sum a + b is a
vector represented by OC , where OC is the diagonal of the parallelogram OACB.
(ii) a + b = b + a (commutative) (iii) (a + b) + c = a + (b + c) (associative)
(iv) a+0 = a = 0+a (v) a + (−a) = 0 = (−a) + a
(vi) | a + b | | a | + | b | (vii) | a − b |  || a | − | b ||

Example # 3 : The two sides of ABC are given by AB = 2 î + 4 ĵ + 4 k̂ , AC = 2 î + 2 ĵ + k̂ . Then find the
length of median through A.
Solution : Let D be mid point of BC
1
In ABC, AB + BD = AD  AB + BC = AD A
2

B D C
AB + (AB + BC)
 = AD
2
AB + AC 4iˆ + 6ˆj + 5kˆ 77
 = AD  | AD | = =
2 2 2

Example # 4 : In a triangle ABC, D, E, F are the mid-points of the sides BC, CA and AB respectively then
A
prove that, AD = – ( BE + CF ).
1
Solution : AD = 3 GD = 3. (GB + GC) where D is mid-point of BC F 2
G
E
2
3 2 2  1
= EB + FC = – ( BE + CF )
2  3 3  B D C

RI
Position vector of a point:  
Let O be a fixed origin, then the position vector of a point P is the vector OP . If a and b are position
vectors of two points A and B, then

A
AB = b − a = position vector (p.v.) of B − position vector (p.v.) of A.
DISTANCE FORMULA
Distance between the two points A (a) and B (b) is AB = a−b

UH
SECTION FORMULA
If a and b are the position vectors of two points A and B, then the p.v. of
na + mb
a point which divides AB in the ratio m: n is given by r =
JA .
m+n
a+b
Note : Position vector of mid point of AB = .
2
Example # 5 : Let O be the centre of a regular pentagon ABCDE and OA = a .
Then AB + 2BC + 3CD + 4DE + 5EA =
LP

Solution : OA = a,OB = b,OC = c,OD = d,OE = e


AB + 2BC + 3CD + 4DE + 5EA = (b − a ) + 2 ( c − b ) + 3 ( d − c ) + 4 ( e − d) + 5 (a − e)
( )
= 5a – a + b + c + d + e = 5a , (since a + b + c + d + +e = 0)
A

Example # 6 : In a triangle ABC, D and E are points on BC and AC respectively, such that BD = 2DC and
BP
AE = 3EC. Let P be the point of intersection of AD and BE. Find using vector method.
PE
NK

Solution : Let the position vectors of points B and C be respectively b and c referred to A as origin of
reference.
BP PD A(0)
Let = and =
PE AP 3
3c 2c + b
SA

1 E
+b
2c + b 3 4 3 
AD = , AE = c  AP = = P 1
3 4  +1  +1 2 µ 1
C(c)
B(b) D
comparing the coefficient of b & c
1 1 3 2
= and =
 +1 3( + 1) 4( + 1) 3( + 1)
solving above equations we get  = 8/3

Self Practice Problems


(4) Express vectors BC , CA and AB in terms of the vectors OA , OB and OC
(5) If a, b are position vectors of the points(1,–1),(–2, m), find the value of m for which a and
b are collinear.
(6) The vertices P, Q and S of a PQS have position vectors p, q and s respectively.
(i) Find the position vector of t of point T in terms of p, q and s , such that
ST : TM = 2 : 1 and M is mid-point of PQ.
(ii) If the parallelogram PQRS is now completed. Express , the position vector
of the point R in terms of p, q and s

(7) In a quadrilateral ABCD, AB = p , BC = q , DA = p – q . If E is the mid point of BC and F is


4
the point on DE such that DF = DE. Show that the points. A,F,C are collinear.
5

(8) Point L, M, N divide the sides BC, CA, AB of ABC in the ratios 1 : 4, 3 : 2, 3 : 7 respectively.
Prove that AL + BM + CN is a vector parallel to CK , when K divides AB in the ratio 1 : 3.

RI
Answers : (4) BC = OC − OB , CA = OA − OC , AB = OB − OA (5) m=2

1
(6) (i) t = (p + q + s) (ii) r = (q − p + s)
3

A
Distance formula
Distance between any two points (x 1, y1, z1) and (x2, y2, z2) is given as (x1 − x 2 )2 + (y1 − y 2 )2 + (z1 − z 2 )2

UH
Distance of a point P from coordinate axes
Let PA, PB and PC are distances of the point P(x, y, z) from the coordinate axes OX, OY and OZ
respectively then PA = y 2 + z 2 , PB = z2 + x2 , PC = x 2 + y 2
JA
Example # 7 : Find the locus of a point which is equidistance from A (0,2,3) and B (2, –2, 1).
Solution : let P (x, y, z) be any point which is equidistance from A (0,2,3) and B (2, – 2, 1)
PA = PB
 (x – 0)2 + (y – 2)2 + (z – 3)2 = (x – 2)2 + (y + 2)2 + (z – 1)2  x – 2y – z + 1 = 0
LP

Example # 8 : Find the locus of a point which moves such that the sum of its distances from points A(0, 0, – )
and B(0, 0, ) is constant.
Solution : Let the variable point whose locus is required be P(x, y, z)
Given PA + PB = constant = 2a (say)
 (x − 0)2 + (y − 0)2 + (z +  )2 + (x − 0)2 + (y − 0)2 + (z −  )2 = 2a
A

 x 2 + y 2 + (z +  )2 = 2a – x 2 + y 2 + (z −  )2
 x2 + y2 + z2 + 2 + 2z = 4a2 + x2 + y2 + z2 + 2 – 2z – 4a x 2 + y 2 + (z −  )2
NK

z2  2
 4z – 4a2 = – 4a x 2 + y 2 + (z −  )2  + a2 – 2z = x2 + y2 + z2 + 2 – 2z
a2
 2  x2 y2 z2
or, x2 + y2 + z2  1 − 2  = a2 – 2  + + =1
 a  a2 −  2 a2 −  2 a2
SA

This is the required locus.

Self practice problems :


(9) One of the vertices of a cuboid is (0, 2, –1) and the edges from this vertex are along the
positive x-axis, positive y-axis and positive z-axis respectively and are of lengths 2, 2, 3
respectively find out the vertices.
(10) Show that the points (0, 4, 1), (2, 3, –1), (4, 5, 0) and (2, 6, 2) are the vertices of a square.
(11) Find the locus of point P if AP2–BP2=20, where A  (2, –1, 3) and B  (–1, – 2, 1).
Answers : (9) (2,2, –1), (2, 4, –1), (2, 4, 2), (2, 2, 2), (0, 2, 2), (0, 4, 2), (0, 4, –1).
(11) x + y + 2z = 6

Centroid of a triangle
 x + x 2 + x 3 y1 + y 2 + y 3 z1 + z2 + z3 
G  1 , , 
 3 3 3 

Incentre of triangle ABC


 ax + bx 2 + cx 3 ay1 + by 2 + cy 3 az1 + bz2 + cz3 
I  1 , ,  Where AB = c, BC = a, CA = b
 a+b+c a+b+c a+b+c 

Example # 9 : Show that the points A(2, 3, 4), B(–1, 2, –3) and C(–4, 1, –10) are collinear. Also find the
ratio in which C divides AB.
Solution : Given A  (2, 3, 4), B  (–1, 2, –3), C  (– 4, 1, –10).
A (2, 3, 4) B (–1, 2, –3)

RI
Let C divide AB internally in the ratio k : 1, then
 −k + 2 2k + 3 −3k + 4  −k + 2
C  , ,   =–4  3k = – 6  k = –2
 k +1 k +1 k +1  k +1
2k + 3 −3k + 4

A
For this value of k, = 1, and = –10
k +1 k +1
Since k < 0, therefore C divides AB externally in the ratio 2 : 1 and points A, B, C are collinear.

UH
Example # 10 :The vertices of a triangle are A(5, 4, 6), B(1, –1, 3) and C(4, 3, 2). The internal bisector of BAC
meets BC in D. Find AD.
Solution : AB = 42 + 52 + 32 = 5 2
AC = 12 + 12 + 42 = 3 2
Since AD is the internal bisector of BAC
JA
BD AB 5
 = =  D divides BC internally in the ratio 5 : 3
DC AC 3
 5  4 + 3  1 5  3 + 3(−1) 5  2 + 3  3   23 12 19 
 D  , ,  D=  , ,
8 
or,
 5 + 3 5 + 3 5 + 3   8 8
2 2 2
 23   12   19  1530
 AD = 5 − 8  +  4 − 8  + 6 − 8  = unit
LP

      8

Example # 11 : If the points P, Q, R, S are (4, 7, 8), (– 1, – 2, 1), (2, 3, 4) and (1,2,5) respectively, show that PQ
and RS intersect. Also find the point of intersection.
Solution : Let the lines PQ and RS intersect at point A.
A

 − + 4 −2 + 7  + 8 
Let A divide PQ in the ratio  : 1, (  –1) then A   , ,
 + 1 
. .... (1)
  +1  +1
 k + 2 2k + 3 5k + 4 
NK

Let A divide RS in the ratio k : 1, then A   , ,


k + 1 
..... (2)
 k +1 k +1
SA

From (1) and (2), we have,


− + 4 k + 2
=  – k –  + 4k + 4 = k + 2 + k + 2  2k + 3 – 3k – 2 = 0 .....(3)
 +1 k +1
−2 + 7 2k + 3
=  –2k – 2 + 7k + 7 = 2k + 3 + 2k + 3  4k + 5 – 5k – 4 = 0 .....(4)
 +1 k +1
 + 8 5k + 4
= ..... (5)
 +1 k +1
Multiplying equation (3) by 2, and subtracting from equation (4), we get –  + k = 0 or,  = k
Putting  = k in equation (3), we get 22 + 3 – 3 – 2 = 0  =1=k
Clearly  = k = 1 satisfies eqn. (5), hence our assumption is correct.
 −1 + 4 −2 + 7 1 + 8  3 5 9
 A  , ,  or, A  , ,  .
 2 2 2  2 2 2
Self practice problems :
(12) Find the ratio in which yz plane divides the line joining the points A (4, 3, 5) and B (7, 4, 5).
(13) Find the co-ordinates of the foot of perpendicular drawn from the point A(1, 2, 1) to the line
joining the point B(1, 4, 6) and C(5, 4, 4).
8 
(14) Two vertices of a triangle are (4, –6, 3) and (2, –2, 1) and its centroid is  , −1, 2  . Find the
 3 
third vertex.
3 7 1
(15) Show that  , ,  is the circumcentre of the triangle whose vertices are
2 2 2
A (2, 3, 2), B (0, 4, 1) and C (3, 3, 0) and hence find its orthocentre.
Answers : (12) 4 : 7 Externally (13) (3, 4, 5) (14) (2, 5, 2) (15) (2, 3, 2)

RI
Direction cosines and direction ratios
(i) Direction cosines : Let    be the angles which a directed line makes with the positive
directions of the axes of x, y and z respectively, then cos , cos cos  are called the

A
direction cosines of the line. The direction cosines are usually denoted by , m, n.

UH
Thus  = cos , m = cos , n = cos .
If , m, n be the direction cosines of a line, then 2 + m2 + n2 = 1
(ii)
JA
(iii) Direction ratios : Let a, b, c be proportional to the direction cosines , m, n then a, b, c are
called the direction ratios.
If a, b, c, are the direction ratios of any line L, then aiˆ + bjˆ + ckˆ will be a vector parallel to the
line L.
If , m, n are direction cosines of line L, then  î + m ĵ + n k̂ is a unit vector parallel to the line
LP

L.
(iv) If , m, n be the direction cosines and a, b, c be the direction ratios of a vector, then (, m, n)
 a b c   −a −b −c 
=  , ,  or  2 , , 
2  2 
 a +b +c a +b +c   a +b +c a +b +c 
2 2 2 2 2 2 2 2 2 2 2 2 2 2 2
a +b +c a +b +c
A

(v) If the coordinates P and Q are (x 1, y1, z1) and (x2, y2, z2), then the direction ratios of line PQ are,
x − x1
a = x2 − x1, b = y2 − y1 & c = z2 − z1 and the direction cosines of line PQ are  = 2
NK

,
| PQ |
y 2 − y1 z − z1
m= and n = 2 .
| PQ | | PQ |
cos 3
Example # 12 : If a line makes angle , ,  with the co-ordinate axes. Then find the value of  .
SA

cos 
cos 3 4cos3  − 3cos 
Solution :  =  = (4cos2 – 3)
cos  cos 
= 4(cos2 + cos2 + cos2) – 3 – 3 – 3 = 4 – 9 = – 5 Ans. –5

Example # 13 : If the direction ratios of two lines are given by mn – 4n + 3m = 0 and  + 2m + 3n = 0 then
find the direction ratios of the lines.
m n m n
Solution : Eliminating  we have m = ± 2 n  = = & = =
–2 2 – 3 2 1 2 2–3 – 2 1
Ans. ( (–2 2 – 3), 2 ,  ), (2 2 – 3)  , 2  –  where   R – {0}

Self practice problems:


(16) Find the direction cosines of a line lying in the xy plane and making angle 30° with x-axis.
(17) A line makes an angle of 60° with each of x and y axes, find the angle which this line makes
with z-axis.
(18) A plane intersects the co-ordinates axes at point A(2, 0, 0), B(0, 4, 0), C(0, 0, 6) ; O is origin.
Find the direction ratio of the line joining the vertex B to the centroid of face ABC.

3 1 2 8
Answers : (16) = ,m=± , n=0 (17) 45° (18) ,– ,2
2 2 3 3

Scalar product (Dot Product) of two vectors :


a . b = a b cos  , (0    )

Note: (a) If  is acute, then a . b > 0 and if  is obtuse, then a . b < 0.

RI
(b) a . b= 0a⊥b (a  0 , b  0)
(c) Maximum value of a . b is | a | | b | (d) Minimum value of a . b is – | a | | b |

A
Geometrical interpretation of scalar product :
B(b)
As shown in Figure, projection of vector OB (or b ) along vector OA ( or a )

UH
b.a
is OL = | b | cos  = = b.aˆ
a
 L
O
| b | cos  = b.aˆ
A(a)
Properties of Dot Product
JA
a . b
(i) Projection of a on b =
|b|
(ii) a . b = b . a (commutative)
(iii) a . (b + c) = a . b + a . c (distributive)
(iv) (ma) . b = a . (mb) = m (a . b) , where m is a scalar.
LP

(v) î . î = ĵ . ĵ = k̂ . k̂ = 1; î . ĵ = ĵ . k̂ = k̂ . î = 0
2
(vi) a . a = a = a2

(vii) If a = a1 î + a2 ĵ + a3 k̂ and b = b1 î + b2 ĵ + b3 k̂ , then a . b = a1b1 + a2b2 + a3b3


A

2 2 2 2 2 2
a = a1 + a2 + a3 , b = b1 + b2 + b3
NK

(viii) ab = | a |2 + | b |2  2 | a || b | cos  , where  is the angle between the vectors

Example # 14 : Find the value of p for which the vectors a = 3iˆ + 2jˆ + 9kˆ and b = ˆi + pjˆ + 3kˆ are
(i) perpendicular (ii) parallel
SA

Solution : (i) (
a ⊥ b  a . b = 0  3iˆ + 2ˆj + 9kˆ . ˆi + pjˆ + 3kˆ = 0 ) ( )
 3 + 2p + 27 = 0  p = – 15
(ii) vectors a = 3i + 2j + 9k and b = i + pj + 3kˆ are parallel iff
ˆ ˆ ˆ ˆ ˆ
3 2 9 2 2
= =  3=  p=
1 p 3 p 3
Example # 15 : If a , b , c are three vectors such that each is inclined at an angle /3 with the other two and
| a | = 1, | b | = 2, | c | = 3, then find the scalar product of the vectors 2 a + 3 b – 5 c and
4 a – 6 b + 10 c .
Solution : Dot products is 82 a – 182 b – 502 c + a . b (–12 + 12) + b . c (30 + 30) + c . a (20 – 20)
 
= 8 – 18 (4) – 50(9) + 60  2.3cos  = 188 – 522 = –334
 3 
Example # 16 : Find the values of x for which the angle between the vectors
a = 2x2 î + 4x ĵ + k̂ and b = 7 î – 2 ĵ + x k̂ is obtuse.

a . b
Solution : The angle  between vectors a and b is given by cos  =
| a || b |
a . b
Now,  is obtuse  cos  < 0  <0  a . b <0 [ | a |,| b |  0 ]
| a || b |
1
 14x2 – 8x + x < 0  7x (2x – 1) < 0  x(2x – 1) < 0  0 < x <
2
Hence, the angle between the given vectors is obtuse if x  (0, 1/2)

Example # 17 : If a = î + ĵ + k̂ and = 2 a – ĵ + 3 k̂ , then find

RI
(i) Component of b along a . (ii) Component of b in plane of a & b but ⊥ to a .
a . b
Solution : (i) Component of b along a is  2 
2
 a ; Here a . b = 2 – 1 + 3 = 4 and | a | = 3
 | a | 

A
a . b 4 4
Hence  2 
 a = 3 a = 3 ( î + ĵ + k̂ )
 | a | 

UH
a . b 1 ˆ
(ii) Component of b in plane of a & b but ⊥ to a is b –  2 
 a. = 2i − 7ˆj + 5kˆ ( )
 |a|  3

Example # 18 : Find the projection of the line joining A(1, 2, 3) and B(–1, 4, 2) on the line having direction ratios
2, 3, –6.
JA
B
Solution : AB = − 2iˆ + 2ˆj − kˆ A
−4 + 6 + 6 8
Projection of AB = − 2iˆ + 2ˆj − kˆ on 2iˆ + 3jˆ − 6kˆ is = 90° 90°
4 + 9 + 36 7 P L M Q

Self Practice Problems :


 |a−b|
LP

(19) If a and b are unit vectors and  is angle between them, prove that tan = .
2 |a+b|
(20) Find the values of x for which the angle between the vectors a = 2x ˆi + 4jˆ + 3kˆ and
b = 2iˆ – 3jˆ + xkˆ is 90º
A

(21) if a, b, c are the pth, qth, rth terms of a HP then find the angle between the vectors
1 1 1
= (q – r) î + (r – q) ĵ + (p – q) k̂ and v = î + ĵ + k̂ .
a b c
NK

(22) The points O, A, B, C, D are such that OA = a , OB = b , OC = 2a + 3b , OD = a + 2b


Given that the length of OA is three times the length of OB . Show that BD and AC are
perpendicular.
(23) ABCD is a tetrahedron and G is the centroid of the base BCD. Prove that
SA

AB2 + AC2 + AD2 = GB2 + GC2 + GD2 + 3GA2

(24) A (2, 3, –2), B (1, 5, 4,), C(0, –1, 2) D (4, 0, 3). Find the projection of line segment AB on CD
line.
(25) The projections of a directed line segment on co-ordinate axes are 3, 4, –12. Find its length and
direction cosines.
Answers : (20) x = 12/7 (21) /2
2 2 3 4 –12
(24) (25) 13, , ,
3 13 13 13
Vector product (Cross Product) of two vectors:
(i) If a , b are two vectors and  is the angle between them, then a x b = a b sin  nˆ , where n̂ is

the unit vector perpendicular to both a and b such that a , b and nˆ forms a right handed screw
system.
(ii) Geometrically a x b = area of the parallelogram whose two adjacent sides are represented by

a and b .

(iii) axbbxa (not commutative)

RI
(iv) (m a)  b = a  (m b) = m (a  b) , where m is a scalar.
(v) a x (b + c) = (a xb) + (a xc) (distributive)
a  b = 0  a and b are parallel (collinear) (a  0 , b  0) i.e. a = Kb , where K is a scalar.

A
(vi)

(vii) ˆi  ˆi = ˆj  ˆj = kˆ  kˆ = 0 ; ˆi  ˆj = k,
ˆ ˆj  kˆ = ˆi, kˆ  ˆi = ˆj
ˆi ˆj kˆ

UH
(viii) If a = a1 î +a2 ĵ + a3 k̂ and b = b1 î + b2 ĵ + b3 k̂ , then a  b = a1 a2 a3
b1 b2 b3

r(a  b)
(x) A vector of magnitude ‘r’ and perpendicular to the plane of a and b is 
| ab |
JA
(xii) If a, b and c are the position vectors of 3 points A, B and C respectively, then the vector area of
1
ABC = (a  b + b  c + c  a) . The points A, B and C are collinear if
2
ab+ b c+ c  a =0
1
LP

(xiii) Area of any quadrilateral whose diagonal vectors are d1 and d2 is given by d1  d2
2

Example # 19 : Given a = î + ĵ – k̂ , b = – î + 2 ĵ + k̂ and c = – î + 2 ĵ – k̂ , then find a unit vector


perpendicular to both a + b and b + c .
A

Solution :  ×  is ⊥ to both  and 


a +b = 3 ĵ , b + c = –2 î + 4 ĵ
NK

 3 ĵ × (–2 î + 4 ĵ ) is ⊥ to both or –6 ĵ × î = 6 k̂  hence a unit vector is k̂ .

Example # 20 : If  = 2 î + 3 ĵ – k̂ ,  = – î + 2 ĵ – 4 k̂ ,  = î + ĵ + k̂ , then find value (  ×  ).(  ×  ).


Solution :  ×  = –10 î + 9 ĵ + 7 k̂ and  ×  = 4 î – 3 ĵ – k̂ their dot product = –40 – 27 – 7 = –74
SA

Example # 21 : Let OA = a + 3 b , OB = 5 a + 4b and OC = 2 a – b where O is origin. Let p denote the area


of the quadrilateral OABC and q denote the area of the parallelogram with OA and OC as
p
adjacent sides. Find .
q
Solution : We have, p = Area of the quadrilateral OABC
1 1 1
 p = | OB  AC | = | OB  (OC − OA) |  p= | (5a + 4b)  (4b − a) |
2 2 2
1
 p= | 20(a  b) − 4(b  a) | = 12 | a  b | .....(i)
2
and q = Area of the parallelogram with OA and OC as adjacent sides
 q = | OA  OC | = | (a  3b)  (2a – b) | = 7 | a  b | .....(ii)
p 12
From (i) and (ii), we get =
q 7
Self Practice Problems :

(26) If p and q are unit vectors forming an angle of 30º. Find the area of the parallelogram
having a = p + 2q and b = 2p + q as its diagonals.

(27) ABC is a triangle and EF is any straight line parallel to BC meeting AC, AB in E, F
respectively. If BR and CQ be drawn parallel to AC, AB respectively to meet EF in R and Q
respectively, prove that  ARB = ACQ.

Answers : (26) 3/4 sq. units

RI
A LINE
Equation of a line
(i) Vector equation: Vector equation of a straight line passing through a fixed point with position
vector a and parallel to a given vector b is r = a +  b where  is a scalar.

A
(ii) Vector equation of a straight line passing through two points with position vectors a & b is
r = a +  ( b − a ).

UH
(iii) The equation of a line passing through the point (x 1, y1, z1) and having direction ratios a, b, c
x − x1 y − y1 z − z1
is = = = r. This form is called symmetric form. A general point on the line
a b c
is given by (x + ar, y + br, z + cr).
(iv) The equation of the line passing through the points (x 1, y1, z1) and (x2, y2, z2) is
JA
x − x1 y − y1 z − z1
= =
x 2 − x1 y 2 − y1 z 2 − z1
(v) Reduction of cartesian form of equation of a line to vector form & vice versa
x − x1 y − y1 z − z1
= =  = r (x1 î + y1 ĵ + z1 k̂ ) +  (a î + b ĵ + c k̂ ).
a b c
(vi) The equations of the bisectors of the angles between the lines r = a +  b and
LP

r = a +  c are : r = a + t bˆ + cˆ and r = a + p ĉ − bˆ .
( ) ( )
Example # 22 : Find the equation of the line through the points (4, –5, 8) and (–1, 2, 7) in vector form as well
as in cartesian form.
A  (4, –5, 8) , B  (–1, 2, 7)
A

Solution : Let

Now a = OA = 4iˆ – 5 ˆj + 8kˆ and b = OB = – î + 2 ĵ + 7 k̂
→ → →
Equation of the line through A( a ) and B( b ) is r = a + t ( b – a )
NK


or r = 4 î – 5 ĵ + 8 k̂ + t (–5 î + 7 ĵ – k̂ ) ..... (1)
x−4 y+5 z–8
Equation of AB in cartesian form is = =
5 –7 1
SA

Example # 23 : Find the equation of the line passing through point (1, 0, 2) having direction ratio 3, –1, 5.
Prove that this line passes through (4, – 1, 7).
x –1 y−0 z– 2
Solution. equation of line is = = ,
3 –1 5
4 –1 −1 − 0 7 – 2
Now = = = 1 , so line passes through point (4, –1,7)
3 –1 5

Example # 24 : Find the equation of the line drawn through point (–1, 7, 0) to meet at right angles the line
x – 2 y +3 z –1
= =
2 −1 2
x – 2 y +3 z –1
Solution : Given line is = = ..... (1)
2 −1 2
Let P  (–1, 7, 0)
Co-ordinates of any point on line (1) may be taken as Q  (2r + 2, – r – 3, 2r + 1)
Direction ratios of PQ are 2r + 3, – r – 10, 2r + 1
Direction ratios of line AB are 2, – 1, 2
Since PQ ⊥ AB
 2(2r + 3) + (– r – 10) (–1) + 2 (2r + 1) = 0  r=–2
Therefore, direction ratios of PQ are – 1 , – 8, – 3
x +1 y – 7 z
Equation of line PQ is = =
1 8 3
Example # 25 : A line passes through the point 3i and is parallel to the vector −ˆi + ˆj + kˆ and another line
ˆ
passes through the point ˆi + ˆj and is parallel to the vector ˆi + kˆ , then find the point of
intersection of lines.
Solution : A point on the first line is 3iˆ + s(−ˆi + ˆj + k)
ˆ ....(i)
ˆ ˆ ˆ
A point on the second line is i + j + t(i + k) ˆ ....(ii)
At the point of intersection (i) and (ii) are same.

RI
 3 – s = 1 + t, s = 1, s = t  s=t=1
hence the point is 3iˆ + (−ˆi + ˆj + k)
ˆ = 2iˆ + ˆj + kˆ Ans. (2,1,1)

Self practice problems:

A
x − 3 y +1 z − 7
(28) Find the equation of the line parallel to line = = and passing through the point
4 –1 5

UH
(2, 3, –2).
x−2 y–3 z+2
Answers : (28) = =
4 1 5

Foot, Reflection, length of perpendicular from a point to a line :


JA
P(x 1,y 1,z 1)


Q(x 2,y 2,z 2) F(x,y,z) x − x 2 y − y 2 z − z2
= =
LP

a b c

R(x,y,z)
A

x − x2 y − y2 z − z2
Let L  = = is a given line and P(x1, y1, z1) is given point as shown in figure.
a b c
Let F(x, y, z) = (ar+x2, br+ y2, cr+ z2) ….(1) be the foot of the point P (x 1, y1, z1) with respect to the line
NK

L. Apply PF.(aiˆ + bjˆ + ck)


ˆ = 0 we get ‘r’. Now put this value of ‘r’ in (1) we get F
Now for calculating the reflection R(x, y, z) of the point P (x 1, y1, z1) with respect to the line L, apply
midpoint formula (midpoint of P & R is F)
PF.(aiˆ + bjˆ + ck)ˆ
PF = PQ sin  =
2 2 2
SA

a +b +c

x +1 y − 3 z + 2
Example # 26 : Find the length of the perpendicular from P (2, – 3, 1) to the line = = .
2 3 −1
Solution : Co-ordinates of any point on given line may be taken as Q  (2r – 1, 3r + 3, – r – 2)
Direction ratios of PQ are 2r – 3, 3r + 6, – r – 3
Direction ratios of AB are 2, 3, – 1
Since PQ ⊥ AB
−15
 2 (2r – 3) + 3 (3r + 6) – 1 (– r – 3) = 0  14r + 15 = 0  r=
14
 −22 −3 −13  531
 Q  , ,   PQ = units.
 7 14 14  14
Self practice problems :
(29) Find the length and foot of perpendicular drawn from point (2,3,4) to the line
x − 4 y z –1
= = . Also find the image of the point in the line.
–2 6 –3
Answers : (29) 3 5 , N  (2, 6, –2),   (2, 9,–8)

Angle between two line :


If two lines have direction ratios a 1, b1, c1 and a2, b2, c2 respectively, then we can consider two vectors
parallel to the lines as a1 î + b1 ĵ + c1 k̂ and a2 î + b2 ĵ + c2 k̂ and angle between them can be given as.

RI
a1a2 + b1b2 + c1c 2
cos  = .
a12 + b12 + c12 a22 + b22 + c 22
(i) The lines will be perpendicular if a1a2 + b1b2 + c1c2 = 0

A
a1 b c
(ii) The lines will be parallel if = 1 = 1
a2 b2 c2

UH
Example # 27 : What is the angle between the lines whose direction cosines are
3 1 3 3 1 3
− , , − and − , ,
4 4 2 4 4 2
Solution : Let  be the required angle, then cos = 12 + m1m2 + n1n2
JA
 3  3   1  1  3  3 3 1 3 1
= −  −  4  +  − 2  .  2  = 16 + 16 − 4 = − 2   = 120°,
 4   +  
   4  4      

Example # 28 : P is a point on line r = 5iˆ + 7ˆj − 2kˆ + s(3iˆ − ˆj + k)


ˆ and Q is a point on the line

( )
r = –3iˆ + 3jˆ + 6kˆ + t −3iˆ + 2ˆj + 4kˆ . If PQ is parallel to the vector, 2iˆ + 7jˆ − 5kˆ , find P and Q
LP

Solution : PQ = OQ – OP is parallel to 2iˆ + 7jˆ − 5kˆ


 – 8iˆ − 4jˆ + 8kˆ + t(−3iˆ + 2jˆ + 4k) ( )
ˆ – s 3iˆ − ˆj + kˆ = 2 = –8 – 3t – 3s

7 = –4 + 2t + s  –5 = 8 + 4t – s
A

solving,  = t = s = –1
 P = (5, 7, –2) – (3, –1, 1) = (2, 8, –3)  Q = (–3, 3, 6) – (–3, 2, 4) = (0, 1, 2)
NK

Self practice problems :


(30) Find the angle between the lines whose direction cosines are given by  + m + n = 0 and 
 2 + m2 – n2 = 0
(31) Let P (6, 3, 2), Q (5, 1, 4), R (3, 3, 5) are vertices of a  find Q.
SA

(32) Show that the direction cosines of a line which is perpendicular to the lines having
directions cosines 1 m1 n1 and 2 m2 n2 respectively are proportional to
m1n2 – m2n1 , n12 – n21, 1m2 – 2m1
Answers : (30) 60° (31) 90°
Skew Lines :
Lines in space which do not intersect and are also not parallel are called skew line.

If lines r = a +  p & r = b +  q are skew lines then (b − a) . (p x q)  0


If lines are not skew lines then they are coplanar which means if (b − a) . (p x q) = 0 , then lines are coplanar.

RI
Shortest distance between two lines
(i) Shortest distance (d) between lines r = a +   p & r =   b +  q

A
(b − a) . (p x q)
is d =
px q

UH
x− y − z− x − ' y − ' z − '
(ii) Shortest distance (d) between two skew lines = = and = =
m n ' m' n'

 ' −   ' −   '− 


JA
is d = m n   (mn − mn)2
' m' n'

For Skew lines the direction of the shortest distance would be perpendicular to both the lines.
If d = 0, the lines are coplanar
LP

(iii) Shortest distance between two parallel lines r1 = a1 + Kb and r2 = a2 + Kb , is given by

b x (a2 − a1 )
d=
b
A

Example # 29 : Find the shortest distance and the vector equation of the line of shortest distance between the
→ →
lines given by r = 3iˆ + 8ˆj + 3kˆ +  (3iˆ − ˆj + kˆ ) and r = −3iˆ − 7ˆj + 6kˆ +  ( −3iˆ + 2ˆj + 4kˆ )
NK

x−3 y−8 z−3


Solution : Equation of given lines in cartesian form is = = = (say L1)
3 −1 1
x+3 y+7 z−6
and = = = (say L2)
−3 2 4
L on L1 is (3 + 3, –  + 8,  + 3) and M on L2 is (– 3 – 3, 2 – 7, 4 + 6)
SA

Let
Direction ratios of LM are 3 + 3 + 6, –  – 2 + 15,  – 4 – 3.
Since LM ⊥ L1
 3 (3 + 3 + 6) – 1 (–  – 2 + 15) + 1 ( – 4 – 3) = 0 or, 11 + 7 = 0 ..... (1)
Again LM ⊥ CD
 – 3 (3 + 3 + 6) + 2 (–  – 2 + 15) + 4 ( – 4 – 3) = 0 or, – 7 – 29 = 0 ..... (2)

A L B
90°

90°
C M D

Solving (1) and (2), we get  = 0,  = 0  L  (3, 8, 3), M  (– 3, – 7, 6)


Hence shortest distance LM = (3 + 3) + (8 + 7) + (3 − 6)2
2 2
= 270 = 3 30 units

Vector equation of LM is r = 3iˆ + 8ˆj + 3kˆ + t ( 6iˆ + 15ˆj − 3kˆ )
x−3 y−8 z−3
Note : Cartesian equation of LM is = = .
6 15 −3
Self practice problems:
x −1 y − 2 z − 3 x−2 y−4 z−5
(33) Find the shortest distance between the lines = = and = = .
2 3 4 3 4 5
Find also its equation.
1
Answers : (33) , 6x – y = 10 – 3y = 6z – 25
6

RI
Scalar triple product (Box Product) (S.T.P.) :
(i) The scalar triple product of three vectors a , b and c is defined as: a x b . c = a b c ,

sin  . cos  where  is the angle between a, b (i.e. a ^b = θ ) and  is the angle between

A
a x b and c (a × b) ^ c = ) . It is (i.e. a  b . c ) also written as a b c  and spelled as box
product.

UH
(ii) Scalar triple product geometrically represents the volume of the parallelopiped whose three
coterminous edges are
JA
represented by a, b and c i.e. V = | [a b c ] |
LP

(iii) In a scalar triple product the position of dot and cross can be interchanged i.e.
a . (b x c) = (a x b) . c  [ a b c] = [ b c a ] = [ c a b ]
(iv) a . (b x c) = −a . (c x b) i.e. [a b c ] = −[ a c b ]
a1 a2 a3
A

(v) If a = a1 î + a2 ĵ + a3 k̂ ; b = b1 î +b2 ĵ +b3 k̂ and c = c1 î + c2 ĵ + c3 k̂ , then [ a b c ] = b1 b2 b3 .


c1 c2 c3
In general, if a = a1 + a2m + a3n ; b = b1 + b2m + b3n and c = c1 + c 2m + c3n
NK

a1 a2 a3

then  a b c  = b b2 b3  m n , where , m and n are non-coplanar vectors.
 1

c1 c 2 c3
SA

(vi) If a , b , c are coplanar, then  [a b c] = 0 .

(vii) If a , b , c are non-coplanar, then [a b c ]  0 for right handed system and [a b c]  0 for left
handed system.

(viii) [iˆ ˆj k]
ˆ =1
(ix) [K a b c] = K [a b c]
(x) [(a + b)c d] = [a c d] + [b c d]
(xi) a − b b − c c − a  = 0 and a + b b + c c + a  = 2 a b c 
     
a.a a.b a.c
2
 
(xii) a b c  = b.a b.b b.c
c.a c.b c.c

Volume of Parallelopiped/ Tetrahedron and their properties :


(a) The volume of the parallelopiped whose three coterminous edges are a , b and c is V = [a b c ]

(b) The volume of the tetrahedron OABC with O as origin and the position vectors of A, B and C being
1 
a, b and c respectively is given by V = a b c 
6 

RI
(c) If the position vectors of the vertices of tetrahedron are a , b , c and d , then the position vector of its
1
centroid is given by (a + b + c + d) .
4
Note : that this is also the point of concurrency of the lines joining the vertices to the centroids of the

A
opposite faces and is also called the centre of the tetrahedron. In case the tetrahedron is regular it is
equidistant from the vertices and the four faces of the tetrahedron.

UH
Example # 30 : The volume of the paralleopiped whose edges are represented by –12 î +  k̂ , 3 ĵ – k,
2 î + ĵ – 15 k̂ is 546, then find  .
−12 0 
Solution : V = | 0 3 −1 |  546 = |12 × 44 – 6|   = –3 , 179
2 1 −15
JA
Example # 31 : Find the volume of the tetrahedron whose four vertices have position vectors a , b , c and d .
Solution : Let four vertices be A, B, C, D with position vectors a , b , c and d respectively.
 DA = ( a – d )  DB = ( b – d )  DC = ( c – d )
1
LP

Hence volume V = [a – d b –d c – d]
6
1 1
= ( a – d ). [( b – d ) × ( c – d )] = (a – d) . [b ×c – b × b + c ×d ]
6 6
1 1
= {[ a b c ] – [ a b d ] + [ a c d ] – [ d b c ]} = {[ a b c ] – [ a b d ] + [ a c d ] – [ b c d ]}
A

6 6
Example # 32 : Prove that vectors r1 = (sec2 A, 1, 1) ; r2 = (1, sec2B, 1) ; r3 = (1, 1, sec2 C) are always non-
coplanar vectors if A, B, C  (0,  ).
NK

sec 2 A 1 1
Solution : Condition of coplanarity gives D = 0  1 sec 2 B 1 =0
2
1 1 sec C
 sec2A [sec2Bsec2C – 1] – 1(sec2c – 1) + 1(1 – sec2B) = 0

SA

(1 + tan2 A)(tan2 B + tan2 C + tan2 B tan2 C) – tan2 C – tan2 B = 0


 tan2 B tan2 C + tan2 A tan2 B + tan2 C tan2A + tan2 A tan2 B tan2 C = 0
divide by tan2 A tan2 B tan2 C
cot2A + cot2B + cot2C = – 1 it is a not possible

Example # 33 : If two pairs of opposite edges of a tetrahedron are mutually perpendicular, show that the third
pair will also be mutually perpendicular.
Solution : Let OABC be the tetrahedron, where O is the origin and co-ordinates of A, B, C are
(x1, y1, z1), (x2, y2, z2), (x3, y3, x3) respectively.
Let OA ⊥ BC and OB ⊥ CA .
We have to prove that OC ⊥ BA .
Now, direction ratios of OA are x 1, y1, z1 and of BC are (x3 – x2), (y3 – y2), (z3 – z2).
 OA ⊥ BC and OB ⊥ CA
 x1(x3 – x2) + y1(y3 – y2) + z1(z3 – z2) = 0 and x2(x1 – x3) + y2(y1 – y3) + z2(z1 – z3) = 0
A (x1, y1, z1)

O (0, 0, 0)

B C
(x2, y2, z2) (x3, y3, z3)

Adding above two equations we get x3(x1 – x2) + y3(y1 – y2) + z3(z1 – z2) = 0
 OC ⊥ BA (direction ratios of OC are x3, y3, z3 and that of BA are (x1 – x2), (y1– y2), (z1 – z2))

Self practice problems :

(34) Show that a . (b + c)  (a + b + c) = 0

RI
(35) One vertex of a parallelopiped is at the point A (1, –1, –2) in the rectangular cartesian co- ordinate. If
three adjacent vertices are at B(–1, 0, 2), C(2, –2, 3) and D(4, 2, 1), then find the volume of the
parallelopiped.

A
(36) Show that the vector a, b, c are coplanar if and only if b + c , c + a , a + b are coplanar.

(37) Show that {( a + b + c ) × ( c – b )} . a = 2 a b c  .

UH
(38) Find the value of m such that the vectors 2iˆ − ˆj + kˆ , ˆi + 2jˆ − 3kˆ and 3iˆ + mjˆ + 5kˆ are coplanar.

(39) Find the value of  for which the four points with position vectors −ˆj − kˆ , 4iˆ + 5jˆ + kˆ , 3iˆ + 9jˆ + 4kˆ , and
JA
−4iˆ + 4jˆ + 4kˆ are coplanar.

Answer : (35) 72 (38) –4 (39) =1

Vector triple product :


LP

Let a , b and c be any three vectors, then the expression a x (b x c ) is a vector & is called a
vector triple product. This vector is perpendicular to a and lies in plane containing vectors b and c

⚫ a x (b x c ) = (a . c)b − (a . b)c
A

⚫ (a x b) x c = (a . c)b − (b . c)a
⚫ In general (a x b) x c  a x (b x c)
NK

Example # 34 : [ a ×(3 b +2 c ) b ×( c – 2 a ) 2 c ×( a –3 b )] =
Solution : Let b × c = p , c × a = q , a × b = r
 [ p q r ] = [ a b c ]2 ....(i)
a × (3 b + 2 c ) = 3 r – 2q etc.
SA

 E = [3 r – 2 q p + 2 r , 2 q + 6 p ]
0 −2 3
= [0 p – 2 q + 3 r , p + 0 q + 2 r , 6 p + 2 q + 0 r ] = 1 0 2 [ a b c ]2 = –18 [ a b c ]2
6 2 0
 3b+a
Example # 35 : If a , b , c are non-coplanar unit vectors such that ( a × b ) × c =   . Then find
 2 
angles which makes c with a & b ( a and b are non-collinear)
 3b+a 3b+a
Solution : (a ×b ) × c =   ( a . c .), b – ( b . c ) a =
 2 2
 
3 1
a.c = , and b .c = – .
2 2
3 1
cos = and cos = – .
2 2
 2
= and =
6 3

Example # 36 : Prove that a  {b  (c  d)} = (b . d)(a  c) – (b. c) (a  d)


Solution : We have, a  {b  (c  d)} = a  {(b . d) c − (b . c) d}
= a  {(b . d) c} − a  {(b . c) d} [by dist. law]
= (b . d) (a  c) − (b . c) (a  d) .

Self Practice Problems :

RI
(40) Prove that a  {a  (a  b)} = (a . a) (b  a) .

(41) Let b and c be noncollinear vectors. If a is a vector such that a . (b + c) = 4 and

A
a  (b  c) = (x2 – 2x + 6) b + c siny, then find x and y.

(42) Find a unit vector coplanar with ˆi + ˆj + 2kˆ and ˆi + 2jˆ + kˆ and perpendicular to ˆi + ˆj + kˆ is

UH
 ˆj – kˆ 
Answer : (41) x = 1 & y = (4n + 1) /2 , n   (42) ±  
 2 

Linear combinations :
JA
Given a finite set of vectors a,b,c,...... , then the vector r = xa + yb + zc + ........ is called a linear
combination of a,b,c,...... for any x, y, z.....  R. We have the following results :

(a) If a,b are non zero, non−collinear vectors, then xa + yb = x'a + y'b  x = x' , y = y'

(b) Fundamental Theorem in plane : Let a,b be non zero, non collinear vectors, then any vector r
LP

coplanar with a,b can be expressed uniquely as a linear combination of a and b


i.e. there exist some unique x, y  R such that xa + yb = r .

(c) If a,b,c are non−zero, non−coplanar vectors, then


A

xa + yb + zc = x'a + y'b + z'c  x = x' , y = y' , z = z'

(d) Fundamental theorem in space: Let a,b,c be non−zero, non−coplanar vectors in space. Then any
NK

vector r can be uniquely expressed as a linear combination of a,b,c i.e. there exist
some unique x,y, z  R such that xa + yb + zc = r .

(e) If x1 ,x2 ,......, xn are n non zero vectors and k1,k2,.....,kn are n scalars and if the linear combination
SA

k1x1 + k 2 x2 + ....... + kn xn = 0  k1 = 0, k 2 = 0 , ....., k n = 0 , then we say that vectors


x1 , x2 , ......,xn are linearly independent vectors.

(f) If k1x1 + k 2 x2 + k3 x3 ...... + kr xr + ...... + kn xn = 0 and if there exists at least one k r  0, then
x1 , x2 , ......, xn are said to be linearly dependent vectors.
If kr  0 then x r is expressed as a linear combination of vectors x1, x2 ,........, xr −1, xr +1, .........., x n
Note :
 In general, in 3 dimensional space every set of four vectors is a linearly dependent system.

 î , ĵ , k̂ are Linearly Independent set of vectors. For K1 î + K2 ĵ + K3 k̂ = 0  K1= K2= K3 = 0

 Two vectors a and b are linearly dependent  a is parallel to b i.e. a  b = 0  linear dependence
of a and b . Conversely if a  b  0 then a and b are linearly independent.
 If three vectors a, b, c are linearly dependent, then they are coplanar i.e. [a b c] = 0. Conversely if
[a b c]  0 then the vectors are linearly independent.

Example # 37 : If a , b , c are three non-coplanar vectors, solve the vector equation r . a = r . b = r . c = 1


Solution : since a , b , c are three non-coplanar vectors therefore a × b , b × c & c × a are also non-
coplanar vectors
Let r = x ( a × b ) + y ( b × c ) + z ( c × a ).
Then, r . a = 1  1 = y [( b × c ) a .]
1 1 1
y= , similarly x = z =  r = (( a × b ) + ( b × c ) + ( c × a ))
[a b c] [a b c] [a b c]

RI
Example # 38 : Given that position vectors of points A, B, C are respectively
a – 2 b + 3 c , 2 a + 3 b – 4 c , – 7 b + 10 c then prove that vectors AB and AC are linearly
dependent.
Solution : Let A, B, C be the given points and O be the point of reference then

A
OA = a – 2 b + 3 c , OB = 2 a + 3 b – 4 c and OC = – 7 b + 10 c
Now AB = p.v. of B – p.v. of A
= OB – OA = ( a + 5 b – 7 c ) and AC = p.v. of C – p.v of A

UH
= OC − OA = – (a + 5b − 7c) = – AB
 AC =  AB
where  = – 1. Hence AB and AC are linearly dependent.
JA
Example # 39 : Prove that the vectors 5 a + 6 b + 7 c , 7 a – 8 b + 9 c and 3 a + 20 b + 5 c are linearly
dependent, where a , b , c being linearly independent vectors.
Solution : We know that if these vectors are linearly dependent , then we can express one of them as a
linear combination of the other two.
Now let us assume that the given vector are coplanar, then we can write
5 a + 6 b + 7 c = ( 7 a – 8 b + 9 c ) + m (3 a + 20 b + 5 c ) where , m are scalars
LP

Comparing the coefficients of a , b and c on both sides of the equation


5 = 7 + 3m, 6 = – 8 + 20 m, 7 = 9 + 5m
1
 = = m. Hence the given vectors are linearly dependent.
2
A

Self Practice Problems :


1 1
(43) Given that x + (p . x) p = q , show that p . x = p . q and find x in terms of p and q .
p2 2
NK

(44) If x . a = 0, x . b = 0 and x . c = 0 for some non-zero vector x , then show that


[a b c] = 0
(r . a) (b  c) (r . b) (c  a) (r . c) (a  b)
(45) Prove that r = + +
[ a b c ] [ a b c ] [ a b c ]
SA

where a, b, c are three non-coplanar vectors

(46) Does there exist scalars u, v, w such that ue1 + ve2 + we3 = ˆi where e1 = kˆ , e2 = ˆj + kˆ ,
e3 = −ˆj + 2kˆ ?

(47) If a and b are non-collinear vectors and A =(x + 4y) a + (2x + y + 1) b and
B = (y – 2x + 2) a + (2x – 3y – 1) b , find x and y such that 3A = 2B .

(48) If vectors a, b,c be linearly independent, then show that


(i) a − 2b + 3c , −2a + 3b − 4c , −b + 2c are linearly dependent
(ii) a − 3b + 2c , −2a − 4b − c , 3a + 2b − c are linearly independent.
(49) Prove that a vector r in space can be expressed linearly in terms of three non-coplanar, non-
zero vectors a, b, c in the form
[r b c] a + [ r c a ] b + [ r a b ] c
r=
[ a b c]
p . q
Answers : (43) x = q –  2 
p (46) No (47) x = 2, y = –1
 2|p| 
Test of collinearity :
Three points A,B,C with position vectors a, b, c respectively are collinear, if & only if there exist scalars
x, y, z not all zero simultaneously such that xa + yb + zc = 0 = 0 , where x + y + z = 0.

RI
Test of coplanarity :
Four points A, B, C, D with position vectors a, b, c, d respectively are coplanar if and only if there exist
scalars x, y, z, w not all zero simultaneously such that xa+yb+zc +wd = 0 , where x + y + z + w = 0.

A
Example # 40 : Prove that four points 2a + 3b − c , a − 2b + 3c , 3a + 4b − 2c and a − 6b + 6c are coplanar.
Solution
PS : Let the given four points be P, Q, R and S respectively. These points are coplanar if the

UH
vectors PQ , PR and PS are coplanar. These vectors are coplanar iff one of them can be
expressed as a linear combination of other two. So let PQ = x PR + y PS
 (
−a − 5b + 4c = x a + b − c ) (
+ y −a − 9b + 7c )
 −a − 5b + 4c = (x – y) a + (x – 9y) b + (–x + 7y) c
JA
 x – y = –1, x – 9y = –5, –x + 7y = 4 [Equating coeff. of a, b, c on both sides]
1 1
Solving the first two equations of these three equations, we get x = – ,y= .
2 2
These values also satisfy the third equation. Hence the given four points are coplanar.

Self Practice Problems :


LP

(50) If a, b, c, d are any four vectors in 3-dimensional space with the same initial point and such
that 3a − 2b + c − 2d = 0 , show that the terminal A, B, C, D of these vectors are coplanar. Find
the point (P) at which AC and BD meet. Also find the ratio in which P divides AC and BD.
A

3a + c
Answers : (50) p = divides AC in 1 : 3 and BD in 1 : 1 ratio
4
NK

A PLANE
If line joining any two points on a surface lies completely on it then the surface is a plane.
OR
If line joining any two points on a surface is perpendicular to some fixed straight line. Then this surface
is called a plane. This fixed line is called the normal to the plane.
SA

Equation of a plane :
(i) Vector form : The equation (r − r0 ) . n = 0 represents a plane containing the point with position
vector is a vector normal to the plane.
The above equation can also be written as r . n = d , where d = r0 . n

(ii) Cartesian form : The equation of a plane passing through the point (x 1, y1, z1) is given by
a (x − x1) + b( y − y1) + c (z − z1) = 0 where a, b, c are the direction ratios of the normal
to the plane.
(iii) Normal form : Vector equation of a plane normal to unit vector and at a distance d from the
origin is r . n = d. Normal form of the equation of a plane is x + my + nz = p, where, ,m, n are
the direction cosines of the normal to the plane and p is the distance of the plane from the
origin.
(iv) General form : ax + by + cz + d = 0 is the equation of a plane, where a, b, c are the
direction ratios of the normal to the plane.

(v) Plane through three points : The equation of the plane through three non −collinear points
x − x3 y − y 3 z − z3
(x1, y1, z1), (x2, y2, z2), (x3, y3, z3) is x1 − x3 y1 − y3 z1 − z3 = 0
x2 − x3 y2 − y3 z 2 − z3
x y z
(vi) Intercept Form : The equation of a plane cutting intercept a, b, c on the axes is + + =1
a b c
Note :
 Equation of yz−plane, xz−plane and xy−plane is x = 0, y = 0 and z = 0

RI
 Transformation of the equation of a plane to the normal form: To reduce any equation
ax + by + cz − d = 0 to the normal form, first write the constant term on the right hand side
and make it positive, then divide each term by a2 + b2 + c 2 , where a, b, c are coefficients of x,

A
y and z respectively e.g.
ax by cz d
+ + =
 a +b +c
2 2 2
 a +b +c
2 2 2
 a +b +c
2 2 2
 a + b2 + c 2
2

UH
Where (+) sign is to be taken if d > 0 and ( −) sign is to be taken if d < 0.

 A plane ax + by + cz + d = 0 divides the line segment joining (x 1, y1, z1) and (x2, y2, z2). in the
 ax + by1 + cz1 + d 
ratio  − 1 
 ax 2 + by 2 + cz2 + d 
JA
 Coplanarity of four points
The points A(x1 y1 z1), B(x2 y2 z2) C(x3 y3 z3) and D(x4 y4 z4) are coplanar then
x 2 − x1 y 2 − y1 z2 − z1
x3 − x1 y3 − y1 z3 − z1 =0
x 4 − x1 y 4 − y1 z 4 − z1
LP

Example # 41 : Find the equation of the plane upon which the length of normal from origin is 10 and direction
ratios of this normal are 3, 2, 6.
Solution : If p be the length of perpendicular from origin to the plane and , m, n be the direction
cosines of this normal, then its equation is
A

x + my + nz = 10 ..... (1)
Direction ratios of normal to the plane are 3, 2, 6
3 2 6
 Direction cosines of normal to the required plane are  = , m = , n =
NK

7 7 7
3 2 6
Equation of required plane is x + y + z = 10 or, 3x + 2y + 6z = 70
7 7 7
Example # 42 :Find the plane through the points (2,–3,3), (–5, 2, 0), (1, – 7, 1)
x–2 y+3 z–3 x–2 y+3 z–3
SA

Solution : –5 – 2 2 + 3 0 – 3 = 0 or –7 5 –3 = 0  2x + y – 3z + 8 = 0
1– 2 –7 + 3 1– 3 –1 –4 –2
Example # 43 : If P be any point on the plane x + my + nz = p and Q be a point on the line OP such that
OP . OQ = p2, show that the locus of the point Q is p(x + my + nz) = x 2 + y2 + z2.
Solution : Let P  (, , ), Q  (x1, y1, z1)
Direction ratios of OP are , ,  and direction ratios of OQ are x 1, y1, z1.
  
Since O, Q, P are collinear, we have = = = k (say) ..... (1)
x1 y1 z1
As P (, , ) lies on the plane x + my + nz = p,
  + m + n = p or k(x1 + my1 + nz1) = p ..... (2)
Given OP . OQ = p2   2 + 2 +  2 x12 + y12 + z12 = p2
or, k 2 (x12 + y12 + z12 ) x12 + y12 + z12 = p2 or, k (x12 + y12 + z12 ) = p2 ....(3)
x1 + my1 + nz1 1
On dividing (2) by (3), we get 2 2 2
= or, p (x1 + my1 + nz1) = x12 + y12 + z12
x1 + y1 + z1 p
Hence the locus of point Q is p (x + my + nz) = x 2 + y2 + z2.

Example # 44 : A moving plane passes through a fixed point (,) and cuts the coordinate axes A, B, C . Find
the locus of the centroid of the tetrahedron OABC.
x y z
Solution : Let the plane be + + = 1, 0 (0,0,0) , A (a, 0,0), B (0, b, 0)
a b c
a b c
C (0,0,c) . Centroid of OABC is  , , 
4 4 4
  
The plane passes through (,,)  + + = 1 ......(i)

RI
a b c
a b c
Centroid , x = ,y = ,z= or a = 4x , b = 4y , c = 4z
4 4 4
  
Now (1) gives the locus of G as + + =4

A
x y z

Self practice problems :

UH
(51) Check whether given points are coplanar if yes find the equation of plane containing them
A  (0, –1, –1), B  (4, 5, 1), C  (3, 9, 4), D  (–4, 4, 4)
(52) Find the plane passing through point (3, 2, 1) and perpendicular to the line joining the
points (2, 4, 3) and (3, –1, 5).
JA
(53) Find the equation of plane passing through the point (2, 4, 6) and making equal intercepts on
the coordinate axes.
(54) Find the equation of plane passing through (1, 2, –3) and (2, 3, 3) and perpendicular to the
plane 2x + y – 3z + 4 = 0.

(55) Find the equation of the plane parallel to 2iˆ + ˆj – kˆ and ˆi − 2jˆ – 3kˆ and passing through (2, 1, 3).
LP

(56) Find the equation of the plane passing through the point (1, 1, – 1) and perpendicular to the
planes x + 2y + 3z – 7 = 0 and 2x – 3y + 4z = 0.
A

Answers : (51) yes, 5x – 7y + 11z + 4 = 0 (52) x – 5y + 2z + 5 = 0


(53) x + y + z =12 (54) 9x – 15y + z + 24 = 0
(55) x–y+z=4 (56) 17x + 2y – 7z = 26
NK

Position of point with respect to plane :


A plane divides the three dimensional space in two equal parts. Two points A (x 1 y1 z1)
and B (x2 y2 z2) are on the same side of the plane ax + by + cz + d = 0 if ax 1 + by1 + cz1 + d and
ax2 + by2 + cz2 + d are both positive or both negative and are opposite side of plane if both of these
SA

values are in opposite sign.

Example # 45 : Show that the points (1, 2, 3) and (2, – 1, 4) lie on opposite sides of the plane
x + 4y + z – 3 = 0.
Solution : Since the numbers 1+ 4 × 2 + 3 – 3 = 9 and 2 – 4 + 4 – 3 = – 1 are of opposite sign, then points
are on opposite sides of the plane.
A plane & a point
P(x 1,y 1, z 1)

F(x, y, z)

R(x,y, z)

Let P  ax + by + cz + d = 0 is a given plane and P(x 1, y1, z1) is given point as shown in figure.

RI
Let F(x, y, z) be the foot of the point P (x 1, y1, z1) with respect to the plane P.
And R(x, y, z) be the reflection of point P (x 1, y1, z1) with respect to the plane P.

Distance of the point (x , y, z) from the plane ax + by + cz+ d = 0 is given by

A
(i)
ax1 + by1 + cz1 + d
.
a2 + b2 + c 2

UH
(ii) The length of the perpendicular from a point having position vector a to plane r . n = d is
|a . n−d|
p= .
|n|
JA
(iii) The coordinates of the foot (F) of perpendicular from the point (x 1, y1, z1) to the plane
x − x1 y − y1 z − z1 (ax1 + by1 + cz1 + d)
ax + by + cz + d = 0 are = = =–
a b c a2 + b2 + c 2
(iv) The coordinates of the Image (R) of point (x 1, y1, z1) to the plane
x − x1 y − y1 z − z1 2(ax1 + by1 + cz1 + d)
ax + by + cz + d = 0 are = = =–
a b c a2 + b2 + c 2
LP

Example # 46 : Find the image of the point P (3, 5, 7) in the plane 2x + y + z = 0.


Solution : Given plane is 2x + y + z = 0 ..... (1)
Direction ratios of normal to plane (1) are 2, 1, 1
Let Q be the image of point P in plane (1). Let PQ meet plane (1) in R then PQ ⊥ plane (1)
A

Let R  (2r + 3, r + 5, r + 7)
Since R lies on plane (1)
 2(2r + 3) + r + 5 + r + 7 = 0 or, 6r + 18 = 0  r=–3
NK

 R  (– 3, 2, 4)
Let Q  (, , )
Since R is the middle point of PQ
+3 +5 +7
 –3=   = – 9 and 2 =   = – 1 and 4 =  =1
2 2 2

SA

Q = (– 9, – 1, 1).

Example # 47 : A plane passes through a fixed point (a, b, c). Show that the locus of the foot of perpendicular
to it from the origin is the sphere x 2 + y2 + z2 – ax – by – cz = 0
Solution : Let the equation of the variable plane be x + my + nz + d = 0 ..... (1)
Plane passes through the fixed point (a, b, c)  a + mb + nc + d = 0 ..... (2)
Let P (, , ) be the foot of perpendicular from origin to plane (1).
Direction ratios of OP are
O(0, 0, 0)

P(, , )

 – 0,  – 0,  – 0 i.e. , , 
From equation (1), it is clear that the direction ratios of normal to the plane i.e. OP are , m, n;
, ,  and , m, n are the direction ratios of the same line OP
   1
 = = = (say)   = k, m = k, n = k ..... (3)
m n k
Putting the values of , m, n in equation (2), we get ka + kb + kc + d = 0 ..... (4)
Since , ,  lies in plane (1)   + m + n + d = 0 ..... (5)
Putting the values of , m, n from (3) in (5), we get k2 + k2 + k2 + d = 0 ..... (6)
or k2 + k2 + k2 – ka – kb – kc = 0 [putting the value of d from (4) in (6)]
or 2 + 2 + 2 – a – b – c = 0
Therefore, locus of foot of perpendicular P (, , ) is x2 + y2 + z2 – ax – by – cz = 0 ..... (7)
Self practice problems :
(57) Find the intercepts of the plane 3x + 4y – 7z = 84 on the axes. Also find the length of
perpendicular from origin to this plane and direction cosines of this normal.
(58) Find : (i) perpendicular distance (ii) foot of perpendicular

RI
(iii) image of (1, 1, 1) in the plane 3x + 4y – 12z + 13 = 0

84 3 4 −7
Answers : (57) a = 28, b = 21, c = – 12, p = ; , ,
74 74 74 74

A
17
(58) (i) (ii) (–1, 1/2, 1) (iii) (–3, 0,1)
2

UH
Angle between two planes :
(i) Consider two planes ax + by + cz + d = 0 and a x + by + cz + d = 0. Angle between these
planes is the angle between their normals. Since direction ratios of their normals are (a, b, c)
and (a, b, c) respectively, hence , the angle between them, is given by
aa'+ bb'+ cc '
JA
cos  =
a + b + c2
2 2
a' 2 + b' 2 + c ' 2
a b c
Planes are perpendicular if aa + bb + cc = 0 and planes are parallel if = =
a' b' c'

n1 . n2
(ii) The angle  between the planes r . n1 = d1 and r . n2 = d2 is given by, cos  =
LP

| n1 | | n2 |
Planes are perpendicular if n1 . n2 = 0 & planes are parallel if n1 =  n2 .

Distance between parallel planes :


A

| d1 − d2 |
Distance between two parallel planes ax + by + cz + d 1 = 0 and ax + by + cz + d2 = 0 is
a2 + b2 + c 2
NK

Example # 48 : Find the distance between the parallel planes 2x – y + 2z + 3 = 0 and 4x – 2y + 4z + 5 = 0


Solution : Given planes are 2x – y + 2z + 3 = 0 and 2x – y + 2z + 5/2 = 0
| 3 – 5/2| 1
Required distance between planes = =
2 2
(2) + (–1) + (2)2 6
SA

Angle bisectors
(i) The equations of the planes bisecting the angle between two given planes
a1x + b1y + c1z + d1 = 0 and a2x + b2y + c2z + d2 = 0 are
a1x + b1y + c1z + d1 a2 x + b2 y + c 2 z + d2
= ……(1)
2 2 2
a +b +c
1 1 1 a22 + b22 + c 22
a1x + b1y + c1z + d1 a2 x + b2 y + c 2 z + d2
= – ……..(2)
2 2 2
a +b +c
1 1 1 a22 + b22 + c 22
(ii) If a1  + b1  + c1  + d1 and a2  + b2  + c2  + d2 are of same/opposite sign then (1)/(2)
gives equation of angle bisector of region containing point (,,  )

(iii) If a1a2 + b1b2 + c1c2 > 0, then equation (1)/(2) gives obtuse/acute angle bisector
and if a1a2 + b1b2 + c1c2 < 0, then equation (1)/(2) gives acute/obtuse angle bisector.

Family of planes
(i) Any plane passing through the line of intersection of non −parallel planes or equation of
the plane through the given line in non symmetric form.
a1x + b1y + c1z + d1 = 0 & a2x + b2y + c2z + d2 = 0 is
a1x + b1y + c1z + d1 +  (a2x + b2y + c2z + d2) = 0, where   R
(ii) The equation of plane passing through the intersection of the planes r . n1 = d1 & r . n2 = d2
is. r (n1 +  n2 ) = d1 + d2 where  is arbitrary scalar

Example # 49 : Find the equation of the plane through the line of intersection of the planes x + 2y + 3z + 2 = 0,

RI
2x + 3y – z + 3 = 0 and perpendicular to the plane x + y + z = 0
Solution : The plane is x + 2y + 3z + 2 +  (2x + 3y – z + 3) = 0
or (1 + 2) x + (2 + 3) y + (3 – ) z + 2 + 3 = 0

A
It is perpendicular to x + y + z = 0
3
 1 + 2 + 2 + 3 + 3 –  = 0 or 2 + 3 = 0   = –
2

UH
Substituting we get 4x + 5y – 9 z + 5 = 0

Example # 50 : Find the equation of the plane through the point (1, 1, 1) which passes through the line of
intersection of the planes x + y + z = 6 and 2x + 3y + 4z + 5 = 0.
Solution : Given planes are x + y + z – 6 = 0 ..... (1)
JA
and 2x + 3y + 4z + 5 = 0 ..... (2)
Given point is P (1, 1, 1).
Equation of any plane through the line of intersection of planes (1) and (2) is
x + y + z – 6 + k (2x + 3y + 4z + 5) = 0 ..... (3)
If plane (3) passes through point P, then
3
1 + 1 + 1 – 6 + k (2 + 3 + 4 + 5) = 0 or, k=
LP

14
From (3) required plane is 20x + 23y + 26z – 69 = 0
Example # 51 Let planes are 2x + y + 2z = 9 and 3x – 4y + 12z + 13 = 0. Which of these bisector planes
bisects the acute angle between the given planes. Does origin lie in the acute angle or obtuse
angle between the given planes ?
A

Solution : Given planes are – 2x – y – 2z + 9 = 0 ..... (1)


and 3x – 4y + 12z + 13 = 0 ..... (2)
NK

−2x − y − 2z + 9 3x − 4y + 12z + 13
Equations of bisecting planes are =
( −2) + ( −1) + ( −2)
2 2 2
32 + ( −4)2 + (12)2
or, 13 [– 2x – y – 2z + 9] = ± 3 (3x – 4y + 12z + 13)
or, 35x + y + 62z = 78, ..... (3) [Taking +ve sign]
and 17x + 25y – 10z = 156 ..... (4) [Taking – ve sign]
SA

Now a1a2 + b1b2 + c1c2 = (– 2) (3) + (– 1) (– 4) + (– 2) (12)


= – 6 + 4 – 24 = – 26 < 0
 Bisector of acute angle is given by 35x + y + 62z = 78
 a1a2 + b1b2 + c1c2 < 0, origin lies in the acute angle between the planes.

Example # 52 : If the planes x – cy – bz = 0, cx – y + az = 0 and bx + ay – z = 0 pass through a straight line,


then find the value of a2 + b2 + c2 + 2abc.
Solution : Given planes are x – cy – bz = 0 ..... (1)
cx – y + az = 0 ..... (2)
bx + ay – z = 0 ..... (3)
Equation of any plane passing through the line of intersection of planes (1) and (2) may be
taken as x – cy – bz +  (cx – y + az) = 0
or, x (1 + c) – y (c + ) + z (– b + a) = 0 ..... (4)
If planes (3) and (4) are the same, then equations (3) and (4) will be identical.
1 + c −(c +  ) −b + a
 = =
b a −1
(i) (ii) (iii)
From (i) and (ii), a + ac = – bc – b
(a + bc)
or, =– ..... (5)
(ac + b)

From (ii) and (iii),


–(ab + c)
c +  = – ab + a2 or = ..... (6)
1 − a2
−(a + bc) −(ab + c)
From (5) and (6), we have = .
ac + b (1 − a2 )

RI
or, a – a3 + bc – a2bc = a2bc + ac2 + ab2 + bc
or, a2bc + ac2 + ab2 + a3 + a2bc – a = 0 or, a2 + b2 + c2 + 2abc = 1.

Self practice problems:

A
(59) Find the equation of plane passing through the line of intersection of the planes
2x – 7y + 4z = 3 and 3x – 5y + 4z = 11 and the point (–2, 1, 3).
(60) Find the equations of the planes bisecting the angles between the planes x + 2y + 2z – 3 = 0,

UH
3x + 4y + 12z + 1 = 0 and sepecify the plane which bisects the acute angle between them.
(61) Show that the origin lies in the acute angle between the planes
x + 2y + 2z – 9 = 0 and 4x – 3y + 12z + 13 = 0
(62) Prove that the planes 12x – 15y + 16z – 28 = 0, 6x + 6y – 7z – 8 = 0 and
JA
2x + 35y – 39z + 12 = 0 have a common line of intersection.
Answers : (59) 15x – 47y + 28z = 7
(60) 2x + 7y – 5z = 21, 11x + 19y + 31z = 18; 2x + 7y – 5z = 21
Angle between a plane and a line:
y − y1 z − z1
x − x1
(i) If  is the angle between line = = and the plane ax + by + cz + d = 0, then
m n
LP

 
sin  =  a + b m + c n .
 
 (a 2
+ b2 + c2 ) 2
+ m2 + n2 
b . n
(ii) Vector form: If  is the angle between a line r = ( a +  b ) and r . n = d then sin  =  .
A

| b | | n | 
m n
(iii) Condition for perpendicularity = = b xn = 0
a b c
NK

(iv) Condition for parallel a + bm + cn = 0 b .n = 0

Condition for a line to lie in a plane


y − y1
x − x1 z − z1
(i) Cartesian form: Line == would lie in a plane
m n
SA

ax + by + cz + d = 0, if ax1 + by1 + cz1 + d = 0 & a + bm + cn = 0.


(ii) Vector form: Line r = a +  b would lie in the plane r . n = d if b . n = 0 & a . n = d

Example # 53 : Find the distance of the point (1, 0, – 3) from the plane x – y – z = 9 measured parallel to the
x−2 y+2 z−6
line = = .
2 3 −6
Solution : Given plane is x – y – z = 9 ..... (1)
x−2 y+2 z−6
Given line AB is = = ..... (2)
2 3 −6
Equation of a line passing through the point Q(1, 0, – 3) and parallel to line (2) is
x −1 y z + 3
= = = r. ..... (3)
2 3 −6
Co-ordinates of any point on line (3) may be taken as
P (2r + 1, 3r, – 6r – 3)
If P is the point of intersection of line (3) and plane (1), then P lies on plane (1),
 (2r + 1) – (3r) – (– 6r – 3) = 9
r=1
or, P  (3, 3, – 9)
Distance between points Q (1, 0, – 3) and P (3, 3, – 9)
B

Q (1, 0, – 3)

PQ = (3 − 1)2 + (3 − 0)2 + ( −9 − (– 3))2 = 4 + 9 + 36 = 7.

Example # 54 : Find the equation of the plane passing through (1, 2, 0) which contains the line

RI
x + 3 y −1 z − 2
= = .
3 4 −2
Solution : Equation of any plane passing through (1, 2, 0) may be taken as
a (x – 1) + b (y – 2) + c (z – 0) = 0 ..... (1)

A
where a, b, c are the direction ratios of the normal to the plane. Given line is
x + 3 y −1 z − 2
= = ..... (2)
3 4 −2

UH
If plane (1) contains the given line, then
3a + 4b – 2c = 0 ..... (3)
Also point (– 3, 1, 2) on line (2) lies in plane (1)
 a (– 3 – 1) + b (1 – 2) + c (2 – 0) = 0
or, – 4a – b + 2c = 0 ..... (4)
JA
a b c
Solving equations (3) and (4), we get = =
8 − 2 8 − 6 −3 + 16
a b c
or, = = = k (say). ..... (5)
6 2 13
Substituting the values of a, b and c in equation (1), we get
6 (x – 1) + 2 (y – 2) + 13 (z – 0) = 0.
or, 6x + 2y + 13z – 10 = 0. This is the required equation.
LP

x −1 y +1 z − 3
Example # 55 : Find the equation of the projection of the line = = on the plane x + 2y + z = 9.
2 −1 4
x −1 y +1 z − 3
Solution : Let the given line AB be = = ..... (1)
A

2 −1 4
Given plane is x + 2y + z = 9 ..... (2)
Let DC be the projection of AB on plane (2)
Clearly plane ABCD is perpendicular to plane (2).
NK

Equation of any plane through AB may be taken as (this plane passes through the point
(1, – 1, 3) on line AB)
a (x – 1) + b (y + 1) + c (z – 3) = 0 ..... (3)
where 2a – b + 4c = 0 ..... (4)
[normal to plane (3) is perpendicular to line (1)]
SA

Since plane (3) is perpendicular to plane (2),


 a + 2b + c = 0 ..... (5)
a b c
Solving equations (4) & (5), we get = = .
−9 2 5
Substituting these values of a, b and c in equation (3), we get
9 (x – 1) – 2 (y + 1) – 5 (z – 3) = 0
or, 9x – 2y – 5z + 4 = 0 ...... (6)
Since projection DC of AB on plane (2) is the line of intersection of plane ABCD and plane (2),
therefore equation of DC will be
9x − 2y − 5z + 4 = 0 .....(i) 
and  ..... (7)
x + 2y + z − 9 = 0 .....(ii)
Let , m, n be the direction ratios of the line of intersection of planes (i) and (ii)
 9– 2m – 5n = 0 ..... (8) and + 2m + n = 0 ..... (9)
m n m n
 = =  = =
−2 + 10 −5 − 9 18 + 2 4 –7 10

Let any point on line (7) is (, , 0)  9 – 2 + 4 = 0


1 17
 + 2 – 9 = 0  = , =
2 4
1 17
x– y–
So equation of line is 2 = 4 = z–0
4 –7 10

Self practice problems :


x−2 y+3 z−6
(63) Find the values of a and b for which the line = = is perpendicular to the plane
a 4 −2

RI
3x – 2y + bz + 10 = 0.

x −1 y − 2 z − 3
(64) Find the equation of the plane containing the lines = = and
2 3 3

A
x−2 y−3 z−4
= = .
3 4 5
x−2 y−3 z−4

UH
(65) Find the plane containing the line = = and parallel to the line
2 3 5
x +1 y −1 −z + 1
= =
1 −2 1
x −1 y−2 z−3 x−4 y −1
(66) Show that the line = = & = = z are intersecting each other.
2 3
JA4 5 2
Find their intersection point and the plane containing the line.
Answers : (63) a = –6, b = 1 (64) 3x – y – z + 2 = 0
(65) 13x + 3y – 7z – 7 = 0 (66) (– 1, – 1, – 1) & 5x – 18y + 11z – 2 = 0

Non-symmetrical form of line :


A straight line in space is characterised by the intersection of two planes which are not parallel and
LP

therefore, the equation of a straight line is a solution of the system constituted by the equations of the
two planes, P1  a1x + b1y + c1z + d1 = 0 and P2  a2x + b2y + c2z + d2 =0. This form is also known as
non-symmetrical form.
A
NK
SA

To find the equation of the line in symmetrical form, we must know (i) its direction ratios (ii) coordinate
of any point on it.
(i) Direction ratios: Let , m, n be the direction ratios of the line. Since the line lies in both the
planes, it must be perpendicular to normals of both planes.
So a1 + b1m + c1n = 0, a2 + b2m + c2n = 0. From these equations, proportional values of
m n
, m, n can be found by cross-multiplication as = =
b1c 2 − b2c1 c1a2 − c 2a1 a1b2 − a2b1
(ii) Point on the line − Note that as , m, n cannot be zero simultaneously, so at least one must
be non-zero. Let a1b2 − a2b1  0, then the line cannot be parallel to xy plane, so it intersect it.
Let it intersect xy-plane in (x1, y1, 0). Then a1x1 + b1y1 + d1 = 0 and a2x1 + b2y1+ d2= 0.Solving
these, we get a point on the line.

Note : If   0, then we can take a point on yz-plane as (0, y 1, z1) and if m  0, then we can take a point on xz-
plane as (x1, 0, z1).

Coplanar lines : Condition of coplanarity if both the lines are in general form Let the lines be
ax + by + cz + d = 0 = ax + by + cz + d & x + y + z +  = 0 = x + y + z + 
a b c d

RI
a' b' c ' d'
They are coplanar if =0
   
' '  ' '

A
Example # 56 : Find the equation of the line of intersection of planes 4x + 4y – 5z = 12, 8x + 12y – 13z = 32
in the symmetric form.

UH
Solution : Given planes are 4x + 4y – 5z – 12 = 0 ..... (1)
and 2x – 3y + 4z = 5 ..... (2)
Let , m, n be the direction ratios of the line of intersection :
then 4– 4m – 3n = 0 ..... (3)
m n m n
and 42– 12m + 13n = 0  = = or, = =
−8 + 9 6 – 4 –3 + 4
JA 1 2 1
Hence direction ratios of line of intersection are 1, 2, 1.
Let the line of intersection meet the xy-plane at P (, , 0).
Then P lies on planes (1) and (2)
  – = 4 or, 2 –3 =5 ..... (5)
 = – 2 ,  = –3
x+2 y+3 z
LP

Hence equation of line of intersection in symmetrical form is = = .


1 2 1

Example # 57 : Find the angle between the lines x – 3y – 4 = 0, 4y – z + 5 = 0 and


x + 3y – 11 = 0, 2y – z + 6 = 0.
x − 3y − 4 = 0 
A

Solution : Given lines are  ..... (1)


4y − z + 5 = 0 
x + 3y − 11 = 0 
and  ..... (2)
2y − z + 6 = 0 
NK

Let 1, m1, n1 and 2, m2, n2 be the direction cosines of lines (1) and (2) respectively
 line (1) is perpendicular to the normals of each of the planes
x – 3y – 4 = 0 and 4y – z + 5 = 0
 1 – 3m1 + 0.n1 = 0 ..... (3) and 01 + 4m1 – n1 = 0 ..... (4)
SA

m1 n
Solving equations (3) and (4), we get 1
= = 1
3−0 0 − ( −1) 4 −0
1 m n
or, = 1 = 1 = k (let).
3 1 4
Since line (2) is perpendicular to the normals of each of the planes
x + 3y – 11 = 0 and 2y – z + 6 = 0,
 2 + 3m2 = 0 ..... (5) and 2m2 – n2 = 0 ..... (6)
n
 2 = – 3m2 or, 2 = m2 and n2 = 2m2 or, 2 = m2.
−3 2
m n
 2
= 2 = 2 = t (let).
−3 1 2
If  be the angle between lines (1) and (2), then cos = 12 + m1m2 + n1n2
= (3k) (– 3t) + (k) (t) + (4k) (2t) = – 9kt + kt + 8kt = 0   = 90°.
x − 3 y +1 z + 2 x–7 y z+7
Example # 58 : Show that the lines = = and = = are coplanar. Also find the
2 −3 1 –3 1 2
equation of the plane containing them.
x − 3 y +1 z + 2 x −7 y z+7
Solution : Given lines are = = = r (say)..... (1) and = = = R (say) ..... (2)
2 −3 1 −3 1 2
If possible, let lines (1) and (2) intersect at P.
Any point on line (1) may be taken as (2r + 3, – 3r – 1, r – 2) = P (let).
Any point on line (2) may be taken as (– 3R + 7, R, 2R – 7) = P (let).
 2r + 3 = – 3R + 7 or, 2r + 3R = 4 ..... (3)
Also – 3r – 1 = R or, – 3r – R = 1 ..... (4)

RI
and r – 2 = 2R – 7 or, r – 2R = – 5. ..... (5)
Solving equations (3) and (4), we get, r = – 1, R = 2
Clearly r = – 1, R = 2 satisfies equation (5).

A
Hence lines (1) and (2) intersect.
 lines (1) and (2) are coplanar.

UH
x − 3 y +1 z + 2
Equation of the plane containing lines (1) and (2) is 2 −3 1 =0
−3 1 2

or, (x – 3) (– 6 – 1) – (y + 1) (4 + 3) + (z + 2) (2 – 9) = 0
JA
or, – 7 (x – 3) – 7 (y + 1) – 7 (z + 2) = 0 or, x – 3 + y + 1 + z + 2 = 0 or, x + y + z = 0.

Self practice problems:

(67) Find the equation of the line of intersection of the plane


LP

x – y + 2z = 5, 3x + y + z = 6.

(68) Prove that the three planes 2x + y – 4z – 17 = 0, 3x + 2y – 2z – 25 = 0, 2x – 4y + 3z + 25 = 0


A

intersect at a point and find its co-ordinates.


NK

4x – 11 4y + 9 z
Answers : (67) = = (68) (3, 7, –1)
–33 5 1
SA
 Marked questions are recommended for Revision.

PART - I : SUBJECTIVE QUESTIONS

Section (A) : Position vector, Direction Ratios & Direction cosines


 
A-1. (i) Let position vectors of points A,B and C are a , b and c respectively. Point D divides line
segment BC internally in the ratio 2 : 1. Find vector AD .

(ii) Let ABCD is parallelogram. Position vector of points A,C and D are a , c and d respectively .

RI
If E divides line segement AB internally in the ratio 3 : 2 then find vector DE .

(iii) Let ABCD is trapezium such that AB = 3 DC . E divides line segement AB internally in the ratio
2 : 1 and F is mid point of DC. If position vector of A,B and C are a,b and c respectively then

A
find vector FE .

In a ABC, AB = 6iˆ + 3jˆ + 3kˆ ; AC = 3iˆ −3ˆj +6kˆ

UH
A-2.
D and D are points trisections of side BC
Find AD and AD .    
A-3. If ABCD is a quadrilateral, E and F are the mid-points of AC and BD respectively, then prove that
JA
AB + AD + CB + CD = 4EF

A-4. Let ABCD is parallelogram where A = (1,2,4) , B = (8,7,9) and D = (6,1,5) . Find direction cosines of line
AC.

A-5. Find the direction cosines , m, n of line which are connected by the relations
 + m + n = 0, 2mn + 2m – n = 0.
LP

 
Section (B) : Dot Product, Projection and Cross Product
 
B-1. Show that the points A, B, C with position vectors 2iˆ − ˆj + kˆ , ˆi − 3jˆ − 5kˆ and 3iˆ − 4jˆ − 4kˆ respectively
A

are the vertices of a right angled triangle. Also find the remaining angles of the triangle.

B-2. If a, b, c are three mutually perpendicular vectors of equal magnitude, prove that a + b + c is equally
NK

inclined with vectors a, b and c .

B-3. (i) Find the projection of b + c on a where a = ˆi + 2jˆ + kˆ , b = ˆi + 3jˆ + kˆ and c = ˆi + kˆ .


(ii) Find the projection of the line segment joining (2, – 1, 3) and (4, 2, 5) on a line which makes
SA

equal acute angles with co-ordinate axes.

(iii) P and Q are the points (–1, 2, 1) and (4, 3, 5) respectively. Find the projection of PQ on a line
which makes angles of 120º and 135º with y and z axes respectively and an acute angle with x-
axis.

2 2
 a b   a−b 
Prove that  2 − 2  = 
 | a | | b | 
B-4.
a 
 b   
B-5. If a, b are two unit vectors and  is the angle between them, then show that:
 1  1
(a) sin = a−b (b) cos = a+b
2 2 2 2
B-6. If two vectors a and b are such that | a | = 2, | b | = 1 and a·b = 1, then find the value of (3a – 5b) ·
(2a + 7b) .

B-7. For any two vectors u & v , prove that


(a) (u.v)2 + | u  v |2 = | u |2 | v |2 & (b) (1+ | u |2 )(1+ | v |2 ) = (1− u.v)2 + | u + v + (u  v) |2

B-8. If the three successive vertices of a parallelogram have the position vectors as,
A (− 3, − 2, 0); B (3, − 3, 1) and C ( 5, 0, 2). Then find
(a) position vector of the fourth vertex D
(b) a vector having the same direction as that of AB but magnitude equal to AC
(c) the angle between AC and BD .

RI
B-9. If a,b,c are three vectors such that | a | = 5, | b | = 12 and | c | = 13, and a + b + c = o , find the value of
a.b + b.c + c.a .

A
B-10. ABCD is a parallelogram in which AB = 3iˆ −6ˆj +3kˆ and AD = 6iˆ +6ˆj +3kˆ . P is a point of AB such
that AP : PB = 1 : 2 and Q is a point on BC such that BQ : QC = 2 : 1. Find angle between DQ and PC.

UH
B-11. Prove using vectors : If two medians of a triangle are equal, then it is isosceles.

B-12. (i) Find the angle between the lines whose direction cosines are given by the equations :
3 + m + 5n = 0 and 6 mn – 2n + 5 m = 0
(ii) Find the angle between the lines whose direction cosines are given by  + m + n = 0 and
JA
2 + m2 = n2.


B-13. Position vectors of A, B, C are given by a, b, c where a  b + b c + c  a = 0. If AC = 2iˆ – 3jˆ + 6kˆ

then find BC if BC = 14.
LP

B-14. A vector c is perpendicular to the vectors 2 ˆi + 3 ˆj − kˆ , ˆi − 2 ˆj + 3kˆ and satisfies the

( )
condition c . 2 ˆi − ˆj + kˆ + 6 = 0. Find the vector c

B-15. (a) Show that the perpendicular distance of the point c from the line joining a and b is
A

bc + c a + a  b
.
b−a
NK

(b) Given a parallelogram ABCD with area 12 sq. units. A straight line is drawn through the mid
point M of the side BC and the vertex A which cuts the diagonal BD at a point ' O '. Use vectors
to determine the area of the quadrilateral OMCD.
 
B-16. P, Q are the mid-points of the non-parallel sides BC and AD of a trapezium ABCD. Show that
SA

APD = CQB.

Section (C) : Line


 
C-1. Find the coordinates of the point when the line through (3, 2, 5) and (–2, 3, –5) crosses the xy plane.
 
x y −1 z−2
C-2. Find the foot of the perpendicular from (1, 6, 3) on the line = = .
1 2 3

C-3. (i) Find the cartesian form of the equation of a line whose vector form is given by .
r = 2iˆ − ˆj + 4kˆ + (iˆ + ˆj − 2k)
ˆ
(ii) Find the vector form of the equation of a line whose cartesian form is given by
2x − 4 3y + 6 –6z + 6
= = .
1 2 1
 
C-4. Find the distance between points of intersection of
x −1 y−2 z−3 x−4 y −1
Lines = = & = =z and
2 3 4 5 2
Lines r = ( î + ĵ − k̂ ) +  (3 î − ĵ ) & r = (4 î − k̂ ) +  (2 î + 3 k̂ )

C-5. Show that the foot of the perpendicular from the origin to the join of A(–9, 4, 5) and B (11, 0, –1) is the
mid point of AB. Also find distance of point (2, 4, 4) from the line AB.

x−3 y−3 z
C-6. Find the equation of the two lines through the origin which intersect the line = = at an

RI
2 1 1
angle of /3.

C-7. The foot of the perpendicular from (a, b, c) on the line x = y = z is the point (r, r, r), then find the value of
r.

A
C-8. Find the shortest distance between the lines :
r = (4iˆ − ˆj) +  (iˆ + 2jˆ − 3k)
ˆ and r = (iˆ − ˆj + 2k)
ˆ +  (2iˆ + 4jˆ − 5k)
ˆ

UH
x−3 y−8 z−3 x+3 y+7 z−6
C-9. Let L1 and L2 be the lines whose equation are = = and = =
3 −1 1 −3 2 4
respectively. A and B are two points on L 1 and L2 respectively such that AB is perpendicular both the
JA
lines L1 and L2.Find points A, B and hence find shortest distance between lines L 1 and L2.

C-10. If r = (iˆ + 2jˆ + 3k)


ˆ +  (iˆ − ˆj + k)
ˆ and r = (iˆ + 2jˆ + 3k)
ˆ +  (iˆ + ˆj − k)
ˆ are two lines, then find the equation
of acute angle bisector of two lines.

C-11. The edges of a rectangular parallelopiped are a, b, c; show that the angles between two of the four
LP

  a2 + b2 – c 2     a2 – b2 + c 2     a2 – b2 – c 2  
diagonals are given by cos−1    2 2 2  or cos −1 
 
  a2 + b2 + c 2   or cos −1 
  2 2 2  .
  a + b + c       a + b + c 
 
C-12.Show that equation of angle bisectors of line r = a + b and r = b + µ a are r = (a + b) +  b a  a b ( )
A

C-13. Prove that the shortest distance between the diagonals of a rectangular parallelepiped whose
NK

bc ca ab
coterminous sides are a, b, c and the edges not meeting it are , ,
2 2 2 2
b +c c +a a + b2
2

Section (D) : STP, VTP, Vector equation, LI/LD


Show that {( a + b + c ) × ( c – b )} . a = 2 a b c  .
SA

D-1.

  
D-2. ˆ nˆ  = pˆ  ( m
Given unit vectors m̂ , n̂ and p̂ such that  m ˆ x nˆ ) =  then find value of
 
ˆ  in terms of .
nˆ pˆ m

D-3. Let a=a1ˆi+a2 ˆj+a3kˆ , b=b1ˆi+b2 ˆj+b3kˆ and c=c1ˆi+c 2 ˆj+c 3 kˆ be three non-zero vectors such that c

is a unit vector perpendicular to both a and b . If the angle between aandb is , then
6
2
a1 a2 a3
b1 b2 b3 is equal to:
c1 c 2 c3
D-4. Examine for coplanarity of the following sets of points
(a) 4 î + 8 ĵ + 12 k̂ , 2 î + 4 ĵ + 6 k̂ , 3 î + 5 ĵ + 4 k̂ , 5 î + 8 ĵ + 5 k̂ .
(b) 3 a + 2 b – 5 c , 3 a + 8 b + 5 c , –3 a + 2 b + c , a + 4 b – 3 c . Where a , b , c are noncoplanar
D-5. The vertices of a tetrahedron are P(2, 3, 2), Q(1, 1, 1), R(3, –2, 1) and S (7, 1, 4).
(i) Find the volume of tetrahedron
(ii) Find the shortest distance between the lines PQ & RS.
D-6. Are the following set of vectors linearly independent?
(i) a = î − 2 ĵ + 3 k̂ , b = 3 î − 6 ĵ + 9 k̂
(ii) a = −2 î − 4 k̂ , b = î − 2 ĵ − k̂ , c = î − 4 ĵ + 3 k̂

D-7. Find value of xR for which the vectors a = (1, –2, 3), b = (–2, 3, – 4), c = (1, – 1, x) form a linearly

RI
dependent system.

D-8. If a , b , c are non-coplanar vectors and v . a = v . b = v . c = 0, then find value of v.

Let a = ˆi + 2 ˆj + 3kˆ , b = 2 ˆi − ˆj + kˆ , c = 3 ˆi + 2 ˆj + kˆ and d = 3 ˆi − ˆj − 2kˆ , then

A
D-9.
(i) if a  (b  c) = pa + qb + rc , then find value of p, q and r.
(ii) find the value of ( a × b ) × ( a × c ). d

UH
1 1
D-10. Given that x + 2
(p . x) p = q , then show that p.x = (p.q) and hence find x in terms of p and q .
p 2

D-11. Let there exist a vector x satisfying the conditions x × a = c  d and x + 2 d = v  d . Find x in ( )
JA
terms of a , c and d

Section (E) : Plane


E-1. Find equation of plane
(i) Which passes through (0, 1, 0), (0, 0, 1), (1, 2, 3)
LP

(ii) Which passes through (0, 1, 0) and contains two vectors ˆi + ˆj − kˆ & 2iˆ − ˆj .

(iii) Whose normal is ˆi + ˆj + kˆ & which passes through (1, 2, 1).


(iv) Which makes equal intercepts on co-ordinate axis and passes through (1, 2, 3)
A

E-2. Find the ratio in which the line joining the points (3, 5,–7) and (–2, 1, 8) is divided by the yz-plane. Find
also the point of intersection of the plane and the line.
NK

E-3. Find the locus of the point whose sum of the square of distances from the planes x + y + z = 0, x – z = 0
and x – 2y + z = 0 is 9.

E-4. The foot of the perpendicular drawn from the origin to the plane is (4, –2, –5), then find the vector
equation of plane.
SA

E-5. Let P (1, 3, 5) and Q(–2, 1, 4) be two points from which perpendiculars PM and QN are drawn to the x-z
plane. Find the angle that the line MN makes with the plane x + y + z = 5.

x –1 y–2 z–3
E-6. The reflection of line = = about the plane x – 2y + z – 6 = 0 is
3 4 5

x −1 y−2 z+3
E-7. Find the equation of image of the line = = in the plane 3x – 3y + 10z = 26.
9 −1 −3

E-8. Find the angle between the plane passing through points (1, 1, 1), (1, − 1, 1), (−7, −3, −5) & x−z plane.

3−y z−2
E-9. Find the equation of the plane containing parallel lines (x − 4) = = and
4 5
(x − 3) =  (y + 2) = z
E-10. Find the distance of the point (2, 3, 4) from the plane 3x + 2y + 2z + 5 = 0, measured parallel to the line
x+3 y−2 z
= =
3 6 2

E-11. If the acute angle that the vector, ˆi + ˆj + kˆ makes with the plane of the two vectors
2 ˆi + 3 ˆj − kˆ and ˆi − ˆj + 2kˆ is cot −1 2 then find the value of  ( + ) –  

E-12. Find the equation of the plane passing through the points (3, 4, 1) and (0, 1, 0) and parallel to the line
x+3 y–3 z–2
= = .
2 7 5

E-13. Find the vector equation of a line passing through the point with position vector (2iˆ – 3jˆ – 5k)
ˆ and

RI
perpendicular to the plane r.(6iˆ – 3jˆ + 5k)
ˆ + 2 = 0.
Also, find the point of intersection of this line and the plane.

A
E-14. Find the equation of the plane passing through the point (1, 2, 1) and perpendicular to the line joining
the points (1, 4, 2) and (2, 3, 5). Also find the coordinates of the foot of the perpendicular and the
perpendicular distance of the point (4, 0, 3) from the above found plane.

UH
E-15. Find the equation of the planes passing through points (1, 0, 0) and (0, 1, 0) and making an angle of
0.25  radians with plane x + y − 3 = 0

E-16. Find the distance between the parallel planes r . (2iˆ − 3jˆ + 6k)
ˆ = 5 and r . (6iˆ − 9jˆ + 18k)
ˆ + 20 = 0.
JA
E-17. If the planes x – cy – bz = 0, cx – y + az = 0 and bx + ay – z = 0 pass through a straight line, then find
the value of a2 + b2 + c2 + 2abc is :

E-18. (i) If n̂ is the unit vector normal to a plane and p be the length of the perpendicular from the origin
to the plane, find the vector equation of the plane.
(ii) Find the equation of the plane which contains the origin and the line of intersection of the
planes r . a = p and r . b = q
LP

PART - II : ONLY ONE OPTION CORRECT TYPE


Section (A) : Position vector, Direction Ratios & Direction cosines
A

 
A-1. ( )
If the vector b is collinear with the vector a = 2 2 , −1, 4 and b = 10, then:
NK

(A) a  b = 0 (B) a  2b = 0 (C) 2a  b = 0 (D) 3a  b = 0

A-2. OABCDE is a regular hexagon of side 2 units in the XY−plane as shown in figure . O being the origin
and OA taken along the X−axis. A point P is taken on a line parallel to Z−axis through the centre of the
hexagon at a distance of 3 units from O in the positive Z direction. Then vector AP is:
SA

D C

E • B

x
O A

  z
(A) − ˆi + 3 ˆj + 5 kˆ (B) ˆi − 3 ˆj + 5kˆ (C) − ˆi + 3 ˆj + 5 kˆ (D) ˆi + 3 ˆj + 5 kˆ
A-3. If the sum of the squares of the distances of a point from the three coordinate axes be 36, then its
distance from the origin is
(A) 6 (B) 3 2 (C) 2 3 (D) 6 2

A-4. A line makes angles    with the coordinate axes. If  + = 90º, then  =
(A) 0 (B) 90º (C) 180º (D) 45°

Section (B) : Dot Product, Projection and Cross Product


1 ˆ
B-1. The vector
2
( )
2 i − 2 ˆj + kˆ is:


(A) a unit vector (B) makes an angle with the vector 2 ˆi − 4 ˆj + 3kˆ
3

RI
(C) parallel to the vector ˆi + ˆj − 1 kˆ (D) perpendicular to the vector 3 ˆi + 2 ˆj − 2kˆ
2

B-2. If | a | = 5, | a – b | = 8 and | a + b | = 10, then | b | is equal to :

A
(A) 1 (B) 57 (C) 3 (D) 57

B-3. If a  b = c , b  c = a , then find value of | 3a + 4b + 12c | if a, b, c are vectors of same magnitude.

UH
(A) 11 (B) 12 (C) 13 (D) 14

B-4. Angle between diagonals of a parallelogram whose side are represented by a = 2iˆ + ˆj + kˆ and
b = ˆi − ˆj − kˆ
JA
 1 3 8 4
(A) cos–1   (B) cos–1   (C) sin–1 (D) tan–1  
2 5 9 3
B-5. A, B, C & D are four points in a plane with position vectors a , b , c & d respectively such that

(a − d) . (b − c ) = (b − d) . (c − a ) = 0. Then for the triangle ABC, D is its :


(A) incentre (B) circumcentre (C) orthocentre (D) centroid
LP

B-6. Given a = 10, b = 2 and a.b = 12, then find a  b .


(A) 12 (B) 16 (C) 8 (D) 32
A

B-7. Unit vector perpendicular to the plane of the triangle ABC with position vectors of the vertices A, B, C,
is (where  is the area of the triangle ABC).
(a x b + b x c + c x a) (a x b + b x c + c x a)
NK

(A) (B)
 2

(C)
(a x b + b x c + c x a) (D)
(a x b + c  b + c x a)
4 2
SA

B-8. ABC is a triangle where A = (2, 3, 5), B = (–1, 2, 2) and C(, 5, ), if the median through A is equally
inclined to the positive axes, then  +  is
(A) 7 (B) 6 (C) 15 (D) 9

Section (C) : Line


C-1. ( )
If a line has a vector equation r = 2 ˆi + 6 ˆj +  ˆi − 3 ˆj , then which of the following statement(s) is/are
NOT correct?
(A) the line is parallel to 2 ˆi + 6 ˆj (B) the line passes through the point 3 ˆi + 3 ˆj
(C) the line passes through the point ˆi + 9 ˆj (D) the line is parallel to XY-plane

C-2. Let a = î + ĵ and b = 2 î – k̂ . The point of intersection of the lines r × a = b × a and


r × b = a × b is :
(A) – î + ĵ + 2 k̂ (B) 3 î – ĵ + k̂ (C) 3 î + ĵ – k̂ (D) î – ĵ – k̂

x −1 y − 2 z x+7 y z+
C-3. The values of ‘’ for which the two lines = = & = = are coplaner
4 1 1  −6 2
(A) 2, 8 (B) 2, –8 (C) 3, 5 (D) 1, 2

x −1 y−2 z−3
C-4. Equation of the angle bisector of the angle between the lines = = &
1 1 1
x −1 y−2 z−3
= = is :
1 1 −1
x −1 y−2 x −1 y−2 z−3
(A) = ;z –3=0 (B) = =
2 2 1 2 3

RI
y−2 z−3 x −1 y−2
(C) x – 1 = 0 ; = (D) = ;z –3=0
1 1 2 3

x y z x −1 y−2 z−3 x+k y −1 z−2

A
C-5. If the lines = = , = = and = = are concurrent then
1 2 3 3 −1 4 3 2 h
1 1
(A) h = – 2, k = – 6 (B) h = , k = 2 (C) h = 6, k = 2 (D) h = 2, k =
2 2

UH
C-6. Points X and Y are taken on the sides QR and RS, respectively of a parallelogram PQRS, so that
QX = 4XR and RY = 4YS. The line XY cuts the line PR at Z. Find the ratio PZ : ZR.
(A) 4 : 21 (B) 3 : 4 (C) 21 : 4 (D) 4 : 3
 
JA
Section (D) : STP, VTP, Vector equation, LI/LD
 

D-1. The value of  a + 2b − c


( ) (a − b ) (a − b − c  is equal to the box product :
)
 
(A) a b c  (B) 2 a b c  (C) 3 a b c  (D) 4 a b c 
       
LP

D-2. For a non zero vector A if the equations A . B = A . C and A  B = A  C hold simultaneously,
then :
(A) A is perpendicular to B − C (B) A = B
(C) B = C (D) C = A
A

D-3. Let a = xiˆ + 12jˆ − kˆ , b = 2iˆ + 2xjˆ + kˆ and c = ˆi + kˆ . If the ordered set  b c a  is left handed, then :
 
NK

(A) x  (2, ) (B) x  (− , − 3) (C) x  (− 3, 2) (D) x  {− 3, 2}

D-4. If a = i + j – k, b = i – j + k, c is a unit vector such that c.a = 0, [c ab] = 0 then a unit vector d both a
and c is perpendicular to
1 1 1 1
SA

(A) (2i – j + k) (B) (j + k) (C) (i + j) (D) (i + k)


6 2 2 2

D-5. If a = – i + j + k and b = 2i + k, then the vector c satisfying the conditions.


(i) that it is coplanar with a and b
(ii) that its projection on b is 0
(A) –3i + 5j + 6k (B) – 3i – 5j + 6k (C) – 6i + 5k (D) – i + 2j + 2k

D-6. If a x b = c x d and a x c = b x d , then the vectors a − d and b − c are :


(A) non-collinear (B) linearly independent
(C) perpendicular (D) parallel
 
D-7. Vector of length 3 unit which is perpendicular to ˆi + ˆj + kˆ and lies in the plane of ˆi + ˆj + kˆ and 2iˆ − 3 ˆj , is
3 3 3 3
(A) ( ˆi − 2jˆ + kˆ ) (B) ( 2iˆ − ˆj − kˆ ) (C) ( 7iˆ + 8 ˆj + kˆ ) (D) ( −7iˆ + 8jˆ − kˆ )
6 6 114 114
 
D-8. If a , b , c be the unit vectors such that b is not parallel to c and a x 2b x c = b , then the angle ( )
that a makes with b and c are respectively :
   2  2  
(A) & (B) & (C) & (D) &
3 4 3 3 2 3 2 3

D-9. If a , b , c are linearly independent vectors, then which one of the following set of vectors is linearly
dependent?
(A) a + b , b + c , c + a (B) a − b , b − c , c − a (C) a xb , b xc , c xa (D) a + 2b + 3c, b − c + a, a + c

RI
D-10. Let a, b, c are three non-coplanar vectors such that r1 = a − b + c , r2 = b + c − a , r3 = c + a + b ,
r = 2a − 3b + 4c . If r = 1r1 + 2 r2 + 3 r3 , then the values of 1 , 2 and 3 respectively are

A
(A) 7, 1, –4 (B) 7 / 2, 1, –1 / 2 (C) 5 / 2, 1, 1/2 (D) –1 / 2, 1, 7 / 2

D-11. Vector x satisfying the relation A.x = c and A  x = B is

UH
cA − (A  B) cA − (A  B) cA + (A  B) cA − 2(A  B)
(A) (B) 2
(C) 2
(D)
|A| |A| |A| | A |2

D-12. The value of r if exist where r = a + b and r  c = d is


 a.d   a.d   a.d   a.d 
JA
(A) a +   b (B) a –   b (C)   a – b (D)   a + b
 b.d   b.d   b.d   b.d 

Section (E) : Plane


E-1. The equation of a plane which passes through (2, − 3, 1) & is perpendicular to the line joining the points
(3, 4, − 1) & (2, − 1, 5) is given by:
LP

(A) x + 5y − 6z + 19 = 0 (B) x − 5y + 6z − 19 = 0
(C) x + 5y + 6z + 19 = 0 (D) x − 5y − 6z − 19 = 0

E-2. The reflection of the point (2, –1, 3) in the plane 3x – 2y – z = 9 is :


 26 15 17   26 −15 17   15 26 −17   26 17 −15 
A

(A)  , ,  (B)  , ,  (C)  , ,  (D)  , , 


 7 7 7   7 7 7   7 7 7   7 7 7 
x − 2 y +1 z − 2
E-3. The distance of the point (− 1, − 5, − 10) from the point of intersection of the line, = =
3 4 12
NK

and the plane, x − y + z = 5, is :


(A) 10 (B) 11 (C) 12 (D) 13
E-4. The distance of the point (1, − 2, 3) from the plane x − y + z = 5 measured parallel to the line,
x y z
= = , is :
2 3 −6
SA

(A) 1 (B) 6/7 (C) 7/6 (D) 1/6


x −1 y−3 z−2
E-5. The distance of the point P(3, 8, 2) from the line = = measured parallel to the plane
2 4 3
3x + 2y – 2z + 17 = 0 is
(A) 2 (B) 3 (C) 5 (D) 7

E-6. If line r = ( î −2 ĵ − k̂ ) + (2 î + ĵ +2 k̂ ) is parallel to the plane r .(3 î −2 ĵ −m k̂ )=14, then the value of m is
(A) 2 (B) – 2
(C) 0 (D) can not be predicted with these informations
E-7. The locus represented by xy + yz = 0 is
(A) A pair of perpendicular lines (B) A pair of parallel lines
(C) A pair of parallel planes (D) A pair of perpendicular planes
E-8. The equation of the plane passing through the point ( 1,−3,−2) and perpendicular to planes
x + 2y + 2z = 5 and 3x + 3y + 2z = 8, is
(A) 2x − 4y + 3z − 8 = 0 (B) 2x − 4y − 3z + 8 = 0 (C) 2x + 4y + 3z + 8 = 0 (D) 2x + 4y + 3z − 8 = 0
E-9. If a plane cuts off intercepts OA = a, OB = b, OC = c from the coordinate axes (where 'O' is the origin),
then the area of the triangle ABC is equal to
1 1
(A) b2c 2 + c 2a2 + a2b2 (B) (bc + ca + ab)
2 2
1 1
(C) abc (D) (b − c)2 + (c − a)2 + (a − b)2
2 2
E-10. Given the vertices A (2, 3, 1), B (4, 1, −2), C (6, 3, 7) & D (−5, −4, 8) of a tetrahedron. The length of the
altitude drawn from the vertex D is:
(A) 7 (B) 9 (C) 11 (D) 13

RI
PART - III : MATCH THE COLUMN
1. Column –  Column – 
(A) If the vectors a = 3 ˆi + ˆj − 2kˆ , b = −ˆi + 3jˆ + 4kˆ and (p) 2

A
c = 4iˆ − 2ˆj − 6kˆ constitute the sides of a ABC
and length of the median bisecting the vector c is , then 2

UH
(B) Let p is the position vector of the orthocentre and g is the (q) 3
position vector of the centroid of the triangle ABC, where
circumcentre is the origin. If p = K g , then K is equal to :
JA
(C) Twice of the area of the parallelogram constructed on (r) 6
the vectors a = p + 2q and b = 2 p + q , where p and
q are unit vectors containing an angle of 30º, is :

(D) Let u , v and w are vector such that u + v + w = 0 . (s) 5


LP

If | u | = 3 , | v | = 4 , | w | = 5 then u.v + v.w + w.u is

2. Match the following set of lines to the corresponding type :


x −1 y − 2 z − 3 x −1 y − 2 z − 3
(A) = = & = = (p) parallel but not coincident
1 2 3 2 2 −2
A

x −1 y − 2 z − 3 x−5 y−2 z−3


(B) = = & = = (q) intersecting
1 2 3 1 2 3
x −1 y − 2 z − 3 x − 3 y − 4 z −1
NK

(C) = = & = = (r) skew lines


2 2 −2 −1 −1 1
x −1 y − 2 z − 3 x y +1 z
(D) = = & = = (s) Coincident
2 2 3 2 3 1
 
Column –  Column – 
SA

3.
(A) The volume of the parallelopiped constructed on
the diagonals of the faces of the given rectangular (p) –3
parallelopiped is m times the volume of the given parallelopiped.
Then m is equal to

(B) If x satisfying the conditions b.x =  & b  x = a

is x =
( 2
− 12 b) +
ab
(q) 2
2
|b| | b |2
then  can be
(C) The points (0, –1, –1), (4, 5, 1), (3, 9, 4) and (r) 4
(–4, 4, k) are coplanar, then k =

(D) In ABC the mid points of the sides AB, BC and CA are (s) 8
respectively (, 0, 0) (0, m ,0) and (0, 0, n).
AB2 + BC2 + CA 2
Then 2
is equal to
+ m2 + n2

 Marked questions are recommended for Revision.

PART - I : ONLY ONE OPTION CORRECT TYPE


1. Let a, b, c be vectors of length 3, 4, 5 respectively. Let a be perpendicular to b + c , b to c + a and

RI
c to a + b . Then a + b + c is equal to :

(A) 2 5 (B) 2 2 (C) 10 5 (D) 5 2

2. Four coplanar forces are applied at a point O. Each of them is equal to k and the angle between two

A
consecutive forces equals 45º as shown in the figure. Then the resultant has the magnitude equal to :

UH
(A) k 2 + 2 2 (B) k 3 + 2 2 (C) k 4 + 2 2 (D) k 4 − 2 2

1
3.
JA
Taken on side AC of a triangle ABC, a point M such that AM = AC . A point N is taken on the side
3
CB such that BN = CB , then for the point of intersection X of AB and MN which of the following
holds good?
1 1 3
(A) XB = AB (B) AX = AB (C) XN = MN (D) XM = 3 XN
3 3 4
 
LP

4. If 3 non zero vectors a, b, c are such that a  b = 2(a  c) , | a | = | c | = 1; | b | = 4 the angle between
1
b and c is cos–1 then b = c + µa where |  | + | µ | is -
4
(A) 6 (B) 5 (C) 4 (D) 0
A

 
5. If  is the angle between the vectors p = a î + b ĵ + c k̂ and vector q = b î + c ĵ + a k̂ then range of  is
(A) [0, /3] (B) [/3, 2/3] (C) [0, 2/3] (D) [0, 5/6]
NK

6. If the unit vectors e1 and e2 are inclined at an angle 2 and | e1 − e2 | < 1, then for   [0, ],  may lie
in the interval :
     5    5 
(A) 0,  (B)  ,  (C)  ,   (D)  , 
 6 6 2  6  2 6 
SA

7. A line makes angles , , ,  with the four diagonals of a cube, then cos 2 + cos2 + cos2 + cos2 is
equal to
(A) 1/3 (B) 2/3 (C) 4/3 (D) 5/3

x y z x y z
8. Consider the lines = = and = = , then the equation of the line which
2 3 5 1 2 3
x y z
(A) bisects the angle between the lines is = =
3 3 8
x y z
(B) bisects the angle between the lines is = =
1 2 3
(C) passes through origin and is perpendicular to the given lines is x = y = – z
(D) none of these

9. Given a = xiˆ + yjˆ + 2kˆ , b = ˆi − ˆj + kˆ , c = ˆi + 2ˆj ; (a ^ b) = , a.c = 4 , then
2
(A) [a b c]2 =| a | (B) [a b c] =| a | (C) [a b c] = 0 (D) [a b c] =| a |2

10. The vectors ˆi + 2 ˆj + 3kˆ , 2 ˆi − ˆj + kˆ and 3 ˆi + ˆj + 4kˆ are so placed that the end point of one vector is the
starting point of the next vector. Then the vectors are :
(A) Not coplaner (B) Coplaner but cannot form a triangle
(C) Coplaner but can form a triangle (D) Coplaner & can form a right angled triangle

11. Let a, b and c be non-coplanar unit vectors equally inclined to one another at an acute angle . Then
[a b c] in terms of  is equal to:

(A) (1 + cos ) cos 2  (B) (1 + cos ) 1− 2cos2 

RI
(C) (1 − cos ) 1 + 2 cos  (D) (1 − sin ) 1 + 2 cos 
 

A
12. Consider a tetrahedron with faces f 1, f2, f3, f4. Let a1 , a2 ,a3 , a4 be the vectors whose magnitudes are
respectively equal to the areas of f 1, f2, f3, f4 and whose directions are perpendicular to these faces in
the outward direction. Then,

UH
(A) a1 + a2 + a3 + a4 = 0 (B) a1 + a3 = a2 + a4
(C) a1 + a2 = a3 + a4 (D) a1 + a2 − a3 + a4 = 0

13. Let r be a vector perpendicular to a + b + c , where [ a b c ] = 2. If


r = ( b × c ) + m( c × a ) + n( a × b ), then ( + m + n) is equal to
JA
(A) 2 (B) 1 (C) 0 (D) –1

14. If a , b , c are three non-coplanar non-zero vectors and r is any vector in space, then
(a  b) × (r  c) + (b  c) × (r  a) + (c  a) × (r  b) is equal to
(A) 2[ a b c ] r (B) 3[ a b c ] r (C) [ a b c ] r (D) 4[ a b c ] r
LP

15. If b and c are two non-collinear vectors such that a || ( b × c ), then ( a × b ) . ( a × c ) is equal to
(A) a2 (b.c) (B) b2 (a.c) (C) c 2 (a.b) (D) – a2 (b.c)

Let 2c + b = 4a + 3d . If  d c a  and a b d are natural numbers with H.C.F. equal to 1 then how
A

16.
many statement are true among below six statement.
(i) a b d  = 2 (ii) a b c  = 3
   
NK

(iii) d c b  = 4 (iv) d c a  = 1
   
(v) ( a  b ) × ( c  d) = 2 c – 3d (vi) ( a  b ) × ( c  d) = 4a – b
(A) 2 (B) 4 (C) 6 (D) 0
SA

17. Let u and v are unit vectors and w is a vector such that (u  v) + u = w and w  u = v then the
value of u v w  is equal to
(A) 1 (B) 2 (C) 0 (D) –1

18. Find the shortest distance between any two opposite edges of a tetrahedron formed by the planes
y + z = 0, x + z = 0, x + y = 0, x + y + z = 3 a .
(A) a (B) 2a (C) a / 2 (D) 2 a
19. A plane meets the coordinate axes in A, B, C and (  ) is the centroid of the triangle ABC, then the
equation of the plane is
x y z x y z 3x 3y 3z
(A) + + =3 (B) + + =1 (C) + + =1 (D) x + y + z =1
        
x −1 y−2 z−3
20. Equation of plane which passes through the point of intersection of lines = = and
3 1 2
x−3 y −1 z−2
= = and at greatest distance from the point (0, 0, 0) is:
1 2 3
(A) 4x + 3y + 5z = 25 (B) 4x + 3y + 5z = 50
(C) 3x + 4y + 5z = 49 (D) x + 7y – 5z = 2

21. The non zero value of ‘a’ for which the lines 2x – y + 3z + 4 = 0 = ax + y – z + 2 and
x – 3y + z = 0 = x + 2y + z + 1 are co-planar is :
(A) – 2 (B) 4 (C) 6 (D) 0

22. A line having direction ratios 3, 4, 5 cuts 2 planes 2x – 3y + 6z – 12 = 0 and 2x – 3y + 6z + 2 = 0 at


point P & Q, then find length of PQ

RI
35 2 35 2 35 2 35 2
(A) (B) (C) (D)
12 24 6 8

23. A line L1 having dierection ratios 1, 0, 1 lies on xz plane. Now this xz plane is rotated about z-axis by an

A
angle of 90°. Now the new position of L1 is L2. The angle between L1 & L2 is :
(A) 30° (B) 60° (C) 90° (D) 45°

UH
PART - II : SINGLE AND DOUBLE VALUE INTEGER TYPE
1. Given f2(x) + g2(x) + h2(x)  9 and U(x) = 3f(x) + 4g(x) + 10h(x), where f(x), g(x) and h(x) are continuous
 x  R. If maximum value of U(x) is N . Then the value of cube root of (N – 1000) is
JA
2. If in a plane A1, A2, A3,......, A20 are the vertices of a regular polygon having 20 sides and O is its centre
19
and  (OA  OA
i=1
i i +1 ) =  (OA 2  OA1 ) then || is

| PA |2 + | PB |2 + | PC |2
LP

3. In an equilateral ABC find the value of where P is any arbitrary point lying on
R2
its circumcircle, is

4. Let A = 2 î + k̂ , B = î + ĵ + k̂ and C = 4 î − 3 ĵ + 7 k̂ . If a vector R =  ˆi – ˆj + kˆ satisfies


A

R x B = C x B and R . A = 0 then  +  +  is

x+2 y−3 z−k


NK

5. A line = = cuts the y-z plane and the x-y plane at A and B respectively. If
1 2 3

AOB = , then 2k, where O is the origin, is
2

6. Given four non zero vectors a, b, c and d . The vectors a, b and c are coplanar but not collinear pair
SA

   
by pair and vector d is not coplanar with vectors a, b and c and ( a b ) = (b c) = , ( d a ) = 
3
 
and ( d b ) =  , if (d c) = cos−1(mcos  + ncos  ) then m – n is :

7. If the circumcentre of the tetrahedron OABC is given by


( ) (
a2 b x c + b 2 ( c x a ) + c 2 a x b ), where

a, b & c are the position vectors of the points A, B, C respectively relative to the origin 'O' such that [
a b c ] = 36 then  is

8. Given three point on x – y plane as O(0, 0), A(1, 0) & B(–1, 0). Point P moving on the given plane
satisfying the condition (PA . PB) + 3 (OA . OB) = 0
If the maximum & minimum values of | PA | | PB | is M & m respectively then the value of M 2 + m2 is

9. If the volume of tetrahedron formed by planes whose equations are y + z = 0, z + x = 0, x + y = 0 and
x + y + z = 1 is t cubic unit, then the value of 3t is

10. If r represents the position vector of point R in which the line AB cuts the plane CDE, where position
vectors of points A, B, C, D, E are respectively a = ˆi + 2ˆj + kˆ , b = 2iˆ + ˆj + 2kˆ , c = −4jˆ + 4kˆ ,
d = 2iˆ − 2ˆj + 2kˆ and e = 4iˆ + ˆj + 2kˆ , then r 2 is :

11. Line L1 is parallel to vector  = −3iˆ + 2ˆj + 4kˆ and passes through a point A(7, 6, 2) and line L 2 is parallel
to a vector  = 2iˆ + ˆj + 3kˆ and passes through a point B(5, 3, 4). Now a line L 3 parallel to a vector

RI
r = 2iˆ − 2jˆ − kˆ intersects the lines L1 and L2 at points C and D respectively, then | 4CD | is equal to :

12. L is the equation of the straight line which passes through the point (2, –1, –1); is parallel to the plane
4x + y + z + 2 = 0 and is perpendicular to the line of intersection of the planes 2x + y = 0 = x – y + z. If

A
the point (3, , ) lies on line L, then | + | is

x+4 y+6 z −1
13. The lines = = and 3x – 2y + z + 5 = 0 = 2x + 3y + 4z – k are coplanar, then the value

UH
3 5 −2
of k is

x −1 y−2 z+3
14. About the line = = the plane 3x + 4y + 6z + 7 = 0 is rotated till the plane passes
2 −3 1
JA
through the origin. Now 4x + y + z = 0 is the equation of plane in new position. The value of  2 + 2
is

15. The value of sec3 , where  is the acute angle between the plane faces of a regular tetrahedron, is

16. R and r are the circum−radius and in−radius of a regular tetrahedron respectively in terms of the length
p 2
k of each edge. If R2 + r2 = k , where p, q   then absolute minimum value of p + q is
LP

17. A line L on the plane 2x + y – 3z + 5 = 0 is at a distance 3 unit from the point P(1, 2, 3). A spider starts
x −1 y − 2 z−3
moving from point A and after moving 4 units along the line = = it reaches to point P.
−3
A

2 1
and from P it jumps to line L along the shortest distance and then moves 12 units along the line L to
reach at point B. The distance between points A and B is
NK

18. The length of edge of a regular tetrahedron D-ABC is 'a'. Point E & F are taken on the edges AD and
BD respectively. Such that E divide DA and F divide BD in the ratio 2 : 1 each. The area of CEF is
 3 2
equal to a , then value of  is :
36
SA

y z x z
19. If 'd' be the shortest distance between the lines + = 1; x = 0 and – = 1; y = 0 and if
b c a c
 1 1 1
2
= + 2 + 2 then  is
d a2 b c
PART - III : ONE OR MORE THAN ONE OPTIONS CORRECT TYPE
1. A vector a has components 2p and 1 with respect to a rectangular cartesian system. The system is
rotated through a certain angle about the origin in the counterclockwise sense. If with respect to the
new system, a has components p + 1 and 1, then
1 1
(A) p = − (B) p = 1 (C) p = − 1 (D) p =
3 3

2. If z1 = a ˆi + b ˆj and z2 = c ˆi + d ˆj are two vectors in ˆi and ˆj system, where z1 = z 2 = r and

z1 . z2 = 0, then w1 = a ˆi + c ˆj and w 2 = b ˆi + d ˆj satisfy :


(A) w1 = r (B) w 2 = r (C) w1 .w 2 = 0 (D) | w1 |  |w 2 |

RI
3. If a, b, c, x, y, z  R such that ax + by + cz = 2, then which of the following is always true
(A) (a2 + b2 + c2)(x2 + y2 + z2)  4 (B) (x2 + b2 + z2)(a2 + y2 + c2)  4
(C) (a2 + y2 + z2)(x2 + b2 + c2)  4 (D) (a2 + b2 + z2)(x2 + y2 + c2)  4
 

A
4. The direction cosines of the lines bisecting the angle between the lines whose direction cosines are 
1, m1, n1 and 2, m2, n2 and the angle between these lines is , are
– 2 m1 – m2 n1 – n2 + 2 m + m2 n + n2

UH
(A) 1 , , (B) 1 , 1 , 1
     
cos cos cos 2cos 2cos 2cos
2 2 2 2 2 2
+ 2 m1 + m2 n1 + n2 − m − m2 n1 − n2
(C) 1 , , (D) 1 2 , 1 ,
     
2 sin 2 sin 2 sin 2sin 2 sin 2sin
2 2 2 2 2 2
JA
5. The value(s) of   [0, 2] for which vector a = ˆi + 3ˆj + (sin2 ) kˆ makes an obtuse angle with the
 ˆ 
z-axis and the vectors b = (tan  ) ˆi − ˆj + 2 sin k and c = (tan ) î + (tan ) ĵ – 3 cos ec kˆ are
2 2
orthogonal, is/are
LP

(A) tan −1 3 (B)  − tan −1 2 (C)  + tan −1 3 (D) 2  − tan −1 2

11
6. The vector ˆi + xjˆ + 3kˆ is rotated through an angle of cos –1 and doubled in magnitude, then it
14
becomes 4iˆ + (4x − 2)jˆ + 2kˆ . The value of ' x ' CANNOT be :
A

2 2 20
(A) − (B) (C) − (D) 2
3 3 17
NK

7. The vertices of a triangle are A (1, 1, 2), B(4, 3, 1) and C(2, 3, 5). A vector representing the bisector of
the angle A is :
(A) 2iˆ – 4kˆ (B) –2iˆ + 4kˆ (C) –2iˆ – 2jˆ − kˆ (D) 2iˆ + 2ˆj + kˆ

8. The vector c , parallel to the internal bisector of the angle between the vectors a = 7 ˆi − 4 ˆj − 4kˆ and
SA

b = − 2 ˆi − ˆj + 2kˆ with  c  = 5 6 , is :
5 ˆ 5 ˆ 5 ˆ 5 ˆ
(A)
3
(
i − 7 ˆj + 2kˆ ) (B)
3
(
i + 7 ˆj − 2kˆ ) (C)
3
(
− i + 7 ˆj − 2kˆ (D) )
3
(
− i − 7 ˆj + 2kˆ )
9. A line passes through a point A with position vector 3 ˆi + ˆj − kˆ and is parallel to the vector 2 ˆi − ˆj + 2kˆ
. If P is a point on this line such that AP = 15 units, then the position vector of the point P is/are
(A) 13 ˆi + 4 ˆj − 9kˆ (B) 13 ˆi − 4 ˆj + 9kˆ (C) 7 ˆi − 6 ˆj + 11kˆ (D) − 7 ˆi + 6 ˆj − 11kˆ

x −1
y +1 z x +1 y−3 z −1
10. Acute angle between the lines =
= and = = where  > m > n, and 
m n m n
, m, n are the roots of the cubic equation x 3 + x2 – 4x = 4 is equal to :
3 65 13 2
(A) cos–1 (B) sin–1 (C) 2cos–1 (D) tan–1
13 9 18 3
x − 2 y +1 z −1
11. The line = = intersects the curve xy = c 2, z = 0 if c is equal to :
3 2 −1
(A) –1 (B) – 5 (C) 5 (D) 1

x − 1 y − 2 z − 3 x − 1 2y − 4 3z − 9 x −  2 y − 2 z − 3
12. Three distinct lines = = , = = , = = are
3 2 1 5 3 1 3 2 
concurrent the value of  may be :
(A) 1 (B) –1 (C) 2 (D) –2
 
x+6 y + 10 z + 14
13. The line = = is the hypotenuse of an isosceles right angle triangle whose opposite
5 3 8
vertex is (7, 2, 4) Then the equation of remaining sides is/are -

RI
x−7 y−2 z−4 x−7 y−2 z−4
(A) = = (B) = =
3 6 2 2 −3 6
x+7 y+2 z+4 x+7 y+2 z+4
(C) = = (D) = =
3 6 2 2 −3 6

A
14. Two lines are    
x −1 y −1 z −1 x −1 y −1 z −1
L1 : ; L2 :

UH
= = = =
1 2 2 2 1 2
Equation of line passing through (2, 1, 3) and equally inclined to L 1 & L2 is/are
x − 2 y −1 z − 3 x−3 y−2 z−5
(A) = = (B) = =
2 2 −3 1 1 2
x − 2 y −1 z − 3 x y +1 z − 6
JA
(C) = = (D) = =
1 1 3 2 2 −3

15. Which of the followings is/are correct :


(A) The angle between the two straight lines r = 3 î – 2 ĵ + 4 k̂ +  (– 2 î + ĵ + 2 k̂ ) and
 4
r = î + 3 ĵ – 2 k̂ +  (3 î – 2 ĵ + 6 k̂ ) is cos–1  
LP

 21 
ˆ (iˆ  r ) + (r.j)
(B) (r.i) ˆ ( ˆj  r ) + (r.k)
ˆ (kˆ  r ) = 0 .
(C) The force determined by the vector r = (1, −8, −7) is resolved along three mutually perpendicular
directions, one of which is in the direction of the vector a = 2 ˆi + 2 ˆj + kˆ . Then the vector component of
A

7 ˆ
the force r in the direction of the vector a is − (2i + 2ˆj + k)
ˆ
3
1
 
NK

(D) The cosine of the angle between any two diagonals of a cube is .
3
16. If the distance between points (, 5, 10) from the point of intersection of the line.
r = (2 î − ĵ + 2 k̂ ) +  (2 î + 4 ĵ + 12 k̂ ) and plane r . ( î − ĵ + k̂ ) = 5 is 13 units, then value of  may be
80
(A) 1 (B) – 1 (C) 4 (D)
SA

63

17. ( )
A vector v =  a ˆj + bkˆ is coplanar with the vectors $i + $j − 2 k$ and ˆi − 2 ˆj + kˆ and is orthogonal to

the vector − 2 ˆi + ˆj + kˆ . It is given that the length of projection of v along the vector ˆi − ˆj + kˆ is equal to
6 3 . Then the value of 2ab may be
(A) 81 (B) 9 (C) –9 (D) –81

18. â and b̂ are two given unit vectors at right angle. The unit vector equally inclined with â , b̂ and
â × b̂ will be:
1 ˆ ˆ ˆ ˆ 1
(A) –
3
(
a+ b + a  b ) (B)
3
(aˆ + bˆ + aˆ  bˆ )
1 1
(C)
3
(aˆ + bˆ − aˆ  bˆ ) (D) –
3
(aˆ + bˆ − aˆ  bˆ )
19. Let a = 2iˆ – ˆj + kˆ , b = ˆi + 2jˆ – kˆ and c = ˆi + ˆj – 2kˆ be three vectors. A vector in the plane of b and c
2
whose length of projection on a is of , is
3
(A) 2i + 3j – 3k (B) 2i + 3j + 3k (C) – 2i – j + 5k (D) i – 5j + 3k
 
20. The position vectors of the angular points of a tetrahedron are A(3iˆ − 2ˆj + k)
ˆ , B(3iˆ + ˆj + 5k)
ˆ , C(4iˆ + 3k)
ˆ
ˆ . Then the acute angle between the lateral face ADC and the base face ABC is :
and D(i)
5 2 5 2

RI
(A) tan–1 (B) tan–1 (C) cot–1 (D) cot–1
2 5 2 5

3
21. If a, b, c and d are unit vectors such that (a  b) . (c  d) =  and a . c = , then

A
2
(A) a , b, c are coplanar if  = 1 (B) Angle between b and d is 30° if  = – 1
(C) angle between b and d is 150° if  = – 1 (D) If  = 1 then angle between b and c is 60°

UH
 
22. The volume of a right triangular prism ABCA 1B1C1 is equal to 3. If the position vectors of the vertices of
the base ABC are A(1, 0, 1), B(2,0, 0) and C(0, 1, 0), then position vectors of the vertex A 1 can be:
(A) (2, 2, 2) (B) (0, 2, 0) (C) (0, − 2, 2) (D) (0, − 2, 0)
JA
23. The coplanar points A , B , C , D are (2 − x , 2 , 2) , (2 , 2 − y , 2) , (2 , 2 , 2 − z) and (1 , 1 , 1)
respectively, then
1 1 1 x −1 y −1 z −1
(A) + + = 1 (B) + + =2
x y z x y z
1 1 1 x y z
(C) + + =1 (D) + + +2=0
1− x 1− y 1− z 1− x 1− y 1 − z
LP

24. Which of the following statement(s) is/are correct :


(A) If a,b,c are non–coplanar and d is any vector, then
[d b c] a + [d c a] b + [d a b] c − [a b c] d = 0
A

(B) If  is incentre of  ABC then | BC | A + | CA |  B+ | AB |  C = 0


(C) Any vector in three dimension can be written as linear combination of three non–coplanar vectors.
a+b+c
NK

(D) In a triangle, if position vector of vertices are a,b,c , then position vector of incentre is .
3

25. Let V = 2 ˆi + ˆj − kˆ & W = ˆi + 3kˆ . If U is a unit vector, then the value of the scalar triple product
 U V W  may be :
 
SA

(A) – 59 (B) 10 + 6 (C) 59 (D) 60

26. If A + B = a , A . a = 1 and A  B = b , then


a  b + a | a |2 −1
( )
(A) A = ab + a
(B) B = 2
| a |2 |a|
b  a + a | a |2 −1
( )
(C) A = ba + a
(D) B = 2
| a |2 |a|

x−4 y−2 z − k2
27. The line = = lies in the plane 2 x − 4 y + z = 1 . Then the value of k
k 1 2
cannot be :
(A) 1 (B) −1 (C) 2 (D) –2

x − x2 y − y2 z − z2
28. Equation of the plane passing through A(x 1, y1, z1) and containing the line = = is
d1 d2 d3
x − x1 y − y1 z − z1 x − x2 y − y2 z − z2
(A) x 2 − x1 y 2 − y1 z2 − z1 = 0 (B) x1 − x 2 y1 − y 2 z1 − z2 = 0
d1 d2 d3 d1 d2 d3
x − d1 y − d2 z − d3 x y z
(C) x1 y1 z1 = 0 (D) x1 − x 2 y1 − y 2 z1 − z2 = 0
x2 y2 z2 d1 d2 d3

RI
x –1 y–2 z–3
29. A line = = intersects the plane x – y + 2z + 2 = 0 at point A. The equation of the
2 3 4
straight line passing through A lying in the given plane and at minimum inclination with the given line
is/are

A
x +1 y +1 z +1
(A) = = (B) 5x – y + 4 = 0 = 2y – 5z – 3
1 5 2
x+2 y+6 z+3
(C) 5x + y – 5z + 1 = 0 = 2y – 5z – 3 (D) = =

UH
1 5 2

30. If the -plane 7x + ( + 4)y + 4z – r = 0 passing through the points of intersection of the planes
2x + 3y− z +1= 0 and x + y − 2z + 3 = 0 and is perpendicular to the plane 3x − y − 2z = 4 and
 12 −78 57 
 , ,  is image of point (1, 1, 1) in −plane, then
JA
    
(A)  = 9 (B)  = – 117 (C)  = – 9 (D)  = 117
31. The planes 2x – 3y – 7z = 0, 3x – 14y – 13z = 0 and 8x – 31y – 33z = 0
(A) pass through origin (B) intersect in a common line
(C) form a triangular prism (D) pass through infinite the many points
 
LP

32. If a,b,c and d are the position vectors of the points A, B, C and D respectively in three dimensional
space no three of A, B, C, D are collinear and satisfy the relation 3a − 2b + c − 2d = 0 , then :
(A) A, B, C and D are coplanar
(B) The line joining the points B and D divides the line joining the point A and C in the ratio 2 : 1.
A

(C) The line joining the points A and C divides the line joining the points B and D in the ratio 1 : 1.
(D) the four vectors a,b,c and d are linearly dependents.
NK

PART - IV : COMPREHENSION
Comprehension # 1
In a parallelogram OABC, vectors a, b, c are respectively the position vectors of vertices A, B, C with
SA

reference to O as origin. A point E is taken on the side BC which divides it in the ratio of 2 : 1 internally.
Also, the line segment AE intersect the line bisecting the angle O internally in point P. If CP, when
extended meets AB in point F. Then

1. The position vector of point P, is


3 a c  a c  a c  a c 
(A)  +  (B)  + 
3 c + 2 a  a c  3 c + 2 a  a c 

2a c  a c  3a c  a c 
(C)  +  (D)  − 
3 c + 2 a  a c  3 c + 2 a  a c 

2. The position vector of point F, is


1 a a 2a a
(A) a + c (B) a + c (C) a + c (D) a − c
3 c c c c

3. The vector AF , is given by


a a 2a 1 a
(A) – c (B) c (C) c (D) c
c c c 3 c

Comprehension # 2
Let a1x + b1y + c1z + d1 = 0 and a2x + b2y + c2z + d2 = 0 be two planes, where d1, d2 > 0. Then origin lies
in acute angle if a1a2 + b1b2 + c1c2 < 0 and origin lies in obtuse angle if a 1a2 + b1b2 + c1c2 > 0.
Further point (x1, y1, z1) and origin both lie either in acute angle or in obtuse angle ,

RI
if (a1x1 + b1y1 + c1z1 + d1) (a2x1 + b2y1 + c2z1 + d2) > 0, one of (x1, y1, z1) and origin lie in acute angle and
the other in obtuse angle, if (a1x1 + b1y1 + c1z1 + d1) (a2x1 + b2y1 + c2z1 + d2) < 0

A
4. Given the planes 2x + 3y – 4z + 7 = 0 and x – 2y + 3z – 5 = 0, if a point P is (1, – 2, 3) and O is origin,
then
(A) O and P both lie in acute angle between the planes

UH
(B) O and P both lie in obtuse angle between the planes
(C) O lies in acute angle, P lies in obtuse angle.
(D) O lies in obtue angle, P lies in acute angle.

5. Given the planes x + 2y – 3z + 5 = 0 and 2x + y + 3z + 1 = 0. If a point P is (2, –1, 2) and O is origin,
JA
then
(A) O and P both lie in acute angle between the planes
(B) O and P both lie in obtuse angle between the planes
(C) O lies in acute angle, P lies in obtuse angle.
(D) O lies in obtue angle, P lies in acute angle.

6. Given the planes x + 2y – 3z + 2 = 0 and x – 2y + 3z + 7 = 0, if the point P is (1, 2, 2) and O is origin,
LP

then
(A) O and P both lie in acute angle between the planes
(B) O and P both lie in obtuse angle between the planes
(C) O lies in acute angle, P lies in obtuse angle.
(D) O lies in obtue angle, P lies in acute angle.
A

Comprehension # 3
If a, b, c & a',b',c' are two sets of non-coplanar vectors such that a.a'=b.b'=c.c' = 1 , then the two
NK

b x c c x a a x b
systems are called Reciprocal System of vectors and a= , b = and c = .
[a b c ] [a b c ] [a b c ]
7. Find the value of a  a + b  b +c  c .
(A) 0 (B) a + b + c (C) a − b + c (D) a + b − c
SA

a+b+c
8. Find value of  such that a  b + b  c + c  a =  .
[abc]
(A) – 1 (B) 1 (C) 2 (D) – 2

9. If [(a × b) × (b × c) (b × c) × (c × a) (c × a) × (a × b)] = [abc] n , then find n.
(A) n = – 4 (B) n = 4 (C) n = – 3 (D) n = 3

Comprehension # 4
The vertices of square pyramid are A(0, 0, 0), B(4, 0, 0), C(4, 0, 4), D(0, 0, 4) and E(2, 6, 6)

10. Volume of the pyramid is :


(A) 32 (B) 16 (C) 8 (D) 4
11. Centroids of triangular faces of square pyramid are
(A) Non-coplanar (B) Coplanar but the plane is not parallel to base plane
(C) Coplanar & plane is parallel to base plane (D) Co-linear

12. The distance of the plane EBC from ortho-centre of ABD is :


12
(A) 2 (B) 5 (C) (D) 10
10
Comprehension # 5
General equation of a sphere is given by x 2 + y2 + z2 + 2ux + 2vy + 2wz + d = 0, where ( −u, −v, −w) is
the centre and u2 + v 2 + w 2 − d is the radius of the sphere.
Let P be a any plane and F is the foot of perpendicular from centre(C) of the sphere to this plane.
If CF > u2 + v 2 + w 2 − d then plane P neither touches nor cuts the sphere.

RI
If CF = u2 + v 2 + w 2 − d then plane P touches the sphere.
If CF < u2 + v 2 + w 2 − d then intersection of plane P and sphere is a circle with
2
radius = u2 + v 2 + w 2 – d – ( CF )

A
13. Find the equation of the sphere having centre at (1, 2, 3) and touching the plane
x + 2y + 3z = 0.

UH
(A) x2 + y2 + z2 – 2x – 4y – 6z = 0 (B) x2 + y2 + z2 – 2x + 4y – 6z = 0
(C) x + y + z – 2x – 4y + 6z = 0
2 2 2
(D) x2 + y2 + z2 + 2x – 4y – 6z = 0

14. A variable plane passes through a fixed point (1, 2, 3). The locus of the foot of the perpendicular drawn
from origin to this plane is:
(A) x2 + y2 + z2 − x − 2y − 3z = 0 (B) x2 + 2y2 + 3z2 − x − 2y − 3z = 0
JA
(C) x + 4y + 9z + x + 2y + 3 = 0
2 2 2
(D) x2 + y2 + z2 + x + 2y + 3z = 0
x −1 y − 2 z − 3
15. Find the length of the chord intercepted on the line = = by the sphere
1 2 3
22
x2 + y2 + z2 – 2x – 2y – z = 0.
3
LP

(A) 56 (B) 54 (C) 9 (D) 6


A
NK
SA
* Marked Questions may have more than one correct option.
 Marked questions are recommended for Revision.
PART - I : JEE (ADVANCED) / IIT-JEE PROBLEMS (PREVIOUS YEARS)
1
1. If a, b, c and d are unit vectors such that (a  b) . (c  d) = 1 and a . c = , then
2
[IIT-JEE-2009, Paper-I, (3, – 1), 80]
(A) a, b, c are non-coplanar (B) b, c, d are non-coplanar
(C) b, d are non-parallel (D) a, d are parallel and b, c are parallel
 

RI
2. Match the statements/expressions given in Column - I with the values given in Column - II
[IIT-JEE-2009, Paper-2, (8, 0), 80]
Column - I Column - II

(A) Root(s) of the equation 2 sin2 + sin22 = 2 (p)

A
6
 
6x  
3x 
(B) Points of discontinuity of the function f(x) =   cos   , (q)
  4

UH
where [y] denotes the largest integer less than or equal to y

(C) Volume of the parallelopiped with its edges represented by the (r)
3
vectors ˆi + ˆj , ˆi + 2jˆ and ˆi + ˆj + kˆ

JA
(D) Angle between vectors a and b where a, b and c are unit (s)
2
vectors satisfying a + b + 3 c = 0 (t) 

3. A line with positive direction cosines passes through the point P(2, –1, 2) and makes equal angles with
the coordinate axes. The line meets the plane 2x + y + z = 9 at point Q. The length of the line segment
PQ equals [IIT-JEE-2009, Paper-2, (3, –1), 80]
LP

(A) 1 (B) 2 (C) 3 (D) 2


4. Let P(3, 2, 6) be a point in space and Q be a point on the line r = (i − j + 2k)
ˆ ˆ ˆ + (−3iˆ + ˆj + 5k)
ˆ . Then the
value of  for which the vector PQ is parallel to the plane x – 4y + 3z = 1 is
[IIT-JEE-2009, Paper-I, (3, – 1), 80]
A

1 1 1 1
(A) (B) – (C) (D) –
4 4 8 8
5. Let P, Q, R and S be the points on the plane with position vectors – 2 î – ĵ , 4 î , 3 î + 3 ĵ and
NK

– 3 î + 2 ĵ respectively. The quadrilateral PQRS must be a [IIT-JEE-2010, Paper-1, (3, –1), 84]
(A) parallelogram, which is neither a rhombus nor a rectangle
(B) square
(C) rectangle, but not a square
SA

(D) rhombus, but not a square

ˆi – 2ˆj 2iˆ + ˆj + 3kˆ


6. If a and b are vectors in space given by a = and b = , then the value of
5 14

( 2a + b ) . (a  b )  (a – 2b ) is [IIT-JEE-2010, Paper-1, (3, 0), 84]

7. Two adjacent sides of a parallelogram ABCD are given by [IIT-JEE-2010, Paper-2, (5, –2), 79]
AB = 2iˆ + 10jˆ + 11kˆ and AD = −ˆi + 2jˆ + 2kˆ . The side AD is rotated by an acute angle  in the plane
of the parallelogram so that AD becomes AD . If AD makes a right angle with the side AB, then the
cosine of the angle  is given by
8 17 1 4 5
(A) (B) (C) (D)
9 9 9 9

x y z
8. Equation of the plane containing the straight line = = and perpendicular to the plane
2 3 4
x y z x y z
containing the straight lines = = and = = is [IIT-JEE-2010, Paper-1, (3, –1), 84]
3 4 2 4 2 3
(A) x + 2y – 2z = 0 (B) 3x + 2y – 2z = 0 (C) x – 2y + z = 0 (D) 5x + 2y – 4z = 0

x  1
9.
 
The number of 3 × 3 matrices A whose entries are either 0 or 1 and for which the system A  y  =
0 
 

RI
 z  0 
has exactly two distinct solutions, is [IIT-JEE-2010, Paper-1, (3, –1), 84]

A
(A) 0 (B) 29 – 1 (C) 168 (D) 2
 

UH
10. If the distance between the plane Ax – 2y + z = d and the plane containing the lines
x –1 y–2 z–3 x–2 y–3 z–4
= = and = = is 6 , then |d| is
2 3 4 3 4 5
[IIT-JEE-2010, Paper-1, (3, 0), 84]
JA
11. If the distance of the point P(1, –2, 1) from the plane x + 2y – 2z = , where  > 0, is 5, then the foot of
the perpendicular from P to the plane is       [IIT-JEE-2010, Paper-2, (5, –2), 79]
8 4 7 4 4 1  1 2 10  2 1 5
(A)  , , −  (B)  , − ,  (C)  , ,  (D)  , − , 
 3 3 3   3 3 3 3 3 3   3 3 2
LP

12. Match the statements in Column-I with those in Column-II. [IIT-JEE-2010, Paper-2, (8, 0), 79]
Column-I Column-II
(A) A line from the origin meets the lines (p) –4
A

8
x–
x – 2 y –1 z +1 3 = y +3 = z –1
= = and
1 –2 1 2 –1 1
at P and Q respectively. If length PQ = d, then d 2 is
NK

(B) The values of x satisfying (q) 0


3
tan–1(x + 3) – tan–1(x – 3) = sin–1   are
5
SA

(C) Non-zero vectors a , b and c satisfy a . b = 0, (r) 4


( b – a).(b + c) = 0 and 2 | b + c | =| b – a | . If a = µb + 4c  
    then possible value of µ are

(D) Let f be the function on [–, ] given by (s) 5


 9x 
sin  
f(0) = 9 and f(x) =  2  for x  0. The value (t) 6
x
sin  
2

2
of
 
–
f(x) dx is
13. Let a = ˆi + ˆj + kˆ , b = ˆi − ˆj + kˆ and c = ˆi − ˆj − kˆ be three vectors. A vector  in the plane of a and b ,
1
whose projection on c is , is given by [IIT-JEE 2011, Paper-1, (3, –1), 80]
3
(A) ˆi − 3jˆ + 3kˆ (B) −3iˆ − 3jˆ − kˆ (C) 3iˆ − ˆj + 3kˆ (D) ˆi + 3jˆ − 3kˆ
 

14*. The vector(s) which is/are coplanar with vectors ˆi + ˆj + 2kˆ and ˆi + 2jˆ + kˆ , and perpendicular to the
vector ˆi + ˆj + kˆ is/are [IIT-JEE 2011, Paper-1, (4, 0), 80]
(A) ˆj − kˆ (B) −ˆi + ˆj (C) ˆi − ˆj (D) −ˆj + kˆ
 

15. Let a = −ˆi − kˆ , b = −ˆi + ˆj and c = ˆi + 2jˆ + 3kˆ be three given vectors. If r is a vector such that
r  b = c  b and r . a = 0 , then the value of r . b is [IIT-JEE 2011, Paper-2, (4, 0), 80]

RI
16. Match the statements given in Column-I with the values given in Column-II
[IIT-JEE 2011, Paper-2, (8, 0), 80]
Column-I Column-II

A

(A) If a = ˆj + 3 kˆ , b = −ˆj + 3 kˆ and c = 2 3 kˆ form a triangle, (p)
6
then the internal angle of the triangle between a and b is

UH
 2
b

(B) If  (f(x) − 3x) dx = a2 – b2, then the value of f   is (q)


a 6 3
2 
5/6

ln3 7/ 6
(C) The value of sec ( x) dx is (r)
3
JA
 1 
(D) The maximum value of Arg   for |z| = 1, z  1 is given by (s) 
 1− z 

(t)
2
17. The point P is the intersection of the straight line joining the points Q(2,3,5) and R(1, –1, 4) with the
plane 5x – 4y – z = 1. If S is the foot of the perpendicular drawn from the point T(2, 1,4) to QR, then the
LP

length of the line segment PS is [IIT-JEE 2012, Paper-1, (3, –1), 70]
1
  (A) (B) 2 (C) 2 (D) 2 2    
2
A

18. If a, b and c are unit vectors satisfying | a – b |2 + | b – c |2 + | c – a |2 = 9, then |2 a + 5 b + 5 c | is


[IIT-JEE 2012, Paper-1, (4, 0), 70]

19. The equation of a plane passing through the line of intersection of the planes x + 2y + 3z = 2 and
NK

2
x – y + z = 3 and at a distance from the point (3, 1, –1) is [IIT-JEE 2012, Paper-2, (3, –1), 66]   
3
(A) 5x – 11y + z = 17 (B) 2x + y = 3 2 – 1
(C) x + y + z = 3 (D) x – 2y = 1 – 2
SA

20. If a and b are vectors such that a + b = 29 and a  (2iˆ + 3jˆ + 4k)
ˆ = (2iˆ + 3jˆ + 4k)
ˆ × b , then a

possible value of (a + b) . (–7iˆ + 2jˆ + 3k)


ˆ is [IIT-JEE 2012, Paper-2, (3, –1), 66]

x −1 y +1 z x +1 y +1 z
21*. If the straight lines   = = and = = are coplanar, then the plane(s)
2 k 2 5 2 k
containing these two lines is(are)         [IIT-JEE 2012, Paper-2, (4, 0), 66]
(A) y + 2z = –1 (B) y + z = –1 (C) y – z = –1 (D) y – 2z = –1

22. Let PR = 3iˆ + ˆj – 2kˆ and SQ = ˆi – 3jˆ – 4kˆ determine diagonals of a parallelogram PQRS and
PT = ˆi + 2jˆ + 3kˆ be another vector. Then the volume of the parallelepiped determined by the vectors
PT, PQ and PS is [JEE (Advanced) 2013, Paper-1, (2, 0)/60]
(A) 5 (B) 20 (C) 10 (D) 30
x + 2 y +1 z
23. Perpendicular are drawn from points on the line = = to the plane x + y + z = 3. The feet of
2 –1 3
perpendiculars lie on the line [JEE (Advanced) 2013, Paper-1, (2, 0)/60]
x y –1 z – 2 x y –1 z – 2 x y –1 z – 2 x y –1 z – 2
(A) = = (B) = = (C) = = (D) = =
5 8 –13 2 3 –5 4 3 –7 2 –7 5

24.* A line l passing through the origin is perpendicular to the lines


l1 : (3 + t) î + (– 1 + 2t) ĵ + (4 + 2t) k̂ , –  < t < 
l2 : (3 + 2s) î + (3 + 2s) ĵ + (2 + s) k̂ , –  < s < 
Then, the coordinate(s) of the point(s) on l2 at a distance of 17 from the point of intersection of l and l1
is(are) [JEE (Advanced) 2013, Paper-1, (4, – 1)/60]

RI
7 7 5 7 7 8
(A)  , ,  (B) (–1, ,–1, 0) (C) (1, 1, 1) (D)  , , 
 3 3 3  9 9 9

 
Consider the set of eight vectors V = a ˆi + bjˆ + ckˆ : a,b,c  –1,1 . Three non-coplanar vectors can be

A
25.
chosen from V in 2 ways. Then p is
p
[JEE (Advanced) 2013, Paper-1, (4, – 1)/60]

UH
y z y z
26.* Two lines L1 : x = 5, = and L2 : x = , = are coplanar. Then  can take value(s)
3– –2 –1 2–
                [JEE (Advanced) 2013, Paper-2, (3, –1)/60]
(A) 1 (B) 2 (C) 3 (D) 4
JA
27. Match List I with List II and select the correct answer using the code given below the
lists : [JEE (Advanced) 2013, Paper-2, (3, –1)/60]
List - I List - II
P. Volume of parallelopiped determined by vectors 1. 100
a,b and c is 2. Then the volume of the parallelepiped
determined by vectors 2(a  b),3(b  c) and (c  a) is
LP

Q. Volume of parallelepiped determined by vectors a,b 2. 30


and c is 5. Then the volume of the parallelepiped
determined by vectors 3(a + b),(b + c) and 2 (c + a) is
A

R. Area of a triangle with adjacent sides determined by 3. 24


vectors a and b is 20. Then the area of the triangle
NK

with adjacent sides determined by vectors ( 2a + 3b )


and (a – b) is

S. Area of a paralelogram with adjacent sides determined by 4. 60


SA

vectors a and b is 30. Then the area of the parallelogram


with adjacent sides determined by vectors (a + b) and a is
Codes :
P Q R S
(A) 4 2 3 1
(B) 2 3 1 4
(C) 3 4 1 2
(D) 1 4 3 2

x –1 y z+3 x– 4 y+3 z+3


28. Consider the lines L1 : = = , L2 = = : and the planes P1 : 7x + y + 2z = 3,
2 –1 1 1 1 2
P2 : 3x + 5y – 6z = 4. Let ax + by + cz = d the equation of the plane passing through the point of
intersection of lines L1 and L2, and perpendicular to planes P1 and P2.
Match List - I with List- II and select the correct answer using the code given below the lists :
[JEE (Advanced) 2013, Paper-2, (3, –1)/60]
List- I List- II
P. a= 1. 13
Q. b= 2. –3
R. c= 3. 1
S. d= 4. –2
Codes :
P Q R S
(A) 3 2 4 1
(B) 1 3 4 2
(C) 3 2 1 4
(D) 2 4 1 3   

RI
29*. Let x,y and z be three vectors each of magnitude 2 and the angle between each pair

of them is . If a is a nonzero vector perpendicular to x and y  z and b is a nonzero
3
vector perpendicular to y and z  x , then [JEE (Advanced) 2014, Paper-1, (3, 0)/60]

A
(A) b = (b.z)(z – x) (B) a = (a.y)(y – z) (C) a.b = –(a.y)(b.z) (D) a = (a.y)(z – y)

UH
30. From a point P(,,), perpendiculars PQ and PR are drawn respectively on the lines y =
x, z = 1 and y = –x, z = –1. If P is such that QPR is a right angle, then the possible
value(s) of  is(are) [JEE (Advanced) 2014, Paper-1, (3, 0)/60]
(A) 2 (B) 1 (C) –1 (D) – 2
JA
31. Let a, b and c be three non-coplanar unit vectors such that the angle between every

pair of them is . If a  b + b  c = pa + qb + rc , where p,q and r are scalars, then the value
3
p2 + 2q2 + r 2
of is [JEE (Advanced) 2014, Paper-1, (3, 0)/60]
q2
LP

32. List I List II


[JEE (Advanced) 2014, Paper-2, (3, –1)/60]
3
P. Let y(x) = cos(3 cos – 1 x), x  [–1, 1], x  ± . Then 1. 1
2
A

1  2 d2 y(x) dy(x) 
y(x) 
(
 x –1
dx
)
2
+x
dx 
 equals
NK

Q. Let A 1 , A 2 ,......, A n (n > 2) be the vertices of a regular polygon of n 2. 2


sides with its centre at the origin. Let ak be the position vector of
n–1 n–1
the point A k , k = 1, 2,...., n. If  (a
k =1
k  ak +1 ) =  (a .a
k =1
k k +1 ) , then

the minimum value of n is


SA

x2 y2
R. If the normal from the point P(h, 1) on the ellipse + = 1 is 3. 8
6 3
perpendicular to the line x + y = 8, then th e value of h is

S. Number of positive solutions satisfying the equation 4. 9


–1  1   1   2
is tan   + tan–1   = tan–1  2 
 2x + 1   4x + 1  x 

P Q R S
(A) 4 3 2 1
(B) 2 4 3 1
(C) 4 3 1 2
(D) 2 4 1 3
33*. In R3, consider the planes P1 : y = 0 and P2 : x + z = 1. Let P3 be a plane, different from P1 and P2, which
passes through the intersection of P1 and P2 . If the distance of the point (0, 1, 0) from P 3 is 1 and the
distance of a point (, , ) from P3 is 2, then which of the following relation is (are) true ?
[JEE (Advanced) 2015, P-1 (4, –2)/ 88]
(A) 2 +  + 2 + 2 = 0 (B) 2 –  + 2 + 4 = 0
(C) 2 +  – 2 – 10 = 0 (D) 2 –  + 2 – 8 = 0

34*. In R3, let L be a straight line passing through the origin. Suppose that all the points on L are at a
constant distance from the two planes P1 : x + 2y – z + 1 = 0 and P2 : 2x – y + z – 1 = 0. Let M be the
locus of the feet of the perpendiculars drawn from the points on L to the plane P 1. Which of the following
points lie(s) on M? [JEE (Advanced) 2015, P-1 (4, –2)/ 88]
 5 2  1 1 1  5 1  1 2
(A)  0, – , –  (B)  – , – ,  (C)  – , 0,  (D)  – , 0, 
 6 3   6 3 6   6 6   3 3 

RI
 
35*. Let PQR be a triangle. Let a = QR, b = RP and c = PQ . If | a | = 12, | b |= 4 3 and b.c = 24 , then
which of the following is(are) true? [JEE (Advanced) 2015, P-1 (4, –2)/ 88]

A
| c |2 | c |2
(A) − | a |= 12 (B) + | a | = 30
2 2
(C) | a  b + c  a |= 48 3 (D) a. b = −72

UH
36. Column-  Column-
[JEE (Advanced) 2015, P-1 (2, –1)/ 88]

(A) In R2, if the magnitude of the projection vector of the vector (P) 1
JA
ˆi + ˆj on 3iˆ + ˆj is 3 and if  = 2 + 3  ,
then possible value(s) of || is (are)

(B) Let a and b be real numbers such that the function (Q) 2
 –3ax 2 – 2 , x  1
f(x) =  is differentiable for all x  R.
 bx + a , x  1
2
LP

Then possible value(s) of a is (are)

(C) Let   1 be a complex cube root of unity. (R) 3


If (3 – 3 + 22)4n + 3 + (2 + 3 – 32)4n + 3 + (–3 + 2 + 32)4n + 3 = 0,
A

then possible value(s) of n is (are)

(D) Let the harmonic mean of two positive real numbers a and b be 4. (S) 4
NK

If q is a positive real number such that a, 5,q, b is an arithmetic


progression, then the value(s) of |q – a| is (are)
(T) 5

37. Column-  Column-


SA

(A) In a triangle XYZ, let a, b and c be the lengths of the sides (P) 1
opposite to the angles X, Y and Z, respectively. If 2(a 2 – b2) = c2
sin(X – Y)
and  = , then possible values of n for which
sin Z
cos(n) = 0 is (are)
(B) In a triangle XYZ, let a, b and c be the lengths of the sides (Q) 2
opposite to the angles X, Y and Z, respectively. If
a
1 + cos2X – 2cos2Y = 2sinXsinY, then possible value(s) of
b
is (are)
(C) In R2, let 3 ˆi + ˆj, ˆi + 3ˆj and  ˆi + (1– ) ˆj be the position vectors (R) 3
of X, Y and Z with respect to the origin O, respectively. If the
distance of Z from the bisector of the acute angle of OX and OY
3
is , then possible value(s) of || is (are)
2

(D) Suppose that F() denotes the area of the region bounded by (S) 5
x = 0, x = 2, y2 = 4x and y = |x – 1| + |x – 2| + x, where
8
  {0, 1}. Then the value(s) of F() + 2 , when  = 0 and
3
 = 1, is (are) [JEE (Advanced) 2015, P-1 (2, –1)/ 88]

38. Suppose that p,q and r are three non-coplanar vectors in R3. Let the components of a vector s
along p,q and r be 4, 3 and 5, respectively. If the components of this vector s along
(–p + q + r ),(p − q + r ) and (–p – q + r ) are x, y and z, respectively, then the value of 2x + y + z is

RI
          [JEE (Advanced) 2015, P-2 (4, 0) / 80]  

39*. Consider a pyramid OPQRS located in the first octant (x  0, y  0, z  0) with O as origin, and OP and
OR along the x-axis and the y-axis, respectively. The base OPQR of the pyramid is a square with

A
OP = 3. The point S is directly above the mid point T of diagonal OQ such that TS = 3. Then

(A) the acute angle between OQ and OS is [JEE (Advanced) 2016, Paper-1, (4, –2)/62]
3

UH
(B) the equation of the plane containing the triangle OQS is x – y = 0
3
(C) the length of the perpendicular from P to the plane containing the triangle OQS is
2
15
(D) the perpendicular distance from O to the straight line containing RS is
2
JA
40. Let P be the image of the point (3, 1, 7) with respect to the plane x – y + z = 3. Then the equation of the
x y z
plane passing through P and containing the straight line = = is
1 2 1
                [JEE (Advanced) 2016, Paper-2, (3, –1)/62]
(A) x + y – 3z = 0 (B) 3x + z = 0 (C) x – 4y + 7z = 0 (D) 2x – y = 0
LP

1 ˆ ˆ
41*. Let û = u1ˆi + u2 ˆj + u3kˆ be a unit vector in R3 and ŵ = (6
)
i + j + 2kˆ . Given that there exists a vector
r r r
u in R3 such that û´ u = 1 and w. ˆ (uˆ ´ u) = 1. Which of the following statements(s) is (are) correct?
r
A

(A) There is exactly one choice for u


r
(B) There are infinitely many choices for such u [JEE (Advanced) 2016, Paper-2, (4, –2)/62]
(C) If û lies in the xy-plane then u1 = u2
NK

(D) If û lies in the xz-plane then 2 u1 = u3

42. Let O be the origin and let PQR be an arbitrary triangle. The point S is such that
OP . OQ + OR . OS = OR . OP + OQ . OS = OQ . OR + OP . OS
Then the triangle PQR has S as its [JEE(Advanced) 2017, Paper-2,(3, –1)/61]
SA

(A) centroid (B) orthocenter


(C) incentre (D) circumcenter

43. The equation of the plane passing through the point (1, 1, 1) and perpendicular to the planes
2x + y – 2z = 5 and 3x – 6y – 2z = 7, is [JEE(Advanced) 2017, Paper-2,(3, –1)/61]
(A) 14x + 2y – 15z = 1 (B) –14x + 2y + 15z = 3
(C) 14x – 2y + 15z = 27 (D) 14x + 2y + 15z = 31

Comprehension (Q.44 & 45)

Let O be the origin, and OX, OY, OZ be three unit vectors in the directions of the sides QR, RP, PQ ,
respectively, of a triangle PQR.

44. If the triangle PQR varies, then the minimum value of cos(P + Q) + cos(Q + R) + cos(R + P) is
[JEE(Advanced) 2017, Paper-2,(3, 0)/61]
3 3 5 5
(A) – (B) (C) (D) –
2 2 3 3

45. | OX  OY| = [JEE(Advanced) 2017, Paper-2,(3, 0)/61]


(A) sin(P + Q) (B) sin(P + R) (C) sin(Q + R) (D) sin2R

46*. Let P1 : 2x + y – z = 3 and P2 : x + 2y + z = 2 be two planes. Then, which of the following statement(s) is
(are) TRUE? [JEE(Advanced) 2018, Paper-1,(4, –2),60]
(A) The line of intersection of P1 and P2 has direction ratios 1, 2, – 1
3x − 4 1 − 3y z
(B) The line = = is perpendicular to the line of intersection of P 1 and P2
9 9 3
(C) The acute angle between P1 and P2 is 60º
(D) If P3 is the plane passing through the point (4, 2, –2) and perpendicular to the line of intersection of

RI
2
P1 and P2, then the distance of the point (2, 1, 1) from the plane P 3 is
3

47. Let a and b be two unit vectors such that a . b = 0 . For some x, y  R, let c = xa + yb + (a  b) . If

A
| c | = 2 and the vector c is inclined at the same angle  to both a and b , then the value of 8 cos2 is
_______. [JEE(Advanced) 2018, Paper-1, (3, 0), 60]

UH
48. Let P be a point in the first octant, whose image Q in the plane x + y = 3 (that is, the line segment PQ is
perpendicular to the plane x + y = 3 and the mid-point of PQ lies in the plane x + y = 3) lies on the
z-axis. Let the distance of P from the x-axis be 5. If R is the image of P in the xy-plane, then the length
of PR is _______ . [JEE(Advanced) 2018, Paper-2,(3, 0)/60]
JA
49. Consider the cube in the first octant with sides OP, OQ and OR of length 1, along the x-axis, y-axis and
 1 1 1
z-axis, respectively, where O(0, 0, 0) is the origin. Let S  , ,  be the centre of the cube and T be
2 2 2
the vertex of the cube opposite to the origin O such that S lies on the diagonal OT. If p = SP ,
q = SQ , r = SR and t = ST , then the value of |( p × q ) × ( r × t )| is ______ .
LP

[JEE(Advanced) 2018, Paper-2,(3, 0), 60]

PART - II : JEE (MAIN) / AIEEE PROBLEMS (PREVIOUS YEARS)


A

→ → →
1. If u, v, w are non-coplanar vectors and p, q are real numbers, then the equality
→ → → → → → → → →
[3 u p v p w ] – [p v w q u ] – [2 w q v q u ] = 0 holds for- [AIEEE 2009 (4, –1), 144]
NK

(1) Exactly two values of (p, q) (2) More than two but not all values of (p, q)
(3) All values of (p, q) (4) Exactly one value of (p, q)
 
x–2 y –1 z+2
2. Let the line = = lies in the plane x + 3y – z +  = 0. Then (, ) equals
3 –5 2
[AIEEE 2009 (4, –1), 144]
SA

(1) (6, – 17) (2) (– 6, 7) (3) (5, – 15) (4) (– 5, 15)

3. The projections of a vector on the three coordinate axes are 6, –3, 2 respectively. The direction cosines
of the vector are. [AIEEE 2009 (4, –1), 144]
6 3 2 6 3 2 6 3 2
(1) 6, –3, 2 (2) , – , (3) , – , (4) – , – ,
5 5 5 7 7 7 7 7 7

4. Let a = ˆj – kˆ and c = ˆi – ˆj – kˆ . Then the vector b satisfying a  b + c = 0 and a.b = 3 is


                    [AIEEE 2010 (4, –1), 144]
ˆ ˆ
(1) 2i – j + 2k ˆ ˆ ˆ
(2) i – j – 2k ˆ ˆ ˆ ˆ
(3) i + j – 2k (4) – ˆi + ˆj – 2kˆ
 
5. If the vectors a = ˆi – ˆj + 2kˆ , b = 2iˆ + 4jˆ + kˆ and c =  ˆi + ˆj + µkˆ are mutually orthogonal, then ( , µ) =
                    [AIEEE 2010 (4, –1), 144]
(1) (2, – 3) (2) (–2, 3) (3) (3, – 2) (4) (–3, 2)

6. Statement -1 : The point A(3, 1, 6) is the mirror image of the point B(1, 3, 4) in the plane x – y + z = 5.
Statement -2 : The plane x – y + z = 5 bisects the line segment joining A(3, 1,6) and B(1, 3, 4).
(1) Statement -1 is true, Statement-2 is true ; Statement -2 is not a correct explanation for Statement -1.
(2) Statement-1 is true, Statement-2 is false. [AIEEE 2009 (4, –1), 144]
(3) Statement -1 is false, Statement -2 is true.
(4) Statement -1 is true, Statement -2 is true; Statement-2 is a correct explanation for Statement-1.
 
7. A line AB in three-dimensional space makes angles 45º and 120º with the positive x-axis and the
positive
y-axis respectively. If AB makes an acute angle  with the positive z-axis, then  equal
                    [AIEEE 2010 (4, –1), 144]
(1) 45º (2) 60º (3) 75º (4) 30º

RI
1 ˆ and b = 1 (2iˆ + 3ˆj − 6k)
8. If a = (3iˆ + k) ˆ , then the value of (2a − b) . [(a  b)  (a + 2b)] is:
10 7
                    [AIEEE 2011, I, (4, –1), 120]

A
(1) – 5 (2) –3 (3) 5 (4) 3

9. The vectors a and b are not perpendicular and c and d are two vectors satisfying : b  c = b  d and

UH
a.d = 0 . Then the vector d is equal to : [AIEEE 2011, I, (4, –1), 120]
 b.c   a.c   b.c   a.c 
(1) b −  (4) c − 
 a.d 
(3) b + 
 a.b 
(2) c + 
 a.b   a.b 
c b c b
       
JA
10. If the vector p î + ĵ + k̂ , î + q ĵ + k̂ and î + ĵ + r k̂ (p  q  r  1) are coplanar, then the value of
pqr – (p + q + r) is- [AIEEE 2011, II, (4, –1), 120]
(1) 2 (2) 0 (3) –1 (4) –2

11. Let a , b , c be three non-zero vectors which are pairwise non-collinear. If a + 3 b is collinear with c
and b + 2 c is collinear with a , then a + 3 b + 6 c is : [AIEEE 2011, II, (4, –1), 120]
LP

(1) a (2) c (3) 0 (4) a + c

y −1 z−3  5 
12. If the angle between the line x = = and the plane x + 2y + 3z = 4 is cos –1   , then 
2   14 
A

equals : [AIEEE 2011, I, (4, –1), 120]


2 3 2 5
(1) (2) (3) (4)
3 2 5 3
NK

13. Statement-1 : The point A(1, 0, 7) is the mirror image of the point B(1, 6, 3) in the line :
x y −1 z − 2
= =
1 2 3
x y −1 z − 2
Statement-2 : The line : = = bisects the line segment joining A(1, 0, 7) and B(1, 6, 3).
SA

1 2 3
(1) Statement-1 is true, Statement-2 is true; Statement-2 is a correct explanation for Statement-1.
(2) Statement-1 is true, Statement-2 is true; Statement-2 is not a correct explanation for Statement-1.
(3) Statement-1 is true, Statement-2 is false.
(4) Statement-1 is false, Statement-2 is true. [AIEEE 2011, I, (4, –1), 120]
14. The distance of the point (1, –5, 9) from the plane x – y + z = 5 measured along a straight line
x = y = z is : [AIEEE 2011, II, (4, –1), 120]
(1) 10 3 (2) 5 3 (3) 3 10 (4) 3 5

x y−2 z−3
15. The length of the perpendicular drawn from the point (3, –1, 11) to the line = = is :
2 3 4
[AIEEE 2011, II, (4, –1), 120]
(1) 29 (2) 33 (3) 53 (4) 66
16. Let â and b̂ be two unit vectors. If the vectors c = aˆ + 2bˆ and d = 5aˆ − 4bˆ are perpendicular to each
other, then the angle between â and b̂ is : [AIEEE-2012, (4, –1)/120]
   
(1) (2) (3) (4)
6 2 3 4

17. A equation of a plane parallel to the plane x – 2y + 2z – 5 = 0 and at a unit distance from the origin is :
                    [AIEEE 2012, (4, –1), 120]
(1) x – 2y + 2z – 3 = 0 (2) x – 2y + 2z + 1 = 0 (3) x – 2y + 2z – 1 = 0 (4) x – 2y + 2z + 5 = 0

x −1 y +1 z −1 x−3 y −k z
18. If the line = = and = = intersect, then k is equal to :
2 3 4 1 2 1
[AIEEE 2012, (4, –1), 120]
2 9

RI
(1) – 1 (2) (3) (4) 0
9 2

19. Let ABCD be a parallelogram such that AB = q , AD = p and BAD be an acute angle. If r is the

A
vector that coincides with the altitude directed from the vertex B to the side AD, then is given by :
[AIEEE-2012, (4, –1)/120]
3(p . q) p . q p . q 3(p . q)
(1) r = 3q − p (2) r = −q +   p (3) r = q −   p (4) r = −3q + p

UH
(p . p)  p . p   p . p  (p . p)

20. Distance between two parallel planes 2x + y + 2z = 8 and 4x + 2y + 4z + 5 = 0 is


[AIEEE - 2013, (4, –1),360]
3 5 7 9
(1) (2) (3) (4)
JA
2 2 2 2

x–2 y–3 z–4 x –1 y–4 z–5


21. If the lines = = and = = are coplanar, then k can have
1 1 –k k 2 1
[AIEEE - 2013, (4, –1),360]
(1) any value (2) exactly one value (3) exactly two values (4) exactly three values
LP

22. If the vectors AB = 3iˆ + 4kˆ and AC = 5iˆ – 2jˆ + 4kˆ are the sides of a triangle ABC, then the length of
the median through A is [AIEEE - 2013, (4, –¼),360]
(1) 18 (2) 72 (3) 33 (4) 45
A

x −1 y − 3 z − 4
23. The image of the line = = in the plane 2x – y + z + 3 = 0 is the line :
3 1 −5
[JEE(Main) 2014, (4, – 1), 120]
NK

x−3 y+5 z−2 x−3 y+5 z−2


(1) = = (2) = =
3 1 −5 −3 −1 5
x+3 y−5 z−2 x+3 y−5 z+2
(3) = = (4) = =
3 1 −5 −3 −1 5
24. The angle between the lines whose direction cosines satisfy the equations  + m + n = 0 and
SA

2 = m2 + n2 is [JEE(Main) 2014, (4, – 1), 120]


   
(1) (2) (3) (4)
6 2 3 4

2
25. If a  b bc c  a  =  a
 
b c  then  is equal to
 [JEE(Main) 2014, (4, – 1), 120]
(1) 0 (2) 1 (3) 2 (4) 3
x–2 y +1 z–2
26. The distance of the point (1,0,2) from the point of intersection of the line = = and the
3 4 12
plane x – y + z = 16, is [JEE(Main) 2015, (4, – 1), 120]
(1) 2 14 (2) 8 (3) 3 21 (4) 13

27. The equation of the plane containing the line 2x – 5y + z = 3, x + y + 4z = 5 and parallel to the plane
x + 3y + 6z = 1, is [JEE(Main) 2015, (4, – 1), 120]
(1) 2x + 6y + 12z = 13 (2) x + 3y + 6z = –7 (3) x + 3y + 6z = 7 (4) 2x + 6y + 12z = –13

28. Let a, b and c be three non-zero vectors such that no two of them are collinear and
1
(a b)  c = | b || c | a . If  is the angle between vectors b and c , then a value of sin is
3
[JEE(Main) 2015, (4, – 1), 120]
2 2 – 2 2 –2 3
(1) (2) (3) (4)
3 3 3 3

x–3 y+2 z+4


29. If the line, = = lies in the plane, lx + my – z = 9, then l2 + m2 is equal to
2 –1 3
[JEE(Main) 2016, (4, – 1), 120]
(1) 18 (2) 5 (3) 2 (4) 26

RI
3
30. Let a, b and c be three unit vectors such that a  b  c = ( ) 2
(b + c) . If b is not parallel to c , then

A
the angle between a and b is [JEE(Main) 2016, (4, – 1), 120]
 2 5 3
(1) (2) (3) (4)
2 3 6 4

UH
31. The distance of the point (1, –5, 9) from the plane x – y + z = 5 measured along the line x = y = z is
[JEE(Main) 2016, (4, – 1), 120]
10 20
(1) 10 3 (2) (3) (4) 3 10
3 3
JA
32. If the image of the point P(1, –2, 3) in the plane, 2x + 3y – 4z + 22 = 0 measured parallel to the line,
x y z
= = is Q, then PQ is equal to : [JEE(Main) 2017, (4, – 1), 120]
1 4 5
(1) 3 5 (2) 2 42 (3) 42 (4) 6 5
 
LP

33. The distance of the point (1, 3, – 7) from the plane passing through the point (1, –1, –1), having normal
x −1 x+2 x−4 x − 2 y +1 z + 7
perpendicular to both the lines = = and = = , is
1 −2 3 2 −1 −1
[JEE(Main) 2017, (4, – 1), 120]
20 10 5 10
A

(1) (2) (3) (4)


74 83 83 74
34. Let a = 2iˆ + ˆj – 2kˆ and b = ˆi + ˆj . Let c be a vector such that c – a = 3, (a  b)  c = 3 and the angle
NK

between c and a  b be 30º. Then a.c is equal to [JEE(Main) 2017, (4, – 1), 120]
25 1
(1) (2) 2 (3) 5 (4)
8 8
SA

35. If L1 is the line of intersection of the planes 2x – 2y + 3z – 2 = 0, x – y + z + 1 = 0 and L 2 is the line of


intersection of the planes x + 2y – z – 3 = 0, 3x – y + 2z – 1 = 0, then the distance of the origin from the
plane, containing the lines L1 and L2 , is : [JEE(Main) 2018, (4, –1), 120]
1 1 1 1
(1) (2) (3) (4)
2 2 2 4 2 3 2

36. Let u be a vector coplanar with the vectors a = 2 î + 3 ˆj – k̂ and b = ĵ + k̂ . If u is perpendicular to a


2
and u . b = 24, then u is equal to : [JEE(Main) 2018, (4, –1), 120]
(1) 256 (2) 84 (3) 336 (4) 315

37. The length of the projection of the line segment joining the points (5, – 1, 4) and (4,–1,3) on the plane ,
x + y + z = 7 is : [JEE(Main) 2018, (4, –1), 120]
1 2 2 2
(1) (2) (3) (4)
3 3 3 3

38. If the lines x = ay + b, z = cy + d and x = a'z + b', y = c'z + d' are perpendicular, then :
[JEE(Main) 2019, Online (09-01-19),P-2 (4, – 1), 120]
(1) ab' + bc' + 1 = 0 (2) bb' + cc' + 1 = 0
(3) cc' + a + a' = 0 (4) aa' + c + c' = 0

  
39. Let a = î + ĵ + 2 k̂ , b = b1î + b2 ĵ + 2 k̂ and c = 5 î + ĵ + 2 k̂ be three vectors such that the projection
      
vector of b on a is a . If a + b is perpendicular to c , then | b | is equal to :
[JEE(Main) 2019, Online (09-01-19),P-2 (4, – 1), 120]

RI
(1) 22 (2) 4 (3) 32 (4) 6

40. A tetrahedron has vertices P(1,2,1), Q(2,1,3), R(–1,1,2) and O(0,0,0) the angle between the faces OPQ

A
and PQR is : [JEE(Main) 2019, Online (12-01-19),P-1 (4, – 1), 120]
 19  7  17   9 
(1) cos−1   (2) cos –1  (3) cos−1   (4) cos−1  
 35   31   31   35 

UH
41. Let S be the set of all real values of  such that a plane passing through the points (– 2 , 1, 1),
(1, –2, 1) and (1, 1, –2) also passes through the point (–1, –1, 1). Then S is equal to -
[JEE(Main) 2019, Online (12-01-19),P-2 (4, – 1), 120]
(1) {1, –1} (2) { 3 } (3) { 3,– 3 } (4) {3, –3}
JA
LP
A
NK
SA
EXERCISE # 1

PART-I
Section (A) :

2c + b – 3a 3c + 5a – 8d 5b + a – 6c
A-1. (i) (ii) (iii)
3 5 6
 6 2 3   –6 –2 –3 
A-2. 5iˆ +ˆj +4kˆ , 4iˆ −ˆj +5kˆ A-4.  7 , 7 , 7 ,  7 , 7 , 7 
   

RI
1 1 2 2 −1 −1
A-5. − , − , or , ,
6 6 6 6 6 6
 

A
Section (B) :
35 6  10 7
B-1. A = cos–1 , B = cos–1 , C = B-3. (i) (ii) (iii) 2 2 – 2

UH
41 41 2 6 3

6 2
B-6. 0 B-8. (a) – î + ĵ + k̂ (b) (6 î – ĵ + k̂ ) (c)
19 3
JA
2  1
B-9. – 169 B-10.  = cos–1 B-12. (i) cos–1   (ii) 60º B-13. ± ( 4iˆ – 6ˆj + 12kˆ )
3 713 6

(
B-14. 3 − ˆi + ˆj + kˆ ) B-15. (b) 5 unit sq.  

 
LP

Section (C) :
 
1 5 
C-1.  2 , 2 ,0  C-2. (1, 3, 5)
 
A

x − 2 y +1 z − 4
C-3. (i) = = (ii) r = (2iˆ − 2jˆ + k)
ˆ + (3iˆ + 4jˆ − k)
ˆ
1 1 −2
 
NK

x y z x y z a+b+c
C-4. 26 C-5. 3 C-6. = = , = = C-7. r=
1 2 −1 −1 1 −2 3

6
C-8. unit     C-9. A = (3, 8, 3), B = (–3, – 7, 6), AB = 3 30
SA

C-10. r = (iˆ + 2ˆj + 3k)


ˆ + t ( ˆj − k)
ˆ , where t is parameter

Section (D) :

1 2
D-2. sin  cos  D-3. (a1 + a22 + a32 ) (b12 + b22 + b32 )
4
D-4. (a) Coplanar (b) Non-coplanar   
 
3
D-5. (i) 1/2 unit3 (ii) unit D-6. (i) No (ii) Yes
35

D-7. x=1 D-8. v =0 D-9. (i) p = 0; q = 10; r = − 3 (ii) – 100


(p . q) p d  (c  d) – 2 | d |2 a
D-10. x = q− D-11. x =
2 | p |2 d.a

Section (E) :
E-1. (i) 4x – y – z + 1 = 0 (ii) x + 2y + 3z – 2 = 0 (iii) x+y+z–4=0
(iv) x+y+z–6=0

E-2. 3 : 2, (0,13/5, 2) E-3. x2 + y2 + z2 = 9 E-4. r . (4iˆ − 2jˆ − 5k)


ˆ = 45

4 x–3 y+2 z–5 x−4 y +1 z−7


E-5. sin–1 E-6. = = E-7. = =
−1 −3

RI
30 3 4 5 9

E-8. /2 E-9. 11x − y − 3z = 35 E-10. 7 E-11. 0

A
E-12. 8x – 13y + 15z + 13 = 0 E-13. ˆ ,  76 , –108 , –170 
r = 2iˆ – 3ˆj – 5kˆ + t (6iˆ – 3jˆ + 5k)  35 35 
 35

UH
E-14. x – y + 3z – 2 = 0 ; (3, 1, 0) ; 11 E-15. x+y ± 2 z=1

5
E-16. unit E-17. 1 E-18. (i) r . n̂ = ±p (ii) r . (aq − pb ) = 0
3
JA
PART - II

Section (A) :
 
A-1. (C) A-2. (C) A-3. (B) A-4. (B)
LP

Section (B) :
 
B-1. (D) B-2. (B) B-3. (C) B-4. (C) B-5. (C) B-6. (B) B-7. (B)
A

B-8. (C)

Section (C) :
NK

C-1. (A) C-2. (C) C-3. (A) C-4. (A) C-5. (D) C-6. (C)
 
Section (D) :

D-1. (C) D-2. (C) D-3. (C) D-4. (B) D-5. (A) D-6. (D)
SA

D-7. (D) D-8. (D) D-9. (B) D-10. (B) D-11. (B) D-12. (B)

Section (E) :

E-1. (A) E-2. (B) E-3. (D) E-4. (A) E-5. (D) E-6. (A) E-7. (D)

E-8. (A) E-9. (A) E-10. (C)

PART - III

1. (A) → (r), (B) → (q), (C) → (q), (D) → (s)

2. (A) → (q), (B) → (p), (C) → (s), (D) → (r)  3. (A)→(q), (B) → (p, r), (C) → (r), (D) → (s)
EXERCISE # 2
1. (D) 2. (C) 3. (C) 4. (A) 5. (C) 6. (A) 7. (C)

8. (C) 9. (D) 10. (B) 11. (C) 12. (A) 13. (C) 14. (A)

15. (A) 16. (C) 17. (A) 18 . (D) 19. (A) 20. (B) 21. (A)

22. (A) 23. (B)


PART - II
1. 5 2. 19 3. 6 4. 9 5. 9 6. 2 7. 72

RI
8. 34 9. 2 10. 18 11. 36 12. 6 13. 4 14. 32

15. 27 16. 17 17. 13 18. 5 19. 4

A
PART - III

1. (AB) 2. (ABC) 3. (ABCD) 4. (BCD) 5. (BD) 6. (ABC)

UH
7. (ABCD) 8. (AC) 9. (BD) 10. (BC) 11. (BC) 12. (B)

13. (AB) 14. (ABCD)15. (ABC) 16. (BD) 17. (D) 18. (AB)
JA
19. (AB) 20. (AD) 21. (AC) 22. (AD) 23. (AB) 24. (ABC)

25. (ABC) 26. (AD) 27. (BCD) 28. (AB) 29. (ABCD) 30. (AD)

31. (ABD) 32. (ACD)


LP

PART - IV

1. (A) 2. (A) 3. (D) 4. (B) 5. (C) 6. (A) 7. (A)


A

8. (B) 9. (A) 10. (A) 11. (C) 12. (C) 13. (A) 14. (A)

15. (A)
NK

EXERCISE # 3
PART - I
SA

1. (C) 2. (A) → (q, s), (B) → (p, r, s, t), (C) → (t), (D) → (r) 3. (C)

4. (A) 5. (A) 6. 5 7. (B) 8. (C) 9. (A)


 
10. 6 11. (A) 12. (A) → (t), (B) → (p, r), (C) → (q) (JEE given q, s) (D) → (r)

13. (C) 14*. (AD) 15. 9 16. (A) → (q), (B) → (p), (C) → (s), (D) → (t)

17. (A) 18. 3 19. (A) 20. (C) 21*. (BC) 22. (C)
23. (D) 24.* (BD) 25. 8
C3 – 24 = 32 26.* (AD) 27. (C) 28. (A)

29. (ABC) 30. (C) 31. (4) 32. (A) 33*. (BD) 34*. (AB)

35*. (ACD) 36. (A) → P,Q ; (B) → P, Q ; (C) → P,Q,S,T ; (D) → Q, T

37. (A) → P,R,S ; (B) → P ; (C) → P,Q ; (D) → S, T 38. BONUS 39*. (BCD)

40. (C) 41. (BC) 42. (B) 43. (D) 44. (A) 45. (A)

RI
46*. (CD) 47. (3) 48. (8) 49. (0.5)

PART - II

A
1. (4) 2. (2) 3. (3) 4. (4) 5. (4) 6. (1) 7. (2)

UH

8. (1) 9. (4) 10. (4) 11. (3) 12. (1) 13. (2) 14. (1)

15. (3) 16. (3) 17. (1) 18. (3) 19. (2) 20. (3) 21. (3)
JA
22. (3) 23. (3) 24. (3) 25. (2) 26. (4) 27. (3) 28. (1)

29. (3) 30. (3) 31. (1) 32. (2) 33. (2) 34. (2) 35. (4)
LP

36. (3) 37. (2) 38. (4) 39. (4) 40. (1) 41. (3)
A
NK
SA
1. Using Vectors prove that
(i) cos(A – B) = cosA cosB + sinA sinB (ii) sin(A + B) = sinA cosB + cosA sinB

2. Using vectors, prove that the altitudes of a triangle are concurrent.


 
3. Prove that the direction cosines of a line equally inclined to three mutually perpendicular lines having
+ 2 + 3 m1 + m2 + m3 n1 + n2 + n3
D.C.’s as 1, m1, n1 ; 2, m2, n2 ; 3, m3, n3 are 1 , , .
3 3 3
4. If A (a) , B (b) , C (c) , D (d) are the position vector of cyclic quadrilateral then find the value of

RI
| a b + b  d + d a | | b  c + c  d + db |
+ . (It is given that no angle of cyclic quadrilateral ABCD is
[(b − a).(d − a)] [(b − c).(d − c)]
right angle)

A
5. Prove that the volume of tetrahedron bounded by the planes,
3
ˆ = 0 , r .(nkˆ + ˆi) = 0 , r .( ˆi + m ˆj) = 0 , r.(
r.(mjˆ + nk) ˆ = p is 2p .
ˆi + m ˆj + n k)
3 mn

UH
6. In a  ABC, let M be the mid point of segment AB and let D be the foot of the bisector of  C. Then
ar( CDM) 1 a−b 1 A −B A +B
prove that = = tan cot .
ar( ABC) 2 a+b 2 2 2
JA
7. Let ABC be a triangle.Points M, N and P are taken on the sides AB, BC and CA respectively such that
AM BN CP
= = =  . Prove that the vectors AN , BP and CM form a triangle. Also find  for which
AB BC CA
the area of the triangle formed by these vectors is the least.

8. In any triangle, show that the perpendicular bisectors of the sides are concurrent.
LP

 
9. Let ABC be an acute-angled triangle AD be the bisector of BAC with D on BC and BE be the altitude
from B on AC. Show that CED > 45º.
 
A

10. In a quadrilateral ABCD, it is given that AB || CD and the diagonals AC and BD are perpendicular to
each other. Show that
(a) AD. BC  AB.CD (b) AD + BC  AB + CD
NK

11. A, B, C, D are four points in space. using vector methods, prove that
AC2 + BD2 + AD2 + BC2  AB2 + CD2 what is the implication of the sign of equality.

12. The direction cosines of a variable line in two near by positions are l, m, n; l + l, m + m, n + n. Show
that the small angle  between the two position is given by ()2 = (l)2 + (m)2 + (n)2.
SA

 a2 + b2 + c 2 
13. In a ABC, prove that distance between centroid and circumcentre is R2 −  
 9 
where R is the circumradius and a, b, c denotes the sides of ABC.
 
14. Prove that the square of the perpendicular distance of a point P (p, q, r) from a line through A(a, b, c)
and whose direction cosines are , m, n is  {(q – b) n – (r – c) m}2.
15. (i) Let 1 & 2 be two skew lines. If P, Q are two distinct points on 1 and R, S are two distinct points
on 2 , then prove that PR can not be parallel to QS.
(ii) A line with direction cosines proportional to (2, 7 – 5) is drawn to intersect the lines
x–5 y–7 z+2 x+3 y–3 z–6
= = and = = . Find the coordinate of the points of
3 –1 1 –3 2 4
intersection and the length intercepted on it. Also find the equation of intersecting straight line.
16. The base of the pyramid AOBC is an equilateral triangle OBC with each side equal to 4 2 . ' O ' is the
origin of reference, AO is perpendicular to the plane of  OBC and | AO | = 2 . Then find the cosine of
the angle between the skew straight lines one passing through A and the mid point of OB and the other
passing through 'O' and the mid point of BC.

17. If D, E, F be three point on BC, CA, AB respectively of a ABC. Such that the line AD, BE, CF are
BD CE AF
concurrent then find the value of . . .
CD AE BF

18. Without expanding the determinant, Prove that


na1 + b1 na2 + b2 na3 + b3 a1 a2 a3
nb1 + c1 nb2 + c 2 nb3 + c 3 = (n3 + 1) b1 b2 b3

RI
nc1 + a1 nc 2 + a2 nc 3 + a3 c1 c 2 c3

19. (i) OABC is a regular tetrahedron D is circumcentre of  OAB and E is mid point of edge AC.
Prove that DE is equal to half the edge of tetrahedron.

A
(ii) If V be the volume of a tetrahedron and V  be the volume of the tetrahedron formed by the
centroids and V = k V  then find the value of k.

Given that u = ˆi − 2jˆ + 3kˆ v = 2iˆ + ˆj + 4kˆ , w = 2iˆ + j + 3kˆ and (u.R − 10) i + (v.R − 20) j + (w.R − 20) k̂ =

UH
20.
0 then find R

21. AB , AC and AD are three adjacent edges of a parallelopiped . The diagonal of the parallelopiped
passing through A and directed away from it is vector a . The vector area of the faces containing
JA
vertices A , B , C and A , B , D are b and c respectively i.e. AB  AC = b and AD  AB = c . If
|a|
projection of each edge AB and AC on diagonal vector a is , then find the vectors AB, AC and
3
AD in terms of a, b, c and | a | .
LP

22. Prove that if the equation r  a = b and r  c = d are consistent (a.d  0,b.c  0,d  b  0 ) then

b.c + a.d = 0
 
23. If a & b are two non collinear vector a . b  0 a  (a  (a  (a..............  (a  (a  b)) =  (a  b)
A

2018 times

24. Let P be an interior point of a triangle ABC and AP, BP, CP meet the sides BC, CA, AB is D, E, F
NK

respectively. Show that


AP AF AE
= +
PD FB EC

27. Lengths of two opposite edges of a tetrahedron are a and b. Shortest distance between these edges is
d and the angle between them is . Prove that its volume is (1/6) abd sin .  
SA

28. If r.a = 0, r . b = 1 and [r a b] = 1 , ab  0 and | a |2 | b |2 – (a . b)2 = 1 then find r in terms of a &


b

29. Let u & v be unit vectors. If w is a vector such that w + ( w x u) = v , then prove that
1
(u  v ). w 
2
and the equality holds if and only if u is perpendicular to v .

30. Let A is set of all possible planes passing through four vertices of given cube. Find number of ways of
selecting four planes from set A, which are linearly dependent and one common point. (If planes P 1 = 0,
P2 = 0, P3 = 0 and P4 = 0 can be writen as aP1 + bP2 + cP3 + dP4 = 0, where all a, b, c, d are not equal
to zero, then we say planes P1, P2, P3, P4 are linearly dependent planes).
31. Let OABC is a regular tetrahedron and P is any point in space. If edge length of tetrahedron is 1 unit,
find the least value of 2(PA 2 + PB2 + PC2 + PO2).

32. Find the minimum value of x 2 + y2 + z2 when ax + by + cz = p.

x−a+d y−a z −a−d x −b+c y −b z−b−c


33. The lines = = , = = are coplanar and these
−  + −  +
determine a single plane if   . Find the equation of the plane in which they lie.

 −1  1  1 
     
34. Consider the plane E : r =  1 +  2 +  0 
 1 0  1 
F is a plane containing the point A (–4, 2, 2) and parallel to E. Suppose the point B is on the plane E,

RI
such that B has a minimum distance from point A. If C(–3, 0, 4) lies in the plane F. Then find the area of
ABC.

35. Through a point P (h, k, ) a plane is drawn at right angles to OP to meet the co-ordinate axes in A, B

A
5
p
and C. If OP = p, show that the area of ABC is , where O is the origin.
2hk

UH
36. If A (a) , B (b) and C (c) are three non collinear points and origin does not lie in the plane of the points
A, B and C, then for any point P (p) in the plane of the ABC, prove that ;
(i) a b c  = p . a x b + b x c + c x a
( )
 
JA
(ii) A point v is on plane of ABC such that vector ov is ⊥ to plane of ABC. Then show that
[a b c](a  b + b  c + c  a)
v = , where  is the vector area of the  ABC.
4 2

37. Prove that the equation of the sphere which passes through the points (1, 0, 0), (0, 1, 0) and (0, 0, 1)
and having radius as small as possible is 3 x2 – 2 x – 1 = 0.
LP

x y z −1
38. Prove that the line = = lies in the plane x + y + z = 1. Find the lines in the plane through the
1 1 −2
 1 
point (0, 0, 1) which are inclined at an angle cos –1   with the line.
A

 6

39. Find the equation of the sphere which has centre at the origin and touches the line
NK

2(x + 1) = 2 – y = z + 3.

40. A mirror and a source of light are situated at the origin O and at a point on OX (x-axis) respectively. A
ray of light from the source strikes the mirror and is reflected. If the Drs of the normal to the plane are
1, –1, 1, then find d.c’s of the reflected ray.
SA

41. A variable plane x + my + nz = p (where , m, n are direction cosines) intersects with co-ordinate axes
at points A, B and C respectively show that the foot of normal on the plane from origin is the
orthocentre of triangle ABC and hence find the coordinates of circumcentre of triangle ABC.

42. A rectangle whose vertices are (5,3,–3), (5,9,9), (0,5,11) and (0,–1,–1) is rotated about its diagonal
 1 2 2
(whose direction cosines are  , ,  in such a way that new position of rectangle is perpendicular to
3 3 3
its old position find the coordinates of new position of the vertices whose position is changed.

43 A solid sphere ‘S’ present in space (above X-Y plane) whose equation is (x – a) 2 + (y – b)2 + (z – c)2 = r2
x y z
(i) A light source lies on the line = = above X-Y plane at infinite distance from X-Y plane.
a b c
If a = c and b = 0 then find the equation of the boundary of shadow of ‘S’ on X-Y plane.
(ii) A light source is at (5,0,3), a = c = 2, b = 0, r = 1. What is the locus name of boundary of
shadow of ‘S’ on X-Y plane.

1 x–2 y–8 z+3 1


4. 0 7. = 15. (ii) (2, 8, –3) & (0, 1, 2); 78 ; = = 16.
2 2 7 –5 2

17. 1 19. (ii) 27 20. R = 10iˆ

ca 1 a  (b − c) 3(b  a) 1 a  (b − c) 3(c  a)


21. AB − AD = 3 ; AC = a + + ; AD = a + –
| a |2 3 | a |2 | a |2 3 | a |2 | a |2

RI
23. – | a |2018 26. xyz = 6k3 28. r = – (a . b) a + | a |2 b + (a  b)

p2 9

A
30. 135 31. 3 32. 33. x – 2y + z = 0 34.
 a2 2

x y z −1 x y z −1

UH
38. = = and = = 39. 9(x2 + y2 + z2) = 5
1 + 15 1 − 15 −2 1 − 15 1 + 15 −2

1 2 2
40. d.c’s of the reflected ray are − , − ,
3 3 3
JA
 p − 2 p p − m2 p p − n2p 
41.  , , 
 2 2m 2n 

42. {(5, –3,3), (5,11,5)} or {(–3,5,–1), (8,3,9)}


LP

43 (i) x2 + 2y2 = 2r2 (ii) Parabola


A
NK
SA

You might also like